Está en la página 1de 729

ANÁLISIS DEL CASO CLÍNICO

IDENTIFICACIÓN DEL REACTIVO


Area: CIRUGÍA
Especialidad: CIRUGÍA ABDOMINAL
Tema: PATOLOGÍA ABDOMINAL RESOLUCIÓN QUIRÚRGICA DE
URGENCIA
Subtema: HERNIA VENTRAL

CASO CLÍNICO CON UNA PREGUNTA

HOMBRE DE 53 AÑOS DE EDAD, DIABÉTICO DE LARGA EVOLUCIÓN, CONTROLADO. REFIERE


ANTECEDENTE DE LAPARATOMÍA EXPLORADORA HACE 3 AÑOS POR TRAUMATISMO CERRADO DE
ABDOMEN. DESDE HACE 6 MESES PRESENTA UN SACO HERNIARIO POR ARRIBA DE LA CICATRIZ
UMBILICAL. EL DÍA DE HOY PRESENTA DOLOR ABDOMINAL LOCALIZADO EN LA ZONA DEL DEFECTO
HERNIARIO, EL CUAL SE VUELVE INCAPACITANTE. A SU LLEGADA SE ENCUENTRA CON FIEBRE, PALIDEZ
EN PIEL, DIAFORÉSIS, TAQUICARDIA, DOLOR ABDOMINAL INTENSO, ABDOMEN EN TABLA, HERNIA
VENTRAL DOLOROSA NO REDUCTIBLE. SE INDICA CIRUGÍA DE URGENCIA

53 años.

diabetes, laparatomía exploradora, hernia


ventral de 6 meses de evolución.

dolor abdominal localizado en el defecto


herniario que evoluciona a incapacitante.

datos que sugieren peritonitis.

1 - EL SIGUIENTE ES EL PROCEDIMIENTO QUIRÚRGICO INDICADO:

PLASTÍA SIN Dado que la respuesta indica la vía laparoscópica para la corrección de la hernia ventral,
MATERIAL se infiere que el paciente se encuentra sin complicaciones. Actualmente se prefiere la vía
PROTÉSICO POR laparoscópica por sus ventajas en el tiempo de estancia hospitalaria y menor dolor
LAPAROSCOPÍA. postoperatorio; solo que no deberá realizarse cuando no exista personal capacitado para
el uso de esta técnica quirúrgica. En cuanto a la colocación de la malla, se prefiere el uso
de este material protésico para aminorar el riesgo de recidiva. EN ESTE PACIENTE NO SE
RECOMIENDA EL USO DE LA TÉCNICA LAPAROSCÓPICA DADO QUE EL CUADRO CLÍNICO
SUGIERE SEPSIS ABDOMINAL.
PLASTÍA SIN EN PRIMER LUGAR DEBERÁS TENER EN CUENTA QUE LOS DATOS CLÍNICOS SUGIEREN LA
MATERIAL PRESENCIA DE PERITONITIS. A considerar: 1. Actualmente se prefiere la VÍA
PROTÉSICO POR LAPAROSCÓPICA por sus ventajas en el tiempo de estancia hospitalaria y menor dolor
CIRUGÍA postoperatorio; se prefiere a la técnica abierta, siempre que este disponible y exista el
ABIERTA personal capacitado. 2. En cuanto a la COLOCACIÓN DE LA MALLA, se prefiere el uso de
este material protésico para aminorar el riesgo de recidiva. 3. Este paciente presenta
datos que sugieren COMPLICACIÓN (PERITONITIS, hernia encarcelada e incluso
DESCARTAR SEPSIS) tales como dolor abdominal incapacitante, fiebre, palidez de piel,
diaforesis, taquicardia, abdomen en tabla; además de una hernia no reducible, lo cual
condiciona la urgencia del manejo. 4. El flujograma de la GPC en cuestión, refiere que,
para los pacientes con hernia ventral complicada, como en este caso y en los cuales exista
“sepsis abdominal”, se debe realizar la HERNIOPLASTIA SIN MALLA.

PLASTÍA CON La técnica laparoscópica está indicada en pacientes sin datos de complicación. ESTE
MATERIAL PACIENTE PRESENTA DATOS QUE SUGIEREN PERITONITIS, POR LO CUAL SU MANEJO
PROTÉSICO POR DEBERÁ SER QUIRÚRGICO ABIERTO.
LAPAROSCOPÍA

PLASTÍA CON La PLASTÍA CON MATERIAL PROTÉSICO POR CIRUGÍA ABIERTA está indicado en aquellos
MATERIAL pacientes con hernia ventral en los que presenten datos de complicación siempre que no
PROTÉSICO POR exista SEPSIS ABDOMINAL. DADO QUE EL PACIENTE PRESENTA DATOS QUE SUGIEREN
CIRUGÍA PERITONITIS, NO ESTÁ JUSTIFICADO EL USO DE MATERIAL PROTÉSICO.
ABIERTA

Bibliografía:
DIAGNÓSTICO Y TRATAMIENTO DE LA HERNIA VENTRAL EN EL ADULTO. EVIDENCIAS Y
RECOMENDACIONES: GUÍA DE PRÁCTICA CLÍNICA. MÉXICO: SECRETARÍA DE SALUD;
27/MARZO/2014.

http://www.cenetec-difusion.com/CMGPC/SS-211-09/ER.pdf
ANÁLISIS DEL CASO CLÍNICO

IDENTIFICACIÓN DEL REACTIVO


Area: MEDICINA INTERNA
Especialidad: REUMATOLOGÍA
Tema: SÍNDROMES DE LAS VASCULITIS
Subtema: LUPUS ERITEMATOSO SISTÉMICO

CASO CLÍNICO CON UNA PREGUNTA

FEMENINO DE 36 AÑOS CON DIAGNÓSTICO DE LUPUS DESDE HACE 3 AÑOS. EL DÍA DE HOY SE
INGRESA CON DIAGNÓSTICO DE LUPUS ERITEMATOSO GENERALIZADO ACTIVO SEVERO.

femenino de 36 años de edad.

tiene diagnostico de lupus desde hace 3


años.

actualmente con actividad severa de la


enfermedad.

-.

-.

2 - EN ESTE MOMENTO ESTA INDICADO LA ADMINISTRACIÓN DE:

METOTREXATE. Aunque hay mucha evidencia que apoya la efectividad del metotrexate en la artritis
reumatoide, "no hay suficiente información para tratar a los pacientes con LES". Las
recomendaciones actuales sugieren su uso como un fármaco para disminuir los
glucocorticoides en las manifestaciones cutáneas y articulares de la enfermad. NO SE
TIENEN EVIDENCIA CLARA DEL EFECTO QUE TIENEN EL METROTEXATE PARA EL
TRATAMIENTO DEL LUPUS ERITEMATOSO.
GLUCOCORTICOIDES. LA BASE DEL TRATAMIENTO DE CUALQUIER MANIFESTACIÓN DEL LES
POTENCIALMENTE LETAL O QUE PUEDA DAÑAR UN ÓRGANO, ES LA
ADMINISTRACIÓN SISTÉMICA DE GLUCOCORTICOIDES. Los glucocorticoides están
indicados en los pacientes con enfermedad moderada a severa ya sea como
monoterapia o en combinación con agentes inmunosupresores. Las dosis de
prednisona van de 0.5 a 1 mg/kg/día en una dosis única generalmente en la mañana.
Cuando estos se combinan con medicamentos inmunosupresores en raras ocasiones
se dan dosis mayores de 0.5 a 0.6 mg/kg/día debido al mayor riesgo de desarrollar
infecciones, incluyendo las infecciones oportunistas. La toxicidad por
glucocorticoides es un problema principal y la disminución de la dosis es la principal
preocupación. Generalmente la disminución de la dosis se inicia después de las
primeras 4 a 6 semanas de tratamiento. El objetivo es una dosis de 0.25 mg/kg cada
otro día a los 2 a 3 meses, que es una dosis aceptable para el uso crónico del
medicamento. El uso concomitante con otro inmunosupresor facilita esta
disminución y disminuye su toxicidad. En los casos en donde se necesiten dosis
mayores de 0.6 mg/kg/día o en enfermedad severa progresiva se pueden usar bolos
por 1 a 3 días de metilprednisolona a dosis de 1 g/día seguido de 0.5 mg/kg/día de
prednisona. Es muy importante proteger a los pacientes que toman glucocorticoides
de la osteoporosis. LOS ESTERIOIDES ESTÁN INDICADOS EN CASOS DE LUPUS
SEVERO.

CICLOFOSFAMIDA. La CICLOFOSFAMIDA, es efectiva para el tratamiento de nefritis lúpica proliferativa


moderada a severa en bolos intermitentes con un perfil tóxico más seguro. También
es el tratamiento de elección para aquellos pacientes con síntomas de lupus
neuropsiquiátrico severo. Los efectos adversos más comunes de la ciclofosfamida
son alopecia reversible y nauseas, otros menos frecuentes son mielotoxicidad,
toxicidad a las gónadas y malignidad. Pueden presentarse varias infecciones
incluyendo infecciones bacterianas y oportunistas como infección por Pneumocystis
jirovecii, infecciones micóticas y Nocardia y reactivación de infecciones latentes
como herpes zoster, tuberculosis e infección por el virus del papiloma humano. El
uso de pulsos intermitentes de ciclofosfamida junto con hidratación adecuada
prácticamente ha eliminado los casos de carcinoma de la vejiga, aunque se puede
observar cistitis hemorrágica. De forma rutinaria se debe indicar mesna, un agente
que reduce la concentración de acroleína y posiblemente otros metabolitos tóxicos
en la vejiga. ESTÁ INDICADA EN LUPUES NEUROPSIQUIÁTRICO Y NEFROPATÍA
LÚPICA.

AZATIOPRINA. En vista de que el tratamiento a base de glucocorticoides y ciclofosfamida tiene


varios efectos adversos se han buscado otros fármacos citotóxicos menos tóxicos. La
AZATIOPRINA (antagonista de la purina) junto con glucocorticoides reduce el
número de exacerbaciones de LES y la dosis de mantenimiento de los esteroides; no
obstante, la eficacia de este método se demora varios meses y además la
ciclofosfamida resulta eficaz en una mayor proporción de pacientes. La azatioprina
diaria por vía oral tiene menos efectos adversos que la ciclofosfamida diaria por la
misma vía, pero la ciclofosfamida intermitente por vía intravenosa tiene menos
efectos adversos que esta última. LOS RESULTADOS CON AZATIOPRIMA SE LOGRA
TRAS VARIOS MESES DE TRATAMIENTO POR LO QUE NO ESTÁ INDICADO EN ESTE
CASO.

Bibliografía:
1. LONGO DL, FAUCI AS, KASPER DL, HAUSERSL, JAMESON JL, LOSCALZOJ. HARRISON. PRINCIPIOS
DE MEDICINA INTERNA, 18A EDICIÓN. MC GRAW HILL. NEW YORK, USA. 2012, PP 2732-2733.
ANÁLISIS DEL CASO CLÍNICO

IDENTIFICACIÓN DEL REACTIVO


Area: PEDIATRÍA
Especialidad: URGENCIAS PEDIÁTRICAS
Tema: INTOXICACIONES
Subtema: POR HIERRO

CASO CLÍNICO SERIADO

LACTANTE DE 1 AÑO 10 MESES DE EDAD, EL CUAL ES LLEVADO AL SERVICIO DE URGENCIAS POR SU


MADRE QUIEN REFIERE HACE MENOS DE 40 MINUTOS HABER NOTADO QUE INGIRIÓ UN FRASCO DE
TABLETAS, NO RECUERDA SU CONTENIDO PERO SABE QUE SE LAS RECETARON PARA EL EMBARAZO. A
SU INGRESO SOMNOLIENTO, POLIPNÉICO, CON REFLEJO NAUSEOSO IMPORTANTE. SE SOSPECHA
INTOXICACIÓN POR HIERRO.

Lactante mayor.

Ingesta de tabletas probablemente hierro


hace 20 minutos.

Somnolencia, polipnea y re ejo nauseoso.

3 - SERÍA DE AYUDA PARA CONFIRMAR LA INGESTA DE HIERRO EN ESTE MOMENTO:

BIOMETRÍA La intoxicación por HIERRO es una de las más comunes durante la infancia, debido a su
HEMÁTICA. accesibilidad y a la adición de diversos alimentos y dulces con este metal. Tras su entrada
al organismo provoca corrosión en la mucosa gastrointestinal tras lo cual se deposita en
las mitocrondrías y los tejidos corporales, volviéndose un problema sistémico con los
siguientes efectos: - Induce vasodilatación y aumento de la permeabilidad capilar que
llevan a la hipotensión. - Acidosis metabólica por disminución de la perfusión periférica y
lesión mitocondrial. - Elevación de las enzimas hepáticas que puede llegar hasta necrosis
del tejido hepático. - Afección del SNC manifestado por somnolencia que puede llegar a
coma. Los signos clásicos de envenenamiento son: náuseas, vómito, diarrea y dolor
abdominal de 30 minutos a 6 hrs después de la ingestión. Raramente se presenta
hematemésis o sangre en heces.
RADIOGRAFÍAS COMO EL HIERRO ES RADIOPACO, LA RADIOGRAFÍA DE ABDOMEN PUEDE AYUDAR A LA
DE ABDOMEN. DETERMINACIÓN DE LA INGESTA DE HIERRO. Las radiografías seriadas pueden ser de
ayuda en el seguimiento de la desintoxicación del menor. OJO: UN RESULTADO NEGATIVO
NO EXCLUYE LA INGESTIÓN, YA QUE SÓLO PUEDEN VERSE LOS COMPRIMIDOS NO
DISUELTOS (dado el tiempo desde la ingesta y la cantidad de medicamento ingerido si
puede ser de gran utilidad).

NIVELES ESTA DETERMINACIÓN DEBE REALIZARSE 4HRS DESPUÉS DE LA INGESTIÓN, POR TAL
SÉRICOS DE MOTIVO NO ES UNA PRUEBA ADECUADA EN ESTE MOMENTO. Niveles séricos inferiores a
HIERRO. 500ng/dl entre las 4 a 8 hrs después de la ingestión se relacionan con un riesgo bajo de
intoxicación. NO EXISTE UNA BUENA CORRELACIÓN ENTRE LOS SIGNOS CLÍNICOS Y LOS
NIVELES SÉRICOS DE HIERRO. Por tanto ante la sospecha de intoxicación por hierro aún
con niveles por debajo de 500 en pacientes con síntomas clínicos asociados deberá
iniciarse manejo con quelante específico.

EXAMEN DE Anteriormente cuando no se disponían de otros métodos más confiables, se realizaba la


ORINA. comparación del color de la orina tras el uso de deferoxamina ya que ésta forma la
ferrioxamina cuando captura el hierro libre y lo excreta vía renal dando a la orina un color
vino rosado. No se considera un dato confiable de una adecuada respuesta a la terapia
pero debe contemplarse como dato clínico.

Bibliografía:
1. BERHMAN R, KLIEGMAN R, JENSON H. NELSON TRATADO DE PEDIATRÍA, 16A EDICIÓN. MC GRAW
HILL. MÉXICO, 2001.

4 - CORRESPONDE AL MANEJO QUE DEBERÁ INDICARSE EN ESTE PACIENTE:

ADMINISTRACIÓN EL CARBÓN ACTIVADO NO ABSORBE EL HIERRO por lo que no debe utilizarse en este
DE CARBÓN caso.
ACTIVADO.

FOMEPIZOL. El FOMEPIZOL es un antídoto competitivo del alcohol deshidrogenasa y ha sido


aprobado para el uso de intoxicación por ETILENGLICOL.

IRRIGACIÓN La irrigación intestinal total sólo puede ser útil para arrastrar los comprimidos al
INTESTINAL intestino pero no revierte la intoxicación.

DESFEROXAMINA La DESFEROXAMINA se considera el ANTÍDOTO ESPECÍFICO PARA LAS


PARENTERAL. INTOXICACIONES CRÓNICAS Y AGUDAS POR HIERRO al ser un quelante específico de
este metal. Está indicada cuando se presentan niveles séricos de hierro mayores de
500ng/dl, tanto en presencia como en ausencia de síntomas, un nivel de hierro sérico
superior a 500ng/dl con síntomas moderados o severos; o la presencia de síntomas
moderados o graves independientemente de las cifras de hierro sérico.

Bibliografía:
1. BERHMAN R, KLIEGMAN R, JENSON H. NELSON TRATADO DE PEDIATRÍA, 16A EDICIÓN. MC GRAW
HILL. MÉXICO, 2001.

FIN DEL CASO CLÍNICO SERIADO


ANÁLISIS DEL CASO CLÍNICO

IDENTIFICACIÓN DEL REACTIVO


Area: MEDICINA INTERNA
Especialidad: INFECTOLOGÍA
Tema: INFECCIONES PULMONARES
Subtema: NEUMONÍA NOSOCOMIAL

CASO CLÍNICO CON UNA PREGUNTA

DURANTE SU CURSO DE INDUCCIÓN AL SERVICIO DE TERAPIA INTENSIVA, SE LE INDICAN LAS MEDIDAS PREVENTIVAS PARA
EVITAR LAS INFECCIONES NOSOCOMIALES. EN ESTE MOMENTO SE LE PIDE HACER ESPECIAL ÉNFASIS EN AQUELLAS
INFECCIONES QUE AFECTAN EL TRACTO RESPIRATORIO DEBIDO A SU ALTA INCIDENCIA EN EL HOSPITAL DONDE SE
ENCUENTRA ROTANDO.

-.

INFECCIONES NOSOCOMIALES.

INFECCIONES QUE AFECTAN EL TRACTO


RESPIRATORIO.

-.

-.

5 - ES LA MEDIDA PREVENTIVA MÁS EFICAZ PARA EVITAR NEUMONÍA EN PACIENTES CON VENTILACIÓN MECÁNICA:

LAVADO SISTEMÁTICO El LAVADO DE MANOS de las personas que atienden a los pacientes en el hospital es el procedimiento más
DE MANOS simple, económico y eficaz en la prevención de las infecciones intrahospitalarias (que incluyen la neumonía
en pacientes con ventilación mecánica), logrando reducir su incidencia hasta en 50% cuando se realiza de
manera adecuada y sistemática.

CULTIVOS RUTINARIOS Los cultivos para detectar la presencia de bacterias en el ambiente hospitalario realizados de forma rutinaria
DEL EQUIPO DE a los pacientes, equipos de ventilación mecánica, dispositivos, accesorios y equipos de anestesia general NO
VENTILACIÓN DISMINUYEN LA APARICIÓN DE NEUMONÍA EN PACIENTES CON VENTILACIÓN MECÁNICA en cambio si
aumentan los costos.

PREFERIR LA Se recomienda, si es necesaria la intubación, LA VÍA OROTRAQUEAL SOBRE LA NASOTRAQUEAL. Esto debido
INTUBACIÓN a que en la ventilación nasotraqueal suele haber mayor infestación del tracto respiratorio bajo por bacterias
NASOTRAQUEAL de las vías aéreas superiores sobre todo en aquellos pacientes con sinusitis concomitante.

REALIZAR CAMBIOS Se recomienda NO realizar cambios rutinarios del circuito, cuando el actual se observa en buenas
RUTINARIOS DEL condiciones. Esto con el fin de disminuir la manipulación de los aparatos accesorios. Es recomendable utilizar
EQUIPO un circuito nuevo para cada paciente, y realizar el cambio por un circuito nuevo si el que se está utilizando se
INDEPENDIENTEMENTE encuentra sucio o dañado.
DE SU ESTADO
Bibliografía:
1. GUÍA DE PRÁCTICA CLÍNICA, PREVENCIÓN, DIAGNÓSTICO Y TRATAMIENTO DE LA NEUMONÍA ASOCIADA A VENTILACIÓN
MECÁNICA. MÉXICO: SECRETARÍA DE SALUD, 2013. 2. GUÍA DE PRÁCTICA CLÍNICA, PREVENCIÓN DE LA NEUMONÍA
ADQUIRIDA EN LA VENTILAIÓN MECÁNICA EN NIÑOS Y ADULTOS EN EL SEGUNDO Y TERCER NIVELES DE ATENCIÓN.
MÉXICO: SECRETARÍA DE SALUD, MARZO 2013.
HTTP://WWW.CENETEC.SALUD.GOB.MX/DESCARGAS/GPC/CATALOGOMAESTRO/098_GPC_NEUMONIAVENTILACION/SSA_098_08_EYR.PDF

http://www.cenetec.salud.gob.mx/descargas/gpc/CatalogoMaestro/IMSS_624_13_NEUM_VENTIL_MECANICA/624GER.pdf
ANÁLISIS DEL CASO CLÍNICO

IDENTIFICACIÓN DEL REACTIVO


Area: MEDICINA INTERNA
Especialidad: GASTROENTEROLOGÍA
Tema: PATOLOGÍA HEPÁTICA Y PANCREATICA
Subtema: PATOLOGIA DE PANCREAS

CASO CLÍNICO CON UNA PREGUNTA

FEMENINA DE 35 AÑOS, HOSPITALIZADA CON DIAGNÓSTICO DE PANCREATITIS AGUDA.

femenino de 35 años de edad

-.

pancreatitis aguda

-.

-.

6 - EL SIGUIENTE FACTOR DE RIESGO ES EL MÁS IMPORTANTE EN LA PATOGENIA DE ESTA ENFERMEDAD:

GÉNERO La pancreatitis es más frecuente en mujeres sin embargo el género no está implicado en la patogenia de la enfermedad.

EDAD El segundo factor de riesgo implicado en la causa pancreatitis aguda es el consumo de alcohol. Corresponde al menos al 30% de los
casos. Y el alcoholismo es la principal causa de pancreatitis crónica.

COLECISTITIS La PANCREATITIS AGUDA es un proceso inflamatorio agudo del páncreas que frecuentemente involucra tejido peripancreático y puede
involucrar órganos y sistemas distantes. Cursa con dolor abdominal, elevación de amilasa y lipasa sérica o hallazgos de pancreatitis en
la tomografía axial computada o en la resonancia magnétia. LA LITIASIS VESICULAR Y EL ABUSO CRÓNICO DE ALCOHOL SON
RESPONSABLES DE 70% DE LOS CASOS DE PANCREATITIS. La obstrucción por litos vesiculares corresponde al 40% de los casos de
pancreatitis aguda, sin embargo de los pacientes que tienen litiasis vesicular solo el 3 a 7% desarrollan pancreatitis. La pancreatitis
biliar es más frecuente en mujeres que en hombres por que la litiasis vesicular es más frecuente en mujeres. La pancreatitis aguda se
presenta con frecuencia cuando los litos tienen un diámetro menor de 5 mm debido a que los litos pequeños es más factible que pasen
a través del conducto cístico y causen obstrucción del ámpula. LA PANCREATITIS BILIAR ES MÁS FRECUENTE EN MUJERES POR LITIASIS
VESICULAR. IMPORTANTE: este es una pregunta 100% tipo ENARM en donde el objetivo es identificar la respuesta MÁS CORRECTA.
Luego entonces, lo primero que hay que hacer es identificar que los dos factores de riego más importantes para pancreatitis los cuales
son: la colelitiasis y el alcohol. Si en las opciones de respuesta viniera también colelitiasis esa sería la opción correcta, sin embargo, no
es así. En lo que respecta al resto de respuestas, la obesidad es un factor de mal pronóstico y gravedad, no un factor de riesgo como tal
que intervenga en la patogenia (motivo de la pregunta); el género y la edad no se consideran como factores de riesgo para este caso,
por lo tanto, la única opción probable es la COLECISTITIS, que deberás recordar es, de acuerdo a la definición de la GPC, la inflamación
de la vesícula biliar ocasionada principalmente por cálculos (litos) y con menor frecuencia por barro (lodo) biliar. En conclusión, de las 4
posibles respuestas LA COLECISTITIS ES LA RESPUESTA MÁS CORRECTA. Como puedes ver, el conocimiento de los factores de riesgo
basta para decidir cuál es y no es, la opción más correcta.

OBESIDAD La hipertrigliceridemia es quizá la tercera causa identificada de pancreatitis aguda seguida de la biliar y luego la alcohólica. Las
concentraciones séricas de triglicéridos mayores de 1000 mg/dl pueden precipitar un ataque agudo de pancreatitis. La
hipertriglieridemia puede causar cerca del 5% de los casos de pancreatitis aguda. LA OBESIDAD ES UN FACTOR DE RIESGO, PERO
PROPIAMENTE DEBE SER CONSIDERADA LA HIPERTRIGLICERIDEMIA COMO EL FACTOR DE RIESGO Y NO LA OBESIDAD.

Bibliografía:
1. GUÍA DE PRÁCTICA CLÍNICA, DIAGNÓSTICO Y REFERENCIA OPORTUNA DE LA PANCREATITIS AGUDA EN EL PRIMER NIVEL DE ATENCIÓN.
MÉXICO: SECRETARIA DE SALUD, NOVIEMBRE 2012. 2. GUÍA DE PRÁCTICA CLÍNICA, DIAGNÓSTICO Y TRATAMIENTO DE LA PANCREATITIS
AGUDA. MÉXICO: SECRETARIA DE SALUD, 2009.
HTTP://WWW.CENETEC.SALUD.GOB.MX/DESCARGAS/GPC/CATALOGOMAESTRO/239_PANCREATITIS_AGUDA/PANCREATITIS_AGUDA_EVR_CENETEC.PDF

http://www.cenetec.salud.gob.mx/descargas/gpc/CatalogoMaestro/011_GPC_PancreatAguda/SSA-011-08_PANCREATITIS_AGUDAEVR.pdf
ANÁLISIS DEL CASO CLÍNICO

IDENTIFICACIÓN DEL REACTIVO


Area: CIRUGÍA
Especialidad: OTORRINOLARINGOLOGÍA
Tema: PATOLOGÍA DEL OÍDO
Subtema: OTITIS EXTERNA

CASO CLÍNICO CON UNA PREGUNTA

HOMBRE DE 23 AÑOS QUE PRESENTA OTALGIA DERECHA MUY INTENSA DE UN DÍA DE EVOLUCIÓN.

Adulto.

El tiempo de evolución es muy importante,


la OTITIS EXTERNA AGUDA tiene una
evolución rápida de menos de 48hr.

Deben interrogarse prurito e hipoacusia.

Es clásica la hiperestesia del pabellón


auricular, otorrea (puede o no estar
presente), MEMBRANA ÍNTEGRA (obligada
para ser otitis externa).

-.

7 - LA AUTOMEDICACIÓN CON ANALGÉSICO TÓPICO A NIVEL ÓTICO EN ESTE CASO, ESTARÍA


CONTRAINDICADA PRINCIPALMENTE PORQUE:

LESIONA EL El ALIVIO DEL DOLOR es un objetivo primordial en el tratamiento integral de la otitis


EPITELIO DEL externa aguda, ya que con frecuencia es severo e interfiere con las actividades diarias o
CONDUCTO laborales del paciente. Dependiendo de la severidad del dolor, se pueden requerir
AUDITIVO. desde anti inflamatorios no esteroideos hasta analgésicos narcóticos. Los analgésicos
tópicos incluso pueden enmascarar el cuadro clínico y la benzocaína se ha asociado con
dermatitis de contacto como complicación. NO TODOS LOS ANALGÉSICOS CAUSAN
ESTE EFECTO, ES ESPECIALMENTE LA BENZOCAÍNA LA QUE SE HA DOCUMENTADO SE
ASOCIA CON DERMATITIS POR CONTACTO.
ELIMINA LA Dependiendo de la SEVERIDAD DEL DOLOR, para el tratamiento de la otitis externa
BARRERA aguda, se puede requerir desde antiinflamatorios no esteroideos hasta analgésicos
MECÁNICA DEL narcóticos. 1. Si el dolor es de LEVE A MODERADO se recomienda: acetaminofen 500
OÍDO EXTERNO. mg VO cada 8 horas en combinación con naproxen 250 mg VO cada 12 horas durante
72hs. 2. Si el dolor es SEVERO se sugiere: dextropropoxifeno 65 mg VO cada 8-12 horas
durante las primeras 24 horas de tratamiento y administrarlo durante un máximo de 48
horas. OJO: No hay ensayos clínicos que demuestren que la analgesia tópica sea eficaz
para el manejo de la otitis externa aguda. NO SE RECOMIENDAN LOS ANALGÉSICOS
ÓTICOS PARA EL MANEJO DE LA OTITIS EXTERNA AGUDA EN NINGÚN CASO.

EXISTE RIESGO DE RECUERDA: siempre que indiques gotas en oído (sin importar el tipo) es necesario
CONTAMINACIÓN verificar que NO ESTE PERFORADA LA MEMBRANA TIMPÁNICA, porque si así fuera
DEL OÍDO MEDIO. contaminamos el oído medio con bacterias del oído externo. EL DATO BÁSICO PARA
CONTESTAR ESTA PREGUNTA SE ENCUENTRA EN SU REDACCIÓN: LA
"AUTOMEDICACIÓN".... ESTARÍA CONTRAINDICADA... PORQUE?... - EN TODOS LOS
CASOS SE CONTRAINDICA LA AUTOMEDICACIÓN CON APLICACIÓN TÓPICA DENTRO
DEL CONDUCTO ANTE EL RIESGO DE QUE LA MEMBRANA SE ENCUENTRE ROTA.

ENMASCARA EL Los analgésicos tópicos pueden incluso enmascarar el cuadro clínico y la benzocaína se
CUADRO CLÍNICO. ha asociado con dermatitis de contacto como complicación. TODAS LAS RESPUESTAS
RESPONDEN EN MAYOR O MENOR MEDIDA A RAZONES POR LAS CUALES SE
CONTRAINDICAN LOS ANALGÉSICOS VÍA TÓPICA; sin embargo, "POR LA SEVERIDAD
DE LA COMPLICACIÓN, LA OPCIÓN ANTERIOR ES MÁS CORRECTA". RECUERDA: en
todo caso de otalgia, antes que cualquier otra cosa, deberás tener en cuenta la
integridad de la membrana para elegir la vía de administración de los medicamento.

Bibliografía:
1. GUÍA DE PRÁCTICA CLÍNICA, DIAGNÓSTICO Y TRATAMIENTO DE LA OTITIS EXTERNA AGUDA EN
ADULTOS. MÉXICO: SECRETARIA DE SALUD; 2010. RECUPERADO DE
HTTP://WWW.CENETEC.SALUD.GOB.MX/CONTENIDOS/GPC/CATALOGOMAESTROGPC.HTML

http://www.cenetec.salud.gob.mx/descargas/gpc/CatalogoMaestro/443_GPC_Otitis_externa_aguda/GER_Otitis_externa_aguda.pdf
ANÁLISIS DEL CASO CLÍNICO

IDENTIFICACIÓN DEL REACTIVO


Area: GINECOLOGÍA Y OBSTETRICIA
Especialidad: OBSTETRICIA
Tema: ENFERMEDAD HIPERTENSIVA DEL EMBARAZO
Subtema: ECLAMPSIA

CASO CLÍNICO SERIADO

MULTIGESTA DE 40 AÑOS DE EDAD EN LA SEMANA 37 DE GESTACIÓN. PREECLAMPSIA EN CONTROL


CON ANTIHIPERTENSIVOS DE PRIMERA LÍNEA Y MEDIDAS GENERALES DESDE HACE 15 DÍAS. ES
LLEVADA AL SERVICIO DE URGENCIAS POR PÉRDIDA DE LA VISIÓN. A SU INGRESO PRESENTA
MOVIMIENTOS TÓNICO CLÓNICOS EN TODO EL CUERPO CON ACTIVIDAD UTERINA Y SANGRADO
TRASVAGINAL.

multigesta años.

preeclamsia de 15 días de evolución en


tratamiento con antihipertensivos.

pérdida de la visión.

convulsión tónico-clónica generalizada,


actividad uterina y sangrado trasvaginal.

-.

8 - LO MÁS PROBABLE ES QUE EL CUADRO CLÍNICO PRESENTE EN LA PACIENTE SEA SECUNDARIO


A:
LIBERACIÓN EXCESIVA No se sabe con certeza el factor etiopatológico de la eclampsia por lo que se han
DE establecido diferentes teorías con base en la observación de los casos y la
NEUROTRANSMISORES investigación clínica. LA TEORÍA DE LOS INTERMEDIARIOS NERVIOSOS está
Y OLEADAS DE basada en la alteración del sistema neurovegetativo y en el bloqueo del sistema
POTENCIAL DE retículo endotelial, demostrable por la hipermeabilidad capilar y por las
ACCIÓN. alteraciones producidas por los "intermediarios neuroquímicos", como la
acetilcolina, la simpatina, y la histamina. El sistema neurovegetativo se sostiene en
estado de equilibrio normal, regulado por las sustancias dichas anteriormente.
Estas sustancias son antagónicas entre sí, pero existen otras que las destruyen, que
son: la aminoxidasa, la histaminasa y la colinesterasa, que respectivamente
destruyen la adrenalina, la histamina y la acetilcolina. Estas diastasas son
secretadas por el sistema retículo endotelial y la placenta es capaz de secretar
histamina y acetilcolina e histaminasa. “Cuando existe una anormalidad en la
producción de las diastasas se rompe el equilibrio; generalmente hay bloqueo de
la aminoxidasa, lo que trae como consecuencias que NO SE DESTRUYA LA
ADRENALINA, la que circulando en mayor cantidad que lo normal, trae consigo la
VASOCONSTRICCIÓN generalizada con la HIPERTENSIÓN ARTERIAL y ATAQUES A
LOS ÓRGANOS, debidos al ESPASMO ARTERIOLAR”. “Los estudiosos de ésta teoría
dicen que en el síndrome eclámptico hay una inadaptación orgánica al complejo
hipófisis placentario diencefálico, con SENSIBILIZACIÓN DEL SISTEMA
NEUROVEGETATIVO A LA ADRENALINA.” Las convulsiones se producen por
liberación excesiva de neurotransmisores excitatorios, en especial glutamato;
despolarización masiva de neuronas en red; y oleadas de potenciales de acción.

HIPOACTIVIDAD En la preeclampsia- eclampsia hay, asimismo, aumento de hormonas de la


HIPOFISIARIA hipófisis: ACTH, la vasopresina, factor antidiurético de Anselmino y Hoffman y
PERSISTENTE. esteroides de las cápsulas suprarrenales (selye), tanto los glucocorticoides como
los mineralocorticoides. Según algunos autores, el ACTH es un factor
importantísimo en el desencadenamiento del síndrome eclámptico y concluyen
que la HIPERACTIVIDAD DE LA HIPÓFISIS con un aumento de ACTH es encontrado
en la mayoría de las toxemias. SE HA ENCONTRADO EN LA MAYOR PARTE DE LAS
TOXEMIAS UNA “HIPERACTIVIDAD HIPOFISIARIA”, pero ello no corresponde
propiamente al cuadro de eclampsia que presenta la paciente.

DILATACIÓN La evidencia del ESPASMO ARTERIOLAR presente en la preeclampsia- eclampsia la


ARTERIOLAR podemos comprobar con el examen del fondo de ojo de una enferma eclámptica,
TEMPORAL. así como con el estudio anatomopatológico de las lesiones encontradas en el
cerebro, hígado y riñones de enfermas muertas por este padecimiento. El cerebro
puede no presentar otras lesiones macroscópicas que la “isquemia de la corteza”,
pero puede haber “edema” y “hemorragia” diminutas múltiples. Al examen
microscópico de las áreas alteradas existen focos de reblandecimiento y arterias
con signos de degeneración grasosa y hialina y presencia de trombos. Hay
también desaparición de células ganglionares y las que quedan se muestran
pignóticas y retraídas. Igualmente han desaparecido las células neuroglia. El área
de lesión parenquimatosa es más extensa que la irrigada por el vaso dañado, lo
que indica que la isquemia es mayor que la lesión vascular. Este hecho hace
suponer que EL TRASTORNO PRIMARIO ES UN ESPASMO ARTERIOLAR y que
cuando la circulación se restablece se forman pequeños trombos por
extravasación de glóbulos rojos debido a la alteración del endotelio. LA
VASOCONTRICCIÓN ES EL PUNTO CLAVE DE LOS SIGNOS CLÍNICOS PRESENTES EN
LA PREECLAMPSIA- ECLAMPSIA.
INHIBICIÓN DE LA Diversas substancias fetales y placentarias penetran en el organismo materno
HISTAMINA VASCULAR obrando como “antígeno” que hace que se produzca el anticuerpo, sensibilizando
CEREBRAL. a la paciente, la que al recibir nueva cantidad de productos extraños, efectúa en su
organismo fenómenos de reacción de antígeno anticuerpo con LIBERACIÓN DE
HISTAMINA Y DE SUSTANCIAS H. La histamina normalmente debe ser destruida
por la histaminasa, por hidrólisis; ya sea por la histaminasa sérica o por la
placentaria. La constante producción de histamina hace que aumente en el
organismo debido a que la histaminasa es insuficiente para producir su
destrucción completa y entonces la histamina llega al tejido nervioso en donde
por su acción específica de “parálisis de los capilares”, con “vasodilatación”, hace
que aumente la circulación cerebral, en un grado mayor que se produzca “edema
cerebral” por la mayor permeabilidad capilar con un “estado progresivo de
disritmia cerebral”. La histamina subsecuentemente continúa sin destruirse y
“excita anormalmente las terminaciones nerviosas parasimpáticas”, haciendo que
liberen acetilcolina. La cual no es destruida por una acción hepática deficiente lo
que lleva a nivel renal a la estimulación de la producción de glucocorticoides y
éstos a su vez de adrenalina produciendo así un círculo vicioso de vasoconstricción
secundaria e isquemia tisular. LA HISTAMINA SE ENCUENTRA AUMENTADA COMO
CONSECUENCIA DE UNA REACCIÓN ANTIGÉNICA DE LA MADRE A LAS
SUSTANCIAS FETALES Y PLACENTARIAS.

Bibliografía:
1. CUNNINGHAM G, LEVENO K, BLOMM S, HAUTH J, RPUSE D, SONG C. WILLIAMS OBSTETRICIA, 23A
EDICIÓN. MC GRAW HILL. USA. 2011 EN ESPAÑOL, PP 722.

9 - AL PERIODO PREVIO A LA CRISIS CONVULSIVA QUE PRESENTA LA PACIENTE SE LE CONOCE


COMO:

AURA. El término “AURA” ha acumulado acepciones de acuerdo a su uso y al contexto. La


palabra proviene del griego y se traduce como “brisa” o viento ligero. Se ha utilizado
para describir los síntomas premonitorios de las crisis epilépticas. Existen diferentes tipos
de aura tales como las motoras (pequeños movimientos involuntarios de las
extremidades), sensoriales, las autonómicas, y las psíquicas. Entre estas últimas se
consideran las de tipo alucinatorio, las oníricas, las intelectuales y las llamadas por
alguno “extáticas”. SE UTILIZA DE FORMA INDISTINTA PARA REFERIRSE A LOS
SÍNTOMAS PREMONITORIOS DE UNA CRISIS CONVULSIVA.

PRÓDROMO DE Fenómenos clínicos relacionados con las crisis epilépticas: 1. PRÓDROMOS: antes de que
LA se produzca una crisis puede presentarse en algunos casos una alteración clínica
CONVULSIÓN. subjetiva u objetiva (como una sensación de malestar o agitación, por ejemplo) que
anuncia que se va a producir una crisis en los próximos minutos, horas o días, estos
síntomas se denominan pródromos. 2. Estado post-crítico y fenómenos postictales: tras
algunos tipos de crisis (sobre todo crisis parciales complejas y crisis generalizadas) el
paciente puede quedar somnoliento, desorientado o confuso durante un tiempo más o
menos prolongado (de minutos a horas), esta fase se denomina estado postcritico y a
veces puede acompañarse también de otros síntomas también transitorios como un
déficit motor (parálisis de todd, tras una crisis focal motora), alteraciones del lenguaje
como afasia o incluso de síntomas de tipo psiquiátrico como agitación, depresión,
síntomas psicóticos, etc. ES CORRECTO LLAMAR PRÓDROMO A LOS SÍNTOMAS PREVIOS
A LA CONVULSIÓN, PERO NO ES ESPECÍFICO DE LA ECLAMPSIA.
ECLAPSISMO. ECLAMPSISMO O ECLAMPSIA INMINENTE: que es la presencia de sintomatología
prodrómica de ataque eclámptico independientemente de las cifras tensionales. CX •
Digestivos: nauseas, vómitos, dolor epigastrio dolor epigastrio; - Visuales: escotomas,
centello, visión oscura. • Neurológicas: cefaleas, Hiperexcitabilidad, híper reflexia,
sensación de vértigo, somnolencia, zumbidos. • Puede o no encontrarse presión arterial
sistólica mayor de 5mmHg con presión arterial diastólica mayor a 115mmHg y/o
proteinuria mayor a 10gr. ECLAMPSISMO ES EL TÉRMINO “ESPECÍFICO” PARA REFERIRSE
A LOS SÍNTOMAS PREMONITORIOS DE LA ECLAMPSIA.

ATAQUE Los SINTOMAS PREMONITORIOS de eclampsia incluyen: • Cefalea intensa persistente 50-
PREMONITORIO. 70 % • Alteraciones visuales 20- 30 % • Dolor epigástrico 20 % • Nauseas – Vómitos 10-
15 % • Desorientación , alteraciones mentales transitorias 5- 10 % LOS SÍNTOMAS
PREMONITORIOS ESTÁN PRESENTES EN SÓLO EL 50% DE LAS ECLAMPSIAS. “Denominar
a los síntomas premonitorios como ataque premonitorio no es un término adecuado ni
específico de ésta patología”.

Bibliografía:
1. CUNNINGHAM G, LEVENO K, BLOMM S, HAUTH J, RPUSE D, SONG C. WILLIAMS OBSTETRICIA, 23A
EDICIÓN. MC GRAW HILL. USA. 2011 EN ESPAÑOL, PP 721-724, 735.

10 - EL TRATAMIENTO INDICADO EN ESTE CASO SERÍA CON:

DIFENILHIDANTOÍNA. Difenilhidantoina sódica: • Impregnación: 10 a 15 mg/kg, lo que equivale a la


administración de 2 a 3 ampolletas de 250 mg. Aforar en solución salina (100 ml) y
administrar a una velocidad no mayor a 50 mg por minuto (con pruebas de
funcionamiento hepático normales). • Mantenimiento: 5 a 6 mg/kg dividido en tres
dosis. SE UTILIZA COMO SEGUNDA ELECCIÓN.

SULFATO DE EL SULFATO DE MAGNESIO ES EL MEDICAMENTO DE ELECCIÓN PARA LA


MAGNESIO. PREVENCIÓN Y TRATAMIENTO DE LAS CRISIS CONVULSIVAS EN LA ECLAMPSIA. El
sulfato de magnesio se recomienda como terapia de primera línea en pacientes con
preeclampsia severa para la prevención de la eclampsia. Se debe suspender a las 24
horas después del nacimiento y en los casos que requiere continuar debe
justificarse. Esta indicado a las siguientes dosis: - Crisis: 4-6 grs intravenosos en 5
minutos diluido en 100 mililitros de solución fisiológica a pasar en 5 minutos . -
Mantenimiento: 1 gr intravenoso por hora (Diluir 900 mililitros de solución
fisiológica con 10 grs de sulfato de magnesio a pasar 100 mililitros por hora) por 24
horas. - Recurrencia de convulsión: 2-4 grs para pasar en 10 min diluido en 100
mililitros de solución fisiológica. El monitoreo para evitar toxicidad se deberá
realizar mediante: reflejo rotuliano, FR >16 rpm, letargia, niveles de magnesio
(objetivo: niveles de 5-7). NOTA: En caso de niveles >7 descontinuar la infusión
Administrar Gluconato de calcio 1 ámpula = 4.64 mEq IV x 1 dosis. IMPORTANTE: El
sulfato de magnesio se asocia con una reducción significativa de las convulsiones
recurrentes y un menor número de muertes maternas, en comparación con otros
anticonvulsivos.

FENOBARBITAL. En caso de no contar con sulfato de magnesio está indicado el uso de


FENOBARBITAL y DIFENILHIDANTOÍNA para el manejo de las convulsiones en la
eclampsia. • FENOBARBITAL: Administrar una ampolleta de 330 mg IM o IV cada 12
horas. SE UTILIZA COMO TERCERA ELECCIÓN.

GLUCONATO DE En caso de intoxicación con sulfato de magnesio utilizar 1 gr de gluconato de calcio


CALCIO. IV diluido en 100 cc de solución fisiológica y pasar en 15 minutos. ES EL ANTÍDOTO
EN CASO DE INTOXICACIÓN POR SULFATO DE MAGNESIO.

Bibliografía:
1. PREVENCIÓN, DIAGNÓSTICO Y TRATAMIENTO DE LA PREECLAMPSIA EN EL SEGUNDO Y TERCER
NIVELES DE ATENCIÓN. EVIDENCIAS Y RECOMENDACIONES. GUÍA DE PRÁCTICA CLÍNICA. MÉXICO,
SECRETARÍA DE SALUD; 09/03/17. 2. CUNNINGHAM G, LEVENO K, BLOMM S, HAUTH J, RPUSE D,
SONG C. WILLIAMS OBSTETRICIA, 23A EDICIÓN. MC GRAW HILL. USA. 2011 EN ESPAÑOL.

http://www.cenetec-difusion.com/CMGPC/S-020-08/ER.pdf

FIN DEL CASO CLÍNICO SERIADO


ANÁLISIS DEL CASO CLÍNICO

IDENTIFICACIÓN DEL REACTIVO


Area: MEDICINA INTERNA
Especialidad: NEUMOLOGÍA
Tema: ENFERMEDADES PULMONARES OBSTRUCTIVAS Y RESTRICTIVAS
Subtema: ENFERMEDAD PULMONAR OBSTRUCTIVA CRÓNICA Y
ENFERMEDAD INTERSTICIALES

CASO CLÍNICO SERIADO

MUJER DE 77 AÑOS CON ANTECEDENTE DE TABAQUISMO INTENSO POR MÁS DE 30 AÑOS, ACUDE A
URGENCIAS POR PRESENTAR TOS CON EXPECTORACIÓN BLANQUECINA, DISNEA DE MEDIANOS
ESFUERZOS QUE CADA VEZ SE VUELVE MÁS INTENSA. REFIERE QUE ÉSTOS SÍNTOMAS SE REPITEN
DESDE HACE 4 AÑOS EN 5-6 OCASIONES POR AÑO. SE REPORTA UNA ESPIROMETRÍA CON PATRÓN
OBSTRUCTIVO NO REVERSIBLE.

MUJER DE 77 AÑOS.

CON ANTECEDENTE DE TABAQUISMO


INTENSO POR MÁS DE 30 AÑOS. FACTOR
DE RIESGO MUY IMPORTANTE.

EXPECTORACIÓN BLANQUECINA, DISNEA


DE MEDIANOS ESFUERZOS QUE CADA
VEZ SE VUELVE MÁS INTENSA. REFIERE
QUE ÉSTOS SíNTOMAS SE REPITEN DESDE
HACE 4 AÑOS EN 5-6 OCASIONES POR
AÑO.

-.

ESPIROMETRÍA CON PATRÓN


OBSTRUCTIVO NO REVERSIBLE,
"DIAGNÓSTICA".

11 - EL DIAGNOSTICO CLÍNICO MAS PROBABLE ES:


COR PULMONALE. En fases avanzadas de la ENFERMEDAD PULMONAR OBSTRUCTIVA CRÓNICA (EPOC)
encontramos insuficiencia respiratoria crónica con hipoxemia mantenida menor a 60
mmHg y/o hipercapnia mayor a 45 mmHg junto con un pH arterial normal. En esta
etapa se puede desarrollar COR PULMONALE CRÓNICO, debido a la hipertensión
pulmonar provocada por una vasoconstricción por la hipoxemia mantenida, y la
dilatación de las cavidades cardiacas derechas. Los síntomas que desarrollan son de
insuficiencia cardiaca derecha: edemas maleolares, ingurgitación yugular,
hepatomegalia dolorosa y aumento del segundo tono cardiaco. En el
electrocardiograma veremos signos de sobrecarga derecha. En la radiografía de tórax
cardiomegalia y aumento hiliar. IMPORTANTE: EL COR PULMONALE PUEDE APARECER
COMO COMPLICACIÓN DEL EPOC EN ETAPAS AVANZADAS. EL PACIENTE NO
PRESENTA DATOS QUE HAGAN SOSPECHAR ESTA COMPLICACIÓN.

EPOC. Se define a la EPOC como una ENFERMEDAD CARACTERIZADA POR LIMITACIÓN AL


FLUJO AÉREO LA CUAL NO ES TOTALMENTE REVERSIBLE Y ES USUALMENTE
PROGRESIVA. Esta limitación se asocia con una respuesta inflamatoria anormal de los
pulmones y la vía aérea cuyos factores de riesgo más importantes son la exposición a
partículas nocivas y gases, principalmente derivados del consumo de tabaco y
exposición a humo de leña. La clásica limitación crónica al flujo aéreo de la EPOC es
causada por la enfermedad de la vía aérea pequeña (bronquiolitis obstructiva) y por
la destrucción del parénquima (enfisema). La contribución relativa de cualquiera de
estas dos entidades a la limitación del flujo aéreo, varía de persona a persona.
Indicadores principales para considerar el diagnóstico de EPOC. 1. Investigar factores
de riesgo (tabaquismo, exposición al humo de leña, exposición a humo, gases, polvos
o sustancias químicas). 2. Síntomas: tos con expectoración y/o disnea. 3. Espirometría
post - broncodilatador Obstrucción FEV1 menor al 80% predicho, FEV1/FVC menor al
70% 4. Radiografía de tórax PA (sólo es útil para descartar patologías como
bronquiectasias, etc). LA PACIENTE CUMPLE CON TODOS LOS CRITERIOS PARA
CONSIDERAR EL DIAGNÓSTICO DE EPOC. EL PATRÓN OBSTRUCTIVO NO REVERSIBLE
DEBE FUNDAMENTAR TU SOSPECHA DIAGNÓSTICA.

BRONQUIECTASIAS. En las BRONQUIECTASIAS hay gran cantidad de esputo purulento, comúnmente


asociado con infecciones bacterianas; estertores gruesos a la auscultación,
acropaquias. La radiografía de tórax muestra dilataciones bronquiales y
engrosamiento de la pared bronquial. SE DESCARTA ESTA PATOLOGÍA EN ESTE
MOMENTO, AUNQUE DEBES CONSIDERAR QUE LAS BRONQUIECTASIAS TAMBIÉN
PUEDEN SER PARTE DE LAS COMPLICACIONES DE LOS PACIENTES CON EPOC.

ASMA. Bajo la denominación de EPOC no deben incluirse los procesos que, aunque cursen
con una obstrucción al flujo aéreo, son de causa conocida o tienen una anatomía
patológica específica, como ocurre con las bronquiectasias, el asma bronquial crónica,
la fibrosis quística o la bronquiolitis obliterante. Para hacer diagnóstico diferencial
deberás considerar los siguientes datos del ASMA: Inicio a temprana edad
(frecuentemente en la infancia), los síntomas varían de un día a otro, predomina por
la noche y primeras horas de la mañana, se asocia con alergia, rinitis y/o eccema.
Historia familiar de asma. Limitación del flujo aéreo principalmente reversible. EL
HECHO DE QUE NO HAYA REVERSIÓN EN LA LIMITACIÓN DEL FLUJO AÉREO
DESCARTA ESTA POSIBILIDAD DIAGNÓSTICA.

Bibliografía:
1. GUÍA DE REFERENCIA RÁPIDA, DIAGNÓSTICO Y TRATAMIENTO DE LA ENFERMEDAD PULMONAR
OBSTRUCTIVA CRÓNICA. MÉXICO: SECRETARIA DE SALUD, 2009 2. GUÍA DE PRÁCTICA CLÍNICA,
DIAGNÓSTICO Y TRATAMIENTO DE LA ENFERMEDAD PULMONAR OBSTRUCTIVA CRÓNICA.
MÉXICO: SECRETARIA DE SALUD, 2009 3. LONGO DL, FAUCI AS, KASPER DL, HAUSERSL, JAMESON
JL, LOSCALZOJ. HARRISON. PRINCIPIOS DE MEDICINA INTERNA, 18A EDICIÓN. MC GRAW HILL. NEW
YORK, USA. 2012, PP 2156-2157. 4. PAPADAKIS MAXINE A, MCPHEE STEPHEN J. DIAGNÓSTICO
CLÍNICO Y TRATAMIENTO. 52ª EDICIÓN. NUEVA YORK. 2013, PP 260-261.

http://www.cenetec.salud.gob.mx/descargas/gpc/CatalogoMaestro/037_GPC_EPOC/IMSS_037_08_EyR.pdf
12 - ADEMÁS DEL TRATAMIENTO FARMACOLÓGICO QUE DEBERÁ COMENZAR EL PACIENTE SERÁ
PRIORITARIO REALIZAR LA SIGUIENTE INTERVENCIÓN:

DEJAR DE El abordaje terapéutico del paciente con EPOC se ha dividido en 5 tipos de


FUMAR. intervenciones, las prioritarias, las convencionales, las recomendables, las cuestionables
y las que requieren discusiones personales y familiares. Las intervenciones a las que
denominamos prioritarias se refieren a aquellas que de manera trascendental modifican
la historia natural de la enfermedad y que por lo tanto, tienen un impacto en la
sobrevida de estos pacientes. Estas intervenciones son: 1) "Dejar de fumar" y/o
exponerse al humo de leña y 2) el uso de oxígeno suplementario en sujetos con
hipoxemia. No se debe iniciar ningún tratamiento diferente si no se han revisado y
satisfecho estas dos intervenciones. Es decir, si un paciente sigue fumando será de poco
valor o inútil cualquier intervención médica, pues cualquiera de ellas se verá rebasada
por la caída anual del FEV1. Asimismo, el valor de esta intervención (haber dejado de
fumar) o la del uso de broncodilatadores se reducirán si el paciente tiene hipoxemia que
no ha sido apropiadamente abordada. Las intervenciones convencionales se refieren a
las formas de abordaje farmacológico que están actualmente disponibles en el mercado
y que por convención, basada en evidencia, proporcionan beneficios clínicos al
paciente, sobre todo al disminuir la falta de aire. EL ABANDONO DEL HÁBITO
TABÁQUICO ES UNA ESTRATEGIA PRIORITARIA PRIMARIA EN EL PACIENTE CON EPOC.
RECUERDA: EL HUMO DEL TABACO ES EL PRINCIPAL DESENCADENANTE PARA EL
PROCESO INFLAMATORIO DEL EPOC.

COMENZAR UN Las intervenciones recomendables son aquellas que se sugiere debe formar parte del
PROGRAMA DE tratamiento del paciente con EPOC, sin embargo, no están disponibles en todas partes
REHABILITACIÓN ni en todos los centros hospitalarios. Por ejemplo, un programa de rehabilitación
PULMONAR. pulmonar bien estructurado solamente existe en algunos centros. Tener un programa
de rehabilitación mejora significativamente la calidad de vida del paciente con EPOC.
Sin embargo, no tenerlo, no implica necesariamente que el paciente empeore. LA
REHABILITACIÓN ES PREFERIBLE PERO NO ESTRICTAMENTE INDISPENSABLE EN EL
MANEJO DEL EPOC.

VACUNAR CON Un programa de vacunación, es también considerado parte de las intervenciones


NEUMOCOCO E recomendables. El objetivo de las vacunas en el manejo de pacientes con EPOC es el
INFLUENZA. mejor control y prevención de las exacerbaciones infecciosas. Las vacunas que se
recomiendan son las que contienen virus inactivados vivos o muertos porque son más
efectivas en pacientes con EPOC de edad avanzada. Las vacunas recomendadas son las
que están diseñadas para prevenir la infección por influenza y la infección por
Streptococcus Pneumonae. EL PROGRAMA DE VACUNACIÓN CONTRA INFLUENZA Y
NEUMOCOCO EN PACIENTE CON EPOC ES UNA ESTRATEGIA RECOMENDABLE, PERO
MÁS QUE DE PRIMERA LÍNEA, ÉSTA APLICA EN LA "PREVENCIÓN DE
COMPLICACIONES", DADO QUE SABEMOS QUE LAS INFECCIONES POR ESTOS AGENTES
SON LA PRINCIPAL CAUSA DE HOSPITALIZACIÓN.

COMENZAR La INMUNOTERAPIA oral se considera una "intervención opcional". Los agentes


INMUNOTERAPIA inmunoestimulantes representan una clase de medicamentos que contienen antígenos
ORAL. derivados de extractos de varias bacterias y que producen estimulación del componente
no específico del sistema inmune. Dentro de una gran gama de inmunoestimulantes, el
OM-85 BV, el agente con mayor número de estudios y al que se tomará como prototipo,
se encuentra constituido por 8 diferentes especies de bacterias usualmente presentes en
las vías aéreas inferiores: Haemophilus influenzae, Streptococcus Pneumonae, Klebsiella
Pneumonae y ozaenae, Staphylococcus Auereus, Streptococo Pyogenes y viridans, y
Neisseria catarrhalis. Su posible mecanismo de acción es activando macrófagos
pulmonares y aumentando la presentación del antígeno a linfocitos T. La activación de
macrófagos produce un incremento de respuesta inmune inespecífica. Al menos en un
ensayo clínico controlado, los inmunoestimulantes han demostrado una disminución en
el número de exacerbaciones, hospitalizaciones y días de estancia. Evidentemente se
requiere de mayor investigación que compruebe los hallazgos anteriores. AUNQUE
ALGUNOS ESTUDIOS DEMUESTRAN LA DISMINUCIÓN DE EXACERVACIONES,
HOSPITALIZACIÓN Y DÍAS DE ESTANCIA HOSPITALARIA NO HAY SUFICIENTE
EVIDENCIA PARA EL USO DE LA INMUNOTERAPIA ORAL.
Bibliografía:
1. GUÍA DE PRÁCTICA CLÍNICA, DIAGNÓSTICO Y TRATAMIENTO DE LA ENFERMEDAD PULMONAR
OBSTRUCTIVA CRÓNICA. MÉXICO: SECRETARIA DE SALUD, 2009. 2. LONGO DL, FAUCI AS, KASPER
DL, HAUSERSL, JAMESON JL, LOSCALZOJ. HARRISON. PRINCIPIOS DE MEDICINA INTERNA, 18A
EDICIÓN. MC GRAW HILL. NEW YORK, USA. 2012, PP 2157. 3. PAPADAKIS MAXINE A, MCPHEE
STEPHEN J. DIAGNÓSTICO CLÍNICO Y TRATAMIENTO. 52ª EDICIÓN. NUEVA YORK. 2013, PP 261.

http://www.cenetec.salud.gob.mx/descargas/gpc/CatalogoMaestro/037_GPC_EPOC/IMSS_037_08_EyR.pdf

FIN DEL CASO CLÍNICO SERIADO


ANÁLISIS DEL CASO CLÍNICO

IDENTIFICACIÓN DEL REACTIVO


Area: MEDICINA INTERNA
Especialidad: INFECTOLOGÍA
Tema: INFECCIONES PULMONARES
Subtema: NEUMONÍAS COMUNITARIAS

CASO CLÍNICO SERIADO

MUJER DE 69 AÑOS DE EDAD, EN CAMA DESDE HACE VARIOS MESES COMO CONSECUENCIA DE UNA
RESECCIÓN DE UN TUMOR CEREBRAL. DESDE HACE 1 MES PRESENTA DETERIORO PROGRESIVO DEL
ESTADO GENERAL, HIPERTERMIA NO CUANTIFICADA Y TOS PRODUCTIVA. EN LA EXPLORACIÓN FÍSICA
SE ENCUENTRA TEMPERATURA DE 38°C, CON PRESENCIA DE TAQUICARDIA, TAQUIPNEA, Y CIANOSIS
DISTAL. HIGIENE DENTAL DEFICIENTE Y HALITOSIS. REFLEJO NAUSEOSO AUSENTE, SE AUSCULTA
SOPLO TUBÁRICO Y ANFÓRICO EN HEMITÓRAX DERECHO.

Femenino de 69 años de edad.

Deterioro de su estado general secundario


a resección de tumor cerebral.

Tos y ebre.

Higiene dental de ciente, ausencia de


re ejo nauseoso, soplo tubÁrico y anfórico
en hemitórax derecho.

--

13 - EL DIAGNÓSTICO CLÍNICO MÁS PROBABLE ES:

BRONCONEUMONÍA Esta nomenclatura ya no se utiliza.


Y PAQUIPLEURITIS
BRONQUIECTASIAS BRONQUIECTASIAS: Dilatación irreversible de las vías respiratorias que afecta al
SOBREINFECTADAS pulmón. Pueden ser localizadas o difusas; se clasifican en cilíndrica o tubular,
varicosa o quística. Tienen origen infeccioso y no infeccioso. El patrón morfológico
de éstas determina en gran medida su etiología quedando del siguiente modo: 1.
Focales: obstrucción de vías respiratorias extrínseca (compresión parenquimatosa) o
intrínseca (tumor de la vía respiratoria, cuerpo extraño, cicatrización, atresia
bronquial). 2. Difusas: patología sistémica o infección. 3. Superiores: fibrosis quística,
radiación. 4. Inferiores: aspiración recurrente, neumopatía fibrótica en fase terminal
o infecciones recurrentes asociadas a inmunodeficiencia. Otras causas son: absceso
pulmonar, infección secundaria a un infarto pulmonar por embolismo o
embolización séptica de endocarditis de la válvula tricuspídea o flebitis supurativa.
CX: tos productiva persistente que se acompaña de expulsión de “esputo espeso y
pegajoso”, estertores y sibilancias, hipocratismo digital (no en todos los casos).
Cuando se asocia a un proceso infeccioso secundario, no siempre se presentan las
manifestaciones clásicas de fiebre y el desarrollo de exudados. RX: imagen en “vías
de tren” por dilatación de las vías respiratorias. TAC: imágenes en “vías de tren” o en
“anillo de sello”. NO JUSTIFICA LOS SOPLOS AUSCULTADOS EN LA PACIENTE,
CUANDO SE ASOCIA A INFECCIÓN SECUNDARIA NO ES TAN EVIDENTE LA
HIPERTERMIA.

NEUMONÍA GRAVE La NEUMONÍA ADQUIRIDA EN LA COMUNIDAD es la infección de los pulmones


ADQUIRIDA EN LA provocada por una gran variedad de microorganismos adquiridos fuera del ámbito
COMUNIDAD hospitalario y que determinan la inflamación del parénquima pulmonar y de los
espacios alveolares. Se presenta en una persona no hospitalizada o dentro de las
primeras 24 a 48 hrs de su internamiento. Generalmente se presenta en pacientes de
edad avanzada, que tienen enfermedades pulmonares o insuficiencia cardíaca
significativa, o en aquellas personas en las que se retrasa la administración de
antibióticos o se prescriben incorrectamente. CX: se asocia con fiebre mayor a 37.8°C,
polipnea (>25x’), producción de esputo durante todo el día, mialgias, sudoración
nocturna en ausencia de inflamación e irritación orofaríngea o rinorrrea, a la
exploración se auscultan estertores. AUNQUE TIENE FACTORES DE RIESGO, NO
JUSTIFICA LOS SOPLOS AUSCULTADOS.

ABSCESO El ABSCESO PULMONAR se refiere a la infección microbiana del pulmón que provoca
PULMONAR necrosis parenquimatosa pulmonar. La mayoría de los abscesos pulmonares son
polimicrobianos con flora anaerobia presente en la boca. CX: El paciente típico es
aquel que broncoaspira debido a "alteraciones en el estado de alerta", además de
tener “pobre higiene bucal con gingivitis”. Se presenta una manera subaguda (días o
semanas), con síntomas que pueden estar presentes por varias semanas
caracterizados por astenia, tos, expectoración y fiebre. La expectoración es de olor
pútrido por la presencia de anaerobios. El "soplo anfórico" es característico de
cavidades con gas en el pulmón (como es el caso del absceso pulmonar) o en caso de
neumotórax. LA EVOLUCIÓN SUBAGUDA DE LA PACIENTE, LAS CARACTERÍSTICAS
DE SU ESTADO BUCAL, Y LA AUSCULTACIÓN DE SOPLO ANFÓRICO SUGIEREN ESTE
DIAGNÓSTICO.

Bibliografía:
1. LONGO DL, FAUCI AS, KASPER DL, HAUSERSL, JAMESON JL, LOSCALZOJ. HARRISON. PRINCIPIOS
DE MEDICINA INTERNA, 18A EDICIÓN. MC GRAW HILL. NEW YORK, USA. 2012, PP 2144-2146. 2. GUÍA
DE PRÁCTICA CLÍNICA, PREVENCIÓN, DIAGNÓSTICO Y TRATAMIENTO DE LA NEUMONÍA
ADQUIRIDA EN LA COMUNIDAD EN ADULTOS. MÉXICO: SECRETARÍA DE SALUD, 2009.

http://www.cenetec.salud.gob.mx/descargas/gpc/CatalogoMaestro/234_IMSS_09_Neumonia_comunidad_adultos/ER_IMSS_234_9.pdf

14 - EL TRATAMIENTO DE ELECCIÓN PARA ESTA PACIENTE CONSISTE EN:


PRACTICAR Este tipo de tratamiento solo es necesario para un 10 a 15 % de pacientes con absceso
RESECCIÓN pulmonar y está reservado para aquellos casos que no responden al tratamiento
QUIRÚRGICA antimicrobiano. Actualmente se plantea, incluso antes de la cirugía, drenaje guiado por
tomografía. LA RESECCIÓN QUIRÚRGICA NO ES EL MANEJO DE PRIMERA ELECCIÓN, SE
RESERVA A CASOS QUE NO RESPONDEN AL TRATAMIENTO, O EN LOS QUE HAY
SOSPECHA DE HEMORRAGIA O NEOPLASIA.

ADMINISTRAR LA ANTIBIOTICOTERAPIA ES EL TRATAMIENTO DE PRIMERA ELECCIÓN PARA EL


CEFTRIAXONA MANEJO DEL ABSCESO PULMONAR. Se considera que casi todos los abscesos
MÁS inespecíficos se deben a anaerobios, sin embargo teniendo en cuenta que la paciente
METRONIDAZOL fue sometida a cirugía previa que le condicionó estancia hospitalaria y además un
deterioro inmunológico, es muy probable que se trata de una infección mixta. Aunque
los gérmenes son anaerobios el metronidazol no tiene cobertura para algunos cocos
anaerobios, por lo que se debe administrar junto con algún antibiótico que los cubra
como la ceftriaxona o amoxicilina con ácido clavulánico. No se recomienda ya el uso de
penicilina debido a la gran resistencia de estos microorganismos.

ADMINISTRAR Los macrólidos en infecciones pulmonares están indicados en neumonía atípica. En caso
CLARITROMICINA de no poder utilizarse penicilina, algún derivado de penicilina o cefalosporinas se
sugiere administrar una fluoroquinolona. La CLARITROMICINA resuelve bien el caso de
los anaerobios para el manejo del absceso pulmonar simple, pero no así la asociación a
otras bacterias.

ADMINISTRAR No esta recomendada debido a la gran resistencia a este antibiótico.


TETRACICLINA

Bibliografía:
1. LONGO DL, FAUCI AS, KASPER DL, HAUSERSL, JAMESON JL, LOSCALZOJ. HARRISON. PRINCIPIOS
DE MEDICINA INTERNA, 18A EDICIÓN. MC GRAW HILL. NEW YORK, USA. 2012, PP 2144-2146.

FIN DEL CASO CLÍNICO SERIADO


ANÁLISIS DEL CASO CLÍNICO

IDENTIFICACIÓN DEL REACTIVO


Area: MEDICINA INTERNA
Especialidad: ENDOCRINOLOGÍA
Tema: TRANSTORNOS DEL METABOLISMO DEL HUESO
Subtema: OSTEOPOROSIS

CASO CLÍNICO CON UNA PREGUNTA

MUJER DE 48 AÑOS CON ÚLTIMA MENSTRUACIÓN HACE 6 MESES.

femenino de 48 años.

ultima menstruación hace 6 meses.

-.

-.

-.

15 - LA PRESENCIA DEL SIGUIENTE MECANISMO, AUMENTA EL RIESGO DE OSTEOPOROSIS POR LA


DISMINUCIÓN DE ESTRÓGENOS EN ESTA PACIENTE.

DISMINUCIÓN DE LA La OSTEOPOROSIS es la enfermedad esquelética crónica y progresiva


CONVERSIÓN DEL 17- caracterizada por masa ósea baja, deterioro de la microarquitectura del tejido
HIDROXICOLECALCIFEROL. óseo, diminución de la fortaleza del hueso, fragilidad ósea y consecuente
incremento del riesgo de fractura. El metabolismo de la vitamina D no esta
relacionado con el mecanismo por el cual la deficiencia de estrógenos
producen osteoporosis. En el caso de deficiencia de vitamina D se produce en
los niños una enfermedad llamada raquitismo y en los adultos osteomalacia. EL
17-HIDROXICOLECALCIFERO ES UN METABOLITO PRODUCTO DEL
METABOLISMO DE LA VITAMINA D. SU DEFICIENCIA NO GUARDA RELACIÓN
CON LA BAJA ESTROGÉNICA QUE OCURRE EN LA MENOPAUSIA.
AUMENTO DE LA La DEFICIENCIA DE ESTRÓGENOS provoca pérdida de la masa ósea por dos
RESORCIÓN ÓSEA. mecanismos diferentes. 1. Activación de nuevos sitios de remodelación ósea y
2. Exageración del desequilibrio entre la formación y la resorción ósea. Los
cambios en la frecuencia de activación solo producen una pérdida transitoria
de hueso ya que posteriormente se alcanza un nuevo equilibrio entre la
formación y la resorción de hueso. Sin embargo el desequilibrio que se
produce en la remodelación produce una disminución permanente de la masa.
El efecto neto de la deficiencia de estrógenos es por un INCREMENTO DEL
RECLUTAMIENTO DE OSTEOCLASTOS Y CON ELLO DE SU ACTIVIDAD.

DISMINUCIÓN DE LA Las pacientes con menopausia independientemente de su causa tienen un


ACTIVIDAD DE LOS AUMENTO EN LA ACTIVIDAD DE LOS OSTEOCLASTOS. No una disminución por
OSTEOCLASTOS lo que esta no es la respuesta correcta.

DISMINUCIÓN DE LA Puede haber alteraciones en la masa ósea como consecuencia de una ingesta
ABSORCIÓN DE CALCIO insuficiente de calcio y esto aumenta el riesgo de osteoporosis, pero la
absorción de calcio no interfiere con el mecanismo por el cual los estrógenos
contribuyen al desarrollo de osteoporosis.

Bibliografía:
1. LONGO DL, FAUCI AS, KASPER DL, HAUSERSL, JAMESON JL, LOSCALZOJ. HARRISON. PRINCIPIOS
DE MEDICINA INTERNA, 18A EDICIÓN. MC GRAW HILL. NEW YORK, USA. 2012, PP 3123-3124. 2. GUÍA
DE PRÁCTICA CLÍNICA. DIAGNÓSTICO Y TRATAMIENTO DE OSTEOPOROSIS EN MUJERES
POSMENOPÁUSICAS. MÉXICO: SECRETARIA DE SALUD; 2013.

http://www.cenetec.salud.gob.mx/descargas/gpc/CatalogoMaestro/IMSS_673_13_Osteoporosisenpostmenopausia/673GER.pdf
ANÁLISIS DEL CASO CLÍNICO

IDENTIFICACIÓN DEL REACTIVO


Area: CIRUGÍA
Especialidad: TRAUMATOLOGÍA Y ORTOPEDIA
Tema: CERVICALGIAS, LUMBALGIAS, TRAUMA MIEMBROS SUP Y
TUMORES
Subtema: LESIONES DE HOMBRO

CASO CLÍNICO CON UNA PREGUNTA

FEMENINA DE 67 AÑOS DE EDAD, PRESENTO CONTUSIÓN DIRECTA EN LA ARTICULACIÓN


GLENOHUMERAL DERECHA, HA PRESENTADO HIPOSENSIBILIDAD EN EL MÚSCULO DELTOIDES.

contusiÓn en hombro.

hiposensibilidad en la regiÓn del deltoides.

16 - POR LOS DATOS CLINICOS USTED SOSPECHA QUE SE HAYA LESIONADO EL:

EL NERVIO El NERVIO RADIAL es un nervio en el cuerpo humano que suple terminaciones nerviosas a
RADIAL. músculos del BRAZO, ANTEBRAZO, MUÑECA Y MANO, así como la sensación cutánea del
dorso de la mano. Procede del fascículo posterior del plexo braquial con fibras nerviosas
procedentes de las raíces espinales C5, C6, C7, C8 y T1. El nervio radial se divide en una
rama profunda, que se convierte en el nervio interóseo posterior y continúa volviéndose
superficial para inervar el dorso de la mano.
EL NERVIO Debido al antecedente de la lesión especifica en el hombro y a la sintomatología de
CIRCUNFLEJO. hiposensibilidad se sospechará de este nervio lesionado. Para realizar el diagnóstico
diferencial, se deberá conocer por lo menos la región inervada por los diferentes nervios
mencionados. El NERVIO AXIAL O CIRCUNFLEJO es un nervio raquídeo mixto que
"pertenece al plexo braquial". Nace del tronco secundario posterior con el nervio radial, en
contacto con el músculo subescapular. Es un nervio que va destinado al HOMBRO, no se
dirige al brazo y pasa entre el deltoides y el húmero rodeando el cuello de éste junto a la
arteria circunfleja posterior. Colaterales Rama motriz para el músculo redondo menor en el
espacio cuadrilátero de Velpeau (nervio del redondo menor). Ramos sensitivos superficiales
para el muñón del hombro. Terminales Termina dando terminales motoras al deltoides por
su cara interna.

EL NERVIO El NERVIO MEDIANO es un nervio raquídeo mixto proveniente del plexo braquial. Nace de
MEDIANO. dos raíces, una del fascículo lateral, y otra del fascículo medial (C5,C6,C7,C8,T1) que forman
una V entre las cuales discurre la arteria axilar.1 Desciende por el borde interno del brazo
junto a la arteria axilar. Cuando llega al compartimento anterior del brazo se sitúa aplicado
a la arteria braquial. En la MUÑECA se sitúa entre los tendones de los músculos palmar
mayor y palmar menor, pasa por debajo del ligamento anular del carpo y se sitúa por
debajo de la eminencia tenar. El nervio mediano permite movilidad y sensibilidad fina con
mucha rapidez y permite la oposición del pulgar.

EL PLEXO Es como tal el origen de los nervios anteriores y cuando éste se afecta en su totalidad, hay
BRAQUIAL. sintomatología no localizada si no generalizada a todo el miembro torácico. RECUERDA:
CUANTO MÁS PROXIMAL SEA LA INCAPACIDAD Y DOLOR DEBERÁS PENSAR EN LESIÓN
DEL PLEXO BRAQUIAL. DE ESTE PLEXO EL NERVIO LESIONADO ESPECÍFICAMENTE PARA EL
ÁREA MENCIONADA ES EL NERVIO CIRCUNFLEJO.

Bibliografía:
1. BRUNICARDI F, ANDERSEN D, BILLIAR T, Y COLS. SCHWARTZ PRINCIPIOS DE CIRUGÍA, 9A
EDICIÓN. MC GRAW HILL. 2011, PP 1581. 2. DOHERTY G. DIAGNÓSTICO Y TRATAMIENTO QUIRÚRGICO,
13A EDICIÓN. MC GRAW HILL LANGE. 2011, PP 1334-1335. 3. PROGRAMA AVANZADO DE APOYO VITAL
EN TRAUMA PARA MÉDICOS. ATLS. COMITÉ DE TRAUMA DEL COLEGIO AMERICANO DE
CIRUJANOS. ESTADOS UNIDOS. 8ª EDICIÓN.
ANÁLISIS DEL CASO CLÍNICO

IDENTIFICACIÓN DEL REACTIVO


Area: GINECOLOGÍA Y OBSTETRICIA
Especialidad: GINECOLOGÍA
Tema: PATOLOGÍA BENIGNA Y MALIGNA DE OVARIO
Subtema: PATOLOGÍA BENIGNA DE OVARIO

CASO CLÍNICO SERIADO

MUJER DE 24 AÑOS DE EDAD, FUR HACE 20 DÍAS, IVSA DESDE HACE 2 AÑOS SIN MÉTODO DE
PLANIFICACIÓN FAMILIAR. ACUDE A URGENCIAS AL PRESENTAR DOLOR ABDOMINAL DESDE HACE 8
HRS, QUE SE PRESENTA SÚBITAMENTE AL ENCONTRARSE SALTANDO EN UN BRINCOLÍN. A LA
EXPLORACIÓN SE ENCUENTRA CON TEMP. 37.5 GRADOS, FACIES DOLOROSA, ABDOMEN CON
RESISTENCIA MUSCULAR, DOLOROSO A LA PALPACIÓN MEDIA Y PROFUNDA EN EL CUADRANTE
INFERIOR DERECHO. REBOTE NEGATIVO. EL TACTO VAGINAL CON CÉRVIX POSTERIOR, CERRADO,
FORMADO. NO SE PUEDE EXPLORAR ÚTERO Y ANEXOS POR RESISTENCIA MUSCULAR. NO HAY
SANGRADO VAGINAL.

24 años de edad, padecimiento más


común en pacientes jóvenes.

Es importante considerar los siguientes


factores de riesgo. 1. Una trompa larga y
ausencia mesosalpinx, más frecuente en la
adolescencia. 2. Afecta más el ovario
derecho que el izquierdo en un 60 %
porque este último es jado por el
sigmoides e impide su movimiento. 3. El
ejercicio puede desencadenar la torsión del
ovario en presencia de los factores
anteriores.

los síntomas claves son: dolor súbito


durante el ejercicio, persistente y su
localización en fosa ilíaca derecha.

abdomen doloroso con resistencia


muscular pero con rebote negativo. a la
exploración de cuello cervical sin datos de
dolor a la movilización

-.
17 - EL EVENTO FISIOPATOLÓGICO QUE MÁS PROBABLEMENTE CONDICIONA EL DOLOR EN EL
MOMENTO DE LA VALORACIÓN DE LA PACIENTE ES:

PROCESO LA TORSIÓN DE LOS COMPONENTES DE LOS ANEXOS CONDICIONA UN ACCIDENTE


ISQUÉMICO. VASCULAR DEL PEDÍCULO. La TORSIÓN DE ANEXOS es común durante la edad fértil con
un pico mayor entre los 20 a 39 años, esta condición también se pude observar, aunque
con menor frecuencia en mujeres postmenopáusicas y durante el embarazo (20-25% de
los casos de torción). Durante la torsión, los anexos (ovario y trompa) rotan con mayor
frecuencia alrededor del ligamento ancho. Si bien se pueden torcer los anexos sanos, el
50 a 80% de los casos se asocia con presencia de tumor ovárico unilateral. La presencia de
ligamentos largos favorece la movilidad de las trompas y con ello el riesgo de torsión. Así
mismo los ovarios crecidos de tamaño, independientemente de la causa, favorecen
también esta patología. COMPROMISO VASCULAR: Cuando la torsión es incompleta
puede quedar conservado el flujo arterial y venoso, entonces el ovario es viable, pero
ante una torsión completa y persistente, en estadio inicial, como la presión venosa es
menor que la presión arterial continúa el riego arterial estando el flujo venoso obstruido.
El flujo arterial en un inicio puede estar conservado porque tiene mayor presión arterial y
porque el ovario recibe una doble circulación de la arteria ovárica y de la arteria uterina,
que en ausencia de flujo en una, el riego sanguíneo lo mantiene la otra. En estas
condiciones el ovario no es viable. En estadio tardío de la torsión completa se afectan el
flujo arterial y el flujo venoso condicionando el edema, el aumento de volumen del
ovario y la necrosis hemorrágica. En ocho horas se condicionan compromiso vascular del
ovario, lo que sin duda condiciona el dolor. El cuadro clínico se caracteriza por dolor
agudo en los cuadrantes inferiores del abdomen, de inicio repentino y progresivo;
circunscrito al lado afectado e irradiado a flanco y muslo del mismo lado. Cuando hay
necrosis suele aparecer febrícula. Algunas pacientes pueden presentar náuseas y vómito.

PROCESO Si bien en un comienzo la torsión condiciona un proceso inflamatorio, y este explicar el


INFLAMATORIO. dolor, las 8 hrs de evolución al momento de la exploración, explican que la opción más
correcta sea el proceso isquémico.

DISTRIBUCIÓN La localización del dolor podría ser explicada por la inervación, sin embargo el dolor es
NERVIOSA DEL producto de un proceso isquémico que se manifiesta a través de la inervación de la zona.
DERMATOMA.

IRRITACIÓN La paciente a la exploración no presenta datos de irritación peritoneal, el rebote es


PERITONEAL. negativo. Puede en este momento descartarse. Si la paciente tuviera abdomen en
madera, ausencia de reflejos abdominales, hiperestesias, hiperbaralgesias y rebote
positivo esta sería la respuesta correcta.

Bibliografía:
1. SCHONGUE J, SCHAFER J, HALVORSON L, HOFFMAN B, BRADSHAW K, CUNNINGHAM G.
WILLIAMS GINECOLOGÍA, DE LA 1A EDICIÓN EN INGLÉS. MC GRAW HILL. USA. 2009, PP 215-216.

18 - EL ESTUDIO MÁS ÚTIL QUE DEBERÁ INDICAR PARA CONFIRMAR SU DIAGNÓSTICO SERÁ:

BIOMETRÍA Si bien debe solicitarse, sus resultados no descartan o apoyan este diagnóstico, su
HEMÁTICA utilidad sería para descartar o apoyar un PROCESO APENDICULAR como diagnóstico
COMPLETA. diferencial.
TOMOGRAFÍA La tomografía computarizada (TAC) y la resonancia magnética (RM) también son
ABDOMINAL. útiles en los casos complicados o en los que se acompaña de un cuadro clínico
ambiguo como el que se observa en la torsión incompleta o crónica. NO
CONSTITUYEN EL ESTUDIO DE PRIMERA ELECCIÓN, SÓLO ESTÁN INDICADOS
CUANDO EXISTE DUDA DIAGNÓSTICA O EN CASOS ESPECIALES QUE SE ASOCIE
OTRA PATOLOGÍA.

RADIOGRAFÍA No nos aporta datos para poder apoyar y mucho menos confirmar nuestra sospecha
SIMPLE DE diagnóstica. Aún que sea muy probable que se solicite, en el caso de tratarse de una
ABDOMEN DE PIE Y torsión ovárica, prácticamente no tendría ninguna utilidad.
EN DECÚBITO.

ULTRASONOGRAFÍA Con base en el cuadro clínico es muy claro que se trata de una TORSIÓN OVÁRICA.
ENDOVAGINAL. Este diagnóstico se fundamenta en: • La edad, comienza súbitamente durante un
evento de ejercicio (brincolin), localización del dolor y eventos acompañantes. • El
diagnóstico diferencial principal sería con apendicitis aguda, sin embargo, ni el dolor
ni los hallazgos en la exploración apoyarían este diagnóstico. Recuerda que esta
patología debe tenerse presente cuando hay dolor bajo vientre de instalación brusca,
náuseas, vómitos, fiebre, tumor palpable, leucocitosis y evidencia sonográfica de una
masa en hipogastrio. El estudio más importante y que permite la confirmación en la
mayoría de los casos es la ultrasonografía Doppler que evidencia el grado de
compromiso vascular del ovario. Desde el punto de vista ecográfico deberá
diferenciarse entre embarazo ectópico, absceso tuboovárico, quiste ovárico
hemorrágico o endometrioma. Además de que la torsión de anexos puede
acompañar a cualquiera de estas patologías. Los hallazgos ultrasonográficos de
anexo torcido incluyen: presencia de folículos múltiples en el borde de un ovario
hipertrófico, congestión y edema ovárico, pedículo torcido con apariencia de ojo de
buey, un remolino o una concha de caracol. La ultrasonografía transvaginal con
Doppler color otorga más datos que la ecografía simple en estos casos, muestra
desde irrigación desordenada hacia los anexos hasta ausencia de flujo ovárico.
RECUERDA: el diagnóstico debe sospecharse por clínica, y se refuerza con el
ultrasonido, el diagnóstico definitivo se hará mediante laparotomía o laparoscopía.
EL MÉTODO DE ELECCIÓN EN ESTE CASO ES LA ULTRASONOGRAFÍA ENDOVAGINAL
DOPPLER.

Bibliografía:
1. SCHONGUE J, SCHAFER J, HALVORSON L, HOFFMAN B, BRADSHAW K, CUNNINGHAM G.
WILLIAMS GINECOLOGÍA, DE LA 1A EDICIÓN EN INGLÉS. MC GRAW HILL. USA. 2009, PP 216-217.

19 - EL TRATAMIENTO INDICADO EN ESTA PACIENTE SERÁ:

REPOSO Y ANALGÉSICOS. Ante el diagnóstico, no tendría sentido esta opción. Sin embargo es importante
que consideres que cuando es destorcido el ovario, no se puede tener el 100%
de certeza de que no hubiera necrosis por lo que esta indicado mantener a la
paciente en vigilancia en espera de algún dato de complicación, como fiebre,
leucocitosis o irritación peritoneal. RECUERDA QUE EN TODOS LOS CASOS DE
DOLOR ABDOMINAL SÚBITO DEBE EVITARSE EL USO DE ANALGÉSICOS PARA
NO ENMASCARAR EL CUADRO HASTA QUE SE CONFIRME EL DIAGNÓSTICO.

APENDICECTOMÍA. En determinado caso podría ser electiva si la cirugía es realizada por un


cirujano, pero en ningún caso podría considerarse una respuesta correcta. NO
CORRESPONDE AL TRATAMIENTO DE ELECCIÓN PUESTO QUE NO SE TRATA DE
UNA APENDICITIS.
SALPINGOOFORECTOMÍA. Anteriormente bajo el criterio de evitar riesgos de tromboembolismos
posterior a la destorsión del ovario, se prefería la ooforectomía o
salpingooforectomía. Sin embargo, actualmente los objetivos del tratamiento
son preservar los anexos dañados, extraer los quistes o tumores concomitantes
y realizar una ooforopexia. OJO. SI SE TUVIERA EVIDENCIA DE NECROSIS O
ROTURA DE LOS ANEXOS CON HEMORRAGIA, ESTA SERÍA LA OPCIÓN
CORRECTA.

DESTORSIÓN Y Hasta la década del 80 la anexectomía era la conducta a seguir para tratar la
OOFOROPEXIA. torsión del ovario por el temor a que la destorsión podía enviar émbolos al
torrente circulatorio, que ocasionaban tromboembolismo pulmonar y
liberación de toxinas que producían reacción anafiláctica grave, además, por la
incapacidad de determinar la viabilidad del ovario. A la luz de los
conocimientos actuales la mayoría de los autores en el mundo recomiendan
estudiar el ovario con US Doppler color, si hay evidencias de compromiso
vascular, seccionar por debajo del nivel de la torsión para prevenir el
embolismo venoso y el shock anafiláctico. Si el ovario es viable se puede
proceder a destorcer el pedículo vascular, quitar el tumor y fijar el ovario con
suturas sin necesidad de realizar ooforectomía, así se preserva la función
ovárica tan importante en la mujer en edad fértil. Después de la operación el
ovario debe estudiarse con US seriado, esto nos permite evaluar el crecimiento
y desarrollo de los folículos, su normalidad refleja que la función ovárica se ha
recuperado. A continuación se numeran los factores de riesgo completos: 1.
Una trompa larga y ausencia mesosalpinx, más frecuente en la adolescencia. 2.
El aumento del peso del ovario incrementa su movilidad sobre todo si está
asociado a un tumor, esto sucede en el 60 % de las torsiones, puede coexistir
con masas que miden de 5 a 12 cm, promedio de 8 cm, es menos frecuente en
neoplasias gigantes. Las variedades histológicas más encontradas son los
quistes funcionales, el cistoadenoma seroso, el cistoadenoma mucinoso, el
quiste dermoides, el fibroma del ovario y cistoadenofibroma seroso. En los
tumores malignos ocurre con una incidencia menor de un 6 % porque las
adherencias cancerosas fijan el ovario. 3. Afecta más el ovario derecho que el
izquierdo en un 60 % porque este último es fijado por los sigmoides e impide
su movimiento. 4. Relajación de los ligamentos suspensores del ovario. 5. En el
síndrome del ovario hiperestimulado por aumento de su peso que lo hace más
móvil, se ve con mayor frecuencia en la primera década de la vida. 6. En las
embarazadas se presenta asociado a un cuerpo lúteo grande, sucede en un 10 a
un 20 % de todas las torsiones del ovario con una incidencia de 5 por cada 10
000 embarazos, más frecuente en el primer trimestre, es raro que ocurra en el
2do y en el 3er trimestre. 7. Los tratamientos con inductores de la ovulación
favorecen la formación de los quistes tecaluteínicos que incrementan el peso
del ovario y por tanto la torsión. 8. La ligadura de la trompa aumenta el riesgo
de torsión del ovario. 9. El ejercicio puede desencadenar la torsión del ovario en
presencia de los factores anteriores. LOS OBJETIVOS DEL TRATAMIENTO DE LA
TORSIÓN DE ANEXOS ES PRESERVAR LOS ANEXOS DAÑADOS, EXTRAER LOS
QUISTES O TUMORES CONCOMITANTES (EN CASO DE HABERLOS) Y
POSIBLEMENTE REALIZAR UNA OOFOROPEXIA. Al encontrar necrosis de los
anexos o rotura con hemorragia, a menudo es necesario extirpar las estructuras
con torsión.

Bibliografía:
1. SCHONGUE J, SCHAFER J, HALVORSON L, HOFFMAN B, BRADSHAW K, CUNNINGHAM G.
WILLIAMS GINECOLOGÍA, DE LA 1A EDICIÓN EN INGLÉS. MC GRAW HILL. USA. 2009, PP 217.

FIN DEL CASO CLÍNICO SERIADO


ANÁLISIS DEL CASO CLÍNICO

IDENTIFICACIÓN DEL REACTIVO


Area: CIRUGÍA
Especialidad: TRAUMATOLOGÍA Y ORTOPEDIA
Tema: CERVICALGIAS, LUMBALGIAS, TRAUMA MIEMBROS SUP Y
TUMORES
Subtema: LESIONES DE HOMBRO

CASO CLÍNICO CON UNA PREGUNTA

FEMENINA DE 67 AÑOS DE EDAD, PRESENTO CONTUSIÓN DIRECTA EN LA ARTICULACIÓN


GLENOHUMERAL DERECHA, HA PRESENTADO HIPOSENSIBILIDAD EN EL MÚSCULO DELTOIDES.

contusiÓn en hombro.

hiposensibilidad en la regiÓn del deltoides.

16 - POR LOS DATOS CLINICOS USTED SOSPECHA QUE SE HAYA LESIONADO EL:

EL NERVIO El NERVIO RADIAL es un nervio en el cuerpo humano que suple terminaciones nerviosas a
RADIAL. músculos del BRAZO, ANTEBRAZO, MUÑECA Y MANO, así como la sensación cutánea del
dorso de la mano. Procede del fascículo posterior del plexo braquial con fibras nerviosas
procedentes de las raíces espinales C5, C6, C7, C8 y T1. El nervio radial se divide en una
rama profunda, que se convierte en el nervio interóseo posterior y continúa volviéndose
superficial para inervar el dorso de la mano.
EL NERVIO Debido al antecedente de la lesión especifica en el hombro y a la sintomatología de
CIRCUNFLEJO. hiposensibilidad se sospechará de este nervio lesionado. Para realizar el diagnóstico
diferencial, se deberá conocer por lo menos la región inervada por los diferentes nervios
mencionados. El NERVIO AXIAL O CIRCUNFLEJO es un nervio raquídeo mixto que
"pertenece al plexo braquial". Nace del tronco secundario posterior con el nervio radial, en
contacto con el músculo subescapular. Es un nervio que va destinado al HOMBRO, no se
dirige al brazo y pasa entre el deltoides y el húmero rodeando el cuello de éste junto a la
arteria circunfleja posterior. Colaterales Rama motriz para el músculo redondo menor en el
espacio cuadrilátero de Velpeau (nervio del redondo menor). Ramos sensitivos superficiales
para el muñón del hombro. Terminales Termina dando terminales motoras al deltoides por
su cara interna.

EL NERVIO El NERVIO MEDIANO es un nervio raquídeo mixto proveniente del plexo braquial. Nace de
MEDIANO. dos raíces, una del fascículo lateral, y otra del fascículo medial (C5,C6,C7,C8,T1) que forman
una V entre las cuales discurre la arteria axilar.1 Desciende por el borde interno del brazo
junto a la arteria axilar. Cuando llega al compartimento anterior del brazo se sitúa aplicado
a la arteria braquial. En la MUÑECA se sitúa entre los tendones de los músculos palmar
mayor y palmar menor, pasa por debajo del ligamento anular del carpo y se sitúa por
debajo de la eminencia tenar. El nervio mediano permite movilidad y sensibilidad fina con
mucha rapidez y permite la oposición del pulgar.

EL PLEXO Es como tal el origen de los nervios anteriores y cuando éste se afecta en su totalidad, hay
BRAQUIAL. sintomatología no localizada si no generalizada a todo el miembro torácico. RECUERDA:
CUANTO MÁS PROXIMAL SEA LA INCAPACIDAD Y DOLOR DEBERÁS PENSAR EN LESIÓN
DEL PLEXO BRAQUIAL. DE ESTE PLEXO EL NERVIO LESIONADO ESPECÍFICAMENTE PARA EL
ÁREA MENCIONADA ES EL NERVIO CIRCUNFLEJO.

Bibliografía:
1. BRUNICARDI F, ANDERSEN D, BILLIAR T, Y COLS. SCHWARTZ PRINCIPIOS DE CIRUGÍA, 9A
EDICIÓN. MC GRAW HILL. 2011, PP 1581. 2. DOHERTY G. DIAGNÓSTICO Y TRATAMIENTO QUIRÚRGICO,
13A EDICIÓN. MC GRAW HILL LANGE. 2011, PP 1334-1335. 3. PROGRAMA AVANZADO DE APOYO VITAL
EN TRAUMA PARA MÉDICOS. ATLS. COMITÉ DE TRAUMA DEL COLEGIO AMERICANO DE
CIRUJANOS. ESTADOS UNIDOS. 8ª EDICIÓN.
ANÁLISIS DEL CASO CLÍNICO

IDENTIFICACIÓN DEL REACTIVO


Area: PEDIATRÍA
Especialidad: INFECTOLOGIA PEDIÁTRICA
Tema: HEPATITIS INFECCIOSA
Subtema: HEPATITIS B

CASO CLÍNICO SERIADO

ADOLESCENTE DE 16 AÑOS DE EDAD, TIENE EL ANTECEDENTE DE HACE 8 MESES CON CUADRO


CLÍNICO DE HEPATITIS. ACUDE A CONSULTA EXTERNA DE PEDIATRÍA POR PRESENTAR NUEVAMENTE
ICTERICIA GENERALIZADA, ASTENIA, ADINAMIA E HIPOREXIA. A LA EXPLORACIÓN CON ATAQUE AL
ESTADO GENERAL, ICTERICIA GENERALIZADA MODERADA, ASÍ COMO HEPATALGIA. DURANTE EL
INTERROGATORIO, EL PACIENTE ACEPTA SER FARMACODEPENDIENTE DE HEROÍNA INTRAVENOSA.

Adolescente de 16 años de edad

Cuadro clínico de hepatitis hace 8 meses.


Farmacodependiente a drogas
intravenosas

Cuadro clínico compatible con hepatitis


por segunda vez en un año

20 - LA PACIENTE REQUIERE DE ESTUDIOS PARA CONFIRMAR EL DIAGNOSTICO. ¿QUÉ ESTUDIOS


LE SOLICITARÍAS?:
TOMOGRAFÍA Hepatitis E. MANIFESTACIONES CLÍNICAS: La infección por hepatitis E es una enfermedad
COMPUTADA aguda cuyos síntomas incluyen ictericia, malestar general, anorexia, fiebre, dolor
Y SEROLOGÍA abdominal y artralgias, también se observan casos de infección subclínica. CAUSAS: El
PARA virus de hepatitis E (Hepatitis E virus, HEV) es un virus de RNA de filamento positivo, sin
HEPATITIS E cubierta y esférico, y es el único agente conocido de hepatitis transmitida por mecanismos
entéricos no-A no-B. Anteriormente se clasificaba el virus de hepatitis E dentro de la
familia Caliciviridae, género Calicivirus; sin embargo, se ha hecho una reasignación para
colocarlo dentro del género “No asignado” de virus similares a hepatitis E, porque HEV
posee algunas características que lo diferencian de los Calicivirus típicos. ASPECTOS
EPIDEMIOLÓGICOS: El contagio del virus se hace por la vía fecal-oral. La enfermedad es
más común en adultos que en niños y tiene un índice extraordinariamente alto de letalidad
en embarazadas. Se han notificado casos en epidemias o de manera esporádica en zonas
de Asia, África y México. Los brotes por lo común dependieron de la ingestión de agua
contaminada. En Estados Unidos de América rara vez se ha notificado la infección por el
virus de hepatitis E, y casi todos los casos publicados se han observado en viajeros que van
a regiones endémicas. Sin embargo, se han señalado casos de infección aguda por este
virus, corroborada al aislar la “Cepa Estadounidense” de él en personas sin el antecedente
reciente de haber viajado fuera de dicho país. La identificación de un virus porcino en
Estados Unidos muy similar a HEV humano plantea la posibilidad de que exista un
reservorio zoonótico del virus. Se desconoce el periodo de transmisibilidad después de
infección aguda, pero durante dos semanas comúnmente hay dispersión del virus por las
heces y viremia. Al parecer no se produce la infección crónica. AL NO GENERAR UN
CUADRO CRÓNICO, LA HEPATITIS E QUEDA DESCARTADA EN AUTOMÁTICO COMO
OPCIÓN DIAGNÓSTICA.

PRUEBAS DE Hepatitis C. MANIFESTACIONES CLÍNICAS: Los signos y los síntomas de la infección por el
FUNCIÓN virus de hepatitis C (Hepatitis C virus, HCV) son prácticamente idénticos a los de las
HEPÁTICA Y hepatitis A o B. El cuadro agudo tiende a ser menos intenso y su comienzo es más
SEROLOGÍA insidioso, y casi todas las infecciones son asintomáticas (80% según datos de la OMS). Se
PARA observa ictericia en menos de 20 % de los pacientes, y las anormalidades en las pruebas de
HEPATITIS C función hepática por lo común son menos intensas que las que surgen en personas con
infección por virus de hepatitis B. La infección persistente con HCV se observa en 50 a 60
% de los niños infectados, incluso sin que haya pruebas bioquímicas de ataque del hígado.
Casi todos los niños con infección crónica son asintomáticos. A pesar de que la hepatitis
crónica aparece en 60 a 70 % de los adultos infectados, algunos datos indican que menos
de 10 % de los niños infectados terminan por mostrar hepatitis crónica, y que menos de 5
% muestran al final cirrosis. La infección por HCV es la principal razón para la práctica de
trasplante de hígado en adultos en Estados Unidos de América. LA HEPATITIS C DA
CUADROS AGUDOS POCO O NADA SINTOMÁTICOS, LO CUAL NO CONCUERDA CON ESTE
CUADRO DONDE LA CLÍNICA ES MUY MARCADA.

PRUEBAS DE Hepatitis B. MANIFESTACIONES CLÍNICAS: Las personas infectadas del virus de hepatitis B
FUNCIÓN (Hepatitis B virus, HBV) pueden mostrar signos y síntomas variados, que incluyen un
HEPÁTICA Y cuadro subagudo con manifestaciones inespecíficas (Anorexia, náuseas o malestar
SEROLOGÍA generalizado), hepatitis clínica con ictericia o la forma letal fulminante. Es frecuente que
PARA haya seroconversión asintomática, y la posibilidad de que surjan los síntomas de la
HEPATITIS B enfermedad depende de la edad de la persona. En niños de corta edad es más frecuente la
infección anictérica o asintomática. En los comienzos de la evolución pueden surgir
manifestaciones extrahepáticas, como artralgias, artritis, erupciones maculosas,
trombocitopenia o acrodermatitis papulosa (Síndrome de Gianotti-Crosti), antes de que
surja la ictericia. Es difícil distinguir entre la hepatitis B aguda y otras formas de hepatitis
aguda por virus con base sólo en los signos y síntomas clínicos o datos inespecíficos de
estudios de laboratorio. La infección crónica por HBV se define como la presencia del
antígeno de superficie de hepatitis B (hepatitis B surface antigen, HBsAg) en el suero
durante seis meses, como mínimo, o por la presencia de dicho antígeno en una persona
que no ha generado anticuerpos de la subclase IgM al antígeno central de hepatitis B
(hepatitis B core antigen, anti-HBc). EL CUADRO CLÍNICO RECIDIVANTE O DE LARGA
DURACIÓN BASTA PARA DESCARTAR HEPATITIS C Y, CONSIDERAR B COMO EL
DIAGNÓSTICO MÁS PROBABLE. ADEMÁS, EN HEPATITIS C EL CUADRO ES MUY LEVE O
NULO, A DIFERENCIA DE LA HEPATITIS B DONDE LA SINTOMATOLOGÍA ES MARCADA.
ULTRASONIDO Hepatitis A. MANIFESTACIONES CLÍNICAS: De manera característica, la infección por el
HEPÁTICO Y virus de hepatitis A (Hepatitis A virus, HAV) incluye un cuadro agudo que cede por sí solo,
SEROLOGÍA compuesto de fiebre, malestar generalizado, ictericia, anorexia y náuseas. La infección
PARA sintomática por dicho virus se observa en cerca de 30% de niños infectados que tienen
HEPATITIS A menos de seis años de vida; pocos de ellos mostrarán ictericia. En niños de mayor edad y
adultos, la infección casi siempre es sintomática y se prolonga algunas semanas; en 70 %
de los casos, en promedio, aparece ictericia. Se observan a veces casos de enfermedad
prolongada o recidivante que puede durar incluso seis meses. La hepatitis fulminante es
rara, pero es más frecuente en sujetos con alguna hepatopatía primaria. No se observa
infección crónica. LA HEPATITIS A SE MANIFIESTA COMO UN CUADRO AGUDO, DE TAL
MANERA QUE NO COINCIDE CON EL TIEMPO DE EVOLUCIÓN DEL CASO.

Bibliografía:
PRINCIPLES AND PRACTICE OF PEDIATRIC INFECTIOUS DISEASES. SARAH S. LONG. CHURCHILL,
LIVINGSTONE. EDICIÓN 2A. 2002. PÁG. 1188-1192.

21 - LO MÁS PROBABLE ES QUE LOS RESULTADOS DE LOS ESTUDIOS REALIZADOS A NUESTRA


PACIENTE REPORTEN TRANSAMINASAS ELEVADAS CON SEROLOGÍA POSITIVA PARA:

INFECCIÓN HEPATITIS B. MÉTODOS DIAGNÓSTICOS. La infección crónica por virus de la hepatitis B (HBV)
POR se define como la presencia del antígeno de superficie de hepatitis B (Hepatitis B surface
HEPATITIS antigen, HBsAg) en el suero durante seis meses, como mínimo, o por la presencia de dicho
B antígeno en una persona que no ha generado anticuerpos de la subclase IgM al antígeno
central de hepatitis B (Hepatitis B core antigen, anti-HBc). Las personas con infección crónica
tienen en su circulación el antígeno de superficie y el anticuerpo contra el antígeno central; en
raras ocasiones también hay anticuerpo contra el antígeno de superficie. Los anticuerpos
contra los antígenos de superficie (s) y central (c) se detectan en personas cuya infección no ha
mostrado resolución, en tanto que el anticuerpo contra el antígeno s sólo aparece en
individuos que han recibido la vacuna contra hepatitis B.

INFECCIÓN HEPATITIS E. MÉTODOS DIAGNÓSTICOS: El diagnóstico de infección aguda por virus de la


POR hepatitis E (HEV) se hace al detectar el anticuerpo inmunoglobulina (Ig) M contra HEV (anti-
HEPATITIS HEV) en suero, o al detectar el RNA de la partícula por reacción en cadena de polimerasa-
E transcriptasa inversa en muestras de suero o excremento. En laboratorios de investigación y
comerciales se practican los métodos basados en la reacción en cadena de polimerasa-
transcriptasa inversa y también los serológicos, para el diagnóstico de infección aguda por
HEV. Sin embargo, ninguno de tales procedimientos ha recibido la aprobación de la Food and
Drug Administration para tal finalidad. Los criterios de los CDC para considerar si es necesario
someter a prueba una muestra de suero de fase aguda para buscar signos de infección por HEV
incluyen un cuadro clínico de comienzo poco preciso con ictericia o una concentración de
transaminasa de alanina sérica cuando menos 2.5 veces mayor que el límite superior de los
resultados normales y negativos para búsqueda de anticuerpo IgM contra el virus de hepatitis
A, anticuerpo IgM contra el antígeno central de hepatitis B y el anticuerpo contra el virus de
hepatitis C. TRATAMIENTO: De sostén.
INFECCIÓN HEPATITIS C. MÉTODOS DIAGNÓSTICOS: Los dos tipos principales de métodos para el
POR diagnóstico de laboratorio de infecciones por virus de la hepatitis C (HCV) son las mediciones
HEPATITIS de anticuerpos en busca de anticuerpo contra HCV y estudios de ácido nucleico (Nucleic acid
C tests, NAT) para detectar RNA de HCV. El diagnóstico por medio de los títulos de anticuerpos
comprende la práctica de un inmunoanálisis enzimático inicial; la repetición de los resultados
positivos se confirma por medio de inmunotransferencia recombinante (Recombinant
immunoblot assay, RIBA). Los dos métodos detectan el anticuerpo de inmunoglobulina (Ig) G;
no se cuenta con métodos para valorar IgM. Los inmunoanálisis enzimáticos actuales tienen
una sensibilidad mínima de 97 % y especificidad mayor de 99 %. Los NAT diagnósticos
aprobados en Estados Unidos de América por la Food and Drug Administration (FDA) para la
detección cualitativa de HCV utilizan la reacción en cadena de polimerasa-transcriptasa inversa
(Reverse transcriptase-polymerase chain reaction, RT-PCR). Es posible detectar el RNA del
virus de hepatitis C en suero o plasma, en término de una a dos semanas después de
exposición al virus y semanas antes de que comiencen las anormalidades en las enzimas
hepáticas o la aparición de anticuerpo contra HCV. En la práctica clínica se utilizan
comúnmente los métodos para identificar RNA de HCV, como la cadena de polimerasa-
transcriptasa inversa, en el diagnóstico inicial de la infección, para identificar la infección en
lactantes desde los comienzos de la vida (transmisión perinatal), momento en que los
anticuerpos séricos de la madre interfieren en la capacidad de detectar los anticuerpos
producidos por el lactante, y para vigilar a pacientes que reciben antivíricos. Sin embargo,
surgen resultados falsos positivos y negativos por el manejo y almacenamiento inadecuados y
por la contaminación de las muestras de prueba. El RNA vírico puede detectarse en forma
intermitente, por lo que un solo método con resultados negativos no es concluyente. Están a la
disposición técnicas cuantitativas para medir la concentración de RNA de HCV. La utilidad
clínica de estos métodos cuantitativos al parecer consiste más bien en que sirven como
indicadores pronósticos de sujetos que reciben antivíricos o que están a punto de recibirlos.

INFECCIÓN HEPATITIS A. MÉTODOS DIAGNÓSTICOS: En el comercio se cuenta con pruebas serológicas


POR para detectar anticuerpos de inmunoglobulina IgM totales específicos contra HAV. La IgM
HEPATITIS sérica aparece desde el comienzo de la enfermedad y desaparece en término de cuatro meses,
A pero puede persistir seis meses o más. La presencia de IgM en suero denota infección actual o
reciente, aunque pueden surgir resultados positivos falsos. Se puede detectar IgG contra HAV
poco después de que aparezca IgM. La presencia del anticuerpo contra HAV sin que surja IgM
como anticuerpo contra HAV denota infección pasada e inmunidad. TRATAMIENTO: Medidas
de sostén.

Bibliografía:
PRINCIPLES AND PRACTICE OF PEDIATRIC INFECTIOUS DISEASES. SARAH S. LONG. CHURCHILL,
LIVINGSTONE. EDICIÓN 2A. 2002. PÁG. 1188-1192.

FIN DEL CASO CLÍNICO SERIADO


ANÁLISIS DEL CASO CLÍNICO

IDENTIFICACIÓN DEL REACTIVO


Area: CIRUGÍA
Especialidad: OTORRINOLARINGOLOGÍA
Tema: PATOLOGÍA DE LA NARÍZ Y SENOS PARANASALES
Subtema: SINUSITIS AGUDA Y CRÓNICA

CASO CLÍNICO SERIADO

HOMBRE DE 25 AÑOS DE EDAD, FUMADOR DESDE HACE 5 AÑOS. INICIA SU PADECIMIENTO ACTUAL
HACE 2 SEMANAS CON RINORREA HIALINA, MALESTAR GENERAL Y CEFALEA, RECIBIENDO AL
COMIENZO SOLO TRATAMIENTO SINTOMÁTICO. A LOS 5 DÍAS PRESENTA AUMENTO DE LOS SÍNTOMAS
AGREGÁNDOSE DESCARGA PURULENTA EN RETROFARINGE, HIPERTERMIA, ANOSMIA Y CEFALEA; SE LE
INDICA AMOXICILINA POR 10 DÍAS, LA CUAL COMPLETA SIN PRESENTAR MEJORÍA. CONSIDERA USTED
QUE EL PACIENTE CURSA EN ESTE MOMENTO CON UNA SINUSITIS AGUDA PERSISTENTE.

Adulto.

Tabaquismo. SINUSITIS AGUDA


PERSISTENTE.

Rinosinusitis viral que se complica con un


proceso bacteriano, PERSISTENCIA DE LOS
SÍNTOMAS CON TRATAMIENTO DE
PRIMERA LÍNEA.

descarga purulenta en retrofaringe, ebre,


anosmia y cefalea.

-.

22 - COMO PARTE DEL ESTUDIO DEL PACIENTE ES IMPORTANTE DISTINGUIR QUE UNA DE LAS
FUNCIONES DE LOS SENOS PARANASALES ES:
BRINDAR Los senos paranasales son estructuras cubiertas por mucosa físicamente contiguas a la
PROTECCIÓN AL cavidad nasal. Sus funciones propuestas incluyen: 1. Actuar como cámaras de resonancia
CEREBRO para la voz. 2. Brindar protección al cerebro contra traumatismos. 3. Humectar y
CONTRA humidificar el aire ambiental. 4. Aligerar el peso del esqueleto facial. La relación de los
TRAUMATISMOS. senos paranasales con el cerebro y las órbitas es de importancia extrema. - Superior y
medialmente, la lámina cribiforme del hueso etmoides sirve como techo a la cavidad
nasal y como piso a la fosa craneal anterior. - Más lateral, las celdillas etmoidales se
separan de la fosa craneal anterior por la fóvea etmoidal, la cual es parte del hueso
frontal. - La lámina papirácea, tan delgada como una hoja de papel, de la región lateral
del hueso etmoides separa los senos etmoidales de la órbita y puede servir como ruta de
diseminación de las infecciones de los senos paranasales a la periórbita. SE CREE QUE
LOS SENOS PARANASALES CUMPLEN SU FUNCIÓN AMORTIGUANDO TRAUMATISMOS
FACIALES QUE PUDIERAN LESIONAR EL CEREBRO.

PRODUCIR Los senos paranasales sirven como cámaras de resonancia de la voz en realidad donde se
SONIDOS producen los sonidos es en las CUERDAS VOCALES mediante vibración de las
TONALES O membranas que las constituyen.
SONOROS.

DAR FORMA Y Hacia el frente, la cavidad nasal se abre al ambiente exterior a través del vestíbulo nasal
PROFUNDIDAD A cubierto por piel. Hacia atrás, el cornete inferior prominente puede observarse en su
LA NARIZ. proyección medial desde la pared nasal lateral. Superior al cornete inferior, el cornete
medio cuelga desde su unión a la base del cráneo. LA FORMA DE LA NARIZ ESTÁ DADA
POR EL HUESO ETMOIDES Y EL TABIQUE NASAL, MIENTRAS QUE EN LA CAVIDAD
NASAL SE ENCUENTRAN LOS CORNETES.

HUMIDIFICAR La eliminación mucociliar nasosinusal se pronostica mediante: 1. Abertura de los senos.


SECRECIONES. 2) Función ciliar. 3) Consistencia mucosa. La alteración de cualquiera de estos tres
factores en el complejo ostiomeatal puede resultar en estasis mucosa, que, en las
condiciones adecuadas, induce el crecimiento bacteriano. SE ESPERA QUE DENTRO DE
LOS SENOS PARANASALES NO SE ACUMULEN SECRECIONES, SU FUNCIÓN ES
HUMIDIFICAR Y HUMECTAR EL AIRE, NO ASÍ LAS SECRECIONES.

Bibliografía:
1. LALWANI K, DIAGNÓSTICO Y TRATAMIENTO EN OTORRINOLARINGOLOGÍA. CIRUGÍA DE CABEZA Y
CUELLO, 2A EDICIÓN. MC GRAWHILL LANGE. 2008, PP 273-274.

23 - CORRESPONDE AL TRATAMIENTO DE ELECCIÓN EN ESTE MOMENTO:

TRIMETOPRIMA LA DURACIÓN DEL TRATAMIENTO EN CUALQUIER CASO DEBE INDICARSE POR 10 A 14


CON DÍAS. El tratamiento de elección en pacientes que no han recibido antibiótico
SULFAMETOXAZOL previamente es con alguno de los siguientes medicamentos: - Amoxicilina 500mg cada
POR 14 DÍAS. 8hrs por 10 a 14 días. - Trimetoprima con Sulfametoxazol 160mg/800mg cada 12hr
por 10 a 14 días.

DUPLICAR LA Cuando existen niveles intermedios de resistencia o susceptibilidad reducida a la


DOSIS DE amoxicilina son candidatos a un duplicado de la dosis, 3 a 4gr al día. NO TENEMOS
AMOXICILINA POR ANTIBIOGRAMA PARA SABER EL GRADO DE RESISTENCIA O SUSCEPTIBILIDAD DE LA
10 DÍAS. BACTERIA POR LO CUAL DEBERÁS ELEGIR EL CIPROFLOXACINO COMO DE ELECCIÓN
EN ESTE CASO.
CIPROFLOXACINO Las alternativas de tratamiento en casos resistentes, de hipersensibilidad o con el uso
POR 10 DÍAS. previo de antibióticos son: - Clindamicina - Rifampicina - QUINOLONAS -
Cefalosporinas En este caso el paciente cuenta ya con un tratamiento por 10 días con
amoxicilina, el objetivo de este reactivo es determinar cual antibiótico deberá usarse
en casos resistentes al tratamiento de elección o que ya han usado tratamiento
antibiótico previo y no han presentado mejoría.

DOXICICLINA POR En caso de alergia o intolerancia a los antibióticos de primera elección deberá
14 DÍAS. indicarse: - Doxiciclina 100mg cada 12hr por 10 a 14 días. - Azitromicina 500mg por 3
días. - Claritromicina 500mg cada 12hrs por 10 a 14 días. - Cefalosporinas o
Quinolonas.

Bibliografía:
1. GUÍA DE PRÁCTICA CLÍNICA, DIAGNÓSTICO Y TRATAMIENTO DE SINUSITIS AGUDA. MÉXICO:
SECRETARIA DE SALUD; 2009. RECUPERADO DE
HTTP://WWW.CENETEC.SALUD.GOB.MX/CONTENIDOS/GPC/CATALOGOMAESTROGPC.HTML 2.
LALWANI K, DIAGNÓSTICO Y TRATAMIENTO EN OTORRINOLARINGOLOGÍA. CIRUGÍA DE CABEZA Y
CUELLO, 2A EDICIÓN. MC GRAWHILL LANGE. 2008, PP 273-281.

http://www.cenetec.salud.gob.mx/descargas/gpc/CatalogoMaestro/080_GPC_Sinusitisaguda/sinusitis_evr_cenetec.pdf

FIN DEL CASO CLÍNICO SERIADO


ANÁLISIS DEL CASO CLÍNICO

IDENTIFICACIÓN DEL REACTIVO


Area: GINECOLOGÍA Y OBSTETRICIA
Especialidad: GINECOLOGÍA
Tema: INFERTILIDAD Y PLANIFICACIÓN FAMILIAR
Subtema: INFERTILIDAD

CASO CLÍNICO SERIADO

MUJER DE 35 AÑOS DE EDAD QUE HA INTENTADO EMBARAZARSE DESDE HACE 2 AÑOS. CICLOS MENSTRUALES
NORMALES. PRESENTA DESDE HACE 12 MESES DOLOR ARTICULAR Y TRES EPISODIOS DE TROMBOSIS VENOSA EN
MIEMBROS PÉLVICOS CON REMISIÓN FAVORABLE. PAREJA DE 40 AÑOS DE EDAD CUYO SEMINOGRAMA REPORTA
MOVILIDAD TIPO "A" SEGÚN LA ORGANIZACIÓN MUNDIAL DE LA SALUD.

35 AÑOS.

INCAPACIDAD PARA EMBARAZARSE EN 2


AÑOS, 3 EPISODIOS DE TROMBOSIS
VENOSA.

DOLOR ARTICULAR DE 12 MESES DE


EVOLUCIóN.

-.

-.

24 - EL DIAGNÓSTICO CLÍNICO MÁS PROBABLE ES:

INFERTILIDAD La ARTRITIS REUMATOIDE (AR) es una enfermedad inflamatoria crónica multisistémica,


SECUNDARIA A predominantemente articular, que afecta principalmente a pequeñas articulaciones de las manos y de
ARTRITIS los pies, de forma simétrica. Presenta un curso variable. El embarazo altera el estado inmune y esto
REUMATOIDE. afecta el curso de la AR. Se ha observado que durante el embarazo disminuyen los efectos de la AR,
pero se incrementan después del parto. Parece no afectar la fertilidad, sin embargo, los pacientes
afectados de artritis reumatoide tienen un deseo mayor de controlarla. NO HAY DATOS CLAROS DE
INFECUNDIDAD EN PACIENTES CON ARTRITIS REUMATOIDE.

ESTERILIDAD El LUPUS ERITEMATOSO SISTÉMICAO es una enfermedad autoinmune y sistémica, que se presenta
SECUNDARIA A con frecuencia en mujeres jóvenes y por tanto en su etapa reproductiva; se asocia a un alto riesgo de
LUPUS morbilidad y mortalidad perinatal. Las complicaciones más frecuentes son los abortos, la muerte
ERITEMATOSO fetal, la prematurez, el retardo del crecimiento intrauterino y el lupus neonatal. HABITUALMENTE NO
SISTÉMICO. COMPROMETE LA FECUNDIDAD PERO SI SE ASOCIA A UN ALTO RIESGO DE MORBILIDAD Y
MORTALIDAD PERINATAL.
SÍNDROME El SÍNDROME ANTIFOSFOLIPÍDICO, una enfermedad autoinmunitaria de manifestaciones clínicas
ANTIFOSFOLÍPIDO diversas (aborto recurrente, preeclampsia, parto pretérmino, trombosis vascular, retraso en el
PRIMARIO. crecimiento intrauterino, óbitos, desprendimiento de la placenta). Es un padecimiento
autoinmunitario distinguido por TROMBOSIS VASCULAR, trombocitopenia, reacción serológica falsa-
positiva para sífilis y pérdidas fetales recurrentes, que aparece en las pacientes (hallazgos
corroborados por laboratorio) con títulos moderados o altos de anticuerpos antifosfolipídicos.
Criterios clínicos mayores: aborto recurrente, muerte fetal en el segundo o tercer trimestre de la
gestación; trombosis venosa, arterial o trombocitopenia. Criterios clínicos menores: reacción
serológica falsa-positiva para sífilis, prueba de Coombs positiva, anormalidades valvulares cardiacas,
livedo reticularis, migraña, úlceras en las piernas, mielopatía, corea, hipertensión pulmonar o
necrosis avascular. LA INCAPACIDAD PARA EMBARAZARSE Y LA PRESENCIA DE TROMBOSIS
FUNDAMENTAN EL DIAGNÓSTICO.

ESTERILIDAD ESTERILIDAD PRIMARIA: pareja que tras un año de coitos no protegidos no ha conseguido un
PRIMARIA. embarazo (para la Federación Internacional de Ginecología y Obstetricia el periodo es de dos años).
CUMPLE CON LA DEFINICIÓN PERO EN ELLA NO SE ESTABLECEN LAS CAUSAS.

Bibliografía:
1. GUÍA DE REFERENCIA RÁPIDA, TRATAMIENTO DEL SÍNDROME DE ANTICUERPOS ANTIFOSFOLIPIDO
PRIMARIO EN EL ADULTO. MÉXICO: SECRETARIA DE SALUD; 2010. 2. GUÍA DE PRÁCTICA CLÍNICA, TRATAMIENTO
DEL SÍNDROME DE ANTICUERPOS ANTIFOSFOLIPIDO PRIMARIO EN EL ADULTO. MÉXICO: SECRETARIA DE
SALUD; 2010.

http://www.cenetec.salud.gob.mx/descargas/gpc/CatalogoMaestro/394_IMSS_10_Acs_Antifosfolipidos/EyR_IMSS_394_10.pdf

25 - PARA CONFIRMAR EL DIAGNÓSTICO DEBERÁ INVESTIGAR:

ANOMALÍAS La presencia de ANOMALÍAS FEMENINAS constituye un factor femenino de la infertilidad y, las


UTERINAS Y DE LA alteraciones en la ovulación un factor endocrinológico. NINGUNA DE ELLAS SE ASOCIA DE MANERA
OVULACIÓN DIRECTA CON EL SÍNDROME ANTIFOSFOLÍPIDOS.

BÚSQUEDA DE El DIAGNÓSTICO DE SÍNDROME DE ANTIFOSFOLÍPIDOS consiste en la determinación de


ANTICUERPOS inmunoglobulinas (IgG, IgM e IgA) dirigidas contra algunos fosfolípidos, como el anticoagulante
ANTIFOSFOLIPIDOS lúpico y el anticardiolipina. El estudio inmunológico, según los lineamientos de la Sociedad
Y SEMINOGRAMA Internacional de Trombosis y Hemostasia, debe ser positivo (al menos en dos ocasiones) para
cualquiera de los dos anticuerpos y con intervalo de seis a ocho semanas de diferencia entre cada
una de las determinaciones.

TROMBOCITOSIS Y Los CRITERIOS CLÍNICOS para el diagnóstico de SÍNDROME DE ANTIFOSFOLÍPIDOS se dividen en


OVULACIÓN mayores y menores. Criterios clínicos mayores: aborto recurrente, muerte fetal en el segundo o
tercer trimestre de la gestación; TROMBOSIS VENOSA, arterial o trombocitopenia. Criterios clínicos
menores: reacción serológica falsa-positiva para sífilis, prueba de Coombs positiva, anormalidades
valvulares cardiacas, livedo reticularis, migraña, úlceras en las piernas, mielopatía, corea,
hipertensión pulmonar o necrosis avascular. LA TROMBOSIS CONSTITUYE UN CRITERIO MAYOR, EL
DIAGNÓSTICO ES INDEPENDIENTE DE LA OVULACIÓN.

PRESENCIA DE De acuerdo al Colegio Americano de Reumatología (ACR-American College of Rheumatology), se


ARTRITIS, FACTOR establece el diagnóstico de Artritis Reumatoidea cuando están presentes cuatro de los siete
REUMATOIDE criterios, siempre y cuando de los criterios uno al cuatro, estén presentes al menos 6 semanas. 1.
Rigidez matutina de al menos una hora de duración. 2. Artritis en 3 o más áreas articulares. 3.
Artritis de las articulaciones de la mano. 4. Artritis simétrica. 5. Nódulos reumatoides. 6. Cambios
radiológicos compatibles con AR. 7. Factor reumatoide positivo.

Bibliografía:
1. GUÍA DE REFERENCIA RÁPIDA, TRATAMIENTO DEL SÍNDROME DE ANTICUERPOS ANTIFOSFOLIPIDO
PRIMARIO EN EL ADULTO. MÉXICO: SECRETARIA DE SALUD; 2010. 2. GUÍA DE PRÁCTICA CLÍNICA, TRATAMIENTO
DEL SÍNDROME DE ANTICUERPOS ANTIFOSFOLIPIDO PRIMARIO EN EL ADULTO. MÉXICO: SECRETARIA DE
SALUD; 2010.

http://www.cenetec.salud.gob.mx/descargas/gpc/CatalogoMaestro/394_IMSS_10_Acs_Antifosfolipidos/EyR_IMSS_394_10.pdf

26 - EL TRATAMIENTO INICIAL MÁS ADECUADO CONSISTIRA EN:


REPOSO POSTCOITO Se ha considerado durante mucho tiempo la medida primaria de reposo postcoito en casos de
POR 30 MINUTOS Y infertilidad con la finalidad de lograr una mayor permanencia vaginal del semen posterior al coito.
ÁCIDO FÓLICO. NO ES UNA MEDIDA ESPECÍFICA, SE CONSIDERA DENTRO DE LAS MEDIDAS GENERALES PARA EL
MANEJO DE LA INFERTILIDAD.

AZATIOPRINA Y La azatioprina es un derivado imidazólico de la 6-mercaptopurina (6-MP), activo por vía oral y
ÁCIDO FOLICO parenteral, con propiedades inmunosupresoras. La azatioprina es utilizada como inmunosupresor
en los pacientes trasplantados, pero también es útil en el tratamiento de la Artritis Reumatoidea
severa, psoriasis artrítica y lupus nefrítico. LA AZATIOPRIMA ES UN INMUNOSUPRESOR, NO ESTÁ
INDICADO EN ESTOS CASOS PUESTO QUE NO SE CONSIDERA LA PRESENCIA DE LUPUS O ARTRITIS
REUMATOIDE.

ÁCIDO Para la prevención, el tratamiento inicial se basaba en GLUCOCORTICOIDES. La combinación de


ACETILSALICÍLICO Y prednisona (40 a 60 mg/día) y ACIDO ACETILSALICILICO (80 a 100 mg/día) fue el tratamiento más
GLUCOCORTICOIDES prescrito con el que se logró un margen del 60 al 75% de efectividad para el síndrome
antifosfolipídico, pero los efectos secundarios eran frecuentes. La explicación de administrar ácido
acetilsalicílico se deba a su efecto inhibidor de la enzima cicloxigenasa, necesaria para la formación
de tromboxano A2 de efecto procoagulante y vasoconstrictor. CORRESPONDEN AL TRATAMIENTO
DE ELECCIÓN PARA LA MUJER INFÉRTIL EN QUIEN SE SOSPECHA SÍNDROME DE
ANTIFOSFOLÍPIDOS.

AUMENTO DE LA Las causas más frecuentes que pueden causar alteraciones de la calidad en el eyaculado y, por
ACTIVIDAD SEXUAL consecuencia, esterilidad en el hombre son: 1. De tipo general: tabaquismo, estrés, consumo
Y PREDNISONA excesivo de alcohol, contaminación ambiental. 2. De tipo mecánico: impotencia o exceso de
actividad sexual masculina. 3. Endocrinas: híper o hipotiroidismo, hipopituitarismo, síndromes
androgenitales y diabetes severa. 4. Enfermedades o infecciones: varicocele, dermatitis y ciertas
enfermedades de transmisión sexual. EL AUMENTO DE LA ACTIVIDAD SEXUAL DETERIORA LA
CALIDAD DE LA EYACULACIÓN, POR LO QUE NO ESTÁ RECOMENDADA COMO MEDIDA EN LA
PAREJA INFÉRTIL.

Bibliografía:
1. GUÍA DE REFERENCIA RÁPIDA, TRATAMIENTO DEL SÍNDROME DE ANTICUERPOS ANTIFOSFOLIPIDO
PRIMARIO EN EL ADULTO. MÉXICO: SECRETARIA DE SALUD; 2010. 2. GUÍA DE PRÁCTICA CLÍNICA, DIAGNÓSTICO
DE LA PAREJA INFÉRTIL Y TRATAMIENTO CON TÉCNICA DE BAJA COPLEJIDAD. MÉXICO: INSTITUTO MEXICANO
DEL SEGURO SOCIAL; 2012.
HTTP://WWW.CENETEC.SALUD.GOB.MX/DESCARGAS/GPC/CATALOGOMAESTRO/IMSS_621_13_DXPAREJAINFERTIL/621GER.PDF

http://www.cenetec.salud.gob.mx/descargas/gpc/CatalogoMaestro/394_IMSS_10_Acs_Antifosfolipidos/EyR_IMSS_394_10.pdf

FIN DEL CASO CLÍNICO SERIADO


ANÁLISIS DEL CASO CLÍNICO

IDENTIFICACIÓN DEL REACTIVO


Area: GINECOLOGÍA Y OBSTETRICIA
Especialidad: GINECOLOGÍA
Tema: CLIMATERIO Y MENOPAUSIA
Subtema: CLIMATERIO Y MENOPAUSIA

CASO CLÍNICO SERIADO

MUJER DE 54 AÑOS DE EDAD, REFIERE MENOPAUSIA HACE 7 AÑOS, CON TERAPIA DE RESTITUCIÓN HORMONAL A BASE DE ESTRÓGENOS.
ACUDE A CONSULTA POR PRESENCIA DE SANGRADO TRANSVAGINAL NO DOLOROSO QUE INICIÓ EL DÍA DE AYER. A LA EXPLORACIÓN CON IMC
32, ADECUADA COLORACIÓN EN PIEL, ABDOMEN GLOBOSO NO SE PALPA CRECIMIENTO UTERINO.

54 años

postmenopáusica, terapia estrogénica


exclusiva.

sangrado postmenopáusico

obesidad.

--

27 - ES EL DIAGNÓSTICO CLÍNICO MÁS PROBABLE EN ESTA PACIENTE:

SANGRADO El SANGRADO UTERINO ANORMAL es la variación del ciclo menstrual normal, e incluye cambios en la regularidad, frecuencia del ciclo, y
UTERINO duración del flujo o en la cantidad de sangrado menstrual asociadas a disturbios médicos que solo pueden ser identificados por una
ANORMAL historia clínica orientada a búsqueda de etiología complementando con métodos de laboratorio, gabinete y determinaciones
hormonales como por ejemplo: coagulopatías, disfunciones ovulatorias o desordenes primarios endometriales Mientras que, la
HEMORRAGIA UTERINA ANORMAL se define como la presentación aumentada en duración y cantidad del sangrado uterino. EL
TÉRMINO SANGRADO UTERINO SE APLICA EN MUJERES EN EDAD REPRODUCTIVA, NO PARA SANGRADO POSTMENOPÁUSICO.

CÁNCER Si bien un sangrado trasvaginal posterior a la menopausia debe hacer sospechar la presencia de cáncer, se deberá tener en cuenta que la
CERVICOUTERINO paciente se encuentra tomando estrógenos sin oposición. EL CÁNCER CERVICOUTERINO NO ES HORMONODEPENDIENTE, POR LO QUE,
AUNQUE RESULTA SIEMPRE OBLIGADO DESCARTAR SU PRESENCIA, NO ES ESTA LA MEJOR RESPUESTA BASADA EN LOS DATOS
CLÍNICOS APORTADOS EN EL CASO. Aun así, deberá promoverse la detección de cáncer cervicouterino en mujeres postmenopaúsicas de
forma rutinaria.

HIPERPLASIA El SANGRADO POSTMENOPÁUSICO se define como la hemorragia uterina que sucede al menos un año después de la menopausia. La
ENDOMETRIAL causa más frecuente de sangrado trasvaginal en la mujer postmenopáusica es la hipertrofia endometrial. A toda mujer postmenopáusica
con sangrado trasvaginal se le debe ofrecer un estudio diagnóstico de imagen con el propósito de evaluar una posible existencia de
pólipos, miomas u otras anomalías estructurales focales, entre las cuales se debe descartar siempre cáncer de endometrio. La
HIPERPLASIA ENDOMETRIAL es un crecimiento excesivo del endometrio que se da por un estímulo mantenido de estrógenos
endógenos o exógenos que no son contrarrestados por la acción de la progesterona. En mujeres con útero en las cuales se les ha
administrado exclusivamente estrógenos sin oposición, existe un mayor riesgo de hiperplasia endometrial y cáncer de endometrio.
Otros factores incluyen: obesidad, diabetes, hipertensión, historia pasada de hiperestrogenismo. IMPORTANTE: el 90% de las mujeres
con cáncer endometrial presentan sangrado trasvaginal como única manifestación clínica de la enfermedad. EL USO DE ESTRÓGENOS
EXÓGENOS SIN OPOSICIÓN CONSTITUYEN EL ANTECEDENTE MÁS IMPORTANTE PARA DIRIGIR TU SOSPECHA DIAGNÓSTICA.

ENDOMETRIOSIS La ENDOMETRIOSIS es la presencia de tejido endometrial funcional, glándulas y estroma fuera de la cavidad uterina, principalmente en
ovarios y en la superficie del peritoneo pélvico, la cual induce una reacción inflamatoria crónica. Se caracteriza por dolor pélvico crónico,
infertilidad y masa anexial. Otros síntomas incluyen dismenorrea, hipermenorrea, dispareunia. NO SE RELACIONA EN NADA CON EL
CUADRO CLÍNICO.
Bibliografía:
1. GUÍA DE PRÁCTICA CLÍNICA, DIAGNÓSTICO Y TRATAMIENTO DE LA HIPERPLASIA ENDOMETRIAL EN MUJERES POSMENOPÁUSICAS EN EL
SEGUNDO NIVEL DE ATENCIÓN. MÉXICO: SECRETARIA DE SALUD; 2012.
HTTP://WWW.CENETEC.SALUD.GOB.MX/DESCARGAS/GPC/CATALOGOMAESTRO/223_SSA_09_HIPERPLASIA_ENDOMETRIAL/GPC_223-
09_HIPERPLASIA_ENDOMETRIALEVR.PDF 2. GUÍA DE PRÁCTICA CLÍNICA, ATENCIÓN DE LOS PADECIMIENTOS GINECOLÓGICOS MÁS
FRECUENTES EN LA POSTMENOPAUSIA. MÉXICO: SECRETARIA DE SALUD; 2012.
HTTP://WWW.CENETEC.SALUD.GOB.MX/DESCARGAS/GPC/CATALOGOMAESTRO/568_GPC_PADECIMIENTOSGINECOLOGICOSPOSTMENOPAUSIA/568GER.PDF
3. GUÍA DE PRÁCTICA CLÍNICA, DIAGNÓSTICO Y TRATAMIENTO DE LA ENDOMETRIOSIS. MÉXICO: SECRETARIA DE SALUD; 2013.
HTTP://WWW.CENETEC.SALUD.GOB.MX/DESCARGAS/GPC/CATALOGOMAESTRO/207_SSA_10_ENDOMETRIOSIS/EYR_SSA_207_09.PDF

28 - CONSTITUYE EL TRATAMIENTO DE ELECCIÓN PARA ESTE CASO:

DIU El DIU levonogestrel es una buena opción terapéutica para el manejo de dos de 4 de las patologías presentadas: 1. Sangrado uterino
LEVONORGESTREL anormal. El tratamiento médico inicial en la paciente con sangrado uterino anormal de causa no anatómica responde adecuadamente a
tratamiento farmacológico, contando con las siguientes opciones: - Antiinflamatorios no esteroideos (AINES). - Anticonceptivos
hormonales orales, ya sea orales, intrauterinos (levonorgestrel en sistema intrauterino), al igual que progestágenos como el acetato de
medroxiprogesterona(AMP). - El Danazol o agonistas de hormonas liberadora de las gonadotropinas. El sistema intrauterino liberador
de levonorgestrel (DIULNG) reduce en 86% a tres meses y 97% a los 12 meses el sangrado menstrual, ya que libera 20 µg de
levonorgestrel directamente sobre la cavidad endometrial por día induciendo atrofia endometrial. 2. Hiperplasia endometrial no atípica
en mujeres PREMENOPÁUSICAS. El tratamiento casi siempre será suficiente con dosis bajas de progestina, a menudo se utiliza
medroxiprogesterina vía oral a dosis de 10 a 20 mg. diarios durante de a 14 días al mes. Otras opciones incluyen la pastilla
anticonceptiva combinada o el DIU de progesterona. La ablación endometrial histeroscópica puede utilizarse como un método curativo,
pero la vigilancia suele ser más difícil con índices de histerectomía posterior altos. EL DIU LEVONORGESTREL ES UNA OPCIÓN
TERAPÉUTICA PARA EL MANEJO DE SANGRADO UTERINO ANORMAL Y DE LA HIPERPLASIA ENDOMETRIAL NO ATÍPICA EN MUJERES
“PREMENOPÁUSICAS”. IMPORTANTE: deberás poner especial atención a la condición menopáusica de la paciente, pues ello dirigirá la
elección de la respuesta en este caso.

ABLACIÓN La ABLASIÓN LAPAROSCÓPICA con láser del tejido endometrióico permite la eliminación del tejido ectópico en pacientes con
LAPAROSCÓPICA ENDOMETRIOSIS. Aunque no es la única opción terapéutica, se ha indicado el uso de AINES para reducir el dolor pélvico, la
administración de anticonceptivos hormonales orales combinados, análogos de la GnRHy el danazol, así como el tratamiento con
progestágenos. CORRESPONDE AL TRATAMIENTO DE LA ENDOMETRIOSIS, NO APLICA PARA ESTE CASO.

ABLACIÓN En general, en mujeres POSTMENOPAÚSICAS con hiperplasia endometrial se siguen dos principales caminos en el manejo de la
HISTEROSCÓPICA enfermedad: control anual sin tratamiento, (hiperplasia endometrial simple) e histerectomía (hiperplasia endometrial atípica). Otras
opciones terapéuticas incluyen: - Con hiperplasia compleja sin atipias, puede tratarse con medroxiprogesterona cíclica a dosis bajas o
con régimen continuo de 2.5 mg. al día. - En pacientes sin atipia la ablación histeroscópica se asocia con un alto nivel de satisfacción en
las pacientes. Permite además el diagnóstico de pólipos endometriales y fibromas submucosos. La histeroscopía con curetaje puede
considerarse cuando existe un reporte de hiperplasia endometrial no atípica (simple o compleja) obtenido tras una biopsia.
IMPORTANTE: su uso es controversial, la misma GPC se contradice en sus indicaciones, se cree que las mujeres sometidas a ablación
terminan en muchos de los casos con histerectomía. SI BIEN LA ABLACIÓN HISTEROSCÓPICA NO CONSTITUYE LA PRIMERA OPCIÓN
TERAPÉUTICA PARA ESTE CASO, ES EL ÚNICO TRATAMIENTO DE LOS 4 QUE SE PUEDE OFRECER A ESTA PACIENTE Y ES POR ESO QUE
DEBERÁS ELEGIRLA COMO CORRECTA. IMPORTANTE: recuerda que las preguntas son tipo ENARM, por lo tanto, deberás elegir de las
cuatro opciones la respuesta más correcta con base en los datos clínicos aportados en el caso. NO OLVIDES QUE, dado que la hiperplasia
en este caso se condicionó por la administración de estrógenos solos, se deberá replantear su uso, y en caso de considerarse necesarios,
utilizarlos siempre acompañados de progestágenos.

CONIZACIÓN La CONIZACIÓN CERVICAL es un método quirúrgico que conste en la extirpación del tejido cervical. Está indicada para lesiones
displásicas del cuello de la matriz. CORRESPONDE AL TRATAMIENTO DE LA DISPLASIA CERVICAL, NO ES DE UTILIDAD EN ESTE CASO.

Bibliografía:
1. GUÍA DE PRÁCTICA CLÍNICA, DIAGNÓSTICO Y TRATAMIENTO DE LA HIPERPLASIA ENDOMETRIAL EN MUJERES POSMENOPÁUSICAS EN EL
SEGUNDO NIVEL DE ATENCIÓN. MÉXICO: SECRETARIA DE SALUD; 2012.
HTTP://WWW.CENETEC.SALUD.GOB.MX/DESCARGAS/GPC/CATALOGOMAESTRO/223_SSA_09_HIPERPLASIA_ENDOMETRIAL/GPC_223-
09_HIPERPLASIA_ENDOMETRIALEVR.PDF 2. GUÍA DE PRÁCTICA CLÍNICA, ATENCIÓN DE LOS PADECIMIENTOS GINECOLÓGICOS MÁS
FRECUENTES EN LA POSTMENOPAUSIA. MÉXICO: SECRETARIA DE SALUD; 2012.
HTTP://WWW.CENETEC.SALUD.GOB.MX/DESCARGAS/GPC/CATALOGOMAESTRO/568_GPC_PADECIMIENTOSGINECOLOGICOSPOSTMENOPAUSIA/568GER.PDF
3. GUÍA DE PRÁCTICA CLÍNICA, DIAGNÓSTICO Y TRATAMIENTO DE LA HEMORRAGIA UTERINA DISFUNCIONAL. MÉXICO: SECRETARIA DE SALUD;
2009.
HTTP://WWW.CENETEC.SALUD.GOB.MX/DESCARGAS/GPC/CATALOGOMAESTRO/322_IMSS_10_HEMORRAGIA_UTERINA_DISFUNCIONAL/EYR_IMSS_322_10.PDF

FIN DEL CASO CLÍNICO SERIADO


ANÁLISIS DEL CASO CLÍNICO

IDENTIFICACIÓN DEL REACTIVO


Area: MEDICINA INTERNA
Especialidad: INFECTOLOGÍA
Tema: INFECCIONES GASTROINTESTINALES
Subtema: TIFOIDEA Y OTRAS SALMONELOSIS

CASO CLÍNICO CON UNA PREGUNTA

MASCULINO DE 34 AÑOS DE EDAD, BAJO TRATAMIENTO MÉDICO POR FIEBRE TIFOIDEA CONFIRMADA
DESDE HACE 10 DÍAS. EL PACIENTE ACUDE POR RECAÍDA, SE CONSIDERA LA POSIBILIDAD DE UNA
COMPLICACIÓN.

Masculino de 34 años de edad.

Fiebre tifoidea hace 10 días recibió


tratamiento.

-.

-.

-.

29 - CORRESPONDE A LA PROBABILIDAD DE COMPLICARSE POR FIEBRE TIFOIDEA:

DEL Las manifestaciones neurológicas se presentan en 2 a 40 % de los pacientes con FIEBRE TIFOIDEA.
30 Estas son meningitis, síndrome de Guillain-Barré, neuritis y síntomas neuropsiquiátricos como delirio.
AL Algunas complicaciones más raras cuya incidencia se reduce si se inicia el tratamiento antibiótico
40% rápidamente son coagulación intravascular diseminada, síndrome hemofagocítico, pancreatitis,
abscesos hepáticos y esplénicos, y granulomas, endocarditis, pericarditis y miocarditis, orquitis,
hepatitis, glomérulonefritis, pielonefritis y síndrome hemolítico urémico. Neumonía severa, artritis,
osteomielitis y parotiditis. NO HAY REFERENCIAS CLARAS EN NUESTRO PAÍS DADO QUE NO
CORRESPONDE A LA COMPLICACIÓN MÁS FRECUENTE.
DEL Después de la resolución de la fiebre en pacientes con FIEBRE TIFOIDEA, la debilidad y pérdida de
20 peso pueden persistir por meses. Un 10 % de los pacientes tienen una recaída leve generalmente
A dentro de las 2 a 3 semanas después de la resolución de la fiebre y están asociadas, con la misma cepa
39% y susceptibilidad al tratamiento. DESPUÉS DEL TRATAMIENTO SE CALCULA QUE UN 10% PUEDE
TENER UN EVENTO DE RECAÍDA DE LA ENFERMEDAD.

DEL La FIEBRE TIFOIDEA es una enfermedad sistémica, febril, aguda, de origen entérico, secundaria a
10 infección por S. Typhi, aunque ocasionalmente puede ser originada por S. paratyphi A, S.
A schotmuelleri o S. hirschfeldii. Afecta únicamente al ser humano, cursa habitualmente con afección
19% sistémica y, en ocasiones, puede originar complicaciones graves como son perforación intestinal y
enterorragia. CX: La fiebre es el síntoma principal, aparece en el 75 a 100% de los casos. La
enfermedad debe sospecharse en casos clínicos que cursen con: fiebre >39°C de más de 72hrs,
cefalea, malestar general y tos seca. La diarrea es más frecuente en niños y en pacientes con
compromiso inmunológico, se acompaña de dolor abdominal y vómito. Otros datos que apoyan la
sospecha diagnóstico son: constipación o diarrea, vómito, dolor abdominal, exantema macular
(rosácea tifoídica), lengua saburral, hepatomegalia, esplenomegalia. El desarrollo de ENFERMEDAD
GRAVE por FIEBRE TIFOIDEA ocurre aproximadamente entre 10 al 15 % de los pacientes y depende de
varios factores. Dentro de estos están los factores del huésped como inmunosupresión, tratamiento
con antiácidos, exposición previa y vacunación. También depende la virulencia y el inóculo de la
bacteria y la elección adecuada del tratamiento antimicrobiano. Las complicaciones más frecuentes
son hemorragia gastrointestinal (10 a 20 %) y perforación intestinal en 1 a 3 % y ocurre más
frecuentemente en la tercer y cuarta semana de la enfermedad. Se producen como resultado de
hiperplasia, ulceración y necrosis de las placas ileocecales de Peyer en el sitio inicial de infiltración por
Salmonella. Ambas complicaciones ponen en riesgo la vida y ameritan tratamiento con reanimación
hídrica inmediata e intervención quirúrgica, junto con tratamiento antibiótico de amplio espectro.
LAS GUÍAS DE PRÁCTICA CLÍNICA APLICABLES PARA NUESTRO PAÍS, ACEPTAN LA PROBABILIDAD
DEL 10 A 17.6% PARA COMPLICACIONES EN PACIENTES QUE CURSAN CON FIEBRE TIFOIDEA, LO
CUAL COINCIDE CON EL RANGO DE ESTA RESPUESTA.

DEL La perforación intestinal ocurre entre 1 a 3 % y es más frecuente en la tercer y cuarta semana de la
5A enfermedad por FIEBRE TIFOIDEA. Se producen como resultado de hiperplasia, ulceración y necrosis
9% de las placas ileocecales de Séller, en el sitio inicial de infiltración por Salmonella. DE ACUERDO A LAS
REFERENCIAS LA PERFORACIÓN INTESTINAL JUNTO CON EL SANGRADO GASTROINTESTINAL SON
LAS COMPLICACIONES MÁS FRECUENTES DE LA FIEBRE TIFOIDEA, EN CONJUNTO REPRESENTAN
HASTA UN 12% DEL TOTAL DE COMPLICACIONES.

Bibliografía:
DIAGNÓSTICO Y TRATAMIENTO PARA LA FIEBRE TIFOIDEA NIÑOS/NIÑAS PRIMERO, SEGUNDO Y
TERCER NIVEL. EVIDENCIAS Y RECOMENDACIONES: GUÍA DE PRÁCTICA CLÍNICA. MÉXICO:
SECRETARÍA DE SALUD; 03/11/2016.

http://www.cenetec-difusion.com/CMGPC/IMSS-259-10/ER.pdf
ANÁLISIS DEL CASO CLÍNICO

IDENTIFICACIÓN DEL REACTIVO


Area: MEDICINA INTERNA
Especialidad: DERMATOLOGÍA
Tema: TRANSTORNOS DESCAMATIVOS
Subtema: PSORIASIS

CASO CLÍNICO SERIADO

HOMBRE DE 31 AÑOS PRESENTA DESDE HACE 6 MESES DERMATOSIS DISEMINADA, BILATERAL,


SIMÉTRICA, EN PIEL CABELLUDA, CODOS Y RODILLAS, CARACTERIZADA POR PLACAS
ERITEMATOESCAMOSAS, BIEN DELIMITADAS, CON ESCAMA GRUESA.

masculino de 31 años de edad.

cuadro de 6 meses de evolución.

dermatosis diseminada, bilateral, simétrica


en piel cabelluda, codos y rodillas.
Caracterizada por placas gruesas
eritemaTOESCAMOSAS, BIEN
DELIMITADAS CON ESCAMA GRUESA.

-.

-.

30 - EL DIAGNÓSTICO CLÍNICO MÁS PROBABLE ES:


DERMATOMIOSITIS. La DERMATOMIOSITIS es una enfermedad de etiología autoinmunitaria que afecta
fundamentalmente al músculo esquelético y a la piel. Se caracteriza por un proceso
inflamatorio no supurativo con predominio de inflamación linfocitaria. Se denomina
polimiositis cuando se respeta la piel y dermatomiositis cuando la polimiositis se
asocia a una erupción cutánea característica. La tercera parte de los casos se asocian a
diferentes enfermedades del tejido conectivo y una décima parte a neoplasias
malignas. La causa es desconocida, pero parece que contribuyen varios factores: 1.
Factores genéticos. Una mayor predisposición genética para HLA DR3 y DRW52. 2.
Mecanismo inmunitario. La presencia de autoanticuerpos circulantes frente antígenos
musculares (anti-JO 1, anti-Mi, anti- PM1, anti-PM/Scl), linfocitos CD8+ y macrófagos
que invaden fibras musculares. 3. Virus. Se han involucrado distintas partículas víricas
en su etiología. Es más frecuente en mujeres (excepto el tipo III) y en edades
comprendidas entre los 45 y 60 años. Las Manifestaciones clínicas de esta patología
son: 1. Lesiones cutáneas. Las pápulas de Gottron, manifestación patognomónica, se
presentan en un 70% de los casos. Consisten en maculo-pápulas eritemato-violáceas
sobre superficies articulares interfalángicas y metacarpofalángicas, incluso en
rodillas, codos y periungueales Con la evolución del proceso pueden aparecer placas
simétricas, formadas por confluencia de las pápulas, con superficie descamativa y
telangiectasias; este hecho se conoce como signo de Gottron, pudiendo evolucionar
dejando pequeñas cicatrices atróficas o hipopigmentadas. El rash heliotropo, se
observa en el 60 % de los casos, es un eritema violáceo, a veces acompañado de
edema, que afecta de forma simétrica a los párpados, Además puede extenderse a
otras áreas faciales, al cuero cabelludo, la parte superior del tórax y la superficie de
extensión de las extremidades. El eritema, puede ser transitorio persistente, y sobre él
pueden aparecer áreas atróficas, que se deben a isquemia, dando lugar a verdaderas
ulceraciones y en niños son frecuentes las calcificaciones subcutáneas. Otras
manifestaciones cutáneas serían fotosensibilidad entre un 50-80%, telangiectasias y
eritema periungueales, eritema psoriasiforme del cuero cabelludo, telangiectasias
gingivales, mucinosis y lesiones lupus-like. Las lesiones de larga duración pueden
evolucionar a una poiquilodermia. 2. Lesiones musculares. Debilidad muscular aguda
o subaguda, simétrica y difusa en músculos proximales de extremidades, tronco y
cuello (cinturón pélvico y escapular), 3. Otros síntomas. Artralgias o artritis transitoria
no erosiva, alteraciones ECG, arritmias o miocarditis, disfagia, fibrosis pulmonar,
fenómeno de Raynaud, afectación renal, etc. Los siguientes criterios son utilizados
para definir la DM/PM: 1. Debilidad muscular proximal y simétrica, con disfagia y
debilidad muscular respiratoria o sin ella. 2. Elevación de las enzimas musculares
séricas (CPK, transaminasas, LDH y aldolasas). 3. Alteraciones EMG características. 4.
Alteraciones en la biopsia muscular. La presencia de dos de estos criterios, en
asociación a los cambios dermatológicos, hace probable el diagnóstico de DM,
mientras que la presencia de tres o más, lo confirma. La PM queda definida si reúne
los cuatro criterios. Microscópicamente, las lesiones cutáneas se caracterizan por
hiperqueratosis variable con acantosis o con atrofia epidérmica y degeneración
vacuolar de la capa basal. En la dermis se observa ectasia vascular, infiltrado
inflamatorio mononuclear perivascular y depósito de ácido hialurónico. En el tejido
subcutáneo puede haber áreas de paniculitis asociadas con degeneración mucoide de
células adiposas (en lesiones precoces) o focos de calcificación (en lesiones antiguas).
No obstante, para efectuar el diagnóstico de dermatomiositis se requiere observar
necrosis segmentaria de fibras musculares esqueléticas, que se manifiesta por pérdida
de la estriación del músculo, fagocitosis de los fragmentos musculares degenerados e
hialinización eosinófila de fibras musculares, así como, infiltrado inflamatorio
mononuclear intersticial, entre fibras musculares y alrededor de los vasos. Las
lesiones más antiguas muestran un cuadro inespecífico de atrofia de fibras
musculares y fibrosis intersticial difusa con escasa inflamación. EN LA
DERMATOMIOSITIS LAS PÁPULAS SON LAS LESIONES DÉRMICAS CARACTERÍSTICA.
DERMATITIS La DERMATITIS ATÓPICA (DA) es un proceso inflamatorio cutáneo crónico,
ATÓPICA. intensamente pruriginoso, de carácter recurrente, usualmente aparece durante la
infancia temprana y la niñez pero puede persistir o comenzar en la vida adulta. La
dermatitis atópica es un proceso multifactorial donde intervienen factores intrínsecos
(alteración genética, trastornos inmunológicos, piel alterada) y factores extrínsecos
que pueden desencadenar o exacerbar el brote (alérgenos alimentarios,
aeroalérgenos, microrganismos). La prevalencia de la dermatitis atópica varia de unas
zonas geográficas a otras y en general es mayor cuanto más al norte se sitúa el área
estudiada y cuanto más alto es su nivel de desarrollo industrial y parece ir
aumentando, no hay un entendimiento claro de los factores determinantes del
incremento. Se han encontrado en algunas poblaciones manifestaciones de dermatitis
atópica en la edad infantil entre un 5 a un 10 %, sin diferencias significativas en
relación con el sexo. Las estadísticas revelan una prevalencia infantil global de 10% a
20% y en los adultos de 1 a 3%. Entre un 70 a 80% de los pacientes tienen una historia
familiar positiva de atopía. La influencia genética es compleja siendo el componente
materno más importante. Los atópicos presentan una respuesta inmunitaria humoral
y celular alterada, que facilita la reacción con antígenos ambientales. Con las
siguientes alteraciones: Aumento de la Ig E sérica, alteración de las subpoblaciones
linfocitarias, alteración de las inmunoglobulinas, aumento de la expresión en la
membrana de las células de Langerhans, alteraciones de la reactividad vascular y
farmacológica, los vasos de estos pacientes tienen tendencia a la vasoconstricción,
alteraciones de la fisiología de la piel, disminución del umbral del prurito ,alteración
en la eliminación de la sudoración ,alteración del manto lipídico de la piel.
Clásicamente la Dermatitis Atópica típica se fija su inicio al final de la fase infantil a
partir de los 10 años o en la pubertad. Sus áreas de predilección son: cara (frente,
parpados, región perioral), cuello (especialmente nuca), parte alta del tórax y
hombros, grandes pliegues flexurales y dorso de las manos. Las lesiones
características son las placas de liquenificación. Pueden aparecer también eczema de
manos, dishidrosis y prurigo nodular. La mayor parte de los pacientes evolucionan
hacia la resolución antes de los 20 años, siendo muy infrecuentes las manifestaciones
de la enfermedad después de los 30 años Las manifestaciones clínicas son la base del
diagnóstico, ya que ni la histología ni ninguna otra prueba de laboratorio son
específicas de la enfermedad. El diagnóstico se fundamenta en criterios clínicos y son
necesarios 3 criterios mayores y 3 criterios menores. Criterios mayores: Prurito,
dermatitis crónica recurrente, morfología y distribución característica, historia
personal o familiar de atopia. Criterios menores: Xerosis o piel seca, queratosis
pilar/exageración de pliegues palmares, pitiriasis alba, dermatitis inespecífica de
manos y pies, eczema del pezón, queilitis, acentuación perifolicular, conjuntivitis
recidivante, intolerancia a la lana y a los disolventes de las grasas, oscurecimiento
periocular, queratocono, catarata subcapsular anterior, pliegue de Dennie-Morgan,
pliegue del cuello, fisuras infraauriculares, edad temprana de inicio, tendencia a
infecciones cutáneas, aumento de los niveles séricos de IgE, reactividad inmediata
tipo I en los tests cutáneos, tendencia a infecciones cutáneas y déficit de la inmunidad
celular, intolerancia a algunos alimentos, reactivación por causa ambiental o
emocional, dermografismo blanco, blanqueamiento retardado,
vasoconstricción/palidez facial, cambio de la temperatura de los dedos, sudoración
anormal con prurito, disminución de la actividad de las glándulas sebáceas,
personalidad atópica. La determinación de la Ig E suele mostrar un nivel aumentado,
pero hay que tener en cuenta que un 20% de los atópicos pueden mostrar
determinaciones normales, y al revés, otras patologías e incluso individuos sanos
pueden mostrar una elevación de esta Ig. Los pacientes con Dermatitis Atópica un 70
a 80% de los pacientes tienen una historia familiar positiva de atopía. La influencia
genética es compleja siendo el componente materno más importante. Un estudio
reciente a gran escala de análisis de micro arreglos de ADN ha demostrado que
cuatro genes involucrados en la diferenciación epidérmica, ubicados en el cromosoma
1q21, muestran diferentes niveles de expresión en lesiones eczematosas de la piel en
comparación con controles. LA DERMATITIS ATÓPICA ES POCO COMÚN QUE TENGA
SU INICIO EN LA EDAD ADULTA, LAS LESIONES SUELEN SER PRURIGINOSAS.

PSORIASIS. La PSORIASIS es una enfermedad crónica, determinada genéticamente, caracterizada


por sucesivos brotes de placas eritemato-descamativas en las superficies extensoras
de la piel y cuero cabelludo, siendo la artropatía la única manifestación extracutánea.
Aunque la etiología es desconocida, existe una clara agregación familiar que apoya la
implicación de factores genéticos. Además, factores ambientales conocidos como los
traumatismos, diálisis, la hipocalcemia, algunas infecciones como la faringitis
estreptocócica o el VIH, fármacos como el litio, betabloqueantes, antipalúdicos,
interferón, interleucina 2, antidiabéticos orales, la retirada de corticoides sistémicos,
algunos antinflamatorios (indometacina) y, por último, las situaciones de estrés,
pueden ser desencadenantes de la enfermedad. La psoriasis se produce como
consecuencia de la interacción de 3 fenómenos que concurren simultáneamente: 1.
aumento de la velocidad de crecimiento epidérmica; 2. proliferación venular
postcapilar en las papilas dérmicas; y 3. Reacción inmune mediada por linfocitos T. La
consecuencia de esta interacción es el fenómeno de la exudación cíclica papilar, que
caracteriza microscópicamente a la enfermedad. Existen dos picos de incidencia: a los
16-22 años y a los 57-60 años, sin diferencia entre sexos, aunque las mujeres
desarrollan antes la enfermedad. La lesión elemental es una placa eritematosa, de
bordes bien definidos y superficie irregular descamativa, que tras el raspado
metódico deja una superficie eritematosa con pequeños puntos sangrantes, rocío
hemorrágico de Auspitz”. Se disponen simétricamente en superficies extensoras y en
cuero cabelludo. Hasta un tercio de los enfermos presentan lesiones en zonas
previamente traumatizadas (fenómeno de Koebner o isomorfismo). Se han descrito
distintas formas clínicas: 1. Psoriasis vulgar: es la forma más frecuente. Las lesiones
son crónicas y se localizan además de en los lugares ya mencionados, en la piel de
abdomen y sacro. 2. Psoriasis en gotas: las lesiones son lenticulares o puntiformes, en
tronco, generalmente en niños y adultos jóvenes, cursando en brotes. Tiene buen
pronóstico y se ha relacionado con infección faríngea estreptocócica previa 3.
Psoriasis invertida suele afectar grandes pliegues con predominio del eritema sobre la
descamación 4. Psoriasis palmoplantar, con placas eritemato descamativas con
fisuración. 5. Psoriasis ungueal: aparece hasta en un 35% en pies y un 50% en manos
de enfermos con psoriasis. Es más frecuente en los casos de eritrodermia o artropatía
asociada, aunque se puede presentar aislada. El espectro clínico va desde lesiones
puntiformes en la lámina ungular (pitas) a onicodistrofia intensa con incluso pérdida
ungueal, pasando por la característica “mancha de aceite” o mancha marrón distal de
la lámina. 6. Psoriasis pustulosa, que puede presentar varias formas: 6.1. Generalizada
tipo von Zumbusch: Puede ser el debut de la enfermedad, asociada frecuentemente a
artropatía, ser la evolución de una forma pustulosa localizada o estar desencadenada
por fármacos (corticoides orales, litio, fenilbutazona), infecciones o embarazo
(impétigo herpetiforme). De forma súbita, aparecen lesiones eritematosas
confluentes que pueden evolucionar a eritrodermia sobre las que surgen brotes
sucesivos de pústulas blanquecinas, anulares, agrupadas o dispersas. Asocia fiebre,
malestar general, y leucocitosis con desviación a la izquierda. Suele respetar palmas y
plantas, aunque es común la afectación ungueal. 6.2. Anular: puede ser generalizada
o localizada. Son lesiones anulares con borde eritematoso y collarete descamativo
interior con alguna pústula aislada; el centro es rosado con descamación 6.3.
Localizadas, con 2 formas: a) Pustulosis palmoplantar, de predominio en adultos y en
el sexo femenino. No está clara su relación con la psoriasis. De hecho, sólo un 24% de
estos enfermos tienen antecedentes de psoriasis; b) Acrodermatitis continúa de
Hallopeau, caracterizada por pústulas en las falanges distales de los dedos (pulgares),
que evolucionan dejando, al vaciarse, una superficie eritematosa brillante atrófica.
Existe afectación ungueal con onicodistrofia e incluso desaparición de la uña por
daño de la matriz. Puede asociarse a otras lesiones a distancia de psoriasis vulgar o
pustuloso e incluso generalizarse a una forma de von Zumbusch. 7. Eritrodermia
psoriásica: eritrodermia exfoliativa no pruriginosa, con afectación del estado general
Las lesiones son de borde neto, con afectación ungueal importante. Suele aparecer en
enfermos con psoriasis crónicas intensas. 8. Artropatía psoriásica: Esta artritis
seronegativa afecta del 5 al 8% de los pacientes con psoriasis. En un 70% afecta de
forma asimétrica a las pequeñas articulaciones de los dedos y en el resto aparece
como poliartritis simétrica, espondilitis anquilosante y artritis mutilante. Ocurre en la
cuarta y quinta década, y en un 50% de los casos de forma aguda. La psoriasis
cutánea precede a la artropatía y la intensidad de la clínica articular es independiente
de la afectación cutánea, aunque más grave en las formas pustulosas. Es muy
frecuente la afectación ungueal, incluso como única manifestación cutánea. La
psoriasis es una enfermedad inmunológica mediada por los linfocitos T y en la que el
recambio celular de los queratinocitos está acelerado. LA DISTRIBUCIÓN ANATÓMICA
DE LAS LESIONES ACOMPAÑADAS DE ESCAMA GRUESA FUNDAMENTAN EL
DIAGNÓSTICO.

DERMATITIS La DERMATITIS SEBORREICA (DS) o eczema seborreico constituye una dermatosis


SEBORREICA. inflamatoria crónica frecuente y recurrente en regiones cutáneas rica en glándulas
sebáceas y eventualmente en algunas áreas intertriginosas, primordialmente en cara,
cuero cabelludo y pecho. La lesión elemental es la placa descamativa y la enfermedad
se agrupa dentro de las afecciones eritematoescamosas. La dermatitis seborreica
afecta entre el 1% y el 3% de la población inmunocompetente. Es más frecuente en
hombres que en mujeres y afecta más a la raza negra. Suele presentar dos picos de
edad, uno entre los adolescentes y adultos jóvenes y otro en pacientes mayores de 50
años. La causa de la dermatitis seborreica se desconoce, sin embargo, diversos
factores como la seborrea, cambios hormonales, alteraciones neurológicas,
microrganismos como las levaduras, variaciones estacionales y el estrés emocional
constituye factores condicionantes o exacerbantes de la dermatosis. El signo clínico
más importantes es la inflamación que se manifiesta clínicamente como finas placas
descamativas grasosas en ocasiones bien delimitadas que varían desde el color
rosado-amarillo hasta el rojo marrón, con predilección a áreas ricas en glándulas
sebáceas como son cuero cabelludo, región centro facial de cara, cejas, región
preesternal, región interescapular y menos frecuente área intertriginosas como axilas,
región inguinal, pliegue inframamario y ombligo. El prurito puede ir desde una leve
molestia hasta sensación de ardor en formas severas o generalizadas. La dermatitis
seborreica estaría producida como consecuencia de una reacción anormal de la piel
frente a la presencia de las levaduras y esta reacción podría estar relacionada con la
capacidad de Malassezia para sintetizar una toxina o estimular la actividad lipasa. Por
lo que se cree que la dermatitis seborreica es causada por una relación alterada entre
estos microrganismos comensales superficiales y el huésped, es más, el éxito de
medicamentos antimicóticos para el tratamiento apoya esta hipótesis ya que la
terapia está basada en controlar y disminuir la producción de sebo, inhibir la
multiplicación de la Malassezia y otras levaduras y desinflamar el área afectada. La
dermatitis seborreica en el adulto requiere que el paciente entienda que es una
enfermedad crónica, que tendrá períodos de mejoría y empeoramiento, por lo que no
debe de buscar la erradicación completa de la afección sino el control. El tratamiento
tópico puede estar dirigido al uso de antinflamatorios. La corticoterapia tópica es
efectiva en el tratamiento de la dermatitis seborreica y su efecto es debido a su
actividad antinflamatoria. Se deben de utilizar corticoesteroides de baja o la mediana
potencia del tipo de la hidrocortisona o mometasona. Los inhibidores de la
calcineurina no están aprobados para el tratamiento de la dermatitis seborreica. La
pomada de succinato de litio es efectiva en el tratamiento de la dermatitis seborreica,
tanto en individuos inmunocompetentes como en enfermos de sida. El succinato de
litio es efectivo contra la Malassezia spp. Otros agentes empleados son los
queratolíticos, como ácido salicílico, el sulfuro de selenio, las fórmulas de alquitrán de
hulla y el propilenoglicol. Los antimicóticos tópicos son empleados para reducir las
colonias de malassezia y disminuir la inflamación. LAS LESIONES EN CODOS Y
RODILLAS NO CORRESPONDEN A DERMATITIS SEBORREICA.

Bibliografía:
1. LONGO DL, FAUCI AS, KASPER DL, HAUSERSL, JAMESON JL, LOSCALZOJ. HARRISON. PRINCIPIOS
DE MEDICINA INTERNA, 18A EDICIÓN. MC GRAW HILL. NEW YORK, USA. 2012, PP 398-399. 2.
PAPADAKIS MAXINE A, MCPHEE STEPHEN J. DIAGNÓSTICO CLÍNICO Y TRATAMIENTO. 52ª EDICIÓN.
NUEVA YORK. 2013, PP 106-107.

31 - EL PACIENTE DEBE SER TRATADO POR MEDIO DE:


FOTOTERAPIA La elección del tratamiento en los pacientes con PSORIASIS se realiza en función de la
Y extensión y localización de las lesiones, así como de los tratamientos previos y la edad del
CORTICOIDES paciente. Entre los tratamientos tópicos DESTACAN LOS CORTICOIDES, EN CURA ABIERTA
TÓPICOS. U OCLUSIVA Y LOS DERIVADOS DE LA VITAMINA D (TACALCITOL Y CALCIPOTRIOL). ESTOS
FÁRMACOS SE UTILIZAN CONJUNTAMENTE CON EMOLIENTES Y BREAS O ALQUITRANES,
ASÍ COMO CON QUERATOLÍTICOS COMO EL ÁCIDO SALICÍLICO. "La helioterapia, y su
variante PUVA (psoraleno + radiación UVA) o radiación UVB de banda estrecha, es un
tratamiento muy empleado, especialmente en las psoriasis eruptivas o en gotas". Dentro
de los tratamientos sistémicos, destaca el empleo de: 1. Retinoides (etretinato y acitretin)
muy útiles en las formas pustulosas, eritrodérmicas y en la artropatía, solos o combinados
con fototerapia (RePUVA); 2. Metotrexate, un potente antiproliferativo y antinflamatorio,
muy eficaz en las formas graves y, sobre todo, en la psoriasis con artropatía, aunque sus
efectos secundarios agudos (Afectación de médula ósea o fibrosis hepática) lo limitan; 3.
Ciclosporina, agente inmunosupresor efectivo en un 70% de enfermos con psoriasis vulgar
crónico intenso, aunque también se utiliza en casos de artropatía, psoriasis pustulosa
generalizada o eritrodermia psoriásica.

LUBRICANTES MEDIDAS GENERALES DEL CUIDADO DE LA PIEL Y LUBRICANTES EN LOS PACIENTES CON
TÓPICOS. DERMATITIS ATÓPICA Los objetivos del tratamiento son: 1.- Medidas generales para evitar
factores desencadenantes: • Exceso de lavado, uso de jabones detergentes. • Ropas
ajustadas, excesivas y fibras sintéticas • Irritantes primarios: polvo, grasas, disolventes,
barnices, etc. • Trabajos inadecuados: carpintería, minería, mecánica, etc. • Climas con
temperaturas extremas. • Estrés emocional y conflictos familiares • Infecciones
intercurrente, bacteriana, vírica o fúngicas. • Alérgenos (alimentos solo en un 20%) 2.-
Cuidado de la piel: • Hidratación.- Es la base del tratamiento de mantenimiento y juega un
papel importante en el brote agudo de la enfermedad, ya que permite disminuir la
cantidad de esteroides tópicos requeridos para el control de la inflamación. • No deben
usarse jabones alcalinos. Se utilizan jabones de pH ácido y aceites de baño. • Los baños de
avena coloidal, con carácter emoliente. • Tras el baño y dentro de los tres minutos se
aplicaran hidratantes y emolientes que permiten retener el agua manteniendo la barrera
suave y flexible. • El uso de esponjas talcos, perfumes, lociones y el secado por fricción
deberán estar proscritos. CORRESPONDEN AL MANEJO BÁSICO DE LA DERMATITIS
ATÓPICA.

PREDNISONA CORTICOIDES TÓPICOS EN PSORIAS: actúan aclarando las placas y reduciendo la


ORAL. inflamación, si bien difícilmente curan las lesiones de forma completa y duradera. Se
utilizan los de baja potencia para zonas delicadas (cara, pliegues y zonas intertriginosas) y
los de potencia más alta para el cuero cabelludo, zonas con placas más gruesas, manos y
pies. Los efectos adversos pueden ser locales o sistémicos. Entre los locales, los más
frecuentes son: disminución del grosor de la epidermis, inhibición de los melanocitos con
aclaramiento de la piel que puede ser crónico, atrofia dérmica, púrpura, telangiectasia,
hipertricosis, acné y dermatitis rosaceiforme, eccemas de contacto. Los efectos sistémicos,
aunque poco frecuentes, pueden ser graves e incluyen la inhibición del eje hipotálamo-
hipofisario y síndrome de Cushing yatrogénico. POR SUS EFECTOS ADVERSOS EL USO DE
ESTEROIDES PARA EL TRATAMIENTO DE LA PSORIASIS SE PREFIERE TÓPICO TRAS HABER
PROBADO ANTES CON MEDIDAS BÁSICAS DEL CONTROL DE LA ENFERMEDAD. LOS
ESTEROIDES SISTÉMICOS SUELEN PROVOCAR GRANDES EXACERBACIONES.
KETOCONAZOL La DERMATITIS SEBORREICA en el adulto requiere que el paciente entienda que es una
EN SHAMPOO. enfermedad crónica, que tendrá períodos de mejoría y empeoramiento, por lo que no
debe de buscar la erradicación completa de la afección sino el control. El tratamiento
tópico puede estar dirigido al uso de antinflamatorios. La corticoterapia tópica es efectiva
en el tratamiento de la dermatitis seborreica y su efecto es debido a su actividad
antinflamatoria. Se deben de utilizar corticoesteroides de baja o la mediana potencia del
tipo de la hidrocortisona o mometasona. Los inhibidores de la calcineurina no están
aprobados para el tratamiento de la dermatitis seborreica. la pomada de succinato de litio
es efectiva en el tratamiento de la dermatitis seborreica, tanto en individuos
inmunocompetentes como en enfermos de sida. El succinato de litio es efectivo contra la
malassezia spp. otros agentes empleados son los queratolíticos, como ácido salicílico, el
sulfuro de selenio, las fórmulas de alquitrán de hulla y el PROPILENOGLICOL. LOS
ANTIMICÓTICOS TÓPICOS SON EMPLEADOS PARA REDUCIR LAS COLONIAS DE
MALASSEZIA Y DISMINUIR LA INFLAMACIÓN PERO NO SON PARTE DEL TRATAMIENTO
ESPECÍFICO.

Bibliografía:
1. GUÍA DE PRÁCTICA CLÍNICA, TRATAMIENTO FARMACOLÓGICO PARA PACIENTES ADULTOS CON
PSORIASIS EN PLACAS. MÉXICO: SECRETARIA DE SALUD, 2013. 2. LONGO DL, FAUCI AS, KASPER DL,
HAUSERSL, JAMESON JL, LOSCALZOJ. HARRISON. PRINCIPIOS DE MEDICINA INTERNA, 18A
EDICIÓN. MC GRAW HILL. NEW YORK, USA. 2012, PP 398-399. 3. PAPADAKIS MAXINE A, MCPHEE
STEPHEN J. DIAGNÓSTICO CLÍNICO Y TRATAMIENTO. 52ª EDICIÓN. NUEVA YORK. 2013, PP 106-107.

http://www.cenetec.salud.gob.mx/descargas/gpc/CatalogoMaestro/IMSS-696-FARMACOLOGICO_PSORIASIS_EN_PLACAS/IMSS-696-13-GER-

TX_FX_PSORIASIS_EN_PLACAS.pdf

FIN DEL CASO CLÍNICO SERIADO


ANÁLISIS DEL CASO CLÍNICO

IDENTIFICACIÓN DEL REACTIVO


Area: MEDICINA INTERNA
Especialidad: REUMATOLOGÍA
Tema: TRANSTORNOS MEDIADOS POR MECANISMOS INMUNITARIOS
Subtema: ARTRITIS REUMATOIDE

CASO CLÍNICO CON UNA PREGUNTA

MUJER DE 45 AÑOS CON DIAGNÓSTICO DESDE HACE 2 AÑOS DE ARTRITIS REUMATOIDE.

MUJER DE 45 AÑOS.

2 AÑOS con ARTRITIS REUMATOIDE.

-.

-.

-.

32 - LA SIGUIENTE TERAPIA FÍSICA NO DEBERÁ SER RECOMENDADA A LA PACIENTE DEBIDO A


QUE NO HA DEMOSTRADO EFICACIA EN ESTE TIPO DE PACIENTES:

TERMOTERAPIA LA APLICACIÓN DE CALOR Y FRÍO COMBINADOS PUEDE PROPORCIONAR UN ALIVIO


COMBINADA SINTOMÁTICO TEMPORAL DEL DOLOR Y LA RIGIDEZ. • La hidroterapia permite maximizar
CON FRÍO. los efectos positivos sobre el control del dolor, función física y autoeficacia. • Se puede
recomendar balneoterapia en casos de afectación poliarticular y sin enfermedad activa,
donde otras terapias más asequibles hayan sido inefectivas.

LÁSER DE BAJA EL LÁSER DE BAJA POTENCIA ES DE UTILIDAD PARA DISMINUIR EL DOLOR. • El láser de
POTENCIA. baja potencia y el TENS, de forma aislada e independiente, son eficaces para la
disminución del dolor a corto plazo. Se recomienda la aplicación de TENS para disminuir
el dolor (el TENS tiene la ventaja de la comodidad de aplicación con aparatos portátiles
para utilizar en el domicilio). • La terapia ocupacional puede ayudar cuando los problemas
en el trabajo son debido a la artritis, modificándose métodos, posturas y con la asistencia
de soportes que ayuden a mejorar la habilidad funcional.
ULTRASONIDO. EL ULTRASONIDO ESTÁ INDICADO EN PACIENTES QUE NO RESPONDEN A OTRAS
ALTERNATIVAS DE TERAPIA FÍSICA. Los datos sobre ultrasonido, electroestimulación
muscular y magnetoterapia aún son insuficientes para recomendarlos de forma habitual,
pero deberían considerarse en casos seleccionados que no han respondido a otras
alternativas.

FRÍO LOCAL. De acuerdo a las GPC vigentes, la aplicación aislada de termoterapia y la aplicación de frío
local (SÓLO) no parecen ofrecer ningún beneficio clínico. • La combinación de parafina
(termoterapia) y ejercicios activos son eficaces para limitar el dolor.

Bibliografía:
1. GUÍA DE PRÁCTICA CLÍNICA, DIAGNÓSTICO, TRATAMIENTO DE LA ARTRITIS REUMATOIDE EN EL
ADULTO. MÉXICO: SECRETARIA DE SALUD, 2010.

http://www.cenetec.salud.gob.mx /descargas/gpc/CatalogoMaestro/195_ARTRITIS_REUMATOIDE/Artritis_reumatoidE_EVR_CENETEC.pdf
ANÁLISIS DEL CASO CLÍNICO

IDENTIFICACIÓN DEL REACTIVO


Area: MEDICINA INTERNA
Especialidad: NEUMOLOGÍA
Tema: ENFERMEDADES PLEURALES
Subtema: DERRAME PLEURAL Y TB

CASO CLÍNICO CON UNA PREGUNTA

MUJER DE 52 AÑOS SIN ANTECEDENTES DE IMPORTANCIA. SE ENCUENTRA HOSPITALIZADA POR


PRESENTAR DERRAME PLEURAL. ACTUALMENTE EN ESTUDIO PARA DETERMINAR LA ETIOLOGÍA
ESPECÍFICA.

MUJER DE 52 AÑOS.

HOSPITALIZADA POR PRESENTAR


DERRAME PLEURAL.

EN ESTUDIO PARA DETERMINAR LA


ETIOLOGÍA ESPECÍFICA.

-.

-.

33 - EL USO DE FIBRINOLÍTICOS EN ESTA PACIENTE ESTARÁ INDICADO EN CASO DE:


DERRAME El DRENAJE PLEURAL (DP) es la acumulación anormal de líquido en el espacio pleural
PLEURAL que constituye un problema frecuente entre diversas enfermedades pulmonares y
SECUNDARIO A extrapulmonares. Los criterios más aceptados para su identificación son: presencia de
LUPUS. líquido de más de 50 mililitros, borramiento de los senos costodiafragmatico y
cardiofrénico de un cm en la radiografía posteroanterior de tórax. La prevalencia se
estima de 400 por 100,000.00 habitantes, la causa más frecuente es la insuficiencia
cardiaca congestiva. Otras etiologías predominantes son la neumonía, la tuberculosis
pleural, neoplasias y el tromboembolismo pulmonar. El DP se ha clasificado
básicamente en trasudados y exudados. En condiciones normales el volumen del
líquido pleural es de 5 a 15 ml. Su entrada y salida del espacio pleural se reabsorbe al
mismo ritmo que su producción pero no en cantidades elevadas. Los signos y síntomas
del DP dependen de la enfermedad subyacente, y de la cantidad de líquido contenido
en el espacio pleural. Al examen clínico los pacientes presentan datos sugestivos del
derrame como: disminución de los movimientos respiratorios, disminución o ausencia
de vibraciones vocales y matidez a la percusión. Debe sospecharse el DP en pacientes
con datos clínicos sugestivos y que en la exploración del tórax se documente
disminución de los movimientos respiratorios, matidez y ausencia de vibraciones
vocales. De acuerdo a la causa del derrame pleural se pueden asociar otros datos
clínicos como fiebre, pérdida de peso, hemoptisis, que pueden orientar a un
diagnóstico específico.

DERRAME PRUEBAS DIAGNÓSTICAS Las pruebas diagnósticas en los pacientes con DP


PLEURAL fundamentalmente consisten en estudios de gabinete como; radiografía de tórax (PA,
TUBERCULOSO. Laterales), tomografía de tórax y en algunos casos ultrasonido torácico. Las pruebas de
laboratorio van desde el aspecto del LP, estudio citológico y bioquímico hasta cultivos y
determinación de pruebas específicas como adenosin desaminasa, PCR, etc. La imagen
de la radiografía de tórax en el DP es usualmente característica. 200 mls de LP
producen borramiento de los ángulos costofrénico y costodiafragmético, sin embargo,
50 a 75 mls de LP puede producir también borramiento del ángulo costofrénico
posterior en la radiografía lateral, provocando el denominado signo del menisco, puede
ubicarse entre la cisura o a nivel mediastinal. Los DP loculados ocurren más
comúnmente en asociación con condiciones que causan inflamación pleural intensa
como son; empiema, tuberculosis o hemotórax. Los derrames loculados ocurren más
comúnmente en asociación con condiciones que causan intensa inflamación pleural
como el empiema, el hemotórax o la tuberculosis. Ocasionalmente, la presencia de
líquido dentro de la cisura puede simular un pseudotumor, situación observada en
pacientes con ICC. Los derrames loculados ocurren más comúnmente en asociación con
condiciones que causan intensa inflamación pleural como el empiema, el hemotórax o
la tuberculosis. Ocasionalmente, la presencia de líquido dentro de la cisura puede
simular un pseudotumor, situación observada en pacientes con ICC. En pacientes con
sospecha clínica de derrame pleural se recomienda realizar en primer lugar radiografía
de tórax (PA y Lateral) Los grandes derrames pueden llegar a opacificar un hemitórax
por completo que provoca desplazamiento del mediastino hacía el lado opuesto y la
mayoría de estos son neoplásicos. Los DP pequeños pueden detectarse
radiográficamente en decúbito lateral sobre el lado afectado. El ultrasonido es un
método útil en el estudio de la patología pleural. Sus indicaciones son: 1. Identificación
de la localización apropiada para efectuar toracocentesis, biopsia pleural o colocación
de sonda. 2. Identificación de loculaciones pleurales. 3. Diferenciación entre derrame y
engrosamiento pleural. El empleo de la tomografía axial computarizada (TAC) es útil en
casos de difícil diagnóstico, donde no se pueden establecer condiciones médicas de
malignidad o para determinar el tamaño, y localización de los derrames loculados. La
TAC tiene una sensibilidad del 94% y una especificidad del 51% para establecer la
etiología maligna de un DP. Se recomienda la TAC, cuando las radiografías de tórax no
pueden distinguir entre la causa benigna o maligna o cuando existe duda diagnóstica.
Se recomienda en los pacientes con DP solicitar estudios de laboratorio como;
biometría hemática, química sanguínea, pruebas de funcionamiento hepático y pruebas
de coagulación.
DERRAME INDICACIONES DE RETIRO. El manejo temprano con antibióticos disminuye la
PLEURAL CON probabilidad de desarrollar derrame paraneumónico y la progresión de un empiema. La
PAQUIPLEURITIS. sonda endopleural es la opción estándar cuando se decide drenar el espacio pleural en
DP masivos, empiemas, quilotórax, derrame pleural recidivantes. La sonda endopleural
debe permanecer en el paciente hasta que: • Volumen menor a 100 mls/24 hrs. • LP
cetrino. • Ausencia de síntomas infecciosos y/o respiratorios. La Toracocentesis es el
procedimiento diagnóstico de mayor relevancia para el estudio del DP y en algunos
casos necesario para el tratamiento. Mediante esta sencilla técnica que consiste en la
punción torácica es posible obtener la cantidad de líquido necesario para los estudios
de investigación mencionados. La técnica consiste en: realizar aseo de la región elegida
previamente mediante imágenes y la percusión del tórax (si liquido pleural es
abundante, la línea axilar anterior, en el quinto espacio intercostal es el sitio más
recomendable) 16 o 18, con una jeringa de 50 mls. Si el procedimiento tiene fines
diagnósticos no es necesario evacuar completamente el líquido. Si la toracocentesis
tiene fines terapéuticos, es recomendable realizarlo colocando una llave de tres vías, lo
que evitará la contaminación y entrada de aire a la cavidad pleural y mantendrá el
control de la evacuación. La indicación de la toracocentesis diagnóstica es en DP
clínicamente significativa con más de 10 mm de engrosamiento por radiografía de
tórax o por ecocardiografía. Ante la sospecha de hemotórax o empiema realizar
toracocentesis de urgencia. El análisis del LP obtenido por toracocentesis permite
identificar si se trata de un exudado o un trasudado de acuerdo a los criterios de Light.

DERRAME El uso de agentes fibrinolíticos se ha empleado en DERRAME PLEURAL multiloculados a


PLEURAL efecto de lisar las paredes y comunicar los lóculos para facilitar el drenaje. Actualmente
MULTILOCULADO. los agentes fibrinolíticos como “primera opción” en aquellos pacientes diagnosticados
con empiema o derrame pleural loculado. Las evidencias muestran que son eficaces y
seguros en población adulta y pediátrica. Se recomienda utilizar estreptoquinasa o
alteplasa en el manejo del empiema en fase II, en población adulta y pediátrica donde
no hay respuesta al manejo conservador. Cuando hay falla en el tratamiento se
recomienda realizar manejo toracoscópico si se cuenta con el recurso.

Bibliografía:
1. DIAGNÓSTICO Y TRATAMIENTO DEL DERRAME PLEURAL. EVIDENCIAS Y RECOMENDACIONES:
GUÍA DE PRÁCTICA CLÍNICA. MÉXICO: SECRETARÍA DE SALUD; 03/11/2016.

http://www.cenetec-difusion.com/CMGPC/IMSS-243-09/ER.pdf
ANÁLISIS DEL CASO CLÍNICO

IDENTIFICACIÓN DEL REACTIVO


Area: MEDICINA INTERNA
Especialidad: ENDOCRINOLOGÍA
Tema: DIABETES MELLITUS
Subtema: NEUROPATÍA DIABÉTICA Y SUS COMPLICACIONES

CASO CLÍNICO CON UNA PREGUNTA

HOMBRE DE 45 AÑOS CON DIABETES MELLITUS DE 18 AÑOS DE EVOLUCIÓN, TRATADO CON INSULINA.
DESDE HACE 1 AÑO PRESENTA NÁUSEAS QUE FRECUENTEMENTE LLEGAN AL VÓMITO DE CONTENIDO
GASTROALIMENTARIO, SACIEDAD TEMPRANA Y DISTENSIÓN ABDOMINAL.

hombre de 45 años de edad.

diabético desde hace 18 años en


tratamiento con insulina.

1 AÑO CON NÁUSEAS Y VÓMITO, saciedad


temprana y distensión abdominal.

-.

-.

34 - EL DIAGNÓSTICO CLÍNICO MÁS PROBABLE ES:

HERNIA HIATAL. La hernia hiatal es una de las causas de enfermedad por reflujo gastroesofágico. Por
lo que los síntomas se refieren en el apartado de abajo.
ENFERMEDAD POR La pirosis es el síntoma clásico de los pacientes con ENFERMEDAD POR REFLUJO
REFLUJO GASTROESOFÁGICO (ERGE). Estos generalmente refieren sensación de ardor
GASTROESOFÁGICO. quemante que se eleva desde el estómago hasta el pecho y se irradia hacia el cuello,
garganta y ocasionalmente a la espalda. De forma característica se presenta en el
periodo postprandial especialmente después de grandes cantidades de alimentos o
después de la ingesta de alimentos muy condimentados, cítricos, grasas, chocolate o
alcohol. El decúbito dorsal puede exacerbar los síntomas. Otros síntomas son
regurgitación ácida, disfagia, odinofagia, eructos, hipo, náuseas y vomito. RECUERDA
SIEMPRE UBICAR EL CASO CLÍNICO SEGÚN LOS DATOS OTORGADOS, ES PROBABLE
QUE SEGUNDARIO A LA GASTROPARECIA EL PACIENTE PRESENTE ADEMÁS ERGE,
PERO DE MANERA SECUNDARIA Y NO COMO CAUSA BÁSICA.

ENFERMEDAD La ENFERMEDAD ÁCIDO PÉPTICA puede involucrar al esófago, estómago y duodeno.


ÁCIDO PÉPTICA. La presentación típica de esta enfermedad son episodios recurrentes de dolor. El
dolor de forma característica se localiza en el epigastrio y puede irradiarse a la
espalda o menos común al tórax u otras regiones del abdomen. En ocasiones los
pacientes refieren el dolor como quemante o penetrante y en otras ocasiones como la
sensación de vacío en estómago. El dolor inicialmente puede mejorar con la ingesta
de alimentos pero regresa en el periodo postprandial. Además del dolor también
pueden referir pirosis o regurgitación ácida a la garganta, nausea y vomito. EL DATO
CENTRAL ES EL DOLOR ABDOMINAL, NO PRESENTE EN EL PACIENTE.

GASTROPARESIA. Los síntomas digestivos más destacados son retraso del vaciamiento gástrico
(gastroparesia) y alteraciones de la motilidad del intestino delgado y el grueso
(estreñimiento o diarrea). La gastroparesia puede presentarse con síntomas de
anorexia, náuseas, vómitos, saciedad precoz y distensión del abdomen. LA
GASTROPARESIA ES UNA PATOLOGÍA FRECUENTEMENTE ASOCIADA A LA DIABETES,
POR LO QUE ESE ANTECEDENTE DA PESO A LA ELECCIÓN DE ÉSTA OPCIÓN.

Bibliografía:
1. LONGO DL, FAUCI AS, KASPER DL, HAUSERSL, JAMESON JL, LOSCALZOJ. HARRISON. PRINCIPIOS
DE MEDICINA INTERNA, 18A EDICIÓN. MC GRAW HILL. NEW YORK, USA. 2012, PP 2985.
ANÁLISIS DEL CASO CLÍNICO

IDENTIFICACIÓN DEL REACTIVO


Area: PEDIATRÍA
Especialidad: INFECTOLOGIA PEDIÁTRICA
Tema: CRUP
Subtema: CRUP

CASO CLÍNICO CON UNA PREGUNTA

ESCOLAR DE 6 AÑOS PRESENTA TOS SECA, DISFONIA Y ESTRIDOR QUE HAN SIDO PROGRESIVOS. LE
DIAGNOSTICA LARINGOTRAQUEITIS AGUDA.

escolar de 6 años

TOS SECA Y DISFONIA.

diagnóstico de laringotraqueitis aguda.

35 - LA SINTOMATOLOGIA QUE PRESENTA LA PACIENTE ES POR:


EDEMA EN PATOGENIA LARINGOTRAQUEITIS. La infección se transmite por contacto de persona a
CUERDAS persona o por secreciones infectadas. La infección viral comienza en la nasofaringe y se
VOCALES Y disemina hacia el epitelio respiratorio de la laringe y la tráquea, donde puede detenerse o
TEJIDO continuar su descenso por el árbol respiratorio. "Ocasiona inflamación difusa, eritema y
SUBGLÓTICO edema en las paredes de la tráquea, y deteriora la movilidad de las cuerdas vocales." La
frecuencia del crup viral a una edad determinada se explica, en parte, por razones
anatómicas: el diámetro de la vía aérea en los niños es mucho más pequeño que en los
adultos; por tanto, ante un mismo grado de inflamación, la obstrucción al flujo aéreo será
mayor. La región subglótica es la parte más estrecha de la vía aérea superior en los niños y
la más afectada en la LA. Además, esta zona está rodeada por un cartílago firme que facilita
que pequeños grados de inflamación causen una obstrucción importante. La ventilación del
niño también resulta afectada por la congestión nasal acompañante y por un aumento de la
velocidad respiratoria durante el llanto. El estrechamiento de la laringe origina la dificultad
respiratoria y produce un ruido ronco inspiratorio que se llama estridor. La inflamación y la
paresia de las cuerdas vocales provocan la afonía de estos cuadros. El edema de la mucosa y
submucosa de la porción subglótica de la vía aérea, asociado a un aumento en la cantidad y
viscosidad de las secreciones, provoca una disminución de la luz traqueal. Al inicio, esta
obstrucción puede compensarse con taquipnea, pero si aumenta, el trabajo respiratorio
será mayor y puede llegar a agotar al paciente. En esta fase de insuficiencia respiratoria
aparece hipoxemia.

AREAS DE TRATAMIENTO LARINGOTRAQUEITIS. En los procesos de laringitis leves, suelen ser


DENUDACIÓN procesos benignos que no requieren medidas terapéuticas. En las laringitis más graves sí
DEL EPITELIO existe consenso en el uso de corticoides y adrenalina nebulizada, aunque continúa el
SUBGLÓTICO debate sobre la dosis y posología de ambos fármacos. Recordar que la nebulización de
sustancias en el tratamiento de las laringitis agudas precisa de un flujo de aire de 4-6 litros
por minuto, para favorecer el depósito de las mismas en la laringe. Es importante destacar
la importancia de evitar en las laringitis situaciones que incrementan y facilitan la agitación
del niño, pues esto conduce a un empeoramiento de la sintomatología (estridor, tos
perruna). Se recomiendan medidas que tranquilicen (estar en brazos de los padres) durante
la exploración y tratamiento de esta patología.

EPIGLOTIS TRATAMIENTO LARINGOTRAQUEITIS. Corticoides La eficacia de los corticoides en el


ENGROSADA tratamiento de las laringitis en sus diferentes modalidades de administración (oral,
Y intramuscular o nebulizada) ha sido demostrada en diferentes estudios comparados con el
ERITEMATOSA uso del placebo: - Disminuye la intensidad de la clínica del paciente. - Disminuye el número
de visitas a urgencias/ ingresos hospitalarios. - Disminuye el uso de adrenalina nebulizada.
No se ha demostrado una eficacia terapéutica superior en la asociación de corticoides
orales/intramusculares, junto a inhalados. Adrenalina La adrenalina estaría indicada en los
casos moderados-graves de laringitis aguda: produce mejoría clínica de los síntomas,
aunque no acorta la estancia hospitalaria hospitalaria, ni el curso de la enfermedad.
Considerada como medida terapéutica previa a la intubación. Es, por tanto, útil en el
tratamiento de las laringitis graves tras administrar corticoides oral/nebulizado, y
considerada como medida terapéutica previa al ingreso hospitalario. Su mecanismo de
acción final es por disminución del edema de la mucosa laríngea; produce una estimulación
de los alfa-receptores en las arteriolas precapilares con disminución final de la presión
hidrostática. Tras su administración, su efecto rápido comienza a los 10 minutos con pico
máximo a las 2 horas.
ABUNDANTE TRATAMIENTO LARINGOTRAQUEITIS. Manejo del niño con laringotraqueitis aguda en
MOCO A atención Primaria. 1. Tranquilizar al niño y evitar procedimientos molestos o dolorosos que
NIVEL no sean necesarios. Procurar que el niño se encuentre cómodo, en brazos de sus padres. 2.
LARINGEO No se debe explorar oro-faringe si el niño no colabora, puede empeorar. 3. Si la laringitis es
leve, tranquilizar a la familia y explicar el cuadro. Recomendar medidas generales de
ambiente húmedo y frío. Así con antiinflamatorios no esteroides. Realizar valoración
individual del paciente si precisa dosis única de dexametasona oral, 0,15 mg/kg:
disminuirán la tos y la afonía, junto a la ansiedad familiar 4. En caso de laringitis moderada
administrar una dosis de dexametasona oral a dosis: 0,3 mg/kg. 5. Si la laringitis es grave
(cianosis, alteración del nivel de conciencia) se procederá a derivar a centro hospitalario.
Previamente al traslado se estabilizará al paciente administrando una dosis de
dexametasona oral a 0,6 mg/kg y L-adrenalina nebulizada 3 mg (3 ml de adrenalina1:1.000
y 2 ml de fisiológico) con oxígeno a flujos bajos (4-6 l/min.), para que las partículas de gran
tamaño se depositen en la laringe. La mejoría tras la administración de adrenalina es
inmediata, pero de duración aproximada de 2 horas, que puede revertir a su situación
original. No debe administrarse sin asociar corticoides. 6. Si existe broncoespasmo
asociado debe añadirse salbutamol inhalado. Indicaciones de ingreso hospitalario. 1
Dificultad respiratoria moderada: episodios de apneas o cianosis, trabajo respiratorio
intenso, dificultad para la alimentación. 2. Afectación del estado general, somnolencia o
agitación 3. Gran ansiedad familiar o incapacidad para un cuidado y vigilancia correctos en
el domicilio.

Bibliografía:
NELSON, TRATADO DE PEDIATRÍA. JESUS KUMATE. MENDEZ EDITORES. EDICIÓN 15A. 1997. PAG. 127.
ANÁLISIS DEL CASO CLÍNICO

IDENTIFICACIÓN DEL REACTIVO


Area: MEDICINA INTERNA
Especialidad: INFECTOLOGÍA
Tema: INFECCIONES RENALES Y DE LAS VÍAS URINARIAS
Subtema: CISTITIS

CASO CLÍNICO CON UNA PREGUNTA

EN LA CLÍNICA DONDE USTED TRABAJA SE IMPLEMENTARÁ UN PROGRAMA DE PREVENCIÓN PRIMARIA


DE INFECCIÓN DE VÍAS URINARIAS.

-.

Se implementara un programa para


prevenir infecciones urinarias en la
población.

-.

-.

-.

36 - LA SIGUIENTE MEDIDA CORRESPONDE A UNA ESTRATEGIA DE PREVENCIÓN PRIMARIA:

USO DE LABSTIX El uso de labstix en orina para tamizaje no es una medida preventiva, más bien
EN ORINA COMO corresponde ya a monitorización, para identificar a las pacientes con infecciones de la
TAMIZAJE EN vía urinaria. En el caso de las embarazadas se utiliza además para identificar
PACIENTES proteinuria además de ser una estrategia específica únicamente para gestantes. LA
EMBARAZADAS PRINCIPAL ESTRATEGIA DE TOMA DE LABSTIX EN EMBARAZADAS ES LA BÚSQUEDA
DE PROTEÍNAS. AÚN ASÍ NO ES UNA MEDIDA QUE APLIQUE PARA TODA LA
POBLACIÓN.

UROCULTIVO A La toma de urocultivo en mujeres en edad fértil qué ya tienen sintomatología urinaria
MUJERES EN es una medida para diagnóstico etiológico de la enfermedad, no es una medida
EDAD FÉRTIL CON preventiva. El urocultivo está indicado sólo en caso de infección recurrente. NO ES UNA
SINTOMATOLOGÍA MEDIDA PREVENTIVA SINO DIAGNÓSTICA.
URINARIA
EXAMEN Esta es una medida para identificar a aquellas pacientes que pudieran tener algún
GENERAL DE problema en la vía urinaria, aunque no es un estudio indicado de forma rutinaria para
ORINA A LA todas las pacientes asintomáticas. NO ES UNA MEDIDA PREVENTIVA SINO
POBLACIÓN DIAGNÓSTICO.
GENERAL EN
PACIENTES
ASINTOMÁTICAS

ORIENTACIÓN La manifestación más común de las infecciones de vía urinarias es la CISTITIS AGUDA,
SOBRE MEDIDAS que se presenta con mayor frecuencia en mujeres que en hombres. Se reconocen como
HIGIÉNICAS Y factores de riesgo uso frecuente de espermicidad, coitos frecuentes, antecedente de
DIETÉTICAS DE LA infecciones del tracto urinario, diabetes mellitus, incontinencia, entre otros. Cuando la
POBLACIÓN población llega a la edad adulta la prevalencia de bacteriuria asintomática aumenta en
las mujeres, tiene una prevalencia en mujeres no embarazadas de 1 a 3 %. De estas
mujeres aproximadamente 25 % desarrollaran infecciones de vías urinarias, estas cifras
evidentemente aumentan con la edad. EL ORIENTAR A LAS PERSONAS SOBRE QUÉ
MEDIDAS IMPLEMENTAR PARA EVITAR EL DESARROLLO DE UNA INFECCIÓN DE VÍAS
URINARIAS "SI" ES UNA MEDIDA DE PREVENCIÓN, EL RESTO SON MEDIDAS
DIAGNÓSTICAS. RECUERDA: la colonización del tracto vaginal y zona periuretral tienen
una fuerte influencia en la génesis de las infecciones urinarias; las medidas higiénicas
son importantes en la profilaxis de éstas.

Bibliografía:
1. LONGO DL, FAUCI AS, KASPER DL, HAUSERSL, JAMESON JL, LOSCALZOJ. HARRISON. PRINCIPIOS
DE MEDICINA INTERNA, 18A EDICIÓN. MC GRAW HILL. NEW YORK, USA. 2012, PP 2388.
ANÁLISIS DEL CASO CLÍNICO

IDENTIFICACIÓN DEL REACTIVO


Area: MEDICINA INTERNA
Especialidad: GASTROENTEROLOGÍA
Tema: PATOLOGÍA HEPÁTICA Y PANCREATICA
Subtema: HEPATOPATÍA ALCOHÓLICA, INSUFICIENCIA HEPÁTICA Y
CIRROSIS

CASO CLÍNICO CON UNA PREGUNTA

PACIENTE MASCULINO DE 45 AÑOS DE EDAD, ALCOHÓLICO DESDE HACE 15 AÑOS, ACUDE A


URGENCIAS CON UN CUADRO CARACTERÍSTICO DE HEPATITIS ALCOHÓLICA. SOLICITA USTED ESTUDIOS
DE LABORATORIO.

masculino de 45 años de edad.

alcoholismo desde hace 15 años.

cuadro clínico compatible con hepatitis


alcohólica.

-.

-.

37 - CON BASE EN EL CUADRO CLÍNICO USTED ESPERARÍA ENCONTRAR LOS NIVELES DE LA


SIGUIENTE ENZIMA POR ARRIBA DE LAS DEL RESTO:
ASPARTATO La HEPATITIS ALCOHÓLICA se asocia con un amplio espectro de manifestaciones
AMINO clínicas. Se piensa que la producción de citocinas es el elemento que rige las
TRANSFERASA. manifestaciones generalizadas de la hepatitis por alcohol. El extremo del espectro está
representado por fiebre, arañas vasculares, ictericia y dolor abdominal que simula
abdomen agudo, aunque muchos pacientes se encuentran completamente
asintomáticos. Identificar las características clínicas de la hepatitis alcohólica resulta
esencial para iniciar una estrategia diagnóstica y terapéutica adecuada. Laboratorio.
Los pacientes con hígado graso alcohólico con frecuencia se identifican a través de las
pruebas habituales. Las alteraciones típicas de laboratorio son inespecíficas y consisten
en elevaciones ligeras de las Aminotransferasas de aspartato (aspartate
aminotransferase, AST), de alanina (alanine aminotransferase, ALT) y la gamma-
glutamiltranspeptidasa (gamma-glutamyl transpeptidase, GGTP), acompañadas de
hipertrigliceridemia, hipercolesterolemia y, ocasionalmente, hiperbilirrubinemia. En la
hepatitis alcohólica, al contrario de lo que sucede en el hígado graso, la AST y la ALT
suelen estar elevadas dos a siete veces. Rara vez superan las 400 UI y la relación
AST/ALT es mayor de uno. La hiperbilirrubinemia es frecuente y se acompaña de
incrementos moderados de la fosfatasa alcalina. Las alteraciones en la función de
síntesis del hepatocito indican una enfermedad más grave. La hipoalbuminemia y la
coagulopatía son frecuentes en la enfermedad hepática avanzada. Se observan
elevaciones de los niveles de aminotransferasas séricas en los pacientes con hepatitis
alcohólica y cirrosis, aun cuando la enfermedad es severa. Los niveles de AST
generalmente se encuentran en cifras menores de 300 a 500 U/L y se asocian con una
elevación trivial de los niveles de ALT lo que resulta en una relación AST/ALT mayor de
2, esto es característico de la enfermedad hepática por alcohol, en parte por deficiencia
de la fosfato piridoxina, que actúa como cofactor de las aminotransferasas en los
pacientes alcohólicos. LA RELACIÓN AST/ALT EN HEPATITIS ALCOHÓLICA ES MAYOR A
2 NO MAYOR A 5.

FOSFATASA En los pacientes con hepatitis alcohólica los niveles de fosfatasa alcalina pueden estar
ALCALINA. dentro de parámetros normales hasta alcanzar valores mayores a 1000 U/L. La
bilirrubina muestra incremento extraordinario a pesar de incremento leve en la
fosfatasa alcalina. Discreta elevación de gamma glutamil transpeptidasa (GGT). LA
FOSFATASA ALCALINA PUEDE ESTAR NORMAL O LIGERAMENTE AUMENTADA.

ALBÚMINA. Los niveles de bilirrubina sérica los pacientes con hepatitis alcohólica pueden ser
normales hasta alcanzar niveles de 20 a 40 mg/dl y los niveles séricos de albumina
pueden ser normales o estar disminuidos a valores tan bajos como 1.0 a 1.5 g/dl. LA
ALBÚMINA PUEDE ENCONTRARSE NORMAL.

GAMMA La mayoría de los pacientes con enfermedad hepática por alcohol están anémicos y
GLUTAMIL tienen cierto grado de trombocitopenia. Los leucocitos por el contrario se encuentran
TRANSPEPTIDASA. normales o aumentados, en ocasiones hasta niveles de una reacción leucemoide. Los
pacientes graves tienen el tiempo de protrombina prolongado y también pueden tener
niveles de creatinina elevados. EN LA HEPATITIS ALCOHÓLICA HAY UNA DISCRETA
ELEVACIÓN DE LA GAMMA GLUTAMIL TRANSPEPTIDASA (GGT), ES UN BUEN
MARCADOR DE INGESTA CRÓNICA DE ALCOHOL PERO ES MUY POCO ESPECÍFICO. •
También es útil en pacientes con Fosfatasa altas, ya que si GGT y fosfatasa están altas,
simbolizan que las fosfatasas son de origen hepático o ilear. Y si la GGT es normal pero
con fosfatasas altas, éstas son óseas o en el caso de embarazada, placentaria. • Es
bastante útil para controlar a los pacientes de manera ambulatoria. • "No aumenta en
todos los bebedores exagerados". • No hay correlación con cantidad de etanol, alguien
puede tomar 2 copas de vino constantemente y tener la GGT elevada. • También
aumenta en: obstrucción biliar y otras alteraciones hepáticas, con drogas
antiepilépticas, anticoagulantes y barbitúricos. "Es el mejor marcador disponible para
el SEGUIMIENTO (no Dx) del paciente con daño hepático por alcohol".

Bibliografía:
1. LONGO DL, FAUCI AS, KASPER DL, HAUSERSL, JAMESON JL, LOSCALZOJ. HARRISON. PRINCIPIOS
DE MEDICINA INTERNA, 18A EDICIÓN. MC GRAW HILL. NEW YORK, USA. 2012, PP 2592-2593. 2. GUÍA
DE PRÁCTICA CLÍNICA, DIAGNÓSTICO Y TRATAMIENTO DE LA INSUFICIENCIA HEPÁTICA CRÓNICA.
MÉXICO: SECRETARIA DE SALUD; 2009.
http://www.cenetec.salud.gob.mx/descargas/gpc/CatalogoMaestro/038_GPC_InsufHepaticaCronica/IMSS_038_08_EyR.pdf
ANÁLISIS DEL CASO CLÍNICO

IDENTIFICACIÓN DEL REACTIVO


Area: MEDICINA INTERNA
Especialidad: ONCOLOGÍA
Tema: OTROS TUMORES
Subtema: TUMORES DE TIROIDES

CASO CLÍNICO CON UNA PREGUNTA

MUJER DE 38 AÑOS DE EDAD CON DIAGNÓSTICO RECIENTE DE CÁNCER DE TIROIDES DIFERENCIADO.

femenina de 38 años de edad.

-.

diagnóstico reciente de cáncer de tiroides


diferenciado.

-.

-.

38 - EL TRATAMIENTO FUNDAMENTAL PARA ESTA PACIENTE CONSISTE EN:

RADIO YODO 131. • Se recomienda la ablación a todos los pacientes con tiroidectomía total o casi total
y vigilancia aún en microcarcinomas. • Así como en aquellos con actividad tumoral
macroscópica que no puede ser resecada por cirugía. • Los beneficios de la ablación
son: erradicar las células tiroideas incluida la actividad tumoral microscópica; facilita
el seguimiento con la tiroglobulina e incrementa el intervalo del periodo libre de
enfermedad. • Los pacientes deber ser referidos después de la cirugía a tratamiento
con dosis ablativa de radioyodo 131. EL MANEJO CON RADIOYODO 131 ES
ABLATIVO Y SE REALIZA DESPUÉS DE UNA CIRUGÍA.
RADIOTERAPIA. - Las indicaciones de la radioterapia externa son: evidencia residual de tumor micro o
macroscópico, particularmente si este residual no tiene suficiente captación de
radioyodo o con estadios IV y extensión extranodal externa. Aumenta el periodo
libre de enfermedad en estadios III y IV. Se puede utilizar en forma paliativa para
enfermedad local irresecable. En caso de metástasis óseas como tratamiento
adyuvante. LA RADIOTERAPIA SE UTILIZA PARA TUMORES RESIDUALES DESPUÉS DE
LA CIRUGÍA.

HORMONOTERAPIA. • La TSH estimula el crecimiento de las células foliculares de la tiroides incluidas las
células neoplásicas. • La supresión de TSH con dosis supra fisiológicas de
Levotiroxina ha demostrado disminución de recidiva tumoral. SE UTILIZA TRAS EL
MANEJO QUIRÚRGICO PARA EVITAR RECIDIVA.

QUIRÚRGICO. LA CIRUGÍA ES EL TRATAMIENTO PRINCIPAL EN CASI TODOS LOS CASOS DE CÁNCER


DE TIROIDES, excepto para algunos cánceres anaplásicos de tiroides. • Si se
diagnóstica cáncer de tiroides mediante una biopsia por aspiración con aguja fina
(FNA), usualmente se recomienda practicar una cirugía para extirpar el tumor y toda
o parte de la glándula tiroides remanente, a la mayoría de los pacientes se le extirpa
la tiroides (tiroidectomía casi total o tiroidectomía total) con extirpación de los
ganglios linfáticos adyacentes. • Algunos médicos recomiendan la disección del
compartimiento central del cuello (extirpación quirúrgica de los ganglios linfáticos
próximos a la glándula tiroides). Aunque no se ha demostrado que mejore la
supervivencia, pudiera disminuir el riesgo de que el cáncer regrese en el área del
cuello. También hace más fácil precisar la etapa del cáncer. Si el cáncer se ha
propagado a otros ganglios linfáticos del cuello, a menudo se hace una disección
cervical radical modificada (una cirugía más extensa para extirpar los ganglios
linfáticos del cuello). EL TRATAMIENTO IDEAL PARA EL CÁNCER BIEN DIFERENCIADO
DE TIROIDES ES EL QUIRÚRGICO.

Bibliografía:
1. GUÍA DE PRÁCTICA CLÍNICA, DIAGNÓSTICO Y TRATAMIENTO DEL TUMOR MALIGNO DE TIROIDES
(CÁNCER DE TIROIDES BIEN DIFERENCIADO). MÉXICO: SECRETARIA DE SALUD, 2009. 2. GUÍA DE
REFERENCIA RÁPIDA, DIAGNÓSTICO Y TRATAMIENTO DEL TUMOR MALIGNO DE TIROIDES
(CÁNCER DE TIROIDES BIEN DIFERENCIADO). MÉXICO: SECRETARIA DE SALUD, 2009.

http://www.cenetec.salud.gob.mx/descargas/gpc/CatalogoMaestro/166_GPC_TUMOR_MALIGNO_TIROIDEO/Gpc_tumor_tiroideo.pdf
ANÁLISIS DEL CASO CLÍNICO

IDENTIFICACIÓN DEL REACTIVO


Area: MEDICINA INTERNA
Especialidad: GERIATRÍA Y GERONTOLOGÍA
Tema: CAÍDAS Y TRANSTORNOS DE LA MARCHA
Subtema: CAÍDAS Y TRANSTORNOS DE LA MARCHA

CASO CLÍNICO CON UNA PREGUNTA

MUJER DE 66 AÑOS DE EDAD, ES LLEVADA POR SU HIJA A LA CONSULTA DE GERIATRÍA PARA SU


VALORACIÓN.

Adulto mayor de 66 años de edad.

-.

los síndromes geriátricos que siempre


deberán valorarse son: incontinencia
urinaria, demencia, inmovilidad y caídas.

la polifarmacia y la interacción
medicamentosa es otro factor de riesgo
que deberá considerarse en este tipo de
pacientes.

-.

39 - LA PREVALENCIA DE PRESENTAR UNA CAÍDA A PARTIR DE LOS 65 AÑOS DE EDAD ES DE:

0A Las caídas se pueden clasificar en dos grupos fundamentales, las accidentales y las no accidentales. •
10%. La caída accidental es cuando un factor extrínseco actúa sobre una persona que está en estado de
alerta y sin ninguna alteración para caminar originando un tropezón o resbalón con resultado de
caída. • Las caídas no accidentales pueden ser de dos tipos: aquellas en las que se produce una
situación de pérdida súbita de conciencia en un individuo activo y aquellas que ocurren en personas
con alteración de la conciencia por su estado clínico, efectos de medicamentosa o dificultad para la
deambulación.
15 A Las caídas deben ser consideradas como un problema de salud en la población adulta mayor, no solo
25%. por su frecuencia sino por sus consecuencias (físicas, sociales y psicológicas). Las caídas constituyen
uno de los síndromes geriátricos más importantes por su elevada incidencia y especialmente por las
repercusiones que va a provocar en la calidad de vida del anciano como del cuidador.

30 A LAS CAÍDAS SON LA FUENTE MÁS IMPORTANTE DE MORBILIDAD Y MORTALIDAD PARA LOS
50%. ADULTOS MAYORES, convirtiéndose en un grave problema de salud pública, por las lesiones que
producen, en la mayoría de los casos responden a una inadaptación entre la persona y su entorno de
origen multifactorial. La prevalencia de caídas en el adulto mayor varía del "30 al 50%" con una
incidencia anual de 25 a 35%. Del 10 al 25% de las caídas en el adulto mayor provocan fracturas, 5%
requiere hospitalización. Las caídas son el 30% de la causa de muerte en los mayores de 65 años. De
acuerdo a la edad y el estado de salud uno de cada tres adultos mayores sufre una caída al año. Del
15 a 28% de los adultos mayores sanos de 60 a 75 años presentan caídas y el 35% en mayores de 75
años. Hay mayor frecuencia de caídas en mujeres que en hombres con una relación en paciente de 75
años o más, 35 hombres por cada 43 mujeres. La mayoría de las caídas ocurre en casa 62%, y vía
pública 26%. El lugar en casa en que con mayor frecuencia ocurren las caídas es: la recamara 27%, el
patio 21%, el baño 14%, la escalera 13%, la cocina 10%. Los mecanismos más frecuentes de caída son:
resbalón 39%, tropiezo 27%, pérdida de equilibrio 23%.

55% Internacionalmente se ha comunicado, que en los adultos mayores que consultan en un policlínico
A general, hay cuatro síndromes que con frecuencia los médicos no pesquisan. Estos son los llamados
60%. gigantes de la geriatría estos son: incontinencia urinaria, demencia, inmovilidad y caídas. La razón de
una caída rara vez es única y la mayoría de las veces la causa es multifactorial, constituyendo un
síndrome clínico. De los fármacos usados por los adultos mayores que sufren caídas,
significativamente es más frecuente el uso de benzodiacepinas y neurolépticos. Sin diferencia en el
uso de antidepresivos y vasodilatadores.

Bibliografía:
1. GUÍA DE PRÁCTICA CLÍNICA, PREVENCIÓN DE CAÍDAS EN EL ADULTO MAYOR EN EL PRIMER
NIVEL DE ATENCIÓN. MÉXICO: SECRETARIA DE SALUD, 2008.

http://www.cenetec.salud.gob.mx/descargas/gpc/CatalogoMaestro/134_ISSSTE_08_caidas_adultomayor/EyR_ISSSTE_134_08.pdf
ANÁLISIS DEL CASO CLÍNICO

IDENTIFICACIÓN DEL REACTIVO


Area: MEDICINA INTERNA
Especialidad: ONCOLOGÍA
Tema: CÁNCER EN SANGRE (LEUCEMIAS Y LINFOMAS)
Subtema: LINFOMA

CASO CLÍNICO CON UNA PREGUNTA

HOMBRE DE 56 AÑOS DE EDAD, DE OFICIO AGRICULTOR, CON ANTECEDENTE DE SER PORTADOR DE


VIH Y LITIASIS RENOURETERAL. EN LOS ULTIMOS 6 MESES SUFRIÓ FRACTURA EN MUÑECA DERECHA Y
TIBIA EN MOMENTOS DISTINTOS. ES LLEVADO AL SERVICIO DE URGENCIAS TRAS RESBALAR Y GOLPEAR
LA CABEZA CON EL SUELO. DURANTE LA EXPLORACIÓN FÍSICA SE ENCUENTRA NEUROLOGICAMENTE
ÍNTEGRO, SÓLO SE DESPIERTA DOLOR A LA MOVILIZACIÓN DE CUELLO, POR LO QUE SE SOLICITA RX DE
COLUMNA CERVICAL. DURANTE EL ESTUDIO, EL RADIÓLOGO NOTA CAMBIOS EN LA ESTRUCTURA
CRANEAL POR LO QUE TOMA LAS SIGUIENTES PROYECCIONES.

hombre d 56 años.

VIH positivo, litiasis renoureteral, agricultor,


2 fracturas distintas en los últimos 6 meses.

-.

-.
cambios en la estructura craneal.

40 - CON BASE EN LOS ANTECEDENTES Y LOS HALLAZGOS RADIOLÓGICOS DEBERÁ


SOSPECHARSE LA ASOCIACIÓN DE LA SIGUIENTE PATOLOGÍA:

MIELOMA MÚLTIPLE. Las lesiones óseas del MIELOMA MÚLTIPLE están dadas por la proliferación de
células tumorales y la activación de osteoclastos que destruyen el hueso. Estas
lesiones suelen ser de tipo líticas y raramente se acompañan de formaciones
osteoblásticas de hueso nuevo. La osteolisis provoca por tanto una movilización
elevada de calcio óseo que suele relacionarse con datos de hipercalcemia aguda y
crónica. Las lesiones óseas localizadas pueden aumentar de tamaño hasta el punto
de aparecer tumoraciones palpables especialmente en cráneo, clavículas y
esternón; cuando hay lesiones en columna pueden provocar compresión medular
como consecuencia de colapsos vertebrales. LA RADIOGRAFÍA DEL PACIENTE
MUESTRA IMÁGENES CLÁSICAS DE MIELOMA MÚLTIPLE EN SACABOCADOS QUE
SE RELACIONA CON LESIONES LÍTICAS EN CRÁNEO. IMPORTANTE: Los estudios de
RX documentan lesiones líticas, osteoporosis o fracturas en 70% de los casos de
mieloma múltiple. Los pacientes con Mieloma Múltiple pueden presentar fracturas
anormales (huesos largos, aplastamiento vertebral) en el 50% de los casos durante
la evolución de su enfermedad.

LINFOMA NO El MIELOMA MÚLTIPLE es una enfermedad neoplásica que afecta a la población


HODGKIN. adulta y que se caracteriza por infiltración de células malignas a médula ósea y
otros tejidos. El 90% de los casos se presenta en mayores de 50 años. Los
pacientes con Virus de la Inmunodeficiencia Humana (VIH), Virus de la Hepatitis
“C” (VHC) agricultores, carpinteros, curtidores de piel y aquellos que tienen
contacto con derivados del petróleo; tienen mayor riesgo de presentar Mieloma
Múltiple.

MACROGLOBULINEMIA Las variantes clínicas del Mieloma Múltiple son: 1. Mieloma sintomático. 2.
DE WALDENSTRÖM. Mieloma asintomático (indolente). 3. Mieloma no secretor. 4. Leucemia de células
plasmáticas. Los criterios diagnósticos de un mieloma múltiple sintomático
incluyen: a. Proteína monoclonal sérica o urinaria. b. Células plasmáticas clonales
en médula ósea o plasmacitoma. Daño a órgano o tejido blanco: anemia,
hipercalemia, insuficiencia renal, hiperviscosidad, amiloidosis, daño óseo o
infecciones recurrentes.

AMILOIDOSIS. En enfermos que tengan anemia crónica normocitica normocrómica, dolor óseo,
hipercalcemia o daño renal de causa no documentada y/o lesiones osteolíticas hay
que considerar el diagnóstico diferencial de Mieloma Múltiple. PARA DIRIGIR EL
DIAGNÓSTICO DEBES CONSIDERAR: - FACTOR DE RIESGO: SER PORTADOR DE VIH
Y AGRICULTOR. - ANTECEDENTES: FRACTURAS (aunque no se describe la causa
puede orientar la sospecha diagnóstica). - LITIASIS RENOURETERAL:
PROBABLEMENTE CAUSADO POR PRECIPITACIÓN DE CALCIO A NIVEL RENAL. -
DATOS RADIOGRÁFICOS DE MIELOMA MÚLTIPLE.

Bibliografía:
1. GUÍA DE PRÁCTICA CLÍNICA, DIAGNÓSTICO Y TRATAMIENTO DE MIELOMA MÚLTIPLE. MÉXICO:
SECRETARIA DE SALUD; 2010. 2. LONGO DL, FAUCI AS, KASPER DL, HAUSERSL, JAMESON JL,
LOSCALZOJ. HARRISON. PRINCIPIOS DE MEDICINA INTERNA, 18A EDICIÓN. MC GRAW HILL. NEW
YORK, USA. 2012. 3. PAPADAKIS MAXINE A, MCPHEE STEPHEN J. DIAGNÓSTICO CLÍNICO Y
TRATAMIENTO. 52ª EDICIÓN. MC GRAW HILL EDUCATION, LANGE. USA. 2013.

http://www.cenetec.salud.gob.mx/descargas/gpc/CatalogoMaestro/409_IMSS_10_Mieloma_multiple/EyR_IMSS_409_10.pdf
ANÁLISIS DEL CASO CLÍNICO

IDENTIFICACIÓN DEL REACTIVO


Area: GINECOLOGÍA Y OBSTETRICIA
Especialidad: OBSTETRICIA
Tema: ENFERMEDADES QUE COMPLICAN EL EMBARAZO
Subtema: ENFERMEDADES MÉDICAS Y TRANSTORNOS DURANTE EL
EMBARAZO

CASO CLÍNICO CON UNA PREGUNTA

MUJER DE 28 AÑOS DE EDAD, MODELO PROFESIONAL, LA CUAL DIO A LUZ A SU PRIMER HIJO HACE 1
SEMANA. ACUDE A CONSULTA ACOMPAÑADA DE SU HERMANA, REFIRIENDO DESDE EL PARTO,
PRESENCIA DE CANSANCIO EXTREMO E INCAPACIDAD PARA CUIDAR A SU HIJO, ADEMÁS SE QUEJA DE
PÉRDIDA DE CONFIANZA Y SENTIRSE INFERIOR QUE OTRAS MUJERES POR SU ESTADO CORPORAL
ACTUAL. DURANTE EL INTERROGATORIO, LA HERMANA INTERVIENE DICIENDO QUE ESTOS
SENTIMIENTOS LOS TIENE DESDE HACE MÁS DE 2 AÑOS, CUANDO INICIO LA RELACIÓN CON EL PADRE
DE SU HIJO, PERO QUE GENERALMENTE INTENTA DISIMULARLO TRABAJANDO; DURANTE TODO ESE
TIEMPO LA HA VISTO TRISTE, CON LLANTO FÁCIL Y FRECUENTEMENTE DESESPERADA.

28 años.

7 días de puerperio.

cansancio, incapacidad para cuidar de su


bebé, pérdida de con anza, sentimientos
de inferioridad. Desde hace 2 años triste,
con llanto fácil y frecuentemente
desesperada, lo cual intenta disimular
trabajando.

41 - EL SIGUIENTE ES EL DIAGNÓSTICO MÁS PROBABLE:


DISTIMIA La DISTIMIA es el estado de ánimo crónicamente deprimido que no cumple los criterios para
el trastorno depresivo mayor, o lo hace solo en períodos muy cortos. A pesar de la fatiga o
falta de energía, del estado de ánimo bajo y de la pérdida parcial de la capacidad de
disfrutar, las personas con distimia “suelen ser capaces de hacer frente a la vida diaria”.
Criterios diagnósticos para distimia: A. Presencia de un período de, al menos, 2 años de
humor depresivo constante o constantemente recurrente. Los períodos intermedios de
ánimo normal, raramente duran más que pocas semanas y no hay episodios de hipomanía.
B. Ninguno o muy pocos episodios individuales de depresión cuando los hay; debe ser lo
suficientemente severo para cumplir los criterios de un trastorno depresivo recurrente leve.
C. Presencia de, por lo menos, tres de los siguientes síntomas, en al menos algunos de los
períodos de depresión: 1. Disminución de la energía o de la actividad. 2. Insomnio. 3.
Pérdida de la confianza en sí mismo o sentimientos de inferioridad. 4. Dificultad para
concentrarse. 5. Llanto fácil. 6. Pérdida de interés o satisfacción por la actividad sexual y
otras actividades placenteras. 7. Sentimientos de desesperanza o desesperación. 8.
Percepción de incapacidad para enfrentar las responsabilidades habituales. 9. Pesimismo
sobre el futuro o cavilaciones sobre el pasado. 10. Aislamiento social. 11. Disminución de la
locuacidad. LA CLAVE ES QUE LA PACIENTE PRESENTA SÍNTOMAS COMPATIBLES CON UN
ESTADO DE ÁNIMO CRÓNICAMENTE DEPRIMIDO DESDE HACE MÁS DE DOS AÑOS, PERO
ESO NO LE IMPIDE TRABAJAR, LO QUE SUGIERE QUE ES CAPAZ DE HACERLE FRENTE A LA
VIDA. IMPORTANTE: a diferencia de la distimia, en el trastorno depresivo mayor las
personas tienen pérdida de placer para la mayoría de cosas y un deterioro funcional
secundario.

DEPRESIÓN DEPRESIÓN POSPARTO: enfermedad depresiva no psicótica que ocurre durante el primer
POSTPARTO año posterior al parto. Para una proporción importante de mujeres, la enfermedad puede
tener su inicio en el período prenatal. El término depresión posparto no debe ser utilizado
como un término genérico para todas las enfermedades mentales que se presenten
posteriores al nacimiento. Los criterios diagnósticos generales para episodio depresivo son
los siguientes: - El episodio depresivo debe durar al menos 2 semanas. - No ha habido
síntomas hipomaníacos o maníacos suficientes para cumplir los criterios de episodio
hipomaníaco o maníaco, en ningún período de la vida del individuo. - Criterio de exclusión
usado con más frecuencia. El episodio no es atribuible al consumo de sustancias psicoactivas
o a ningún trastorno mental orgánico. LOS SÍNTOMAS EN GENERAL TIENEN UNA
DURACIÓN MAYOR A 2 AÑOS, POR LO QUE DEBERÁN CONSIDERARSE COMO
POSIBILIDADES DIAGNÓSTICAS EN TIEMPO, A LA DISTIMIA O EL TRASTORNO DEPRESIVO
MAYOR. DE CUALQUIER MODO, SI NOS REFERIMOS ÚNICAMENTE A LOS RELACIONADOS
CON EL EVENTO OBSTÉTRICO, SOLO HA TRANSCURRIDO UNA SEMANA DESDE SU INICIO,
POR LO QUE NO SE PUEDE CONSIDERAR UN EPISODIO DEPRESIVO.

TRASTORNO La DEPRESIÓN MAYOR (también llamada depresión clínica o trastorno depresivo mayor), se
DEPRESIVO caracteriza porque la persona que lo sufre presenta un estado de ánimo depresivo o la
MAYOR pérdida de placer en la mayoría de las actividades, así como una serie de síntomas
emocionales, cognitivos, físicos y de conducta asociados. La gravedad del trastorno está
determinada tanto por el número y la gravedad de los síntomas, como por el grado de
deterioro funcional. NO SE REFIERE LA EXISTENCIA DE UN DETERIORO FUNCIONAL EN LA
PACIENTE, NI PÉRDIDA DE PLACER, NI LA MAYORÍA DE SUS ACTIVIDADES. DE HECHO, LA
PACIENTE PERMANECIÓ TRABAJANDO A PESAR DE SU ÁNIMO DEPRESIVO.

MELANCOLÍA Es importante diferenciar a las madres que sufren de depresión posparto de aquellas con
PUERPERAL melancolía puerperal, ya que un diagnóstico erróneo puede llevar a un tratamiento
innecesario. Para distinguir los síntomas de la depresión sobre los síntomas de la
"melancolía del bebé" o melancolía puerperal, el tiempo (>2 semanas de duración, durante
todo el día casi todos los días) y la gravedad (deterioro funcional) deben ser evaluados. SI
LA PACIENTE NO TUVIERA EL ANTECEDENTE DE SÍNTOMAS A LO LARGO DE 2 AÑOS, ES
MUY PROBABLE QUE NOS ENCONTRÁRAMOS ANTE EL DIAGNÓSTICO DE UNA
MELANCOLÍA PUERPERAL.

Bibliografía:

http://www.cenetec-difusion.com/CMGPC/SS-666-14/ER.pdf
ANÁLISIS DEL CASO CLÍNICO

IDENTIFICACIÓN DEL REACTIVO


Area: PEDIATRÍA
Especialidad: URGENCIAS PEDIÁTRICAS
Tema: INTOXICACIONES
Subtema: INGESTIÓN POR CÁUSTICOS

CASO CLÍNICO CON UNA PREGUNTA

PREESCOLAR DE 2 AÑOS DE EDAD, SE ENCONTRABA SÓLO EN LA COCINA, PRESENTA LLANTO INTENSO


Y AGUDO. LO REVISA LA MADRE Y OBSERVA BOCA EDEMATOSA E HIPEREMIA ALREDEDOR DE ÉSTA, ASÍ
COMO EN LA MANO DERECHA. POSTERIORMENTE PRESENTA SIALORREA, TOS Y DOLOR FARÍNGEO
POR LO QUE ES LLEVADO A URGENCIAS.

preescolar de 2 años de vida

se encontraba sólo en la cocina. en la


cocina

PRESENTA LLANTO INTENSO, AGUDO,


SÚBITO. Y POSTERIORMENTE SE AGREGA
TOS Y DOLOR FARÍNGEO

PRESENCIA DE BOCA EDEMATOSA, CON


HIPEREMIA EN MANO DERECHA Y
PERIBUCAL

----

42 - SU DIAGNÓSTICO PROBABLE ES EL DE:


LESIÓN POR Las SUSTANCIAS CÁUSTICAS son compuestos químicos que provocan lesiones en los
CÁUSTICOS tejidos por contacto. Constituyen alrededor del 5% de todas las exposiciones
accidentales a sustancias tóxicas y afectan sobre todo a los niños más pequeños. El
grupo de sustancias cáusticas comprende desatascadores de cañerías lejía, tabletas
Clinitest, limpiadores de inodoros y baterías pequeñas. Las quemaduras por álcalis
provocan lesiones y necrosis por licuefacción, que penetran profundamente en los
tejidos, sobre todo en el esófago. Las quemaduras por ácidos provocan necrosis por
coagulación, afectan principalmente es estómago y provocan lesiones graves de la
mucosa gástrica. Manifestaciones clínicas. Los niños con este tipo de intoxicación se
pueden presentar con llanto continuo, rechazo de las comidas o bebidas, babeo,
ulceraciones bucofaríngeas, estridor vómitos y dolor abdominal. Las intoxicaciones
graves se pueden asociar con un síndrome de dificultad respiratoria aguda, edema de
las vías aéreas y choque. Estudios diagnósticos. Los análisis clínicos deben incluir
hemograma completo, estudio de electrolitos séricos, pruebas de función renal,
coagulograma y glucemia. En todos los niños con síntomas respiratorios están
indicadas la oximetría de pulso y una radiografía de tórax. En los pacientes con dolor
abdominal se debe ordenar una radiografía simple del abdomen para evaluar la
posibilidad de perforación y la presencia de aire libre en la cavidad abdominal. El
examen endoscópico permitirá determinar la gravedad de la lesión, dirigir el
tratamiento y establecer el pronóstico. Diagnóstico diferencial: La identificación del
agente cáustico específico (ácido o álcali), la determinación de la cantidad ingerida y la
concentración y el pH de la sustancia son esenciales para prever su toxicidad potencial.
Al igual que en otros niños con dificultad respiratoria o problemas en las vías aéreas
superiores, es necesario descartar la presencia de epiglotitis, absceso faríngeo y
traqueítis. También se debe descartar la aspiración de un cuerpo extraño. El tratamiento
inicial puede comprender la dilución inmediata con agua o leche. En la medida de lo
posible se debe consultar con un centro de toxicología local para obtener información
acerca de la dilución apropiada en el domicilio. A menudo, los padres o cuidadores bien
intencionados administran un volumen demasiado grande de agua o leche, que
provoca distensión gástrica y vómitos. En niños con ulceraciones orales o antecedentes
inequívocos de ingestión de una cantidad importante se debe considerar la posibilidad
de una esofagoscopia en las 12 horas posteriores a la ingestión. La ingestión de
desatascador de cañerías líquido puede provocar quemaduras esofágicas en ausencia
de ulceraciones bucofaríngeas. El lavado gástrico está contraindicado y la
administración de carbón activado es ineficaz. Las indicaciones para administrar
corticoides y antibióticos dependen de la gravedad de las quemaduras esofágicas.

INTOXICACIÓN Los hidrocarburos se encuentran en los limpiadores y lustradores domésticos como


POR solventes, combustibles y aditivos. La toxicidad de estos compuestos se debe
HIDROCARBUROS principalmente a la escasa tensión superficial y la baja presión de vapor asociadas, lo
que les permite extenderse sobre superficies extensas como las de los pulmones y
provocar una neumonitis química. En los niños pequeños la ingestión de hidrocarburos
por lo general es accidental, mientras que los adolescentes generalmente inhalan estos
compuestos en forma de sustancias volátiles (pegamento) o de solventes para pintura
(tolueno). Manifestaciones clínicas: La ingestión de hidrocarburos puede provocar
atragantamiento, tos y náuseas mientras se ingiere el producto y vómitos
inmediatamente después. La aspiración del producto durante la deglución puede
provocar una neumonitis pos aspiración. Se observó que la mera presencia en la
hipofaringe de una sustancia que contiene hidrocarburos puede causar una neumonitis
química secundaria a la propagación del producto hacia las superficies contiguas de la
vía aérea. Además de los hallazgos pulmonares pueden aparecer síntomas nerviosos
centrales transitorios como consecuencia de la absorción sistémica de algunos
hidrocarburos. En casos raros se producen lesiones hepáticas, renales o miocárdicas. A
menudo es difícil cuantificar la cantidad de hidrocarburos ingerida por un niño. La
aspiración intratraqueal de menos de 1 ml de algunos compuestos, como el aceite
mineral, puede provocar una neumonitis grave potencialmente fatal. Otros compuestos
son más difíciles de aspirar y no se absorben bien en el tubo digestivo. En general,
sustancias como el asfalto o el alquitrán, los lubricantes (ej., aceite industrial, aceite
doméstico, la grasa industrial densa) y la vaselina líquida no son tóxicas por vía oral.
Los adolescentes que abusan de los hidrocarburos pueden presentar efectos tóxicos
neurológicos agudos, como euforia, desinhibición, desorientación y alucinaciones. Esta
práctica también aumenta el riesgo de neumonitis química y efectos cardiotóxicos
potencialmente fatales. Lamentablemente, la incidencia del consumo abusivo de
inhalantes y la mortalidad asociada con esta práctica muestran una tendencia creciente.
El abuso crónico puede provocar complicaciones renales y secuelas cognitivas
permanentes. Estudios diagnósticos: En todo niño con antecedentes de ingestión de
hidrocarburos y síntomas respiratorios están indicadas la oximetría de pulso y una
radiografía de tórax para evaluar la presencia de hipoxia o una neumonitis por
aspiración. En un adolescente que inhaló intencionalmente una sustancia volátil están
indicados un ECG y la monitorización de los parámetros cardíacos. Además, se deben
controlar las concentraciones séricas de electrólitos, las pruebas funcionales renales y el
estado acidobásico del paciente. Diagnóstico diferencial .En todo niño con antecedentes
de ingestión de hidrocarburos es importe identificar el compuesto ingerido en la
medida en que esta información puede afectar el tratamiento y el pronóstico. En un
niño con síntomas respiratorios se debe investigar la posibilidad de neumonía,
hiperreactividad de las vías aéreas o aspiración de cuerpo extraño. También se deben
tener presentes otras sustancias químicas que se pueden asociar con un compromiso
respiratorio, como insecticidas (organofosfatos), herbicidas (paraquat) e irritantes
pulmonares (cloro gaseoso). En todo adolescente que a inhale intencionalmente
sustancias volátiles se debe investigar el consumo de otras drogas prohibidas, como
cocaína, anfetaminas, opiáceos o marihuana. El tratamiento: Los niños asintomáticos
con un examen físico normal se deben mantener en observación en el DE durante 4-6
horas. Si después de transcurrido ese lapso no se observan alteraciones, el paciente
puede recibir el alta con la recomendación de retornar si aparecen fiebre, taquipnea o
tos. El tratamiento de la neumonitis por hidrocarburos es esencialmente de apoyo. Se
desaconseja la administración profiláctica de antibióticos. La administración de
corticoides para los tratamientos de la aspiración de hidrocarburos se asoció con un
aumento de la tasa de morbilidad y tampoco se recomienda. La inducción de vómitos
con jarabe de ipecacuana o el lavado gástrico están contraindicados. La administración
de carbón activado también se desaconseja, salvo en casos en los que el compuesto
ingerido contenga un aditivo peligroso que pueda provocar toxicidad sistémica. La
ingestión de gasolina, queroseno, producto líquido para encender el carbón vegetal y
alcoholes minerales en general no se asocia con síntomas sistémicos. El tratamiento de
adolescentes que inhalaron hidrocarburos volátiles comprende las medidas de apoyo y
las maniobras de apoyo vital cardíaco convencionales según necesidad. La acción
sensibilizante cardíaca de los hidrocarburos volátiles hizo que algunos autores
desaconsejaran la administración de estimulantes cardíacos, como la epinefrina, y
propusieran utilizar B-bloqueadores para el tratamiento de las taquiarritmias
ventriculares, pero este enfoque requiere estudios más detallados.
INTOXICACIÓN El cloro es de color amarillento-verdoso, 2.5 veces más pesado que el aire y con un olor
POR CLORO penetrante. Se utiliza como agente blanqueador, en la purificación del agua (riesgo de
intoxicación en piscinas), se usa en la industria química y de plásticos (escapes o fugas
en la propia industria productora y transformadora, o durante el transporte, por rotura
de la cisterna y fuga del gas a la atmósfera), la mayor parte de intoxicaciones, sin
embargo, ocurren por mezcla de ácido clorhídrico con hipoclorito (salfumán+ lejía) o al
mezclar amoníaco con lejía que desprende vapores de cloro (cloramina) con una
potente acción cáustica. Como en todas las intoxicaciones por gases la prevalencia tiene
unos niveles bajos (alrededor de un 3% del total de las intoxicaciones graves y el 9% del
total de las intoxicaciones no medicamentosas. Es un potente agente oxidante que
destruye con rapidez y de forma amplia los tejidos con los que se pone en contacto al
formarse ácido clorhídrico. Esta propiedad se ve incrementada por su gran
hidrosolubilidad Concentraciones en el aire entre 3-6 ppm determinan un síndrome
irritativo de las mucosas. Con una concentración ambiental de 3-6 ppm se produce un
síndrome irritativo: lagrimeo, escozor en los ojos y nariz, odinofagia, tos irritativa,
opresión en tórax, hemoptisis, cefaleas, y sensación de falta de aire. En exposiciones
prolongadas tiene efecto corrosivo sobre los dientes. En intoxicaciones graves:
sibilancias y edema pulmonar tras un período de latencia de horas, con insuficiencia
respiratoria aguda grave. Aparición de un síndrome irritativo respiratorio tras
exposición a vapores por manipulación y mezcla de cáusticos. Muchos de los efectos
quedan limitados a la región nasofaríngea, donde el gas o los vapores liberados son
absorbidos y fijados por la humedad de las mucosas. Viene determinada por el
desarrollo de edema pulmonar, horas después de la exposición. Se puede presentar
hipoxemia en la gasometría arterial con necesidad de ventilación mecánica. Su
tratamiento es inespecífico (ver medidas generales de tratamiento de la intoxicación
por gases irritantes). Recientemente se ha señalado la utilidad de la administración de
bicarbonato sódico en nebulización para evitar el daño pulmonar (evaluado por el
desarrollo de edema pulmonar y necesidad de ventilación mecánica). Así se reducen las
estancias y el número de hospitalizaciones.
PICADURA POR Todos los himenópteros, presentan un temible aguijón en el último segmento de su
HIMENÓPTEROS, abdomen. Excepto la abeja, el resto de insectos del grupo retienen el aguijón y pueden
picar repetidamente. Su veneno, eficaz mecanismo de defensa, puede dar lugar a
reacciones alérgicas, aunque puede resultar mortal en individuos hipersensibilizados
ante dichas sustancias químicas. El orden Hymenoptera incluye avispas, abejas,
abejorros y hormigas, (probablemente existan más de 100,000 especies en el mismo), y
sin duda estos insectos causan al hombre más picaduras que ningún otro grupo de
animales venenosos. La gravedad del cuadro depende del número de picaduras (hasta
1000 en algunos casos), exposición anterior, edad y estado general del paciente. Los
cuadros clínicos que comprende su espectro van desde manifestaciones sólo locales a
un shock anafiláctico de aparición inmediata, o reacciones de hipersensibilidad
retardada parecidas a la enfermedad del suero que pueden aparecer 10-14 días después
de la picadura. Los venenos de estos insectos son una mezcla de sustancias de tipo
proteínico con antigenicidad variable, cininas, histamina y serotonina. La reacción del
hombre a las picaduras de insectos es muy variable. En general los efectos más graves
suelen producirse en picaduras de cabeza, cara y cuello. Puede observarse una pápula
indolora pasajera o un dolor quemante difuso intenso junto con prurito, parestesias,
zonas de anestesia y cefalea. A veces urticaria, edema y eritema; incluso edema
angioneurótico intenso. A nivel muscular puede llegar a observarse debilidad, espasmo,
contracturas o parálisis. En casos graves disnea, cianosis, disfagia, parálisis ocular,
fiebre, náuseas, alteraciones de la frecuencia y el ritmo cardíaco y shock. Mención
aparte merece la prevención; Según las directivas del Insect Allergy Committee de la
American Academy of Allergy, todos los sujetos con antecedentes de reacción grave a
picaduras de insectos deben ser desensibilizados. Para identificar de la manera más
precisa dicha hipersensibilidad deben usarse cutirreacciones con los venenos
específicos; y se debe instituir la desensibilización 14 días después de una picadura
grave. A los pacientes sensibilizados debe aconsejárseles usar ropa de manga larga,
evitar riesgos innecesarios y llevar siempre a mano un botiquín que contenga tabletas
de 10 mg de isoproterenol para uso sublingual, adrenalina en aerosol para inhalación, y
pinzas para extraer el aguijón. El tratamiento local ha de basarse en la extracción del
aguijón en caso de que haya quedado retenido, y siempre que esto sea posible; así
como el lavado de la herida con agua y jabón. Desbridar en caso de celulitis o gangrena.
La inyección de Lidocaína local puede aliviar el dolor de la zona en caso de dolor
persistente de la misma en casos no complicados. El frío tópico también es aconsejable.
En las formas leves los antihistamínicos tópicos y orales son recomendables hasta la
desaparición de la sintomatología. En caso de picaduras múltiples, la toxina total
inyectada puede ser suficiente para producir síntomas generales graves, ello puede
hacer necesaria la terapia de sostén; sedación, líquidos. La inyección de gluconato de
calcio debe valorarse si aparece sintomatología neurológica. Como medida precautoria
puede indicarse mantener al paciente en observación 6-8 horas, vigilando así cualquier
empeoramiento del estado general; y teniendo especial precaución con la aparición de
hipotensión, broncoespasmo o edema de glotis. Ciertamente el tratamiento inmediato
es la clave del éxito; y en las reacciones alérgicas no debe dudarse en la administración
de Adrenalina 0.5 ml en solución al 1/1000 vía subcutánea en intervalos entre 5 y 20
minutos dada su breve acción.

Bibliografía:
1. NELSON TRATADO DE PEDIATRÍA. BEHRMAN / KLIEGNMAN. ELSEVIER. EDICIÓN 7MA. 2004. PAG.
2371.
ANÁLISIS DEL CASO CLÍNICO

IDENTIFICACIÓN DEL REACTIVO


Area: MEDICINA INTERNA
Especialidad: NEUROLOGÍA
Tema: ENFERMEDAD DESMIELINIZANTE
Subtema: ESCLEROSIS MÚLTIPLE Y SX DE GUILLIAN BARRE

CASO CLÍNICO CON UNA PREGUNTA

MUJER DE 42 AÑOS DE EDAD, INTERNADA DESDE HACE 10 DÍAS POR DEBILIDAD PROGRESIVA,
SIMÉTRICA Y ASCENDENTE DE MIEMBROS INFERIORES E HIPORREFLEXIA, SIN FIEBRE ASOCIADA.
REFIERE CUADRO GRIPAL 15 DÍAS ANTES DEL INICIO DE LOS SÍNTOMAS. EL DÍA DE HOY SE TOMARÁ
NUEVA MUESTRA DE LÍQUIDO CEFALORRAQUÍDEO, YA QUE LA DE SU INGRESO FUE NORMAL.

¥ 42 años.

¥ cuadro gripal 15 días antes del inicio de


los síntomas.

¥ debilidad progresiva, simétrica y


ascendente de miembros inferiores e
hiporre exia.

43 - EL HALLAZGO MÁS PROBABLE EN ESTA NUEVA MUESTRA DE LÍQUIDO CEFALORRAQUÍDEO


SERÁ:

PLEOCITOSIS Ante la presencia de PLEOCITOSIS en el líquido cefalorraquídeo en pacientes con


sospecha de Síndrome de Guillain- Barré, se tiene que considerar la asociación o
presencia de otras entidades nosológicas como la borreliosis de Lyme, infección por
VIH y neoplasias, para las cuales será necesario realizar los estudios pertinentes,
según sea el caso. LA PRESENCIA DE PLEOCITOSIS DESCARTARÍA LA ENFERMEDAD DE
GUILLAIN BARRE, Y OBLIGARÍA A BUSCAR OTROS DIAGNÓSTICOS.
PROTEINORRAQUIA TOMANDO EN CUENTA QUE NUESTRO PROBABLE DIAGNÓSTICO ES EL SÍNDROME
DE GUILLIAN BARRÉ o polirradiculoneuropatía inflamatoria aguda que es una
enfermedad autoinmune asociada en la mayoría de los casos por una “infección viral”
o bacteriana. La presentación clínica habitual se caracteriza por una debilidad
simétrica de más de una extremidad, rápidamente progresiva, de comienzo distal y
avance ascendente, a veces llegando a afectar la musculatura bulbar respiratoria, así
como nervios craneales motores y que cursa con disminución o pérdida de los reflejos
osteotendinosos y con signos sensitivos leves o ausentes. Se recomienda investigar
síndrome de Guillain-Barré en aquellos pacientes que presentan los siguientes datos
clínicos: • Debilidad progresiva en más de una extremidad • Arreflexia o hiporreflexia
• Progresión simétrica y ascendente • Parestesias, disestesias • Compromiso de pares
craneales • Disfunción autonómica • Ausencia de fiebre al inicio de la enfermedad El
LCR puede no mostrar alteraciones en las primeras 48hrs de instaurada la
enfermedad, pero para la segunda semana, el 90% de los casos presenta niveles de
proteínas elevados, siendo evidente desde la primera semana en el 25% de los casos.
EN LOS PACIENTES CON GUILLAIN BARRÉ ES COMÚN ENCONTRAR ELEVACIÓN DE
LAS PROTEÍNAS EN EL LIQUIDO CEFALORRAQUÍDEO.

GLUCORRAQUIA El término GLUCORRAQUIA hace referencia a la presencia de glucosa en el líquido


cefalorraquídeo. Normalmente se espera una glucosa de 45-85mg/dl en el líquido
cefalorraquídeo. Se encuentra elevada en pacientes con diabetes y en algunos
tumores cerebrales. Se ve disminuida generalmente en procesos infecciosos, sobre
todo asociados a bacterias, micobacterias y hongos. LA GLUCOSA DEL LCR NO SUELE
VERSE AFECTADA EN SINDROME DE GUILLAIN BARRE.

PRESIÓN DE INICIO La presión de inicio durante la obtención de líquido -cefalorraquídeo se calcula en 70-
ELEVADA 180 mmH2O. Se eleva de forma muy marcada en infecciones bacterianas; moderada,
en infecciones por micobacterias u hongos; y ligeramente en infecciones por
espiroquetas o virus, en esta última puede ser normal. LA ELEVACIÓN DE LA PRESIÓN
DE INICIO ELEVADA, SUELE ASOCIARSE CON INFECCIONES.

Bibliografía:
1. DIAGNÓSTICO Y TRATAMIENTO DEL SÍNDROME DE GUILLAIN-BARRÉ EN EL SEGUNDO Y TERCER
NIVEL DE ATENCIÓN. REFERENCIA RÁPIDA: GUÍA DE PRÁCTICA CLÍNICA. MÉXICO: SECRETARÍA DE
SALUD. 2016. 2. PAPADAKIS MAXINE A, MCPHEE STEPHEN J. DIAGNÓSTICO CLÍNICO Y
TRATAMIENTO. 52ª EDICIÓN. NUEVA YORK. 2013.

http://www.cenetec-difusion.com/CMGPC/IMSS-089-09/ER.pdf
ANÁLISIS DEL CASO CLÍNICO

IDENTIFICACIÓN DEL REACTIVO


Area: MEDICINA INTERNA
Especialidad: CARDIOVASCULAR
Tema: CARDIOPATÍA ISQUÉMICA
Subtema: INFARTO AGUDO DEL MIOCARDIO

CASO CLÍNICO SERIADO

MUJER DE 71 AÑOS, QUE ACUDE AL SERVICIO DE URGENCIAS AL PRESENTAR DOLOR RETROESTERNAL


OPRESIVO DE MÁS DE 4 HRS. DE DURACIÓN, ACOMPAÑADO DE DIAFORESIS. A LA EXPLORACIÓN SE
ENCUENTRA TA 100/70 MMHG, FC 99/MIN, FR 22/MIN, RUIDOS CARDÍACOS RÍTMICOS.

MUJER DE 71 AÑOS.

-.

DOLOR RETROESTERNAL OPRESIVO DE


MÁS DE 4 HRS DE DURACIÓN,
ACOMPAÑADO DE DIAFORESIS.

TA 100/70 MMHG, FC 99/MIN, FR 22/MIN,


RUIDOS CARDÍACOS RÍTMICOS.

-.

44 - PARA REALIZAR EL DIAGNÓSTICO DEFINITIVO SOLICITARÍA NIVELES DE:


TROPONINAS. TROPONINAS Es posible encontrar troponina sérica en muchas condiciones clínicas como
insuficiencia cardíaca, hipotensión asociada a arritmias, enfermos en estado crítico,
miocarditis, pericarditis, tromboembolia pulmonar, hipertensión arterial pulmonar crónica
grave, traumatismo cardíaco, estados avanzados de insuficiencia renal, etc. • Su principal
valor es la alta sensibilidad para identificar infartos pequeños, se eleva entre 4 a 10 horas
después del inicio de los síntomas, con elevación máxima a las 12 ó 48 horas y puede
persistir hasta por 21días. • Se recomiendan tres determinaciones, una basal en urgencias,
a las 6 y 12 horas. Cualquier valor elevado de troponina se relaciona con mayor riesgo para
eventos adversos cardiovasculares. La Troponina I, como se ha dicho anteriormente, existe
en tres formas moleculares distintas (isoformas) encontrados en fibras de músculo rápidas,
fibras de músculo lentas y corazón (cTnI), ésta última expresada tanto en aurículas como
en ventrículos y siendo de mayor interés en el SCA. La cTnI tiene 30 residuos extra en el
extremo amino terminal y se libera precozmente al torrente sanguíneo después de un SCA;
parece persistir en plasma durante, al menos, 5 a 7 días. LA TROPONINA I ES EL
MARCADOR MÁS CARDIOESPECÍFICO DE TODOS POR LO QUE ES EL ÚNICO QUE HARÁ EL
DIAGNÓSTICO DEFINITIVO.

ISOENZIMA La isoenzima MB de la fosfocinasa de creatinina tiene la ventaja, respecto a la CK total, de


MB DE que no aparece en concentraciones relevantes en tejido extracardíaco y es muchísimo más
CREATININ específica. AUNQUE ES MÁS ESPECÍFICA QUE OTRAS NO ES LA PRUEBA CONFIRMATORIA
FOSFOCINASA. DE PRIMERA ELECCIÓN.

CREATININ En un plazo de 4 a 8 h, aumentan los niveles de fosfocinasa de creatinina (creatine


FOSFOCINASA. phosphokinase, CK), que se normalizarán entre las 48 y 72 h). • Un inconveniente
importante de medir la CK total es que no señala de manera específica la presencia de
STEMI (ST segment elevation myocardial infarction), dado que el nivel de la enzima puede
aumentar con el traumatismo de músculos de fibra estriada. • Después de una inyección
intramuscular, por ejemplo, aumenta dos o tres veces la cifra total de CK; tal ambigüedad
puede originar el diagnóstico erróneo de STEMI en las personas a quienes se ha aplicado
por vía intramuscular un narcótico contra el dolor retroesternal de origen no cardíaco. •
Otras causas posibles del incremento de CK total son: 1) trastornos de músculos de fibra
estriada, como distrofias musculares, miopatías y polimiositis; 2) cardioversión eléctrica; 3)
hipotiroidismo; 4) accidente apoplético; 5) operaciones quirúrgicas, y 6) daño de músculo
estriado a causa de traumatismo, convulsiones e inmovilización duradera. ES MUY
INESPECÍFICA POR LO QUE NO SE DEBE TOMAR EN CUENTA PARA EL DIAGNÓSTICO
CONFIRMATORIO.

MIOGLOBINA. MIOGLOBINA • La mioglobina es liberada en la sangre unas horas después de haber


comenzado el IAM. • Dicho pigmento es uno de los primeros marcadores cardíacos que
aumentan por encima de los límites normales después del IAM, pero no tiene especificidad
cardíaca y se excreta rápidamente en la orina, de modo que los niveles en sangre retornan
a sus intervalos normales en las primeras 24 h de haber comenzado el infarto. • Es el
biomarcador más rápido (1 a 2 horas) para demostrar lesión celular aguda, su elevación
máxima se observa entre las 6 y 12 horas después del inicio de los síntomas y regresa a la
normalidad en las siguientes 24 horas. AUNQUE ES UNA DE LOS PRIMEROS MARCADORES
EN APARECER ES MUY INESPECÍFICA POR LO QUE NO SE PUEDE MANEJAR PAR EL
DIAGNÓSTICO DEFINITIVO.

Bibliografía:
1. LONGO DL, FAUCI AS, KASPER DL, HAUSERSL, JAMESON JL, LOSCALZOJ. HARRISON. PRINCIPIOS
DE MEDICINA INTERNA, 18A EDICIÓN. MC GRAW HILL. NEW YORK, USA. 2012, PP 2023.

45 - EL SIGUIENTE EVENTO FISIOPATOLÓGICO EXPLICA LA SITUACIÓN CLÍNICA DE LA PACIENTE:


CAMBIOS EN La activación plaquetaria induce un cambio conformacional en la plaqueta, que conlleva
RECEPTORES la exposición y la activación de los receptores de la GP IIb/IIIa plaquetarios almacenados
IIB/IIIA. en el interior de la plaqueta, hacia su superficie. Una vez activado el receptor IIb/IIIa se
une al fibrinógeno que es su ligando primario, las plaquetas se agregan y forman un
verdadero trombo. • La conversión del fibrinógeno en fibrina por parte de la trombina,
estabilizará finalmente el trombo. La unión del fibrinógeno al receptor IIb/IIIa es el único
mediador de la agregación plaquetaria y es independiente de los mecanismos de
activación plaquetaria. Por lo tanto, a la unión del fibrinógeno al receptor IIb/IIIa se le ha
llamado la vía final común de la formación del trombo plaquetario. LA ACTIVACIÓN DE
LOS RECEPTORES IIB/IIIA ESTÁ INVOLUCRADA EN LA GÉNESIS DEL TROMBO
PLAQUETARIO INDEPENDIENTEMENTE DE LA AFECCIÓN CORONARIA QUE PROVOQUE.

ACTIVACIÓN DE Los tromboxanos A2 estimulan la acción trombocitaria y tienen un efecto vasoconstrictor.


TROMBOXANO Son los antagonistas de las prostaciclinas. • El tromboxano A2 se libera después de la
A2. adhesión de los trombocitos al endotelio vascular dañado. EL PROCESO DE ACTIVACIÓN
DEL TROMBOXANO A2 ES SECUNDARIO A LA PRESENCIA DE UN ENDOTELIO DAÑADO.

ACTIVACIÓN Las plaquetas se adhieren al colágeno, al subendotelio y a las superficies artificiales.


PLAQUETARIA. Después de adherirse se explayan en las superficies. En la mayoría de los casos la
secuencia es adhesión, disminución y liberación del contenido de los gránulos. LAS
PLAQUETAS NO SE ADHIEREN AL ENDOTELIO NI A LAS CÉLULAS ENDOTELIALES
NORMALES, ÉSTAS DEBEN HABER SUFRIDO UN DAÑO PREVIO.

RUPTURA DE En los casos frecuentes de IM se invoca la siguiente secuencia de acontecimientos: 1. El


PLACA primer fenómeno es el cambio morfológico brusco de la placa ateromatosa (es decir, la
ATEROMATOSA. rotura), que se manifiesta por hemorragia dentro de la placa, erosión o ulceración, o por
rotura o fisuración. 2. Al quedar al descubierto el colágeno subendotelial y el contenido
de la placa necrosada, se produce adhesión, agregación, activación de potentes factores
agravantes de las plaquetas como el tromboxano A2, serotonina, factores plaquetarios 3
y 4; se estimula el vaso espasmo. 3. Otros mediadores activan la vía extrínseca de la
coagulación mediante tras la ruptura del trombo. 4. Con frecuencia el Trombo evoluciona
en cuestión de minutos, y obstruye totalmente la luz del vaso coronario responsable.
TODOS LOS MECANISMOS MENCIONADOS ANTES SON CONSECUENCIA DE LA RUPTURA
DE PLACA ATEROMATOSA DE OTRO MODO. POR LO QUE ÉSTA ES LA VERDADERA
GÉNESIS DEL SÍNDROME CARDIOVASCULAR.

Bibliografía:
1. LONGO DL, FAUCI AS, KASPER DL, HAUSERSL, JAMESON JL, LOSCALZOJ. HARRISON. PRINCIPIOS
DE MEDICINA INTERNA, 18A EDICIÓN. MC GRAW HILL. NEW YORK, USA. 2012, PP 2021-2022.

46 - EL SIGUIENTE TRATAMIENTO SERÍA EL MÁS INDICADO EN ESTE PACIENTE:


NITROGLICERINA NITRATOS • En el escenario de un infarto tienen dos efectos importantes, por una parte
ENDOVENOSA. reducen pre y postcarga a través de vasodilatación arterial y venosa, mejoran el flujo
coronario mediante relajación de las arterias epicárdicas y dilatación de la circulación
colateral con lo que se obtiene una mejor proporción de flujo epicárdico y
subendocárdico. • La nitroglicerina y todos sus derivados han demostrado efectos
benéficos en enfermedades vasculares caracterizadas por disfunción endotelial y
deficiencia de óxido nítrico. Sin embargo, en la fase aguda la evidencia actual establece
un beneficio clínico modesto, requiriéndose tratar 1,000 pacientes para evitar 3 ó 4
fallecimientos. • No se recomiendan con tensión arterial sistólica < 90 mm Hg,
bradicardia o taquicardia. En presencia de un infarto con extensión al ventrículo
derecho con datos clínicos y/o ecocardiográficos de disfunción ventricular importante
pueden inducir hipotensión grave ya que este ventrículo requiere una precarga
adecuada para mantener el gasto. • La nitroglicerina (0.4 mg) y el dinitrato de
isosorbide (5 mg) se utilizan comúnmente por vía sublingual. En la fase aguda la
absorción por esta vía tiene limitaciones ya que la salivación se encuentra alterada por
la actividad simpática, aumento de la frecuencia respiratoria, etcétera. Un bolo
endovenoso de dinitrato de isosorbide es más efectivo (1 mg) y seguro que esta vía. Una
infusión permite al clínico titular la dosis de acuerdo a la tensión arterial y frecuencia
cardíaca del paciente. NO ESTÁ INDICADA DE PRIMERA LÍNEA, DEPENDERÁ MUCHO DE
LA EVOLUCIÓN DEL PACIENTE SI SE INDICA COMO COADYUVANTE EN EL MANEJO DEL
PACIENTE. “Los Nitratos se pueden utilizar en fase aguda por vía endovenosa para
manejo de síntomas, hipertensión arterial sistémica y congestión pulmonar, aunque el
impacto sobre la mortalidad es bajo (disminución de 5%)."

ACTIVADORES TERAPIA FIBRINOLÍTICA • Por la accesibilidad y significativa reducción en mortalidad y


DE eventos adversos demostrada en cientos de miles de pacientes, LA TERAPIA
PLASMINÓGENO. FIBRINOLÍTICA COMO EL "TRATAMIENTO ESTÁNDAR DE ORO DEL INFARTO AGUDO". •
Con la fibrinólisis el objetivo principal es restaurar inmediatamente el libre tránsito por
la arteria coronaria. • En Estados Unidos, la Food and Drug Administration ha aprobado
fibrinolíticos como el activador de plasminógeno hístico (tissue plasminogen activator,
tPA), la estreptocinasa, la tenecteplasa (TNK) y la reteplasa (rPA) para empleo
intravenoso en casos de STEMI. • Los fármacos en cuestión "estimulan la conversión de
plasminógeno en plasmina", que ejercerá una acción lítica en los trombos de fibrina.

HEPARINAS DE HEPARINA Las principales ventajas sobre la heparina convencional son: a) inhibir tanto
BAJO PESO la actividad de la trombina como la del factor Xa, con lo que se disminuye la generación
MOLECULAR. de trombina. b) inducir una mayor liberación del inhibidor de la vía del factor tisular y c)
no es neutralizada por el factor plaquetario 4. • En pacientes con cualquier variedad de
SCA, el empleo de heparinas de bajo peso molecular más aspirina han reducido
significativamente la mortalidad cuando se han comparado con aspirina sola.

INHIBIDORES Los inhibidores de la enzima convertidora de angiotensina se han utilizado ampliamente


DELA ENZIMA en el tratamiento de los supervivientes de infartos de miocardio, pacientes con
CONVERTIDORA hipertensión o cardiopatía isquémica crónica incluyendo a la angina de pecho y aquéllos
DE con riesgo vascular, como pacientes con diabetes. En un gran estudio clínico (Heart
ANGIOTENSINA. Outcomes Prevention Evaluation Study, Estudio sobre Prevención y Valoración de los
Resultados cardíacos) se demostró que la administración hasta de 10 mg/día de
ramiprilo reduce los eventos adversos mayores (muerte, infarto del miocardio y
apoplejía) de la angina y la necesidad de revascularizar en un grupo de pacientes de alto
riesgo con ateroesclerosis cardiovascular (incluyendo cardiopatía isquémica crónica) y
función normal del ventrículo izquierdo. ESTÁN INDICADOS EN EL MANEJO POSTERIOR
DEL INFARTO.

Bibliografía:
1. LONGO DL, FAUCI AS, KASPER DL, HAUSERSL, JAMESON JL, LOSCALZOJ. HARRISON. PRINCIPIOS
DE MEDICINA INTERNA, 18A EDICIÓN. MC GRAW HILL. NEW YORK, USA. 2012, PP 2027-2028.

FIN DEL CASO CLÍNICO SERIADO


ANÁLISIS DEL CASO CLÍNICO

IDENTIFICACIÓN DEL REACTIVO


Area: MEDICINA INTERNA
Especialidad: PSIQUIATRÍA
Tema: ESQUIZOFRENIA
Subtema: ESQUIZOFRENIA

CASO CLÍNICO SERIADO

MUJER DE 22 AÑOS DE EDAD QUE ACUDE AL SERVICIO DE URGENCIAS, CUENTA CON EL ANTECEDENTE
DE HACE UN MES DE HABER PRESENTADO INTENTO SUICIDA DE ALTA LETALIDAD. SUS PADRES LA
REFIEREN AISLADA, SIN AMIGOS, CON RENDIMIENTO ACADÉMICO BUENO HASTA HACE 8 MESES EN
QUE ABANDONÓ LA ESCUELA ADUCIENDO QUE SUS COMPAÑEROS CONTROLABAN SUS
PENSAMIENTOS Y LA OBLIGABAN A AUTOLESIONARSE. A LA EXPLORACIÓN SE ENCUENTRA INQUIETA,
DESESPERADA, MUESTRA SOLILOQUIOS Y SUSPICACIA. NO ESTABLECE CONTACTO VISUAL CON EL
MÉDICO ENTREVISTADOR.

mujer de 22 años.

un mes presento intento suicida, padres la


describe aislada, sin amigos.

ideas delirantes de que sus compañeros


controlaban sus pensamientos y le daban
órdenes.

inquieta, desesperada, mutismo y se


observa suspicaz incluso sin contacto
visual, SOLILOQUIOS.

-.

47 - EL DIAGNÓSTICO CLÍNICO MÁS PROBABLE ES:

PERSONALIDAD El único síntoma que acompaña a la PERSONALIDAD LÍMITE seria la presencia de


LÍMITE. micropsicosis pero por la duración de un mes se descartaría, pero no se mencionan los
esfuerzos frenéticos para evitar el abandono, las relaciones interpersonales inestables, la
alteración de la identidad, inestabilidad afectiva, sentimientos crónicos de vacío, lo que
si igual llamaría la atención sería el intento de suicido, pero no se presenta en un
contexto de impulsividad.
AUTISMO. Siempre para los diagnósticos psiquiátricos debemos descartar los transtonos
generalizados del desarrollo, para los cuales necesitamos antecedentes de su
psicodesarrollo, como síntomas específicos de la interacción social, la alteración de la
comunicación y los patrones de comportamiento, intereses y actividades restingidos,
repetitivos y estereotipados generando en conjunto un funcionamiento anormal antes
de los 3 años. POR LA EDAD DE PRESENTACIÓN SE DESCARTA.

ESQUIZOFRENIA. La ESQUIZOFRENIA es un trastorno psiquiátrico crónico y grave, clasificado dentro del


grupo de los trastornos psicóticos. Es un trastorno del pensamiento manifestado en
diferentes áreas como la del afecto, la percepción, la motivación, la conducta y la
actividad motora. Los síntomas afectan múltiples procesos psicológicos y causan
percepciones anormales del entorno en forma de alucinaciones, verificación de la
realidad a través de juicios aberrantes que resultan en creencias extraordinarias y
delirantes (delirios), procesos del pensamiento (asociaciones laxas), sentimientos (afecto
plano, afecto inapropiado), conducta (catatonia, desorganización), cognición (atención,
concentración), motivación y juicio. Los pacientes además padecen problemas
cognositivos que afectan la memoria, a las funciones ejecutivas y pueden presentar un
comportamiento extraño o desorganizado. De acuerdo al DSM-IV debe persistir por lo
menos durante 6 meses, con por lo menos 1 mes de síntomas de fase activa (dos o más:
"ideas delirantes", alucinaciones, lenguaje desorganizado, comportamiento gravemente
desorganizado, o catatónico y "síntomas negativos"). ENTRE LOS CRITERIOS
DIAGNÓSTICOS DEL DSM IV-TR PARA ESQUIZOFRENIA SE HABLA DE UN PERÍODO DE
UN MES DE SÍNTOMAS COMO SON IDEAS DELIRANTES, ALUCINACIONES LAS CUALES
ESTA PACIENTE PRESENTA AMBOS, LA DISFUNCIÓN ESCOLAR Y SOCIAL ES CLARA, ASÍ
COMO NO SE REFIEREN SÍNTOMAS AFECTIVOS, ABUSO DE SUSTANCIAS NI LA
RELACIÓN CON UN TRASTORNO GENERALIZADO DEL DESARROLLO LO CUAL FUE
COMENTADO EN LA RESPUESTA ANTERIOR.

TRASTORNO DE El TRASTORNO DE IDEAS DELIRANTES nos habla de nunca cumplir el criterio A para
IDEAS esquizofrenia, el cual habla de las alucinaciones, lenguaje desorganizado,
DELIRANTES. comportamiento catatónico o síntomas negativos de la enfermedad durante un mes a
seis meses, en este trastorno las ideas no son extrañas, la actividad psicosocial no esta
deteriorada significativamente por lo que diferenciamos de la respuesta corrrecta.

Bibliografía:
1. GUÍA DE PRÁCTICA CLÍNICA, DIAGNÓSTICO Y TRATAMIENTO DE LA ESQUIZOFRENIA EN EL
PRIMER Y SEGUNDO NIVEL DE ATENCIÓN. MÉXICO: SECRETARIA DE SALUD, 2009.

http://www.cenetec.salud.gob.mx/descargas/gpc/CatalogoMaestro/222-09_Esquizofrenia/SSA-222-09_Esquizofrenia_-_RER_xCorregidax.pdf

48 - LA CONDUCTA MÉDICA QUE DEBERÁ ESTABLECERSE COMO MEDIDA TERAPÉUTICA EN ESTE


CASO.

ANTIPSICÓTICOS Los ANTIPSICÓTICOS son la base del manejo del paciente con ESQUIZOFRENIA, se
ATÍPICOS. utilizan tanto para el tratamiento inmediato, como para el de mantenimiento. Su
principal eficacia está dirigida al control de las alucinaciones, ideas delirantes y
trastornos ideatorios. En general, estos medicamentos se unen a los receptores D2 y
D3 de la dopamina. El tratamiento de primera elección en la ESQUIZOFRENIA en el
primer episodio es con ANTIPSICÓTICOS ATÍPICOS, recomendados basándose en el
hecho de mejor tolerabilidad y riesgo reducido de disquinesia tardía. IMPORTANTE:
en pacientes con riesgo suicida elevado y persistente, se debe considerar el uso
temprano de clozapina.

TERAPIA La TERAPIA COGNITIVO- CONDUCTUAL muestra más beneficios que los cuidados
COGNITIVO- estándar en la prevención de progresión de la psicosis en la intervención precoz,
CONDUCTUAL. disminuyendo la prescripción de medicación antipsicótica y reduciendo los síntomas.
SON UN COMPONENTE IMPORTANTE DEL TRATAMIENTO PERO NO BASTA CON ESTE
DE PRIMERA INTENSIÓN.
BENZODIAZEPINAS. En el primer episodio no se deben usar antipsicóticos típicos ni benzodiazepinas para
tratar la ansiedad y las alteraciones del sueño.

ESTABILIZADORES Si el episodio de ezquizofrenia es de tipo maníaco, se debe iniciar con un


DEL ÁNIMO. estabilizador del ánimo junto con un antipsicótico atípico a bajas dosis. Si es de tipo
depresivo, iniciar con dosis bajas de atípico con un inhibidor de la recaptura de
serotonina. EN CUALQUIER TIPO DE ESPISODIO DE EZQUIZOFRENIA EL
TRATAMIENTO DE BASE SERÁN LOS ANTIPSICÓTICOS ATÍPICOS.

Bibliografía:
1. GUÍA DE PRÁCTICA CLÍNICA, DIAGNÓSTICO Y TRATAMIENTO DE LA ESQUIZOFRENIA EN EL
PRIMER Y SEGUNDO NIVEL DE ATENCIÓN. MÉXICO: SECRETARIA DE SALUD, 2009.

http://www.cenetec.salud.gob.mx/descargas/gpc/CatalogoMaestro/222-09_Esquizofrenia/SSA-222-09_Esquizofrenia_-_RER_xCorregidax.pdf

FIN DEL CASO CLÍNICO SERIADO


ANÁLISIS DEL CASO CLÍNICO

IDENTIFICACIÓN DEL REACTIVO


Area: PEDIATRÍA
Especialidad: INFECTOLOGIA PEDIÁTRICA
Tema: INFECCIONES NEONATALES
Subtema: SEPSIS Y CHOQUE SÉPTICO

CASO CLÍNICO CON UNA PREGUNTA

RECIÉN NACIDO OBTENIDO POR PARTO EN UN CENTRO DE SALUD, A LOS TRES DÍAS DE NACIDO SE
PRESENTA FIEBRE DE 38.7 °C, RECHAZO A LA VÍA ORAL E IRRITABILIDAD. ACUDE A URGENCIAS
ENCONTRÁNDOSE A LA EXPLORACIÓN FC DE 180XMIN, FR 60XMIN, TEMPERATURA DE 39°C, IRRITABLE Y
CON DESHIDRATACIÓN LEVE.

Tres días de vida, muy importante mayor o


menor de tres días

Obtenido por parto en un centro de salud

Rechazo a la vía oral e irritabilidad

Temperatura de 39 °C, taquicardico,


polipneico, irritable y deshidratado.
Padecimiento sistémico

49 - SE INICIA SU TRATAMIENTO EMPÍRICO CON LOS SIGUIENTES ANTIBIÓTICOS:


AMPICILINA+ La SEPSIS se incluye dentro del síndrome de respuesta inflamatoria sistémica (SRIS), en
GENTAMICINA presencia o como resultado de infección sospechada o confirmada. El espectro clínico de la
sepsis comienza cuando una infección sistémica (bacteremia, viremia o fungemia) o
localizada (meningitis, neumonía, pielonefritis, entre otras) produce una afección
sistémica y pueden progresar a una sepsis grave, choque séptico y muerte. IMPORTANTE:
Lo primero, es determinar que se trata de un padecimiento con compromiso sistémico que
involucra al sistema cardiovascular, respiratorio y neurológico. La presencia de fiebre a
esta edad, más el compromiso sistémico, nos confirma desde el punto de vista clínico que
se trata de una SEPSIS. Cuando hablamos de una SEPSIS NEONATAL se espera que el
esquema antibiótico cubra gérmenes Gram Positivos y negativos, y también Listeria, por lo
que se utiliza por lo general AMPICILINA Y AMINOGLUCÓSIDOS. La recomendación de la
GPC establece la asociación de AMPICILINA + GENTAMICINA, que tienen además acción
sinérgica. OJO: la administración de gentamicina necesita monitoreo de niveles
antibióticos por su nefrotoxicidad y ototoxicidad dosis-dependiente, lo que no ocurre con
la cefotaxima.

AMPICILINA+ La utilización de AMPICILINA + CEFOTAXIMA está indicada cuando existe meningitis


CEFOTAXIMA asociada (o no se pudo obtener LCR y existe sospecha de meningitis asociada) y cuando
hay colonización del canal vaginal materno por H. influenzae al final de la gestación. NO
EXISTE EVIDENCIA DE NEUROINFECCIÓN O ALGUNA OTRA INDICACIÓN PARA INICIAR
ESTE ESQUEMA.

VANCOMICINA SE INDICA en neonatos mayores de 7 días y menores de 3 meses, en los cuales exista el
+ AMIKACINA antecedente de una vía central, se recomienda sustituir a la ampicilina por Cloxacilina o
vancomicina, quedando el esquema CLOXACILINA + AMINOGLUCÓSIDO o VANCOMICINA
+ AMINOGLUCÓSIDO. NO APLICA PARA ESTE CASO, YA QUE TIENE 3 DÍAS DE VIDA POR
LO QUE NO ESTAN INDICADOS A ESTE EDAD. ADEMÁS DE QUENO SE REFIERE CON EL
ANTECEDENTE DE COLOCACIÓN DE VÍA CENTRAL.

AMPICILINA + Como alternativa a los aminoglucósidos puede emplearse ceftazidima o IMIPENEM-


IMIPENEM. CILASTINA, que son efectivos frente a microorganismos Gram negativos, incluyendo
Pseudomonas, aunque NUNCA DEBEN UTILIZARSE DE FORMA RUTINARIA.

Bibliografía:
1. PREVENCIÓN, DIAGNÓSTICO Y TRATAMIENTO DE SEPSIS Y CHOQUE SÉPTICO DEL RECIÉN
NACIDO, EN EL SEGUNDO Y TERCER NIVEL DE ATENCIÓN. EVIDENCIAS Y RECOMENDACIONES.
GUÍA DE PRÁCTICA CLÍNICA. MÉXICO: SECRETARÍA DE SALUD; NOVIEMBRE 2012. 2. MANUAL DE
CUIDADOS NEONATALES. JOHN P. CLOHERTY. MOSSON. EDICIÓN 4A. 2005. PÁG. 332-333

http://www.cenetec-difusion.com/CMGPC/SS-283-12/ER.pdf
ANÁLISIS DEL CASO CLÍNICO

IDENTIFICACIÓN DEL REACTIVO


Area: MEDICINA INTERNA
Especialidad: ONCOLOGÍA
Tema: CÁNCER DE PIEL
Subtema: CARCINOMA ESPINO CELULAR Y MELANOMA

CASO CLÍNICO CON UNA PREGUNTA

HOMBRE DE 55 AÑOS, DE OCUPACIÓN CAMPESINO, ORIGINARIO DEL ESTADO DE SINALOA. ACUDE A


CONSULTA CON LESIÓN HIPERCRÓMICA EN LA PLANTA DEL PIE DERECHO. USTED CONSIDERA EL
DIAGNÓSTICO DE PROBABLE MELANOMA.

HOMBRE DE 55 AÑOS.

CAMPEsINO.

LESIÓN HIPERCRÓMICA EN LA PLANTA


DEL PIE DERECHO.

DX PBLE MELANOMA.

-.

50 - EN ESTE CASO PARA REALIZAR EL DIAGNÓSTICO SE DEBERÁ INDICAR UNA:


BIOPSIA EN El MELANOMA es un tumor maligno originado a partir de los melanocitos epidérmicos.
SACABOCADO. Suele desarrollarse en la piel y en raras ocasiones puede hacerlo en otras localizaciones
como los epitelios de las mucosas, en la retina o las leptomeninges. Se presenta como una
neoformación plana exofítica por lo regular pigmentada, curable en etapas iniciales pero a
diferencia de otros tipos de cáncer cutáneo, el melanoma es el tumor que muestra una
marcada tendencia a producir metástasis linfáticas o hematógenas. A menudo tiene un
crecimiento temprano lento durante la cual la lesión es curable si se detecta y se retira
oportunamente. Se presenta predominantemente de individuos de tez blanca y se plantea
que su incidencia causal esta correlacionada con la exposición al sol. La edad promedio de
una persona con diagnóstico de melanoma es de 45 años. IMPORTANTE: DEBIDO A SU
TENDENCIA PRODUCIR METÁSTASIS SE REMIENDA LA RESECCIÓN COMPLETA DE
CUALQUIER LESIÓN SOSPECHOSA. DIAGNÓSTICO DEL MELANOMA. La BIOPSIA POR
SACABOCADO o por incisión, deben limitarse a lesiones localizadas en áreas anatómicas
funcionales específicas, como palmas, plantas, dedos, regiones subungulares, genitales,
áreas con gran riesgo de compromiso cosmético o funcional, como cara y orejas, o lesiones
de gran tamaño en donde no sea posible la biopsia por escisión con cierre primario del
defecto. En estos casos, la biopsia debe tomarse de las áreas más representativas de la
lesión (zonas más pigmentadas, elevadas, nódulos) y debe ser guiada por dermatoscopía.
PUEDE REALIZARSE LA BIOPSIA EN SACABOCADO EN PLANTA DEL PIE PERO NO ES LA
PRIMERA OPCIÓN.

BIOPSIA POR Las BIOPSIA POR AFEITADO profundo son aceptadas sólo en lesiones melanocíticas
RASURADO. benignas o con baja sospecha de melanoma y en casos de léntigo maligno, en el cual no se
requieren biopsias tan profundas porque el tumor está confinado a la epidermis. LA
BIOPSIA POR RASURADO SÓLO ESTÁ INDICADA EN LESIONES MELANOCÍTICAS BENIGNAS
CON BAJA SOSPECHA DE MALIGNIDAD.

BIOPSIA Las lesiones de gran tamaño en áreas anatómicas especiales (genitales, dedos, región
INCISIONAL. subungular, cara, orejas, palmas, plantas) sin posibilidad de cierre primario está indicada la
BIOPSIA INCISIONAL amplia del área más representativa de la lesión (nódulos, región más
pigmentada o elevada) orientada por dermatoscopÍa. SI BIEN LA BIOPSIA INCISIONAL
ESTÁ INDICADA EN ÉSTE TIPO DE PACIENTE POR SER LA LESIÓN EN EL PIE, NO ES LA
PRIMERA OPCIÓN GENERAL ANTE LA SOSPECHA DE MELANOMA.

BIOPSIA "La American Academy of Dermatology y la National Comprehensive Cancer Network


EXCISIONAL. (NCCN) recomiendan realizar biopsia por escisión con márgenes laterales de 1 a 3 mm en
todas las lesiones sospechosas de melanoma." • En las lesiones con posibilidad de cierre
primario del defecto biopsia por escisión con márgenes laterales de 1 a 3 mm. LA BIOPSIA
EXCISIONAL ESTÁ INDICADA EN "TODA" LESIÓN SOSPECHOSA DE MELANOMA COMO
PRIMERA OPCIÓN, ESTA CONSTITUYE EN MUCHAS OCASIONES EL TRATAMIENTO DE LAS
LESIONES QUE RESULTAN BENIGNAS O DE AQUELLAS DELIMITADAS A ESA ÁREA.

Bibliografía:
1. PREVENCIÓN PRIMARIA Y DETECCIÓN OPORTUNA DEL MELANOMA CUTÁNEO EN POBLACIÓN
GENERAL EN EL PRIMER NIVEL DE ATENCIÓN MÉDICA. REFERENCIA RÁPIDA: GUÍA DE PRÁCTICA
CLÍNICA. MÉXICO: SECRETARÍA DE SALUD. 03/11/2016. 2. GUÍA DE PRÁCTICA CLÍNICA, ABORDAJE
DIAGNÓSTICO DE MELANOMA MALIGNO. MÉXICO: SECRETARIA DE SALUD, 2010. 3. LONGO DL,
FAUCI AS, KASPER DL, HAUSERSL, JAMESON JL, LOSCALZOJ. HARRISON. PRINCIPIOS DE MEDICINA
INTERNA, 18A EDICIÓN. MC GRAW HILL. NEW YORK, USA. 2012, PP 726, 728. 4. PAPADAKIS MAXINE A,
MCPHEE STEPHEN J. DIAGNÓSTICO CLÍNICO Y TRATAMIENTO. 52ª EDICIÓN. NUEVA YORK. 2013, PP
102-103.

http://www.cenetec-difusion.com/CMGPC/IMSS-099-08/ER.pdf
ANÁLISIS DEL CASO CLÍNICO

IDENTIFICACIÓN DEL REACTIVO


Area: PEDIATRÍA
Especialidad: URGENCIAS PEDIÁTRICAS
Tema: URGENCIAS AMBIENTALES
Subtema: AHOGAMIENTO Y CASI AHOGAMIENTO

CASO CLÍNICO SERIADO

PREESCOLAR FEMENINO DE 5 AÑOS DE EDAD. ES LLEVADA A URGENCIAS POR CAER EN UNA ALBERCA
Y TENER CASI AHOGAMIENTO POR INMERSIÓN.

preescolar de 5 años de edad

cayó en una alberca, con casi ahogamiento


por inmersión

--

--

--

51 - LA PACIENTE DEBE DE ESTAR EN ESTRECHA VIGILANCIA PARA EVITAR PRINCIPALMENTE LA


SIGUIENTE COMPLICACIÓN:

EDEMA DEFINICIÓN: El casi ahogamiento es la supervivencia mayor de 24 horas, tras sofocación


PULMONAR por sumersión en agua. • La fisiopatología del casi ahogamiento se resume con la
aspiración de agua dulce o salada, lo que conduce a un alveolo lleno de líquido pero aun
perfundido, lo que produce una sustancial mezcla pulmonar venosa. - Cuando se trata de
agua salada hipertónica atrae líquidos adicionales del plasma hacia los pulmones. Con
ambos tipos de agua, el edema pulmonar ocurre y se asocia a la anormalidad de
ventilación-perfusión. • El lavado del surfactante, da lugar a la aparición de atelectasias,
desequilibrio de la ventilación/perfusión y disminución de la distensibilidad pulmonar.
Dicho factor, más la rotura de células alveolares, dan lugar a la aparición de edema
pulmonar. • El edema pulmonar y el bajo gasto son producto por lo general delas lesiones
pulmonares producidas por la aspiración de agua con extravasación de líquidos al pulmón,
dando lugar a hipovolemia. EL EDEMA PULMONAR ES LA PRINCIPAL COMPLICACIÓN DEL
CASI AHOGAMIENTO SIENDO LOS PULMONES LOS ÓRGANOS MÁS AFECTADOS.
HEMÓLISIS Existen complicaciones tempranas y tardías que a continuación te enumero:
COMPLICACIONES TEMPRANAS: 1. Debido a la liberación masiva de catecolaminas en las
víctimas de casi ahogamiento es frecuente la hiperglicemia aun en pacientes no diabéticos,
pudiendo empeorar la encefalopatía. 2. La hipotermia severa es una complicación
frecuente. 3. Convulsiones. 4. La ingestión de agua contaminada puede causar neumonía y
absceso pulmonar, estas complicaciones son raras.

EDEMA COMPLICACIONES TARDÍAS: 1. Siempre deberán ser manejadas en el tercer nivel e


CEREBRAL incluyen: SDRA (síndrome de dificultad respiratoria aguda), encefalopatía anóxico-
isquémica persistente, neumonía aspirativa, absceso pulmonar, neumotórax,
neumomediastino, neumopericardio y pulmón de shock. Mioglobinuria o hemoglobinuria,
falla renal debido a necrosis tubular aguda, coagulopatía (especialmente asociada con
hipotermia), sepsis, empiema y barotrauma secundario a las altas presiones del ventilador.
• "La principal complicación a largo plazo es el daño neurológico": subnormalidad,
disfunción cerebral mínima, cuadriplejia espástica, síndrome extrapiramidal, atrofia
cortical y daño neuromuscular periférico. EL EDEMA CEREBRAL (INICIALMENTE
ASTROCÍTICO) "SECUNDARIO A LA HIPOXEMIA" PRESENTA MICROVACUOLIZACIÓN
COMO UNO DE LOS PRIMEROS PASOS EN LA CASCADA DE DAÑOS. Recuerda: Todas las
complicaciones están precedidas del daño pulmonar.

INSUFICIENCIA PRONÓSTICO: Los siguientes son los principales signos pronósticos: • Edad: peor
MIOCÁRDICA pronóstico sí es menor de 3 años. • Lugar de la sumersión: mayor riesgo en agua dulce que
salada. • Temperatura del agua: mejor pronóstico en agua fría. • Duración de la inmersión:
normalmente desconocida; estimable por el método del «paréntesis» (tiempo transcurrido
desde la última vez que se vio al niño. Mal pronóstico si es mayor a 5 minutos). • Tiempo
transcurrido hasta el primer esfuerzo respiratorio espontaneo: entre 15 y 30 minutos tras
el rescate, menos del 10% tiene retraso mental y tetraplejía espástica. Más de 60 minutos,
el 60-80% presentan secuelas neurológicas graves. • Si se realizó reanimación
cardiopulmonar antes de 10 minutos y si fue por personal entrenado: un 30% de posibles
complicaciones, se evitan con reanimación cardiopulmonar adecuada en el lugar del
accidente. • Puntuación de la escala de Glasgow: si es menor de 5, tiene un 80% de
mortalidad o secuelas neurológicas graves. Si es mayor de 6, grupo de bajo riesgo, no
estando indicadas medidas severas contra el edema cerebral. • Pupilas midriáticas y
arreactivas al ingreso: fallecen y el 16% de los supervivientes quedan con daño cerebral
severo. • PH arterial: normalmente no se realiza antes de 1 a 3 horas tras el rescate; si es
menor de 7,00, signo de mal pronóstico. • Medición de P.I.C.: buen método para predecir
supervivencia o muerte, pero no se relaciona con lesión cerebral residual.

Bibliografía:
APLS THE PEDIATRIC EMERGENCY MEDICINE RESOURCE. GAUSCHE-HILL M, FUCHS S, YAMAMOTO
L. JONES AND BARTLETT PUBLISHERS. EDICIÓN 4. 2004. PAG. 224-225.

52 - EL OBJETIVO PRINCIPAL DEL TRATAMIENTO EN ESTA PACIENTE SERÁ:

EL MANEJO LA El manejo de la hipotermia es un tema de discusión, ya que por un lado "favorece


HIPOTERMIA el pronóstico" al disminuir el metabolismo cerebral secundario a hipoxemia; pero,
por otro, la hipotermia es un "factor de riesgo" porque también causa arritmias
mortales.
REVERTIR LA Se debe de iniciar el tratamiento con lo más sencillo, lo cual consiste en sacar
HIPOXEMIA inmediatamente del agua a la víctima y ventilarlo, inmovilizar la columna cervical
ante la posibilidad de traumatismo y, si existe, corregir la hipotermia. Se
administra oxígeno al 100%, posteriormente se ajusta de acuerdo a la gasometría
arterial u oximetría de pulso. Si la vía aérea está comprometida o existe alguna
indicación, se debe intubar al paciente e iniciar asistencia mecánica ventilatoria
(AMV) con la adición de presión positiva al final de la espiración (PEEP) según se
requiera, aunque existen estudios en los que se ha utilizado exitosamente la
administración continua con presión positiva a través de ventilación mecánica no
invasiva. También se ha utilizado la oxigenación mediante dispositivos de
membrana extracorpórea. Las indicaciones para intubar e inicio de la asistencia
mecánica ventilatoria son: apnea, dificultad respiratoria grave, hipoxemia
refractaria al oxígeno. LA PRIMERA ACCIÓN Y PRINCIPAL VA DIRIGIDA A
MEJORAR LA FUNCIÓN VENTILATORIA Y REVERTIR LA HIPOXEMIA.

EL MANEJO DEL El tratamiento de la hiperhidratación, para evitar el edema pulmonar y la


EDEMA CEREBRAL hipertensión intracraneal consiste en la: restricción de líquidos, perfundiendo
soluciones isotónicas a 1/3 de necesidades básales, furosemida a 0.5-1 mg/Kg y en
ocasiones, manitol a 0.25-0.5 g,kg./dosis. EL MANEJO PREVENTIVO DEL EDEMA
PULMONAR Y CEREBRAL SONN MEDIDAS ESPECÍFICAS EN EL MANTENIMIENTO
DEL PACIENTE UNA VEZ QUE SE ENCUENTRA ESTABLE.

CORREGIR LA Las primeras medidas a aplicar en la U.C.I. son: Corregir la hipotermia, vaciar el
ALTERACIONES contenido gástrico, corregir las alteraciones hidroelectrolíticas y, las víctimas por
HIDROELECTROLÍTICAS ahogamiento en agua salada, pueden requerir la corrección de hipovolemia e
hipotensión, con expansores plasmáticos. Deben evaluarse los signos indicativos
de: 1. Estado hemodinámico: color de piel, temperatura diferencial, llenado
capilar. 2. Estado neurológico: puntuación de la escala de Glasgow. 3. Función
respiratoria: apnea, distress, etc. Auscultación. 4. Función renal: gasto urinario. LA
CORRECCIÓN DE LAS ALTERACIONES HIDROELECTROLÍTICAS Y METABÓLICAS
SON MEDIDAS DE SOSTÉN EN EL PACIENTE CASI AHOGADO.

Bibliografía:
APLS THE PEDIATRIC EMERGENCY MEDICINE RESOURCE. GAUSCHE-HILL M, FUCHS S, YAMAMOTO
L. JONES AND BARTLETT PUBLISHERS. EDICIÓN 4. 2004. PAG. 226-227.

FIN DEL CASO CLÍNICO SERIADO


ANÁLISIS DEL CASO CLÍNICO

IDENTIFICACIÓN DEL REACTIVO


Area: PEDIATRÍA
Especialidad: URGENCIAS PEDIÁTRICAS
Tema: URGENCIAS HEMATOLÓGICAS PEDIÁTRICAS
Subtema: PURPURA TROMBOCITOPÉNICA IDIOPÁTICA

CASO CLÍNICO CON UNA PREGUNTA

ESCOLAR FEMENINA DE 6 AÑOS, ACUDE A URGENCIAS POR PRESENTAR LESIONES PETEQUIALES EN


EXTREMIDADES INFERIORES, LE DIAGNOSTICAN PÚRPURA TROMBOCITOPÉNICA IDIOPÁTICA.

escolar femenina de 6 años

--

LESIONES PETEQUIALES EN
EXTREMIDADES INFERIORES

DIAGNOSTICAN PURPURA
TROMBOCITOPÉNICA IDIOPÁTICA.

--

53 - EL TRATAMIENTO MÁS EFICAZ EN ESTA PACIENTE ES CON:

FACTOR DE DEFINICIÓN. La PÚRPURA TROMBOCITOPÉNICA INMUNOLÓGICA O IDIOPÁTICA


TRANSFERENCIA (PTI), es una enfermedad hemorrágica autoinmune caracterizada por la destrucción
prematura de plaquetas debido a la unión de un autoanticuerpo, habitualmente de
la clase IgG a las glicoproteínas plaquetarias (GPIIb-IIIa) y la posterior depuración
por el sistema fagocítico mononuclear.
ANTIBIÓTICOS • En la PTI de niños la enfermedad habitualmente está precedida por infecciones
virales o bacterianas. • El inicio es súbito a los pocos días (<6 semanas) de una
enfermedad infecciosa. En más del 70% de los niños la enfermedad se resuelve
dentro de los primeros 6 meses. • Lo clásico es que en niños sanos previamente, un
mes posterior a padecer una enfermedad viral se presentan petequias, púrpura y
equimosis y sangrado de piel y mucosas, de ser predominio en la parte alta de las
extremidades inferiores, por ejemplo epistaxis nasal que puede ser de difícil control,
existen ampollas sangrantes en la boca y en los labios, se presenta hemorragia
intracraneal de forma muy rara con signos de sangrado en mucosas, retina, vías
urinarias y gastrointestinales.

PLASMA FRESCO • Recuento plaquetario con cifras disminuidas. (Valores normales 150 a 450 x109/L).
CONGELADO Se considera trombocitopenia cuando se tienen cifras <150x 109/L. • Frotis de
sangre periférica para excluir trastornos como leucemias, mielodisplasia, anemia y
megaloblástica.

INMUNOGLOBULINA La primera línea de tratamiento de la PTI (PÚRPURA TROMBOCITOPÉNICA


INTRAVENOSA TROMBÓTICA) son los glucocorticoides y en algunos casos inmunoglobulinas. • Los
criterios para iniciar el tratamiento son: cifras de plaquetas bajos y manifestaciones
clínicas de hemorragia. • La inmunoglobulina intravenosa (IgG IV) produce
incremento en la cifra de plaquetas en aproximadamente el 75% de los pacientes, sin
embargo la respuesta es transitoria. • La IgG IV no ha demostrado efecto a largo
plazo. • Esquemas de IgG: 1g x Kg/día (1 a 2 dosis) o 0.4g x Kg/día por 5 días. LA
INMUNOGLOBULINA IV HA RESULTADO MUY EFICAZ EN EL TRATAMIENTO DE LA
PÚRPURA TROMBOCITOPÉNICA IDIOPÁTICA.

Bibliografía:
NELSON TRATADO DE PEDIATRIA. RICHARD E. BEHRMAN. ELSEVIER. EDICIÓN 17. 2004. PAG. 1671.
ANÁLISIS DEL CASO CLÍNICO

IDENTIFICACIÓN DEL REACTIVO


Area: GINECOLOGÍA Y OBSTETRICIA
Especialidad: GINECOLOGÍA
Tema: INFERTILIDAD Y PLANIFICACIÓN FAMILIAR
Subtema: ANTICONCEPCIÓN DE EMERGENCIA

CASO CLÍNICO CON UNA PREGUNTA

ADOLESCENTE DE 18 AÑOS DE EDAD, ACUDE A CONSULTA REFIRIENDO HACE 4 DÍAS HABER TENIDO
COITO SIN PROTECCIÓN Y TEME LA POSIBILIDAD DE UN EMBARAZO, FECHA DE ÚLTIMA REGLA HACE 16
DÍAS. SOLICITA MÉTODO ANTICONCEPTIVO DE EMERGENCIA.

adolescente18 años.

coito sin protección hace 4 días, “TIEMPO


DE OVULACIÓN ESTIMADO HACE 2 DÍAS”.

solicita método anticonceptivo de


emergencia.

-.

-.

54 - ES ALTAMENTE EFECTIVO PARA PREVENIR EL EMBARAZO EN ESTA PACIENTE

APLICAR El término ANTICONCEPCIÓN DE EMERGENCIA, o anticoncepción poscoital, se refiere


HORMONAL a métodos anticonceptivos que se pueden utilizar para prevenir embarazos en los días
INYECTABLE inmediatamente posteriores a la relación sexual. Se estableció para situaciones de
COMBINADO. emergencia creadas por una relación sexual sin protección, la falla o el uso incorrecto
de otros anticonceptivos (como la omisión de una píldora anticonceptiva o la rotura
de un preservativo) y para casos de violación o relaciones sexuales forzadas. • Existen
dos métodos de anticoncepción de emergencia: las píldoras anticonceptivas de
emergencia y los dispositivos intrauterinos de cobre. • Si se los coloca en los cinco días
posteriores a la relación sexual sin protección, los dispositivos intrauterinos de cobre
son el método más eficaz de anticoncepción de emergencia. • SÓLO EXISTEN 2
MÉTODOS ANTICONCEPTIVOS DE EMERGENCIA: LAS PÍLDORAS DE EMERGENCIA Y EL
DIU DE COBRE.
TOMAR 1 TABLETA La OMS recomienda la píldora de LEVONORGESTREL para la anticoncepción de
DE emergencia. Lo ideal es que la mujer tome una sola dosis de esta píldora de
LEVONOGESTREN progestágeno solo (1.5 mg) dentro de los cinco días posteriores (120 horas) a la
DE 0.75MG. relación sexual sin protección. Otra posibilidad es que tome dos dosis de
levonorgestrel (de 0.75 mg cada una, con un intervalo de 12 horas). SE INDICA 1
PÍLDORA DE 1.5MG O “2” DE 0.75MG Y NO 1 COMO SE DESCRIBE EN LA RESPUESTA.

ADMINISTRAR 2 El régimen recomendado por la OMS para las píldoras anticonceptivas de emergencia
TABLETAS DE es una dosis de 1.5 mg de levonorgestrel administrada dentro los cinco días (120
ANTICONCEPTIVOS horas) posteriores a la relación sexual sin protección. LAS PÍLDORAS
ORALES ANTICONCEPTIVAS ESTÁN HECHAS A BASE DE LEVONOGESTREL Y NO DE
COMBINADOS. HORMONALES COMBINADOS. IMPORTANTE: : la anticoncepción de emergencia
hormonal oral, está indicada idealmente dentro de las 72hrs tras el acto sexual de
riesgo, según la licencia del producto. La GPC establece que puede recomendarse
hasta el quinto día, pero de ser así, entre las 72-120hrs tras la relación sexual de
riesgo, se deberá informar a la mujer que la eficacia puede ser limitada; ya que entre
mayor es el tiempo que transcurre entre el acto sexual y su administración, menor es
la efectividad anticonceptiva.

INSERCIÓN DE DIU DIU DE COBRE. Este método es altamente eficaz para prevenir el embarazo. Un DIU
DE COBRE. liberador con cobre (DIU-Cu) puede ser usado dentro de los 5 días después del coito
sin protección, como un anticonceptivo de emergencia. Sin embargo, cuando se
puede estimar el tiempo de la ovulación, el DIU-Cu puede ser insertado más allá de los
5 días después del coito, si es necesario, siempre que la inserción no se realice después
de más de 5 días desde la ovulación. Los criterios de elegibilidad para el intervalo de
inserción del DIU-Cu también se aplican para la inserción del DIU-Cu como
anticoncepción de emergencia. ACLARACIÓN: • El DIU no está indicado durante el
embarazo y no debe ser usado debido al riesgo de infección pélvica grave y aborto
séptico espontáneo. • En violación: Los DIU no protegen contra ITS/VIH/EPI. Entre las
mujeres con infección por clamidia o gonorrea, debe evitarse el riesgo potencial
aumentado de EPI con la inserción del DIU. La preocupación es menor para otras ITS.
SE RECOMIENDA EL DIU COMO EL MÉTODO MÁS EFECTIVO DE ANTICONCEPCIÓN DE
EMERGENCIA CON BAJA INCIDENCIA DE EFECTOS SECUNDARIOS.

Bibliografía:
1. GUÍA DE PRÁCTICA CLÍNICA, MANEJO DE ANTICONCEPTIVOS TEMPORALES HORMONALES EN
MUJERES EN EDAD REPRODUCTIVA , EN EL PRIMER Y SEGUNDO NIVELES DE ATENCIÓN. MÉXICO:
SECRETARIA DE SALUD; MARZO 2014. 2. GUÍA DE PRÁCTICA CLÍNICA, CONSULTA Y ASESORÍA
MÉDICA PARA EL USO DE LA ANTICONCEPCIÓN DE EMERGENCIA. MÉXICO: SECRETARIA DE SALUD;
2009.

http://www.cenetec.salud.gob.mx /descargas/gpc/CatalogoMaestro/201_SSA_09_Anticoncepcion_emergencia/GPC_201-09_Anticoncepcixn_de_emergenciaEVR.pdf
ANÁLISIS DEL CASO CLÍNICO

IDENTIFICACIÓN DEL REACTIVO


Area: GINECOLOGÍA Y OBSTETRICIA
Especialidad: OBSTETRICIA
Tema: DIABETES GESTACIONAL
Subtema: DIABETES GESTACIONAL

CASO CLÍNICO SERIADO

SECUNDIGESTA DE 33 AÑOS DE EDAD CON ANTECEDENTE DE PRODUCTO MACROSÓMICO EN LA


PRIMERA GESTA. ACUDE AL CENTRO DE SALUD SOLICITANDO ATENCIÓN PRENATAL. SE CALCULA UN
EMBARAZO DE 14 SEMANAS DE GESTACIÓN. A LA EXPLORACIÓN FÍSICA GANANCIA PONDERAL
DURANTE EL EMBARAZO DE 5KG, FONDO UTERINO ACORDE CON LA EDAD GESTACIONAL, SIN
PÉRDIDAS TRASVAGINALES. GLUCÉMIA CAPILAR EN AYUNO DE 112MG/DL. SE SOLICITAN LABORATORIOS
PRENATALES GENERALES Y PRUEBA DE CURVA DE TOLERANCIA ORAL A LA GLUCOSA.

secundigesta de 33 años

producto macrosómico en primera gesta.

-.

incremento ponderal de 5kg en 14


semanas

glucemia capilar de 112mg/dl.

55 - EN CASO DE RESULTAR POSITIVA LA PRUEBA SE DEBERÁ COMENZAR EL MANEJO CON:


TERAPIA EL PILAR DEL TRATAMIENTO DE LA DIABETES GESTACIONAL ES LA NUTRICIÓN, EL
NUTRICIONAL Y EJERCICIO Y EL AUTOCONTROL. Los objetivos del manejo nutricional son: lograr una
EJERCICIO. ganancia ponderal adecuada, mantener el control glucémica y evitar la cetonuria y/o
episodios de hipoglucemia. Existen bibliografías que describen hasta un 90% de
mujeres con diabetes gestacional controladas únicamente con apego estricto a la
dieta. Se recomienda el ejercicio leve como la caminata de 20 a 45 minutos 3 veces
por semana sobre todo en la primera hora postprandial. El tratamiento
farmacológico de la diabetes gestacional se debe considerar cuando la dieta y el
ejercicio no logran las cifras meta (menor de 95mg/dl en ayuno y 120mg/dl 2 horas
postprandial) para el control de la glucosa en sangre en un periodo de 2 semanas. EL
TRATAMIENTO FARMACOLÓGICO ESTÁ INDICADO CUANDO NO SE HAN CUMPLIDO
METAS CON TERAPIA NUTRICIONAL Y EJERCICIO, NO ANTES.

HIPOGLUCEMIANTES No existe evidencia de grandes diferencias en el control de mujeres con diabetes


ORALES. gestacional tratadas con hipoglucemiantes orales con respecto a aquellas con
manejo insulínico sin embargo, LOS HIPOGLUCEMIANTES ORALES NO SE
CONSIDERAN SEGUROS DURANTE EL EMBARAZO por lo que deberá evitarse su uso
en este periodo.

INSULINOTERAPIA. LA INSULINOTERAPIA ESTÁ INDICADA CUANDO NO SE HAN CUMPLIDO METAS


CON MEDIDAS NUTRICIONALES Y DE EJERCICIO. Los análogos de insulina LISPRO y
ASPART, así como la INSULINA HUMANA DE ACCIÓN RÁPIDA, han sido investigados
en el embarazo demostrando efectividad, seguridad clínica, mínimo transporte a
través de la placenta y no se ha informado teratogenicidad. La INSULINA DE ACCIÓN
INTERMEDIA (NPH) es también segura y se utiliza en combinación con las de acción
rápida.

INHIBIDORES DELA La ACARBOSA un inhibidor del alfaglucosidasa, es un oligosacárido que se utiliza


ALFA GLUCOSIDASA. para el control de la diabetes. Dado que no se disponen de estudios confiables en el
uso de éste fármaco durante el embarazo. No se contempla siquiera en las guías
actuales de manejo de la diabetes gestacional como opción terapéutica.

Bibliografía:
1. DIAGNÓSTICO Y TRATAMIENTO DE LA DIABETES EN EL EMBARAZO. EVIDENCIAS Y
RECOMENDACIONES. GUÍA DE PRÁCTICA CLÍNICA. MÉXICO: SECRETARÍA DE SALUD; 03/11/2016. 2.
CUNNINGHAM G, LEVENO K, BLOMM S, HAUTH J, RPUSE D, SONG C. WILLIAMS OBSTETRICIA, 23A
EDICIÓN. MC GRAW HILL. USA. 2011 EN ESPAÑOL.

http://www.cenetec-difusion.com/CMGPC/IMSS-320-10/ER.pdf

56 - LA META DE CONTROL GLUCÉMICO POSPRANDIAL A UNA HORA CONTEMPLA UNA GLUCEMIA


DE:

MENOR A Metas terapéuticas de glucosa plasmática durante el embarazo. 1. DIABETES GESTACIONAL. -


140MG/DL. Glucosa en ayuno: menor a 95mg/dl. - Glucosa 1 hr posprandial: igual o menor a 140mg/dl. -
Glucosa 2 hrs posprandial: igual o menor a 120mg/dl. - HbA1c: igual o menor a 6%. - Glucosa
antes de dormir y en la madrugada: 60mg/dl. 2. DIABETES PREGESTACIONAL: - Glucosa en
ayuno: menor a 90mg/dl. - Glucosa 1 hr posprandial: igual o menor a 130-140mg/dl. - Glucosa
2 hrs posprandial: igual o menor a 120mg/dl. - HbA1c: igual o menor a 6%. - Glucosa antes de
dormir y en la madrugada: 60mg/dl. OJO: DEBES LEER CON CUIDADO LA INSTRUCCIÓN
SOLICITADA EN EL REACTIVO DONDE SE ESPECIFICA POSPRANDIAL A “1 HORA”.

MENOR A Si el crecimiento fetal es igual o mayor al percentil 90 las metas de glucemia serán más
130MG/DL. estrictas: - Menor o igual a 80mg/dl en ayuno y 110mg/dl dosis a las dos horas del
posprandio.
MENOR A Dado que el control de la glucemia es muy estricto durante el embarazo se deberá capacitar a
120MG/DL. la mujer en el AUTOMONITOREO capilar sobre todo para evitar hipoglucemias y restricción en
el crecimiento intrauterino. Esta estrategia ha demostrado mejor control metabólico en la
mujer con diabetes gestacional y mejores resultados perinatales para el binomio. El
automonitoreo se debe realizar siempre que sea posible al menos 3 veces al día; de no poderse
realizar con esta frecuencia, se deberá realizar semanalmente en la unidad de salud una toma
de glucemia central en ayuno y 1hr posprandial. SE ESPERA TENER MENOS DE 120MG/DL
POSTPRANDIAL A LAS 2 HORAS.

MENOR A RECUERDA: Toda paciente con descontrol metabólico (glucemia en ayuno mayor a 140mg/dl o
110MG/DL. postprandial a la hora, mayor o igual a 180mg/dl) requieren manejo hospitalario para control
metabólico.

Bibliografía:
1. DIAGNÓSTICO Y TRATAMIENTO DE LA DIABETES EN EL EMBARAZO. EVIDENCIAS Y
RECOMENDACIONES. GUÍA DE PRÁCTICA CLÍNICA. MÉXICO: SECRETARÍA DE SALUD; 03/11/2016.

http://www.cenetec-difusion.com/CMGPC/IMSS-320-10/ER.pdf

FIN DEL CASO CLÍNICO SERIADO


ANÁLISIS DEL CASO CLÍNICO

IDENTIFICACIÓN DEL REACTIVO


Area: PEDIATRÍA
Especialidad: URGENCIAS PEDIÁTRICAS
Tema: DESHIDRATACIÓN, DESEQUILIBRIO HIDROELECTROLÍTICO Y
CHOQUE
Subtema: DESEQUILIBRIO ACIDOBÁSICO

CASO CLÍNICO SERIADO

ESCOLAR DE 8 AÑOS DE EDAD. CON ANTECEDENTE DE VIOLENCIA INTRAFAMILIAR. EN LOS DOS


ÚLTIMOS MESES HA CURSADO CON CUADROS DE DOLOR ABDOMINAL EN 4 OCASIONES, EL CUAL ES
INCAPACITANTE Y EN OCASIONES SE ACOMPAÑA DE EVACUACIONES DIARRÉICAS.

escolar de 8 años de edad

--

cuadros de dolor abdominal en 4


ocasiones, incapacitante y en ocasiones
acompañado de evacuaciones diarréicas

--

--

57 - EL DIAGNÓSTICO PROBABLE DE ESTE PACIENTE ES:

AMIBIASIS AMIBIASIS: es una infección ocasionada por Entamoeba histolytica o Entamoeba dispar
INTESTINAL que no produce síntomas en 90% de los individuos afectados. • La infección se localiza en
la mucosa del intestino grueso, donde sólo la especie E. histolytica y, en particular, las
cepas invasoras dañan el tejido y ocasionan enfermedades intestinales o extraintestinales
que afectan otros órganos. • CX: Los pacientes con amibiasis intestinal sintomática o colitis
amibiana refieren dolor abdominal tipo cólico de varias semanas de evolución, en
ocasiones tienen pérdida de peso y diarrea, que puede ser acuosa con abundante moco y
poca materia fecal acompañada o no de sangre, cuando ésta existe se habla de disentería
amibiana. El inicio suele ser insidioso y los síntomas pueden ser muy heterogéneos, lo que
dificulta el diagnóstico. EL CUADRO CLÍNICO ES INSIDIOSO, LA DIARREA CARACTERÍSTICA
CON MOCO ABUNDANTE Y EN OCASIONES SANGRE NO CORRESPONDE AL CUADRO
CLÍNICO DE NUESTRO PACIENTE.
INFECCIÓN GIARDIASIS INTESTINAL: es la infección enteral por Giardia lamblia, protozoo flagelado;
POR GIARDIA es elevada en el mundo e incide con mayor frecuencia en los países de clima tropical. Su
distribución varía de acuerdo a las condiciones económicas, higiénicas y sociales que
conllevan a la desnutrición y la incultura, además se presenta con incidencia elevada en
niños con inmunodeficiencias. • El cuadro clínico de la enfermedad se presenta en la
población infantil con diversidad de manifestaciones clínicas, favorecidas por el curso
prolongado que frecuentemente esta parasitosis exhibe. • El cuadro clínico es variable, los
niños se ven con frecuencia más afectados que los adultos, aunque en todas las edades
pueden aparecer síntomas que van desde diarreas ligeras, flatulencia, anorexia,
retortijones y dolores epigástricos hasta esteatorrea y auténtico síndrome de mal
absorción. • Del 10 al 20 % de pacientes portadores de Giardia lamblia, pueden ser
asintomáticos. TIENE CUADRO CLÍNICO INCIDIOSO Y POCO ESPECÍFICO CON UN CLÁSICO
SÍNDROME DE MALA ABSORCIÓN INTESTINAL.

GASTRITIS HELICOBACTER PYLORI: es una bacteria Gram negativa espirilada o que se describe en
POR forma de golondrina en vuelo, que coloniza el epitelio gástrico por medio de sus pedículo,
HELICOBACTER se mantiene bajo la protección de capa de moco, lo cual la protege adecuadamente de la
PYLORI acidez gástrica, así mismo metaboliza urea para liberar amonio y en esta forma es capaz
de neutralizar el ácido agresivo del medio gástrico. • Estudios serológicos retrospectivos y
prospectivos han demostrado que la incidencia de HP aumenta con la edad, siendo en los
países desarrollados detectado muy raramente antes de los veinte años, mientras que en
los países tercermundistas la incidencia en niños de diez años es arriba del 50% esto
correlaciona la incidencia del microorganismo infectante con las condiciones sanitarias
malas. • La asociación de HP con gastritis crónica es en la actualidad aceptada como la
causa más importante de ésta. NO ES UNA ENFERMEDAD PROPIA EN NIÑOS.

DOLOR Se estima que entre el 10% y el 18% de los niños en edad escolar presentan el síndrome
ABDOMINAL conocido como Dolor abdominal recurrente (DAR). DEFINICIÓN: Éste se define como la
RECURRENTE presencia de tres o más episodios de dolor abdominal, lo suficientemente importantes
como para limitar las actividades y funciones del niño durante, al menos, un período de
tres meses previamente a la consulta. • La incidencia es mayor en niñas que en niños,
particularmente durante la adolescencia; antes de esta edad las diferencias no son tan
marcadas. • En verdad, una gran mayoría de los niños afectados por DAR, (85 a 95%), no
presenta evidencia de trastorno orgánico y, en muchos, sus episodios de dolor están
claramente relacionados con el "stress", tanto escolar como familiar. DADO QUE LA
DIARREA NO TIENE CARACTERÍSTICAS DE SER INFECCIOSA Y NO HAY MAS DATOS
ASOCIADOS EL DIAGNÓSTICO ADECUADO ES UN DOLOR ABDOMINAL RECURRENTE.

Bibliografía:
PEDIATRÍA DE RUDOLPH. RUDOLPH COLIA. INTERAMARICANA. EDICIÓN 21. 2004. PAG. 1507-08.

58 - EL PACIENTE DEBE DE INICIAR SU TRATAMIENTO CON:

METRONIDAZOL TX AMIBIASIS: • El tratamiento de la amibiasis intestinal invasora y del absceso hepático


es el metronidazol, antibiótico del que se han acumulado más de 25 años de experiencia
de alta eficacia terapéutica. • La amibiasis no invasora, o asintomática, puede tratarse
con paromomicina. Los nitroimidazoles, como el metronidazol, se recomiendan sólo para
la amibiasis invasora. El 90% de los pacientes con disentería amibiana reaccionan
positivamente a los nitroimidazoles. TX GIARDIASIS: • Los medicamentos de elección
para la giardiasis sintomática son: metronidazol, furazolidona y tinidazol, en otros países
se recomienda la quinacrina sin embargo en México se tiene poca experiencia al respecto
además de tener muchos efectos colaterales. Actualmente tras estudios clínicos se ha
comprobado también la eficacia del albendazol. CORRESPONDE AL TRATAMIENTO DE LA
AMIBIASIS Y GIARDIASIS.
OMEPRAZOL Está indicado en esofagitis por reflujo gastroesofágico, úlcera duodenal, úlcera gástrica
benigna (incluyendo las producidas por AINEs), síndrome de Zollinger-Ellison y
tratamiento de úlcera gástrica y duodenal asociada a Helicobacter pylori. SE UTILIZA EN
EL MANEJO DE GASTRITIS DE DIFERENTES ETIOLOGÍAS.

SECNIDAZOL El Secnidazol es el tratamiento de elección para un control eficaz, seguro y rápido de


amebiasis intestinal y extraintestinal, tricomoniasis y giardiasis. También activo contra
Helicobacter pylori, causante de gastritis, duodenitis y úlcera péptica. • Mantiene niveles
terapéuticamente activos durante más de 72 horas.

APOYO • Si la sospecha diagnóstica está fundadamente apuntando a una causa psicogénica


PSICOLÓGICO como etiología, el médico debe explicar sencilla, pero clara y razonadamente al paciente
y a sus padres que el origen del dolor no está en alguna "enfermedad" del abdomen ni
de otro sitio, pero que el dolor es real (para evitar que sea malinterpretado como una
simulación). • La terapia debe estar enfocada básicamente a despejar las ansiedades del
grupo familiar, con lo que se tendrá recorrida una parte importante del camino hacia las
metas terapéuticas. EL MANEJO DEBE SER INTEGRAL TANTO PARA EL NIÑO COMO AL
NÚCLEO FAMILIAR EN EL QUE SE DESENVUELVE.

Bibliografía:
PEDIATRÍA DE RUDOLPH. RUDOLPH COLIA. INTERAMARICANA. EDICIÓN 21. 2004. PAG. 1507-08.

FIN DEL CASO CLÍNICO SERIADO


ANÁLISIS DEL CASO CLÍNICO

IDENTIFICACIÓN DEL REACTIVO


Area: MEDICINA INTERNA
Especialidad: ONCOLOGÍA
Tema: CÁNCER EN SANGRE (LEUCEMIAS Y LINFOMAS)
Subtema: LEUCEMIA

CASO CLÍNICO CON UNA PREGUNTA

HOMBRE DE 53 AÑOS DE EDAD QUIEN DESDE HACE 3 MESES PRESENTA ANOREXIA, PÉRDIDA DE PESO
Y FATIGA. DESDE HACE 3 SEMANAS PRESENTA EPISTAXIS Y GINGIVORRAGIA RECURRENTES. A LA
EXPLORACIÓN FÍSICA SE OBSERVAN EQUIMOSIS EN TODO EL CUERPO Y LINFADENOPATÍAS EN AXILAS
Y REGIÓN INGUINAL. LA BIOMETRÍA HEMÁTICA MUESTRA LEUCOPENIA Y LOS DATOS HISTOLÓGICOS
SUGIEREN LEUCEMIA MIELOIDE AGUDA.

53 años.

LEUCEMIA.

anorexia, pérdida de peso y fatiga, así como


epistaxis y gingivorragia.

equimosis en todo el cuerpo y


linfadenopatías en axilas e ingles.

biometría hemática con leucopenia.


Histología compatible con Leucemia
Mieloide aguda.

59 - SE RELACIONA DIRECTAMENTE CON ESTE TIPO DE LEUCEMIA:

EXPOSICIÓN A La LEUCEMIA MIELOIDE AGUDA (LMA) es la neoplasia mieloide más frecuente en adultos,
BENCENO. su incidencia se incrementa de forma importante en personas mayores de 65 años. La LMA
es un grupo heterogéneo de leucemias que proceden de las líneas celulares precursoras
mieloides, eritroides, megacariocítica y monocítica. Las leucemias resultan de
transformación clonal de precursores hematopoyéticos, a través de la adquisición de
arreglos cromosómicos y múltiples mutaciones genéticas. Entre los factores de riesgo para
la LMA se encuentran la edad (>60 años), exposición a bencenos, radioterapia,
quimioterapia, síndrome mielodisplásico, anemia de Fanconi y Trisomia 21. EL BENCENO
ES UN COMPUESTO QUÍMICO IMPORTANTE EN LAS LESIONES DE MÉDULA ÓSEA ENTRE
LAS QUE DESTACAN APLASIA MEDULAR Y LEUCEMIA MIELOIDE AGUDA.
INTOXICACIÓN La INTOXICACIÓN CRÓNICA POR PLOMO se caracteriza por neuropatía periférica (motora,
CRÓNICA POR brazos y piernas), retraso en el desarrollo, problemas cognitivos, hipertensión arterial
PLOMO. sistémica y nefropatía (síndrome de Fanconi). Las manifestaciones en el sistema
hematopoyético son: anemia; el plomo induce estrés oxidativo como mecanismo
subyacente ANEMIA APLÁSICA. LA INTOXICACIÓN POR PLOMO SE RELACIONA MÁS CON
ANEMIA APLÁSICA DONDE SE AFECTA LA PRODUCCIÓN DE TODAS LAS LÍNEAS
HEMATOPOYÉTICAS.

ALCOHOLISMO EL CONSUMO DE ALCOHOL ES UNA DE LAS CAUSAS MÁS COMUNES DE MACROCITOSIS


CRÓNICO. NO ANÉMICA. Todo paciente con macrocitosis debe ser interrogado sobre el consumo de
alcohol. Es probable que esta alteración eritrocitaria sea secundaria a las siguientes causas.
- Folatos deficientes por baja ingesta. - Reticulocitosis por hemólisis o hemorragia
gastrointestinal. - Enfermedad hepática. - Macrocitosis por intoxicación alcohólica.
Cuando la macrocitosis se presenta acompañada de anemia es probable que el paciente
alcohólico este cursando además con deficiencia de folatos. EL ALCOHOLISMO NO SE
ASOCIA A ESTE TIPO DE ANEMIA, EL TABAQUISMO “SI”, SE CREE INCREMENTA EL RIESGO.

INTOXICACIÓN Las manifestaciones clínicas de INTOXICACIÓN POR ARSÉNICO varían, según la exposición
POR sea aguda o crónica; sin embargo, incluyen síntomas generales como malestar general,
ARSÉNICO. fatiga, cefalea, fiebre, dolor abdominal, anorexia, así como hiperhidrosis, melanodermia,
hiperqueratosis, síntomas gastrointestinales, daño hepático, PANCITOPENIA, enfermedad
vascular periférica, enfermedad pulmonar obstructiva crónica y restrictiva, neuropatía
sensoriomotora periférica simétrica, hematuria, oliguria o anuria. LA MANIFESTACIÓN
SANGUÍNEA DE LA INTOXICACIÓN POR ARSÉNICO ES LA PANCITOPENIA.

Bibliografía:
1. GUÍA DE PRÁCTICA CLÍNICA, DIAGNOSTICO Y TRATAMIENTO DE LA LEUCEMIA MIELOIDE AGUDA.
MÉXICO: SECRETARIA DE SALUD; 2010. 2. GUÍA DE PRÁCTICA CLÍNICA, DIAGNÓSTICO Y
TRATAMIENTO DE LA INTOXICACIÓN POR METALES: PLOMO, MERCURIO, ARSÉNICO Y TALIO EN EL
PRIMER, SEGUNDO Y TERCER NIVELES DE ATENCIÓN. MÉXICO: SECRETARIA DE SALUD; 2010. 3.
LONGO DL, FAUCI AS, KASPER DL, HAUSERSL, JAMESON JL, LOSCALZOJ. HARRISON. PRINCIPIOS
DE MEDICINA INTERNA, 18A EDICIÓN. MC GRAW HILL. NEW YORK, USA. 2012

http://www.cenetec.salud.gob.mx/descargas/gpc/CatalogoMaestro/276_IMSS_10_Leucemia_Mieloide_Aguda/EyR_IMSS_276_10.pdf
ANÁLISIS DEL CASO CLÍNICO

IDENTIFICACIÓN DEL REACTIVO


Area: GINECOLOGÍA Y OBSTETRICIA
Especialidad: GINECOLOGÍA
Tema: LEUCORREA
Subtema: VAGINOSIS BACTERIANA

CASO CLÍNICO SERIADO

MUJER DE 32 AÑOS DE EDAD, QUE PRESENTA DESDE HACE 3 SEMANAS SECRECIÓN TRANSVAGINAL
PROFUSA, DE COLOR BLANCO GRISÁCEO. SE REALIZA ESTUDIO MICROSCÓPICO DE LA SECRECIÓN
DONDE SE OBSERVAN CÉLULAS CLAVE.

Mujer de 32 años de edad.

-.

Secreción transvaginal profusa color blanco


grisáceo.

-.

Estudio de microscopio donde se observan


"células CLAVE".

60 - EL DIAGNÓSTICO MÁS PROBABLE ES:

INFECCIÓN POR El diagnóstico de TRICOMONIASIS se caracteriza por presentar leucorrea abundante,


TRICHOMONA. maloliente y espumosa, con prurito y ardor vulvovaginal así como dispareunia, disuria y
flujo, la mucosa está hiperémica, moteada por petequias (Cérvix en “fresa”) o zonas
hemorrágicas, "leucorrea olorosa, espuma blanca o amarrilla". Es importante considerar
que hasta un tercio de las mujeres son totalmente asintomáticas. RECUERDA. La
infección por Trichomona puede producir al igual que la vaginosis bacteriana, pH de la
secreción vaginal mayor a 4.5 y olor a pescado. El diagnóstico diferencial en ocasiones
sólo puede hacerse, a través de la visión directa de la secreción y observación del
parásito o de las células guía. En la práctica, dado que el tratamiento es el mismo, se
suele dar manejo sin determinar la diferenciación. EN LA INFECCIÓN POR TRICHOMONA
LA EXPLORACIÓN AL MICROSCOPIO GENERALMENTE MUESTRA FORMAS FLAGELADAS.
INFECCIÓN POR El diagnóstico de las infecciones genitourinarias por CHLAMYDIA, se caracteriza por
CHLAMYDIA. secreción escasa o nula, y cuando la hay es mucopurulenta y fétida con sangrado
poscoito. En las infecciones gonocócicas, se observa al microscopio su peculiar
característica de agrupación en racimos, cadena o paquetes cuboidales. EN LA
INFECCIÓN POR CHLAMYDIA, AL MICROSCOPIO SE OBSERVAN COCOS GRAM
NEGATIVOS EN CADENA O RACIMOS, PUEDE HACER FORMAS INTRACELULARES.

CANDIDIASIS La colonización vaginal por CÁNDIDA es relativamente frecuente entre las mujeres
VULVOVAGINAL. atendidas en clínicas de enfermedades de transmisión sexual. Muchas de ellas también
presentan colonización en la zona anorrectal. Sin embargo, solo la mitad tienen
sintomatología de vulvovaginitis candidiásica, que incluye inflamación vulvar y vaginal,
fisuras y existencia de un exudado adherente a la mucosa, blanquecino y amarillento,
con grumos (Cottage cheese). El pH vaginal se mantiene en 4.5. No siempre tiene la
candidiasis el carácter de ETS, pudiendo ser una infección endógena. En la patogenia de
esta infección actúan como factores predisponentes la diabetes, el embarazo, el uso de
contraceptivos orales, la obesidad, el empleo reciente de antimicrobianos y la utilización
de corticoides. La forma de presentación se clasifica en complicada o no complicada. La
primera es recurrente, grave, puede estar producida por otras especies de Candida
diferentes a C. Albicans y, se produce en diabéticas no controladas, inmunodeprimidas y
embarazadas. LA CARACTERÍSTICA DE LOS HONGOS CUANDO SE OBSERVAN AL
MICROSCOPIO ES LA PRESENCIA DE HIFAS.

VAGINOSIS VAGINOSIS BACTERIANA. Dentro de las diferentes patologías infecciosas que se pueden
BACTERIANA. presentar en el canal cervicovaginal se considera que la vaginosis es la más frecuente, ya
que en clínicas de enfermedades de transmisión sexual se le encuentra en un 32 a 64 %
en medicina familiar del 12 a 25 % y de 10 a 26% en la práctica obstétrica. En relación al
género que se ha reportado que hasta el 80 % de los hombres y el 51% de las mujeres
han tenido enfermedades de transmisión sexual. Cuadro clínico. Los signos
característicos encontrados son: una secreción blanca o blanca-grisácea" que se percibe
generalmente después de la relación sexual, descarga vaginal excesiva, el olor fétido
aminado (Pescado), a causa de la producción de aminas por las múltiples bacterias de la
vagina que puede ser más marcado después de la relación sexual sin protección, debido
a que el flujo seminal alcalino favorece más el olor. Las pacientes no siempre se quejan
de prurito vulvar (58 %), molestias vaginales o una dispareunia. Diagnóstico clínico. El
diagnóstico clínico generalmente se lleva a cabo mediante los antecedentes clínicos y el
olor. El examen pélvico debe llevarse a cabo para determinar la producción de secreción
anómala, además de verificar o destacar la presencia de alguna otra enfermedad. El
diagnóstico de vaginosis se basa en la presencia de cuando menos tres de los cuatro
criterios clínicos propuestos por Amsel y colegas en el Simposio Internacional sobre
Vaginosis en Estocolmo, las cuales han sido aceptadas como parámetro para indicar la
presencia de la enfermedad. 1. Descarga fina, blanca adherente y homogénea 2. pH
superior a 4.5 3. Prueba de amina positiva 4. Células indicadoras (Células clave) en
preparación salina La presencia de 2 de los 4 criterios clínicos, aunados a la presencia de
una prueba de “Olor” a amina positiva y el hallazgo microscópico de células clave,
permite hacer un diagnóstico exacto y rápido de la vaginosis, según Thomason y
colaboradores. LAS CARACTERÍSTICAS DE LA LEUCORREA Y LA PRESENCIA DE "CÉLULAS
CLAVE" DIRIGEN ESTE DIAGNÓSTICO.

Bibliografía:
1. GUÍA DE PRÁCTICA CLÍNICA, DIAGNÓSTICO Y TRATAMIENTO DE LA VAGINITIS INFECCIOSA EN
MUJERES EN EDAD REPRODUCTIVA EN EL PRIMER NIVEL DE ATENCIÓN. MÉXICO: SECRETARIA DE
SALUD; DICIEMBRE 2014.

http://www.cenetec.salud.gob.mx/descargas/gpc/CatalogoMaestro/081_GPC_Vaginitisinfec1NA/Vaginitis_ER_CENETEC.pdf

61 - LA PACIENTE REFIERE SER ALÉRGICA AL TRATAMIENTO DE ELECCIÓN, POR TAL MOTIVO


DEBERÁ USTED INDICAR:
METRONIDAZOL. El tratamiento de elección para la VAGINITIS BACTERIANA es: 1) Metronidazol 500 mg,
vía oral dos veces al día por 7 días; o 2) Metronidazol 2 g vía oral en una sola dosis. EL
TRATAMIENTO DE ELECCIÓN PARA LA VAGINOSIS BACTERIANA ES EL METRONIDAZOL.
DADO QUE LA PACIENTE ES ALÉRGICA NO DEBERÁ TENERSE EN CUENTA COMO
RESPUESTA CORRECTA.

CEFTRIAXONA. Tratamiento de infección por CHLAMYDIA TRACHIMATIS, Infección no complicada. •


Azitromicina 1 gr, dosis única. • Doxiciclina 100 mg, tres veces al día por siete días. •
Ofloxacino 200 mg, tres veces al día por siete días. • Minociclina 100 mg, una vez al día
por nueve días. El Center for Disease Control CDC recomienda: • Azitromicina 1 gr oral,
dosis única. • Doxiciclina 100 mg oral, dos veces al día por siete días. El CDC recomienda
como régimen alternativo: • Eritromicina 500 mg oral, cuatro veces al día por siete días.
• Eritromicina etilsuccinato 800 mg oral, cuatro veces al día por siete días. • Ofloxacino
300 mg oral, dos veces al día por siete días. • Levofloxacino 500 mg oral, una vez al día
por siete días. B. Infección no complicada en el embarazo: • Eritromicina 500 mg, cuatro
veces al día por siete días. • Amoxicilina 500 mg, tres veces al día por siete días. •
Azitromicina 1 gr oral, dosis única. El CDC recomienda como régimen alternativo: •
Eritromicina 500 mg oral, cuatro veces al día por siete días. • Eritromicina 250 mg oral,
cuatro veces al día por 14 días. • Eritromicina etilsuccinato 800 mg, cuatro veces al día
por siete días. • Eritromicina etilsuccinato 400 mg, cuatro veces al día por siete día.

CLINDAMICINA. La prevalencia de VAGINOSIS BACTERIANA (VB) es mayor en mujeres con enfermedad


inflamatoria pélvica (EIP): Dar tratamiento a mujeres sintomáticas con VB. La VB se ha
asociado con la presencia de endometritis posterior a cesáreas u otros procedimientos
quirúrgicos ginecológicos: Dar tratamiento a mujeres asintomáticas con VB que se van a
someter a algún procedimiento quirúrgico ginecológico (Inserción de DIU, biopsia
endometrial, cesárea electiva, legrados). La VB se ha asociado con abortos, parto
pretérmino y ruptura prematura de membranas. En embarazadas sintomáticas con
antecedente de parto pretérmino y/o ruptura prematura de membranas (RPM), está
indicado el tratamiento con Metronidazol oral. No se observa una disminución en la
frecuencia de recaídas o re-infección de VB, cuando se da tratamiento a la pareja
masculina: No está indicada la detección ni el tratamiento en la(s) pareja(s) en el caso de
VB. El tratamiento de elección para VB es: 1) Metronidazol 500 mg, vía oral dos veces al
día por 7 días; o 2) Metronidazol 2 g vía oral en una sola dosis. Como tratamiento
alternativo en caso de intolerancia al Metronidazol o alergia: 1) Clindamicina crema
vaginal 2%, una vez al día por 7 días o 2) Clindamicina 300mg, oral 2 veces al día por 7
días. EL TRATAMIENTO DE ELECCIÓN DE LA VAGINOSIS BACTERIANA ES EL
METRONIDAZOL, LA CLINDAMICINA SE UTILIZA COMO MANEJO ALTERNO.

TINIDAZOL. Unas cuantas pacientes poseen cepas de TRICHOMONA que son altamente resistentes al
Metronidazol, pero estos microorganismos por lo general son sensibles al Tinidazol. En
las pacientes con infecciones recurrentes o en aquellas que no responden al tratamiento
inicial y que lo han seguido al pie de la letra, se deben realizar cultivos y pruebas de
sensibilidad. El Tinidazol oral a dosis de 500 mg cada 8hrs durante 7 días o cada 6 hrs.
durante 14 días, ha sido eficaz en las pacientes con microorganismos resistentes. EL
TINIDAZOL ES LA SEGUNDA ALTERNATIVA TERAPÉUTICA EN CASO DE TRICOMONIASIS.

Bibliografía:
1. GUÍA DE PRÁCTICA CLÍNICA, DIAGNÓSTICO Y TRATAMIENTO DE LA VAGINITIS INFECCIOSA EN
MUJERES EN EDAD REPRODUCTIVA EN EL PRIMER NIVEL DE ATENCIÓN. MÉXICO: SECRETARIA DE
SALUD; DICIEMBRE 2014.

http://www.cenetec.salud.gob.mx/descargas/gpc/CatalogoMaestro/081_GPC_Vaginitisinfec1NA/Vaginitis_ER_CENETEC.pdf

FIN DEL CASO CLÍNICO SERIADO


ANÁLISIS DEL CASO CLÍNICO

IDENTIFICACIÓN DEL REACTIVO


Area: MEDICINA INTERNA
Especialidad: NEFROLOGÍA
Tema: INSUFICIENCIA RENAL CRÓNICA, NEFROPATÍA DIABÉTICA E
HIPERTENSA
Subtema: IRCT Y UREMIA

CASO CLÍNICO CON UNA PREGUNTA

HOMBRE DE 60 AÑOS QUE ACUDE A CONSULTA CON RESULTADO DE TASA DE FILTRACIÓN


GLOMERULAR DE 35 ML/MIN/1.73M2.

HOMBRE DE 60 AÑOS.

-.

-.

-.

TASA DE FILTRACIÓN GLOMERULAR DE 35


ML/MIN/1.73M2. ESTADIO 3 DE LA
ENFERMEDAD RENAL CRÓNICA.

62 - CON BASE EN ESTE RESULTADO SU PLAN DE ACCIÓN TERAPÉUTICO CONSISTIRÁ EN:


REDUCCIÓN DE La IRC se divide en cinco estadios según la TFG y la evidencia de daño renal. El
FACTORES DE ESTADIO 1 se caracteriza por la presencia de daño renal con TFG normal o aumentada,
RIESGO PARA es decir mayor o igual a "90ml"/min/1.73m2. Por lo general la enfermedad es
ENFERMEDAD asintomática. Las guías de la National Kidney Foundation clasifican a los pacientes que
RENAL CRÓNICA. tienen diabetes y microalbuminuria con una TFG normal en el estadio 1. El ESTADIO 2
se establece por la presencia de daño renal asociada con una ligera disminución de la
TFG entre "89 y 60" ml/min/1.73m2. Usualmente el paciente no presenta síntomas y el
diagnóstico se realiza de manera incidental. Aproximadamente el 75% de los
individuos mayores de 70 años se encuentran en este estadio. La función renal global
es suficiente para mantener al paciente asintomático, debido a la función adaptativa
de las nefronas. El correcto plan de actuación en ambos estadios radica en el
diagnóstico precoz y en el inicio de medidas preventivas con el fin de evitar la
progresión. SEGUIMIENTO EN ESTADIOS 1 Y 2. Tratamiento de morbilidad asociada.
Intervenciones para tratar la progresión. Reducción de factores de riesgo. Utilizar
nefroprotectores. Vigilar signos de alarma.

MANEJO LO PRIMERO ES IDENTIFICAR CON BASE EN LA TFG EL ESTADIO EN EL QUE SE


NEFROPROTECTOR ENCUENTRA EL PACIENTE Y CON BASE EN ESTE DETERMINAR EL MANEJO COMO
Y TRATAMIENTO SIGUE: El ESTADIO 3 es una disminución moderada de la TFG entre "30 y 59"
DE ml/min/1.73m2. Se ha dividido el estadio 3 en dos etapas. La etapa temprana 3a,
COMPLICACIONES. pacientes con TFG entre 59 y 45 ml/min/1.73m2 y la etapa tardía 3b con TFG entre 44 y
30 ml/min/1.73m2. Al disminuir la función renal, se acumulan sustancias tóxicas en el
torrente sanguíneo que ocasionan uremia. Los pacientes comúnmente presentan
síntomas y complicaciones típicas de la como hipertensión, anemia y alteraciones del
metabolismo óseo. Algunos de los síntomas incluyen fatiga relacionada con la anemia,
edema por retención de agua corporal, dificultad para conciliar el sueño debido a
prurito y calambres musculares, cambios en la frecuencia urinaria, espuma cuando hay
proteinuria y coloración oscura que refleja hematuria. Se aumentan los riesgos de
enfermedad cardiovascular. SEGUIMIENTO. Identificar y modificar factores de
progresión. "Detectar Y TRATAR complicaciones de la ERC". "Utilizar
nefroprotectores". Ajustar fármacos de acuerdo a TFG Evitar nefrotoxinas Revisión por
el nefrólogo una vez al año.

PREPARAR PARA El ESTADIO 4 se refiere a daño renal avanzado con una disminución grave de la TFG
TERAPIA DE entre "15 y 302 ml/min/1.73m2. Los pacientes tienen un alto riesgo de progresión al
REEMPLAZO estadio 5 y de complicaciones cardiovasculares. A los síntomas iniciales del estadio
RENAL. anterior se agregan náusea, sabor metálico, aliento urémico, anorexia, dificultad para
concentrarse y alteraciones nerviosas como entumecimiento u hormigueo de las
extremidades. SEGUIMIENTO. Preparar para tratamiento sustitutivo de la función renal
(DP, HD o TR). Control individualizado. Revisión por Nefrólogo cada 1-3 meses.

COMENZAR ESTADIO 5: FG menor a 15 ml/min/1,73 m2. Cursa con osteodistrofia renal y trastornos
PROGRAMA DE endocrinos y dermatológicos sobreañadidos a las alteraciones previas. Dicho estadio
DIÁLISIS. corresponde al síndrome urémico, en el que además de las medidas previas es
obligada la valoración del inicio del tratamiento renal sustitutivo: diálisis —
peritoneal/hemodiálisis— o trasplante renal. Es conocido que los pacientes
pertenecientes al estadio 5 no reciben una atención adecuada en estadios anteriores, y
que en un alto porcentaje son remitidos tardíamente a los servicios de nefrología
desde los centros de Atención Primaria y especializada. Entre los motivos destacan:
edad avanzada, severa comorbilidad, ausencia de síntomas, factores económicos y un
diagnóstico tardío. SEGUIMIENTO. Revisión conjunta entre medicina interna y
nefrología. Inicio oportuno del tratamiento sustitutivo.

Bibliografía:
1. GUÍA DE PRÁCTICA CLÍNICA, DIAGNÓSTICO Y TRATAMIENTO DE LA ENFERMEDAD RENAL
CRÓNICA TEMPRANA. MÉXICO: SECRETARIA DE SALUD, 2009. 2. GUÍA DE REFERENCIA RÁPIDA,
DIAGNÓSTICO Y TRATAMIENTO DE LA ENFERMEDAD RENAL CRÓNICA TEMPRANA. MÉXICO:
SECRETARIA DE SALUD, 2009.

http://www.cenetec.salud.gob.mx /descargas/gpc/CatalogoMaestro/335_IMSS_09_Enfermedad_Renal_Cronica_Temprana/EyR_IMSS_335_09.pdf
ANÁLISIS DEL CASO CLÍNICO

IDENTIFICACIÓN DEL REACTIVO


Area: CIRUGÍA
Especialidad: URGENCIAS
Tema: TRAUMA CRANEOENCÉFALICO
Subtema: TRAUMA CRANEOENCEFÁLICO

CASO CLÍNICO CON UNA PREGUNTA

MUJER DE 23 AÑOS DE EDAD QUE ES LLEVADA AL SERVICIO DE URGENCIAS POSTERIOR A TRAUMA EN


CRÁNEO. DURANTE SU VALORACIÓN NEUROLÓGICA USTED LA CLASIFICA CON TRAUMA
CRANEOENCEFÁLICO MODERADO.

mujer de 23 años.

-.

-.

se clasi ca el trauma craneoencefálico


como moderado.

-.

63 - TODOS LOS PACIENTES CON ESTE GRADO DE TRAUMA TENDRÁN EN COMÚN:

QUE DEBEN SER El manejo inicial del Traumatismo Craneoencefálico (TCE moderado) consiste en: -
INGRESADOS A Evaluación inicial y resucitación siguiendo la secuencia ABCDE. - TAC para descartar la
UNA UNIDAD DE presencia de hematomas que requieren de tratamiento neuroquirúrgico. - Admisión a
CUIDADOS una unidad de atención médica para su observación. - Prevención de la lesión cerebral
INTENSIVOS PARA secundaria mediante la prevención o mantenimiento de la presión arterial y de la
SU VIGILANCIA. oxemia. - Vigilar y mantener estabilizados ABCDE. - Admisión hospitalaria de larga
estancia, a menos que haya una rápida mejoría clínica, TAC normal y sin otros factores
de riesgo. - Consulta neuroquirúrgica en caso de que no exista mejoría clínica o con
TAC anormal. - Determinar la presencia de amnesia postraumática. TODO PACIENTE
CON TCE GRAVE DEBE INGRESAR A UNA UNIDAD DE CUIDADOS INTENSIVOS PARA
SU ADECUADA MONITORIZACIÓN Y MANEJO; EN EL CASO DE TCE MODERADO, ESTÁ
SUGERIDO PERO NO ES OBLIGATORIO, BASTA CON UNA ESTANCIA HOSPITALARIA
CON VIGILANCIA ESTRECHA.
LA PRESENCIA DE Aproximadamente 10% de los pacientes con trauma craneoencefálico vistos en la sala
HEMIPARESIAS de urgencias tienen traumatismo craneoencefálico moderado. Ellos son aún capaces
DURANTE LA de seguir órdenes sencillas, pero habitualmente están confusos o somnolientos y
EXPLORACIÓN pueden tener déficit neurológicos focales tales como hemiparesia. Aproximadamente
NEUROLÓGICA. 10 a 20% de estos pacientes se deterioran y caen en coma. EN NINGÚN CASO EL 100%
DE LOS PACIENTES CON ÉSTE GRADO DE TRAUMA PRESENTA HEMIPARESIAS.

TENER GLASGOW La ESCALA DE COMA DE GLASGOW (ECG) se usa como una medida clínica objetiva de
MENOR DE 9 trauma cerebral. Los pacientes que abren los ojos espontáneamente, obedecen
órdenes y están orientados, tienen un puntaje de 15, mientras que pacientes que están
flácidos, no abren los ojos ni hablan tienen un puntaje mínimo de 3 puntos. - Una
puntuación de Glasgow de 8 o menos se considera como TCE severo. - LOS PACIENTES
CON TRAUMA CEREBRAL Y PUNTAJE DE ECG DE 9 A 12 SON CATEGORIZADOS COMO
MODERADOS. - Aquellos con un puntaje de ECG de 13 a 15 son designados como
leves. LOS PACIENTES CON UN GLASGOW MENOS A 9 DEBERÁN SER CLASIFICADOS
COMO GRAVES, POR LO QUE ESTA RESPUESTA NO CORRESPONDE AL CASO.

QUE DEBERÁ El TRAUMATISMO CRANEOENCEFÁLIC (TCE) es un impacto súbito, que involucra un


COMPLEMENTARSE intercambio brusco de energía mecánica entre el cráneo y una superficie, por lo tanto
SIEMPRE SU provoca lesiones físicas directas sobre el cráneo y su contenido que, a su vez, se
EVALUACIÓN CON manifestarán con un deterioro de las funciones neurológicas y neuroquímicas de
TAC DE CRÁNEO. grado variable, que irán acorde con la severidad de la lesión. Se debe solicitar
tomografía axial computada craneal en pacientes con TCE y alguno de los siguientes
factores de riesgo: - Glasgow <13 en cualquier momento tras el traumatismo (TCE
moderado-grave). - Glasgow igual a 13 o 14 a las 2 hrs del traumatismo. - Sospecha
de fractura craneal abierta o fractura con hundimiento - Cualquier signo de fractura
de la base del cráneo. - Crisis convulsiva postraumática. - Déficit neurológico focal. -
Cefalea persistente generalizada. - Vómito en dos o más episodios. - Evidencia de
traumatismo por encima de la clavícula. - Amnesia anterógrada de >30 minutos. Al
momento del ingreso a urgencias, se obtiene una breve historia y se asegura una
estabilización cardiopulmonar antes de la evaluación neurológica. EN TODOS LOS
PACIENTES CON TCE MODERADO Y GRAVE, DEBE OBTENERSE UNA TAC DE CRÁNEO y
se debe contactar a un neurocirujano. Se recomienda hacer una TAC de seguimiento
en 12 o 24 horas si la TAC inicial es anormal o el paciente presenta deterioro en su
estado neurológico.

Bibliografía:
1. GUÍA DE PRÁCTICA CLÍNICA, DETECCIÓN Y MANEJO INICIAL DE LA LESIÓN CRANEAL
TRAUMÁTICA AGUDA EN EL ADULTO EN EL PRIMER NIVEL DE ATENCIÓN. MÉXICO: SECRETARIA DE
SALUD; 2013. 2. GUÍA DE REFERENCIA RÁPIDA, DETECCIÓN Y MANEJO INICIAL DE LA LESIÓN
CRANEAL TRAUMÁTICA AGUDA EN EL ADULTO EN EL PRIMER NIVEL DE ATENCIÓN. MÉXICO:
SECRETARIA DE SALUD; 2013. 3. COMITÉ DE TRAUMA DEL COLEGIO AMERICANO DE CIRUJANOS,
PROGRAMA AVANZADO DE APOYO VITAL EN TRAUMA PARA MÉDICOS. ATLS, 7A EDICIÓN. USA. PP
167.

http://www.cenetec.salud.gob.mx/descargas/gpc/CatalogoMaestro/016_GPC_TCEenelAdulto/SSA_016_08_EyR.pdf
ANÁLISIS DEL CASO CLÍNICO

IDENTIFICACIÓN DEL REACTIVO


Area: PEDIATRÍA
Especialidad: INFECTOLOGIA PEDIÁTRICA
Tema: BRONQUIOLITIS
Subtema: BRONQUIOLITIS

CASO CLÍNICO CON UNA PREGUNTA

LACTANTE DE 1 AÑO, EL CUAL PRESENTA, TOS PRODUCTIVA, RINORREA HIALINA Y DIFICULTAD


RESPIRATORIA POR LO QUE ES LLEVADO AL SERVICIO DE URGENCIAS. A LA EXPLORACION PRESENTA
HIPOVENTILACION DE AMBOS CAMPOS PULMONARES CON SIBILANCIAS BILATERALES, S-A DE 2-3.

lactante de 1 año de edad.

cuadro catarral previo

HIPOVENTILACION DE AMBOS CAMPOS


PULMONARES CON SIBILANCIAS
BILATERALES, S-A DE 2-3. cuadro
característico por edad y clínica con
bronquiolitis.

64 - EL CUADRO CLÍNICO DEL PACIENTE PUEDE SER EXPLICADO POR::


OBSTRUCCIÓN "La lesión más importante EN LA BRONQUIOLITIS, se asienta en el bronquiolo, con edema
POR EDEMA Y de la pared, obstrucción de la luz con moco y detritus celulares." También existe un
ACUMULACIÓN infiltrado inflamatorio en el intersticio peribronquiolar que comprime extrínsecamente la
DE MOCO vía aérea, factor de obstrucción que se agrega al intramural e intraluminal. No existe
uniformidad en las lesiones, se hallan bronquiolos totalmente obstruídos, otros
parcialmente y aún otros permeables, lo que lleva a zonas alveolares mal ventiladas,
atelectasiadas o hiperinsufladas, así como otras con hiperventilación compensatoria, lo
que genera trastornos de la perfusión. El desequilibrio ventilación/ perfusión origina
hipoxemia que según la gravedad puede llegar a ser progresiva y comprometer
seriamente al niño.

OCUPACIÓN DE BRONQUIOLITIS Y VIRUS SINCITIAL RESPIRATORIO. MANIFESTACIONES CLÍNICAS: El virus


LOS ALVEOLOS sincitial respiratorio (respiratory syncytial virus, RSV) origina un cuadro agudo en las vías
CON DETRITUS respiratorias en personas de cualquier edad. En los lactantes y en niños de corta edad,
CELULARES RSV pudiera ser la causa más importante de bronquiolitis y neumonía. En las primeras
semanas de vida, en particular en los productos pretérmino, la infección por RSV puede
ocasionar signos mínimos en las vías respiratorias, y las principales manifestaciones son
letargia, irritabilidad, poco apetito, a veces acompañado de episodios apneicos. Muchos
de los pequeños infectados por RSV que habían estado sanos no necesitan
hospitalización, y muchos de los que están hospitalizados mejoran con medidas de apoyo
y pueden ser dados de alta antes de cinco días. Los cuadros que agravan el peligro de
infección grave o letal por RSV incluyen cardiopatías congénitas cianóticas o complicadas,
en particular aquellas que causan hipertensión pulmonar, neumopatías primarias, en
particular displasia broncopulmonar; premadurez y enfermedades por inmunodeficiencia
o tratamiento que origina inmunosupresión en cualquier edad. No se conoce en detalle el
vínculo entre la bronquiolitis con RSV en los comienzos de la vida y la enfermedad
reactiva de vías respiratorias que surge ulteriormente. Después de la bronquiolitis por
dicho virus, algunos niños terminarán por mostrar anormalidades a largo plazo en la
función pulmonar y presentar sibilancias repetitivas. El vínculo en cuestión quizá refleje
una predisposición basal a un cuadro reactivo de vías respiratorias y no consecuencia
directa de la infección por virus sincitial respiratorio. Casi todos los niños han padecido la
infección como mínimo una vez para cuando tienen dos años de edad, y es frecuente que
haya brotes de reinfección durante toda la vida. Los niños de mayor edad y los adultos
por lo común muestran enfermedad de vías respiratorias altas, pero también pueden
presentar una infección más grave en la porción baja de dichas vías. Se observa a veces
exacerbación del asma u otras neumopatías crónicas.

OBSTRUCCIÓN BRONQUIOLITIS CAUSAS: El agente patógeno es un paramixovirus de RNA con cubierta


POR NECROSIS que no posee neuraminidasa ni glucoproteínas de superficie de tipo hemaglutinina. Se
DEL EPITELIO han identificado dos graves tipos (A y B), y a menudo circulan conjuntamente. No se ha
RESPIRATORIO precisado la importancia clínica y epidemiológica de las variaciones en las cepas, pero las
pruebas sugieren que las diferencias antigénicas pueden modificar la susceptibilidad a la
infección y que algunas cepas pueden ser más virulentas que otras. ASPECTOS
EPIDEMIOLÓGICOS: Los seres humanos son la única fuente de infección. La transmisión
por lo regular se hace por contacto directo o cercano con secreciones contaminadas, que
pueden incluir partículas u objetos inanimados. El virus persiste en superficies del entorno
durante varias horas y 30 min o más en las manos. La infección en personal hospitalario y
otros sujetos puede ocurrir por autoinoculación con secreciones contaminadas. Es
importante poner en práctica rigurosamente las normas de erradicación de infecciones
para disminuir el peligro de transmisión de RSV, de tipo nosocomial. La propagación
nosocomial de RSV a personas que reciben un órgano en trasplante o pacientes con
anormalidades cardiopulmonares o cuadros de inmunodeficiencia se ha vinculado con
enfermedad grave y letal en niños y adultos. El virus sincitial respiratorio suele afectar en
epidemias anuales durante el invierno y comienzos de la primavera en clima templado.
Sin embargo, durante todo el año pueden surgir casos esporádicos de la infección. Es
frecuente que se propague entre miembros del círculo familiar y contactos en la unidad
de cuidado pediátrico, incluidos adultos. El periodo de dispersión o secreción del virus
suele ser de tres a ocho días, pero puede durar más, especialmente en lactantes de corta
edad en que dicho fenómeno puede continuar incluso tres a cuatro semanas. El periodo
de incubación va de dos a ocho días, y es más frecuente que abarque cuatro a seis días.
DISMINUCIÓN BRONQUIOLITIS MÉTODOS DIAGNÓSTICOS: Se cuenta en el comercio con métodos
DEL FLUJO diagnósticos rápidos que incluyen técnicas de inmunofluorescencia y inmunoanálisis
SANGUÍNEO enzimático para detectar el antígeno vírico en muestras de nasofaringe, y por lo común
PULMONAR generan resultados fidedignos. La sensibilidad de tales técnicas en comparación con el
cultivo varía entre 53 y 96%, y muchas están en límites de 80 a 90%. Aislar el virus de las
secreciones nasofaríngeas en cultivos celulares requiere el transcurso de tres a cinco días,
pero los resultados y la sensibilidad varían de un laboratorio a otro porque los métodos
de aislamiento son difíciles y lentos y RSV es un virus relativamente lábil. Es necesario
consultar a un laboratorio de virología experto en cuanto a métodos óptimos de reunión
y transporte de las muestras. Cabe recurrir a estudios serológicos hechos en muestras de
suero de fase aguda y de convalecencia para confirmar la presencia de la infección; sin
embargo, es poca la sensibilidad del diagnóstico serológico de la infección en lactantes de
corta edad. Se ha aplicado la reacción en cadena de polimerasa para detectar RSV en
muestras clínicas, pero no se le practica comercialmente. TRATAMIENTO: El tratamiento
primario es de sostén y debe incluir hidratación, valoración clínica cuidadosa del estado
respiratorio, que incluya medición de la saturación de oxígeno y empleo de dicho gas
suplementario, y si es necesario, ventilación mecánica. La ribavirina posee actividad
antivírica in vitro contra RSV, pero por lo común no se recomienda su aplicación en
aerosol en la infección por dicho virus. En estudios en que los testigos han recibido
placebo, no se ha demostrado la disminución constante de la necesidad de ventilación
mecánica, duración de la permanencia en la unidad de cuidado intensivo para niños ni en
la duración de la hospitalización entre quienes recibieron ribavirina. Contribuyen a esta
controversia factores como el costo alto del fármaco, la aplicación en aerosol (ineficaz), la
preocupación por los posibles efectos tóxicos en profesionales y asistenciales expuestos y
los resultados antagónicos de estudios de eficacia. Las decisiones en cuanto a la
administración de dicho fármaco deben hacerse con base en las circunstancias clínicas
particulares y en la experiencia del médico. Corticosteroides. En niños hospitalizados que
tienen bronquiolitis por RSV no son eficaces ni están indicados los corticosteroides.
Antimicrobianos. Los agentes de esta categoría rara vez están indicados porque son raras
la infección pulmonar bacteriana y la bacteriemia en pequeños hospitalizados con
bronquiolitis o neumonía por virus sincitial respiratorio.

Bibliografía:
NELSON TRATADO DE PEDIATRÍA. NAPOLEÓN GONZÁLEZ SALDAÑA. MC GRAW HILL
INTERAMERICANA. EDICIÓN 7A. 2003. PAG. 471-478.
ANÁLISIS DEL CASO CLÍNICO

IDENTIFICACIÓN DEL REACTIVO


Area: CIRUGÍA
Especialidad: URGENCIAS
Tema: CHOQUE
Subtema: CHOQUE HIPOVOLÉMICO TRAUMÁTICO

CASO CLÍNICO CON UNA PREGUNTA

MUJER DE 30 AÑOS CON POLITRAUMATISMO POR CHOQUE AUTOMOVILÍSTICO. ES LLEVADA POR


AMBULANCIA A CENTRO TRAUMATOLÓGICO, DONDE A SU LLEGADA SE LE ENCUENTRA CON DATOS DE
CHOQUE GRADO IV.

MUJER DE 30 AÑOS.

POLITRAUMATIZADA POR CHOQUE


AUTOMOVILÍSTICO.

-.

PARA REALIZAR LA CLASIFICACIÓN DESDE


EL PUNTO DE VISTA CLÍNICO ES IDEAL
TENER LOS SIGUIENTES DATOS:
FRECUENCIA CARDIACA, TENSIÓN
ARTERIAL, PRESIÓN DE PULSO Y
DIURESIS.

-.

65 - LA REPOSICIÓN PRINCIPAL DE LÍQUIDOS BASADOS EN LA GRAVEDAD DE LA CLASIFICACIÓN


DEL GRADO DE CHOQUE DEBERÁ REALIZARSE CON:
NO REQUIERE HEMORRAGIA GRADO I Pérdida sanguínea de hasta 15% (750ml). Los síntomas son
REPOSICIÓN. mínimos, en situaciones no complicadas pudiera ocurrir una mínima taquicardia. No
existen cambios en la PA, presión del pulso o frecuencia respiratoria. En pacientes sanos
esta pérdida puede no requerir tratamiento, en ellos, el llenado capilar y otros
mecanismos suelen restaurarse dentro de las primeras 24 horas. Sin embargo, en
presencia de otras condiciones, esta cantidad de pérdida de sangre puede producir
síntomas clínicos. El remplazo de las pérdidas primarias con cristaloides, en estos casos,
corrige el estado circulatorio, habitualmente sin requerir de una trasfusión de sangre.
PÉRDIDA DE SANGRE:………….Hasta 750ml PÉRDIDA DE SANGRE(% DE VOLUMEN DE
SANGRE): Hasta 15%. FRECUENCIA DE PULSO: <100 x min PRESIÓN ARTERIAL: Normal
PRESIÓN DE PULSO: Normal o aumentada FRECUENCIA RESPIRATORIA: 14 a 20 X min
GASTO URINARIO: >30 ESTADO MENTAL: Ligeramente ansioso IMPORTANTE: tras la
reanimación inicial con cristaloides, se debe valorar la respuesta del paciente, esta
puede clasificarse en respuesta rápida, respuesta transitoria o ninguna respuesta.
Cuando la respuesta es transitoria es posible que el paciente requiera de mayores
reposiciones con soluciones, o en caso de pérdidas grado III la administración de
sangre. Cuando la respuesta es nula, aún en hemorragias leves, deberá considerarse la
presencia de un sangrado activo. RECUERDA: la valoración y evolución de estos
pacientes es dinámica, dada la condición del trauma, por lo que deben ser reevaluados
constantemente para ajustar el manejo según las necesidades y la respuesta. En
pérdidas mayores o hemorragias que no responden a la administración de cristaloides,
la necesidad de aporte de sangre es inmediata.

EXCLUSIVAMENTE HEMORRAGIA GRADO II Pérdida sanguínea entre un 15% y un 30% (750-1500ml). La


CRISTALOIDES. sintomatología incluye taquicardia, taquipnea y disminución en la presión del pulso.
Esta disminución en la presión del pulso es debida a un aumento en la diástole por
efecto de catecolaminas. Estos producen un aumento en el tono y resistencias
vasculares periféricas. A nivel de SNC se presenta ansiedad, hostilidad y pánico. El gasto
solo se afecta medianamente, y el flujo urinario es aproximadamente de 20 a 30
ml/hora. Estos pacientes pueden ser estabilizados con cristaloides. PÉRDIDA DE
SANGRE: 750 a 1500ml PÉRDIDA DE SANGRE (% DE VOLUMEN DE SANGRE): 15 a 30%
FRECUENCIA DE PULSO: <100 x min PRESIÓN ARTERIAL: Normal PRESIÓN DE PULSO:
Disminuida FRECUENCIA RESPIRATORIA: 20 a 30 x min GASTO URINARIO: 20 a 30
ml/hora ESTADO MENTAL: Medianamente ansioso

CRISTALOIDES HEMORRAGIA GRADO III Pérdida sanguínea entre un 30% y 40% (1500-2000ml). Puede
MÁS ser devastadora. Presentan muestras de una inadecuada perfusión como son
TRANSFUSIÓN DE taquicardia más acentuada, taquipnea, alteraciones en el SNC y una caída en la presión
SANGRE. sistólica. Siempre requerirán de una transfusión sanguínea, además de los cristaloides.
La cual esta basada en la respuesta del paciente a la reposición de líquidos. PÉRDIDA DE
SANGRE: 1500 a 2000ml PÉRDIDA DE SANGRE (% DE VOLUMEN DE SANGRE): 30% a
40%. FRECUENCIA DE PULSO: >120 x min PRESIÓN ARTERIAL: Disminuida PRESIÓN DE
PULSO: Disminuida FRECUENCIA RESPIRATORIA: 30 a 40 x min. GASTO URINARIO: 5 a
15 ml/hora ESTADO MENTAL: Ansioso, confuso LA TRANSFUSIÓN SANGUÍNEA EN ESTE
CASO, ESTÁ CONSIDERADA CON BASE A LA RESPUESTA TRAS LA ADMINISTRACIÓN DE
CRISTALOIDES.
TRANSFUSIÓN DE HEMORRAGIA GRADO IV Pérdida sanguínea de más de un 40% (>2000 ml). Pone en
SANGRE. peligro la vida si no es tratada enérgicamente. Se presenta una taquicardia marcada,
una disminución en la presión sistólica y una presión del pulso muy angosta o muchas
veces no se obtiene la presión diastólica. Gasto urinario ausente y existe una depresión
del SNC. La piel se encuentra fría y pálida. Pérdidas de más del 50% se manifiestan con
pérdida de la conciencia, pulso y de la PA. La reposición de líquidos idealmente debería
consistir en la reposición de la sangre perdida con sangre. RECUERDA: la transfusión
sanguínea además de intervenir en la reposición de líquidos, provee mejor perfusión a
los tejidos ya que contiene células sanguíneas, lo cual la hace ideal en casos severos
donde no está en juego sólo el volumen, sino también la funcionalidad de los tejidos
por falta de oxigenación. PÉRDIDA DE SANGRE: > 2000ml PÉRDIDA DE SANGRE (% DE
VOLUMEN DE SANGRE): > 40% FRECUENCIA DE PULSO: >140 x min PRESIÓN
ARTERIAL: Disminuida PRESIÓN DE PULSO: Disminuida FRECUENCIA RESPIRATORIA:
>35 x min GASTO URINARIO: Nulo o despreciable ESTADO MENTAL: Confuso, letárgico
LA BASE DEL TRATAMIENTO EN ESTADO DE CHOQUE GRADO IV ES LA REPOSICIÓN DE
LÍQUIDOS MEDIANTE TRANSFUSIÓN SANGUÍNEA. A DIFERENCIA DE OTROS GRADOS
NO DEBERÁ INICIARSE LA REPOSICIÓN CON CRISTALOIDES Y ESPERAR RESPUESTA,
SINO TRASFUNDIR DE INMEDIATO. OJO: aunque en este grado también se aplican
cristaloides como parte del manejo general, NO SON LA BASE del manejo, la sangre si
lo es. IMPORTANTE: ESTOS REACTIVOS SON TIPO ENARM POR LO QUE DEBES PONER
ESPECIAL ATENCIÓN EN LO QUE LAS PREGUNTAS SOLICITAN, PARA ESTE CASO: LA
REPOSICIÓN “PRINCIPAL”… DEBERÁ REALIZARSE CON… LA CLAVE ESTÁ EN ELEGIR EL
MEDIO DE REPOSICIÓN DE VOLUMEN MÁS IMPORTANTE PARA ESTE GRADO.

Bibliografía:
1. PROGRAMA AVANZADO DE APOYO VITAL EN TRAUMA PARA MÉDICOS. ATLS. COMITÉ DE TRAUMA
DEL COLEGIO AMERICANO DE CIRUJANOS. ESTADOS UNIDOS. 8ª EDICIÓN, 2008.
ANÁLISIS DEL CASO CLÍNICO

IDENTIFICACIÓN DEL REACTIVO


Area: MEDICINA INTERNA
Especialidad: HEMATOLOGÍA
Tema: TRANSTORNOS DE LA HEMOSTASIA
Subtema: HEMOFILIA A Y B

CASO CLÍNICO CON UNA PREGUNTA

HOMBRE DE 23 AÑOS SIN ANTECEDENTES DE IMPORTANCIA. DESDE HACE 3 DÍAS PRESENTA AUMENTO
DE VOLUMEN EN LA RODILLA DERECHA ACOMPAÑADO DE DOLOR INTENSO. ES VALORADO POR EL
SERVICIO DE ORTOPEDIA EN DONDE SE HACE EL DIAGNÓSTICO DE HEMARTROSIS.

HOMBRE DE 23 AÑOS.

-.

HACE 3 DÍAS PRESENTA AUMENTO DE


VOLUMEN EN LA RODILLA DERECHA
ACOMPAÑADO DE DOLOR INTENSO.

DIAGNÓSTICO DE HEMARTROSIS.

-.

66 - EL TRATAMIENTO QUE DEBE RECIBIR DE FORMA INMEDIATA ES:

PLASMA FRESCO Se recomienda que todo paciente con hemofilia hereditaria sea tratado con
CONGELADO concentrados de factores de coagulación (CFC) derivados de plasma tratados con
doble proceso de inactivación viral o CFC recombinantes. Los recombinantes
indicados, de preferencia, que sean de tercera generación. No se recomienda el uso
de plasma fresco congelado y/o crioprecipitados como terapia de reemplazo. ESTA
ES LA ÚLTIMA OPCIÓN DE TRATAMIENTO, EN CASO DE QUE NO SE CUENTE CON EL
CONCENTRADO DE FACTORES, CON POBRES RESULTADOS.
FACTOR VIII DEFINICIÓN DE HEMOFILIA. La HEMOFILIA es un desorden hemorrágico infrecuente,
LIOFILIZADO originado por mutaciones en el cromosoma X, que genera una disminución o
ausencia de actividad funcional de los factores VIII y IX. Alrededor de un tercio de
dichas mutaciones son espontáneas, sin antecedente familiares. Su consecuencia es
la aparición de hemorragias internas y/o externas de severidad variable según su
localización. La hemofilia A es más frecuente que la B (80-85% y de 15-20%
respectivamente). La expresión clínica de la hemofilia es la hemorragia en diversas
localizaciones del organismo: articulaciones, músculos en miembros inferiores y
superiores, hemorragias internas, aparato digestivo, urinario y otros en menor
frecuencia. Entre estos últimos, la localización en el Sistema Nervioso Central (SNC)
es la hemorragia más severa de la hemofilia y que ocasiona mayor morbi-
mortalidad. Los sitios más afectados por hemorragia son las articulaciones (70% a
80%), más frecuente en rodillas, tobillos y codos, menos frecuente en hombros,
cadera y carpos; en músculos (10 a 20%), otros sitios (5%) y hemorragia intracraneal
? 5%. La afección de las articulaciones origina una patología característica
denominada artropatía hemofílica que desarrolla lesiones articulares progresivas,
que conducen a una severa limitación de la función articular, artralgias y serias
secuelas invalidantes. Por su frecuencia y evolución crónica, la artropatía hemofílica
es la complicación de la enfermedad con mayor morbilidad, siendo el objetivo
primario del tratamiento, la prevención de su desarrollo. TRATAMIENTO: Se
recomienda que todo paciente con hemofilia hereditaria sea tratado con CFC
(concentrados de factores de coagulación) derivados de plasma tratados con doble
proceso de inactivación viral o CFC recombinantes. Los recombinantes indicados, de
preferencia, que sean de tercera generación. En caso de no contar aún con
concentrados recombinantes, emplear concentrados de factores de la coagulación
de origen plasmático que al menos tengan dos procesos de inactivación viral. Los
pacientes con hemorragia deben recibir tratamiento con terapia sustitutiva del
factor deficiente lo más pronto posible, de preferencia en las dos horas siguientes de
la manifestación inicial. En caso de hemorragia grave o que pone en peligro la vida
del paciente debe iniciarse la terapia de sustitución inmediatamente, después se
realizarán los exámenes y el traslado a unidades médicas que cuenten con
especialistas y el equipo necesario para su manejo integral. TRATAMIENTO
SUSTITUTIVO. En el tratamiento de la HEMOFILIA TIPO A el factor VIII se administra
cada 8 horas mínimo por 24 horas, luego puede continuar la dosis cada 12 hrs.,
manteniendo un nivel mínimo de factor de 50% hasta 3 semanas y no menos de 2.
En el tratamiento de la HEMOFILIA TIPO B, se administra factor IX cada 12 horas
mínimo por 24 horas, luego puede continuar la dosis cada 12 a 18 hrs, manteniendo
un nivel mínimo de factor de 50% hasta 3 semanas y no menos de 2. Se recomienda
que todos los pacientes con hemofilia sean tratados en el 100% de los casos con
concentrados de factores de coagulación. La desmopresina no es eficaz para el
tratamiento de pacientes con EVW tipo 3 y podría exacerbar la trombocitopenia que
a menudo se presenta en pacientes con EVW tipo 2B.

DESMOPRESINA SI NO SE CUENTA CON EL CONCENTRADO DE FACTOR, LA DESMOPRESINA ES UNA


BUENA OPCIÓN TERAPÉUTICA PARA LA HEMOFILIA A. Existen otras opciones
farmacológicas para el tratamiento del paciente con hemofilia como son la
desmopresina (DDAVP) y los antifibrinolíticos ácido épsilon aminocapróico y ácido
tranexámico. La DDAVP es un análogo sintético de la vasopresina, su mecanismo de
acción es multifactorial: incrementa los niveles plasmáticos de Factor VIII y del
Factor de Von Willebrand, estimula la adhesión plaquetaria e incrementa la
expresión del factor tisular. La DDAVP puede ser el tratamiento de elección para
pacientes con hemofilia A leve o moderada que han respondido a la prueba
terapéutica, evitando costos y riesgos secundarios de los CFC. LA DDAVP NO AFECTA
LOS NIVELES SÉRICOS DE FACTOR IX POR LO TANTO NO ESTÁ INDICADA EN
HEMOFILIA B. Sus principales efectos secundarios son la retención de líquidos y la
hipertensión. Su uso continuo por más de 72 hrs produce taquifilaxia. Se
recomienda usar DDAVP como tratamiento en hemorragias leves, en el paciente con
hemofilia A leve o moderada que responda a la misma. La dosis por vía nasal es de
150 ?g en cada narina en personas con peso > 40kg, o una sola aplicación si pesa <
40 Kg. Por vía parenteral la dosis es de 0.3 ?g/kg c/12 h. Sin exceder de 3 días.
ANTIFIBRINOLÍTICOS LOS ANTIFIBRINOLÍTICOS SON UNA BUENA OPCIÓN TERAPÉUTICA “ADYUVANTE”
PARA LA HEMOFILIA. Los antifibrinolíticos como el ácido tranexámico y el ácido
épsilon aminocapróico (AEAC) son útiles como terapia coadyuvante en pacientes con
hemofilia tanto A como B, inhiben la fibrinolisis disminuyendo la activación del
plasminógeno e incrementa la estabilidad del coágulo. Se recomienda en
hemorragia de mucosas, ya sea oral, nasal o menorragia, utilizar ácido épsilon
aminocapróico, administrado por vía oral o intravenoso, cada 4 a 6 horas, máximo
24 g/día en adultos. Está contraindicado en coagulación intravascular diseminada,
enfermedad tromboembólica y hemorragia de vías urinarias. OJO: Aunque no existe
evidencia contundente para el uso de antifibrinolíticos en el tratamiento de
hemorragias orales se recomienda emplear éstos como enjuague bucal, en caso de
gingivorragia o extracciones dentales.

Bibliografía:
1. DIAGNÓSTICO Y TRATAMIENTO HEMOFILIA A Y B EN POBLACIÓN MAYOR O IGUAL DE 16 AÑOS
DEL SEGUNDO Y TERCER NIVEL DE ATENCIÓN. EVIDENCIAS Y RECOMENDACIONES. GUÍA DE
PRÁCTICA CLÍNICA. MÉXICO: SECRETARÍA DE SALUD. 16/03/2017. 2. LONGO DL, FAUCI AS, KASPER
DL, HAUSERSL, JAMESON JL, LOSCALZOJ. HARRISON. PRINCIPIOS DE MEDICINA INTERNA, 18A
EDICIÓN. MC GRAW HILL. NEW YORK, USA. 2012, PP 975-977.

http://www.cenetec-difusion.com/CMGPC/IMSS-178-09/ER.pdf
ANÁLISIS DEL CASO CLÍNICO

IDENTIFICACIÓN DEL REACTIVO


Area: MEDICINA INTERNA
Especialidad: CARDIOVASCULAR
Tema: CARDIOPATÍA ISQUÉMICA
Subtema: INFARTO AGUDO DEL MIOCARDIO

CASO CLÍNICO CON UNA PREGUNTA

HOMBRE DE 45 AÑOS CON ANTECEDENTE RECIENTE DE INFARTO AGUDO AL MIOCARDIO QUE NO


PRESENTO ELEVACIÓN DEL SEGMENTO ST. DURANTE SU VALORACIÓN POSTERIOR SE DETECTA RIESGO
ALTO DE MUERTE SÚBITA.

HOMBRE DE 45 AÑOS

RECIENTE DE INFARTO AGUDO AL


MIOCARDIO QUE NO PRESENTO
ELEVACIÓN DEL SEGMENTO ST

SE DETECTA RIESGO ALTO DE MUERTE


SÚBITA

--

--

67 - CON EL FIN DE DISMINUIR ESTE RIESGO SE DEBERÁ INDICAR EL SIGUIENTE MANEJO:

WARFARINA MANEJO FRMACOLÓGICO DEL PACIENTE POSTINFARTADO. Todos los pacientes post
infartados deberán tener un tratamiento combinado con los siguientes fármacos: •
Inhibidores de la enzima convertidora de angiotensina • Ácido acetilsalicílico •
Betabloqueadores • Estatinas

CLOPIDOGREL • Una combinación de ácido acetilsalicílico y CLOPIDOGREL SON RECOMENDADOS PARA EL


MANEJO DEL PACIENTE SIN ELEVACIÓN DEL SEGMENTO ST, CON RIESGO MODERADO O
ALTO PARA MUERTE SÚBITA. • Esta combinación podría ser usada o continuada por doce
meses después del episodio coronario agudo. Posteriormente se retirará el clopidogrel y se
continuara con ácido acetil salicílico únicamente. SE ELIGE EL CLOPIDOGREL COMO
CORRECTA YA QUE CUMPLE CON LOS REQUISITOS DE DISMINUIR MUERTE SÚBITA Y ESTAR
INDICADO EN IAM SIN ELEVASIÓN DEL SEGMENTO ST.
METOPROLOL Los betabloqueadores deberán ser utilizados de manera indefinida después de un infarto
agudo del miocardio en pacientes con disfunción ventricular izquierda, sintomáticos o
asintomáticos, con el fin de prevenir una muerte súbita. AUNQUE ESTÁN INDICADOS NO ES
ESPECÍFICA DE ÉSTE PACIENTE.

ENALAPRIL Se debe prescribir un IECA a todos los pacientes con angina inestable e insuficiencia
cardíaca, disfunción de ventrículo izquierdo (fracción de eyección menor del 40%,
hipertensión arterial o diabetes). NO HAY INDICACIÓN PARA PRESCRIBIR UN IECA EN EL
PACIENTE.

Bibliografía:
1. GUÍA DE PRÁCTICA CLÍNICA, PREVENCIÓN SECUNDARIA Y REHABILITACIÓN CARDÍACA
POSTINFARTO DEL MIOCARDIO EN EL PRIMER NIVEL DE ATENCIÓN. MÉXICO: SECRETARIA DE
SALUD, 2008. 2. GUÍA DE REFERENCIA RÁPIDA, PREVENCIÓN SECUNDARIA Y REHABILITACIÓN
CARDÍACA POSTINFARTO DEL MIOCARDIO EN EL PRIMER NIVEL DE ATENCIÓN. MÉXICO:
SECRETARIA DE SALUD, 2008.

http://www.cenetec.salud.gob.mx/descargas/gpc/CatalogoMaestro/152_GPC_REHABILITACION_CARDIACA/SSA-152-08_REHABILITACIxN_POST_IMAEVR.pdf
ANÁLISIS DEL CASO CLÍNICO

IDENTIFICACIÓN DEL REACTIVO


Area: PEDIATRÍA
Especialidad: CRECIMIENTO Y DESARROLLO
Tema: CRECIMIENTO Y DESARROLLO NORMAL
Subtema: EL RECIEN NACIDO

CASO CLÍNICO CON UNA PREGUNTA

MUJER CON EMBARAZO DE 35 SEMANAS. ACUDE A SU REVISIÓN MENSUAL Y LE PIDE A USTED LE


INFORME SOBRE EL CRECIMIENTO DE SU HIJO.

--

EMBARAZO DE 35 SDG.

--

--

--

68 - USTED LE EXPLICA QUE EL PRINCIPAL FACTOR DEL CRECIMIENTO PRENATAL ESTÁ DADO POR:

HORMONAS El feto es considerado autónomo desde el punto de vista de su función tiroidea a partir de
TIROIDEAS la segunda mitad de la gestación. La falta de hormonas tiroideas no influencia la longitud
del feto aunque sí y de una manera importante la mineralización del esqueleto óseo y el
desarrollo del sistema nervioso central. • En condrocitos fetales en cultivo, nosotros
demostramos la presencia de receptores para la T3, así como efectos biológicos de esta
hormona. Concentraciones plasmáticas fetales de T3 estimulan parámetros relacionados
con la mineralización del cartílago y no tienen efectos sobre la síntesis de ADN. • Estos
datos experimentales se corresponden con las observaciones clínicas en el hipotiroidismo
congénito. EL PRINCIPAL EFECTO PARA FETAL RADICA EN LA MINERALIZACIÓN ÓSEA Y EL
DESARROLLO NEURAL PERO NO INFLUYE EN LA TALLA FETAL.
EL El lactógeno placentario humano (LPh) es una hormona peptídica, relacionada con la
LACTÓGENO hormona de crecimiento que es sintetizada exclusivamente por la placenta. El gen del
PLACENTARIO lactógeno placentario pertenece a la familia del gen de la hormona de crecimiento. La
síntesis de lactógeno placentario puede ser detectada en el sincitiotrofoblasto tan
temprano como entre el 5º y 10º días postimplantación. En la sangre materna sus niveles se
incrementan con el desarrollo del embarazo, siendo detectable desde la 6ª semana. • Es una
hormona con efectos anabólicos en la madre que tiene como objetivo final incrementar la
biodisponibilidad de glucosa y aminoácidos en la circulación placentaria y fetal. • En
resumen el lactógeno placentario sería una hormona con acciones anabolizantes a nivel
materno y fetal, y con posibles acciones directas sobre diversos tejidos fetales, entre ellos el
cartílago de crecimiento. Regularía el crecimiento fetal de una forma global incrementando
la biodisponibilidad de glucosa y aminoácidos en la circulación materna y fetal y facilitando
su captación por las células fetales. Además de esta acción global sobre el crecimiento un
posible efecto directo sobre el crecimiento del sistema esquelético también ha de
contemplarse a raíz de sus efectos sobre síntesis de ADN en el cartílago epifiseal. SE HAN
DESCRITO DELECIONES EN SU GEN QUE NO HAN CONDICIONADO RETRASO DE
CRECIMIENTO INTRAUTERINO Y HAN DADO LUGAR A RECIÉN NACIDOS NORMALES.

LA 1. HORMONA DE CRECIMIENTO PLACENTARIO: En las células del sincitiotrofoblasto


HORMONA placentario se expresa el gen de la hormona de crecimiento (GH2), sintetizándose hormona
DE de crecimiento. La hormona de crecimiento placentaria difiere de la hormona de
CRECIMIENTO crecimiento hipofisaria en 13 aminoácidos y es más básica. • Los efectos biológicos de la
hormona de crecimiento placentaria no son aún bien conocidos. Es capaz de unirse a las
proteínas de transporte de la hormona hipofisaria, así como a sus receptores y sus
concentraciones en plasma guardan buena relación con las de IGF-I materno. • Una función
anabolizante en la madre en el sentido de permitir la biodisponibilidad de nutrientes en la
circulación fetoplancentaria y regular de esta forma el crecimiento fetal ha sido sugerida.
Los factores que controlan su secreción son igualmente desconocidos y un efecto directo
sobre los tejidos fetales no ha sido demostrado, tal como sugiere el hecho de que no esté
presente en la circulación fetal. • Sin embargo, el hecho de que sea sintetizada por la
placenta y que ésta exprese receptores para la hormona de crecimiento, sugiere que podría
estar implicada también en la regulación del crecimiento placentario. SU EFECTO
BIOLÓGICO FETALES NO ESTÁN BIEN DEFINIDOS, PERMITE LA BIODISPONIBILIDAD DE
NUTRIMENTOS MATERNO-FETALES Y EL CRECIMIENTO PLACENTARIO PERO NO SE DEFINE
COMO PRINCIPAL FACTOR DE CRECIMIENTO INTRUTERINO. 2. HORMONA DE
CRECIMIENTO: El papel de la hormona de crecimiento en la regulación del crecimiento fetal
es materia de discusión. Receptores para la hormona de crecimiento han sido identificados
en células precursoras de la médula ósea durante la embriogénesis y en diversos tejidos
fetales humano. • El análisis de la longitud al nacimiento en recién nacidos con deleciones
del gen de la hormona de crecimiento, en el síndrome de resistencia periférica a la acción
de la hormona de crecimiento, e incluso en otras series con déficit congénito idiopático de
hormona de crecimiento ha mostrado que estos recién nacidos presentan una longitud
inferior a la de los recién nacidos normales, replanteándose el efecto de la hormona de
crecimiento en el crecimiento fetal global y en el crecimiento del esqueleto óseo. DURANTE
EL DESARROLLO FETAL LA NUTRICIÓN Y LOS FACTORES TISULARES DE CRECIMIENTO SON
SUS PRINCIPALES AGENTES REGULADORES, LA DEFICIENCIA DE GH PUEDE SER
COMPENSADA POR OTROS FACTORES.
EL FACTOR Factores de crecimiento con acción similar a la insulina IGF-I e IGF-II: durante el desarrollo
SIMILAR A LA fetal prácticamente todos los tejidos tienen la capacidad de sintetizar IGF-I e IGF II y sus
INSULINA proteínas de transporte. • El IFG II es el principal factor de crecimiento durante el desarrollo
TIPO 1 previo a la implantación y ejerce sus efectos tanto a través del receptor de IGF I como el
suyo propio. • En estadios posteriores del desarrollo embrionario y fetal se expresaría de
manera mucho más importante el IGF I y éste es el principal factor regulador del
crecimiento estimulando la síntesis de proteínas y la multiplicación celular. • Ante la
deleción parcial del gen de IGF I se ha podido comprobar en clínica humana la importancia
del IGF I tanto en el crecimiento intrauterino como en el postnatal, así como el papel
relevante de IGF I en la diferenciación y crecimiento de órganos fetales. • Los pacientes con
ésta alteración tienen un severo retraso de crecimiento intrauterino con una longitudes
menores a la esperada, peso bajo, perímetro craneal inferior a las semanas de gestación.
Además asociación importante a malformaciones como micrognatia, microcefalia, sordera
sensorial, clinodactilia bilateral y línea palmar única. El retraso de crecimiento se acentúa de
forma severa durante el desarrollo postnatal. Más recientemente se ha sugerido también
una relación entre polimorfismos del gen del IGF I y retraso de crecimiento intrauterino. EL
FACTOR DE CRECIMIENTO CON ACCIÓN SIMILAR A LA INSULINA IGF-1 ES EL PRINCIPAL
FACTOR QUE DETERMINA EL CRECIMIENTO Y DESARROLLO INTRAUTERINO CON EFECTOS
DETERMINANTES AUN EN LA EDAD POSTNATAL.

Bibliografía:
FISIOLOGÍA MÉDICA. W.F. GANONG. EL MANUAL MODERNO. EDICIÓN 18. 2002. PAG. 443.

Carrascosa A. Crecimiento intrauterino: factores reguladores. An Pediatr 2003;58(Supl 2):55-73.


ANÁLISIS DEL CASO CLÍNICO

IDENTIFICACIÓN DEL REACTIVO


Area: GINECOLOGÍA Y OBSTETRICIA
Especialidad: GINECOLOGÍA
Tema: PATOLOGÍA DE VULVA Y VAGINA
Subtema: CONDILOMATOSIS

CASO CLÍNICO SERIADO

MUJER DE 28 AÑOS DE EDAD CON ANTECEDENTE DE MADRE FALLECIDA POR CÁNCER


CERVICOUTERINO, ACUDE A CONSULTA PARA TOMA DE CITOLOGÍA CERVICAL. A LA EXPLORACIÓN SE
OBSERVAN TUMORACIONES EXOFÍTICAS, MÚLTIPLES MENORES A 10 MM, DE COLOR ROSADO EN
PERINÉ Y LABIOS MAYORES. SE INTRODUCE ESPEJO VAGINAL OBSERVANDO CUELLO ANTERIOR
ERITEMATOSO CON ECTOPRION IMPORTANTE.

Femenina 30 años.

-.

-.

tumoraciones exofíticas, múltiples


menores a 1 cm, de color rosado

-.

69 - LO MÁS PROBABLE ES QUE LA PACIENTE PRESENTE UNA INFECCIÓN POR LOS SIGUIENTES
SUBTIPOS VIRALES:

6 IMPORTANTE: LA ÚNICA LESIÓN COMPROBABLE DE INFECCIÓN POR VIRUS DEL PAPILOMA HUMANO
Y (VPH) A LA EXPLORACIÓN CLÍNICA, ES LA PRESENCIA DE CONDILOMAS (VERRUGAS). En el caso de las
11. verrugas anogenitales o condilomas acuminados, que aparecen sobre o alrededor de los genitales y el
ano tanto en los hombres como en las mujeres, se encuentran generalmente asociados los tipos de
VPH 6 y 11, que son considerados de importancia clínica y principales agentes etiológicos de estas
lesiones; sin embargo no están asociados con el desarrollo del cáncer cervical. LOS SUBTIPOS 6 Y 11
ESTÁN ALTAMENTE ASOCIADOS CON LA PRODUCCIÓN DE CONDILOMAS GENITALES.
16 Con base a la clasificación de Bethesda, las lesiones de ALTO RIESGO se relacionan con el VPH subtipos
Y oncogénicos: 16, 18, 31, 33, 35, 39, 45, 51, 52, 56 y 58. Las lesiones de bajo riesgo para desarrollar
45. cáncer son producidas por subtipos de VPH como son: 6, 11, 40, 42, 53, 54 y 57. LOS SUBTIPOS 16 Y 45
SE CONSIDERAN ONCOGÉNICOS.

18 La infección persistente por VPH de alto riesgo y que no desaparece durante años, aumenta el riesgo
Y de desarrollar malignidad. EL SUBTIPO 18 DEL VPH SE CONSIDERA UNO DE LOS PRINCIPALES
35. ASOCIADO A ONCOGENICIDAD.

31 Basados en su asociación con el cáncer cervical y las lesiones precursoras, los VPH pueden agruparse
Y en tipos de alto y bajo riesgo oncogénico. Los tipos 6, 11, 42, 43 y 44, están comúnmente asociados
40. con el condiloma acuminado y las displasias leves, razón por la cual se les ha denominado de ¨bajo
riesgo oncogénico, mientras que los tipos 16, 18, 31, 33, 34, 35, 39, 45, 51, 52, 56, 58, 59, 66, 68 y 70
están relacionados con la Neoplasia Intraepitelial Cervical (NIC), carcinoma in situ y el carcinoma
invasor y son considerados de ¨alto riesgo oncogénico. EL SUBTIPO 31 SE CONSIDERA DE ALTA
ONCOGENICIDAD.

Bibliografía:
1. GUÍA DE PRÁCTICA CLÍNICA, TRATAMIENTO DEL CONDILOMA ACUMINADO EN MUJERES EN
EDAD REPRODUCTIVA EN LOS TRES NIVELES DE ATENCIÓN. MÉXICO: SECRETARIA DE SALUD;
JUNIO 2013.

http://www.cenetec.salud.gob.mx/descargas/gpc/CatalogoMaestro/issste_658_13_condilomaacuminadomujeres/gpc_er_condiloma_acuminado.pdf

70 - EL MANEJO TERAPÉUTICO IDEAL PARA LAS TUMORACIONES EXOFÍTICAS ENCONTRADAS EN


LA PACIENTE SERÁ CON:

ESCISIÓN El TRATAMIENTO QUIRÚRGICO se prefiere en lesiones de mayor tamaño; entre las técnicas
QUIRÚRGICA. más utilizadas se encuentran la escisión quirúrgica, la electrofulguración con
electrocauterio, y en las últimas décadas, la vaporización con láser de CO2. La escisión
quirúrgica se prefiere en pacientes con condilomas de gran tamaño o numerosas lesiones.
En verrugas de MÁS DE DIEZ MILÍMETROS DE DIÁMETRO, la escisión quirúrgica debe
considerarse como la primera opción de tratamiento. La técnica se realiza de preferencia
con tijeras pequeñas debido a que resulta más útil porque limita y minimiza el daño en la
piel sana intervenida y además provee de tejido para diagnóstico histopatológico. ES EL
TRATAMIENTO DE PRIMERA ELECCIÓN PARA LESIONES MAYORES A 10MM.

INTERFERON. El INTERFERÓN puede administrarse, ya sea de forma sistémica o intralesional, mostrando


mayor efectividad cuando se inyecta directamente en las lesiones. Debido a su “elevado
costo” y a la presentación frecuente de “efectos adversos sistémicos”, el uso de Interferón
debe considerarse como una última opción de tratamiento, sobre todo para condilomas
recalcitrantes o en aquellas lesiones crónicas que no responden a las demás opciones
terapéuticas ES UNA DE LAS ÚLTIMAS OPCIONES TERAPÉUTICAS EN CONDILOMATOSIS,
ACTUALMENTE NO ESTÁ RECOMENDADO EN LAS GPC.

APLICACIÓN En la actualidad existe la vacuna contra el VPH cuyo principal objetivo está encaminado a la
LOCAL DE LA prevención del cáncer cervicouterino y NO EXISTEN REPORTES RECIENTES SOBRE SU USO
VACUNA D EL PROFILÁCTICO EN CONDILOMATOSIS PERIANAL, ADEMÁS DE QUE NO SE APLICA DE
VIRUS DEL MANERA LOCAL.
PAPILOMA
HUMANO.
IMIQUIMOD. Los CONDILOMAS son lesiones resultantes de la infección con virus del papiloma humano
(VPH) variedad acuminada, conocido como virus del condiloma. Tienen diversas
morfologías y su aspecto varía desde pápulas planas hasta la "lesión verrugosa exofítica"
clásica llamado condiloma acuminado. Estas verrugas se ubican en diversos tejidos en la
parte inferior del aparato reproductor, uretra, ano o boca. Casi siempre se diagnostican por
inspección clínica y no es necesario realizar una biopsia a menos que se sospeche de la
coexistencia de una neoplasia. Los síntomas asociados con condilomas acuminados varían
en función del número de lesiones y su localización. Los pacientes con un pequeño número
de verrugas a menudo son asintomáticos. Otros pacientes pueden tener sangrado, prurito,
ardor, dolor, secreción vaginal. Los condilomas en ocasiones pueden formar grandes masas
exofíticas que pueden interferir con la defecación, el coito, o un parto vaginal. Las lesiones
que afectan al conducto anal proximal también pueden causar estenosis. La secreción
vaginal y el prurito son las manifestaciones más frecuentes en caso de condilomas floridos;
también puede haber hemorragia después del coito. EL CONDILOMA ACUMINADO ES LA
LESIÓN CLÁSICA DE ESTE PADECIMIENTO, SE CARACTERIZA POR LESIONES EXOFÍTICAS TAL
COMO SE DESCRIBEN EN EL CUADRO CLÍNICO. TRATAMIENTO MÉDICO. Entre los agentes
tópicos para el tratamiento de las verrugas genitales está la Podofilina, el ACIDO
TRICLOROACÉTICO y Bicloroacético. 1. Autoaplicación: El tratamiento más comúnmente
utilizado es la auto aplicación de podofilina, el cual es un agente antimitótico, interfiriendo
en la división celular y dañando los tejidos en los cuales las células se están reproduciendo.
Se utiliza la Podofilina al 10 a 25 % en tintura de benzoína. 2. Aplicación médica: se debe de
aplicar por un médico el ácido bicloroacético o ácido Tricloroacético (Proteolíticos) solución
al 50% a 80%, con menor concentración en cérvix y vagina, y con mayor concentración en
vulva. Se realiza la aplicación semanal hasta que las lesiones desaparezcan. Ya se demostró
que la aplicación de interferón dentro de la lesión es efectiva en casos que no responden al
tratamiento. Se pueden utilizar agentes quimioterapéuticos como el ungüento de 5-
fluorouracilo o bleomicina en forma de inyecciones dentro de la lesión como tratamiento de
segunda línea. TRATAMIENTO INMUNOLÓGICO. El IMIQUIMOD es un producto tópico
inmunomodulador con actividad antiviral, ya que induce la formación de citocinas en el
sitio de la lesión, entre ellos el Interferón. Además, se ha visto que el ARN-m L1 y el ADN
del VPH se encuentran significativamente disminuidos después del tratamiento. La crema de
Imiquimod al 5% se aplica tres veces por semana y el tiempo promedio para la desaparición
de las lesiones es de 12 semanas. La recidiva varía del 6 al 19% a 3 meses de seguimiento. Se
recomienda en “lesiones pequeñas” o después de la escisión quirúrgica del condiloma,
aplicándolo sobre el área cicatrizada. TRATAMIENTO QUIRÚRGICO. Se recomienda en
lesiones extensas, particularmente aquellas que son pediculadas. Las opciones incluyen
criocirugía, destrucción electroquirúrgica, escisión y vaporización con láser. - La
electrocirugía con asa diatérmica se recomienda para pacientes con condiloma acuminado
genital, anal u oral, como aplicación única. - La vaporización está indicada en lesiones muy
extensas tanto anales como genitales, su costo es elevado y tienen una baja accesibilidad.
POR EL TAMAÑO Y CARACTERÍSTICAS DE LAS LESIONES, EL IMIQUIMOD CORRESPONDE AL
TRATAMIENTO DE ELECCIÓN EN ÉSTE CASO. IMPORTANTE: recuerda que no es el
tratamiento de primera elección, pero el resto de las opciones no corresponden al manejo
adecuado de estas lesiones.

Bibliografía:
1. GUÍA DE PRÁCTICA CLÍNICA, TRATAMIENTO DEL CONDILOMA ACUMINADO EN MUJERES EN
EDAD REPRODUCTIVA EN LOS TRES NIVELES DE ATENCIÓN. MÉXICO: SECRETARIA DE SALUD;
JUNIO 2013.

http://www.cenetec.salud.gob.mx/descargas/gpc/CatalogoMaestro/issste_658_13_condilomaacuminadomujeres/gpc_er_condiloma_acuminado.pdf

FIN DEL CASO CLÍNICO SERIADO


ANÁLISIS DEL CASO CLÍNICO

IDENTIFICACIÓN DEL REACTIVO


Area: MEDICINA INTERNA
Especialidad: DERMATOLOGÍA
Tema: LESIONES EXUDANTES O COSTROSAS
Subtema: DERMATITIS POR CONTACTO

CASO CLÍNICO CON UNA PREGUNTA

MUJER DE 20 AÑOS CON DIAGNÓSTICO ACTUAL DE ANOREXIA Y RAQUITISMO SECUNDARIO.

femenina de 20 años de edad.

anorexia y raquitismo.

-.

-.

-.

71 - EN ESTA PACIENTE DEBERÁ INDICARSE QUE SE EXPONGA AL SOL DEBIDO A QUE EN LA PIEL
SE LLEVA A CABO PARCIALMENTE EL METABOLISMO DEL:
CALCIFEROL. LA VITAMINA D (CALCIFEROL) ES EL ÚNICO QUE PODEMOS FORMAR EN LA PIEL POR
EFECTOS DE LOS RAYOS SOLARES SIN LA NECESIDAD DE SER ABSORBIDA. LA VITAMINA D,
O VITAMINA SOLAR (CALCIFEROL) NECESITA DE LA PRESENCIA DE GRASAS EN LA DIETA Y
DE BILIS EN EL ORGANISMO, PARA QUE PUEDA SER ABSORBIDA EN EL TEJIDO ADIPOSO,
HÍGADO, BAZO Y BAJO LA PIEL. POR LO TANTO LA SÍNTESIS DE VITAMINA D DEPENDE DEL
GRADO DE EXPOSICIÓN AL SOL Y DE LA INTENSIDAD DE PIGMENTACIÓN DE LA PIEL. LA
PIEL OSCURA O DE COLOR DEJA PASAR MENOS LOS RAYOS ULTRAVIOLETA Y SINTETIZA
MENOS VITAMINA D. No la destruye la cocción pero sí que se deteriora u oxida al entrar en
contacto con el oxígeno y la luz. La vitamina D es la encargada de regular el paso de calcio
(Ca2+) a los huesos. Por ello si la vitamina D falta, este paso no se produce y los huesos
empiezan a debilitarse y a curvarse produciéndose malformaciones irreversibles: el
raquitismo. Esta enfermedad afecta especialmente a los niños. La Vitamina D representa un
papel importante en el mantenimiento de órganos y sistemas a través de múltiples
funciones, tales como: la regulación de los niveles de calcio y fósforo en sangre,
promoviendo la absorción intestinal de los mismos a partir de los alimentos y la reabsorción
de calcio a nivel renal. Con esto contribuye a la formación y mineralización ósea, siendo
esencial para el desarrollo del esqueleto. Sin embargo, en dosis muy altas, puede conducir a
la resorción ósea.

RETINOL. El RETINOL es un componente de los pigmentos visuales, y es esencial para la integridad de


la retina. La deficiencia de vitamina A afecta a la visión provocando xeroftalmía. Contribuye
en el desarrollo óseo normal, así como de los tejidos blandos y las células epiteliales que
forman el esmalte en el desarrollo de los dientes. Es importante en el mantenimiento de los
tejidos epiteliales normales y en la diferenciación de las células basales dentro de las células
mucosas epiteliales. Existen dos fuentes de vitamina A en la dieta: Vitamina preformada o
retinol. Está presente en alimentos de origen animal. El retinol se encuentra con frecuencia
en los mismos alimentos que las proteínas y es soluble en grasa. La encontramos
principalmente en el hígado, el pescado azul, los huevos, el queso y la leche. Provitamina o
carotenoides: Provienen principalmente de fuentes vegetales y se convierte en vitamina A en
el hígado. Los betacarotenos son los más activos. Se encuentra en vegetales de hoja verde
(lechuga, espinaca) y de color anaranjado (calabaza, zanahoria, etc).

TOCOFEROL. El TOCOFEROL o VITAMINA E es una vitamina liposoluble que actúa como "antioxidante" a
nivel de la síntesis del pigmento hemo, que es una parte esencial de la hemoglobina de los
glóbulos rojos. Todas las acciones de los tocoferoles parecen estar determinadas por su
carácter de agente antioxidante, y que en particular previene las reacciones de peroxidación
de lípidos (enranciamiento). El enranciamiento de lípidos insaturados consiste en una serie
compleja de reacciones. Al final los radicales oxigenados dan lugar a su vez a una serie de
compuestos (aldehídos, ácidos y cetonas) que son los responsables de las características
desagradables de los productos enranciados, como el mal olor. Además, inducen en otras
estructuras (proteínas de membrana, por ejemplo) alteraciones que comprometen
gravemente su función. Los tocoferoles actúan rompiendo la cadena de reacciones, actuando
de forma que ofrecen un hidrógeno fácilmente sustraíble a los radicales oxigenados,
impidiendo así que sea sustraído de los lípidos.

ASCORBATO. La VITAMINA C o ASCORBATO es una vitamina hidrosoluble sensible al calor. Químicamente


hablando, se trata de ácido L-ascórbico y sus sales, los ascorbatos (los más comunes, que son
ascorbatos de sodio y de calcio).La vitamina C es un cofactor enzimático implicado en
diversas reacciones fisiológicas (hidroxilación). Es necesaria para la síntesis del colágeno y de
los glóbulos rojos, y contribuye al buen funcionamiento del sistema inmunitario. También
juega un papel en el metabolismo del hierro, en la transformación de dopamina en
noradrenalina y en la biosíntesis de carnitina. Bajo su forma oxidada, atraviesa la barrera
hemato-encefálica para acceder al cerebro y a varios órganos. Muy frágil en solución, se
destruye al contacto con el aire, por la luz o el calor. Se trata de un antioxidante, molécula
capaz de contrarrestar la acción nociva de oxidantes como los radicales libres. Con este fin,
también se emplea la forma R- del ácido ascórbico, la cual, al contrario que la forma L-, no
presenta actividad vitamínica.

Bibliografía:
1. GUÍA DE PRÁCTICA CLÍNICA, PREVENCIÓN, DIAGNÓSTICO Y TRATAMIENTO DE RAQUITISMO
CARENCIAL. MÉXICO: SECRETARIA DE SALUD, 2010. 2. GUÍA DE PRÁCTICA CLÍNICA, PREVENCIÓN,
DIAGNÓSTICO Y TRATAMIENTO DE RAQUITISMO CARENCIAL. MÉXICO: SECRETARIA DE SALUD, 2010.

http://www.cenetec.salud.gob.mx/descargas/gpc/CatalogoMaestro/459-11_Raquitismo_carencial/GER_RAQUITISMO.pdf
ANÁLISIS DEL CASO CLÍNICO

IDENTIFICACIÓN DEL REACTIVO


Area: CIRUGÍA
Especialidad: TRAUMATOLOGÍA Y ORTOPEDIA
Tema: CERVICALGIAS, LUMBALGIAS, TRAUMA MIEMBROS SUP Y
TUMORES
Subtema: LESIONES DE HOMBRO

CASO CLÍNICO CON UNA PREGUNTA

FEMENINA DE 67 AÑOS DE EDAD, PRESENTO CONTUSIÓN DIRECTA EN LA ARTICULACIÓN


GLENOHUMERAL DERECHA, HA PRESENTADO HIPOSENSIBILIDAD EN EL MÚSCULO DELTOIDES.

contusiÓn en hombro.

hiposensibilidad en la regiÓn del deltoides.

16 - POR LOS DATOS CLINICOS USTED SOSPECHA QUE SE HAYA LESIONADO EL:

EL NERVIO El NERVIO RADIAL es un nervio en el cuerpo humano que suple terminaciones nerviosas a
RADIAL. músculos del BRAZO, ANTEBRAZO, MUÑECA Y MANO, así como la sensación cutánea del
dorso de la mano. Procede del fascículo posterior del plexo braquial con fibras nerviosas
procedentes de las raíces espinales C5, C6, C7, C8 y T1. El nervio radial se divide en una
rama profunda, que se convierte en el nervio interóseo posterior y continúa volviéndose
superficial para inervar el dorso de la mano.
EL NERVIO Debido al antecedente de la lesión especifica en el hombro y a la sintomatología de
CIRCUNFLEJO. hiposensibilidad se sospechará de este nervio lesionado. Para realizar el diagnóstico
diferencial, se deberá conocer por lo menos la región inervada por los diferentes nervios
mencionados. El NERVIO AXIAL O CIRCUNFLEJO es un nervio raquídeo mixto que
"pertenece al plexo braquial". Nace del tronco secundario posterior con el nervio radial, en
contacto con el músculo subescapular. Es un nervio que va destinado al HOMBRO, no se
dirige al brazo y pasa entre el deltoides y el húmero rodeando el cuello de éste junto a la
arteria circunfleja posterior. Colaterales Rama motriz para el músculo redondo menor en el
espacio cuadrilátero de Velpeau (nervio del redondo menor). Ramos sensitivos superficiales
para el muñón del hombro. Terminales Termina dando terminales motoras al deltoides por
su cara interna.

EL NERVIO El NERVIO MEDIANO es un nervio raquídeo mixto proveniente del plexo braquial. Nace de
MEDIANO. dos raíces, una del fascículo lateral, y otra del fascículo medial (C5,C6,C7,C8,T1) que forman
una V entre las cuales discurre la arteria axilar.1 Desciende por el borde interno del brazo
junto a la arteria axilar. Cuando llega al compartimento anterior del brazo se sitúa aplicado
a la arteria braquial. En la MUÑECA se sitúa entre los tendones de los músculos palmar
mayor y palmar menor, pasa por debajo del ligamento anular del carpo y se sitúa por
debajo de la eminencia tenar. El nervio mediano permite movilidad y sensibilidad fina con
mucha rapidez y permite la oposición del pulgar.

EL PLEXO Es como tal el origen de los nervios anteriores y cuando éste se afecta en su totalidad, hay
BRAQUIAL. sintomatología no localizada si no generalizada a todo el miembro torácico. RECUERDA:
CUANTO MÁS PROXIMAL SEA LA INCAPACIDAD Y DOLOR DEBERÁS PENSAR EN LESIÓN
DEL PLEXO BRAQUIAL. DE ESTE PLEXO EL NERVIO LESIONADO ESPECÍFICAMENTE PARA EL
ÁREA MENCIONADA ES EL NERVIO CIRCUNFLEJO.

Bibliografía:
1. BRUNICARDI F, ANDERSEN D, BILLIAR T, Y COLS. SCHWARTZ PRINCIPIOS DE CIRUGÍA, 9A
EDICIÓN. MC GRAW HILL. 2011, PP 1581. 2. DOHERTY G. DIAGNÓSTICO Y TRATAMIENTO QUIRÚRGICO,
13A EDICIÓN. MC GRAW HILL LANGE. 2011, PP 1334-1335. 3. PROGRAMA AVANZADO DE APOYO VITAL
EN TRAUMA PARA MÉDICOS. ATLS. COMITÉ DE TRAUMA DEL COLEGIO AMERICANO DE
CIRUJANOS. ESTADOS UNIDOS. 8ª EDICIÓN.
ANÁLISIS DEL CASO CLÍNICO

IDENTIFICACIÓN DEL REACTIVO


Area: PEDIATRÍA
Especialidad: URGENCIAS PEDIÁTRICAS
Tema: MALFORMACIONES CONGÉNITAS INTESTINALES QUE REQUIEREN
ATENCIÓN DE URGENCIA
Subtema: ATRESIA INTESTINAL

CASO CLÍNICO SERIADO

RECIÉN NACIDO DE 2 DÍAS DE VIDA, PRESENTA VÓMITOS BILIARES Y AUSENCIA DE EVACUACIONES


DESDE SU NACIMIENTO. A LA EXPLORACIÓN: MASA PALPABLE EN CUADRANTE INFERIOR DERECHO.
RADIOGRAFÍA DE ABDOMEN CON DISTENSIÓN DE ASAS, AUSENCIA DE AIRE DISTAL E IMAGEN
GRANULAR ESPUMOSA EN CUADRANTE INFERIOR DERECHO.

recién nacido de 2 dias de vida

--

PRESENCIA DE VÓMITOS BILIARES Y


AUSENCIA DE EVACUACIONES DESDE SU
NACIMIENTO

MASA ABDOMINAL PALPABLE EN


CUADRANTE INFERIOR DERECHO

RADIOGRAFíA DE ABDOMEN CON


DISTENSIÓN DE ASAS, AUSENCIA DE AIRE
DISTAL E IMAGEN GRANULAR ESPUMOSA
EN CUADRANTE INFERIOR DERECHO

72 - PARA REALIZAR EL DIAGNÓSTICO DEL PACIENTE SE DEBERÁN SOLICITAR:

ELECTROLITOS Los Electrolitos de orina, tales como sodio y cloruro, son necesarios sus determinaciones y
URINARIOS control en pacientes con alteración en la función renal como lo es azotemia prerrenal
discierne de daño renal aguda tubular, que se producen secundaria a nefrotoxinas o
isquemia.
ELECTROLITOS La determinación de los electrólitos en el líquido de ascitis es necesaria dado a que en el
DE LÍQUIDO líquido de ascitis hay gran cantidad de proteínas y electrolitos en el líquido intraperitoneal,
DE ASCITIS por lo que es necesario en estos pacientes la determinación de éstos y corregir sus
alteraciones. Las ascitis están presente secundaria a las patologías de origen hepático (por
ejemplo en la cirrosis hepática).

ELECTROLITOS Los electrolitos séricos se determinan en pacientes que lo requieran o presente alguna
SÉRICOS patología que puedan alterarlos como lo es la deshidratación, secundaria a pérdidas o poca
ingesta, en hiperplasia suprarrenal congenital en pacientes con malformaciones
congénitas, en recién nacidos etc.

ELECTROLITOS LO PRIMERO QUE DEBES HACER ES DETERMINAR EL DIAGNÓSTICO PROBABLE QUE EN


EN SUDOR ESTE CASO ES: FIBROSIS QUÍSTICA. RECUERDA: El ileo y el tapón meconial se asocian
frecuentemente a fibrosis quística. Es una prueba que mide el nivel de cloruro en el sudor.
Aunque las pruebas genéticas se han convertido en métodos importantes para diagnosticar
la FIBROSIS QUÍSTICA en niños, la prueba del cloruro en sudor aún es el estándar.

Bibliografía:
MANUAL DE CUIDADOS NEONATALES. JOHN P CLOHERTY. MASSON. EDICIÓN 4. 2005. PAG. 763.

73 - EL MANEJO A SEGUIR EN ESTE PACIENTE, ES REALIZARLE:

ESTIMULACIÓN RECTAL La estimulación rectal no es un método necesario en esta patología,


generalmente es cuando hay ausencia de evacuaciones o estreñimiento, y
que la materia fecal se encuentra en el ámpula rectal. NO SERÁ DE
UTILIDAD PUES SEGURAMENTE HAY UN TAPÓN MECONIAL.

ENEMA CONTRASTADO CON Este es un estudio contrastado que nos ayuda a estudiar la anatomía del
BARIO intestino grueso, y para el estudio de patología como lo son en el caso de la
invaginación intestinal, malrotación intestinal, hernia diafragmáticas etc.
NO TIENE UTILIDAD TERAPÉUTICA EN ESTE CASO.

SERIE Estudio contrastado que nos sirve para el estudio de la anatomía del tubo
ESOFAGOGASTRODUODENAL digestivo alto, así mismo es un estudio indispensable para el diagnóstico de
otras patologías como lo son estenosis esofágica, reflujo gastroesofágico,
atresias duodenales, hipertrofia pilórica, acalasia etc. NO TIENE UTILIDAD
TERAPÉUTICA EN ESTE CASO.

ENEMA CONTRASTADO CON La gastrografina es una solución acuosa hiperosmolar de diatrizoato de


DIATRIZOATO meglumina. La solución hiperosmolar absorbe líquido hacia la luz del
intestino a partir del plasma, lo que resulta en una diarrea osmótica que se
suspende después de haber expulsado la solución hiperosmolar. ENEMA
CONTRASTADO CON DIATRIZOATO ES UNA FORMA NO QUIRÚRGICA DE
TRATAR EL TAPÓN MECONIAL.

Bibliografía:
MANUAL DE CUIDADOS NEONATALES. JOHN P CLOHERTY. MASSON. EDICIÓN 4. 2005. PAG. 763.

FIN DEL CASO CLÍNICO SERIADO


ANÁLISIS DEL CASO CLÍNICO

IDENTIFICACIÓN DEL REACTIVO


Area: MEDICINA INTERNA
Especialidad: NEUMOLOGÍA
Tema: ENFERMEDADES PLEURALES
Subtema: DERRAME PLEURAL Y TB

CASO CLÍNICO CON UNA PREGUNTA

HOMBRE DE 55 AÑOS DE MEDIO RURAL QUE ACTUALMENTE SE ENCUENTRA EN TRATAMIENTO


INTENSIVO PARA TUBERCULOSIS PULMONAR. PRESENTA NEURITIS PERIFÉRICA COMO APARENTE
EFECTO ADVERSO A LOS MEDICAMENTOS.

HOMBRE DE 55 AÑOS.

EN TRATAMIENTO INTENSIVO PARA


TUBERCULOSIS PULMONAR.

-.

PRESENTA NEURITIS PERIFéRICA COMO


APARENTE EFECTO ADVERSO A LOS
MEDICAMENTOS.

-.

74 - LA NEURITIS PERIFÉRICA EN PACIENTES ESTE PACIENTE ES UN EFECTO ADVERSO POTENCIAL


DE:

LA Efectos adversos de los fármacos antituberculosos de primera línea: PIRAZINAMIDA.


PIRAZINAMIDA. Hepatitis. Trastornos gastrointestinales. Artralgia. Hipersensibilidad cutánea.
Hiperuricemia. Gota. Fotosensibilidad.

LA RIFAMPICINA. Efectos adversos de los fármacos antituberculosos de primera línea: RIFAMPICINA.


Hepatitis. Colostasis. Reacción de hipersensibilidad. Intolerancia digestiva. Fiebre.
Interacción medicamentosa. Trombopenia. Anemia hemolítica. Necrosis tubular.
Nefritis intersticial. Síndrome gripal.
LA ISONIACIDA. Efectos adversos de los fármacos antituberculosos de primera línea: ISONIACINA.
Hepatitis. "Neuropatía periférica". Reacción de Hipersensibilidad. Fiebre. Pseudolupus.
Vértigo. Convulsiones. Psicosis. Ataxia cerebelosa. Neuritis óptica. Anemia.
Agranulocitosis, Ginecomastia. Artralgias. Pelagra. La NEUROPATÍA PERIFÉRICA que
aparece con el uso de isoniacida se observa hasta en 2% de los pacientes que reciben
5mg/kg de peso. Al parecer el antifímico en cuestión interviene en el metabolismo de
la piridoxina (vitamina B6). El riesgo de neurotoxicidad por isoniacida es mayor en
sujetos con trastornos previos que también conllevan el peligro de neuropatía como la
infección por VIH; en quien tienen Diabetes Mellitus, abusan del alcohol o están
desnutridos; y entre quienes reciben simultáneamente otros fármacos que pueden ser
neuropáticos como la estavudina. ES EL ÚNICO FÁRMACO ANTIFÍMICO DE PRIMERA
LÍNEA QUE TIENE COMO EFECTO ADVERSO LA NEUROPATÍA PERIFÉRICA.

LA Efectos adversos de los fármacos antituberculosos de primera línea: ESTREPTOMICINA.


ESTREPTOMICINA. Toxicidad auditiva, vestibular y renal. Reacción de hipersensibilidad. Bloqueo
neuromuscular. Citopenias.

Bibliografía:
1. LONGO DL, FAUCI AS, KASPER DL, HAUSERSL, JAMESON JL, LOSCALZOJ. HARRISON. PRINCIPIOS
DE MEDICINA INTERNA, 18A EDICIÓN. MC GRAW HILL. NEW YORK, USA. 2012, PP 1374.
ANÁLISIS DEL CASO CLÍNICO

IDENTIFICACIÓN DEL REACTIVO


Area: MEDICINA INTERNA
Especialidad: ENDOCRINOLOGÍA
Tema: DIABETES MELLITUS
Subtema: DIABETES MELLITUS TIPO II

CASO CLÍNICO SERIADO

HOMBRE DE 48 AÑOS, CON ANTECEDENTES FAMILIARES DE DIABETES. DESDE HACE 4 MESES


PRESENTA PÉRDIDA DE PESO NO CUANTIFICADA, POLIURIA Y POLIDIPSIA. A LA EXPLORACIÓN
PRESENTA IMC DE 35.2, ACANTOSIS NIGRICANS EN CUELLO Y OBESIDAD CENTRAL, SIN
DESHIDRATACIÓN. SE DOCUMENTA: GLUCOSA EN AYUNO 129MG/DL, UNA SEGUNDA MUESTRA
REPORTA 130MG/DL.

hombre de 48 años de edad

antecedentes familiares de diabetes

perdida de peso no cuanti cada desde


hace 4 meses, poliuria y polidipsia

IMc 35.2, acantosis nigricans en cuello y


obesidad central.

glucosa 129 mg/dl y segunda muestra de


130 mg/dl.

75 - LA CAUSA QUE EXPLICA LA PRESENCIA DE ACANTOSIS NIGRICANS EN ESTE PACIENTE ES LA:

RESISTENCIA A La mayoría de los casos de acantosis nigricans son idiopáticos y están asociados con
LA INSULINA A obesidad. La mayoría de estos pacientes tienen resistencia a la insulina clínica o
NIVEL subclínica. La interacción entre las cantidades circulantes excesivas de insulina que
PERIFÉRICO. estimule a los receptores del factor de crecimiento similar a la insulina en los
queratinocitos puede conllevar a su desarrollo. En ocasiones raras la acantosis nigricans
puede presentarse con un patrón autosómico dominante asociado a algunas
endocrinopatías o anomalías congénitas. LA ACANTOSIS NIGRICANS ES UN DATO
ALTAMENTE SUGESTIVO DE RESISTENCIA A LA INSULINA.
PÉRDIDA DE LA La Acantosis Nigricans (AN), es un proceso caracterizado por engrosamiento cutáneo e
PRIMERA FASE hiperpigmentación pardo-grisácea en forma de placas generalmente simétricas y con
DE LA ACCIÓN preferencia topográfica por los pliegues y caras de flexión de extremidades. LA PÉRDIDA
DE LA INSULINA. DE LA PRIMERA FASE DE LA ACCIÓN DE LA INSULINA TIENEN QUE VER CON EL
DESARROLLO DE LA HIPERGLUCEMIA POSTPRANDIAL y no con la aparición de acantosis
nigricans.

DISMINUCIÓN La insulina no es una hormona que estimule la lipólisis, por el contrario estimula el
DE LA ACCIÓN deposito de ácidos grasos en el tejido adiposo. Su deficiencia activa la lipólisis.
ANTILIPOLÍTICA
DE LA INSULINA.

LIPÓLISIS LA LIPOLISIS NO TIENE EFECTOS SOBRE LA APARICIÓN DE LA ACANTOSIS NIGRICANS.


INCREMENTADA. Se conoce que hay una hiperactividad epidérmica y fibroblástica, con resistencia
insulínica periférica y elevación de los niveles de insulinemia en la que pueden influir
factores genéticos y/o adquiridos. En la pseudoacantosis el factor fundamental es el roce
repetido que estimula a los queratinocitos y melanocitos.

Bibliografía:
1. LONGO DL, FAUCI AS, KASPER DL, HAUSERSL, JAMESON JL, LOSCALZOJ. HARRISON. PRINCIPIOS
DE MEDICINA INTERNA, 18A EDICIÓN. MC GRAW HILL. NEW YORK, USA. 2012, PP 2988.

76 - EL DIAGNÓSTICO MÁS PROBABLE EN ESTE PACIENTE ES:

DIABETES La Acantosis Nigricans es una alteración en la piel que se caracteriza por engrosamiento
MELLITUS TIPO 2. importante e hiperpigmentación con una textura aterciopelada que con mayor
frecuencia aparece en sitios de flexión. El 80% de la Acantosis Nigricans es de origen
benigno el resto puede asociarse con neoplasias. En el caso de los pacientes con
diabetes esta alteración en la piel se ha asociado con resistencia a la insulina. La
resistencia a la insulina es el mecanismo fisiopatológico principal en el desarrollo de
diabetes tipo 2. ESTE PACIENTE TIENE EN DOS OCASIONES CIFRAS DE GLUCOSA EN
AYUNO SUPERIORES A 126 MG/DL JUNTO CON SÍNTOMAS RELACIONADOS CON
HIPERGLUCEMIA POR LO QUE CUMPLE CON LOS CRITERIOS DIAGNÓSTICOS DE
DIABETES TIPO 2. La DIABETES TIPO 2 se deriva de un conjunto de trastornos diversos
caracterizado por diversos grados de resistencia a la insulina, menor secreción de
insulina y una mayor producción de glucosa. En esta los factores genéticos y
ambientales se combinan para causar, tanto pérdida de las células beta, como
resistencia a la insulina. Se le ha considerado por mucho tiempo la diabetes de los
adultos; ahora es cada vez más común ver este tipo de enfermedad a edades más
tempranas sobre todo en adolescentes obesos. LA OBESIDAD ES EL FACTOR
AMBIENTAL ASOCIADO A LA DIABETES MELLITUS TIPO 2 MÁS IMPORTANTE DE LA
RESISTENCIA A LA INSULINA. Hay pacientes que si bien no son obesos tienen un
incremento en la grasa visceral o también llamada obesidad metabólica.

DIABETES La DIABETES TIPO 1 no se asocia a resistencia a la insulina que es mecanismo que se ha


MELLITUS TIPO 1. propuesto para el desarrollo de acantosis nigricans. La DIABETES TIPO 1 es relacionada
con patogenia inmunitaria hasta en el 95% de los casos (tipo 1A) y el otro 5% se
considera de tipo idiopática (tipo 1B). Ambas, se acompañan de destrucción variable de
las células pancreáticas. Si bien, la mayoría de los pacientes con este padecimiento
desarrollan la enfermedad antes de los 30 años de edad, la destrucción autoinmunitaria
de las células pancreáticas puede suceder a cualquier edad, en este sentido, se calcula
que un 5 a 10% se diagnostica después de los 30 años. LOS DATOS DEL CASO CLÍNICO
NO COINCIDEN CON UNA DIABETES TIPO 1.
SÍNDROME Los pacientes con síndrome metabólico al igual que los diabéticos tipo 2 pueden tener
METABÓLICO. acantosis nigricans. El síndrome metabólico está conformado por una serie de factores
de riesgo, como la "hipertensión arterial", la "dislipidemia", la "intolerancia a la
glucosa por la resistencia a la insulina" y la "obesidad visceral", elevando la
probabilidad de padecer enfermedad cardiovascular. NO TENEMOS SUFICIENTES
DATOS PARA FUNDAMENTAR EL CONJUNTO DE ELEMENTOS QUE CONFORMAN EL
SÍNDROME METABÓLICO.

INTOLERANCIA A La intolerancia a los carbohidratos es un estado de prediabetes, también producido por


LOS resistencia a la insulina por lo que es posible que algunos pacientes con intolerancia a
CARBOHIDRATOS. los carbohidratos tengan acantosis nigricans, sin embargo lo mismo que en la respuesta
anterior, debido a los valores de glucosa en dos ocasiones asociada a síntomas de
hiperglucemia este pacientes ya tiene diagnostico de DM2. LOS VALORES DE GLICEMIA
CAPILAR SE ENCUENTRAN POR ARRIBA DE 125MG/DL POR LO TANTO NO
CORRESPONDE A ESTE DIAGNÓSTICO.

Bibliografía:
1. GUÍA DE PRÁCTICA CLÍNICA. DIAGNÓSTICO, METAS DE CONTROL AMBULATORIO Y REFERENCIA
OPORTUNA DE PREDIABETES Y DIABETES MELLITUS TIPO 2 EN ADULTOS EN EL PRIMER NIVEL DE
ATENCIÓN. MÉXICO: SECRETARIA DE SALUD; 21/MARZO/2013. 2. LONGO DL, FAUCI AS, KASPER DL,
HAUSERSL, JAMESON JL, LOSCALZOJ. HARRISON. PRINCIPIOS DE MEDICINA INTERNA, 18A
EDICIÓN. MC GRAW HILL. NEW YORK, USA. 2012, PP 2988-2989.

http://www.cenetec.salud.gob.mx/descargas/gpc/CatalogoMaestro/093_GPC_Diabmellitus2/SSA_093_08_EyR.pdf

77 - EL TRATAMIENTO DE PRIMERA ELECCIÓN INDICADO EN ESTE PACIENTE ES:

DIETA, DISMINUCIÓN Las medidas para reducir de peso tales como la dieta y el ejercicio son de vital
DE PESO Y USO DE importancia para mejorar la sensibilidad a la insulina por lo que son medidas que se
INSULINA. deben implementar en todos los pacientes con diabetes tipo 2, síndrome
metabólico o prediabetes. En este caso debido a las cifras de glucosa en ayuno
estaría indicado iniciar con dieta, ejercicio y algún medicamento que mejore la
sensibilidad a la insulina. En caso de no haber mejoría con el tratamiento anterior se
valorara un ajuste del mismo.

DIETA Y La dieta, el ejercicio y la pérdida de peso son medidas que se deben implementar en
DISMINUCIÓN DE todos los pacientes que tengan alguna patología asociada a resistencia a la insulina.
PESO Y EJERCICIO Este paciente ya tiene criterios diagnósticos de diabetes tipo 2 por lo que aparte de
AERÓBICO. las medidas higiénico dietéticas es recomendado iniciar tratamiento antidiabético.
IMPORTANTE: NO SE RECOMIENDA EL MANEJO ÚNICO CON CAMBIOS EN EL
ESTILO DE VIDA EN PACIENTES CON DM2 RECIÉN DIAGNOSTICADA, AÚN CUANDO
EL VALOR INICIAL DE HBA1C ESTÉ CERCA DEL VALOR ÓPTIMO.
DIETA, EJERCICIO E La dieta y el ejercicio son muy importantes en los pacientes con diabetes tipo 2,
HIPOGLUCEMIANTES. actualmente es bien sabido que por su efecto en la reducción de peso disminuyen la
resistencia a la insulina. ES CORRECTO INICIAR TRATAMIENTO FARMACOLÓGICO
CON METFORMINA (A NO SER QUE ESTE CONTRAINDICADO), JUNTO CON
CAMBIOS EN EL ESTILO DE VIDA EN TODO PACIENTE DIABÉTICO, AL MOMENTO
DEL DIAGNÓSTICO. Se sugiere iniciar con metformina a dosis de 425mg diarios e
incrementar de manera gradual cada tercer a quinto día hasta alcanzar la dosis meta
para evitar efectos gastrointestinales. Cuando los niveles de HbA1c bajales al
momento del diagnóstico de la DM2, superan el 8%, está indicado combinar otro
antidiabético. IMPORTANTE: - Actualmente no se recomienda iniciar con terapia
combinada con sulfonilureas como la glibenclamida por el riesgo de hipoglucemia;
se sugiere considerar primero otro grupo de medicamentos. - Las tiazolidinedionas
tampoco con una buena opción de inicio en la terapia combinada con metformina
debido a que incrementan el riesgo de desarrollar fracturas, falla cardíaca o cáncer
de vejiga, además del aumento de peso corporal. - Los inhibidores SGL-2 en adultos
con diagnóstico reciente de DM2, no son buena opción para la terapia combinada,
aún cuando favorecen la pérdida de peso; únicamente se deberán utilizar como
alternativa en caso de que no se puedan utilizar inhibidores DPP-4. - En pacientes
con DM2 de reciente diagnóstico que no alcancen metas con metformina como
monoterapia, se recomienda como primer paso a la terapia combinada, un inhibidor
DDP-4 (sitagliptina, saxagliptina y linagliptina), debido a que este grupo de
medicamentos tiene el mejor perfil de efectividad y seguridad. Una segunda opción
de terapia combinada es metformina-sulfonilurea en pacientes con bajo riesgo de
hipoglucemia o la combinación de metformina- inhibidor SGLT-2.

INSULINA Y No esta indicado en este momento el inicio de insulinoterapia. Debido al


EJERCICIO AERÓBICO mecanismo de producción de la enfermedad es importante reducir la resistencia a la
insulina, para este caso existen medicamentos como las biguanidas y
tiazolidinedionas.

Bibliografía:
1. DIAGNÓSTICO Y TRATAMIENTO FARMACOLÓGICO DE LA DIABETES MELLITUS TIPO 2 EN EL
PRIMER NIVEL DE ATENCIÓN. EVIDENCIAS Y RECOMENDACIONES: GUÍA DE PRÁCTICA CLÍNICA.
MÉXICO: CENETEC; 2018. 2. LONGO DL, FAUCI AS, KASPER DL, HAUSERSL, JAMESON JL, LOSCALZOJ.
HARRISON. PRINCIPIOS DE MEDICINA INTERNA, 18A EDICIÓN. MC GRAW HILL. NEW YORK, USA.
2012, PP 2989-2992, 2995-3000.

http://www.cenetec-difusion.com/CMGPC/GPC-IMSS-718-18/ER.pdf

FIN DEL CASO CLÍNICO SERIADO


ANÁLISIS DEL CASO CLÍNICO

IDENTIFICACIÓN DEL REACTIVO


Area: MEDICINA INTERNA
Especialidad: GERIATRÍA Y GERONTOLOGÍA
Tema: MALTRATO EN EL ADULTO MAYOR
Subtema: MALTRATO EN EL ADULTO MAYOR

CASO CLÍNICO SERIADO

HACE UNOS DÍAS LE FUE ENTREGADA UNA QUEJA POR PROBABLE MALTRATO EN EL ASILO ANEXO AL
HOSPITAL DONDE USTED LABORA. DERIVADO DE ESTA SITUACIÓN SE LE HA ENCOMENDADO VALORAR
EL ESTADO DE SALUD DE TODOS LOS INTERNOS DEL ASILO Y REALIZAR LA BÚSQUEDA INTENCIONADA
DE ALGÚN TIPO DE MALTRATO Y/O DEPRESIÓN.

Adultos mayores.

-.

Sospecha de maltrato.

-.

-.

78 - ES EL TIPO DE MALTRATO MÁS COMÚN QUE SE ESPERA ENCONTRAR EN ESTE GRUPO DE


EDAD:

MALTRATO El MANTRATO AL ADULTO MAYOR lo constituye cualquier acto y omisión acto y omisión
ECONÓMICO. que tenga como resultado un daño, que vulnere o ponga en peligro la integridad física o
psíquica, así como el principio de autonomía y respeto de sus derechos fundamentales del
individuo de 60 años y más, el cual puede ocurrir en el medio familiar, comunitario o
institucional. El maltrato se puede dar de una o varias esferas de la vida, entre las más
frecuentes se encuentran: - Maltrato físico. - Maltrato psicológico o emocional. - Maltrato
financiero. - Maltrato sexual. - Negligencia. - Abandono. El MALTRATO FINANCIERO O
ECONÓMICO afecta a individuos de todos los estratos socioeconómicos. Se caracteriza por
el uso ilegal o indebido de fondos, propiedades o bienes, sin autorización del adulto mayor
o mediante un poder notarial.
MALTRATO El MALTRATO PSICOLÓGICO O EMOCIONAL: consiste en la acción de infligir angustia, dolor
PSICOLÓGICO. o estrés mediante palabras o actos. ES EL TIPO MÁS COMÚN DE MALTRATO EN EL ADULTO
MAYOR y requiere un abordaje integral por múltiples presentaciones, lo que dificulta su
diagnóstico. En el año 2006 se realizó una encuesta a personas adultas mayores de la
Ciudad de México, que incluyo 618 adultos mayores, la cual reporto una prevalencia de
algún tipo maltrato en este grupo de edad del 16.2%: - Maltrato psicológico 12.7%, - Abuso
sexual 0.9%, - Maltrato económico 3.9%, - Maltrato físico 3.7%, - Negligencia y abandono
3.5%.

MALTRATO El MALTRATO FÍSICO: consiste en la acción de utilizar la fuerza física y ocasionar dolor, daño
FÍSICO. o discapacidad física. Es probablemente el más fácil de reconocer, pero en muchas de las
ocasiones el maltrato se muestra renuente o en incapaz de reportarto. Se relaciona más con
las características del maltratador, como el antecedente de substancias o malas relaciones
personales.

MALTRATO El ABANDONO es el acto de desamparo injustificado hacia el adulto mayor, con el que se
POR tienen responsabilidad que deriva de las disposiciones legales y que pone en peligro la
ABANDONO. salud. El abandono social es la falta de atención y cuidados por parte de la familia, lo cual
afecta el aspecto psicológico y emocional, que se manifiesta con depresión e ideas de
muerte.

Bibliografía:
1. GUÍA DE PRÁCTICA CLÍNICA, DETECCIÓN Y MANEJO DEL MALTRADO EN EL ADULTO MAYOR.
MÉXICO: SECRETARIA DE SALUD; 2013.

http://www.cenetec.salud.gob.mx/descargas/gpc/CatalogoMaestro/057_GPC_MaltratoAdultoMayor/MAM_EVR_CENETEC.pdf

79 - ES UN FACTOR DE RIESGO PARA QUE SE PRESENTE EL MALTRATO INSTITUCIONAL:

AISLAMIENTO FACTORES DE RIESGO AMBIENTALES Y FACTORES FAMILIARES: - Familia disfuncional. -


SOCIAL. Relaciones conflictivas. - Aislamiento social. - Red de apoyo deficiente.

COLAPSO DEL FACTORES DE RIESGO PARA EL CUIDADOR/MALTRATADOR: - Colapso del cuidador. -


CUIDADOR. Enfermedad psiquiátrica o problemas psicológicos. - Uso de substancias. - Dificultades
financieras o desempleo.

SÍNDROME FACTORES DE RIESGO PARA EL MALTRATAO INSTITUCIONAL: - Inadecuada capacitación,


DE FATIGA experiencia y supervisión de los cuidadores. - Escasez de personal. - Salarios bajos. -
LABORAL. Síndrome de fatiga laboral. - Transferencia negativa del personal.

CONDUCTA FACTORES DEL RIESGO DEL ADULTO MAYOR. - Dependencia funcional. - Deterioro
AGRESIVA. cognitivo. - Conducta agresiva. - Insuficientes recursos financieros. - Enfermedad
psiquiátrica. - Sexo femenino. - Edad avanzada. - Aislamiento social. - Historia familiar de
violencia.

Bibliografía:
1. GUÍA DE PRÁCTICA CLÍNICA, DETECCIÓN Y MANEJO DEL MALTRADO EN EL ADULTO MAYOR.
MÉXICO: SECRETARIA DE SALUD; 2013.

http://www.cenetec.salud.gob.mx/descargas/gpc/CatalogoMaestro/057_GPC_MaltratoAdultoMayor/MAM_EVR_CENETEC.pdf

80 - CORRESPONDE A LA ESCALA QUE LE PERMITIRÁ EVALUAR LA PRESENCIA DE DEPRESIÓN EN


ESTOS ANCIANOS:
ESCALA DE La ESCALA DE BARTHEL evalúa actividades básicas de la vida diaria.
BARTHEL.

ESCALA DE La ESCALA DE LAWTON Y BRODY evalúa actividades instrumentales de la vida diaria.


LAWTON Y BRODY.

ESCALA DE La ESCALA DE YASEVAGE o Escala Geriátrica de Depresión (GDS) es ideal para este
YASEVAGE. caso.

ESCALA DE ZARIT. La ESCALA PARA EVALUAR LA SOBRECARGA DEL CUIDADOR DE ZARIT permite
identificar a cuidador en riesgo de colapso.

Bibliografía:
1. GUÍA DE PRÁCTICA CLÍNICA, DETECCIÓN Y MANEJO DEL MALTRADO EN EL ADULTO MAYOR.
MÉXICO: SECRETARIA DE SALUD; 2013.

http://www.cenetec.salud.gob.mx/descargas/gpc/CatalogoMaestro/057_GPC_MaltratoAdultoMayor/MAM_EVR_CENETEC.pdf

FIN DEL CASO CLÍNICO SERIADO


ANÁLISIS DEL CASO CLÍNICO

IDENTIFICACIÓN DEL REACTIVO


Area: PEDIATRÍA
Especialidad: URGENCIAS PEDIÁTRICAS
Tema: URGENCIAS NEUMOLÓGICAS PEDIÁTRICAS
Subtema: ASMA

CASO CLÍNICO SERIADO

FEMENINA DE 3 AÑOS DE EDAD, DE MANERA SÚBITA HACE 30 MIN PRESENTA TOS, DISNEA, CIANOSIS,
SIBILANCIAS Y ALETEO NASAL.

femenina de 3 años de edad

--

DE MANERA SÚBITA HACE 30 MIN


PRESENTA TOS, DISNEA, CIANOSIS,
SIBILANCIAS Y ALETEO NASAL

--

--

81 - SEGÚN EL CUADRO CLÍNICO SU SOSPECHA DIAGNÓSTICA ES EL DE:

CUERPO • Cuando un cuerpo extraño pasa a la vía aérea, el organismo reacciona de forma inmediata,
EXTRAÑO EN automática, con el reflejo de la tos. La tos violenta puede hacer que el objeto sea expulsado
LAS VÍAS al exterior solucionando el problema. • El cuadro clínico aparece de forma súbita Se
AÉREAS describen tres fases clínicas distintas tras la aspiración de un cuerpo extraño. 1. Período
inmediato postaspiración, se manifiesta por una tos súbita, violenta, con asfixia, cianosis,
ahogo, estridor y sibilancias. 2. Período asintomático que puede ser de minutos a meses de
duración, dependiendo de la localización del cuerpo extraño, del grado de obstrucción que
produce y de la reacción inflamatoria que genere. 3. Se manifiesta la patología derivada de
la reacción de cuerpo extraño generada: inflamación-infección, dando síntomas como tos
crónica, expectoración, fiebre, sibilancias y, a veces, hemoptisis. ES UN CUADRO MUY
FRECUENTE EN PREESCOLARES, EL PACIENTE PRESENTA EL CUADRO CARACTERÍSTICO DE
LA FASE AGUDA. Ten en cuenta, que no tiene ningún otro antecedente.
ESTENOSIS La ESTENOSIS SUBGLÓTICA se define como un estrechamiento parcial o completo de la
SUBGLÓTICA subglotis, la cual está localizada a nivel del cartílago cricoides, pudiendo ser congénita o
SECUNDARIA adquirida. • Las formas adquiridas de estenosis subglótica son más severas que las de origen
congénito. La forma adquirida secundaria a intubación endotraqueal tiene mayor riesgo de
ocurrir durante la primera semana y disminuye levemente después de este tiempo. NO HAY
ANTECEDENTES QUE RESPALDEN ÉSTA POSIBILIDAD DIAGNÓSTICA.

ESTENOSIS ESTENOSIS SUBGLÓTICA CONGÉNITA • No es frecuente que la sintomatología se manifieste


SUBGLÓTICA en el nacimiento sino en las primeras semanas o meses. Lo más frecuente es que el síntoma,
CONGÉNITA casi el único, sea una disnea aislada, de predominio inspiratorio y sin alteraciones de la
deglución ni disfonía. Es rarísimo que aparezca disfonía y alteraciones del grito. Se sospecha
al aparecer episodios de laringitis subglótica o croup persistente o de repetición en poco
espacio de tiempo y de carácter subagudo. En realidad, cualquiera que sea la malformación
estenosante se puede manifestar o agravar ante una infección rinofaríngea intercurrente y
sólo la disnea puede indicar un tratamiento urgente. No es frecuente que la sintomatología
sea tan evidente como cuando presenta un estridor asociando disnea y un ruido
respiratorio. LOS SÍNTOMAS APARECEN EN LOS PRIMERO MESES DE VIDA.

TRAQUEÍTIS La TRAQUEITIS BACTERIANA es una causa considerada poco habitual de obstrucción grave
BACTERIANA de la vía aérea superior, de origen infeccioso. CX: Los niños afectados suelen presentar una
alteración grave del estado general, con aspecto séptico, fiebre alta, insuficiencia
respiratoria, estridor y presencia de secreciones purulentas en la tráquea. LOS NIÑOS
SUELEN TENER ASPECTO SÉPTICO CON FIEBRE O HIPOTERMIA CARACTERÍSTICA NO
PRESENTE EN NUESTRO PACIENTE.

Bibliografía:
TRATADO DE PEDIATRÍA. NELSON. MC GRAW HILL. EDICIÓN 16. 2000. PAG. 1400-1401.

82 - EL TRATAMIENTO QUE DEBE DE INICIAR DE INMEDIATO LA PACIENTE ES CON:

DILATACIONES A ESTENOSIS SUBGLÓTICA ADQUIRIDA • Las dilataciones son útiles en las primeras
TRAVÉS DE semanas posteriores a la lesión y luego su utilidad es menor puesto que ya ha ocurrido
LARINGOSCOPÍA maduración del tejido colágeno. • Algunos estudios mencionan la inyección de
DIRECTA esteroides en la zona de la estenosis; sin embargo este procedimiento también puede
ocasionar lesiones secundarias. • Otros métodos terapéuticos son la
electrocauterización del tejido de granulación y la colocación de soportes o stents
intraluminales luego de la dilatación. • El manejo endoscópico también puede hacerse
con láser de CO2, pero los tejidos vecinos son muy frágiles y se pueden ocasionar
lesiones iatrogénicas. CORRESPONDE AL MANEJO DE LA OBSTRUCCIÓN SUBGLÓTICA
ADQUIRIDA O AGUDA.

ENDOSCOPÍA Y La BRONCOSCOPÍA RÍGIDA es el método de elección para la extracción de cuerpos


EXTRACCIÓN DE extraños en la vía aérea, ya que es más eficaz y segura. Además permite ventilar al
CUERPO paciente y el uso de instrumental más variado en cuanto al tamaño. La
EXTRAÑO fibrobroncoscopia se utiliza sólo en caso de duda diagnóstica, por ejemplo en los casos
en los que, sin un antecedente claro, presenta sintomatología crónica como tos crónica,
sibilancias de origen no asmático, neumonías recurrentes, atelectasias, bronquiectasias,
etc. Sus limitaciones son debidas a la falta de un instrumental adecuado para la
extracción y a que no permite una ventilación adecuada por lo que hay riesgo de
hipoxemia e hipercapnia.

NEBULIZACIONES En general se debe considerar a la BRONQUIOLITIS como un padecimiento autolimitado


FRÍAS CON cuyo manejo debe establecerse fundamentalmente con medidas de sostén. • La base
OXÍGENO principal del tratamiento es la oxigenoterapia, que generalmente corrige la hipoxemia
con oxígeno en cámara húmeda o nebulizador neumático (Púritan) en concentraciones
del 30 al 40%.
TRAQUEOSTOMÍA TRAQUEOSTOMÍA Es una operación muy conocida desde la antigüedad, consiste en
comunicar el exterior (medio aéreo) con la luz traqueal, generalmente al nivel de la
parte baja y medial del cuello. 1. Traqueostomía de urgencia Es la más realizada y está
indicada en las insuficiencias respiratorias agudas altas como las producidas por:
Cuerpos extraños laríngeos, Neoplasias Laríngeas, Edemas de la laringe, Parálisis de
cuerdas vocales, Edemas de la base de la lengua, Traumatismos laríngeos, Epiglotis,
Difteria laríngea, Estenosis laríngea o subglótica, Traumatismos craneoencefálicos,
Malformaciones congénitas. AUNQUE ESTÁ INDICADA EN CUERPOS EXTRAÑOS NO
HAY DATOS DE QUE SE ENCUENTRE ABOCADO A LARINGE.

Bibliografía:
TRATADO DE PEDIATRÍA. NELSON. MC GRAW HILL. EDICIÓN 16. 2000. PAG. 1403.

FIN DEL CASO CLÍNICO SERIADO


ANÁLISIS DEL CASO CLÍNICO

IDENTIFICACIÓN DEL REACTIVO


Area: MEDICINA INTERNA
Especialidad: NEUROLOGÍA
Tema: ENFERMEDADES CEREBROVASCULARES
Subtema: ANEURISMA Y HEMORRAGIA CEREBAL

CASO CLÍNICO CON UNA PREGUNTA

FEMENINA DE 49 AÑOS, CON DIAGNÓSTICO DE HEMORRAGIA SUBARACNOIDEA POR RUPTURA DE


ANEURISMA CEREBRAL. ES INTERVENIDA QUIRÚRGICAMENTE REALIZÁNDOSE CLIPAJE DE ANEURISMA.
UNA SEMANA DESPUÉS PRESENTA HEMIPARESIA IZQUIERDA PROGRESIVA HASTA LLEGAR A
HEMIPLEJIA.

FEMENINA DE 49 AÑOS.

HEMORRAGIA SUBARACNOIDEA POR


RUPTURA DE ANEURISMA CEREBRAL, con
cirugía de CLIPAJE DE ANEURISMA.

UNA SEMANA DESPUÉS PRESENTA


HEMIPARESIA IZQUIERDA PROGRESIVA
HASTA LLEGAR A HEMIPLEJIA.

-.

-.

83 - EL EVENTO FISIOPATOLÓGICO QUE EXPLICA LA HEMIPLEJÍA ACTUAL DE LA PACIENTE ES:

LA INCIDENCIA DEL VASOESPASMO EN EVC La enfermedad cerebrovascular (ECV) es la


HIDROCEFALIA tercera causa de muerte en países industrializados. De todos los tipos de EVC, la
SECUNDARIA. hemorragia subaracnoidea (HSA) es responsable de 22% a 29% de la mortalidad.
Alrededor de 30% de los pacientes con hemorragia subaracnoidea (HSA) secundaria a
ruptura de aneurisma cerebral, desarrollan vasoespasmo arterial y con ello el déficit
neurológico asociado aumenta. Esta complicación empeora el pronóstico de los
pacientes, puesto que un 25% de ellos mueren y otro 30% a 35% sufren de déficit
neurológico permanente. Entonces, el vasoespasmo cerebral es el factor modificable más
importante para mejorar la tasa de morbimortalidad en pacientes con hemorragia
subaracnoidea secundaria a ruptura de aneurisma.
EL El vasoespasmo cerebral posterior a hemorragia subaracnoidea por ruptura aneurismática
VASOESPASMO es la principal causa de muerte y de déficit neurológico permanente en supervivientes de
SECUNDARIO. Hemorragia Subaracnoidea y, por tanto, es el factor más frecuentemente modificable
para mejorar el resultado en este grupo de pacientes. Aparece de 4 a 14 días después de
la hemorragia. EL VASOESPASMO ES LA PRINCIPAL COMPLICACIÓN DE HEMORRAGIA
SUBARACNOIDEA, PUEDE PROVOCAR UN DÉFICIT NEUROLÓGICO ADICIONAL O
INCREMENTAR LOS SIGNOS CLÍNICOS YA EXISTENTES.

LA DEFINICIÓN DEL VASOESPASMO • El vasoespasmo cerebral es una condición reversible


HIPONATREMIA que cursa con reducción del calibre de la luz de una arteria en el espacio subaracnoideo
PERSISTENTE. cerebral, con la consiguiente disminución del flujo sanguíneo a las áreas perfundidas por
el vaso comprometido.

UNA EMBOLIA CLASIFICACIÓN DEL VASOESPASMO 1. Angiográfico: adelgazamiento de la columna de


CEREBRAL medio de contraste en las arterias cerebrales mayores. • La clasificación angiográfica
SECUNDARIA. determina el porcentaje de reducción del diámetro del vaso, comparado con el observado
en la angiografía inicial. • Se denomina vasoespasmo grave a un estrechamiento mayor
de 75%; moderado de 50% a 75% y leve de 25% a 50%. • El vasoespasmo angiográfico
usualmente inicia del día 3° al 5o de la HSA, se observa la mayor estrechez de la luz
arterial desde el día 5 al día 7, y se prolonga en promedio hasta el día 14; se resuelve en
el curso de 2 a 4 semanas. Es observado en 30 - 70% de las arteriografías realizadas en el
día 7 después de la HSA. • Es de notar que, al menos, buena parte del estrechamiento
vascular sucede en vasos de pequeño calibre, que no son visibles con angiografía cerebral
convencional y que pueden causar parte del compromiso neurológico del paciente. 2.
Sintomático: denominado también déficit neurológico tardío, se refiere al síndrome
originado por el compromiso isquémico de una región cerebral, por estrechamiento de
uno o varios vasos cerebrales, que origina la aparición de un nuevo deterioro del estado
de conciencia, afasia o déficit motor. • Ha sido asociado con pronóstico neurológico muy
pobre en supervivientes de HSA. • Dada la alta frecuencia de esta patología en HSA y la
baja tasa de respuesta clínica a las terapias conocidas en la actualidad, el énfasis del
tratamiento está no sólo en el diagnóstico temprano sino en la instauración de una
terapia profiláctica como parte del manejo, que intente reducir el número de pacientes
que desarrollen esta complicación.

Bibliografía:
1. LONGO DL, FAUCI AS, KASPER DL, HAUSERSL, JAMESON JL, LOSCALZOJ. HARRISON. PRINCIPIOS
DE MEDICINA INTERNA, 18A EDICIÓN. MC GRAW HILL. NEW YORK, USA. 2012, PP 2263.
ANÁLISIS DEL CASO CLÍNICO

IDENTIFICACIÓN DEL REACTIVO


Area: PEDIATRÍA
Especialidad: INFECTOLOGIA PEDIÁTRICA
Tema: INFECCIONES NEONATALES
Subtema: SEPSIS Y CHOQUE SÉPTICO

CASO CLÍNICO CON UNA PREGUNTA

RECIÉN NACIDO MASCULINO DE 6 DÍAS DE VIDA EXTRAUTERINA, OBTENIDO POR PARTO EN SU


DOMICILIO, NO ACUDIÓ A MÉDICO, POR LO QUE DESCONOCE APGA; SILVERMAN, SOMATOMETRIA.
ACUDE A URGENCIAS POR QUE EL RECIÉN NACIDO PRESENTA FIEBRE NO CUANTIFICADA, RECHAZO A
LA VÍA ORAL E IRRITABILIDAD. A LA EXPLORACIÓN CON FIEBRE DE 39°C, FC DE 180 X MIN, FR 55 X MIN,
PULSOS NORMALES, TINTE ICTÉRICO GENERALIZADO XXX, IRRITABLE, SECRECIÓN VERDOSA EN AMBOS
OJOS, PETEQUIAS EN EXTREMIDADES, SECRECIÓN PURULENTA ABUNDANTE A TRAVÉS DEL MUÑÓN
UMBILICAL Y HEPATOMEGALIA.

Es muy importante los días de vida


extrauterina, para cuando nos piden
clasi car entre sepsis temprana y tardía.

El antecedente de parto en su domicilio no


nos garantiza la asepsia y antisepsia del
procedimiento, considerando al paciente ,
si hubiera llegado al hospital como
potencialmente infectado

Es muy importante la irritabilidad, el


rechazo a la vía oral. Estos son datos
característicos en pacientes sépticos a esta
edad

Es muy importante que identi ques que


existe compromiso sistémico a nivel
cardiovascular, respiratorio, mucocutáneo,
gastrohepático, hemático e infeccioso

84 - EL DIAGNÓSTICO PROBABLE ES:


SÍNDROME DE SÍNDROME DE RESPUESTA INFLAMATORIA SISTÉMICA (SRIS). Es la presencia de por lo
RESPUESTA menos 2 de los siguientes criterios, uno de los cuales deberá ser temperatura o recuento
INFLAMATORIA leucocitario anormal: 1. Temperatura mayor a 38,5°C o menor de 36°C. 2. Taquicardia
SISTÉMICA definida como una FC media mayor a 2 DS sobre la normal para su edad en la ausencia de
estímulos externos, uso medicinas crónicas o de estímulos dolorosos o una taquicardia
inexplicable que persiste por un período de más de media a una hora o en niños menores
de 1 año: Bradicardia definida como una FC media menor percentil a 10 para la edad en la
ausencia de estímulos vagales, beta bloqueadores o cardiopatía congénita, o una
depresión persistente por más de media a una hora inexplicable. 3. Frecuencia respiratoria
media mayor a 2DS sobre la normal para su edad o ventilación mecánica por un
padecimiento agudo no relacionado a una enfermedad neuromuscular, ni a anestesia
general. 4. Recuento leucocitario elevado o disminuido para la edad (No secundario a
quimioterapia) o mayor a 10% neutrófilos inmaduros. RECUERDA: el SRIS puede ser
secundario a un sin número de etiologías (trauma, quemadura, intoxicación, etc.), no es
exclusivo de un proceso infeccioso. CUANDO ES DE ORIGEN INFECCIOSO RECIBE EL
NOMBRE ESPECÍFICO DE SEPSIS.

SEPSIS SEPSIS La SEPSIS se incluye dentro del síndrome de respuesta inflamatoria sistémica
(SRIS), en presencia o como resultado de infección sospechada o confirmada. El espectro
clínico de la sepsis comienza cuando una infección sistémica (bacteriemia, viremia o
fungemia) o localizada (meningiis, neumonía, pielonefritis, entre otras) produce una
afección sistémica y pueden progresar a una sepsis grave, choque séptico y muerte. EN
ESTE CASO ES CLARO QUE EL PACIENTE PRESENTA UN SÍNDROME DE RESPUESTA
INFLAMATORIA SISTÉMICA SECUNDARIO A INFECCIÓN (SEPSIS), Y SE DIFERENCIA DE LAS
OTRAS OPCIONES DIAGNÓSTICAS (SEPSIS SEVERA Y CHOQUE SÉPTICO) AL NO TENER EL
DATO DE HIPOTENSIÓN. IMPORTANTE: - Es indispensable que primero identifiques que el
paciente presenta compromiso sistémico, que involucra, el sistema cardiovascular (FC de
180), respiratorio (FR de 55), neurológico (irritabilidad), gastrohepático (Ictericia y
hepatomegalia), hematológico (Petequias en ambos miembros e ictericia), cutáneo
(Muñón umbilical) con datos claros de un proceso infeccioso (Secreción ocular, rechazo a
la vía oral, fiebre, petequias, hepatomegalia, muñón umbilical) como causa determinante
de éste cuadro clínico. - Con base en la identificación del compromiso sistémico
secundario a un proceso infeccioso, deberás determinar cómo siguiente paso el grado del
compromiso, por lo que será muy importante diferenciar entre sepsis, sepsis severa y
choque séptico. Esta clasificación, además de orientar el tratamiento también implicará
pronóstico del paciente. REPASO: Según la edad de presentación puede ser clasificada de
manera arbitraria en sepsis temprana, si aparece en los primeros 3 días de vida, que es
debida generalmente a microorganismos adquiridos de vía materna y sepsis tardía, la cual
se presenta después de los 3 días de vida extrauterina y es causada frecuentemente por
microorganismos adquiridos después del nacimiento; esta última puede ser de
adquisición nosocomial o de la comunidad. DADO QUE SE TRATA DE UN SRIS
SECUNDARIO A UN PROCESO INFECCIOSO, LA FORMA CORRECTA DE DEFINIRLO ES
SEPSIS.

SEPSIS SEVERA SEPSIS SEVERA – Sepsis más disfunción orgánica, hipotensión o hipoperfusión. Aquí
podría haber confusión, por la gravedad evidente del paciente y su disfunción orgánica.
Sin embargo "no contamos con el dato de hipotensión y para apoyar con objetividad la
hipoperfusión", esperaríamos encontrar o que se mencionara la disminución en la
intensidad de los pulsos, además de palidez de los miembros inferiores. Recuerda. Que
cuando hay hipotensión, los lechos cutáneos son los primeros que se sacrifican para
mantener la perfusión a nivel cerebral, cardiaco y renal.

CHOQUE CHOQUE SÉPTICO – Sepsis severa y disfunción orgánica cardiovascular, con hipotensión
SÉPTICO arterial a pesar de reposición de líquidos que requiere apoyo Inotrópico. "No cumpliría
con estos criterios". NOTA: En recién nacidos (De 0 a 7 días) se considera anormal una
taquicardia mayor a 180, bradicardia menor a 100, FR mayor a 50, leucocitosis mayores a
34,000/mm3, presión sanguínea sistólica menor a 65 mm/Hg. En el recién nacido (De 8 a
28 días) se considera anormal una FC mayor a 180 y menor a 100, FR mayor a 40,
leucocitos mayores de 19,500 ó menores de 5000 /mm3 y presión sanguínea sistólica
mayor a 75 mm/Hg. La temperatura debe ser tomada rectal, vesical, oral o por sensor en
catéter central.
Bibliografía:
1. PREVENCIÓN, DIAGNÓSTICO Y TRATAMIENTO DE SEPSIS Y CHOQUE SÉPTICO DEL RECIÉN
NACIDO, EN EL SEGUNDO Y TERCER NIVEL DE ATENCIÓN. EVIDENCIAS Y RECOMENDACIONES.
GUÍA DE PRÁCTICA CLÍNICA. MÉXICO: SECRETARÍA DE SALUD; NOVIEMBRE 2012. MANUAL DE
CUIDADOS NEONATALES. WALTER WILSON. MANUAL MODERNO. EDICIÓN 2. 2002. PÁG. 480.

http://www.cenetec-difusion.com/CMGPC/SS-283-12/ER.pdf
ANÁLISIS DEL CASO CLÍNICO

IDENTIFICACIÓN DEL REACTIVO


Area: CIRUGÍA
Especialidad: CIRUGÍA ABDOMINAL
Tema: PANCREATITIS
Subtema: PANCREATITIS CRÓNICA

CASO CLÍNICO SERIADO

MASCULINO DE 51 AÑOS, ALCOHÓLICO DESDE HACE 15 AÑOS. ACUDE A LA CONSULTA POR LA PRESENCIA DE
DOLOR CRÓNICO INTENSO EN EPIGASTRIO, EVACUACIONES GRASOSAS Y ABUNDANTES,ASÍ COMO LA
PÉRDIDA SOSTENIDA DE PESO. A LA EXPLORACIÓN FÍSICA DESTACA LA PÉRDIDA DE MASA MUSCULAR Y DEL
TEJIDO ADIPOSO SUBCUTÁNEO.

masculino de 51 años.

alcoholismo de 15 años.

DOLOR CRÓNICO EN EPIGASTRIO,


EVACUACIONES GRASOSAS Y PÉRDIDA DE
PESO.

PÉRDIDA DE MASA MUSCULAR Y DE


TEJIDO ADIPOSO SUBCUTÁNEO.

-.

85 - EL DIAGNÓSTICO CLÍNICO MAS PROBABLE ES:

HEPATITIS Las manifestaciones clínicas del HÍGADO GRASO ALCOHÓLICO son sutiles detectados en muchos
ALCOHÓLICA. casos como hallazgo durante la consulta por otra causa. Es muy probable que el único hallazgo
clínico manifiesto sea la hepatomegalia. Ésta puede condicionar dolor en cuadrante superior
derecho del abdomen, náuseas y en muy raras ocasiones ictericia. Por su parte la HEPATITIS
ALCOHÓLICA se asocia con diversas manifestaciones. El extremo clásico se manifiesta con fiebre,
telangiectasias, ictericia y dolor abdominal; aunque existen pacientes asintomáticos que la
padecen. La hipertensión portal condiciona ascitis o hemorragia (por várices esofágicas), esta
última manifestada con evacuaciones melénicas. LAS EVACUACIONES GRASOSAS (ESTEATORREA)
NO CON COMPATIBLES CON ESTE DIAGNÓSTICO.
PANCREATITIS La PANCREATITIS CRÓNICA es más frecuente en hombres, entre los 30 y 50 años, Existe una
CRÓNICA. relación lineal entre la exposición al alcohol y el desarrollo de Pancreatitis Crónica. El DOLOR es el
síntoma más común de la pancreatitis crónica. Por lo general, se localiza a mitad del epigastrio,
pero puede situarse o afectar cualquiera de los cuadrantes superiores izquierdo o derecho del
abdomen. En ocasiones, se percibe en la parte media baja del abdomen y con frecuencia se
describe como penetrante hacia la espalda. Las más de las veces, el dolor es constante y
terebrante, pero no tipo cólico. Se puede exacerbar con la alimentación o el consumo de alcohol.
El dolor puede acompañarse de náuseas o vómitos, pero el síntoma concurrente más común es la
anorexia. Cuando disminuye la capacidad exocrina del páncreas 10% por debajo de lo normal se
presentan DIARREA Y ESTEATORREA. Los pacientes describen una evacuación suelta (pero no
acuosa) de mal olor y voluminosa, que puede ser de color pálido y flotar en la superficie del agua.
El pronóstico de pacientes con PANCREATITIS CRÓNICA depende de la causa de la enfermedad, el
desarrollo de complicaciones, la edad y el estado socioeconómico del paciente. El tratamiento
médico del DOLOR CRÓNICO O RECURRENTE EN LA PANCREATITIS CRÓNICA requiere de
analgésicos, de suprimir el consumo de alcohol e instituir tratamiento con enzimas orales.
También son coadyuvantes de la terapéutica médica los procedimientos intervencionistas para
bloquear la conducción de nervios aferentes viscerales o tratar obstrucciones del conducto
pancreático principal. En el Tratamiento endoscópico se colocan prótesis en el conducto
pancreático, en las estenosis proximales del conducto pancreático, se puede realizar
descompresión de un escape del conducto pancreático y el drenaje de los pseudoquistes
pancreáticos que pueden cateterizarse a través del conducto pancreático principal. El tratamiento
quirúrgico solo debe considerarse cuando fracasa la terapéutica médica de los síntomas. LAS
EVACUACIONES GRASOSAS (ESTEATORREA) ESTÁN FRECUENTEMENTE ASOCIADAS A ESTA
PATOLOGÍA.

ÚLCERA La ÚLCERA PÉPTICA es un defecto de la mucosa de por lo menos de 0.5 cm de diámetro (lesiones
DUODENAL. menores se les denomina erosiones), que penetra la muscularis mucosae. Las úlceras gástricas se
dividen en proximales, las que se encuentran en el cuerpo y las distales, que se encuentra en el
antro, siendo la localización más frecuente la curvatura menor. Las úlceras duodenales por lo
regular se encuentran en la parte anterior y/o posterior del bulbo. Úlcera péptica es la
denominación de la enfermedad ulcerosa confinada al “estómago y al duodeno”, resultante del
daño de la mucosa por el desequilibrio entre los factores protectores (riego sanguíneo, secreción
de bicarbonato de la mucosa, uniones intercelulares estrechas de la mucosa y recambio epitelial) y
los factores dañinos (infección por H. Pylori, alcohol, tabaco, estado de choque, AINES, esteroides,
desnutrición e inmunosupresión). El dolor en la ÚLCERA DUODENAL se localiza en el abdomen
alto, transfictivo, intenso y de inicio súbito, que se exacerba con los movimientos, se irradia hacia
el abdomen bajo derecho o a hombros. Pueden existir datos de irritación peritoneal. Las
evacuaciones generalmente son melénicas (20%). NO CORRESPONDE AL CUADRO CLÍNICO, SE
ESPERARÍA ENCONTRAR EVACUACIONES MELÉNICAS (SIEMPRE QUE SEA SANGRANTE), LAS
EVACUACIONES ESTEATORREICAS NO SON CARACTERÍSTICAS DE ESTA PATOLOGÍA.

GASTRITIS La GASTRITIS EROSIVA CRÓNICA es el desarrollo de lesiones erosivas hemorrágicas después de la


EROSIVA exposición de la mucosa gástrica a substancias tóxicas o posterior a una reducción significante del
CRÓNICA. flujo sanguíneo de la mucosa. Es la patología más frecuente en mayores de 50 años. Puede ser
secundaria a irritantes como los fármacos, especialmente la aspirina y otros antiinflamatorios no
esteroideos (AINES), a la Enfermedad de Crohn y a infecciones bacterianas y virales. Es más
frecuente en personas que abusan del alcohol. Dentro de los síntomas pueden estar presentes las
náuseas y el DOLOR en la parte alta del abdomen. Sin embargo, muchas personas (como los
consumidores crónicos de aspirinas) no sienten dolor. Si la GASTRITIS EROSIVA CRÓNICA se
complica con úlceras sangrantes, las heces pueden ser melénicas, así mismo pueden presentar
hematemésis o sangrado en posos de café. El estudio de elección para el DIAGNÓSTICO de un
sangrado de tubo digestivo alto es la endoscopia (esófago-gastro-duodenoscopia). Si es preciso,
se puede realizar una biopsia (obtención de una muestra del revestimiento del estómago para su
examen). En el TRATAMIENTO los inhibidores de bomba de protones son más efectivos que los
bloqueadores H2 en el manejo de la gastritis. El omeprazol es el fármaco de primera elección. SE
ASOCIA EN ALGUNOS CASOS CON LA PÉRDIDA DE PESO, PERO NO ASÍ CON EL TIPO DE
EVACUACIONES DESCRITAS.

Bibliografía:
1. DOHERTY G. DIAGNÓSTICO Y TRATAMIENTO QUIRÚRGICO, 13A EDICIÓN. MC GRAW HILL LANGE. 2011, PP
505-507. 2. LONGO DL, FAUCI AS, KASPER DL, HAUSERSL, JAMESON JL, LOSCALZOJ. HARRISON.
PRINCIPIOS DE MEDICINA INTERNA, 18A EDICIÓN. MC GRAW HILL. NEW YORK, USA. 2012.
86 - EL TRATAMIENTO INDICADO EN ESTE CASO ES:

CONTROL DEL Este manejo se lleva a cabo como parte del manejo en pacientes con PANCREATITIS CRÓNICA
DOLOR Y en la que mediante estudios complementarios se ha detectado la insuficiencia endocrina y
CORRECCIÓN DE exocrina. LA ADMINISTRACIÓN DE ENZIMAS PANCREÁTICAS SON EL ELEMENTO BÁSICO DE
LAS LOS PACIENTES CON PANCREATITIS CRÓNICA. Se requiere de una administración suficiente
INSUFICIENCIAS de lipasa para corregir la digestión deficiente y corregir la esteatorrea. Si bien el tratamiento
ENDÓCRINA Y no logra la remisión total de esteatorrea en todos los pacientes, se ha notado mejoría en
EXÓCRINA. cuanto a la diarrea la restaura y la digestión, lo cual lleva al paciente a una ganancia ponderal
notoria. El tratamiento del dolor por su parte, es controversial en estos casos. Existen estudios
que respaldan la opinión de que la enzimoterapia no mejora el dolor en estos pacientes. Sin
embargo, otras investigaciones sugieren que la mejoría del dolor deriva del mejoramiento de
la dispepsia. Se han utilizado diversos recursos farmacológicos y quirúrgicos para el manejo
del dolor pero ninguno cuenta con el sustento necesario para elegirse con preferencia.

INHIBIDORES DE Los inhibidores de bomba de protones son más efectivos que los bloqueadores H2 en el
BOMBA DE manejo de la GASTRITIS. Los inhibidores de bomba de protones han demostrado mantener el
PROTONES. PH intragástrico por arriba de 6 por más de 72 hrs. cuando se dan a dosis adecuadas. Cuando
el PH se mantiene por arriba de 4 ayuda a restituir la mucosa gástrica.

PROCINÉTICOS Y Es el manejo utilizado en la ÚLCERA DUODENAL principalmente, ya que el sucralfato


LA incrementa la secreción de moco, bicarbonato, la síntesis endógena de prostaglandinas
ADMINISTRACIÓN consecuentemente presenta disminución del reflujo. No se utiliza en ulceras producidas por
ORAL DE AINES.
SUCRALFATO.

DIETA BAJA EN Manejo utilizado en la HEPATITIS ALCOHÓLICA además de utilizar corticoesteroides, se


PROTEÍNAS Y utilizan complementos vitamínicos principalmente el complejo B y ácido fólico, entre otros.
ANTIBIÓTICOS
INTRALUMINALES.

Bibliografía:
1. DOHERTY G. DIAGNÓSTICO Y TRATAMIENTO QUIRÚRGICO, 13A EDICIÓN. MC GRAW HILL LANGE. 2011, PP
505-507. 2. LONGO DL, FAUCI AS, KASPER DL, HAUSERSL, JAMESON JL, LOSCALZOJ. HARRISON.
PRINCIPIOS DE MEDICINA INTERNA, 18A EDICIÓN. MC GRAW HILL. NEW YORK, USA. 2012.

FIN DEL CASO CLÍNICO SERIADO


ANÁLISIS DEL CASO CLÍNICO

IDENTIFICACIÓN DEL REACTIVO


Area: MEDICINA INTERNA
Especialidad: CARDIOVASCULAR
Tema: URGENCIAS CARDIOVASCULARES
Subtema: REANIMACIÓN CARDIOPULMONAR

CASO CLÍNICO CON UNA PREGUNTA

HOMBRE DE 56 AÑOS, QUE ES LLEVADO A LA SALA DE URGENCIAS CON APARENTE PARO


CARDIORRESPIRATORIO. SE INICIAN MANIOBRAS DE REANIMACIÓN Y MONITORIZACIÓN DEL PACIENTE. LA FC
ES 0/MIN.

HOMBRE DE 56 AÑOS.

APARENTE PARO CARDIORRESPIRATORIO.

-.

SE INICIAN MANIOBRAS DE REANIMACIÓN


Y MONITORIZACIÓN DEL PACIENTE. LA FC
ES 0/MIN.

-.

87 - EL MEDICAMENTO A UTILIZAR DE PRIMERA ELECCIÓN EN ESTE CASO SERA:

ATROPINA. Es una droga anticolinérgica natural compuesta por ácido trópico y tropina, capaz de atravesar la
barrera hematoencefálica y ejercer algunos efectos sobre el SNC. • Efecto: Aumenta el
automatismo del NSA, mejora la conducción del impulso en el NAV. • Indicaciones: ASISTOLIA,
AESP con frecuencia <60, bradicardia sintomática. Bloqueo AV con QRS estrecho. Es el
tratamiento de elección en la bradicardia sintomática a dosis de 0,5mg cada 5 minutos según
necesidad. Bloquea la acción de la acetil-colina sobre los nodos sinusal y A-V, aumentando la
frecuencia cardiaca y la conducción A-V. La asistolia es una arritmia casi siempre fatal y pareciera
que la atropina tendría un valor limitado en este contexto. La recomendación es no pasarse de la
máxima dosis vagolítica de 3 mg o de 0,04 mg/Kg. SU PRINCIPAL INDICACIÓN ES LA
BRADICARDIA, TIENE POCO EFECTO EN ASISTOLIA.
ADRENALINA. La ADRENALINA es una catecolamina endógena. - Indicaciones: Fármaco de elección en todas las
formas de paro cardiaco, disminuye el umbral de desfibrilación, se usa en el tratamiento de la
bradicardia sintomática si no disponemos de marcapasos. - El mejor efecto beneficioso de la
adrenalina, en el paro cardiorrespiratorio, es la vasoconstricción periférica que conduce a mejoría
de la perfusión coronaria y cerebral. - Indicaciones: 1. Paro cardiaco: FV, TV sin pulso, asistolia,
actividad eléctrica sin pulso. 2. Bradicardia sintomática: Luego de colocar atropina, dopamina y
marcapaso transcutáneo. 3. Hipotensión severa. 4. Anafilaxia, reacción alérgica severa: en
conjunto con altos volúmenes de líquidos I.V, corticoesteroides y antihistamínicos. LA
ADRENALINA ES EL MEDICAMENTO DE PRIMERA ELECCIÓN EN TODO PARO CARDIO
RESPIRATORIO.

AMIODARONA. Es un derivado del benzofuran que tiene dos átomos de yodo en la molécula, siendo el yodo
esencial para la actividad antiarrítmica. • Indicaciones: La amiodarona es efectiva para una amplia
variedad de arritmias ventriculares y supraventriculares, en dosis bajas (100-300 mg por día) es
efectiva en el control de las taquicardias paroxísticas supraventriculares, fibrilación auricular y
flúter auricular. La amiodarona es más ampliamente usada en la taquicardia ventricular
recurrente mantenida y/o fibrilación ventricular. NO HAY DATOS DE ARRÍTMIA EN EL PACIENTE
POR LO QUE NO ESTÁ INDICADA LA AMIODARONA.

DOPAMINA. Es una catecolamina natural, actúa estimulando la liberación de norepinefrina endógena. •


Indicaciones: está indicada en el tratamiento del shock séptico y cardiogénico. NO HAY DATOS
QUE RESPALDEN UN CHOQUE SÉPTICO O CARDIOGÉNICO POR LO QUE SE LIMITA SU USO EN
ÉSTE PACIENTE.

Bibliografía:
1. LONGO DL, FAUCI AS, KASPER DL, HAUSERSL, JAMESON JL, LOSCALZOJ. HARRISON. PRINCIPIOS DE
MEDICINA INTERNA, 18A EDICIÓN. MC GRAW HILL. NEW YORK, USA. 2012, PP 2244-2245.
ANÁLISIS DEL CASO CLÍNICO

IDENTIFICACIÓN DEL REACTIVO


Area: PEDIATRÍA
Especialidad: CRECIMIENTO Y DESARROLLO
Tema: CRECIMIENTO Y DESARROLLO NORMAL
Subtema: CRECIMIENTO Y DESARROLLO FETAL

CASO CLÍNICO SERIADO

EN LA COMUNIDAD DONDE SE ENCUENTRA REALIZANDO SU SERVICIO SOCIAL SE HA OBSERVADO UNA ALTA


INCIDENCIA DE PRODUCTOS CON RESTRICCIÓN EN EL CRECIMIENTO INTRAUTERINO (RCIU) EN EL ÚLTIMO
AÑO, RAZÓN POR LA CUAL SE LE SOLICITA QUE, MEDIANTE EL ANÁLISIS DE FACTORES DE RIESGO
GENERALES PARA ESTE CASO, PROPONGA UNA ESTRATEGIA QUE AYUDE A REDUCIR ESTE FENÓMENO.

peso bajo al nacer.

88 - DEBERÁ CONSIDERARSE, COMO PRIMER PUNTO, QUE PARA REALIZAR EL DIAGNÓSTICO EL PESO
DEL PRODUCTO DEBERÁ ESTAR:

DEBAJO DE El parámetro usualmente empleado para valorar el crecimiento fetal es el peso; sin embargo, el
LA parámetro óptimo para determinar el crecimiento de un feto de acuerdo a las condiciones o factores
PERCENTILA que potencialmente pueden afectar su desarrollo (maternas, paternas y medio ambientales), es la
20 comparación del peso fetal estimado con estándares de peso fetal para condiciones similares a esta
(curvas personalizadas de peso fetal).

DEBAJO DE Se define como PEQUEÑO PARA LA EDAD GESTACIONAL al feto que ha fallado en alcanzar algún
LA parámetro biométrico o un valor de peso fetal estimado para alguna edad gestacional determinada.
PERCENTILA
15
DEBAJO DE Diversos valores (percentiles 2.5,3.5, 5. 20. 15 y 25: o desviaciones estándar: 1.0, 1.5 y 2) han sido
LA sugeridos para definir el límite normal bajo. El punto de corte más utilizado para establecer el límite
PERCENTILA normal bajo es el percentil 10 tanto para el peso fetal como para la circunferencia abdominal. Por lo
10 tanto, TODO FETO QUE SE ENCUENTRE POR DEBAJO DEL PERCENTIL 10 PARA EL PESO ESPERADO,
PARA SU EDAD SERÁ CONSIDERADO PEQUEÑO PARA LA EDAD GESTACIONAL. Recuerda: la
importancia de estas determinaciones radica en que la restricción del crecimiento intrauterino se
asocia con un incremento en la morbi-mortalidad neonatal con secuelas a corto y largo plazo.

DEBAJO DE Actualmente se define como RESTRICCIÓN DEL CRECIMIENTO INTRAUTERINO a la incapacidad del
LA feto para alcanzar su potencial genético de crecimiento, estadísticamente se estima cuando el peso
PERCENTILA y/o circunferencia abdominal es inferior al percentil 10 de los rangos de referencia en percentiles
5 correspondientes a su edad gestacional.

Bibliografía:

http://www.cenetec-difusion.com/CMGPC/IMSS-500-11/ER.pdf

89 - ES UNA MEDIDA QUE CON SEGURIDAD REDUCIRÍA EL RIESGO DE RESTRICCIÓN DEL CRECIMIENTO
INTRAUTERINO EN UNA COMUNIDAD:

ÁCIDO FÓLICO La administración de ÁCIDO FÓLICO preconcepcional reduce significativamente los defectos
PRECONCEPCIONAL del tubo neural; sin embargo, no muestra una asociación importante en la posibilidad de
presentar mortalidad perinatal y peso bajo. EL MAYOR BENEFICIO DEL ÁCIDO FÓLICO
PRECONCEPCIONAL ES LA PREVENCIÓN DE LOS DEFECTOS DEL TUBO NEURAL.

PREVENCIÓN DEL El embarazo en adolescentes y en mujeres en edad avanzada, particularmente si son


EMBARAZO EN LA primigestas se asocian con bajo peso al nacer. LA IMPLEMENTACIÓN DE TALLERES DE SALUD
ADOLESCENCIA REPRODUCTIVA PARA LA PREVENCIÓN DEL EMBARAZO EN ADOLESCENTES ES UNA
ESTRATEGIA RECOMENDADA PARA REDUCIR LA INCIDENCIA GENERAL DE PRODUCTOS
CON PESO BAJO.

HIERRO No se recomienda suplementar la dieta de la embarazada con hierro de forma rutinaria ya


SUPLEMENTARIO que no se asocia con ningún beneficio para el binomio, incluso puede tener efectos no
TEMPRANO placenteros para la madre. Se sugiere indicar hierro solo en aquellos casos en que lo
requieran de acuerdo a los resultados observados en la biometría hemática. EL HIERRO NO
SE RECOMIENDA DE FORMA RUTINARIA Y NO HA DEMOSTRADO EFECTOS BENÉFICOS EN EL
BINOMIO.

SUSPENDER EL Suspender el consumo de tabaco CUATRO MESES ANTES DEL EMBARAZO produce recién
TABAQUISMO EN nacidos de peso similares a los de la población no fumadora. Suspender el consumo del
EL PRIMER tabaco tempranamente durante el embarazo podría evitar el bajo peso al nacimiento. LA
TRIMESTRE DEL SUSPENSIÓN DEL TABAQUISMO, ENTRE MÁS TEMPRANA, MAYOR ES LA POSIBILIDAD DE
EMBARAZO REDUCCIÓN DEL RIESGO DE PESO BAJO; SIN EMBARGO, LOS RESULTADOS REALES SE
LOGRAN POR COMPLETO SI ESTA ESTRATEGIA SE TOMA PREVIO AL EMBARAZO.

Bibliografía:

http://www.cenetec-difusion.com/CMGPC/IMSS-500-11/ER.pdf

FIN DEL CASO CLÍNICO SERIADO


ANÁLISIS DEL CASO CLÍNICO

IDENTIFICACIÓN DEL REACTIVO


Area: CIRUGÍA
Especialidad: TRAUMATOLOGÍA Y ORTOPEDIA
Tema: CERVICALGIAS, LUMBALGIAS, TRAUMA MIEMBROS SUP Y TUMORES
Subtema: LESIONES DE ANTEBRAZO

CASO CLÍNICO CON UNA PREGUNTA

MUJER DE 63 AÑOS ES LLEVADA A URGENCIAS POR SUFRIR CAÍDA DESDE SU PROPIA ALTURA Y PRESENTAR
DEFORMIDAD EN ANTEBRAZO. A SU LLEGADA SE LE SOLICITA UNA RADIOGRAFÍA AP Y LATERAL DE LA
MUÑECA OBTENIÉNDOSE LAS SIGUIENTES IMÁGENES.

Mujer de 63 años.

Caída desde su propia altura y deformidad


de antebrazo

Radiografía AP y lateral de muñeca.

90 - EL MECANISMO DE LESIÓN MÁS PROBABLE DE ESTA FRACTURA ES:


CAÍDA SOBRE LA La imagen radiológica nos presenta una FRACTURA DE COLLES, este tipo de fracturas son tanto
MANO EN extraarticulares como intraarticulares del radio con diversos grados de angulación dorsal,
HIPEREXTENSIÓN desplazamiento dorsal, traslación radial y acortamiento radial. Clínicamente se ha descrito
Y DESVIACIÓN como deformidad en “dorso de tenedor”. Más del 90% de las fracturas del extremo distal del
RADIAL. radio presentan este patrón. El mecanismo de lesión de la fractura de Colles es caída sobre la
mano en HIPEREXTENSIÓN Y DESVIACIÓN RADIAL. Según la clasificación de Frykman (basada
en el patrón de afectación intraarticular), ésta fractura sería una tipo I debido a que el trazo de
fractura es extraarticular.

CAÍDA SOBRE LA Este mecanismo produce una fractura con angulación volar (vértice dorsal) del radio distal con
MANO CON LA una deformidad “en pala de jardinero”, con desplazamiento volar de la mano y del fragmento
MUÑECA EN distal del radio. También es conocida como Fractura de Smith. Generalmente presenta un
FLEXIÓN Y patrón de fractura inestable, que frecuentemente requiere de una reducción abierta y fijación
ANTEBRAZO EN interna debido a la dificultad para mantener una reducción cerrada adecuada.
SUPINACIÓN.

CAÍDA SOBRE Este mecanismo produce una fractura-luxación o una subluxación de la muñeca en la cual el
MUÑECA EN margen dorsal o volar del radio distal acompaña en su desplazamiento al carpo y la mano, se
DORSIFLEXIÓN Y conoce también como Fractura de Barton. Casi todas las fracturas de este tipo son inestables y
ANTEBRAZO EN necesitan reducción abierta con fijación interna mediante una placa de neutralización para
PRONACIÓN. conseguir una reducción anatómica estable.

COMPRESIÓN Este mecanismo de lesión produce una fractura-avulsión en la cual los ligamentos extrínsecos
DEL ESCAFOIDES permanecen unidos al fragmento de la estiloides. También pueden ser secundarios a
SOBRE LA traumatismo directo. Con mucha frecuencia se asocia a lesiones de los ligamentos
APÓFISIS intercarpianos. Y también suelen requerir de reducción abierta con fijación interna. Esta
ESTILOIDES. fractura también es conocida como fractura de Chofer, Fractura de Hutchinson.

Bibliografía:
1. KOVAL K.,EGOL K, ZUCKERMAN J. MANUAL DE FRACTURAS. 4ª EDICIÓN.WOLTERS KLUWER HEALTH. 2011.
PP:257-280

2. HTTP://WWW.CENETEC.SALUD.GOB.MX/DESCARGAS/G PC/CATALOGOMAESTRO/193_GPC_FRACTURA_ANTEBRAZO/FRACTURAS_ANTEBRAZO_CENETEC.PDF
ANÁLISIS DEL CASO CLÍNICO

IDENTIFICACIÓN DEL REACTIVO


Area: CIRUGÍA
Especialidad: CIRUGÍA ABDOMINAL
Tema: PANCREATITIS
Subtema: PANCREATITIS AGUDA

CASO CLÍNICO CON UNA PREGUNTA

MASCULINO DE 38 AÑOS, POSTOPERADO DE PANCREATECTOMÍA POR CÁNCER DE PÁNCREAS.

masculino de 38 años de edad.

operado de pancreatectomía por cáncer de


páncreas.

-.

-.

-.

91 - LA PÉRDIDA DE LA FUNCIÓN ENDÓCRINA DEL PÁNCREAS TRAERÍA COMO PRINCIPAL


CONSECUENCIA LA:

PERDIDA DE PROCURA SIEMPRE LEER MUY BIEN LA PREGUNTA QUE EN ESTE CASO INDICA "PÉRDIDA DE LA
SECRECIÓN DE FUNCIÓN ENDÓCRINA". Como parte de las funciones EXÓCRINAS del páncreas, éste sintetiza
LIPASA. enzimas pancreáticas y las almacena para liberarlas a estímulos de colecistocinina y vagales. Las
enzimas pancreáticas son proteolíticas, lipolíticas y amilolíticas. La lipasa y amilasa son
almacenadas y secretadas en su forma activa; mientras que las enzimas proteolíticas son
secretadas como precursores inactivos. LA SECRECIÓN DE ENZIMAS (LIPASA) CORRESPONDE A
LA FUNCIÓN EXÓCRINA DEL PÁNCREAS.

ACIDIFICACIÓN El JUGO PANCREÁTICO está compuesto por agua y bicarbonato de sodio, éste sirve para
DEL QUIMO neutralizar la acidez del quimo proveniente del estómago, para que puedan actuar las enzimas en
ALIMENTARIO. el intestino. Para que el Quimo no dañe el duodeno, está el páncreas (compuesto por células), el
tejido funciona como glandular digestiva, el páncreas fabrica unas proteínas llamadas “enzimas
digestivas” que funcionan a PH Básico. El páncreas se contrae, y esto contrarresta la acidez del
quimo. EL PÁNCREAS EMITE SECRECIONES EXTERNAS (FUNCIÓN EXÓCRINA) FORMADAS POR
UNA SOLUCIÓN CLARA Y ALCALINA QUE CONTIENEN ENZIMAS DIGESTIVAS.
MALA Las vitaminas son de dos tipos: - Vitaminas hidrosolubles: son la mayor parte y tienen un
ABSORCIÓN DE mecanismo de absorción mediante difusión pasiva (rápido) - Vitaminas liposolubles: son la
VITAMINAS Y vitamina F, vitamina A, vitamina D, vitamina E y vitamina K. Requieren para su absorción la
MINERALES. presencia de bilis y de enzimas pancreáticas lipolíticas (al igual que las grasas); por tanto, si hay
un déficit de absorción de grasas, también se ven afectadas las vitaminas liposolubles. Las
vitaminas y los minerales, la mayor parte de ellas se absorben en el duodeno y el yeyuno (parte
alta intestino delgado), pero algunos se absorben en la parte final del intestino delgado, en el
íleo como la vitamina B12, ésta vitamina se une al factor intrínseco, el cual será secretado por las
células parietales del estómago y se absorberá en el íleon. Es importante para la maduración final
de los eritrocitos . LA PANCREATECTOMÍA TOTAL NO INTERFIERE EN GRAN PARTE EN LA
ABSORCIÓN DE LAS VITAMINAS YA QUE LAS VITAMINAS HIDROSOLUBLES SE ABSORBEN EN EL
INTESTINO DELGADO Y LAS LIPOSOLUBLES NO REQUIEREN ENZIMAS PANCREÁTICAS PARA
ABSORBERSE.

ALTERACIÓN EL PÁNCREAS INTERVIENE EN LA ABSORCIÓN Y METABOLISMO DE LOS CARBOHIDRATOS POR


EN EL MEDIO DE SU FUNCIÓN EXOCRINA Y ENDÓCRINA, RESPECTIVAMENTE. 1. Función exocrina
METABOLISMO (absorción). En la dieta, los carbohidratos se encuentran en forma de almidón, disacáridos
DE LOS (sacarosa y lactosa) y glucosa. Únicamente son absorbidos en el intestino, y sólo lo hacen en
HIDRATOS DE forma de monosacáridos. Por lo tanto, antes de su absorción, el almidón y los disacáridos
CARBONO. deberán ser digeridos a monosacáridos por medio de la AMILASA PANCREÁTICA y las
disacaridasas del borde en cepillo intestinal. 2. FUNCIÓN ENDÓCRINA (metabolismo). El
metabolismo de los carbohidratos tiene vital importancia para las células al proveer la energía
requerida para la mayor parte de los procesos metabólicos a este nivel. La función endócrina del
páncreas consiste en facilitar el almacenamiento del material alimenticio mediante la liberación
de insulina, después de las comidas, y proporcionar un mecanismo para movilizarlo, por medio
de la liberación de glucosa, durante los periodos de ayuno. La INSULINA, un producto de la
secreción endócrina del páncreas, tiene como funciones principales “estimular las acciones
anabólicas de los carbohidratos”, grasas, proteínas y ácidos nucleicos; además, aminora los
procesos de glucogenólisis, lipólisis, proteólisis, gluconeogénesis, ureagénesis y cetogénesis. AL
REALIZAR LA PANCREATECTOMÍA SE PERDIÓ LA FUNCIÓN ENDÓCRINA, LO CUAL TRAE COMO
CONSECUENCIA UN MAL METABOLISMO DE LOS HIDRATOS DE CARBONO. La glucosa es un
carbohidrato que funge como el principal sustrato del metabolismo energético en el ser humano,
por lo tanto si te fueras al ejemplo más básico para recordar este reactivo sería: PÉRDIDA DE
“FUNCIÓN ENDÓCRINA PÁNCREAS" = DIABETES = GLUCOSA = CARBOHIDRATO.

Bibliografía:
1. DOHERTY G. DIAGNÓSTICO Y TRATAMIENTO QUIRÚRGICO, 13A EDICIÓN. MC GRAW HILL LANGE. 2011, PP
493-495. 2. LONGO DL, FAUCI AS, KASPER DL, HAUSERSL, JAMESON JL, LOSCALZOJ. HARRISON.
PRINCIPIOS DE MEDICINA INTERNA, 18A EDICIÓN. MC GRAW HILL. NEW YORK, USA. 2012.
ANÁLISIS DEL CASO CLÍNICO

IDENTIFICACIÓN DEL REACTIVO


Area: GINECOLOGÍA Y OBSTETRICIA
Especialidad: OBSTETRICIA
Tema: ENFERMEDAD HIPERTENSIVA DEL EMBARAZO
Subtema: PREECLAMPSIA

CASO CLÍNICO CON UNA PREGUNTA

ACUDE A SU CONSULTA MUJER DE 18 AÑOS, QUE CURSA CON EMBARAZO GEMELAR DE 29 SEMANAS DE
GESTACIÓN. HA INCREMENTADO 2 KGS DE PESO EN LA ÚLTIMA SEMANA. A LA EXPLORACIÓN SE ENCUENTRA
CON EDEMA DISTAL, SANGRADO FÁCIL DE ENCÍAS, TA DE 140/90 MMHG Y FRECUENCIA CARDIACA DE 100
POR MINUTO. SE REPORTA PROTEINURIA ++.

18 años.

-.

incremento de peso de 2 kilos en la última


semana.

edema distal, sangrado fácil de las encías,


ta de 140/90mmhg.

proteinuria ++.

92 - EL DIAGNÓSTICO CLÍNICO ES:

SÍNDROME El SÍNDROME NEFRÓTICO es una entidad clínica definida por cinco características: -
NEFRÓTICO. Proteinuria (> 3.5 g/24 h), - Hipoalbuminemia (< 3.5 g/dL), - Edema, - Hipercolesterolemia y
- Lipiduria. A diferencia de la preeclampsia en estadios avanzados cursa con un estado de
hipercoagulabilidad y trombosis secundaria a una mayor agregación plaquetaria. NO
EXISTEN SUFICIENTES CRITERIOS PARA ESTE DIAGNÓSTICO.

SÍNDROME El SÍNDROME ANTIFOSFOLÍPIDOS es una enfermedad del sistema inmune en la cual existen
ANTIFOSFOLÍPIDOS. anticuerpos con aparente especificidad por fosfolípidos y donde la característica mayor es la
trombosis. Para hacer el diagnóstico se requiere 1 criterio mayor (aborto recurrente, muerte
fetal 2º - 3er trimestre, trombosis venosa, trombosis arterial, trombocitopenia) + alteración
reproductiva o vascular más una prueba serológica positiva. Anticuerpos anticardiolipina
(ACL) positivo y/o anticoagulante lúpico (ACP) IgG o IgM en títulos moderados o altos. EL
SÍNDROME ANTIFOSFOLÍPIDOS SE ASOCIA CON ABORTO RECURRENTE Y MUERTE FETAL, LA
PACIENTE NO TIENE ANTECEDENTES QUE SUGIERAN ESTE PADECIMIENTO.
PREECLAMPSIA. La preeclampsia es un trastorno multisistémico cuyos criterios clínicos no han cambiado en
la última década: edad gestacional mayor de 20 semanas, presión arterial mayor de 140/90
mmHg, tira reactiva con 1+ o muestra aislada de orina con 30mg de proteínas en dos
muestras de 4 a 6 h. En ausencia de proteinuria, el diagnóstico de preeclampsia podría
establecerse cuando la hipertensión gestacional es asociada con síntomas cerebrales
persistentes, epigastralgia o dolor en cuadrante superior derecho con náusea o vómito o
bien trombocitopenia con alteraciones en las concentraciones de las enzimas hepáticas. Se
caracteriza por una reducción de la perfusión sistémica generada por vasoespasmo y
activación de los sistemas de coagulación lo que puede producir la hemorragia de encías y
epistaxis frecuente. Para su diagnóstico deberán considerarse las siguientes condiciones
específicas: 1. Hipertensión arterial: presión sistólica igual o mayor a 140 y/o presión
diastólica igual o mayor a 90. 2. Proteinuria: Mayor o igual a 300 mg de proteínas en orina
de 24 horas, o presencia de 1+ en tira reactiva en una muestra tomada al azar o relación
proteína-creatinina > 0.3. 3. En ausencia de proteinuria, con presencia de condiciones
adversas: trombocitopenia (100mil plaquetas/microlitro), creatinina (mayor a 1.1mg/dl en
ausencia de enfermedad renal), alteraciones hepáticas (al doble de la concentración normal),
edema agudo pulmonar. RECUERDA: existen diversos factores de riesgo asociados a
preeclampsia entre los que figuran la hipertensión crónica o enfermedad renal preexistente,
diabetes mellitus preexistente, enfermedades del tejido conectivo como lupus, trombofilia
congénita o adquirida, obesidad, edad mayor a 40 años, embarazo gemelar, embarazo triple,
antecedente de preeclampsia, embarazo por semen de donador y donación de oocito. EL
EMBARAZO GEMELAR INCREMENTA HASTA EN UN 20% EL RIESGO DE PREECLAMPSIA, LA
PACIENTE CUMPLE CON LOS CRITERIOS CLÍNICOS PARA SER CLASIFICADA CON ESTE
PADECIMIENTO.

HIPERTENSIÓN LA PRESENCIA DE PROTEINURIA DESCARTA COMPLETAMENTE LA POSIBILIDAD DE QUE SE


GESTACIONAL. TRATE DE HIPERTENSIÓN GESTACIONAL. Los TRASTORNOS HIPERTENSIVOS DEL
EMBARAZO (HDP) representan la complicación más común en el embarazo, afectando
aproximadamente el 15% de los embarazos y representan casi el 18% de todas las muertes
maternas en el mundo. IMPORTANTE!!! Con base en la última modificación a la GPC para
preeclampsia (2017) los desórdenes hipertensivos en el embarazo se han reclasificado como:
a) HIPERTENSIÓN PREEXISTENTE: Es definida como la hipertensión presente antes del
embarazo o que se desarrolla antes de las 20 semanas de gestación. - Con condiciones
comórbidas: por ejemplo, Diabetes mellitus I o II pregestacional o enfermedad renal,
garantizan un control de la presión sanguínea fuera del embarazo por su asociación con
riesgo cardiovascular elevado. - Con evidencia de preeclampsia: Es también conocida como
“preeclampsia sobreagregada" y es definida por el desarrollo de 1 o más de los siguientes
criterios antes de la semana 20 de gestación: hipertensión resistente (necesidad de 3
antihipertensivos para el control de la presión arterial), ó proteinuria de nueva aparición o
empeoramiento de la misma, ó 1 o más condiciones adversas, ó 1 o más condiciones severas.
b) HIPERTENSIÓN GESTACIONAL: Esta definida como la hipertensión que se desarrolla por
primera vez después de la semana 20 de gestación con presión arterial sistólica ?140mmHg
y/o presión arterial diastólica ?90mmHg. - Con condición comórbida: por ejemplo, Diabetes
mellitus I o II pregestacional o enfermedad renal, garantizan un control de la presión
sanguínea fuera del embarazo por su asociación con riesgo cardiovascular elevado. - Con
evidencia de preeclampsia.: mismos criterios que para la evidencia en hipertensión
preexistente. c) PREECLAMPSIA: La evidencia de preeclampsia puede aparecer muchas
semanas después del inicio de la hipertensión gestacional. Está definida como hipertensión
que se presenta en el embarazo (TA sistólica ? 140 mmHg o TA diastólica ? a 90mmHg en 2
ocasiones con un margen de al menos 4 horas después de la semana 20 de gestación en una
mujer con presión arterial normal antes del embarazo) y uno o más de los siguientes puntos:
Proteinuria de nueva aparición (igual o mayor a 300 mg en recolección de orina de 24 hrs) ó
Labstix con lectura de 1+ solo si no se dispone de otro método cuantitativo ó Relación
proteína/creatinina ? a 0.28 mg/dL ó en ausencia de proteinuria, 1 ó más condiciones
adversas. d) OTRAS FORMAS HIPERTENSIVAS: - Hipertensión transitoria: La elevación de la
presión sanguínea puede deberse a estímulos ambientales o el dolor del parto, por ejemplo.
- Hipertensión de bata blanca: La presión sanguínea esta elevada en el consultorio (TA
sistólica ? a 140 mmHg ó TA ? a 90mmHg) pero es consistentemente normal fuera del
consultorio (menor a 135/85 por monitoreo ambulatorio o en su domicilio). e)
HIPERTENSIÓN ENMASCARADA: La presión sanguínea es consistentemente normal en el
consultorio (TA sistólica menor a 140 mmHg o TAD menor a 90 mmHg) pero se eleva
consistentemente fuera del consultorio (Mayor a 135/85 mmHg) por monitoreo ambulatorio
o monitoreo en casa.

Bibliografía:
1. PREVENCIÓN, DIAGNÓSTICO Y TRATAMIENTO DE LA PREECLAMPSIA EN EL SEGUNDO Y TERCER
NIVELES DE ATENCIÓN. EVIDENCIAS Y RECOMENDACIONES. GUÍA DE PRÁCTICA CLÍNICA. MÉXICO,
SECRETARÍA DE SALUD; 09/03/17.

http://www.cenetec-difusion.com/CMGPC/S-020-08/ER.pdf
ANÁLISIS DEL CASO CLÍNICO

IDENTIFICACIÓN DEL REACTIVO


Area: CIRUGÍA
Especialidad: OFTALMOLOGÍA
Tema: ENFERMEDADES DE LA RETINA
Subtema: RETINOPATÍA DIABÉTICA

CASO CLÍNICO CON UNA PREGUNTA

HOMBRE DE 59 AÑOS DE EDAD, DIABÉTICO DESDE HACE 10 AÑOS. ACUDE A CONSULTA DE CONTROL DONDE
USTED REALIZA OFTALMOSCOPIA, ENCONTRANDO LOS SIGUIENTES HALLAZGOS: PAPILA ÓPTICA DE LÍMITES
Y COLORACIÓN NORMAL, EXCAVACIÓN PAPILAR CON CARACTERÍSTICAS FISIOLÓGICAS. RELACIÓN ENTRE EL
CALIBRE ARTERIAL Y VENOSO A/V=3/4. PRESENCIA DE ARROSARIAMIENTO VENOSO EN DOS CUADRANTES,
MÁCULA NORMAL.

Adulto de 59 años de edad.

Antecedente de diabetes de 10 años de


evolución.

-.

La clave, es la presencia de arrosariamiento


venoso en dos cuadrantes.

-.

93 - CON BASE EN EL FONDO DE OJO, USTED CONSIDERARÁ QUE EL PACIENTE CURSA CON
RETINOPATÍA:
MÍNIMA. El ojo del paciente diabético puede afectarse por diversos procesos patológicos, de los cuales el
más importante es la RETINOPATÍA DIABÉTICA. La diabetes es la principal causa de pérdida de la
visión, siendo la ceguera 25 veces más común en diabéticos que en controles. Las lesiones suelen
aparecer a partir de los 10 años del diagnóstico de la diabetes tipo I, mientras que en los
pacientes con diabetes tipo II, hay lesiones visibles en el momento del diagnóstico hasta en el
39% de los pacientes, lo que significa que la enfermedad ha evolucionado varios años sin ser
diagnosticada, el 90% de los diabéticos tipo I muestra algún tipo de retinopatía a los 15 años del
diagnóstico de la diabetes y el 10% presenta signos proliferativos. La Retinopatía diabética tiene
por lo general un curso progresivo, aunque en sus primeras fases ciertas lesiones pueden remitir
de forma espontánea. La RETINOPATÍA NO PROLIFERATIVA MÍNIMA (leve o de fondo), consiste
en la aparición de microaneurismas, hemorragias en punto, manchas o flama, exudados duros,
blandos, anormalidades microvasculares intrarretinianas (AMIR) y arrosariamiento. En esta
primera fase la reducción de la agudeza visual puede ser moderada o nula, excepto cuando
aparece edema de la mácula o los exudados y las hemorragias aparecen en esta localización.
PUEDE HABER ARROSARIAMIENTO MÍNIMO PERO NO CON LESIÓN EN MÁS DE UN
CUADRANTE. EL DATO CLÁSICO EN ESTE LA RETINOPATÍA MÍNIMA SON LOS
MICROANEURISMAS SOLAMENTE.

MODERADA. En la RETINOPATÍA DIABÉTICA MODERADA, los "capilares presentan engrosamiento" de la


membrana basal, microaneurismas con hemorragias moderadas en los 4 cuadrantes o severas en
menor de 4 (-3), degeneración de los pericitos, falta de perfusión y obstrucción. También hay
aumento de la permeabilidad vascular, con escapes locales desde los microaneurismas o bien
generalizados, que dan origen a los "exudados duros". Puede aparecer edema retiniano más
común en el polo posterior y región de la mácula que es también el sitio donde predominan los
exudados. También se observa arrosariamiento venoso leve en 1 cuadrante y microvasculares
intrarretinianas (AMIR) leves en 1-4 cuadrantes. EN LA RETINOPATÍA DIABÉTICA MODERADA
ADEMÁS DE LOS MICROANEURISMAS SE PUEDEN OBSERVAR EXUDADOS DUROS Y
ENGROSAMIENTO DE LOS CAPILARES.

SEVERA. En la RETINOPATÍA AVANZADA (severa), hay obliteración de vasos con grandes parches de
capilares acelulares que proceden de la oclusión de arteriolas terminales, aparición de nidos de
"microaneurismas" y "vasos tortuosos" que se denominan anormalidades microvasculares
intraretinianas (AMIR). El "exudado blando y algodonoso" es la expresión oftalmológica de la
oclusión capilar reciente (infarto retinal). Cuando existe IRMA, las dilataciones venosas, las
hemorragias retinianas, los exudados algodonosos son intensos, la retinopatía no proliferativa
se considera severa o preproliferativa. LA RETINOPATÍA DIABÉTICA NO PROLIFERATIVA SEVERA
SE CARACTERIZA POR CUALQUIERA DE LOS SIGUIENTES HALLAZGOS: - Presenta más de 20
hemorragias retinianas en cada cuadrante (complejos microaneurina-hemorragia severos). -
Arrosariamiento venoso en 2 o más cuadrantes. - AMIR en un cuadrante o más. EL HECHO DE
QUE EL ARROSARIAMIENTO ABARQUE 2 CUADRANTES ES LA BASE DEL DIAGNÓSTICO.

PROLIFERATIVA. La "aparición de NEOVASOS" en la superficie de la retina indica el ESTADIO PROLIFERATIVO.


Entre las causas que conducen a la pérdida de la agudeza visual, la Retinopatía Proliferativa se
considera la más importante en la diabetes tipo I, y el Edema Macular en la diabetes tipo II. LA
RETINOPATÍA DIABÉTICA PROLIFERATIVA, SE CARACTERIZA POR UNO O MÁS DE LOS
SIGUIENTES HALLAZGOS: NEOVASOS, HEMORRAGIA VÍTREA O SUBHIALOIDEA.

Bibliografía:
1. GUÍA DE REFERENCIA RÁPIDA, DIAGNÓSTICO Y TRATAMIENTO DE RETINOPATÍA DIABÉTICA. MÉXICO:
SECRETARIA DE SALUD; 2009. 2. GUÍA DE PRÁCTICA CLÍNICA, DIAGNÓSTICO Y TRATAMIENTO DE
RETINOPATÍA DIABÉTICA. MÉXICO: SECRETARIA DE SALUD; 2009.

http://www.cenetec.salud.gob.mx/descargas/gpc/CatalogoMaestro/171_GPC_RETINOPATIA_DIABETICA/Imss_171ER.pdf
ANÁLISIS DEL CASO CLÍNICO

IDENTIFICACIÓN DEL REACTIVO


Area: PEDIATRÍA
Especialidad: INFECTOLOGIA PEDIÁTRICA
Tema: INFECCIONES VÍAS RESPIRATORIAS ALTAS
Subtema: OTITIS MEDIA AGUDA Y CRÓNICA

CASO CLÍNICO CON UNA PREGUNTA

LACTANTE DE AÑO Y MEDIO, ACUDE A URGENCIAS POR LA PRESENCIA DE FIEBRE DE 39°C, IRRITABILIDAD Y
REFIERE DOLOR EN OÍDO DERECHO. CUENTA CON EL ANTECEDENTE DE RINORREA HIALINA Y TOS
PRODUCTIVA HACE 72 HRS. A LA EXPLORACIÓN EL OÍDO DERECHO CON HIPEREMIA EN EL CONDUCTO,
ABOMBAMIENTO E HIPEREMIA DE LA MEMBRANA Y DISMINUCIÓN DE LA MOVILIDAD A LA NEUMATOSCOPIA
DEL TÍMPANO DERECHO.

LACTANTE DE AÑO Y MEDIO

FIEBRE DE 39°C, IRRITABILIDAD Y REFIERE


DOLOR EN OíDO DERECHO. CUENTA CON
EL ANTECEDENTE DE RINORREA HIALINA
Y TOS PRODUCTIVA HACE 72 HRS

OíDO DERECHO CON HIPEREMIA EN EL


CONDUCTO, ABOMBAMIENTO E
HIPEREMIA DE LA MEMBRANA Y
DISMINUCIóN DE LA MOVILIDAD A LA
NEUMATOSCOPIA DEL TÍMPANO
DERECHO. LA OTOTSCOPÍA ES
DIAGNÓSTICA, SUMADA A LOS SÍNTOMAS
CARACTERÍSTICOS

94 - LA PACIENTE CURSA CON EL DIAGNÓSTICO DE:


OTITIS LA OTOSCOPIA PRESENTADA ES POSITIVA PARA OTITIS MEDIA AGUDA (OMA) La membrana
MEDIA timpánica normal es ligeramente cóncava y presenta un color gris perla o rosado pálido, siendo
AGUDA ligeramente transparente y permitiendo una cierta visualización de las estructuras de oído medio. •
Es de valoración especialmente difícil en el período neonatal, por la estrechez del conducto y la
opacidad de la membrana timpánica. • La hiperemia aislada sin otros signos (abombamiento,
matidez, coloración azul o amarilla) no tiene ningún valor, especialmente si el niño llora. Puede
significar simplemente miringitis (inflamación de la membrana timpánica), que frecuentemente
acompaña a un proceso catarral vírico, sin derrame asociado en oído medio. La miringitis puede ser
una fase inicial de la OMA, por lo que el niño que la presente debe ser seguido evolutivamente, u
observarse en la fase de resolución de una OMA. La forma ampollosa (miringitis bullosa) es una
forma de presentación de OMA, con la misma etiología, aunque en este caso un posible agente
etiológico adicional es Mycoplasma Pneumonae. • La no visualización del triángulo luminoso no
tiene ningún valor. • El color azul o amarillo son sugestivos de otitis media. El tímpano azulado se
visualiza más frecuentemente en OME, y el amarillo en OMA, pero esta regla no es, en absoluto,
patognomónica. • La hiperemia asociada a matidez es sugestiva de otitis media. • Las placas blancas
en la membrana suelen traducir procesos crónicos o cicatriciales (timpanoesclerosis). • La retracción
timpánica marcada es sugestiva de OME. • Son casi patognomónicos de OMA el abombamiento
timpánico (excepcionalmente presente en otitis media con efusión) y la presencia de una
supuración aguda procedente de oído medio. • Los clásicos signos “del trago” o “del pabellón” no
tienen valor en el diagnóstico de OMA. En cambio, son altamente sospechosos de otitis externa. • Si
estos signos otoscópicos se acompañan de sintomatología clínica aguda y, fundamentalmente, de
otalgia o su equivalente de irritabilidad en lactantes, se hará el diagnóstico de OMA. SIGNOS Y
SÍNTOMAS. Específicos • Otalgia • Equivalente a la otalgia en lactantes • Irritabilidad • Tracción del
pabellón auricular • Hipoacusia de corta evolución sin tapón de cerumen • Supuración aguda
proveniente del oído medio Inespecíficos • Fiebre • Vómitos • Diarrea

MASTOIDITIS La MASTOIDITIS AGUDA es una infección de la apófisis mastoides, el hueso prominente que se
AGUDA encuentra detrás del oído. Este trastorno suele ocurrir cuando una otitis media no tratada o tratada
inadecuadamente se extiende desde el oído medio hasta el hueso que lo circunda (la apófisis
mastoides). Síntomas Por lo general, los síntomas aparecen al cabo de dos o más semanas de
haberse desarrollado la otitis media aguda, a medida que la infección diseminada destruye la parte
interna de la apófisis mastoides. Es posible que se forme un absceso en el hueso. La piel que
recubre la apófisis mastoides puede tornarse roja, hinchada y dolorosa, y el oído externo se
desplaza hacia un lado y abajo. Otros síntomas son fiebre, dolor alrededor del oído y en su interior
una secreción profusa y cremosa por el oído, todo lo cual suele empeorar. El dolor tiende a ser
persistente y pulsátil. La pérdida de la audición es progresiva. La tomografía computadorizada (TC)
demuestra que las celdillas (los espacios en el hueso que normalmente contienen aire) de la apófisis
mastoides están llenas de líquido. A medida que la mastoiditis avanza, los espacios se agrandan.
Una mastoiditis mal tratada puede producir sordera, infección de la sangre (sepsis), meningitis,
abscesos cerebrales o la muerte.

OTITIS La OTITIS MEDIA SUPURADA debe tratarse siempre con ATB, y recibir el mismo manejo terapéutico
MEDIA CON que la OMA con membrana timpánica integra, salvo en la duración del tratamiento antibiótico, que
DERRAME debe ser siempre de 10 días. La perforación de la membrana timpánica con evacuación de exudado
purulento y eventualmente de sangre es un evento tan común en la evolución de la OMA que debe
considerarse una forma de presentación clínica más que una complicación. Generalmente, el dolor
desaparece después de la perforación. El manejo terapéutico deberá hacerse siguiendo el mismo
algoritmo de actuación de la OMA sin supuración, puesto que la presencia de supuración no tiene
ninguna repercusión en cuanto a la sospecha del germen implicado, aunque en este caso aumenta
el aislamiento de Streptococcus pyogenes y estafilococo. Un inicio muy agudo con otalgia intensa y
supuración rápida se relacionó clásicamente como muy sugestivo de neumococo, pero recientes
estudios han demostrado que esta evolución es compatible con el HI y otros gérmenes. El
tratamiento ATB deberá prolongarse siempre durante 10 días, independientemente de la edad del
niño. OJO. EN ESTA HAY PERFORACIÓN DE LA MEMBRANA CON DERRAME DEL CONTENIDO.
OTITIS OTITIS EXTERNA La clínica se caracteriza fundamentalmente por el dolor, precedido a veces de
EXTERNA prurito, con los clásicos signos del trago y del pabellón muy positivos. Suele ser unilateral.
Habitualmente, hay disociación clínico-otoscópica, puesto que el niño tiene un dolor muy superior
a los escasos o nulos signos inflamatorios que pueden visualizarse en fases iniciales.
Posteriormente, aparece hiperemia de intensidad variable de la piel del conducto auditivo externo
y estrechamiento hasta del 50% de la luz del mismo, con dificultades para visualizar la membrana
timpánica. Es frecuente la hipoacusia y no es rara la supuración. La fiebre no es común y cuando
aparece suele traducir infección mixta estafilococo-pseudomonas. Puede acompañarse de
adenopatías preauriculares. El diagnóstico diferencial con la OMA no siempre es fácil,
especialmente cuando hay supuración que impide la valoración de la membrana timpánica. El dolor
suele mejorar cuando hay perforación en la OMA y en cambio empeora con la otorrea en la otitis
externa. En caso de duda, el timpanómetro es un aliado imprescindible, al verificarnos si existe o no
perforación timpánica con la valoración del volumen de oído medio. Los signos del trago y del
pabellón son propios de la otitis externa y no de la OMA. En la OMA suele haber un antecedente
catarral y es más frecuente la fiebre. La otitis externa es más propia del verano.

Bibliografía:
NELSON TRATADO DE PEDIATRÍA. BEHRMAN. MC GRAW HILL INTERNACIONAL. EDICIÓN 15A. 1996. PAG.
2258-2259.
ANÁLISIS DEL CASO CLÍNICO

IDENTIFICACIÓN DEL REACTIVO


Area: CIRUGÍA
Especialidad: CIRUGÍA ABDOMINAL
Tema: ENFERMEDAD ÁCIDO PÉPTICA Y SUS COMPLICACIONES
Subtema: COMPLICACIONES DE LA ENFERMEDAD ÁCIDOPÉPTICA

CASO CLÍNICO SERIADO

HOMBRE DE APROXIMADAMENTE 30 AÑOS DE EDAD, ES ENCONTRADO EN LA CALLE CON PÉRDIDA DEL


ESTADO DE ALERTA E INTENSO ALIENTO ALCOHÓLICO. ES LLEVADO AL SERVICIO DE URGENCIAS DEL
HOSPITAL. A LA EXPLORACIÓN FÍSICA CON MALA HIGIENE, RESTOS DE VÓMITO EN SU ROPA, GLASGOW DE 13-
14, TA 90/55, FC 115X´, FR 26X´, TEMPERATURA 38.9°C, EL ABDOMEN SE ENCUENTRA DISTENDIDO, REFLEJO
DOLOROSO A LA PALPACIÓN MEDIA Y PROFUNDA PRINCIPALMENTE EN EPIGASTRIO, REBOTE
GENERALIZADO, PÉRDIDA DE LA MATIDEZ HEPÁTICA Y TIMPANISMO GENERALIZADO.

Masculino adulto joven de 30 años de edad.

encontrado en la calle, pÉrdida del estado


de alerta, alcoholismo positivo (aliento
alcÓholico).

-.

se encuentra CON GLASGOW DE 13-14,


presenta restos alimenticios en su ropa,
con hipotensión arterial de 90/55 mmhg,
taquicardia de 115xmin,fr de 26xmin, ebre
de 38.9°C. presenta distensión abdominal,
dolor a la palpación media y profunda de
predominio en epigastrio, rebote
generalizado,hay pÉrdida de la matidez
hepática y timpanismo generalizado.

-.

95 - PARA APOYAR SU DIAGNÓSTICO DEBERÁ SOLICITAR DE MANERA INICIAL:


ULTRASONIDO LO PRIMERO ES INTEGRAR EL DIAGNÓSTICO DEL PACIENTE EL CUAL EN ESTE CASO SERÍA UNA
ABDOMINAL. ÚLCERA GÁSTRICA PERFORADA. TRAS ELLO DEBES PONER ESPECIAL ATENCIÓN A LO
SOLICITADO EN LA PREGUNTA "APOYO DIAGNÓSTICO INICIAL". El ULTRASONIDO puede ser útil
en el diagnóstico de algunas entidades causantes de abdomen agudo. Sin embargo, debe tenerse
en mente que su sensibilidad y especificidad son dependientes del operador. Su principal uso es
en el diagnóstico de la ENFERMEDAD BILIAR. Se ha empleado también para el diagnóstico de
apendicitis, cada vez con mayor frecuencia, especialmente en niños. Por ejemplo en el hallazgo de
un apéndice mayor de 7 mm constante en varias imágenes sugiere el diagnóstico de apendicitis.
NO ES DE AYUDA EN ESTE CASO YA QUE NO APORTA DATOS EN PERFORACIÓN DE ÚLCERA
GÁSTRICA.

ESTUDIOS Los estudios contrastados rara vez están indicadas en el estudio diagnóstico de pacientes con
CON MEDIO abdomen agudo. LAS PLACAS SIMPLES Y CONTRASTADAS CON "MATERIAL HIDROSOLUBLE",
DE SON UNA EXCELENTE ALTERNATIVA PARA EL DIAGNÓSTICO DE ÚLCERA PÉPTICA PERFORADA.
CONTRASTE. IMPORTANTE: solo se pueden utilizar los medios de contraste hidrosoluble, no otros tipos de
contrastes. En la opción no se especifica el tipo, y aunque así fuera es necesario "iniciar" con
radiografías simples.

TOMOGRAFÍA Preferentemente la TOMOGRAFÍA AXIAL COMPUTARIZADA (TAC) debe realizarse empleando


COMPUTADA. medio intravenoso y oral. Permite evaluar detalladamente las vísceras sólidas y las estructuras
retroperitoneales (páncreas, aorta). Es de gran valor en el diagnóstico de abscesos
intraperitoneales y en casos seleccionados puede ser útil para realizar drenaje percutáneo. Cuando
se sospecha diverticulitis, la tomografía puede ser de gran utilidad para confirmar el diagnóstico y
de hecho constituye el estudio de elección .En casos de apendicitis de difícil diagnóstico la TAC
puede ser de gran valor y está indicada casi de rutina. Si bien la TAC está siendo empleada cada
vez con mayor frecuencia en la evaluación y diagnóstico de muchas condiciones abdominales, en
ningún caso debe remplazar la completa evaluación y el juicio clínico. No está indicada de inicio
en nuestro paciente, dado a que existen otros exámenes de gabinete que nos pueden aportar
datos más específicos, que son más accesibles y de menor costo. ANTE LA SOSPECHA CLÍNICA DE
ÚLCERA PÉPTICA PERFORADA EL ESTUDIO DIAGNÓSTICO DE PRIMERA ELECCIÓN ES LA
TOMOGRAFÍA AXIAL COMPUTARIZADA CON MEDIO DE CONTRASTE HIDROSOLUBLE, EN BUSCA
DE AIRE LIBRE INTRAABDOMINAL O FUGA DE MEDIO DE CONTRASTE. OJO: ES EL ESTUDIO DE
ELECCIÓN PERO NO EL" INICIAL" POR EL COSTO QUE IMPLICA, SI LAS RADIOGRAFÍAS SIMPLES
DE ABDOMEN Y TÓRAX SON DUDOSAS SE PROCEDERÁ A ESTE MÉTODO.

RADIOGRAFÍA La PERFORACIÓN DE UNA ÚLCERA PÉPTICA es una de las complicaciones de la Enfermedad Ácido-
DE TÓRAX Y Péptica. El síntoma principal es "dolor abdominal" (70% de los casos, generalmente muy intenso),
ABDOMEN. inicialmente epigástrico y posteriormente generalizado tipo transfictivo, intenso, de inicio súbito,
se exacerba con los movimientos y se irradia hacia el abdomen bajo derecho o a hombros,
acompañado de distensión abdominal y datos de “irritación peritoneal” (56.7%), vómito y
deshidratación (50%), oliguria (30%) y choque séptico (20%). Es frecuente encontrar entonces
“fiebre”, “taquicardia” e “íleo”. El diagnóstico clínico se realiza palpando el abdomen, que casi
siempre presenta hipersensibilidad exquisita, rigidez y rebote. En la percusión se encuentra
"pérdida de la matidez hepática". EL INICIO DEL DOLOR ES DE MANERA SÚBITA CON
TAQUICARDIA, DIAFORESIS Y DATOS DE IRRITACIÓN PERITONEAL. En los pacientes en los que se
sospecha una ÚLCERA PÉPTICA PERFORADA es necesario La evaluación radiológica inicial en
busca de neumoperitoneo, lo cual se efectúa con TELERRADIOGRAFÍA DE TÓRAX Y PLACA SIMPLE
DE ABDOMEN de pie; si las condiciones lo permiten, el paciente previamente debe colocarse de
pie o sentado por 5 a 10 minutos. CORRESPONDE AL MÉTODO DIAGNÓSTICO DE "PRIMERA
LÍNEA" EN LA EVALUACIÓN DE ÚLCERA PÉPTICA PERFORADA, DEBES CONSIDERAR QUE LA
PREGUNTA ESPECIFICA "MÉTODO DIAGNÓSTICO INICIAL". IMPORTANTE: En un principio se
menciona que el paciente fue encontrado en estado de alerta; sin embargo, a la exploración su
Glasgow es de 13-15. EN CASO DE QUE EL PACIENTE PRESENTARA UNA PÉRDIDA REAL DEL
ESTADO DE ALERTA AL MOMENTO DE LA EXPLORACIÓN O INCAPACIDAD PARA PONERLO DE PIE
SE DEBE ELEGIR LA TOMOGRAFÍA YA QUE LAS RADIOGRAFÍAS DEBEN TOMARSE DE PIE.

Bibliografía:
1. GUÍA DE PRÁCTICA CLÍNICA, TRATAMIENTO DE LA PERFORACIÓN DE ÚLCERA PÉPTICA EN PACIENTES
ADULTOS EN EL 2° NIVEL DE ATENCIÓN. MÉXICO: SECRETARIA DE SALUD; 2012. RECUPERADO DE
HTTP://WWW.CENETEC.SALUD.GOB.MX/CONTENIDOS/GPC/CATALOGOMAESTROGPC.HTML

http://www.cenetec.salud.gob.mx/descargas/gpc/CatalogoMaestro/549_GPC_uelcerapepticaperforada/GPCxEYRxLCERAxPEPTICAxPERFORADA.pdf
96 - EL DIAGNÓSTICO CLÍNICO MÁS PROBABLE ES:

APENDICITIS LA APENDICITIS AGUDA es la inflamación del apéndice vermicular secundaria a la obstrucción de


AGUDA la luz. Y se considera complicada cuando ya se originó gangrena y rotura del órgano lleno de pus,
COMPLICADA. lo que puede condicionar flemón, abscesos apendiculares, peritonitis por diseminación, pileflebitis
supurativa y obstrucción intestinal. Los pacientes con una apendicitis aguda complicada lucen
enfermos, postrados, tóxicos, deshidratados y con distensión abdominal. A la Exploración física
puede palparse plastrón, hay hipersensibilidad en todo el cuadrante inferior derecho presencia de
rebote y rigidez muscular. La fiebre y el pulso también siguen a la gravedad del paciente. Es uno
de los diagnósticos diferenciales de la úlcera péptica complicada. La apendicitis aguda no tiene
relación con la ingesta de alcohol como en el caso de nuestro paciente. LOS ANTECEDENTES DEL
PACIENTE NO SE RELACIONAN CON APENDICITIS, ADEMÁS DE QUE EL TIPO DE DOLOR Y SU
UBICACIÓN NO COINCIDE CON EL CUADRO CLÍNICO DESCRITO.

VÓLVULO EL VÓLVULO INTESTINAL es la torsión anormal o rotación de una porción del intestino sobre su
INTESTINAL. mesenterio, que ocasiona oclusión de la luz, obstrucción y compromiso vascular. Es una
obstrucción mecánica que ocurre debido a una torsión anatómica del intestino alrededor de un
pedículo mesentérico estrecho. Puede producirse en cualquier parte del tubo gastrointestinal,
pero se observa en porciones con mesenterio largo, estrecho en su raíz y móviles principalmente.
La mayoría ocurre en el colon izquierdo en un 45% a un 80%, en el colon derecho en un 15% a
30% y transverso y ángulo esplénico en un 2 a 5%. El sitio más usual del vólvulo es el colon
sigmoideo, constituyendo una forma bien reconocida, sobre todo en nuestra zona, como
complicación del megacolon. Las manifestaciones son dolor abdominal tipo cólico, distensión
abdominal, vómitos y estreñimiento. Muchos pacientes tienen episodios previos de dolor
abdominal y distensión. Cuentan con leucocitosis, fiebre y datos de irritación peritoneal ante la
presencia de gangrena. NO GUARDA RELACIÓN CON EL ALCOHOLISMO, NO JUSTIFICA LA
PÉRDIDA DE MATIDEZ HEPÁTICA.

ULCERA En este paciente su diagnóstico es una ÚLCERA PÉPTICA COMPLICADA con perforación, este
PÉPTICA diagnóstico se sustenta en: - Los antecedentes y factores de riesgo como lo son la edad que
COMPLICADA. generalmente se presentan en la vida adulta temprana o edad madura y es más frecuente en los
varones. El tabaquismo y el alcoholismo son antecedentes de gran importancia. - Los datos
clínicos obtenidos en la exploración física los cuales son dolor abdominal muy intenso, fiebre,
taquicardia, generalmente se encuentran deshidratados y con íleo. El abdomen suele presentar
hipersensibilidad exquisita, rigidez y rebote. Y en la percusión se encuentra pérdida de la matidez
hepática. EL ANTECEDENTE DE ALCOHOLISMO + DATOS CLÍNICOS DEL PACIENTE SON LA BASE
DE LA SOSPECHA DIAGNÓSTICA. IMPORTANTE: en los casos clínicos se te aportan siempre
antecedentes que dan peso a la sospecha diagnóstica, te recomiendo leer con cuidado los casos
para no pasar por alto su presencia.

PANCREATITIS En los pacientes quienes presentan un cuadro de PANCREATITIS AGUDA inicia con dolor en el
AGUDA. epigastrio, intenso y persistente que a menudo se irradia a la espalda. Es común que se presente
después de una comida copiosa. Se acompaña de náuseas, vómitos y arqueo persistentes. En la
exploración física abdominal se encuentra hipersensibilidad más intensa en epigastrio, pero en
ocasiones generalizada, los ruidos intestinales se encuentran disminuidos o ausentes.
Generalmente no hay masas palpables a menos que se trate de una complicación de la pancreatitis
como son los pseudoquistes o abscesos. En la pancreatitis hemorrágica se encuentra un abdomen
distendido, la temperatura se encuentra discretamente elevada (37.7 a 38.3 °C). El diagnóstico
suele confirmarse con las enzimas pancreáticas (lipasa y amilasa) la cuales se encuentran elevadas.
Aunque con frecuencia el cuadro clínico sugiere el diagnóstico correcto y los datos de laboratorio
suelen confirmarlo, es importante resaltar que la pancreatitis es un diagnóstico de exclusión. ESTE
PUEDE SER UN BUEN DIAGNÓSTICO DIFERENCIAL EN ESTE PACIENTE POR EL ANTECEDENTE
ALCOHÓLICO. SI BIEN LA UBICACIÓN DEL DOLOR ES SIMILAR, SOLO EN LA ÚLCERA PÉPTICA
PERFORADA SE PRESENTA EL HALLAZGO DE PÉRDIDA DE MATIDEZ HEPÁTICA.

Bibliografía:
1. GUÍA DE PRÁCTICA CLÍNICA, TRATAMIENTO DE LA PERFORACIÓN DE ÚLCERA PÉPTICA EN PACIENTES
ADULTOS EN EL 2° NIVEL DE ATENCIÓN. MÉXICO: SECRETARIA DE SALUD; 2012. RECUPERADO DE
HTTP://WWW.CENETEC.SALUD.GOB.MX/CONTENIDOS/GPC/CATALOGOMAESTROGPC.HTML 2.
BRUNICARDI F, ANDERSEN D, BILLIAR T, Y COLS. SCHWARTZ PRINCIPIOS DE CIRUGÍA, 9A EDICIÓN. MC
GRAW HILL. 2011, PP 921.

http://www.cenetec.salud.gob.mx/descargas/gpc/CatalogoMaestro/549_GPC_uelcerapepticaperforada/GPCxEYRxLCERAxPEPTICAxPERFORADA.pdf
97 - EL TRATAMIENTO INDICADO EN ESTE CASO ES:

RESECCIÓN ES MUY IMPORTANTE ACERTAR A LOS PRIMEROS REACTIVOS DE UN CASO CLÍNICO


INTESTINAL Y SERIADO MEDIANTE EL ANÁLISIS DEL CASO Y LA CORRELACIÓN DE LAS RESPUESTAS,
COLOSTOMÍA EN RECUERDA SEGUIR SIEMPRE LA MISMA LÍNEA DE TU DIAGNÓSTICO. El tratamiento
BOLSA DE quirúrgico que se recomienda para el VÓLVULO DEL SIGMOIDES en un paciente que se ha
HARTMANN. preparado de manera adecuada es una resección con anastomosis primaria, pero en la
intervención quirúrgica de urgencia en un intestino no preparado suele ser necesario resecar
el colón sigmoides dañado y hacer una colostomía en bolsa de Hartmann. Se reseca todo el
intestino no viable; la mortalidad depende de la presencia de intestino gangrenado. ES EL
TRATAMIENTO DE ELECCIÓN EN CASO DE VÓLVULO SIGMOIDEO.

CIERRE CON El manejo médico inicial de la ÚLCERA PERFORADA incluye ayuno estricto, resucitación
PARCHE DE hídrica con fluidos intravenosos, inhibidores H2 o inhibidores de bomba de protones,
GRAHAM. antibióticos de amplio espectro, succión nasogástrica y manejo de la sepsis. Se recomienda
que terapia antimicrobiana cubra bacterias entéricas gram negativas, anaerobios, flora oral y
hongos. El tratamiento de elección en úlcera péptica perforada, es cirugía con abordaje
abierto o laparoscópico. El objetivo del tratamiento de la perforación de úlcera gástrica es
identificar tempranamente la perforación y corregirlo de inmediato. Se colocan puntos de
sutura (ya sea seda o lo que elija el cirujano), seromusculares en el sitio de la perforación y se
coloca un segmento de epiplón en el sitio quirúrgico para proporcionar un apoyo biológico a
lo que se le conoce como PARCHE DE GRAHAM (parche de epiplón). Otros autores
recomiendan la vagotomía proximal con parche de Graham como tratamiento de elección
para la úlcera perforada. La técnica definitiva a utilizar dependerá de la experiencia del
cirujano y de las condiciones clínicas del paciente. La cirugía abierta se recomienda en: a)
Paciente inestable con perforación y sangrado simultáneo b) Paciente con choque a la
admisión c) Atención tardía, más de 24 horas d) Paciente con comorbilidad cardiovascular o
respiratoria, que comprometa la habilidad del paciente para tolerar el pneumoperitoneo e)
Pacientes con antecedente de cirugía abdominal, que pudiera dificultar el acceso por
laparoscopía f) Localización poco accesible de la úlcera (conversión de cirugía laparoscópica a
abierta) g) Perforación mayor de 6 mm (conversión de cirugía laparoscópica a abierta) h)
Úlcera con bordes friables o infiltrativos (conversión de cirugía laparoscópica a abierta) EL
OBJETIVO PRIMARIO Y MÁS IMPORTANTE DE LA ÚLCERA PERFORADA, ES CERRAR LA
PERFORACIÓN, POR TAL MOTIVO DE EXISTIR LA OPCIÓN QUIRÚRGICA, COMO EN ESTE
CASO, SIEMPRE SERÁ LA DE ELECCIÓN.

APENDICECTOMÍA. La APENDICECTOMÍA es el único tratamiento para los pacientes con apendicitis aguda no
complicada y complicada. Y se debe de realizar en cuanto se disponga de los estudios
mínimos necesarios para un buen ejercicio médico. Se puede realizar la apendicectomía
abierta o por laparoscopia, esto lo decidirá el cirujano.

COLOCACIÓN DE El manejo médico inicial de la ÚLCERA PERFORADA incluye ayuno estricto, resucitación
SONDA, hídrica con fluidos intravenosos, inhibidores H2 o inhibidores de bomba de protones,
ANALGÉSICOS Y antibióticos de amplio espectro, succión nasogástrica y manejo de la sepsis. Se recomienda
ANTIBIÓTICOS. que terapia antimicrobiana cubra bacterias entéricas gram negativas, anaerobios, flora oral y
hongos. LOS ANALGÉSICOS NO FORMAN PARTE DEL MANEJO INICIAL EN ESTE CASO. El
tratamiento de la PANCREATITIS AGUDA no complicada es médico y se dirige a restablecer el
equilibrio de líquidos y electrólitos así como evitar la estimulación secretora del páncreas. El
manejo médico varía dependiendo de la literatura consultada y hay controversia en cuanto a
la colocación de la sonda nasogástrica, uso de antibióticos y analgésicos. En la mayoría de los
hospitales se coloca la sonda nasogástrica a excepción de los pacientes con afección leve sin
vómitos o sin dolor importante. Ayuno hasta que desaparezca el íleo o disminuya el dolor. Se
refiere que los antibióticos en las pancreatitis solo deben de utilizarse para tratar
complicaciones supurativas específicas. El tratamiento de estos pacientes con analgésicos se
inicia con dosis pequeñas de medicamentos orales, como derivados de la codeina solos o
combinados con acetaminofén. ESTAS MEDIDAS CORRESPONDEN AL MANEJO INICIAL DE LA
PANCREATITIS.

Bibliografía:
1. GUÍA DE PRÁCTICA CLÍNICA, TRATAMIENTO DE LA PERFORACIÓN DE ÚLCERA PÉPTICA EN PACIENTES
ADULTOS EN EL 2° NIVEL DE ATENCIÓN. MÉXICO: SECRETARIA DE SALUD; 2012. RECUPERADO DE
HTTP://WWW.CENETEC.SALUD.GOB.MX/CONTENIDOS/GPC/CATALOGOMAESTROGPC.HTML 2.
BRUNICARDI F, ANDERSEN D, BILLIAR T, Y COLS. SCHWARTZ PRINCIPIOS DE CIRUGÍA, 9A EDICIÓN. MC
GRAW HILL. 2011, PP 921.

http://www.cenetec.salud.gob.mx/descargas/gpc/CatalogoMaestro/549_GPC_uelcerapepticaperforada/GPCxEYRxLCERAxPEPTICAxPERFORADA.pdf

FIN DEL CASO CLÍNICO SERIADO


ANÁLISIS DEL CASO CLÍNICO

IDENTIFICACIÓN DEL REACTIVO


Area: MEDICINA INTERNA
Especialidad: GERIATRÍA Y GERONTOLOGÍA
Tema: MALTRATO EN EL ADULTO MAYOR
Subtema: MALTRATO EN EL ADULTO MAYOR

CASO CLÍNICO CON UNA PREGUNTA

MUJER DE 68 AÑOS DE EDAD, MÉDICO JUBILADA DEL SEGURO SOCIAL. DURANTE SU VALORACIÓN USTED
ENCUENTRA DATOS RECIENTES DE ABUSO FÍSICO Y EMOCIONAL.

femenina de 68 años (adulta mayor).

jubilada.

-.

datos recientes de abuso físico y


emocional.

-.

98 - EN ESTA PACIENTE EL PROCESO A SEGUIR ES:

HACER LA PAUTA A SEGUIR POR EL MÉDICO FAMILIAR EN LA CONSULTA EXTERNA, CUANDO SE


REPORTE DE SOSPECHA DE ABUSO FÍSICO Y SOCIAL A UNA PERSONA ADULTA MAYOR TANTO POR EL
LESIONES. INTERROGATORIO Y EL EXAMEN CLÍNICO; DEBE DE REALIZARSE EL PARTE DE LESIONES SI
PROCEDE Y COMUNICARLO AL JUEZ, INFORMAR DE LOS PASOS LEGALES PARA FORMALIZAR
DENUNCIAS Y QUE QUEDE REGISTRADO EN LA HISTORIA.
CONFRONTAR Durante la entrevista al cuidador, evitar hacer preguntas que lo hagan sentir acusado y
AL CUIDADOR Y provoquen que se ponga a la defensiva (no culparlo, no amenazarlo y ocupar técnicas de
AMENAZARLO persuasión o negociación). Se deben implementar algunas estrategias e intervenciones de ayuda
CON orientadas a la persona que maltrata, cuando esta es el cuidador o familiar son: 1. Reducir el
DENUNCIARLO. estrés de la familia o del cuidador, a través de medidas como compartir el cuidado con otros
miembros de la familia o facilitar si fuera preciso cambios al domicilio de otro familiar más
capacitado, contactar con grupos de apoyo ( voluntariado, asociaciones de enfermos o de
familiares de enfermos), garantizar períodos de descanso al cuidador, dar apoyo técnico y
emocional al cuidador. 2. Si el cuidador padece de un problema psiquiátrico o de consumo de
alcohol o drogas, debe realizarse entrevista intentando conocer el estado emocional de la
persona y facilitar apoyo y tratamiento de problemas psiquiátricos o toxicomanías si las hubiera.
EL MÉDICO NO DEBE CONFRONTAR AL CUIDADOR, ES MUY IMPORTANTE REALIZAR EL
REPORTE DE LA SOSPECHA DE MALTRATO DE PRIMERA INSTANCIA.

HOSPITALIZAR En los caso de riesgo inmediato, debe remitirse siempre a los servicios especializados y/o
AL PACIENTE Y sociales; el resto se manejará en función de la valoración realizada de mutuo acuerdo
AVISAR A profesional y paciente, respetando siempre que sea posible los deseos de éste. Si el anciano no
TRABAJO está capacitado, se da conocimientos a los servicios sociales y de protección al adulto. Pero no
SOCIAL. hospitalizarlo, a menos que las condiciones clínicas y psicológicas del adulto mayor así lo
ameriten. LA HOSPITALIZACIÓN ESTÁ INDICADA EN PACIENTES QUE AMERITEN URGENCIA
MÉDICA O TENGAN UN RIESGO INMEDIATO.

DERIVARLO A La acción que debe seguir inicialmente el médico, es el de realizar el reporte de lesiones y
UNA CASA DE enviarlo a la autoridad correspondiente, así mismo presentarlo a trabajo social y esperar que la
CUIDADO autoridad determine el destino del adulto mayor. Pero no es el médico quien determina esto.
ESPECIALIZADO. TRAS EL REPORTE SE TOMAR LA DECISIÓN DE ENVIÓ A UN CENTRO ESPECIALIZADO PARA SU
CUIDADO, SEGÚN SEA EL CASO.

Bibliografía:
1. GUÍA DE PRÁCTICA CLÍNICA, DETECCIÓN Y MANEJO DEL MALTRATO EN EL ADULTO MAYOR. MÉXICO:
SECRETARIA DE SALUD, 2013.

http://www.cenetec.salud.gob.mx/descargas/gpc/CatalogoMaestro/057_GPC_MaltratoAdultoMayor/MAM_EVR_CENETEC.pdf
ANÁLISIS DEL CASO CLÍNICO

IDENTIFICACIÓN DEL REACTIVO


Area: MEDICINA INTERNA
Especialidad: NEUMOLOGÍA
Tema: ENFERMEDADES PULMONARES OBSTRUCTIVAS Y RESTRICTIVAS
Subtema: ENFERMEDAD PULMONAR OBSTRUCTIVA CRÓNICA Y
ENFERMEDAD INTERSTICIALES

CASO CLÍNICO CON UNA PREGUNTA

HOMBRE DE 55 AÑOS CON ANTECEDENTE DE EPOC DESDE HACE 2 AÑOS. INGRESA CON RETENCIÓN
URINARIA, SECUNDARIA A HIPERTROFIA PROSTÁTICA.

HOMBRE DE 55 AÑOS.

EPOC DESDE HACE 2 AÑOS.

RETENCIÓN URINARIA, SECUNDARIA A


HIPERTROFIA PROSTÁTICA. muy
importante para considerar el tratamiento
del epoc.

-.

-.

99 - A PARTIR DE ESTE MOMENTO LOS SIGUIENTES MEDICAMENTOS ESTARÁN CONTRAINDICADOS EN


EL PACIENTE:

AGONISTAS B2 DE AGONISTAS B2 DE CORTA ACCIÓN. Un grupo de pacientes con EPOC se benefician con el
CORTA ACCIÓN. uso de este tipo de broncodilatadores principalmente aquéllos con respuesta de más del
12% en el FEV1 después de salbutamol. "Se recomiendan también en aquellos pacientes con
hipertrofia prostática en quienes el uso de anticolinérgicos puede producir retención
urinaria". En pacientes con EPOC de moderado a grave o que no mejoran con el uso de un
solo broncodilatador, se deben recomendar Salbutamol combinado con bromuro de
ipratropio. LOS AGONISTAS B2, SE RECOMIENDAN EN AQUELLOS PACIENTES CON
HIPERTROFIA PROSTÁTICA EN QUIENES EL USO DE ANTICOLINÉRGICOS PUEDE PRODUCIR
RETENCIÓN URINARIA.
ANTICOLINÉRGICOS. ANTICOLINÉRGICOS DE CORTA ACCIÓN. Diversos estudios encaminados a investigar el
mejor tratamiento broncodilatador para pacientes con EPOC, demostraron que los agentes
anticolinérgicos en comparación con los agonistas B2 adrenérgicos de corta acción,
producen un mayor efecto sobre el FEV1, tolerancia al ejercicio y disnea. Este efecto es aún
mayor cuando bromuro de ipratropio se combina con salbutamol. ANTICOLINÉRGICOS DE
LARGA ACCIÓN. Otro agente que antagoniza la acción de la acetilcolina es el bromuro de
tiotropio. A diferencia del tiempo de acción limitado de 4 a 6 hrs del bromuro de ipratropio,
el tiotropio tiene un efecto prolongado de hasta 24 horas. Su efectividad radica en que
permanece sobre los receptores M1 y M3 un tiempo más prolongado, lo cual explica su
potente efecto broncodilatador de acción prolongada. El tiotropio ha probado mejorar la
disnea y la calidad de vida, así como disminuir el número de exacerbaciones y
hospitalizaciones. OJO: SIEMPRE QUE SE SOSPECHE DE HIPERTROFIA PROSTÁTICA
BENIGNA DEBE INTERROGARSE SOBRE LA UTILIZACIÓN DE FÁRMACOS QUE AFECTAN LA
MICCIÓN COMO: ANTICOLINÉRGICOS, ANTIDEPRESIVOS, DIURÉTICOS,
ALFABLOQUEANTES.

B2 AGONISTAS DE Los BRONCODILATADORES AGONISTAS B2 ADRENÉRGICOS de acción prolongada que


ACCIÓN existen son el salmeterol y el formoterol. Su eficacia reside en su vida prolongada (la cual es
PROLONGADA. de 12 horas) y que son más selectivos (estimulan predominantemente los receptores B2). El
efecto de salmeterol y formoterol sobre el FEV1, la disnea y la tolerancia al ejercicio es más
prolongado que el efecto de bromuro de ipratropio. También los B2 agonistas de larga
acción han demostrado tener un efecto sobre la calidad de vida en pacientes con EPOC. LOS
AGONISTAS B2, SE RECOMIENDAN EN AQUELLOS PACIENTES CON HIPERTROFIA
PROSTÁTICA EN QUIENES EL USO DE ANTICOLINÉRGICOS PUEDE PRODUCIR RETENCIÓN
URINARIA.

XANTINAS. XANTINAS Las xantinas son otro tipo de broncodilatadores que relajan el músculo liso del
bronquio al aumentar el AMPC. Otras propiedades farmacológicas interesantes son: La
estimulación de la movilidad ciliar, la disminución de la fatiga diafragmática, la disminución
de la presión arterial pulmonar, la estimulación del músculo cardíaco y del SNC. Sin
embargo, sólo se ha demostrado su utilidad clínica en pacientes con EPOC en combinación
con otros broncodilatadores. Por lo controvertido de los resultados de los estudios dirigidos
a evaluar la utilidad de las xantinas y por sus efectos adversos, las guías para el manejo y
tratamiento de EPOC, sólo recomiendan su uso en pacientes en quienes no mejoran después
del tratamiento por periodo prolongado con la combinación de salbutamol más bromuro de
ipratropio o bien en pacientes con exacerbación. Los niveles séricos de aminofilina deben
ser medidos en forma periódica siempre que se prescriba. De esta forma se podrán alcanzar
concentraciones terapéuticas y evitar efectos colaterales indeseables (8- 12 ng/mL). AL
IGUAL QUE LOS B2 AGONISTAS SUS EFECTOS ADVERSOS SON TAQUIARRITMIAS,
PALPITACIONES, INQUIETUD, TEMBLORES... NO SE RELACIONA CON EFECTOS ADVERSOS A
NIVEL URINARIO.

Bibliografía:
1. LONGO DL, FAUCI AS, KASPER DL, HAUSERSL, JAMESON JL, LOSCALZOJ. HARRISON. PRINCIPIOS DE
MEDICINA INTERNA, 18A EDICIÓN. MC GRAW HILL. NEW YORK, USA. 2012, PP 581. 2. GOLDAN D, TASHJIAN
A, ARMSTRONG E, AMSTRONG A. PRINCIPIO DE FARMACOLOGÍA, 3ª EDICIÓN. WOLTERS KLUWER LWW.
ESPAÑA. 2012.
ANÁLISIS DEL CASO CLÍNICO

IDENTIFICACIÓN DEL REACTIVO


Area: GINECOLOGÍA Y OBSTETRICIA
Especialidad: OBSTETRICIA
Tema: COMPLICACIONES DEL EMBARAZO Y DEL PARTO
Subtema: INDUCCIÓN DEL TRABAJO DE PARTO

CASO CLÍNICO SERIADO

MUJER DE 31 AÑOS, MULTIGESTA, CON EMBARAZO DE 36 SDG POR FUM CONFIABLE. ACUDE AL SERVICIO DE URGENCIAS POR
APARENTE ACTIVIDAD UTERINA. DURANTE LA VALORACION USTED REGISTRA ACTIVIDAD UTERINA REGULAR DE TRES
CONTRACCIONES DOLOROSAS EN MEDIA HORA CON DURACIÓN DE 30 SEGUNDOS. A LA EXPLORACION VAGINAL ENCUENTRA
DILATACION CERVICAL DE 5 CM, CON 90% DE BORRAMIENTO Y ENCUENTRA AL PRODUCTO ENCAJADO.

mujer de 31 años.

36 SDG. POR FUM CONFIABLE.

APARENTE ACTIVIDAD UTERINA.

2 CONTRACCIONES EN MEDIA HR DE 10
SEGUNDOS CADA UNA, MODIFICACIONES
CERVICALES.

-.

100 - PARA CONSIDERAR SI ESTA PACIENTE SE ENCUENTRA EN FASE ACTIVA DE TRABAJO DE PARTO DEBE TOMAR EN CUENTA
PRINCIPALMENTE:

LA DILATACIÓN La fase activa finaliza con dilatación completa (se producen unas subfases en función del ritmo de la dilatación: primero
CERVICAL. aparece una aceleración respecto a la fase de latencia, se pasa de 2 a 4 cm; luego el ritmo se acelera aún más ( fase de
velocidad máxima) pasando de 4 cm a 6 cm, y en la cual baja mucho la presentación; finalmente desde los 9 cm a la
dilatación completa (10 cm), el proceso se enlentece (fase de desaceleración), que en todo caso es la más breve de las tres).
Se puede considerar de manera confiable que la dilatación del cuello uterino de 3 a 5 cm o mayor, en presencia de
contracciones uterinas, es representativo del umbral del trabajo de parto. LA DILATACIÓN CERVICAL ES UN PARÁMETRO
CONFIABLE PARA DETERMINAR LA FASE ACTIVA.

EL El borramiento y reblandecimiento cervical puede iniciar desde algunas horas antes del parto y evoluciona durante todo el
BORRAMIENTO primer periodo del trabajo de parto, es muy variable en cada mujer no es característico en ninguna fase del parto y termina
CERVICAL. invariablemente con la dilatación completa.

FRECUENCIA DE Durante la fase activa las contracciones son más frecuentes (2/10 minutos) y dolorosas. Es una fase de duración variable que
CONTRACCIONES. está en relación con la paridad (es más rápida en las multíparas dilatan 1,5 cm/hora, mientras que las primigestas dilatan 1
cm/hora). LA FRECUENCIA CON LA QUE APARECEN LAS CONTRACCIONES Y LA DILATACIÓN ES UNA MUESTRA CLARA DE
QUE EL TRABAJO DE PARTO SE ENCUENTRA EN FASE ACTIVA.

DURACIÓN DE Las contracciones efectivas deben tener una duración de entre 60-90 segundos y con una frecuencia de 3-4 en 10 minutos,
CONTRACCIONES. la duración de las contracciones no es un indicador confiable ya que durante la fase latente pueden alcanzar los 60 segundos
sin tener la frecuencia necesaria aún para determinar una fase activa.
Bibliografía:
1. GUÍA DE PRÁCTICA CLÍNICA, VIGILANCIA Y MANEJO DEL TRABAJO DE PARTO EN EMBARAZO DE BAJO RIESGO. MÉXICO:
SECRETARIA DE SALUD; 2014. 2. CURRENT DIAGNOSIS & TREATMENT. OBSTETRICS & GYNECOLOGY DE CHERNEY/ NATHAN/
GOODWIN/ LAUFER. MC GRAW-HILL. EDICIÓN 10. 2007. PAG. 388.

http://www.cenetec.salud.gob.mx/descargas/gpc/CatalogoMaestro/052_GPC_VigilanciaManejodelParto/IMSS_052_08_EyR.pdf

101 - UNA VEZ VALORADA LA PACIENTE SE DEBERÁ INDICAR:

INDUCTORES La administración de corticoides entre las 24 y 34 semanas de gestación es un medida eficaz para disminuir la morbi-mortalidad
DE perinatal secundaria a la prematuridad. AL SER UN EMBARAZO DE 36 SDG YA NO ESTA INDICADA LA APLICACIÓN DE
MADURACIÓN INDUCTORES DE MADURACIÓN PULMONAR.
PULMONAR.

ÚTERO Solo se realizará uteroinhibición de ataque durante 48 a 72 horas para el trabajo de parto pretérmino. Los objetivos
INHIBICIÓN. fundamentales de la uteroinhibición son: retardar el parto hasta lograr el efecto de los esteroides en la maduración pulmonar,
permitir el transporte de la materna cuando sea necesario, evitar el parto pretérmino por causas temporales autolimitadas
(ejemplos: píelonefritis, cirugía abdominal, etc.), en las cuales es improbable que causen recurrencia de trabajo de parto
pretérmino. LA UTEROINHIBICIÓN SE RECOMIENDA EN EMBARAZOS MAYORES A 24 Y MENORES DE 36 SEMANAS DE
GESTACIÓN. NO ESTÁ INDICADA EN LA PACIENTE AL ENCONTRARSE EN LA 36 SDG.

CONDUCCIÓN CONDUCCIÓN DEL TRABAJO DE PARTO: es la intervención médica para regularizar la intensidad, frecuencia y duración de las
DEL TRABAJO contracciones uterinas mediante el uso de la oxitocina, con el propósito de completar el trabajo de parto. Este procedimiento
DE PARTO. permite regularizar la dinámica de las contracciones espontáneas que se consideran inadecuadas por falta de avance en la
dilatación del cuello uterino que tiene como objetivo lograr el parto. El medicamento de elección en la conducción del parto es
la oxitocina. DADO QUE LA PACIENTE HA COMENZADO YA TRABAJO DE PARTO ESTÁ INDICADA LA CONDUCCIÓN DEL
MISMO.

CITAR EN UNA La terapia espectante está indicada entre las 26 y 33 sem, se indica reposo, antibióticos e inductores de la maduración
SEMANA. pulmonar. Con más de 36 semanas siempre se indica la inducción después de un período de observación de 12 a 24 hr., y se
busca la posibilidad del trabajo de parto espontáneo, lo que ocurre en más de 60 % de los casos. EN LA SEMANA 36 SDG NO
ESTÁ INDICADA LA TERAPIA ESPECTANTE, CITAR EN 1 SEMANA A LA PACIENTE AUMENTA EL RIESGO DE ÓBITO FETAL.

Bibliografía:
1. GUÍA DE PRÁCTICA CLÍNICA, INDUCCIÓN DEL TRABAJO DE PARTO EN EL SEGUNDO NIVEL DE ATENCIÓN. MÉXICO: SECRETARIA DE
SALUD; MARZO 2013. 2. GUÍA DE PRÁCTICA CLÍNICA, VIGILANCIA Y MANEJO DEL TRABAJO DE PARTO EN EMBARAZO DE BAJO
RIESGO. MÉXICO: SECRETARIA DE SALUD; 2014.
HTTP://WWW.CENETEC.SALUD.GOB.MX/DESCARGAS/GPC/CATALOGOMAESTRO/052_GPC_VIGILANCIAMANEJODELPARTO/IMSS_052_08_EYR.PDF3.
CUNNINGHAM G, LEVENO K, BLOMM S, HAUTH J, RPUSE D, SONG C. WILLIAMS OBSTETRICIA, 23A EDICIÓN. MC GRAW HILL. USA. 2011
EN ESPAÑOL, PP 501-502.

http://www.cenetec.salud.gob.mx/descargas/gpc/CatalogoMaestro/218_SSA_09_TDP/GPC_218-09_Induccion_Trabajo_de_PartoEVR.pdf

FIN DEL CASO CLÍNICO SERIADO


ANÁLISIS DEL CASO CLÍNICO

IDENTIFICACIÓN DEL REACTIVO


Area: PEDIATRÍA
Especialidad: INFECTOLOGIA PEDIÁTRICA
Tema: HEPATITIS INFECCIOSA
Subtema: HEPATITIS A

CASO CLÍNICO CON UNA PREGUNTA

ESCOLAR DE 8 AÑOS DE EDAD, CON ANTECEDENTE DE CONSUMO DE ALIMENTOS INSALUBRES


SOSPECHOSOS HACE 2 SEMANAS. ES LLEVADO A CONSULTA PORQUE DESDE HACE 3 DÍAS PRESENTA FIEBRE
EN PICOS, ASTENIA Y ADINAMIA. EL DÍA DE HOY SE AGREGA DOLOR ABDOMINAL INTENSO, ICTERICIA Y
VÓMITOS.

8 años.

consumo de alimentos insalubres


sospechosos hace 2 semanas.

ebre en picos, astenia, adinamia, dolor


abdominal intenso, ictericia y vómitos.

pble hepatitis A.

102 - LA SIGUIENTE MEDIDA PROFILÁCTICA CONFIERE INMUNIDAD ACTIVA PARA EVITAR EL CONTAGIO
EN LA FAMILIA:
VACUNA ANTI-VHA DE LO PRIMERO EN ESTE CASO SERÁ: 1. Reconocer cuál de las respuestas corresponden
VIRUS INACTIVOS específicamente a una medida profiláctica que confieren INMUNIDAD ACTIVA
(VACUNAS). Se denomina INMUNIDAD ACTIVA cuando el propio organismo crea sus
anticuerpos, existen dos formas de adquirir inmunidad activa. - INMUNIDAD ACTIVA
NATURAL: se genera por estimulación directa del sistema inmunológico del individuo
ante la presencia de la enfermedad. - INMUNIDAD ACTIVA ARTIFICIAL: se genera por la
sensibilización del sistema inmunológico mediante la introducción de microorganismos
atenuados, inactivos o sus fracciones conocidas como vacunas. 2. Determinar el
diagnóstico probable: HEPATITIS A. HEPATITIS A. De manera característica, la infección
por el virus de hepatitis A (Hepatitis A virus, HAV) incluye un cuadro agudo que cede por
sí solo, compuesto de fiebre, malestar generalizado, ictericia, anorexia y náuseas. La
infección sintomática por dicho virus se observa en cerca de 30% de niños infectados que
tienen menos de seis años de vida; pocos de ellos mostrarán ictericia. En niños de mayor
edad y adultos, la infección casi siempre es sintomática y se prolonga algunas semanas;
en 70% de los casos, en promedio, aparece ictericia. Se observan a veces casos de
enfermedad prolongada o recidivante que puede durar incluso seis meses. La hepatitis
fulminante es rara, pero es más frecuente en sujetos con alguna hepatopatía primaria; no
se observa infección crónica. MEDIDAS PROFILÁCTICAS PARA HEPATITIS A. Los mejores
métodos para evitar infecciones por virus de hepatitis A (HAV) son la mejoría en la
sanidad (p. Ej., en los abastos de agua y en la preparación de alimentos) y en la higiene
personal (p. Ej., lavarse perfectamente las manos después de cambiar pañales en centros
pediátricos), vacunación contra hepatitis A, y aplicación del concentrado
inmunoglobulínico (Ig) en escuelas, centros pediátricos y labores. Es importante aislar
temporalmente durante una semana de haber comenzado la enfermedad, a los niños y a
adultos con infección aguda por HAV que trabajan en el manejo de alimentos o que
atienden o trabajan en centros de atención pediátrica. PROFILAXIS EN CONTACTOS: tanto
la Inmunoglobulina G como la vacuna contra la VHA han demostrado eficacia para la
prevención post-exposición de la infección por VHA siempre y cuando se apliquen en las
dos primeras semanas posteriores al inicio de los síntomas o del contacto con el caso
índice. OJO: en este caso para elegir la respuesta correcta debes tener en cuenta lo que la
pregunta solicita “inmunidad activa”, es por eso que se elige a la vacuna como respuesta
correcta. VACUNA DE HEPATITIS A. Es una preparación purificada del virus de la hepatitis
A “inactivado”, se preparan a partir de virus adaptado en cultivo celular, propagado en
fibroblastos humanos y se le purifica de lisados celulares, se inactiva en formol y se
adsorbe a un coadyuvante de hidróxido de aluminio. En Estados Unidos de América, se ha
aprobado el uso de vacunas contra hepatitis para niños de dos años de edad y mayores;
hay presentaciones para niños y adultos que se administran en un plan posológico de dos
dosis. Las presentaciones para adultos son recomendables para jóvenes de 19 años y
personas mayores. Las vacunas aprobadas se aplican por vía intramuscular.

CONCENTRADO LAS INMUNOGLOBULINAS CONFIEREN INMUNIDAD “PASIVA” Y NO “ACTIVA” COMO LO


INMUNOGLOBULÍNICO SOLICITA LA PREGUNTA. La INMUNIDAD PASIVA consiste en la transferencia de los
anticuerpos ya formados. Comprende todos los productos de la sangre, además de otros
productos como inmunoglobulina intravenosa, productos del plasma, etc. Al igual que la
inmunidad activa, la inmunidad pasiva se puede adquirir de manera natural o artificial. -
INMUNIDAD PASIVA NATURAL: se adquiere durante el embarazo, a través del paso
placentario de IgG y durante la lactancia, a través del calostro. - INMUNIDAD PASIVA
ARTIFICIAL: es la transferencia de anticuerpos ya formados de un individuo a otro de la
misma o distinta especie, cuya duración es de aproximadamente tres meses. Los
principalmente usados son: inmunoglobulina, hiper-inmunoglobulina humana homóloga
y los faboterápicos. CONCENTRADO INMUNOGLOBULÍNICO. El concentrado
inmunoglobulínico por aplicación intramuscular, si se administrar en término de dos
semanas de exposición a HAV tiene una eficacia superior a 85% para evitar la infección
sintomática. IMPORTANTE. La protección con la gammaglobulina es inmediata, mientras
que la vacuna tarda de 2 a 4 semanas en proveer protección.

INMUNOGLOBULINA G LAS INMUNOGLOBULINAS CONFIEREN INMUNIDAD “PASIVA” Y NO “ACTIVA” COMO LO


CONTRA VHA HIPER- SOLICITA LA PREGUNTA. Los principalmente usados son: inmunoglobulina, hiper-
INMUNE inmunoglobulina humana homóloga y los faboterápicos. En el caso de la hepatitis A está
indicada la inmunoglobulina G anti-VHA.
VACUNA DE LA VACUNA PARA LA PREVENCIÓN DE LA ENFERMEDAD DE LA HEPATITIS A ESTÁ HECHA
ANTÍGENOS DE CON VHA INACTIVADOS. La vacuna contra la hepatitis que contiene antígeno de
SUPERFICIE DEL VHA superficie del virus es contra la HEPATITIS B (AgsHB), el cual es obtenido y purificado por
tecnología de ADN recombinante.

Bibliografía:
RUDOLPH'S PEDIATRICS. ABRAHAM M. RUDOLPH. APPLETON AND LANGE. EDICIÓN 20A. 1996. PÁG. 648.
MANUAL DE VACUNACIÓN. CENTRO NACIONAL PARA LA SALUD DE LA INFANCIA Y ADOLESCENCIA. 24 DE
NOVIEMBRE DE 2017.

https://www.gob.mx/salud%7Ccensia/documentos/manual-de-vacunacion-edicion-2017
ANÁLISIS DEL CASO CLÍNICO

IDENTIFICACIÓN DEL REACTIVO


Area: PEDIATRÍA
Especialidad: URGENCIAS PEDIÁTRICAS
Tema: MALFORMACIONES CONGÉNITAS INTESTINALES QUE REQUIEREN
ATENCIÓN DE URGENCIA
Subtema: ENFERMEDAD DE HIRSCHSPRUNG

CASO CLÍNICO SERIADO

PREESCOLAR DE 5 AÑOS CON ENCOPRESIS, MANCHADO FECAL EN SU ROPA INTERIOR Y SANGRADO AL


FINAL CADA EVACUACION, REFIERE INGESTA DE MAS DE 1 LITRO DE LECHE DIARIO. A LA EXPLORACIÓN SE
ENCUENTRA BIEN HIDRATADA, ABDOMEN TIMPÁNICO, BLANDO, DEPRESIBLE, PERISTALSIS NORMAL, CON
DOLOR A LA PALPACIÓN MEDIA Y PROFUNDA EN TODO MARCO CÓLICO DE PREDOMINIO IZQUIERDO. TACTO
RECTAL CON ÁMPULA LLENA.

preescolar de 5 años de edaD

--

presenta encopresis, con manchado fecal


en su ropa interior y sangrado al nal de
cada evacuación. Ingesta de 1 litro de leche

se encuentra con adecuado estado de


hidratación, abdomen blando, depresible,
peristalsis normal, timpánico,, con dolor a la
palpación media y profunda en todo el
marco cólico con predominio del lado
izquierdo. tacto rectal con ámpula llena

---

103 - EL DIAGNÓSTICO MÁS PROBABLE DE LA PACIENTE ES:


PSEUDOOBSTRUCCIÓN La PSEUDOOBSTRUCCIÓN INTESTINAL es un síndrome clínico, relativamente infrecuente,
INTESTINAL caracterizado por episodios recurrentes de signos y síntomas de obstrucción intestinal sin
patología estructural evidenciable. La pseudoobstrucción puede afectar a cualquier
segmento del tracto gastrointestinal y la clínica variará dependiendo de la zona afectada.
Cuando afecta exclusivamente al colon se denomina pseudoobstrucción de colon, inercia
colónica, y se manifiesta fundamentalmente por estreñimiento refractario al tratamiento
farmacológico. La característica clínica fundamental es la presencia de crisis suboclusivas.
Las crisis suboclusivas clínicamente se manifiestan por dolor abdominal de tipo cólico con
distensión abdominal acompañado generalmente de náuseas y vómitos y ausencia de
emisión de heces y gases. En la exploración física destaca un abdomen distendido,
timpánico a la percusión, difusamente doloroso a la palpación sin signos de peritonismo y
con ruidos intestinales aumentados, ausentes o disminuidos. En una RX de abdomen
simple aparecerán asas de intestino delgado o delgado y grueso dilatadas con niveles
hidroaéreos dependiendo de la localización de la oclusión y con ausencia de aire en la
porción distal a la misma. En los períodos de intercrisis de la pseudoobstrucción intestinal
la Rx de abdomen puede ser estrictamente normal.

ESPINA BÍFIDA ESPINA BÍFIDA OCULTA, es un disrafismo de las estructuras óseas debido al cierre
OCULTA incompleto de los arcos vertebrales. La malformación más sutil, limitada al defecto
vertebral, se denomina espina bífida oculta. Cuando a esta anomalía se asocian
malformaciones subyacentes de la médula espinal sin discontinuidad de la piel, el
complejo malformativo se designa como disrafismo espinal oculto. Este término incluye
un amplio espectro clínico de malformaciones, a saber: quistes dermoides o
epidermoides, quistes entéricos intraespinales, lipomas lumbosacros, la diastematomielia,
el mielocistocele terminal y el síndrome de la médula anclada, que es la anomalía más
frecuente. Acontece en al menos un 5% de la población. Es asintomática, se localiza
habitualmente a nivel lumbosacro, y se pone de manifiesto mediante una radiografía de
columna en la que se evidencia el cierre incompleto del arco vertebral posterior. No tiene
relación con la enuresis nocturna monosintomática; sin embargo, cuando la enuresis es
polisintomática se deben excluir alteraciones subyacentes de la médula espinal propias
del disrafismo espinal oculto. En estos casos, suelen observarse a nivel lumbosacro
alteraciones cutáneas, tales como áreas de piel atrófica o hiperpigmentada,
hemangiomas, lipomas subcutáneos, o bien hipertricosis localizada, apéndices cutáneos,
fístulas o senos dérmicos. El disrafismo espinal oculto puede ser asintomático o bien se
puede sospechar por la presencia de manifestaciones clínicas comunes más o menos
evidentes: cifoscoliosis, lumbociática, pies equinovaros, asimetría de las extremidades
inferiores, signo de Babinski, pérdida parcheada de la sensibilidad, úlceras tróficas y
trastornos de los esfínteres. En ocasiones, cuando la malformación oculta está en
comunicación con el exterior a través de fístulas o senos dérmicos puede ser la causa de
meningitis recurrente. Mediante la ultrasonografía, la TAC y la RMI se pueden deducir el
nivel de la lesión y su relación con el tejido funcional. Las pruebas preoperatorias
pertinentes en estos casos son el EMG, los PESS y la urodinámica. Considerando que en
estos pacientes no se puede predecir la evolución, los neurocirujanos pediátricos, en la
actualidad, son partidarios de la intervención profiláctica precoz, para evitar alteraciones
irreparables, liberando la médula espinal de la lesión donde esté anclada. La cirugía se
lleva a cabo utilizando láser de CO2.

ENFERMEDAD DE La ENFERMEDAD DE HIRSCHSPRUG (EH) es considerada una enfermedad congénita


HIRSCHSPRUG caracterizada por una ausencia de células ganglionares en el plexo mientérico de
Auerbach y en el submucoso de Meissner, en el recto y otros segmentos del colon en
forma ascendente (de caudal a cefálico). Esta alteración produce una anormalidad de la
motilidad intestinal, que se manifiesta más frecuentemente como una obstrucción
intestinal La EH puede ser clasificada según el segmento intestinal comprometido. Así
puede dividirse en: 1.- Segmento corto: cuando no compromete más allá de la unión
rectosigmoidea; 2.- Ultracorto: si sólo afecta esfínter interno o algunos centímetros
próximos a dicho esfínter; 3.- Segmento largo cuando el segmento agangliónico afecta
más allá de la unión rectosigmoidea. La EH forma parte de los trastornos conocidos como
disganglionismos que incluyen también el hipoganglionismo y la displasia neuronal
intestinal. La incidencia varía según la etnia, estimándose en 1.5 por 10.000 nacidos vivos
en población caucásica, 2.1 por 10.000 nacidos vivos en afroamericanos, 1.0 por 10.000
nacidos vivos en hispanos y 2.8 por 100.000 nacidos vivos en asiáticos. Es más frecuente
en raza blanca y en recién nacidos de término. En hermanos la incidencia es de
aproximadamente del 3.5%, aumentando según la longitud del segmento afectado hasta
un 20%. La EH predomina en varones en una relación de 3-5:1. Esta relación disminuye a
2:1 cuando el segmento afectado es más largo. El 70% de los casos de EH es un defecto
aislado, constituyéndose en un diagnóstico único. Los casos restantes pueden ser
asociados a otras patologías. Un ejemplo de ellos es la asociación con Síndrome de Down,
siendo 10 veces más frecuente en estos pacientes el hallazgo de una aganglionosis
congénita. Otras anomalías no congénitas que también se han asociado son: hidrocefalia,
defecto septal ventricular, agenesia renal, ano imperforado, divertículo de Meckel,
poliposis colónica, criptorquidia, epilepsia y resistencia a la insulina. La EH es producto de
una falla en la migración craneocaudal de las células ganglionares provenientes de la
cresta neural, fenómeno producido entre las 5º y la 12º semana de gestación. Mientras
más precoz se produzca la alteración en la migración celular, mayor será la longitud del
segmento afectado. También se cree que podría estar implicado un defecto en la matriz
extracelular de la pared intestinal que impediría que las células procedentes de la cresta
neural la colonizaran Dentro de los síntomas que permiten una sospecha precoz, se
encuentra el estreñimiento o constipación, definida en el recién nacido como el retraso en
la eliminación de meconio mayor a 48 horas asociada a distensión abdominal, y en los
niños mayores como deposiciones infrecuentes de consistencia aumentada. El 98% de los
lactantes elimina el meconio en las primeras 48 horas de vida. Los prematuros eliminan
más tardíamente el meconio, pero la EH es rara en prematuros. De los pacientes con EH,
sólo el 60% elimina el meconio después de las 48 horas, por lo que este signo no es
patognomónico de la enfermedad. La mayoría de los niños que presentan aganglionosis
congénita, son sintomáticos los primeros días o las primeras semanas luego del
nacimiento. Alrededor de dos tercios de los pacientes presenta síntomas dentro de los
tres primeros meses de vida y 80% desarrolla síntomas dentro del primer año de vida.
Sólo un 10% de los pacientes inicia síntomas entre los 3 y 14 años de edad y en general se
trata de pacientes con enfermedad de segmento ultracorto. Los recién nacidos y lactantes
pequeños presentan con frecuencia signos de obstrucción intestinal, distensión
abdominal, vómitos biliosos e intolerancia a la alimentación. La inspección anal y la
radiografía pueden orientarnos hacia una causa mecánica de obstrucción, pero no
descarta EH. Si la obstrucción no tiene una causa mecánica, además de pensar en una EH,
debe plantearse el diagnóstico diferencial con hipotiroidismo, insuficiencia suprarrenal,
hipokalemia, hipercalcemia, hipomagnesemia, y en casos excepcionales alteraciones
neuromusculares. Cuando la sintomatología es poco evidente, puede presentarse como
un cuadro de constipación crónica, con historia de dificultad en la eliminación de
deposiciones, masas fecales palpables en fosa ilíaca izquierda y un tacto rectal en que no
se encuentran deposiciones en la ampolla rectal y esfínter anal hipertónico. En muchas
ocasiones la estimulación rectal provoca salida explosiva de heces líquidas de olor fétido.
Por lo tanto, frente a pacientes con constipación crónica, en los cuales se ha descartado
causa mecánica de obstrucción intestinal, que no cede a las medidas dietéticas ni
farmacológicas, debe plantearse el diagnóstico de EH. También puede encontrarse
dilatación de asas intestinales, adelgazamiento de la pared abdominal, alteraciones de la
nutrición y el crecimiento. En niños mayores, los síntomas más comunes incluyen
constipación crónica progresiva, impactación fecal recurrente, mal incremento ponderal y
malnutrición La primera orientación diagnóstica está dada por la radiología, la que puede
ser de gran ayuda al dar un primer acercamiento al diagnóstico. Se solicita radiografía
abdominal anteroposterior y lateral, que muestran distribución normal del aire intestinal,
recto vacío y a veces signos de obstrucción (distensión del colon proximal a la
obstrucción que se identifica por las haustras y ausencia de gas distal a la obstrucción). El
enema contrastado ha demostrado un Generalmente las anormalidades en el enema
contrastado motivan la solicitud de biopsia de recto para un diagnóstico definitivo La
manometría anorrectal consiste en la distensión rectal mediante un balón con presión
controlada, y la posterior medición de los cambios de presión provocados en el esfínter
anal externo e interno.
CONSTIPACIÓN La constipación funcional representa el 97% de los casos pediátricos. Ha sido definida
FUNCIONAL recientemente por un grupo de expertos en los criterios de Roma III, según se describe a
continuación. La constipación funcional en niños menores de cuatro años requiere la
presencia de 1 dos o más de las siguientes manifestaciones durante al menos un mes: 1)
dos o menos evacuaciones por semana, 2) “al menos un episodio semanal de
incontinencia fecal después del control de esfínteres”, 3) postura retencionista o
retención voluntaria de las evacuaciones, 4) dolor abdominal cólico, 5) “masa fecal en
recto”, 6) heces de gran diámetro que pueden obstruir el inodoro. Los síntomas
acompañantes como irritabilidad, disminución del apetito o saciedad temprana,
desaparecen luego de evacuar el intestino. Interrogatorio: Debe incluir información
acerca del momento en que se presentó la primera evacuación meconial, tiempo de
evolución del trastorno, patrón de las evacuaciones, sangrado, manchado fecal, consumo
de medicamentos (codeína, diuréticos, metilfenidato, fenitoína, clorhidrato de
imipramina, antiácidos), traumatismos. Exploración física: Debe abarcar el hábito externo,
peso y talla; abdomen; región lumbosacra; examen neurológico: tono y fuerza muscular,
reflejos osteotendinosos, cremasteriano, anal y cutáneo abdominales; tiroides; ano. Tacto
rectal: debe realizarse a todo lactante con constipación para descartar alteraciones
anatómicas y buscar datos que orienten a enfermedad de Hirschsprung tales como
evacuación explosiva al retirar el dedo o ámpula rectal vacía. Signos de alarma: Fiebre,
distensión abdominal, vómito, náusea, enterocolitis del lactante, pérdida de peso, escaso
crecimiento pondoestatural, primera evacuación luego de las 48 horas de vida, heces de
pequeño diámetro, ámpula rectal vacía, estenosis anal o aumento del tono esfinteriano,
infecciones urinarias a repetición. ? Estudios auxiliares Radiografía de abdomen: Permite
diferenciar entre incontinencia fecal retencionista y no retencionista. Enema de bario:
Indicado ante la sospecha de malformaciones, estenosis colónica o enfermedad de
Hirschsprung. Debe realizarse sin preparación. La ausencia de zona de transición no
descarta enfermedad de Hirschsprung. Resonancia magnética de columna vertebral: Para
pacientes con alteraciones en la exploración neurológica, que sugieran compromiso
medular. Tránsito colónico: Diferencia inercia colónica de obstrucción a la salida.
Manometría ano-rectal: Útil para diferenciar constipación funcional de enfermedad de
Hirschs-prung (ausencia del reflejo recto-anal inhibitorio). En prematuros es
recomendable realizar directamente la biopsia rectal. La ausencia del reflejo recto-anal
inhibitorio puede deberse a acalasia del esfínter anal interno (biopsia con células
ganglionares presentes). La manometría ano-rectal permite diagnosticar anismo
(contracción paradójica). Manometría colónica: Permite diferenciar retención fecal
funcional de pseudo-obstrucción intestinal. Biopsia rectal: Estándar de oro para
diagnosticar enfermedad de Hirschsprung (células ganglionares ausentes y troncos
nerviosos hipertróficos). LA PRESENCIA DE ÁMPULA RECTAL JUSTIFICA EL DIAGNÓSTICO
DE CONSTIPACIÓN FUNCIONAL.

Bibliografía:
1. LA SALUD DEL NIÑO Y ADOLESCENTE. MARTÍNEZ Y MARTÍNEZ. MANUAL MODERNO. EDICIÓN 4TA. 2001.
PAG.1630-1634.

104 - LA FISIOPATOLOGÍA DE ESTE PADECIMIENTO ES POR:

LESIONES Las alteraciones de la inducción dorsal se deben al fracaso de la fusión del canal neural,
ESPINALES persistiendo, en consecuencia, la continuidad entre el neuroectodermo posterior y el
ectodermo cutáneo, que en su forma completa se denomina disrafismo. El término "disrafismo
espinal" comprende un grupo heterogéneo de MALFORMACIONES CONGÉNITAS DE LA
MÉDULA ESPINAL QUE SE CARACTERIZA POR LA FUSIÓN IMPERFECTA DE LAS ESTRUCTURAS
NEURALES, ÓSEAS Y MESENQUIMATOSAS DE LA LÍNEA MEDIA.
ALTERACIÓN La pseudoobstrucción intestinal es uno de los trastornos en la motilidad o propulsivos de
FUNCIONAL origen nervioso o muscular es la pseudoobstrucción intestinal crónica, esta entidad infrecuente
MOTORA y grave, se caracteriza por episodios repetitivos o continuos de obstrucción intestinal, en
INTESTINAL ausencia de lesiones obstructivas. El motivo es una alteración funcional motora intestinal en
diversas zonas, del aparato digestivo. La pseudoobstrucción crónica intestinal (SCI) puede
clasificarse en dos tipos: 1.-Miopatía visceral: en la que la alteración reside en la musculatura
lisa del intestino. Se caracteriza por presentar cambios que consisten en degeneración de las
células musculares lisas y reemplazamiento por tejido fibroso. La consecuencia es una
contractilidad ordenada, pero débil e inefectiva. Puede asociarse a afectación de la
musculatura lisa de otras localizaciones. 2.-Neuropatía visceral: engloba procesos que pueden
afectar a cualquier eslabón de la regulación nerviosa del intestino desde el plexo mientérico al
sistema nervioso central. La neuropatía visceral se puede limitar al tracto gastrointestinal o
asociarse a alteraciones en el sistema nervioso central o autónomo. Se caracteriza por
presentar unas contracciones eficaces, pero con un patrón desorganizado y no propulsivo.
Cuando afecta al sistema nervioso entérico se caracteriza por cambios degenerativos con
alteraciones en el número o en la morfología de las neuronas del plexo mientérico rodeadas o
no de infiltrado inflamatorio. Tanto las miopatías como las neuropatías pueden dividirse, en
cuanto al tipo de presentación, en familiares o esporádicas, y, en cuanto a la etiología, en
primarias, si la alteración se limita a la musculatura intestinal o al plexo nervioso entérico, o
secundarias, si el trastorno gastrointestinal forma parte de un proceso sistémico. En ocasiones
pueden ser segmentarias, como en los casos de megaduodeno o de dilatación cólica aguda o
síndrome de Ogilvie.

ESFUERZO O EL ESTREÑIMIENTO ES EL RESULTADO CON FRECUENCIA DE INTENTOS REPETIDOS DE


RETENCIÓN DE RETENCIÓN DE LAS HECES, por lo que cuanto antes se realice el diagnóstico y se inicie
HECES tratamiento, mejor es el pronóstico. Niños que han estado estreñidos durante años pueden
haber tenido comportamientos retentivos durante mucho tiempo y cuando son evaluados, el
recto está dilatado y acomodado hasta tal punto que no es necesario ningún comportamiento
retentivo para retrasar el paso de las heces La incontinencia fecal retentiva es una de las formas
más comunes de presentación del estreñimiento funcional y se encuentra en el 84% de los
casos. Puede emplearse como un marcador objetivo de severidad y monitorización de la
eficacia del tratamiento.

AUSENCIA DE Es una enfermedad congénita caracterizada por una ausencia de células ganglionares en el
LAS CÉLULAS plexo mientérico de Auerbach y en el submucoso de Meissner, en el recto y otros segmentos
AGANGLIONARES del colon en forma ascendente (de caudal a cefálico).

Bibliografía:
1. LA SALUD DEL NIÑO Y ADOLESCENTE. MARTÍNEZ Y MARTÍNEZ. MANUAL MODERNO. EDICIÓN 4TA. 2001.
PAG. 833-837.

105 - EL TRATAMIENTO INCIAL QUE DEBE RECIBIR LA PACIENTE ES CON:

EXTIRPACIÓN La operación consiste en la extirpación de los tejidos anómalos (lipoma, dermoide, bandas
DE LOS fibrosas), para dejar libre la médula y las raíces nerviosas. La disección de los tejidos se realiza
TEJIDOS con microscopio quirúrgico, y a veces con monitorización intraoperatoria de la función nerviosa
ANÓMALOS Y (electromiografía, potenciales evocados), técnicas que disminuyen la posibilidad de añadir un
CIERRE DE LAS daño neurológico durante la intervención. El cierre de las cubiertas meníngeas se realiza de
CUBIERTAS manera impermeable para evitar que gotee líquido cefalorraquídeo y pueda producirse una
MENÍNGEAS. meningitis postoperatoria. Igualmente es muy importante reponer el hueso que se levanta
(laminotomía osteoplástica) y recubrir el defecto con músculo y piel sana. CORRESPONDE AL
TRATAMIENTO DE LA ESPINA BÍFIDA.

TRATAMIENTO El tratamiento de la EH es quirúrgico y está orientado a la remoción del segmento agangliónico,


QUIRÚRGICO y anastomosar el segmento normal proximal con el recto distal o canal anal. EL TRATAMIENTO
QUIRÚRGICO PARA LA CONSTIPACIÓN FUNCIONAL SE UTILIZA EN CASOS EXCEPCIONALES, NO
ES DE PRIMERA LÍNEA.
MEJORAR TRATAMIENTO CONSTIPACIÓN FUNCIONAL 1) Desimpactación: representa el primer paso en el
HÁBITOS paciente con impactación fecal. Si bien el polietilenglicol (peg) con electrolitos es útil, el produc-
HIGIÉNICOS Y to sin electrolitos es mejor aceptado por carecer de sabor; dosis: 1.5 g/kg/día. Los enemas
DIETÉTICOS evacuantes son eficaces; las soluciones de fosfatos pueden provocar hiperfosfatemia e
hipocalcemia grave, sobre todo en niños pequeños. En ciertos casos, será conveniente limpiar el
colon mediante la administración de peg por vía oral o por sonda nasogástrica (30-40 ml/kg/
hora cada seis a ocho horas). La desimpactación manual deberá realizarse bajo sedación o
anestesia general. 2) Mantenimiento: el objetivo es garantizar un patrón normal de evacuaciones.
Es imprescindible el uso de laxantes durante un tiempo mínimo de seis meses. A) Laxantes
osmóticos: preferidos por producir menos efectos adversos. • Peg sin electrolitos: eficaz, inocuo y
bien aceptado, aún en lactantes. Más eficaz que la lactulosa e igual que la leche de magnesia
pero mejor aceptado.dosis: 0.5-0.8 mg/kg/día. • Lactulosa: disacárido artificial. Dosis: 2-3
ml/kg/día. • Leche de magnesia: se absorbe el 20%.en pacientes con insuficiencia renal
incrementa el riesgo de hipermagnesemia, hipofosfatemia e hipocalcemia. Dosis: 1-3 ml/kg/día.
B) Laxantes estimulantes: la evidencia acerca de su eficacia es insuficiente. El consumo
prolongado de senósidos se asocia a pérdida de la inervación intrínseca y atrofia de la capa
muscular lisa. Se ha reportado también melanosis coli, hepatitis idiosincrática y osteoartropatía
hipertrófica. C) Laxantes lubricantes: aceite mineral, vaselina líquida, glicerina • No
recomendados en lactantes, niños con daño neurológico o vómitos, por riesgo de aspiración.
Dosis: 1-3 ml/ kg/día. D) Otros medicamentos: • Probióticos: las evidencias son insuficientes •
Cisaprida: es eficaz pero poco disponible por sus efectos adversos. • Fibra comercial: psyllium
plantago: evidencias insuficientes. Puede utilizarse en mayores de cuatro años. Dosis: 4-6 años: 9-
11 g/día; 7-10 años: 12-15 g/día; 11-14 años: 16-19 g/día. 3) Modificaciones dietéticas la
alimentación deberá ser adecuada para edad con fibra proveniente de frutas, verduras, cereales y
leguminosas. 4) Biorretroalimentación: se apoya en la manometría anorectal y la
electromiografía. Útil para enseñar la mecánica correcta de la defecación. 5) Tratamiento
quirúrgico: reservado para casos graves de constipación funcional, refractarios a múltiples
tratamientos. Debe ir precedido de estudios de motilidad ano-rectal y de ser posible, colónica.
Las modalidades dependen de la extensión del trastorno funcional: colectomía subtotal,
colectomía total o pull-through. Los pacientes con aca-lasia del esfínter anal interno, pueden
beneficiarse de la inyección de toxina botulínica. En algunos casos, la realización de enemas
anterógrados mediante cecostomía ayuda a mantener el intestino limpio. 6) Terapia emocional:
recomendada para casos con escasa adhesión al tratamiento y niños con problemas emocionales.
APOYO El tratamiento de los pacientes con pseudoobstrucción intestinal (SCI) incluye lo siguiente.
PSICOLÓGICO , Tratamiento etiológico En los casos de SCI es posible cuando ésta es secundaria, pero pocas veces
LAXANTES Y se obtiene una mejoría sustancial del cuadro abdominal. Medidas de soporte Los pacientes con
ERITROMICINA. SCI pueden presentar un amplio espectro en cuanto a la gravedad de los síntomas, y también
episodios repetidos de su enfermedad. Durante el episodio agudo se mantendrá el paciente con:
Dieta absoluta. Aspiración nasogástrica o nasoentérica para descomprimir el tubo digestivo.
Corrección de los trastornos electrolíticos. Medicación procinética (véase más adelante).
Nutrición parenteral total en los casos en que exista desnutrición grave u oclusión prolongada.
Tratamiento antibiótico por vía oral en los casos en que se compruebe la existencia de
sobrecrecimiento bacteriano. Tratamiento analgésico, evitando medicación opiácea. En los
períodos de remisión el paciente requerirá: Soporte nutricional: en los casos con síntomas leves
se limitarán a modificaciones dietéticas sencillas, como tomar dietas fraccionadas pobres en
grasas y en fibra y con suplementos de hierro, folatos, calcio y vitaminas. En ocasiones se
requerirán suplementos de dietas en forma líquida. Cuando el estómago es un obstáculo para
una alimentación normal por la presencia de náuseas y vómitos se puede considerar la
posibilidad de realizar nutrición enteral en intestino delgado a través de una sonda de
yeyunostomia de implantación quirúrgica. Cuando la nutrición enteral no es bien tolerada o
insuficiente se puede recurrir a nutrición parenteral domiciliaria o a pautas de nutrición
parenteral hospitalaria en los períodos de mayor desnutrición. La nutrición parenteral a largo
término puede presentar complicaciones graves, como sepsis, trombosis de accesos venosos y
complicaciones hepáticas, particularmente en niños. Sobrecrecimiento bacteriano: cuando existe
sobrecrecimiento secundario la estasis crónica intestinal se tratará con antibióticos de amplio
espectro por vía oral fundamentalmente: tetraciclinas (500 mg/12 h), metronidazol (500 mg/8 h),
ciprofloxacino (500 mg/12 h) o amoxicilina-ácido clavulánico (500-125 mg/12 h). En los casos de
sobrecrecimiento bacteriano crónico se realizarán tratamientos con pautas cíclicas de 7-10 días al
mes con antibióticos de forma rotatoria. Tratamiento farmacológico: Se utiliza para mejorar la
capacidad contráctil del tubo digestivo: Cisaprida: actúa como agonista de los receptores 5HT4,
que facilitan la liberación de acetilcolina en el sistema nervioso entérico y estimulan la regulación
de la contracción. Se administra a dosis de 10-20 mg, 4 veces al día, preferiblemente antes de las
comidas y a la hora de acostarse. Eritromicina: es un antibiótico macrólido que actúa como
agonista de la motilina, induce fases III del complejo motor interdigestivo y acorta el tiempo de
tránsito orocecal. Resulta eficaz en el tratamiento agudo de la gastroparesia, pero el beneficio en
el tratamiento en la SCI no ha sido probado. Octreótido: análogo de la somatostatina de acción
sostenida. Tiene dos posibles utilidades: a) induce frentes de actividad en el intestino delgado,
por lo que tiene efecto beneficioso en la propulsión, mejorando también el sobrecrecimiento
bacteriano. Se administra a dosis de 50 g s.c. por la noche, y b) como analgésico visceral solo o
en combinación con procinéticos. Neostigmina: es un inhibidor de la acetilcolinesterasa que se ha
introducido recientemente en el tratamiento del síndrome de Ogilvie o dilatación cólica aguda a
dosis de 2,5 mg por vía intravenosa. EL APOYO EMOCIONAL ES DE AYUDA EN LA CONSTIPACIÓN
FUNCIONAL, PERO EL USO DE ANTIBIÓTICOS NO SE JUSTIFICA.

Bibliografía:
1. LA SALUD DEL NIÑO Y ADOLESCENTE. MARTÍNEZ Y MARTÍNEZ. MANUAL MODERNO. EDICIÓN 4TA. 2001.
PAG. 833-837.

FIN DEL CASO CLÍNICO SERIADO


ANÁLISIS DEL CASO CLÍNICO

IDENTIFICACIÓN DEL REACTIVO


Area: MEDICINA INTERNA
Especialidad: INMUNOALERGIA
Tema: RINITIS ALÉRGICA
Subtema: RINITIS ALÉRGICA

CASO CLÍNICO CON UNA PREGUNTA

HOMBRE DE 22 AÑOS QUE ACUDE A LA CONSULTA REFIRIENDO OBSTRUCCIÓN NASAL BILATERAL


INTERMITENTE Y RINORREA HIALINA ABUNDANTE, QUE SE PRESENTA PRINCIPALMENTE POSTERIOR A
CAMBIOS DE TEMPERATURA INTENSOS.

hombre de 22 años.

-.

OBSTRUCCIÓN NASAL BILATERAL


INTERMITENTE Y RINORREA HIALINA
ABUNDANTE QUE SE PRESENTA
PRINCIPALMENTE POSTERIOR A
"CAMBIOS DE TEMPERATURA INTENSOS".

-.

-.

106 - EL DIAGNÓSTICO CLÍNICO MÁS PROBABLE ES:

RINITIS La rinitis alérgica es un problema de salud que se caracteriza por una reacción inmunológica
ALÉRGICA. mediada por IgE en la mucosa nasal, desencadenada por aeroalergenos. El diagnóstico clínico de
rinitis alérgica se establece con presencia de: prurito nasal y faríngeo, estornudos en salva, moco
nasal y síntomas conjuntivales. La mayoría de los pacientes tiene síntomas transitorios por las
mañanas. NO SE RELACIONA CON CAMBIOS DE TEMPERATURA SINO MÁS BIEN CON LA
EXPOSICIÓN A UN ALERGENO.

RINITIS VIRAL. Las rinitis infecciosas pueden ser bien de carácter viral o bien de carácter bacteriano;
generalmente las rinitis infecciosas agudas son debidas a infecciones virales. LAS RINITIS
VASOMOTORAS "NO" TIENEN UN ORIGEN INFECCIOSO.
RINITIS Las RINITIS VASOMOTORAS tienen una hiperrespuesta nasal ante cambios de temperatura, al
VASOMOTORA. humo del tabaco o a fuertes olores, con lo que se intensifican los síntomas. Corresponden al tipo
de rinitis no alérgicas no infecciosas ya que no hay un agente biológico que desencadene la
reacción ni producción de IgE. LA RINITIS QUE SE PRESENTAN CON CAMBIOS DE TEMPERATURA
SE LLAMA RINITIS VASOMOTORA.

RINITIS Las personas con rinitis alérgica estacional tienen exacerbaciones periódicas de sus síntomas, los
ESTACIONAL. cuales ocurren en ciertas estaciones donde existen alto grado de polinización, especialmente en
climas templados (primavera para árboles, primavera y verano para pastos y otoño para malezas)
sin embargo habrá que recordar que esto dependerá del área específica donde resida el individuo
y las condiciones climatológicas de su área. LA RINITIS EN EL PACIENTE NO SE RELACIONA CON
TEMPORADAS ESTACIONALES.

Bibliografía:
1. LONGO DL, FAUCI AS, KASPER DL, HAUSERSL, JAMESON JL, LOSCALZOJ. HARRISON. PRINCIPIOS DE
MEDICINA INTERNA, 18A EDICIÓN. MC GRAW HILL. NEW YORK, USA. 2012, PP 2716.
ANÁLISIS DEL CASO CLÍNICO

IDENTIFICACIÓN DEL REACTIVO


Area: MEDICINA INTERNA
Especialidad: INFECTOLOGÍA
Tema: INFECCIONES RENALES Y DE LAS VÍAS URINARIAS
Subtema: PIELONEFRITIS

CASO CLÍNICO SERIADO

HOMBRE DE 36 AÑOS DE EDAD, CON ANTECEDENTE DE LUMBALGIA DE ESFUERZO DE 2 DÍAS DE EVOLUCIÓN


QUE NO MEJORA CON EL TRATAMIENTO. ACUDE A LA CONSULTA FAMILIAR REFIRIENDO DESDE HACE 24 HRS
LA PRESENCIA DE ASTENIA, ADINAMIA E HIPOREXIA, HIPERTERMIA NO CUANTIFICADA, NÁUSEAS SIN LLEGAR
A VÓMITO, ARTRALGIAS, MIALGIAS Y SÍNTOMAS DE IRRITACIÓN URINARIA. A LA EXPLORACIÓN PACIENTE
FEBRIL CON PALIDEZ EN PIEL Y TEGUMENTOS, FRECUENCIA CARDIACA DE 98 LATIDOS POR MINUTO,
ABDOMEN DOLOROSO DE PREDOMINIO EN REGIÓN SUPRAPÚBICA. EL UROTEST MUESTRA LEUCOCITURIA
+++, HEMATURIA +, CETONAS +.

Masculino de 36 años.

lumbalgia de 2 días de evolución que no


mejora con el tratamiento.

ASTENIA, ADINAMIA E HIPOREXIA,


hipertermia, náuseas, artralgias, mialgias y
síntomas de irritación urinaria.

febril, abdomen doloroso de predominio en


región suprapúbica.

leucocituria +++, hematuria +, cetonas +


(PUEDEN PRESENTARSE EN TODOS LOS
PADECIMIENTOS DESCRITOS).

107 - CORRESPONDE AL DIAGNÓSTICO CLÍNICO MÁS PROBABLE EN ESTE PACIENTE:

CISTITIS La CISTITIS se caracteriza por una inflamación de la vejiga generalmente por bacterias coleiformes
AGUDA (E. coli) en menor proporción por gram positivos. La vía de infección suele ser ascendente a través
de la uretra. EN VARONES LA CISTITIS NO ES COMÚN, CUANDO APARECE ES NECESARIO PENSAR
EN PROCESOS PATOLÓGICOS ASOCIADOS tales como prostatitis, retención urinaria crónica o
cálculos infectados.
PROSTATITIS La principal causa de PROSTATITIS AGUDA es la infecciosa, entre los agentes patológicos más
AGUDA comunes se encuentra la E. coli y las Pseudomonas. La vía de infección más común suele ser la
ascendente por medio de la uretra, es favorecida en muchos de los casos por el reflujo de orina
infectada a través de los conductos prostáticos. El cuadro clínico se caracteriza por dolor perineal,
sacro o subrapúbico, fiebre y síntomas de irritación durante la micción; puede acompañarse
además de grados variables de obstrucción que algunas veces favorecen la retención urinaria. A la
exploración hay fiebre y próstata dolorosa a la palpación. OJO: COMO DATO IMPORTANTE, EL
TACTO RECTAL, EN CASO DE REALIZARSE, DEBE HACERSE DE MANERA MUY SUAVE PUES LA
MANIPULACIÓN VIGOROSA PUEDE DESENCADENAR SEPTICEMIA. LOS SÍNTOMAS DE
PIELONERITIS Y PROSTATITIS SON MUY PARECIDOS, SÓLO QUE EN EL CASO DE LA ÚLTIMA EL
ATAQUE AL ESTADO GENERAL ES PREDOMINANTE Y EN LA PRIMERA SUELE HABER DOLOR
PERINEAL.

PIELONEFRITIS La PIELONEFRITIS AGUDA hace referencia a una inflamación aguda del parénquima y la pelvis
AGUDA renal. Generalmente es de origen infeccioso, se asocia principalmente a E. coli, Proteus, Klepsiella,
Enterobacter y Pseudomonas; las bacterias gram positivas son menos frecuentes. Por lo general la
infección suele ser por vía ascendente proveniente de las vías urinarias inferiores. Los síntomas
clínicos incluyen: fiebre, dolor lumbar, escalofrío intenso y síntomas de irritación a la micción. En
gran parte de los pacientes se presentan náuseas, vómito y diarrea. Suele haber taquicardia. El
urocultivo siempre es positivo y generalmente muestra un desarrollo abundante; el urocultivo
también puede ser positivo. EL FUERTE ATAQUE AL ESTADO GENERAL HACE PENSAR EN UNA
PIELONEFRITIS AGUDA MÁS QUE EN CUALQUIERA DE LOS OTROS DIAGNÓSTICOS. EL
ANTECEDENTE DE LUMBALGIA SUELE SER IMPORTANTE EN ESTOS CASOS AÚN CUANDO NO SE
DESCRIBEN GIORDANOS.

EPIDIDIMITIS Gran parte de las EPIDIDIMITIS AGUDAS se producen por agentes infecciosos y pueden coincidir
AGUDA con aquellas adquiridas por transmisión sexual (en varones adultos jóvenes) y las no transmitidas
por vía sexual (en adultos mayores y asociadas a prostatitis e infecciones urinarias). El cuadro
clínico clásicamente aparece tras un esfuerzo físico, traumatismo o actividad sexual. Los síntomas
pueden asociarse a aquellos causados por uretritis (secreción uretral e inflamación del meato
urinario) o cistitis (datos de irritación vesical). El dolor característico parte del escroto y suele
irradiarse por el cordón espermático y hacia el flanco correspondiente; suele acompañarse de
fiebre y aumento de volumen en la región escrotal. Una vez que se establece bien la inflamación el
epidídimo es posible palparlo percibiendo una masa dolorosa sobre el testículo. EL DOLOR NO
PARTE DEL ESCROTO POR LO QUE NO COINCIDE CON ESTE DIAGNÓSTICO.

Bibliografía:
1. GUÍA DE PRÁCTICA CLÍNICA, DIAGNÓSTICO Y TRATAMIENTO DE PIELONEFRITIS AGUDA NO
COMPLICADA EN EL ADULTO. MÉXICO: SECRETARIA DE SALUD, JULIO 2014. 2. PAPADAKIS MAXINE A,
MCPHEE STEPHEN J. DIAGNÓSTICO CLÍNICO Y TRATAMIENTO. 52ª EDICIÓN. MC GRAW HILL EDUCATION,
LANGE. USA. 2013.

http://www.cenetec.salud.gob.mx/descargas/gpc/CatalogoMaestro/451_GPC_Pielonefritis_aguda/GER_Pielonefritis_aguda.pdf

108 - ES EL TRATAMIENTO AMBULATORIO QUE DEBERÁ INDICAR USTED EN ESTE MOMENTO:

NITROFURANTOÍNA La CISTITIS AGUDA puede tratarse con un sinfín de medicamentos. En el caso de la cistitis no
complicada, la Nitrofurantoína y las fluoroquinolonas suelen ser los medicamentos de
elección. Los analgésicos como la fenazopiridina únicamente se indican para dar alivio
sintomático al paciente.

AMPICILINA + En la PROSTATITIS generalmente se inicia el manejo vía parenteral, sin estudio


AMIKACINA bacteriológico este incluye la administración de ampicilina más un aminoglucósido. Si la
fiebre no se presenta en un lapso de 24 a 48hrs puede cambiarse por tratamiento oral (por
ejemplo con quinolonas).

DOXICICLINA El tratamiento de la EPIDIDIMITIS de origen sexual (muy probablemente por la edad del
paciente), es a base de antibióticos por 10 a 21 días; el esquema más común es una
aplicación única de ceftriaxona (250mg) más 10 días de doxiciclina a dosis de 100mg cada
12hrs.
CIPROFLOXACINO Con base en la última modificación a la GPC para el diagnóstico y tratamiento de la
PIELONEFRITIS NO COMPLICADA EN EL ADULTO, el tratamiento empírico deberá
contemplar el agente patógeno más frecuente, que es el E. coli, y con base en ello iniciar con
fluoroquinolonas por 7 a 10 días. Se recomienda hospitalizar a los pacientes que presenten: -
Estado de choque. - Deshidratación. - Intolerancia a la vía oral (náuseas y vómito). - Sin
mejoría a las 72hrs de haber iniciado el tratamiento ambulatorio. - Necesidad de tratamiento
intravenoso y reanimación hídrica. En el caso de HOSPITALIZADOS deberá considerase el uso
de fluoroquinolonas, aminoglucósidos con o sin ampicilina, cefalosporinas de espectro
extendido con o sin aminogluósidos o un carbapenem. LAS FLUOROQUINOLONAS ESTÁN
INDICADAS COMO TRATAMIENTO EMPÍRICO PARA LA PIELONEFRITIS.

Bibliografía:
1. GUÍA DE PRÁCTICA CLÍNICA, DIAGNÓSTICO Y TRATAMIENTO DE PIELONEFRITIS AGUDA NO
COMPLICADA EN EL ADULTO. MÉXICO: SECRETARIA DE SALUD, JULIO 2014. 2. PAPADAKIS MAXINE A,
MCPHEE STEPHEN J. DIAGNÓSTICO CLÍNICO Y TRATAMIENTO. 52ª EDICIÓN. MC GRAW HILL EDUCATION,
LANGE. USA. 2013.

http://www.cenetec.salud.gob.mx/descargas/gpc/CatalogoMaestro/451_GPC_Pielonefritis_aguda/GER_Pielonefritis_aguda.pdf

FIN DEL CASO CLÍNICO SERIADO


ANÁLISIS DEL CASO CLÍNICO

IDENTIFICACIÓN DEL REACTIVO


Area: MEDICINA INTERNA
Especialidad: ENDOCRINOLOGÍA
Tema: PATOLOGÍA DE LA GLÁNDULA TIROIDES
Subtema: HIPERTIROIDISMO

CASO CLÍNICO SERIADO

MUJER DE 18 AÑOS, PRESENTA DESDE HACE 4 MESES NERVIOSISMO, INTOLERANCIA AL CALOR Y PÉRDIDA
DE PESO. A LA EXPLORACIÓN PRESENTA EXOFTALMOS, TIROIDES CON AUMENTO DE TAMAÑO, BORDES
REGULARES Y DIFUSOS, PIEL CALIENTE Y TEMBLOR FINO.

Mujer de 18 años de edad.

-.

4 MESES DE EVOLUCIóN CON


NERVIOSISMO, INTOLERANCIA AL CALOR
Y PERDIDA DE PESO.

EXOFTALMOS, aumento de tamaño de la


glándula tiroides, piel caliente y temblor
no.

-.

109 - LA CAUSA MÁS PROBABLE DEL CRECIMIENTO TIROIDEO EN ESTA PACIENTE SE DEBE A LA
PRESENCIA DE ANTICUERPOS:

ANTI- ÚNICAMENTE LOS ANTICUERPOS CONTRA EL RECEPTOR DE TSH SON ESPECÍFICOS DE


TIROPEROXIDASA. ENFERMEDAD AUTOINMUNE. Existe en la población en general algunas personas con
elevación de los anticuerpos antitiroglobulina y anti TPO sin que tengan enfermedad
tiroidea. "Los anticuerpos antiperoxisoma al igual que los antitiroglobulina se encuentran
elevados en pacientes con tiroiditis de Hashimoto".

CONTRA EL La enfermedad de Graves es una enfermedad autoinmune en la que los autoanticuerpos se


RECEPTOR DE TSH. unen y estimulan al receptor de TSH. Es la causa más común de tirotoxicosis. La activación
del receptor estimula el crecimiento y la función de las células tiroideas. LA PRODUCCIÓN
DE ACTICUERPOS CONTRA EL RECEPTOR DE TSH ES LA CAUSA MÁS COMÚN DE
TIROTOXICOSIS.
CONTRA También se pueden encontrar alteración en algunos otros autoantígenos. Los anticuerpos
TIROGLOBULINA. antiperoxidasa tiroideos (TPO, antes conocidos como anticuerpos antimicrosomales) y los
anticuerpos antitiroglobulina se pueden encontrar también en el suero de las personas con
tiroiditis linfocitos y en muchos pacientes con enfermedad de Graves. Los anticuerpos TPO
inhiben la actividad enzimática y estimulan la citotoxicidad mediada por células NK. LOS
ANTICUERPOS ANTITIROGLOBULINA NO PARECEN JUGAR UN PAPEL EN LA
DESTRUCCIÓN AUTOINMUNE DE LA GLÁNDULA TIROIDES.

ANTIMICROSÓMICOS. Los anticuerpos antiperoxidasa tiroides eran antes llamados antimicrosomales. ES UN


TÉRMINO EN DESUSO Y NO CORRESPONDE AL TIPO MÁS FRECUENTE.

Bibliografía:
1. LONGO DL, FAUCI AS, KASPER DL, HAUSERSL, JAMESON JL, LOSCALZOJ. HARRISON. PRINCIPIOS DE
MEDICINA INTERNA, 18A EDICIÓN. MC GRAW HILL. NEW YORK, USA. 2012, PP 2922-2923. 2. PAPADAKIS
MAXINE A, MCPHEE STEPHEN J. DIAGNÓSTICO CLÍNICO Y TRATAMIENTO. 52ª EDICIÓN. NUEVA YORK. 2013.
3. GUÍA DE PRÁCTICA CLÍNICA. DIAGNÓSTICO Y TRATAMIENTO DE LA ENFERMEDAD DE GRAVES EN >18
AÑOS. MÉXICO: SECRETARIA DE SALUD; 2011.

http://www.cenetec.salud.gob.mx/descargas/gpc/CatalogoMaestro/514_GPC_Enf_Graves/GER_EnfermedadGraves.pdf

110 - EL DIAGNÓSTICO MÁS PROBABLE EN ESTA PACIENTE ES:

ENFERMEDAD DE La ENFERMEDAD DE GRAVES es causada por una activación inadecuada del sistema
GRAVES. inmunológico (autoanticuerpos) que elige como blanco a los receptores TSH de las células
foliculares, resultando en una síntesis y secreción excesiva de hormona tiroidea, caracterizada
por hiperplasia glandular difusa (bocio), hiperfunción glandular (tirotoxicosis), oftalmopatía
infiltrativa y dermopatía. Es la causa más común de hipertiroidismo endógeno. Se caracteriza
por la siguiente triada clínica: 1. Hipertiroidismo secundario a crecimiento difuso y aumento
de la función de la glándula tiroides. 2. Oftalmopatía infiltrativa que da como resultado
exoftalmos. 3. Dermopatía infiltrativa localizada, también a veces se denomina mixedema
pretibial.

HIPERTIROIDISMO Este tipo de tirotoxicosis es el que se origina de la ingesta, generalmente es crónica, de


FICTICIO. grandes cantidades de hormona tiroidea. Esto se produce en persona que tienen alguna
enfermedad psiquiátrica. NO HAY ANTECEDENTE DE INGESTA DE HORMONAS TIROIDEAS.

ADENOMA Este no es el caso de la paciente ya que en la paciente se reporta en la exploración física


TÓXICO. crecimiento difuso de la glándula tiroides y no un solo nódulo que es lo que se encuentra en
los pacientes que tienen un adenoma tóxico. En estos casos ese nódulo la mayoría de las veces
es hiperfuncionante.

HIPERTIROIDISMO Esta ya no es una nomenclatura que se utilice hoy en día y hacía referencia a los pacientes que
APÁTICO. tenía hipertiroidismo pero que presentaban síntomas atípicos como depresión, letárgia,
fatiga, pérdida de apetito entre otros y este tipo de manifestaciones es mas frecuente en
ancianos.

Bibliografía:
1. LONGO DL, FAUCI AS, KASPER DL, HAUSERSL, JAMESON JL, LOSCALZOJ. HARRISON. PRINCIPIOS DE
MEDICINA INTERNA, 18A EDICIÓN. MC GRAW HILL. NEW YORK, USA. 2012, PP 2922-2923. 2. PAPADAKIS
MAXINE A, MCPHEE STEPHEN J. DIAGNÓSTICO CLÍNICO Y TRATAMIENTO. 52ª EDICIÓN. NUEVA YORK. 2013,
PP 1110. 3. GUÍA DE PRÁCTICA CLÍNICA. DIAGNÓSTICO Y TRATAMIENTO DE LA ENFERMEDAD DE GRAVES
EN >18 AÑOS. MÉXICO: SECRETARIA DE SALUD; 2011.

http://www.cenetec.salud.gob.mx/descargas/gpc/CatalogoMaestro/514_GPC_Enf_Graves/GER_EnfermedadGraves.pdf

FIN DEL CASO CLÍNICO SERIADO


ANÁLISIS DEL CASO CLÍNICO

IDENTIFICACIÓN DEL REACTIVO


Area: MEDICINA INTERNA
Especialidad: REUMATOLOGÍA
Tema: SÍNDROMES DE LAS VASCULITIS
Subtema: LUPUS ERITEMATOSO SISTÉMICO

CASO CLÍNICO CON UNA PREGUNTA

MUJER DE 32 AÑOS QUE INGRESA A URGENCIAS CON DIAGNÓSTICO DE LUPUS ERITEMATOSO


GENERALIZADO ACTIVO, CON DATOS DE INSUFICIENCIA RENAL AGUDA, ENCEFALITIS, FIEBRE Y LEUCOPENIA
SEVERA.

mujer de 32 años de edad

ingresa a urgencias con datos de


insu ciencia renal aguda, encefalitis, ebre
y leucopenia severa

111 - EN ESTE CASO SE DEBERÁ INICIAR DE INMEDIATO EL SIGUIENTE TRATAMIENTO:

CICLOFOSFAMIDA La ciclofosfamida es una sustancia alquilan y se usa de manera tradicional para la nefritis
INTRAVENOSA potencialmente letal del lupus, en particular en los pacientes en los que la biopsia renal
presenta una nefritis proliferativa o membranoproliferativa grados III, IV y V según la OMS.
Los estudios de ciclofosfamida que han tenido éxito también han utilizado glucocorticoides
concomitantes; la respuesta a la ciclofosfamida comienza entre tres y 16 semanas después
del inicio del tratamiento por lo que se debe dar junto con glucocorticoides. LA
CICLOFOSFAMIDA TIENE UN EFECTO A LARGO PLAZO POR LO QUE NO ESTÁ INDICADA EN
ESTE MOMENTO, ADEMAS DE QUE DEBE UTILIZARSE COMBINADA CON ESTEROIDES.
DOSIS ALTAS DE La base del tratamiento de cualquier manifestación del lupus que es potencialmente letal o
GLUCOCORTICOIDES puede dañar un órgano, es la administración de glucocorticoides a dosis de 0.5 a 2
mg/kg/día por vía oral o 1 000 mg de metilprednisolona por vía intravenosa diariamente
durante tres días seguido de 0.5 a 1 mg/kg/día de prednisona o su equivalente. En la
actualidad, las dosis altas se utilizan durante cuatro a seis semanas. Más tarde, se reducen
gradualmente pero con la mayor rapidez que permita la situación clínica, hasta obtener una
dosis de mantenimiento de 5 a 10 mg/día de prednisona, prednisolona o su equivalente o
entre 10 y 20 mg cada tercer día. EL TRATAMIENTO DE ELECCIÓN EN LUPUS EN CUALQUIER
FASE SON LOS GLUCOCORTICOIDES.

METOTREXATE A El metotrexate es un medicamento que se utiliza en pacientes con artritis reumatoide. En la


DOSIS TOPE actualidad no existe suficiente evidencia que apoye la efectividad del metotrexate para
tratar a pacientes con LES. Se puede indicar como un medicamento para reducir las dosis de
glucocorticoides en pacientes con manifestaciones cutáneas y articulares. NO HAY
EVIDENCIA CLÍNICA QUE RESPALDE EL USO DE METROTEXATE EN LES.

AZATIOPRINA Quizá la adición de azatioprina (antagonista de la purina) a los glucocorticoides reduce el


DOSIS ALTAS número de exacerbaciones de LES y la dosis de mantenimiento de los esteroides; no
obstante, la eficacia de este método se demora varios meses por lo que no está indicado
para los casos con enfermedad activa. SE PUEDE UTILIZAR EN EL MANEJO CRÓNICO DEL LES
AUNADO A ESTEROIDES.

Bibliografía:
1. LONGO DL, FAUCI AS, KASPER DL, HAUSERSL, JAMESON JL, LOSCALZOJ. HARRISON. PRINCIPIOS DE
MEDICINA INTERNA, 18A EDICIÓN. MC GRAW HILL. NEW YORK, USA. 2012, PP 2733. 2. PAPADAKIS MAXINE
A, MCPHEE STEPHEN J. DIAGNÓSTICO CLÍNICO Y TRATAMIENTO. 52ª EDICIÓN. NUEVA YORK. 2013, PP 834.
ANÁLISIS DEL CASO CLÍNICO

IDENTIFICACIÓN DEL REACTIVO


Area: MEDICINA INTERNA
Especialidad: HEMATOLOGÍA
Tema: TRANSTORNOS DE LA HEMOSTASIA
Subtema: ENF DE VON WILLEBRAND

CASO CLÍNICO SERIADO

MUJER DE 30 AÑOS DE EDAD LA CUAL ES SOMETIDA A EXTRACCIÓN DENTAL, CON PRESENCIA DE


SANGRADO LEVE PERO PERSISTENTE QUE DIFICULTA LA HEMOSTASIA, NO PRESENTA
COMPLICACIONES SISTÉMICAS. ES REFERIDA AL HEMATÓLOGO EL CUAL SOLICITA ESTUDIOS DE
COAGULACIÓN, REPORTÁNDOSE EN PRIMERA INSTANCIA TIEMPO DE SANGRADO PROLONGADO.

MUJER DE 30 AÑOs.

SOMETIDA A EXTRACCIÓN DENTAL,


PRESENTANDO DIFíCIL HEMOSTASIA, SIN
COMPLICACIONES SISTÉMICAS.

-.

-.

ESTUDIOS DE COAGULACIÓN
REPORTáNDOSE EN PRIMERA INSTANCIA
TIEMPO DE SANGRADO PROLONGADO.

112 - EL ESTUDIO QUE SE REQUIERE PARA CONFIRMAR EL DIAGNÓSTICO ES:


PRUEBA DE IMPORTANTE. Lo primero que tienes que hacer en este caso es determinar el
COFACTOR DE diagnóstico probable, el cual es ENFERMEDAD DE VON WILLEBRAND (EvW), basado
RISTOCETINA en que: 1. Es el defecto hemorrágico más frecuente. 2. A diferencia de la hemofilia, la
localización más frecuente de hemorragias son las mucosas. 3. En obligado
sospecharla en presencia de hemorragias de difícil control (10 minutos o más), como
es el caso, durante extracciones dentales o cirugías. OJO: el planteamiento de este
caso clínico se basa en un TIEMPO DE HEMORRAGIA (tiempo de sangrado)
prolongado, lo cual habla de un trastorno en la hemostasia, pero no es de utilidad
para dirigir el diagnóstico, “puede ser un distractor”. Anteriormente, se consideraba al
tiempo de hemorragia como el estándar de oro dentro de las pruebas de escrutinio;
actualmente, como se refiere en la GPC más reciente (2017), su valor diagnóstico ha
sido descartado por la ISTH (International Society of Thrombosis and Hemostasis)
debido a las dificultades para la estandarización, reproductibilidad, falta de
sensibilidad y especificidad. Por lo tanto, no es necesaria esta prueba para realizar el
diagnóstico de EvW. La ENFERMEDAD DE VON WILLEBRAND (EvW) es el defecto
hemorrágico más frecuente descrito en la literatura médica con una distribución
mundial y sin predominio de sexo y que se presenta igual en todos los grupos étnicos.
Se caracteriza por ser un padecimiento hemorrágico, hereditario que se presenta por
la deficiencia del Factor Von Willebrand (FvW). El FACTOR DE VON WILLEBRAND
(FvW) tiene dos funciones en la hemostasia: 1. Hemostasia primaria: es esencial para la
formación del coágulo plaquetario por sus funciones en la adhesión y agregación
plaquetaria a través de los grandes multímeros del FvW. 2. Hemostasia secundaria;
integra un complejo con el factor VIII por medio de una unión no covalente
protegiendo a este factor de la degradación enzimática. Se recomienda el uso del
puntaje de hemorragia para la EvW, mejor conocido como BLEEDING SCORE (BS) para
establecer la gravedad de la hemorragia e incrementar la sensibilidad y especificidad
en el diagnóstico, el cual consiste en un listado de síntomas asociados a hemorragia, a
los cuales se les otorga una puntuación que va de 0 a 4, según su frecuencia o
características clínicas específicas (puede leerse en la GPC 2017 de esta enfermedad).
Se aplica en hombres y mujeres con sospecha de la enfermedad con historia personal
o familiar de “hemorragias significativas”, entendiéndose por ésta cuando el/la
paciente tiene por lo menos 2 síntomas, sin requerimiento de transfusión sanguínea o
un síntoma que requiere transfusión sanguínea, o un síntoma recurrente, cuando
menos en 3 ocasiones diferentes. En lo que respecta a la historia familiar positiva
compatible con EvW cuando exista el antecedente de un familiar de primer grado o 2
familiares de segundo grado con historia familiar de hemorragias mucocutáneas
significativas y pruebas de laboratorio compatibles con EvW. PRUEBAS
CONFIRMTORIAS PARA EvW: - Factor de Von Willebrand antigénico (FvW:Ag). -
Factor de Willebrand Cofactor de Ristocetina (FvW:Co R) (simplemente denominado
Cofactor de Ristocetina o prueba de la actividad del FvW). - Determinación de Factor
VIII coagulante (FVIII:C). Este tipo de pruebas se recomiendan en toda persona con
alta sospecha de EvW. Se recomienda sospechar EvW en paciente que cursa con
hemorragias mucocutáneas y actividad del FvW (FvW:CoR). Se recomienda buscar la
relación del cofactor de ristocetina entre la fracción antigénica (FvW:CoR/FvW:Ag). Si
el cociente es < de 0.6 hay que sospechar se trata de un tipo 2 A, 2B o 2M. Si es > 0.6
puede tratarse del Tipo 1, a más cercano a la unidad es más probable que se trate de
una persona sin la enfermedad. LA PRUEBA DE COFACTOR DE RISTOCETINA ES UNA
PRUEBA CONFIRMATORIA DE LA ENFERMEDAD DE VON WILLEBRAND.

TIEMPO DE Las PRUEBAS DE TAMIZAJE (escrutinio) para EvW son: - Biometría hemática completa
PROTROMBINA (BHC) con cuenta de plaquetas y frotis de sangre periférica. - Tiempo de Protrombina
(TP). - Tiempo de Tromboplastina Parcial activado (TTPa). NOTA: el tiempo de
hemorragia antes se incluía en este rubro, actualmente no se solicita. ESTAS PRUEBAS
NO DETERMINAN LA EvW PERO EVALÚAN LA POSIBILIDAD DE QUE ESTÉ PRESENTE.
El TP y TTPa “no son trascendente como pruebas de tamizaje específicas para el
diagnóstico de EvW”. “El beneficio de realizar estos estudios es para considerar otras
posibilidades diagnósticas, fuera de EvW”, como sería en deficiencia de FXI. TANTO EL
TP COMO EL TTPa NO CORRESPONDEN A PRUEBAS CONFIRMATORIAS DE LA
ENFERMEDAD DE VON WILLEBRAND.
TIEMPO DE El TTPa, en la mayoría de los casos de EvW es normal, aunque, dependiendo de la
TROMBOPLASTINA actividad plasmática del FVIII, puede estar prolongado en los tipos 2N y 3. Es decir, el
PARCIAL TTPa normal no excluye el diagnóstico de EvW, siempre que esté prolongado obliga a
considerar los tipos 2N y 3. EL TTPa NO ES UNA PRUEBA CONFIRMATORIA DE EVW,
SE CONSIDERA UNA PRUEBA DE TAMIZAJE SOLAMENTE QUE NO ES ESPECÍFICA DE
LA ENFERMEDAD, PERO CONTRIBUYE A DESCARTAR OTRAS POSIBILIDADES
DIAGNÓSTICAS.

NIVELES DE La PROTEÍNA C es una glucoproteína plasmática que se trasforma en sustancia


PROTEINA S Y C anticoagulante cuando se activa por la trombina. Actúa como anticoagulante al incidir
e inactivar los factores V y VIII activados. La PROTEÍNA S es similar a la proteína C, es
una glucoproteína que experimenta modificaciones postraduccionales que dependen
de la vitamina K. Las deficiencias cualitativas o cuantitativas de la proteína C y/o la
proteína S, o la resistencia a la acción de la proteína C activada por una mutación
específica en su sitio de escisión predeterminado en el factor Va (factor V de Leiden),
origina estados de HIPERCOAGULABILIDAD. EL ESTUDIO DE LOS NIVELES DE
PROTEÍNA S Y PROTEÍNA C SE SOLICITAN EN ESTADOS DE HIPERCOAGULABILIDAD
EN LOS QUE SE SOSPECHE DEFICIENCIA DE ESTAS PROTEÍNAS. NO SON DE UTILIDAD
EN LA EvW.

Bibliografía:
1. DIAGNÓSTICO Y TRATAMIENTO ENFERMEDAD DE VON WILLEBRAND HEREDITARIA. PERSONAS
DE TODAS LAS EDADES. SEGUNDO Y TERCER NIVEL DE ATENCIÓN. EVIDENCIAS Y
RECOMENDACIONES. GUÍA DE PRÁCTICA CLÍNICA. 09/03/2017. 2. LONGO DL, FAUCI AS, KASPER DL,
HAUSERSL, JAMESON JL, LOSCALZOJ. HARRISON. PRINCIPIOS DE MEDICINA INTERNA, 18A
EDICIÓN. MC GRAW HILL. NEW YORK, USA. 2012.

http://www.cenetec-difusion.com/CMGPC/IMSS-408-10/ER.pdf

113 - EL TRATAMIENTO QUE DEBE INDICARSE ANTES DE LA SIGUIENTE EXTRACCIÓN ES:

PLASMA FRESCO Prevención y tratamiento de hemorragias. En términos generales, el tratamiento de la


CONGELADO. EVW puede dividirse en dos tipos: terapias coadyuvantes para proporcionar un beneficio
hemostático indirecto, y tratamientos que incrementan las concentraciones plasmáticas
de FVW y FVIII. LA INDICACIÓN DE PLASMA FRESCO CONGELADO SERÍA ÚNICAMENTE
EN CASOS DE SUMA URGENCIA, CUANDO NO SE CONTARÁ CON NINGÚN OTRO TIPO
DE OPCIÓN TERAPÉUTICA.

CONCENTRADOS Concentrado de FVW/FVIII. En pacientes con EVW en quienes la desmopresina no sea


DE FACTOR VIII. eficaz o esté contraindicada, o en casos en los que se anticipa un riesgo de hemorragia
elevado, o cuando la duración del apoyo hemostático requerido sea mayor a 2-3 días, las
concentraciones de FVW y FVIII pueden restablecerse mediante la infusión de
concentrados de estas proteínas derivados de plasma. La imposibilidad de inactivar virus
en el crioprecipitado (el hemoderivado que anteriormente se prefería para el
tratamiento de la EVW) y la falta de cualquier concentrado de FVW recombinante
aprobado ha generado el extenso uso de varios productos de FVW/FVIII derivados de
plasma. EL CONCENTRADO DE FACTOR DE VON WILLEBRAND ES EL TRATAMIENTO
MÁS EFECTIVO PARA LA EVW. EN EL CASO DEL CONCENTRADO DE FACTOR VIII NO ES
ESPECÍFICO, DEBE APLICARSE CON EL CONCENTRADO DE FVW LO CUAL RESULTA EN
UNA EFECTIVIDAD DE HASTA EL 90% EN EL TRATAMIENTO DE LA ENFERMEDAD.
ÁCIDO Los ANTIFIBRINOLÍTICOS inhiben la activación del plasminógeno y la actividad de la
TRANEXÁMICO. plasmina, por lo tanto, previenen la lisis del coágulo. Se recomienda el uso de
antifibrinolíticos orales, parenterales o tópicos en HEMORRAGIAS LEVES LOCALIZADAS
EN MUCOSAS donde hay gran actividad fibrinolítica y con deficiencia leve del FvW,
como en procedimientos dentales, gingivorragia, epistaxis, menorragia. Con aplicación
de cada 6 a 8 horas. La dosis del ácido aminocaproico (amicar) es de 25 a 50 mg/kg por
dosis. La duración del tratamiento dependerá de la respuesta hemostática. En cirugías,
tanto mayores como menores, en especial si involucran mucosas, se pueden emplear
ambos agentes antifibrinolíticos, como terapia adjuvante junto a reemplazo con
concentrado de FVIII/FVW. Los antifibrinolíticos se pueden administrar en forma
sistémica o local, entre ellos se encuentran al ácido épsilon aminocapróico y el ácido
tranexámico; pueden administrarse VO, IV o tópica. LOS ANTIFIBRINOLÍTICOS ESTÁN
INDICADOS EN HEMORRAGIA LEVE DE MUCOSA, LO CUAL QUIERE DECIR QUE SE
ENCUENTRA ACTIVA. LA GPC ACTUAL (2017) NO ES CLARA SOBRE SU USO COMO
MEDIDA PROFILÁCTICA. RECUERDA: aunque algunos autores sugieren que puede ser
utilizada como medida profiláctica, las respuestas deberán apegarse lo más posible a la
GPC siempre que exista una opción más correcta.

DESMOPRESINA. La DESMOPRESINA (1-deamino-8-D-arginina vasopresina ó DDAVP) es un análogo


sintético de la hormona antidiurética vasopresina. En la actualidad se cuenta con más de
25 años de experiencia clínica en el uso de la desmopresina para el tratamiento de la
EVW, y se ha utilizado ampliamente en todas las vías de administración: intravenosa,
subcutánea e intranasal. El efecto hemostático pico de la dosis normal de desmopresina
ocurre entre 0.5 y 1 hora después de su administración, con un incremento promedio de
FVW/FVIII de 2 a 4 veces por arriba de las concentraciones basales. No obstante, dada la
relativamente impredecible naturaleza de la respuesta a la desmopresina, todos los
pacientes con EVW deberían someterse a una prueba de administración terapéutica a fin
de determinar su nivel de respuesta individual. Si se documenta un beneficio
hemostático inicial adecuado (incremento de más de tres veces en el FVW:CoR y el
FVW:Ag a concentraciones de mayor a 0.30 U/mL), este método de tratamiento puede
usarse para la PREVENCIÓN DE HEMORRAGIAS RELACIONADAS CON CIRUGÍAS
MENORES E INTERVENCIONES DENTALES, ASÍ COMO PARA EL TRATAMIENTO DE
HEMORRAGIAS MENSTRUALES GRAVES. Si se requirieran dosis repetidas de
desmopresina, éstas no deberían administrarse más de una vez al día y, aún así, es muy
probable que los tratamientos subsecuentes den por resultado respuestas reducidas
(menor a70% de los incrementos iniciales de FVW y FVIII). AUNQUE LA PACIENTE NO HA
PRESENTADO COMPLICACIONES Y AL PARECER EL EVENTO ANTERIOR TUVO
COMPLICACIONES, LA DESMOPRESINA ES IDEAL COMO MEDIDA PROFILÁCTICA. NOTA:
recuerda que requiere de una prueba de administración terapéutica.

Bibliografía:
1. DIAGNÓSTICO Y TRATAMIENTO ENFERMEDAD DE VON WILLEBRAND HEREDITARIA. PERSONAS
DE TODAS LAS EDADES. SEGUNDO Y TERCER NIVEL DE ATENCIÓN. EVIDENCIAS Y
RECOMENDACIONES. GUÍA DE PRÁCTICA CLÍNICA. 09/03/2017. 2. LONGO DL, FAUCI AS, KASPER DL,
HAUSERSL, JAMESON JL, LOSCALZOJ. HARRISON. PRINCIPIOS DE MEDICINA INTERNA, 18A
EDICIÓN. MC GRAW HILL. NEW YORK, USA. 2012.

http://www.cenetec-difusion.com/CMGPC/IMSS-408-10/ER.pdf

FIN DEL CASO CLÍNICO SERIADO


ANÁLISIS DEL CASO CLÍNICO

IDENTIFICACIÓN DEL REACTIVO


Area: PEDIATRÍA
Especialidad: INFECTOLOGIA PEDIÁTRICA
Tema: NEUMONÍAS
Subtema: NEUMONÍAS

CASO CLÍNICO CON UNA PREGUNTA

ESCOLAR DE 7 AÑOS DE EDAD, CON ANTECEDENTE DE 8 DÍAS CON INFECCIÓN DE VÍAS AÉREAS SUPERIORES,
NO EVOLUCIÓN ADECUADA, CON PRESENCIA DE DIFICULTAD RESPIRATORIA. ACUDE A URGENCIAS Y
DIAGNOSTICAN NEUMONÍA ADQUIRIDA EN LA COMUNIDAD.

ESCOLAR DE 7 AÑOS

ANTECEDENTE DE 8 DIAS CON INFECCION


DE VIAS AEREAS SUPERIORES

MUY IMPORTANTE LA PRESENCIA DE


DIFICULTAD RESPIRATORIA. DIAGNÓSTICO
DE NEUMONÍA COMUNITARIA

114 - EL TRATAMIENTO ANTIBIÓTICO QUE DEBE INICIAR EL PACIENTE ES:


AZITROMICINA Tratamiento AMBULATORIO del paciente de 3 meses a 18 años de edad con neumonía adquirida
en la comunidad (NAC) de CURSO ATÍPICO: - Azitromicina oral, dosis inicial de 10 mg/kg/día
máximo 500 mg día, dosis de mantenimiento 5 mg/kg/día máximo 250 mg por 5 días. -
Alternativas: Claritromicina oral (15 mg/kg/día en 2 dosis, máximo 1 g al día por 7- 14 días).
Eritromicina oral (40 mg/kg/día en 4 dosis) por 10 días. Tratamiento del paciente
HOSPITALIZADO pediátrico de todas las edades con neumonía de origen bacteriano con CURSO
ATÍPICO: - Azitromicina (en adición a b-lactámico, si el diagnostico de neumonía atípica es
posible). - Alternativas: claritromicina o eritromicina; doxiciclina para niños mayores de 7 años de
edad; levofloxacina para niños quienes han terminado su crecimiento o que no presenten
adecuada tolerancia a macrólidos. IMPORTANTE: En caso de NAC con manifestaciones de
neumonía atípica, en pacientes escolares y adolescentes, se recomienda EL USO DE MACRÓLIDOS
COMO PRIMER ESQUEMA ANTIBIÓTICO. Se recomienda adicionar el uso de macrólidos en las/los
pacientes de cualquier edad que no hayan respondido al tratamiento de primera línea, o en
quienes se sospeche infección por Mycoplasma pneumoniae o Chlamydophila pneumoniae. LA
AZITROMICINA ES EL MEDICAMENTO DE ELECCIÓN PARA EL MANEJO DEL PACIENTE
PEDIÁTRICO CON NEUMONÍA DE CURSO ATÍPICO, LO CUAL NO COINCIDE CON ESTE CASO.

AMOXICILINA Tratamiento AMBULATORIO del paciente de 3 meses a 18 años de edad con neumonía adquirida
en la comunidad (NAC) de origen bacteriano: - Amoxicilina dosis (90 mg/kg/día en 2 dosis
máxima de 4 g día) por 10 días. - En niños con sospecha de NAC de origen bacteriano que
clínicamente o radiográficamente se sospeche de una NAC de curso atípico, se podrá añadir al
tratamiento un macrólido. - Alternativas: Amoxicilina con clavulanato oral dosis de (90
mg/kg/día en 2 dosis máxima de 4 g al día) por 10 días. - En presencia de alergia a ?-lactámicos:
Azitromicina Claritromicina Eritromicina. LA AMOXICILINA ES EL MEDICAMENTO DE PRIMERA
LÍNEA PARA EL MANEJO “AMBULATORIO” DE LA NEUMONÍA ADQUIRIDA EN LA COMUNIDAD
DE ORIGEN BACTERIANO.

PENICILINA LA CLAVE ES DETERMINAR SI SE PODRÁ DAR TRATAMIENTO AMBULATORIO U HOSPITALARIO.


EL ÚNICO DATO QUE SE DA ES QUE EL PACIENTE PRESENTA DIFICULTAD RESPIRATORIA. ESTE
SÓLO DATO OBLIGA AL CLÍNICO A HOSPITALIZAR AL PACIENTE Y COMENZAR TRATAMIENTO
INTRAHOSPITALARIO. Una vez establecido el diagnóstico debes decidir si el tratamiento puede
ser ambulatorio o requiere de hospitalización. 1. Tratamiento del paciente HOSPITALIZADO
pediátrico de todas las edades con neumonía de origen bacteriano “con vacunación completa”: -
Ampicilina 150 a 200 mg/kg/día, intravenosa en 4 dosis por 10 a 14 días. - Penicilina G sódica
cristalina 250,000 a 300,000 UI/kg/día, intravenosa en 4 a 6 dosis por 10 a 14 días. - Adicionar
vancomicina o clindamicina dosis de 40 mg/kg/día cada 6-8 hrs en sospecha de Staphylococcus
aureus resistente a meticilina (SARM) de la comunidad. LAS PENICILINAS (AMPICILINA Y
PENICILINA G) SON EL TRATAMIENTO DE ELECCIÓN PARA EL MANEJO DE NIÑOS Y
ADOLESCENTES CON NEUMONÍA DE ORIGEN BACTERIANO ADQUIRIDO EN LA COMUNIDAD.
OJO: el uso de este esquema de tratamiento es ideal siempre que el niño tenga completo el
esquema de vacunación contra H. influenza y S. pneumoniae y cuando la resistencia local a
penicilina en cepas invasivas de S. pneumoniae sea mínima. 2. Tratamiento del paciente
HOSPITALIZADO pediátrico de todas las edades con neumonía de origen bacteriano “con
vacunación incompleta”: - Cefotaxima 150 mg/kg/día, intravenosa en 3 dosis por 10 a 14 días. -
Ceftriaxona 50-100 mg/kg/día, intravenosa en 2 dosis por 10 a 14 días. - Adicionar vancomicina o
clindamicina por sospecha de SARM de la comunidad. – - Alternativa de tratamiento:
Levofloxacina dosis 6 meses-5 años de edad: 16–20mg/kg/día en 2 dosis 5 – 18 años de edad: 8-
10 mg/kg/dosis cada 24h por 10 a 14 dias (dosis máxima 750 mg día). - Adicionar vancomicina o
clindamicina en sospecha de SARM comunitario. OJO: el uso de este esquema de tratamiento es
ideal para aquellos pacientes con esquema de vacunación contra H. influenza y S. pneumoniae
incompleto y cuando la resistencia local a penicilina en cepas invasivas de S. pneumoniae sea
significativa.

GENTAMICINA LOS AMINOGLUCÓSIDOS NO SE CONTEMPLAN COMO OPCIONES TERAPÉUTICAS EN EL MANEJO


DE LA NEUMONÍA ADQUIRIDA EN LA COMUNIDAD EN PACIENTES PEDIÁTRICOS. RESUMEN.
Circunstancias que hacen aconsejable el ingreso hospitalario en la NAC (neumonía adquirida en la
comunidad). • Existencia de alguno de los criterios de gravedad • Menores de 6 meses • Estado
general afectado • Patología de base: neumopatía, cardiopatía, inmunodepresión, malnutrición. •
Dificultad para el control evolutivo • Dudas acerca del cumplimiento terapéutico • Distrés
respiratorio, taquipnea importante o hipoxemia/cianosis (saturación 92%) • Dificultades para la
alimentación • Coexistencia de otros problemas: diarrea, broncoespasmo • Afectación radiológica
extensa, derrame pleural, neumotórax • Sospecha de estafilococo, gramnegativos entéricos o
germen inhabitual • Necesidad de identificación bacteriológica y antibiograma • Falta de
respuesta a las 48-72 horas de tratamiento ambulatorio • Problemática de índole socioeconómica
Bibliografía:
1. DIAGNÓSTICO Y TRATAMIENTO DE LA NEUMONÍA ADQUIRIDA EN LA COMUNIDAD EN PACIENTES DE 3
MESES A 18 AÑOS EN EL PRIMER Y SEGUNDO NIVEL DE ATENCIÓN. REFERENCIA RÁPIDA: GUÍA DE
PRÁCTICA CLÍNICA. MÉXICO: SECRETARÍA DE SALUD, 1 DE DICIEMBRE DE 2015. 2. NELSON TRATADO DE
PEDIATRÍA. NAPOLEÓN GONZÁLEZ SALDAÑA. MC GRAW HILL INTERAMERICANA. EDICIÓN 7A. 2003. PAG.
261-367.

http://www.cenetec-difusion.com/CMGPC/S-120-08/ER.pdf
ANÁLISIS DEL CASO CLÍNICO

IDENTIFICACIÓN DEL REACTIVO


Area: GINECOLOGÍA Y OBSTETRICIA
Especialidad: OBSTETRICIA
Tema: HEMORRAGIAS DE LA PRIMERA MITAD DEL EMBARAZO
Subtema: ABORTO

CASO CLÍNICO SERIADO

MUJER DE 26 AÑOS, GESTA 2, ABORTO 1, RESUELTO POR LEGRADO INSTRUMENTADO HACE 4 MESES. ACUDE
A URGENCIAS REFIRIENDO EMBARAZO DE 15 SDG Y PRESENTAR SALIDA DE LÍQUIDO TRANSVAGINAL
MODERADO, ADEMÁS DE ESCASO SANGRADO. A LA EXPLORACIÓN SE ENCUENTRA CUELLO UTERINO
DILATADO.

-.

GESTA 2, ABORTO 1 RESUELTO POR


LEGRADO INSTRUMENTADO HACE 4
MESES, EMBARAZO 15 sdG.

SALIDA DE LÍQUIDO TRASVAGINAL


MODERADO Y ESCASO SANGRADO.

CUELLO UTERINO DILATADO.

-.

115 - EN ESTE CASO EL ANTECEDENTE DE LEGRADO PREVIO ES IMPORTANTE EN LA PATOGENIA DEL


ESTADO ACTUAL PORQUE:

PUEDE El ABORTO HABITUAL también llamado PÉRDIDA GESTACIONAL REPETIDA, RECURRENTE o


CONVERTIRSE EN ABORTO RECURRENTE es la pérdida “espontánea” del embarazo en dos o más ocasiones de
ABORTADORA forma consecutiva o alterna. OJO: el caso clínico refiere que un aborto que se resolvió o
HABITUAL. ameritó legrado instrumentado, de momento no sabemos si fue espontáneo o provocado. EN
CASO DE DOCUMENTARSE EL ABORTO PREVIO COMO ESPONTÁNEO, LA PACIENTE PUEDE
SER “DIAGNOSTICADA” CON UN ABORTO RECURRETE, SIN EMBARGO, ESO NO RESPONDE A
LA PREGUNTA QUE SE SOLICITA: “El antecedente de legrado previo es importante en la
PATOGENIA ACTUAL porque…” EN CONCLUSIÓN: SER ABORTADORA HABITUAL, NO FORMA
PARTE DE LA PATOGENIA ACTUAL, ES MÁS BIEN LA RESULTANTE.
PUDO PROVOCAR Para contestar esta pregunta deberás poner especial atención en lo que se solicita: EL
INCOMPETENCIA ANTECEDENTE DE LEGRADO TIENE IMPORTANCIA EN LA PATOGENIA ACTUAL PORQUE…
CERVICAL. luego entonces, el primer análisis rápido de las respuestas será: 1. Convertirse en abortadora
habitual no es parte de la patogenia actual, sino una consecuencia. 2. La incompetencia
cervical si es parte de la patogenia; el legrado previo condiciona lesión cervical, la lesión
cervical condiciona incompetencia y, ésta a su vez, condiciona el cuadro clínico actual
(aborto). 3. La cervicovaginitis o endometriosis son complicaciones inmediatas graves de un
legrado instrumentado, el tiempo de evolución no concuerda, además de que el cuadro
clínico no refiere datos de infección, lo cual elimina esta opción inmediatamente. 4. La
paciente se encuentra embarazada por lo tanto no se puede hablar de infertilidad secundaria
al legrado previo. La INCOMPETENCIA CERVICAL, también llamada insuficiencia ístmico-
cervical consiste en un defecto del cierre del cérvix uterino a nivel del orificio cervical interno.
La incompetencia provoca una dilatación progresiva e indolora del cuello uterino con la
consecuente expulsión embrionaria o fetal en el embarazo. Ésta patología puede ser
congénita o adquirida. Dentro de las causas adquiridas se encuentra la lesión del cuello por la
salida de un feto en el parto, traumatismo al emplear fórceps o la dilatación quirúrgica del
cuello antes de realizar un legrado uterino. DESPUÉS DE UNA DILATACIÓN O LEGRADO
PUEDE QUEDAR COMO SECUELA UNA INSUFICIENCIA CERVICAL.

LE PUDO La infección puerperal se da en los primeros 10 días del puerperio, excluyendo las primeras
PROVOCAR UNA 24hrs siendo las principales la endometritis, infecciones de origen urinario, e infecciones de la
CERVICOVAGINITIS herida quirúrgica en caso de cesárea. Aunque por definición el puerperio incluye las primeras
O ENDOMETRITIS semanas que siguen del parto entendiendo por éste al conjunto de fenómenos activos y
QUE HA pasivos que desencadenados a partir de la semana 20 de la gestación, que tienen por objeto
PERSISTIDO la expulsión del producto mismo de la gestación, la placenta y sus anexos; DESPUÉS DE UN
HASTA LA FECHA. ABORTO DEBEMOS TOMAR LAS MISMAS CONSIDERACIONES QUE EL LA INFECCIÓN
PUERPERAL PARA PENSAR QUE ÉSTA SEA SECUNDARIA AL PROCEDIMIENTO, DENTRO DE
LOS PRIMEROS 10 DÍAS DESPUÉS DE REALIZADO EL ABORTO INSTRUMENTADO. Recuerda
que la manifestación más frecuente de infección del aborto es la endometritis y se presenta
en la mayoría de los casos dentro de la primera semana después del aborto médico inducido.

PUEDE Dentro de las complicaciones tardías del legrado instrumentado está la infertilidad
PROVOCARLE relacionada con la evacuación por curetaje por la lesión que provoca en el endometrio
INFERTILIDAD AL dificultando la implantación. NO COINCIDE CON EL CASO CLÍNICO YA QUE ÉSTE TIENE UNA
LESIONAR AL EVOLUCIÓN DE 15 SDG.
ÚTERO.

Bibliografía:
1. CUNNINGHAM G, LEVENO K, BLOMM S, HAUTH J, RPUSE D, SONG C. WILLIAMS OBSTETRICIA, 23A
EDICIÓN. MC GRAW HILL. USA. 2011 EN ESPAÑOL, PP 218-219. GUÍA DE PRÁCTICA CLÍNICA. DIAGNÓSTICO Y
TRATAMIENTO DEL ABORTO ESPONTÁNEO Y MANEJO INICIAL DEL ABORTO RECURRENTE. MÉXICO:
SECRETARÍA DE SALUD; 2009.

http://www.cenetec-difusion.com/CMGPC/IMSS-088-08/ER.pdf

116 - PARA REALIZAR EL DIAGNÓSTICO DEFINITIVO SERÁ NECESARIO REALIZAR:

ULTRASONIDO. La ecografía transvaginal es útil para corroborar la "vitalidad fetal" y situación del
embarazo, confirma lo ya explorado por tacto vaginal, acortamiento cervical con o sin
presencia de bolsa en el trayecto cervical. SI BIEN EL ULTRASONIDO ES UN "MÉTODO
DIAGNÓSTICO CONFIRMATORIO" RECUERDA INDICARLO SIEMPRE POSTERIOR A LA
VALORACIÓN CLÍNICA COMO PARTE DEL PROTOCOLO DE VALORACIÓN FETAL.

HISTEROSALPINGOGRAFÍA La HISTEROSALPINGOGRAFÍA es la visualización radiológica de la cavidad uterina y de


UNA VEZ CORROBORADA las trompas, mediante la introducción de un contraste radiopaco a través del cérvix.
LA VITALIDAD FETAL. Está contraindicado en embarazo, infección del tracto genito-urinario, hemorragia (ya
sea por menstruación o cualquier otra causa) y sensibilidad al contraste. Es útil para
obtener información sobre las cavidades tubárica y uterina (tamaño, forma, posición)
y su posible patología, su principal indicación es para demostrar la permeabilidad
tubaria. RECUERDA QUE LA HISTEROGRAFÍA ES UN MÉTODO DIAGNÓSTICO
CONTRAINDICADO EN EL EMBARAZO.
TACTO VAGINAL. La historia clínica y exploración del paciente deben fundamentar el diagnóstico
clínico. Al tacto vaginal se encuentra acortamiento y dilatación cervical con o sin
protrusión amniótica. EL ANTECEDENTE DE MANIPULACIÓN CERVICAL, CÉRVIX
ACORTADO AL TACTO VAGINAL Y LA DILATACIÓN INDOLORA DEL CUELLO UTERINO
SUGIEREN UNA INCOMPETENCIA CERVICAL. OJO: En el caso clínico se describe que ya
se realizó un tacto vaginal encontrándose el cuello dilatado lo cual sugiere una
amenaza de aborto. El estudio ideal para realizar el DIAGNÓSTICO DEFINITIVO en este
caso es el ultrasonido.

CULTIVO CERVICAL. El cultivo cervical consiste en la toma de muestra de secreción cervical para análisis
biológico, se utiliza para determinar el agente patógeno en las enfermedades
infecciosas del tracto genital. NO CORRESPONDE CON LOS DATOS CLÍNICOS YA QUE
NO HAY SOSPECHA DIAGNÓSTICA EN ÉSTE MOMENTO DE CERVICITIS.

Bibliografía:
1. CUNNINGHAM G, LEVENO K, BLOMM S, HAUTH J, RPUSE D, SONG C. WILLIAMS OBSTETRICIA, 23A
EDICIÓN. MC GRAW HILL. USA. 2011 EN ESPAÑOL, PP 218-219.

117 - EN CASO DE ENCONTRARSE VIVO EL PRODUCTO EL TRATAMIENTO DE ELECCIÓN CONSISTE EN:

REPOSO No existe evidencia consistente que indique que el reposo en cama mejora el pronóstico de
ABSOLUTO. una amenaza de aborto, sin embargo proporciona beneficios secundarios a la paciente y
puede ser recomendado. NO SE PUEDE CONSIDERAR AL REPOSO COMO UNA ESTRATEGIA
FRANCA DE TRATAMIENTO SÓLO COMO COADYUVANTE EN EL MANEJO CLÍNICO DEL
BINOMIO.

TOCOLÍTICO. Existen tres indicaciones principales para el uso de tocolíticos en el tratamiento del parto
prematuro: profilaxis (tratamiento basado en la existencia de un factor de riesgo de
actividad uterina exclusivamente, en ausencia de cambios cervicales, para prevenir el parto
prematuro); tratamiento agudo (administración de fármacos parenterales por vía
intravenosa o subcutánea para el control inmediato de los episodios agudos de parto
prematuro durante períodos que oscilan entre 24 y 72 horas), y mantenimiento (utilización
de fármacos orales o subcutáneos para tocólisis a largo plazo, tras la resolución de la
amenaza de parto prematuro, con el objeto de prevenir la recurrencia de la actividad
uterina). Su indicación es exclusiva durante la amenaza de parto pretérmino a partir de la
semana 22 de gestación. LA PACIENTE NO CUMPLE CON LOS CRITERIOS DE EDAD
GESTACIONAL PARA LA UTILIZACIÓN DE TOCOLÍTICOS.

CERCLAJE. El CERCLAJE CERVICALl está indicado en historia previa de parto pretérmino o abortos de
repetición, diagnóstico de incompetencia cervical durante el embarazo y malformaciones
uterinas. Una vez diagnosticada la incompetencia cervical el tratamiento de elección es
quirúrgico y consiste en un cerclaje uterino. Se basa en reforzar el cérvix mediante algún tipo
de sutura en bolsa de tabaco que permita el cierre del mismo y evite la protrusión de
membranas. Se recomienda usar de forma profiláctica durante la semana 14 a 16 y de
urgencia "en cualquier momento del segundo trimestre". EL CERCLAJE CERVICAL ES EL
TRATAMIENTO IDEAL PARA MANEJAR LA INCOMPETENCIA CERVICAL.

ANTIMICROBIANOS Dentro de las complicaciones del cerclaje y de la incompetencia misma del cuello uterino
Y HORMONALES. está la infección con corioamnionitis por la exposición de membranas y la manipulación, por
lo que LOS ANTIMICROBIANOS ESTÁN INDICADOS COMO MEDIDA PROFILÁCTICA PERO NO
CONSTITUYEN LA ESTRATEGIA TERAPÉUTICA ESPECÍFICA DE LA INCOMPETENCIA CERVICAL.

Bibliografía:
1. GUÍA DE PRÁCTICA CLÍNICA, INDICACIONES Y MANEJO DEL CERCLAJE CERVICAL. MÉXICO: SECRETARIA
DE SALUD; 2011. 2. GUÍA DE REFERENCIA RÁPIDA, INDICACIONES Y MANEJO DEL CERCLAJE CERVICAL.
MÉXICO: SECRETARIA DE SALUD; 2011. 3. CUNNINGHAM G, LEVENO K, BLOMM S, HAUTH J, RPUSE D, SONG
C. WILLIAMS OBSTETRICIA, 23A EDICIÓN. MC GRAW HILL. USA. 2011 EN ESPAÑOL, PP 218-219.

http://www.cenetec.salud.gob.mx/descargas/gpc/CatalogoMaestro/539_GPC_Cerclajecervical/GER_CerclajeCervical.pdf
FIN DEL CASO CLÍNICO SERIADO
ANÁLISIS DEL CASO CLÍNICO

IDENTIFICACIÓN DEL REACTIVO


Area: GINECOLOGÍA Y OBSTETRICIA
Especialidad: OBSTETRICIA
Tema: HEMORRAGIAS DE LA SEGUNDA MITAD DEL EMBARAZO
Subtema: DESPRENDIMIENTO PREMATURO DE PLACENTA

CASO CLÍNICO SERIADO

MUJER DE 26 AÑOS GESTA 3, PARTOS 2. CURSA CON EMBARAZO DE 36 SDG. SE PRESENTA A URGENCIAS
PORQUE HACE 30 MINUTOS PRESENTO SALIDA DE LÍQUIDO TRANSVAGINAL ABUNDANTE. POSTERIORMENTE
SE AGREGA DOLOR ABDOMINAL Y SANGRADO TRANSVAGINAL DE COLOR ROJO OSCURO ABUNDANTE. A LA
EXPLORACIÓN SE ENCUENTRA CON TENSIÓN ARTERIAL DE 80/40 MMHG, FRECUENCIA CARDIACA 120 X',
ÚTERO HIPERSENSIBLE, FONDO UTERINO A 35 CM POR ARRIBA DE LA SÍNFISIS DEL PUBIS.

-.

EMBARAZO 36SDG, SALIDA DE LIQUIDO


TRAnSVAGINAL Y SANGRADO ROJO
OSCURO ABUNDANTE.

DOLOR ABDMINAL.

ta 80/40, FRECUENCIA CARDÍACA 120X',


ÚTERO HIPERSENSIBLE.

-.

118 - PARA OBTENER EL DIAGNÓSTICO DEFINITIVO DE ESTA PACIENTE EN ESTE MOMENTO DEBERÁ
REALIZAR:
ULTRASONIDO Las principales causas de hemorragia en el tercer trimestre del embarazo son el
PÉLVICO. desprendimiento prematuro de placenta y la placenta previa. OJO: la placenta previa se
presenta característicamente como sangrado intermitente e indoloro, rojo rutilante. Dado que
la paciente presenta un cuadro clínico que sugiere DESPRENDIMIENTO PREMATURO DE
PLACENTA; hemorragia genital obscura o líquido amniótico sanguinolento abundante, dolor
abdominal súbito y constante localizado en el útero, hipertonía y polisistolia uterina, datos de
sufrimiento fetal acompañado o no de datos de choque hipovolémico; el método diagnóstico
debe estar dirigido hacia esta patología. Sólo en la minoría de los casos se observa en el
ultrasonido el desprendimiento de la placenta con acumulación retroplacentaria de sangre. Por
consiguiente, una ultrasonografía con datos normales no excluye la posibilidad de
desprendimiento prematuro de la placenta. Los criterios ultrasonográficos de desprendimiento
de placenta son: colección entre placenta y líquido amniótico, movimiento en "jello" de la
lámina coriónica con la actividad fetal, colección retroplacentaria, hematoma marginal,
hematoma subcoriónico, aumento heterogéneo en el grosor de la placenta, hematoma intra
amniótico. COMO AUXILIAR DIAGNÓSTICO ES ÚTIL EL ULTRASONIDO ABDOMINAL PERO NO
ES DEFINITIVO NI 100% CONCLUYENTE. EL ULTRASONIDO ABDOMINAL ESTÁ INDICADO SI
LAS CONDICIONES CLÍNICAS LO PERMITE, DADO QUE LA PACIENTE TIENE SIGNOS CLAROS DE
CHOQUE HIPOVOLÉMICO NO ESTÁ INDICADO.

ULTRASONIDO Ante datos clínicos de desprendimiento de placenta y si las condiciones lo permiten se realiza
ENDOVAGINAL. ultrasonido abdominal, el ultrasonido endovaginal no está indicado. EN CASO DE REQUERIR
ULTRASONIDO COMO AUXILIAR EN EL DIAGNÓSTICO LA INDICACIÓN ES REALIZARLO POR
VÍA ABDOMINAL.

COMPLETAR El diagnóstico de desprendimiento prematuro de placenta es clínico y la condición se debe


EXPLORACIÓN sospechar en toda gestante con embarazo mayor de 20 semanas que presenta sangrado por
OBSTÉTRICA. genitales o dolor abdominal o ambos, una historia de trauma o aquellas que presentan trabajo
de parto pre término no explicado. EL DIAGNÓSTICO ES CLÍNICO Y NO PUEDE ESPERAR A LOS
MÉTODOS AUXILIARES DIAGNÓSTICOS PARA EMPEZAR CON EL MANEJO TERAPÉUTICO
DEBIDO AL RIESGO DE HIPOVOLEMIA Y MUERTE FETAL.

ESPECULOSCOPÍA La especuloscopia con presencia de líquido amniótico en fondo de saco posterior, asociada a
Y cristalografía son métodos diagnósticos empleados para comprobar la ruptura de membranas;
CRISTALOGRAFÍA. sin embargo, la presencia de hemorragia abundante no permiten su utilización para éste fin.

Bibliografía:
1. GUÍA DE PRÁCTICA CLÍNICA, DIAGNÓSTICO Y TRATAMIENTO DE LA HEMORRAGIA OBSTÉTRICA EN LA
SEGUNDA MITAD DEL EMBARAZO Y PUERPERIO INMEDIATO. MÉXICO: SECRETARIA DE SALUD; 2009. 2.
GUÍA DE REFERENCIA RÁPIDA, DIAGNÓSTICO Y TRATAMIENTO DE LA HEMORRAGIA OBSTÉTRICA EN LA
SEGUNDA MITAD DEL EMBARAZO Y PUERPERIO INMEDIATO. MÉXICO: SECRETARIA DE SALUD; 2009. 3.
CUNNINGHAM G, LEVENO K, BLOMM S, HAUTH J, RPUSE D, SONG C. WILLIAMS OBSTETRICIA, 23A
EDICIÓN. MC GRAW HILL. USA. 2011 EN ESPAÑOL, PP 761-767.

http://www.cenetec.salud.gob.mx/descargas/gpc/CatalogoMaestro/162_GPC_HEMORRAGIA_OBSTETRICA/Imss_162ER.pdf

119 - EL TRATAMIENTO DE ELECCIÓN EN ESTE MOMENTO SERÁ:

INDUCTO La inducto-conducción es la estimulación de las contracciones uterinas antes de que se


CONDUCCIÓN desencadene el trabajo de parto de manera espontánea. LA PACIENTE YA MUESTRA DATOS
INMEDIATA. DE ACTIVIDAD UTERINA POR LO QUE ÉSTA MEDIDA NO ESTA INDICADA ADEMÁS DE QUE SE
UTILIZA PARA ESTIMULAR EL NACIMIENTO VÍA VAGINAL LO CUAL NO ESTÁ INDICADO EN
ESTE CASO.

UTEROINHIBICIÓN. El objetivo principal de la uteroinhibición en el trabajo de parto pretérmino es retrasar el


parto a menos 48hrs el parto para la administración de inductores de la maduración
pulmonar. NO ES FUNCIONAL LA UTEROINHIBICIÓN EN ÉSTE CASO YA QUE AUNQUE SE
LOGRARA INHIBIR LA ACTIVIDAD UTERINA LOS DATOS CLÍNICOS DE LA PACIENTE INDICAN
COMPLICACIONES HEMODINÁMICAS QUE PONEN EN RIESGO LA VIDA DEL BINOMIO Y
OBLIGAN A ACTUAR INMEDIATAMENTE EN LA RESOLUCIÓN DEL EMBARAZO.
PARTO POR En el caso de desprendimiento prematuro de placenta con fetos vivos debe realizarse la
CESÁREA. cesárea de forma inmediata ya que se ha demostrado que hay un mayor porcentaje de fetos
vivos tras en procedimiento comparados con el parto vaginal. DADO QUE EL EMBARAZO SE
DESARROLLA EN LA SEMANA 36 Y CON HEMORRAGIA ACTIVA DEBE REALIZARSE CESÁREA
DE URGENCIA.

REPOSO Y Sólo en casos de embarazos pretérmino entre la semana 20-34 de gestación con
VIGILANCIA desprendimiento parcial y sin sangrado activo está indicado el manejo conservador para
OBSTÉTRICA. administrar inductores de maduración fetal. NO ES UNA OPCIÓN YA QUE LA HEMORRAGIA
ADEMÁS DE SER ACTIVA ES ABUNDANTE Y PONE EN RIESGO LA VIDA DEL BINOMIO.

Bibliografía:
1. GUÍA DE PRÁCTICA CLÍNICA, DIAGNÓSTICO Y TRATAMIENTO DE LA HEMORRAGIA OBSTÉTRICA EN LA
SEGUNDA MITAD DEL EMBARAZO Y PUERPERIO INMEDIATO. MÉXICO: SECRETARIA DE SALUD; 2009. 2.
GUÍA DE REFERENCIA RÁPIDA, DIAGNÓSTICO Y TRATAMIENTO DE LA HEMORRAGIA OBSTÉTRICA EN LA
SEGUNDA MITAD DEL EMBARAZO Y PUERPERIO INMEDIATO. MÉXICO: SECRETARIA DE SALUD; 2009. 3.
CUNNINGHAM G, LEVENO K, BLOMM S, HAUTH J, RPUSE D, SONG C. WILLIAMS OBSTETRICIA, 23A
EDICIÓN. MC GRAW HILL. USA. 2011 EN ESPAÑOL, PP 767-769.

http://www.cenetec.salud.gob.mx/descargas/gpc/CatalogoMaestro/162_GPC_HEMORRAGIA_OBSTETRICA/Imss_162ER.pdf

FIN DEL CASO CLÍNICO SERIADO


ANÁLISIS DEL CASO CLÍNICO

IDENTIFICACIÓN DEL REACTIVO


Area: GINECOLOGÍA Y OBSTETRICIA
Especialidad: GINECOLOGÍA
Tema: LEUCORREA
Subtema: CANDIDIASIS VAGINAL

CASO CLÍNICO CON UNA PREGUNTA

MUJER DE 38 AÑOS DE EDAD, CURSANDO SU PRIMER TRIMESTRE DE EMBARAZO, QUE ACUDE CON PRUEBA
DE LABORATORIO CONFIRMATORIA DE MONILIASIS VAGINAL. AL INTERROGATORIO REFIERE INTENSO
PRURITO, SECRECIÓN BLANQUECINA ABUNDANTE CON GRUMOS.

Mujer de 38 años de edad.

Embarazo del primer trimestre.

Prurito y secreción blanquecina en grumos.

-.

Moniliasis vaginal.

120 - EL TRATAMIENTO INDICADO EN ESTE CASO SERÁ CON:

ITRACONAZOL. RECUERDA QUE LOS IMIDAZOLES ESTÁN CONTRAINDICADOS DURANTE EL EMBARAZO. El


Fluconazol excepcionalmente puede desarrollar Síndrome de Stevens–Johnson principalmente en
población de alto riego con sida y en pacientes oncológicos. Se han descrito en usos prolongados
y a altas dosis con Itraconazol hipopotasemia e hipertensión moderada. El efecto secundario más
frecuente y único entre los Azoles del voriconazol, es el trastorno reversible de la visión
(Fotopsias) que puede ocurrir hasta en el 30% de los pacientes. No se recomiendan utilizar este
grupo de fármacos, ni en el embarazo, ni en la lactancia por el posible efecto teratogénico,
embriotóxico y su posible paso a la leche materna. OJO. Tratamiento de elección para Vaginosis
por Cándida recurrente: Inducción: Itraconazol oral 200mg cada 12 horas por un día (Dosis única)
o Miconazol crema 2 %, Una aplicación intravaginal diaria por 14 días. Mantenimiento:
Ketoconazol* tabletas de 200mg, media tableta al día por 6 meses o Itraconazol* oral 50 a 100
mg diario por 6 meses o Fluconazol* cápsulas de 100 mg, una vez a la semana por 6 meses. OJO:
DEBES CONSIDERAR LA CONDICIÓN GESTANTE DE LA PACIENTE PARA ELEGIR EL TRATAMIENTO
DE ELECCIÓN.
NISTATINA La CANDIDIASIS VULVOVAGINAL se presenta con prurito y ardor vulvar intenso; una secreción
LOCAL. vaginal "blanca", "parecida a queso", y "eritema vulvar". Es posible que haya una sensación
quemante después de la micción, sobre todo si hay excoriación de la piel por el rascado. El pH
vaginal es menor a 4.5 y en la visión directa de la secreción, se observan micelios y yemas
detectados en la preparación con solución de KOH a 10% (Preparación en fresco). La prueba de
aminas siempre es negativa. Todos los Azoles tópicos y orales así como la Nistatina local, tienen
una efectividad alrededor del 80% en el tratamiento de Candidiasis Vulvovaginal no complicada.
1. Tratamiento tópico: - Miconazol crema 2%, una aplicación (5 gramos) en vulva y vagina al día,
durante 7 días o - Nistatina óvulos o tabletas vaginales de 100 000 U, una aplicación vaginal al
día, durante 14 días. IMPORTANTE: los Azoles tópicos pueden causar irritación Vulvovaginal,
misma que debe considerarse si persisten los síntomas. Daña los condones y diafragmas de látex.
2. Tratamiento oral. - Fluconazol cápsulas 250 mg en una dosis única ó Itraconazol cápsulas 200
mg cada 12 horas por 1 día. IMPORTANTE.: Contraindicados en el embarazo y lactancia. El
tratamiento de la pareja masculina asintomática, de pacientes con candidiasis vulvovaginal, no
disminuye la frecuencia de recurrencia de esta, por lo tanto, no está indicado dar tratamiento a
la(s) pareja(s) masculina(s), si ésta(s) se encuentra(n) asintomática(s). VAGINITIS RECURRENTE
POR CANDIDA SP. Tratamiento de elección para VC recurrente: - Inducción: Itraconazol oral
200mg cada 12 horas por un día (dosis única) o Miconazol crema 2%, 1 aplicación intravaginal
diaria por 14 días. - Mantenimiento: Ketoconazol tabletas de 200mg, media tableta al día por 6
meses o Itraconazol oral 50 a 100 mg diario por 6 meses o Fluconazol cápsulas de 100 mg, una
vez a la semana por 6 meses (no se use en embarazo o lactancia). A CONSIDERAR DURANTE EL
EMBARAZO: Las mujeres embarazadas asintomáticas con vaginitis por cándida no requieren
tratamiento. Use sólo tratamientos locales por 14 días en caso de que la infección sea sintomática
durante el embarazo. EL TRATAMIENTO DE ELECCIÓN DE CANDIDIASIS VAGINAL ES CON
NISTATINA A DOSIS DE 100,000UI VAGINAL POR 14 DÍAS PARA MUJERES EMBARAZADAS.

ANFOTERICINA No es indicación en esta paciente; está indicado en el tratamiento de la Candidiasis invasiva


B. grave, de Micosis sistémicas graves, tales como: Aspergilosis, Criptococosis, Histoplasmosis,
Fusariosis, Zigomicosis, Blastomicosis, Coccidioidomicosis y Fungemia por Malassezia spp,
además en el tratamiento y profilaxis de la Leishmaniasis visceral. LA ANFOTERICINA SÓLO SE
UTILIZA EN FORMAS GRAVES DE CANDIDIASIS.

NO DEBE Recuerda. Las mujeres embarazadas asintomáticas (que no es el caso de está paciente), no esta
INDICARSE indicado el tratamiento.
NINGÚN
TRATAMIENTO.

Bibliografía:
1. GUÍA DE REFERENCIA RÁPIDA, DIAGNÓSTICO Y TRATAMIENTO DE CANDIDOSIS VULVOVAGINAL EN
MUJERES MAYORES A 12 AÑOS DE EDAD. MÉXICO: SECRETARIA DE SALUD; 2010.

http://www.cenetec.salud.gob.mx/descargas/gpc/CatalogoMaestro/IMSS_609_13_CANDIDOSISVULVOVAGINAL/609GER.pdf
ANÁLISIS DEL CASO CLÍNICO

IDENTIFICACIÓN DEL REACTIVO


Area: MEDICINA INTERNA
Especialidad: NEFROLOGÍA
Tema: NEFROPATÍAS Y NEFROLITIASIS
Subtema: NEFROPATÍAS Y CALCULOS RENALES

CASO CLÍNICO SERIADO

HOMBRE DE 35 AÑOS DE EDAD, PRESENTA DESDE HACE 4 HORAS DOLOR SÚBITO E INTENSO EN FLANCO
DERECHO, CON IRRADIACIÓN A LAS REGIONES LUMBAR E INGUINAL IPSILATERAL. SE REPORTAN
LABORATORIOS CON CREATININA 1.1 MG/DL, BUN DE 20, EGO: DU 1.015, PH 5.4, ERITROCITOS ++++, LEUCOCITOS
+, BACTERIAS +. ULTRASONIDO RENAL REPORTA DILATACIÓN PIELOCALICIAL RENAL DERECHA.

HOMBRE DE 35 AÑOS DE EDAD.

-.

DOLOR SÚBITO E INTENSO EN FLANCO


DERECHO, CON IRRADIACIÓN A LAS
REGIONES LUMBAR E INGUINAL
IPSILATERAL. SINTOMATOLOGÍA CLÁSICA
DEL CÓLICO RENOURETERAL.

-.

CREATININA 1.1 MG/DL, BUN DE 20, EGO:


DU 1.015, PH 5.4, ERITROCITOS ++++,
LEUCOCITOS +, BACTERIAS +.
ULTRASONIDO RENAL REPORTA
DILATACIÓN PIELOCALICIAL RENAL
DERECHA. EL USG REPORTA
CLARAMENTE UNA DILATACIÓN,
SECUNDARIA A OBSTRUCCIÓN.

121 - POR FRECUENCIA LOS CÁLCULOS ENCONTRADOS EN EL PACIENTE ESTARÁN COMPUESTOS POR:
ÁCIDO La LITIASIS URINARIA es definida como la presencia de cálculos que se forman en el aparato urinario,
ÚRICO. según define la OMS, se describe como un proceso físico, por medio de una compleja cascada de
eventos que ocurre en el filtrado glomerular a través de la nefrona, y que se desarrollan en las
cavidades renales debido a sobresaturación de sales como son iones disueltos y moléculas que se
precipitan en solución formando cristales o núcleos, los cuales pueden fluir en la orina o quedar
retenidos en el riñón o anclados en las cavidades renales, promoviendo el crecimiento y la
agregación de solutos, fenómeno conocido como epitaxia, hasta llevar a la formación del cálculo. Los
cálculos de ácido úrico son radiolúcidos y también más frecuentes en varones. La mitad de los
pacientes que los sufren experimentan gota; la litiasis de ácido úrico suele ser familiar, con gota o sin
ella. NO SE REFIERE EL ANTECEDENTE DE HIPERURICEMIA.

CISTINA. LOS CÁLCULOS DE CISTINA SON POCO FRECUENTES; su radioopacidad se debe al azufre que
contienen. Aparecen en la orina como placas hexagonales planas.

ESTRUVITA. Los cálculos de estruvita son frecuentes y potencialmente peligrosos. Estos cálculos, que se forman
casi siempre en "mujeres" o en enfermos que precisan cateterismo crónico de vejiga, son el resultado
de una infección de las vías urinarias por bacterias productoras de ureasa, generalmente especies de
Proteus. Los cálculos pueden crecer hasta alcanzar un gran tamaño y llenar la pelvis y los cálices
renales, en la forma de "asta de venado". Son radiopacos y tienen densidad interna variable. En la
orina, los cristales de estruvita son prismas rectangulares que han sido comparados con tapas de
ataúdes. LOS CÁLCULOS DE ESTRUVITA SON MÁS FRECUENTES EN MUJERES.

CALCIO. Sales de calcio, ácido úrico, cistina y estruvita son la base de la mayor parte de los cálculos renales
detectados en el hemisferio occidental. Los cálculos de OXALATO DE CALCIO y los de FOSFATO
CÁLCICO constituyen "75 a 85% del total", y ambos compuestos pueden estar mezclados en el mismo
cálculo. El fosfato de calcio de los cálculos suele ser hidroxiapatita [Ca5 (PO4)3OH] o, con menos
frecuencia, brushita (CaHPO4H2O). Los cálculos de calcio son más frecuentes en los varones; la edad
media de inicio es del tercero al cuarto decenios de la vida. Alrededor de 50% de las personas que
producen un solo cálculo de calcio acaban desarrollando otro en los 10 años siguientes. La cifra
media de formación de nuevos cálculos en pacientes que antes han formado uno es de
aproximadamente un cálculo cada dos o tres años. Con frecuencia la litiasis de calcio es una
enfermedad familiar. LA MAYOR PARTE DE LOS CÁLCULOS RESPONSABLES UROLITIASIS ESTÁN
FORMADOS DE CALCIO.

Bibliografía:
1. GUÍA DE PRÁCTICA CLÍNICA, DIAGNÓSTICO Y TRATAMIENTO DE LA UROLITIASIS EN EL ADULTO. MÉXICO:
SECRETARIA DE SALUD, 2009. 2. PAPADAKIS MAXINE A, MCPHEE STEPHEN J. DIAGNÓSTICO CLÍNICO Y
TRATAMIENTO. 52ª EDICIÓN. NUEVA YORK. 2013, PP 946.

http://www.cenetec.salud.gob.mx/descargas/gpc/CatalogoMaestro/215_SSA_09_urolitiasis/EYR_SSA_215_09.pdf

122 - EL ESTUDIO DIAGNÓSTICO MÁS SENSIBLE PARA CORROBORAR EL DIAGNÓSTICO DEL PACIENTE ES:
TAC HELICOIDAL NO El cólico renal (CR) es una urgencia urológica frecuente, que comporta una de las formas
CONTRASTADA. más angustiantes de dolor en el ser humano, por lo que requiere de un rápido diagnóstico y
tratamiento. Más de un 12% de la población sufrirá un CR durante su vida, siendo la tasa de
recurrencia alrededor de un 50%. Es causado por una obstrucción uretral aguda, parcial o
completa, que en la gran mayoría de casos suele ser debida a un cálculo, provocando una
distensión aguda del sistema colector. TOMOGRAFÍA HELICOIDAL SIN CONTRASTE: varios
estudios recientes han confirmado a esta exploración como una modalidad precisa para la
evaluación del cólico renal, con una sensibilidad de hasta el 98% y una especificidad de
hasta un 100%. No requiere de contraste, siendo especialmente atractiva en pacientes
alérgicos o con insuficiencia renal previa. Puede visualizar pequeños cálculos, incluidos los
radiotransparentes, y si la litiasis es suficientemente pequeña para no ser identificada, se
observan múltiples signos indirectos como dilatación uretral y renal, trazos de líneas de
grasa perirrenal, líquido perirrenal y el signo del anillo en las partes blandas (visualización
de un círculo de tejido blando que envuelve al cálculo, que representa la pared edematosa
del uréter, permitiendo diferenciar los cálculos de los flebolitos). Es un procedimiento
rápido de tan solo 5 minutos, y que en algunos sistemas sanitarios se ha determinado que
no cuesta más que la UIV33. Además, cuando no existe litiasis identifica con exactitud otras
alteraciones urinarias y no urinarias. POR SU ALTA SENSIBILIDAD Y ESPECIFICIDAD SE
CONSIDERA ACTUALMENTE EL ESTUDIO DIAGNÓSTICO DE ELECCIÓN EN LOS CENTROS
HOSPITALARIOS DONDE SE CUENTE CON EL ESTUDIO.

ULTRASONOGRAMA ECOGRAFÍA ABDOMINAL: es un método incruento, rápido, portátil, repetible, relativamente


RENAL. barato y que no usa radiaciones ionizantes ni material de contraste. Nos aporta información
sobre el estado de la vía por encima de la obstrucción, informando del grado de ectasia
pielocalicial del riñón afecto (se considera que una separación mayor de 5 mm de los ecos
en el seno renal constituye un signo indirecto de obstrucción), pudiendo diagnosticar,
además, litiasis radiotransparentes no visibles en la radiografía simple, o procesos tumorales
que comprometan el vaciado uretral. El uso del Doppler permitirá estudiar el índice de
resistencia renal (parámetro que permite detectar obstrucción con alta sensibilidad y
especificidad, observándose una disminución del índice del riñón obstruido respecto al
contralateral, y que incluso permitiría diferenciar, si el proceso es de instauración aguda o
crónica) y la evaluación de los jets ureterales (presencia y simetría de los eyaculados de
ambos uréteres). Como deben transcurrir horas para que se desarrolle una pielocaliectasia
manifiesta después de una obstrucción súbita e incluso completa, no resulta sorprendente
que la ecografía pase por alto el 20-30% de les obstrucciones agudas causadas por un
cálculo uretral. Además, se pueden establecer diagnósticos falsamente positivos de
obstrucción en pacientes con pielonefritis, ectasias crónicas residuales, reflujo
vesicoureteral, dilatación residual después de aliviar la obstrucción o sobredistención de la
vejiga, sobretodo en mujeres. También debe destacarse la disminución de su eficacia
diagnóstica en uréter medio y distal, siendo útil para valorar causas obstructivas en el uréter
yuxtavesical con la vejiga repleccionada. Por tanto, resulta de gran utilidad la combinación
de la radiografía simple de abdomen y de la ecografía, con demostración de los cálculos y la
pielocaliectasia, en el diagnóstico de cólico renal agudo. ES EL MÁS OCUPADO POR SU
COSTO, NO ES EL MÁS ESPECÍFICO NI SENSIBLE.

RADIOGRAFÍA DE RADIOGRAFÍA SIMPLE DE ABDOMEN: es el estudio de imagen más rápido y sencillo en los
ABDOMEN. pacientes con cólico renal. Esta prueba detecta los cálculos ureterales con una sensibilidad
que varía del 45 al 59%, por lo que la su utilidad es limitada. La superposición del intestino y
de los huesos oculta algunos cálculos, y los flebolitos (perlas venosas calcificadas, redondas,
múltiples, con frecuencia muy excéntricas, con menor densidad central, y muy frecuentes en
los espacios paravesicales), se pueden confundir con litiasis. Además, la radiografía simple
no detecta cálculos radiotransparentes (ácido úrico puro), litiasis menores de 2 mm o
microlitiasis (popularmente llamadas “arenilla”), ni la causa obstructiva cuando no es
litiásica (hasta un 10% de casos), y la presencia de una calcificación abdominal no es un
signo seguro de que esté localizada en el aparato urinario. Permite ver cálculos a partir de 2
mm si son de calcio y a partir de 3-4 mm si son de densidad más baja (estruvita o cistina).
Además, podemos observar, una ligera escoliosis de concavidad hacia el lado afecto, un
discreto aumento de tamaño de la silueta del riñón afectado y posibles imágenes radiopacas
sugestivas de cálculos a nivel renal o ureteral. LA SENSIBILIDAD DE LOS RX PARA EL
DIAGNÓSTICO DE LITIASIS RENOURETERAL ES MUY BAJA (menos del 60%).
UROGRAFÍA UROGRAFÍA INTRAVENOSA: durante mucho tiempo se ha considerado el método
EXCRETORA. diagnóstico de elección para estudiar a los pacientes con cólico renal. Está al alcance de
todos los hospitales, es fiable, barato y relativamente inocuo. Los hallazgos clásicos en la
obstrucción renal aguda consisten en, retraso en la aparición del nefrograma, que adquiere
una densidad cada vez mayor en las imágenes posteriores, retraso en la aparición del
contraste en el sistema pielocalicial, dilatación ureteral proximal en la zona de la
obstrucción y posible identificación de la causa. Tiene el inconveniente de las reacciones
alérgicas debidas al contraste, que han disminuido con los contrastes de baja osmolaridad.
Muchos centros siguen considerando la UIV como el método de referencia para diagnosticar
el cólico renal agudo. No obstante, con la introducción de la ecografía Doppler con
medición del índice de resistencia renal y la TAC helicoidal sin contraste con reconstrucción
tridimensional, la UIV tenderá a usarse cada vez menos. EN CASO DE NO ESTAR LA TAC
HELICOIDAL, ESTA SERÍA LA RESPUESTA CORRECTA.

Bibliografía:
1. GUÍA DE REFERENCIA RÁPIDA, DIAGNÓSTICO Y TRATAMIENTO DE LA UROLITIASIS EN EL ADULTO.
MÉXICO: SECRETARIA DE SALUD, 2009. 2. RESUMEN DE EVIDENCIAS Y RECOMENDACIONES,
DIAGNÓSTICO Y TRATAMIENTO DE LA UROLITIASIS EN EL ADULTO. MÉXICO: SECRETARIA DE SALUD, 2009.

http://www.cenetec.salud.gob.mx/descargas/gpc/CatalogoMaestro/215_SSA_09_urolitiasis/EYR_SSA_215_09.pdf

FIN DEL CASO CLÍNICO SERIADO


ANÁLISIS DEL CASO CLÍNICO

IDENTIFICACIÓN DEL REACTIVO


Area: CIRUGÍA
Especialidad: TRAUMATOLOGÍA Y ORTOPEDIA
Tema: CERVICALGIAS, LUMBALGIAS, TRAUMA MIEMBROS SUP Y TUMORES
Subtema: CERVICALGIA

CASO CLÍNICO SERIADO

RECIÉN NACIDO DE 28 DÍAS, LLEVADO A CONSULTA POR TENER LA CABEZA INCLINADA AL LADO IZQUIERDO
Y LA BARBILLA GIRADA A LA DERECHA. NO DOLOR NI SINTOMATOLOGÍA ACOMPAÑANTE.

reciÉn nacido.

cabeza inclinada al lado izquierdo y la


barbilla girada a la derecha.

123 - EL DIAGNÓSTICO PROBABLE DEL RECIÉN NACIDO ES:

AGENESIA DE En el caso en que se sospeche AGENESIA DEL MÚSCULO TRAPECIO como etiología causal de la
TRAPECIO. deformidad comentada, seria evidente a la exploración física la "ausencia de la masa muscular del
trapecio" puesto que en la parte posterior del cuello el musculo trapecio contralateral daría la
imagen de una tumoración en comparación a donde no se encuentra el musculo en cuestión. Cabe
mencionar que la función del trapecio es la de rotador y elevador de la cabeza por lo que si faltara
uno de los dos el contralateral ejercería tracción contraria (rotación) con respecto al agenésico, así
pues se pudiera tomar esta respuesta como válida pero nuevamente haciendo hincapié en los
datos que nos menciona el caso clínico, no podríamos concluir en que esta fuera la mejor
respuesta, además de que una agenesia de este musculo sería muy POCO FRECUENTE en el recién
nacido en comparación con la frecuencia con que se presenta la tortícolis.
TUMORACIÓN NO SE COMENTA EN EL CASO PRESENCIA DE TUMORACIÓN, es importante saber que en
C1-C2. ocasiones se presentaran distractores como el presente pero debemos enfocarnos al
planteamiento del problema y a los datos que nos están ofreciendo. Para determinar una
tumoración, en primer lugar se debería mencionar en la exploración física la presencia de una
masa a nivel cervical palpable o de lo contrario algún estudio de imagen que nos hiciera sospechar
de tal patología lo cual no se presenta en el caso.

KLIPPEL-FEIL. El SÍNDROME DE KLIPPEL-FEIL (BREVÍCOLIS) es una rara enfermedad, reportada originalmente en


1912 por Maurice Klippel y André Feil, que se caracteriza por la FUSIÓN CONGÉNITA DE 2 A 7
VÉRTEBRAS CERVICALES. De hecho, el síndrome de Klippel-Feil ocurre en un grupo heterogéneo
de pacientes unificado solo por la presencia de un defecto congénito en la formación o la
segmentación de la columna cervical. Los signos más comunes del trastorno son un "cuello corto",
"baja implantación del cabello" en la parte posterior de la cabeza, y la "movilidad restringida de la
columna vertebral superior". Anomalías asociadas • escoliosis (curvatura de lado a lado de la
columna vertebral) • espina bífida • anomalías de los riñones y las costillas • paladar hendido •
algunos problemas respiratorios • malformaciones del corazón El trastorno también puede estar
asociado con anomalías de la cabeza y la cara, el esqueleto, los órganos sexuales, los músculos, el
cerebro y la médula espinal, los brazos, las piernas y los dedos ya que en el caso clínico se
menciona claramente la posición del cuello la cual es típica de la torticolis y en el sx de klippel aun
que se afecta el cuello se menciona como cuello corto no debemos tomar esta como una opción
adecuada de respuesta.

TORTÍCOLIS La TORTÍCOLIS es un tipo de distonía (contracciones musculares prolongadas) en que los músculos
CONGÉNITA. del cuello, particularmente el músculo esternocleidomastoideo, se contraen involuntariamente y
hacen que se incline la cabeza. Existe la siguiente teoría respecto a la etiología de la torticolis:
Como consecuencia de la hiperextensión del músculo en presentaciones podálicas o en expulsivos
difíciles, se rompe la fascia o el propio músculo, formándose un hematoma que, al ser invadido
después por fibrina y fibroblastos, acaba cicatrizando con acortamiento del músculo. Aunque se
puede detectar una masa ya al nacer en la zona media del músculo, es más frecuente no
evidenciarla hasta los 10-14 días de vida. Suele ser de 1-2 cm de diámetro, dura, inmóvil, bien
delimitada y sin aspecto inflamatorio o cambios de coloración en la piel circundante. En las 2-4
semanas siguientes aumenta de tamaño la masa, disminuyendo posteriormente, para desaparecer
a los 5-8 meses. Existe un 40% de casos en los que no se presenta masa, solo contractura muscular.
LA CONTRACTURA DEL MÚSCULO TRAS EL NACIMIENTO INCLINA LA CABEZA HACIA EL LADO
AFECTO Y LA ROTA EN DIRECCIÓN OPUESTA, NO PUDIENDO SER MOVIDA PASIVAMENTE A SU
POSICIÓN NORMAL así pues se explica la presentación clínica del paciente en el que se menciona
la inclinación de la cabeza y la rotación del cuello hacia el lado contra lateral por lo tanto esta es la
respuesta válida.

Bibliografía:
1. BRUNICARDI F, ANDERSEN D, BILLIAR T, Y COLS. SCHWARTZ PRINCIPIOS DE CIRUGÍA, 9A EDICIÓN. MC
GRAW HILL. 2011, PP 1416. 2. DOHERTY G. DIAGNÓSTICO Y TRATAMIENTO QUIRÚRGICO, 13A EDICIÓN. MC
GRAW HILL LANGE. 2011, PP 1013-1014.

124 - EL TRATAMIENTO QUE DEBE RECIBIR EL PACIENTE ES:

EJERCICIOS DE El tratamiento de la TORTÍCOLIS EN EL RECIÉN NACIDO es la rehabilitación precoz mediante


ESTIRAMIENTO. movimientos de flexo-extensión, lateralización y rotación del cuello, realizados con el paciente
en decúbito supino sobre una superficie dura. Si después de 6 meses de una rehabilitación
constante y bien realizada no se consigue una mejoría importante y el paciente sigue con su
malposición, evolucionando a la plagiocefalia, entonces está indicada la cirugía (sección de
elongación del esternocleidomastoideo).

RESECCIÓN En el determinado caso en que se sospeche de una TUMORACIÓN, se deberá realizar


QUIRÚRGICA Y primeramente una toma de biopsia para determinar el procedimiento a seguir, si las condiciones
FUSIÓN ÓSEA. del paciente y la estirpe del tumor lo permiten estaría indicada la resección del mismo y
dependiendo de la estabilidad de la columna cervical posterior podríamos pensar en hacer una
fusión vertebral aun que ese no es el caso del presente tema.
RESECCIÓN EN MEMBRANAS CUTÁNEAS. No se presenta dicha opción de diagnóstico previamente aunque
Z DE puede ser la causa de la inclinación del cuello y rotación de la cabeza en cuyo caso si estaría
MEMBRANAS indicada la realización de Z plastias cutáneas para elongar la zona con lo cual se resolvería tal
CUTÁNEAS. deformidad.

FIJACIÓN Casos como el KLIPPEL-FEIL en los que puede existir una anomalía en la segmentación cervical
VERTEBRAL. pudiera existir la posibilidad de tratamiento quirúrgico como la corrección de la alineación
cervical en el plano sagital y coronal así como su posterior fusión vertebral aunque este no es el
caso en cuestión.

Bibliografía:
1. BRUNICARDI F, ANDERSEN D, BILLIAR T, Y COLS. SCHWARTZ PRINCIPIOS DE CIRUGÍA, 9A EDICIÓN. MC
GRAW HILL. 2011, PP 1416. 2. DOHERTY G. DIAGNÓSTICO Y TRATAMIENTO QUIRÚRGICO, 13A EDICIÓN. MC
GRAW HILL LANGE. 2011, PP 1013-1014.

FIN DEL CASO CLÍNICO SERIADO


ANÁLISIS DEL CASO CLÍNICO

IDENTIFICACIÓN DEL REACTIVO


Area: MEDICINA INTERNA
Especialidad: PSIQUIATRÍA
Tema: TRANSTORNOS DE ANSIEDAD
Subtema: TRANSTORNO OBSESIVO-COMPULSIVO

CASO CLÍNICO CON UNA PREGUNTA

FEMENINA DE 56 AÑOS DE EDAD QUE VISITA POR DÉCIMA VEZ AL MÉDICO HABIENDO VISITADO POR LO
MENOS A 8 MÉDICOS MÁS EN LOS ÚLTIMOS 2 AÑOS DEBIDO A QUE DICE NO PRESENTAR NINGUNA MEJORÍA
DE SUS SÍNTOMAS. DESDE HACE MÁS DE 10 AÑOS PRESENTA CERVICALGIA Y DOLOR EN LAS PIERNAS, POR
LO QUE NO HACE EJERCICIO, TAMBIÉN REFIERE DISTENSIÓN ABDOMINAL, MAREO Y VÉRTIGO FRECUENTES.
NUNCA TUVO PAREJA NI HIJOS. A LA EXPLORACIÓN FÍSICA USTED LA ENCUENTRA NORMAL Y LOS EXÁMENES
DE LABORATORIO SE REPORTAN NORMALES.

femenina de 56 años.

8 mÉdicos visitados en los Últimos 2 años.

10 años de cervicalgia y dolor en las piernas.

normal.

normales.

125 - EL DIAGNÓSTICO CLÍNICO MÁS PROBABLE ES:


TRASTORNO POR Los rasgos principales de la SOMATIZACIÓN son síntomas físicos múltiples, recurrentes y
SOMATIZACIÓN. frecuentemente cambiantes de al menos dos años de duración. La mayoría de pacientes tienen
una larga y complicada historia de contactos con servicios de salud de atención primaria y
especializada, durante los cuales se han llevado a cabo múltiples pruebas negativas y
operaciones exploratorias infructuosas. Criterios a considerar para el trastorno de
somatización: 1. Diversas molestias físicas, en menores de 30 años, con mayor frecuencia en
mujeres, de forma crónica, que lleva a buscar atención médica y que puede provocar deterioro
en la productividad y desarrollo personal y social del paciente. 2. Criterios principales: - Cuatro
síntomas dolorosos (relacionado con cuatro localizaciones o funciones distintas). - Dos
síntomas digestivos (distintos a dolor). - Un síntoma sexual. - Un síntoma seudoneurológico
(al menos un síntoma o déficit que sugiera proceso neurológico no limitado al dolor). 3.
Alguno de los siguientes: - Después de una exploración exhaustiva no es integra ningún
síndrome orgánico que justifique los síntomas. - En presencia de una enfermedad orgánica, los
datos clínicos son de mayor severidad y discordantes con la exploración física y los datos de
laboratorio. 4. Los trastornos no son intencionados ni fingidos como ocurre en el trastorno
simulado o de fingimiento. EL TIEMPO DE EVOLUCIÓN CON MÚLTIPLES SÍNTOMAS CON
PRUEBAS NEGATIVAS PARA PATOLOGÍA ORGÁNICA SON LA BASE PARA SOSPECHAR QUE SE
TRATA DE SOMATIZACIÓN.

FIBROMIALGIA. La FIBROMIALGIA está considerada como un síndrome doloroso crónico generalizado de más
de 3 meses de duración, que afecta al sistema músculo esquelético y presenta una elevada
sensibilidad a la presión de determinadas zonas. Este síntoma está acompañado de sueño no
reparador y fatigabilidad que afecta de forma severa a la salud. NO COINCIDE CON EL
CUADRO CLÍNICO NI EL TIEMPO DE EVOLUCIÓN DE LA PACIENTE.

HIPOCONDRIASIS. En los pacientes con HIPOCONDRIASIS, lo esencial es una preocupación persistente por la
posibilidad de tener uno o más trastornos físicos graves y progresivos. Los pacientes
manifiestan continuamente quejas somáticas o preocupación por su aspecto físico. NO SE
ENCUENTRA EN LA PACIENTE UNA PREOCUPACIÓN POR SU ASPECTO FÍSICO.

TRASTORNO En el TRASTORNO CONVERSIVO, el aspecto común es la pérdida parcial o completa de la


CONVERSIVO. integración normal entre ciertos recuerdos del pasado, conciencia de la propia identidad,
ciertas sensaciones inmediatas y el control de los movimientos corporales. NO HAY PÉRDIDA
DE RECUERDOS.

Bibliografía:
1. LONGO DL, FAUCI AS, KASPER DL, HAUSERSL, JAMESON JL, LOSCALZOJ. HARRISON. PRINCIPIOS DE
MEDICINA INTERNA, 18A EDICIÓN. MC GRAW HILL. NEW YORK, USA. 2012, PP 3541. 2. PAPADAKIS MAXINE
A, MCPHEE STEPHEN J. DIAGNÓSTICO CLÍNICO Y TRATAMIENTO. 52ª EDICIÓN. NUEVA YORK. 2013, PP 1045.
ANÁLISIS DEL CASO CLÍNICO

IDENTIFICACIÓN DEL REACTIVO


Area: CIRUGÍA
Especialidad: CIRUGÍA ABDOMINAL
Tema: PANCREATITIS
Subtema: PANCREATITIS AGUDA

CASO CLÍNICO SERIADO

HOMBRE DE 56 AÑOS DE EDAD, ACUDE AL SERVICIO DE URGENCIAS POR PRESENCIA DE DOLOR


ABDOMINAL INTENSO, INCAPACITANTE Y QUE IMPIDE LA DEAMBULACIÓN. A LA EXPLORACIÓN PRESENTA
ABDOMEN DISTENDITO, EXTREMADAMENTE DOLOROSO A LA PALPACIÓN Y LA PERISTALSIS SE ENCUENTRA
DISMINUIDA. SE SOLICITA RX ABDOMINAL ENCONTRÁNDOSE LA SIGUIENTE IMAGEN.

56 años.

-.

dolor abdominal intenso, incapacitante,


que impide la deambulación.

abdomen globoso, distendido, peristalsis


disminuida.

-.

126 - LA IMAGEN RADIOLÓGICA OBTENIDA APOYA EL DIAGNÓSTICO DE:


APENDICITIS La APENDICITIS AGUDA es la inflamación del apéndice cecal o vermiforme, que inicia con
AGUDA. obstrucción de la luz apendicular, lo que trae como consecuencia un incremento de la presión
intramural por el acúmulo de moco asociado con poca elasticidad de la serosa. En la Rx de
abdomen puede encontrarse la presencia de borramiento del psoas derecho, escoliosis antiálgica,
borramiento de la grasa preperitoneal y prevesical, aire libre, fecalito, nivel hidroaéreo en fosa
iliaca derecha, neumoápendice, ileo generalizado, etc. LA RADIOGRAFÍA NO PRESENTA ESTOS
SIGNOS RADIOGRÁFICOS.

ÚLCERA La ÚLCERA PÉPTICA consiste en una solución de continuidad de la mucosa gástrica o duodenal
PÉPTICA que se extiende en profundidad a través de la mucosa muscular y que permanece en función de la
PERFORADA. actividad ácido-péptica. Se le llama ULCERA PÉPTICA COMPLICADA al daño necrótico de la
mucosa que se extiende más allá de la mucosa muscular produciendo una lesión excavada
secundaria que puede causar sangrado o perforación. El examen radiológico en el caso de una
ÚLCERA PERFORADA presenta como signos más importante neumoperitoneo caracterizado por
aire libre bajo el diafragma. NO CORRESPONDE A LOS HALLAZGOS RADIOGRÁFICOS
PRESENTADOS.

PANCREATITIS. La PANCREATITIS aguda es el proceso inflamatorio agudo del páncreas que frecuentemente
involucra tejido peripancreático y puede involucrar órganos y sistemas distales. Los pacientes
habitualmente refieren dolor abdominal intenso en la región superior del abdomen, transfictivo
al dorso, acompañado de anorexia y vómito. - La Rx de tórax permite visualizar elevación de
hemidiafragmas, derrame pleural, atelectasia o neumonía; la presencia de derrame pleural en uno
o ambos pulmones se asocia con pobre desenlace. - Las Rx simples de abdomen pueden mostrar
cálculos biliares, asa centinela (un segmento del intestino lleno de aire, generalmente en
cuadrante superior izquierdo), signo de supresión del colon o signo de colon cortado (segmento
de porción transversa de colon lleno de gas que termina de manera súbita en el área de
inflamación pancreática), o aumento del espacio gastrocólico. LA RADIOGRAFÍA MUESTRA
CLARAMENTE AUMENTO DEL ESPACIO GASTROCÓLICO, COMPATIBLE ÚNICAMENTE CON ESTE
DIAGNÓSTICO.

COLELITIASIS La COLECISTITIS es la inflamación de la vesícula biliar ocasionada principalmente por cálculos


AGUDA. (litos), con menor frecuencia por barro (lodo) biliar y en raras ocasiones ninguna de estas
condiciones está en el paciente. La colelitiasis es la presencia de litos (cálculos) en la vesícula
biliar. Ante la sospecha de cualquiera de estas patologías el método diagnóstico de elección es el
ultrasonido. Los Rx pueden mostrar imagen con vesícula en porcelana en casos de colecistitis
crónica, o la presencia de litos en solo algunos de los casos (sobre todo cuando son de gran
tamaño). NO CORRESPONDE A LA IMAGEN MOSTRADA, EL DIAGNÓSTICO DE COLELITIASIS ES
MUY COMPLICADO POR ESTE MÉTODO YA QUE DEPENDE DEL TAMAÑO Y CONTENIDO CÁLCICO
DE LOS LITOS.

Bibliografía:
1. GUÍA DE PRÁCTICA CLÍNICA, DIAGNÓSTICO Y TRATAMIENTO DE LA PANCREATITIS AGUDA. MÉXICO:
SECRETARIA DE SALUD; 2009. RECUPERADO DE
HTTP://WWW.CENETEC.SALUD.GOB.MX/CONTENIDOS/GPC/CATALOGOMAESTROGPC.HTML 2. LONGO DL,
FAUCI AS, KASPER DL, HAUSERSL, JAMESON JL, LOSCALZOJ. HARRISON. PRINCIPIOS DE MEDICINA
INTERNA, 18A EDICIÓN. MC GRAW HILL. NEW YORK, USA. 2012. 3. PAPADAKIS MAXINE A, MCPHEE
STEPHEN J. DIAGNÓSTICO CLÍNICO Y TRATAMIENTO. 52ª EDICIÓN. MC GRAW HILL EDUCATION, LANGE.
USA. 2013.

http://www.cenetec.salud.gob.mx/descargas/gpc/CatalogoMaestro/239_PANCREATITIS_AGUDA/Pancreatitis_aguda_evr_cenetec.pdf

127 - UNA VEZ ESTABLECIDO EL DIAGNÓSTICO, DEBERÁ INDICARSE EL SIGUIENTE TRATAMIENTO:

REPOSO PANCREATITIS. - En general, en todos los grados de pancreatitis se recomienda el reposo


PANCREÁTICO. pancreático mediante la supresión de líquidos y alimentos vía oral y reposo en cama. -
Actualmente se recomienda la nutrición enteral temprana mediante sonda nasoyeyunal o
transpilórica y en casos de cirugía con sonda de yeyunostomía. - Se recomienda la
intervención quirúrgica en pancreatitis necrótica infectada o colangitis aguda ya que de no
realizarse, la mortalidad es cercana al 100%. En necrosis pancreática estéril debe tratarse
médicamente, ya que en estos casos no se recomienda el debridamiento o drenaje. EL REPOSO
PANCREÁTICO ES LA PIEDRA ANGULAR EN EL MANEJO DE LA PANCREATITIS AGUDA.
COLECISTECTOMÍA COLELITIASIS AGUDA: La colecistectomía laparoscópica es el tratamiento de elección en la
LAPAROSCÓPICA. mayoría de los pacientes; sin embargo, el tratamiento dependerá de la gravedad de la
colecistitis y de la habilidad del cirujano.

HEMOSTASIA ÚLCERA PÉPTICA PERFORADA: El sangrado de la úlcera péptica, puede cesar


ENDOSCÓPICA. espontáneamente hasta en 80% de los casos; cuando el sangrado es leve, no hay repercusión
hemodinámica y el paciente se puede recuperar sin intervención específica. Cuando hay
sangrado profuso con alteraciones hemodinámicas el primer paso es estabilizar al paciente,
administrar inhibidores de la bomba de protones y realizar la hemostasia endoscópica de la
lesión.

APENDICECTOMÍA APENDICITIS AGUDA: la apendicectomía laparoscópica se considera de primera elección sobre


ABIERTA. todo en mujeres jóvenes en edad fértil, además de la menor incidencia de complicaciones. Su
uso versus apendicectomía abierta, dependerá de la habilidad del cirujano y la gravedad de la
apendicitis.

Bibliografía:
1. GUÍA DE PRÁCTICA CLÍNICA, DIAGNÓSTICO Y TRATAMIENTO DE LA PANCREATITIS AGUDA. MÉXICO:
SECRETARIA DE SALUD; 2009. RECUPERADO DE
HTTP://WWW.CENETEC.SALUD.GOB.MX/CONTENIDOS/GPC/CATALOGOMAESTROGPC.HTML 2. LONGO DL,
FAUCI AS, KASPER DL, HAUSERSL, JAMESON JL, LOSCALZOJ. HARRISON. PRINCIPIOS DE MEDICINA
INTERNA, 18A EDICIÓN. MC GRAW HILL. NEW YORK, USA. 2012. 3. PAPADAKIS MAXINE A, MCPHEE
STEPHEN J. DIAGNÓSTICO CLÍNICO Y TRATAMIENTO. 52ª EDICIÓN. MC GRAW HILL EDUCATION, LANGE.
USA. 2013.

http://www.cenetec.salud.gob.mx/descargas/gpc/CatalogoMaestro/239_PANCREATITIS_AGUDA/Pancreatitis_aguda_evr_cenetec.pdf

FIN DEL CASO CLÍNICO SERIADO


ANÁLISIS DEL CASO CLÍNICO

IDENTIFICACIÓN DEL REACTIVO


Area: MEDICINA INTERNA
Especialidad: GERIATRÍA Y GERONTOLOGÍA
Tema: MALTRATO EN EL ADULTO MAYOR
Subtema: MALTRATO EN EL ADULTO MAYOR

CASO CLÍNICO CON UNA PREGUNTA

MUJER DE 84 AÑOS DE EDAD, QUE ES LLEVADA A URGENCIAS CON DIAGNÓSTICO DE CRISIS HIPERTENSIVA.
DURANTE SU VALORACIÓN USTED LA ENCUENTRA CON LIENDRES Y PIOJOS EN EL PELO.

Mujer de 84 años de edad.

llega al hospital con crisis hipertensiva.

-.

SE ENCUENTRA CON LIENDRES Y PIOJOS


EN EL PELO.

-.

128 - ESTE DATO EN LA PACIENTE PODRÍA APOYAR LA PRESENCIA DEL SIGUIENTE TIPO DE ABUSO:

PSICOLÓGICO El MALTRATO AL ADULTO MAYOR lo constituye cualquier acto u omisión que tenga como
Y resultado un daño o que vulnere o ponga en peligro la integridad física o psíquica, así como el
EMOCIONAL. principio de autonomía y respeto de los derechos fundamentales del individuo de 60 años y más, el
cual puede ocurrir en el medio familiar, comunitario o institucional. El maltrato se puede dar en
una o varias esferas de la vida, entre las más frecuentes se encuentran: maltrato físico, psicológico
o emocional, financiero, sexual, negligencia y abandono. Algunos de los hallazgos que podemos
encontrar en pacientes sometidos a ABUSO EMOCIONAL, son sensación de resignación y
desesperación con referencias vagas al maltrato, introversión, conducta pasiva, de indefensión,
cambio de comportamiento llamativo reciente, cambios en el patrón de alimentación habitual,
manifestados por baja de peso. Ansiedad, temblor, actitud temerosa de algo y/o alguien,
autoinculpación acerca de su situación actual, pérdida de memoria llamativa no explicable por otra
razón.
FÍSICO. Los principales hallazgos en las persona mayores por MALTRATO FÍSICO son higiene pobre,
vestimenta sucia o inapropiada, pérdida de peso, caquexia. Se pueden encontrar alteraciones en los
signos vitales tensión arterial baja, taquicardia, hipo o hipertermia. Algunos signos en la
exploración física son equimosis en cavidad oral, quemaduras en labios, falta de piezas o fracturas
dentales, desviación del tabique nasal, lesiones en región cervical, hemorragias oculares, alopecia
traumática, quemaduras, laceraciones, hematomas, úlceras por presión en mal estado, heridas
cortantes, coexistencia de lesiones recientes con lesiones de aspecto antiguo, fracturas costales,
neumotórax, rotura de vísceras, fracturas óseas, lesiones musculares, lesiones en área genital,
confusión, desorientación, sobre-sedación, temor, ansiedad, sintomatología depresiva.

NEGLIGENCIA En la víctimas de NEGLIGENCIA Y ABANDONO, se pueden encontrar con frecuencia úlceras por
Y presión, deshidratación, sensación de impotencia, prendas de vestir inadecuadas, "falta de
ABANDONO. higiene", ausencia de servicios básicos en su vivienda, falta apoyo en salud (gafas, audífonos o
prótesis dentales) piojos y sarna, desnutrición, encierro, descuido general, mal olor, preescripciones
médicas mal hechas, condiciones médicas no tratadas, pérdida de peso.

FINANCIERO. En éste tipo de abuso, generalmente a la exploración física, no encontramos mucha evidencia, solo
si ya se conoce con anterioridad al adulto mayor podría verse en su ropa y calzado desgaste.

Bibliografía:
1. GUÍA DE PRÁCTICA CLÍNICA, DETECCIÓN Y MANEJO DEL MALTRATO EN EL ADULTO MAYOR. MÉXICO:
SECRETARIA DE SALUD, 2013.

http://www.cenetec.salud.gob.mx/descargas/gpc/CatalogoMaestro/057_GPC_MaltratoAdultoMayor/MAM_EVR_CENETEC.pdf
ANÁLISIS DEL CASO CLÍNICO

IDENTIFICACIÓN DEL REACTIVO


Area: PEDIATRÍA
Especialidad: INFECTOLOGIA PEDIÁTRICA
Tema: GASTROENTERITIS INFECCIOSA
Subtema: GEPI BACTERIANA Y PARASITARIAS

CASO CLÍNICO CON UNA PREGUNTA

ESCOLAR DE 10 AÑOS DE EDAD, VIVE EN HACINAMIENTO EN MEDIO RURAL. ACUDE A LA CONSULTA Y LE


DIAGNOSTICA AMIBIASIS INTESTINAL. USTED INICIA TRATAMIENTO CON METRONIDAZOL.

escolar de 10 años

hacinamiento y proveniente de medio rural

amibiasis intestinal, recibiendo manejo con


metronidazol

129 - DADO A QUE SU MECANISMO DE ACCIÓN ES:

UNIÓN A LA BETA MEBENDAZOL. Mecanismo de acción: • Unión a la beta tubulina con polimerización de
TUBULINA CON microtúbulos con pérdidas de vesículas secretorias y alteración de la captación de glucosa
POLIMERIZACIÓN DE (larvas/adultos). o Otros: inhibición de la fumarato reductasa, desacoplamiento de la
MICROTÚBULOS fosforilación oxidativa o Espectro: o Áscaris, Ancylostoma, Necátor, Trichuris.
CON PÉRDIDAS DE Farmacocinética: • Absorción rápida, pero incompleta (baja biodisponibilidad; efecto de
VESÍCULAS primer paso) Absorción oral, mejor con alimentos o Metabolitos poco activos (excepto
SECRETORIAS Y albendazol) Metabolismo Hepático, Sulfóxido albendazol (activo) • Distribución SNC •
ALTERACIÓN DE LA Excreción biliar: mebendazol Renal: albendazol • Toxicidad: Trastornos GI Fiebre, rash,
CAPTACIÓN DE eritema. Gastrointestinales, citopenias, colestasis, teratogénesis, embriotoxicidad Efectos
GLUCOSA adversos: en casos de infestación masiva y eliminación de vermes, puede producir dolor
abdominal y diarrea. En tratamientos prolongados pueden producir aumento de
transaminasas, supresión de medula ósea y alopecia. • Interacciones: puede reducir niveles
de ácido Valproico y carbamazepina. • Contraindicaciones: embarazo (produce
teratogénesis en animales) y en alérgicos. No usar en menores de 2 años.
ES UN PROFÁRMACO METRONIDAZOL • Espectro antiparasitario: Entamoeba histolytica, Giardia lamblia y
QUE SE ACTIVA POR Trichomonas vaginalis. • Espectro antimicrobiano: cocos anaerobios y bacilos
NITRORREDUCCIÓN gramnegativos anaerobios. Fragilis, Clostridium, Fusobacterium, Peptococcus, Helicobacter
E INTERACTUA CON y Peptostreptococcus. MECANISMO DE ACCIÓN • Profármaco activado en el interior de
EL DNA DEL células sensibles reduciendo su grupo nitro por la enzima ferrodoxina del parásito,
PARÁSITO formando un compuesto reactivo que interfiere en el transporte de electrones y rompe el
DNA. EFECTOS ADVERSOS • Frecuentes: cefalea, náuseas, boca seca, sabor metálico • Pocos
frecuentes: vómitos, diarrea, insomnio, debilidad, mareos, estomatitis, disuria, orina
oscura, vértigo, ataxia, leucopenia. (Por vía e.v. puede dar convulsiones o neuropatía
periférica) PRECAUCIONES --CONTRAINDICACIONES • Potencia el efecto de la warfarina
(inhibe el metabolismo) • Intolerancia al alcohol por inhibición de la aldehído
deshidrogenasa (antabus). • Evitar 1º trimestre embarazo, en casos de lactancia suspender
h/24 hrs después. • Evitar en pacientes con alteraciones neurológicas, renales o hepáticas.

ACTIVACIÓN PIRANTEL. Activación persistente del Receptor nicotínico excitatorio de Acetilcolina del
PERSISTENTE DEL músculo de nematodos, despolarización y PARÁLISIS ESPÁSTICA por incremento de la
RECEPTOR conductancia al sodio y al potasio. FARMACOCINÉTICA • Poca absorción oral, eliminación
NICOTÍNICO por heces excreción urinaria (inalterado + metabolitos) • USOS: Enterobiasis, ascaridiasis,
EXCITATORIO DE Uncinariasis. EFECTOS ADVERSOS: • Gastrointestinales, cefaleas, fiebre, mareos. • No hay
ACETILCOLINA DEL pruebas de que sea inofensivo para el embarazo o el lactante CONTRAINDICACIONES: •
MÚSCULO DE Menores de dos años, Embarazo.
NEMÁTODOS

PARÁLISIS POR IVERMECTINA MECANISMO DE ACCIÓN • Ocasiona parálisis por activación de un canal de
ACTIVACIÓN DE UN cloro (abierto por glutamato) generando hiperpolarización y parálisis tónica. También
CANAL DE CLORO activa canales de cloro sensibles al GABA en nematodos. FARMACOCINÉTICA •
(ABIERTO POR Administración oral, subcutánea, tópica. Alta distribución en la grasa y el hígado.
GLUTAMATO) Eliminación por heces. • Usos: Oncocercosis, filariasis, Estrongiloidiasis, escabiosis,
GENERANDO pediculosis.
HIPERPOLARIZACIÓN
Y PARÁLISIS TÓNICA

Bibliografía:
ATENCIÓN PRIMARIA EN PEDIATRIA. ROBERT A. HOEKELMAN. HARCOURT Y MOSBY. EDICIÓN 3A. 1998.
PAG. 388.
ANÁLISIS DEL CASO CLÍNICO

IDENTIFICACIÓN DEL REACTIVO


Area: CIRUGÍA
Especialidad: URGENCIAS
Tema: TRAUMA DE TÓRAX
Subtema: HEMOTÓRAX

CASO CLÍNICO SERIADO

MASCULINO DE 27 AÑOS DE EDAD, SUFRE ACCIDENTE AL CHOCAR SU MOTO CONTRA EL MURO DE


CONTENCIÓN. A SU LLEGADA AL HOSPITAL SE ENCUENTRA CON TEMPERATURA DE 36°C, TENSIÓN ARTERIAL
90/40 MMHG, FRECUENCIA CARDÍACA 130 POR MINUTO, FRECUENCIA RESPIRATORIA DE 24 POR MINUTO. SE
ENCUENTRA DESORIENTADO, CON PALIDEZ DE TEGUMENTOS, AUSENCIA DE RUIDOS RESPIRATORIOS,
MATIDEZ A LA PERCUSIÓN Y CREPITACIÓN DEL 4° AL 8° ARCO COSTAL EN HEMITÓRAX DERECHO.

Masculino de 27 años de edad.

choque en moto.

-.

Datos de bajo gasto caracterizados por


hipotensión, taquicardia y palidez de
tegumentos. Compromiso ventilatorio
caracterizado por polipnea, ausencia de
ruidos respiratorio, matidez a la percusión
en hemitórax derecho y fracturas costales.
Hemorragia más compromiso ventilatorio
es igual a hemotórax masivo.

-.

130 - EL DIAGNÓSTICO PRINCIPAL DEL PACIENTE ES:


TAPONAMIENTO El TAPONAMIENTO CARDIACO resulta de un trauma cardiaco penetrante, por acumulación de
CARDÍACO sangre en el saco pericárdico, lo cual lleva a un efecto restrictivo en las cavidades derechas con
disminución del llenado cardiaco y del volumen de eyección, lo que puede conducir a la muerte.
Beck: Describió la tríada para el diagnóstico; hipotensión, velamiento de los ruidos cardiacos e
ingurgitación de las venas del cuello, signos que se observan apenas en menos de la mitad de
los pacientes con taponamiento; otros pacientes se pueden presentar asintomáticos. El área
precordial, la cual ha sido denominada como el “Triángulo de la muerte”, va desde las
clavículas, bajando por las líneas medioclaviculares hasta el epigastrio. Todas las lesiones
penetrantes en esta localización se denominan heridas precordiales y en todo paciente afectado
se debe descartar una lesión cardiaca. NO SE NOS OFRECEN DATOS CON RESPECTO A LOS
RUIDOS CARDÍACOS, DEBES TENER EN CUENTA QUE EL TAPONAMIENTO CARDIACO NO
JUSTIFICA LA MATIDEZ EN TÓRAX NI LA AUSENCIA DE RUIDOS RESPIRATORIOS.

ROTURA La ROTURA TRAUMÁTICA DE DIAFRAGMA se presenta en casos de trauma penetrante


TRAUMÁTICA toracoabdominal, pero también en el trauma cerrado. Hay pérdida de la continuidad del
DEL DIAFRAGMA músculo diafragma, con herniación de las vísceras abdominales al tórax en razón de la presión
negativa de éste. En el trauma cerrado, cuando hay fracturas de las 4 o 5 últimas costillas, se
debe sospechar lesión del diafragma. La presentación clínica de las roturas traumáticas del
diafragma, abarca una amplia gama de formas que van desde el paciente asintomático, con
hemodinamia estable, hasta pacientes con severo compromiso hemodinámico y múltiples
injurias asociadas. Es fundamental la información aportada por el personal de atención
prehospitalaria, así como de ser posible por el mismo paciente, para establecer el mecanismo
lesional que nos permita sospechar una eventual rotura diafragmática. En pacientes con trauma
cerrado, los elementos de sospecha son: colisión de vehículos a alta velocidad, siendo datos
importantes, la dirección del impacto, posición del paciente dentro del vehículo, severidad del
daño del automóvil (como deformidad de la cabina y del volante, así como tiempo requerido
para extraer al paciente del vehículo). Los criterios establecidos por Bowditch, para el
diagnóstico de hernia diafragmática aguda son: a) Prominencia con inmovilidad del hemitórax
izquierdo. b) Desplazamiento del área cardíaca a derecha. c) Ausencia de murmullo alvéolo
vesicular en el hemitórax izquierdo. d) "Presencia de ruidos hidroaereos en hemitórax
izquierdo". e) "Timpanismo a la percusión de hemitórax izquierdo". Esta signología
patognomónica, es prácticamente excepcional y ello obedece a dos razones. En primer lugar, no
todas las roturas diafragmáticas se acompañan de herniación visceral aguda. En segundo lugar
es habitual que la clínica del paciente esté dominada por los síntomas y signos de las lesiones
asociadas. EL DATO CARACTERÍSTICO A LA PERCUSIÓN ES EL TIMPANISMO, SE PUEDEN
ESCUCHAR RUIDOS HIDROAÉREOS A LA AUSCULTACIÓN.
TÓRAX El trauma de tórax contribuye con cerca del 25% de las muertes en los pacientes traumatizados;
INESTABLE es de tener en cuenta que los traumatismos cerrados alcanzan una mortalidad mucho mayor
cuando son severos. El TÓRAX INESTABLE o tórax batiente o volet costal, se define como la
fractura de 3 o 4 costillas en más de dos segmentos de su longitud ocasionando un fenómeno
paradójico en la caja torácica colapsándolo en inspiración y expandiéndolo en espiración. Es
importante reconocer que se asocia a múltiples lesiones torácicas como contusión pulmonar,
hemotórax, neumotórax y a nivel extra torácico con lesiones abdominales, encefálicas y lesiones
en extremidades inferiores. La frecuencia del tórax inestable es baja alcanzando en las
diferentes series con un total de 851 pacientes, un rango entre el 1.5% y 10.1% en su mayoría
derivados de accidentes de tránsito; otras asociadas son caídas, accidentes de trabajo,
agresiones y lesiones deportivas. Después de evaluar el A.B.C en el paciente se define si
presenta insuficiencia respiratoria representada en taquipnea y aumento del trabajo
respiratorio, alteración del estado de conciencia, agitación psicomotriz, etc. En el tórax se
evalúa la existencia de movimientos paradójicos, se ausculta en busca de alteraciones del
murmullo vesicular que representen hemotórax o neumotórax; una clave para observar el tórax
inestable es mirar de perfil en movimiento torácico; esto se puede realizar en un plano
horizontal sobre los pies. El estudio de imagen inicial en estos pacientes; es las radiografía
portátil de tórax la cual se realiza en proyección antero posterior, con el inconveniente de ser
deficiente en la evaluación de los arcos posteriores y laterales de las costillas, si el paciente
presenta compromiso multiorgánico con gran severidad del trauma, se sugiere realizar
tomografía torácica y abdominal simple y con contraste con la ventaja adicional de poder
evaluar estructuras vasculares y sólidas en las diferentes cavidades. Son las lesiones más
frecuentes en el trauma de tórax. Se pueden presentar en el trauma cerrado o en heridas por
proyectil de arma de fuego. Su diagnóstico se realiza por medio del examen físico, con la
palpación se localiza la zona dolorosa y en algunas circunstancias se encuentra crepitación o
signo de la tecla en la costilla afectada. En la radiografía de tórax no siempre se ven las
fracturas, especialmente si no están desplazadas, la radiografía de reja costal tiene poca
aplicación. La gammagrafía ósea es muy útil después de las primeras 24 horas. Cuando se
lesionan las dos primeras costillas se debe sospechar lesión de grandes vasos, y cuando se
fracturan las últimas se debe descartar lesión diafragmática o intra-abdominal. En su manejo, el
control del dolor es lo más importante. EL PACIENTE PRESENTA MUY PROBABLEMENTE
TAMBIÉN ESTE DIAGNÓSTICO, SIN EMBARGO, EL DIAGNÓSTICO PRINCIPAL Y GENERADOR DEL
CUADRO CLÍNICO ES EL HEMOTÓRAX MASIVO.

HEMOTÓRAX El HEMOTÓRAX MASIVO, se produce por la rápida acumulación de sangre en el espacio pleural.
MASIVO Su principal causa es la lesión del hilio pulmonar o de los vasos sistémicos de la reja costal. El
paciente se puede presentar con inestabilidad hemodinámica y ausencia de ventilación en un
hemitórax; el cuadro clínico es secundario a la hipovolemia y a la alteración V/Q que lleva a
shock y dificultad respiratoria severa. Clínicamente estos pacientes presentan "datos de bajo
gasto", con "hipotensión", "taquicardia", palidez y pulsos disminuidos a la exploración de tórax,
este se caracterizará por la "ausencia de murmullo vesicular" y "matidez a la percusión de
tórax", tal y como presenta nuestro paciente. LA AUSENCIA DE RUIDOS RESPIRATORIOS Y
MATIDEZ EN TÓRAX SUSTENTAN ESTE DIAGNÓSTICO.

Bibliografía:
1. COMITÉ DE TRAUMA DEL COLEGIO AMERICAN DE CIRUJANOS, PROGRAMA AVANZADO DE APOYO
VITAL EN TRAUMA PARA MÉDICOS. ATLS, 7A EDICIÓN.. USA. PP 111.

131 - EL TRATAMIENTO QUE REQUIERE DE INMEDIATO EL PACIENTE ES CON:


TORACOSTOMÍA CON EL TRATAMIENTO DEL HEMOTÓRAX CONSISTE EN EL MANEJO DEL SHOCK,
SONDA DESCOMPRESIÓN DEL ESPACIO PLEURAL CON UN TUBO DE TORACOTOMÍA, LUEGO DE
UNA RÁPIDA ESTABILIZACIÓN SE LLEVA A CIRUGÍA. El tratamiento del HEMOTÓRAX debe
iniciarse con una toracocentesis o toracostomía con sonda (o ambas), tras lo cual será
necesario ubicar el sangrado e ingresar a cirugía. La TORACOSTOMÍA POR SONDA es el
procedimiento que consiste en la inserción de una sonda para drenar aire o líquido (en este
caso sangre) del espacio pleural. La colocación de un TUBO ENDOTORÁCICO es una técnica
sencilla que todo médico debe conocer. Los riesgos de esta técnica son mínimos siempre
que respetemos sus indicaciones. INDICACIONES • Neumotórax a tensión. • Neumotórax
postraumático y posquirúrgico. • Neumotórax yatrogénico secundario a canalización de
vías centrales. • Neumotórax en ventilación mecánica. • Neumotórax espontáneo mayor
del 20% o situación clínica comprometida. • Hemotórax. • Derrame pleural no controlado.
• Quilotórax. • Empiema, líquido pleural con pH menor de 7.0, pus o infección.
CONTRAINDICACIONES Coagulopatía grave (Deberá ser corregida previamente, excepto
en situaciones de emergencia). COMPLICACIONES - Hemorragia; lesión del paquete
vasculonervioso intercostal, o de la arteria mamaria interna. Debemos introducir el tubo
inmediatamente por encima del borde superior de la costilla y alejado de la línea
paraesternal. - Laceración pulmonar; secundaria a la inserción brusca del tubo con trocar,
produciendo hemotórax o neumotórax yatrogénico. (Por este motivo, es preferible realizar
la penetración al espacio pleura) mediante el dedo índice o la pinza de disección roma. -
Colocación intra-abdominal; con lesión de hígado, bazo, etcétera, por elección de un
espacio intercostal demasiado bajo. Por ello, no colocaremos un tubo endotorácico por
debajo del 62 espacio intercostal. - Colocación subcutánea; introducción del tubo entre la
fascia endotorácica y la pleura parietal. Observaremos que no oscila, siempre debemos
comprobar con el dedo índice que hemos penetrado en la cavidad pleural. - Edema de
pulmón ex-vacuo. Se produce en drenajes importantes evacuados de forma brusca. En
estos casos se procederá a drenar lentamente la cavidad pleural mediante pinzamientos
sucesivos del tubo de tórax. IMPORTANTE: Con base en el material extraído a parir del
tubo pleural, el ATLS del American College of Surgeons, define el HEMOTÓRAX MASIVO
como el drenaje de 1.500 ml de sangre en el espacio pleural en el momento de la
colocación del tubo o un drenaje mayor de 200 ml por hora en 2 a 4 horas.

PLASTÍA El abordaje abdominal, es considerado unánimemente, como la vía de elección en el caso


DIAFRAGMÁTICA de la ROTURA DIAFRAGMÁTICA en su fase aguda, ya que tiene la ventaja fundamental de
CON ABORDAJE permitir la completa exploración y tratamiento de las lesiones abdominales asociadas. Por
TORÁCICO otra parte, en la fase aguda la reducción de las vísceras herniadas y la reparación
diafragmática se realizan con facilidad desde el abdomen. El abordaje torácico, en la fase
aguda, sólo se realiza cuando es necesario realizar una toracotomía de emergencia,
generalmente por sangrado masivo y en el curso de ella se diagnostica la lesión
diafragmática.

PERICARDIOCENTESIS Los pacientes con HERIDAS PRECORDIALES se dividen en tres clases: 1. Paciente estable. 2.
Paciente inestable (Hipotensión y taquicardia). 3. Paciente en estado agónico. De acuerdo
con esta clasificación, se establece el diagnóstico de posible lesión cardiaca de la siguiente
manera: 1. Tipo 1: por medio de eco cardiografía, ecografía o ventana pericárdica. 2. Tipo
2: por medio de pericardiocentesis o la ventana pericárdica. 3. Tipo 3: se requiere
toracotomía de urgencia (Anterolateral izquierda). Si por algún otro método se confirma la
presencia de líquido en el saco pericárdico, se debe hacer una toracotomía izquierda
anterolateral o una esternotomía para la reparación de la herida en el corazón.
PLASTÍA El objetivo del manejo médico del TÓRAX INESTABLE, es mantener adecuada ventilación y
DIAFRAGMÁTICA los elementos claves son el manejo del dolor y el mejoramiento de la oxigenación. La
CON ABORDAJE ventilación es otro de los pilares del manejo en la inmovilización interna del tórax. La
ABDOMINAL presión positiva fue la primera estrategia utilizada, inicialmente descrita por; Avery en
1956. Cullen planteó el uso de ventilación mecánica intermitente con buenos resultados.
En la década de los sesentas y setentas se pregonaba la utilización de traqueotomías
tempranas con ventilación mecánica con resultados favorables para la disfunción
respiratoria, pero con aumento de la incidencia de neumonías y otras infecciones asociadas
a ventilación mecánica prolongada. Las indicaciones de ventilación mecánica son: 1. PO2
<60, PAFI < 250 Shunt >15% (Centros con monitorea hemodinámica invasiva), Diferencia
alvéolo arterial >350 mm/Hg, Frecuencia respiratoria <8 o >35 por minuto, Asociación con
shock, Lesiones asociadas graves, Trauma cráneo encefálico severo, Enfermedad pulmonar
previa. Más de 8 costillas fracturadas ISS (Índice de severidad del trauma) >de 23 y >de 65
años (Freedland). El inicio de la ventilación mecánica es uno de los factores que muestran
impacto en la mortalidad disminuyéndola de un 68% hasta el 7% 2. En la ventilación
mecánica se pueden utilizar sistemas invasivos y no invasivos. Sin embargo, se sugiere la
intubación profiláctica en estos pacientes.

Bibliografía:
1. COMITÉ DE TRAUMA DEL COLEGIO AMERICAN DE CIRUJANOS, PROGRAMA AVANZADO DE APOYO
VITAL EN TRAUMA PARA MÉDICOS. ATLS, 7A EDICIÓN.. USA. PP 111. 2. SOTONE C, HUMPHRIES R,
DIAGNÓSTICO Y TRATAMIENTO EN MEDICIAN DE URGENCIAS. 7ª EDICIÓN. MÉXICO. MC GRAW HILL
LANGE. 2013.

FIN DEL CASO CLÍNICO SERIADO


ANÁLISIS DEL CASO CLÍNICO

IDENTIFICACIÓN DEL REACTIVO


Area: MEDICINA INTERNA
Especialidad: ENDOCRINOLOGÍA
Tema: TRANSTORNOS DEL METABOLISMO DEL HUESO
Subtema: OSTEOPOROSIS

CASO CLÍNICO CON UNA PREGUNTA

FEMENINA DE 50 AÑOS DE EDAD, DIABÉTICA DESDE HACE 15 AÑOS, CON MAL CONTROL DE SU
ENFERMEDAD. ACUDE URGENCIAS POR PRESENTAR DOLOR INTENSO EN EXTREMIDAD INFERIOR DERECHA
ASÍ COMO LIMITACIÓN DE LA MOVILIDAD DE LA MISMA EXTREMIDAD. GLUCEMIA DE 340 MG/DL. EXAMEN
GENERAL DE ORINA CON PROTEINURIA Y ERITROCITOS 1 A 2 POR CAMPO. LA RADIOGRAFÍA DE LA
EXTREMIDAD MUESTRA FISURA EN EL DEDO MEDIO DEL PIE DERECHO Y DESMINERALIZACIÓN EN HUESOS
CIRCUNDANTES.

FEMENINA DE 50 AÑOS.

- DM MAL CONTROLADA. - DATOS DE


PATOLOGIA RENAL.

- DOLOR.

Rx muestra desmineralización y fractura


sin antecedentes traumáticos.

132 - EN ESTA PACIENTE LA DESMINERALIZACIÓN OSEA ES SECUNDARIA A LA DISMINUCIÓN EN:

VITAMINA En este particular caso hay que hacer un análisis de los antecedentes de la paciente, es decir estudiar el
A. caso con detenimiento, el caso plantea que es una paciente con: • DM mal controlada (glucemia de
340 mg/dl). • Existen ya problemas renales secundarios a esta DM mal controlada (EGO con
proteinuria). • Y la clave del caso, la presencia de una fractura sin antecedentes traumáticos y
desmineralización ósea.

VITAMINA Hay que pensar en las causas de una DESMINERALIZACIÓN ÓSEA: El hueso está formado por una
B12. matriz proteica y sales que se depositan sobre ella. La desmineralización consiste en la pérdida de esta
sustancia mineral. Debida a dos motivos: disminución de la matriz proteica (osteoporosis), déficit de
depósito de sales minerales (osteomalacia). En ocasiones la desmineralización puede ser debida a
ambas situaciones (osteoporomalacia).
VITAMINA IMPORTANTE PARA COMPRENDER: La Vitamina D2 es la que se encuentra en las fórmulas
C. farmacéuticas, la D3 es el compuesto natural más importante para el hombre; se produce a partir de la
7-dehidrocolesterol en la piel. De ahí la vitamina D se metaboliza en el hígado convirtiéndose en 25-
hidroxivitamina D. Luego pasa al RIÑON donde se hidroxila para producir la 1.25-hidroxivitamina D
que es la FORMA DE LA VITAMINA DE CON MAYOR ACCIÓN METABÓLICA. CONCLUSIÓN: Cuando
existe daño renal es imposible lograr las formas activas más importantes de la Vitamina D y en
consecuencia puede haber complicaciones tales como desmineralización ósea.

VITAMINA VITAMINA D Dependiente: • Ingesta inadecuada de vitamina D. • Falta de exposición al sol (radiación
D. UV) y no formación de vitamina D endógena. • Mala absorción intestinal de vitamina D. • Desórdenes
del metabolismo de la Vi-D. • Defectos en los receptores de 1,25 Vi-D. VITAMINA D Independiente : •
Defecto tubular renal con hipofosfatemia o con acidosis. • Acidosis crónica. • Intoxicación por
aluminio. • Uso crónico de anticonvulsivantes. - INSUFICIENCIA RENAL O HEMODIÁLISIS CRÓNICA
POR LO TANTO EN ESTA PACIENTE LA RESPUESTA BUSCADA ES LA DEFICIENCIA DE VITAMINA D
DEBIDO A PATOLOGÍA RENAL.

Bibliografía:
1. BRUNICARDI F, ANDERSEN D, BILLIAR T, Y COLS. SCHWARTZ PRINCIPIOS DE CIRUGÍA, 9A EDICIÓN. MC
GRAW HILL. 2011, PP 1377.
ANÁLISIS DEL CASO CLÍNICO

IDENTIFICACIÓN DEL REACTIVO


Area: GINECOLOGÍA Y OBSTETRICIA
Especialidad: GINECOLOGÍA
Tema: CLIMATERIO Y MENOPAUSIA
Subtema: PROLAPSO VAGINAL Y CISTOCELE

CASO CLÍNICO SERIADO

MUJER DE 52 AÑOS DE EDAD, CON ANTECEDENTE DE GESTA 4, PARTOS 4. ACUDE A LA CONSULTA


REFIRIENDO SENSACIÓN DE CUERPO EXTRAÑO EN LA VAGINA, QUE AUMENTA CON EL ESFUERZO, URGENCIA
URINARIA, DOLOR EN HIPOGÁSTRICO, CONSTIPACIÓN Y SANGRADO OCASIONAL POST- COITO DESDE HACE
APROXIMADAMENTE 1 AÑO.

Femenina de 52 años, postmenopáusica

Multigesta con constipación

Datos clave. Prolapso de órganos pélvicos:


Sensación de cuerpo extraño en vagina,
que aumenta con el esfuerzo, urgencia
urinaria

-.

-.

133 - EL DIAGNÓSTICO CLÍNICO MÁS PROBABLE DE ÉSTA PACIENTE ES:

INCONTINENCIA La INCONTINENCIA URINARIA (IU) es la pérdida de orina asociada al esfuerzo, objetivamente


URINARIA demostrable y capaz de ocasionar un problema higiénico-social. La INCONTINENCIA URINARIA
ASOCIADA AL DE ESFUERZO o urgencia, acompaña comúnmente al prolapso de los órganos pélvicos en las
CLIMATERIO. mujeres posmenopáusicas. Esto se explica por los cambios hormonales presentes en estas
mujeres y que producen: Las modificaciones fisiológicas del aparato genitourinario después de
la menopausia son histológicas, morfológicas y funcionales: A. Reducción del índice de
colágeno. B. Involución de fascias y ligamentos. C. Involución de músculos estriados pélvicos. D.
Pérdida del tono y la disminución de la longitud de la vagina, por efecto de carencia
estrogénica. COMO PUEDES LEER EN EL CASO, LA PACIENTE SI PRESENTA INCONTINENCIA
URINARIA, SIN EMBARGO ESTA FORMA PARTE O ES CONSECUENCIA DEL PROLAPSO DE LOS
ÓRGANOS PÉLVICOS, PREDOMINANDO EN ESTE CASO EL CISTOCELE.
MIOMATOSIS Recuerda que la mayoría de los MIOMAS son asintomáticos; llegan a presentar las pacientes
UTERINA DE sensación de plenitud o presión en el bajo abdomen, compresión de vísceras pelvianas,
GRANDES hemorragia o flujo no relacionados con la menstruación, dificultad o dolor al orinar, aumento
ELEMENTOS. del perímetro, abdomen agudo en torsión, aborto espontáneo. LA CLAVE DEL DIAGNÓSTICO ES
LA SENSACIÓN DEL CUERPO EXTRAÑO A NIVEL VAGINAL, ASOCIADO A LA SINTOMATOLOGÍA
URINARIA, LO QUE NO SUCEDE EN NINGÚN CASO EN LA MIOMATOSIS CUYA LOCALIZACIÓN
ES PURAMENTE UTERINA. Como puedes ver se agregaron algunos datos al caso clínico para que
considerarás también esta posibilidad. IMPORTANTE. La miomatosis uterina es la causa más
común de tumores pélvicos sólidos en mujeres, y ocurre en 20 a 40% de mujeres en "edad
reproductiva". Pueden ser clínicamente evidentes en el 20 a 25% de los casos; el pico mayor de
presentación tiene lugar en la cuarta a quinta décadas, presentándose hasta en 80 % de los
especímenes de patología de los úteros resecados de manera quirúrgica, independientemente
del diagnóstico preoperatorio.

CISTOCELE. La paciente presenta factores de riesgo muy claros para PROLAPSO DE ÓRGANOS PÉLVICOS y
específicamente CISTOCELE, además el cuadro clínico es característico. FACTORES DE RIESGO.
Son muchos los factores de riesgo presentes para desarrollar el estado del prolapso de órganos
pélvicos. Sin duda el parto vaginal es el más sólido factor de riesgo en esta entidad ocasionado
por lesión muscular y nerviosa, así como por ruptura de los tejidos. Los partos traumáticos
(Maniobras de Kristeller o uso de fórceps, periodo expulsivo prolongado, productos
macrosómicos, etc.) aumentan considerablemente el riesgo. Existen además factores
promotores como el estreñimiento y el pujo excesivo, labores cotidianas o deporte rutinario con
exceso de presión intra-abdominal, obesidad, la realización de procedimientos quirúrgicos
previos como la histerectomía (aún mayor en totales con técnicas extrafasciales y radicales),
radiación pélvica, enfermedades propias de la colágena, bronquitis, enfisema, etc., asociadas a
tabaquismo, déficit estrogénico y envejecimiento. Las modificaciones fisiológicas del aparato
genitourinario después de la menopausia son histológicas, morfológicas y funcionales.
DIAGNÓSTICO. Prolapso de la pared vaginal anterior (Cistocele). EL ÚNICO SÍNTOMA
ESPECÍFICO PARA EL PROLAPSO DE LA PARED VAGINAL ANTERIOR (CISTOCELE) ES LA
SENSACIÓN DE CUERPO EXTRAÑO A NIVEL VAGINAL. Pueden existir ulceraciones mucosas con
metrorragias o leucorreas. La incontinencia urinaria es frecuente, sobre todo en caso de
debilidad de los soportes de la uretra, en caso hipermovilidad uretral. En caso de cistocele, la
paciente puede describir una disminución secundaria de las perdidas urinarias asociada con el
desarrollo del cistocele, que enmascara la incontinencia; así también acompañado de disuria, y
polaquiuria diurna o nocturna.

CACU Las mujeres con CÁNCER INVASOR DE CUELLO UTERINO a menudo tienen como manifestación
METASTÁSICO. inicial alguno o varios de los siguientes síntomas: Hemorragias intermenstruales, hemorragias
poscoitales, menstruaciones más abundantes, flujo seropurulento abundante, flujo fétido,
cistitis recurrente, frecuencia y urgencia miccional, dolor de espalda y en la parte inferior del
abdomen. En los estadios avanzados, las pacientes pueden presentar cansancio por anemia
intensa, uropatía obstructiva, edema de las extremidades inferiores, hematuria, obstrucción
intestinal y caquexia. EN ESTE CASO ESPERARÍAMOS ENCONTRAR LOS SÍNTOMAS DE UN CACU
AVANZADO QUE JUSTIFIQUEN LA PRESENCIA DE UNA MASA A NIVEL VAGINAL, NO
CONCUERDAN CON LO ENCONTRADO EN LA PACIENTE.

Bibliografía:
1. GUÍA DE PRÁCTICA CLÍNICA, DIAGNÓSTICO Y TRATAMIENTO DEL PROLAPSO DE LA PARED VAGINAL
ANTERIOR CISTOCELE E INCONTINENCIA URINARIA DE ESFUERZO. MÉXICO: SECRETARIA DE SALUD; 2009.
2. GUÍA DE REFERENCIA RÁPIDA, DIAGNÓSTICO Y TRATAMIENTO DEL PROLAPSO DE LA PARED VAGINAL
ANTERIOR CISTOCELE E INCONTINENCIA URINARIA DE ESFUERZO. MÉXICO: SECRETARIA DE SALUD; 2009.
3. SCHONGUE J, SCHAFER J, HALVORSON L, HOFFMAN B, BRADSHAW K, CUNNINGHAM G. WILLIAMS
GINECOLOGÍA, DE LA 1A EDICIÓN EN INGLÉS. MC GRAW HILL. USA. 2009, PP 538-541.

http://www.cenetec.salud.gob.mx/descargas/gpc/CatalogoMaestro/263_GPC_CISTOCELE/Cistocele_EVR_CENETEC_IUE_VERIF_MZO.pdf

134 - DURANTE LA EXPLORACIÓN FÍSICA DE LA PACIENTE ESPERA ENCONTRAR:


PÉRDIDA El signo principal de la INCONTINENCIA URINARIA DE ESFUERZO es la pérdida involuntaria de
INVOLUNTARIA orina secundaria al aumento de la presión intra-abdominal ocasionada al toser, estornudar,
DE ORINA ponerse de pie, sentarse o realizar un esfuerzo con los músculos abdominales. • El diario
SECUNDARIA AL miccional se considera un método práctico y confiable de obtención de información sobre el
AUMENTO DE LA comportamiento miccional, que nos ayuda a diferenciar entre incontinencia urinaria de
PRESIÓN esfuerzo e incontinencia urinaria de urgencia. • La exploración física de la incontinencia
INTRABDOMINAL. urinaria de esfuerzo consiste en buscar la salida de orina durante la maniobra de Valsalva. •
Interrogar a las pacientes sobre la pérdida involuntaria de orina al realizar un esfuerzo y
diferenciar ésta de la pérdida de orina relacionada a la urgencia miccional. • Realizar la prueba
de Q-tip, para diagnosticar incontinencia urinaria de esfuerzo secundaria a hipermovilidad
uretral. ES PROBABLE QUE LA PACIENTE PRESENTE INCONTINENCIA URINARIA, PERO EN ESTE
CASO SERÍA ASOCIADA EL CISTOCELE.

AUMENTO DE En la MIOMATOSIS UTERINA el diagnóstico se basa usualmente en el hallazgo de un útero


TAMAÑO aumentado de tamaño, móvil con contornos irregulares a la exploración bimanual o como un
UTERINO. hallazgo incidental en el ultrasonido. EL CISTOCELE NO JUSTIFICA UN AUMENTO EN EL
TAMAÑO DEL ÚTERO.

DESCENSO DE LA La exploración física consiste en una REVISIÓN BIMANUAL en posición ginecológica


PARED VAGINAL evidenciando la PROTRUSIÓN DE LA PARED VAGINAL ANTERIOR (CISTOCELE) con la maniobra
ANTERIOR. de Valsalva, realizándola de pie en caso de no visualizar el prolapso. • El tamaño o severidad
del prolapso de la pared vaginal anterior (Cistocele) puede ser medido y registrado utilizando
la clasificación de Prolapso de Órganos Pélvicos.

INFILTRACIÓN Los CÁNCERES MÁS INVASIVOS pueden ser exofíticos, endofíticos o una combinación de
VAGINAL Y DE ambos. Los carcinomas exofíticos por lo general invaden superficialmente, y la mayor parte de
PARAMETRIOS. su masa se proyecta hacia la luz vaginal como un hongo o una protuberancia proliferativa
similar a una coliflor, con excrecencias polipoides o papilares. Los cánceres endofíticos pueden
infiltrar ampliamente el estroma, distorsionando el cuello uterino, con poco crecimiento
visible en la superficie. Estas lesiones pueden extenderse hacia el endocérvix al tiempo que
dejan intacto el epitelio escamoso del cuello uterino hasta que la lesión rebasa los 5 ó 6 cm de
diámetro. Dan lugar a un cuello sumamente aumentado de tamaño, irregular y en forma de
barril, con una superficie rugosa, papilar o granulosa. Tales cánceres pueden mantenerse
asintomáticos durante mucho tiempo. Los tumores que son a la vez exofíticos y endofíticos
generalmente están ulcerados, con infiltración profunda del estroma subyacente. En todos los
tipos, la hemorragia al tacto y la necrosis son las características clínicas predominantes.
También es común el flujo maloliente, debido a la infección agregada del tejido necrótico por
anaerobios. Al proseguir la invasión, puede afectar directamente a la vagina, el parametrio, la
pared lateral de la pelvis, la vejiga urinaria y el recto. La compresión de los uréteres por
afección local avanzada provoca obstrucción uretral, hidronefrosis (Aumento del volumen de
los riñones) y, a la larga, insuficiencia renal. Además de la invasión local se produce metástasis
de los ganglios linfáticos regionales. El cáncer metastásico de los ganglios paraaórticos puede
atravesar la cápsula de los ganglios e invadir directamente las vértebras y las raíces nerviosas.
La invasión directa de ramas de la raíz del nervio ciático provoca dolor de espalda; la
compresión sobre las venas de la pared pélvica y los vasos linfáticos causa edema de los
miembros inferiores. Puede ocurrir diseminación hematógena en las vértebras lumbares y en
el músculo psoas sin invasión ganglionar. En las etapas avanzadas de la enfermedad se
presentan metástasis a distancia, que suelen afectar a los nódulos paraaórticos, los pulmones,
el hígado, los huesos y otro tipo de estructuras. LA INFILTRACIÓN DE VAGINA Y
PARAMETRIOS CORRESPONDE A UNA CARACTERÍSTICA DEL CÁNCER CERVICOUTERINO EN
ESTADIOS AVANZADOS.

Bibliografía:
1. GUÍA DE PRÁCTICA CLÍNICA, DIAGNÓSTICO Y TRATAMIENTO DEL PROLAPSO DE LA PARED VAGINAL
ANTERIOR CISTOCELE E INCONTINENCIA URINARIA DE ESFUERZO. MÉXICO: SECRETARIA DE SALUD; 2009.
2. GUÍA DE REFERENCIA RÁPIDA, DIAGNÓSTICO Y TRATAMIENTO DEL PROLAPSO DE LA PARED VAGINAL
ANTERIOR CISTOCELE E INCONTINENCIA URINARIA DE ESFUERZO. MÉXICO: SECRETARIA DE SALUD; 2009.
3. SCHONGUE J, SCHAFER J, HALVORSON L, HOFFMAN B, BRADSHAW K, CUNNINGHAM G. WILLIAMS
GINECOLOGÍA, DE LA 1A EDICIÓN EN INGLÉS. MC GRAW HILL. USA. 2009, PP 534-535.

http://www.cenetec.salud.gob.mx/descargas/gpc/CatalogoMaestro/263_GPC_CISTOCELE/Cistocele_EVR_CENETEC_IUE_VERIF_MZO.pdf

135 - EL TRATAMIENTO INDICADO EN ESTA PACIENTE CONSISTE EN:


RADIOTERAPIA. El tratamiento convencional del CARCINOMA CERVICOUTERINO puede incluir cirugía,
radioterapia o una combinación de ambas. Los carcinomas cervicouterinos tempranos
(Estadios I y IIA) pueden tratarse con alguna de las dos opciones. La radioterapia es el
tratamiento preferido una vez que la enfermedad se ha extendido más allá de los límites del
cuello uterino y los fondos de sacos vaginales, cuando la cirugía no es eficaz. El tratamiento
del carcinoma cervicouterino con radioterapia a menudo puede incluir una combinación de
radioterapia externa (Para toda la pelvis) y radiación intracavitaria (Para la parte central de la
enfermedad). En caso de neoplasias localmente avanzadas, como los estadios IIB y III,
combinar la radiación intracavitaria y la externa ofrece un mejor control de la enfermedad y
mejora la supervivencia, en comparación con la radioterapia externa por sí sola. LA
RADIOTERAPIA FORMA PARTE DEL MANEJO DEL CÁNCER CERVICOUTERINO EN ESTADIOS
AVANZADOS.

CIRUGÍA PÉLVICA El CISTOCELE amerita tratamiento cuando es sintomático y si se asocia con incontinencia
RECONSTRUCTIVA. urinaria se realiza tratamiento conjunto. Se puede realizar con COLPOPERINEOPLASTIA
ANTERIOR en casos de defecto central o mediante reparación paravaginal a la línea
iliopectínea en casos de defectos laterales con la operación de Richardson (Vía vaginal o
abdominal). La operación de Burch resuelve parcialmente cistoceles I y II. En descensos
uterinos grado II se puede realizar operación de Manchester o Histerectomía vaginal. En los
grados mayores se realizará histerectomía vaginal o elegir cirugías más conservadoras en caso
de riesgo alto quirúrgico, pacientes de edad avanzada, o sin deseo de función copuladota
como la histerocolpectomía o colpocleisis. En los prolapsos de cúpula, cuando la paciente
mantiene vida sexual activa la operación de Nichols es la elección. Si se corrige el defecto vía
abdominal está indicada la colposacropexia con fascia antóloga o material sintético. En
pacientes con edad avanzada sin deseo de vida sexual la colpocleisis y colpectomía son la
opción. En los casos de enterocele sintomático su tratamiento se prefiere por vía vaginal, la
técnica depende de otros trastornos de la estática pelvi genital encontrados. Por vía
abdominal se puede realizar técnica de Moschowitz o la obliteración total del fondo de saco
de Douglas. En casos de rectocele se realizará colpoperineorrafia, con defectos moderados
debe, además realizarse plastia de la cuña perineal con miorrafia de los músculos transversos
superficiales del periné. El tratamiento no quirúrgico consistiría en: El tratamiento
conservador del prolapso de la pared vaginal anterior (Cistocele) está indicado para las
pacientes con grados leves de prolapso, sin paridad satisfecha, con alto riesgo quirúrgico y
para las que no desean tratamiento quirúrgico. • La asociación del tratamiento quirúrgico del
prolapso de la pared vaginal anterior (Cistocele) con los ejercicios de Kegel, proporcionan un
mejor resultado que cuando se realiza únicamente el tratamiento quirúrgico. • El uso de un
pesario es una alternativa no quirúrgica que puede considerarse en pacientes con prolapso de
la pared vaginal anterior (Cistocele), sin importar el estadio. • Incontinencia urinaria de
esfuerzo. • Los ejercicios de Kegel se recomiendan como tratamiento no invasivo en mujeres
con incontinencia urinaria de esfuerzo. DADO QUE LA PACIENTE PRESENTA UN CISTOCELE
SINTOMÁTICO ES INDISPENSABLE OTORGAR MANEJO QUIRÚRGICO.
HISTERECTOMÍA. Los MIOMAS UTERINOS asintomáticos se manejan de manera expectante. El tratamiento
dependerá del tamaño del mioma, su localización, sintomatología, edad e historia
reproductiva y obstétrica. Terapia médica. El objetivo de la terapia médica es aliviar la
sintomatología. Para las mujeres que prefieren manejo conservador, existen múltiples
terapias médicas. Tiene como ventaja evitar las complicaciones del manejo quirúrgico y
permitir la conservación del útero. Estas terapias deben ser tomadas como la primera línea de
tratamiento. Las terapias disponibles son: Estrógenos y progestinas: La combinación o
progestinas solas, son a menudo la primera línea de tratamiento de estas pacientes. A pesar
de que esta terapia produce atrofia endometrial y estabilización, no han mostrado disminuir
el tamaño del mioma. Inhibidores de la síntesis esteroidea: Agonistas de la hormona
liberadora de gonadotrofinas (análogos de GnRH): Es la terapia actual más exitosa para el
manejo médico de los miomas. Estos medicamentos hacen una regulación a la baja de los
receptores de GnRH a nivel de la hipófisis, causando una reducción profunda de hormona
folículo estimulante (FSH), hormona luteinizante (LH) y esteroides ováricos y, por lo tanto,
causan un incremento repentino inicial, causando hiperestrogenismo y posteriormente
producen un estado de hipoestrogenismo. Esto ocasiona amenorrea y declinación en el
tamaño del mioma de 35 a 65%. También se pueden utilizar: Antagonistas de la hormona
liberadora de gonadotrofinas, inhibidores de la aromatasa, moduladores de los receptores
esteroideos (Tamoxifeno, mifepristona, asoprisnil, etc.), terapia androgénica (Danazol). Las
indicaciones para el manejo quirúrgico, incluyen: • Sangrado uterino anormal que no
responde a tratamiento conservador. • Alto nivel de sospecha de malignidad. • Crecimiento
posterior a la menopausia. • Infertilidad cuando existe distorsión de la cavidad endometrial u
obstrucción de la trompa uterina. • Pérdida gestacional recurrente por distorsión de la
cavidad endometrial. • Presión y dolor pélvico que interfieran con la calidad de vida. •
Síntomas del tracto urinario, como frecuencia y/o obstrucción. • Deficiencia de hierro y
anemia secundaria a pérdida sanguínea crónica. Histerectomía: La histerectomía continúa
siendo el tratamiento más común para la miomatosis uterina, debido a que es el único que
cura y elimina la posibilidad de recurrencia. ES UNA ESTRATEGIA QUIRÚRGICA INDICADA EN
CASO DE LESIÓN A NIVEL UTERINO, N CORRESPONDE CON EL CASO CLÍNICO.

EJERCICIOS DE INCONTINENCIA URINARIA DE ESFUERZO. Los ejercicios de Kegel; se recomiendan como


KEGEL. tratamiento no invasivo en mujeres con incontinencia urinaria de esfuerzo. El principio de los
ejercicios de Kegel es fortalecer los músculos del piso pélvico y, en consecuencia, mejorar el
funcionamiento del esfínter uretral para disminuir las pérdidas de orina. Aparte de los
beneficios relacionados con las pérdidas de orina, estos ejercicios también se han
recomendado para recuperar el tono de los músculos y de la vagina después del parto,
encontrándose además que su práctica habitual tiene efectos beneficiosos en la sexualidad. El
objetivo es contraer y relajar de manera repetida el músculo pubococcígeo. Ejercitar los
músculos del suelo de la pelvis durante cinco minutos tres veces al día puede significar una
gran diferencia en el control de la vejiga. El ejercicio fortalece los músculos que sostienen la
vejiga y mantienen los otros órganos de la pelvis en su lugar. Miomectomía abdominal: La
miomectomía es el tratamiento preferido cuando se desea la preservación del útero. La
evidencia sugiere que este procedimiento resuelve en 81% la menorragia. Indicaciones:
Interferencia con la fertilidad o predisposición a pérdidas gestacionales recurrentes, debido a
la naturaleza y/o localización de los miomas. LOS EJERCICIOS DE KEGEL CORRESPONDEN AL
TRATAMIENTO NO QUIRÚRGICO DE LA INCONTINENCIA URINARIA.

Bibliografía:
1. GUÍA DE PRÁCTICA CLÍNICA, DIAGNÓSTICO Y TRATAMIENTO DEL PROLAPSO DE LA PARED VAGINAL
ANTERIOR CISTOCELE E INCONTINENCIA URINARIA DE ESFUERZO. MÉXICO: SECRETARIA DE SALUD; 2009.
2. GUÍA DE REFERENCIA RÁPIDA, DIAGNÓSTICO Y TRATAMIENTO DEL PROLAPSO DE LA PARED VAGINAL
ANTERIOR CISTOCELE E INCONTINENCIA URINARIA DE ESFUERZO. MÉXICO: SECRETARIA DE SALUD; 2009.
3. SCHONGUE J, SCHAFER J, HALVORSON L, HOFFMAN B, BRADSHAW K, CUNNINGHAM G. WILLIAMS
GINECOLOGÍA, DE LA 1A EDICIÓN EN INGLÉS. MC GRAW HILL. USA. 2009, PP 549-553.

http://www.cenetec.salud.gob.mx/descargas/gpc/CatalogoMaestro/263_GPC_CISTOCELE/Cistocele_EVR_CENETEC_IUE_VERIF_MZO.pdf

FIN DEL CASO CLÍNICO SERIADO


ANÁLISIS DEL CASO CLÍNICO

IDENTIFICACIÓN DEL REACTIVO


Area: MEDICINA INTERNA
Especialidad: REUMATOLOGÍA
Tema: TRANSTORNOS ARTICULARES
Subtema: GOTA

CASO CLÍNICO CON UNA PREGUNTA

HOMBRE DE 19 AÑOS DE EDAD, CON ANTECEDENTE FAMILIAR DE GOTA, REFIERE TRAS INGESTA ALTA DE
FERMENTADOS PRESENTA DOLOR E INFLAMACIÓN DE MUÑECA DERECHA Y UNA RODILLA CON
INCAPACIDAD PARA LA DEAMBULACIÓN. ES ESTUDIO DE LÍQUIDO SINOVIAL MUESTRA PRESENCIA DE
CRISTALES DE MONOHIDRATO DE URATO MONOSÓDICO.

hombre de 19 años de edad.

antecedentes familiares de gota.

dolor e in amación en muñeca derecha y


una rodilla con incapacidad para la
deambulación.

-.

-.

136 - CORRESPONDE AL RECEPTOR QUE PROCESA ESTOS CRISTALES A NIVEL CELULAR.

DE La GOTA es una enfermedad metabólica caracterizada por artritis mono articular aunque
INFLAMOSOMAS también puede presentarse como poli articular, en donde existen depósitos de cristales de urato
NALP-3. mono sódico en diferentes tejidos como en la sinovia, bursas, tendones y los riñones y es el
resultado el aumento sostenido de ácido úrico en plasma. Estos depósitos pueden observarse en
forma de tofos a la exploración física o radiológica. Es una enfermedad metabólica de
naturaleza heterogénea, a menudo familiar, vinculada con cantidades anormales de urato en el
cuerpo; se caracteriza en fase temprana por una artritis aguda recurrente, la mayoría de las
veces monoarticular, y más tarde por artritis deformante crónica.
DE CANALES La HIPERURICEMIA se define como la concentración sérica de ácido úrico mayor de 6.8mg/dl. A
RÁPIDOS DE esta concentración disminuye la solubilidad del ácido úrico en los fluidos orgánicos por lo que
PARED. pueden formarse depósitos en el líquido sinovial y tejidos, aunque esto no sucede en todos los
individuos. Son concentración depende tanto de la edad como del género. Se considera
primaria cuando existe algún defecto enzimático de la síntesis de purinas; y secundaria por
sobreproducción o disminución de la excreción del ácido úrico. Se debe a la sobreproducción o
subexcreción de ácido úrico (a menudo ambas). IMPORTANTE: La ingestión de alcohol favorece
la hiperuricemia por aumento en la producción de urato y disminución de la excreción renal de
ácido úrico.

ACTIVOS DE La lesión característica de la gota es el tofo, un depósito nodular de cristales de monohidrato de


URATOS. urato monosódico en asociación con una reacción a cuerpo extraño. Se ha encontrado urato en
tejidos sinoviales y líquido sinovial durante la artritis aguda; en realidad se cree que la
inflamación aguda en la gota se activa mediante fagocitosis de cristales de urato por los
monocitos y sinoviocitos.

SEMEJANTES A Una vez dentro de las células, los cristales de gota se procesan por los "receptores semejantes a
TOLL. Toll" y activa los inflamosomas NALP-3, que a su vez liberan varios agentes quimiotácticos y
citocinas capaces de mediar la inflamación.

Bibliografía:
1. GUÍA DE PRÁCTICA CLÍNICA, PREVENCIÓN, DIAGNÓSTICO, TRATAMIENTO Y REFERENCIA OPORTUNA DE
HIPERURICEMIA Y GOTA. MÉXICO: SECRETARIA DE SALUD, 2009. 2. PAPADAKIS MAXINE A, MCPHEE
STEPHEN J. DIAGNÓSTICO CLÍNICO Y TRATAMIENTO. 52ª EDICIÓN. NUEVA YORK. 2013, PP 812-813.

http://www.cenetec.salud.gob.mx/descargas/gpc/CatalogoMaestro/216_SSA_09_Hiperuricemia_Gota/SSA-216-09_GOTAEVR.pdf
ANÁLISIS DEL CASO CLÍNICO

IDENTIFICACIÓN DEL REACTIVO


Area: CIRUGÍA
Especialidad: PROCTOLOGÍA Y UROLOGÍA
Tema: PATOLOGÍA DE LA PROSTATA
Subtema: CÁNCER DE PROSTATA

CASO CLÍNICO CON UNA PREGUNTA

HOMBRE DE 40 AÑOS DE EDAD QUE ACUDE A REVISIÓN GENERAL A SU CONSULTORIO. ACTUALMENTE SE


ENCUENTRA ASINTOMÁTICO.

HOMBRE DE 40 AÑOS.

MUY IMPORTANTE QUE NO SE REFIERE


ANTECEDENTE FAMILIAR DE CÁNCER DE
PRÓSTATA.

ASINTOMÁTICO.

-.

-.

137 - DURANTE LA EXPLORACIÓN FÍSICA USTED REALIZARÍA TACTO RECTAL PARA VALORAR LA
PROSTATA, EN CASO DE QUE EL PACIENTE CONTARÁ CON EL SIGUIENTE FACTOR DE RIESGO:
EXPOSICIÓN A El CÁNCER DE PRÓSTATA es el crecimiento anormal y desordenado de las células del epitelio
RADIACIÓN glandular que tienen capacidad de diseminarse. Los factores de riesgo confirmado para cáncer
ULTRAVIOLETA. de próstata son: antecedentes familiares, raza negra, edad avanzada. DETECCIÓN TEMPRANA.
De acuerdo a los resultados del estudio PLCO (próstata, pulmón, colorrectal y ovario) realizado
en Estados Unidos, después de 7 a 10 años de seguimiento, la tasa de mortalidad por cáncer de
próstata es muy baja y no difiere significativamente entre el grupo con y sin detección de cáncer
de próstata. El ERSPC (Estudio Europeo Aleatorizado para la Detección de Cáncer de Próstata) en
Europa, demostró que la detección basada en el APE reduce la tasa de mortalidad por cáncer de
próstata en 20% pero se asocia con un elevado riesgo de sobre diagnóstico. No se recomienda la
realización de tamizaje de cáncer de próstata a población masculina asintomática entre 50 a 69
años de edad empleando la prueba de APE y el TR, debido a que se tiene evidencia de que no es
efectivo en la reducción de la mortalidad por cáncer de próstata; a que su implantación
representaría un elevado impacto presupuestal y a que se expondría a los pacientes a riesgos
innecesarios. Dada la importancia creciente de esta enfermedad para la población mexicana se
recomienda estudiar la posibilidad de otros programas de sensibilización, prevención y
detección oportuna del cáncer de próstata. Dichos programas deberán acompañarse de difusión
de medidas de prevención y de promoción de conductas saludables entre la población en
general. La Sociedad Americana de Cáncer (ACS) y la Asociación Americana de Urología (AUA)
recomiendan que los hombres con factores de riesgo alto para cáncer de próstata, como son
historia familiar y raza negra, comiencen una detección temprana (APE y TR) antes de los 50
años. La AUA recomienda que sea a los 40 años, mientras que la ACS recomienda a los 45 años.
NO SE RECOMIENDA LA DETECCIÓN EN PACIENTES SIN FACTORES DE RIESGO. IMPORTANTE: la
relación entre cáncer de próstata y la exposición a radiación ultravioleta no se encuentra
confirmada, se encuentra aún en debate.

PROMISCUIDAD. Factores de riesgo en debate aún: alimentación, conducta sexual, consumo de alcohol,
exposición a radiaciones ultravioleta, exposición ocupacional.

ALCOHOLISMO. OJO. El cáncer de próstata familiar se presenta en el 9% de los pacientes confirmados; es


definido cuando dos o más familiares afectados han desarrollado la enfermedad a edad
temprana (antes de los 55 años). Los pacientes con cáncer de próstata hereditario usualmente lo
presentan seis a siete años antes que aquellos con cáncer espontáneo.

ANTECEDENTE Los factores que determinan el riesgo de desarrollar esta enfermedad en forma clínica no son
FAMILIAR bien conocidos, sin embargo, se han identificado algunos, de los cuales el factor hereditario
DIRECTO. parece ser el más importante; si un familiar de primera línea tiene la enfermedad, el riesgo se
incrementa al doble. Si dos o mas familiares de primera línea son afectados el riesgo se
incrementa de 5 a 11 veces. Realizar tacto rectal y APE a pacientes masculinos mayores de 40
años con antecedentes familiares de línea directa de cáncer de próstata, para realizar el
diagnóstico oportuno. NO SE RECOMIENDA EL TAMIZAJE GENERALIZADO CON ANTÍGENO
PROSTÁTICO Y/O TACTO RECTAL A LA POBLACIÓN GENERAL. EN PACIENTES CON HISTORIA
FAMILIAR DE CÁNCER PROSTÁTICO Y RAZA NEGRA SE RECOMIENDA UNA DETECCIÓN
TEMPRANA ANTES DE LOS 50 AÑOS.

Bibliografía:
1. GUÍA DE PRÁCTICA CLÍNICA, DIAGNÓSTICO, PREVENCIÓN Y DETECCIÓN TEMPRANA DEL CÁNCER DE
PRÓSTATA EN EL PRIMER NIVEL DE ATENCIÓN. MÉXICO: SECRETARIA DE SALUD; 21 MARZO 2013.

http://www.cenetec.salud.gob.mx/descargas/gpc/CatalogoMaestro/021_GPC_CaProstata/SSA_021_08_EyR.pdf
ANÁLISIS DEL CASO CLÍNICO

IDENTIFICACIÓN DEL REACTIVO


Area: MEDICINA INTERNA
Especialidad: INMUNOALERGIA
Tema: RINITIS ALÉRGICA
Subtema: RINITIS ALÉRGICA

CASO CLÍNICO SERIADO

MUJER DE 25 AÑOS CON ANTECEDENTE DE RINITIS ALÉRGICA, PRESENTA DESDE HACE 5 DÍAS RINORREA
PURULENTA, FIEBRE, OBSTRUCCIÓN NASAL Y DOLOR FACIAL DE TIPO OPRESIVO DE UNA SEMANA DE
EVOLUCIÓN.

mujer de 25 años.

RINITIS ALÉRGICA.

5 DÍAS CON RINORREA PURULENTA,


FIEBRE, OBSTRUCCIÓN NASAL, DOLOR
FACIAL OPRESIVO DE una SEMANA DE
EVOLUCIÓN.

-.

-.

138 - EL SITIO ANATÓMICO DE AFECCIÓN MÁS FRECUENTE DE LAS SINUSITIS AGUDAS BACTERIANAS ES:

EL SENO SINUSITIS MAXILAR - En las formas agudas se acompaña de cefalea suborbitaria que irradia a
MAXILAR. maxilar y a órbita, con algia facial localizada a la presión sobre la pared anterior del seno maxilar.
Rinorrea mucopurulenta que sale a la fosa por meato medio. - En las formas de origen dentario
es típica la rinorrea fétida, por presencia de gérmenes anaerobios.

EL SENO SINUSITIS ETMOIDAL. - Es rara que se presente aislada y suele asociarse a la sinusitis maxilar. La
ETMOIDAL. cefalea se localiza en la raíz nasal y en el ángulo interno de la órbita, siendo dolorosa la presión a
dicho nivel. - La rinorrea mucopurulenta sale a la fosa por el meato medio.
EL SENO SINUSITIS FRONTAL. - La cefalea intensa supraorbitaria suele ser a menudo pulsátil. En ocasiones
FRONTAL. se acompaña de fotofobia y cierta obnubilación mental. - La rinorrea mucopurulenta aparece en
la fosa por el meato medio. Son muy típicas las formas barotraumáticas por cambios bruscos de
presión (vuelo en avión, inmersiones).

COMPLEJO - La sinusitis es uno de los motivos de consulta médica más frecuentes. Afecta tanto a niños
OSTEOMEATAL. como a adultos. LA AFECCIÓN DE UN ÚNICO SENO ES POCO HABITUAL. - El complejo
osteomeatal (COM) se encuentra en el meato medio y es un espacio reducido donde
desembocan, antes de llegar a la cavidad nasal, los senos etmoidales anteriores, los maxilares y el
frontal. El infundíbulo es el espacio del COM que se congestiona fácilmente y provoca el
desarrollo de la sinusitis. EL COMPLEJO OSTEOMEATAL, QUE ES LA ZONA DE CONFLUENCIA DE
VARIOS SENOS EN LA FOSA NASAL, ES LA ZONA QUE CON MAYOR FRECUENCIA SE AFECTA.

Bibliografía:
1. PAPADAKIS MAXINE A, MCPHEE STEPHEN J. DIAGNÓSTICO CLÍNICO Y TRATAMIENTO. 52ª EDICIÓN.
NUEVA YORK. 2013, PP 214.

139 - PARA CONFIRMAR EL DIAGNÓSTICO DEL PACIENTE SE DEBE SOLICITAR EL SIGUIENTE ESTUDIO:

RADIOGRAFÍA. RADIOGRAFÍA DE SENOS (Proyecciones de Cadwell y Waters). - En general el uso de la radiología


simple no está indicado en el diagnóstico de la sinusitis. Un velamiento periférico de la cavidad se
ve frecuentemente, siendo un hallazgo poco específico. Son hallazgos más específicos la presencia
de una opacificación total del seno o de niveles hidroaéreos (pero sólo se observan estos
hallazgos en un 60% de los pacientes con sinusitis). EN CASO DE RINOSINUSITIS AGUDA (ANTES
DE 10 DÍAS) NO SE JUSTIFICA EL USO DE RADIOGRAFÍAS.

TOMOGRAFÍA LA PRUEBA RADIOLÓGICA CONFIRMATORIA QUE SE DEBE SOLICITAR EN CASO DE SOSPECHA DE


COMPUTADA. SINUSITIS ES LA TOMOGRAFÍA COMPUTADA (TC), YA QUE LA RX SIMPLE DE SENOS SE HA
MOSTRADO UN ALTO ÍNDICE DE FALSOS POSITIVOS Y NEGATIVOS. TC DE SENOS: - La TC revela
la presencia de niveles hidroaéreos, engrosamientos, mucosos, obstrucción o bloqueo del
complejo ostiomeatal y factores anatómicos predisponentes. Para la correcta valoración de estos
hallazgos hay que recordar que hasta en un 30-60% de pacientes asintomáticos se puede
encontrar como hallazgo engrosamientos de la mucosa de alrededor de 3 mm. - La TC será
obligada para la valoración de la presencia de complicaciones en la evolución. - La diferencia de la
tomografía computarizada respecto a las radiografías simples y a la resonancia magnética, es la
capacidad de obtener imágenes de cortes axiales y coronales. LA TC REPRESENTA UN MEDIO
RÁPIDO Y EFICAZ DE VALORAR TODOS LOS SENOS PARANASALES Y ACELERAR EL TRATAMIENTO
APROPIADO. Es inespecífica para definir la etiología de la enfermedad. OJO: La pregunta
específica prueba confirmatoria, sin embargo debes considerar que cuando la clínica justifica el
diagnóstico no es necesaria ninguna prueba más.

RESONANCIA - La RESONANCIA MAGNÉTICA (RM) tiene un papel reservado a la evaluación de complicaciones


MAGNÉTICA. intracraneales u orbitarias, y para el diagnóstico diferencial con neoplasias sinusales. - La RM está
indicada sólo cuando la TC no diferencia adecuadamente las imágenes orbitarias o intracraneanas
ya que la RM puede ayudar a distinguir mejor los planos tisulares o un tumor.

CULTIVO DE PUNCIÓN SINUSAL: Es la técnica gold-standard pero debido a su invasividad no se practica


SECRECIÓN habitualmente. EL CULTIVO DE SECRECIÓN OBTENIDA POR PUNCIÓN SINUSAL ES EL ESTÁNDAR
NASAL. DE ORO, LA SECRECIÓN NASAL NO ES DE UTILIDAD EN ÉSTE SENTIDO YA QUE NO COMPRUEBA
LA EXISTENCIA DE LA MISMA EN LA REGIÓN SINUSAL. Debes considerar que el cuadro clínico es
suficiente para determinar el diagnóstico por lo que la prueba confirmatoria deberá ser lo menos
invasiva posible.

Bibliografía:
1. PAPADAKIS MAXINE A, MCPHEE STEPHEN J. DIAGNÓSTICO CLÍNICO Y TRATAMIENTO. 52ª EDICIÓN.
NUEVA YORK. 2013, PP 214.
140 - EL TRATAMIENTO INDICADO EN ESTE PACIENTE DEBE SER CON:

MACRÓLIDOS. - La antibioticoterapia para la sinusitis aguda y aguda recurrente generalmente es empírica,


y está basada en el conocimiento de que los patógenos más comunes son Streptococcus
pneumoniae y Haemophilus influenzae en el 76%; anaerobios, otras especies de
estreptococos, Moraxella catarrhalis y Staphylococcus aureus, entre el 3 y 7%. Es importante
prescribir el antibiótico adecuado y la dosis y duración del mismo, para evitar en lo posible
resistencias bacterianas. - El tiempo ideal de un antibiótico para sinusitis aguda es de 10 a
14 días. La principal razón de fracaso del tratamiento de la sinusitis aguda es seleccionar un
antibiótico inadecuado o tratar el problema como alérgico en vez de infeccioso, esto último
sucede más en pacientes pediátricos. SÓLO LOS PACIENTES ALÉRGICOS A
BETALACTÁMICOS SE TRATARAN CON MACRÓLIDOS COMO CLARITROMICINA O
AZITROMICINA.

CEFALOSPORINAS. Las Cefalosporinas de 2ª generación (cefuroxima axetilo o cefpodoxima proxetilo) son de


segunda elección. - En pacientes con infección grave con mala respuesta al tratamiento
inicial, sospecha o presencia de complicación se debe recurrir al tratamiento vía parenteral
con cefalosporinas de 3ª generación (ceftriaxona o cefotaxima [iv], que pueden asociarse a
vancomicina. - En el caso de los niños, para pacientes que vomitan o rechazan antibióticos
orales se ofrecería cefriaxona.

AMINOPENICILINAS. El esquema recomendado en México de acuerdo a las GPC para el diagnóstico y tratamiento
de la sinusitis aguda es: - Amoxicilina 500mg c/8hrs durante 10 a 14 días por vía oral. En
caso de alergia utilizar: - Trimetropima/Sulfametoxazol 160/800mg c/12hrs durante 10 a 14
días. EL MANEJO CON AMOXICILINA (UNA AMINOPENICILINA) ES EL MANEJO DE PRIMERA
LÍNEA DE LA SINUSITIS.

QUINOLONAS. La Amoxicilina + Ac. Clavulánico constituyen el manejo de primera elección, en alérgicos a


betalactámicos puede emplearse fluorquinolonas como el levofloxaciono y el
moxifloxacino, dado el elevado número de resistencias del neumococo a algunos
macrólidos.

Bibliografía:
1. GUÍA DE PRÁCTICA CLÍNICA, DIAGNÓSTICO Y TRATAMIENTO DE SINUSTIS AGUDA. MÉXICO: SECRETARIA
DE SALUD; 2009.

http://www.cenetec.salud.gob.mx/descargas/gpc/CatalogoMaestro/080_GPC_Sinusitisaguda/sinusitis_evr_cenetec.pdf

FIN DEL CASO CLÍNICO SERIADO


ANÁLISIS DEL CASO CLÍNICO

IDENTIFICACIÓN DEL REACTIVO


Area: GINECOLOGÍA Y OBSTETRICIA
Especialidad: GINECOLOGÍA
Tema: DOLOR PELVICO
Subtema: ENDOMETRIOSIS

CASO CLÍNICO SERIADO

MUJER DE 25 AÑOS DE EDAD, CASADA Y SIN PODER EMBARAZARSE DESDE HACE 3 AÑOS. REFIERE HISTORIA
DE DOLOR PÉLVICO CRÓNICO, DISMENORREA Y DISPAREUNIA. NUNCA HA UTILIZADO MÉTODO
ANTICONCEPTIVO Y SÓLO HA TENIDO UNA PAREJA SEXUAL.

Mujer de 25 años de edad.

Casada, SIN MÉTODO DE PLANIICACIÓN


FAMILIAR, INFERTILIDAD.

Dismenorrea, infertilidad, dolor pélvico


crónico y dispareunia.

-.

-.

141 - PARA CONFIRMAR EL DIAGNÓSTICO DE LA PACIENTE SE DEBERÁ:

REALIZAR LO PRIMERO QUE DEBES HACER EN ESTA PACIENTE ES IDENTIFICAR EL DIAGNÓSTICO


BIOPSIA PROBABLE EN ESTE CASO ES ENDOMETRIOSIS PUES SE ASOCIA INFERTILIDAD + DISMENORREA
ENDOMETRIAL. + DISPAREUNIA. BIOPSIA ENDOMETRIAL es un procedimiento en el cual se toma una muestra
de tejido del revestimiento del útero (Endometrio), y se examina bajo el microscopio en
búsqueda de cualquier tipo de células anormales o signos de cáncer. El examen se hace para
encontrar la causa de: • Períodos menstruales anormales (Sangrado profuso, prolongado o
irregular). • Sangrado después de la menopausia. • Sangrado a causa de medicamentos de
hormonoterapia. • Engrosamiento del revestimiento uterino observado en una ecografía. • El
examen generalmente se lleva a cabo en mujeres de más de 35 años. • Se usa para investigar
cáncer endometrial. NO CUMPLE CON CRITERIOS PARA REALIZAR ESTE PROCEDIMIENTO.
SOLICITAR El ULTRASONIDO TRASVAGINAL se debe realizar en el protocolo de estudio de la paciente,
ULTRASONIDO porque nos permite evaluar estructuras con mejor resolución, particularmente si se encuentran
PÉLVICO O tumores anexos, pero este estudio "no nos confirma el diagnostico". Permite la exploración no
TRANSVAGINAL. invasiva de la pelvis. Por su bajo costo y su inocuidad ha sido ampliamente utilizada. La
ecografía abdominal posee una gran sensibilidad en la definición de las masas pélvicas aunque
su valor es más limitado para clarificar el origen de las mismas, la ecografía transvaginal ha
permitido mejorar la capacidad diagnóstica de las masas pélvicas y aunque se han intentado
descubrir hallazgos ultrasonográficos característicos de endometriosis, su diagnóstico por
medios no invasivos sigue planteando problemas. Tienen un valor limitado en el diagnóstico de
endometriosis peritoneal, per es buena herramienta si las lesiones se sospechan a nivel de
ovarios. Tienen una sensibilidad del 83% y una especificidad de 98%. Se recomienda como
estudio de gabinete “inicial”. IMPORTANTE: La clave de esta pregunta es: que se solicita el
estudio que permita CONFIRMAR EL DIAGNÓSTICO y esto sólo sería a través de la laparoscopía,
que permite la visualización directa de las lesiones y toma de muestras para estudio
histopatológico. NO ES CONFIRMATORIA DE ENDOMETRIOSIS SÓLO COMO PARTE DEL
PROTOCOLO.

REALIZAR La ENDOMETRIOSIS es la presencia de tejido endometrial funcional, glándulas y estroma fuera


LAPAROSCOPIA. de la cavidad uterina. Los sitios mayormente afectados en orden de frecuencia son: ovarios,
fondo de seco de Douglas, ligamento ancho y ligamentos útero-sacros. Las manifestaciones
clínicas de esta son: dolor pélvico crónico, infertilidad y masa anexial. Los síntomas pueden
incluir dismenorrea, “dispareunia”, disuria disquecia, dolor lumbar bajo y dolor pélvico crónico
(dolor pélvico y abdominal no cíclico de más de 6 meses). A la exploración física bimanual es
posible encontrar dolor pélvico, útero fijo, ligamentos utero-sacros dolorosos u ovarios
aumentados de tamaño. La LAPAROSCOPÍA es una técnica que nos permite observar la cavidad
abdominal y pélvica. Permite intervenciones quirúrgicas, por lo que también se considera un
sistema de cirugía de mínima invasión. En ginecología es útil, porque a través de éste
procedimiento se confirma el diagnóstico de endometriosis y es posible además otorgar
tratamiento mediante ablación de las lesiones. Todas las lesiones mayores de 4cm deberán ser
sospechosas de lesiones malignas. IMPORTANTE: la visualización directa de la lesión es
suficiente para el diagnóstico, aunque no excluye la necesidad de examen histológico. EL
ESTÁNDAR DE ORO PARA DIAGNÓSTICAR ENDOMETRIOSIS ES MEDIANTE LA EVALUACIÓN
HISTOLÓGICA DE LAS LESIONES, lo cual requiere de realizar laparoscopía.

SOLICITAR Si se pueden solicitar niveles de hormonas ováricas con la finalidad de investigar la función
NIVELES DE ovárica pero en este caso no nos hace el diagnóstico de endometriosis. NO SON ÚTILES PARA EL
HORMONAS DIAGNÓSTICO DE ENDOMETRIOSIS.
OVÁRICAS.

Bibliografía:
1. GUÍA DE PRÁCTICA CLÍNICA, DIAGNÓSTICO Y TRATAMIENTO DE LA ENDOMETRIOSIS. MÉXICO:
SECRETARIA DE SALUD; MARZO 2013.

http://www.cenetec.salud.gob.mx/descargas/gpc/CatalogoMaestro/207_SSA_10_ENDOMETRIOSIS/EyR_SSA_207_09.pdf

142 - DADO QUE LA PACIENTE DESEA EMBARAZARSE EL SIGUIENTE TRATAMIENTO ESTÁ INDICADO EN
ESTE MOMENTO:
FERTILIZACIÓN La FECUNDACIÓN IN VITRO (FIV o IVF por sus siglas en inglés) es una técnica por la cual la
IN VITRO. fecundación de los ovocitos por los espermatozoides se realiza fuera del cuerpo de la madre. La
FIV es el principal tratamiento para la esterilidad cuando otros métodos de reproducción asistida
no han tenido éxito. El proceso implica el control hormonal del proceso ovulatorio, extrayendo
uno o varios ovocitos de los ovarios maternos, para permitir que sean fecundados por
espermatozoides en un medio líquido. El ovocito fecundado (el pre-embrión) pueden entonces
ser transferidos al útero de la mujer, para iniciar un embarazo, siempre antes de los 14 días desde
que el ovocito es fecundado, tiempo durante el cual, el cigoto recibe el nombre de pre-embrión,
después de esas 2 semanas recibe el nombre de embrión hasta los 57 días desde la fecundación
del óvulo. A partir de esos 57 días, ya empieza a tener forma humana y recibe el nombre de feto.
NO TIENE INDICACIÓN PARA ESTE TIPO DE TRATAMIENTO DE FERTILIZACIÓN YA QUE NO HA
PASADO ANTES POR MÉTODOS DE FERTILIZACIÓN ASISTIDA. Ojo: en mujeres con diagnóstico de
endometriosis que son candidatas a fertilización in vitro, la supresión con agonistas de la GnRH
más terapia hormonal de soporte por 3 a 6 meses antes de la fertilización in vitro está asociada a
un incremento de la tasa de embarazo. LA FERTILIZACIÓNIN VITRO ES EL ÚLTIMO RECURSO Y SI
SE APOYA PREVIO DE HORMONALES TIENE MAYOR ÉXITO.

AGONISTAS DE Después de una adecuada evaluación inicial de la ENDOMETRIOSIS y falla en la respuesta con
GNRH. anticonceptivos orales combinados y AINE, es apropiada la terapia empírica con un periodo de 3
meses de agonista de la GnRh. NO ES EL TRATAMIENTO DE PRIMERA LÍNEA.
QUIRÚRGICO. QUIRÚRGICO. ENDOMETRIOSIS. a. TRATAMIENTO MÉDICO GENERAL. 1. AINES: primera línea en
el manejo de pacientes con dismenorrea + dolor pélvico, independientemente de su origen, no
precisamente asociado a endometriosis. 2. ANTICONCEPTIVOS ORALES COMBINADOS:
administrados en forma continua, constituyen el tratamiento de primera línea para pacientes con
dolor pélvico “asociado a endometriosis”. La supresión ovárica con estos medicamentos se deberá
llevar a cabo por un lapso de 6 meses. La evidencia sugiere que si transcurren 3 meses de
tratamiento y no se observa reducción de dolor, deben suspenderse. 3. ANÁLOGOS DE LA
HORMONA LIBERADORA DE GONADOTROFINA (GnRH): constituye una segunda opción al
tratamiento del dolor pélvico asociado a endometriosis. . Después de una adecuada evaluación
inicial y falla terapéutica con anticonceptivos orales combinados y AINE, es apropiada a terapia
empírica con 3 meses de GnRH. No se recomiendan por más de 6 meses. El medicamento más
representativo es la leuprolida, otros son: el buserelin, goserelin, histrelin, nafareliny triptorelin.
OJO: tienen efectos adversos derivado de la inhibición de la producción de estrógenos, tales
como pérdida ósea. Se recomienda el uso de terapia complementaria con estrógenos a partir de
los tres meses de tratamiento. 4. DANAZOL: administrado de 3 a 6 meses, reduce el dolor pélvico
asociado a endometriosis. Su uso es limitado por los efectos androgénicos que produce, además
de tener un impacto desfavorable en las concentraciones circulantes de lípidos. 5. PROGESTINA:
sola (oral, intramuscular o subcutánea), también se puede considerar como primera línea de
tratamiento en el dolor pélvico asociado a endometriosis, aunque la evidencia sugiere que no ha
resultado más efectiva que los hormonales combinados, los análogos de GnRH o el Danazol.
IMPORTANTE: - La supresión ovárica con anticonceptivos combinados o con análogos de la GnRH
en infertilidad asociada a endometriosis NO ES EFECTIVA. - En los casos de “endometriosis
severa”, el tratamiento médico es insuficiente. - La terapia médica mejora los índices de dolor,
pero tiene una alta recurrencia de los síntomas. - Cuando el tratamiento médico ha fallado, el
manejo quirúrgico “definitivo” está indicado, siempre que no se desee fertilidad futura. b.
TRATAMIENTO QUIRÚRGICO GENERAL. 1. ABLACIÓN DE LA INERVACIÓN (ligamentos útero-
sacros): por sí sola no reduce el dolor asociado a endometriosis. 2. ABLACIÓN DE LESIONES
ENDOMETRIÓSICAS: sólo aplica para endometriosis mínima a moderada y se prefiere junto con
ablación de la inervación. 3. NEURECTOMÍA PRESACRA: reduce el dolor solo en la línea media. Se
ha asociado con constipación y disfunción urinaria postquirúrgica. Sólo puede utilizarse como
terapia coadyuvante al manejo quirúrgico. 4. HISTERECTOMÍA TOTAL ABDOMINAL CON
SALPINGOOFERECTOMÍA BILATERAL + ESCISIÓN DE IMPLANTES: se recomienda sólo en caso de
recurrencia de dolor pélvico, falla terapéutica al tratamiento médico postquirúrgico y siempre
que exista paridad satisfecha. 5. CISTECTOMÍA: recomendada en endometriomas mayores a 3cm
en mujeres con dolor pélvico. IMPORTANTE: - La ablación de inervación no resulta efectiva para
el tratamiento del dolor por si sola. - La combinación de ablación endometriósica y de inervación
reduce el dolor a 6 meses. - EL HALLAZGO INSIDENTAL DE ENDOMETRIOSIS EN MUJERES
ASINTOMÁTICAS NO REQUIERE TRATAMIENTO MÉDICO NI QUIRÚRGICO. - Siempre tomar
biopsias para descartar malignidad. c. TRATAMIENTO QUIRÚRGICO EN ENDOMETRIOSIS
ASOCIADA A ESTERILIDAD. 1. ABLACIÓN DE LESIONES + ADHERENCIAS: se recomienda la cirugía
laparoscópica quirúrgica con ablación de lesiones endometriósicas y adherenciolisis en casos de
infertilidad asociada a endometriosis mínima a moderada, ya que mejora la fertilidad (mayores
tazas de embarazo). IMPORTANTE: - Las técnicas de reproducción asistida son una opción para el
manejo de casos de infertilidad generada por endometriosis, con sus indicaciones individuales
para cada caso. PARA ESTA PACIENTE SE RECOMIENDA LAPAROSCOPÍA QUIRÚRGICA CON
ABLACIÓN DE LESIONES ENDOMETRIÓSICAS Y ADHERENCIOLISIS.

DANAZOL. El DANAZOL está indicado para tratar los síntomas asociados a la endometriosis y/o para reducir
la extensión del foco endometriósico. Puede ser utilizado en conjunto con la cirugía o como
Monoterapia hormonal en pacientes que no responden a otros tratamientos. SU PRINCIPAL
INDICACIÓN ES EL ALIVIO DEL DOLOR Y NO EL MANEJO DE LA INFERTILIDAD.

Bibliografía:
1. GUÍA DE PRÁCTICA CLÍNICA, DIAGNÓSTICO Y TRATAMIENTO DE LA ENDOMETRIOSIS. MÉXICO:
SECRETARIA DE SALUD; MARZO 2013.

http://www.cenetec.salud.gob.mx/descargas/gpc/CatalogoMaestro/207_SSA_10_ENDOMETRIOSIS/EyR_SSA_207_09.pdf

FIN DEL CASO CLÍNICO SERIADO


ANÁLISIS DEL CASO CLÍNICO

IDENTIFICACIÓN DEL REACTIVO


Area: PEDIATRÍA
Especialidad: URGENCIAS PEDIÁTRICAS
Tema: PATOLOGÍA NEONATAL
Subtema: HIPERBILIRRUBINEMIA E INCOMPATIBILIDAD A GRUPO Y RH

CASO CLÍNICO CON UNA PREGUNTA

RECIÉN NACIDA DE 4 DIAS, QUE INGRESA POR URGENCIAS POR ICTERICIA +++, LAS CIFRAS DE BILIRRUBINA
INDIRECTA SE ENCUENTRAN ELEVADAS.

recién nacida 4 días de vida.

---

ictericia +++

ictericia+++

bilirrubina directa elevada

143 - LA CONCENTRACION ELEVADA DE BILIRRUBINA ES DEBIDA A UNA DEFICIENCIA DEL


METABOLISMO DE LOS:

ÁCIDOS ÁCIDOS ALDÁRICOS: Intervienen en la oxidación de los monosacáridos. En los ácidos aldáricos,
ALDÁRICOS se oxidan los dos átomos de C con alcoholes primarios. Es una oxidación enérgica y afecta al C
del grupo carbonilo y al del OH primario. El de la glucosa se denomina “ácido
glucárico”o“sacárico”. Conjuntamente con los ácidos aldáricos y aldónicos, aparecen unos
compuestos que químicamente son lactonas, que se forman mediante una reacción de
esterificación intermolecular con liberación de una molécula de agua. Por lo tanto, reacciona el
grupo ácido del C1 con uno de los grupos alcohólicos de la aldosa.

ÁCIDOS Intervienen en la oxidación de los monosacáridos junto con los ácidos aldáricos. En los ácidos
GLUCÁRICOS aldáricos, se oxidan los dos átomos de C con alcoholes primarios. Es una oxidación enérgica y
afecta al C del grupo carbonilo y al del OH primario. El de la glucosa se denomina “ácido
glucárico”o“sacárico”. Conjuntamente con los ácidos aldáricos y aldónicos, aparecen unos
compuestos que químicamente son lactonas, que se forman mediante una reacción de
esterificación intermolecular con liberación de una molécula de agua. Por lo tanto, reacciona el
grupo ácido del C1 con uno de los grupos alcohólicos de la aldosa.
ÁCIDOS En el metabolismo la bilirrubina se une a proteínas citoplasmáticas y es transportada al retículo
GLUCURÓNICOS endosplásmico. En el retículo endoplásmico del hepatocito, LA BILIRRUBINA ES CONJUGADA
CON EL ÁCIDO GLUCURÓNICO MEDIANTE LA URIDIN-DI FOSFATO-GLUCURONILTRANSFERASA
(UDGP-TRANSFERASA), DANDO LUGAR A MONO Y DIGLUCURÓNIDOS DE BILIRRUBINA.
Mediante este proceso de conjugación, la bilirrubina pierde sus efectos tóxicos sobre el
organismo. La bilirrubina conjugada es excretada a través de la membrana del hepatocito al
canalículo biliar, posteriormente se agrega a la bilis y llega hasta duodeno a través del árbol
biliar. La bilirrubina conjugada o directa es hidrosoluble y por lo tanto, puede eliminarse por la
orina. Cuando se ve alterado este proceso es cuando se eleva la bilirrubina. LA DEFICIENCIA EN
EL METABOLISMO DEL ÁCIDO GLUCURÓNICO PROVOCA QUE NO SE CONJUGUE LA
BILIRRUBINA, ENCONTRÁNDOSE ENTONCES NIVELES ALTOS DE BILIRRUBINA INDIRECTA.

DEL SORBITOL La vía del poliol o sorbitol es una cascada de reacciones químicas en la cual se obtiene fructosa a
partir de la glucosa, pasando por el sorbitol con la ayuda de la enzima aldosa reductasa.

Bibliografía:
BIOQUÍMICA DE HARPER. MURRAY. EL MANUAL MODERNO. EDICIÓN 15. 2001. PAG. 423.
ANÁLISIS DEL CASO CLÍNICO

IDENTIFICACIÓN DEL REACTIVO


Area: PEDIATRÍA
Especialidad: URGENCIAS PEDIÁTRICAS
Tema: URGENCIAS QUIRÚRGICAS Y PATOLOGIAS DE RESOLUCIÓN
QUIRÚRGICA
Subtema: ESCROTO AGUDO

CASO CLÍNICO CON UNA PREGUNTA

LACTANTE DE 2 MESES DE EDAD CON AUMENTO DE VOLUMEN EN ESCROTO DERECHO DESDE EL


NACIMIENTO., QUE NO INCREMENTA DE TAMAÑO CON EL ESFUERZO NI EL LLANTO. A LA EXPLORACIÓN
ENCUENTRA AUMENTO DE VOLUMEN EN ESCROTO DERECHO, NO SE PALPA TESTÍCULO, SE ENCUENTRA
TENSO Y TRANSILUMINA. TESTÍCULO IZQUIERDO NORMAL.

lactante de 2 meses de edad

re ere aumento de volumen en el escroto


desde su nacimiento

aumento en el escroto derecho , no


incrementa con el llanto ni el esfuerzo

Aumento de volumen en escroto derecho,


no se palpa testículo, se encuentra tenso y
transilumina. Testículo izquierdo normal.

--

144 - EL DIAGNOSTICO DEL PACIENTE ES:

HIDROCELE El Hidrocele comunicante es el acúmulo del líquido peritoneal en la túnica vaginal a través de una
COMUNICANTE persistencia del peritoneovaginales. Esto indica que hay comunicación con la cavidad abdominal
se debe de considerar como hernia inguinal y tratarse como tal. El cuadro clínico característico es
la tumefacción escrotal que aparece y desaparece dependiendo del nivel de actividad y relajación.
A menudo la presión suave reduce el líquido de la hidrocele desde el escroto a la cavidad
abdominal, pero es típico que el líquido reaparezca en forma súbita cuando incremento en la
presión intraabdominal. Su diagnóstico es clínico, la transiluminación es positiva. El tratamiento
es quirúrgico y es la obliteración o cierre de la persistencia del peritoneovaginales.
HIDROCELE NO El hidrocele no comunicante es el acúmulo del líquido peritoneal en la túnica vaginal, pero no
COMUNICANTE relacionado con proceso peritoneovaginalis permeable. Es muy común en recién nacidos
masculinos y es autolimitado, por lo regular se resuelve en 6 a 12 meses. Este tipo de hidroceles
pueden estar presentes al nacimiento o desarrollarse meses o años después sin una razón obvia.
La evolución es de un tamaño estable sin cambios rápidos de tamaño. El diagnóstico es clínico,
transiluminación positiva. Su tratamiento es la vigilancia, no requiere tratamiento médico. Puede
desaparecer entre los 6 a 12 meses de edad, si posterior a esta edad continua hay que descartar
no se trate de un hidrocele comunicante.

HERNIA La hernia inguinal indirecta es la más frecuente en los pacientes pediátricos. Su incidencia es del 1
INGUINAL a 5 %. Más frecuentes en los niños que en las niñas con una proporción de 8:1 a 10:1.La hernia
ENCARCELADA inguinal se produce por la persistencia del proceso peritoneovaginalis permeable. Por este
proceso permeable se introducen vísceras lo más frecuente es que sean asas intestinales,
apéndice, divertículo de Meckel, ovarios, trompas etc... Una hernia inguinal encarcelada es una
complicación de la hernia sino ha sido operada. Una hernia encarcelada se presenta cuando el
contenido herniario queda atrapado en el saco herniario, sin compromiso circulatorio. Produce
dolor e imposibilidad para reducirla. El tratamiento es con cirugía.

TUMOR Tumor testicular es una neoplasia poco frecuente en la edad pediátrica, es sólo el 2 al 5% de
TESTICULAR todos los tumores testiculares. La edad de mayor incidencia es a los dos años, aunque hay un pico
elevado en los adolescentes. Existen varios tipos de tumores, los de células germinales provocan
cerca de las tres cuartas partes de las neoplasias testiculares en niños. Los tumores testiculares en
niños aparecen como masas indoloras e insensibles. Por lo general son sólidas, no transilumina. Y
hasta un tercio de los pacientes presentan hidrocele secundaria. El diagnóstico clínico debe de
complementarse con estudios de laboratorio y gabinete, y el estudio patológico se realizará
después de la resección de la tumoración con lo que tendremos el diagnóstico definitivo.

Bibliografía:
PEDIATRIC SURGERY. ASHCRAFT, KEITH. W.B. SAUNDERS. EDICIÓN 3A. 2000. PAG. 657.
ANÁLISIS DEL CASO CLÍNICO

IDENTIFICACIÓN DEL REACTIVO


Area: PEDIATRÍA
Especialidad: URGENCIAS PEDIÁTRICAS
Tema: PATOLOGÍA NEONATAL
Subtema: TAQUIPNEA TRANSITORIA DEL RECIEN NACIDO

CASO CLÍNICO SERIADO

RECIEN NACIDO PREMATURO DE 32 SEMANAS DE GESTACION, ESTANDO HOSPITALIZADO PRESENTA APNEAS.

recién nacido prematuro de 32 semanas de


gestación.

estando hospitalizado presenta apneas.

----

----

----

145 - USTED LE EXPLICA A LOS PADRES QUE LA APNEA DEL PACIENTE ESTA RELACIONADA CON SU
INMADUREZ Y POR SU PATOGENIA DEBERÁ CONSIDERARSE COMO:

CENTRAL La APNEA CENTRAL PURA resulta de depresión del centro respiratorio y se evidencia por ausencia
completa de actividad de los músculos respiratorios (hay ausencia de flujo aéreo y cese de
movimientos respiratorios). La apnea central puede ser inducida en el recién nacido pretérmino
como respuesta fisiológica normal a hipoxia, hipercapnia o acidosis. La depresión ventilatoria, el
segundo componente de la respuesta bifásica a la hipoxia, es tanto causa como consecuencia de la
apnea en el recién nacido pretérmino. Las apneas centrales también ocurren como respuesta refleja
a la estimulación de receptores ubicados en la mucosa laríngea y a la estimulación de los receptores
de estiramiento mecánico localizados en el tejido pulmonar. DE LAS CUATRO OPCIONES DE
RESPUESTA, ESTA ES LA ÚNICA QUE SE REFIERE A UNA EXPLICACIÓN PATOGÉNICA. LA APNEA
PRIMARIA O SECUNDARIA SE TRATA DE UNA CLASIFICACIÓN ETIOLÓGICA, QUE EN ESTE CASO NO
PUEDEN CONSIDERARSE COMO RESPUESTAS CORRECTAS DADO QUE NO CONTAMOS CON
ELEMENTOS O MÁS DATOS CLÍNICOS QUE NOS PERMITIERAN CONFIRMAR O DESCARTAR
CUALQUIERA DE ESTAS DOS POSIBILIDADES.
SECUNDARIA Las APNEAS SECUNDARIAS se ven tanto en pretérminos como en términos. Las causas más
frecuentes de apnea secundaria son: • Infección: es la causa más frecuente. Podría determinar
depresión del SNC. • Reflujo gastroesofágico (RGE): muy discutido. Si bien los RN con apneas
tienen mayor frecuencia de RGE, la gran mayoría de las apneas no se produce en coincidencia con el
episodio de RGE. • Desórdenes metabólicos: sobre todo alteraciones del calcio y la glucosa. •
Distermias. NO CONTAMOS CON DATOS CLÍNICOS QUE PUDIERAN APOYAR O DESCARTAR SI SE
TRATA DE UNA APNEA SECUNDARIA.

IDIOPÁTICA APNEA OBSTRUCTIVA: En esta existen los movimientos respiratorios pero no hay flujo aéreo. En las
apneas obstructiva y mixta la ausencia de flujo de aire se da por oclusión de la faringe, el orificio
laríngeo o ambos. Normalmente la vía aérea superior permanece abierta por los músculos faríngeos
y laríngeos, incluidos el abductor de la laringe, los laríngeos aductores y el geniogloso. Sin
embargo, la presión intraluminal negativa durante la inspiración podría fácilmente colapsar la
complaciente faringe del recién nacido prematuro, resultando en obstrucción del flujo de aire.
Aunque la apnea obstructiva pura es poco común, el cierre de la vía aérea ocurre en 47% de las
apneas centrales y en 100% de ellas cuando duran más de veinte segundos. La pérdida del tono
muscular es la causa de la obstrucción de la vía aérea durante la apnea central. OJO. EN EL TEMA DE
APNEAS, IDIOPÁTICA ES UTILIZADA COMO SINÓNIMO DE PRIMARIA.

PRIMARIA APNEA PRIMARIA, idiopática o de la prematuridad, por inmadurez de los mecanismos de


regulación de la respiración. El diagnóstico de apnea primaria en el recién nacido prematuro es
clínico, se confirma al excluir que alguna patología sea la causa. La apnea en el recién nacido
prematuro se caracteriza por: No asociarse a una patología que la explique, excepto su condición
de prematuro. Aparece generalmente entre el 2°-3° día de vida Es muy raro que se presente por
primera vez después de la segunda semana de vida. En el recién nacido prematuro los eventos de
apnea ocurren durante el sueño activo, se supone que durante el cese del impulso respiratorio
central se pierde el tono muscular de la vía aérea superior. Entre los eventos de apnea el recién
nacido prematuro se puede encontrar en adecuadas condiciones generales. La detección de la
apnea se realizara durante la inspección del recién nacido o por medio del monitor de signos vitales
al observar: • Ausencia de respiración por mas de 20 segundos o Ausencia de respiración menor de
20 segundos acompañada de bradicardia o cianosis. El recién nacido con factores de riesgo para
presentar apnea deberá mantenerse con vigilancia continua, especialmente durante el sueño activo,
preferentemente con monitor de signos vitales para detectar los eventos de apnea. DE ACUERDO A
LA DEFINICIÓN CLÍNICA NO PODEMOS CONFIRMARLA, AL NO CONTAR CON DATOS CLÍNICOS
QUE NOS PERMITAN DIFERENCIAR ENTRE APNEA PRIMARIA O SECUNDARIA.

Bibliografía:
MANUAL DE CUIDADOS NEONATALES. JOHN P. CLOHERTY. MASSON. EDICION 4TA. 2005. PAG. 452.

146 - EL MANEJO DE LA APNEA DE LA PACIENTE PREMATURA ES CON:

CAFEÍNA Por casi treinta años, las metilxantinas (teofilina, cafeína, aminofilina), estimulantes potentes del
SNC, han sido las piedras angulares de la terapia para los pacientes con apnea de la pre-maturez.
Estudios recientes demuestran que tanto la teofilina como la cafeína son efectivas en reducir las
apneas 2-7 días después de iniciado el tratamiento con ellas. ACTUALMENTE, LA METILXANTINA
PREFERIDA ES LA CAFEÍNA, POR SU MENOR TOXICIDAD. LA CAFEÍNA TIENE ELIMINACIÓN
MUCHO MÁS LENTA, SU VIDA MEDIA ES DE CIEN HORAS COMPARADA CON TREINTA DE LA
TEOFI¬LINA. EN LOS NEONATOS UNA CANTIDAD SIGNIFICATIVA DE TEOFILINA ES METILADA A
CAFEÍNA. LAS METILXANTINAS REDUCEN LAS APNEAS POR MÚLTIPLES MECANISMOS. LA
CAFEÍNA Y LA TEOFILINA AUMENTAN LA VENTILACIÓN MINUTO, LA SENSIBILIDAD AL CO2 Y
ESTIMULAN EL CENTRO RESPIRATORIO. Ellas también estimulan los músculos esqueléticos,
mejorando la contracción diafragmática y, en general, de todos los músculos respiratorios. A
pesar de ser utilizadas desde hace mucho tiempo, muy poco se conoce de su seguridad a corto y
largo plazo. Las metilxantinas se unen a los receptores de adenosina y bloquean la acción de este
neuro¬transmisor en el cerebro. La función del sistema de receptores de adenosina en el cerebro
inmaduro no está totalmente establecida, pero se cree que la adenosina es neuroprotectora
durante la isquemia. Si esto es así, el uso de metilxantinas podrían empeorar el daño tisular
hipóxico en los niños en riesgo de hipoxemia recurrente. Las metilxantinas tienen efecto sobre la
alimentación, el crecimiento, y la conducta de los recién nacidos tratados con ellas.
ESTEROIDES El SÍNDROME DE ASPIRACIÓN DE MECONIO puede causar dificultad respiratoria grave en el
recién nacido, lo que se asocia a una alta morbilidad y mortalidad. Se supone que ocurre una
neumonitis química causada por la bilis, los ácidos biliares y las secreciones pancreáticas que
contiene el meconio. Por lo tanto, se ha planteado la hipótesis de que los corticosteroides pueden
ser beneficiosos para el tratamiento de esta enfermedad por efecto de sus propiedades
antiinflamatorias. Así mismo estos fármacos son utilizados en la maduración pulmonar en los
recién nacidos prematuros.

FENOBARBITAL El FENOBARBITAL es la droga de elección para el control de las CONVULSIONES EN EL PERIODO


NEONATAL. Se administra una dosis inicial de 20 mg/Kg, con dosis de mantenimiento de 3 a 5
mg/Kg dividido en dos dosis cada 12 horas parenteral o enteral. Idealmente debe monitorizarse
niveles séricos manteniéndolos alrededor de 20 mgm %. Es importante anotar que el diluente del
fenobarbital es muy hiperosmolar, razón por la cual las dosis calculadas deben ser diluidas y
administradas mínimo durante 10 minutos.

SURFACTANTE La administración de SURFACTANTE exógeno por vía intratraqueal, es una medida eficaz que ha
conseguido reducir la mortalidad y complicaciones del RN con ENFERMEDAD DE MEMBRANA
HIALINA.

Bibliografía:
MANUAL DE CUIDADOS NEONATALES. JOHN P. CLOHERTY. MASSON. EDICIÓN 4TA. 2005. PAG. 454.
DIAGNÓSTICO Y TRATAMIENTO DE APNEA DEL PREMATURO. CATÁLOGO MAESTRO DE GUÍAS DE
PRÁCTICA CLÍNICA: IMSS-724-14

http://www.cenetec.salud.gob.mx/descargas/gpc/CatalogoMaestro/IMSS-724-14-Apneadelprematuro/724GRR.pdf

FIN DEL CASO CLÍNICO SERIADO


ANÁLISIS DEL CASO CLÍNICO

IDENTIFICACIÓN DEL REACTIVO


Area: MEDICINA INTERNA
Especialidad: INFECTOLOGÍA
Tema: NEUROINFECCIONES
Subtema: MENINGITIS

CASO CLÍNICO CON UNA PREGUNTA

HOMBRE DE 50 AÑOS DE EDAD, SIN ANTECEDENTES DE IMPORTANCIA, QUE PRESENTA CUADRO CLÍNICO COMPATIBLE CON MENINGITIS, A QUIEN SE LE REALIZA
PUNCIÓN LUMBAR, LA CUAL REPORTA CITOQUÍMICO Y CITOLÓGICO COMPATIBLE CON INFECCIÓN BACTERIANA.

Hombre de 50 años de edad.

-.

Cuadro compatible con meningitis.

-.

Citológico y citoquÍmico compatible con


infección bacteriana.

147 - EL TRATAMIENTO EMPÍRICO DE ELECCIÓN PARA ESTE PACIENTE CONSISTE EN ADMINISTRARLE:

CEFTRIAXONA Y La MENINGITIS BACTERIANA AGUDA, adquirida en la comunidad en el adulto inmunocompetente, es producto de la presencia de bacterias en el espacio
VANCOMICINA subaracnoideo, con una consecuente reacción inflamatoria que comprende a su vez al parénquima cerebral y las meninges. Esta enfermedad es una verdadera urgencia
médica lo que conlleva a que el objetivo sea comenzar el tratamiento antibiótico en los primeros 60 minutos de la llegada del paciente al servicio de urgencias. En
aquellos pacientes en que se sospeche una meningitis bacteriana, debe comenzarse un tratamiento antimicrobiano empírico antes de conocer los resultados de la
tinción de Gram. y de los cultivos del LCR. S. pneumoniae y N. meningitidis que son los microorganismos que con mayor frecuencia producen las meningitis
extrahospitalarias. El tratamiento antimicrobiano empírico del adulto inmunocompetente con menigitis bacteriana adquirida en la comunidad debe ser a base de una
cefalosporina de tercera generación y como alternativa meropenem o cloranfenicol. La ceftriaxona o la cefotaxima proporcionan buena cobertura contra S.
pneumoniae, estreptococos del grupo B y H. influenzae sensibles, y una razonable cobertura contra N. meningitidis. Debido a la aparición de cepas de S. pneumoniae
resistentes a penicilina y a cefalosporinas, el tratamiento empírico debera incluir una cefalosporina de tercera generación (p. Ej., ceftriaxona o cefotaxima) más
vancomicina. 1. Ceftriaxona 2g cada 12 a 24hrs o cefotaxima 2g cada 6 u 8hrs. - Alternativa: meropenem 2gr cada 8hrs, o cloranfenicol 1gr cada 6hrs. 2. Vancomicina
(se debe sumar en caso de sospecha de resistencia a penicilina ocefalosporinas) 60mg/kg/24hrs, dosis carga y continuar con 15mg/kg/día. LA COMBINACIÓN DE
CETRIAXONA Y VANCOMICINA SON IDEALES PARA EL MANEJO DE LA MENINGITIS BACTERIANA, SOBRE TODO PENSANDO EN LA ALTA RESISTENCIA QUE EXISTE DEL
NEUMOCOCO A LA PENICILINA Y CEFALOSPORINAS.

CEFUROXIMA Y No se recomienda la combinación de dos medicamentos que tienen mecanismos de acción similares. La idea de realizar dichas combinaciones es potenciar la acción
TAZOBACTAM bactericida o la combinación de un bactericida con un bacteriostático para tener mayor potencia contra los gérmenes. RECUERDA ELEGIR SIEMPRE UNA
CEFALOSPORINA COMO BASE DEL MANEJO, EL USO DE TAZOBACTAM NO ESTÁ JUSTIFICADO.

AMPICILINA Y En el caso que se sospeche infección por Listeria monocytogenes está indicada esta combinación. Sin embargo, siempre que tienes un paciente con cierta patología
GENTAMICINA tienes que pensar primero en los gérmenes más comunes que puedan causar la enfermedad y dar ese tratamiento, de lo contrario no se cubrirá el espectro antibiótico
para la mayoría de los pacientes generando mayores complicaciones. EXISTE UNA ALTA RESISTENCIA A PENICILINAS, POR LO TANTO NO ESTÁN INDICADAS PARA EL
MANEJO DE LA MENINGITIS.

CLORANFENICOL No es un esquema indicado para el tratamiento de meningitis. EL CLORANFENICOL ESTÁ INDICADO COMO ALTERNATIVA A LAS CEFALOSPORINAS.
Y
DICLOXACILINA

Bibliografía:
1. GUÍA DE PRÁCTICA CLÍNICA. DIAGNÓSTICO, TRATAMIENTO Y PREVENCIÓN DE LA MENINGITIS AGUDA BACTERIANA ADQUIRIDA EN LA COMUNIDAD EN PACIENTES
ADULTOS INMUNOCOMPETENTES, MÉXICO: SECRETARÍA DE SALUD; 2010.
HTTP://WWW.CENETEC.SALUD.GOB.MX/DESCARGAS/GPC/CATALOGOMAESTRO/310_GPC_DIAGNOSTICO_TRATAMIENTO_MENINGITIS_AGUDA_BACTERIANA/RER_DIAGNOSTICO_Y_TRATA
2. LONGO DL, FAUCI AS, KASPER DL, HAUSERSL, JAMESON JL, LOSCALZOJ. HARRISON. PRINCIPIOS DE MEDICINA INTERNA, 18A EDICIÓN. MC GRAW HILL. NEW YORK, USA.
2012, PP 3415.
ANÁLISIS DEL CASO CLÍNICO

IDENTIFICACIÓN DEL REACTIVO


Area: GINECOLOGÍA Y OBSTETRICIA
Especialidad: GINECOLOGÍA
Tema: AFECCIONES BENIGNAS Y MALIGNAS DEL UTERO
Subtema: HIPERPLASIA ENDOMETRIAL Y CÁNCER DE ENDOMETRIO

CASO CLÍNICO SERIADO

MUJER DE 69 AÑOS DE EDAD, GESTA 3 PARA 3, FUR HACE 20 AÑOS. ACUDE A CONSULTA POR PRESENTAR,
DESDE HACE 8 MESES SANGRADO TRANSVAGINAL EN MODERADA CANTIDAD. PAPANICOLAOU HACE 6
MESES NORMAL, A LA EXPLORACIÓN SE ENCUENTRA VAGINA ATRÓFICA, SANGRADO TRANSCERVICAL
ESCASO CON CÉRVIX DE 3 CM, CERRADO, FORMADO Y CON ÚTERO DE 6X4 CM.

Edad mayor de 55 años de edad, recuerda


que el 80% de los tumores endometriales
se presentan a esta edad.

fur (fecha de ultima regla) de 20 años.

sangrado transvaginal de 8 meses de


evolución.

La exploración esperada sería, vagina


estrecha, se reduce en tamaño,
desaparecen los fondos de saco cérvico-
vaginales, a menudo se forma una
estrechez anular en el tercio superior que
di culta la observación del cuello uterino.
Se atro a el epitelio disminuyendo el
espesor, la mucosa se muestra seca y
brillante El útero involuciona en conjunto,
pero es más evidente en el cuerpo que en
el cuello uterino. Esta atro a se evidencia
en la disminución global de sus diámetros.

-.

148 - PARA CONFIRMAR SU DIAGNÓSTICO, USTED DEBERÁ REALIZAR:


CITOLOGÍA OJO: DEBES PONER ESPECIAL ATENCIÓN A LA PREGUNTA QUE SOLICITA "CONFIRMACIÓN
EXFOLIATICA DIAGNÓSTICA". De inicio tienes que dirigir la posibilidad diagnóstica a CÁNCER
CERVICAL. ENDOMETRIAL por la edad de aparición y los datos clínicos. EL FROTIS DE PAPANICOLAOU
NO HA SIDO UNA HERRAMIENTA SENSIBLE PARA DIAGNOSTICAR CÁNCER ENDOMETRIAL,
50% de las mujeres, con esta neoplasia tiene resultados normales. En mujeres de 40 años de
edad o más, en ocasiones se registran células endometriales benignas en un frotis
sistemático. En premenopáusicas, a menudo este es un hallazgo de poca importancia, sobre
todo si el frotis se realiza después de la menstruación. Sin embargo, las posmenopáusicas
con tales datos tienen un riesgo de 3 a 5% de cáncer endometrial. El hallazgo de células
endometriales debe hacernos sospechar la posibilidad de cáncer de endometrio, sin
embargo cuando se sospecha esta posibilidad, el estudio indicado es la toma de biopsia, ya
sea por aspiración o mediante legrado uterino. NO ES DIAGNÓSTICO DE CÁNCER
ENDOMETRIAL SIN EMBARGO DEBES CONSIDERAR ÉSTA POSIBILIDAD SIEMPRE QUE EN UN
ESTUDIO DE CITOLOGÍA SE REPORTEN CÉLULAS ENDOMETRIALES.

ULTRASONOGRAFÍA Sería de utilidad para descartar o apoyar el diagnóstico de HIPERPLASIA ENDOMETRIAL,


PÉLVICA. siempre y cuando fuera transvaginal, esperando encontrar mediciones endometriales de
5mm o menos. Aquellas pacientes con endometrio mayor a este tamaño requieren de
biopsia para confirmar el diagnóstico. ESTE MÉTODO NOS OFRECE DATOS A CERCA DEL
GROSOR ENDOMETRIAL PERO NO ESTABLECE EL DIAGNOSTICO ETIOLÓGICO DE ÉSTE
HALLAZGO.

NIVELES DE CA-125. Los niveles de CA-125 son útiles para evaluar la RESPUESTA AL TRATAMIENTO en pacientes
con estadios avanzados de CÁNCER DE ENDOMETRIO. "En ningún caso debe considerarse
diagnóstico". Recuerda que concentraciones elevadas en el suero de CA-125 las podemos
encontrar en el 80% de las mujeres con carcinoma de ovario, en el 26% de las mujeres con
tumores ováricos benignos y en el 66% de las pacientes con condiciones no neoplásicas
incluyendo estados como el primer trimestre del embarazo, menstruación, endometriosis,
adenomiosis, fibrosis uterina, salpingitis aguda, enfermedades hepáticas como la cirrosis y
en inflamaciones del peritoneo, pericardio y pleura. ES UN MARCADOR TUMORAL
PRESENTE EN MUCHOS TIPOS DE CÁNCER LO QUE LO CONVIERTE EN POCO ESPECÍFICO.
DILATACIÓN CON Una BIOPSIA ENDOMETRIAL es la prueba que se realiza con más frecuencia para el cáncer
LEGRADO UTERINO. endometrial, y es muy precisa en mujeres posmenopáusicas. Puede llevarse a cabo en el
consultorio médico. En este procedimiento, se inserta un tubo flexible muy delgado y
flexible en el útero por el cuello uterino. Luego, se extrae por succión una pequeña cantidad
de endometrio a través del tubo, procedimiento que toma alrededor de un minuto o menos.
Si la muestra de la biopsia endometrial no proporciona suficiente tejido, o si la biopsia
sugiere cáncer, pero los resultados son inciertos, deberá efectuarse una dilatación y un
legrado. En este procedimiento ambulatorio, el cuello uterino se agranda (dilatado) y se
utiliza un instrumento especial para extraer tejido del interior del útero mediante raspado.
Este procedimiento demora aproximadamente una hora, y puede requerir anestesia general
o sedación consciente. Generalmente se usa una dilatación y un legrado en un área
quirúrgica ambulatoria. La mayoría de las mujeres experimentan algo de molestia después
de este procedimiento. EL DIAGNÓSTICO HISTOPATOLÓGICO POR BIOPSIA ENDOMETRIAL
ES EL ESTÁNDAR DE ORO PARA EL DIAGNÓSTICO DE CA ENDOMETRIAL. REPASO CÁNCER
DE ENDOMETRIO. El CÁNCER DE ENDOMETRIO es la neoplasia glandular maligna que se
origina en la capa interna (endometrio) del cuerpo uterino. La mayoría de estos cánceres son
adenocarcinomas que derivan de células que producen y liberan moco u otros líquidos. Se
caracteriza por producir síntomas aún en sus estadios más precoces; el síntoma más común
es el sangrado uterino anormal. Debe sospecharse en mujeres con las siguientes
características: - Sangrado uterino anormal en pacientes con factores de riesgo. - Sangrado
uterino anormal con antecedente de terapia estrogénica sin oposición, hormonoterapia con
tamoxifeno. - Cualquier sangrado uterino en la postmenopausia. Las mujeres con sangrado
profuso e irregular deberán ser sometidas a ultrasonido transvaginal para la búsqueda
intencionada de cáncer endometrial siempre que tengan: - Peso mayor a 90kg en mayores
de 45 años. - Antecedente de hiperplasia “atípica” o carcinoma endometrial. La patogenia de
este tipo de cáncer sigue dos tipologías: 1. Adenocarcinoma de tipo I: corresponde al 75% de
los cánceres de endometrio, depende de estrógenos, son de bajo grado y tienen como
precursor a la hiperplasia endometrial atípica. 2. Cáncer endometrial tipo II: tienen rasgos
histológicos serosos o de células claras, no se asocia con lesiones precursoras, tienen una
evolución clínica más agresiva. Cerca del comienzo de la menopausia, se produce una
disminución de estrógeno circulante, la cual origina síntomas inaceptables, que afectan la
salud y el bienestar de las mujeres. La terapia de reemplazo hormonal (con estrógeno sin
oposición o terapias combinadas de estrógeno y progestágeno) constituye un método
efectivo para el tratamiento de estos síntomas. EL ESTRÓGENO SIN OPOSICIÓN,
ADMINISTRADO A MUJERES CON ÚTERO, PUEDE INDUCIR LA ESTIMULACIÓN
ENDOMETRIAL Y AUMENTAR EL RIESGO DE HIPERPLASIA ENDOMETRIAL Y CARCINOMA.
Este riesgo puede reducirse al agregar progestágeno, sin embargo, es posible que este
agente, a su vez, ocasione ciertos síntomas como por ejemplo, sangrado vaginal y
oligometrorragia, que pueden comprometer el cumplimiento con el tratamiento. El
diagnóstico definitivo de cáncer endometrial se realiza a través de estudio histopatológico
de la biopsia endometrial, la cual se puede obtener por diversos métodos. - Biopsia
endometrial ambulatoria (cánula de Pipelle) considerada el primer procedimiento
diagnóstico. - LEGRADO UTERINO FRACCIONADO (endometrial y endocervical). -
Histeroscopía, la cual debe ser realizada por un especialista. La ecografía endovaginal es el
estudio ideal para iniciar la exploración complementaria en caso de hemorragia uterina
anormal (pre o post menopáusica), en él se puede observar engrosamiento significativo del
endometrio, y observar las características del contenido de la cavidad, en caso de haberlo.
Otros estudios como tomografía axial, resonancia magnética, urografía excretora,
cistoscopía, rectosigmoidoscopía, se reservan para pacientes con enfermedad extrauterina
con histología desfavorable o grado histológico 3. El marcador tumoral Ca125 se solicita
sólo en casos de sospecha de cáncer con extensión extra uterina, y se utiliza para la
monitorización en la evolución del paciente.

Bibliografía:
1. GUÍA DE REFERENCIA RÁPIDA, DIAGNÓSTICO Y TRATAMIENTO DEL CÁNCER DE ENDOMETRIO. MÉXICO:
SECRETARIA DE SALUD; 2010. 2. GUÍA DE PRÁCTICA CLÍNICA, DIAGNÓSTICO Y TRATAMIENTO DEL CÁNCER
DE ENDOMETRIO. MÉXICO: SECRETARIA DE SALUD; 2010.

http://www.cenetec.salud.gob.mx/descargas/gpc/CatalogoMaestro/478_GPC_CxncerEndometrio/GER_Cxncer_de_Endometrio.pdf

149 - EN CASO DE CONFIRMAR SU DIAGNÓSTICO, EL TRATAMIENTO SIEMPRE DEBERÁ INCLUIR:


HISTERECTOMIA Dependiendo de la estadificación, en prácticamente todos los estadios se debe realizar la
CON histerectomía, pudiéndose llevar acabo extrafascial o radical, dependiendo del
OOFORECTOMÍA. involucramiento de las estructuras adyacentes. Incluso la estadificación requiere del
procedimiento quirúrgico para poderse hacer. El tratamiento principal para el cáncer
endometrial es una operación (histerectomía) para extirpar el útero y el cuello uterino. La
extirpación de los ovarios y las trompas de Falopio en realidad no es parte de una
histerectomía. Este procedimiento a menudo se hace junto con una histerectomía en la
misma operación. Para el cáncer endometrial, la extirpación del útero, sin los ovarios o las
trompas de Falopio, rara vez es recomendada. Para determinar la etapa del cáncer, también
se necesitará la extirpación de los ganglios linfáticos en la pelvis y alrededor de la aorta. Una
histerectomía radical se hace cuando el cáncer endometrial se ha propagado al cuello
uterino o al área que circula al cuello uterino (El parametrio). En esta operación, se extirpa
todo el útero, los tejidos contiguos al útero (El parametrio y los ligamentos uterosacrales),
así como la parte superior de la vagina (Adyacente al cuello uterino). Para el cáncer
endometrial, se hace una salpingo-ooforectomía bilateral al mismo tiempo. TODOS LOS
CASOS DE CÁNCER ENDOMETRIAL INCLUYEN LAVADO PERITONEAL, HISTERECTOMÍA
EXTRAFACIAL, SALPINGOOFERECTOMÍA BILATERAL, LINFADENECTOMÍA PÉLVICA
BILATERAL Y PARA AÓRTICA. RECUERDA QUE DADO QUE ES UN CÁNCER
HORMONODEPENDIENTE DEBEN EXTIRPARSE SIEMPRE LOS OVARIOS.

QUIMIOTERAPIA. La QUIIMIOTERAPIA con paclitaxel, doxorrubicina y cisplatino es la opción ADYUVANTE DE


ELECCIÓN para el cáncer endometrial avanzado. Como puedes ver adyuvante al
procedimiento quirúrgico.

RADIOTERAPIA. Por lo general, la RADIOTERAPIA primaria se considera sólo en casos raros, cuando una
paciente es candidata excepcionalmente pobre para intervención quirúrgica. La radioterapia
consiste del uso de radiación de alta energía (Por ejemplo, Rayos X) para destruir las células
cancerosas. La radioterapia puede administrarse colocando materiales radiactivos dentro del
cuerpo en el lugar cercano al tumor. A esto se le llama radioterapia interna o braquiterapia.
Otra opción consiste en administrar radiación desde una máquina fuera del cuerpo en un
procedimiento parecido a tener una radiografía. A esto se le llama radioterapia de rayos
externos. En algunos casos, se administran en secuencia tanto la braquiterapia como la
radioterapia de rayos externos. La etapa y el grado del cáncer ayudan a determinar qué
áreas necesitan ser expuestas a la radioterapia y qué métodos se deben emplear. LA
RADIOTERAPIA NO SE UTILIZA MUCHO EN CÁNCER ENDOMETRIAL.

HORMONOTERAPIA. Una de las características únicas del cáncer endometrial, es su capacidad de respuesta
hormonal. En casos raros, se utiliza progestina para el tratamiento primario de mujeres con
riesgo quirúrgico excesivo. Esta podría ser la única opción paliativa disponible en unas
cuantas circunstancias excepcionales. SÓLO EN ALGUNOS CASOS DONDE EL RIESGO
QUIRÚRGICO ES MUY ALTO, DEBE SER A BASE DE PROGESTINAS.

Bibliografía:
1. GUÍA DE REFERENCIA RÁPIDA, DIAGNÓSTICO Y TRATAMIENTO DEL CÁNCER DE ENDOMETRIO. MÉXICO:
SECRETARIA DE SALUD; 2010. 2. GUÍA DE PRÁCTICA CLÍNICA, DIAGNÓSTICO Y TRATAMIENTO DEL CÁNCER
DE ENDOMETRIO. MÉXICO: SECRETARIA DE SALUD; 2010. 3. SCHONGUE J, SCHAFER J, HALVORSON L,
HOFFMAN B, BRADSHAW K, CUNNINGHAM G. WILLIAMS GINECOLOGÍA, DE LA 1A EDICIÓN EN INGLÉS. MC
GRAW HILL. USA. 2009, PP 699-701.

http://www.cenetec.salud.gob.mx/descargas/gpc/CatalogoMaestro/478_GPC_CxncerEndometrio/GER_Cxncer_de_Endometrio.pdf

FIN DEL CASO CLÍNICO SERIADO


ANÁLISIS DEL CASO CLÍNICO

IDENTIFICACIÓN DEL REACTIVO


Area: CIRUGÍA
Especialidad: TRAUMATOLOGÍA Y ORTOPEDIA
Tema: CERVICALGIAS, LUMBALGIAS, TRAUMA MIEMBROS SUP Y TUMORES
Subtema: LESIONES DE HOMBRO

CASO CLÍNICO CON UNA PREGUNTA

MASCULINO DE 75 AÑOS DE EDAD, QUE SUFRE CAÍDA APOYÁNDOSE CON EL BRAZO DERECHO EXTENDIDO,
PRESENTANDO DOLOR Y LIMITACIÓN A LA FUNCIÓN DE ESA EXTREMIDAD. POR LO QUE ACUDE A LA
CONSULTA Y USTED LE DIAGNOSTICA FRACTURA DEL CUELLO DEL HUMERO.

antecedente traumático.

dolor y limitación funcional de la


extreMidad afectada.

Los pacientes con cualquier tipo de


fractura presentarán dolor a la palpación
en la zona en la que se encuentra la
fractura.

Las radiografías necesarias para


diagnosticar dicha fractura serán ap y lat
del hombro.

150 - LA INMOVILIZACIÓN SIMPLE EN ESTE PACIENTE ESTARÍA INDICADA SI PRESENTA:


ÍNDICE DE KATZ El índice de Katz es un instrumento que evalúa aspectos de las actividades de la vida diaria de
DISFUNCIONAL. los pacientes que han perdido su autonomía y el proceso de recuperación, surgió para definir
el concepto de dependencia en sujetos con fractura de cadera. Este índice valora la capacidad
para realizar el cuidado personal valorando independencia o dependencia en bañarse,
vestirse, usar el retrete, trasladarse, mantener la continencia y alimentarse. Se correlaciona
con el grado de movilidad y confinamiento en casa tras el alta hospitalaria, probabilidad de
muerte, hospitalización e institucionalización. Fue construida para uso específico en población
mayor de 65 años. Es de fácil aplicación (habitualmente menos de 5 minutos). Tiene valor
predictivo sobre la estancia media hospitalaria, la institucionalización y la mortalidad a corto y
largo plazo. EL ÍNDICE DE KATZ NO TIENE NADA QUE VER CON LA INDICACIÓN DE
INMOVILIZAR O NO EN ESTE CASO, PUES SÓLO EVALÚA EL GRADO DE DEPENDENCIA PARA
REALIZAR ACTIVIDADES PERSONALES EN PACIENTES QUE HAN TENIDO FRACTURAS
PREVIAS.

DESPLAZAMIENTO En el caso de las fracturas del cuello humeral, cabe mencionar que hay dos zonas llamadas
MÍNIMO. cuello humeral, uno es el cuello anatómico y el otro se llama cuello quirúrgico. El primero se
encuentra proximal a la superficie articular de la cabeza humeral y el segundo está localizado
en la metáfisis proximal del húmero. En cualquier caso si una fractura del humero presenta un
"desplazamiento" deberá tomarse en consideración la inmovilización externa e incluso
interna. Entiéndase por externa desde el vendaje, cabestrillo, inmovilizador de hombro y por
interna a la cirugía, en la cual se realizara una reducción abierta y fijación de la fractura con
material de osteosíntesis, por lo tanto, la mejor respuesta deberá ser el desplazamiento de la
fractura como motivo para inmovilizar LAS FRACTURAS CON DESPLAZAMIENTO MÍNIMO
SUELEN INMOVILIZARSE CON CABESTRILLO PARA ELIMINAR LA MOVILIZACIÓN DEL
HOMBRO, EN EL CASO DE AQUELLAS DESPLAZADAS REQUIEREN MANEJO QUIRÚRGICO CON
FIJACIÓN INTERNA. IMPORTANTE: si bien las fracturas no desplazadas también son
candidatas a este tipo de inmovilización deberás elegir la respuesta más correcta de las 4 y
esta es la única que coincide para el caso clínico.

LUXACIÓN DEL En este caso no estaría indicada la inmovilización si no primeramente la reducción de la


HOMBRO. luxación por lo que no es esta la mejor opción de respuesta. IMPORTANTE: la luxación de
hombro puede acompañar a todos los tipos de fracturas (no desplazadas, con desplazamiento
mínimo, con compromiso vascular...) por tanto, no puede ser considerado un criterio para la
inmovilización en este caso.

LESIÓN VASCULO- Las lesiones vasculo nerviosas se presentan en el caso de las fracturas del humero, cuando se
NERVIOSA. afecta el cuello anatómico. Deberás recordar que la circulación de la cabeza humeral depende
de la arteria circunfleja que corre a través de esta zona del humero, provocando
posteriormente una necrosis avascular de la cabeza humeral. En este caso estará indicado el
TRATAMIENTO QUIRÚRGICO DE LA FRACTURA. Hay que tomar en cuenta que primero debe
evaluarse el desplazamiento de la fractura y posteriormente se sospechara la lesión
vasculonerviosa, entonces esta sería la segunda mejor opción de respuesta.

Bibliografía:
1. BRUNICARDI F, ANDERSEN D, BILLIAR T, Y COLS. SCHWARTZ PRINCIPIOS DE CIRUGÍA, 9A EDICIÓN. MC
GRAW HILL. 2011, PP 1574. 2. DOHERTY G. DIAGNÓSTICO Y TRATAMIENTO QUIRÚRGICO, 13A EDICIÓN. MC
GRAW HILL LANGE. 2011, PP 883.
ANÁLISIS DEL CASO CLÍNICO

IDENTIFICACIÓN DEL REACTIVO


Area: MEDICINA INTERNA
Especialidad: CARDIOVASCULAR
Tema: VASCULOPATÍA HIPERTENSIVA
Subtema: COMPLICACIONES DEL PACIENTE CON VASCULOPATÍA
HIPERTENSIVA

CASO CLÍNICO CON UNA PREGUNTA

FEMENINA DE 50 AÑOS CON DIAGNÓSTICO DE HIPERTENSIÓN ARTERIAL DE 10 AÑOS DE EVOLUCIÓN CON


MAL APEGO A TRATAMIENTO. SE SOSPECHA DE ARTERIOPATÍA DE LAS EXTREMIDADES PÉLVICAS.

MUJER DE 53 AÑOS.

HIPERTENSIÓN ARTERIAL DE 10 AÑOS DE


EVOLUCIÓN.

-.

-.

-.

151 - CORRESPONDE AL SÍNTOMA MÁS CLÁSICO DE ESTA PATOLOGÍA:

ESPASMOS ARTERIOPATIA DE LAS EXTREMIDADES INFERIORES (PAD) Los vasos sanguíneos, además de
MUSCULARES. contribuir a la patogenia de la hipertensión, pueden recibir los efectos de la enfermedad
ateroesclerótica que es consecuencia de la hipertensión de larga evolución.

CLAUDICACIÓN Los individuos con lesiones estenóticas de las extremidades inferiores posiblemente no tengan
INTERMITENTE. síntomas, pero UNO DE LOS SÍNTOMAS CLÁSICOS DE LA ARTERIOPATIA DE LAS EXTREMIDADES
INFERIORES ES LA "CLAUDICACIÓN INTERMITENTE"; se caracteriza por dolor molesto en las
pantorrillas o en la región de las nalgas durante la marcha, que cede con el reposo.

LUMBALGIA Los sujetos hipertensos con arteriopatía de las extremidades pélvicas están expuestos a un mayor
CON riesgo de presentar en lo futuro enfermedades cardiovasculares.
INFLAMACIÓN
DEL NERVIO
CIÁTICO.
PARESTESIAS El índice tarsal/braquial es un recurso útil para valorar la presión de la enfermedad arterial y se
DE LOS define por la proporción entre la presión sistólica medida en el tarso y en el brazo, por métodos
MIEMBROS no penetrantes. El índice menor a 09 se considera como confirmatorio del diagnóstico de PAD y
INFERIORES. depende de la estenosis mayor a 50% en uno de los principales vasos de la extremidad pélvica,
cuando menos. Algunos estudios sugieren que el índice tarsal/braquial más del 0.8 proviene d un
incremento en la presión arterial, en particular sistólica.

Bibliografía:
1. LONGO DL, FAUCI AS, KASPER DL, HAUSERSL, JAMESON JL, LOSCALZOJ. HARRISON. PRINCIPIOS DE
MEDICINA INTERNA, 18A EDICIÓN. MC GRAW HILL. NEW YORK, USA. 2012, PP 2046-2047.
ANÁLISIS DEL CASO CLÍNICO

IDENTIFICACIÓN DEL REACTIVO


Area: MEDICINA INTERNA
Especialidad: REUMATOLOGÍA
Tema: TRANSTORNOS MEDIADOS POR MECANISMOS INMUNITARIOS
Subtema: ARTRITIS REUMATOIDE

CASO CLÍNICO CON UNA PREGUNTA

HOMBRE DE 52 AÑOS CON DIAGNÓSTICO RECIENTE DE ARTRITIS REUMATOIDE.

hombre de 52 años de edad.

artritis reumatoide.

-.

-.

-.

152 - DECIDE COMENZAR MANEJO CON EL MEDICAMENTO CUYO MECANISMO DE ACCIÓN CONSISTE EN
INHIBIR LA VÍA DE LOS FOLATOS:

SALES DE ORO. EL TRATAMIENTO CON SALES DE ORO SE PUEDE INDICAR EN PACIENTES CON AR, SIN
EMBARGO NO ES EL TRATAMIENTO DE PRIMERA ELECCIÓN PARA ESTA ENFERMEDAD Y NO
CORRESPONDE AL MECANISMO DE ACCIÓN. • Mecanismo de acción: Parece que inhiben las
actividades inflamatorias de linfocitos, monocitos, y, en menor grado, de neutrófilos.
Disminuyen la producción de interleuquina-1 (IL-1), interleuquina-6 (IL-6) y factor de necrosis
tumoral alfa (TNF-a), así como el número de macrófagos sinoviales.

TAMOXIFENO. El tamoxifeno es un modulador de los receptores de estrógenos. Estos medicamentos producen


efectos benéficos en algunos tejidos como el hueso, cerebro e hígado durante la
postmenopausia pero ejercen efectos antagónicos en la mama y endometrio.

METROTEXATO. EL METOTREXATE ES UN ANTIMETABOLITO QUE ES UN "ANÁLOGO DEL ÁCIDO FÓLICO" QUE


INHIBE COMPETITIVAMENTE A LA DIHIDRO REDUCTASA. El inicio del tratamiento de un
paciente con artritis reumatoide consiste en AINES, junto con dosis bajas de esteroide en lo que
el fármaco modificador de la enfermedad alcanza su efecto máximo y un FAME. El metotrexate
es el medicamento que se utiliza con mayor frecuencia en pacientes con AR.
COLCHICINA. La COLCHICINA tiene efectos antimitóticos inhibiendo la división celular en la fase G1 al
interferir con los microtúbulos y la formación del huso, también puede afectar la motilidad de
los neutrófilos y su adhesión al endotelio vascular al modular la expresión de moléculas de
adhesión endotelial.

Bibliografía:
1. LONGO DL, FAUCI AS, KASPER DL, HAUSERSL, JAMESON JL, LOSCALZOJ. HARRISON. PRINCIPIOS DE
MEDICINA INTERNA, 18A EDICIÓN. MC GRAW HILL. NEW YORK, USA. 2012, PP 455, 870. 2. GUÍA DE
PRÁCTICA CLÍNICA, DIAGNÓSTICO, TRATAMIENTO DE LA ARTRITIS REUMATOIDE EN EL ADULTO. MÉXICO:
SECRETARIA DE SALUD, 2010.

http://www.cenetec.salud.gob.mx/descargas/gpc/CatalogoMaestro/195_ARTRITIS_REUMATOIDE/Artritis_reumatoidE_EVR_CENETEC.pdf
ANÁLISIS DEL CASO CLÍNICO

IDENTIFICACIÓN DEL REACTIVO


Area: MEDICINA INTERNA
Especialidad: GASTROENTEROLOGÍA
Tema: PATOLOGÍA HEPÁTICA Y PANCREATICA
Subtema: PATOLOGIA DE PANCREAS

CASO CLÍNICO SERIADO

MASCULINO DE 43 AÑOS ALCOHÓLICO, QUE ACUDE A SU CONSULTORIO POR PRESENTAR DOLOR ABDOMINAL TRANSFICTIVO EN LA
MITAD SUPERIOR DEL ABDOMEN, DE INTENSIDAD SEVERA, ACOMPAÑADO DE NAUSEAS QUE PRECEDE A VÓMITOS EN NÚMERO DE 10 DE
CONTENIDO GASTROBILIAR EN LAS ÚLTIMAS 12 HRS. LOS LABORATORIOS REPORTAN BILIRRUBINAS TOTALES DE 4MG/DL, FOSFATASA
ALCALINA DE 359 UI/DL Y ELEVACIÓN LIGERA DE ENZIMAS HEPÁTICAS.

masculino de 43 años de edad.

alcohólico.

dolor abdominal trans ctivo en la mitad


superior del abdomen, severo, con nauseas
y vómito gastrobiliar desde hace 12 horas.

-.

bilirrubinas totales 4 mg/dl, fosfatasa


alcalina 359 UI/DL,y ligera elevaciÓn de
enzimas hepÁticas.

153 - ES EL DIAGNÓSTICO CLÍNICO MAS PROBABLE:

HEPATÍTIS La HEPATITIS ALCOHÓLICA se asocia con un amplio espectro de manifestaciones clínicas. Se piensa que la producción de citocinas es
ALCOHÓLICA. el elemento que rige las manifestaciones generalizadas de la hepatitis por alcohol. El extremo del espectro está representado por
fiebre, arañas vasculares, ictericia y dolor abdominal que simula abdomen agudo, aunque muchos pacientes se encuentran
completamente asintomáticos. Identificar las características clínicas de la hepatitis alcohólica resulta esencial para iniciar una
estrategia diagnóstica y terapéutica adecuada. EXISTEN DOS TEORÍAS POR LAS QUE LA INGESTA CRÓNICA DE ALCOHOL PUEDE
PRECIPITAR PANCREATITIS. 1. La primera es que el alcohol modula la función exocrina para aumentar la litogenicidad del líquido
pancreático lo que conlleva a la formación de tapones de proteínas y litos. El contacto crónico con estos litos en los conductos
pancreáticos produce ulceración y cicatrización de los mismos que resulta finalmente en obstrucción y estancamiento de las
secreciones pancreáticas con posterior desarrollo de la enfermedad. 2. La segunda teoría se basa en el desarrollo de fibrosis de novo
sin pancreatitis aguda, esta propone el desarrollo de fibrosis de tal vez pancreatitis agudas subclínicas recurrentes. La inflamación y la
necrosis de los episodios iniciales de pancreatitis aguda producen cicatrización alrededor de los conductos que conlleva a destrucción
de los ductos y por lo tanto estancamiento dentro de los mismos con formación de litos subsecuente. ES POSIBLE QUE EL PACIENTE
TENGA UNA HEPATITIS ALCOHÓLICA DE FONDO, PERO NO CORRESPONDE CON EL CUADRO CLÍNICO AGUDO QUE PRESENTA EN
ESTE MOMENTO.
PANCREATITIS La PANCREATITIS AGUDA es un proceso inflamatorio agudo del páncreas que frecuentemente involucra tejido peripancreático y
AGUDA. puede involucrar órganos y sistemas distantes. Cursa con dolor abdominal, elevación de amilasa y lipasa sérica o hallazgos de
pancreatitis en la tomografía axial computada o en la resonancia magnétia. - Pancreatitis leve: asociada con disfunción orgánica
mínima y una recuperación sin complicaciones. La falta de mejoría en 48 a 72hrs después de iniciar el tratamiento obliga a investigar
complicaciones de la pancreatitis. La característica macroscópica e histológica de la pancreatitis aguda leve es el edema intersticial. -
Pancreatitis aguda grave: está asociada con falla orgánica (respiratoria, renal, hepática, cardiovascular, hematológica y neurológica) y
complicaciones locales como necrosis, absceso o pseudoquiste. - Falla orgánica: se presenta con estado de choque, insuficiencia
respiratoria, falla renal, sangrado gastrointestinal, y algunas otras complicaciones sistémicas como coagulación intravascular
diseminada, productos de degradación de la fibrina o trastornos metabólicos graves. El alcohol causa al menos 30% de los casos de
pancreatitis aguda y es la etiología más común de pancreatitis crónica. Solo 10% de los pacientes alcohólicos crónicos desarrollan
pancreatitis crónica. Mientras que la pancreatitis aguda puede ser precipitada por litiasis vesicular de manera inmediata,
generalmente se requieren años de exposición al alcohol para que la produzca. CX: Los pacientes habitualmente refieren dolor
abdominal intenso en la región superior del abdomen, transfictivo al dorso, acompañado de náusea y vómito. EL CUADRO CLÍNICO
DE PANCRETATITIS ES COMPATIBLE CON LOS DATOS PRESENTADOS EN EL PACIENTE.

COLECISTITIS REPASO PANCREATITIS: La diferenciación entre la pancreatitis alcohólica y por litos es importante. La pancreatitis alcohólica se
CRÓNICA presenta con mayor frecuencia en hombres de aproximadamente 40 años de edad, el primer episodio clínico generalmente ocurre de
LITIÁSICA 5 a 10 años después de ingesta intensa de alcohol. Por el contrario la pancreatitis biliar es más frecuente en mujeres y el primer
AGUDIZADA. episodio clínico por lo general es después de los 40 años y en estos casos es imperativo realizar colecistectomía.

ULCERA REPASO PANCREATITIS. Algunos hallazgos clínicos señalan algunas causas específicas de la pancreatitis. Por ejemplo la presencia de
DUODENAL hepatomegalia, angiomas cutáneos e hipoplasia tenar e hipotecar sugieren pancreatitis alcohólica. Los xantomas eruptivos y la
PERFORADA. lipemia retínales sugieren pancreatitis por hipertrigliceridemia. Dolor en las parótidas e inflamación son características de paperas.

Bibliografía:
1. GUÍA DE PRÁCTICA CLÍNICA, DIAGNÓSTICO Y REFERENCIA OPORTUNA DE LA PANCREATITIS AGUDA EN EL PRIMER NIVEL DE ATENCIÓN.
MÉXICO: SECRETARIA DE SALUD, NOVIEMBRE 2012. 2. GUÍA DE PRÁCTICA CLÍNICA, DIAGNÓSTICO Y TRATAMIENTO DE LA PANCREATITIS
AGUDA. MÉXICO: SECRETARIA DE SALUD, 2009.
HTTP://WWW.CENETEC.SALUD.GOB.MX/DESCARGAS/GPC/CATALOGOMAESTRO/239_PANCREATITIS_AGUDA/PANCREATITIS_AGUDA_EVR_CENETEC.PDF

http://www.cenetec.salud.gob.mx/descargas/gpc/CatalogoMaestro/011_GPC_PancreatAguda/SSA-011-08_PANCREATITIS_AGUDAEVR.pdf

154 - EL TRATAMIENTO INICIAL CONSISTE EN:

HIDRATAR AL Los pacientes con PANCREATITIS AGUDA requieren hidratación intravenosa adecuada y analgesia adecuada para disminuir o
PACIENTE. eliminar el dolor. Se debe dejar a los pacientes en ayuno hasta que se les quite la náusea y el vómito completamente. El dolor
abdominal se puede tratar con opiáceos que deben ajustarse diariamente de acuerdo a las necesidades clínicas del paciente. La
colocación de una sonda nasogástrica no debe usarse de manera rutinaria ya que no es útil en los casos de pancreatitis leve, solo
debe indicarse para tratar íleo gástrico o intestinal o nausea y vómito refractarios. De manera similar no se deben usar los
inhibidores de la bomba de protones o de los receptores H2 ya que no proporcionan ningún beneficio. Se debe monitorizar a los
pacientes cuidadosamente buscando signos tempranos de falla orgánica como hipotensión, insuficiencia renal o pulmonar
monitorizando los signos vitales y el gasto urinario. Cuando se presente taquipnea se debe monitorizar la saturación de oxígeno
y si es necesario se debe tomar una gasometría, si existe hipoxemia suplementar con O2. Todos los pacientes que muestren
datos tempranos de falla orgánica deben ser trasladados a una terapia intensiva. EL REPOSO DEL TUBO DIGESTIVO ES LA BASE
DEL TRATAMIENTO DE LA PANCREATITIS, ES IMPORTANTE INICIAR EL MANEJO CON HIDRATACIÓN DEL PACIENTE LO MÁS
PRONTO POSIBLE. Lo mas importante en el tratamiento de la pancreatitis aguda es la hidratación intravenosa, ya que uno de los
marcadores de severidad de la enfermedad es el secuestro de líquidos. Es importante mantener un volumen intravascular
adecuado. Los pacientes con enfermedad severa pueden llegar a requerir un volumen hasta de 5 a 10 L al día de solución salina
isotónica los primeros días. En aquellos pacientes que no se puede estimar el gasto cardíaco se debe colocar un catéter de Swan
Ganz para evitar tanto la hidratación subóptima como la falla cardíaca congestiva.

COLOCAR PARCHE La reparación de una úlcera péptica perforada puede efectuarse con un simple cierre con sutura o con parche de epiplón
DE GRAHAM. (Graham) suturado o grapado. EL PARCHE DE GRAHAM CORRESPONDE AL MANEJO ENDOSCÓPICO QUIRÚRGICO DE LA ÚLCERA
PÉPTICA NO TIENE RELACIÓN CON PANCREATITIS.

ADMINISTRAR El uso de antibióticos profilácticos en los pacientes con PANCREATITIS LEVE está CONTRAINDICADO. Sin embargo existe
ANTIBIOTICOS. controversia sobre el uso de estos en pacientes con pancreatitis severa, debido a que los pacientes con pancreatitis necrótica
tienen mayor riesgo de infección y esta empeora por mucho el pronóstico.

PRACTICAR Hasta la actualidad existe controversia sobre el la realización de colangiopancreatografía endoscópica retrograda en los
COLECISTECTOMÍA. pacientes con pancreatitis biliar para la extracción de litos, en la fase aguda de la enfermedad. Existen estudios que lo
recomiendan y en otros los resultados son controvertidos e incluso promueven el desarrollo de complicaciones. LA
COLECISTECTOMÍA NO ESTA INDICADA EN LA PANCREATITIS AGUDA CUANDO SU ORIGEN ES BILIAR PERO SI
POSTERIORMENTE PARA EVITAR RECIDIVAS.

Bibliografía:
1. GUÍA DE PRÁCTICA CLÍNICA, DIAGNÓSTICO Y TRATAMIENTO DE PANCREATITIS AGUDA. MÉXICO: SECRETARIA DE SALUD; 2009.

http://www.cenetec.salud.gob.mx/descargas/gpc/CatalogoMaestro/239_PANCREATITIS_AGUDA/Pancreatitis_aguda_evr_cenetec.pdf

FIN DEL CASO CLÍNICO SERIADO


ANÁLISIS DEL CASO CLÍNICO

IDENTIFICACIÓN DEL REACTIVO


Area: MEDICINA INTERNA
Especialidad: HEMATOLOGÍA
Tema: TRANSTORNOS DE LA HEMOSTASIA
Subtema: COAGULACIÓN INTRAVASCULAR DISEMINADA

CASO CLÍNICO CON UNA PREGUNTA

MASCULINO DE 40 AÑOS SIN ANTECEDENTES DE IMPORTANCIA QUE ES TRATADO CON ANTIBIÓTICO POR UN
PROCESO INFECCIOSO GASTROINTESTINAL CON COAGULACIÓN INTRAVASCULAR DISEMINADA LA CUAL FUE
RESUELTA FAVORABLEMENTE. POSTERIOR AL TRATAMIENTO Y DURANTE LOS ESTUDIOS DE CONTROL SE
ENCUENTRA LEUCOPENIA SEVERA.

MASCULINO DE 40 AÑOS.

TRATADO CON ANTIBIÓTICO POR UN


PROCESO INFECCIOSO
GASTROINTESTINAL.

-.

-.

POSTERIOR AL TRATAMIENTO Y DURANTE


LOS ESTUDIOS DE CONTROL SE
ENCUENTRA LEUCOPENIA SEVERA.

155 - EL ANTIBIÓTICO QUE MUY PROBABLEMENTE RECIBIÓ Y QUE SE ASOCIA A ESTA COMPLICACIÓN DE
MANERA DOSIS DEPENDIENTE ES EL:
CLORANFENICOL. CLORANFENICOL Antibiótico bacteriostático aislado originalmente del Streptomyces
venezuelae. En la actualidad, dada la sencillez de su estructura química, se obtiene por
síntesis. Tiene un espectro de acción bastante amplio, pero debido a su toxicidad, su uso
actual ha quedado limitado al tratamiento de aquellas infecciones sensibles que
comprometan la vida del paciente, para las que no exista otra alternativa terapéutica.
Tianfenicol es un antibiótico del mismo grupo que cloranfenicol, obtenido por sustitución del
grupo NO2 de éste por un -SO2; parece producir menos anemia aplásica. Mecanismo de
acción y resistencia: Es bacteriostático, excepto para Haemophilus influenzae, para el que es
bactericida. El fármaco penetra por difusión facilitada al interior de la bacteria donde se une
a la fracción 50S del ribosoma impidiendo la transpeptidación entre los aminoácidos de la
cadena peptídica, con lo que impide la elongación de la cadena en crecimiento. El mecanismo
de resistencia más importante es extracromosómico, y se debe a un plásmido adquirido por
conjugación que transmite la capacidad para acetilar el antibiótico. El cloranfenicol acetilado
no se une al ribosoma. Espectro antibacteriano: Es activo frente a una gran variedad de
gérmenes, aunque muchos de ellos han desarrollado resistencia. Destaca la gran sensibilidad
a cloranfenicol de Haemophilus influenzae, Salmonella, y la mayoría de los anaerobios.
También son sensibles Micoplasmas, Rickettsias y Chlamydias. Reacciones adversas:
Toxicidad hematológica, que puede cursar como 2 cuadros distintos de depresión medular:
1º) toxicidad directa y dependiente de la dosis (reversible, afecta sobre todo a la serie roja,
incidencia de 1/20 pacientes tratados, usualmente empieza a los 7-10 días de iniciado el
tratamiento); 2º) toxicidad idiosincrásica, no-dependiente de la dosis, (baja incidencia de
1/30.000 tratamientos, produce un cuadro de aplasia medular, que puede aparecer incluso
meses después de finalizado el tratamiento y que suele ser fatal). Síndrome gris del recién
nacido, se debe a inmadurez hepática del niño para metabolizar el cloranfenicol. Se
caracteriza por náuseas, vómitos, flacidez, acidosis, hipotermia y color grisáceo. Indicaciones:
Siempre que no existan otras opciones más seguras: • Tratamiento de infecciones por
Salmonella • Meningitis por H. influenzae • Infecciones por B. fragilis • Infecciones por
rickettsias EL CLORANFENICOL SE ASOCIA CON APLASIA MEDULAR CON LO QUE PUEDE
OBSERVARSE LEUCOPENIA CONSIDERA ADEMÁS SU USO EN GASTROENTERITIS AGUDA.

TRIMETROPINA REACCIONES ADVERSAS DE LA TRIMETROPINA CON SULFAMETOXAZOL. Puede presentarse


MAS diarrea, mareos, dolor de cabeza, pérdida de apetito, náuseas o vómito, sensibilidad cutánea
SULFAMETOXAZOL. a la luz solar, picores o rash cutáneo (hipersensibilidad) y con una incidencia menos
frecuente: dolor articular y muscular (síndrome de Stevens-Johnson) dificultad al tragar
(síndrome de Lyell), fiebre (discrasias sanguíneas, hipersensibilidad), piel pálida o dolor de
garganta o hemorragias o hematomas no habituales (discrasias sanguíneas), color amarillo
en los ojos o en la piel (hepatitis). Púrpura, neutropenia, muy raramente agranulocitosis.
CIERTAMENTE PUEDE PROVOCAR NEUTROPENIA, SIN EMBARGO EN ORDEN DE FRECUENCIA
NO ES EL EFECTO ADVERSO MÁS COMÚN.

VANCOMICINA. REACCIONES ADVERSAS DE LA VANCOMICINA. La incidencia de reacciones tóxicas ha


disminuido a medida que se han desarrollado mejores procesos de manufacturación y se han
definido mejor los esquemas de dosificación. Puede producirse flebitis, por lo que se
aconseja la dilución del fármaco. Una reacción asociada a la administración i.v. rápida de
vancomicina es el enrojecimiento de cara, cuello y parte superior de tronco, acompañado de
prurito y hormigueo; raras veces hipotensión. (Red man syndrome). Puede evitarse con una
infusión lenta de 60 minutos asociada a antihistamínicos. Aunque semeja ser de causa
alérgica, se trata de un cuadro histamínico. Con los procedimientos actuales de purificación
los efectos tóxicos severos de vancomicina son raros. El riesgo potencial más severo es sobre
el nervio auditivo, lo que es poco frecuente con las concentraciones séricas habituales. El
riesgo aumenta en tratamientos prolongados o cuando se la asocia a aminoglucósidos.
Ocasionalmente produce toxicidad vestibular, aunque este efecto es raro con preparados
más purificados. Las nuevas presentaciones no son nefrotóxicas, aunque la asociación con
otras drogas nefrotóxicas como aminoglucósidos aumenta el riesgo de esa toxicidad.
Erupciones alérgicas y fiebre son raras. La neutropenia solo se observó en un 2%, se ha
descrito trombocitopenia y eosinofilia. A PESAR DE QUE ESTE MEDICAMENTO TAMBIÉN
PRODUCE LEUCOPENIA, SU ADMINISTRACIÓN POR UN PROCESO GASTROINTESTINAL
AMBULATORIO, SERÍA TAN POCO PROBABLE QUE LA DESCARTA.
TETRACICLINA. TETRACICLINAS Conjunto de antibióticos obtenidos a partir de varias especies de
Streptomyces (clortetraciclina, oxitetraciclina, tetraciclina) o bien por semisíntesis
(tetraciclina, demeclociclina, metaciclina, doxiciclina y minociclina). Todos los antibióticos del
grupo comparten una serie de características comunes (estructura química, espectro
antimicrobiano, mecanismo de acción y toxicidad). Las principales diferencias radican en su
perfil farmacocinético, lo que permite agrupar a las tetraciclinas en tres categorías: 1º) las de
vida media corta (6-8 h), como clortetraciclina, oxitetraciclina y tetraciclina); 2º) las de vida
media intermedia (12-14 h), como demeclociclina y metaciclina); y 3º) las de vida media larga
(16-18 h), como doxiciclina y minociclina, las más liposolubles. Estructura química: Derivan
del núcleo tetracíclico octahidronaftaleno. Son inestables en solución (epitetraciclinas).
Mecanismo de acción y resistencia: El antibiótico accede al interior celular por un doble
mecanismo de difusión pasiva y transporte activo. Una vez allí, se une a la subunidad 30S del
ribosoma bacteriano bloqueando la fijación del aminoacil-tRNA al sitio aceptor (A) del
complejo formado por el mRNA y la subunidad 50S del ribosoma. De esta forma, actúan
como bacteriostáticos al impedir la adición de nuevos aminoácidos a la cadena peptídica en
crecimiento. La resistencia a tetraciclinas se establece lentamente y son inducibles. El
mecanismo de resistencia se basa en un proceso por el que la bacteria impide la penetración
del antibiótico desde el exterior. La resistencia es cruzada para todas las tetraciclinas y se
transmite por plásmidos. Espectro: Amplio, actúan sobre bacilos y cocos Gram (+), bacilos
Gram (-) [H. influenzae, Brucella, Legionella Pneumophila, Helicobacter pylori, Borrelia
recurrentis], así como sobre Rickettsia, Mycoplasma, Chlamydia y Espiroquetas. Reacciones
adversas: Molestias gastrointestinales (las más frecuentes); excepcionalmente degeneración
grasa hepática. Erupciones cutáneas e hipersensibilidad a la luz. Toxicidad renal (síndrome de
Fanconi). Alteraciones dentarias, óseas y de las uñas por depósito del antibiótico.
Superinfección por Proteus, Pseudomona y Candida. Alteraciones neurológicas (toxicidad
vestibular de minociclina). Contraindicada su utilización en niños menores de 8 años y en
mujeres embarazadas. Indicaciones: Primera elección en cólera, brucelosis, infecciones por
Rickettsias, Chlamydia y Espiroquetas. Alternativa en infecciones por Legionella y
Micoplasma. Tratamiento del acné grave. LAS ALTERACIONES HEMATOLÓGICAS NO
CORRESPONDEN A EFECTOS ADVERSOS COMUNES TRAS LA ADMINISTRACIÓN DE
TETRACICLINAS.

Bibliografía:
1. LONGO DL, FAUCI AS, KASPER DL, HAUSERSL, JAMESON JL, LOSCALZOJ. HARRISON. PRINCIPIOS DE
MEDICINA INTERNA, 18A EDICIÓN. MC GRAW HILL. NEW YORK, USA. 2012, PP 888, 893, 905.
ANÁLISIS DEL CASO CLÍNICO

IDENTIFICACIÓN DEL REACTIVO


Area: CIRUGÍA
Especialidad: OFTALMOLOGÍA
Tema: ENFERMEDADES DE LA RETINA
Subtema: RETINOPATÍA DIABÉTICA

CASO CLÍNICO CON UNA PREGUNTA

HOMBRE DE 63 AÑOS DE EDAD, CON DIAGNÓSTICO DE DIABETES DESDE HACE 15 AÑOS. DURANTE EL
ÚLTIMO AÑO, CON MUY MAL APEGO AL TRATAMIENTO Y DESCONTROL SECUNDARIO. REFIERE VISIÓN
NUBLADA EN LOS MOMENTOS DE MAYOR DESCONTROL GLUCÉMICO.

adulto mayor de 63 años DE EDAD.

Diabetes con más de 10 años de evolución.


Umbral considerado para la presencia de
retinopatía, paciente mal controlado.

Visión nublada, que se intensi ca en los


momentos de descontrol glucémico.

-.

-.

156 - DURANTE LA EXPLORACIÓN OFTALMOSCÓPICA, USTED DEFINIRÁ A LA CAPACIDAD QUE TENGA EL


PACIENTE DE DETECTAR PEQUEÑAS DIFERENCIAS DE PROFUNDIDAD ENTRE DOS OBJETOS COMO:

AGUDEZA VISUAL AGUDEZA VISUAL. Es la capacidad de discriminar como diferentes dos puntos u objetos
ABSOLUTA. próximos. Refleja de forma gruesa la función visual (transparencia de medios y función
retiniana en el área macular, estado refractaria y trastornos neurooftalmológicos) AGUDEZA
VISUAL ABSOLUTA. Se define como la agudeza visual que obtenemos cuando el error de
refracción se corrige en condiciones de ciclopejía, mediante una lente situada en el punto focal
anterior del ojo.

AGUDEZA VISUAL AGUDEZA VISUAL ESTEREOSCÓPICA. Capacidad de detectar pequeñas diferencias de


ESTEREOSCÓPICA. profundidad entre dos objetos. DEBES USAR LA LÓGICA DE LAS PALABRAS PARA AYUDARTE
A CONTESTAR ESTA PREGUNTA, RECUERDA QUE ESTEREOSCÓPICO HACE REFERENCIA A LAS
VISIONES 3D LO QUE INCLUYE PROFUNDIDAD A LAS IMÁGENES 2D; DE AHÍ QUE SIEMPRE
QUE TE HABLEN DE PROFUNDIDAD EN LA VISIÓN DEBERÁS ELEGIR ESTA RESPUESTA COMO
CORRECTA.
AGUDEZA VISUAL AGUDEZA VISUAL RELATIVA/ CON CORRECCIÓN/ CORREGIDA. Agudeza visual máxima que el
RELATIVA. ojo posee con refracción completamente corregida y con las lentes puestas en la posición
habitual.

AGUDEZA VISUAL AGUDEZA VISUAL SIN CORRECCIÓN. Agudeza visual máxima que puede alcanzar un ojo sin
SIN CORRECCIÓN. necesidad de corrección óptica.

Bibliografía:
1. GUÍA DE PRÁCTICA CLÍNICA, DIAGNÓSTICO Y TRATAMIENTO DE RETINOPATÍA DIABÉTICA. MÉXICO:
SECRETARIA DE SALUD; 2009. 2. GUÍA DE REFERENCIA RÁPIDA, DIAGNÓSTICO Y TRATAMIENTO DE
RETINOPATÍA DIABÉTICA. MÉXICO: SECRETARIA DE SALUD; 2009.

http://www.cenetec.salud.gob.mx/descargas/gpc/CatalogoMaestro/171_GPC_RETINOPATIA_DIABETICA/Imss_171ER.pdf
ANÁLISIS DEL CASO CLÍNICO

IDENTIFICACIÓN DEL REACTIVO


Area: CIRUGÍA
Especialidad: CIRUGÍA ABDOMINAL
Tema: PATOLOGÍA ABDOMINAL RESOLUCIÓN QUIRÚRGICA DE URGENCIA
Subtema: HERNIA INGUINAL

CASO CLÍNICO CON UNA PREGUNTA

MASCULINO DE 42 AÑOS DE EDAD QUE INGRESA A URGENCIAS POR PRESENTAR COMPLICACIÓN


SECUNDARIA A HERNIA INGUINAL DERECHA.

masculino de 42 años.

-.

Hernia inguinal complicada.

-.

-.

157 - EN LOS PACIENTES CON HERNIA INGUINAL LA COMPLICACIÓN MAS TEMIDA ES:

INCARCERACIÓN. La HERNIA INGUINAL INCARCERADA es una HERNIA IRREDUCTIBLE. Sin embargo, ello no
debe implicar una urgencia quirúrgica, ya que son comunes los estados crónicos de
incarceración por el tamaño del cuello de la hernia en relación con su contenido o por
adherencias al saco herniario. En éstas la irreductibilidad se acompaña de un trastorno del
tránsito intestinal, con un cierto grado de obstrucción intestinal, pero sin existir compromiso
vascular ni isquemia intestinal. La incarceración se suele seguir de la estrangulación. El
tratamiento recomendado de una hernia incarcerada es la reparación quirúrgica, pero no es
urgente porque no existe una complicación que ponga en peligro la vida. LA HERNIA
INCARCERADA NO SE REDUCE PERO NO PONE EN RIESGO LA VIABILIDAD DE SU
CONTENIDO.
ESTRANGULACIÓN. LA COMPLICACIÓN MÁS IMPORTANTE Y TEMIDA EN LOS PACIENTES CON HERNIA
INGUINAL ES LA ESTRANGULACIÓN. Es un estado importante que pone en peligro la vida
porque el contenido de la hernia se torna isquémico e inviable. Las características clínicas de
una obstrucción estrangulada son notables. Además de que el paciente desarrolla una hernia
irreductible y una obstrucción intestinal, los signos clínicos indican que tuvo lugar una
estrangulación. La hernia está tensa, muy sensible y la piel que la recubre puede tener un
tinte rojizo o azulado. No existen ruidos intestinales dentro de la hernia. El paciente suele
tener leucocitosis con desviación a la izquierda, se encuentra tóxico, deshidratado y febril.
Los gases en sangre arterial muestran acidosis metabólica. Su tratamiento es quirúrgico de
Urgencia, por el peligro de necrosis (muerte) intestinal y perforación. Esto último es una
complicación temible de la hernia complicada, ya que parte de su tratamiento es la resección
intestinal del segmento afectado. EL PRINCIPAL PROBLEMA EN LAS HERNIAS
ENCARCELADAS ES QUE HAY QUE VALORAR LA VIABILIDAD DE LOS TEJIDOS DEL
CONTENIDO HERNIARIO.

TORSIÓN La TORSIÓN TESTICULAR ocurre por una incompleta fijación testicular. Se presenta a
TESTICULAR. cualquier edad, siendo más frecuente en dos períodos: la vida intrauterina (torsión
extravaginal, 5% de los casos) a partir de la semana 28a de gestación y la pubertad (torsión
intravaginal), entre los 8 y los 18 años. Esta torsión es dos veces más frecuente a izquierda.
Entre los factores predisponente están: el traumatismo (20%), la criptorquidia, los tumores
testiculares y el antecedente de torsión testicular contralateral previa (30% torsión bilateral
asincrónica). El diagnóstico es clínico, y se basa en el inicio brusco, aumento de volumen,
signos inflamatorios locales y reflejo cremasteriano abolido. Si el cuadro es de larga data y
tiene compromiso vascular avanzado, el testículo aparece de consistencia dura, e indoloro. La
ecotomografía testicular, con Doppler color y los estudios de perfusión nuclear son de
utilidad, pero de difícil disponibilidad y no descartan en un 100% el diagnóstico de torsión
testicular. Por las gravísimas consecuencias, ante la sospecha clínica y para no retardar el
tratamiento, la exploración quirúrgica es obligatoria. Ésta consiste en confirmar el
diagnóstico, reducir la torsión y practicar una fijación testicular bilateral. En el caso de daño
testicular irreversible se realiza la orquidectomía y la fijación testicular contralateral. El
pronóstico depende de la precocidad de la consulta, el grado de la sospecha diagnóstica y el
tratamiento oportuno. El daño vascular aparece después de 6 horas de iniciada la torsión. En
la práctica clínica, solo el 25% de los casos de torsión testicular son tratadas a tiempo. ES
UNA PATOLOGÍA QUE AMERITA URGENCIA, SE PRESENTA MÁS COMÚNMENTE EN EDADES
TEMPRANAS, POR LO QUE NO SE RELACIONA CON EL CASO CLÍNICO.

VARICOCELE. El VARICOCELE es la dilatación de las venas del cordón espermático, determinada por la
dilatación de la vena espermática, usualmente en el lado izquierdo (70% a 75%), seguido en
el lado derecho (15% a 20%), o bilateralmente (10%). Al parecer su principal causa es la
obstrucción que producen las valvas, aumentando la presión y causando reflujo venoso;
igualmente el aumento de la presión y el reflujo venoso suceden con la llegada de la vena
espermática o gonadal izquierda a la vena renal en el lado izquierdo. El Varicocele está
presente en un 15% de la población y aproximadamente el 40% de los hombres que
presentan infertilidad, tienen algún grado de varicocele. La incidencia de dolor o malestar
asociada al varicocele es del 2% al 10%.El tratamiento del varicocele implica siempre un
procedimiento quirúrgico, por lo que es necesario precisar las indicaciones de cirugía para un
grupo de pacientes que no consulta por la más importante y significativa patología clínica
descripta en el adulto: la infertilidad. El tratamiento del varicocele implica siempre un
procedimiento quirúrgico, por lo que es necesario precisar las indicaciones de cirugía para un
grupo de pacientes que no consulta por la más importante y significativa patología clínica
descrita en el adulto: la infertilidad. EL VARICOCELE SE PUEDE ASOCIAR A LA HERNIA PERO
NO AMERITA URGENCIA QUIRÚRGICA.

Bibliografía:
1. GUÍA DE PRÁCTICA CLÍNICA, DIAGNÓSTICO Y TRATAMIENTO DE HERNIAS INGUINALES Y FEMORALES.
MÉXICO: SECRETARIA DE SALUD; NOVIEMBRE 2012. RECUPERADO DE
HTTP://WWW.CENETEC.SALUD.GOB.MX/CONTENIDOS/GPC/CATALOGOMAESTROGPC.HTML 2. DOHERTY G.
DIAGNÓSTICO Y TRATAMIENTO QUIRÚRGICO, 13A EDICIÓN. MC GRAW HILL LANGE. 2011, PP 626.

http://www.cenetec.salud.gob.mx/descargas/gpc/CatalogoMaestro/015_GPC_HerniasIng.yFem/SSA_015_08_EyR.pdf
ANÁLISIS DEL CASO CLÍNICO

IDENTIFICACIÓN DEL REACTIVO


Area: CIRUGÍA
Especialidad: PROCTOLOGÍA Y UROLOGÍA
Tema: DISFUNCIÓN ERÉCTIL
Subtema: DISFUNCIÓN ERÉCTIL

CASO CLÍNICO CON UNA PREGUNTA

MASCULINO DE 50 AÑOS, DIABÉTICO DESDE HACE OCHO AÑOS, TRATADO CON GLIBENCLAMIDA DESDE
HACE DOS AÑOS. PRESENTA, DE MANERA PROGRESIVA DIFICULTAD PARA TENER ERECCIONES.

masculino de 50 años.

diabético desde hace 8 años, tratado con


glibenclamida. DIFICULTAD PROGRESIVA
PARA TENER ERECCIONES.

La disfunción eréctil se ha de nido como la


incapacidad para obtener y mantener una
erección con la su ciente rigidez que
permita una penetración sexual
satisfactoria para alcanzar el orgasmo y la
eyaculación en al menos 50% de las
relaciones sexuales durante un periodo de
3 meses.

-.

-.

158 - EL MECANISMO FISIOPATOLÓGICO QUE MÁS PROBABLMENTE AFECTA LA FUNCIÓN SEXUAL DE


ÉSTE PACIENTE ES:
DEGENERACIÓN REPASO DE DISFUNCIÓN ERÉCTIL Y DIABETES. PREVALENCIA. Es difícil conocer la verdadera
FIBRÓTICA DE LOS prevalencia de la Disfunción Eréctil (DE) debido a la diversidad de definiciones, lo que
CUERPOS conduce a imprecisión en el diagnóstico. Sin embargo, se ha estimado una prevalencia
CAVERNOSOS. global de 19.2% en hombres de 30 a 80 años de edad y más específicamente de 39, 48, 57 y
67%, entre los varones de 40, 50, 60 y 70 años, respectivamente, incrementándose la
gravedad a medida que avanza la edad. Tratándose de pacientes con diabetes mellitus, la DE
es tres veces más común que en los hombres no diabéticos y parece afectarlos más
tempranamente. En general, se encuentra DE en 35 a 75% de los pacientes diabéticos de
todas las edades. La incidencia incrementa con la edad avanzada, la duración de la diabetes
y el deterioro del control metabólico, mientras que la severidad de la disfunción eréctil está
correlacionada con el control glucémico, la duración de la enfermedad y la presencia de
complicaciones inducidas por la diabetes mellitus, siendo el tabaquismo un factor
contribuyente. SI TE DAS CUENTA CONFORME INCREMENTA LA EDAD Y EL TIEMPO DE
EVOLUCIÓN ES COMÚN ENCONTRAR LA ASOCIACIÓN A DISFUNCIÓN ERÉCTIL EN ESTOS
PACIENTES.

EFECTO DEL REPASO DE DISFUNCIÓN ERÉCTIL Y DIABETES. La disfunción erectil (DE) está asociada a las
TRATAMIENTO CON siguientes manifestaciones que, por otra parte, se relacionan también con la diabetes
EL mellitus: - Psicológicas. En el paciente hay un deterioro de la imagen de sí mismo y de la
HIPOGLUCEMIANTE. autoestima, debido a su percepción de incapacidad para lograr una erección, presentando
altos niveles de sumisión, abnegación, conformismo, dependencia y timidez. - Otras
disfunciones sexuales. La DE puede estar asociada a deseo sexual hipoactivo y a eyaculación
precoz. - Estilo de vida. La disfunción eréctil influye en la rutina diaria, en la interacción
social y en la calidad de vida del paciente. La insatisfacción sexual se asocia con
insatisfacción en otras áreas de la vida. - Calidad de vida. La DE tiene una influencia negativa
en el bienestar del individuo y en su calidad de vida. EL CONTROL GLUCÉMICO DE LOS
PACIENTES (HIPOGLUCEMIANTES ORALES) EN GENERAL SE HA ASOCIADO A UNA MENOR
PRESENCIA DE COMPLICACIONES CRÓNICAS DE LA ENFERMEDAD.

NEUROANGIOPATÍA Una actividad sexual adecuada requiere la adquisición y el mantenimiento de la erección


DIABÉTICA. peniana, fenómeno que fundamentalmente es vascular, aunque inducido por estímulos
neurológicos y psicológicos. Cualquier alteración en los distintos eventos que conducen a la
erección da lugar a la disfunción eréctil, situación observada con una alta frecuencia en los
pacientes diabéticos. En realidad se desconoce la patogénesis exacta de la disfunción eréctil
en la diabetes mellitus pero se presume que es multifactorial, teniendo como base una
alteración neurológica, vascular o una combinación de ambas. Dentro de las causas que
subyacen en la patogénesis de la DE en los pacientes con DM se ha considerado en primer
término a la alteración vascular, ya que se observa una disfunción endotelial que se presenta
tempranamente y está estrechamente relacionada con microangiopatía y enfermedad
macrovascular que afectan al músculo liso. Otra alteración importante en la diabetes
mellitus es la neuropatía. Los grandes nervios parasimpáticos de los órganos pélvicos son los
más vulnerables de los nervios autónomos, lo cual puede explicar por qué la falla eréctil es a
menudo la manifestación clínica más temprana y común de la neuropatía autónoma
diabética. Ansiedad. Hay ansiedad sexual, miedo al fracaso y preocupación sobre una
disminución de la percepción subjetiva de la excitación sexual y del placer. LA NEUROPATÍA
AUTÓNOMA DIABÉTICA PUEDE PROVOCAR AFECCIONES GENITOURINARIAS QUE
COMPRENDEN CISTOPATÍA, DISFUNCIÓN ERÉCTIL Y DISFUNCIÓN SEXUAL FEMENINA. LA
DISFUNCIÓN ERÉCTIL Y LA EYACULACIÓN RETRÓGRADA SUELEN SER LOS PRIMEROS
INDICIOS DE NEUROPATÍA DIABÉTICA.
EFECTO PSICÓGENO TRATAMIENTO. Lo primero a considerar en el tratamiento es la eliminación de factores
POR DEPRESIÓN. contribuyentes de la enfermedad vascular, como la hiperglucemia, el sedentarismo o la
ingesta de alcohol. También debe considerarse el uso de medicamentos con menores efectos
adversos sobre la función eréctil para tratar las alteraciones coexistentes como
antihipertensivos y antidepresivos y se debe investigar el funcionamiento del sistema
cardiovascular antes de iniciar cualquier tratamiento para DE. - Inhibidores de la 5-
fosfodiesterasa: Sin duda, el tratamiento farmacológico de primera línea actualmente son
los inhibidores de la 5-fosfodiesterasa que incluyen al sildenafil, vardenafil y tadalafil. -
Apomorfina. Esta sustancia por vía sublingual activa predominantemente receptores
dopaminérgicos D2 del hipotálamo, activando vías neurogénicas centrales involucradas en la
erección. - Dispositivos de vacío. En pacientes en quienes la terapia oral está contraindicada
los dispositivos de vacío son una buena alternativa. Se crea una presión negativa de 200 a
250 mmHg, lo que produce llenado de los cuerpos cavernosos; la bomba se retira y se coloca
un anillo de constricción por no más de 30 minutos para mantener la erección.
Prostaglandina E1, papaverina, fentolamina. - Cuando se contraindica el uso de inhibidores
de la 5-FDE, no hay respuesta a ellos, se presentan reacciones intolerables o se prefiere no
utilizar el dispositivo de vacío. Hay una segunda línea de tratamiento que consiste en la
aplicación de inyecciones intracavernosas de prostaglandina E1 (PGE1, alprostadil) que
estimula a la enzima adenilato ciclasa e incrementa los niveles de AMP cíclico. - Tratamiento
psicológico. La terapia médica, psicológica y sexual son herramientas importantes que
deben ser consideradas simultáneamente para el tratamiento integral del paciente; por esta
razón es esencial contar con un equipo multidisciplinario.

Bibliografía:
1. LONGO DL, FAUCI AS, KASPER DL, HAUSERSL, JAMESON JL, LOSCALZOJ. HARRISON. PRINCIPIOS DE
MEDICINA INTERNA, 18A EDICIÓN. MC GRAW HILL. NEW YORK, USA. 2012.
ANÁLISIS DEL CASO CLÍNICO

IDENTIFICACIÓN DEL REACTIVO


Area: GINECOLOGÍA Y OBSTETRICIA
Especialidad: OBSTETRICIA
Tema: COMPLICACIONES DEL EMBARAZO Y DEL PARTO
Subtema: RESTRICCIÓN DEL CRECIMIENTO INTRAUTERINO

CASO CLÍNICO CON UNA PREGUNTA

MUJER DE 20 AÑOS DE EDAD, CON ANTECEDENTE DE GESTA 2, CESÁREA 1, POR BAJA RESERVA FETAL Y
RESTRICCIÓN DEL CRECIMIENTO INTRAUTERINO.

20 AÑOS.

GESTA 2, CESÁREA 1 POR BAJA RESERVA


FETAL Y RESTRICCIÓN DEL CRECIMIENTO
INTRAUTERINO.

-.

-.

-.

159 - EL MECANISMO FISIOLÓGICO A TRAVÉS DEL CUAL SE ASEGURA EL PASO DE GLUCOSA A TRAVÉS DE
LA PLACENTA ES:

ARRASTRE El paso de moléculas de la circulación materna a la fetal o viceversa se denomina TRANSFERENCIA


POR EL PLACENTARIA. Por la existencia de la Barrera Placentaria, el paso de estas moléculas esta
SOLVENTE. controlado por lo que se requiere mecanismos de transporte. ARRASTRE POR EL SOLVENTE: El
crecimiento fetal supone aumento progresivo de espacios intercelulares. Paso paracelular. Ejemplo:
ELECTROLITOS.

DIFUSIÓN DIFUSIÓN SIMPLE: Regida por la Ley de Fick, la velocidad de transferencia disminuye con el tamaño
SIMPLE. de la molécula y la hidrosolubilidad. Ejemplos: ÁCIDOS GRASOS, ELECTROLITOS Y GASES.

DIFUSIÓN DIFUSIÓN FACILITADA: Aumenta de forma especifica el coeficiente de difusión de la sustancia. Se


FACILITADA. precisa de transportador estereoespecífico. El transporte de difusión facilitada coexiste con un
componente de difusión simple. Ejemplos: GLUCOSA, LACTATO.
TRANSPORTE TRANSPORTE ACTIVO: Contra gradiente, especifico y dependiente de energía. Mediados por
ACTIVO. transportadores. Ejemplos: CATIONES, VITAMINAS HIDROSOLUBLES Y AMINOÁCIDOS.

Bibliografía:
1. CUNNINGHAM G, LEVENO K, BLOMM S, HAUTH J, RPUSE D, SONG C. WILLIAMS OBSTETRICIA, 23A
EDICIÓN. MC GRAW HILL. USA. 2011 EN ESPAÑOL, PP 87.
ANÁLISIS DEL CASO CLÍNICO

IDENTIFICACIÓN DEL REACTIVO


Area: MEDICINA INTERNA
Especialidad: REUMATOLOGÍA
Tema: SÍNDROMES DE LAS VASCULITIS
Subtema: LUPUS ERITEMATOSO SISTÉMICO

CASO CLÍNICO CON UNA PREGUNTA

MUJER DE 46 AÑOS QUE INGRESO AL HOSPITAL CON DIAGNÓSTICO DE ENCEFALITIS EN ESTUDIO.


ACTUALMENTE SE SOSPECHA LA POSIBILIDAD DE CURSAR CON LUPUS ERITEMATOSO GENERALIZADO.

mujer de 46 años de edad.

Encefalitis EN ESTUDIO.

se sospecha que este cursando con les.

-.

-.

160 - ANTICUERPOS MÁS SENSIBLES QUE SE DEBERÁN SOLICITAR PARA EL DIAGNÓSTICO DE LUPUS
ERITEMATOSO SISTÉMICO:

ANTIHISTONA. Si los ANTICUERPOS ANTINUCLEARES son positivos en un paciente con datos clínicos
compatibles de LES, se deben solicitar estudios complementarios. Estos son anti-dsDNA,
anti-Smith y los niveles séricos de complemento. Los anticuerpos anti-dsDNA y anti-Smith
son muy específicos de lupus, y los niveles aumentados de anti-dsDNA y de complemento
disminuidos puede indicar actividad de la enfermedad. También se deben solicitar
anticuerpos antifosfolípido, incluyendo el anticoagulante lúpico y los anticuerpos
anticardiolipina. Si estos se encuentran elevados confirman el diagnóstico de lupus e
indican un aumento en el riesgo de enfermedad tromboembólica venosa y arterial así
como posibles complicaciones durante el embarazo.
ANTIMICROSOMALES. La característica del LUPUS ERITEMATOSO SISTÉMICO (LES) es la producción de
autoanticuerpos dirigidos contra el DNA, RNA, proteínas citoplásmicas y nucleares. "LOS
ANTICUERPOS ANTINUCLEARES SE OBSERVAN EN EL 100% DE LOS PACIENTES", los
anticuerpos anti DNA en 60 a 70% de los casos dependiendo del estudio empleado. Los
anticuerpos contra las proteínas de unión-RNA anti-U1 RNP y los anticuerpos anti-Sm se
presentan en aproximadamente 70% y 40% de los pacientes respectivamente. LOS
ANTICUERPOS ANTINUCLEARES SON EL MEJOR ESTUDIO DE DETECCIÓN PARA LES.

ANTINUCLEARES. Los anticuerpos antinucleares tienen una "alta sensibilidad" para el diagnóstico de LES,
pero pueden tener baja especificidad. Los anticuerpos antinucleares pueden ser positivos
en personas sana o en muchas otras enfermedades. Por esto la prueba de para detección
de anticuerpos antinucleares debe realizarse en aquellos pacientes en los que se sospecha
LES en un contexto clínico y/o con otros estudios de laboratorio que lo sugieran. Una
prueba de fluorescencia negativa para ANA (FANA) descarta la existencia de LES. Si se
solicita la prueba de FANA, es muy poco probable el diagnóstico de LES si los títulos son
menores de 1:160. LA TITULACIÓN DE ANTICUERPOS ANTINUCLEARES ESTÁN INDICADOS
EN SOSPECHA DE LEP EN PACIENTES SINTOMÁTICOS.

ANTIFOSFOLÍPIDOS. Otros autoanticuerpos que se pueden encontrar en los pacientes con LES son: los
anticuerpos contra Ro y La se encuentran en 30 a 40% y 15 a 20% de los pacientes
respectivamente. El anticuerpo antifosfolípido que se mide con mayor frecuencia son las
anticardiolipinas, que se encuentran en 50 a 60% de los pacientes y el anticoagulante
lúpico en 20% de los casos. La presencia de anticuerpos antifosfolípido se asocia con
aumento del riesgo de trombosis, desarrollo de corea, necrosis avascular, epilepsia,
migraña y livedo reticularis. Los anticuerpos antiribosoma P están presentes en 15% de los
pacientes. Un mayor porcentaje de los pacientes con elevación de este anticuerpo
presentan psicosis. El factor reumatoide se encuentra positivo en 12% a 29% de los
pacientes y en general se asocia con los anticuerpos anti Ro y La. LOS ANTICUERPOS
ANTIFOSFOLÍPIDOS PUEDEN ESTAR PRESENTES EN LES PERO EN MENOR PROPORCIÓN
QUE LOS ANTINUCLEARES.

Bibliografía:
1. LONGO DL, FAUCI AS, KASPER DL, HAUSERSL, JAMESON JL, LOSCALZOJ. HARRISON. PRINCIPIOS DE
MEDICINA INTERNA, 18A EDICIÓN. MC GRAW HILL. NEW YORK, USA. 2012, PP 2726, 2730. 2. PAPADAKIS
MAXINE A, MCPHEE STEPHEN J. DIAGNÓSTICO CLÍNICO Y TRATAMIENTO. 52ª EDICIÓN. NUEVA YORK. 2013,
PP 833-834.
ANÁLISIS DEL CASO CLÍNICO

IDENTIFICACIÓN DEL REACTIVO


Area: MEDICINA INTERNA
Especialidad: NEUROLOGÍA
Tema: ENFERMEDAD DESMIELINIZANTE
Subtema: ESCLEROSIS MÚLTIPLE Y SX DE GUILLIAN BARRE

CASO CLÍNICO SERIADO

MASCULINO DE 38 AÑOS, MÉDICO DE PROFESIÓN. DESDE HACE 2 MESES PRESENTA SINTOMATOLOGÍA A


NIVEL DEL SISTEMA NERVIOSO, SOSPECHANDO ACTUALMENTE EN ESCLEROSIS MÚLTIPLE. REALIZA USTED
PUNCIÓN LUMBAR SOLICITANDO ELECTROFORESIS DE PROTEÍNAS.

MASCULINO DE 38 AÑOS. Edad de inicio es


entre 20 y 50 años.

-.

DESDE HACE 2 MESES PRESENTA


SINTOMATOLOGÍA A NIVEL DEL SISTEMA
NERVIOSO, SOSPECHANDO
ACTUALMENTE EN ESCLEROSIS MúLTIPLE.

-.

REALIZA USTED PUNCIÓN LUMBAR


SOLICITANDO ELECTROFORESIS DE
PROTEÍNAS.

161 - LA INMUNOGLOBULINA QUE SE ENCUENTRA ELEVADA EN EL LIQUIDO CEFALORRAQUIDEO DE LOS


PACIENTES CON ESCLEROSIS MULTIPLE ES LA:

A. La ESCLEROSIS MÚLTIPLE es una enfermedad crónica, inflamatoria, desmielinizante del sistema nervioso
central, constituye una de las causas más comunes de incapacidad neurológica en el adulto joven. Es
caracterizada por la recurrencia de eventos neurológicos multifocales y signos y síntomas con grado variable
de recuperación. Eventualmente, la mayoría de los pacientes desarrolla un curso clínico progresivo de la
enfermedad; puede cursar también con episodios de exacerbaciones y remisiones. La triada clásica de la EM es:
inflamación, desmielinización y gliosis (cicatricial).
E. EPIDEMIOLOGÍA Y FACTORES DE RIESGO. Es la causa más común de discapacidad neurológica no traumática
en adultos jóvenes, se estima que existen entre 300,000 y 350,000 personas afectadas en EUA. La prevalencia
varía geográficamente, logrando una prevalencia de 5 a 30 por 100,000 habitantes en Norteamérica y Europa.
La edad de inicio es entre 20 y 50 años de edad. La relación hombre mujer es de 1:2.3. Aunque casi todos lo
casos son esporádicos, la relación en primer grado con una persona con EM aumenta el riesgo de 20 a 40 veces,
comparado con el de la población general, con un riesgo absoluto menor al 5%. Estudios recientes muestran
que no existe evidencia suficiente que asocie a las vacunas como causa ni como desencadenante de los brotes
de la EM. En el estudio llamado “In The Nurses Health Study”, se demostró que no había relación entre las
pacientes que recibieron la vacuna contra hepatitis B y las que presentaron esclerosis múltiple. Los estudios de
la base de datos de Canadá y Europa demostraron que no existía mayor riesgo de exacerbaciones en pacientes
que fueron inmunizados contra hepatitis B, tétanos o influenza.

D. ETIOLOGÍA. El HLA-DR en el cromosoma 6p21 (que codifica para proteínas que participan en la presentación
de antígenos de las células T), es el factor de susceptibilidad más importante que se ha identificado hasta la
fecha. Los polimorfismos de HLA-DR no se asocian con el curso y severidad de la EM a pesar de su importante
contribución a la susceptibilidad de la enfermedad. Existe evidencia epidemiológica de que la exposición
ambiental juega un papel importante para el desarrollo de EM. La mayor incidencia en personas de nivel
socioeconómico alto, se explica por la deficiente exposición a agentes infecciosos en edades tempranas. Se han
identificado en el suero de los pacientes con EM (sarampión, varicela, rubéola, VEB, HVS e influenza C) títulos
altos de anticuerpos contra diversos virus. En un estudio realizado Chlamydia pneumoniae fue aislada en el
64% de los pacientes con EM, comparado con el 11% del grupo control, identificándose como un agente causal
potencial.

G. SIGNOS Y SÍNTOMAS. La ESCLEROSIS MÚLTIPLE (EM) causa una gran variedad de signos y síntomas
neurológicos, los cuales pueden ser agudos (desarrollándose en horas o días) o crónicos y progresivos. Los
síntomas más comunes de presentación son disfunción sensorial, neuritis óptica, diplopía, paresia de piernas,
ataxia, fenómeno de Lhermitte (parestesias en extremidades con flexión de cuello) alteraciones en el estado
cognitivo y afectivo, vejiga neurogénica y alteraciones intestinales. Diagnóstico No existe actualmente ninguna
prueba específica para realizar el diagnóstico de EM. La clasificación diagnóstica más común hace énfasis en el
creciente papel de la RM, para demostrar diseminación de las lesiones en tiempo y espacio. La RM puede ser
utilizada para demostrar la diseminación en tiempo por la aparición de lesiones nuevas en T2 o hipercaptantes
de al menos tres meses después de la RM inicial. Las anormalidades en el líquido cefalorraquídeo (LCR) que
confirman el diagnóstico de la EM son: la síntesis de "IgG intratecal" (incluyendo IgG oligoclonal la cual no está
presente en el suero) y un nivel elevado de IgG en LCR. Un estudio reciente demostró que la presencia de
anticuerpos en LCR dirigidos contra glucoproteínas de oligodendrocitos de la mielina y de la proteína ácida de
la mielina son factores predictivos para el desarrollo clínico definitivo de le EM en pacientes que han
presentado eventos desmielinizantes en el SNC. Los potenciales evocados visuales son útiles en el diagnóstico
ya que proveen evidencia electrofisiológica de lesiones ópticas previas en pacientes que han tenido un solo
episodio desmielinizante. LAS ANORMALIDADES DEL LÍQUIDO CEFALORRAQUÍDEO EN LA MIELOSIS MÚLTIPLE
INCLUYEN PLEOCITOSIS DE MONONUCLEARES Y MAYOR CONCENTRACIÓN DE IgG SINTETIZADA DE FORMA
INTRATECAL.

Bibliografía:
1. LONGO DL, FAUCI AS, KASPER DL, HAUSERSL, JAMESON JL, LOSCALZOJ. HARRISON. PRINCIPIOS DE
MEDICINA INTERNA, 18A EDICIÓN. MC GRAW HILL. NEW YORK, USA. 2012, PP 3399. 2. GUÍA DE PRÁCTICA
CLÍNICA, DIAGNÓSTICO Y TRATAMIENTO DE PACIENTES CON PRIMER BROTE Y SOSPECHA DE
ESCLEROSIS MÚLTIPLE. MÉXICO: SECRETARIA DE SALUD, 2008.

http://www.cenetec.salud.gob.mx/descargas/gpc/CatalogoMaestro/348_SSA_09_Brote_Esclerosis_Multiple/EYR_SSA_348_09.pdf

162 - SE CONFIRMA EL DIAGNÓSTICO DE SOSPECHA, PRESENTANDO ACTUALMENTE UN ATAQUE AGUDO


POR LO QUE DEBERÁ SER TRATADO POR MEDIO DE:

PLASMAFERESIS. DIAGNÓSTICO DIFERENCIAL. La posibilidad de una alternativa diagnóstica debe


considerarse particularmente cuando: 1) los síntomas son localizados exclusivamente en la
fosa posterior, unión cráneo cervical, o columna vertebral; 2) si el paciente es menor de 15
años o mayor de 60 años; 3) el curso es progresivo desde el inicio (primaria progresiva); 4)
si el paciente nunca ha experimentado alteraciones visuales o vesicales y; 5) cuando los
resultados de laboratorio son atípicos. El diagnóstico diferencial de la EM incluye
alteraciones que pueden ser multifocales en el SNC, como lesiones metastásicas; aquellas
enfermedades con notables alteraciones neurológicas que imitan clínicamente a la EM y
otras condiciones que causan disfunción lenta y progresiva de la columna cervical.
ACETATO DE CURSO Y PRONÓSTICO. El 85% de los pacientes cursa con periodos de remisión-
GLATIRAMER. exacerbación de la EM, caracterizada por ataques limitados o exacerbaciones de las
alteraciones neurológicas. Los pacientes se recuperan completa o parcialmente,
encontrándose neurológicamente estables entre cada episodio. Aproximadamente el 50%
de estos pacientes desarrollan EM secundariamente progresiva, en la cual existe un notable
y gradual deterioro neurológico entre cada episodio. Cerca del 15% de los pacientes inicia
con EM primariamente progresiva caracterizada por inicio insidioso o progresión continua y
gradual de los síntomas. Ocasionalmente presentan una meseta o fluctuaciones menores
pero sin ser remisiones.

INTERFERON. PRINCIPIOS DEL TRATAMIENTO. Los pacientes con EM enfrentan un pronóstico incierto y
deben ser informados sobre su enfermedad, esto se logra con un manejo multidisciplinario
que involucre a neurólogos y trabajadores de la salud relacionados, como enfermeras y
trabajadores sociales, con expertos en EM y diversas organizaciones de ayuda para
pacientes con EM. Se debe buscar apoyo psicosocial, ya que la depresión es común en los
pacientes, existiendo según un estudio incremento en el suicidio.

GLUCOCORTICOIDES. TRATAMIENTO DE LAS EXACERVACIONES AGUDAS DE ESCLEROSIS MÚLTIPLE. Las


exacerbaciones agudas de déficit neurológico que afectan las funciones del paciente o su
calidad de vida son generalmente tratadas con dosis altas de corticosteroides vía IV (1 g por
día en dosis única o dividida de metilprednisolona) por 3 ó 5 días. Un estudio sugiere una
ventaja de los corticosteroides IV sobre los orales en el tratamiento de la neuritis óptica. El
cambio de plasma ha demostrado cambios beneficios sobre las exacerbaciones
devastadoras que no responden a esteroides. EL TRATAMIENTO AGUDO DE LA
ENFERMEDAD SE BASA EN LA ADMINISTRACIÓN DE GLUCOCORTICOIDES.

Bibliografía:
1. GUÍA DE PRÁCTICA CLÍNICA, DIAGNÓSTICO Y TRATAMIENTO DE PACIENTES CON PRIMER BROTE Y
SOSPECHA DE ESCLEROSIS MÚLTIPLE. MÉXICO: SECRETARIA DE SALUD, 2008. 2. LONGO DL, FAUCI AS,
KASPER DL, HAUSERSL, JAMESON JL, LOSCALZOJ. HARRISON. PRINCIPIOS DE MEDICINA INTERNA, 18A
EDICIÓN. MC GRAW HILL. NEW YORK, USA. 2012, PP 3402.

http://www.cenetec.salud.gob.mx/descargas/gpc/CatalogoMaestro/348_SSA_09_Brote_Esclerosis_Multiple/EYR_SSA_348_09.pdf

FIN DEL CASO CLÍNICO SERIADO


ANÁLISIS DEL CASO CLÍNICO

IDENTIFICACIÓN DEL REACTIVO


Area: CIRUGÍA
Especialidad: URGENCIAS
Tema: TRAUMA CRANEOENCÉFALICO
Subtema: TRAUMA CRANEOENCEFÁLICO

CASO CLÍNICO CON UNA PREGUNTA

MUJER DE 23 AÑOS DE EDAD, SE ENCUENTRA HOSPITALIZADA DESDE HACE 2 DÍAS CON DIAGNÓSTICO DE
TRAUMA CRANEOENCEFÁLICO GRAVE. LA PACIENTE SE MANTIENE INTUBADA PERO DESDE HACE 24 HRS SE
INTERRUMPIERON LOS RELAJANTES MUSCULARES Y LA SEDACIÓN. A LA EXPLORACIÓN LA ENCUENTRA CON
PUPILAS MIÓTICAS Y ARREFLÉCTICAS.

23 años de edad.

-.

-.

intubada, sin relajación ni sedación. el


hallazgo en la paciente es la presencia de
pupilas mióticas y arre écticas.

no se re ere tac, aun que como recordarás


en todos los pacientes con tce grave debe
realizarse esta sin demora.

163 - LO MÁS PROBABLE ES QUE LA PACIENTE ESTE CURSANDO CON UNA:


HERNIA UNCAL. HERNIA UNCAL: Se produce al protruir el uncus del lóbulo temporal hacia medial. Su clínica
dependerá de la compresión de estructuras neurovasculares. • III NERVIO: En más de un 90%
de los casos el III nervio afectado es ipsilateral a la lesión, en un pequeño porcentaje es del
lado contralateral, en este caso, habitualmente se debe a un hematoma subdural. La
explicación clásica ha sido que el III nervio, luego de salir del troncoencéfalo es comprimido
por la hernia. Sin embargo, algunos autores atribuyen su aparición a isquemia mesencefálica.
La clínica del III nervio es progresiva, lo primero que aparece es una "MIDRIASIS REFLÉCTICA,
LUEGO ARREFLÉCTICA (PUPILA DE HUTCHINSON)" y por último un compromiso de todos los
músculos extraoculares dependientes de éste nervio. La explicación de esta progresión es la
anatomía del nervio. En un corte transversal, las fibras parasimpáticas, que provocan miosis,
se ubican en un fascículo periférico, por lo tanto, son las primeras en afectarse en una
compresión externa. • ESCOTADURA DE KERNOHA: La hernia uncal puede comprimir el
pedúnculo cerebral mesencefálico contralateral contra el borde libre de la tienda del cerebelo,
esto provoca HEMIPARESIA Y SIGNO DE BABINSKI IPSILATERAL A LA HERNIA. Su
fisiopatología es el compromiso de las fibras corticonucleares a nivel supratentorial. •
COMPRESIÓN VASCULAR: En esta zona existen 2 arterias susceptibles de ser comprimidas por
una hernia uncal: la arteria cerebral posterior y la arteria coroidea anterior. Su compresión
puede provocar infartos cerebrales en estos territorios. • HEMORRAGIAS DE DURET: Se
describen principalmente en el TCE grave. Corresponden a hemorragias lineares
mesencefálicas y pontinas altas. Su patogenia es desconocida, aparentemente se producen
por ruptura de venas o arterias luego de una deformación del troncoencéfalo debido a hernia
uncal. La mayoría son descripciones de autopsia, recientemente han sido descritas en
tomografía axial computada (TAC) y resonancia nuclear magnética (RNM). • Otros signos: La
compresión directa del mesencéfalo puede provocar alteraciones de la oculomotilidad y
RIGIDEZ DE DESCEREBRACIÓN O DECORTICACIÓN. La progresión puede continuar como un
síndrome de hernia transtentorial. Las alteraciones del patrón respiratorio son premonitores
de muerte. Debe considerarse que una vez aparecido alguno de los signos descritos de hernia
uncal, la progresión hacia la muerte puede ser muy rápida. La búsqueda de estos signos en
pacientes que potencialmente pueden desarrollarla debe ser obsesiva y su tratamiento precoz
y agresivo. EN LA HERNIA UNCAL ES COMÚN ENCONTRAR RIGIDEZ, Y SEGÚN LA
ESTRUCTURA LESIONADA SIGNO DE BABINSKI.

HERNIA HERNIA TRANSTENTORIAL: Se produce al protruir parénquima cerebral hacia la escotadura


TRANSTENTORIAL. de la tienda del cerebelo, se comprimen en forma progresiva: diencéfalo, mesencéfalo, puente
y bulbo raquídeo. La fenomenología va cambiando según la estructura comprometida,
siguiendo el patrón clásico de progresión rostro-caudal. Los signos más patognomónicos son
la respuesta motora, las alteraciones pupilares y la pérdida de los reflejos propios de cada
estructura. En el caso de los patrones respiratorios, clásicamente se les consideraba a cada uno
de ellos como patognomónico de una estructura comprometida, hoy se reconoce la ausencia
de especificidad. La excepción es la respiración apneúsica o de Biot, la que frecuentemente
indica una lesión bulbar e inminente paro respiratorio. - Diencéfalo. Datos de decorticación,
pupilas reactivas. - Mesencéfalo. Datos de descerebración, pupilas en posición media y fija. -
Puente. Sin patrón motor, "pupilas puntiformes". LA PACIENTE ESTA PRESENTANDO UNA
HERNIA TRANSTENTORIAL, CON COMPROMISO A NIVEL DEL PUENTE EN EL MOMENTO DE
LA EXPLORACIÓN. Bulboraquídeo, sin patrón motor, pupilas en posición media y fija,
respiración de Biot. IMPORTANTE: no existe patrón motor en estos casos, sólo alteraciones
pupilares.

HERNIA HERNIA CEREBELOSA ASCENDENTE: Es infrecuente, se observa en tumores de fosa posterior e


CEREBELOSA infartos cerebelosos con efecto de masa. Estas lesiones pueden producir compresión del
ASCENDENTE. acueducto de Silvio y IV ventrículo, provocando hidrocefalia triventricular, el drenaje
supraventricular de esta hidrocefalia, sin tener la precaución de descomprimir la fosa
posterior, puede producir una gradiente de presión transtentorial, favoreciendo la aparición
de esta hernia. Clínicamente se presenta con compromiso de conciencia cuantitativa y
alteración de la oculomotilidad, lo más frecuente es la paresia de mirada ascendente
(espontáneamente mira hacia abajo), o más característico, la paresia de mirada descendente
(espontáneamente mira hacia arriba). También se observa rigidez de descerebración y
decorticación. Puede provocar infarto de la arteria cerebral posterior. LA HERNIA
CEREBELOSA ASCENDENTE SE CARACTERIZA POR PRESENCIA DE RIGIDEZ Y CAMBIOS EN LA
OCULOMOTILIDAD, AMBOS NO PRESENTES EN LA PACIENTE.
HERNIA HERNIA CEREBELOSA DESCENDENTE: Es infrecuente pero mortal. No presenta clínica
CEREBELOSA premonitoria antes del paro respiratorio por compresión directa de las amígdalas cerebelosas
DESCENDENTE. sobre el bulbo raquídeo. La causa más frecuente es una masa cerebelosa que es
descompensada al realizar una punción lumbar (PL), debido a estos casos es que se
recomienda, salvo excepciones, realizar la PL después de tener imágenes del encéfalo. LA
HERNIA CEREBELOSA DESCENDENTE SE PRESENTA COMO UN PARO RESPIRATORIO SÚBITO.

Bibliografía:
1. COMITÉ DE TRAUMA DEL COLEGIO AMERICANO DE CIRUJANOS, PROGRAMA AVANZADO DE APOYO
VITAL EN TRAUMA PARA MÉDICOS. ATLS, 7A EDICIÓN. USA. PP 160.
ANÁLISIS DEL CASO CLÍNICO

IDENTIFICACIÓN DEL REACTIVO


Area: MEDICINA INTERNA
Especialidad: GASTROENTEROLOGÍA
Tema: ENFERMEDAD ÁCIDO PÉPTICA
Subtema: ENFERMEDAD POR REFLUJO GASTROESOFÁGICO

CASO CLÍNICO CON UNA PREGUNTA

HOMBRE DE 45 AÑOS DE EDAD, EJECUTIVO, REFIERE LA TOMA DIARIA DE AINES POR AUTOMEDICACIÓN
DESDE HACE VARIOS AÑOS. ALCOHOLISMO REFIERE DE TIPO SOCIAL. DESDE HACE 4 MESES PRESENTA
DOLOR EPIGÁSTRICO, DE MODERADA INTENSIDAD, TIPO URENTE, SIN IRRADIACIONES. A LA EXPLORACIÓN
FÍSICA ABDOMEN GLOBOSO A EXPENSAS DE PANÍCULO ADIPOSO CON DOLOR A LA PALPACIÓN PROFUNDA
EN EPIGASTRIO.

hombre de 45 años de edad.

TOMA AINES POR AUTOMEDICACIÓN


DESDE HACE VARIOS AÑOS.
ALCOHOLISMO SOCIAL

dolor en epigastrio de 4 meses de


evolución, moderado, urente.

DOLOR EN EPIGASTRIO A LA PALPACIÓN


PROFUNDA.

-.

164 - EL ESTUDIO MÁS ÚTIL PARA LA CONFIRMACIÓN DEL DIAGNÓSTICO SERÍA:

SERIE Los ESTUDIOS CONTRASTADOS del tubo digestivo alto se utilizan con frecuencia
ESOFAGOGASTRODUODENAL aun sin embargo su sensibilidad y la especificidad de la esofagografía son muy
bajas comparadas con la endoscopía. Actualmente, el esofagograma NO se
considera una prueba diagnóstica para la ERGE. En caso de realizarse, cuando se
detecta alguna alteración en este estudio se debe continuar con una endoscopía. La
serie esofagogastroduodenal se recomienda como parte del estudio preoperatorio
de la ERGE para detallar la anatomía y función de la zona. LOS ESTUDIOS
CONTRASTADOS NO SE RECOMIENDAN EN LA ACTUALIDAD COMO PRUEBA
DIAGNÓSTICA PARA LA ERGE.
ULTRASONIDO ABDOMINAL En términos generales el ultrasonido no es un estudio adecuado para valorar
vísceras huecas. REPASO: La ENFERMEDAD POR REFLUJO GASTROESOFÁGICO se
considera como la condición que aparece cando el reflujo del contenido del
estómago produce síntomas molestos y/o complicaciones. La enfermedad por
reflujo se manifiesta de diferentes formas: esofagitis erosiva, esofagitis no erosiva,
pirosis funcional.

GAMMAGRAFÍA No está indicado la gammagrafía, además es un estudio muy caro y con muchas
limitaciones, no es el de elección. REPASO: Las principales manifestaciones
asociadas son: - Síntomas digestivos: regurgitación, pirosis, vómito, disfagia,
odinofagia, hematemesis, dispepsia, trastornos nutricionales, disminución
ponderal. - Síntomas extraesofágicos: apena, cianosis sibilancias, estridor, tos
crónica, asma, bronquiolitis, neumonía, laringotraqueitis, Crup, trastornos de
fonación, otitis, dolor torácico, escurrimiento nasal posterior, deterioro del esmalte
dentario.

ENDOSCOPÍA La ENDOSCOPIA es el método más "sensible" y "específico" para el estudio de


enfermedades del tubo digestivo superior, ya que permite la visualización directa
de la mucosa, facilita la documentación fotográfica de los defectos mucosos y
permite hacer biopsias de los tejidos para descartar lesiones o infección por H.
pylori. El examen endoscópico es especialmente útil para identificar lesiones
demasiado pequeñas para ser detectadas en la exploración radiológica, para
estudiar alteraciones radiológicas atípicas o para determinar si una úlcera es el
origen de una hemorragia. Está indicada de inmediato en todo paciente con ERGE
complicado o con síntomas de alarma como disfagia, vómito persistente,
hemorragia gastrointestinal, anemia ferropénica, pérdida de peso no intencionada
o tumoración epigástrica. RECUERDA: la endoscopía permite la visualización
directa de la mucosa y la toma de biopsias en caso de encontrar alteraciones.
DEBIDO A QUE TIENEN MAYOR ESPECIFICIDAD Y SENSIBILIDAD QUE OTROS
ESTUDIOS, ADEMÁS DE QUE PERMITE LA VISUALIZACIÓN DIRECTA Y TOMA DE
BIOPSIAS, LA ENDOSCOPÍA ES EL ESTUDIO DE ELECCIÓN PARA LA ERGE. NOTA: si
bien, los lineamientos actuales (GPC) para el reflujo gastroesofágico en el adulto
no refieren explícitamente que la endoscopía sea el método diagnóstico de
elección, si lo indica como el método de primera línea tras la prueba terapéutica
fallida. El uso de pHmetría con impedancia se reserva para todos los casos que, tras
la endoscopía, se diagnostiquen con esofagitis no erosiva y, para aquellos con
esofagitis erosiva que no respondan al tratamiento médico aún cuando se utilicen
diferentes tipos de fármacos. Por su parte, la manometría se utiliza como último
recurso diagnóstico con el fin de realizar diagnóstico diferencial.

Bibliografía:
1.DIAGNÓSTICO Y TRATAMIENTO DE LA ENFERMEDAD POR REFLUJO GASTROESOFÁGICO EN EL ADULTO.
EVIDENCIAS Y RECOMENDACIONES; GUÍA DE PRÁCTICA CLÍNICA. MÉXICO, CENETEC; 2018. 2.
DIAGNÓSTICO Y TRATAMIENTO QUIRÚRGICO DE LA ENFERMEDAD POR REFLUJO GASTROESOFÁGICO EN
EL ADULTO. GUÍA DE PRÁCTICA CLÍNICA. MÉXICO: SECRETARIA DE SALUD, NOVIEMRE 2012.

http://www.cenetec-difusion.com/CMGPC/SS-012-08/ER.pdf
ANÁLISIS DEL CASO CLÍNICO

IDENTIFICACIÓN DEL REACTIVO


Area: CIRUGÍA
Especialidad: TRAUMATOLOGÍA Y ORTOPEDIA
Tema: CERVICALGIAS, LUMBALGIAS, TRAUMA MIEMBROS SUP Y TUMORES
Subtema: LUMBALGIA

CASO CLÍNICO CON UNA PREGUNTA

HOMBRE DE 48 AÑOS DE EDAD, ALBAÑIL DESDE LOS 13 AÑOS. COMIENZA SU PADECIMIENTO HACE 1 AÑO
CON LUMBALGIA DE ESFUERZO LA CUAL HA AUMENTADO PAULATINAMENTE HASTA LLEGAR A SER
INCAPACITANTE. ACTUALMENTE CON DOLOR LANCINANTE IRRADIADO A MIEMBROS INFERIORES QUE NO
MEJORA CON ANALGÉSICOS, REFIERE PIE DERECHO “CAÍDO” DESDE HACE DOS SEMANAS CON
DISMINUCIÓN DE LA SENSIBILIDAD.

MASCULINO 48 AÑOS.

riesgo laboral que implica el levantamiento


de objetos pesados.

lumbalgia crónica que no mejora con


analgésicos, dé cit neurológico (pie caído,
cambios en la sensibilidad).

165 - EN CASO DE REQUERIR TRATAMIENTO QUIRÚRGICO ESTARÍA INDICADO REALIZAR UNA:

FUSIÓN RAQUÍDEA LO PRIMERO ES INTEGRAR EL DIAGNÓSTICO CLÍNICO: SE TRATA DE UNA LUMBALGIA


CRÓNICA SECUNDARIA A HERNIA DISCAL. La FUSIÓN RAQUÍDEA es una técnica quirúrgica
que se utiliza en aquellos pacientes con INESTABILIDAD RAQUÍDEA originada por
enfermedad, intervención quirúrgica o ambas. Esta técnica consiste en fijar las vértebras
adyacentes una con la otra. La fusión se logra cuando el cuerpo vertebral forma una masa
sólida de hueso que incorpora las vértebras adyacentes y elimina el movimiento vertebral
normal. ALGUNAS VECES PUEDE RECOMENDARSE LA FUSIÓN CUANDO LAS
HERNIACIONES DE DISCO SON RECURRENTES EN EL MISMO NIVEL O HAY DOLOR
SECUNDARIO A LA INESTABILIDAD ARTICULAR.
INSTRUMENTACIÓN La INSTRUMENTACIÓN RAQUÍDEA se refiere al uso de dispositivos de fijación interna para
RAQUÍDEA la INMOVILIZACIÓN RAQUÍDEA SEGMENTARIA en todos los niveles de la columna
vertebral. Para este fin se utilizan barras y placas que se fijan mediante ganchos tornillos o
alambres a la columna vertebral. La idea es lograr una FUSIÓN ÓSEA y dar ESTABILIDAD A
LA COLUMNA. Todos ellos al final fallan porque se aflojan y rompen si no se produce la
fusión ósea.

ARTRODESIS La ARTRODESIS consiste en la eliminación del movimiento o de la inestabilidad mediante la


incorporación de componentes relevantes en una masa sólida de hueso. Está indicada en
cualquier SEGMENTO FUSIONADO para que tenga estabilidad. Por lo tanto, SI NO HAY
ARTRODESIS NO SE LOGRA UNA FUSIÓN VERTEBRAL. Para que se produzca la artrodesis es
necesario que los osteoblastos formen hueso nuevo a través de la anomalía inestable. La
colocación de autoinjertos sirve como puentes para estos osteoblastos facilitando así la
artrodesis.

MICRODISCECTOMÍA. LA MICRODISCECTOMÍA ES LA INTERVENCIÓN QUIRÚRGICA ESTÁNDAR PARA PACIENTES


CON DISCOS LUMBARES HERNIARIOS. Incluye una laminotomía para obtener acceso al
espacio discal. La raíz nerviosa y el saco tecal se protegen, mientras el disco fragmentado
se identifica y remueve. El tratamiento quirúrgico de la HERNIA DISCAL está indicado
cuando los síntomas persisten más de 6 a 8 semanas, hay una deficiencia motora
progresiva o personas con dolor incapacitante que no mejora con analgésicos.

Bibliografía:
1. BRUNICARDI F, ANDERSEN D, BILLIAR T, Y COLS. SCHWARTZ PRINCIPIOS DE CIRUGÍA, 9A EDICIÓN. MC
GRAW HILL. 2011, PP 1546-1548.
ANÁLISIS DEL CASO CLÍNICO

IDENTIFICACIÓN DEL REACTIVO


Area: GINECOLOGÍA Y OBSTETRICIA
Especialidad: GINECOLOGÍA
Tema: PATOLOGÍA BENIGNA Y MALIGNA DE OVARIO
Subtema: PATOLOGÍA BENIGNA DE OVARIO

CASO CLÍNICO SERIADO

MUJER DE 21 AÑOS DE EDAD, QUE ACUDE A CONSULTA POR PRESENTAR ALTERACIONES MENSTRUALES CON
EVIDENCIA DE ANOVULACIÓN. A LA EXPLORACIÓN SE ENCUENTRA PESO DE 70 KILOGRAMOS, ACNÉ FACIAL E
HIRSUTISMO ABDOMINAL. MENCIONA QUE SU PRINCIPAL DESEO ES EMBARAZARSE.

Mujer en edad reproductiva.

el elemento clave del caso cuando


decidimos el tratamiento es el deseo de la
paciente de embarazarse y contar con
alteraciones menstruales y anovulación.

alteraciones menstruales, sobrepeso, acnÉ


e hirsutismo, síndrome de ovarios
poliquísticos como primera opción de
inmediato.

-.

-.

166 - PARA CONFIRMAR EL DIAGNÓSTICO USTED SOLICITARÁ:

TOMOGRAFÍA LO PRIMERO QUE DEBERÁS HACER EN ESTE TIPO DE CASOS ES ESTABLECER LA SOSPECHA
DE ABDOMEN. DIAGNÓSTICA, QUE EN ESTE CASO SE TRATA DE UN HIPERANDROGENISMO, PROBABLEMENTE
RELACIONADO CON SÍNDROME DE OVARIO POLIQUÍSTICO (SOP). El estudio de imagen de
elección para el estudio de las SUPRARRENALES es la TOMOGRAÍA AXIAL COMPUTARIZADA de
alta resolución. No debe indicarse si no se ha documentado en primera instancia la presencia de
anormalidades en los tests endocrinos para evitar el riesgo de encontrarnos con incidentalomas,
ya que hasta el 4.36% de la población pudiera presentarlos sin que esto represente un riesgo para
su salud. La presencia de un tumor adrenal mayor de 3 cm sugiere malignidad. SU USO SÓLO SE
JUSTIFICA CUANDO SE SOSPECHA PATOLOGÍA SUPRARRENAL, NO PARA APOYAR EL
DIAGNÓSTICO DE SÍNDROME DE OVARIO POLIQUÍSTICO.
ULTRASONIDO La ULTRASONOGRAFÍA (USG) pélvica es el método de elección para la evaluación inicial de los
PÉLVICO. anexos. Característicamente se trata de folículos de 8 mm con aumento del estroma central,
aunque con la técnica vaginal pueden detectarse quistes de 3-5 mm. Los quistes ováricos
aparecen irregularmente en las pacientes con síndrome de ovario poliquístico encontrándose
diferencias significativas de acuerdo a las series estudiadas (25-80%). Resulta sorprendente
observar que los estudios con ecografía han demostrado que al menos 23% de las mujeres
jóvenes tienen ovarios con morfología poliquística, si bien muchas de ellas carecen de otros
síntomas de síndrome de ovario poliquístico. Además, los ovarios a menudo tienen aspecto
poliquístico en otras enfermedades que se acompañan de andrógenos excesivos como hiperplasia
suprarrenal congénita, síndrome de Cushing y uso de andrógenos exógenos. LA MORFOLOGÍA
POLIQUÍSTICA NO ESTABLECE POR SÍ SOLA EL DIAGNÓSTICO, PUES ESTE ASPECTO PUEDE
ACOMPAÑAR A OTRO TIPO DE ENFERMEDADES.

BIOPSIA Se recomienda tomar una BIOPSIA ENDOMETRIAL en las mujeres mayores de 35 años de edad
ENDOMETRIAL. con HEMORRAGIA ANORMAL y en las más jóvenes con hemorragia anovulatoria resistente al
tratamiento hormonal. NO TIENEN ANTECEDENTES QUE JUSTIFIQUEN UNA BIOPSIA A ESTA
EDAD.

NIVELES Al momento de enfrentar un caso de HIPERANDROGENISO nuestra obligación más importante es


SÉRICOS DE descartar la presencia de algunas patologías, que si bien no son frecuentes, requieren de
HORMONAS. abordajes particulares y tratamientos específicos. En este grupo incluimos a los tumores benignos
y malignos de las adrenales y el ovario, así como los casos de hiperplasia suprarrenal congénita
en su variedad no clásica, entre otras. El SÍNDROME DE OVARIO POLIQUÍSTICO (SOP) es un
diagnóstico de exclusión. Por consiguiente es necesario excluir de manera sistemática otros
trastornos potencialmente graves con manifestaciones clínicas del SOP. Por esa razón se deberán
solicitar siempre: • Tirotropina. Descartar hipo o hipertiroidismo. • Prolactina. Descartar
hiperprolactinemia que provoca anovulación al inhibir la secreción pulsátil de GnRH del
hipotálamo. • Testosterona. Específicamente, en una mujer en la que aparecen o empeoran los
signos de virilización de manera repentina en los últimos meses, se debe contemplar la
posibilidad de un tumor ovárico o suprarrenal productor de hormonas. • Sulfato de
Dehidroepiandrosterona. Este es producido casi exclusivamente en la glándula suprarrenal. Por lo
tanto una concentración elevada es altamente sugestiva de neoplasia suprarrenal. •
Gonadotropinas. Durante la valoración de la amenorrea, se mide la concentración de FSH y LH
para excluir una insuficiencia ovárica precoz e hipogonadismo hipogonadotrópico. • 17-
hidroxiprogesterona. El término hiperplasia suprarrenal congénita describe a varios trastornos
autosómicos recesivos que son el resultado de la deficiencia completa o parcial de una enzima
que participa en la síntesis de cortisol y aldosterona. Produce datos de hiperandrogenismo. •
Cortisol. Para descartar síndrome de Cushing, que causa datos de hiperandrogenismo. Todos
estos estudios permitirán descartar las patologías con las que debe hacerse diagnóstico
diferencial obligadamente. A manera de escrutinio se solicitará: Determinación en ayunas y
durante los primeros 7 días del ciclo de: a) LH y FSH en pool (el promedio de 3 muestras
recolectadas con 20 minutos de diferencia) y Prolactina. b) Testosterona total y libre. c)
Dehidroepiandrosterona sulfato (DHEAS) y 17 alfa-hidroxiprogesterona (17-HPO). En el 66% de
los pacientes se documenta una relación LH: FSH (mayor a 2.5:1), la cual es característica de SOP.
Actualmente para hacer diagnóstico, las pacientes deben llenar dos de los tres siguientes
criterios: 1) Oligoovulación, anovulación o ambas, 2) Hiperandrogenismo (clínico, bioquímico o
ambos) y 3) Ovario poliquístico en la ecografía. Por consiguiente se comenta nuevamente que el
SOP es un diagnóstico de exclusión. DADO QUE EN PRESENCIA DE OVARIOS POLIQUÍSTICO
DEBERÁN EXCLUIRSE OTRAS ENFERMEDADES ASOCIADAS A ÉSTOS, ES INDISPENSABLE PARA
“CONFIRMAR” EL DIAGNÓSTICO LA VALORACIÓN DE NIVELES HORMONALES. IMPORTANTE:
existen diversos criterios clínicos para la sospecha del SOP, en la GPC se describen tres, de las
cuales todas incluyen la exclusión de otros trastornos (lo cual se logra con los niveles séricos de
hormonas) y uno de ellos excluye el hallazgo ultrasonográfico. Esto le da mayor peso al estudio
hormonal.

Bibliografía:
1. SCHONGUE J, SCHAFER J, HALVORSON L, HOFFMAN B, BRADSHAW K, CUNNINGHAM G. WILLIAMS
GINECOLOGÍA, DE LA 1A EDICIÓN EN INGLÉS. MC GRAW HILL. USA. 2009, PP 391-393.

167 - EN CASO DE CONFIRMAR SU DIAGNÓSTICO EL TRATAMIENTO SERÍA CON:


METFORMINA. IMPORTANTE, SI VUELVES AL CASO CLÍNICO, EL MOTIVO DE CONSULTA ES PORQUE DESEA
EMBARAZARSE, DE AHÍ SE DESPRENDE EL TRATAMIENTO ADECUADO EN ESTE MOMENTO. La
METFORMINA aumenta la SENSIBIILIDAD PERIFÉRICA A LA INSULINA al reducir la producción
hepática de glucosa y aumentar la sensibilidad de los tejidos destinatarios a la insulina. Reduce
los andrógenos en mujeres tanto delgadas como obesas, con lo que aumenta el índice de
ovulación espontánea. En varios estudios se ha demostrado que hasta 40% de las mujeres con
anovulación y síndrome de ovario poliquístico logra ovular y muchas de ellas se embarazan con
metformina como único tratamiento, sin embargo su uso con este fin aún no ha sido
autorizado. Se utiliza en ocasiones combinado con citrato de clomifeno. FORMA PARTE DEL
MANEJO DEL SOP, PERO NO ESTÁ INDICADO PARA INDUCIR LA OVULACIÓN.

ACETATO DE Los ANTIANDRÓGENOS son inhibidores competitivos de la fijación de los andrógenos a sus
CIPROTERONA. receptores. Si bien estos fármacos son efectivos en el tratamiento del HIRSUTISMO, conllevan
ciertos riesgos por diversos efectos colaterales. A menudo las pacientes padecen metrorragias.
En México y EUA, el antiandrógeno preferido es éste. COMO PODRÁS LEER, EL OBJETIVO DE
INDICAR ESTE MEDICAMENTO ES PARA TRATAR EL HIRSUTISMO.

CITRATO DE La elección del tratamiento para cada síntoma del Síndrome de Ovario Poliquístico depende de
CLOMIFENO. los objetivos de la mujer y de la magnitud de la disfunción endocrina. Así las mujeres con
anovulación que desean un embarazo reciben un tratamiento completamente distinto que el de
las adolescentes con irregularidades menstruales y acné. En este caso estaría indicado:
Tratamiento de la anovulación. El citrato de clomifeno, solo o en combinación con la pérdida de
peso, sigue siendo, dada su seguridad y simplicidad, el tratamiento de primera elección en la
infertilidad de origen anovulatorio asociado al SOP. Actúa predominantemente como un
antiestrógeno uniéndose a los receptores hipotalámicos, lo cual disminuye el mecanismo de
retroalimentación negativa ejercida por los estrógenos endógenos. Esto aumenta la secreción de
GnRH, la que a su vez estimula la secreción de LH y FSH. El tratamiento con citrato de clomifeno
comienza durante la fase proliferativa precoz (del 2do al 5to día del ciclo menstrual) con la
administración de 50 mg/ día durante 5 días. Si la anovulación persiste, la dosis puede ser
incrementada de 50 en 50 mg al día hasta llegar a una dosis máxima de 250 mg/día y al
contrario que las gonadotrofinas exógenas, el aumento en la dosis de clomifeno no suele
asociarse con un aumento del reclutamiento folicular. Como la finalidad del tratamiento es la
inducción de la ovulación y el embarazo, la falta de respuesta terapéutica a pesar de las dosis
máximas hace necesaria la adición de otros fármacos. ES EL TRATAMIENTO DE ELECCIÓN PARA
PACIENTES CON ESTERILIDAD POR CICLOS ANOVULATORIOS, SE PUEDE ADMINISTRAR EN
COMBINACIÓN CON METFORMINA, SE TIENE MEJOR RESPUESTA EN LOGRAR NACIDOS VIVOS
CON CLOMIFENO SOLO QUE CON METFORMINA SOLA.

ISOTRETINOÍNA. La ISOTRETINOÍNA es un análogo de la vitamina A altamente efectivo para el ACNÉ resistente al


tratamiento. A pesar de su eficacia, es teratógeno durante el primer trimestre del embarazo. Las
malformaciones se ubican en el cráneo, cara corazón, SNC y timo. DADA SU EFECTO
TERATÓGENO ESTÁ CONTRAINDICADO EN MUJERES QUE BUSCAN EMBARAZARSE EN ESTE
MOMENTO.

Bibliografía:
1. SCHONGUE J, SCHAFER J, HALVORSON L, HOFFMAN B, BRADSHAW K, CUNNINGHAM G. WILLIAMS
GINECOLOGÍA, DE LA 1A EDICIÓN EN INGLÉS. MC GRAW HILL. USA. 2009, PP 396,450.

FIN DEL CASO CLÍNICO SERIADO


ANÁLISIS DEL CASO CLÍNICO

IDENTIFICACIÓN DEL REACTIVO


Area: PEDIATRÍA
Especialidad: INFECTOLOGIA PEDIÁTRICA
Tema: ARTRITIS SEPTICA Y OSTEOMIELITIS
Subtema: OSTEOMIELITIS

CASO CLÍNICO CON UNA PREGUNTA

PREESCOLAR DE 3 AÑOS DE EDAD, EL CUÁL PRESENTO CUADRO DE ABDOMEN AGUDO SECUNDARIO A


APENDICITIS PERFORADA DE VARIOS DÍAS DE EVOLUCIÓN QUE REQUIRIÓ RESOLUCIÓN QUIRÚRGICA. UNA
SEMANA POSTERIOR ACUDE A LA CONSULTA EXTERNA DONDE LE DIAGNOSTICAN OSTEOMIELITIS DE FÉMUR
DERECHO.

preescolar de 3 años.

Es muy importante el antecedente de


apendicitis complicada con hospitalización.

Osteomielitis de fémur derecho.

168 - EN ESTA PACIENTE LA VÍA DE DISEMINACIÓN QUE FAVORECIÓ LA OSTEOMIELITIS FUE:

LINFÁTICA. Etiología y factores predisponentes. —Aproximadamente el 75 % de todos los casos son


causados por Staphylococus aureus hemolítico. El patógeno que le sigue en frecuencia es el
Streptococus hemolítico, grupo A, y en casos más raros, por otras especies de
estreptococos, neumococos, gonococos, colibacilos, Hemofilus influenzae, etc. Este tipo de
diseminación requiere siempre una relación anatómica. Cuello y cara es un ejemplo.
LINFOHEMATÓGENA. Manifestaciones clínicas: I. — En lactantes: A) De instalación generalmente súbita con
síntomas sistémicos alarmantes de toxicidad. B) Ocasionalmente de instalación insidiosa,
caracterizada únicamente por irritabilidad y seguida usualmente en pocas horas por fiebre
y hallazgos loca les significativos. C) Tiende a la inmovilidad de la extremidad afectada y
articulaciones adyacentes. D) Generalmente hay dolor antes de que aparezcan signos de
inflamación. E) El rubor y el edema usualmente indican presencia de absceso extracortical.
F) Por lo general no se ven cambios radiológicos de importancia antes de la primera
semana. II. — Niños mayores de dos años: A) Por lo general manifestaciones iniciales
menos dramáticas que en el lactante. B) Fiebre con escalofríos. C) Dolor severo, constante,
pulsante en la región afectada y algunas veces signos de inflamación. D) Limitación de
movimientos articulares, en menor grado que en el lactante. E) Por lo general no se ven
cambios radiológicos de importancia antes de los 10 a 14 días.

HEMATÓGENA. Clasificación de la OSTEOMIELITIS. Puede clasificarse de acuerdo al mecanismo de


introducción de la infección o bien basándose en su etiología microbiana. I. De acuerdo al
mecanismo de introducción. A) Osteomielitis primaria: Es causada por implantación directa
de los microbios en el hueso. • Heridas penetrantes. • Fracturas compuestas o abiertas. •
Intervención quirúrgica ósea. • Infusiones o inyecciones intramedulares. B) Osteomielitis
secundaria: La vía de infección es usualmente, a través del sistema arterial; una excepción
notable es la diseminación del sistema genitourinario, a través de las venas a los huesos
pélvicos o a la columna vertebral. La infección, a través del sistema linfático es rara.
Ocasionalmente resulta por extensión directa de una infección de tejidos blandos o
articulaciones contiguas. DADO EL FOCO INFECCIOSO POR EL CUAL SE ENCONTRÓ
HOSPITALIZADO EL PACIENTE INICIALMENTE, NO EXISTE RELACIÓN ANATÓMICA DE
CONTIGÜIDAD NI LINFÁTICA, DESCARTÁNDOSE ABSOLUTAMENTE ESTAS OPCIONES DE
RESPUESTA. La OSTEOMIELITIS HEMATÓGENA es la más frecuente y la que se acompaña de
infección y destrucción tisular más extensas.

CONTIGUA. Los microorganismos suelen llegar al hueso por vía hematógena, pero puede ocurrir
invasión por propagación directa de una infección adyacente en tejidos blandos, o por
contaminación del hueso cuando alguna lesión de tejidos blandos deja al descubierto los
huesos subyacentes; por ejemplo: fracturas compuestas, desgarros traumáticos extensos y
heridas por bala.

Bibliografía:
DIAGNÓSTICO Y TRATAMIENTO DE ENFERMEDADES INFECCIOSAS. JOHN P. CLOHERTY. MASSON.
EDICIÓN 4A. 2005. PÁG. 330.
ANÁLISIS DEL CASO CLÍNICO

IDENTIFICACIÓN DEL REACTIVO


Area: MEDICINA INTERNA
Especialidad: GASTROENTEROLOGÍA
Tema: ENFERMEDAD ÁCIDO PÉPTICA
Subtema: ULCERA PÉPTICA

CASO CLÍNICO CON UNA PREGUNTA

HOMBRE DE 38 AÑOS DE EDAD CON DIAGNÓSTICO DE GASTRITIS CRÓNICA, HACE 6 MESES SE LE INDICA UN
INHIBIDOR DE LOS RECEPTORES H2 POR 15 DÍAS EL CUAL SE AUTOMEDICA DE FORMA PERMANENTE HASTA
EL DÍA DE HOY. ACUDE A CONSULTA AL NOTAR CRECIMIENTO DE AMBAS MAMAS Y DISMINUCIÓN DE LA
LÍBIDO.

36 años.

Consumo prolongado de un inhibidor de


los receptores de H2

crecimiento mamario y disminución de la


líbido.

-.

-.

169 - EL MEDICAMENTO QUE MÁS PROBABLEMENTE ESTUVO TOMANDO EL PACIENTE ES:

RANITIDINA Los antagonistas de los receptores H2 se consideran eficaces para el tratamiento de la enfermedad
ácido péptica sin embargo han sido desplazados por los inhibidores de la bomba de protones,
quedando estos últimos como el tratamiento de elección. Se dispone de 4 antagonistas de los
receptores H2: cimetidina, ranitidina, famotidina y nizatidina. Todos ellos con una mayor eficacia
para inhibir la producción gástrica nocturna.

CIMETIDINA La CIMETIDINA fue el primer inhibidor de los receptores H2 para el tratamiento de la úlcera
péptica; se utiliza poco debido a sus efectos adversos, inhibe el metabolismo hepático del
citocromo P450 e incrementa la concentración sérica de teofilina, warfarina, lidocaína y fenitoína.
Puede tener efectos ANTIANDROGÉNICOS leves que causan ginecomastia e impotencia sobre todo
en pacientes con tratamientos prolongados o a altas dosis. Otros efectos adversos relacionados con
esta sal son la elevación en los valores séricos de las aminotransferasas, creatinina y prolactina.
FAMOTIDINA Los efectos adversos más frecuentes de los inhibidores de los receptores H2 son: pancitopenia,
neutropenia, anemia y trombocitopenia. Con excepción de la nizatidina todos se unen al citocromo
450 del hígado.

OMEPRAZOL El Omeprazol es un inhibidor de la bomba de protones, se une de forma covalente a ésta


inhibiéndola de manera irreversible. EL OMEPRAZOL NO CORRESPONDE AL GRUPO DE LOS
INHIBIDORES DE LOS RECEPTORES H2.

Bibliografía:
1. LONGO DL, FAUCI AS, KASPER DL, HAUSERSL, JAMESON JL, LOSCALZOJ. HARRISON. PRINCIPIOS DE
MEDICINA INTERNA, 18A EDICIÓN. MC GRAW HILL. NEW YORK, USA. 2012. 2. PAPADAKIS MAXINE A,
MCPHEE STEPHEN J. DIAGNÓSTICO CLÍNICO Y TRATAMIENTO. 52ª EDICIÓN. MC GRAW HILL EDUCATION,
LANGE. USA. 2013.
ANÁLISIS DEL CASO CLÍNICO

IDENTIFICACIÓN DEL REACTIVO


Area: GINECOLOGÍA Y OBSTETRICIA
Especialidad: GINECOLOGÍA
Tema: LEUCORREA
Subtema: CANDIDIASIS VAGINAL

CASO CLÍNICO SERIADO

MUJER DE 48 AÑOS DE EDAD, CON ANTECEDENTE DE CÉRVICO VAGINITIS DE REPETICIÓN Y QUE TOMA
TRATAMIENTO DE REEMPLAZO HORMONAL. ACUDE A CONSULTA PORQUE DESDE HACE 2 MESES PRESENTA
PRURITO INTENSO VULVAR Y LEUCORREA GRUMOSA, REFIERE COMO ANTECEDENTE HABER TOMADO UN
TRATAMIENTO DE CATORCE DÍAS CON METRONIDAZOL. A LA EXPLORACIÓN FÍSICA SE OBSERVA PIEL VULVAR
ENGROSADA, EDEMATOSA Y SECA.

Mujer de 48 años de edad.

Antecedente de cérvico vaginitis de


repetición, tratamiento previo con
metronidazol por 14 días.

Hace 2 meses presenta prurito intenso


vulvar y leucorrea grumosa.

Piel vulvar engrosada, edematosa y seca.

-.

170 - EL DIAGNÓSTICO CLÍNICO MÁS PROBABLE ES:


LIQUEN El LIQUEN PLANO EROSIVO es una lesión que afecta mucosa gingival y vulvovaginal. Ocurre en el
PLANO 45 % de los casos en mujeres menores de 50 años y en el 70 % de los casos en mayores de 60 años.
EROSIVO. Es muy doloroso. Las relaciones sexuales cuando son posibles culminan con sangrado. La
dispareunia es severa. En algunos casos se presenta como erosiones rojas con bordes blancos o
grisáceos en la zona vestibular. Se puede observar una vaginitis descamativa, vagina con erosiones,
atrofia, flujo maloliente y finaliza con la aparición de sinequias (Adherencias) y cicatrices. Síntomas:
Dolor, dispareunia severa, a veces con imposibilidad de mantener relaciones sexuales. Incapacidad
que lleva a depresión y angustia. Lamentablemente esta es una condición crónica, destructiva y de
muy difícil manejo. Es una enfermedad crónica, recalcitrante que produce dolor y prurito
controlado parcialmente por el tratamiento con corticoides. Cursa con exacerbaciones, lenta
cicatrización e infecciones secundarias recurrentes. Diagnóstico: Clínico observando las lesiones en
boca y piel. Biopsia e inmunofluorescencia ( a veces las biopsias pueden no ser específicas) LA
EVOLUCIÓN DEL LIQUEN PLANO EROSIVO ES CRÓNICA Y DE APARICIÓN INSIDIOSA.

CLIMATERIO. OJO: QUEDA CLARO QUE LA PACIENTE SE ENCUENTRA EN LA FASE DE CLIMATERIO DADO QUE
ESTÁ EN TRATAMIENTO CON RESTITUCIÓN HORMONAL. Cuando la paciente se encuentra en etapa
de climaterio, hay disminución de la lubricación vaginal, presentando incluso atrofia genitourinaria.
Si bien la paciente pudiera estar cursando con el climaterio y los hallazgos clínicos compatibles
también con esta etapa, es claro que la paciente presenta, además un proceso infeccioso
caracterizado por la secreción y la sintomatología acompañante. LOS CAMBIOS ATRÓFICOS DE LA
VAGINA SON CLÁSICOS EN LA ETAPA CLIMATÉRICA Y EN MUCHAS OCASIONES FAVORECEN LA
APARICIÓN DE INFECCIONES COMO LA CANDIDIASIS. OJO: - LA ATROFIA VAGINAL NO JUSTIFICA
LA PRESENCIA DE LEUCORREA. - DADO QUE LA PACIENTE ESTA EN TRATAMIENTO HORMONAL SE
ESPERARÍA QUE NO TUVIERA SÍNTOMAS ASOCIADOS A LA ATROFIA VAGINAL.

VULVITIS TE ENCUENTRAS ANTE UN CUADRO CLÁSICO DE CANDIDIASIS. La paciente tiene un cuadro


INFECCIOSA. infeccioso provocado por Cándida, por el tipo las características de la leucorrea la cual es grumosa,
este hongo provoca prurito y esto origina engrosamiento de la piel y sequedad. - Los "síntomas"
usualmente observados en pacientes con Candidiasis Vulvovaginal (CVV) son: escurrimiento vaginal,
prurito vulvar, dolor vulvar, dispareunia y disuria. - Los "signos" usualmente observados en
pacientes con CVV son: eritema vulvar, edema vulvar, fisuras vulvovaginales, escurrimiento vaginal
blanco y grumoso, y lesiones satélites (máculo-papulares) e ídes.

NEOPLASIA El CÁNCER VULVAR es sobre todo una enfermedad de ancianas, pero se ha observado también en
VULVAR. mujeres premenopáusicas. Estos cánceres son poco comunes y comprenden cerca de 5 % de todas
las neoplasias malignas ginecológicas. Por lo general, los tumores vulvares causan síntomas de
prurito e irritación. Es probable que las mujeres al principio ignoren síntomas menores, lo que
contribuye a retrasar el diagnóstico. En el 50 % de los casos las neoplasias intraepiteliales vulvares
son asintomáticas, mientras que en las pacientes restantes, suele acompañarse de picazón,
sensación de quemado, dolor y la presencia de una masa palpable en la vulva, la cual no presenta la
paciente, pudiéndose descartar esta posibilidad diagnóstica. NO CORRESPONDE A LA EDAD MÁS
FRECUENTE DE APARICIÓN. RECUERDA SIEMPRE BUSCAR EL DIAGNÓSTICO MÁS FRECUENTE Y
TENER EN CUENTA ESTAS POSIBILIDADES SÓLO COMO SECUNDARIAS.

Bibliografía:
1. GUÍA DE REFERENCIA RÁPIDA, DIAGNÓSTICO Y TRATAMIENTO DE CANDIDOSIS VULVOVAGINAL EN
MUJERES MAYORES A 12 AÑOS DE EDAD. MÉXICO: SECRETARIA DE SALUD; 2010. 2. SCHONGUE J, SCHAFER
J, HALVORSON L, HOFFMAN B, BRADSHAW K, CUNNINGHAM G. WILLIAMS GINECOLOGÍA, DE LA 1A
EDICIÓN EN INGLÉS. MC GRAW HILL. USA. 2009, PP 62-63.

http://www.cenetec.salud.gob.mx/descargas/gpc/CatalogoMaestro/IMSS_609_13_CANDIDOSISVULVOVAGINAL/609GRR.pdf

171 - EL SIGUIENTE FACTOR ES FUNDAMENTAL EN LA FISIOPATOLOGÍA DE LA ENFERMEDAD:


MENOPAUSIA. Consecuencias de la MENOPAUSIA: 1. Amenorrea, infertilidad. 2. Síntomas vasomotores. 3.
Atrofia urogenital. 4. Efectos sobre la sexualidad: • Asociados a hipoestrogenismo: Disminución
del trofismo vaginal, dispareunia y trastorno de la integridad sensorial. • Asociados a
hipoandrogenismo: Disminución de la libido y del funcionamiento del órgano eréctil femenino. 5.
Cambios cognitivos: En estadíos iniciales de la post-menopausia se pueden observar cambios
adversos en la memoria y en la capacidad de concentración, cambios en la forma del pensamiento
y en la capacidad de análisis. 6. Síntomas de tipo depresivo: El climaterio puede provocar
síntomas de tipo depresivo sin constituir una depresión crónica o depresión mayor. La TH no es
tratamiento de depresión crónica ni de depresión mayor, pero puede ser un coadyuvante a la
terapia específica. 7. Trastornos del sueño: Asociado a síntomas vasomotores. 8. Riesgo de
demencia: Hay controversias al comparar los estudios de efectos de TH, según ésta se inicie cerca
del comienzo de la vida posmenopáusica, o se inicie después de quince años de la menopausia
(Estudio WHIMS). Mientras el tratamiento precoz y mantenido parece reducir la incidencia de
diversas demencias a largo plazo, la TH puede aumentar levemente el riesgo de demencia,
vascular y no vascular, y la incidencia de accidente vascular encefálico (AVE) en mujeres mayores
de 65 años. 9. Osteoporosis: Debida al aumento de la resorción ósea, que depende directamente
del grado de hipoestrogenismo. Condiciona un mayor riesgo de fracturas, especialmente de
columna, cadera y antebrazo. 10. Riesgo cardiovascular: El hipoestrogenismo aumenta el riesgo
coronario afectando negativamente diversos factores de riesgo cardiovascular. RECUERDA QUE
LA MENOPAUSIA ES EL CESE DE LA MENSTRUACIÓN POR EL DECLIVE DE LA PRODUCCIÓN
HORMONAL FEMENINA, EN ESTE CASO ES EL FACTOR QUE SE ASOCIA AL CLIMATERIO.

AUTOINMUNE. La etiología del LÍQUEN PLANO es desconocida. Se han propuestos múltiples mecanismos
asociados de los cuales el inmune es el más evidente. Existe una alteración inmunológica en la piel
de la vulva. Es decir existe una activación de linfocitos T, contra las células de la piel
(Queratinocitos básales). CORRESPONDE AL FACTOR ASOCIADO AL LÍQUEN PLANO.
IMPORTANTE: Las mujeres con cierto grado de inmunocompromiso (ejem. diabetes) se asocian
fuertemente a candidiasis vulvovaginal, sin embargo, no existen antecedentes en este caso que
sugieran que esta sea un factor asociado.

USO DE La paciente tiene un cuadro infeccioso ocasionado por "hongos". Se han reportado como factores
ANTIBIÓTICOS de riesgo el embarazo, el uso de anticonceptivos orales, diabetes Mellitus y el uso de antibióticos
DE AMPLIO antibacterianos, también están vinculados a un incremento en la incidencia de infecciones por
ESPECTRO. levaduras. En este caso el TRATAMIENTO DE CATORCE DÍAS CON METRONIDAZOL pudo haber
sido el factor de riesgo principal para la colonización por cándida. Los síntomas de la candidiasis
pueden variar dependiendo del área que este afecte. Infecciones de la vagina o vulva pueden
causar picazón severa, ardor, dolor, irritación, y una descarga blanquecina o blanco grisáceo con
consistencia como de requesón, a menudo con una apariencia similar a grumos como la paciente
del caso clínico. EL USO DE ANTIBIÓTICOS SISTÉMICOS ES UN FACTOR DE RIESGO PARA
PRESENTAR CANDIDIASIS VULVOVAGINAL DADO QUE LOS HONGOS APROVECHAN
SITUACIONES COMO EL QUE LOS ANTIBIÓTICOS BARRIERAN CON LA FLORA NORMAL.

VIRUS DEL La incidencia de CÁNCER VULVAR aumenta en forma constante con la edad, y alcanza su nivel
PAPILOMA máximo en el séptimo decenio de la vida. Con base en las características etiológicas e
HUMANO. histopatológicas, se sospecha que el carcinoma epidermoide se desarrolla por dos vías separadas.
Una conduce sobre todo a carcinomas no queratinizados y afecta principalmente a mujeres
jóvenes. En esta vía, se identifica la infección con virus del papiloma humano de alto riesgo (HR-
HPV). Predominan los serotipos 16 y 18, aunque también hay informes de los serotipos 31, 33 y
45. La segunda vía, rara vez se relaciona con HR-HPV, ocurre en mujeres mayores, conduce sobre
todo a carcinoma epidermoide queratinizado, diferenciado y se desarrolla sobre un fondo de
trastornos epiteliales no neoplásicos, como liquen escleroso. ES LA PRINCIPAL CAUSA ASOCIADA
A CÁNCER CERVICOUTERINO Y CÁNCER VULVAR, NO CORRESPONDE AL CASO CLÍNICO.

Bibliografía:
1. GUÍA DE REFERENCIA RÁPIDA, DIAGNÓSTICO Y TRATAMIENTO DE CANDIDOSIS VULVOVAGINAL EN
MUJERES MAYORES A 12 AÑOS DE EDAD. MÉXICO: SECRETARIA DE SALUD; 2010. 3. SCHONGUE J, SCHAFER
J, HALVORSON L, HOFFMAN B, BRADSHAW K, CUNNINGHAM G. WILLIAMS GINECOLOGÍA, DE LA 1A
EDICIÓN EN INGLÉS. MC GRAW HILL. USA. 2009, PP 62-63.

http://www.cenetec.salud.gob.mx/descargas/gpc/CatalogoMaestro/IMSS_609_13_CANDIDOSISVULVOVAGINAL/609GRR.pdf

172 - EL TRATAMIENTO INDICADO EN ESTE PACIENTE ES MEDIANTE:


CORTICOTERAPIA Tratamiento del LIQUEN PLANO erosivo. Es muy difícil; se aconseja evitar irritantes, analgésicos
LOCAL. para calmar el dolor, sedación. Se aplican corticoides locales, en casos graves pueden darse en
forma sistémica. Para enfermedad vaginal se utilizan corticoides como la hidrocortisona, que
suele ser muy útil para prevenir las adherencias y para curar las erosiones vaginales. En
pacientes que no tienen una vida sexual regular, es necesario que usen diariamente dilatadores
vaginales para impedir la formación de adherencias. Se indican corticoides sistémicos solo en
casos recalcitrantes con prurito intenso y lesiones mucocutáneas extensas. Se propone también
el uso de Ciclosporina. En casos de adherencias persistentes, deben ser tratadas
quirúrgicamente, pero desafortunadamente pueden recurrir. Luego de la cirugía deben utilizar
dilatadores vaginales hasta la cicatrización completa. Control de las infecciones sobre
agregadas (Doxiciclina más fluconazol). EL USO DE CORTICOTERAPIA LOCAL ESTÁ INDICADA
PARA EL MANEJO DEL LIQUEN PLANO. IMPORTANTE: recuerda que el uso de esteroides
tópicos ante una infección por hongos puede empeorar el cuadro, por lo que por ningún
motivo deberán aplicarse en pacientes con vulvovaginitis por cándida.

TERAPIA EL GERMEN ASOCIADO A ESTE CUADRO ES LA CÁNDIDA ALBICANS, UN HONGO DIPLOIDE


ANTIMICOTICA. ASEXUADO EN FORMA DE LARVA, POR TAN MOTIVO LA TERAPIA ANTIMICÓTICA ES LO MÁS
ADECUADO PARA ESTE CASO. Todos los Azoles tópicos y orales así como la Nistatina local,
tienen una efectividad alrededor del 80% en el tratamiento de Candidiasis Vulvovaginal no
complicada. 1. Tratamiento tópico: - Miconazol crema 2%, una aplicación (5 gramos) en vulva y
vagina al día, durante 7 días o - Nistatina óvulos o tabletas vaginales de 100 000 U, una
aplicación vaginal al día, durante 14 días. IMPORTANTE: los Azoles tópicos pueden causar
irritación Vulvovaginal, misma que debe considerarse si persisten los síntomas. Daña los
condones y diafragmas de látex. 2. Tratamiento oral. - Fluconazol cápsulas 250 mg en una dosis
única ó Itraconazol cápsulas 200 mg cada 12 horas por 1 día. IMPORTANTE.: Contraindicados
en el embarazo y lactancia. El tratamiento de la pareja masculina asintomática, de pacientes
con candidiasis vulvovaginal, no disminuye la frecuencia de recurrencia de esta, por lo tanto,
no está indicado dar tratamiento a la(s) pareja(s) masculina(s), si ésta(s) se encuentra(n)
asintomática(s). VAGINITIS RECURRENTE POR CANDIDA SP. Tratamiento de elección para VC
recurrente: - Inducción: Itraconazol oral 200mg cada 12 horas por un día (dosis única) o
Miconazol crema 2%, 1 aplicación intravaginal diaria por 14 días. - Mantenimiento:
Ketoconazol tabletas de 200mg, media tableta al día por 6 meses o Itraconazol oral 50 a 100
mg diario por 6 meses o Fluconazol cápsulas de 100 mg, una vez a la semana por 6 meses (no
se use en embarazo o lactancia). EMBARAZO: Las mujeres embarazadas asintomáticas con
vaginitis por cándida no requieren tratamiento. Use sólo tratamientos locales por 14 días en
caso de que la infección sea sintomática durante el embarazo.

TERAPIA ATROFIA UROGENITALl: La sequedad vaginal, dispareunia, infecciones vaginales, prurito


HORMONAL vaginal, disuria y urgencia miccional mejoran con tratamiento hormonal (TH). Si la única
LOCAL A BASE DE indicación de TH es la atrofia urogenital, se debe considerar el uso de estrógenos locales. La TH
ESTRÓGENOS. debe ser sólo utilizada cuando exista una indicación clara para su uso. La paciente sintomática
es la principal beneficiada del tratamiento, no existe un tratamiento alternativo a los
estrógenos o estrógeno/progestina tan eficaz en el alivio de la sintomatología y en reducción
de fracturas. La indicación de un tratamiento prolongado debe ser revisada anualmente y
requiere de una vigilancia estrecha de la paciente.

VULVECTOMÍA. Casi 90 % de los TUMORES VULVARES corresponden a carcinoma epidermoide. Por


consiguiente, todo el conocimiento sobre factores pronósticos, patrones de diseminación y
datos de supervivencia se basa en la revisión de las mujeres con este tipo de tumor. Casi todos
los cánceres epidermoides tienen un buen pronóstico cuando se detectan en etapa temprana.
El tratamiento tradicional incluye ablación radical de la vulva y ganglios linfáticos inguinales.
En ocasiones se aplica radioterapia adyuvante.

Bibliografía:
1. GUÍA DE REFERENCIA RÁPIDA, DIAGNÓSTICO Y TRATAMIENTO DE CANDIDOSIS VULVOVAGINAL EN
MUJERES MAYORES A 12 AÑOS DE EDAD. MÉXICO: SECRETARIA DE SALUD; 2010. 2. SCHONGUE J, SCHAFER
J, HALVORSON L, HOFFMAN B, BRADSHAW K, CUNNINGHAM G. WILLIAMS GINECOLOGÍA, DE LA 1A
EDICIÓN EN INGLÉS. MC GRAW HILL. USA. 2009, PP 62-63.

http://www.cenetec.salud.gob.mx/descargas/gpc/CatalogoMaestro/IMSS_609_13_CANDIDOSISVULVOVAGINAL/609GRR.pdf

FIN DEL CASO CLÍNICO SERIADO


ANÁLISIS DEL CASO CLÍNICO

IDENTIFICACIÓN DEL REACTIVO


Area: GINECOLOGÍA Y OBSTETRICIA
Especialidad: GINECOLOGÍA
Tema: TRANSTORNOS MENSTRUALES
Subtema: HEMORRAGIA UTERINA ANORMAL DE TIPO ANATÓMICO

CASO CLÍNICO SERIADO

FEMENINA DE 26 AÑOS DE EDAD, ANTECEDENTE DE TÍA CON PATOLOGÍA TIROIDEA, DESCONOCE EL TIPO.
REFIERE 2 AÑOS DE VIDA SEXUAL ACTIVA SIN LOGRAR UN EMBARAZO. AGO: MENARCA A LOS 16 AÑOS, RITMO
IRREGULAR (PASAN HASTA 6 MESES SIN MENSTRUACIÓN). REFIERE ASTENIA, PIEL SECA, CAÍDA DE CABELLO,
CEFALEA INTENSA RECURRENTE. A LA EXPLORACIÓN FÍSICA INQUIETA, PIEL Y MUCOSAS SECAS, MAMAS
SIMÉTRICAS TURGENTES, PERIMETRO ABDOMINAL 90CM, DISTRIBUCIÓN DEL VELLO GENITAL ROMBOIDE.
LABORATORIOS: HB 9.5, HTO 29%, TSH 3.5, T4 1.3, PROLACTINA 27, PROGESTERONA 1, TESTOTERONA 150.

Femenina de 26 años.

INFERTILIDAD. Oligomenorrea.

astenia, piel seca, caída de cabello, cefalea


intensa recurrente.

OBESIDAD CENTRÍPTA, DISTRIBUCIÓN DE


VELLO GENITAL ANDROIDE.

Normales.

173 - EL DIAGNÓSTICO CLÍNICO MÁS PROBABLE ES:


HIPOTIROIDISMO El HIPOTIROIDISMO se refiere a la disminución de las hormonas tiroideas y tiene
diferentes orígenes (congénito, autoinmunitario, tiroiditis atrófica). En un inicio algunos
pacientes pueden presentar síntomas leves, esta fase recibe el nombre de hipotiroidismo
subclínico o leve. Más adelante, los niveles de T4 descienden y los niveles de TSH
aumentan todavía más; los síntomas se hacen mucho más evidentes en ésta fase
(habitualmente TSH>10 mUU/L), que se denomina hipotiroidismo clínico (hipotiroidismo
manifiesto). CX: cansancio, debilidad, sequedad de piel, sensación de frío, caída de pelo,
dificultad para concentrarse y mala memoria, estreñimiento, disnea, voz ronca,
menorragia (más adelante oligomenorrea o amenorrea), parestesias, déficit auditivo,
extremidades frías, mixedema, alopecia difusa, bradicardia, edema periférico, retraso de
la relajación de los reflejos tendinosos, síndrome de túnel del carpo, derrames de
cavidades serosas. Pruebas de función tiroidea: 1. HIPOTIROIDISMO PRIMARIO:
TSH>10mUI/L, T4 <0.9ngL. 2. HIPOTIROIDISMO SECUNDARIO: TSH<1mUI/L, T4 <0.9ngL.
3. HIPOTIROIDISMO SUBCLÍNICO: TSH 4.5-10mUI/L, T4 0.9-1.9ngL (normal). LAS
HORMONAS TIROIDEAS DE LA PACIENTE SE ENCUENTRAN DENTRO DE PARÁMETROS
NORMALES.

HIPERPROLACTINEMIA El exceso de secreción de prolactina (HIPERPROLACTINEMIA) se presenta tanto en


varones como en mujeres y se manifiesta clínicamente por disfunciones sexuales o
reproductivas y galactorrea. La hiperprolactinemia (HPRL) llega a producir un estado de
hipogonadismo hipogonadotropo al suprimir la pulsatilidad de la secreción de
Gonadoliberina (GnRH) en el hipotálamo,de Gonadotropinas Folitropina (FSH) y
Lutropina (LH) en la hipófisis, con la consiguiente disminución de las concentraciones en
sangre de progesterona y estradiol en la mujer y testosterona en el varón. Los cambios en
la menstruación son proporcionales a los niveles de prolactina. Una HPRL moderada
(hasta 40 ng/ml) frecuentemente se manifiesta con deficiencias de fase lútea con ciclos
cortos y “spotting” premenstrual. El cambio posterior, cuando los niveles de PRL se
incrementan por encima de valores de alto riesgo de prolactinoma (por encima de 50
ng/ml), puede ocasionar anovulación, con ciclos oligomenorreicos y usualmente
hipomenorreas. Casi en el 100% de los casos con niveles mayores de 100 ng/mL hay
amenorrea. Entonces, la alteración progresiva de la secreción de las gonadotropinas da
como resultado un estado hipoestrogenizado, con amenorrea, infertilidad, dispareunia, y
a largo plazo, osteoporosis. LAS CIFRAS DE PROLACTINA NO COINCIDEN CON ÉSTA
PATOLOGÍA.

SÍNDROME DE OVARIO El SÍNDROME DE STEIN-LEVENTHAL u OVARIO POLIQUÍSTICO se caracteriza por ciclos


POLIQUÍSTICO anovulatorios que dan como resultado ovarios poliquísticos hipertróficos, amenorrea u
oligomenorrea, obesidad, hirsutismo e infertilidad. Sus principales características son
datos de Hiperandrogenismo (hirsutismo, acné) y trastornos menstruales.
Histológicamente se caracteriza por la presencia de pequeños quistes foliculares llenos de
líquido debajo de la engrosada y fibrosa corteza superficial. CX: Se asocia con la obesidad,
generalmente central, y anormalidades metabólicas como resistencia a la insulina. Las
pacientes deben llenar 2 de 3 criterios siguientes: 1. Oligoovulación, anovulación o
ambas. 2. Hiperandrogeniso (clínico, bioquímico o ambos) 3. Ovario poliquístico en la
ecografía. Descripción: - Resistencia insulínica: se manifiesta por mayor grado de
resistencia insulínica e hiperinsulinemia, situación que puede condicionar hiperglucemias.
- Anovulación: la secreción inapropiada de gonadotropinas provoca ciclos anovulatorios,
con mayor producción de hormona luteinizante (LH) y de hormona foliculoestimulante
(FSH). - Hiperandrogenismo: la hiperilsulinemia y la elevación de LH estimula la
producción de andrógenos en las células de la teca ovárica. - Hirsutismo: se manifiesta
con la presencia de vello grueso, oscuro y terminal distribuido en un patrón masculino. -
Acné: el aumento de andrógenos hiperestimula los receptores androgénicos en la unidad
pilosebásea con lo cual aumenta la producción de sebo lo cual favorece la formación de
comedones. Las pruebas de laboratorio revelan elevación sérica de andrógenos, aumento
en la proporción de hormona luteinizante contra la hormona del folículo, anormalidades
en los lípidos y resistencia a la insulina. La testosterona es una hormona androgénica
producida por los testículos. Las mujeres producen una cantidad mucho menor, que tiene
funciones en la regulación de aspectos como su humor, apetito sexual y sensación de
bienestar. En las mujeres, los ovarios producen la mayor parte de la testosterona.
VALORES NORMALES Hombres: 300-1.000 ng/dl Mujeres: 20-80 ng/dl LA PACIENTE
MUESTRA DATOS DE OLIGOOVULACIÓN E HIPERANDROGENISMO POR LABORATORIO.
SÍNDROME DE El SÍNDROME DE CUSHING se define como un conjunto de signos y síntomas resultantes
CUSHING de la elevación persistente, inapropiada y mantenida de los niveles circulantes en sangre
de glucocorticoides o hipercortisolismo. CX: Obesidad centrípeta por su localización
preferente en peritoneo, mediastino y tejido celular subcutáneo que afecta
preferentemente la cara. Retraso del crecimiento y pubertad. Osteopenia que puede
conducir a fracturas. Estrías rojo vinoso, de más de 1 cm de tamaño de eje transversal por
atrofia cutánea. HIRSUTISMO. Hiperpigmentación. Depresión con ansiedad. CEFALEAS.
ALTERACIONES MENSTRUALES (AMENORREA U OLIGOMENORREA). Úlcera péptica o
duodenal. LA PACIENTE CUMPLE CON MUCHOS DE LOS CRITERIOS AQUÍ DESCRITOS SIN
EMBARGO DE ACUERDO A SU EDAD, LA CAUSA MÁS FRECUENTE DE SÍNDROME DE
CUSHING ES LA ADMINSITRACIÓN EXÓGENA DE GLUCOCORTICOIDES Y NO EXISTE TAL
ANTECEDENTE.

Bibliografía:
1. GUÍA DE PRÁCTICA CLÍNICA, DIAGNÓSTICO Y TRATAMIENTO DEL SÍNDROME DE OVARIOS
POLIQUÍSTICOS. MÉXICO: SECRETARIA DE SALUD; 2010. 2. SCHONGUE J, SCHAFER J, HALVORSON L,
HOFFMAN B, BRADSHAW K, CUNNINGHAM G. WILLIAMS GINECOLOGÍA, DE LA 1A EDICIÓN EN INGLÉS. MC
GRAW HILL. USA. 2009, PP 383-391.

http://www.cenetec.salud.gob.mx/descargas/gpc/CatalogoMaestro/453_GPC_Ovario_poliquistico/GER_Sx_ovario_poliquxstico.pdf

174 - EL TRATAMIENTO INDICADO EN ESTE CASO SERÁ CON:

BROMOCRIPTINA. La BROMOCRIPTINA es una agonista sintético de la Dopamina químicamente emparentado


con los alcaloides del Ergot. Se utiliza en el tratamiento de varios síndromes de
hiperprolactinemia, así como en el Parkinson, la infertilidad, la acromegalia y los adenomas
de la pituitaria que excretan prolactina. CORRESPONDE AL TRATAMIENTO DE LA
HIPERPROLACTINEMIA.

LEVOTIROXINA. En el HIPOTIROIDISMO se debe iniciar la sustitución de tiroxina con Levotiroxina de 1.6 a


1.8mcg/kg/día o dosis de 25mcg diarios por 2 meses con reevaluación para incremento de la
dosis cada 2 a 4 semanas hasta llegar al Eutiroidismo clínico o bioquímico. Repetir pruebas
cada 8 a 12 semanas de inicio del tratamiento hasta lograr una TSH entre 0.5 y 4.5mUI/ml.
CORRESPONDE AL TRATAMIENTO DE ELECCIÓN PARA HIPOTIROIDISMO.

SOMATOSTATINA. Existen 2 tipos de fármacos para el manejo del SÍNDROME DE CUSHING: Aquellos que actúan
suprimiendo la secreción de ACTH y los que inhiben la secreción de cortisol. Los primeros son
poco eficaces a pesar de tener una acción selectiva bien sobre la secreción o sobre la acción
periférica de ACTH (Ciproheptadina, Bromocriptina, Reserpina, Somatostatina y Valproato
sódico). Sin embargo los fármacos que suprimen la secreción de cortisol son de gran utilidad
y representan una buena alternativa cuando el tratamiento médico es lo único disponible. De
todos ellos el más utilizado, por ser el menos tóxico es el Ketoconazol. CORRESPONDE AL
MANEJO FARMACOLÓGICO DEL SÍNDROME DE CUSHING.

ANTICONCEPTIVOS Los ANTICONCEPTIVOS ORALES “son el tratamiento de primera línea” en pacientes con
ORALES. Síndrome de ovario Poliquístico sin deseo de embarazo y pueden utilizarse por periodos
mayores a 6 meses. IMPORTANTE: Dado que la oligo o amenorrea se ha asociado a síndrome
de ovario poliquítico pude predisponer a hiperplasia endometrial y más tarde a carcinoma.
Por lo que se recomienda el tratamiento con PROGESTÁGENOS para inducir una hemorragia
por deprivación por lo menos cada 3 a 4 meses. AUNQUE LA PACIENTE DESEA
EMBARAZARSE, ES EL ÚNICO MANEJO ADECUADO PARA ÉSTA PATOLOGÍA DESCRITO EN LA
OPCIONES TERAPÉUTICAS.

Bibliografía:
1. GUÍA DE PRÁCTICA CLÍNICA, DIAGNÓSTICO Y TRATAMIENTO DEL SÍNDROME DE OVARIOS
POLIQUÍSTICOS. MÉXICO: SECRETARIA DE SALUD; 2010. 2. SCHONGUE J, SCHAFER J, HALVORSON L,
HOFFMAN B, BRADSHAW K, CUNNINGHAM G. WILLIAMS GINECOLOGÍA, DE LA 1A EDICIÓN EN INGLÉS. MC
GRAW HILL. USA. 2009, PP 383-391.

http://www.cenetec.salud.gob.mx/descargas/gpc/CatalogoMaestro/453_GPC_Ovario_poliquistico/GER_Sx_ovario_poliquxstico.pdf
175 - LA COMPLICACIÓN MÁS ESPERADA EN ESTE CASO SERÍA:

RESISTENCIA A En los últimos 20 años se ha demostrado claramente que el SÍNDROME DE OVARIO


LA INSULINA. POLIQUÍSTICO (SOPQ) tiene consecuencias metabólicas relacionadas a la resistencia a la
insulina y esta alteración juega un papel importante en la patogénesis de las anormalidades
reproductivas de este trastorno. La asociación entre Diabetes e Hiperandrogenismo fue
mostrada por primera vez por Achard y Thiers en 1921 con la descripción de “diabetes de la
mujer barbuda. En 1980 Burghen sugirió por primera vez la relación entre el
Hiperandrogenismo e Hiperinsulinemia, particularmente en mujeres obesa. Desde entonces, en
los últimos 20 años, la relación entre SOPQ y resistencia a la insulina ha sido ampliamente
estudiada. A pesar de que las alteraciones metabólicas pueden estar presentes precozmente en
mujeres jóvenes y delgadas, diversos estudios han demostrado claramente que las mujeres con
SOPQ tienen un mayor riesgo para desarrollar intolerancia a la glucosa y DM-2, el cual a su vez,
está relacionado con el avance de la edad y desarrollo de obesidad . LA RESISTENCIA A LA
INSULINA Y LA DIABETES MELLITUS SON LAS PRINCIPALES COMPLICACIONES RELACIONADAS
CON EL OVARIO POLIQUÍSTICO.

DISMINUCIÓN La HIPERPROLACTINEMIA puede ser debida a grandes tumores hipofisiarios llamados


DE LA VISIÓN. PROLACTINOMAS, cuando son grandes llegan a comprimir las vías ópticas ocasionando
disminución de la visión. El tratamiento de elección en éste caso es quirúrgico ya que es
urgente la descompresión de las vías ópticas para salvar la visión.

FRACTURAS. Las FRACTURAS, se producen frecuentemente en el SÍNDROME DE CUSHING por: • Bloqueo de


la absorción intestinal de calcio, como consecuencia de la inhibición de la acción de la vitamina
D en la luz intestinal. • Inhibición de la vitamina D en la luz intestinal. • Inhibición de la
Hidroxilación hepática de la vitamina D. • inhibición de la reabsorción tubular de calcio.
CORRESPONDE A UNA COMPLIACIÓN FRECUENTE DEL SÍNDROME DE CUSHING.

COMA El COMA MIXEDEMATOSO sigue teniendo una elevada mortalidad a pesar del tratamiento
MIXEDEMATOSO. intensivo. Las manifestaciones clínicas consisten en disminución del nivel de conciencia, en
ocasiones acompañada de convulsiones, así como las demás características del hipotiroidismo.
La hipotermia puede alcanzar los 23°C. Puede haber antecedentes de hipotiroidismo tratado
con mal cumplimiento por parte del paciente, pero también es posible que no se haya
establecido aún el diagnóstico de hipotiroidismo. El coma mixedematoso ocurre casi siempre
en el anciano y suele desencadenarse por factores que alteran la respiración, como fármacos
(especialmente sedantes, anestésicos y antidepresivos), neumonía, insuficiencia cardíaca
congestiva, infarto del miocardio, hemorragia digestiva o accidentes cerebrovasculares. ES
UNA COMPLICACIÓN DE HIPOTIROIDISMO.

Bibliografía:
1. GUÍA DE PRÁCTICA CLÍNICA, DIAGNÓSTICO Y TRATAMIENTO DEL SÍNDROME DE OVARIOS
POLIQUÍSTICOS. MÉXICO: SECRETARIA DE SALUD; 2010. 2. SCHONGUE J, SCHAFER J, HALVORSON L,
HOFFMAN B, BRADSHAW K, CUNNINGHAM G. WILLIAMS GINECOLOGÍA, DE LA 1A EDICIÓN EN INGLÉS. MC
GRAW HILL. USA. 2009, PP 394.

http://www.cenetec.salud.gob.mx/descargas/gpc/CatalogoMaestro/453_GPC_Ovario_poliquistico/GER_Sx_ovario_poliquxstico.pdf

FIN DEL CASO CLÍNICO SERIADO


ANÁLISIS DEL CASO CLÍNICO

IDENTIFICACIÓN DEL REACTIVO


Area: MEDICINA INTERNA
Especialidad: ONCOLOGÍA
Tema: CÁNCER DE PULMÓN, GASTROINTESTINAL Y SNC
Subtema: CÁNCER GASTROINTESTINAL

CASO CLÍNICO CON UNA PREGUNTA

HOMBRE DE 48 AÑOS DE EDAD CON DIAGNÓSTICO DE CÁNCER GÁSTRICO, ACTUALMENTE CON


TRATAMIENTO A BASE DE 5-FLUOROURACILO.

HOMBRE DE 48 AÑOs.

dx CÁNCER GÁSTRICO. TX A BASE DE 5-


FLUOROURACILO.

-.

-.

-.

176 - LA ACCIÓN ANTINEOPLÁSICA DEL MEDICAMENTO ES SECUNDARIA A LA:

FORMACIÓN DE El CÁNCER DE ESTÓMAGO O CÁNCER GÁSTRICO es un tipo de crecimiento tisular maligno


ENLACES producido por la proliferación contigua de células anormales con capacidad invasora y
COVALENTES CON destrucción de otros tejidos y órganos, en particular esófago e intestino delgado. La mayor
EL DNA. parte de estos tumores son de tipo adenocarcinomas. La quimioterapia como parte del
manejo del paciente con cáncer gástrico se otorga con en diferentes combinaciones, teniendo
todas en común en fluorouracilo. El 5-FLUOROURACILO también conocido como 5-FU en un
potente antimetabolito utilizado en algunas formas de cáncer, algunos de éstos, de la piel, sin
embargo, las indicaciones de uso dependen de la vía de administración y se maneja en
presentaciones tópicas, crema y solución; o sistémicas, solución inyectable por vía endovenosa
gota a gota.
DETENCIÓN DE LA INDICACIONES • El 5-fluorouracilo se utiliza en enfermedad de Bowen, basaliomas
CÉLULA EN FASE superficiales solos o acompañados. Estados precancerosos y en situaciones en que la radiación
G2. sea imposible o no se pueda intervenir quirúrgicamente. • Se ha notado que el 5-fluorouracilo
no tiene eficacia alguna en espinaliomas. La solución inyectable se utiliza en ciertos casos
como paliativo de los carcinomas de seno, recto y colon cuando no pueda intervenirse
quirúrgicamente. Carcinoma de colon, recto, mama, estómago, páncreas, vejiga y próstata. •
Las indicaciones de los antineoplásicos, su dosificación y pautas de administración están en
constante revisión. En su forma tópica está indicado para enfermedades cutáneas
precancerosas y carcinomas superficiales de células basales.

INHIBICIÓN DE LA FARMATOCINÉTICA • Atraviesa la barrera hematoencefálica y los metabolitos activos se


TOPOISOMERASA- localizan dentro de la célula. Se metaboliza con rapidez (1 hora) en los tejidos y produce un
DNA. metabolito activo, el monofosfato de fluoxuridina. • La degradación catabólica tiene lugar en
el hígado. Se elimina en forma primaria por vía respiratoria 60% a 80%, como dióxido de
carbono, y en forma secundaria por vía renal 15%, inalterado, en la primera hora.

INHIBICIÓN DE LA EFECTO FARMACOLÓGICO. • EL 5-FLUOROURACILO INHIBE LA TIMIDILATO SINTETASA A


TIMIDILATO TRAVÉS DE SU METABOLITO, 5-FLUORO-2 -DEOXIURIDINA MONOFOSFATO. Este, forma un
SINTETASA. complejo ternario covalente con la timidilato sintetasa y el 5,10-metilentetrahidrofolato, al
sustituir a la deoxiuridina monofosfato. Esta unión impide la producción de deoxitimidina
monofosfato, mientras los niveles de folatos endógenos sean correctos. • Se considera que el
5-fluorouracilo tiene acción específica de la fase S del ciclo de división celular. La actividad se
produce como resultado de su conversión a un metabolito activo en los tejidos e incluye la
inhibición de la síntesis de DNA y RNA. • Interviene en la síntesis de ADN e inhibe en poco
grado la formación de ARN. Ambas acciones se combinan para promover un desequilibrio
metabólico que resulta en la muerte de la célula. La actividad inhibitoria del fármaco, por su
analogía con el ácido nucleico uracilo, tiene afectación sobre el veloz crecimiento de las
células neoplásicas que aprovechan preferentemente la molécula del uracilo para la biosíntesis
del ácido nucleico. Los efectos de una privación de ADN y ARN atacan más a las células que
crecen y se multiplican sin control que a las normales.

Bibliografía:
1. LONGO DL, FAUCI AS, KASPER DL, HAUSERSL, JAMESON JL, LOSCALZOJ. HARRISON. PRINCIPIOS DE
MEDICINA INTERNA, 18A EDICIÓN. MC GRAW HILL. NEW YORK, USA. 2012, PP 703. 2. GUÍA DE PRÁCTICA
CLÍNICA, DIAGNÓSTICO Y TRATAMIENTO DEL ADENOMA GÁSTRICO EN PACIENTES ADULTOS. MÉXICO:
SECRETARIA DE SALUD, 2010.

http://www.cenetec.salud.gob.mx/descargas/gpc/CatalogoMaestro/167_GPC_CA_GASTRICO/Gpc_cancergastrico.pdf
ANÁLISIS DEL CASO CLÍNICO

IDENTIFICACIÓN DEL REACTIVO


Area: GINECOLOGÍA Y OBSTETRICIA
Especialidad: OBSTETRICIA
Tema: HEMORRAGIAS DE LA PRIMERA MITAD DEL EMBARAZO
Subtema: ABORTO

CASO CLÍNICO CON UNA PREGUNTA

MUJER DE 32 AÑOS, GESTA 3, PARA 2, CURSA CON EMBARAZO DE 15 SDG ACUDE AL SERVICIO DE OBSTETRICIA AL PRESENTAR
SALIDA DE LÍQUIDO TRANSVAGINAL. A LA EXPLORACIÓN ENCUENTRA CÉRVIX CON 1CM DE DILATACIÓN, SIN SALIDA EVIDENTE DE
LÍQUIDO. SE REALIZA ULTRASONIDO QUE REPORTA AUSENCIA DE LÍQUIDO AMNIÓTICO.

-.

embarazo de 15 SDG.

salida de lÍquido trasvaginal.

Cérvix con 1cm de dilatación.

ausencia de líquido amniótico.

177 - EN ESTE MOMENTO A LA PACIENTE SE LE DEBERÁ INDICAR:

HISTERECTOMÍA En algunos casos se prefiere recurrir a una histerotomía o histerectomía abdominal para la resolución de un aborto en
EN BLOQUE. lugar del legrado o la inducción médica en casos como: patología uterina importante asociada, interrupción del embarazo
con control de la natalidad definitivo, o en complicaciones secundarias al legrado tal como la perforación uterina. LA
HISTERECTOMÍA NO ES EL TRATAMIENTO IDEAL DEL ABORTO sólo en casos especiales ya mencionados.
DILATACIÓN El ABORTO es la terminación espontánea o provocada de la gestación antes de la vigésima semana, contando desde el
MAS LEGRADO primer día de la última menstruación normal, o expulsión del producto de la gestación con peso menor a 500mg.
UTERINO. Conceptos: ABORTO INCOMPLETO: la placenta se desprende del útero de manera total o parcial produciendo hemorragia,
el orificio cervical se encuentra abierto y permite la salida de sangre. El feto y la placenta pueden permanecer total o
parcialmente dentro del útero, expulsarse juntos o por separado. ABORTO INEVITABLE: en este caso no es posible revertir
las causas del aborto, puede existir rotura de membranas con dilatación del cérvix. AUNQUE NO SE REFIERE EL ESTADO
FETAL, LA FRANCA ROTURA DE MEMBRANAS JUNTO CON DILATACIÓN CERVICAL SUGIEREN LA PRESENCIA DE UN
ABORTO INEVITABLE EN ESTA PACIENTE. El aborto puede entonces resolverse por dos vías: aborto médico o quirúrgico.
1. ABORTO MÉDICO. Se le denomina INDUCCIÓN o INDUCTOCONDUCCIÓN y hace referencia a la aplicación de fármacos
para estimular la contracción uterina, la vigilancia de la evolución de la dilatación y la consecuente expulsión fetal. Está
indicada en aquellos abortos inevitables (como es el caso), en evolución o incompletos con o sin modificaciones
cervicales. El tratamiento médico tiene mayor éxito (70 a 96%) cuando se prescribe a dosis altas de misoprostol 1200 a
1400 gr. En gestaciones de 13 a 15 semanas la dosis inicial para la inducción del aborto es de 400 ug vaginal, repetir la
dosis entre 6 a 12 horas si no ha habido respuesta. Si no hay respuesta a las 24 hrs duplicar la dosis hasta un máximo de 4
dosis. OJO: si resultase efectiva para expulsar por completo la unidad fetoplacentaria, no es necesario realizar legrado, a
menos que exista sangrado excesivo y persistente, inestabilidad hemodinámica, la evidencia de tejido retenido infectado
o sospecha de enfermedad trofoblástica gestacional. 2. ABORTO QUIRÚRGICO. Se puede realizar mediante LEGRADO
UTERINO INSTRUMENTADO (LUI) o ASPIRACIÓN MANUAL ENDOUTERINA (AMEU). Son indicaciones de LUI
instrumentado: a) Tratamiento del aborto en cualquiera de sus variedades siempre que la altura uterina sea mayor o igual
a 12cm (aborto tardío) y exista una dilatación mayor o igual a 1cm. b) Aborto séptico hasta 6 a 8hrs después de iniciado
el tratamiento antibiótico. Son indicaciones de AMEU: a) Tratamiento el aborto en cualquiera de sus variedades siempre y
cuando se tenga una altura uterina mayor a 11cm y dilatación cervical menor o igual a 1cm. b) Aborto séptico hasta 6 a
9hrs después de haber iniciado el tratamiento antibiótico. El aborto quirúrgico, en abortos menores a 12-14SDG, puede
no dilatarse el cuello siempre que sea posible introducir la cánula de tamaño justo a través del orificio cervical. Siempre
que el cérvix se encuentra cerrado o insuficiente dilatado la dilatación es esencial y, se considera recomendable en todas
las mujeres con más de 12 a 14 SDG. IMPORTANTE: PARA PREPARAR EL CÉRVIX SE PUEDE USAR DILATADORES
OSMÓTICOS Y AGENTES FARMACOLÓGICOS. - DILATACIÓN FARMACOLÓGICA: se realiza mediante la administración de
misoprostol. - DILATADORES OSMÓTICOS: permiten la dilatación cervical de manera mecánica. OJO: El misoprostol
produce menos dilatación que los dilatadores osmóticos, pero tiene la ventaja de ser un procedimiento que se completa
en un único día en la mayoría de las mujeres. En gestaciones entre las 12 a 19SDG pueden utilizarse ambas opciones de
manera indiferente. En gestaciones mayores a 20 SDG se prefieren los dilatadores osmóticos. A TENER EN CUENTA: - En la
práctica clínica generalmente se realiza dilatación + legrado en la mayoría de los abortos tardíos (mayores a 14SDG), esto
bajo la creencia de que, si no se efectúa el legrado de forma rutinaria, se favorece la retención de tejidos, infecciones y
sangrado. - En caso de que se te presentara una respuesta con los términos inductoconducción del aborto/inducción del
aborto + legrado uterino, se limitaría sólo a la dilatación farmacológica del cérvix dejando de lado la posibilidad a
dilatación mecánica (mediante dilatadores osmóticos), por tal motivo esta respuesta resulta muy adecuada pues
DILATACIÓN + LEGRADO UTERINO incorpora tanto a la dilatación farmacológica como la mecánica. PARA ESTE CASO,
PUESTO QUE SE TRATA DE UNA GESTACIÓN DE 15 SDG, SE ASUME QUE LA ALTURA UTERINA ESTÉ POR ARRIBA DE LOS
12CM Y LAS CONDICIONES CERVICALES SON ADECUADAS PARA PROCEDER A DILATACIÓN MÁS LEGRADO UTERINO.

EXTRACCIÓN El manejo está dirigido a la extracción fetal, la placenta y sus anexos con la finalidad de evitar la retención de tejidos que
FETAL. favorezcan la infección. Por tanto, LA EXTRACCIÓN FETAL ES SÓLO UNA PARTE DE LA TERAPIA INDICADA PARA LA
RESOLUCIÓN DEL ABORTO.

INDUCCIÓN DE PARTO es el conjunto de fenómenos activos y pasivos que desencadenados a partir de la SEMANA 20 de la gestación, que
TRABAJO DE tienen por objeto la expulsión del producto mismo de la gestación, la placenta y sus anexos a través de la vía natural (o
PARTO. canal del parto en la mujer). Dado que la paciente se encuentra en la semana 15 del embarazo sin datos de viabilidad
fetal se considera un aborto y por lo tanto EL TÉRMINO ADECUADO ES INDUCCIÓN DEL ABORTO.

Bibliografía:
- DIAGNÓSTICO Y TRATAMIENTO DEL ABORTO ESPONTÁNEO Y MANEJO INICIAL DE ABORTO RECURRENTE. EVIDENCIAS Y
RECOMENDACIONES: GUÍA DE PRÁCTICA CLÍNICA. MÉXICO: SECRETARIA DE SALUD; 2009. - CUNNINGHAM G, LEVENO K, BLOMM
S, HAUTH J, RPUSE D, SONG C. WILLIAMS OBSTETRICIA, 23A EDICIÓN. MC GRAW HILL. USA. 2011. - MANUAL DE PRÁCTICA CLÍNICA
PARA UN ABORTO SEGURO. ORGANIZACIÓN MUNDIAL DE LA SALUD (OMS), 2014.
HTTP://APPS.WHO.INT/IRIS/BITSTREAM/HANDLE/10665/134747/9789243548715_SPA.PDF;JSESSIONID=68CDB2F6D22A64E382C7E4DE7C87B1D7?
SEQUENCE=1

http://www.cenetec-difusion.com/CMGPC/IMSS-088-08/ER.pdf
ANÁLISIS DEL CASO CLÍNICO

IDENTIFICACIÓN DEL REACTIVO


Area: CIRUGÍA
Especialidad: OTORRINOLARINGOLOGÍA
Tema: PATOLOGÍA DEL OÍDO
Subtema: HIPOACUSIA

CASO CLÍNICO CON UNA PREGUNTA

MUJER DE 48 AÑOS DE EDAD, QUE ACUDE A LA CONSULTA EXTERNA AL PERCIBIR DISMINUCIÓN DE LA


CAPACIDAD AUDITIVA.

-.

-.

disminución de la capacidad auditiva.

-.

-.

178 - PARA SOLICITAR E INTERPRETAR UNA AUDIOMETRÍA, ES NECESARIO SABER QUE LA ESTRUCTURA
QUE INTERVIENE EN EL AJUSTE DE LA IMPEDANCIA DEL SONIDO ES:

LA El oído es el órgano sensorial en donde se aloja el receptor especializado para la audición y para el
MEMBRANA equilibrio. El ÓRGANO DE CORTI en la Cóclea, transforma las vibraciones en impulsos nerviosos. La
TENTORIA. MEMBRANA TENTORIA es una estructura gelatinosa, hialina, situada por encima del ÓRGANO DE
CORTI, que se extiende desde la lámina espiral hasta el nivel de las células de Hensen. Las puntas
de los estereocilios de las células pilosas externas están embebidas en una masa gelatinosa llamada
membrana tectoria, la cual se encuentra en la región superior del órgano de Corti. CONSTITUYE A
LA FORMACIÓN DEL ESTÍMULO ACÚSTICO CUANDO SE PRODUCE UNA DEFLEXIÓN DE ELLA.
LOS La VIBRACIÓN DEL TÍMPANO es mayor según la frecuencia, siendo mejor en la zona central, entre
HUESECILLOS. las frecuencias 1000-3000 ciclos/s y en el martillo directamente sobre los 4000 ciclos/s. El
mecanismo de amplificación del sonido está determinado por el tímpano, que es 17 veces más
grande que la ventana oval, lo que contribuye a mejorar la audición en 27 dbs. El mecanismo de
palanca de los huesecillos contribuye a mejorar en 3 dbs, lo que permite ajustar la impedancia de
la interfase aire-líquido perilinfático del oído interno por medio de la platina del estribo que actúa
como pistón. LOS HUESECILLOS TIENEN LA FUNCIÓN DE AJUSTAR LA IMPEDANCIA AIRE-LIQUIDO
PARA LA TRANSMISIÓN DEL IMPULSO AUDITIVO.

EL MÚSCULO El MÚSCULO ESTAPEDIO es parte de la pared del oído medio y se inserta en el cuello del estribo. Su
ESTAPEDIO. función es halar la cabeza del estribo hacia atrás. FORMA PARTE DEL MECANISMO DE FUNCIÓN DE
LOS HUESECILLOS PERO NO LOGRA LOS AJUSTES DE IMPEDANCIA POR SI SOLO REQUIERE DEL
CONJUNTO DE ESTRUCTURAS. ESTA ESTRUCTURA SE RELACIONA MÁS BIEN CON EL REFLEJO
ACÚSTICO.

EL ÓRGANO Una vez que el impulso sonoro llega a la ventana oval en el OÍDO INTERNO, se produce un
DE CORTI. movimiento de la perilinfa determinando una onda denominada "onda viajera" y que tiene un
punto de mayor vibración dependiendo de la frecuencia de estímulo en una determinada zona de
la cóclea, existiendo así una distribución tonotópica dentro de las dos y media espiras de ésta. De
este modo las frecuencias altas estimulan mejor la membrana basal que sostiene el órgano de Corti
de la base de la cóclea, y las frecuencias graves estimulan más el ápice de la cóclea, que está más
alejado. El segundo fenómeno que ocurre es la transducción de la energía mecánica en eléctrica,
por medio de las células ciliadas. Así, con la onda vibratoria, son estimulados los cilios de estas
células que están en contacto con la membrana tectoria del órgano de Corti, generando por este
mecanismo un estímulo nervioso. EL ÓRGANO DE CORTI TIENE LA DIFÍCIL FUNCIÓN DE
CONVERTIR EL ESTIMULO MECÁNICO EN NERVIOSO (ELÉCTRICO).

Bibliografía:
1. COMITÉ DE TRAUMA DEL COLEGIO AMERICANO DE CIRUJANOS, PROGRAMA AVANZADO DE APOYO
VITAL EN TRAUMA PARA MÉDICOS. ATLS, 7A EDICIÓN. USA. PP 600-603. 2. LALWANI K, DIAGNÓSTICO Y
TRATAMIENTO EN OTORRINOLARINGOLOGÍA. CIRUGÍA DE CABEZA Y CUELLO, 2A EDICIÓN. MC GRAWHILL
LANGE. 2008.
ANÁLISIS DEL CASO CLÍNICO

IDENTIFICACIÓN DEL REACTIVO


Area: MEDICINA INTERNA
Especialidad: CARDIOVASCULAR
Tema: CARDIOPATÍA ISQUÉMICA
Subtema: INFARTO AGUDO DEL MIOCARDIO

CASO CLÍNICO CON UNA PREGUNTA

MUJER DE 50 AÑOS DE EDAD, CON ANTECEDENTE DE HIPERTENSIÓN DESDE HACE 5 AÑOS. ACTUALMENTE
CON SOSPECHA DE CARDIOPATÍA ISQUÉMICA AÚN EN ESTUDIO.

MUJER DE 50 AÑOS DE EDAD.

HIPERTENSIÓN DESDE HACE 5 AÑOS.

SOSPECHA DE CARDIOPATÍA ISQUÉMICA


AÚN EN ESTUDIO.

-.

-.

179 - EL MEDICAMENTO INICIAL DE ELECCIÓN PARA LOS PACIENTES HIPERTENSOS QUE DESARROLLAN
CARDIOPATÍA ISQUÉMICA ES:

EL PRAZOSIN. RELACIÓN DE HIPERTENSIÓN CON CARDIOPATÍA ISQUÉMICA. • Existe una estrecha relación entre
la HTA y diversas enfermedades cardíacas como la cardiopatía isquémica, la insuficiencia cardíaca,
la hipertrofia ventricular izquierda o determinadas arritmias como la fibrilación auricular. • La
presencia de HTA empeora el pronóstico de la cardiopatía isquémica y el tratamiento
antihipertensivo es eficaz en la prevención secundaria en estos pacientes.

LA ANTAGONISTAS DE CALCIO DIHIDROPIRIDÍNICOS. • En caso de contraindicación o intolerancia a


FELODIPINA. los betabloqueantes o si la monoterapia no es suficiente se asociarán antagonistas del calcio
dihidropiridínicos de acción prolongada. Los calcioantagonistas dihidropiridínicos (nifedipina,
amlodipina, nimodipina, nicardipina, FENODIPINO) también pueden ser útiles en adición al
tratamiento estándar tanto en casos de HTA no controlada como con cifras normal-altas de PA.
LOS CALCIO ANTAGONISTAS DIHIDROPIRIDÍNICOS SON LA SEGUNDA OPCIÓN TERAPÉUTICA
PARA LA HIPERTENSIÓN EN PACIENTES CON CARDIOPATÍA ISQUÉMICA.
EL BETABLOQUEADORES • Los betabloqueadores disminuyen la frecuencia cardíaca y el efecto
METOPROLOL. adverso del exceso de adrenalina en el corazón. El uso de betabloqueadores después de un infarto
de miocardio se ha asociado con mejor supervivencia. LOS BETABLOQUEADORES SON LOS
FÁRMACOS DE ELECCIÓN PARA LOS PACIENTES CON HTA Y ANGINA ESTABLE O IAM PREVIO.

EL • Inhibidores de la Enzima Convertidora de Angiotensina (ECA) y Bloqueadores de los Receptores


CAPTOPRIL. de Angiotensina II. Los inhibidores de la ECA y los bloqueadores de los receptores de angiotensina
II, son medicamentos que reducen la resistencia de los vasos sanguíneos contra la cual el corazón
tiene que bombear sangre. Estos medicamentos han sido efectivos en prevenir el agrandamiento
del corazón después de infartos de miocardio grandes. • Numerosos estudios han demostrado la
utilidad del tratamiento con fármacos IECA tras un IAM, especialmente en casos de disfunción
ventricular o IAM anterior extenso. LOS IECAS SE HAN USADO SIN MEJORÍAS IMPORTANTES CON
RESPECTO A LOS TRATAMIENTOS DE BASE POR LO QUE NO SE CONSIDERAN MEDICAMENTOS DE
PRIMERA LÍNEA.

Bibliografía:
1. LONGO DL, FAUCI AS, KASPER DL, HAUSERSL, JAMESON JL, LOSCALZOJ. HARRISON. PRINCIPIOS DE
MEDICINA INTERNA, 18A EDICIÓN. MC GRAW HILL. NEW YORK, USA. 2012, PP 2030.
ANÁLISIS DEL CASO CLÍNICO

IDENTIFICACIÓN DEL REACTIVO


Area: GINECOLOGÍA Y OBSTETRICIA
Especialidad: GINECOLOGÍA
Tema: LEUCORREA
Subtema: CANDIDIASIS VAGINAL

CASO CLÍNICO SERIADO

MUJER DE 52 AÑOS DE EDAD CON FUM DE HACE 7 DÍAS. ACUDE A CONSULTA POR PRESENTAR PRURITO
VULVAR INTENSO QUE SE ACOMPAÑA DE ARDOR VULVAR DURANTE LA MICCIÓN. A LA EXPLORACIÓN SE
ENCUENTRA ERITEMA VULVAR ACENTUADO, HUELLAS DE RASCADO INTENSO Y LESIONES PAPULOSAS
PERIFÉRICAS BIEN DEFINIDAS, LA SECRECIÓN ES BLANQUECINA Y GRUMOSA.

Mujer de 52 años de edad.

FUM de hace 7 días.

Prurito vulvar intenso que se acompaña de


ardor vulvar durante la micción.

Eritema vulvar acentuado, huellas de


rascado intenso y lesiones papulosas
periféricas bien de nidas y secreción
blanquecina grumosa.

-.

180 - EL AGENTE INFECCIOSO QUE MÁS PROBABLEMENTE ESTA ASOCIADO AL CUADRO CLÍNICO DE LA
PACIENTE ES:

VIRUS DEL El HERPES GENITAL es una enfermedad de transmisión sexual (ETS) causada por los virus del
HERPES herpes simple tipo 1 (VHS-1), y tipo 2 (VHS-2). La mayoría de los herpes genitales son causados
SIMPLE. por el tipo VHS-2. La mayoría de las personas infectadas por el VHS-1 ó VHS-2, no presentan
signos ni síntomas de la infección o presentan síntomas mínimos. Cuando se manifiestan los
signos, usualmente lo hacen en forma de una o más ampollas en los genitales o el recto o
alrededor de los mismos. Las ampollas se rompen formando úlceras dolorosas (llagas) que
pueden tardar de dos a cuatro semanas en curarse la primera vez que se presentan. Típicamente,
puede presentarse otro brote semanas o meses después del primero, pero casi siempre es menos
intenso y de más corta duración. A pesar de que la infección puede permanecer en forma
indefinida en el organismo, la cantidad de brotes tiende a disminuir a medida que pasan los años.
RECUERDA QUE EN ESTE CASO LA CARACTERÍSTICA SON VESÍCULAS "DOLOROSAS".
CHLAMYDIA La CERVICITIS MUCOPURULENTA es en la mujer el equivalente a la uretritis en el varón. Es muy
TRACHOMATIS. importante su diagnóstico para prevenir complicaciones como la endometritis y la salpingitis y,
en la mujer embarazada el parto prematuro, la infección puerperal y la iniciación o promoción de
una neoplasia cervical. Para afirmar que existe una CMP, el exudado del cérvix obtenido con una
torunda de algodón blanco, tras una primera limpieza de la mucosidad, debe manchar la torunda
de color amarillento o verdoso extendido sobre el portaobjetos, visto al microscopio (x 1.000)
debe contener al menos 10 polimorfonucleares por campo, en 5 campos no adyacentes
observados de forma consecutiva. C. trachomatis es el microorganismo aislado con mayor
frecuencia en la CMP, seguido de N. gonorrhoeae, herpes simple y T. vaginalis, estos dos últimos
producen una exocervicitis, Chlamydia y gonococo infectan el endocérvix. El diagnóstico
etiológico se establece mediante cultivos del exudado endocervical, que debe obtenerse tras la
limpieza previa del orificio externo del cérvix. CIERTAMENTE ESTE TIPO DE INFECCIÓN PUEDE
DAR SÍNTOMAS URINARIOS PERO LA SECRECIÓN ES TÍPICAMENTE AMARILLO-VERSOSA.

CANDIDA Los síntomas de CANDIDIASIS VULVOVAGINAL incluyen prurito, descenso vaginal, dolor vaginal,
ALBICANS. dispareunia y disuria externa. El diagnóstico de candidiasis se puede hacer mediante inspección
visual, determinación del pH vaginal, microscopía, Papanicolaou, prueba de látex y cultivo de
secreción cervicovaginal. El flujo vaginal de una real infección por hongos puede tener diferentes
apariencias. Puede estar ausente, muy discreto, o muy fluido, blanco, con presencia de placas en
la pared vaginal, típicamente como ‘requesón’. Se debe de sospechar de cándida si la paciente
tiene un rash geográfico simétrico en la vulva o en el área perineal. Una forma algo atípica de
presentación de la cándida, es aquella paciente que tiene una irritación inexplicable y disconfort
o aquella sin historia de dispareunia que inicia molestias de quemazón intra o poscoital,
irritación, disconfort. Este problema suele presentarse en mujeres peri y posmenopáusicas. LA
PRESENCIA DE PRURITO VAGINAL QUE SE CONFIRMA CON HUELLAS DE RASCADO Y LAS
CARACTERÍSTICAS DE LA LEUCORREA ORIENTAN ESTE DIAGNÓSTICO.

TRICHOMONA TRICOMONIASIS T. vaginalis constituye una de las enfermedades de transmisión sexual más
VAGINALIS. frecuentes en el mundo, en la embarazada se ha asociado a parto prematuro y recién nacido de
bajo peso. Produce leucorrea profusa, espumosa, amarillo-verdosa y maloliente, con abundantes
polimorfonucleares, pH alcalino y prurito vaginal. En la exploración, la vagina está inflamada y el
cérvix enrojecido y edematoso con aspecto de frambuesa. En el varón, la infección por T.
vaginalis es menos frecuente. La sintomatología que produce es de uretritis y, con frecuencia, la
infección es asintomática. El diagnóstico por microscopia en fresco es un método específico en
caso de vaginitis purulenta, pero poco sensible para detectar a las pacientes asintomáticas. El
cultivo es un método específico y sensible, superado por los métodos moleculares, especialmente
por la reacción en cadena de la polimerasa (PCR). La citología, especialmente el Papanicolau, se
considera un método inadecuado por su baja sensibilidad y pobre valor predictivo positivo. LA
LEUCORREA EN ESTE CASO ES CLÁSICA ABUNDANTE, AMARILLO VERDOSA Y FÉTIDA.

Bibliografía:
1. GUÍA DE REFERENCIA RÁPIDA, DIAGNÓSTICO Y TRATAMIENTO DE CANDIDOSIS VULVOVAGINAL EN
MUJERES MAYORES A 12 AÑOS DE EDAD. MÉXICO: SECRETARIA DE SALUD; 2010.

http://www.cenetec.salud.gob.mx/descargas/gpc/CatalogoMaestro/IMSS_609_13_CANDIDOSISVULVOVAGINAL/609GRR.pdf

181 - EL TRATAMIENTO MÁS INDICADO EN ESTE CASO ES:

AZITROMICINA. Tratamiento de la CERVICITIS MUCOPURULENTA Está indicada la Azitromicina 1 g en dosis


única o Doxiciclina 100 mg cada 12 horas por vía oral durante 7 días. Como régimen alternativo
podemos emplear Levofloxacino 500 mg al día durante 7 días. Tanto la doxiciclina como el
levofloxacino están contraindicados en la infección en mujeres embarazadas, la azitromicina es
eficaz y segura. Por los riesgos de infección del neonato debe repetirse el cultivo a las tres
semanas de finalizado el tratamiento. LA AZITROMICINA ES EL TRATAMIENTO DE ELECCIÓN EN
CASO DE CERVICITIS MUCOPURULENTA.

METRONIDAZOL. Tratamiento TRICOMONIASIS. Un tratamiento eficaz es el Metronidazol que, en dosis única de


2 g o dosis múltiples de 500 mg cada 12 horas durante 7 días, se ha mostrado eficaz en más del
90% de las pacientes, al igual que los nuevos compuestos Tinidazol y Ornidazol. Este
tratamiento se recomienda también en las mujeres embarazadas, en las que no se ha mostrado
teratogénico. PARA EL MANEJO DE LA TRICOMONIASIS SE DEBE ADMINISTRAR
METRONIDAZOL ORAL.
ACICLOVIR. HERPES GENITAL Se desconoce la cura del herpes genital pero la evolución de los síntomas se
puede modificar si se inicia el tratamiento sistémico con Aciclovir o sus análogos tan pronto
comienzan los síntomas. El tratamiento puede reducir la formación de nuevas lesiones, la
duración del dolor, el tiempo necesario hasta la resolución de las lesiones y la eliminación viral.
Sin embargo, no parece modificar la historia natural de la enfermedad recurrente. No se
recomienda el tratamiento tópico con Aciclovir ya que sólo produce un acortamiento mínimo
de la duración de los episodios sintomáticos. Infecciones recurrentes. La mayoría de los
pacientes con un primer episodio de infección por herpes genital presentará episodios
recurrentes de las lesiones genitales. El tratamiento antiviral episódico o supresivo acortará la
duración de las lesiones genitales. Muchos pacientes se benefician al recibir el tratamiento
antiviral por lo tanto, se deben analizar con todos los pacientes las opciones de tratamiento
antiviral. Muchos pacientes con enfermedad recurrente se benefician al recibir tratamiento
episódico, si se inicia el tratamiento durante el período prodrómico o dentro del primer día de
aparición de las lesiones. Si se opta por el tratamiento de los episódicos recurrentes, se deberá
suministrar al paciente el tratamiento antiviral o una receta para obtener el medicamento de
manera que se inicie el tratamiento ante el primer signo de lesiones genitales o el pródromos.
LOS ANTIVIRALES ESTÁN INDICADOS PARA EL MANEJO DEL HERPES UROGENITAL.

MICONAZOL. CANDIDIASIS VULVOVAGINAL. Todos los Azoles tópicos y orales así como la Nistatina local,
tienen una efectividad alrededor del 80% en el tratamiento de Candidiasis Vulvovaginal no
complicada. 1. Tratamiento tópico: - Miconazol crema 2%, una aplicación (5 gramos) en vulva y
vagina al día, durante 7 días o - Nistatina óvulos o tabletas vaginales de 100 000 U, una
aplicación vaginal al día, durante 14 días. IMPORTANTE: los Azoles tópicos pueden causar
irritación Vulvovaginal, misma que debe considerarse si persisten los síntomas. Daña los
condones y diafragmas de látex. 2. Tratamiento oral. - Fluconazol cápsulas 250 mg en una dosis
única ó Itraconazol cápsulas 200 mg cada 12 horas por 1 día. IMPORTANTE.: Contraindicados en
el embarazo y lactancia. El tratamiento de la pareja masculina asintomática, de pacientes con
candidiasis vulvovaginal, no disminuye la frecuencia de recurrencia de esta, por lo tanto, no
está indicado dar tratamiento a la(s) pareja(s) masculina(s), si ésta(s) se encuentra(n)
asintomática(s). VAGINITIS RECURRENTE POR CANDIDA SP. Tratamiento de elección para VC
recurrente: - Inducción: Itraconazol oral 200mg cada 12 horas por un día (dosis única) o
Miconazol crema 2%, 1 aplicación intravaginal diaria por 14 días. - Mantenimiento: Ketoconazol
tabletas de 200mg, media tableta al día por 6 meses o Itraconazol oral 50 a 100 mg diario por 6
meses o Fluconazol cápsulas de 100 mg, una vez a la semana por 6 meses (no se use en
embarazo o lactancia). EMBARAZO: Las mujeres embarazadas asintomáticas con vaginitis por
cándida no requieren tratamiento. Use sólo tratamientos locales por 14 días en caso de que la
infección sea sintomática durante el embarazo. EL MICONAZOL ESTÁ INDICADO COMO PARTE
DEL MANEJO TÓPICO DE LA CANDIDIASIS VULVOVAGINAL.

Bibliografía:
1. GUÍA DE REFERENCIA RÁPIDA, DIAGNÓSTICO Y TRATAMIENTO DE CANDIDOSIS VULVOVAGINAL EN
MUJERES MAYORES A 12 AÑOS DE EDAD. MÉXICO: SECRETARIA DE SALUD; 2010.

http://www.cenetec.salud.gob.mx/descargas/gpc/CatalogoMaestro/IMSS_609_13_CANDIDOSISVULVOVAGINAL/609GRR.pdf

FIN DEL CASO CLÍNICO SERIADO


ANÁLISIS DEL CASO CLÍNICO

IDENTIFICACIÓN DEL REACTIVO


Area: PEDIATRÍA
Especialidad: INFECTOLOGIA PEDIÁTRICA
Tema: HIV Y SIDA
Subtema: HIV Y SIDA

CASO CLÍNICO SERIADO

LACTANTE DE 4 MESES, HIJO DE MADRE DE 17 AÑOS, ADICTA A HEROÍNA Y EN REHABILITACIÓN DESDE HACE
UN AÑO. SIN CONTROL PRENATAL, OBTENIDO POR PARTO, REPORTÁNDOSE AL NACIMIENTO COMO SANO.
INICIA SU PADECIMIENTO HACE UNA SEMANA CON PRESENCIA DE FIEBRE DE 38°C, DISTENSIÓN
ABDOMINAL, ADENOMEGALIAS CERVICALES Y AXILARES EVACUACIONES DISMINUIDAS DE CONSISTENCIA
CON MOCO SIN SANGRE DE 5 A 7 EN 24 HRS. MOTIVO POR LO QUE ACUDE A URGENCIAS. A LA EXPLORACIÓN
CON MAL ESTADO GENERAL, CAQUÉCTICO, PÁLIDO, HIPOACTIVO, CON ADENOMEGALIAS CERVICALES Y
AXILARES BILATERALES DE 1 CM, BLANDAS, MÓVILES, NO DOLOROSAS, HEPATOMEGALIA 5, 5,6 CM DEL
BORDE COSTAL, ESPLENOMEGALIA DE 6 CM. LLAMA LA ATENCIÓN LA PRESENCIA DE MONILIASIS INTENSA
ORAL Y EN LA ZONA DEL PAÑAL.

LACTANTE DE 4 MESES. FORMA DE


COMIENZO PRECOZ DE VIH

MUY IMPORTANTE QUE LA MADRE SEA


ADICTA A LA HEROÍNA. SIN CONTROL
PRENATAL, OBTENIDO POR PARTO,
APARENTEMENTE SANO

FIEBRE DE 38°C, DISTENSIÓN


ABDOMINAL, ADEnOMEGALIAS
CERVICALES Y AXILARES EVACUACIONES
DISMINUIDAS DE CONSISTENCIA CON
MOCO SIN SANGRE DE 5 A 7 EN 24 HRS

MAL ESTADO GENERAL, CAQUéCTICO,


PáLIDO, HIPOACTIVO, CON
ADENOMEGALIAS CERVICALES Y
AXILARES BILATERALES DE 1 CM,
BLANDAS, MÓVILES, NO DOLOROSAS,
HEPATOMEGALIA 5,5,6 CM DEL BORDE
COSTAL, ESPLENOMEGALIA DE 6 CM.
LLAMA LA ATENCIÓN LA PRESENCIA DE
MONILIASIS INTENSA ORAL Y EN LA ZONA
DEL PAÑAL. ES CLARO QUE EL PACIENTE
PRESENTA UN COMPROMISO INFECCIOSO
SISTÉMICO, CON DETERIORO
IMPORTANTE E INFECCIONES
OPORTUNISTAS. SIEMPRE DEBERÁ
CONSIDERARSE HIV

182 - CON LOS ANTECEDENTES Y EL CUADRO CLÍNICO LA SOSPECHA DIAGNÓSTICA MAS PROBABLE ES:

INFECCIÓN POR Deben distinguirse dos síndromes importantes de la infección por citomegalovirus (CMG)
CITOMAGALOVIRUS en pacientes pediátricos: 1. La ENFERMEDAD POR CMV EN LA EDAD PEDIÁTRICA que es la
infección sintomática producida por un virus DNA de la familia de los Herpesvirus en
pacientes menores de 16 años, que causa una gran variedad de síntomas que pueden ser
leves hasta muy graves e incluso llevar a la muerte a individuos previamente sanos. Lo más
frecuente es que la infección sea asintomática y permanezca latente en estos individuos
toda la vida; sin embargo, la primoinfección o reactivación pueden causar enfermedad y su
gravedad dependerá del estado inmunológico del paciente. La infección postnatal por CMV
puede producirse por contacto con secreciones del tracto genial materno durante el parto,
alimentación con leche materna, trasfusiones sanguíneas o a través de líquidos biológicos
de pacientes infectados que excretan el virus, especialmente orina y saliva. La mayoría de
las infecciones adquiridas en neonatos o lactantes tiene un curso asintomático; sin
embargo, en el recién nacido prematuro, que tiene menor cantidad de anticuerpos
transferidos, tiene mayor riesgo de presentar síntomas. Las principales manifestaciones
consisten en neumonitis, hepatitis, enteritis y menos frecuentemente, linfadenopatías y
meningitis aséptica. 2. El SÍNDROME DE INFECCIÓN CONGÉNITA POR CMV se refiere al
diagnóstico de la infección congénita por este virus durante las primeras tres semanas de
vida en presencia de manifestaciones clínicas. El 90% de los neonatos infectados de manera
congénita se muestran asintomáticos al nacimiento y, se espera que, entre el 5 al 17%
desarrolle síntomas como pérdida auditiva neurosensorial, coriorretinitis o déficit
neurológico que puede manifestarse desde los primeros 2 días de vida. Del 100% de los
nacidos sintomáticos, el 20% muere y el 80% de sobrevivientes puede desarrollar secuelas
neurológicas. Además, un tercio de estos niños cursan con prematurez y la mitad de los
niños son pequeños para la edad gestacional, por lo que se ha propuesto la búsqueda
sistemática de este virus en recién nacidos, principalmente aquellos que nacen antes de las
32 SDG y que tienen peso inferior a 1,500 gr. Cuando se sospecha enfermedad congénita
por CMV, los signos y síntomas que deben investigarse incluyen: exantema, petequias,
retraso en el crecimiento intrauterino, microcefalia, hepatoesplenomegalia e ictericia. En
los estudios de laboratorio se deberá buscar de forma intencionada daño hepático y
reticuloendotelial, con hiperbilirrubinemia conjugada y transaminasas elevadas. A largo
plazo es frecuente observar alteraciones en el sistema nervioso central, hipoacusia,
dificultades para el aprendizaje, microcefalia y en raras ocasiones, disminución de la
agudeza visual. POR EL TIEMPO DE EVOLUCIÓN PODRÍA TRATARSE DE UNA INFECCIÓN
POSTNATAL POR CMG; SIN EMBARGO, UNA DE LAS PRINCIPALES MANIFESTACIONES
SERÍA LA NEUMONITIS Y EL CASO NO DESCRIBE AFECTACIÓN RESPIRATORIA, ADEMÁS, SI
BIEN LAS INFECCIONES POR CMV SUELEN ACOMPAÑARSE DE LINFADENOPATÍA, ÉSTE NO
ES UN DATO CLAVE NI TAN MARCADO COMO EN EL SIDA. OJO: recuerda que los pacientes
con infección por VIH a menudo pueden presentar, como parte de los signos graves de la
enfermedad, infecciones por CMV; por lo tanto, aunque no es el caso, los pacientes pueden
presentar manifestaciones por CMV aunadas a otras altamente sugestivas de infección por
VIH.
SÍNDROME DE La mayoría de las infecciones por VIH en menores de 13 años se deben a transmisión
INMUNODEFICIENCIA vertical (transmisión perinatal); entendiéndose por ésta a la infección transmitida de una
ADQUIRIDA madre infectada por el VIH a su hijo durante el embarazo, el trabajo de parto, el
nacimiento, o la lactancia materna (postparto). Actualmente se considera que el riesgo de
esta transmisión en ausencia de cualquier tipo de intervención varía entre 15 al 40%. La
PATOGÉNESIS de la transmisión vertical del VIH es multifactorial y se considera que está
mediada por las microtransfusiones sanguíneas que se producen durante las contracciones
uterinas, cuando existe corioamnionitis, o por el ascenso del virus a través de las
secreciones vaginales después de la ruptura de las membranas y su absorción por el tracto
digestivo del feto. La transmisión de la infección por el VIH de madre a hijo es factible de
acuerdo con los niveles de la carga viral de la madre que correlacionan con el riesgo de
transmisión aún en mujeres que reciben tratamiento antirretroviral (ARV); sin embargo,
aunque el riesgo de transmisión perinatal en mujeres con carga viral indetectable parece
ser extremadamente bajo, se ha identificado la transmisión en mujeres con carga viral no-
detectable o con niveles muy bajos debido a que, además de la carga viral plasmática,
existen otros factores que intervienen en la transmisión como niveles bajos de RNA y DNA
del VIH en secreciones genitales de mujeres con carga viral indetectable, coinfecciones del
tracto genital y penetración variable de los diferentes ARV en el tracto genital. Además,
actualmente se sabe que, si existe exposición al VIH a través del tracto genital durante el
nacimiento, existe riesgo de transmisión perinatal del VIH. Por lo tanto, todas las mujeres
infectadas por el VIH deben recibir consejería y tratamiento ARV independientemente de
sus niveles de carga viral y el seguimiento durante el embarazo idealmente deberá incluir
carga viral hasta lograr niveles de indetectabilidad antes del nacimiento (alrededor de la
semana 36 a 37). La HISTORIA NATURAL de la infección por el VIH en la infancia es
variable; se estima que alrededor del 25% de los lactantes infectados progresan
rápidamente a SIDA, o mueren durante el primer año de vida, aunque algunos sobreviven
hasta los 12 años en ausencia de tratamiento ARV. Las MANIFESTACIONES tempranas de
enfermedad en el niño con infección por el VIH constituyen un predictor de mortalidad
significativo. En la mayoría de los lactantes la exploración física suele ser normal al nacer.
Los síntomas iniciales pueden ser sutiles con linfadenopatía y hepatoesplenomegalia, o
inespecíficos, como retraso del crecimiento, diarrea crónica y recidivantes, síntomas
respiratorios o candidiasis oral, y quizá solo se distingan por su persistencia. En niños, a
diferencia de los adultos, son más comunes las infecciones bacterianas recidivantes, la
tumefacción parotídea crónica, la neumonitis intersticial linfoide y el comienzo precoz de
un deterioro neurológico. En pediatría se ha definido un sistema de clasificación clínica
basado en el estado clínico y el grado de alteración inmunológica. Clínicamente es posible
clasificar a los pacientes en: - Categoría A (síntomas leves): niños con al menos dos
síntomas leves como linfadenopatía, parotiditis, hepatomegalia, esplenomegalia,
dermatitis, sinusitis u otitis media recidivantes y persistentes. - Categoría B (síntomas
moderados): incluye a niños con neumonitis intersticial linfoide (NIL), candidiasis
orofaríngea que persiste por más de dos meses, diarrea recidivante o crónica, fiebre
persistente durante más de 1 mes, hepatitis, estomatitis herpética recidivante, esofagitis o
neumonitis por virus del herpes simple (VHS), varicela diseminada (con afección visceral),
cardiomegalia o nefropatía. - Categoría C (síntomas graves): comprende a niños con
enfermedades oportunistas (p. ejem., candidiasis esofágica o del aparato respiratorio
inferior, criptosporidiosis, infección diseminada por micobacterias o citomegalovirus,
neumonía por Pneumocystis o toxoplasmosis cerebral), infecciones bacterianas
recidivantes (sepsis, meningitis, neumonía), encefalopatía, tumores malignos y pérdida de
peso intensa. Basado en el grado de alteración inmunológica, los menores de 12 meses
pueden clasificarse como: - Sin evidencia de inmunosupresión: con linfocitos TCD4
mayores o iguales a 1,500. - Con evidencia de inmunosupresión moderada: con linfocitos
TCD4 entre 750 y 1,499. - Con inmunosupresión grave: con linfocitos TCD4 menores a 750.
ES MUY PROBABLE QUE SE TRATE DE UN SÍNDROME DE INMUNODEFICIENCIA HUMANA
POR LA SUMA DE SIGNOS Y SÍNTOMAS QUE MANIFIESTA EL PACIENTE, ADEMÁS DEL
ANTECEDENTE DE MADRE CON FACTORES DE RIESGO PARA INFECCIÓN POR VIH.
HEPATITIS El término HEPATITIS NEONATAL involucra a los padecimientos que derivan de una
NEONATAL colestasis intrahepática, por lo que puede manifestarse de varias formas, las principales
son: - Hepatitis neonatal idiopática: puede presentarse con un patrón familiar o de forma
espontánea de causa desconocida. Se cree que la forma espontánea deriva de una
enfermedad metabólica o vírica no identificada, mientras que, las formas familiares
pueden surgir a consecuencia de una alteración genética o metabólica. - Síndrome de
Aagenae: es una forma de colestasis intrahepática familiar asociada con linfedema de
extremidades inferiores. Se cree que está asociada a la disminución del flujo linfático
hepático o a una hipoplasia linfática hepática. Los episodios de colestasis se acompañan de
elevación de aminotransferasas, fosfatasa alcalina y ácidos grasos. - Síndrome de Zellweger
(cerebrohepatorrenal): es un trastorno autosómico recesivo infrecuente que se caracteriza
por una degeneración progresiva del hígado y los riñones. Los niños afectados presentan
hipotonía generalizada grave y afección intensa de la función neurológica con retraso
psicomotor. La forma de la cabeza es anómala y la cara inusual, existe hepatomegalia,
quistes en la corteza renal, calcificaciones en forma de puntos en las rótulas y el trocánter
mayor, así como anomalías oculares. Un hallazgo histopatológico es la ausencia de
peroxisomas en las células hepáticas. - Enfermedad neonatal por depósito de hierro
(hemocromatosis neonatal): se caracteriza por ser una enfermedad de progresión rápida
debida al depósito de hierro en el hígado, corazón y los órganos endócrinos sin incremento
en el sistema retículoendotelial. Clínicamente, los pacientes muestran falla multiorgánica.
Los hallazgos de laboratorio incluyen hipoglucemia, hiperbilirrubinemia, hipoalbuminemia
e hipoprotrombinemia profunda. COMO TE DARÁS CUENTA EL TÉRMINO HEPATITIS
NEONATAL ENGLOBA DIVERSOS SÍNDROMES QUE TIENEN EN COMÚN LA COLESTASIS
INTRAHEPÁTICA. EN ESTE CASO EN ESPECIAL, ESTE TRASTORNO NO JUSTIFICA EL TOTAL
DE LOS SIGNOS Y SÍNTOMAS DEL PACIENTE, ESPECIALMENTE LAS LINFADENOPATÍAS.

INFECCIÓN POR El Virus del Epstein Barr (EBV) es un virus ubicuo que causa infecciones, principalmente en
VIRUS EPSTEIN- niños y adolescentes, lo que resulta en un espectro de expresión clínica que va desde
BARR formas asintomáticas, autolimitadas, mononucleosis infecciosa (MI), hasta
linfoproliferación; ésta última característica de individuos con inmunocompromiso y
asociada con enfermedades malignas (linajes de células B y células epiteliales). La infección
primaria en la infancia ocurre estrechamente después de la desaparición de anticuerpos
maternos, permaneciendo latente en el hospedero, favoreciendo una fuerte respuesta
inmune. EN LACTANTES SANOS, LA PRIMOINFECCIÓN SUELE CURSAR ASINTOMÁTICA. La
MONONUCLEOSIS INFECCIOSA es el síndrome mejor conocido de los causados por el Virus
del Epstein Barr (EBV). Es más común en adolescentes; como se mencionó antes en
lactantes y niños pequeños la infección suele cursar asintomática, mientras que para
personas mayores el comienzo suele ser insidioso y vago. La triada clínica clásica se
caracteriza por fiebre, linfadenopatía y datos clínicos de faringitis. Durante la exploración
física es común encontrar adenopatías generalizadas, esplenomegalia y hepatomegalia.
Aunque poco frecuente, cuando hay exantema, este suele ser maculopapular. SI BIEN EL
PACIENTE REUNE VARIOS DE LOS SIGNOS CLÁSICOS DE LA INFECCIÓN POR EL EBV, SE
DEBE CONSIDERAR QUE EN PACIENTES SANOS (INMUNOCOMPETENTES) A ESA EDAD LA
INFECCIÓN SUELE CURSAR ASINTOMÁTICA. OJO: la inmunosupresión permite la
reactivación de infecciones por el EBV latentes o la presencia de primoinfección
sintomática en lactantes.

Bibliografía:
1. PREVENCIÓN, DIAGNÓSTICO Y TRATAMIENTO EN EL BINOMIO MADRE-HIJO CON INFECCIÓN POR EL
VIH. GUÍA DE EVIDENCIAS Y RECOMENDACIONES: GUÍA DE PRÁCTICA CLÍNICA. MÉXICO. SECRETARÍA DE
SALUD; 03/11/2016. 2. KLIEGMAN R, STANTON B, GEME J, SCR N, BEHRMAN R. NELSON TRATADO DE
PEDIATRÍA, 19ª EDICIÓN. ELSEVIER. ESPAÑA 2013. 3. PREVENCIÓN, DIAGNÓSTICO Y TRATAMIENTO DE LA
INFECCIÓN POR CITOMEGALOVIRUS EN LA EDAD PEDIÁTRICA. GUÍA DE EVIDENCIAS Y
RECOMENDACIONES: GUÍA DE PRÁCTICA CLÍNICA. MÉXICO. SECRETARÍA DE SALUD; 2012.
HTTP://WWW.CENETEC-DIFUSION.COM/CMGPC/IMSS-610-13/ER.PDF 4. PREVENCIÓN, DIAGNÓSTICO Y
TRATAMIENTO DE LA MONONUCLEOSIS INFECCIOSA. GUÍA DE EVIDENCIAS Y RECOMENDACIONES: GUÍA
DE PRÁCTICA CLÍNICA. MÉXICO. SECRETARÍA DE SALUD; 2010. HTTP://WWW.CENETEC-
DIFUSION.COM/CMGPC/IMSS-485-11/ER.PDF

http://www.cenetec-difusion.com/CMGPC/IMSS-246-12/ER.pdf

183 - EL ESTUDIO QUE SE REQUIERE PARA CONFIRMAR EL DIAGNÓSTICO ES:


AMPLIFICACIÓN El diagnóstico de la infección por el VIH EN EL LACTANTE debe realizarse mediante el uso de
VIROLÓGICA DE pruebas de amplificación virológica que incluyen DNA y RNA. Los anticuerpos maternos
DNA PARA VIH. cruzan la placenta y pueden detectarse en todos los recién nacidos expuestos al VIH, por lo
cual no deben solicitarse estudios que detectan anticuerpos desde la etapa de recién nacido
hasta los 18 meses de vida extrauterina. Los ensayos que detectan el antígeno p24 se usan
con menor frecuencia que las técnicas de amplificación del DNA o RNA para el diagnóstico
de la infección por el VIH debido a su relativa pobre sensibilidad y la ausencia de pruebas
comerciales, con una sensibilidad de 97% a 100% en los primeros seis meses de vida. Los
estudios virológicos que se requieren para el diagnóstico de la infección por el VIH en el
recién nacido y el lactante menor de 18 meses de vida, con exposición al VIH deben
efectuarse en tres periodos diferentes durante los primeros seis meses de vida: a. entre 14 a
21 días. b. de 1 a 2 meses, y c. 4 a 6 meses. Una prueba virológica positiva debe ser
confirmada lo antes posible con un segundo estudio virológico en muestras diferentes. Para
establecer el diagnóstico definitivo de la infección por el VIH en un niño menor de 18 meses
de edad, se requieren dos resultados positivos mediante estudios virológicos como PCR ya
sea del RNA y/o DNA del VIH en determinaciones de sangre diferentes, lo que se presenta
con mayor frecuencia en los primeros seis meses de vida. La infección por el VIH puede
descartarse en el lactante expuesto en la etapa perinatal mediante dos o más estudios
virológicos positivos después de las ocho semanas de vida extrauterina, en los periodos de
seguimiento establecidos (14 a 21 días, al mes de vida o antes de los 6 meses). La exclusión
definitiva puede realizarse en niños que no recibieron lactancia materna con dos o más
estudios virológicos negativos a los seis meses de vida. Algunos expertos recomiendan
efectuar además en estas mismas condiciones, una prueba de detección de anticuerpos a los
18 meses para descartar en forma definitiva la infección. LAS PRUEBAS IDEALES PARA
REALIZAR EL DIAGNÓSTICO DE INFECCIÓN POR VIH EN LACTANTES SON LAS TÉCNICAS DE
AMPLIFICACIÓN DE DNA Y RNA VIRAL.

IGM PARA El diagnóstico de la infección por citomegalovirus (CMV) en el recién nacido, se establece
CITOMEGALOVIRUS mediante la detección del virus en orina, en cultivo y por PCR, este último es el método
diagnóstico más utilizado por su rapidez (24 a 48hrs), con una gran sensibilidad (94.5%) y
especificidad (99%). La PCR para CMV en salida y muestras de sangre seca (tamiz neonatal)
tiene una sensibilidad del 99% y una especificidad del 100%, por lo que se considera el
método de elección en infección en el recién nacido. Fuera del periodo neonatal, se
recomienda realizar estudios diagnósticos siempre que exista sospecha clínica de la
enfermedad. La tinción con anticuerpos mononucleares puede acortar el tiempo de
diagnóstico de la infección. En niños mayores de un año, la presencia de IgM anti-CMV más
un cuadro clínico sugestivo, así como el incremento de IgG anti-CMV en dos muestras con
diferencia de dos semanas entre ellas, son recomendables. En recién nacidos o lactantes
durante los primeros 9 a 12 meses, la presencia de IgG anti-CMV se traduce en transmisión
transplacentaria e implica una infección activa. AUNQUE ES EVIDENTE QUE ESTA NO ES LA
RESPUESTA CORRECTA, SE DEBE CONSIDERAR REALIZAR PRUEBAS DIAGNÓSTICAS PARA
CMV EN NIÑOS CON VIH Y CUADRO CLÍNICO SUGESTIVO DE INFECCIÓN POR ESTE VIRUS.
RECUERDA: aunque se puede solicitar la detección de anticuerpos IgG, la mejor prueba para
el diagnóstico de infección por CMV a esta edad sería una amplificación de DNA viral
mediante PCR.

IGM PARA En general, la colestasis intrahepática involucrada en todas las formas de hepatitis neonatal
ANTÍGENO C DEL se traduce en elevación en la concentración de ácidos biliares. Específicamente para esta
VIRUS DE respuesta, la evaluación del paciente con sospecha o infección crónica por el virus de la
HEPATITIS B hepatitis B (VHB) debe incluir pruebas de replicación del virus HBeAg, antiHBeAg y DNA del
VHB. También se recomienda solicitar el anticuerpo IgM para fracción core (Anti-HBc). EL
DIAGNÓSTICO MÁS PROBABLE PARA ESTE CASO ES UN SÍNDROME DE
INMUNODEFICIENCIA HUMANA, POR LO QUE ESTE MEDIO DIAGNÓSTICO NO ES DE
UTILIDAD PARA CONFIRMARLO.
ANTÍGENO En pacientes con sospecha clínica de mononucleosis infecciosa por infección del virus del
TEMPRANO DEL Epstein Barr (EBV) debe solicitarse BHC con determinación de linfocitos atípicos y búsqueda
VIRUS EPSTEIN- de linfocitosis para el incremento de probabilidad de MI, así como el cálculo del índice
BARR Linfocitos totales/leucocitos totales. Los tres criterios clásicos de laboratorio para la
confirmación de mononucleosis infecciosa son: linfocitosis, presencia de linfocitos atípicos
igual o mayor al 10% y prueba serológica positiva para EBV. Dado que la infección por EBV es
perenne, existen diversos sistemas de anticuerpos contra diferentes antígenos que pueden
indicar un determinado estadio de la infección: - Los antígenos tempranos (AE) son
producidos en los estadios iniciales de la infección lítica del virus, antes de la síntesis del
DNA. - Los antígenos tardíos son producidos después de la síntesis de DNA e incluyen los
antígenos capsulares (VCA) que son las proteínas estructurales de la misma. - Durante la
infección latente solo pocos antígenos son producidos y entre ellos el EBNA1. La
determinación de IgM-VCA es el ensayo serológico más específico que por sí mismo indica
infección aguda y es suficiente para confirmar el diagnóstico. Por su parte, la elevación de
IgG-VCA usualmente es máxima durante el estadio agudo de la enfermedad, declinando
gradualmente en los meses siguientes y permaneciendo estable el resto de la vida, por lo
tanto, aunque la IgG-VCA es positiva durante la fase aguda, su determinación única no puede
distinguir entre una infección reciente o una antigua. EL ESTUDIO DE ANTÍGENOS
ESPECÍFICOS PARA EL VIRUS DEL EPSTEIN BARR ES DE UTILIDAD PARA EL DIAGNÓSTICO DE
MONONUCLEOSIS INFECCIOSA, NO ASÍ PARA SIDA.

Bibliografía:
1. PREVENCIÓN, DIAGNÓSTICO Y TRATAMIENTO EN EL BINOMIO MADRE-HIJO CON INFECCIÓN POR EL
VIH. GUÍA DE EVIDENCIAS Y RECOMENDACIONES: GUÍA DE PRÁCTICA CLÍNICA. MÉXICO. SECRETARÍA DE
SALUD; 03/11/2016. 2. KLIEGMAN R, STANTON B, GEME J, SCR N, BEHRMAN R. NELSON TRATADO DE
PEDIATRÍA, 19ª EDICIÓN. ELSEVIER. ESPAÑA 2013. 3. PREVENCIÓN, DIAGNÓSTICO Y TRATAMIENTO DE LA
INFECCIÓN POR CITOMEGALOVIRUS EN LA EDAD PEDIÁTRICA. GUÍA DE EVIDENCIAS Y
RECOMENDACIONES: GUÍA DE PRÁCTICA CLÍNICA. MÉXICO. SECRETARÍA DE SALUD; 2012.
HTTP://WWW.CENETEC-DIFUSION.COM/CMGPC/IMSS-610-13/ER.PDF 4. PREVENCIÓN, DIAGNÓSTICO Y
TRATAMIENTO DE LA MONONUCLEOSIS INFECCIOSA. GUÍA DE EVIDENCIAS Y RECOMENDACIONES: GUÍA
DE PRÁCTICA CLÍNICA. MÉXICO. SECRETARÍA DE SALUD; 2010. HTTP://WWW.CENETEC-
DIFUSION.COM/CMGPC/IMSS-485-11/ER.PDF

http://www.cenetec-difusion.com/CMGPC/IMSS-246-12/ER.pdf

FIN DEL CASO CLÍNICO SERIADO


ANÁLISIS DEL CASO CLÍNICO

IDENTIFICACIÓN DEL REACTIVO


Area: PEDIATRÍA
Especialidad: INFECTOLOGIA PEDIÁTRICA
Tema: ARTRITIS SEPTICA Y OSTEOMIELITIS
Subtema: OSTEOMIELITIS

CASO CLÍNICO CON UNA PREGUNTA

ESCOLAR DE 8 AÑOS DE EDAD, CON DIAGNOSTICO DE OSTEOMIELITIS DE FÉMUR.

Escolar de 8 años de edad

Diagnóstico de osteomielitis

184 - EL TRATAMIENTO DE ELECCIÓN EN ESTE PACIENTE SERÁ CON:


DICLOXACILINA En la OSTEOMIELITIS, la etiología es infecciosa, siendo el germen causal más frecuente
“STAPHYLOCOCCUS AUREUS” en todas las edades. La población más afectada en orden de
frecuencia son los lactantes, niños y adultos. El tratamiento antibiótico inicial es generalmente
empírico, seleccionando el antibiótico en función de la edad y patología de base del paciente. El
tratamiento inicial de la osteomielitis debe realizarse con antibióticos parenterales para asegurar
una adecuada concentración de antibiótico en el hueso. La duración del tratamiento antibiótico
depende de la extensión de la infección, la respuesta clínica y la presencia de factores de riesgo o
patología asociada. En general es de 3 a 6 semanas, Se ha demostrado que tratamientos
inferiores a 3 semanas se asocian a un alto de recidivas (19%) frente al 2% en los niños que
reciben tratamientos índice largos. Se inicia con antibioticoterapia empírica por vía intravenosa,
de acuerdo a germen causal más común, según el grupo de edad y los factores de riesgo.
Mayores de 5 años Staphylococcus sp, Streptococcus sp (principalmente Streptococcus
pneumoniae), por lo que la DICLOXACILINA o Cefalotina son buenas opciones. IMPORTANTE: la
GPC menciona en mayores de 5 años a la CEFALOTINA como el medicamento de elección, sin
embargo, en la práctica siempre se indica primero dicloxacilina y, en este caso, la cefalotina no se
incluye en las opciones de respuesta. En el caso de que tuvieras las dos opciones, la opción más
correcta sería CEFALOTINA. IMPORTANTE: AUNQUE HOY LOS LINEAMIENTOS EN NUESTRO PAÍS
INDICAN QUE LA CEFALOTINA ES EL MEDICAMENTO SUGERIDO PARA EL MANEJO DE LA
OSTEOMIELITIS EN MAYORES DE 5 AÑOS, LAS PENICILINAS SIEMPRE SE HAN CONSIDERADO
LOS FÁRMACOS DE PRIMERA LÍNEA DEBIDO A QUE ES ACTIVO CONTRA S. PNEUMONIAE Y
ESTREPTOCOCOS DEL GRUPO A (PRINCIPALES AGENTES A ESTA EDAD), DADO QUE NO ESTÁ
COMO RESPUESTA CEFALOTINA, ÉSTA ES LA OPCIÓN MÁS CORRECTA.

CEFUROXIMA De un mes a cinco años el Staphylococcus sp, Haemophillus influenzae, Streptococcus sp, son los
gérmenes más comunes; por lo que la cefuroxima (25-50 mg/kg cada 8 horas) o penicilina G
sódica (12,500-75,000IU/kg cada 4-6 horas) con de primera elección. LA CEFUROXIMA ES EL
MEDICAMENTO DE ELECCIÓN EN LACTANTES Y NIÑOS MENORES DE 5 AÑOS.

DICLOXACILINA Las CEFALOSPORINAS DE TERCERA GENERACIÓN son resistentes a muchas beta-lactamasa y, por
MÁS tanto, son muy activas frente a enterobacterias. Este tipo de medicamentos son menos activos
CEFOTAXIMA frente a gram positivos que los fármacos de primera generación, aunque algunas veces pueden
tener buena acción contra P. pneumoniae. La suma de dicloxacilina + cefotaxima, resultan, por
etiología, ser un buen tratamiento alternativo en menores de 5 años, en quienes se sospecha la
posibilidad de Gram negativos. LAS CEFALOSPORINAS DE TERCERA GENERACIÓN SUMADAS A
UN MEDICAMENTO RESISTENTE A BETA-LACTAMASA COMPLETAN EL ESQUEMA PARA
PROTEGER CONTRA GRAM NEGATIVOS. PUESTO QUE NUESTRO PACIENTE ES MAYOR A 5 AÑOS,
ES POCO PROBABLE QUE ESTOS GÉRMENES SE ENCUENTREN PRESENTES.

CLINDAMICINA En pacientes alérgicos puede considerarse esta posibilidad. Es muy importante que recuerdes los
antibióticos antiestafilocócicos. • Clindamicina. • Fosfomicina. • Rifampicina. • Vancomicina. LA
CLINDAMICINA ES UNA BUENA OPCIÓN TERAPÉUTICA EN NIÑOS CON ALERGIA A LA
PENICILINA.

Bibliografía:
1. MANUAL DE INFECTOLOGÍA CLÍNICA. WALTER WILSON. MANUAL MODERNO. EDICIÓN 2. 2000. PÁG. 221.
2. GUÍA DE PRÁCTICA CLÍNICA, PREVENCIÓN, DIAGNÓSTICO OPORTUNO Y TRATAMIENTO DE LA
OSTEOMIELITIS HEMATÓGENA AGUDA EN POBLACIÓN GENERAL PARA EL PRIMER Y SEGUNDO NIVELES
DE ATENCIÓN. MÉXICO: SECRETARIA DE SALUD; 25/SEPTIEMBRE/2014. 3. KLIEGMAN R, STANTON B, GEME
J, SCR N, BEHRMAN R. NELSON TRATADO DE PEDIATRÍA, 19ª EDICIÓN. ELSEVIER. ESPAÑA 2013.
HTTP://HIMFG.COM.MX/DESCARGAS/DOCUMENTOS/PLANEACION/GUIAS/GTRATAOSTEOMIELITIS.PDF

http://www.cenetec-difusion.com/CMGPC/SS-111-08/ER.pdf
ANÁLISIS DEL CASO CLÍNICO

IDENTIFICACIÓN DEL REACTIVO


Area: CIRUGÍA
Especialidad: URGENCIAS
Tema: TRAUMA CRANEOENCÉFALICO
Subtema: TRAUMA CRANEOENCEFÁLICO

CASO CLÍNICO CON UNA PREGUNTA

HOMBRE DE 33 AÑOS DE EDAD, QUE ES ASALTADO AFUERA DE SU DOMICILIO, RECIBIENDO GOLPE


CONTUSO CON TUBO EN CRÁNEO. A SU LLEGADA AL HOSPITAL USTED LO ENCUENTRA DESPIERTO, CON
GLASGOW QUE FLUCTÚA ENTRE 14 Y 15. DURANTE LA EXPLORACIÓN USTED PALPA UNA FRACTURA HUNDIDA
DE CRÁNEO.

hombre de 33 años.

golpe contuso en cabeza.

sin datos de deterioro neurológico severo.

glasgow que alterna entre 14 y 15, además


de fractura hundida, lo que se traduce
como un alto riesgo de deterioro
neurológico.

-.

185 - EL SIGUIENTE HALLAZGO DURANTE EL SEGUIMIENTO Y EVALUACIÓN DEL PACIENTE, JUSTIFICARÍA


LA INDICACIÓN DE HIPERVENTILACIÓN:

APARICIÓN DE OJO. La presencia de parestesias o hemiparesias, junto con el desarrollo de dilatación pupilar o
PARESIAS Y pérdida del estado de alerta, son consideradas parte del deterioro neurológico agudo. ES MUY
PARESTESIAS. IMPORTANTE QUE RECONOZCAS QUE EL DESARROLLO DE UNA PUPILA DILATADA,
HEMIPARESIA O PÉRDIDA DEL ESTADO DE ALERTA MIENTRAS EL PACIENTE ESTA SIENDO
OBSERVADO, SON ELEMENTOS CLÍNICOS QUE DEBEN SER CONSIDERADOS PARA DETERMINAR
DETERIORO NEUROLÓGICO AGUDO, LO QUE HACE QUE ESTA OPCIÓN TAMBIÉN PUEDA SER
CONSIDERADA COMO CORRECTA. TE PEDIMOS LEAS EL ANÁLISIS DE LA SIGUIENTE OPCIÓN
PARA QUE CONOZCAS CON BASE EN QUE DECIDIMOS LA RESPUESTA CORRECTA.
DETERIORO La HIPERVENTILACIÓN debe ser utilizada con cautela en pacientes con daño cerebral severo y
NEUROLÓGICO solamente cuando se presenta DETERIORO NEUROLÓGICO "AGUDO". Se prefiere mantener
AGUDO. normocarbia en la mayoría de los pacientes. La hiperventilación actúa por medio de la reducción
de PaCO2, lo que causa vasoconstricción cerebral. La hiperventilación agresiva y prolongada
puede, en realidad, producir isquemia grave al causar vasoconstricción y, con ello, reducir la
perfusión cerebral. Esto es particularmente cierto si la PaCO2 cae por debajo de 30 mmHg. La
hiperventilación debe utilizarse con moderación y por periodos lo más breve posible. En general,
debe buscarse mantener la PaCO2 en 35 mmHg o más. Breves periodos de hiperventilación
(PaCO2 de 25 a 30 mmHg) son aceptables si se necesita tratar un deterioro neurológico agudo.
DEBE QUEDARTE CLARO QUE COMO PARTE DEL MANEJO DE LA HIPERTENSIÓN
INTRACRANEANA, DE MANERA "AGUDA" PUEDE UTILIZARSE LA HIPERVENTILACIÓN,
CONSIDERANDOSE SU EFECTO TRANSITORIO, NO MAYOR A 6 HORAS. Un examen neurológico
completo debe incluir una evaluación de las funciones sensitivas y motoras de las extremidades, y
también una reevaluación constante del estado de conciencia, además del tamaño y reflejos
pupilares. La reevaluación del paciente debe incluir los siguientes factores: - Nivel de conciencia. -
Respuesta pupilar, movimientos extraoculares y presencia o ausencia de posturas de
decorticación de descerebración. - Función motora general que incluya respuesta a estímulos. -
Reflejos tendinosos profundos que incluya el reflejo de Babinski. IMPORTANTE: las demás
opciones de respuesta son manifestaciones que pueden presentarse en casos de hipertensión
intracraneana, lo que hace a las opciones a y c correctas también. CONSIDERAMOS A ESTA
OPCIÓN LA CORRECTA DEBIDO A QUE ESTA INTEGRA TODOS LOS DATOS DE DETERIORO
NEUROLÓGICO "AGUDO" QUE INCLUYE A LOS HALLAZGOS CLÍNICOS DESCRITOS EN LAS
OPCIONES A Y C.

INTEGRACIÓN SÍNTOMAS Y SIGNOS CLÁSICOS DE HIPERTENSIÓN INTRARANEAL (HTIC) 1. CEFALEA:


DE LA TRIADA Clásicamente ha sido descrita como holocranea y de predominio matinal. Sin embargo,
DE CUSHING. habitualmente el tipo de cefalea es muy diverso. Su patogenia es el compromiso meníngeo o
vascular por el factor etiológico de la HTIC o la HTIC per se. El parénquima cerebral no posee
receptores para detectar dolor. 2. VÓMITO: El tipo de vómito clásicamente descrito es explosivo,
es decir, no precedido de náuseas. En la práctica clínica, lo habitual es que sí esté precedida de
náuseas. 3. EDEMA DE PAPILA: Se encuentra presente hasta en el 50% de pacientes con HTIC de
evolución subaguda o crónica. En pacientes con HTIC aguda se observa sólo en el 2% de los casos,
se ha postulado que estos pacientes tendrían una HTIC crónica subclínica con sintomatología
reciente. SEXTO NERVIO: La paresia del músculo recto externo uni o bilateral se debe
frecuentemente a compromiso del VI nervio. Se le denomina falso signo localizatorio pues no se
debe a una lesión de su núcleo en el puente, sino a una compresión del nervio en su trayecto.
TRIADA DE CUSHING: Cushing, a principios del siglo XX, realizó experimentos en animales, a
quienes les inflaba un balón intracraneano, estos animales presentaban una tríada:
HIPERTENSIÓN ARTERIAL, BRADICARDIA Y TRASTORNOS RSPIRATORIOS. En la práctica clínica,
rara vez se observa la tríada completa. Es más frecuente en HTIC de fosa posterior o HTIC
supratentorial aguda con grave compromiso de fosa posterior. COMPROMISO DE LA
CONSCIENCIA: Ropper describió en la tomografía axial computada de cerebro una relación entre
la desviación de línea media, a nivel de la glándula pineal, y el grado de compromiso de
conciencia TRATAMIENTO DE EMERGENCIA SIN MONITORIZACIÓN DE LA PIC Indicación: Un
paciente grave con compromiso vital, habitualmente en el servicio de urgencia o en una unidad
de tratamiento intensivo que presenta clínica de un aumento de la PIC global o de una hernia
cerebral. Objetivo: El objetivo es intentar "comprar tiempo" mientras se realiza un diagnóstico
preciso. En caso de evidenciar una lesión que provoca HTIC se debe mantener el tratamiento
hasta lograr monitorizar la PIC o someterlo a un tratamiento etiológico definitivo, por ejemplo,
drenar un hematoma. TRATAMIENTO: Cabeza en 30 grados y posición neutra. Proteger la vía
aérea. Si es necesario intubar, siempre debe usarse lidocaína 1mg/K un minuto antes de usar los
inductores para evitar el Valsalva y el enclavamiento secundario. La Succinilcolina aumenta la PIC,
mientras que la secuencia de intubación rápida disminuye el FSC. Se recomienda el siguiente
esquema: Lidocaína / Etomidato / Midazolam / Rocuronio. Hiperventilar al paciente. Su objetivo
es bajar la presión parcial de CO2 a 30 mmHg, esto provoca Vasocontricción Intracerebral lo que
disminuye el FSC y así la PIC. Su efecto es transitorio (hasta 6 horas), se utiliza mientras se inicia
un tratamiento específico. Debe controlarse con gases en sangre arterial y evitar una presión
parcial de CO2 menor a 30 mmHg ya que la vasocontricción puede llegar a un punto en que se
produce isquemia cerebral. Manitol: Se usa 1 gr/K en 30 minutos. Sodio hipertónico: Se usa 2 a 4
gramos de sodio en bolo intravenoso (ver tratamiento específico). Oxígeno: No se ha demostrado
un efecto beneficioso, se indica O2 para saturar sobre 90%. Suero fisiológico. Evitar hiperglicemia
y deshidratación. TAC de cerebro: Diagnóstico de lesiones potencialmente tratables en forma
urgente por equipo neuroquirúrgico (hematoma, hidrocefalia, tumor).
PREVENCIÓN La ACTIVIDAD CONVULSIVA que se presenta en el período inicial posterior a un traumatismo
DE CRISIS craneoencefálico puede causar daño cerebral secundario como resultado del aumento en las
CONVULSIVAS necesidades metabólicas, de la hipertensión intracraneana y del incremento en la liberación de
SECUNDARIAS. neurotransmisores. Los fármacos antiepilépticos pueden reducir las crisis convulsivas después del
traumatismo craneoencefálico, pero es necesario realizar más investigaciones para establecer si
esto lleva a una reducción en las muertes o la discapacidad. El TRAUMATISMO
CRANEOENCEFÁLICO GRAVE puede lesionar al cerebro. El daño puede empeorar con las crisis
convulsivas (descargas eléctricas anormales al cerebro) que se presentan después de la lesión
inicial. A menudo, los fármacos antiepilépticos se utilizan para tratar de prevenir las crisis
convulsivas y el daño adicional en las personas que han tenido una lesión cerebral traumática. La
revisión reveló que el uso de fármacos antiepilépticos en las primeras etapas después de la lesión
cerebral traumática reduce las crisis convulsivas. Sin embargo, todavía se necesitan más pruebas
para determinar si reducen la muerte o la discapacidad. LA PREVENCIÓN DE LAS CRISIS
CONVULSIVAS NO TIENE RELACIÓN ALGUNA CON LA HIPERVENTILACIÓN. ES IMPORTANTE QUE
RECONOZCAS LA INDICACIÓN DE LOS ANTICOMICIALES EN EL MANEJO DE LOS PACIENTES CON
TRAUMATISMO CRANEOENCEFÁLICO GRAVE.

Bibliografía:
1. COMITÉ DE TRAUMA DEL COLEGIO AMERICANO DE CIRUJANOS, PROGRAMA AVANZADO DE APOYO
VITAL EN TRAUMA PARA MÉDICOS. ATLS, 7A EDICIÓN. USA. PP 171. 2. SOTONE C, HUMPHRIES R,
DIAGNÓSTICO Y TRATAMIENTO EN MEDICIAN DE URGNEICAS. 7ª EDICIÓN. MÉXICO. MC GRAW HILL
LANGE. 2013.
ANÁLISIS DEL CASO CLÍNICO

IDENTIFICACIÓN DEL REACTIVO


Area: PEDIATRÍA
Especialidad: INFECTOLOGIA PEDIÁTRICA
Tema: GASTROENTERITIS INFECCIOSA
Subtema: GEPI VIRAL

CASO CLÍNICO SERIADO

LACTANTE DE 10 MESES DE EDAD, CON PADECIMIENTO DE 24 HRS DE EVOLUCIÓN, HA PRESENTADO 16


EVACUACIONES LÍQUIDAS, NO MOCO, NO SANGRE, VÓMITOS EN 7 OCASIONES DE CONTENIDO ALIMENTARIO.
A LA EXPLORACIÓN: TA 50/20, FC 170/MIN, FR 35/MIN, HIPOACTIVO, PULSOS APENAS PALPABLES, LLENADO
CAPILAR DE 5 SEG, OJOS HUNDIDOS, MUCOSA ORAL SECA, FONTANELA ANTERIOR HUNDIDA, ABDOMEN CON
PERISTALSIS INCREMENTADA, NO DATOS DE IRRITACIÓN PERITONEAL.

LACTANTE DE 10 MESES DE EDAD

24 HRS. DE EVOLUCIÓN

16 EVACUACIONES LíQUIDAS, NO MOCO,


NO SANGRE, VóMITOS EN 7 OCASIONES
DE CONTENIDO ALIMENTARIO

TA 50/20, FC 170/MIN, FR 35/MIN,


HIPOACTIVO,PULSOS APENAS PALPABLES,
LLENADO CAPILAR DE 5 SEG, OJOS
HUNDIDOS, MUCOSA ORAL SECA,
FONTANELA ANTERIOR HUNDIDA,
ABDOMEN CON PERISTALSIS
INCREMENTADA, NO DATOS DE
IRRITACIÓN PERITONEAL.

--

186 - LO MÁS PROBABLE ES QUE EL PACIENTE PRESENTE UNA GASTROENTERITIS SECUNDARIA A:


INFECCIÓN La DIARREA AGUDA se define como la disminución en la consistencia de las evacuaciones
VIRAL (líquidas o semilíquidas) o el incremento de la frecuencia de las mismas (más de 3 en 24hrs), que
puede acompañarse de fiebre o vómitos, su duración es por lo general menor de 7 días y no
mayor de 14 días. NOTA: no se considera diarrea a las heces de consistencia sólida, ni a las de
menor constancia o pastosa en bebés que son amamantados. El término GASTROENTERITIS se
emplea para definir a las infecciones del tracto gastrointestinal causadas por bacterias, VIRUS
(etiología más frecuente en niños) o parásitos, cuyos síntomas más frecuentes son la diarrea y el
vómito, pudiéndose presentar también fiebre y dolor abdominal. EL ROTAVIRUS ES LA PRIMERA
CAUSA DE DIARREA Y EN MENORES DE 5 AÑOS, AFECTANDO PRIORITARIAMENTE A MENORES
DE 1 AÑO, con un índice alto de mortalidad por esta causa, sobre todo en países de bajos
recursos. Dicha infección es más frecuente en los meses invernales en los climas templados. La
enfermedad tiende a ser más severa en niños de 3 a 24 meses, aunque el 25% de los casos graves
ocurre en mayores de 2 años y. casi todos los niños, presentan índices serológicos de infección a
los 4 o 5 años. Se disemina fácilmente por vía fecal-oral y es frecuente observar brotes en
hospitales y guarderías. La infección suele tener un periodo de incubación menor a 48hrs (rango
entre 1 a 7 días), característicamente los pacientes presentan fiebre (leve a moderada), vómitos y
diarrea acuosa frecuente (50 a 60% de los casos). El vómito y la fiebre son los primeros síntomas
en aparecer y desaparecen al segundo día; mientras que la diarrea suele persistir por 5 a 7 días.
OJO: las deposiciones no suelen contener sangre macroscópica ni leucocitos. La deshidratación
puede ser severa y rápidamente progresiva. Además de los lactantes, los niños con algún grado
de desnutrición, inmunocompromiso y con enfermedades subyacentes suelen ser blanco fácil este
tipo de infección y suelen presentar formas graves y prolongadas de la enfermedad. ESTE
DIAGNÓSTICO ES EL MÁS PROBABLE PRIMERO, PORQUE LOS VIRUS SON LA PRIMERA CAUSA DE
DIARREA GRAVE EN NIÑOS, LUEGO PORQUE LA INFECCIÓN POR ROTAVIRUS SUELE
ACOMPAÑARSE DE VÓMITOS Y FINALMENTE, PORQUE LA DIARREA ES ACUOSA, INTENSA, SIN
MOCO Y SIN SANGRE.

INTOLERANCIA La INTOLERANCIA A LA LACTOSA pueden deberse a una deficiencia molecular de lactasa que se
A LA LACTOSA manifiesta como diarrea crónica y, que puede ser de varios tipos. 1. La DEFICIENCIA CONGÉNITA
DE LACTASA es infrecuente y se asocia con síntomas que se producen con la exposición a la
lactosa de la leche. 2. La HIPOLACTASIA PRIMARIA DE TIPO ADULTO está causada por una
disminución fisiológica de la actividad lactasa, que se produce tras el destete en la mayoría de los
mamíferos. La lactasa en el borde de cepillo intestinal es escasa durante la vida fetal, su actividad
incrementa a finales de la vida fetal y alcanza su máximo desde el nacimiento y hasta el término
de los 3 años, para luego disminuir gradualmente con la edad. La reducción gradual varía con
entre grupos étnicos, al grado que se estima que al menos el 80% de la población adulta no
blanca en EU la padece. 3. La INTOLERANCIA SECUNDARIA A LA LACTOSA se produce tras la
lesión de la mucosa del intestino delgado (enfermedad celíaca, infección por rotavirus) y suele ser
de tipo transitoria y, mejora con la curación de la mucosa. Puede diagnosticarse por prueba de H2
en el aliento o por determinación de la actividad de lactasa en la mucosa obtenida por biopsia del
intestino delgado. LA INTOLERANCIA A LA LACTOSA PROVOCA UNA DIARREA CRÓNICA
RELACIONADA CON LA INGESTA ESPECÍFICA A LÁCTEOS, LO CUAL NO CONCUERDA CON EL
CASO.

INFECCIÓN Uno de los principales agentes parasitarios causantes de infección en niños es la ENTAMOEBA
PARASITARIA HISTOLYTICA. Aunque el 90% de las personas infectadas permanece asintomática, esta infección
puede desde una disentería aguda o fulminante, con fiebre, escalofríos y diarrea sanguinolenta o
mucoide (disentería amebiana) o hasta un ligero malestar abdominal intestinal acompañado de
diarrea con sangre y moco (colitis amebiana) que alterna con periodos de estreñimiento y
remisión, o bien, como un ameboma o una enfermedad extraintestinal (absceso amebiano
hepático). La COLITIS AMEBIANA suele tener un periodo de incubación de unas dos semanas,
aunque puede manifestarse de forma diferida durante varios meses. Tiene un inicio gradual con
dolores abdominales de tipo cólico y aumento de deposiciones (6 a 8 al día), la diarrea suele
asociarse a tenesmo y casi siempre las heces son hemo-positivas (con sangre), aunque no siempre
sea de tipo macroscópica. Los signos constitucionales (3A: Astenia, Adelgazamiento y Anorexia,
algunos autores agregan mialgias, febrícula, cefalea y diaforesis) suelen estar ausentes. Aunque
afecta a todas las edades muestra una mayor incidencia entre niños de 1 a 5 años. En lactantes y
niños pequeños pueden presentarse más frecuentemente formas graves con más afección
extraintestinal y elevadas tasas de mortalidad, especialmente en climas tropicales. La disentería
amebiana puede conducir a deshidratación y alteraciones electrolíticas. CLÁSICAMENTE LA
DIARREA POR AGENTES PARASITARIOS NO ES TAN INTENSA (MENOS DE 10 DEPOSICIONES/DÍA)
Y SUELE ACOMPAÑARSE DE PRESENCIA DE SANGRE Y/O MOCO EN HECES. LA EVOLUCIÓN ES
PROLONGADA, INSIDIOSA Y EN MUCHOS DE LOS CASOS RECIDIVANTE. Otros agentes
parasitarios comúnmente involucrados con gastroenteritis son: Criptosporidium, Cyclospora
cayetanensis, Angiostrongulus cantonensis y Giardia lamblia.
INTOXICACIÓN El nombre de ENFERMEDAD DE ORIGEN ALIMENTARIO abarca tanto las intoxicaciones
ALIMENTARIA alimentarias como las infecciones alimentarias, se aplica a todas aquellas enfermedades que se
contraen por el consumo de alimentos; incluyen también las causadas por contaminantes
químicos, como metales y otros compuestos orgánicos. Las causas más frecuentes son: 1. Toxinas
elaboradas por proliferación bacteriana en los alimentos antes de su consumo (Clostridium
botulinu, Staphylococcus aureus y Bacilus cereus). 2. Infecciones por bacterias, virus o parásitos
(amebiasis, brucelosis, enteritis por Campylobacter, diarrea por E. coli, hepatitis A, listeriosis,
salmonelosis, shigelosis, toxoplasmosis, gastroenteritis vírica, teniasis, triquinosis e infecciones
por vibrioides. 3. Toxinas producidas por algunas especies de algas nocivas (ciguatera,
intoxicaciones de tipo paralizante, neurotóxico, diarreico o causante de amnesia por consumo de
mariscos) o presentes en algunos tipos de peces (intoxicación por pez globo, intoxicación por
azaspirácido). La INTOXICACIÓN ALIMENTARIA ESTAFILOCÓCICA (no infección), tienen un
comienzo abrupto con náuseas, cólicos, vómito y postración intensos, a menudo se acompaña de
diarrea e hipotermia e hipotensión arterial. Rara vez causa la muerte. Dura uno o dos días,
aunque raramente puede prolongarse en los casos graves. La clave en estos casos es el tiempo
que transcurre entre la ingesta del alimento sospechoso y la aparición de los síntomas, lo cual
suele suceder en un promedio de 2 a 4hrs (intervalo de 30 minutos a 8hrs). El causal son las
enterotoxinas de Staphylococcus aureus, las cuales suelen ser muy estables a temperaturas de
ebullición y resisten algunos procedimientos térmicos de procesamiento de alimentos. LA
INTOXICACIÓN ALIMENTARIA NO CORRESPONDE A LA CAUSA MÁS COMÚN PARA ESTE GRUPO
DE EDAD, ADEMÁS “NO” SE REFIERE LA INGESTA DE ALIMENTOS SOSPECHOSOS NI OTROS
MIEMBROS DE LA FAMILIA QUE HAYAN INGERIDO ALIMENTOS EN COMÚN Y TENGAN UN
CUADRO CLÍNICO SIMILAR, LO CUAL ES COMÚN EN ESTOS CASOS.

Bibliografía:
1. PREVENCIÓN, DIAGNÓSTICO Y TRATAMIENTO DE DIARREA AGUDA EN PACIENTES DE 2 MESES A 5 AÑOS
DE EDAD, EN EL PRIMER Y SEGUNDO NIVEL DE ATENCIÓN. EVIDENCIAS Y RECOMENDACIONES: GUÍA DE
PRÁCTICA CLÍNICA. MÉXICO, CENETEC; 2018. 2. KLIEGMAN R, STANTON B, GEME J, SCR N, BEHRMAN R.
NELSON TRATADO DE PEDIATRÍA, 19ª EDICIÓN. ELSEVIER. ESPAÑA 2013. 3. HEYMANN, DAVID L. EL
CONTROL DE LAS ENFERMEDADES TRANSMISIBLES, 19A EDICIÓN. OPS. WASHINGTON, D.C. 2011.

http://www.cenetec-difusion.com/CMGPC/GPC-SS-156-18/ER.pdf

187 - DEBERÁ INDICARSE EL SIGUIENTE TRATAMIENTO DE INMEDIATO:


TERAPIA HÍDRICA Los principales objetivos de tratamiento de la GASTROENTERITIS VIRAL son prevenir la
INTRAVENOSA deshidratación y tratarla si ya se ha producido; como secundario se deberá procurar mantener
el estado nutricional del paciente. IMPORTANTE: los fármacos antivirales y los antibióticos NO
proporcionan ningún beneficio en estos casos. Se ha comprobado que el uso de terapia con
microorganismos probióticos (Lactobacillus) sólo es útil en casos leves no asociados a
deshidratación. OJO: la GPC no recomienda el uso preventivo de bebidas o preparados con
probióticos, prebióticos, simbióticos, ni leches fermentadas para disminuir la incidencia y el
número de episodios diarreicos; pero si promueve su uso como coadyuvante en el proceso
infeccioso agudo. Se recomienda la rehidratación oral con VSO (vida suero oral) de baja
osmolaridad como terapia de primera línea en el manejo de los niños con diarrea aguda. En
caso de intolerancia a la vía oral, se recomienda administrar VSO a través de sonda
nasogástrica. PLANES DE REHIDRATACIÓN. -PLAN A. Pacientes con diarrea aguda sin datos de
deshidratación. Continúe la alimentación habitual, administre más bebidas de lo usual para
prevenir la deshidratación. VSO 75ml (media taza) posterior a cada evacuación o vómito, en
menores de 1 año y, 150m (1 taza) en mayores de 1 año). No suspender la lactancia materna.
Evitar el consumo de bebidas carbonatadas. Consulta médica oportuna. - PLAN B: Pacientes
con diarrea aguda con datos de deshidratación, sin datos de choque. Terapia de hidratación
oral de suero vida oral de la siguiente manera. 1. Si conoce el peso del paciente: 50-100ml/kg
de peso durante 2 a 4hrs, fraccionado en 8 dosis cada 30 minutos. 2. Si no conoce el peso del
paciente: menores de 4 meses (200 a 400ml), 4 a 11 meses (600 a 1200ml), 2 a 5 años (800 a
1400ml). Con valoración y reclasificación del grado de deshidratación a las 3 o 4hrs. -PLAN C:
pacientes con diarrea aguda, inconscientes, con llenado capilar mayor a 2 segundos, con
pulsos rápidos (signos de choque). 1. Estabilice la vía aérea del paciente e inicia REPOSICIÓN
HÍDRICA INTRAVENOSA INMEDIATA. 2. Suministre solución salina o lactato de Ringer de
acuerdo al siguiente esquema: primera hora (60ml/kg/hora dividido en cargas de 20ml/kg),
segunda hora (25ml(kg/hr), tercera hora (25ml/kg/hr). 3. Reevalúe el pulso radial al finalizar
la primera carga, si aún es débil pasar la siguiente carga en 20 minutos, si mejora continúe el
volumen indicado para la primera y segunda hora. PUESTO QUE EL PACIENTE TIENE DATOS
CLAROS DE DESHIDRATACIÓN SEVERA Y CHOQUE, ES PRECISO INICIAR CON
REHIDRATACIÓN POR VIA INTRAVENOSA. NOTA: esto aplica igual para cualquier tipo de
etiología, lo más importante será revertir la deshidratación.

ANTIBIÓTICO IMPORTANTE: - La presencia de sangre en heces generalmente indica infección por patógenos
INTRAVENOSO invasivos como Shigella, Campylobacter jejuni, Salmonella o Entamoeba histolytica, sin
embargo, cuando existen alteraciones del volumen de líquidos y electrolitos en un paciente
con diarrea aguda, es más importante corregirlas que identificar el agente causal. - No se
recomienda el uso de antibióticos de forma rutinaria en diarrea aguda ya que predispone a
prolongar la enfermedad. - El uso de antibióticos se recomienda sólo en casos de diarrea
aguda causada por patógenos específicos o en situaciones especiales (diarrea con sangre y
moco, con duración mayor de 7 días, inmunocomprometidos, factores de riesgo para sepsis o
en bacteremia). EN ESTE CASO LA ETIOLOGÍA MÁS PROBABLE ES LA DE TIPO VIRAL, POR LO
QUE LOS ANTIBIÓTICOS NO SERÍAN DE UTILIDAD Y, AUNQUE ASÍ LO FUERAN, LO PRIMERO
ES SIEMPRE LA REPOSICIÓN HÍDRICA ANTES QUE LA ADMINISTRACIÓN DE TRATAMIENTO
ETIOLÓGICO.

ANTIPARASITARIO La AMEBIASIS SINTOMÁTICA debe tratarse con compuestos de acción sistémica como es el
INTRAVENOSO caso del metronidazol (30mg/kg/día, fraccionados en 3 dosis durante 8 a 10 días), seguidos
de un amebicida intraluminal (furoato de diloxanida, clefamina, etofamina, paromomicina y
teclozán) para eliminar los microorganismos que pudieran sobrevivir en el colon.
IMPORTANTE: la administración parenteral dependerá de la tolerancia a la vía oral. LOS
ANTIPARASITARIOS NO SERÍAN DE UTILIDAD PARA ESTE CASO DONDE, CLARAMENTE, SE
TRATA DE UNA GASTROENTERITIS DE TIPO VIRAL.

AYUNO Y Como recomendación de la OMS, en todos los niños con diarrea crónica debe ofrecerse una
REINICIO DE dieta libre de lactosa como parte inicial del manejo. La lactosa en estos casos puede sustituirse
ALIMENTACIÓN por maltodextrina o por una combinación de carbohidratos complejos. En el caso específico
CON LECHE de INTOLERANCIA A LA LACTOSA, está indicado reducir la ingesta de lactosa y añadir
DESLACTOSADA comprimidos de lactasa, en caso de no haber mejora suprimir por completo la lactosa en la
dieta. IMPORTANTE: en las infecciones por ROTAVIRUS puede haber una intolerancia
temporal a la lactosa, por lo que esta puede ser una buena estrategia para acompañar el
manejo de pacientes con gastroenteritis viral, pero no antes de la reposición de líquidos.

Bibliografía:
1. PREVENCIÓN, DIAGNÓSTICO Y TRATAMIENTO DE DIARREA AGUDA EN PACIENTES DE 2 MESES A 5 AÑOS
DE EDAD, EN EL PRIMER Y SEGUNDO NIVEL DE ATENCIÓN. EVIDENCIAS Y RECOMENDACIONES: GUÍA DE
PRÁCTICA CLÍNICA. MÉXICO, CENETEC; 2018. 2. KLIEGMAN R, STANTON B, GEME J, SCR N, BEHRMAN R.
NELSON TRATADO DE PEDIATRÍA, 19ª EDICIÓN. ELSEVIER. ESPAÑA 2013. 3. HEYMANN, DAVID L. EL
CONTROL DE LAS ENFERMEDADES TRANSMISIBLES, 19A EDICIÓN. OPS. WASHINGTON, D.C. 2011.

http://www.cenetec-difusion.com/CMGPC/GPC-SS-156-18/ER.pdf

FIN DEL CASO CLÍNICO SERIADO


ANÁLISIS DEL CASO CLÍNICO

IDENTIFICACIÓN DEL REACTIVO


Area: CIRUGÍA
Especialidad: CIRUGÍA ABDOMINAL
Tema: PATOLOGÍA ABDOMINAL RESOLUCIÓN QUIRÚRGICA DE URGENCIA
Subtema: HERNIA INGUINAL

CASO CLÍNICO CON UNA PREGUNTA

MASCULINO DE 59 AÑOS DE EDAD, CON ANTECEDENTE DE ANGINA INESTABLE POR LA CUAL TOMA ÁCIDO
ACETILSALICÍLICO DIARIO. TAMBIÉN ES DIABÉTICO CONTROLADO CON METFORMINA Y GLIBENCLAMIDA.
ACTUALMENTE SE ENCUENTRA HOSPITALIZADO PROGRAMADO PARA HERNIOPLASTÍA INGUINAL.

MASCULINO DE 59 AÑOS DE EDAD.

TOMA ÁCIDO ACETILSALICÍLICO.

PROGRAMADO PARA HERNIOPLASTÍA


INGUINAL.

-.

-.

188 - ESTE PACIENTE DEBERÁ EVITAR LA INGESTIÓN DE ÁCIDO ACETILSALICÍLICO DURANTE EL


PREOPERATORIO INMEDIATO YA QUE FAVORECERÍA EN ÉL LA PRESENTACIÓN DE:

HIPOGLUCEMIA. El uso de los salicilatos, inhibe la síntesis de la prostaglandina E2 y aumentan de forma


indirecta la secreción de insulina. De esta manera, los salicilatos reducen los niveles de
glucosa en sangre. Este mecanismo explica porque los salicilatos aumentan los efectos
hipoglucemiantes de las sulfonilureas, aunque esta potenciación puede ser también debida
a un desplazamiento de las sulfonilureas de las proteínas del plasma a las que se encuentran
unidas. Por eso en estos casos se debe de suspender el ácido acetilsalicílico y evitar la
hipoglucemia que puede ser severa y traer consigo complicaciones importantes. DEBES
TENER EN CUENTA QUE EL EFECTO HIPOGLUCEMIANTE SE PRESENTA EN DOSIS ALTAS Y
LAS DOSIS PROFILÁCTICAS DE ÁCIDO ACETILSALICÍLICO SON MUY BAJAS.
ACIDOSIS Los salicilatos en dosis tóxicas estimulan el sistema nervioso central directamente, causando
METABÓLICA. hiperpnea y un trastorno metabólico con acumulación de ácidos orgánicos. También
disminuyen la protrombina al dificultar la utilización de la vitamina K en el hígado. En
presencia de ácido gástrico, la aspirina produce lesiones en la mucosa con la hemorragia
consecuente. Las manifestaciones más importantes en el envenenamiento por salicilatos son
la hiperpnea, y los trastornos en el equilibrio ácido básico. La sintomatología se hace
evidente después de 3 horas de la ingesta, con vómito, dolor urente en el abdomen,
hiperpnea que puede ser ligera o moderada, letargia, zumbido de oídos y vértigo. Si el
paciente no recibe atención médica, el cuadro evoluciona con la presencia de convulsiones,
cianosis, oliguria, edema pulmonar e insuficiencia respiratoria. DATOS DE LABORATORIO
Gases arteriales. Valores inferiores a 8 mEq/L de bicarbonato sanguíneo, son indicativos de
una acidosis severa. El pH, la PaO2 y la PaCO2 permiten establecer la fase de la intoxicación
FASES DE LA INTOXICACIÓN 1. Alcalosis respiratoria: dura aproximadamente 12 horas 2.
Alcalosis repiratoria y aciduria paradójica 3. Acidosis metabólica LA ALCALOSIS
METABÓLICA SE PRESENTA EN CASOS GRAVES DE INTOXICACIÓN POR SALICILATOS,
NUEVAMENTE RECUERDA QUE LAS DOSIS PROFILÁCTICAS PARA ANGINA DE PECHO SON
PEQUEÑAS.

ANTICOAGULACIÓN. JUSTO ES ÉSTA EL EFECTO QUE SE BUSCA CUANDO SE INDICA ÁCIDO ACETILSALICÍLICO EN
PACIENTES CON SÍNDROME METABÓLICO, Y ES TAMBIÉN LA RAZÓN POR LA QUE DEBE
SUSPENDERSE PREVIO A INTERVENCIONES QUIRÚRGICAS O DENTALES. El ácido
acetilsalicílico dentro de sus efectos también es antiagregante plaquetario dosis bajas de
aspirina, de 81 mg diarios, producen una leve prolongación en el tiempo de sangrado, que
se duplica si la administración de la aspirina continúa durante una semana. El cambio se
debe a la inhibición irreversible de la COX de las plaquetas, por lo que se mantiene durante
toda la vida de las mismas (entre 8 y 10 días). Esa propiedad anticoagulante hace que la
aspirina sea útil en la reducción de la incidencia de infartos en algunos pacientes. 40 mg de
aspirina al día son suficientes para inhibir una proporción adecuada de tromboxano A2, sin
que tenga efecto inhibitorio sobre la síntesis de prostaglandina I2, por lo que se requerirán
mayores dosis para surtir efectos antiinflamatorios. Como en este caso el paciente tiene el
antecedente de angina inestable manejada con el ácido acetilsalicílico, dicho medicamento
tendrá que suspenderse.

HIPERGLUCEMIA. En grandes dosis, los salicilatos desacoplan la fosforilización oxidativa, ocasionando una
depleción del glucógeno hepático y muscular y, por tanto, hiperglucemia y glucosuria. Por
lo tanto, después de una dosis muy alta, la aspirina puede causar bien hipoglucemia o bien
hiperglucemia. NO SE DEBERÁN ELEGIR LAS RESPUESTAS QUE INCLUYEN HIPOGLUCEMIA E
HIPERGLUCEMIA PUES SABEMOS QUE LAS DOSIS DE ÁCIDO ACETILSALICÍLICO PARA EL
MANEJO PROFILÁCTICO DE LA ANGINA SON PEQUEÑAS.

Bibliografía:
1. GOLAN D, TASHJIAN A. AMSTRONG E, AMSTRONG A. PRINCIPIOS DE FARMACOLOGÍA, 3A EDICIÓN.
LIPPINCOTT CILLIAMS & WILKINS. 2012, PP 754-755.
ANÁLISIS DEL CASO CLÍNICO

IDENTIFICACIÓN DEL REACTIVO


Area: GINECOLOGÍA Y OBSTETRICIA
Especialidad: GINECOLOGÍA
Tema: LEUCORREA
Subtema: VAGINOSIS BACTERIANA

CASO CLÍNICO CON UNA PREGUNTA

A 24-YEAR-OLD SEXUALLY ACTIVE WOMAN REPORTS INCREASED VAGINAL DISCHARGE. PELVIC EXAMINATION
SHOWS A GREEN FROTHY DISCHARGE. MICROSCOPIC EXAMINATION OF A WET MOUNT OF THE DISCHARGE
SHOWS MOTILE UNICELLULAR ORGANISMS 10-30 MICROMETER IN LENGTH.

Una mujer sexualmente activa de 24 años.

presenta un aumento en sus secreciones


vaginales El examen pélvico revela una
secreción verde y espumosa.

-.

El examen microscópico de una


preparación en fresco de la secreción
revela organismos unicelulares móviles de
10 a 30 ?MICRÓMETROS de longitud.

-.

189 - WHICH OF THE FOLLOWING FEATURES OF THIS DISORDER BEST EXPLAINS WHY THE INFECTION
CAN BE EFFECTIVELY TREATED WITH METRONIDAZOLE?

FACULTATIVELY PREGUNTA: ¿Cuál de las siguientes características de esta patología explica mejor el por qué
AEROBIC la infección puede ser efectivamente tratadas con metronidazol? Facultativamente aerobio,
no es una característica de las Trichomonas.
MICROAEROPHILIC Microaerófilo, o es una característica de las Trichomonas. REPASO TRICOMONIASIS. El
diagnóstico de TRICOMONIASIS se caracteriza por presentar leucorrea abundante, maloliente
y espumosa, con prurito y ardor vulvovaginal, así como dispareunia, disuria y flujo, la mucosa
está hiperémica, moteada por petequias; cérvix en “fresa” o zonas hemorrágicas, "leucorrea
olorosa, espuma blanca o amarrilla". Es importante considerar que hasta un tercio de las
mujeres son totalmente asintomáticas. Diagnóstico diferencial por laboratorio. Se puede
iniciar desde el examen de la secreción durante la toma de una muestra. Esta prueba se lleva a
cabo colocando una gota de KOH al 10% en el espejo vaginal mezclando el fluido vaginal con
una gota de KOH, el cual debido a que tiene propiedades alcalinas, causa la producción de
aminas dependientes del metabolismo de las bacterias anaeróbicas [2]. Ya en el microscopio,
primero se localiza el campo en potencia baja (Fijación del objetivo 10x) para detectar
trichomonas o yemas de levaduras y seudohifas. Luego se ubica el campo en potencia alta
(Fijación del objetivo 40x) para detectar los organismos relacionados con vaginosis: Células
clave, leucocitos, lactobacilos, además de otras bacterias en el medio. Los hallazgos
microscópicos típicos, permiten diferenciar las secreciones normales de las de origen
infeccioso. Régimen recomendado para las infecciones vaginales. • Metronidazol, 2 g por vía
oral, en dosis única. • Tinidazol 2 g por vía oral, en dosis única. Nota: - La tasa de curación que
se registra para las mujeres varía entre el 82% y el 88% pero puede aumentar al 95% si las
parejas sexuales reciben tratamiento simultáneo. Régimen alternativo. • Metronidazol 400 mg
o 500 mg por vía oral, dos veces al día durante 7 días. • Tinidazol 500 mg por vía oral, dos
veces al día durante 5 días Los pacientes que no logran la cura con la repetición del ciclo de
Metronidazol pueden recibir un tratamiento que consiste en la administración de 2 g de
Metronidazol diarios por vía oral, más 500 mg aplicados por vía intravaginal todas las noches
durante 3 a 7 días. Las preparaciones de Metronidazol para uso vaginal están disponibles en
muchos países, pero sólo se recomiendan para el tratamiento de las infecciones refractarias y
no como tratamiento primario de la Trichomoniasis. Un régimen alternativo consiste en la
administración de 400 mg ó 500 mg de Metronidazol por vía oral, dos veces al día durante
siete días.

STRICTLY AEROBIC Aerobio estricto, no es una característica de las Trichomonas. REPASO IMIDAZOLES. Los
IMIDAZOLES son intermediarios de la biosíntesis de la histidina que se forma desde el
Imidazol glicerol fosfato con la pérdida de agua. Su molécula ha servido de base para el
desarrollo de numerosos fármacos. Es un ejemplo de medicamento antibacteriano y
antiprotozoario el "Metronidazol", siendo de primera elección en el tratamiento de las
pacientes con cervicovaginitis bacteriana o por Trichomona. El METRONIDAZOL, ejerce su
acción antibacteriana y antiprotozoaria por desestructuración del ADN. Tras ingresar en la
célula mediante difusión pasiva es químicamente reducido por proteínas del metabolismo
anaerobio (proteínas de transporte de electrones de bajo potencial redox). Estas proteínas
son exclusivas de algunos parásitos y de bacterias anaerobias y algunas microaerófilas. El
Metronidazol reducido produce pérdida de la estructura helicoidal del ADN, rotura de la
cadena e inhibición de la síntesis de ácidos nucleicos y muerte celular, generando compuestos
que son tóxicos para la célula. El Metronidazol se distribuye bien en la mayoría de los líquidos
y tejidos corporales debido al bajo peso molecular y a la escasa unión del fármaco a proteínas
plasmáticas. Su concentración terapéutica se alcanza en secreciones vaginales, líquido
seminal, saliva, leche, líquidos de derrame, miometrio, trompas de Falopio, entre otros. Su
dosis para adultos por vía oral es de 2 g por 2 días o 500 mg cada 12 horas, durante 7 días y
su dosis máxima es de 4 g por día; por vía intravaginal en gel es de 5 g durante 7 días. No se
recomienda su uso en el primer trimestre de embarazo (en otras fases del embarazo se ha
consumido, al parecer sin efectos adversos manifestados. IMPORTANTE: Se considera que el
95% de Trichomonas son sensibles al tratamiento con metronidazol.

STRICTLY IMPORTANTE: LO PRIMERO QUE DEBERÁS HACER EN ESTE CASO ES DETERMINAR EL TIPO DE
ANAEROBIC INFECCIÓN DE LA CUAL SE TRATA, EN ESTE CASO ES UNA TRICOMONIASIS, EL HALLAZGO DE
ORGANISMOS UNICELULARES MÓVILES + SECRECIÓN VERDE ESPUMOSA LO SUSTENTAN.
REPASO TRICOMONA. La TRICHOMONAS VAGINALIS es un protozooo flagelado, tienen por
característica ser flagelado, móvil y piriforme. Su tamaño es de aproximadamente 10
micrómetros. Carece de mitocondrias por lo que realiza oxidación anaerobia a partir de unos
organelos hidrogenosomas. LA CARENCIA DE MITOCONDRIAS VUELVE A LA TRICOMONA EN
UN ANAEROBIO ESTRICTO. IMPORTANTE: EL METRONIDAZOL ES UN MEDICAMENTO
ALTAMENTE EFICAZ PARA EL TRATAMIENTO DE ORGANISMOS ANAEROBIOS, POR ESA
RAZÓN ES IDEAL PARA EL MANEJO DE LA TRICOMONIASIS DE ESTE CASO CLÍNICO.

Bibliografía:
1. GUÍA DE PRÁCTICA CLÍNICA, DIAGNÓSTICO Y TRATAMIENTO DE LA VAGINITIS INFECCIOSA EN MUJERES
EN EDAD REPRODUCTIVA EN EL PRIMER NIVEL DE ATENCIÓN. MÉXICO: SECRETARIA DE SALUD;
DICIEMBRE 2014. 2. GOLAN D, TASHJIAN A. AMSTRONG E, AMSTRONG A. PRINCIPIOS DE FARMACOLOGÍA,
3A EDICIÓN. LIPPINCOTT CILLIAMS & WILKINS. 2012. 3. LONGO DL, FAUCI AS, KASPER DL, HAUSERSL,
JAMESON JL, LOSCALZOJ. HARRISON. PRINCIPIOS DE MEDICINA INTERNA, 18A EDICIÓN. MC GRAW HILL.
NEW YORK, USA. 2012.

http://www.cenetec.salud.gob.mx/descargas/gpc/CatalogoMaestro/081_GPC_Vaginitisinfec1NA/Vaginitis_ER_CENETEC.pdf
ANÁLISIS DEL CASO CLÍNICO

IDENTIFICACIÓN DEL REACTIVO


Area: MEDICINA INTERNA
Especialidad: DERMATOLOGÍA
Tema: TRANSTORNOS DESCAMATIVOS
Subtema: DERMATITIS ATÓPICA

CASO CLÍNICO SERIADO

PACIENTE MASCULINO DE 21 AÑOS DE EDAD, PRESENTA DERMATOSIS CRÓNICA, CON EXACERBACIONES Y


REMISIONES, DISEMINADA EN LOS PLIEGUES DE FLEXIÓN DE LOS CODOS, HUECOS POPLÍTEOS Y EL CUELLO.
SE CARACTERIZA POR PLACAS DE ECZEMA CON ERITEMA Y ESCAMA, MUY PRURIGINOSAS.

masculino de 21 años de edad.

-.

presenta dermatosis crónica, con


exacerbaciones y remisiones, diseminada
en los pliegues de exión de los codos,
huecos poplíteos y el cuello.se caracteriza
por placas de eczema con eritema y
escama, muy pruriginosas.

-.

-.

190 - EL DIAGNÓSTICO CLÍNICO MÁS PROBABLE ES:


DERMATITIS Las DERMATITIS POR CONTACTO (DxC) son un conjunto de patrones de respuesta inflamatoria de
POR la piel que ocurren como resultado del contacto de la misma con factores externos como alérgenos
CONTACTO. e irritantes. La palabra dermatitis sugiere un criterio amplio de dermatosis inflamatoria, ya que se
trata de un espectro que excede al eczema. De todas maneras las manifestaciones clínicas más
frecuentes de las Dermatitis por Contacto Irritativa y Alérgica son las formas eccematosas y su
etiología sólo puede dilucidarse en base a la conjunción de los antecedentes y las pruebas del
parche. Las dermatitis por contacto se clasifican en: 1- Dermatitis irritativa por contacto (DIC) 2-
Dermatitis alérgica por contacto (DAC) 3- Reacciones inmediatas por contacto (RIC) 4- Reacciones
fotoalérgicas y fototóxicas por contacto (FAxC y FTxC) 5- Reacciones no eccematosas por contacto
(RNEC) 6- Dermatitis por contacto sistémica (DCS) La dermatitis por contacto irritativa es la más
frecuente y en menor porcentaje el resto de las dermatitis de contacto: La DCS la forma más
frecuente de dermatitis por contacto en todas las edades y mucho más en los niños. Las lesiones se
circunscriben a la zona donde se produce el contacto de la piel con los irritantes. En las formas
agudas pueden verse pápulas o placas eritematosas y edematosas y más adelante vesículas o
ampollas, exudación, erosiones y costras. En las formas acumulativas o crónicas, además de este
último grupo de lesiones, puede predominar un aspecto descamativo, liquenificación y formación
de grietas o fisuras. Subjetivamente puede producir prurito, escozor, quemazón o incluso dolor. En
los niños pequeños la orina y las heces, cuyo contacto con la piel es responsable de la dermatitis del
pañal, y la saliva, que origina lesiones en la cara y el cuello, son los principales desencadenantes. En
niños más mayores y adolescentes la saliva causa también queilitis descamativa y lesiones
periorales. La orina y las heces continúan actuando como irritantes en casos de incontinencia o
enuresis. Otros irritantes a tener en cuenta en la infancia son jabones, detergentes, pinturas,
disolventes, el agua, la tierra, las tizas, plantas, maderas y soluciones azucaradas. Algunas de ellas se
manipulan en actividades escolares o complementarias. Los niños atópicos son muy propensos a
reacciones irritativas por alimentos como tomate, naranjas, limones, piña, fresas, etc. y por la lana y
otras prendas textiles Para realizar el diagnóstico es indispensable una historia clínica así como la
exploración que son la base del diagnóstico de las dermatitis por contacto. La analítica no aporta
datos significativos y la biopsia muestra una imagen de dermatitis espongiótica y en ocasiones
necrosis epidérmica; ambas pueden servir para descartar otros procesos, pero raramente orientan a
un diagnóstico no sospechado por la clínica. En las dermatitis por contacto irritativas y fototóxicas
no existen otras pruebas complementarias con valor diagnóstico utilizables en la práctica diaria En
las dermatitis por contacto irritativas y fototóxicas no existen otras pruebas complementarias con
valor diagnóstico utilizables en la práctica diaria. Para el diagnóstico de la dermatitis por contacto
alérgica se utilizan las pruebas epicutáneas o del parche, consistentes en la aplicación de los
supuestos alérgenos responsables sobre la piel del paciente. Aunque en determinadas situaciones se
emplean pruebas abiertas y otras modalidades, lo más común es mantener las sustancias
investigadas en contacto con la piel durante 48 horas, empleando apósitos o recipientes adecuados.
Pasado este tiempo se retiran los alérgenos y se evalúa la respuesta al cabo de 1/2-1hora,
evaluación que se repite a las 72-96 horas. La respuesta positiva implica una reacción eccematosa en
la zona de aplicación y su periferia. Las lesiones de las dermatitis por contacto foto alérgica también
pueden reproducirse mediante pruebas de foto parche, similar a las anteriores, pero en las que tras
retirar los alérgenos se irradia la zona de aplicación con una o varias longitudes de onda lumínica.
NO HAY ANTECEDENTE DE AGENTE DE CONTACTO QUE HAYA PRODUCIDO DICHAS LESIONES.
DERMATITIS La DERMATITIS ATÓPICA (DA) es un proceso inflamatorio cutáneo crónico, intensamente
ATÓPICA. pruriginoso, de carácter recurrente, usualmente aparece durante la infancia temprana y la niñez
pero puede persistir o comenzar en la vida adulta. La dermatitis atópica es un proceso multifactorial
donde intervienen factores intrínsecos (alteración genética, trastornos inmunológicos, piel alterada)
y factores extrínsecos que pueden desencadenar o exacerbar el brote (alérgenos alimentarios,
aeroalérgenos, microrganismos). La prevalencia de la dermatitis atópica varia de unas zonas
geográficas a otras y en general es mayor cuanto más al norte se sitúa el área estudiada y cuanto
más alto es su nivel de desarrollo industrial y parece ir aumentando, no hay un entendimiento claro
de los factores determinantes del incremento. Se han encontrado en algunas poblaciones
manifestaciones de dermatitis atópica en la edad infantil entre un 5 a un 10 %, sin diferencias
significativas en relación con el sexo. Las estadísticas revelan una prevalencia infantil global de 10%
a 20% y en los adultos de 1 a 3%. Entre un 70 a 80% de los pacientes tienen una historia familiar
positiva de atopía. La influencia genética es compleja siendo el componente materno más
importante. Los atópicos presentan una respuesta inmunitaria humoral y celular alterada, que
facilita la reacción con antígenos ambientales. Con las siguientes alteraciones: Aumento de la Ig E
sérica, alteración de las subpoblaciones linfocitarias ,alteración de las inmunoglobulinas, aumento
de la expresión en la membrana de las células de Langerhans, alteraciones de la reactividad vascular
y farmacológica, los vasos de estos pacientes tienen tendencia a la vasoconstricción, alteraciones de
la fisiología de la piel, disminución del umbral del prurito ,alteración en la eliminación de la
sudoración ,alteración del manto lipídico de la piel. Clásicamente la Dermatitis Atópica típica se fija
su inicio al final de la fase infantil a partir de los 10 años o en la pubertad. Sus áreas de predilección
son: cara (frente, parpados, región perioral), cuello (especialmente nuca), parte alta del tórax y
hombros, grandes pliegues flexurales y dorso de las manos. Las lesiones características son las
placas de liquenificación. Pueden aparecer también eczema de manos, dishidrosis y prurigo
nodular. La mayor parte de los pacientes evolucionan hacia la resolución antes de los 20 años,
siendo muy infrecuentes las manifestaciones de la enfermedad después de los 30 años Las
manifestaciones clínicas son la base del diagnóstico, ya que ni la histología ni ninguna otra prueba
de laboratorio son específicas de la enfermedad. El diagnóstico se fundamenta en criterios clínicos y
son necesarios 3 criterios mayores y 3 criterios menores. Criterios mayores: Prurito, dermatitis
crónica recurrente, morfología y distribución característica, historia personal o familiar de atopia.
Criterios menores: Xerosis o piel seca, queratosis pilar/exageración de pliegues palmares, pitiriasis
alba, dermatitis inespecífica de manos y pies, eczema del pezón, queilitis, acentuación perifolicular,
conjuntivitis recidivante, intolerancia a la lana y a los disolventes de las grasas, oscurecimiento
periocular, queratocono, catarata subcapsular anterior, pliegue de Dennie-Morgan, pliegue del
cuello, fisuras infraauriculares, edad temprana de inicio, tendencia a infecciones cutáneas, aumento
de los niveles séricos de IgE, reactividad inmediata tipo I en los tests cutáneos, tendencia a
infecciones cutáneas y déficit de la inmunidad celular, intolerancia a algunos alimentos, reactivación
por causa ambiental o emocional, dermografismo blanco, blanqueamiento retardado,
vasoconstricción/palidez facial, cambio de la temperatura de los dedos, sudoración anormal con
prurito, disminución de la actividad de las glándulas sebáceas, personalidad atópica. La
determinación de la Ig E suele mostrar un nivel aumentado, pero hay que tener en cuenta que un
20% de los atópicos pueden mostrar determinaciones normales, y al revés, otras patologías e
incluso individuos sanos pueden mostrar una elevación de esta Ig. LA DISTRIBUCIÓN DE LAS
LESIONES SON DE GRAN PESO PARA ORIENTAR ESTE DIAGNÓSTICO.

DERMATITIS La DERMATITIS SEBORRÉICA (DS) o eczema seborreico constituye una dermatosis inflamatoria
SEBORRÉICA. crónica frecuente y recurrente en regiones cutáneas rica en glándulas sebáceas y eventualmente en
algunas áreas intertriginosas, primordialmente en cara, cuero cabelludo y pecho. La lesión
elemental es la placa descamativa y la enfermedad se agrupa dentro de las afecciones
eritematoescamosas. La dermatitis seborreica afecta entre el 1% y el 3% de la población
inmunocompetente. Es más frecuente en hombres que en mujeres y afecta más a a la raza negra.
Suele presentar dos picos de edad, uno entre los adolescentes y adultos jóvenes y otro en pacientes
mayores de 50 años. La causa de la dermatitis seborreica se desconoce, sin embargo, diversos
factores como la seborrea, cambios hormonales, alteraciones neurológicas, microrganismos como
las levaduras, variaciones estacionales y el estrés emocional constituyen factores condicionantes o
exacerbantes de la dermatosis. El signo clínico más importantes es la inflamación que se manifiesta
clínicamente como finas placas descamativas grasosas en ocasiones bien delimitadas que varían
desde el color rosado-amarillo hasta el rojo marrón, con predilección a áreas ricas en glándulas
sebáceas como son cuero cabelludo, región centro facial de cara, cejas, región preesternal, región
interescapular y menos frecuente área intertriginosas como axilas, región inguinal, pliegue
inframamario y ombligo. El prurito puede ir desde una leve molestia hasta sensación de ardor en
formas severas o generalizadas. LA DERMATITIS SEBORRÉICA SE CARACTERIZA POR PLACAS
GRASOSAS CON GRAN ERITEMA, NO APLICA PARA ESTE CASO.
PSORIASIS. La PSORIASIS es una enfermedad crónica, determinada genéticamente, caracterizada por sucesivos
brotes de placas eritemato-descamativas en las superficies extensoras de la piel y cuero cabelludo,
siendo la artropatía la única manifestación extracutánea. Aunque la etiología es desconocida, existe
una clara agregación familiar que apoya la implicación de factores genéticos. Además, factores
ambientales conocidos como los traumatismos, diálisis, la hipocalcemia, algunas infecciones como
la faringitis estreptocócica o el VIH, fármacos como el litio, betabloqueantes, antipalúdicos,
interferón, interleucina 2, antidiabéticos orales, la retirada de corticoides sistémicos, algunos
antinflamatorios (indometacina) y, por último, las situaciones de estrés, pueden ser
desencadenantes de la enfermedad. La psoriasis se produce como consecuencia de la inter acción de
3 fenómenos que concurren simultáneamente: 1. aumento de la velocidad de crecimiento
epidérmica; 2. proliferación venular postcapilar en las papilas dérmicas; y 3. Reacción inmune
mediada por linfocitos T. La consecuencia de esta interacción es el fenómeno de la exudación cíclica
papilar, que caracteriza microscópicamente a la enfermedad. Es una enfermedad universal con una
prevalencia de alrededor del 2% de la población europea, y con una incidencia de 150.000 casos
nuevos/año. Puede debutar a cualquier edad, aunque cuando existen antecedentes familiares
aparece de forma más temprana. Existen dos picos de incidencia: a los 16-22 años y a los 57-60
años, sin diferencia entre sexos, aunque las mujeres desarrollan antes la enfermedad. La lesión
elemental es una placa eritematosa, de bordes bien definidos y superficie irregular descamativa,
que tras el raspado metódico deja una superficie eritematosa con pequeños puntos sangrantes,
rocío hemorrágico de Auspitz”. Se disponen simétricamente en superficies extensoras y en cuero
cabelludo. Hasta un tercio de los enfermos presentan lesiones en zonas previamente traumatizadas
(fenómeno de Koebner o isomorfismo). Se han descrito distintas formas clínicas: 1. Psoriasis vulgar:
es la forma más frecuente. Las lesiones son crónicas y se localizan además de en los lugares ya
mencionados, en la piel de abdomen y sacro. 2. Psoriasis en gotas: las lesiones son lenticulares o
puntiformes, en tronco, generalmente en niños y adultos jóvenes, cursando en brotes. Tiene buen
pronóstico y se ha relacionado con infección faríngea estreptocócica previa 3. Psoriasis invertida
suele afectar grandes pliegues con predominio del eritema sobre la descamación 4. Psoriasis
palmoplantar, con placas eritematodescamativas con fisuración. 5. Psoriasis ungueal: aparece hasta
en un 35% en pies y un 50% en manos de enfermos con psoriasis. Es más frecuente en los casos de
eritrodermia o artropatía asociada, aunque se puede presentar aislada. El espectro clínico va desde
lesiones puntiformes en la lámina ungular (pitas) a onicodistrofia intensa con incluso pérdida
ungueal, pasando por la característica “mancha de aceite” o mancha marrón distal de la lámina. 6.
Psoriasis pustulosa, que puede presentar varias formas: 6.1. Generalizada tipo von Zumbusch:
Puede ser el debut de la enfermedad, asociada frecuentemente a artropatía, ser la evolución de una
forma pustulosa localizada o estar desencadenada por fármacos (corticoides orales, litio,
fenilbutazona), infecciones o embarazo (impétigo herpetiforme). De forma súbita, aparecen
lesiones eritematosas confluentes que pueden evolucionar a eritrodermia sobre las que surgen
brotes sucesivos de pústulas blanquecinas, anulares, agrupadas o dispersas. Asocia fiebre, malestar
general, y leucocitosis con desviación a la izquierda. Suele respetar palmas y plantas, aunque es
común la afectación ungueal. 6.2. Anular: puede ser generalizada o localizada. Son lesiones anulares
con borde eritematoso y collarete descamativo interior con alguna pústula aislada; el centro es
rosado con descamación 6.3. Localizadas, con 2 formas: a) Pustulosis palmoplantar, de predominio
en adultos y en el sexo femenino. No está clara su relación con la psoriasis. De hecho, sólo un 24%
de estos enfermos tienen antecedentes de psoriasis; b) Acrodermatitis continua de Hallopeau,
caracterizada por pústulas en las falanges distales de los dedos (pulgares), que evolucionan
dejando, al vaciarse, una superficie eritematosa brillante atrófica. Existe afectación ungüeal con
onicodistrofia e incluso desaparición de la uña por daño de la matriz. Puede asociarse a otras
lesiones a distancia de psoriasis vulgar o pustuloso e incluso generalizarse a una forma de von
Zumbusch. 7. Eritrodermia psoriásica: eritrodermia exfoliativa no pruriginosa, con afectación del
estado general Las lesiones son de borde neto, con afectación ungueal importante. Suele aparecer
en enfermos con psoriasis crónicas intensas. 8. Artropatía psoriásica: Esta artritis seronegativa
afecta del 5 al 8% de los pacientes con psoriasis. En un 70% afecta de forma asimétrica a las
pequeñas articulaciones de los dedos y en el resto aparece como poliartritis simétrica, espondilitis
anquilosante y artritis mutilante. Ocurre en la cuarta y quinta década, y en un 50% de los casos de
forma aguda. La psoriasis cutánea precede a la artropatía y la intensidad de la clínica articular es
independiente de la afectación cutánea, aunque más grave en las formas pustulosas. Es muy
frecuente la afectación ungueal, incluso como única manifestación cutánea. LA PSORIASIS
CARACTERÍSTICAMENTE MUESTRA LESIONES EN CUERO CABELLUDO, NO COINCIDE CON EL
CUADRO CLÍNICO DEL PACIENTE.

Bibliografía:
1. GUÍA DE PRÁCTICA CLÍNICA, DIAGNÓSTICO Y MANEJO DE LA DERMATITIS ATÓPICA DESDE EL
NACIMIENTO HASTA LOS 16 AÑOS DE EDAD EN EL PRIMER NIVEL DE ATENCIÓN. MÉXICO: SECRETARIA DE
SALUD, 2009. 3. LONGO DL, FAUCI AS, KASPER DL, HAUSERSL, JAMESON JL, LOSCALZOJ. HARRISON.
PRINCIPIOS DE MEDICINA INTERNA, 18A EDICIÓN. MC GRAW HILL. NEW YORK, USA. 2012, PP 395.
http://www.cenetec.salud.gob.mx/descargas/gpc/CatalogoMaestro/033_GPC_DermAtopPed/IMSS_033_08_EyR.pdf

191 - EL FACTOR PRINCIPALMENTE IMPLICADO EN LA FISIOPATOLOGÍA DE LA ENFERMEDAD ES:

ACELERACIÓN EN La PSORIASIS es una enfermedad inmunológica mediada por los linfocitos T y en la que el
EL RECAMBIO DE recambio celular de los queratinocitos está acelerado. CORRESPONDE A LA FISIOPATOLOGÍA
LOS DE LA PSORIASIS.
QUERATINOCITOS.

EXPOSICIÓN A La DERMATITIS POR CONTACTO se debe a la acción irritante directa de las sustancias
ALERGENOS E desencadenantes, que provocan una reacción inflamatoria en la piel, sin que medien
IRRITANTES mecanismos inmunológicos. Aunque las sustancias irritantes lo son para todos los individuos,
CUTÁNEOS. la sensibilidad a su acción es muy variable. Los niños, sobre todo los lactantes, se encuentran
entre los más vulnerables a los irritantes. La constitución atópica predispone también a sufrir
dermatitis por irritantes. Algunos de los alérgenos que causan la dermatitis por contacto son:
Sulfato de níquel, alcoholes de la lana, sulfato de neomicina, dicromato potásico, benzocaína,
mezcla de fragancias,colofonia,resina epoxi, clioquinol, bálsamo del Perú, diclorhidrato de
etilendiamina, cloruro de cobalto, resina de p-terc-butilfenol, formaldehído, mezcla de
parabenos, mezcla carbas, fenilisopropil-para-fenilendiamina,quaternium 15,
mercaptobenzotiazol, para-fenilendiamina, formaldehído, mezcla de mercaptos tiomersal,
mezcla de tiuranes, mercurio, mezcla de lactonas serquiterpénicas, pivalato de tixocortol.
CORRESPONDE AL MECANISMO FISIOPATOLÓGICO DE LAS DERMATITIS POR CONTACTO.

PREDISPOSICIÓN "En un 70 a 80% de los pacientes con DERMATITIS ATÓPICA tienen una historia familiar
GENÉTICA. positiva de atopía." La influencia genética es compleja siendo el componente materno más
importante. Un estudio reciente a gran escala de análisis de micro arreglos de ADN ha
demostrado que cuatro genes involucrados en la diferenciación epidérmica, ubicados en el
cromosoma 1q21, muestran diferentes niveles de expresión en lesiones eczematosas de la piel
en comparación con controles. LOS PACIENTES CON DERMATITIS ATÓPICA TIENEN UNA ALTA
ASOCIACIÓN FAMILIAR.

LA INFECCIÓN La DERMATITIS SEBORRÉICA estaría producida como consecuencia de una reacción anormal
POR MALASSEZIA de la piel frente a la presencia de las levaduras y esta reacción podría estar relacionada con la
SP. capacidad de Malassezia para sintetizar una toxina o estimular la actividad lipasa. Por lo que
se cree que la dermatitis seborreica es causada por una relación alterada entre estos
microrganismos comensales superficiales y el huésped, es mas, el éxito de medicamentos
antimicóticos para el tratamiento apoya esta hipótesis ya que la terapia esta basada en
controlar y disminuir la producción de sebo, inhibir la multiplicación de la Malassezia y otras
levaduras y desinflamar el área afectada. LA MALASSEZIA ESTIMULA LA ACTIVIDAD LIPASA
DE LA PIEL PROVOCANDO DERMATITIS SEBORREICA.

Bibliografía:
1. GUÍA DE PRÁCTICA CLÍNICA, DIAGNÓSTICO Y MANEJO DE LA DERMATITIS ATÓPICA DESDE EL
NACIMIENTO HASTA LOS 16 AÑOS DE EDAD EN EL PRIMER NIVEL DE ATENCIÓN. MÉXICO: SECRETARIA DE
SALUD, 2009. 2. LONGO DL, FAUCI AS, KASPER DL, HAUSERSL, JAMESON JL, LOSCALZOJ. HARRISON.
PRINCIPIOS DE MEDICINA INTERNA, 18A EDICIÓN. MC GRAW HILL. NEW YORK, USA. 2012, PP 395.

http://www.cenetec.salud.gob.mx/descargas/gpc/CatalogoMaestro/033_GPC_DermAtopPed/IMSS_033_08_EyR.pdf

192 - EL TRATAMIENTO DE PRIMERA ELECCIÓN EN ESTE CASO ES CON:


QUERATOLÍTICOS , La elección del tratamiento se realiza en función de la extensión y localización de las
ALQUITRÁN DE lesiones de psoriasis, así como de los tratamientos previos y la edad del paciente. Entre los
HULLA Y tratamientos tópicos DESTACAN LOS CORTICOIDES, EN CURA ABIERTA U OCLUSIVA Y LOS
CORTICOESTEROIDES DERIVADOS DE LA VITAMINA D (TACALCITOL Y CALCIPOTRIOL). ESTOS FÁRMACOS SE
TÓPICOS. UTILIZAN CONJUNTAMENTE CON EMOLIENTES Y BREAS O ALQUITRANES, ASÍ COMO CON
QUERATOLÍTICOS COMO EL ÁCIDO SALICÍLICO. La helioterapia, y su variante PUVA
(psoraleno + radiación UVA) o radiación UVB de banda estrecha, es un tratamiento muy
empleado, especialmente en las psoriasis eruptivas o en gotas. Dentro de los tratamientos
sistémicos, destaca el empleo de: 1) Retinoides (etretinato y acitretino) muy útiles en las
formas pustulosas, eritrodérmicas y en la artropatía, solos o combinados con fototerapia
(RePUVA); 2) Metotrexate, un potente antiproliferativo y antinflamatorio, muy eficaz en las
formas graves y, sobre todo, en la psoriasis con artropatía, aunque sus efectos secundarios
agudos (Afectación de médula ósea o fibrosis hepática) lo limitan; 3) Ciclosporina, agente
inmunosupresor efectivo en un 70% de enfermos con psoriasis vulgar crónico intenso,
aunque también se utiliza en casos de artropatía, psoriasis pustuloso generalizado o
eritrodermia psoriásica.

CHAMPÚ CON El sulfuro de selenio, un agente antiinfeccioso, alivia el prurito y la descamación del cuero
SULFURO DE cabelludo y remueve las partículas secas y escamosas que se denominan comúnmente caspa
SELENIO. o seborrea. También se usa para tratar la tiña multicolor, una infección fúngica de la piel.

LUBRICANTES Y MEDIDAS GENERALES DEL CUIDADO DE LA PIEL Y LUBRICANTES EN LOS PACIENTES CON
PROTECCIÓN DE PIEL DERMATITIS ATÓPICA Los objetivos del tratamiento son: 1.- Medidas generales para evitar
EN GENERAL. factores desencadenantes: • Exceso de lavado, uso de jabones detergentes. • Ropas
ajustadas, excesivas y fibras sintéticas • Irritantes primarios: polvo, grasas, disolventes,
barnices, etc. • Trabajos inadecuados: carpintería, minería, mecánica, etc. • Climas con
temperaturas extremas. • Estrés emocional y conflictos familiares • Infecciones
intercurrente, bacteriana, vírica o fúngicas. • Alérgenos (alimentos solo en un 20%) 2.-
Cuidado de la piel: • Hidratación.- Es la base del tratamiento de mantenimiento y juega un
papel importante en el brote agudo de la enfermedad, ya que permite disminuir la cantidad
de esteroides tópicos requeridos para el control de la inflamación. • No deben usarse
jabones alcalinos. Se utilizan jabones de pH ácido y aceites de baño. • Los baños de avena
coloidal, con carácter emoliente. • Tras el baño y dentro de los tres minutos se aplicaran
hidratantes y emolientes que permiten retener el agua manteniendo la barrera suave y
flexible. • El uso de esponjas talcos, perfumes, lociones y el secado por fricción deberán
estar proscritos. LA BASE DEL TRATAMIENTO DE LA DERMATITIS ATÓPICA CONSISTE EN
EVITAR LOS IRRITANTES CUTÁNEOS (PROTECCIÓN DE LA PIEL EN GENERAL), MANTENER
UNA HIDRATACIÓN CUTÁNEA ADECUADA, USAR CON SENSATEZ ANTIINFLAMATORIOS
TÓPICOS Y TRATAR TEMPRANAMENTE LAS LESIONES INFECTADAS.

KETOCONAZOL EN La DERMATITIS SEBORREICA EN EL ADULTO en el adulto requiere que el paciente entienda


CREMA Y que es una enfermedad crónica, que tendrá períodos de mejoría y empeoramiento, por lo
PROTECTORES que no debe de buscar la erradicación completa de la afección sino el control. El
SOLARES. tratamiento tópico puede estar dirigido al uso de antinflamatorios. La corticoterapia tópica
es efectiva en el tratamiento de la dermatitis seborreica y su efecto es debido a su actividad
antinflamatoria. Se deben de utilizar corticoesteroides de baja o la mediana potencia del
tipo de la hidrocortisona o mometasona. Los inhibidores de la calcineurina no están
aprobados para el tratamiento de la dermatitis seborreica. la pomada de succinato de litio
es efectiva en el tratamiento de la dermatitis seborreica, tanto en individuos
inmunocompetentes como en enfermos de sida. El succinato de litio es efectivo contra la
malassezia spp. otros agentes empleados son los queratolíticos, como ácido salicílico, el
sulfuro de selenio, las fórmulas de alquitrán de hulla y el PROPILENOGLICOL. LOS
ANTIMICÓTICOS TÓPICOS SON EMPLEADOS PARA REDUCIR LAS COLONIAS DE
MALASSEZIA Y DISMINUIR LA INFLAMACIÓN

Bibliografía:
1. GUÍA DE PRÁCTICA CLÍNICA, DIAGNÓSTICO Y MANEJO DE LA DERMATITIS ATÓPICA DESDE EL
NACIMIENTO HASTA LOS 16 AÑOS DE EDAD EN EL PRIMER NIVEL DE ATENCIÓN. MÉXICO: SECRETARIA DE
SALUD, 2009. 2. LONGO DL, FAUCI AS, KASPER DL, HAUSERSL, JAMESON JL, LOSCALZOJ. HARRISON.
PRINCIPIOS DE MEDICINA INTERNA, 18A EDICIÓN. MC GRAW HILL. NEW YORK, USA. 2012, PP 395.

http://www.cenetec.salud.gob.mx/descargas/gpc/CatalogoMaestro/033_GPC_DermAtopPed/IMSS_033_08_EyR.pdf

FIN DEL CASO CLÍNICO SERIADO


ANÁLISIS DEL CASO CLÍNICO

IDENTIFICACIÓN DEL REACTIVO


Area: GINECOLOGÍA Y OBSTETRICIA
Especialidad: OBSTETRICIA
Tema: HEMORRAGIA POSTPARTO Y PUERPERIO ANORMAL
Subtema: TVP Y TROMBOEMBOLIA PULMONAR EN EL EMBARAZO Y
PUERPERIO

CASO CLÍNICO SERIADO

MUJER DE 38 AÑOS DE EDAD, CON ANTECEDENTE PERSONAL Y FAMILIAR DE TROMBOEMBOLIA VENOSA NO


COMPLICADA ASOCIADA AL EMBARAZO. ACUDE A CONSULTA CON PRUEBA INMUNOLÓGICA DE EMBARAZO
POSITIVA, CON AMENORREA DE 5 SEMANAS. DURANTE LA EXPLORACIÓN SE OBSERVAN VENAS VARICOSAS
TURGENTES EN AMBAS PIERNAS. SE DECIDE INICIAR TROMBOPROFILAXIS ANTE EL ALTO RIESGO DE
DESARROLLAR UN NUEVO EVENTO TROMBÓTICO.

38 años (factor de riesgo)

¥ personal y familiar de tromboembolia


venosa no complicada (OJO EL
ANTECEDENTE PERSONAL POR SI SOLO
CONFIERE UN ALTO RIESGO).

venas varicosas en ambas piernas (factor


de riesgo)

193 - EL SIGUIENTE MEDICAMENTO NO SE DEBERÁ INDICAR POR SU ALTO GRADO DE


TERATOGENICIDAD:

HEPARINA NO Cuando la heparina de bajo peso molecular (HBPM) no se puede utilizar o cuando se prefiera la
FRACCIONADA heparina no fraccionada (HNF), por ejemplo, en pacientes con disfunción renal, se puede utilizar
HNF a través de dos alternativas: - Terapia inicial intravenosa seguida de una dosis ajustada
subcutánea cada 12hrs. - Oferta subcutánea de dosis ajustada de HNF con terapia subcutánea, la
dosis debe ajustarse para prolongar mínimo el intervalo de TTPa en rango terapéutico (6hrs
después de la aplicación), aunque se reconoce que el control de TTPa es menos fiable en el
embarazo. LA HEPARINA NO FRACCIONADA ESTÁ INDICADA EN EL EMBARAZO COMO
ALTERNATIVA AL TRATAMIENTO DE PRIMERA ELECCIÓN.
WARFARINA Para mujeres embarazadas que reciben anticoagulación para el tratamiento de la
tromboembolia venosa, se recomienda la heparina de bajo peso molecular (HBPM) sobre los
antagonistas de la vitamina K durante todo el embarazo, y durante el embarazo tardío cuando
el parto es inminente. En el caso de aquellas mujeres que requieren de antagonistas de la
vitamina K a largo plazo y están intentando embarazarse, son candidatas para la sustitución de
HBPM, por lo que se debe sugerir se realicen las pruebas de embarazo frecuente, y cuando se
logre el embarazo sustituir los antagonistas de la vitamina K por la HBPM en lugar de sustituirla
al intentar embarazarse. La evidencia muestra que la sustitución de la vitamina K antes de las 6
semanas parece eliminar el riesgo de posibles embriopatías y es segura las primeras 6 semanas
de gestación. EN GENERAL, LOS ANTAGONISTAS DE LA VITAMINA K, COMO LA WARFARINA,
NO SE CONSIDERAN PARA EL TRATAMIENTO DE LA TROMBOSIS VENOSA EN EL EMBARAZO YA
QUE SU EXPOSICIÓN DURANTE EL PRIMER TRIMESTRE PUEDE CAUSAR EMBRIOPATIA.

HEPARINA DE La HEPARINA DE BAJO PESO MOLECULAR (HBPM) tiene un lugar preferente en el manejo y
BAJO PESO profilaxis de la tromboembolia venosa (TEV) ya que: - Es el agente farmacológico preferido
MOLECULAR sobre la Heparina no fraccionada (HNF) para la tromboprofilaxis posparto. - Se recomienda en
mujeres embarazadas con TEV aguda en dosis ajustada, sobre las dosis ajustadas de HNF. - Se
recomienda para las mujeres embarazadas con TEV agudo sobre los antagonistas de la vitamina
K durante el tratamiento antes del parto. LA HEPARINA DE BAJO PESO MOLECULAR ES EL
FÁRMACO DE ELECCIÓN PARA LA PROFILAXIS Y EL MANEJO DE LA TROMBOEMBOLIA VENOSA
DURANTE EL EMBARAZO Y PUERPERIO.

FONDAPARINUX En mujeres embarazadas, se sugiere el uso de fondaparinux e inhibidores directos de trombina


parenteral, para mujeres con reacciones alérgicas severas a la heparina que no pueden recibir
danaparoide (por ejemplo, trombocitopenia inducida por heparina). LOS INHIBIDORES
DIRECTOS DE LA TROMBINA SON UNA BUENA ALTERNATIVA DURANTE EL EMBARAZO
CUANDO LA PACIENTE TIENE REACCIÓN ALÉRGICA A LA HEPARINA.

Bibliografía:

http://www.cenetec-difusion.com/CMGPC/SS-544-11/ER1.pdf

194 - ES LA EMBRIOPATÍA MÁS FRECUENTEMENTE ASOCIADA A LA ADMINISTRACIÓN DE ESTE


MEDICAMENTO:

HIPOPLASIA El SÍNDROME FETAL POR HIDANTOÍNA se asocia a la administración de éste y otros


DIGITAL anticonvulsivantes durante el embarazo. Se caracteriza por la presencia de rasgos faciales que
DISTAL + incluyen nariz respingada, hipoplasia medio facial y labio superior largo con borde bermellón
HIPOPLASIA delgado e hipoplasia digital distal.
MEDIO
FACIAL

TOXICIDAD La administración materna de AMINOGLUCÓSIDOS puede generar concentraciones sanguíneas


RENAL + fetales tóxicas renales y óticas. Se sabe que los efectos son dosis dependientes y que, en muchos de
TOXICIDAD los casos, pueden evitarse con el uso de dosis menores divididas.
ÓTICA

HIPOPLASIA Los antagonistas de la vitamina K atraviesan la placenta, y la exposición durante el primer trimestre
DEL TERCIO a este fármaco puede causar embriopatía. También se asocian a pérdida fetal y anticoagulación fetal
MEDIO durante el parto. La exposición a la warfarina entre la sexta y novena semanas del embarazo provoca
FACIAL + la llamada EMBRIOPATÍA POR WARFARINA QUE SE CARACTERIZA POR HIPOPLASIA NASAL Y DE LA
EPÍFISIS REGIÓN INTERMEDIA DE LA CARA, ADEMÁS DE PUNTEADO EN LAS VÉRTEBRAS Y EPÍFISIS
PUNTEADA FEMORALES. Se cree que este efecto se debe a la inhibición de la carboxilación posterior a la
ÓSEA traducción de las proteínas de coagulación. Se sabe que el efecto de este fármaco es dependiente de
la dosis. Durante el segundo y tercer trimestre los defectos relacionados con la warfarina son
debidos principalmente a hemorragia, lo cual causa crecimiento no armónico y deformación por
cicatrización en uno o varios órganos. Estos defectos pueden ser regionales extensos e incluyen la
displasia medial dorsal del sistema nervioso con agenesia del cuerpo calloso, malformación de
Dandy-Walker y atrofia cerebral media; displasia ventral media, como microftalmia, atrofia óptica y
ceguera, retraso del desarrollo y retraso mental.
PALADAR Existen varios reportes que relacionan la administración de ANTIMICÓTICOS (fluconazol,
HENDIDO + itraconazol) con malformaciones congénitas. Algunos estudios sugieren que los recién nacidos
FUSIÓN expuestos a fluconazol durante la gestación presentan anormalidades craneales, paladar hendido,
HUMERAL fusión humeral radial y otros trastornos del brazo. Por su parte el itraconazol se ha relacionado con
RADIAL defectos en las extremidades.

Bibliografía:
1. CUNNINGHAM G, LEVENO K, BLOMM S, HAUTH J, RPUSE D, SONG C. WILLIAMS OBSTETRICIA, 23A
EDICIÓN. MC GRAW HILL. USA. 2011.

http://www.cenetec-difusion.com/CMGPC/SS-544-11/ER1.pdf

FIN DEL CASO CLÍNICO SERIADO


ANÁLISIS DEL CASO CLÍNICO

IDENTIFICACIÓN DEL REACTIVO


Area: MEDICINA INTERNA
Especialidad: HEMATOLOGÍA
Tema: ANEMIAS
Subtema: ANEMIA POR DEFICIENCIA DE HIERRO

CASO CLÍNICO CON UNA PREGUNTA

MUJER DE 56 AÑOS EN ESTUDIO ACTUALMENTE POR PROBABLE ANEMIA FERROPRIVA.

MUJER DE 56 AÑOS.

-.

-.

ANEMIA FERROPRIVA.

-.

195 - ACUDE CON RESULTADOS DE LABORATORIO DONDE USTED ESPERARÍA ENCONTRAR EL


SIGUIENTE HALLAZGO PARA CONFIRMAR SU SOSPECHA DIAGNÓSTICA:

ELEVACIÓN DEL A todos los pacientes con ANEMIA MICROCÍTICA HIPOCRÓMICA se les debe solicitar: - Hierro
RECEPTOR DE sérico total: disminuido. - Capacidad total de fijación del hierro: aumentada. - Porcentaje de
TRANSFERRINA. saturación de la transferrina: disminuida. - Ferritina sérica: disminuida. El RECEPTOR DE
TRANSFERRINA (TfR) está presente en los reticulocitos y se separa o es liberado de la membrana
cuando el reticulocito madura. En la deficiencia de hierro tisular se produce un incremento
proporcional del número de receptores de transferrina. Puede ser útil como un marcador precoz
de deficiencia de hierro, como la ferritina sérica, pero también permite diferenciar entre la
anemia ferropénica y la de la enfermedad crónica en la que permanece normal. LA FERROPENIA
AUMENTA LAS CONCENTRACIONES DEL RECEPTOR DE TRANSFERRINA.

HIERRO SÉRICO La concentración de hierro sérico generalmente disminuye cuando las reservas de hierro se
ELEVADO. depleccionan. Sin embargo, su valor está sujeto a variaciones diurnas, con valores más altos a
última hora del día y puede estar influenciado por su ingesta en las comidas. EL HIERRO SÉRICO
SE ENCUENTRA DISMINUIDO EN EL CASO DE ANEMIA FERROPRIVA.
FERRITINA La FERRITINA es un compuesto de almacenamiento del hierro y su valor sérico se correlaciona
ELEVADA. con las reservas totales en el organismo, disminuyendo por debajo de 10-12 ng/dl cuando éstas
se depleccionan. Es un reactante de fase aguda, por lo que puede estar aumentada en caso de
infección o inflamación. La concentración de ferritina aumenta rápidamente tras iniciar un
tratamiento con hierro y alcanza valores normales mucho antes de que se repongan por
completo los depósitos de hierro, por lo que no es válida para evaluar su eficacia. LA
DISMINUCIÓN DE FERRITINA ES DIAGNÓSTICA DE ANEMIA POR DEFICIENCIA DE HIERRO.

AUMENTO DE La SATURACIÓN DE TRANSFERRINA (Tfsat) indica el porcentaje de puntos de unión al hierro


LA ocupados y refleja el trasporte de hierro más que su acumulación. Su disminución indica un
SATURACIÓN hierro sérico bajo con respecto al número de receptores y, por tanto, sugiere reservas de hierro
DE bajas. La Tsfat varía por los mismos factores que influyen en la TIBC. LA ELEVACIÓN DE
TRANSFERRINA. SATURACIÓN DE TRANSFERRINA PUEDE PROVOCAR UNA SOBRECARGA DE HIERRO A LOS
TEJIDOS.

Bibliografía:
1. LONGO DL, FAUCI AS, KASPER DL, HAUSERSL, JAMESON JL, LOSCALZOJ. HARRISON. PRINCIPIOS DE
MEDICINA INTERNA, 18A EDICIÓN. MC GRAW HILL. NEW YORK, USA. 2012, PP 847. 2. GUÍA DE PRÁCTICA
CLÍNICA, PREVENCIÓN, DIAGNÓSTIO Y TRATAMIENTO DE LA ANEMIA POR DEFICIENCIA DE HIERRO.
MÉXICO: SECRETARIA DE SALUD, 2010.

http://www.cenetec.salud.gob.mx/descargas/gpc/CatalogoMaestro/415_IMSS_10_Anemia_def_hierro_May2a/EYR_IMSS_415_10.pdf
ANÁLISIS DEL CASO CLÍNICO

IDENTIFICACIÓN DEL REACTIVO


Area: CIRUGÍA
Especialidad: CIRUGÍA ABDOMINAL
Tema: PATOLOGÍA ABDOMINAL RESOLUCIÓN QUIRÚRGICA DE URGENCIA
Subtema: ALTERACIONES ELECTROLÍTICAS Y ACIDO BÁSICAS EN EL
PACIENTE QUIRÚRGICO

CASO CLÍNICO CON UNA PREGUNTA

MUJER DE 35 AÑOS DE EDAD, POSTOPERADA DE REDUCCIÓN QUIRÚRGICA DE FRACTURA EXPUESTA DE


FÉMUR, SIN COMPLICACIONES. ACTUALMENTE EN EL ÁREA DE RECUPERACIÓN DONDE SE LE OBSERVA
SUMAMENTE ANSIOSA Y CON DOLOR, A PESAR DE LA ADMINISTRACIÓN DE MEDICAMENTOS. LA
GASOMETRÍA REPORTA PH ELEVADO.

35 años.

postoperada de reducción quirúrgica de


fémur sin complicaciones (no guarda
relación con alteraciones ácido-base)

196 - EL DESEQUILIBRIO ÁCIDO-BÁSICO QUE PRESENTA LA PACIENTE ES SECUNDARIO A:


RETENCIÓN DE La ALCALOSIS METABÓLICA es uno de los trastornos del equilibrio ácido-base en que una
BICARBONATO concentración baja de hidrógenos circulantes y el consecuente aumento de la concentración de
bicarbonato, eleva el pH del plasma sanguíneo por encima del rango normal (7.35-7.45). En la
alcalosis metabólica se halla un pH arterial >7,45 y un bicarbonato plasmático >25 mmol/l como
alteración primaria y un aumento de la pC02, por hipoventilación compensatoria. La alcalosis
metabólica se produce a causa de ganancia neta de [HCO3–] o de pérdida de ácidos no volátiles
(en general HCl) procedentes del líquido extracelular. La alcalosis metabólica se manifiesta por pH
arterial alto, aumento de la [HCO3–] sérica y aumento de la PaCO2 a causa de la hipoventilación
alveolar compensadora. A menudo conlleva hipocloremia e hipopotasemia. También podemos
encontrar una retención de HCO3 como mecanismo compensador de una acidosis respiratoria. El
trastorno entraña una fase generadora, en la que la pérdida de ácido suele causar alcalosis, y una
fase de "mantenimiento", en la que el riñón es incapaz de compensar mediante la eliminación de
bicarbonato (HCO3–). Los riñones y los pulmones regulan el estado ácido/base, los riñones
poseen una impresionante capacidad de eliminar HCO3, y la persistencia de una alcalosis
metabólica representa un fracaso de los riñones para eliminar HCO3 de la forma habitual. Si a un
individuo normal se le administra una carga de bicarbonato sódico, se produce un alza del pH de
corta duración, ya que el riñón es capaz de responder rápidamente eliminando el exceso de
bicarbonato. Así, para que una alcalosis metabólica se mantenga en el tiempo son necesarias dos
condiciones: 1) Una pérdida continua de hidrogeniones o ingreso mantenido de bases. La pérdida
de H+ puede deberse entre otras causas a vómitos, o sondas nasogástricas, como también al uso
excesivo de diuréticos. El ingreso exagerado de bases puede deberse a administración terapéutica
de bicarbonato o de sustancias como lactato (soluciones endovenosas), acetato (diálisis) y citrato
(transfusiones). 2) UNA ALTERACIÓN EN LA FUNCIÓN RENAL QUE IMPIDA LA EXCRECIÓN DE
BICARBONATO. Ésta puede deberse a las siguientes condiciones: Hipovolemia: en estos casos
prima la reabsorción de Na+, ión que se reabsorbe junto con bicarbonato. Si no se corrige la
hipovolemia, la alcalosis persiste a pesar de que haya cesado la pérdida de H+ o el ingreso de
bases. Aumento de mineralocorticoides (aldosterona) como en el síndrome de Cushing. En estos
casos hay retención de Na+ con pérdidas exageradas de H+ y K+, que impiden la eliminación de
bicarbonato. En ellos es necesario corregir la hipocalcemia o el hiperaldosteronismo para
normalizar el pH. Hipocalcemia: en estos casos hay una pérdida exagerada de H+ por la orina que
impide compensar la alcalosis. Hipocloremia: cuando este anión disminuye, de aumenta el
bicarbonato para mantener el equilibrio isoeléctrico. Lo inverso también es cierto y, cuando se
retiene bicarbonato para compensar una acidosis respiratoria, el Cl baja. Es importante tener
presente este punto durante la corrección de la acidosis respiratoria, ya que, si no se suministra Cl
al paciente, llega el momento en que el bicarbonato acumulado durante la compensación no
puede seguirse eliminando, ya que debe mantenerse el equilibrio isoeléctrico. Con ello se produce
una alcalosis que inhibe la ventilación, imposibilitando la corrección total de la acidosis
respiratoria. En cualquiera de estas condiciones existe compensación respiratoria, que consiste en
una disminución de la ventilación alveolar con aumento de la PaCO2 y el subsiguiente aumento
de protones en sangre al desplazarse el equilibrio del tampón bicarbonato provocando una caída
del pH que vuelve a rangos fisiológicos. Sin embargo, ésta es limitada, ya que la hipercapnia
produce hipoxemia que estimula la ventilación. Por esta razón, la PaCO2 como compensación rara
vez sube de 55 mmHg. En el caso de la alcalosis metabólica habremos de centrarnos en el análisis
de los datos obtenidos acerca de la presión arterial de CO2 y el pH. Esto es debido a que el pH y la
relación ácido-base del tampón bicarbonato (CO2 actuaría como ácido y bicarbonato como base)
es muy estrecha, siendo los valores normales de 7,35-7,45 para el pH y de 35-45 mmHg para la
PCO2. Todas las variaciones que sobrepasen esos umbrales indicarán una alcalosis (si el pH es
mayor) o una acidosis (si el pH es menor). Si observamos un aumento del bicarbonato estaremos
ante una alcalosis metabólica. Si lo que se observa es un descenso de la PCO2 estaremos también
ante una alcalosis, pero en este caso respiratoria porque será debida a hiperventilación (los
pulmones son muy eficientes eliminando O2). Otras pruebas que medirían el pH y los niveles de
CO2 y bicarbonato en el riñón serían un análisis de orina o un análisis básico de química
sanguínea. Se medirán en relación con el riñón porque la alcalosis metabólica es debida a la
acumulación de bicarbonato en el mismo debido a múltiples factores, como pueden ser: vómitos,
diarrea, diuréticos, fibrosis quística, síndromes como el de Cushing o Bartther, etc. En el caso de
trastornos que conllevan la pérdida de cloro (vómitos prolongados, diuréticos…) o ingesta
insuficiente de cloro estaremos ante una alcalosis hipoclorémica. Es debido a que la pérdida de
cloro tiene como consecuencia la retención de bicarbonato en el riñón (alcalosis). LA ALCALOSIS
METABÓLICA SE CARACTERIZA POR UN PH ALTO Y UN HCO3 ALTO. LA RETENCIÓN DE
BICARBONATO A NIVEL RENAL FAVORECERÍA LA APARICIÓN DE ESTE TRASTORNO.
PÉRDIDA DE La ALCALOSIS RESPIRATORIA es uno de los trastornos del equilibrio ácido-base en que una mayor
CO2 frecuencia de respiración (hiperventilación) eleva el pH del plasma sanguíneo, a lo cual se le
denomina alcalosis. Para unos investigadores, la alcalosis respiratoria es el disturbio ácido básico
más prevalente en pacientes seriamente enfermos, mientras que, para otros, la acidosis
metabólica sería el trastorno más frecuente. La alcalosis respiratoria aguda ocurre rápidamente.
Durante la alcalosis respiratoria aguda, el paciente pierde el conocimiento momento en el cual la
tasa de ventilación volverá a la normalidad. La alcalosis respiratoria generalmente ocurre cuando
bajo el efecto de algún estímulo que hace que una persona comience a hiperventilar. El aumento
de la respiración produce un aumento de la respiración alveolar, expulsando las emisiones de CO2
de la circulación (hipocapnia, disminución del CO2). Esto altera la dinámica del equilibrio químico
de dióxido de carbono en el sistema circulatorio y el sistema reacciona bajo el principio de Le
Chatelier. Los iones circulantes de hidrógeno y bicarbonato reaccionan para formar ácido
carbónico (H2CO3) para hacer más CO2 a través de la enzima anhidrasa carbónica de modo que
CO2+H2O -- > H2CO3 -- > H+ + HCO3-. El resultado neto de esta reacción es la disminución de la
concentración de iones hidrógeno de la circulación, y un aumento consecuente del pH. También
se ha notado un aumento compensador en la concentración del calcio ionizado en el plasma
sanguíneo. La alcalosis respiratoria puede ser causada accidentalmente por iatrogenia durante
una excesiva ventilación mecánica. Otras causas incluyen: Dolor, la ansiedad, la histeria, el estrés,
Accidente cerebrovascular, hemorragia subaracnoidea, la meningitis. El uso de medicamentos y
drogas como el doxapram, la aspirina, la cafeína , el abuso al café, El mudarse a regiones de gran
altitud sobre el nivel del mar donde la baja presión atmosférica estimula un aumento en la
frecuencia ventilatoria, enfermedades pulmonares tales como neumonía donde la respiración es
regulada por un estímulo hipóxico en vez de los niveles de CO2, el determinante normal de tal
regulación, fiebre, que estimulan el centro respiratorio en el tronco cerebral, embarazo, actividad
sexual, que puede inducir la respiración excesiva debido a la excitación sexual. Las
manifestaciones clínicas se relacionan con la disminución de los niveles de dióxido de carbono
arteriales, e incluyen parestesia periférica. Además, la alcalosis produce trastornos en el balance
del ion calcio, y causar así síntomas de hipocalcemia, tales como tetania y desmayos, sin que se
note una disminución de los niveles séricos de calcio total. LA ALCALOSIS METABÓLICA ES
CARACTERÍSTICA DE LOS ESTADOS DE HIPERVENTILACIÓN, COMO ES EL CASO. La estimulación
directa del centro respiratorio puede estar condicionada en estos casos por: dolor, síndrome de
ansiedad/hiperventilación, delirio hiperactivo, fiebre, hemorragia subraracnoidea, neuroinfección,
enfermedad vascular cerebral, trauma o tumores del SNC.
RETENCIÓN DE La ACIDOSIS RESPIRATORIA es uno de los trastornos del equilibrio ácido-base en la que la
C02 disminución en la frecuencia de las respiraciones o hipoventilación, provoca una concentración
creciente del dióxido de carbono en el plasma sanguíneo y la consecuente disminución del pH de
la sangre. El dióxido de carbono es producido constantemente a medida que el cuerpo metaboliza
energía, el cual se acumulará en la circulación con rapidez si los pulmones no lo eliminan
adecuadamente. La hipoventilación alveolar conduce así a un aumento de la presión parcial de
CO2 arterial (PaCO2), un trastorno llamado hipercapnia (el valor normal de la PaCO2 es de 36-44
mm Hg). El aumento en la PaCO2 a su vez disminuye la relación entre el bicarbonato (HCO3) y la
PaCO2 con la consecuente disminución en la acidez o pH sanguíneo. La acidosis respiratoria
puede ser aguda o crónica. En la acidosis respiratoria aguda, la PaCO2 se encuentra elevada por
encima del límite superior del rango de referencia normal (más de 6,3 kPa o 45 mmHg) con una
concomitante acidemia (pH < 7,35). En la acidosis respiratoria crónica, la PaCO2 se encuentra
también elevada por encima del límite superior del rango normal, pero persiste en el tiempo con
un pH sanguíneo normal (7,35 a 7,45) o valores casi normales gracias a la compensación renal y a
niveles elevados de bicarbonato (HCO-3 > 30 mEq/L). Etiología: La acidosis respiratoria aguda se
produce cuando aparece un trastorno súbito de la ventilación y antes de que los riñones puedan
retornar el cuerpo a un estado de equilibrio. Este fracaso ventilatorio puede ser causado por la
depresión del centro respiratorio a nivel del sistema nervioso central por enfermedad cerebral o
drogas (como la morfina), incapacidad para ventilar adecuadamente debido a una enfermedad
neuromuscular (p. ej., miastenia gravis, esclerosis lateral amiotrófica, síndrome de Guillain-Barré,
o distrofia muscular), u obstrucción de las vías respiratorias relacionada con la exacerbación del
asma o la enfermedad pulmonar obstructiva crónica (EPOC).Por su parte, la acidosis respiratoria
crónica puede ser secundaria a muchos trastornos, incluyendo la EPOC. La hipoventilación en la
EPOC involucra múltiples mecanismos, incluyendo la disminución en la receptividad a la hipoxia y
la hipercapnia, una mayor discrepancia entre la perfusión y la ventilación que conduce a una
mayor ventilación con espacio muerto y una disminución de la función del diafragma como
consecuencia de la fatiga y la hiperinflación. Acidosis respiratoria crónica también puede ser
secundaria al síndrome de hipoventilación por obesidad (llamado síndrome de Pickwick),
trastornos neuromusculares tales como la esclerosis lateral amiotrófica y graves defectos
ventilatorio restrictivos como se observarán en la fibrosis intersticial y deformidades torácica. Las
enfermedades pulmonares que principalmente causan anormalidad en el intercambio de gas
alveolar usualmente no causan hipoventilación, pero tienden a causar estimulación de la
ventilación e hipocapnia secundaria a la hipoxia. La hipercapnia solo se produce si se hay una
enfermedad grave o fatiga muscular respiratoria. Respuesta fisiológica: El metabolismo del
cuerpo rápidamente genera una gran cantidad de ácido volátil (H2CO3) y ácido no volátil. El
metabolismo de las grasas y carbohidratos conduce a la formación de una gran cantidad de CO2.
El CO2 se combina con agua para formar ácido carbónico (H2CO3). Los pulmones normalmente
excretan la fracción volátil a través de la ventilación sin acumulación de ácido en la sangre. En
situaciones normales, la PaCO2 se mantiene dentro de un rango de 39 a 41 mm Hg. Una
alteración significativa en la asistencia respiratoria que afecte a la eliminación de CO2 puede
causar un trastorno respiratorio ácido-base. La ventilación alveolar está bajo el control de los
centros respiratorios centrales, que se encuentran en el puente troncoencefálico y el bulbo
raquídeo. La ventilación es influenciada y regulada por los quimiorreceptores para la PaCO2,
PaO2 y del pH ubicados en el tronco encefálico y en las arterias aorta y carótidas, así como por
impulsos neurales de receptores de estiramiento en el pulmón, así como impulsos de la corteza
cerebral. El fracaso ventilatorio aumenta rápidamente la PaCO2.En la acidosis respiratoria aguda,
la compensación se produce en 2 pasos: La respuesta inicial es buffering celular que se produce en
cuestión de minutos a horas. El almacenamiento en búfer celular eleva bicarbonato de plasma
(HCO3) solo ligeramente, aproximadamente 1 mEq/L para cada 10 mm de Hg aumento PaCO2.El
segundo paso es una compensación renal que se produce más de 3 a 5 días. Con compensación
renal, se incrementa la excreción renal de ácido carbónico y se incrementa la reabsorción de
bicarbonato. EL PH BAJO Y CO2 ALTO SE RELACIONAN CON UNA ACIDOSIS RESPIRATORIA,
PRINCIPALMENTE RELACIONADOS CON UN FALLO RESPIRATORIO (HIPOVENTILACIÓN). EN ESTA
PACIENTE SUCEDE JUSTO LO CONTRARIO HIPERVENTILACIÓN.
PÉRDIDA DE La ACIDOSIS METABÓLICA es uno de los trastornos del equilibrio ácido-base, caracterizado por un
HCO3 incremento en la acidez del plasma sanguíneo y es, por lo general, una manifestación de
trastornos metabólicos en el organismo. El identificar la enfermedad desencadenante es la clave
para la corrección del trastorno. La acidosis metabólica puede ser causada por: - Aumento en la
generación de H+ de origen endógeno (por ejemplo, cetonas) o ácidos exógenos (por ejemplo,
salicilatos, etilenglicol, metanol). - Incapacidad de los riñones para excretar el hidrógeno
producido por la ingesta de proteínas de la dieta (acidosis tubular renal Tipo I, IV) La PÉRDIDA DE
BICARBONATO (HCO3) es debido a la pérdida-a través del riñón (acidosis tubular renal tipo II) o
del tracto gastrointestinal (diarrea). Cuadro clínico: Los síntomas de una acidosis metabólica no
son específicos, y el diagnóstico puede ser complicado a menos que el paciente presente
indicaciones claras para el muestreo de gases en sangre arterial. Los síntomas incluyen el dolor de
pecho, palpitaciones, dolor de cabeza, alteración del estado mental, incluyendo la ansiedad severa
debido a hipoxia, disminución de la agudeza visual, náuseas, vómitos, dolor abdominal, alteración
del apetito y pérdida de peso (a largo plazo), debilidad muscular y dolor de los huesos. Los que
están en una situación de acidosis metabólica suelen presentar la respiración de Kussmaul, una
respiración profunda, rápida, asociada con cetoacidosis diabética clásica. Las respiraciones rápidas
y profundas aumentan la cantidad de dióxido de carbono exhalado, lo que conlleva a una
reducción de los niveles de dióxido de carbono sérico, causando algún grado de compensación. La
acidemia extrema conduce a complicaciones neurológicas y cardíacas: - Neurológicas: letargo,
estupor, coma, convulsiones - Cardíacos: arritmias (taquicardia ventricular), disminución en la
respuesta a la epinefrina; ambas conducen a la hipotensión arterial El examen físico revela
ocasionalmente signos de enfermedad, pero por lo demás resulta normal. En la intoxicación por
glicol de etileno se reportan alteraciones en los nervios craneales. El edema de la retina puede ser
un signo de intoxicación por metanol (alcohol metílico). La acidosis metabólica crónica conduce a
la osteoporosis y puede causar fracturas. LA ACIDOSIS METABÓLICA, SE DA PRINCIPALMENTE
POR PÉRDIDA EXCESIVA DE BICARBONATO, CUYO EJEMPLO MÁS CLARO ES LA DIARREA.

Bibliografía:
1. DIAGNÓSTICO Y TRATAMIENTO DEL DESEQUILIBRIO ÁCIDO-BASE. EVIDENCIAS Y RECOMENDACIONES:
GUÍA DE PRÁCTICA CLÍNICA. MÉXICO: SECRETARIA DE SALUD; 2010. 2. BRUNICARDI F, ANDERSEN D,
BILLIAR T, Y COLS. SCHWARTZ PRINCIPIOS DE CIRUGÍA, 9A EDICIÓN. MC GRAW HILL. 2011, PP 58-59. 3.
DOHERTY G. DIAGNÓSTICO Y TRATAMIENTO QUIRÚRGICO, 13A EDICIÓN. MC GRAW HILL LANGE. 2011, PP
90-91, 559-562.

http://www.cenetec-difusion.com/CMGPC/IMSS-411-10/ER.pdf
ANÁLISIS DEL CASO CLÍNICO

IDENTIFICACIÓN DEL REACTIVO


Area: CIRUGÍA
Especialidad: PROCTOLOGÍA Y UROLOGÍA
Tema: PATOLOGÍA DE COLÓN, ANO Y RECTO
Subtema: HEMORROIDES

CASO CLÍNICO SERIADO

MUJER DE 40 AÑOS DE EDAD, OBESA Y CON ANTECEDENTE DE CONSTIPACIÓN CRÓNICA. ACUDE A LA


CONSULTA PORQUE DESDE HACE 6 MESES PRESENTA SANGRADO FRESCO MODERADO E INTERMITENTE AL
FINAL DE CADA EVACUACIÓN, QUE SE ACOMPAÑA EN OCASIONES CON LOS ESFUERZOS DE PROTRUSIÓN DE
UNA MASA DE 2 CM DE DIÁMETRO A TRAVÉS DEL ANO Y QUE LE RESULTA DIFÍCIL REDUCIRLA CON
MANIOBRAS DIGITALES.

MUJER DE 40 AÑOS.

Constipación crónica.

6 MESES CON SANGRADO FRESCO


MODERADO E INTERMITENTE AL FINAL
DE CADA EVACUACIÓN que se acompaña
en ocasiones con esfuerzos de protrusión
de una masa de 2cm de diámetro a través
del ano, difícil a la reducción digital.

-.

-.

197 - EL DIAGNÓSTICO CLÍNICO MÁS PROBABLE ES:


HEMORROIDES La ENFERMEDAD HEMORROIDAL se define como la dilatación de los paquetes hemorroidales
GRADO I. localizados en la parte terminal del recto, conducto anal y ano. Con la distensión de las
anastomosis arteriovenosas por deterioro, existe destrucción de los sistemas de fijación del tejido
conjuntivo y el consecuente desplazamiento de los cojinetes. Las hemorroides son estructuras
fisiológicas, ocasionan síntomas cuando presentan alteraciones estructurales del tejido
hemorroidal (dilatación e ingurgitación) y cambios en los tejidos de sostén adyacentes. Las
hemorroides se clasifican desde el punto de vista anatómico en internas y externas; las primeras
se originan en el tercio distal del recto, justo por arriba de la línea dentada y están revestidas de
epitelio cilíndrico. Las externas se encuentran por debajo de la línea dentada hasta el margen del
conducto anal y se encuentran cubiertas de piel. La clasificación de las hemorroides no contempla
el componente externo y solo clasifica a las hemorroides internas. La clasificación queda como
sigue: - GRADO I. Vasos hemorroidales eminentes. Sin prolapso y mínima dilatación venosa. Se
manifiesta con hemorragia.

HEMORROIDES - GRADO II. Prolapso con Valsalva y reducción espontánea. Se manifiesta con hemorragia.
GRADO II.

HEMORROIDES - GRADO III: Prolapso con Valsalva y requiere reducción digital. Se manifiesta con prurito,
GRADO III. hemorragia y prolapso. CUMPLE CON LOS CRITERIOS DADO QUE SE DA CON EL ESFUERZO Y
AUNQUE DIFÍCIL SE REDUCE DE FORMA DIGITAL.

HEMORROIDES - GRADO IV: Prolapso crónico y reducción digital inefectiva. Se manifiesta con hemorragia,
GRADO IV. prolapso y prurito.

Bibliografía:
1. DIAGNÓSTICO Y TRATAMIENTO DE LA ENFERMEDAD HEMORROIDAL EN LA ADULTEZ. EVIDENCIAS Y
RECOMENDACIONES: GUÍA DE PRÁCTICA CLÍNICA. MÉXICO: SECRETARÍA DE SALUD, CENETEC; 2015.

http://www.cenetec-difusion.com/CMGPC/SS-208-09/ER.pdf

198 - EL TRATAMIENTO INDICADO EN ESTE CASO ES:

FLEBOTÓNICOS POR En general hay poca evidencia sobre la efectividad del tratamiento en la patología
VÍA ORAL. hemorroidal con venotónicos (flebotónicos). El mas estudiado ha sido la diosmina. SE
UTILIZAN COMO MEDICAMENTO COADYUVANTE EN EL MANEJO DE LAS HEMORROIDES
PERO NO COMO TERAPÉUTICA DEFINITIVA.

HEMORROIDECTOMÍA. La hemorroidectomía ha demostrado mejores tasas de respuesta pero también se asocia


con más complicaciones (infecciones, sangrado) y más dolor que la ligadura con banda
elástica, por lo que ésta técnica debe reservarse para pacientes que no responden a la
ligadura en banda. LA HEMORROIDECTOMÍA ESTÁ INDICADA EN HEMORROIDES GRADO
IV Y EN AQUELLAS QUE NO RESPONDEN A TÉCNICAS DE CIRUGÍA MENOR.
LIGADURA CON Las técnicas de cirugía menor más utilizadas para el manejo de la ENFERMEDAD
BANDA DE HULE. HEMORROIDAL es la escleroterapia, la fotocoagulación con infrarrojos (CIR) o con láser,
la electrocoagulación, la criocirugía y la ligadura con banda elástica (LBE). La ligadura con
banda elástica es una técnica quirúrgica de cirugía menor; tiene eficacia similar a la
fotocoagulación con infrarrojos. Se muestra superior al resto de las técnicas de cirugía
menor (escleroterapia, fotocoagulación con infrarrojo o láser, electrocoagulación,
criocirugía) y es el tratamiento de elección en hemorroides grado I a III. EL GRADO III DE
HEMORROIDES QUE PRESENTA NUESTRA PACIENTE DEBE SER TRATADO CON UNA
TÉCNICA DE CIRUGÍA MENOR. DE ENTRE ELLAS LA DE ELECCIÓN ES LA LIGADURA CON
BANDA ELÁSTICA. Se deberá elegir la ligadura con banda de hule en esta paciente
(hemorroides grado III) bajo los siguientes criterios: - Para las hemorroides grado III con
bajo dolor, el tratamiento de elección es la hemorroidopexia “o” la ligadura de arteria
guiada por USG. - En pacientes con hemorroides grado III se debe considerar la
hemorroidectomía como opción terapéutica de elección sólo si existe recurrencia (baja
recurrencia). - No se refiere el antecedente de tratamientos previos, y por tanto, no se
puede hablar de recurrencia. - Al existir la opción de ligadura de con banda, esta deberá
elegirse como correcta pues es un método terapéutico que ofrece buenos resultados con
pocos efectos adversos, además de no encontrarse la hemorroidopexia como opción de
respuesta. NOTA: será importante reconocer la diferencia entre hemorroidectomía y
hemorroidopexia. - HEMORROIDOPEXIA: tiene como finalidad la exéresis de la parte
mucosa prolapsada y la fijación de la misma en el canal interior del canal anal, se puede
realizar mediante grapas (máquina de autosutura). - HEMORROIDECTOMÍA: tiene como
finalidad eliminar los tejidos enfermos; la resección se puede realizar por diversas
técnicas.

DIETA RICA EN FIBRA, El tratamiento higiénico-dietético consiste en: aumento de la ingesta de fibra, laxantes,
REDUCCIÓN DE PESO Y ejercicio, reducción de peso y evitar sedestación prolongada; esta indicado como
MEDICAMENTOS tratamiento inicial de los pacientes con hemorroides sintomáticas. Los tratamientos
TÓPICOS. tópicos están hechos a base de anestésicos y corticoides, están indicados como manejo
sintomático y no terapéutico. TODAS ESTAS MEDIDAS CONSTITUYEN EL MANEJO
SINTOMÁTICO GENERAL DE TODAS LAS HEMORROIDES PERO NO COMO MANEJO
TERAPÉUTICO DEFINITIVO.

Bibliografía:
1. DIAGNÓSTICO Y TRATAMIENTO DE LA ENFERMEDAD HEMORROIDAL EN LA ADULTEZ. EVIDENCIAS Y
RECOMENDACIONES: GUÍA DE PRÁCTICA CLÍNICA. MÉXICO: SECRETARÍA DE SALUD, CENETEC; 2015. 2.
BRUNICARDI F, ANDERSEN D, BILLIAR T, Y COLS. SCHWARTZ PRINCIPIOS DE CIRUGÍA, 9A EDICIÓN. MC
GRAW HILL. 2011.

http://www.cenetec-difusion.com/CMGPC/SS-208-09/ER.pdf

FIN DEL CASO CLÍNICO SERIADO


ANÁLISIS DEL CASO CLÍNICO

IDENTIFICACIÓN DEL REACTIVO


Area: PEDIATRÍA
Especialidad: URGENCIAS PEDIÁTRICAS
Tema: URGENCIAS CARDIOVASCULARES PEDIÁTRICAS
Subtema: TETRALOGÍA DE FALLOT

CASO CLÍNICO SERIADO

LACTANTE DE 4 MESES DE EDAD CON TETRALOGÍA DE FALLOT CLÁSICA CON UNA ANATOMÍA CARDÍACA
DESFAVORABLE POR LO QUE SU MANEJO ES CONSERVADOR. EN LOS ÚLTIMOS 15 DÍAS HA PRESENTADO 3
CRISIS DE HIPOXIA POR LO QUE SE DECIDE INTENTAR UN TRATAMIENTO PALIATIVO TEMPORAL.

Lactante de 4 meses.

TETRALOGÍA DE FALLOT CLÁSICA.

3 crisis de hipoxia en 15 días.

Se decide intentar tratamiento paliativo


temporal.

199 - LA ALTERACIÓN ESTRUCTURAL QUE CARACTERIZA A LA VARIEDAD CLÁSICA QUE PRESENTA EL


MENOR ES:

ESTENOSIS La TETRALOGÍA DE FALLOT (TF) se debe a la desviación anterocefálica del septum de salida, se
PULMONAR. caracteriza por: 1. Cabalgamiento aórtico. 2. Defecto septal interventricular (sub-aórtico), 3.
Obstrucción del conducto de salida del ventrículo derecho: puede ser infundibular, valvular,
supravalvular o combinación de todos. 4. Hipertrofia del ventrículo derecho. La TF se clasifica en 4
variedades. - TF CLÁSICA: se acompaña de ESTENOSIS PULMONAR la cual puede ser a nivel
subvalvular, valvular o supravalvular. Es el tipo más frecuente.

AUSENCIA DE - TF CON AUSENCIA DE VÁLVULA PULMONAR, con severa displasia de la válvula e importante
LA VÁLVULA dilatación de arterias pulmonares. Representa del 3 al 5% de los casos de TF y comúnmente la
PULMONAR. mobimortalidad se asocia a problemas respiratorios y ventilatorios por compresión bronquial.
CANAL - TF CON CANAL AURICULO-VENTRICULAR COMÚN, es una rara lesión; se presenta en el 2% de los
AURICULO- casos con TF. Esta asociación conlleva un riego quirúrgico mayor durante la corrección.
VENTRICULAR
COMÚN.

ATRESIA - TF CON ATRESIA PULMONAR: frecuentemente asociado a hipoplasia de ramas de la arteria


PULMONAR. pulmonar y colaterales aortopulmonares.

Bibliografía:
1. GUÍA DE PRÁCTICA CLÍNICA, TRATAMIENTO DE LA TETRALOGÍA DE FALLOT. MÉXICO: SECRETARIA DE
SALUD; 2011.

http://www.cenetec.salud.gob.mx/descargas/gpc/CatalogoMaestro/497_GPC_Tetralogxa_de_Fallot/IMSS-497-11-GER_Fallot.pdf

200 - EL TRATAMIENTO PALIATIVO DE ELECCIÓN PARA ESTE CASO ES:

VALVULOPLASTÍA EL objetivo del tratamiento paliativo es aumentar el flujo sanguíneo pulmonar y por ende la
CON BALÓN. saturación periférica, permitiendo el crecimiento de la arteria pulmonar del niño hasta llevarlo
a la corrección total. Las formas de tratamiento paliativo menos utilizadas son: - Valvuloplastía
con balón: tiene como objetivo mejorar el flujo pulmonar e inducir el crecimiento de las
arterias pulmonares. - Parche en el tracto de salida del ventrículo derecho con circulación
extracorpórea.

COLOCACIÓN DE La colocación de STENT en el conducto arterioso es una ALTERNATIVA AL MANEJO


STENT EN EL CONVENCIONAL en cardiopatías dependientes del conducto. Este procedimiento es factible en
CONDUCTO neonatos como medida paliativa incrementando flujo a la arteria pulmonar, sin embargo, si
ARTERIOSO. existe estenosis de rama en la inserción con el conducto arterioso está contraindicada. Aún con
la aplicación de Stent puede haber reestenosis que en muchos casos requiere de ser dilatado
nuevamente. LA COLOCACIÓN DE STENT EN EL CONDUCTO ARTERIOSO ES UNA ALTERNATIVA
RAZONABLE A LA COLOCACIÓN DE FÍSTULA SISTÉMICO PULMONAR MODIFICADA.

FÍSTULA El procedimiento paliativo actual de elección es la FÍSTULA SISTÉMICO PULMONAR


SISTÉMICO MODIFICADA tipo Blalock-Taussing, utilizando un injerto de Gore-Tex entre la subclavia y la
PULMONAR arteria pulmonar. Sus ventajas son las siguientes: - Presevación de la subclavia. - Idoneidad
MODIFICADA. para su realización por ambos lados. - Tasa de permeabilidad excelente. - Menor incidencia de
iatrogenia arterial pulmonar y sistémica. - Facilidad de cierre en el momento de la corrección
quirúrgica. - Mortalidad menor al 1%.

VALVULOPLASTÍA Es preferible la cirugía paliativa en menores de 6 meses sintomáticos para posteriormente


PULMONAR realizar la cirugía correctiva. Los pacientes con TF sintomáticos menores de 6 meses con
PERCUTÁNEA. anatomía no favorable se les puede ofrecer: - Fístula sistémico pulmonar. - Valvuloplastía
pulmonar percutánea la cual puede disminuir la obstrucción y la sintomatología; favorece el
crecimiento de las ramas de la arteria pulmonar evitando las secuelas de una fístula sistémico
pulmonar paliativa. OJO: - EN ESTE MOMENTO SE SOLICITA ESPECÍFICAMENTE EL MANEJO
PALIATIVO DE ELECCIÓN YA QUE LA ANATOMÍA NO ES FAVORABLE. - EN CASO DE QUE SE
PREGUNTARA CUAL ES EL “TRATAMIENTO DE ELECCIÓN” PARA LA TF SERÍA LA CORRECCIÓN
COMPLETA, SIEMPRE Y CUANDO LA ANATOMÍA SE DESCRIBA FAVORABLE.

Bibliografía:
1. GUÍA DE PRÁCTICA CLÍNICA, TRATAMIENTO DE LA TETRALOGÍA DE FALLOT. MÉXICO: SECRETARIA DE
SALUD; 2011.

http://www.cenetec.salud.gob.mx/descargas/gpc/CatalogoMaestro/497_GPC_Tetralogxa_de_Fallot/IMSS-497-11-GER_Fallot.pdf

FIN DEL CASO CLÍNICO SERIADO


ANÁLISIS DEL CASO CLÍNICO

IDENTIFICACIÓN DEL REACTIVO


Area: MEDICINA INTERNA
Especialidad: INFECTOLOGÍA
Tema: NEUROINFECCIONES
Subtema: CISTICERCOSIS

CASO CLÍNICO CON UNA PREGUNTA

FEMENINO DE 55 AÑOS DE EDAD, QUE ACTUALMENTE SE ENCUENTRA EN ESTUDIO POR PRESENCIA DE


CRISIS CONVULSIVAS DE INICIO RECIENTE. SE REALIZA TOMOGRAFÍA DE CRÁNEO OBSERVÁNDOSE
IMÁGENES COMPATIBLES CON NEUROCISTICERCOSIS.

Femenino de 55 años de edad.

-.

Crisis convulsivas de inicio reciente.

-.

TAC de cráneo con imágenes que sugieren


neurocisticercosis.

201 - EL TRATAMIENTO INDICADO EN ESTE CASO SERÍA A BASE DE:


PRAZIQUANTEL La TENIASIS es una infección intestinal causada por la forma adulta de grandes tenias de
O ALBENDAZOL cualquier especie (incluye T. saginata); mientras que la CISTICERSOSIS es producto de la
infección tisular por la forma larvaria de la T. solium. El Praziquantel es un antihelmíntico de
amplio espectro que se utiliza para el tratamiento de infecciones por tremátodos y cestodos. Los
pacientes con neurocisticercosis complicada deben recibir dosis altas de Praziquantel (50 a 100
mg/kg/día) por 15 a 30 días, dividido en 3 dosis o también se puede indicar Albendazol a dosis
de 10 a 15 mg/kg/día, dividida en 2 dosis con una dosis máxima de 800 mg por día por un
periodo de 8 días, esto con la finalidad de alcanzar niveles séricos del fármaco suficientes para
eliminar algún quiste con vida. Generalmente al eliminar los quistes se produce una reacción
inflamatoria local intensa, lo que puede producir una exacerbación de los síntomas. Por lo
anterior, el tratamiento debe acompañarse de tratamiento anticonvulsivo apropiado con un
adecuado control de las crisis convulsivas y, en caso de presentar hidrocefalia, esta debe
corregirse mediante una derivación. También se puede reducir la reacción inflamatoria con uso
concomitante de corticoestoroides (Dexametasona), sin embargo, su uso no elimina
necesariamente el riesgo de complicaciones severas como infarto cerebral, elevación de la
presión intracraneana. Generalmente no está indicado el uso de ambos para el tratamiento de la
neurocisticercosis, es más, se han realizado alguna meta análisis en donde se ha demostrado que
el uso de corticoesteroides aumente las concentraciones séricas de Albendazol y disminuye las
de praziquantel y esto favorece el uso de Albenzadol en lugar de Praziquantel. Pero es la única
respuesta que tiene el tratamiento indicado. La mayoría de los expertos recomiendan la terapia
con albenzadol o praziquantel para pacientes con cisticercos múltiples o con imagen sin realce.
El albenzadol se prefiere sobre el praziquantel, porque tiene menos interacciones entre
fármacos con los anticonvulsivos.

METRONIDAZOL El Mebendazol es un medicamento que no es soluble en agua y etanol. Tiene una baja
O MEBENDAZOL biodisponibilidad, además de tener un metabolismo de primer paso importante a nivel hepático
que llega a ser de hasta 80 % por lo que no alcanza concentraciones adecuadas para el
tratamiento de neurocisticercosis. Esta indicado únicamente en el tratamiento de infecciones
intestinales por helmintos. EL METRONIDAZOL NO TIENEN ACCIÓN SOBRE LA
NEUROCISTICERCOSIS.

PIRIMETAMINA La Pirimetamina en combinación con sulfonamidas de acción corta es efectiva para el


O IVERMECTINA tratamiento con malaria, toxoplasmosis e isosporiasis. Y la Ivermectina se utiliza para
tratamiento por infecciones por helmintos y ectoparásitos. Es el tratamiento de elección para
oncocercosis, strongiloidiasis, larva migrans cutánea y escabiasis. NINGUNO DE LOS DOS
MEDICAMENTOS ESTÁN INDICADOS EN CISTICERCOSIS.

NICLOSAMIDA La Niclosamina es un antihelmíntico que no se absorbe de forma adecuada y el Tiabendazol;


O TIABENDAZOL prácticamente ya no se utiliza, debido a su alta frecuencia de reacciones adversas.

Bibliografía:
1. LONGO DL, FAUCI AS, KASPER DL, HAUSERSL, JAMESON JL, LOSCALZOJ. HARRISON. PRINCIPIOS DE
MEDICINA INTERNA, 18A EDICIÓN. MC GRAW HILL. NEW YORK, USA. 2012, PP 1762. 2. HEYMANN, DAVID L.
EL CONTROL DE LAS ENFERMEDADES TRANSMISIBLES, 19A EDICIÓN. OPS. WASHINGTON, D.C. 2011, PP
692.
ANÁLISIS DEL CASO CLÍNICO

IDENTIFICACIÓN DEL REACTIVO


Area: MEDICINA INTERNA
Especialidad: GASTROENTEROLOGÍA
Tema: ENFERMEDAD ÁCIDO PÉPTICA
Subtema: DISPEPSIA FUNCIONAL Y GASTRITIS

CASO CLÍNICO CON UNA PREGUNTA

FEMENINA DE 45 AÑOS QUE ACUDE POR PRESENTAR SINTOMATOLOGÍA COMPATIBLE CON REFLUJO
GASTROESOFÁGICO IMPORTANTE POR LO MENOS 4 VECES POR SEMANA, ACOMPAÑADA DE ANSIEDAD E
INSOMNIO EVIDENTE, AFECTANDO SU CALIDAD DE VIDA. SE REALIZA ENDOSCOPIA LA CUAL SE REPORTA
COMO NORMAL.

femenino de 45 años.

-.

sintomatologia de re ujo gastroesofágico


por lo menos 4 veces por semana
acompañandose de ansiedad e insomnio
que afecta su calidad de vida.

-.

endoscopia normal.

202 - CON BASE EN EL CUADRO CLÍNICO LA PACIENTE DEBE SER TRATADA POR MEDIO DE:

AMITRIPTILINA Algunos autores han recomendado el uso de antidepresivos tricíclicos para el tratamiento más
que de reflujo gastroesofágico para pacientes con ulcera gástrica, sin embargo hasta el momento
los efectos secundarios sobrepasan a los beneficios aportados al paciente por lo que no se deben
administrar para tratar esta enfermedad.

CISAPRIDA Los procinéticos son fármacos que mejoran los síntomas de reflujo al aumentar la presión del
esfínter esofágico inferior y acelerar el vaciamiento gástrico. Sin embargo siempre deben
combinarse con algún fármaco que disminuya la secreción de ácido gástrico de lo contrario no
son tan eficaces.
SUCRALFATO La combinación de sucralfato con un inhibidor de bomba de protones se utiliza en los casos de
úlcera gástrica o duodenal. El inhibidor de bomba de protones reduce la secreción de ácido
gástrico mientras que el sucralfato se convierte en una pasta viscosa que se une a los lugares de
úlcera activa. En pacientes con reflujo gastroesofágico, el uso de sucralfato, solo estaría indicado
cuando este es secundario a sales biliares y no deberá combinarse con un inhibidor de bomba de
protones, ya que cuando existe reflujo biliar el uso de inhibidores de bomba de protones
disminuye la efectividad del sucralfato.

OMEPRAZOL La ENFERMEDAD POR REFLUJO GASTROESOFÁGICO se considera como la condición que aparece
cando el reflujo del contenido del estómago produce síntomas molestos y/o complicaciones. La
enfermedad por reflujo se manifiesta de diferentes formas: esofagitis erosiva, esofagitis no
erosiva, pirosis funcional. Las principales manifestaciones asociadas son: - Síntomas digestivos:
regurgitación, pirosis, vómito, disfagia, odinofagia, hematemesis, dispepsia, trastornos
nutricionales, disminución ponderal. - Síntomas extraesofágicos: apena, cianosis sibilancias,
estridor, tos crónica, asma, bronquiolitis, neumonía, laringotraqueitis, Crup, trastornos de
fonación, otitis, dolor torácico, escurrimiento nasal posterior, deterioro del esmalte dentario. La
reducción del la acidez del jugo gástrico no impide el reflujo pero mejora los síntomas y permite
a la mucosa sanar. Es por ello que se considera que EL TRATAMIENTO DE ELECCIÓN SON LOS
INHIBIDORES DE LA BOMBA DE PROTONES (IBP). Estos son comparativamente eficaces:
omeprazol (20 mg/día), lansoprazol (30 mg/día), pantoprazol (40 mg/día), esomeprazol (40 mg/
día) o rabeprazol (20 mg/día) durante ocho semanas. Los antagonistas de los receptores H2
también pueden ser utilizados pero con mejor eficacia.

Bibliografía:
1. LONGO DL, FAUCI AS, KASPER DL, HAUSERSL, JAMESON JL, LOSCALZOJ. HARRISON. PRINCIPIOS DE
MEDICINA INTERNA, 18A EDICIÓN. MC GRAW HILL. NEW YORK, USA. 2012, PP 2434-2435.
ANÁLISIS DEL CASO CLÍNICO

IDENTIFICACIÓN DEL REACTIVO


Area: MEDICINA INTERNA
Especialidad: ENDOCRINOLOGÍA
Tema: TRANSTORNOS DEL METABOLISMO Y DE LAS VITAMINAS
Subtema: OBESIDAD

CASO CLÍNICO CON UNA PREGUNTA

MASCULINO DE 40 AÑOS CON ANTECEDENTE DE CARDIOPATÍA ISQUÉMICA ACUDE A VALORACIÓN


NUTRICIONAL PARA DISMINUIR FACTORES DE RIESGO.

masculino de 40 años.

antecedente de cardiopatía isquémica.

-.

"factores de riesgo".

-.

203 - CORRESPONDE AL SITIO DE DEPÓSITO DE TEJIDO ADIPOSO QUE CONFIERE UN MAYOR RIESGO DE
MORBILIDAD EN ÉSTE PACIENTE.

MUSCULAR. En los últimos años se ha comprobado que un exceso de peso graso está estrechamente
relacionado con el riesgo de sufrir diferentes enfermedades, tales como problemas
cardiovasculares, hipertensión, diabetes tipo 2, obesidad, sobrepeso, ciertos tipos de cáncer,
dislipidemia, síndrome metabólico, alteraciones en el sistema inmune, etc.

SUBCUTÁNEO. Al día de hoy la obesidad está considerada como la epidemia del siglo XXI. Los últimos estudios
que se vienen realizando en nuestro país reflejan que la prevalencia de la obesidad infantil se ha
duplicado en los últimos diez años. La obesidad, entre otros problemas médicos, está
estrechamente asociada con mayor riesgo de sufrir enfermedad cardiovascular. Ésta puede afectar
no sólo a las grandes arterias y favorecer el desarrollo de infarto del miocardio y de accidente
cerebrovascular, sino que además puede favorecer las alteraciones microvasculares que participan
en el desarrollo de retinopatía, nefropatía e insuficiencia cardíaca.
ABDOMINAL. UN EXCESO DE MASA GRASA ABDOMINAL Y VISCERAL ESTÁ CONSIDERADO, EN LOS ESTUDIOS
EPIDEMIOLÓGICOS, COMO UNO DE LOS MÁS IMPORTANTES FACTORES DE RIESGO DE
ENFERMEDAD CARDIOVASCULAR. • Durante los últimos años se han llevado a cabo gran número
de investigaciones acerca de la acumulación de masa grasa en diferentes regiones corporales,
observándose una estrecha relación entre ésta y diversas patologías. ES MÁS CORRECTO
ESPECIFICAR "VISCERAL" YA QUE ES AHÍ DONDE SE PRODUCE EL PROCESO PATOLÓGICO DE
REACCIÓN INFLAMATORIA CAUSAL DE LA ENFERMEDAD CARDIOVASCULAR.

VISCERAL. • La grasa visceral está contenida en la parte interna de las cavidades corporales, envolviendo
órganos, sobre todo abdominales y está compuesta por la grasa mesentérica y la grasa de los
epiplones. • El tejido adiposo visceral está constituido por adipocitos de un tamaño muy reducido,
con poca capacidad de almacenamiento, es más vascularizado, y tiene mayor inervación simpática
y gran número de receptores adrenérgicos, lo que facilita una "mayor actividad metabólica". • Los
adipocitos viscerales hipertróficos hiperplásicos tienen menor densidad de receptores para
insulina y mayor densidad de receptores lo que condiciona el aumento de las tasas de lipólisis, que
facilita la diapédesis de monocitos hacia el estroma adiposo visceral, que inicia un ciclo
proinflamatorio con repercusión local y sistémica. "LA DISFUNCIÓN ENDOTELIAL, FAVORECIDA
POR EL PROCESO INFLAMATORIO, PUEDE SER EL VÍNCULO DE UNIÓN ENTRE LA OBESIDAD
VISCERAL Y LA ENFERMEDAD CARDIOVASCULAR".

Bibliografía:
1. LONGO DL, FAUCI AS, KASPER DL, HAUSERSL, JAMESON JL, LOSCALZOJ. HARRISON. PRINCIPIOS DE
MEDICINA INTERNA, 18A EDICIÓN. MC GRAW HILL. NEW YORK, USA. 2012, PP 1993-1994. 2. DIAGNÓSTICO Y
TRATAMIENTO DEL SOBREPESO Y OBESIDAD EXÓGENA. EVIDENCIAS Y RECOMENDACIONES: GUÍA DE
PRÁCTICA CLÍNICA. MÉXICO: CENETEC; 2018.

http://www.cenetec-difusion.com/CMGPC/GPC-IMSS-046-18/ER.pdf
ANÁLISIS DEL CASO CLÍNICO

IDENTIFICACIÓN DEL REACTIVO


Area: PEDIATRÍA
Especialidad: URGENCIAS PEDIÁTRICAS
Tema: URGENCIAS ORTOPÉDICAS
Subtema: LUXACIONES Y FRACTURAS DE MIEMBRO INFERIOR

CASO CLÍNICO SERIADO

ADOLESCENTE DE 16 AÑOS HABITUADA A CORRER CON FRECUENCIA EN COMPETENCIAS, REFIERE DOLOR


EN EL EMPEINE, QUE AUMENTA CON LA ACTIVIDAD FÍSICA NORMAL Y DESDE HACE 5 DÍAS HA
INCREMENTADO EL DOLOR EN INTENSIDAD Y DURACIÓN.

16 años

deportista (atletismo)

dolor en el empeine

podra presentar dolor a la palpaciÓn de la


zona lesionada

204 - EL DIAGNÓSTICO MAS PROBABLE ES:

ARTRITIS El término ARTRITIS se utiliza para definir a la inflamación de las articulaciones. Existen muchos tipos
de artritis, el más común en la infancia es la artritis idiopática juvenil, la cual se denomina también
artritis reumatoide juvenil; esta enfermedad constituye además la enfermedad crónica más frecuente
de la infancia. Se desconoce su origen, pero se sabe que tiene una patogenia autoinmnitaria.
Clínicamente se caracteriza por “edema intraarticular” o por la presencia de dos o más de los
siguientes síntomas: “limitación de la amplitud del movimiento”, “sensibilidad o dolor con el
movimiento” y “aumento de calor o eritema”. Los síntomas iniciales pueden ir desde sutiles hasta
agudos severos, y a menudo se manifiestan con “rigidez matutina” con cojera o efecto de gel
después de la inactividad. LA SOLA PRESENCIA DE DOLOR NO ES SUFICIENTE PARA SOSPECHAR
ARTRITIS EN ESTA PACIENTE.
BURSITIS Las bursas o bolsas serosas están en diferentes partes del pie, entre el hueso y el tendón, en las zonas
donde existe un mayor roce entre estos elementos. La finalidad de las bursas, es evitar la fricción
excesiva de los tendones sobre el hueso y así facilitar el movimiento de los tendones. Se denomina
BURSITIS a la inflamación de las bolsas sinoviales, la cual puede estar condicionada por diferentes
motivos; la causa más frecuente es por pequeños traumatismos (microtraumatismos) repetidos en un
mismo punto, generalmente relacionados con el deporte y la actividad laboral diaria. Las
deformidades de los pies y el uso de calzado inadecuado pueden aumentar el rozamiento que
soportan estas bolsas y con ello aparece la irritación y la inflamación de éstas. Algunas enfermedades
del metabolismo, así como las enfermedades reumáticas también favorecen la aparición de esta
enfermedad. Las infecciones se producen cuando un germen invade la bursa ya sea por una herida
cercana o bien a través de la sangre. Clínicamente, la bursitis se manifiesta con dolor e hinchazón
localizada. El dolor aumenta al contraer el tendón cercano a la bursa inflamada y frecuentemente al
rozar con el calzado. Las bursas que más frecuentemente se ven afectadas son, aquellas alrededor del
talón, cercanas al calcáneo y al tendón de Aquiles (retroaquílea, retrocalcánea), pero las
metatarsianas también pueden verse afectadas. Cuando el causante es un germen, la inflamación
suele ser mayor, y en ocasiones se acompaña de fiebre. El tratamiento de la bursitis depende de la
causa, cuando ésta es bien conocida, habrá que realizar un tratamiento dirigido de la enfermedad.
Están bien indicadas las medidas generales mediante antiinflamatorios y cambio de calzado.
Adicionalmente se pueden realizar inyecciones con corticoides (infiltraciones) en el interior de la
bursa. Cuando la causa es infecciosa, se deberá drenar el contenido y/o extirpar la bolsa infectada
mediante cirugía, siempre acompañado de tratamiento con antibióticos. DEBIDO AL ANTECEDENTE
DE PRACTICAR DEPORTE (ATLETISMO) DE MANERA FRECUENTE SE DEBERÁ SOSPECHAR EN
PRIMERA INSTANCIA DE LA INFLAMACIÓN DE LAS BURSAS, DEBIDO AL TRABAJO EXCESIVO DE LOS
TENDONES PARA REALIZAR LOS MOVIMIENTOS PROPIOS DE LA MARCHA. ES POR ESO QUE
DEBERÁS ELEGIR A LA BURSITIS COMO LA MEJOR RESPUESTA PARA ESTE CASO.

MIOSITIS La MIOSISTIS OSIFICANTE es una entidad en la cual se forma hueso extraesquelético (ectópico) que
OSIFICANTE comienza en zonas de fascitis, y es “secundaria a intervención quirúrgica, traumatismo, quemaduras
o lesión neurológica”. El hueso neoformado suele organizarse tanto en forma lineal como trabecular,
y contiene osteoblastos y osteoclastos que realizan las funciones de remodelación activa, puede
incluso tener elementos medulares bien desarrollados. Cuando la formación de hueso ectópico
ocurre en un trastorno hereditario se denomina fibrodisplasia osificante progresiva. DADO QUE EN
EL CASO NO SE REFIERE ESPECÍFICAMENTE ANTECEDENTES DE LESIONES PREVIAS EN EL PIE
(TRAUMATISMOS, QUEMADURAS, CIRUGÍA), ES MUY POCO PROBABLE QUE LA SINTOMATOLOGÍA
DEL PACIENTE ESTÉ ASOCIADA A UNA MIOSITIS OSIFICANTE.

FRACTURA Las FRACTURAS DE ESTRÉS son producto de microtraumatismos repetitivos, en corredores, “la zona
DE ESTRÉS mayormente afectada es la tibia”, aunque los metatarsianos también pueden afectarse en este tipo
de atletas. Estas fracturas están condicionadas por microfracturas que en un principio pasan
desapercibidas pero que con el tiempo van ocasionando dolor, aumento de volumen y disminución
de los arcos de movilidad por dolor. El dolor tiende a ser insidioso (mal definido) con la actividad y
que evoluciona hacia el empeoramiento. Hablando específicamente de la fractura por estrés de los
metatarsianos, durante la exploración es posible encontrar hipersensibilidad de la parte media de la
diáfisis del metatarsiano, sobre todo del 2º al 3er metatarsiano. Cuando el paciente refiere un dolor
vago, muy mal definido, es posible que la fractura se encuentre en el hueso navicular o escafoides, a
diferencia de otras fracturas, es muy difícil su identificación durante la exploración, pero algunas
veces suele dar datos de hipersensibilidad alrededor del escafoides. La reacción perióstica es posible
que “no se observe en las radiografías cuando el paciente solicita atención por la presencia de
dolor”, y no es sino hasta 2 o 3 semanas después de iniciado el cuadro clínico cuando exista una
evidencia radiográfica de la lesión ósea. La gammagrafía ósea y la resonancia magnética son estudios
de elección para el diagnóstico en estos casos. El tratamiento para este tipo de fracturas consiste en
inmovilización y descarga por 8 semanas. Cumplido el tratamiento, se recomienda documentar la
consolidación por medio de tomografía computada. DEBES CONSIDERAR A LA FRACTURA POR
ESTRÉS COMO LA SEGUNDA MEJOR OPCIÓN DIAGNÓSTICA PARA ESTE CASO. LA RAZONES POR LAS
QUE DEBERÁS CONSIDERARLA ASÍ ES PORQUE: 1. En corredores es más frecuente la fractura de
estrés en tibia que en pie. 2. Aunque existe un poco de controversia, las fracturas de estrés se
presentan más a menudo en adultos con alguna patología agregada, como la osteoporosis o la
diabetes (enfermedad de Charcot). 3. En corredores, estas lesiones se dan más a menudo en adultos
jóvenes, entre los 18 y 25 años. 4. Los infantes y adolescentes presentan cierta elasticidad ósea que
permite soportar mejor las cargas sobre el hueso que las personas adultas, donde el tejido es más
rígido y más propenso a fracturas.

Bibliografía:
1. BHERMAN R. NELSON TRATADO DE PEDIATRÍA, 15ª EDICIÓN. MC GRAW HILL. INTERAMERICANA. 1997. 2.
KLIEGMAN R, STANTON B, GEME J, SCR N, BHERMAN R. NELSON TRATADO DE PEDIATRÍA, 19ª EDICIÓN.
ELSEVIER. ESPAÑA 2013. 3. LONGO DL, FAUCI AS, KASPER DL, HAUSERSL, JAMESON JL, LOSCALZOJ.
HARRISON. PRINCIPIOS DE MEDICINA INTERNA, 18A EDICIÓN. MC GRAW HILL. NEW YORK, USA. 2012.

205 - EL DIAGNÓSTICO SE CONFIRMA CON:

RADIOGRAFÍA La radiografía se deberá solicitar precisamente para realizar el diagnóstico diferencial entre
ANTERIOR Y fractura o afección de partes blandas (bursitis o tendinitis). Las precisamente podrán mostrar la
LATERAL DE PIE presencia de solución de continuidad en algún hueso del pie y con ello diagnosticar la fractura,
que en este caso pudiera ser por estrés, o bien en el caso de no evidenciar la fractura se
corrobora el diagnostico de lesión de partes blandas.

GAMMAGRAMA Éste método diagnóstico está reservado para el caso en el que sospecha de patologías de
ÓSEO origen infeccioso como ya se ha mencionado en preguntas anteriores (infecciones óseas) por lo
tanto este no será un estudio que deberá solicitar ante un caso como el presente.

DETERMINACIÓN Están indicados como coadyuvante en el diagnóstico de enfermedades autoinmunes lo cual no


DE es el caso del presente estudio por lo tanto podemos descartar esta opción de respuesta.
ANTICUERPOS
ANTINUCLEARES

EXAMEN FÍSICO El examen físico detallado de la paciente desde el momento en que entra al consultorio nos
dará la pauta a seguir puesto que la marcha será claudicante, antiálgica, con facies de dolor,
incluso con limitación de la marcha por el mismo dolor, en seguida el interrogatorio que nos
revelara el hecho de que puede existir una lesión a nivel del pie, la observación de la zona nos
deberá hacer sospechar sobre lesión en la zona del empeine y finalmente la palpación nos
orientara a hacer un diagnóstico diferencial entre fractura o inflamación de las bursas o
tendinitis esta es la opción de respuesta más acertada al planteamiento del caso.

Bibliografía:
NELSON TRATADO DE PEDIATRÍA. BHERMAN RE. MC GRAW HILL. INTERAMERICANA. EDICIÓN 15A. 1997.
PAG. 2444-5.

206 - EL TRATAMIENTO QUE DEBE DE INICIARSE EN ESTE PACIENTE ES CON:

PRACTICAR La bursectomía estará indicada en aquellos casos en que sospeche de etiología infecciosa
BURSECTOMÍA de la bursitis ayudado de antibioticoterapia, siendo esta la última opción de respuesta
para el caso en cuestión.

COLOCACIÓN DE En los casos en que el aumento de volumen sea severo pudiera estar indicado la
FÉRULA Y REPOSO inmovilización de la extremidad y aplicación de vendaje algodonoso para ayudar al
tratamiento farmacológico así como el reposo de la misma. Por lo tanto esta podrá ser la
tercera opción de respuesta para el presente caso.

ADMINISTRACIÓN DE Los esteroides tienen excelente efecto en las enfermedades inflamatorias o de tipo
ESTEROIDES traumático en que se presenta aumento de columna secundario, pero no deberán ser la
primera elección de tratamiento en pacientes Siempre se deberá seguir una línea y niveles
de tratamiento es decir en primer lugar los AINES, analgésico puros y en segundo lugar los
corticoesteroides, ambos por su efecto analgésico y antiinflamatorio siendo entonces esta
la segunda opción de respuesta.
ADMINISTRACIÓN DE Como se ha mencionado en ocasiones anteriores, el primer paso a seguir en las lesiones
ANTIINFLAMATORIOS traumáticas o por estrés es el reposo seguido de la administración de analgésico,
NO ESTEROIDEOS antiinflamatorio e incluso miorelajantes. En el presente caso se deberá administrar
analgésico no estará del todo indicada la inmovilización debido a que parte del proceso
del tratamiento de las bursitis es la movilización temprana mediante la ayuda de terapia
física y rehabilitación. Por lo tanto esta será le respuesta más adecuada a la secuencia de la
pregunta.

Bibliografía:
NELSON TRATADO DE PEDIATRÍA. BHERMAN RE. MC GRAW HILL. INTERAMERICANA. EDICIÓN 15A. 1997.
PAG. 2444-5.

FIN DEL CASO CLÍNICO SERIADO


ANÁLISIS DEL CASO CLÍNICO

IDENTIFICACIÓN DEL REACTIVO


Area: MEDICINA INTERNA
Especialidad: INMUNOALERGIA
Tema: URTICARIA Y ANGIOEDEMA
Subtema: URTICARIA Y ANGIOEDEMA

CASO CLÍNICO CON UNA PREGUNTA

MUJER DE 22 AÑOS DE EDAD, ESTUDIANTE UNIVERSITARIA. REFIERE QUE EN PERÍODO DE EXÁMENES ES


COMÚN QUE LE APAREZCAN RONCHAS EN LAS PIERNAS, LAS CUALES TIENEN UNA SUPERFICIE ELEVADA, UN
HALO ERITEMATOSO Y LE GENERAN PRURITO. ALGUNAS VECES SE AUTOMEDICA LORATADINA CON LO QUE
LAS LESIONES MEJORAN. GENERALMENTE APARECEN Y DESAPARECEN SIN DEJAR HUELLA, PERO ESTA VEZ
DESAPARECEN, REAPARECEN Y CAMBIAN DE LUGAR DESDE HACE 6 SEMANAS.

22 años.

: lesiones tipo urticaria asociada a estrés,


desde hace 6 semanas=URTICARIA
CRÒNICA.

207 - EN ESTE MOMENTO SE DEBERÁN INDICAR:


ANTIHISTAMÍNICOS LO PRIMERO QUE DEBERÁS HACER ES DETERMINAR EL DIAGNÓSTICO QUE, EN ESTE CASO,
DE SEGUNDA LO MÁS PROBABLE ES QUE SE TRATE DE UNA URTICARIA CRÓNICA. La URTICARIA
GENERACIÓN CRÓNICA es una reacción delimitada de la piel que se distingue por la aparición de ronchas,
con edema central y un halo eritematoso, acompañada de prurito o sensación de
quemadura, con naturaleza migratoria, que desaparece en 1 a 24 hrs, no deja huella y
persiste por más de 6 semanas. Para su tratamiento, SE UTILIZARÁN ANTIHISTAMÍNICOS DE
SEGUNDA GENERACIÓN NO SEDANTES (loratadina, cetirizina, desloratadina, levocetirizina,
epinastina, fexofenadina) a dosis habituales. IMPORTANTE: la levocetiricina se recomienda
más que la desloratadina a dosis de 5mg/día para el manejo de la urticaria crónica. El
algoritmo a seguir en el manejo de esta patología es el siguiente: 1. Antihistamínicos de
segunda generación. 2. Si persisten los síntomas después de 2 semanas: incrementar la dosis
4 veces. 3. Si persisten los síntomas después de 1 a 4 semanas: agregar antagonistas de
leucotrienos o cambio de antihistamínico, además de esteroides sistémicos por 3 a 7 días en
exacerbaciones. 4. Si persisten los síntomas por 1 a 4 semanas: agregar ciclosporina A o
dapsona o omalizumb, más esteroides en exacerbaciones por 3 a 7 días. IMPORTANTE:
Deberás poner especial atención a los datos que se te ofrecen en el caso clínico. En primer
lugar, el caso clínico dice que “algunas veces” la paciente se automedica loratadina, más no
que se encontrara tomándola de forma constante a lo largo de 6 semanas. Lo ideal para este
caso será iniciar la administración “adecuada” diaria de antihistamínicos de segunda
generación a dosis estándar, de no mejorar después de dos semanas, incrementar la dosis 4
veces; de persistir los síntomas de 1 a 4 semanas, entonces sí, agregar inhibidores
leucotrienos o cambiar de antihistamínico, además de administrar esteroides sistémicos por
3 a 7 días, durante las exacerbaciones.

INHIBIDORES DE Se ha propuesto que en pacientes con síntomas persistentes después de una a cuatro
LEUCOTRIENOS semanas, a pesar del tratamiento con antihistamínicos, se agregue un antagonista de
leucotrienos o se cambie de antihistamínico. NO SE UTILIZAN SINO HASTA VER SI LA
PACIENTE MEJORA CON LA ADMINISTRACIÒN DE ANTIHISTAMÌNICOS.

ANTIHISTAMÍNICOS Los antihistamínicos de primera generación son útiles en la noche, cuando el paciente tiene
DE PRIMERA problemas de insomnio; sólo se indica si no se conduce automóvil. EN GENERAL NO SE
GENERACIÓN RECOMIENDA EL USO DE ANTIHISTAMÍNICOS DE PRIMERA GENERACIÓN.

ESTEROIDES Durante las exacerbaciones se indicará esteroides sistémicos de tres a siete días. LOS
SISTÉMICOS ESTEROIDES NO ESTÁN INDICADOS PARA EL MANEJO DE PRIMERA LÍNEA.

Bibliografía:
1. GUÍA DE PRÁCTICA CLÍNICA. DIAGNÓSTICO Y TRATAMIENTO DE LA URTICARIA CRÓNICA EN ADULTOS
EN LOS TRES NIVELES DE ATENCIÓN. MÉXICO: SECRETARÍA DE SALUD.2013.

http://www.cenetec.salud.gob.mx/descargas/gpc/CatalogoMaestro/issste_661_13_urticariacronica/urticaria_060613.pdf
ANÁLISIS DEL CASO CLÍNICO

IDENTIFICACIÓN DEL REACTIVO


Area: CIRUGÍA
Especialidad: CIRUGÍA ABDOMINAL
Tema: SANGRADO DE TUBO DISGESTIVO ALTO Y BAJO
Subtema: VARICES ESOFÁGICAS

CASO CLÍNICO CON UNA PREGUNTA

HOMBRE DE 53 AÑOS DE EDAD, ES ENCONTRADO EN SU DOMICILIO INCONSCIENTE Y CON DATOS CLÍNICOS


COMPATIBLES DE CHOQUE. LO ENCONTRARON SOBRE UN CHARCO DE SANGRE Y CON RASGOS DE SANGRE
EN BOCA Y NARÍZ, APARENTEMENTE SECUNDARIO A HEMATEMESIS. SE OBSERVA A LA EXPLORACIÓN TINTE
ICTÉRICO, TELANGIECTASIAS Y ERITEMA PALMAR.

masculino de 53 años de edad.

encontrado en su domicilio con datos


clínicos de choque. restos de sangre en
boca y nariz( probable secundario a
hematemesis).

-.

-.

-.

208 - LA CAUSA MAS PROBABLE DE LA HEMORRAGIA ENCONTRADA EN EL PACIENTE SI SE CONFIRMA


LA HEMATEMÉSIS ES:
ULCERA Las ÚLCERAS DUODENALES, antrales y gástricas son las responsables de 60%, 20% y 20% de las
GÁSTRICA perforaciones por enfermedad ácido-péptica, respectivamente. Entre un tercio y la mitad de las
PERFORADA. úlceras perforadas están relacionadas con el uso de AINES y usualmente ocurren en pacientes
ancianos. La perforación supone la penetración de la úlcera de todas las capas del estómago o
duodeno alcanzando la cavidad peritoneal. Se produce en un 5% de los ulcerosos y es más
frecuente en varones. El 90% de las perforaciones se localizan en la cara anterior del bulbo
duodenal y en un tercio de los pacientes es la primera manifestación de la enfermedad ulcerosa. La
clínica consiste en la aparición de un dolor brusco muy intenso epigástrico, puede irradiar a espalda
o extenderse al resto del abdomen. Hay contractura muscular, silencio a la auscultación y signo de
descompresión positivo. La radiografía simple de abdomen muestra neumoperitoneo en dos tercios
de los casos. El diagnóstico debe ser clínico y no retardarse. La mortalidad oscila entre el 10 y el
40%. Algunos no registran historia de enfermedad ácido péptica, pero en un interrogatorio
detallado se pueden encontrar claves: generalmente son personas de edad avanzada que reciben
AINES. El diagnóstico rápido es esencial, pues el pronóstico mejora si el paciente se trata en las
primeras seis horas; la mortalidad aumenta luego de 12 horas de la perforación. La historia clínica y
el examen físico son fundamentales, porque la perforación es un diagnóstico clínico. La presencia
de aire libre en la radiografía de tórax o eventualmente en la de abdomen vertical o decúbito
lateral son sugestivas o diagnósticas de perforación; sin embargo, cerca del 20% de los pacientes
con perforación no muestran aire libre en la radiografía. La evidencia de medio de contraste
hidrosoluble libre en la cavidad en unas vías digestivas o en la tomografía axial computadorizada
son confirmatorias, pero usualmente no son necesarias en la mayoría de los casos y pueden
demorar la cirugía. El tratamiento óptimo es la cirugía. Pero el manejo no operatorio, que incluye
succión nasogástrica, líquidos endovenosos, antibióticos y antiácidos por vía endovenosa puede ser
exitoso en pacientes seleccionados por ser de alto riesgo quirúrgico en los cuales el cuadro clínico
cede rápidamente en respuesta al manejo médico. Los pacientes mayores de 70 años tienen menor
probabilidad de responder al manejo no operatorio. Las úlceras gástricas perforadas se asocian con
una mayor mortalidad; la gastrectomía parcial es el procedimiento preferido. Algunos consideran
que al cierre con parche se debe agregar una vagotomía gástrica proximal como forma definitiva
de terapia en aquellos pacientes que requerirían una segunda intervención. LA GRAN PÉRDIDA
HEMÁTICA APARENTEMENTE POR HEMATEMESIS NO SE JUSTIFICA POR ESTE DIAGNÓSTICO.

VARICES Al momento del diagnóstico, más de la mitad de los pacientes con CIRROSIS han desarrollado
ESOFÁGICAS. VÁRICES GASTROESOFÁGICAS y con la progresión de la enfermedad se presentan hasta en un 90%
de los pacientes. Las várices están presentes en aproximadamente 40% de los pacientes con cirrosis
compensada y en el 60% de los que tienen ascitis. La presencia de várices se correlaciona con la
gravedad de la enfermedad hepática: 20 a 40% de los pacientes con clase funcional A de Child-Pugh
tienen várices gastroesofágicas, comparado con más del 85% de los pacientes con clase C. Las
VÁRICES ESOFÁGICAS son un conjunto de venas longitudinales y tortuosas situadas
preferentemente en el tercio inferior del esófago, que cursan a través de varios niveles desde la
lámina propia hasta la submucosa profunda, pueden progresar hacia la parte superior del esófago o
hacia el estómago. Se comunican por medio de venas perforantes con una circulación colateral
paraesofágica extensa. Se forman como consecuencia de HIPERTENSIÓN PORTAL. Se desarrolla
hipertensión portal en prácticamente todos los pacientes con cirrosis. La ruptura y sangrado de las
várices son una complicación mayor de la hipertensión portal y conlleva una alta mortalidad. La
frecuencia es igual en ambos sexos. En adultos la causa más común es la cirrosis hepática. En niños,
la trombosis portal y cirrosis biliar secundaria son causas comunes. El sangrado de las várices es
aproximadamente el 10 a 30% de las hemorragias digestivas del tracto superior. Las várices
esofágicas no producen dolor ni molestias, excepto cuando sangran. El sangrado de las várices es
una complicación grave del daño hepático crónico y puede manifestarse de las siguientes maneras:
Vómitos con sangre (hematemésis), evacuaciones de color negro y mal olor (melena), lipotimia o
desmayo, anemia crónica, en casos de pequeños sangrados por gastropatía hipertensiva portal. La
frecuencia es igual en ambos sexos. En el examen físico se pueden encontrar: Palidez, presión
arterial baja, incremento de pulso y cambios en la presión sanguínea postural sugieren pérdida de
sangre, disnea, taquipnea. Puede presentar ictericia por insuficiente función hepática.
Telangiectasias, ginecomastia, eritema palmar, ascitis, edema, atrofia testicular. Venas dilatadas
radiales hacia afuera desde el ombligo (cabeza de medusa). Circulación colateral en abdomen.
Hígado puede ser de pequeño tamaño, esplenomegalia. Tacto rectal presenta melena. Solo un 40%
de las várices se manifiestan radiológicamente. Todo paciente con cirrosis debería ser sometido a
una endoscopia de rutina para la detección de várices. EL PACIENTE PRESENTA SIGNOS CLÍNICOS
DE HIPERTENSIÓN PORTAL POR LO QUE DEBE SOSPECHARSE DE UNA HEMORRAGIA SECUNDARIA
A RUPTURA DE VÁRICES ESOFÁGICAS.
ÚLCERA La ÚLCERA PÉPTICA (UP) es una enfermedad heterogénea atribuible a una serie de factores, que de
DUODENAL. forma aislada o en combinación, actúan produciendo un desequilibrio entre los elementos
agresivos y defensivos de la mucosa gastroduodenal que conlleva a la aparición de lesiones en el
estómago y/o en el duodeno. En la úlcera duodenal (UD) la acción del ácido supondría el factor
agresivo. Entre los factores patogénicos más conocidos están los AINES, la infección por
Helicobacter pylori, las alteraciones del vaciamiento gástrico y el reflujo biliar duodeno-gástrico.
Mecanismos patogénicos en la úlcera duodenal (UD): predominan los fallos en los factores
agresivos de la mucosa, aumento de la secreción de ácido y pepsina, aumento del número de
células G antrales, respuesta exagerada en la liberación de gastrina, liberación selectiva de la
gastrina G-17, fallo de los mecanismos de inhibición duodenal de la secreción del ácido,
hiperpepsinogenemia, vaciado gástrico rápido, alteración a nivel de los receptores duodenales para
el ácido. La incidencia máxima de la úlcera duodenal se produce entre los 55 y los 65 años, mientras
que en el caso de la gástrica esta incidencia alcanza una meseta a los 25 años en el varón y a los 45
en la mujer. Clásicamente se ha considerado un patrón clínico típico de la enfermedad ulcerosa, que
consistía en la presencia de ardor, “hambre dolorosa” o molestia epigástrica que aparecía de una a
tres horas tras las comidas, período en el que los alimentos ya han sido evacuados y por tanto no
tamponan la acidez gástrica. También se consideraba típicamente que la sintomatología se aliviaba
con la nueva ingestión o con la toma de alcalinos. Los síntomas típicos son ardor y dolor localizado
en el epigastrio, que despierta por la noche y evoluciona por temporadas. El paciente ulceroso
puede tener además nauseas, vómitos, diarrea, estreñimiento, hinchazón abdominal, alteración del
ritmo intestinal, flatulencia, meteorismo, pirosis, pesadez gástrica, sensación de gases, anorexia,
pérdida de peso y anemia. Es importante establecer si el paciente tiene Helicobacter pylori, se debe
de realizar una endoscopia para la toma de biopsias y corroborar la presencia de la úlcera. Su
tratamiento obvio si cuenta con Helicobacter dar tratamiento de erradicación, suspender
medicamentos nocivos a la mucosa, medidas higiénico-dietéticas, e iniciar antagonistas H2 o
inhibidores de la bomba de protones. LA GRAN PÉRDIDA HEMÁTICA APARENTEMENTE POR
HEMATEMESIS NO SE JUSTIFICA POR ESTE DIAGNÓSTICO.

ECTASIA Prominencia anormal de los capilares de la mucosa antral gástrica, que se acompaña de
VASCULAR adelgazamiento de la misma mucosa, con presencia de bandas rojas brillantes y atrofia gástrica en
ANTRAL el antro del estómago, que se irradia longitudinalmente entre el píloro y este último, recordando la
GÁSTRICA. visión por endoscopio a las bandas de la superficie de una sandía, lo cual corresponde a los
hallazgos típicos de la ectasia vascular del antro gástrico”. Es más común en las mujeres de mayor
edad, con una relación 9:1 respecto a los hombres, y con una edad media de presentación de 69.1
años con un rango de 40-90. Aunque se desconoce la verdadera incidencia, se estima que esta
patología es responsable entre 1,2 y el 8% del sangrado de vías digestivas altas, y en casos de
insuficiencia renal crónica alcanza hasta un 24%. Existen muy pocos estudios prospectivos dirigidos
a conocer la prevalencia o incidencia de esta patología. El hecho de no reconocer a la ectasia
gástrica como causa de sangrado gastrointestinal o confundirla con otras enfermedades (gastritis
hemorrágica aguda, carcinoma gástrico, etc.) se suma a la dificultad para identificar la verdadera
frecuencia de su presentación. La etiología de esta condición permanece desconocida y se considera
una malformación arteriovenosa; actualmente se especula que la formación del "estómago de
sandía" es el resultado de episodios recurrentes de prolapso de la mucosa antral. La
hipercontractilidad antral prolapsa la mucosa fijada laxamente a través del píloro, con posterior
traumatismo e isquemia de la mucosa. La mayoría de los pacientes con ectasia vascular antral cursa
habitualmente con pérdida crónica y oculta de sangre o con hemorragia aguda recurrente que
desencadena anemia severa ferropénica. Las presentaciones clínicas más comunes son: Anemia por
deficiencia férrica (88%), Heces guayaco positivo (42%), Melenas (15%) Hematoquezia 1%.El
sangrado gastrointestinal oculto es dependiente de transfusiones, que en promedio alcanza a
requerir diez unidades en un período de doce meses. Aunque la pérdida de sangre raramente
excede los 250 mg/día, el remplazo férrico por sí solo, no puede mantener los niveles de
hemoglobina a nivel normal. Hasta un 50% de los pacientes presenta sangrados recurrentes, los
cuales son dados por: No obliteración de las ectasias, presencia de múltiples ectasias, ectasias no
diagnosticadas en otras áreas del tubo digestivo, aparición de nuevas ectasias .Se han descrito
terapias conservadoras con aporte de fierro y transfusiones sanguíneas según necesidad, terapia
farmacológica con estrógenos y progesterona, esteroides, octreótide, antiácidos, inhibidores H2,
que no han mostrado ser efectivas. La cirugía (antrectomia) y las terapia endoscópicas: YAG láser,
argón plasma y electrocoagulación bipolar, son las únicas terapias que han mostrado se efectivas.
La cirugía no se encuentra exenta de mortalidad y se le considera un tratamiento radical e invasivo,
aunque con las suturas mecánicas actuales y el adecuado soporte nutricional la mortalidad de esta
cirugía invasiva ha disminuido considerablemente en rangos de 1-2%. Las terapias endoscópicas,
son menos invasivas, pueden realizarse en forma ambulatoria, y en la mayoría de los pacientes se
requiere más de una sesión para el manejo del sangrado y se han descrito recurrencias. NO SE
RELACIONA CON EL CUADRO CLÍNICO,
Bibliografía:
1. DIAGNÓSTICO Y TRATAMIENTO DE VÁRICES ESOFÁGICAS EN EL ADULTO. EVIDENCIAS Y
RECOMENDACIONES. GUÍA DE PRÁCTICA CLÍNICA. MÉXICO, CENETEC; 2017. 2. BRUNICARDI F, ANDERSEN
D, BILLIAR T, Y COLS. SCHWARTZ PRINCIPIOS DE CIRUGÍA, 9A EDICIÓN. MC GRAW HILL. 2011, PP 1111-114.

http://www.cenetec-difusion.com/CMGPC/IMSS-087-08/ER.pdf
ANÁLISIS DEL CASO CLÍNICO

IDENTIFICACIÓN DEL REACTIVO


Area: MEDICINA INTERNA
Especialidad: GASTROENTEROLOGÍA
Tema: ENFERMEDADES DEL INTESTINO DELGADO Y GRUESO
Subtema: INTESTINO IRRITABLE

CASO CLÍNICO CON UNA PREGUNTA

MUJER DE 28 AÑOS PRESENTA DESDE HACE 10 DÍAS DOLOR ABDOMINAL EN EL CUADRANTE INFERIOR
DERECHO. CUENTA CON EL ANTECEDENTE DE ESTREÑIMIENTO DESDE HACE 2 AÑOS QUE EMPEORA CON
ESTADOS DE NERVIOSISMO Y MEJORA CON LA SALIDA DE GASES. SE REALIZAN ESTUDIOS DE AMILASA,
ULTRASONIDO DE HÍGADO Y VÍAS BILIARES ADEMÁS DE COLONOSCOPIA, LAS CUALES SE REPORTAN
NORMALES.

mujer de 28 años de edad

estreñimiento desde hace 2 años que


empeora con estres y mejora con salida de
gases

dolor abdominal de 10 días de evolución en


cuadrante inferior derecho.

-.

amilasa sérica, usg hepático y de vías


biliares y colonoscopía normales.

209 - FÁRMACO INDICADO EN ESTE MOMENTO:

PANCREATINA LO PRIMERO EN ESTE CASO ES DEFINIR EL DIAGNÓSTICO CLÍNICO QUE EN ESTE CASO ES UN
SÍNDROME DE INTESTINO IRRITABLE. El SÍNDROME DE INTESTINO IRRITABLE (SII) se define
como una afección crónica de dolor o molestia abdominal asociado a alteraciones en el hábito
intestinal, durante al menos 3 días por mes en los últimos 3 meses, sin causa orgánica que lo
justifique. Las ENZIMAS PANCREÁTICAS están indicadas únicamente en aquellos pacientes que
tienen "insuficiencia pancreática exocrina" y esta se suele manifestar principalmente con diarrea
no con estreñimiento.
BROMURO DE Los grupos farmacológicos más utilizados en el control del Síndrome de Intestino Irritable (SII)
PINAVERIO son: - Antiespasmódicos. - Antidiarréicos. - Laxantes. - Antidepresivos. - Antibióticos. - Agentes
serotóninérgicos. Los ANTIESPASMÓDICOS incluye a aquellos que afectan directamente la
relajación del músculo liso como la mebeverina y pinaverio, y aquellos que actúan a través de las
vías anticolinérgicas como la diciclomina y la hiosciamina. Se indican en los pacientes con
síndrome de colon irritable ya que se tiene la hipótesis que la alteración en la motilidad intestinal
produce dolor, distensión y alteraciones en la defecación. Estos agentes pueden ser benéficos en
aquellos pacientes con dolor abdominal postprandial, distensión y urgencia fecal. Debido a que
los síntomas intermitentes de esta enfermedad se recomienda que se utilicen estos medicamentos
solo cuando sean necesarios o cuando se pueda anticipar algún estresor. Es importante comentar
que siempre los pacientes con síndrome de intestino irritable necesitan modificaciones en los
estilos de vida, antes de la prescripción de fármacos. Dentro de los antiespasmódicos que han
mostrado utilidad en el tratamiento SII encontramos: 1. Agentes relajantes directos del músculo
liso: - Trimebutina 100-200mg cada 8 horas. - Mebeverina 200mg cada 8 horas. - Pargeverina
10mg cada 8 horas. 2. Agentes anticolinérgicos o antimuscarínicos derivados de la escopolamina. -
Bromuro de butilhioscina 10mg cada 8 horas. - Dicicloverina 10mg cada 8 horas. 3. Agentes
anticolinérgicos, antimuscarínicos o antagonistas del calcio derivados de amonio. - Bromuro de
otilonio 40mg cada 8 horas. 4. Agentes bloqueadores de los canales del calcio. - Bromuro de
pinaverio 100 mg cada 8 o 12 horas. LA GPC VIGENTE EN NUESTRO PAÍS RECOMIENDA EL USO DE
BROMURO DE PINAVERIO Y BROMURO DE BUTIHIOSINA COMO PRINCIPALES
ANTIESPASMÓFICOS PARA EL TRATAMIENTO DE LOS SÍNTOMAS DEL SÍNDROME DE COLON
IRRITABLE. IMPORTANTE: Los medicamentos antiespasmódicos pueden ser utilizados como
tratamiento inicial en el control de los síntomas en SII, además pueden ser acompañado de
loperamida o psyllium plantago dependiendo de los síntomas predominantes (diarrea o
constipación). LOS ANTIESPASMÓDICOS CONSTITUYEN EL MANEJO INICIAL DEL SII.

SERTRALINA La SERTRALINA es un potente inhibidor de la recaptura de serotonina con indicaciones aprobadas


para el manejo de la depresión, así como de diversos trastornos de ansiedad. No se ha evaluado la
eficacia de esta clase de antidepresivos en el tratamiento del síndrome de intestino irritable y se
necesitan mayores datos confirmatorios de la eficacia de los inhibidores de la recaptura de
serotonina en el tratamiento del síndrome de intestino irritable. LA ÚLTIMA MODIFICACIÓN A LA
GPC PARA SII CONTEMPLA QUE: - Los antidepresivos fueron utilizados inicialmente en el manejo
de SII basado en la observación que la depresión y ansiedad son comorbilidades frecuentes en
estos pacientes. - Se recomienda utilizar antidepresivos en dosis menores a las usadas en
depresión para mejorar el dolor de origen visceral. - Los antidepresivos tiene propiedades
analgésicas independientemente de los efectos en mejorar el estado de ánimo; los tricíclicos, por
sus propiedades anticolinérgicas, también disminuyen el tiempo de tránsito intestinal, por lo que
proporcionan beneficio en SII con predominio de diarrea. - Se recomienda que por sus efectos en
el tránsito intestinal, los antidepresivos tricíclicos sean usados cautelosamente en pacientes con
SII con predominio de constipación. - En caso de SII con predominio de constipación o mixto, se
recomienda utilizar un antidepresivo inhibidores selectivo de recaptura de serotonina. - Para
pacientes con SII en quienes la depresión sea un comórbido los antidepresivos inhibidores
selectivos de recaptura de serotonina deben ser utilizados. EN CONCLUSIÓN LOS
ANTIDEPRESIVOS TRICÍCLICOS PUEDEN SER UTILIZADOS ANTE COMORBILIDAD DEPRESIVA EN
PACIENTES CON SII. OJO: NO CORRESPONDE AL MANEJO INICIAL DEL SII.

TEGASEROD Es muy importante hacer hincapié sobre el tratamiento farmacológico de los pacientes con
síndrome de intestino irritable solo como adyuvantes. El tratamiento depende de si se presenta
con diarrea o estreñimiento. En los pacientes con predominio de estreñimiento el tegaserod, que
es un agonista selectivo y parcial de los receptores 5-HT4, posee efecto estimulante
gastrointestinal al facilitar la transmisión colinérgica entérica. El tegaserod mejora de forma
significativa el dolor abdominal y la función intestinal. Desafortunadamente fue retirado del
mercado por la FDA por sus efectos cardiovasculares. ACTUALMENTE EL TEGASEROD NO SE
ENCUENTRA EN EL MERCADO POR SUS EFECTOS ADVERSOS, NO ESTA INDICADO ACTUALMENTE
PARA EL MANEJO DEL SÍNDROME DE INTESTINO IRRITABLE.

Bibliografía:
1. GUÍA DE PRÁCTICA CLÍNICA, DIAGNÓSTICO Y TRATAMIENTO DEL SÍNDROME DE COLON IRRITABLE.
MÉXICO: SECRETARIA DE SALUD; MARZO 2015. 2. LONGO DL, FAUCI AS, KASPER DL, HAUSERSL, JAMESON
JL, LOSCALZOJ. HARRISON. PRINCIPIOS DE MEDICINA INTERNA, 18A EDICIÓN. MC GRAW HILL. NEW YORK,
USA. 2012, PP 2499-2501.

http://www.cenetec.salud.gob.mx/descargas/gpc/CatalogoMaestro/042_GPC_ColonIrritable/IMSS_042_08_EyR.pdf
ANÁLISIS DEL CASO CLÍNICO

IDENTIFICACIÓN DEL REACTIVO


Area: MEDICINA INTERNA
Especialidad: ENDOCRINOLOGÍA
Tema: PATOLOGÍA DE LA GLÁNDULA SUPRARRENAL
Subtema: INSUFICIENCIA SUPRARRENAL PRIMARIA Y SECUNDARIA

CASO CLÍNICO CON UNA PREGUNTA

MUJER DE 35 AÑOS HOSPITALIZADA, CON DIAGNÓSTICO DE INSUFICIENCIA SUPRARRENAL.

mujer de 35 años de edad..

-.

hospitalizada con diagnóstico de


insu ciencia suprarrenal.

-.

-.

210 - EL SIGUIENTE HALLAZGO CLÍNICO LE PERMITIRÍA CON UN ALTO ÍNDICE DE PRECISIÓN


DIFERENCIAR ENTRE SI ES PRIMARIA O SECUNDARIA:

HIPERPIGMENTACIÓN. 1. La INSUFICIENCIA SUPRARRENAL PRIMARIA se da principalmente a causa de


suprarrenalitis autoinmunitaria. Otras causas menos comunes son la destrucción de las
glándulas suprarrenales a consecuencia de infección (la tuberculosis es común en países
en vías de desarrollo), hemorragia infiltración. Es extremadamente raro, pero también
contribuyen las causas innatas. 2. La INSUFICIENCIA SUPRARRENAL SECUNDARIA se debe
a la disfunción del componente hipotálamo-hipofisiario. Estos casos se asocian en su
mayoría a supresión iatrogénica (cirugía o radiación). Otras causas menos comunes son
apoplejía hipofisiaria ya sea por un adenoma hipofisiario infartado o por reducción
transitoria del aporta sanguíneo (enfermedad de Sheehan). En los pacientes con
insuficiencia suprarrenal secundaria la disminución en los niveles de ACTH además de la
disminución de su efecto sobre las glándulas suprarrenales para la síntesis de cortisol
"tampoco estimulan la síntesis de melanina por los melanocitos", por lo que estos
pacientes no tienen la hiperpigmentación característica de los pacientes con insuficiencia
suprarrenal primaria. PRIMARIA=HIPERPIGMENTACIÓN SECUNDARIA= PIEL PÁLIDA.
PÉRDIDA DE PESO. Este hallazgo puede presentarse tanto en la insuficiencia suprarrenal primaria como
secundaria.

NAUSEA Y VÓMITO. Este hallazgo puede presentarse tanto en la insuficiencia suprarrenal primaria como
secundaria.

DIARREA. Este hallazgo puede presentarse tanto en la insuficiencia suprarrenal primaria como
secundaria.

Bibliografía:
1. LONGO DL, FAUCI AS, KASPER DL, HAUSERSL, JAMESON JL, LOSCALZOJ. HARRISON. PRINCIPIOS DE
MEDICINA INTERNA, 18A EDICIÓN. MC GRAW HILL. NEW YORK, USA. 2012, PP 2956.
ANÁLISIS DEL CASO CLÍNICO

IDENTIFICACIÓN DEL REACTIVO


Area: MEDICINA INTERNA
Especialidad: HEMATOLOGÍA
Tema: ANEMIAS
Subtema: ANEMIAS HEMOLÍTICAS

CASO CLÍNICO CON UNA PREGUNTA

HOMBRE DE 40 AÑOS DE EDAD, QUE INGRESA CON DIAGNÓSTICO DE SÍNDROME ANÉMICO. A LA


EXPLORACIÓN SE ENCUENTRA ICTERICIA EN ESCLERÓTICAS, PALIDEZ LEVE DE TEGUMENTOS Y
ESPLENOMEGALIA. SE REPORTAN EXÁMENES DE LABORATORIO CON HEMOGLOBINA DE 8 G/DL,
LEUCOCITOS 9000, PLAQUETAS 275 000, RETICULOCITOS 8.8%. EL FROTIS DE SANGRE PERIFÉRICO
CORROBORA LA PRESENCIA DE RETICULOCITOS Y LA PRESENCIA DE ESFEROCITOS.

HOMBRE DE 40 AÑOS DE EDAD.

-.

-.

ICTERICIA EN ESCLERÓTICAS, PALIDEZ


LEVE DE TEGUMENTOS Y
ESPLENOMEGALIA. DATOS DE ANEMIA PB
HEMOLÍTICA.

HEMOGLOBINA DE 8 G/DL, LEUCOCITOS


9000, PLAQUETAS 275 000,
RETICULOCITOS 8.8%. EL FROTIS DE
SANGRE PERIFÉRICO CORROBORA LA
PRESENCIA DE RETICULOCITOS Y LA
PRESENCIA DE ESFEROCITOS. ANEMIA
REGENERATIVA, CON PRESENCIA DE
ESFEROCITOS, IGUAL A ANEMIA
HEMOLÍTICA.

211 - EL TRATAMIENTO INDICADO EN ESTE MOMENTO EN EL PACIENTE ES CON:


ESPLENECTOMIA. ESPLENECTOMÍA en ANEMIA HEMOLÍTICA AUTOINMUNE. Las indicaciones son la dependencia
de dosis de prednisona superiores a 20 mg al día o la aparición de efectos adversos a los
corticoides intratables. Consigue remisiones completas en aproximadamente el 50% de las
AHAI idiopáticas y el 30% de las secundarias. En caso de recaída debe descartar la existencia de
un bazo accesorio. Debido al riesgo se sobreinfección por gérmenes encapsulados debe
realizarse una vacunación en las dos semanas previas frente a neumococo y meningococo y
tratamiento urgente con antibioterapia ante cualquier episodio febril. CUANDO EL
TRATAMIENTO DE PRIMERA LÍNEA PARA LA ANEMIA HEMOLÍTICA AUTOINMUNE
(CORTICOESTEROIDES) NO TIENE LA RESPUESTA ESPERADA SE PUEDE REALIZAR
ESPLENECTOMÍA O APLICACIÓN DE INMUNOSUPRESORES.

DANAZOL. INMUNOSUPRESORES en ANEMIA HEMOLÍTICA AUTOINMUNE. Son la alternativa en los casos


refractarios a corticoterapia y esplenectomía. Los más utilizados son azatioprina (1,5 mg/kg al
día) y ciclofosfamida (2 mg/kg al día) por vía oral o en pautas de choque de 1.000-1.500
mg/m2 cada 3 semanas. El tratamiento debe mantenerse un mínimo de tres meses para
conseguir la máxima disminución de síntesis de anticuerpos, antes de ser considerado como
fracaso terapéutico. OJO. Medicamentos que en principio parecían prometedores, como el
danazol o los andrógenos atenuados sintéticos, han quedado en mera anécdota. CUANDO EL
TRATAMIENTO DE PRIMERA LÍNEA PARA LA ANEMIA HEMOLÍTICA AUTOINMUNE
(CORTICOESTEROIDES) NO TIENE LA RESPUESTA ESPERADA SE PUEDE REALIZAR
ESPLENECTOMÍA O APLICACIÓN DE INMUNOSUPRESORES.

TRANSFUSIÓN TRANSFUSIÓN en ANEMIA HEMOLÍTICA AUTOINMUNE. La indicación tiene que estar basada
DE en criterios clínicos. El nivel de hemoglobina (Hb) en el que aparecen síntomas graves varía en
CONCENTRADO función de la velocidad del descenso de Hb y el grado de compensación cardíaca. Es
ERITROCITARIO. fundamental la comunicación entre el clínico responsable y el banco de sangre, ya que el clínico
determinará la urgencia de la transfusión y aportará datos de antecedentes del paciente
(transfusiones o embarazos). Una vez iniciada la transfusión debe realizarse con una adecuada
monitorización. Es importante recordar que el paciente siempre va a recibir sangre
incompatible, pero por eso la transfusión no debe rechazarse ni demorarse. El mayor problema
es la presencia de aloanticuerpos frente a diferentes sistemas antigénicos que pueden estar
siendo enmascarados por los autoanticuerpos, por lo que es muy importante realizar técnicas
de absorción en pacientes con antecedentes de transfusión o embarazo. La presencia de
aloanticuerpos se da en un 32% de los casos. La mejoría de la sintomatología se produce con
transfusiones de escaso volumen (1 unidad o media). EL TRATAMIENTO DE LA ANEMIA
AUTOINMUNE CASI SIEMPRE INCLUYE LA TRANSFUSIÓN SANGUÍNEA SIN EMBARGO ESTO ES
UN PROBLEMA CUANDO EL ANTICUERPO QUE INTERVIENE ES INESPECÍFICO, PUES TODAS LAS
UNIDADES DE SANGRE QUE SE SOMETAN A PRUEBAS DE COMPATIBILIDAD CRUZADA SERÁN
INCOMPATIBLES. POR ESTA RAZÓN NO DEBERÁ ELEGIRSE COMO DE PRIMERA LÍNEA.
PREDNISONA. POR FRECUENCIA DEBEREMOS CONSIDERAR PRINCIPALMENTE LA PRESENCIA DE UNA
ANEMIA HEMOLÍTICA AUTOINMUNE POR AUTOANTICUERPOS CALIENTES. Anemia hemolítica
por autoanticuerpos calientes. Es la forma más frecuente (75% de los casos), con una incidencia
de 0.2-1 casos por 100,000 habitantes y año. Se presentan a cualquier edad (aunque son más
frecuentes en adultos) y predominan en el sexo femenino. Puede ser idiopática o secundaria. El
embarazo, aunque con mucha menor frecuencia, también puede favorecer la aparición de
AHAI5. Es importante recordar que la detección de autoanticuerpos no significa
necesariamente la aparición de AHAI. Tratamiento. ESTEROIDES: Debido a la variabilidad de la
presentación clínica desde indolente a urgencia vital, el tratamiento varía desde la transfusión
inmediata al tratamiento inmunomodulador. En las AHAI secundarias, es preciso tener en
cuenta el tratamiento de la enfermedad de base, aunque una vez iniciada la crisis hemolítica, su
evolución clínica sigue un curso propio e independiente al de la misma. Además del
tratamiento de soporte (oxigenoterapia, sueroterapia, transfusiones), el objetivo es controlar la
población de células B productoras de los anticuerpos. Corticoides. Son el tratamiento de
elección en la AHAI. Consigue remisiones hasta en el 80% de los casos idiopáticos y el 50% de
los secundarios. El efecto principal de los corticoides es interferir en la capacidad de los
macrófagos de fagocitar los hematíes recubiertos por IgG, C3b e iC3b. También interfieren en
la expresión y función de los receptores Fc de los macrófagos. Además disminuyen la síntesis
de anticuerpos, aunque este efecto aparece más tardíamente, tras semanas de tratamiento. Las
dosis iniciales de prednisona son de 1-2 mg/kg al día hasta que la Hb alcanza un valor superior
a 10 g/dl. Si no se aprecia este aumento de Hb, la dosis inicial de prednisona debe
administrarse durante un mínimo de tres semanas antes de considerarla ineficaz. Después, la
dosis de corticoides se reduce progresivamente (5-10 mg por semana) dependiendo de la
gravedad del cuadro al diagnóstico hasta alcanzar la mínima dosis necesaria para mantener el
estado de remisión, y si es posible retirarlos. Si la dosis de mantenimiento es superior a 15 mg
al día, deben considerarse otros tratamientos. EL TRATAMIENTO DE PRIMERA LÍNEA DE LA
ANEMIA HEMOLÍTICA AUTOINMUNE ES LA ADMINISTRACIÓN DE CORTICOESTEROIDES
(PREDNISONA).

Bibliografía:
1. LONGO DL, FAUCI AS, KASPER DL, HAUSERSL, JAMESON JL, LOSCALZOJ. HARRISON. PRINCIPIOS DE
MEDICINA INTERNA, 18A EDICIÓN. MC GRAW HILL. NEW YORK, USA. 2012, PP 882.
ANÁLISIS DEL CASO CLÍNICO

IDENTIFICACIÓN DEL REACTIVO


Area: MEDICINA INTERNA
Especialidad: CARDIOVASCULAR
Tema: VASCULOPATÍA HIPERTENSIVA
Subtema: HTA

CASO CLÍNICO SERIADO

HOMBRE DE 36 AÑOS DE EDAD, CON ANTECEDENTE DE DIABETES MELLITUS DE 1 AÑO DE EVOLUCIÓN,


PRESENTA ELEVACIÓN DE CIFRAS TENSIONALES EN REPETIDAS OCASIONES QUE NO CORRIGEN CON
MEDIDAS NO FARMACOLÓGICAS. ACUDE A CONSULTA DE CONTROL PARA INICIO DE MANEJO
FARMACOLÓGICO DE LA PRESIÓN ARTERIAL. A LA EXPLORACIÓN IMC 26.5, TA 165/105, NO EDEMA, SIN LESIÓN
A ÓRGANO BLANCO CLINICAMENTE PERCEPTIBLE EN ESTE MOMENTO.

Hombre de 36 años.

Diabetes Mellitus 2.

Sobrepeso, HIPERTENSIÓN TIPO 2.

-.

-.

212 - CORRESPONDE AL TIPO DE HIPERTENSIÓN QUE PRESENTA EL PACIENTE:

PRESIÓN De acuerdo a la última modificación de la NOM 030 y las GPC vigentes en nuestro país, la presión
ARTERIAL arterial se clasifica por cifras de acuerdo a los siguientes criterios: 1. PRESIÓN ARTERIAL
NORMAL FRONTERIZA O NORMAL ALTA: - Sistólica de 130 a 139 mmHg. - Diastólica de 85 a 89 mmHg.
ALTA.

HIPERTENSIÓN 2. PRESIÓN ARTERIAL GRADO I: - Sistólica de 140 a 159 mmHg. - Diastólica de 90 a 99 mmHg.
GRADO 1.

HIPERTENSIÓN 3. PRESIÓN ARTERIAL GRADO II: - Sistólica de 160 a 179 mmHg. - Diastólica de 100 a 109 mmHg.
GRADO 2.
HIPERTENSIÓN 4. PRESIÓN ARTERIAL GRADO III: - Sistólica mayor o igual a 180 mmHg. - Diastólica mayor o
GRADO 3. igual a 110 mmHg.

Bibliografía:
1. GUÍA DE PRÁCTICA CLÍNICA, DIAGNÓSTICO Y TRATAMIENTO DE LA HIPERTENSIÓN ARTERIAL EN EL
PRIMER NIVEL DE ATENCIÓN. MÉXICO: SECRETARIA DE SALUD, 08/07/14.

http://www.cenetec.salud.gob.mx/descargas/gpc/CatalogoMaestro/076-GCP__HipertArterial1NA/HIPERTENSION_EVR_CENETEC.pdf

213 - ES EL RIESGO CARDIOVASCULAR QUE TIENEN EL PACIENTE BASADO EN SU HIPERTENSIÓN Y LOS


FACTORES DE RIESGO ASOCIADOS:

BAJO. Para poder interpretar la gradación es necesario que tengas en cuenta la clasificación vigente de la
presión arterial (European Heart Journal, 2013): - OPTIMA: <120/<80. - NORMAL: 120-129/80-84. -
NORMAL ALTA: 130-139/ 85-89. - Hipertensión GRADO 1: 140-159/ 90-99. - Hipertensión GRADO 2:
160-179/ 100-109. - Hipertensión GRADO 3: >=180/ >=110. - Hipertensión SISTÓLICA AISLADA:
>=140/ <90. De acuerdo a la última modificación (2013) de la GRADACIÓN DE RIESGO
CARDIOVASCULAR DE ACUERDO A LAS CIFRAS DE PRESIÓN ARTERIAL Y LA PRESENCIA DE
FACTORES DE RIESGO Y DAÑO A ÓRGANO BLANCO (DOB) los pacientes se clasifican en: a. Riesgo
bajo. b. Riesgo moderado. c. Riesgo bajo moderado. d. Riesgo moderado alto. e. Riesgo alto. f.
Riesgo alto a muy alto. g. Riesgo muy alto. Los pacientes con RIESGO BAJO son aquellos: - Sin
factores de riesgo con HAS Grado I. - Con 1 o 2 factores de riesgo con presión normal alta. EL
PACIENTE TIENE UNA HIPERTENSIÓN GRADO 2 POR LO QUE NO ENCAJA EN ESTA CLASIFICACIÓN.

MODERADO. Los pacientes con RIESGO MODERADO son aquellos: - Sin factores de riesgo con HAS Grado 2. - Con
1 o 2 factores de riesgo con HAS Grado 1. Los pacientes con RIESGO BAJO A MODERADO son
aquellos: - Con >=3 factores de riesgo con presión normal alta. Los pacientes con RIESGO
MODERADO A ALTO son aquellos: - Con 1 o 2 factores de riesgo con HAS grado 2. - Con >=3
factores de riesgo con HAS grado 1. - Con daño a órgano blanco (DOB), enfermedad renal crónica
(ERC) estadio 3 o diabetes con presión normal alta. EL PACIENTE TIENE UNA HIPERTENSIÓN GRADO
2 POR LO QUE NO ENCAJA EN ESTA CLASIFICACIÓN.

ALTO. Los pacientes con RIESGO ALTO son aquellos: - Sin factores de riesgo con HAS grado 3. - Con 1 o 2
factores de riesgo con HAS grado 3. - Con >=3 factores de riesgo con HAS grado 2 o 3. - Con daño a
órgano blanco (DOB), enfermedad renal crónica (ERC) estadio 3 o diabetes con HAS grado 1 o 2.
DADO QUE EL PACIENTE TIENEN COMO ÚNICO ANTECEDENTE DIABETES, Y TIENEN UNA
HIPERTENSIÓN GRADO 2; SU RIESGO CARDIOVASCULAR DEBE SER CONSIDERADO COMO ALTO.
IMPORTANTE: el IMC del paciente es 26.5= sobrepeso, se considera factor de riesgo obesidad= IMC
>30.

MUY ALTO. Los pacientes con RIESGO ALTO A MUY ALTO son aquellos: - Con daño a órgano blanco (DOB),
enfermedad renal crónica (ERC) estadio 3 o diabetes con HAS grado 3. Los pacientes con RIESGO
MUY ALTO son aquellos: - Con enfermedad cardiovascular (EVC) sintomática, enfermedad renal
crónica (ERC) estadio >=4 o diabetes con daño a órgano blanco (DOB)/factores de riesgo (FR). "ES
MUY IMPORTANTE QUE REVISES LA MODIFICACIÓN A LA GRADACIÓN DE RIESGO EN LA
ACTUALIZACIÓN DE LA GPC 2014"

Bibliografía:
1. GUÍA DE PRÁCTICA CLÍNICA, DIAGNÓSTICO Y TRATAMIENTO DE LA HIPERTENSIÓN ARTERIAL EN EL
PRIMER NIVEL DE ATENCIÓN. MÉXICO: SECRETARIA DE SALUD, 08/07/14.

http://www.cenetec.salud.gob.mx/descargas/gpc/CatalogoMaestro/076-GCP__HipertArterial1NA/HIPERTENSION_EVR_CENETEC.pdf

214 - ES EL FÁRMACO DE ELECCIÓN PARA INICIAR EL TRATAMIENTO ANTIHIPERTENSIVO DEL PACIENTE:


DIURÉTICO EL TRATAMIENTO FARMACOLÓGICO ESTÁ RECOMENDADO EN TODO PACIENTE CON
TIAZIDA. HIPERTENSIÓN CON RIESGO CARDIOVACULAR ALTO POR DAÑO A ÓRGANO BLANCO (DOB),
DIABETE MELLITUS (DM), ENFERMEDAD CARDIOVASCULAR (EVC) O ENFERMEDAD RENAL
CRÓNICA (ERC). El DIURÉTICO TIAZIDA está indicado como tercera línea en el manejo de la
hipertensión en el paciente con diabetes.

CALCIO Se recomienda el uso de terapia combinada de IECA o ARA II + calcioantagonistas de efecto


ANTAGONISTA. prolongado en pacientes que lo requieras, especialmente cuando existe asociación de DM con
albuminuria o microalbuminuria. CORRESPONDE A LA SEGUNDA LÍNEA DE TRATAMIENTO
PARA EL MANEJO DE LA HIPERTENSIÓN EN PACIENTES CON DIABETES YA SEA DE FORMA
ÚNICA O COMBINADA CUANDO EXISTE ALBUMINURIA O MICROALBUMINURIA.

INHIBIDOR DE En los pacientes hipertensos con DM, iniciar el tratamiento cuando la PAS se encuentra >=
LA ENZIMA 140mmHg. En el paciente con DM se recomienda, como orden de preferencia de la elección de
CONVERTIDORA un fármaco (de primera línea), los siguientes: 1. IECA o ARA II. 2. Calcio antagonistas, y 3.
DE Tiazidas a dosis bajas. OJO: en pacientes con DM con evidencia de proteinuria o
ANGIOTENSINA. microproteinuria se prefiere el uso de ARA II. LOS INHIBIDOR DE LA ENZIMA CONVERTIDORA
DE ANGIOTENSINA (IECAs) SON LOS MEDICAMENTOS DE PRIMERA LÍNEA EN EL MANEO DE LA
HIPERTENSIÓN EN LOS PACIENTES CON DIABETES.

BETA En pacientes con HIPERTENSIÓN + CARDIOPATÍA ISQUÉMICA se recomienda mantener cifras


BLOQUEADOR. <130/80 mmHg. En pacientes con angina estable se recomienda el uso de beta-bloqueadores o
calcio antagonistas. En pacientes con infarto reciente se recomienda el uso de beta-
bloqueadores. LOS BETA BLOQUEADORES SON EL TRATAMIENTO DE PRIMERA ELECCIÓN EN
PACIENTES CON CARDIOPATÍA ISQUÉMICA.

Bibliografía:
1. GUÍA DE PRÁCTICA CLÍNICA, DIAGNÓSTICO Y TRATAMIENTO DE LA HIPERTENSIÓN ARTERIAL EN EL
PRIMER NIVEL DE ATENCIÓN. MÉXICO: SECRETARIA DE SALUD, 08/07/14.

http://www.cenetec.salud.gob.mx/descargas/gpc/CatalogoMaestro/076-GCP__HipertArterial1NA/HIPERTENSION_EVR_CENETEC.pdf

FIN DEL CASO CLÍNICO SERIADO


ANÁLISIS DEL CASO CLÍNICO

IDENTIFICACIÓN DEL REACTIVO


Area: CIRUGÍA
Especialidad: CIRUGÍA ABDOMINAL
Tema: PATOLOGÍA ESOFÁGICA
Subtema: ENFERMEDAD POR REFLUJO GASTROESOFÁGICO

CASO CLÍNICO CON UNA PREGUNTA

MUJER DE 60 AÑOS DE EDAD CON OBESIDAD MÓRBIDA. PRESENTA DESDE HACE 6 MESES CUADROS
INTERMITENTES DE PIROSIS, REGURGITACIÓN Y DISFAGIA OCASIONAL, QUE CEDEN CON INHIBIDORES DE LA
BOMBA DE PROTONES. SE LE INDICA UNA SERIE ESÓFAGOGASTRODUODENAL, PARA COMPLEMENTAR SU
ESTUDIO, OBTENIÉNDOSE LA SIGUIENTE IMAGEN:

Mujer de 60 años de edad.

Obesidad mórbida.

Cuadro intermitente de pirosis,


regurgitaciones y disfagia ocasional.

-.

Serie esófagogastroduodenal.

215 - EL DIAGNÓSTICO RADIOLÓGICO MÁS PROBABLE ES EL DE:


HERNIA HIATAL POR Las HERNIAS HIATALES se dividen en cuatro tipos principales: - TIPO I/DESLIZANTES: son
DESLIZAMIENTO. las más comunes de las hernias hiatales, representan hasta 95%, se caracterizan por la
debilidad y elongación de las estructuras frenoesofágicas que mantienen en localización
abdominal la unión gastroesofágica, con su consecuente migración hacia tórax. - TIPO
II/PARAESOFÁGICAS: éstas representan a las hernias paraesofágicas propiamente dichas,
se caracterizan por la presencia de un saco herniario que contiene el fundus gástrico y que
suele localizarse entre las 12 y 4 horas del hiato esofágico en su visión abdominal. Ocurren
debido a la debilidad de la membrana pleuroperitoneal, lo que permite que la pared
anterior del estómago se hernie hacia el espacio virtual que existe entre el esófago y el
ligamento frenoesofágico, asociado a la preservación de la unión gastroesofágica en
territorio abdominal, a diferencia de la tipo I. - TIPO III/MIXTAS: en la que se encuentran
características de la tipo I y II y son las más comunes entre aquellas que se catalogan como
paraesofágicas. - TIPO IV/COMPLEJAS: se definen como la migración intratorácica de
cualquier otro órgano intraabdominal. Se considera la expresión máxima de los defectos II
y III asociados a un aumento de la presión intraabdominal. Teniendo especial atención por
las HERNIAS POR DESLIZAMIENTO, la mayoría de los pacientes no presentan síntomas, por
eso es difícil determinar la incidencia real de esta patología. Se caracteriza por una
dislocación cefálica del cardias en el mediastino posterior. Su incidencia es siete veces
mayor que la hernia paraesofágica. Los síntomas se deben a las anomalías funcionales
relacionadas con el reflujo gastroesofágico como pirosis, regurgitación y disfagia. Estos
pacientes presentan una deficiencia mecánica del esfínter esofágico inferior que origina
reflujo del jugo gástrico hacia el esófago causando pirosis y regurgitación y la disfagia
puede ser secundaria a edema de la mucosa, anillo de Schatzki, estenosis o a la
imposibilidad para organizar la actividad peristáltica en el cuerpo del esófago como
consecuencia de la enfermedad. EL ESTUDIO CON MEDIO DE CONTRASTE DEMUESTRA
QUE LA CAUSA DE LA DISFAGIA EN ESTOS INDIVIDUOS ES LA OBSTRUCCIÓN AL PASO DEL
BOLO DEGLUTIDO POR EL IMPACTO DEL DIAFRAGMA SOBRE EL ESTÓMAGO HERNIARIO.
El estudio contrastado (SEGD) es el método de elección para el diagnóstico de hernias
hiatales y la imagen radiológica característica es cuando se encuentran más de 1-2 cm de
anillos mucosos por encima del diafragma o por medio de la identificación de 5 o más
pliegues mucosos que representan más de 1-2 cm por encima del diafragma y que sea
demostrado en un estudio de contraste simple. En estudios dinámicos, las ondas
peristáltica del esófago terminan por encima del hiato; delimitando la unión
esofagogástrica. EN ESTA IMAGEN RADIOLÓGICA DE LA SEGD (FASE ESOFÁGICA) CON
MEDIO DE CONTRASTE BARITADO SE OBSERVAN LOS PLIEGUES GÁSTRICOS POR ARRIBA
DEL HIATO O BIEN COMO UNA BOLSA SUPRADIAFRAGMÁTICA SIN PERISTALSIS NO
ALINEADA CON EL CUERPO ESOFÁGICO, CON PARTE DEL ESTÓMAGO EN TÓRAX, IMAGEN
CARACTERÍSTICA DE UNA HERNIA HIATAL POR DESLIZAMIENTO.

REFLUJO La Enfermedad por Reflujo Gastroesofágico (ERGE) es el trastorno gastrointestinal de


GRASTROESOFÁGICO. mayor incidencia (75% de las enfermedades esofágicas). Se ha considerado que la Pirosis
es el síntoma principal pero también presentan regurgitación y disfagia. Así mismo pueden
presentar síntomas atípicos como tos, sibilancias, dolor torácico, disfonía y erosiones
dentales.No obstante, si el diagnóstico se basa solo en los síntomas, únicamente será
acertado en el 70% de los pacientes. En la actualidad, el esofagograma no se considera una
prueba diagnóstica de ERGE, debido a que tiene una sensibilidad (26%) y especificidad
(50%) muy bajas comparadas con la endoscopia, por lo que actualmente la confirmación
del diagnóstico es con Endoscopia. Incluso en el esofagograma puede presentar reflujo de
bario sin que exista reflujo anormal. La serie esófagogastroduodenal (SEGD) se solicita
como parte del estudio preoperatorio de la ERGE para detallar la anatomía y la función de
la zona. Los puntos que se valoran en una SEGD son: 1.La presencia de esófago corto 2.La
presencia de hernia hiatal 3.La función propulsiva y de vaciamiento del esófago y
estómago EN ESTA IMAGEN RADIOLÓGICA SE OBSERVA MEDIO DE CONTRASTE EN
ESÓFAGO Y ESTÓMAGO, PERO SOLO ES UNA IMAGEN RADIOLÓGICA, PARA DESCARTAR
ERGE ES NECESARIO LA SECUENCIA COMPLETA DE LA SEGD, ADEMÁS SON EVIDENTES
LOS PLIEGUES MUCOSOS POR ARRIBA DEL HIATO Y EL ESTOMAGO EN TÓRAX. POR LO
QUE ESTA IMAGEN SUSTENTA LA PRESENCIA DE UNA HERNIA HIATAL.
ACALASIA. La ACALASIA es una discinesia esofágica primaria caracterizada por la falta de peristalsis,
la mayoría de los pacientes sufren de hipertensión del esfínter esofágico inferior, el cual no
se relaja de manera apropiada en respuesta a la deglución. Se desconoce aún la etiología
de la ACALASIA , pero existen dos hipótesis: 1.- Se trata de una enfermedad degenerativa
de las neuronas y 2.- Surge por infecciones virales en neuronas u otros agentes infecciosos.
Dentro de las manifestaciones clínicas se encuentra: a) La disfagia que es el síntoma más
común, tanto para alimentos sólidos como a líquidos. b) La regurgitación es el segundo
síntoma más frecuente, ocurre en posición supina y puede ocasionar broncoaspiración. c)
Sólo un 40% de los pacientes padecen pirosis, pero no a causa del reflujo esofágico, sino
por éxtasis y fermentación de la comida no digerida en la porción distal del esófago. d) El
dolor torácico que pueden referir algunos pacientes debido a la distensión esofágica al
ingerir los alimentos. El estudio inicial es una serie esófagogastroduodenal (SEGD) o
esofagograma baritado, en donde se encuentra un estrechamiento progresivo del
diámetro esofágico y un esfínter esofágico inferior cerrado que da la imagen radiológica
típica de “pico de ave” o “de copa de champaña”. En un Esofagograma dinámico hay
pérdida de la peristalsis principalmente hacia los 2/3 distales del esófago, mal vaciamiento
de la comida y la saliva lo que produce un nivel de aire-líquido en lo alto de la columna de
bario En las etapas crónicas de la enfermedad, hay importante dilatación, tortuosidad y
redundancia del esófago que semejan al colon sigmoides y la imagen de “pico de pájaro”
en la unión gastroesofágica. La fluoroscopia es particularmente necesaria para el
diagnóstico de la enfermedad en sus fases iniciales, cuando el esófago no está todavía
dilatado, ya que las imágenes estáticas de las radiografías pueden ser normales y a
intervalos regulares después del tratamiento ha sido propuesta para monitorear el éxito
del terapéutico. LA IMAGEN CLÁSICA EN ESTUDIOS CONTRASTADOS DE PACIENTES CON
ACALASIA ES EN PICO DE PÁJARO. LO CUAL NO CONCUERDA CON LA IMAGEN
MOSTRADA.

ANILLO DE SCHATZKI. El ANILLO DE SCHATZKI es un reborde circunferencial delgado que se ubica en la mucosa
del tercio inferior del esófago, en la unión escamocolumnar. De manera frecuente se asocia
a una hernia hiatal y puede o no presentar reflujo gastroesofágico. La mayoría de los
pacientes son asintomáticos, pero algunos pueden presentar disfagia sobre todo durante
la ingestión apresurada de alimentos sólidos , esto secundario a que el diámetro del anillo
sea menor de 12mm. El diagnóstico se realiza mediante un estudio contrastado (SEGD) y
endoscopia. Los datos encontrados en la serie esofagogastroduodenal, en su fase esofágica
es un "defecto de llenado circunferencial en el esófago distal". Se diferencia de las
estenosis inflamatorias (pépticas) debido a que éstas abarcan todas las capas del intestino
y el anillo de Schatzki se encuentra confinado únicamente a la mucosa. Su tratamiento
consiste en dilatar el anillo bajo endoscopia e iniciar tratamiento para reflujo
gastroesofágico en caso de que lo presente. SE DESCARTA ESTA PATOLOGÍA DEBIDO A
QUE NO PRESENTA CUADRO CLÍNICO NI IMAGEN RADIOLÓGICA CARACTERÍSTICA DE
ESTA ENTIDAD EN EL ESTUDIO CONTRASTADO.

Bibliografía:
1.-DOHERTY G. DIAGNÓSTICO Y TRATAMIENTOS QUIRÚRGICOS.13ª. EDICION. LANGE MC GRAW HILL. 2011.
2.- BRUNICARDI F. SCHWARTZ PRINCIPIOS DE CIRUGIA. 9ª EDICIÓN.MCGRAW HILL.2011. 3. GUÍA DE
PRÁCTICA CLÍNICA, DIAGNÓSTIC Y TRATAMIENTO QUIRÚRGICO DE LA HERNIA PARAESOFÁGICA. MÉXICO:
SECRETARIA DE SALUD; DICIEMBRE 2011. RECUPERADO DE
HTTP://WWW.CENETEC.SALUD.GOB.MX/CONTENIDOS/GPC/CATALOGOMAESTROGPC.HTML

http://www.cenetec.salud.gob.mx/descargas/gpc/CatalogoMaestro/452_GPC_Herniaparaesofxgica/HParaesofagica.ER.pdf
ANÁLISIS DEL CASO CLÍNICO

IDENTIFICACIÓN DEL REACTIVO


Area: GINECOLOGÍA Y OBSTETRICIA
Especialidad: GINECOLOGÍA
Tema: PATOLOGÍA BENIGNA Y MALIGNA DE OVARIO
Subtema: PATOLOGÍA BENIGNA DE OVARIO

CASO CLÍNICO CON UNA PREGUNTA

MUJER DE 21 AÑOS DE EDAD, QUE ACUDE A LA CONSULTA CON DATOS COMPATIBLES DE SÍNDROME DE
OVARIO POLIQUÍSTICO.

El SÍNDROME DE OVARIO POLIQUÍSTICO


es la endocrinopatía más frecuente en la
mujer en edad fértil, en la que alcanza de 4
a 12%.

es igual de frecuente en todas las razas y


nacionalidades.

como podrás estudiar, los síntomas son en


relación al hiperandrogenismo y a la
resistencia a la insulina.

-.

Escrutinio del síndrome de ovarios


poliquísticos (SOP): Determinación en
ayunas y durante los primeros 7 días del
ciclo de: a) LH y FSH en pool (el promedio
de 3 muestras recolectadas con 20 minutos
de diferencia) y Prolactina. b) Testosterona
total y libre. c) Dehidroepiandrosterona
sulfato (DHEAS) y 17
alfahidroxiprogesterona (17-HPO). En el 66%
de los pacientes se documenta una
relación LH:FSH (>2.5:1), la cual es
característica de SOP.

216 - EN ESTA PATOLOGÍA SERÁ IMPORTANTE CONSIDERAR QUE LA HORMONA LUTEINIZANTE:


SE MANTIENE SÍNDROME DE OVARIO POLIQUÍSITO (SOP). Las dos principales anormalidades hormonales
ELEVADA halladas en los pacientes con SOP incluyen la elevación de los niveles circulantes de hormona
DURANTE EL CICLO luteinizante (LH) e insulina. La hipótesis más aceptable sugiere un sinergismo entre ambas, lo
MENSTRUAL. cual termina por provocar en el ovario, atresia folicular e incapacidad para la formación de
estrógenos. En este contexto, la hiperestimulación ovárica por la insulina, provocaría
hiperandrogenismo. La fisiopatología del SOP es compleja y en muchos sentidos no del todo
conocida. La resistencia a la insulina tiene un papel central en más del 50% de los casos,
generalmente asociada a obesidad. En el resto de los casos las teorías involucran
directamente al hipotálamo, a través de cambios en la frecuencia y magnitud de los picos de
la hormona liberadora de gonadotropinas, lo cual aumentaría los niveles circulantes de LH, o
al ovario y las suprarrenales bajo mecanismos más oscuros. Escrutinio del síndrome de
ovarios poliquísticos (SOP): Determinación en ayunas y durante los primeros 7 días del ciclo
de: a) LH y FSH en pool (el promedio de 3 muestras recolectadas con 20 minutos de
diferencia) y Prolactina. b) Testosterona total y libre. c) Dehidroepiandrosterona sulfato
(DHEAS) y 17 alfa-hidroxiprogesterona (17-HPO). En el 66% de los pacientes se documenta
una relación LH: FSH (mayor a 2.5:1), la cual es característica de SOP. EL SÍNDROME DE
OVARIO POLIQUÍSTICO SE CARACTERIZA POR ELEVACIÓN SOSTENIDA DE LA HORMONA
LUTEINIZANTE E INSULINA.

FLUCTÚA El síndrome de ovarios poliquísticos es una entidad frecuente en la mujer joven. La


MOSTRANDO UN importancia de su diagnóstico radica en la exclusión de patologías potencialmente graves,
PICO MÁXIMO como las neoplasias ováricas y adrenales o la hiperplasia adrenal congénita y en el reto que
ALREDEDOR DEL representa para los clínicos, en cuanto a prevención se refiere, por su nexo fisiopatológico
DÍA 15. con los estados de resistencia a la insulina e hiperinsulinemia compensatoria. Hoy sabemos
que estas mujeres tienen riesgos significativamente elevados de diabetes tipo 2, hipertensión
arterial y cardiopatía isquémica, lo que repercute en un aumento en su mortalidad.

SUS NIVELES SE CICLO MENSTRUAL. El ciclo menstrual puede dividirse en cuatro fases, refiriéndose cada una
ENCUENTRAN POR al estado de los folículos y de los óvulos. Sería más adecuado considerar como primera fase
DEBAJO DE LOS DE el período de menstruación, porque puede identificarse con facilidad, pero, desde el punto
LA HORMONA de vista fisiológico, representa la última. La primera fase, denominada fase folicular, se
FOLÍCULO extiende, más o menos, desde el cuarto hasta el decimocuarto día después de comenzar la
ESTIMULANTE. menstruación. (Al contar los días del ciclo, se considera que el primer día es también el
primero de la menstruación). Durante esta fase, un folículo madura y aumenta de tamaño. El
final de esta fase corresponde a la rotura del folículo y la liberación del óvulo (fase
ovulatoria). Durante la siguiente fase (fase lútea), se forma un grupo de células de color
amarillo rojizo, denominado cuerpo lúteo, en el folículo abierto. La fase final, marcada por la
menstruación, supone la expulsión del revestimiento interior del útero (endometrio),
constituido para acoger y nutrir un huevo fertilizado. La regulación del ciclo menstrual lleva
consigo la interacción entre los niveles de las siguientes hormonas. La fase folicular del ciclo
se inicia cuando la glándula pituitaria secreta la hormona folículo-estimulante (FSH), que
induce el aumento de la producción de estrógenos en los ovarios y estimula la maduración
de varios folículos. El elevado nivel de estrógenos resultante, mediante el bucle de
retroalimentación negativa, induce la disminución de la producción de FSH en la pituitaria y
dispara la producción de la hormona luteinizante (HL), cuya función principal consiste en
provocar la ovulación. Durante cierto tiempo, la FSH y la HL inducen una producción aún
mayor de estrógenos, que ayudan a reducir la cantidad de FSH. En este punto, empieza a
predominar la HL, provocando la rotura del folículo y la liberación del óvulo. A continuación,
se forma el cuerpo lúteo en el folículo abierto. El cuerpo lúteo produce gran cantidad de
progesterona. Cuando los niveles de progesterona son suficientemente elevados, inhiben,
mediante el bucle de retroalimentación negativa, la producción de HL y, a la vez, estimulan la
producción de FSH, comenzando de nuevo el ciclo.

MUESTRA Manifestaciones clínicas principales del Síndrome de Ovario Poliquístico (SOP). • Acné. •
CONSTANTEMENTE Hirsutismo. • Virilización. • Irregularidad menstrual y anovulación. • Acantosis Nigricans. Al
NIVELES BAJOS. momento de enfrentar un caso de hiperandrogenismo nuestra obligación más importante es
descartar la presencia de algunas patologías, que si bien no son frecuentes, requieren de
abordajes particulares y tratamientos específicos. En este grupo incluimos a los tumores
benignos y malignos de las adrenales y el ovario, así como los casos de hiperplasia
suprarrenal congénita en su variedad no clásica, entre otras. El SOP es un diagnóstico de
exclusión y no es necesaria la presencia de alteraciones en los niveles hormonales o quistes
en los ovarios para considerar su presencia; basta integrar el síndrome durante la historia
clínica y hacer con base en un protocolo de escrutinio un buen diagnóstico diferencial.

Bibliografía:
1. GUÍA DE REFERENCIA RÁPIDA, DIAGNÓSTICO Y TRATAMIENTO DEL SÍNDROME DE OVARIOS
POLIQUÍSTICOS. MÉXICO: SECRETARIA DE SALUD; 2010. 2. GUÍA DE REFERENCIA RÁPIDA, DIAGNÓSTICO Y
TRATAMIENTO DEL SÍNDROME DE OVARIOS POLIQUÍSTICOS. MÉXICO: SECRETARIA DE SALUD; 2010.

http://www.cenetec.salud.gob.mx/descargas/gpc/CatalogoMaestro/453_GPC_Ovario_poliquistico/GER_Sx_ovario_poliquxstico.pdf
ANÁLISIS DEL CASO CLÍNICO

IDENTIFICACIÓN DEL REACTIVO


Area: GINECOLOGÍA Y OBSTETRICIA
Especialidad: GINECOLOGÍA
Tema: LEUCORREA
Subtema: VAGINOSIS BACTERIANA

CASO CLÍNICO CON UNA PREGUNTA

MUJER DE 34 AÑOS DE EDAD, SEXOSERVIDORA, ACUDE A CONSULTA POR SOSPECHA DE ENFERMEDAD DE


TRANSMISIÓN SEXUAL. USTED CONFIRMA EL DIAGNÓSTICO DE CERVICITIS POR CHLAMYDIA TRACHOMATIS.

Mujer de 34 años de edad.

Sospecha de INfECCIÓN DE TRANSMISIÓN


SEXUAL Sexoservidora.

-.

-.

Cervicitis por chlamydia trachomatis.

217 - EN ESTE CASO ESTÁ INDICADO EL USO DE:

NITROIMIDAZOLES. El METRONIDAZOL, ejerce su acción antibacteriana y antiprotozoaria por


desestructuración del ADN. Tras ingresar en la célula mediante difusión pasiva es
químicamente reducido por proteínas del metabolismo anaerobio (Proteínas de
transporte de electrones de bajo potencial redox). Estas proteínas son exclusivas de
algunos parásitos y de bacterias anaerobias y algunas microaerófilas. El Metronidazol
reducido produce pérdida de la estructura helicoidal del ADN, rotura de la cadena e
inhibición de la síntesis de ácidos nucleicos y muerte celular, generando compuestos que
son tóxicos para la célula. El Metronidazol se distribuye bien en la mayoría de los líquidos
y tejidos corporales debido al bajo peso molecular y a la escasa unión del fármaco a
proteínas plasmáticas. Su concentración terapéutica se alcanza en secreciones vaginales,
líquido seminal, saliva, leche, líquidos de derrame, miometrio, trompas de Falopio, entre
otros. Su dosis para adultos por vía oral es de 2 g por 2 días o 500 mg cada 12 horas,
durante 7 días y su dosis máxima es de 4 g por día; por vía intravaginal en gel es de 5 g
durante 7 días. No se recomienda su uso en el primer trimestre de embarazo (en otras
fases del embarazo se ha consumido, al parecer sin efectos adversos manifestados. EL
METRONIDAZOL ESTÁ INDICADO EN EL TRATAMIENTO DE LA VAGINOSIS BACTERIANA Y
LA TRICOMONIASIS.
FLUOROQUINOLONAS. Las Quinolonas y Fluoroquinolonas actúan sobre las enzimas bacterianas de tipo girasa y
topoisomerasa del ADN. - OFLOXACINO 200mg 2 veces al día o 400mg una vez al día por
7 días. SE PUEDEN UTILIZAR EN ESTA PACIENTE PERO NO SON EL TRATAMIENTO DE
PRIMERA ELECCIÓN. OJO: RECUERDA SIEMPRE ELEGIR EL TRATAMIENTO DE PRIMERA
ELECCIÓN A MENOS QUE EN EL CASO CLÍNICO SE INDIQUEN FACTORES QUE LO
CONTRAINDIQUEN COMO POR EJEMPLO EMBARAZO O ALERGIAS.

MACRÓLIDOS. LA AZITROMICINA Y DOXICICLINA SON EL TRATAMIENTO DE PRIMERA ELECCIÓN PARA


LA VAGINOSIS POR CHLAMYDIA. Los macrólidos inhiben la síntesis proteica mediante la
unión a la subunidad ribosomal 50S, inhibiendo la translocación del aminoacil ARNt.
Tiene también efectos sobre el nivel de la peptidil transferasa. Sus acciones pueden
provocar un efecto bacteriostático o bactericida, según la especie bacteriana atacada, la
concentración del antibiótico alcanzada en el sitio de infección o la fase de crecimiento en
que se encuentran las bacterias durante el ataque del antibiótico. La Doxiciclina es un
fármaco bacteriostático de espectro amplio, que actúa por inhibición de la síntesis de
proteínas y bloquea la unión del RNA de transferencia al complejo ribosómico del RNA
mensajero. La unión reversible se produce en la sub-unidad ribosómica 30S de los
organismos sensibles. No se inhibe la síntesis de la pared celular bacteriana.

SULFAMIDAS. SULFAMIDAS. Están estructuralmente relacionadas con PABA, y compiten con él por la
enzima dihidropteroato sintetasa que interviene en el metabolismo del ácido fólico. El
ácido fólico es imprescindible para la síntesis de precursores de los ácidos nucleicos
bacterianos. Las células de los mamíferos requieren ácido fólico preformado, ya que no
pueden sintetizarlo y, por tanto, no se ven afectadas por la acción de las sulfamidas. La
actividad antibacteriana es inhibida en presencia de pus o restos de tejido necrótico
(Reducen la necesidad de la bacteria de sintetizar ácido fólico). Las diaminopirimidinas
(Como el trimetoprima), al igual que las sulfamidas, interfieren en el metabolismo del
ácido fólico, por lo que combinadas tienen efecto sinérgico. NO TIENEN NINGUNA
UTILIDAD EN EL TRATAMIENTO DE LA PACIENTE.

Bibliografía:
1. GUÍA DE PRÁCTICA CLÍNICA, PREVENCIÓN Y DIAGNÓSTICO OPORTUNO DE LA INFECCIÓN DEL TRACTO
GENITOURINARIO INFERIOR POR CHLAMYDIA TRACHOMATIS. MÉXICO: SECRETARIA DE SALUD; MARZO
2013. 2. GOLAN D, TASHJIAN A. AMSTRONG E, AMSTRONG A. PRINCIPIOS DE FARMACOLOGÍA, 3A EDICIÓN.
LIPPINCOTT CILLIAMS & WILKINS. 2012, PP 592.

http://www.cenetec.salud.gob.mx/descargas/gpc/CatalogoMaestro/006_GPC_Inf.porChlamydia/SSA_006_08_EyR.pdf
ANÁLISIS DEL CASO CLÍNICO

IDENTIFICACIÓN DEL REACTIVO


Area: MEDICINA INTERNA
Especialidad: REUMATOLOGÍA
Tema: TRANSTORNOS ARTICULARES
Subtema: GOTA

CASO CLÍNICO CON UNA PREGUNTA

MASCULINO DE 43 AÑOS, CON DIAGNÓSTICO DE GOTA DESDE HACE 5 AÑOS.

Masculino de 43 años de edad.

tiene diagnostico de gota desde hace 5


años.

-.

-.

-.

218 - ACTUALMENTE SE ENCUENTRA EN CONTROL Y MANEJO CON DOSIS ALTA DE ALOPURINOL QUE
PREVIENE LA SÍNTESIS DE URATO, POR INHIBICIÓN NO COMPETITIVA DE LA:

TIROXIN Las indicaciones para el uso de alopurinol son hiperuricemia asociada con aumento en la
HIDROXILASA. producción de ácido úrico, excreción urinaria de 1000 mg o más de ácido úrico en orina de 24
horas, nefropatía por ácido úrico, nefrolitiasis, profilaxis antes del tratamiento con citotóxicos,
ineficacia de los agentes uricosúricos, gota con insuficiencia renal con un índice de filtración
de glomerular < 60 ml/min y alergia a los agentes uricosúricos.

DOPA Los fármacos antihiperuricémicos proporcionan un método de control definitivo de la


DESCARBOXILASA hiperuricemia. Es importante tratar y prevenir los ataques agudos de gota con agentes
L AMINOACIDO. antiinflamatorios, sin embargo es el control de la hiperuricemia a largo plazo lo que
finalmente modifica las manifestaciones de la gota. Una vez iniciado el tratamiento con
fármacos para reducir los niveles de urato ya no se debe suspender el tratamiento y se deben
dar a dosis suficientes para mantener los niveles séricos menores de 6.8 mg/dl y de preferencia
entre 5 y 6 mg/dl.
XANTINO El alopurinol es un medicamento que inhibe a la xantina oxidasa. Esta enzima cataliza la
OXIDASA. oxidación de hipoxantina a xantina y de xantina a ácido úrico. EL ALOPURINOL, INHIBIDOR DE
OXIDASA DE XANTINA, ES EL HIPOURICEMIANTE MÁS UTILIZADO y el mejor para disminuir la
concentración de uratos séricos en personas que producen de manera excesiva, en quienes
forman cálculos de urato y en individuos con alguna nefropatía.

ARABINOSIL Para evitar el desarrollo de un ataque agudo de gota por el alopurinol se pueden dar dosis
TRANSFERASA. profilácticas de colchicina o AINES 2 semanas antes del inicio del alopurinol. Otra alternativa
es iniciar las dosis de alopurinol a 50 a 100 mg al día e irlo incrementando en esa misma dosis
cada semana hasta que se alcance la dosis adecuada para el control del ácido úrico.
Aproximadamente el 20% de los pacientes que toman alopurinol reportan efectos secundarios
y de estos 5% tienen que suspender el medicamento. Los efectos secundarios más comunes
son intolerancia gastrointestinal y rash cutáneo. Otros efectos secundarios incluyen fiebre,
necrolisis epidérmica tóxica, alopecia, supresión de la médula ósea con leucopenia o
trombocitopenia, agranulocitosis, anemia aplásica, hepatitis granulomatosa, ictericia y
vasculitis. La reacción más severa es hipersensibilidad al alopurinol que puede incluir fiebre,
rash cutáneo, eosinofilia, hepatitis, insuficiencia renal progresiva y muerte.

Bibliografía:
1. LONGO DL, FAUCI AS, KASPER DL, HAUSERSL, JAMESON JL, LOSCALZOJ. HARRISON. PRINCIPIOS DE
MEDICINA INTERNA, 18A EDICIÓN. MC GRAW HILL. NEW YORK, USA. 2012, PP 2838.
ANÁLISIS DEL CASO CLÍNICO

IDENTIFICACIÓN DEL REACTIVO


Area: PEDIATRÍA
Especialidad: INFECTOLOGIA PEDIÁTRICA
Tema: GASTROENTERITIS INFECCIOSA
Subtema: GEPI BACTERIANA Y PARASITARIAS

CASO CLÍNICO CON UNA PREGUNTA

PACIENTE FEMENINA DE 6 AÑOS DE EDAD, LA CUAL PRESENTA PALIDEZ DE TEGUMENTOS. DOLOR


ABDOMINAL TIPO CÓLICO, EVACUACIONES DIARREICAS ALTERNANDO CON CONSTIPACIÓN. SE REALIZA
ESTUDIO DE LA MATERIA FECAL DONDE SE OBSERVAN ABUNDANTES HUEVOS DE ÁSCARIS LUMBRICOIDES.

FEMENINA DE 6 AÑOS DE EDAD

--

PALIDEZ DE TEGUMENTOS, DOLOR


ABDOMINAL TIPO CóLICO,
EVACUACIONES DIARRÉICAS
ALTERNANDO CON CONSTIPACIÓN

--

ESTUDIO DE LA MATERIA FECAL DONDE


SE OBSERVAN ABUNDANTES HUEVOS DE
áSCARIS LUMBRICOIDES

219 - EL TRATAMIENTO QUE DEBE INICIAR LA PACIENTE ES CON:

PIRANTEL ASCARIS LUMBRICOIDES. • Es el nematodo más grande que parasita el tubo intestinal. • Infesta
principalmente el intestino delgado. • Se adquiere al ingerir huevecillos viables procedentes de
suelos o alimentos contaminados con heces. • Los medicamentos más adecuados para ésta
parasitosis son: piperazina, tetramisol, pirantel y mebendazol. PIRANTEL. Activación persistente
del Receptor nicotínico excitatorio de Acetilcolina del músculo de nematodos, despolarización y
PARÁLISIS ESPÁSTICA por incremento de la conductancia al sodio y al potasio.
FARMACOCINÉTICA. Poca absorción oral, eliminación por heces excreción urinaria (inalterado +
metabolitos). USOS: Enterobiasis, ascaridiasis, Uncinariasis. EFECTOS ADVERSOS: •
Gastrointestinales, cefaleas, fiebre, mareos. • No hay pruebas de que sea inofensivo para el
embarazo o el lactante CONTRAINDICACIONES: • Menores de dos años, Embarazo.
ALBENDAZOL ASCARIS LUMBRICOIDES. El tratamiento recomendado en caso de parasitosis asintomáticas y
sintomáticas incluye una sola dosis de fármacos como el albendazol, pamoato de pirantel o
mebendazol, durante tres días. Pocos datos sugieren que los fármacos mencionados son inocuos
en niños menores de dos años de vida, pero hay que comparar sus riesgos y beneficios antes de
administrarlos. Es una medida útil para evaluar el tratamiento, aunque no esencial, practicar de
nuevo un estudio de muestras de excrementos tres semanas después del tratamiento para saber
si se eliminaron los vermes. El caso de obstrucción intestinal parcial o completa atribuible a un
enorme número de vermes, se puede administrar por una sonda gastrointestinal una solución de
citrato de piperacina (75 mg/kg por día, que no exceda de 3.5 g), aunque no se cuenta con él en
muchos países, incluido Estados Unidos. No se recomienda el pamoato de pirantel porque
puede empeorar la obstrucción. Si no se consigue la piperacina, la obstrucción puede mostrar
resolución con medidas conservadoras (aspiración nasogástrica, soluciones intravenosas), y en
ese momento podrán administrarse albendazol o mebendazol. A veces se necesita alguna
intervención quirúrgica para corregir la obstrucción de intestinos o vías biliares o para tratar el
vólvulo o la peritonitis que es consecuencia de la perforación. ACTUALMENTE EN MÉXICO Y EN
EL RED BOOK SE CONSIDERA AL ALBENDAZOL COMO LA PRIMERA OPCIÓN TERAPEÚTICA EN
PACIENTES DE ESTA EDAD.

METRONIDAZOL METRONIDAZOL El metronidazol es un nitroimidazol con propiedades antibacterianas y


antiprotozoarias, que se utiliza para tratar las infecciones producidas por Trichomonas vaginalis,
así como las amebiasis y giardiasis. Es uno de los fármacos más eficaces frente a las bacterias
anaerobias y, en combinación con otros antibióticos, se utiliza para la erradicación del
Helicobacter pylori. También es utilizado para el tratamiento de la enfermedad de Crohn, la
diarrea asociada a los tratamientos antibióticos y la rosácea. EL METRONIDAZOL NO TIENE
EFECTOS TERAPÉUTICOS SOBRE EL ÁSCARIS LUMBRICOIDES.

IVERMECTINA IVERMECTINA MECANISMO DE ACCIÓN • Ocasiona parálisis por activación de un canal de cloro
(abierto por glutamato) generando hiperpolarización y parálisis tónica. También activa canales
de cloro sensibles al GABA en nematodos. FARMACOCINÉTICA • Administración oral,
subcutánea, tópica. Alta distribución en la grasa y el hígado. Eliminación por heces. USOS:
Oncocercosis, filariasis, Estrongiloidiasis, escabiosis, pediculosis. NO TIENE USO EN
PARASITOSIS POR ASCARIS LUMBRICOIDES.

Bibliografía:
THE PHARMACOLOGICAL BASIS OF THERAPEUTICS. BRUNTON LL, LAZO JS, PARKER KL. MCGRAW HILL.
11TH. 2005. PAG. 1074.
Salir

MUJER DE 41 AÑOS DE EDAD, CON ÚNICO ANTECEDENTE DE IMPORTANCIA


MATERNO DE GLAUCOMA DE ÁNGULO ABIERTO. ACUDE AL OFTALMÓLOGO HACE
UN MES REPORTÁNDOSE LA VALORACIÓN SIN ALTERACIONES.

220 - A PARTIR DE ESTE MOMENTO, USTED LE RECOMENDARÁ AL PACIENTE


REALIZARSE REVALORACIONES OFTALMOLÓGICAS CADA:

6 MESES.
12 MESES.
18 MESES.
24 MESES.

Bibliografía:
1. DIAGNÓSTICO Y TRATAMIENTO DEL PACIENTE ADULTO CON GLAUCOMA DE ÁNGULO ABIERTO. EVIDENCIAS
Y RECOMENDACIONES: GUÍA DE PRÁCTICA CLÍNICA. MÉXICO: SECRETARÍA DE SALUD, 17 DE MARZO DE 2016.

http://www.cenetec-difusion.com/CMGPC/IMSS-164-09/ER.pdf

SEGURA DUDA NI IDEA


Salir

HOMBRE DE 65 AÑOS DE EDAD, CON ANTECEDENTE DE ESPLENECTOMÍA POR


ACCIDENTE AUTOMOVILÍSTICO HACE SEIS AÑOS. INGRESA AL SERVICIO DE
URGENCIAS CON UN CUADRO CLÍNICO COMPATIBLE CON NEUMONÍA ADQUIRIDA
EN LA COMUNIDAD.

221 - LA ETIOLOGÍA BACTERIANA MÁS PROBABLE EN ESTE CASO ES POR:

PSEUDOMONA AERUGINOSA
LEGIONELLA PNEUMOPHILA
MYCOPLASMA PNEUMONIAE
STREPTOCOCCUS PNEUMONIAE

Bibliografía:
1. PREVENCIÓN, DIAGNÓSTICO Y TRATAMIENTO DE LA NEUMONÍA ADQUIRIDA EN LA COMUNIDAD EN
ADULTOS. EVIDENCIAS Y RECOMENDACIONES: GUÍA DE PRÁCTICA CLÍNICA. MÉXICO, CENETEC; 2017. 2. GUÍA
DE PRÁCTICA CLÍNICA, PREVENCIÓN, DIAGNÓSTICO Y TRATAMIENTO DE LA NEUMONÍA ADQUIRIDA EN LA
COMUNIDAD EN ADULTOS. MÉXICO: SECRETARÍA DE SALUD, DICIEMBRE 2009. 3. LONGO DL, FAUCI AS,
KASPER DL, HAUSERSL, JAMESON JL, LOSCALZOJ. HARRISON. PRINCIPIOS DE MEDICINA INTERNA, 18A
EDICIÓN. MC GRAW HILL. NEW YORK, USA. 2012, PP 2131-2135.

http://www.cenetec-difusion.com/CMGPC/IMSS-234-09/ER.pdf

SEGURA DUDA NI IDEA

222 - EL TRATAMIENTO DE ELECCIÓN EN ESTE CASO ES CON:

BETALACTÁMICO + AMINOGLUCÓSIDO
MONOTERAPIA CON AMINOPENICILINA
BETALACTÁMICO + QUINOLONA
MONOTERAPIA CON QUINOLONA

Bibliografía:
1. PREVENCIÓN, DIAGNÓSTICO Y TRATAMIENTO DE LA NEUMONÍA ADQUIRIDA EN LA COMUNIDAD EN
ADULTOS. EVIDENCIAS Y RECOMENDACIONES: GUÍA DE PRÁCTICA CLÍNICA. MÉXICO, CENETEC; 2017. 2.
LONGO DL, FAUCI AS, KASPER DL, HAUSERSL, JAMESON JL, LOSCALZOJ. HARRISON. PRINCIPIOS DE
MEDICINA INTERNA, 18A EDICIÓN. MC GRAW HILL. NEW YORK, USA. 2012, PP 2135.

http://www.cenetec-difusion.com/CMGPC/IMSS-234-09/ER.pdf

SEGURA DUDA NI IDEA


ANÁLISIS DEL CASO CLÍNICO

IDENTIFICACIÓN DEL REACTIVO


Area: MEDICINA INTERNA
Especialidad: CARDIOVASCULAR
Tema: ALTERACIONES DEL RITMO
Subtema: BRADIARRITMIAS Y ALTERACIONES DE LA CONDUCCIÓN
AURICULOVENTRICULAR

CASO CLÍNICO CON UNA PREGUNTA

MUJER DE 56 AÑOS, HIPERTENSA QUE INGRESA CON DATOS DE DESHIDRATACIÓN POSTERIOR A LA INGESTA
DE SOBREDOSIS ACCIDENTAL DE FUROSEMIDE. SE REALIZA ELECTROCARDIOGRAMA QUE MUESTRA
EFECTOS DE LA HIPOKALEMIA SECUNDARIA.

mujer de 56 años.

hipertensa, ingesta de sobredosis


accidental de furosemide.

-.

datos de deshidratación.

SE REALIZA ELECTROCARDIOGRAMA QUE


MUESTRA EFECTOS DE LA HIPOKALEMIA
SECUNDARIA.

223 - EL MECANISMO FISIOPATOLÓGICO QUE EXPLICA EL DESARROLLO DE ESTAS ALTERACIONES


ELECTROCARDIOGRÁFICAS ES:

AUMENTO DE LA MECANISMO DE ACCIÓN DEL FURASEMIDE: La furosemida bloquea el sistema de


AUTOMATICIDAD. cotransporte de la Na+K+2Cl– localizado en la membrana de las células luminales de la rama
gruesa ascendente del asa de Henle. La acción diurética resulta de la inhibición de la
reabsorción del cloruro de sodio en este segmento del asa. Su efecto antihipertensivo se
atribuye a un aumento de la excreción de sodio, a una reducción del volumen sanguíneo y a
la disminución de la respuesta del músculo liso vascular a estímulos vasoconstrictores.
HIPOKALEMIA: se define como el nivel sérico de potasio inferior a 3.5mEq/l, es un trastorno
hidroelectrolítico frecuente.
ALTERACIÓN EN LA FISIOPATOLOGÍA CARDIACA DE LA HIPOKALEMIA: - En el tejido cardíaco, la hipokalemia
PROPAGACIÓN DE acelera la velocidad de conducción y la pendiente de despolarización de las células
IMPULSOS DE marcapasos, provocando arritmias. - La asociación de infarto de miocardio más hipokalemia
DESCARGA. conduce a mayor riesgo de desarrollo de taquicardia y fibrilación ventricular. RECUERDA
QUE... "Las arritmias se deben a alteraciones en la iniciación de los impulsos (automatismo)
o a alteraciones en la propagación de los impulsos (conducción)." LA HIPOKALEMIA CAUSA
ALTERACIONES EN LA VELOCIDAD DE CONDUCCIÓN DICHO DE OTRO MODO,
ALTERACIONES EN LA PROPAGACIÓN DE LOS IMPULSOS.

ALTERACIÓN DE LA DATOS ELECTROCARDIOGRÁFICOS DE HIPOKALEMIA: - Como consecuencia de la


FASE 2 DEL hipokalemia se producen trastornos en la conducción y el ritmo cardíaco. - Se reflejan
POTENCIAL DE electrocardiograficamente como disminución del voltaje de la onda T, depresión del
ACCIÓN. segmento ST y aparición de onda U prominente cuando el potasio plasmático es menor a
3mEq/l.

PRESENCIA DE POTENCIAL DE ACCIÓN DE RESPUESTA RÁPIDA CARDÍACA: En las células que presentan
BLOQUEOS este potencial de acción se identifican cinco fases. - Fase cero: De ascenso rápido, debido al
UNIDIRECCIONALES. ingreso abrupto de sodio por la activación de los canales rápidos. - Fase uno: Repolarización
temprana dada por la inactivación de la corriente de sodio y a la activación de corrientes
transitorias de potasio hacia fuera y cloro hacia adentro. - Fase dos: Fase de meseta. El
evento iónico principal es la corriente lenta de ingreso de calcio. El ingreso de calcio activa
la liberación de calcio por el retículo sarcoplásmico, evento fundamental para el
acoplamiento electromecánico. - Fase tres: Fase de repolarización rápida debido a la
activación de canales de POTASIO, permitiendo corriente hacia fuera. - Fase cuatro. Reposo
eléctrico, extrusión activa de sodio y recuperación del POTASIO que salió de la célula gracias
a la bomba sodio potasio. EL POTASIO ACTÚA PRINCIPALMENTE EN LA FASE 3 CON LA
SALIDA DE ÉSTE A TRAVÉS DE LOS CANALES ESPECÍFICOS Y EN LA FASE 4 CON LA
RECUPERACIÓN DE ÉSTE ELECTROLITO.

Bibliografía:
1. LONGO DL, FAUCI AS, KASPER DL, HAUSERSL, JAMESON JL, LOSCALZOJ. HARRISON. PRINCIPIOS DE
MEDICINA INTERNA, 18A EDICIÓN. MC GRAW HILL. NEW YORK, USA. 2012, PP 353-354, 1838.
ANÁLISIS DEL CASO CLÍNICO

IDENTIFICACIÓN DEL REACTIVO


Area: GINECOLOGÍA Y OBSTETRICIA
Especialidad: GINECOLOGÍA
Tema: AFECCIONES BENIGNAS Y MALIGNAS DEL UTERO
Subtema: CERVICITIS AGUDA Y CRÓNICA

CASO CLÍNICO SERIADO

MUJER DE 49 AÑOS DE EDAD, QUE ACUDE A LA CONSULTA POR PRESENTAR SANGRADO INTERMENSTRUAL,
POSTCOITAL, DISPAREUNIA Y DISURIA DESDE HACE 2 AÑOS, SIN SINTOMATOLOGÍA MENOPÁUSICA. CUENTA
CON OCLUSIÓN TUBARIA BILATERAL; AL TACTO VAGINAL SE ENCUENTRA CÉRVIX IRREGULAR CON ESCASO
SANGRADO RECIENTE, CUERPO UTERINO Y ANEXOS NORMALES.

Femenina de 49 años de edad.

-.

Sintomatología ginecológica y urinaria,


muy característica de la cervicitis aguda y
crónica.

Cérvix irregular con escaso sangrado


reciente, cuerpo uterino y anexos normales.

-.

224 - PARA CORROBORAR SU SOSPECHA DIAGNÓSTICA, USTED DEBERÁ REALIZAR:

LEGRADO Técnica por la que se obtienen muestras del interior del útero (Legrado endometrial), en el
UTERINO diagnóstico del cáncer de endometrio. Se realiza el legrado uterino fraccionado, mediante la
FRACCIONADO. separación del material del endometrio y del endocérvix. ESTÁ INDICADO EN SOSPECHA DE
PATOLOGÍA ENDOMETRIAL.

TACTO RECTAL. En el caso de la paciente, el examen anogenital externo, de vagina y cuello uterino se
justificaría en "búsqueda de vesículas, úlceras de poca profundidad y úlceras botonoides", y de
la región inguinal para detectar ganglios linfáticos inflamados o hipertrofiados, y la palpación
abdominal inferior y bimanual para evaluar la sensibilidad y la existencia de masas pelvianas,
deben ser parte del examen clínico, para descartar los cuadros infecciosos. EN EL CASO DE LA
PACIENTE SERÍA COMPLEMENTARIO; NO COMO PARTE DEL PROCEDIMIENTO DIAGNÓSTICO
PRINCIPAL Y SIEMPRE QUE SE SOSPECHE PATOLOGÍA TUMORAL.
HISTEROSCOPIA. La histeroscopia es un procedimiento clínico, que le permite a un ginecólogo ver el interior del
útero por medio de una endoscopia. Este procedimiento puede realizarse con fines de
diagnóstico o para tratamientos de patologías intrauterinas, y como método de intervención
quirúrgicos. DADO QUE ES UN MÉTODO INVASIVO NO DEBERÁ REALIZARSE DE PRIMERA
INSTANCIA EN NINGÚN CASO.

ESPECULOSCOPIA. La ESPECULOSCOPIA consiste en la introducción de un especulo en la vagina permite observar


las paredes de la vagina, el tipo de flujo, el tipo de sangrado que presenta y ante todo permite
una visión directa del cuello de la matriz (Cérvix). El cuello de la matriz es la parte que
comunica el interior del útero con la vagina, al estar en contacto con el exterior puede
afectarse por varias patologías infecciosas, pero a la vez, su condición de "Exterior" permite su
estudio y detección de los posibles problemas de forma precoz. Así, el cáncer de cuello uterino
es posible diagnosticarlo en etapas precancerígenas, de tal forma, que el tratamiento precoz
previene la aparición del cáncer de cuello. Una proporción significativa de mujeres con
cervicitis son asintomáticas, la cervicitis en estas mujeres puede ser detectada incidentalmente
durante un examen físico. Cuando está presente, los síntomas son inespecíficos. Todas las
mujeres tienen: Descarga vaginal purulenta o mucopurulenta y/o sangrado íntermenstrual o
postcoital, algunas mujeres también tienen uno o más de los siguientes síntomas: Disuria,
dispareunia, irritación vulvovaginal. Los síntomas urinarios son generalmente debido a la
infección concomitante por la uretra, que se produce en aproximadamente en el 15 % de las
mujeres con infección por Chlamydia cervical. RECUERDA SIEMPRE INICIAR POR LA CLÍNICA.
LA VAGINA Y EL CUELLO UTERINO SE EXPLORAN MEDIANTE LA INSERCIÓN DE UN ESPEJO
VAGINAL.

Bibliografía:
1. SCHONGUE J, SCHAFER J, HALVORSON L, HOFFMAN B, BRADSHAW K, CUNNINGHAM G. WILLIAMS
GINECOLOGÍA, DE LA 1A EDICIÓN EN INGLÉS. MC GRAW HILL. USA. 2009, PP 3-4.

225 - EL TRATAMIENTO MÁS INDICADO EN ESTE CASO ES:

LEGRADO UTERINO No, está indicada en la paciente del caso clínico porque el sangrado no es intrauterino;
INSTRUMENTAL. es de origen cervical.

CONIZACIÓN CERVICAL. Procedimiento quirúrgico para remover una pieza en forma de cono de tejido
proveniente del cuello uterino. Una CONIZACIÓN CERVICAL se utiliza para diagnosticar,
y con frecuencia tratar una enfermedad del cuello uterino como un cáncer o cambios
precancerosos. El procedimiento se realiza después de que una mujer se haya realizado
una prueba de Papanicolaou, cuyos resultados no fueron normales; esta prueba detecta
cambios cancerosos o células anómalas en el cuello del útero. En el caso de la paciente
no contamos con estudios previos que justifiquen la utilización de éste tratamiento. Está
indicado como manejo de la cervicitis crónica una vez que se ha otorgado tratamiento
sin mejoría. PARA TOMAR LA DECISIÓN DE REALIZAR UN CONO DEBERÁ CONTARSE
CON UN PROTOCOLO DIAGNÓSTICO PREVIO QUE JUSTIFIQUE ESTE PROCEDIMIENTO.

ELECTROFULGURACIÓN. La ELECTROFULGURACIÓN.- es un procedimiento seguro que se utiliza de manera


rutinaria en intervenciones quirúrgicas para extirpar tejidos dañinos o indeseados.
Igualmente, se puede utilizar para cauterizar y sellar vasos sanguíneos, lo cual ayuda a
reducir o detener un sangrado; existen actualmente otras alternativas de tratamiento
como: La Criocirugía y Terapia con láser. ESTÁ BIEN INDICADO EN EL CASO DE
ECTOPRIÓN SECUNDARIO A CERVICITIS CRÓNICA DESPUÉS DE HABER OTORGADO
TRATAMIENTO ESPECÍFICO.
TRATAMIENTO La mayoría de las mujeres con CERVICITIS, debe recibir tratamiento antibiótico empírico
ANTIBIÓTICO en el momento de la evaluación inicial, sin esperar los resultados de las pruebas de
EMPÍRICO. laboratorio que definan la etiología específica, especialmente si el seguimiento es
incierto. Los objetivos de dicho tratamiento son: aliviar los síntomas y la prevención de
la infección del tracto genital superior. El régimen de tratamiento empírico para la
cervicitis, debe incluir la cobertura de la Chlamydia especialmente para las mujeres de 25
años de edad, ya que la prevalencia de esta infección es mayor en este grupo de edad.
Será importante mencionar que la CHLAMYDIA TRACHOMATIS se considera el patógeno
más importante de los causantes de infecciones de transmisión sexual en países
desarrollados; típicamente se manifiesta como una cervicitis y uretritis. (LA PACIENTE
PRESENTA SÍNTOMATOLOGÍA GINECOLÓGICA + URINARIA) La CERVICITIS POR
CHLAMYDIA se acompaña de sensación de urgencia urinaria y disuria; puede además
estar asociada a salpingitis aguda e infección pélvica. Es común la hemorragia después
del coito o manchado intermenstrual secundaria a erosión del cuello uterino. NINGUNA
DE LAS OTRAS OPCIONES DE RESPUESTA CUMPLEN CON LOS REQUISITOS PARA SER
REALIZADAS EN ESTE MOMENTO, LO IDEAL ES DAR TRATAMIENTO ESPECÍFICO PERO
NO PODEMOS ESPERAR A LOS RESULTADOS DE CULTIVOS POR LO QUE SE DEBERÁ
INICIAR TRATAMIENTO EMPÍRICO POR DIAGNÓSTICO CLÍNICO. IMPORTANTE: si bien
no se refiere la presencia de leucorrea, principal signo clínico de cervicitis aguda y
crónica, el resto de los signos clínicos son altamente compatibles con este tipo de
enfermedad, sobre todo teniendo en cuenta que existe sintomatología urinaria asociada.
OJO: aunque por la presencia de irregularidad cervical y sangrado llegaras a sospechar
que el diagnóstico es un cáncer invasivo, recuerda que estos se pueden asociar con una
infección secundaria, que en este caso, se sustenta en la presencia de síntomas urinarios,
y el tratamiento antibiótico empírico también sería la opción en este momento como
primera línea. EN CONCLUSIÓN: - El legrado no aplica como tratamiento en ningún
momento pues el sangrado no es de origen uterino. - Está indicado dar TRATAMIENTO
ANTIBIÓTICO EMPÍRICO ante la sospecha de una cervicitis crónica o de una lesión
cervical asociada a infección (justificado por sintomatología ginecológica +
“sintomatología urinaria”). - En caso de considerarlo necesario, realizar en una segunda
intensión colposcopía y conización o electrofulguración, según sea el caso. RECUERDA:
la infección por chlamydia condiciona friabilidad de los tejidos, por lo que en cualquiera
de los casos será mejor ofrecer tratamiento antibiótico, siempre que sea posible, antes
de cualquier procedimiento invasivo.

Bibliografía:
1. GUÍA DE PRÁCTICA CLÍNICA, DIAGNÓSTICO Y TRATAMIENTO DE LA VAGINITIS INFECCIOSA EN MUJERES
EN EDAD REPRODUCTIVA EN EL PRIMER NIVEL DE ATENCIÓN. MÉXICO: SECRETARIA DE SALUD;
DICIEMBRE 2014. 2. GUÍA DE PRÁCTICA CLÍNICA, PREVENCIÓN Y DIAGNÓSTICO OPORTUNO DE LA
INFECCIÓN DEL TRACTO GENITOURINARIO INFERIOR POR CHLAMYDIA TRACHOMATIS. MÉXICO:
SECRETARIA DE SALUD; MARZO 2013.

http://www.cenetec.salud.gob.mx /descargas/gpc/CatalogoMaestro/081_GPC_Vaginitisinfec1NA/Vaginitis_ER_CENETEC.pdf

FIN DEL CASO CLÍNICO SERIADO


ANÁLISIS DEL CASO CLÍNICO

IDENTIFICACIÓN DEL REACTIVO


Area: MEDICINA INTERNA
Especialidad: NEUROLOGÍA
Tema: ENFERMEDADES CEREBROVASCULARES
Subtema: INFARTO CEREBRAL (TROMBÓTICOS O EMBÓLICOS)

CASO CLÍNICO SERIADO

HOMBRE DE 25 AÑOS DE EDAD QUE ES LLEVADO A URGENCIAS POR UN FAMILIAR QUIEN REFIERE UN
CUADRO CLÍNICO DE DEPRESIÓN DE POR LO MENOS 2 MESES DE EVOLUCIÓN. DOS HORAS ANTES DE SU
INGRESO ES ENCONTRADO EN SU CASA MUY DORMIDO Y JUNTO A EL CAJAS VACÍAS DE MEDICAMENTO NO
ESPECIFICADO. A LA EXPLORACIÓN LO ENCUENTRA CON RESPIRACIÓN SUPERFICIAL, SIN RESPUESTA A
ESTÍMULOS VERBALES, NI DOLOROSOS.

HOMBRE DE 25 AÑOS.

CUADRO CLÍNICO DE DEPRESIÓN DE POR


LO MENOS 2 MESES DE EVOLUCIÓN.

DOS HORAS ANTES DE SU INGRESO ES


ENCONTRADO EN SU CASA MUY
DORMIDO Y JUNTO A EL CAJAS VACÍAS DE
MEDICAMENTO NO ESPECIFICADO.

A LA EXPLORACIÓN LO ENCUENTRA CON


RESPIRACIÓN SUPERFICIAL, SIN
RESPUESTA A ESTÍMULOS VERBALES, NI
DOLOROSOS.

-.

226 - EL ESTUDIO DE GABINETE DE URGENCIA INDICADO PARA REALIZAR EL DIAGNOSTICO


DIFERENCIAL EN ESTE PACIENTE ES:

ELECTROENCEFALOGRAMA. INTOXICACIÓN POR BENZODIAZEPINAS • Las benzodiacepinas son medicamentos de


amplio uso con propiedades ansiolíticas, hipnóticas y sedantes. • Las
benzodiacepinas potencian la acción inhibitoria del neurotransmisor ácido gaba
amino butírico (GABA), favoreciendo el ingreso de iones de cloro a la célula, lo cual
genera hiperpolarización celular y disminuye la excitabilidad neuronal.
ANGIOGRAFÍA CEREBRAL. CUADRO CLÍNICO DE INTOXICACIÓN POR BENZODIAZEPINAS Los síntomas de
depresión del sistema nervioso central (SNC) suelen iniciarse rápidamente por vía
venosa, o a los 30–120 minutos por vía oral, dependiendo del compuesto. Los
síntomas más comunes son sedación, ataxia, somnolencia, disartria, nistagmus y
pupilas mióticas o intermedias. Puede haber además hiporreflexia, hipotermia e
hipotensión con taquicardia compensatoria. La náusea y el vómito son más comunes
en los niños. La aparición de coma debe hacer sospechar la coingestión de otros
depresores. Ocasionalmente pueden observarse algunos efectos paradójicos como
agresión, excitación, psicosis o deterioro neurológico importante, siendo ancianos y
niños más susceptibles a este tipo de manifestaciones. La inmovilidad prolongada
por inconciencia puede generar rabdomiólisis o escaras. Muy ocasionalmente el
compromiso respiratorio desencadena hipoxia y acidemia secundaria.

TOMOGRAFÍA AUXILIARES DE IMAGEN EN LA INTOXICACIÓN POR BENZODIAZEPINAS • La


COMPUTADA DE CRÁNEO. tomografía axial computadorizada (TAC) de cráneo, punción lumbar y radiografía de
tórax se indican si se sospechan complicaciones pulmonares u otras causas de
depresión del SNC. LA TOMOGRAFÍA DE CRÁNEO ESTÁ INDICADA COMO PARTE DEL
PROTOCOLO DE TODO PACIENTE CON DATOS DE DÉFICIT NEUROLÓGICO.

RESONANCIA MAGNÉTICA DIAGNÓSTICO DE INTOXICACIÓN POR BENZODIAZEPINAS • Debe solicitarse la


DE CRÁNEO. prueba rápida cualitativa para benzodiacepinas en orina o el examen cuantitativo,
teniendo en cuenta que los niveles urinarios se correlacionan pobremente con la
clínica. • En algunos casos puede ser necesario la toma de ionograma, BUN y
creatinina para descartar causas diferentes a las tóxicas que comprometan el estado
de conciencia. • Si se sospecha hipoxemia se debe ordenar oximetría de pulso o gases
arteriales. • La glicemia es mandatoria para descartar hipoglicemia como parte del
diagnóstico diferencial en un paciente con deterioro neurológico. • El citoquímico de
orina y la CPK son de utilidad para evaluar rabdomiólisis cuando el paciente está en
estado de coma.

Bibliografía:
1. LONGO DL, FAUCI AS, KASPER DL, HAUSERSL, JAMESON JL, LOSCALZOJ. HARRISON. PRINCIPIOS DE
MEDICINA INTERNA, 18A EDICIÓN. MC GRAW HILL. NEW YORK, USA. 2012, PP 3239, 3240.

227 - SE REALIZA ESTUDIO TOXICOLÓGICO REPORTANDO INTOXICACIÓN POR BENZODIAZEPINAS. EL


TRATAMIENTO DE ELECCIÓN EN ESTE CASO SERÍA:

FLUMAZENIL. ANTÍDOTO EN INTOXICACIÓN POR BENZODIAZEPINAS • El FLUMAZENIL en ampollas de 5mL (0.1


mg/mL) es el antídoto específico para la intoxicación por benzodiacepinas y su uso debe
restringirse a casos seleccionados, tales como: 1. Pacientes con sedación profunda (aquella que
mantiene al paciente en estado inconsciente). 2. Pacientes con depresión respiratoria (respiración
lenta o con insuficiente fuerza, de tal forma que no puede proporcionar una ventilación y perfusión
adecuada de los pulmones). 3. Pacientes en coma (falta de respuesta a estímulos, sin fenómeno de
despertar). o Luego de aplicado el flumazenil sus efectos se inician en 1-2 minutos y persisten
durante 1-5 horas dependiendo de la cantidad y el tipo de benzodiacepina ingerida. La dosis inicial
es de 0.2mg IV (0.01mg/kg en niños), y en caso de no obtenerse respuesta se suministran bolos de
0.3 mg, hasta un máximo de 3 mg en adultos y 1mg en niños. Si con esta dosis no ha revertido la
sedación pueden descartarse las benzodiacepinas como causa de la intoxicación. DADO QUE EL
PACIENTE SE ENCUENTRA INCONSCIENTE CON RESPIRACIÓN SUPERFICIAL Y SIN RESPUESTA A
ESTÍMULOS ESTÁ INDICADO EL USO DE FLUMAZENIL COMO ANTÍDOTO. Recuerda.... debes poner
suma atención en las indicaciones ya que eso guiará tu responder en otros casos en los que no se
presenten los datos descritos. A continuación otras medidas específicas que debes tener en cuenta.
NALOXONA. MEDIDAS TERAPÉUTICAS EN INTOXICACIÓN POR BENZODIAZEPINAS. • En caso de no contar con
flumazenil, la medicación alternativa es la aminofilina (ampollas de 240 mg/5 mL) administrada en
bolo IV de 6 mg/kg disuelta en 30 mL de DAD o solución salina, para pasar en 30 minutos,
disminuyendo la dosis de carga a la mitad si el paciente es mayor de 60 años, si no está
adecuadamente oxigenado o presenta enfermedad hepática o cardiaca. • No inducir el vómito,
pues no ha demostrado utilidad y está absolutamente contraindicada en pacientes con algún grado
de deterioro neurológico por el riesgo de broncoaspiración. • Suspender la vía oral hasta que el
paciente esté consciente. • Intubación orotraqueal, según el estado neurológico, antes de iniciar la
descontaminación del tracto gastrointestinal. • Suministrar catártico: sulfato de magnesio 250
mg/kg, hasta un máximo total de 30 g, en solución al 25%. Puede utilizarse como alternativa
manitol al 20% en dosis de 1 g/kg por vía oral (5mL/kg). "El catártico se suministra en dosis única
luego de la primera dosis de carbón activado" y sólo se repite si en las siguientes cuatro horas no
hay deposiciones.

LAVADO • Oxigeno suplementario, según la condición del paciente. • Si se presenta hipotensión que no
GÁSTRICO Y mejora con la administración de solución salina al 0,9%, administrar dopamina 2-5 mcgr/kg/ min,
DIURESIS titulando hasta obtener el efecto deseado. Es poco probable que dosis superiores a 20 mcgr/kg
FORZADA. tengan algún beneficio. • LOS DIURÉTICOS NO AUMENTAN LA VELOCIDAD DE ELIMINACIÓN DE
LAS BENZODAZEPINAS POR LO QUE SU EMPLEO CON TAL FIN NO ESTÁ JUSTIFICADO.

CARBÓN • Lavado gástrico en pacientes que consulten dentro de la hora siguiente a la intoxicación y
ACTIVADO Y conserven intacto el reflejo nauseoso y tusígeno o tengan intubación orotraqueal. • Carbón
DIÁLISIS activado 1 g/kg de peso, en solución al 25%, vía oral, cada ocho horas durante 24 horas. • La
PERITONEAL. DIÁLISIS PERITONEAL "es de limitada efectividad en la remoción de drogas". Para la mayoría de los
tóxicos no constituye una ventaja adicional respecto de la diuresis forzada con manipulación acido-
básica.

Bibliografía:
1. PAPADAKIS MAXINE A, MCPHEE STEPHEN J. DIAGNÓSTICO CLÍNICO Y TRATAMIENTO. 52ª EDICIÓN.
NUEVA YORK. 2013, PP 1579.

FIN DEL CASO CLÍNICO SERIADO


ANÁLISIS DEL CASO CLÍNICO

IDENTIFICACIÓN DEL REACTIVO


Area: CIRUGÍA
Especialidad: PROCTOLOGÍA Y UROLOGÍA
Tema: PATOLOGÍA DE COLÓN, ANO Y RECTO
Subtema: ABSCESO ANORRECTAL

CASO CLÍNICO SERIADO

MUJER DE 38 AÑOS QUE ACUDE A URGENCIAS AL PRESENTAR SALIDA DE MATERIAL PURULENTO POR EL
ANO. HACE APROXIMADAMENTE 25 DÍAS PRESENTÓ UN ABSCESO GLÚTEO QUE DRENÓ Y CURÓ
ESPONTÁNEAMENTE.

-.

absceso glúteo hace 25 días que dreno y


curó espontáneamente.

salida de material purulento por el ano.

-.

-.

228 - EL DIAGNÓSTICO CLÍNICO MÁS PROBABLE EN ESTE PACIENTE ES:

ENFERMEDAD Las FÍSTULAS ANORRECTALES pueden aparecer por enfermedad inflamatoria intestinal por la
DE CROHN. complicación de diverticulos inflamados (diverticulitis). LAS 3 PRIMERAS RESPUESTAS
CORRESPONDEN A ENTIDADES PATOLÓGICAS QUE COMO COMPLICACIÓN PUEDEN RESULTAR
EN FÍSTULAS PERIANALES; SIN EMBARGO, LA PRESENCIA DE MATERIAL PURULENTO EN RECTO
NO FUNDAMENTA EL DIAGNÓSTICO DE NINGUNA DE ELLAS. La ENFERMEDAD DE CROHN es un
trastorno inflamatorio crónico transmural que puede involucrar cualquier parte del tracto
gastrointestinal con preferencia por el íleon, ciego y colon. Se caracteriza por diarrea crónica,
dolor abdominal, pérdida de peso, fiebre y sangrado rectal. Una de las características de esta
enfermedad es la formación de fístulas las cuales suelen derivar a colon, vejiga, vagina o piel. En
el caso de las fístulas a vagina estas pueden encontrarse a cualquier nivel (baja, media o alta),
pudiendo ser rectovaginales, colovaginales y enterovaginales. UNA CARACTERÍSTICA
PRIMORDIAL DE LA ENFERMEDAD DE CROHN ES SU CAPACIDAD PARA FISTULIZAR; SIN
EMBARGO, NO EXISTEN ANTECEDENTES PARA SOSPECHAR ESTA ENFERMEDAD.
COLITIS La COLITIS ULCEROSA se caracteriza por una inflamación de la mucosa, generalmente se ubica en
ULCERATIVA recto en sentido proximal hasta afectar el colon en diferentes medidas. La mayor parte de los
CRÓNICA. casos se circunscribe a recto o rectosigmoides; algunas veces, aunque poco frecuente se extiende
a todo el colon y llega hasta el íleon. Una tercera parte de los casos de ENFERMEDAD DE CROHN
tiene afección perianal caracterizada por, fisuras anales, abscesos perianales y fístulas, el
diagnóstico clínico diferencial se basará en que LA COLITIS ULCERATIVA GENERALMENTE NO
PRODUCE FÍSTULAS. IMPORTANTE: se escribe “enfermedad inflamatoria intestinal” para referir a
la enfermedad de crohn y a la colitis ulcerativa, es por ello que ambas entidades sin diferenciar se
pronuncian en relación a muchas patologías. La realidad es que una diferencia entre estas es que
la colitis ulcerosa generalmente no produce fístulas.

DIVERTICULITIS La ENFERMEDAD DIVERTICULAR se caracteriza por sintomatología clínica variada, como dolor
COMPLICADA. abdominal crónico en el cuadrante inferior izquierdo o sangrado de los divertículos. Los síntomas
derivan principalmente a trastornos en la motilidad. En este grado de la enfermedad no hay
fiebre ni leucocitosis. En la DIVERTICULITIS se presenta inflamación de los divertículos, es un
cuadro agudo caracterizado por dolor, sensibilidad y presencia de masa en cuadrante inferior
izquierdo, fiebre y leucocitosis. La DIVERTICULITIS COMPLICADA se define como aquella que se
acompaña de un absceso, fístula, obstrucción o perforación libre intrabdominal. SI BIEN LA
DIVERTICULITIS SE ASOCIA CON FÍSTULAS EN SU FORMA COMPLICADA, NO EXISTEN
ANTECEDENTES QUE SUGIERAN ESTA PATOLOGÍA.

FÍSTULA La FÍSTULA ANAL es una enfermedad caracterizada por la presencia de un conducto de paredes
PERIANAL. fibrosas afectadas que comunica una cripta anal o el lumen del recto con la piel. El orificio
localizado en la cripta anal se denomina primario o interno, y el cutáneo, secundario o externo.
Las fístulas simples son generalmente interesfintéricas o transesfintéricas bajas, generalmente
con un trayecto único entre el oficio primario y el secundario. La fístula compleja tiene un
trayecto que comprende más del 30% a 50% del esfínter externo (transesfintérica alta,
supraesfintérica o extraesfintérica), es de localización anterior en mujeres, la padece un paciente
con incontinencia, existe enfermedad inflamatoria intestinal o el antecedente de cirugías previas
anorrectales. Se presentan en el 50% de los casos que han desarrollado absceso anorrectal. Los
antecedentes de importancia en este caso son: patología intestinal asociada, eventos que
comprometen la función del esfínter, cirugía anorrectal, trauma obstétrico, “procesos infecciosos
en el área anorrectal”. Por lo general se presentan con descarga a través del orifico externo, en
ocasiones con dolor. A la exploración física se observa el orifico externo con descarga y se palpa
trayecto con fibrosis. Casi siempre es el resultado de un absceso anterior, éste absceso en general
se desarrrolla en las glándulas anales pero no es improbable que se genere a partir de una
infección en el piso de la pelvis o la región glútea. TODO ABSCESO NO RESUELTO TIENDE A
FISTULARSE YA QUE EL MATERIAL PURULENTO QUE CONTIENE BUSCA UN TRAYECTO DE
SALIDA. DADO QUE LA PACIENTE NO RECIBIÓ TRATAMIENTO PREVIO, LA RECIDIVA DEL
ABSCESO ES GRANDE CON UNA CONSECUENTE FÍSTULA QUE PUEDE DRENAR A CUALQUIER
PARTE DEL PISO PÉLVICO EN ÉSTE CASO RECTO.

Bibliografía:
1. GUÍA DE PRÁCTICA CLÍNICA, DIAGNÓSTICO Y TRATAMIENTO QUIRÚRGICO DE LA FÍSTULA ANAL EN EL
ADULTO. MÉXICO: SECRETARIA DE SALUD; 12 DICIEMBRE 2013. 2. BRUNICARDI F, ANDERSEN D, BILLIAR T, Y
COLS. SCHWARTZ PRINCIPIOS DE CIRUGÍA, 9A EDICIÓN. MC GRAW HILL. 2011, PP 1064-1065. 4. DOHERTY G.
DIAGNÓSTICO Y TRATAMIENTO QUIRÚRGICO, 13A EDICIÓN. MC GRAW HILL LANGE. 2011, PP 615-616.

http://www.cenetec.salud.gob.mx/descargas/gpc/CatalogoMaestro/200_SSA_09_fistula_anal/GPC_SSA_200-09_FISTULA_ANAL_No_sexalar_actualizada.pdf

229 - EL TRATAMIENTO INDICADO EN ESTE PACIENTE SERÁ CON:

FISTULOTOMÍA CON Los objetivos fundamentales del tratamiento de la FÍSTULA ANAL son: curarla, preservar la
MARSUPIALIZACIÓN. función de continencia anal, minimizar los defectos de cicatrización y ofrecer al paciente
una recuperación rápida. El manejo es quirúrgico e incluye: fistulostomía, fistulectomía con
o sin reparación de esfínteres, sedal, colocación de setón de corte o de drenaje, avance de
colgajo de mucosa rectal y aplicación de fibrina. LA FISTULOTOMÍA ES EL MANEJO
INDICADO PARA LA FÍSTULA PERIRECTAL, CUANDO SE COMPLETA CON SUTURA DE LA
PIEL A LOS BORDES DEL TEJIDO DE GRANULACIÓN RECIBE EL NOMBRE DE
MARSUPIALIZACIÓN Y CONSTITUYEN EL MÉTODO TERAPÉUTICO MÁS USADO
ACTUALMENTE.
ANTIBIÓTICOS Y Usamos el término COLOSTOMÍA para referirnos al abocamiento del intestino grueso hacia
COLOSTOMÍA. el exterior, practicado quirúrgicamente con miras a derivar parcial o totalmente el tránsito
intestinal. Las colostomías se efectúan con fines terapéuticos. Pueden ser transitorias o
definitivas: las segundas tienen por objetivo derivar el tránsito intestinal mientras
tratamos la lesión distal. UNA FÍSTULA PERINANAL NO ES CONDICIÓN SUFICIENTE PARA
REALIZAR UN PROCEDIMIENTO TAN INVASIVO COMO LA COLOSTOMÍA, LOS
ANTIBIÓTICOS SI ESTÁN INDICADOS PERO NO SUSTITUYEN AL MANEJO QUIRÚRGICO.

INMUNOSUPRESORES El tratamiento de la fístula perianal es siempre quirúrgico no es recomendable su demora


Y ANTIBIÓTICOS. basándose en la toma de antibióticos o antiinflamatorios, por el riesgo de progresión y
propagación de la infección ya que se puede complicar con sepsis perianal. LA TERAPIA
EXPECTANTE CON ADMINISTRACIÓN DE INMUNOSUPRESORES Y ANTIBIÓTICOS PUEDE
CONDUCIR A COMPLICACIONES SERIAS EN LA REGIÓN ANAL. EL MANEJO DE LA FÍSTULA
PERIANAL SIEMPRE SERÁ QUIRÚRGICO.

DRENAJE ABIERTO. Una fístula que no es posible reparar por fistulostomía puede necesitar que se ponga un
drenaje especial – un setón – durante al menos seis semanas, después de la cual se hace
una reparación quirúrgica definitiva. El setón de corte es una técnica quirúrgica para
aquellas fístulas perianales en las que se encuentra comprometido alguno o los dos
esfínteres anales. CIERTAMENTE EL SETÓN ES UN DRENAJE ABIERTO DE LA FÍSTULA PERO
SE RESERVA A AQUELLOS CASOS EN QUE LAS FÍSTULAS SON COMPLICADAS O EL
ABORDAJE POR FISTULOSTOMÍA NO HA SIDO SUFICIENTE.

Bibliografía:
1. GUÍA DE PRÁCTICA CLÍNICA, DIAGNÓSTICO Y TRATAMIENTO QUIRÚRGICO DE LA FÍSTULA ANAL EN EL
ADULTO. MÉXICO: SECRETARIA DE SALUD; 12 DICIEMBRE 2013. 2. BRUNICARDI F, ANDERSEN D, BILLIAR T, Y
COLS. SCHWARTZ PRINCIPIOS DE CIRUGÍA, 9A EDICIÓN. MC GRAW HILL. 2011, PP 1064-1065. 3. DOHERTY G.
DIAGNÓSTICO Y TRATAMIENTO QUIRÚRGICO, 13A EDICIÓN. MC GRAW HILL LANGE. 2011, PP 615-616.

http://www.cenetec.salud.gob.mx/descargas/gpc/CatalogoMaestro/200_SSA_09_fistula_anal/GPC_SSA_200-09_FISTULA_ANAL_No_sexalar_actualizada.pdf

FIN DEL CASO CLÍNICO SERIADO


ANÁLISIS DEL CASO CLÍNICO

IDENTIFICACIÓN DEL REACTIVO


Area: MEDICINA INTERNA
Especialidad: NEFROLOGÍA
Tema: INSUFICIENCIA RENAL AGUDA
Subtema: INSUFICIENCIA RENAL AGUDA PRERRENAL

CASO CLÍNICO SERIADO

MUJER DE 72 AÑOS HIPERTENSA DESDE HACE 30 AÑOS, ACTUALMENTE EN CONTROL CON CAPTOPRIL.
POSTERIOR A LA INGESTA DE SALMÓN AHUMADO PRESENTA 15 EVACUACIONES LÍQUIDAS ABUNDANTES Y 10
VÓMITOS EN LAS ÚLTIMAS 24 HORAS. EN LA EXPLORACIÓN SE LE ENCUENTRA CON DATOS DE
DESHIDRATACIÓN SEVERA. EL LABORATORIO REPORTA NA 150 MEQ/L, K 2.3MEQ/L, DENSIDAD URINARIA 1.025,
UREA DE 60, CREATININA 2.2.

ADULTO MAYOR DE 72 AÑOS DE EDAD.

HIPERTENSA DESDE HACE 30 AÑOS,


ACTUALMENTE EN CONTROL CON
CAPTOPRIL. RECUERDA EL CAPTOPRIL ES
UN FÁRMACO CONSIDERADO FACTOR DE
RIESGO PARA Insu ciencia renal aguda
PRERRENAL.

15 EVACUACIONES LÍQUIDAS
ABUNDANTES Y 10 VÓMITOS EN LAS
ÚLTIMAS 24 HORAS. "PÉRDIDAS
INCREMENTADAS".

DESHIDRATACIÓN SEVERA. POR LO


MENOS DEL 13 AL 15% DE PÉRDIDAS.

NA 150 MEQ/L, K 2.3MEQ/L, DENSIDAD


URINARIA 1.025, UREA DE 60, CREATININA
2.2. MUY IMPORTANTE EL INCREMENTO
DE AZOADOS Y DE LA DENSIDAD
URINARIA.

230 - EL DIAGNÓSTICO CLÍNICO MÁS PROBABLE ES DE INSUFICIENCIA:


PRE-RENAL. ES MUY CLARO QUE LA PACIENTE PRESENTA UNA INSUFICIENCIA RENAL AGUDA SECUNDARIA A
DESHIDRATACIÓN SEVERÁ, LO QUE IMPLICA HIPOPERFUSIÓN RENAL QUE POR DEFINICIÓN
DEBERÁ SER CONSIDERADA INSUFICIENCIA RENAL AGUDA DE ORIGEN PRERRENAL. POR LA EDAD
DE LA PACIENTE, DEBERÁS CONSIDERAR DISTINTOS FACTORES QUE HACEN MÁS PROPENSOS AL
PACIENTE ANCIANO A ESTE PADECIMIENTO Y QUE TAMBIÉN DIFICULTAN SU DIAGNÓSTICO. El
diagnóstico de la enfermedad renal en el anciano resulta especialmente complejo por varias
razones, entre las que cabe destacar: a) Inespecificidad de los síntomas y la frecuente ausencia de
aquellos que son habituales en los jóvenes, aunque en general la presentación de la enfermedad
renal en adultos de edad avanzada o frente a los más jóvenes no es significativamente diferente. b)
Concurrencia de enfermedades no renales cuya presentación clínica y síntomas se superponen,
contribuyendo al retraso diagnóstico y a la infravaloración de la participación de la enfermedad
renal. c) Difícil interpretación de los parámetros analíticos (urinarios y séricos) en el contexto de los
cambios fisiológicos de la función renal relacionados con el envejecimiento (por ej., reducción
fisiológica del filtrado glomerular, escaso valor de la creatinina sérica como índice aislado de la
función renal). La IRA es un síndrome clínico, potencialmente reversible, caracterizado por el rápido
deterioro de la función renal, en ocasiones horas, días o semanas, y cuyo elemento común se
traduce en un aumento de la concentración de productos nitrogenados en sangre, representados
principalmente por la urea y la creatinina, tal y como se muestra en nuestro paciente. FRA (Falla
Renal Aguda) prerrenal. Caracterizado por un descenso de la presión de filtración glomerular,
secundaria a diferentes situaciones clínicas, y, en consecuencia, una hipoperfusión renal, con
alteración de los mecanismos reguladores que, en caso de mantenerse sin corrección, pueden
derivar en un FRA intrínseco o parenquimatoso. Pruebas complementarias: 1. Osmolaridad y sodio
urinarios: ayudan a establecer el diagnóstico diferencial entre patología funcional y estructural. Así
en el FRA prerrenal se ponen en marcha mecanismos reguladores renales que contribuyen al ahorro
de sodio y agua, dando lugar a una orina concentrada, con una eliminación reducida de sodio
(osmolaridad urinaria > 400, Na en orina < 20 mEq/L Causas de IRA (Insuficiencia Renal Aguda)
prerrenal. 1. Disminución del gasto cardiaco. • Shock, IAM (Infarto Agudo al Miocardio),
valvulopatías, endocarditis, arritmias, insuficiencia cardiaca congestiva, embolismo pulmonar,
taponamiento cardiaco, miocarditis. 2. Vasodilatación periférica • Hipotensores, nitritos, sepsis,
hipercapnia, hipoxemia. Disminución del volumen circulante efectivo. • Por pérdidas reales del
líquido extracelular (LEC): hemorragia, vómitos, diarrea, quemaduras, diuréticos, nefropatías pierde
sal, insuficiencia suprarrenal. • Por redistribución del líquido extracelular (LEC): síndrome nefrótico,
hepatopatías graves, obstrucción intestinal, pancreatitis, malnutrición, rabdomiólisis. 3.
Alteraciones de la dinámica arteriolar glomerular. • Inhibidores de la síntesis de prostaglandinas
(AINEs), sustancias b-adrenérgicas, síndrome hepatorrenal, sepsis, inhibidores de la enzima de
conversión de la angiotensina (IECAS), antagonistas de los receptores de angiotensina II (ARA II).

RENAL FALLO RENAL AGUDO INTRÍNSECO O PARENQUIMATOSO. Tal y como su nombre indica, implica
INTRÍNSECA. daño a nivel del parénquima renal secundario al resultado de la actuación de diferentes procesos. El
FRA parenquimatoso engloba las siguientes entidades fisiopatológicas: la necrosis tubular aguda,
las enfermedades tubulointersticiales, las glomerulopatías, la necrosis cortical y las alteraciones a
nivel vascular. Causas de IRA intrínseca o parenquimatosa: A) Necrosis tubular aguda 1. Isquémica:
secundaria a factores que provoquen hipoperfusión. 2. Nefrotóxica: antibióticos (aminoglucósidos,
anfotericina B), contrastes, quimioterápicos, hemólisis, mioglobinurias, ácido úrico, oxalato y
discrasias de células plasmáticas. B) Enfermedades tubulointersticiales. • Idiopáticas, infecciosas,
inmunológicas, neoplásicas y por fármacos. C) Glomerulopatías • Glomerulonefritis agudas,
vasculitis y enfermedades del tejido conectivo. D) Necrosis cortical. • Coagulación intravascular
diseminada, aborto séptico. E) Alteraciones renovasculares • Trombosis, embolias, vasculitis,
traumatismos, enfermedades del tejido conectivo e hipertensión arterial.

RENAL POR LA INSUFICIENCIA RENAL POR TOXINAS, FORMA PARTE DE LA INTRÍNSECA, RENAL O
TOXINAS. PARENQUIMATOSA. AL ENCONTRARSE AMBAS OPCIONES, SE DESCARTAN AMBAS.

POST- Causas de INSUFICIENCIA RENAL AGUDA postrenal: A) Obstrucciones uretrales intrínsecas •


RENAL. Nefrolitiasis, neoplasias, coágulos, disfunción vesical por fármacos (anticolinérgicos), necrosis
papilar. B) Obstrucciones uretrales extrínsecas • Neoplasias, fibrosis retroperitoneal, radioterapia,
hiperplasia benigna de próstata, traumatismos. C) Obstrucciones intratubulares • Depósitos de
ácido úrico, cristales de oxalato, cristalización de fármacos (Aciclovir, metotrexate, sulfamidas),
cadenas ligeras.

Bibliografía:
1. LONGO DL, FAUCI AS, KASPER DL, HAUSERSL, JAMESON JL, LOSCALZOJ. HARRISON. PRINCIPIOS DE
MEDICINA INTERNA, 18A EDICIÓN. MC GRAW HILL. NEW YORK, USA. 2012, PP 2294-2296, 2298. 2.
PAPADAKIS MAXINE A, MCPHEE STEPHEN J. DIAGNÓSTICO CLÍNICO Y TRATAMIENTO. 52ª EDICIÓN. NUEVA
YORK. 2013, PP 902-903.
231 - EL FACTOR FISIOPATOLÓGICO DESENCADENANTE DEL PADECIMIENTO ACTUAL DEL ENFERMO ES:

OBSTRUCCIÓN INSUFICIENCIA RENAL AGUDA (IRA) POSRENAL U OBSTRUCTIVA Las causas son lesiones que
URETERAL. produzcan un obstáculo en la vía urinaria que impida la salida de la orina formada,
provocando un aumento de presión que se transmite retrógradamente, comprometiendo el
filtrado glomerular. Supone un 5% de las causas de IRA. Pueden ser lesiones extrarrenales de
uréteres-pelvis (litiasis, tumores, fibrosis...), vejiga (litiasis, coágulos, tumores, prostatismo,
vejiga neurógena), uretra (estenosis, fimosis) o también lesiones intrarrenales (depósito de
cristales, coágulos, cilindros). Para que estas causas produzcan una IRA es necesario que la
obstrucción sea grave, prolongada y que afecte a tracto urinario distal (meato uretral
externo, cuello de la vejiga) o bien a los uréteres de manera bilateral o unilateral en paciente
con un único riñón funcionante. LA OBSTRUCCIÓN URETERAL PROVOCA UNA INSUFICIENCIA
POSTRRENAL.

ACTIVACIÓN DE FISIOPATOLOGÍA INSUFICIENCIA RENAL AGUDA PRERRENAL. La azoemia prerrenal


BARORECEPTORES. representa una respuesta fisiológica a la hipoperfusión renal que, a su vez, desencadena
disminución de la presión de ultrafiltración glomerular lo que, en conjunto, favorece la
disminución de la filtración glomerular y la acumulación de desechos nitrogenados y puede
asociarse con trastornos hidroelectrolíticos o no hacerlo. La reversibilidad de la azoemia
prerrenal depende de la pronta restauración del flujo sanguíneo renal y, por ende, de la
presión de ultrafiltración glomerular. La hipoperfusión grave puede conducir a isquemia del
parénquima renal y falla renal intrínseca; por consiguiente, dentro del espectro de
manifestaciones de la hipoperfusión renal debe incluirse la azoemia prerrenal y la IRA
intrínseca secundaria. Con base en lo anterior, se considera que la azoemia prerrenal puede
complicar u originarse en cualquier enfermedad que curse con hipovolemia, disminución del
gasto cardíaco, vasodilatación sistémica o vasoconstricción renal selectiva. "La hipovolemia
conlleva a disminución de la tensión arterial media efectiva, frente a lo cual se ACTIVACIÓN
DE LOS BARORRECEPTORES arteriales y cardiacos, lo que desencadena una serie coordinada
de respuestas neurohumorales, cuyo objetivo final es restaurar el volumen sanguíneo y la
presión arterial". Las sustancias vasoactivas endógenas actúan en concierto para preservar la
función cardíaca y cerebral, estimulando la vasoconstricción en lechos vasculares ‘no
esenciales’ (circulación musculocutánea y esplácnica), activando la sensación de sed y
promoviendo la retención renal de sal y agua. El riñón es altamente susceptible a la
hipoperfusión; la lesión isquémica aguda renal es mucho más frecuente que en cerebro,
miocardio o hígado. La perfusión glomerular, la presión de ultrafiltración y la tasa de
filtración se preservan durante la hipoperfusión leve gracias a la activación de varios
mecanismos compensatorios como la vasodilatación arteriolar aferente favorecida por un
reflejo local miogénico y por la acción de prostaglandinas vasodilatadores (prostaglandina F2
y prostaciclina); además, se produce vasoconstricción arteriolar eferente por efecto de la
angiotensina II, lo que mantiene estable la presión intraglomerular al aumentar la fracción de
filtración, que finalmente preserva la tasa de filtración glomerular. Durante los estados de
hipoperfusión grave y sostenida se sobrecargan los mecanismos de equilibrio descritos y, por
ende, la tasa de filtración glomerular no puede mantenerse y se desarrolla insuficiencia renal
aguda prerrenal. En términos prácticos, la azoemia prerrenal se presenta en respuesta a la
inadecuada perfusión renal que, generalmente, es secundaria a pérdida de agua, sangre o
pérdida del tono vascular, lo cual genera incremento de la reabsorción tubular de agua y la
consecuente reabsorción de nitrógeno ureico, que a su vez explica la disociación en la
relación creatinina: BUN. La normalización de la volemia favorece una rápida respuesta de los
mecanismos renales de defensa y, por consiguiente, no podemos considerar realmente la
presencia de azoemia prerrenal.
DISFUNCIÓN Desde una óptica clinicopatológica, las causas de INSUFICIENCIA RENAL (IRA) renal,
ENDOTELIAL POR parenquimatosa o intrínseca se dividen así: 1) patología de grandes vasos, 2) compromiso de
TOXINAS. la microcirculación renal y glomérulos, 3) falla renal aguda isquémica o nefrotóxica, y 4)
enfermedades tubulointersticiales. En la mayoría de los casos, los elementos desencadenantes
son la isquemia o las nefrotoxinas, las cuales clásicamente inducen necrosis tubular aguda.
INJURIA DIRECTA (NEFROTÓXICOS). Dentro de las causas de injuria directa, la más
importante es el uso de antibióticos nefrotóxicos o potencialmente nefrotóxicos, solos o en
combinación con otras drogas nefrotóxicas. Entre estas tenemos principalmente a los
aminoglucósidos usados solos o en combinación con cefalosporinas o furosemida y
anfotericina B. Se ha descrito algunos factores de riesgo para desarrollar nefrotoxicidad por
aminoglucósidos, dentro de los cuales se señalan el nivel sérico, el sexo femenino y la
presencia de enfermedad hepática e hipotensión arterial. Las sustancias de contraste, también
pueden producir IRA principalmente en pacientes con insuficiencia renal crónica de base,
diabéticos y con mieloma múltiple. La fisiopatología no es muy clara pero probablemente
esté relacionado a la toxicidad directa de la sustancia de contraste sobre los túbulos renales o
a isquemia renal. Se ha observado en pacientes sometidos a urografía excretoria, arteriografía
e incluso venografía y su incidencia en pacientes con función renal normal es entre 0.6 y 1.4%,
mientras que en diabéticos con insuficiencia renal, la incidencia excede el 90%. Otras drogas
importantes a tener en cuenta no por su efecto tóxico directo renal sino por su efecto
inhibidor de prostaglandinas (prostaglandinas producen un efecto antagónico a angiotensina
en la arteriola aferente), son los antiinflamatorios no esteroideos que pueden producir IRA en
pacientes que tienen estimulado el eje renina-angiotensina- aldosterona, tal como ocurre en
los pacientes con lupus eritematoso sistémico, en pacientes con hipoalbuminemia crónica o
con insuficiencia renal preexistente. LA DISFUNCIÓN ENDOTELIAL POR TOXINAS PROVOCA
UNA INSUFICIENCIA RENAL PARENQUIMATOSA.

ISQUEMIA INJURIA RENAL INDIRECTA (hipoperfusión renal). La hipoperfusión renal prolongada, es la


MEDULAR. causa más frecuente de necrosis tubular aguda (NTA) y es observada en pacientes sometidos
a cirugía mayor, trauma, hipovolemia severa, sepsis y quemados. La NTA que ocurre en
trauma es debida generalmente a un efecto combinado de hipovolemia y mioglobina u otras
toxinas liberadas por daño tisular. En los pacientes con quemaduras de más del 15% de su
superficie corporal, la NTA se produce por hipovolemia, rabdomiólisis, sepsis y antibióticos
nefrotóxicos. En algunos casos de intoxicación por metanfetamina (éctasis), se ha descrito
IRA y la etiología probable es la asociación de rabdomiólisis e hipotensión arterial. La
metanfetamina, estimulante de los últimos años produce hipertermia, trastornos
cardiovasculares, hipertensión arterial e isquemia cardiaca, renal, cerebral y de otros órganos.
SI NO SE CORRIGE LA HIPOVOLEMIA Y LA LESIÓN EN ESTE PACIENTE, PUEDE EVOLUCIONAR
A UNA ISQUEMIA MEDULAR, PERO SÓLO EN CASOS SOSTENIDOS DONDE NO RECIBEN
TRATAMIENTO.

Bibliografía:
1. LONGO DL, FAUCI AS, KASPER DL, HAUSERSL, JAMESON JL, LOSCALZOJ. HARRISON. PRINCIPIOS DE
MEDICINA INTERNA, 18A EDICIÓN. MC GRAW HILL. NEW YORK, USA. 2012, PP 2294-2296, 2298. 2.
PAPADAKIS MAXINE A, MCPHEE STEPHEN J. DIAGNÓSTICO CLÍNICO Y TRATAMIENTO. 52ª EDICIÓN. NUEVA
YORK. 2013, PP 902-903.

232 - ES EL TRATAMIENTO INDICADO EN EL PACIENTE:

NEFROSTOMÍA. Manejo de la INSUFICIENCIA RENAL AGUDA (IRA) POSTRENAL U OSTRUCTIVA. El tratamiento


definitivo es desobstruir. La mayoría de las causas que la producen son quirúrgicas, por lo que
será necesaria la consulta con el urólogo. De manera transitoria se procederá al sondaje vesical en
el caso de patología prostática. En caso de obstrucción uretral con hidronefrosis será necesario la
realización de nefrectomía percutánea.

HEMODIALISIS. Las indicaciones de DIÁLISIS en la INSUFICIENCIA RENAL AGUDA (IRA) son: hiperpotasemia o
hiponatremia graves, acidosis metabólica con bicarbonato plasmático menor de 10 mEq/l,
sobrecarga de líquidos con edema pulmonar o insuficiencia cardiaca, pericarditis urémica,
encefalopatía urémica, diátesis hemorrágica urémica y azotemia severa (urea mayor a 250 mg/dl
o creatinina mayor a 10 mg/dl). EN EL CASO CLÍNICO NO SE REFIERE NINGUNA DE ÉSTAS.
DOPAMINA FALLO RENAL AGUDO (FRA) INTRÍNSECO O PARENQUIMATOSO. Una vez establecida la necrosis
MÁS tubular aguda (NTA), no existe actualmente un tratamiento que cambie su curso. Se aplican las
FUROSEMIDE. mismas medidas generales —monitorización de constantes, vigilancia de volúmenes, corrección
de alteraciones electrolíticas y de acidosis si ésta está presente—, tratando aquellos factores
prerrenales, con un control de hemodinámica estricto. Es importante tener en cuenta que frente a
una situación de oliguria, la reposición hidrosalina debe ser igual a las pérdidas cuantificadas más
las insensibles, pudiendo convertir dicha situación en poliúrica con la ayuda de bolos de
diuréticos —de asa— y de dopamina a dosis vasodilatadoras si no se obtiene la respuesta
deseada. En el caso de las NIA asociada a nefrotóxicos, el FRA desaparece tras la retirada del
fármaco, pudiendo utilizarse esteroides si esta medida no es suficiente. En las vasculitis y las
glomerulonefritis, el tratamiento de elección se basa en esteroides e inmunosupresores
(ciclofosfamida).

SOLUCIONES Manejo del paciente con INSUFICIENCIA RENAL AGUDA (IRA) PRERRENAL. • Dieta rica en
ISOTÓNICAS. hidratos de carbono y con aporte de proteínas de alto valor biológico entre 0,6-0,8 gr/Kg/día. •
Monitorizar tensión arterial, frecuencia cardiaca y medición de la ingesta y pérdida de agua y sal.
El mecanismo de control mas simple es el peso diario. • El sondaje vesical será necesario si se
precisa la medición de diuresis horaria. • Canalizar vía periférica y central y monitorizar presión
venosa central (PVC), para ajustar el aporte de líquidos a una PVC entre 4 y 8 cmH2O. •
REPOSICIÓN DE VOLUMEN. Si no existe contraindicación, se puede realizar una rehidratación
rápida (en unos 30 minutos) con 500-1000 ml de suero salino fisiológico, controlando la presión
arterial, la presión venosa central y vigilando la respuesta clínica y diurética. Si existe una pérdida
hemática grave se usará concentrado de hematíes. • Una vez corregida la volemia, el volumen
urinario aumenta y se debe continuar con reposición de líquidos a ritmo de diuresis. • Si la causa
que ha provocado la IRA es una disminución del volumen circulante efectivo, se aplicaran los
protocolos de tratamiento del shock, de la insuficiencia cardiaca, cirrosis hepática etc. • Si no
existe otra complicación, la restauración de la perfusión renal mejora rápidamente la IRA
prerrenal. LA CAUSA DE LA IRA EN EL PACIENTE ES LA HIPOVOLEMIA POR TANTO LO MÁS
IMPORTANTE EN ÉSTE MOMENTO SERÁ LA REPOSICIÓN DE VOLUMEN.

Bibliografía:
1. LONGO DL, FAUCI AS, KASPER DL, HAUSERSL, JAMESON JL, LOSCALZOJ. HARRISON. PRINCIPIOS DE
MEDICINA INTERNA, 18A EDICIÓN. MC GRAW HILL. NEW YORK, USA. 2012, PP 2305-2306.

FIN DEL CASO CLÍNICO SERIADO


ANÁLISIS DEL CASO CLÍNICO

IDENTIFICACIÓN DEL REACTIVO


Area: MEDICINA INTERNA
Especialidad: ENDOCRINOLOGÍA
Tema: TRANSTORNOS DEL METABOLISMO Y DE LAS VITAMINAS
Subtema: OBESIDAD

CASO CLÍNICO CON UNA PREGUNTA

MUJER DE 36 AÑOS DE EDAD CON OBESIDAD GRADO 2, DESDE HACE 3 MESES EN TRATAMIENTO
NUTRICIONAL Y ACTIVIDAD FÍSICA PROGRAMADA. ACTUALMENTE CON MEJORÍA EN CIFRAS TENSIONALES,
LÍPIDOS Y GLICEMIA SÉRICA, PERO SIN LOGRAR LA REDUCCIÓN DE PESO ESPERADA. SE DECIDE INICIAR
TRATAMIENTO FARMACOLÓGICO PARA LA REDUCCIÓN DE PESO CON ORLISTAT.

mujer de 36 años.

obesidad GRADO 2 que no mejora


signi cativamente con dieta y ejercicio.

se inicia terapia farmacológica para


reducción de peso con ORLISTAT.

233 - CORRESPONDE AL MECANISMO DE ACCIÓN DEL MEDICAMENTO INDICADO EN ESTA PACIENTE:

INHIBIDOR EN Los ANOREXIGENICOS son medicamentos que suprimen el apetito con acción a nivel central.
LA RECAPTURA Actúan en la región ventromedial y lateral del hipotálamo en el sistema nervioso central. Para
DE entender el mecanismo de acción de estos medicamentos es necesario saber que el apetito se
SEROTONINA. encuentra regulado por la intensificación de la neurotransmisión de tres monoaminas:
noradrenalina, serotonina y en menor grado, dopamina. La SIBUTRAMINA es un INHIBIDOR DE
LA RECAPTURA DE SEROTONINA, pertenece al grupo de los anorexigenicos y se diferencia de
los primeros fármacos en que no guarda relación con las anfetaminas ni causa adicción. Hasta
hace algunos años era el único inhibidor del apetito autorizado para su venta, pero desde 2010
se retiró del mercado por el incremento de infartos al miocardio no letales y aplopejía no letal
en individuos con enfermedad cardiovascular preexistente.
INHIBIDOR DE La ACARBOSA es un INHIBIDOR DE LA GLUCOSIDASA ALFA. Reduce la hiperglucemia
LA postprandial retrasando la absorción de glucosa, no afecta la utilización de glucosa ni la
GLUCOSIDASA secreción de insulina. SE UTILIZA COMO COADYUVANTE EN EL TRATAMIENTO DE LA DIABETES
ALFA. PERO NO ESPECÍFICAMENTE PARA LA OBESIDAD.

ANTAGONISTA El RIMONABANT es un ANTAGONISTA SELECTIVO DEL RECEPTOR CANNABINOIDE, el primero


SELECTIVO DEL en su clase. Se utilizó para el tratamiento de la obesidad por su efecto al suprimir el apetito. Se
RECEPTOR cree que el sistema endocanabinoide encefálico controla la ingesta de alimentos, refuerza la
CANNABINOIDE. motivación para ingerir alimentos que producen placer o alto factor estimulante y regula
algunas otras vías del apetito. Fue retirado del mercado por sus efectos adversos neurológicos y
psiquiátricos (convulsiones, depresión, ansiedad y hasta intentos suicidas).

INHIBIDOR DE El ORLISTAT es el primer medicamento autorizado para el tratamiento de la obesidad que actúa
LA LIPASA a nivel intestinal y no a nivel del sistema nervioso central. Es un INHIBIDOR potente de
INTESTINAL. reversibilidad lenta DE LA LIPASA pancreática, gástrica y de la carboxiléster y de fosfolipasa A2,
necesarias para la hidrólisis de las grasas de los alimentos convirtiéndolas en ácidos grasos y
monoacilgliceroles. Actúa tanto a nivel gástrico como intestinal. La dosis terapéutica es de
120mg tres veces al día, con lo que se logra una absorción sólo del 30% de los alimentos. Al
indicar orlistat en los pacientes obesos se debe tener en cuenta que debido a que bloquea la
absorción de grasa proveniente de la dieta, sus principales efectos adversos están relacionados
con cambios gastrointestinales tales como: heces con grasa, urgencia fecal y dolor abdominal.

Bibliografía:
1. LONGO DL, FAUCI AS, KASPER DL, HAUSERSL, JAMESON JL, LOSCALZOJ. HARRISON. PRINCIPIOS DE
MEDICINA INTERNA, 18A EDICIÓN. MC GRAW HILL. NEW YORK, USA. 2012. 2. PAPADAKIS MAXINE A,
MCPHEE STEPHEN J. DIAGNÓSTICO CLÍNICO Y TRATAMIENTO. 52ª EDICIÓN. MC GRAW HILL EDUCATION,
LANGE. USA. 2013.
ANÁLISIS DEL CASO CLÍNICO

IDENTIFICACIÓN DEL REACTIVO


Area: PEDIATRÍA
Especialidad: INFECTOLOGIA PEDIÁTRICA
Tema: HEPATITIS INFECCIOSA
Subtema: HEPATITIS B

CASO CLÍNICO CON UNA PREGUNTA

PRIMIGESTA DE 24 AÑOS DE EDAD CON ANTECEDENTE DE INFECCIÓN POR HEPATITIS B. ACTUALMENTE CON
36 SDG, REPORTÁNDOSE HACE 1 SEMANA TÍTULOS POSITIVOS DE ANTÍGENO S DE LA HEPATITIS B EN SANGRE.
SE ENCUENTRA PROGRAMADA PARA CESÁREA EN 2 SEMANAS POR PRODUCTO PÉLVICO.

Primigesta.

Infección por hepatitis B.

Títulos positivos de AgsHB.

234 - PARA REDUCIR LA PROBABILIDAD DE TRANSMISIÓN AL NEONATO, DEBERÁ INDICARSE:

INMUNOGLOBULINA El VIRUS DE LA HEPATITIS B (VHB) se transmite por vía perinatal, percutánea y por
G EN LA MADRE Y EL contacto sexual, así como por contacto estrecho persona-persona, presumiblemente
NEONATO. cuando existen heridas en piel, particularmente en niños en áreas endémicas. La
transmisión vertical del VHB se reduce en un 90% con la aplicación de la inmunoglobulina
específica para la hepatitis B.

VACUNA CONTRA La transmisión vertical ocurre en un 90% de los embarazos de madres con HBeAg positivo y
LA HEPATITIS B EN cerca del 10% de aquellas que tienen HBsAg positivo. Se considera que más del 90% de los
EL NEONATO. recién nacidos infectados llegan a ser portadores crónicos. En México la vacuna universal
contra hepatitis B en recién nacidos y lactantes está incluida en el esquema básico de
vacunación nacional, las primera dosis se aplica en las primeras 12 a 24hrs de nacido en
niños de término o prematuros de más de 2kg. EL INCLUIR LA VACUNA DE LA HEPATITIS B
EN EL ESQUEMA BÁSICO DE VACUNACIÓN ES UNA ESTRATEGIA ESPECÍFICA PARA LA
PREVENCIÓN DE LA ENFERMEDAD COMO PROFILAXIS “PREEXPOSICIÓN”.
INMUNOGLOBULINA La PROFILAXIS POSTEXPOSICIÓN está indicada en el periodo perinatal, infección sexual
G Y VACUNA aguda, exposición sexual por portador crónico, contacto domiciliario con portador crónico,
CONTRA LA contacto domiciliario con caso agudo con exposición aguda conocida de sangre positiva y
HEPATITIS B EN EL en exposición accidental de mucosas o percutánea a sangre con AgsHB. En el periodo
NEONATO. neonatal, si la madre es portadora de AgsHB o AgeHB, la transmisión vertical puede
evitarse hasta en un 95% con la inmunoprofilaxis activa y pasiva concurrente en el neonato
con inmunoglobulina específica y vacuna contra el VHB. Después de lo cual se deberá
completar el esquema normal de vacunación en el niño. La administración de la
inmunoglobulina G específica previene el estado de portador crónico de AgsHB en lactantes
nacidas de madres portadoras de dicho antígeno.

INMUNOGLOBULINA Los antivirales no han resultado ser eficaces para el tratamiento ni para la profilaxis de las
G ESPECÍFICA Y hepatitis de tipo viral. IMPORTANTE: LA ALIMENTACIÓN AL SENO MATERNO NO
ANTIVIRALES. AUMENTA LA POSIBILIDAD DE ADQUIRIR INFECCIÓN POR EL VHB.

Bibliografía:
1. GUÍA DE PRÁCTICA CLÍNICA, DIAGNÓSTICO Y TRATAMIENTO DE LA INFECCIÓN CRÓNICA POR VIRUS DE
LA HEPATISIS B. MÉXICO: SECRETARIA DE SALUD; 2009. 2. GONZÁLEZ N, TORALES A, GÓMEZ D.
INFECTOLOGÍA CLÍNICA PEDIÁTRICA, 8A EDICIÓN. MC GRAW HILL. MÉXICO, 2011. 3. NORMA OFICIAL
MEXICANA NOM-036-SSA2-2012, PREVENCIÓN Y CONTROL DE ENFERMEDADES. APLICACIÓN DE
VACUNAS, TOXOIDES, FABOTERÁPICOS (SUEROS) E INMUNOGLOBULINAS EN EL HUMANO.

http://www.cenetec.salud.gob.mx /descargas/gpc/CatalogoMaestro/419_GPC_Diagnostico_tratamiento_HEPATITIS_POR_VIRUS_B/GPC_Hepatitis_B.pdf
Salir

FEMENINA DE 7 AÑOS DE EDAD, ACUDE AL CENTRO DE SALUD POR PRESENTAR


DE UNA SEMANA A LA FECHA ASTENIA, ADINAMIA, HIPOREXIA, Y EL DÍA DE HOY
SE AGREGA ICTERICIA EN CONJUNTIVAS Y TEGUMENTOS, FIEBRE NO
CUANTIFICADA, EVACUACIONES DISMINUIDAS DE CONSISTENCIA CON MOCO SIN
SANGRE Y LA ORINA OBSCURA.

237 - CON EL CUADRO CLÍNICO PRESENTE EN ESTA FEMENINA, EL DIAGNOSTICO


MÁS PROBABLE ES:

SÍNDROME URÉMICO HEMOLÍTICO


SHIGELOSIS
SALMONELOSIS
HEPATITIS
Bibliografía:
PRINCIPLES AND PRACTICE OF PEDIATRIC INFECTIOUS DESIASES. SARAH S. LONG. CHURCHILL,
LIVINGSTONE. EDICIÓN 2A. 2002. PÁG. 1188-1192.

SEGURA DUDA NI IDEA

238 - EN ESTE PADECIMIENTO CUAL SERÍA EL AGENTE ETIOLÓGICO MÁS


FRECUENTE:

SHIGELLA SONNEI
VIRUS DE HEPATITIS A
SALMONELLA NO TYPHI
E. COLI

Bibliografía:
PRINCIPLES AND PRACTICE OF PEDIATRIC INFECTIOUS DESIASES. SARAH S. LONG. CHURCHILL,
LIVINGSTONE. EDICIÓN 2A. 2002. PÁG. 1188-1192.

SEGURA DUDA NI IDEA


ANÁLISIS DEL CASO CLÍNICO

IDENTIFICACIÓN DEL REACTIVO


Area: PEDIATRÍA
Especialidad: INFECTOLOGIA PEDIÁTRICA
Tema: HEPATITIS INFECCIOSA
Subtema: HEPATITIS A

CASO CLÍNICO SERIADO

FEMENINA DE 7 AÑOS DE EDAD, ACUDE AL CENTRO DE SALUD POR PRESENTAR DE UNA SEMANA A LA
FECHA ASTENIA, ADINAMIA, HIPOREXIA, Y EL DÍA DE HOY SE AGREGA ICTERICIA EN CONJUNTIVAS Y
TEGUMENTOS, FIEBRE NO CUANTIFICADA, EVACUACIONES DISMINUIDAS DE CONSISTENCIA CON MOCO SIN
SANGRE Y LA ORINA OBSCURA.

Femenina de 7 años de edad

Una semana de evolución, caracterizado


por astenia, adinamia e hiporexia

Fiebre, ictericia y orina oscura (Coluria).


Cuadro compatible con hepatitis

237 - CON EL CUADRO CLÍNICO PRESENTE EN ESTA FEMENINA, EL DIAGNOSTICO MÁS PROBABLE ES:
SÍNDROME Secuelas tardías de la infección por cepas de E. Coli productoras de toxina Shiga (STEC). El
URÉMICO síndrome hemolítico-urémico es una secuela grave de la infección entérica por STEC,
HEMOLÍTICO especialmente por E. Coli O157:H7. El síndrome mencionado se define por la tríada de anemia
hemolítica microangiopática, trombocitopenia y disfunción renal aguda. En muchos niños con
diarrea causada por E. Coli O157:H7, surgen en las dos semanas siguientes al comienzo de la
diarrea, cambios hematológicos microangiopáticos leves que ceden por sí solos;
trombocitopenia, nefropatía o la tríada de conjunto. La púrpura trombocitopénica, aparece en
adultos, puede surgir después de la infección por STEC e incluye ataque del sistema nervioso
central y fiebre; puede tener un comienzo más gradual que HUS y es parte de un espectro
patológico que a menudo ha sido llamado TTP-HUS. Muchos casos de HUS en niños en Estados
Unidos son causados por E. Coli O157:H7, pero gran parte de los casos de TTP en los adultos no
tienen causa conocida. Como se describe, "normalmente aparece dos semanas después del
comienzo del cuadro enteral, que no presenta nuestro paciente". La microangiopatía causa la
trombocitopenia y la insuficiencia renal aguda, además de hemolisis, lo que da lugar a la
ictericia.

SHIGELOSIS Shigella, infecciones. MANIFESTACIONES CLÍNICAS: Las especies de Shigella infectan más bien el
colon y originan manifestaciones clínicas que van desde la expulsión de excrementos acuosos o
laxos con síntomas generales mínimos o sin ellos, hasta un cuadro más grave que incluye fiebre,
cólicos o dolor abdominales, tenesmo y excrementos mucoides con sangre o sin ella. El cuadro
inicial varía con cada especie de Shigella; los niños atacados con Shigella Sonnei por lo común
presentan diarrea acuosa, los afectados por Shigella Flexneri, Boydii o Dysenteriae típicamente
muestran diarrea sanguinolenta y síntomas generales graves. Entre las complicaciones raras
están bacteriemia, síndrome de Reiter (después de infección por S. Flexneri), síndrome
hemolítico-urémico (Después de infección por S. Dysenteriae tipo 1), megacolon tóxico y
perforación y encefalopatía tóxica (Síndrome de Ekiri). "En general, para estos casos, el cuadro
clínico se circunscribe a nivel intestinal".

SALMONELOSIS Salmonella, infecciones. (Salmonelosis) MANIFESTACIONES CLÍNICAS: Los microorganismos del


género salmonella no tifoideos ocasionan el estado de portador asintomático, gastroenteritis,
bacteriemia e infecciones focales (Como meningitis y osteomielitis). Las categorías patológicas
mencionadas no son mutuamente excluyentes, sino que representan todo un espectro de
enfermedades. El cuadro más común que acompaña a la infección por Salmonella no tifoidea es
la gastroenteritis en que las manifestaciones más frecuentes son diarrea, cólicos abdominales y
dolor a la palpación en el vientre y fiebre. El sitio de infección por lo regular es el intestino
delgado, aunque puede haber colitis. Surge a veces bacteriemia sostenida o intermitente y se
identifican infecciones focales, incluso en l0% de sujetos con bacteriemia proveniente de la
infección por Salmonella. En la Salmonella serotipo Typhi y otros serotipos de este
microorganismo pueden causar un cuadro bacteriémico tardío y de duración larga que se conoce
como fiebre intestinal o tifoidea. El cuadro comienza de manera gradual, con manifestaciones
como fiebre, síntomas generalizados (Cefalea, malestar general, anorexia y letargia), dolor
espontáneo y a la palpación en el vientre, hepatomegalia y esplenomegalia, “Manchas color de
rosa” y cambios en el estado psíquico. La fiebre intestinal puede ser un cuadro febril leve
impreciso en niños de corta edad en que a veces hay bacteriemia sostenida o intermitente. El
estreñimiento puede ser una de las primeras manifestaciones. En los niños a menudo surge
diarrea. La bacteriemia recurrente por Salmonella es un cuadro definitorio del síndrome de
inmunodeficiencia adquirida (SIDA) en adolescentes y adultos infectados por el virus de
inmunodeficiencia humana (VIH). "En este caso la presencia de coluria e ictericia descartan esta
respuesta".

HEPATITIS Hepatitis A. MANIFESTACIONES CLÍNICAS: De manera característica, la infección por el virus de


hepatitis A (Hepatitis A virus, HAV) incluye un cuadro agudo que cede por sí solo, compuesto de
fiebre, malestar generalizado, ictericia, anorexia y náuseas. La infección sintomática por dicho
virus se observa en cerca de 30 % de niños infectados que tienen menos de seis años de vida;
pocos de ellos mostrarán ictericia. En niños de mayor edad y adultos, la infección casi siempre es
sintomática y se prolonga algunas semanas; en 70 % de los casos, en promedio, aparece ictericia.
Se observan a veces casos de enfermedad prolongada o recidivante que puede durar incluso seis
meses. La hepatitis fulminante es rara, pero es más frecuente en sujetos con alguna hepatopatía
primaria. No se observa infección crónica. La coluria y acolia son datos característicos por la
inflamación hepática que altera la excreción entero hepática de la bilirrubina.

Bibliografía:
PRINCIPLES AND PRACTICE OF PEDIATRIC INFECTIOUS DESIASES. SARAH S. LONG. CHURCHILL,
LIVINGSTONE. EDICIÓN 2A. 2002. PÁG. 1188-1192.
238 - EN ESTE PADECIMIENTO CUAL SERÍA EL AGENTE ETIOLÓGICO MÁS FRECUENTE:

SHIGELLA SHIGELOSIS. CAUSAS: Las especies de Shigella son bacilos Gram Negativos de la familia
SONNEI Enterobacteriaceae. Se han identificado cuatro especies con más de 40 serotipos. Entre las shigellas
aisladas y señaladas en Estados Unidos de 1989 a 2000, 78% correspondieron a la especie S. Sonnei,
19 % a S. Flexneri, 2 %, S. Boydii, y 1%. S. Dysenteriae. Esta última variedad de Shigella es rara en
Estados Unidos pero muy difundida en zonas rurales de África y el subcontinente indio. ASPECTOS
EPIDEMIOLÓGICOS: Los seres humanos son los hospedadores naturales de Shigella, aunque pueden
ser infectados otros primates. El mecanismo principal de transmisión es la vía fecal-oral. Están
expuestos a un mayor peligro de infección los niños de cinco años o menores que acuden a
instituciones de cuidado infantil, quienes los cuidan y otras personas que viven en apiñamiento. El
hecho de viajar a países de escasos recursos y con sanidad inadecuada puede hacer que el viajero
quede expuesto al peligro de infección. La transmisión requiere que haya apenas 10 a 200
microorganismos para que ocurra la infección. Otras formas de transmisión incluyen ingestión de
alimentos o agua contaminados, contacto con un objeto inanimado también contaminado y
contacto sexual. Las moscas domésticas también pueden ser vectores por el transporte físico de
excrementos infectados. Las infecciones por S. Flexneri, S. Boydii y S. Dysenteriae son más comunes
en niños de mayor edad y en adultos, y suelen provenir de fuentes por fuera de Estados Unidos de
América. La transmisión se produce todo el tiempo en que esté el microorganismo en los
excrementos. Incluso sin terapia antimicrobiana el estado de portador suele cesar en término de
cuatro semanas de haber comenzado la enfermedad, y es raro que el estado crónico dure más de
un año. El periodo de incubación varía de uno a siete días, pero de manera típica es de dos a cuatro
días.

VIRUS DE HEPATITIS A. CAUSAS: La partícula infectante es un virus de RNA clasificado como miembro del
HEPATITIS A grupo Picornavirus. ASPECTOS EPIDEMIOLÓGICOS: El mecanismo común de transmisión es el de
persona a persona, que es consecuencia de la contaminación fecal y la ingestión del patógeno (La
vía fecal-oral). La edad en que se produce la infección varía con el estado socioeconómico y las
condiciones de vida. En países en desarrollo donde la infección es endémica, casi todos los
individuos se infectan en el primer decenio de su vida. La cifra más alta de ataque se observó en
niños de cinco a 14 años, y las más bajas, en adultos mayores de 40 años. En los últimos decenios,
los casos notificados de infección por hepatitis A; han seguido una distribución geográfica
desigual, es decir, las cifras más altas se han observado en un número preciso de estados y
comunidades. Entre los casos de infección de hepatitis A notificados a los CDC, las fuentes de
infección identificadas incluyen contacto personal muy cercano con una persona infectada con el
virus de hepatitis A, contactos domésticos o personales en un centro de atención pediátrico, viajes
internacionales, un brote en que se sabe que el germen fue transmitido por alimentos o agua,
actividad homosexual en varones y el uso de drogas inyectables. Rara vez la partícula se transmite
por transfusión de sangre, o de la madre a su recién nacido (Es decir, transmisión vertical). En
contadas ocasiones la infección ha provenido de primates no humanos y que no nacieron en
cautiverio. En cerca de la mitad de los casos notificados es imposible detectar su origen. Es
probable que muchos de estos casos de origen desconocido hayan provenido del contagio fecal-
oral con personas que tenían infecciones asintomáticas y en particular niños de corta edad. Casi
todos los casos de infección y enfermedad por HAV surgen en el contexto de epidemias
comunitarias, en que la infección predominantemente se transmite en núcleos caseros y familias
“Extendidas”. Se sabe de brotes de origen común transmitidos por alimentos; los transmitidos por
agua son raros. Pocas veces hay transmisión nosocomial, pero se han señalado casos de brotes
surgidos por transmisión de pacientes hospitalizados a profesionales asistenciales. Además, se ha
sabido de brotes en unidades de cuidado intensivo de neonatos, de recién nacidos infectados por
sangre transfundida y que más adelante transmitieron el virus a otros neonatos y al personal de la
unidad. En centros de atención pediátrica, la enfermedad sintomática reconocida (ictérica) aparece
más bien en los contactos adultos de niños. Casi todos los niños infectados en centros pediátricos
son asintomáticos o tienen manifestaciones inespecíficas. Por tal razón, antes de que se identifique
el primer caso (caso índice) suele propagarse la infección por el virus dentro y fuera del centro
mencionado. Los brotes ocurren más a menudo en grandes centros de ese tipo y los que atienden
niños en pañales. En la mayoría de personas infectadas, en la primera a segunda semanas antes de
que comience el cuadro clínico, se advierten los máximos títulos de HAV en las heces, fase en que
los pacientes tienen más probabilidades de transmitir el virus. Más adelante disminuye el peligro
de transmisión y llega a su mínimo una semana después de que comenzó la ictericia. Sin embargo,
es posible detectar el virus en los excrementos por lapsos más largos, especialmente en neonatos y
niños de corta edad. El periodo de incubación es de 15 a 50 días, con un promedio de 25 a 30 días.
SALMONELLA SALMONELOSIS. CAUSAS: Las salmonellas son bacilos Gram Negativos que pertenecen a la familia
NO TYPHI Enterobacteriaceae. En la actualidad se conocen más de 2 460 serotipos; muchos de los que son
patógenos para el ser humano se dividen en los grupos antigénicos O, que van de la A a E.
Salmonella serotipo Typhi se clasifica en el serogrupo D. En el año 2000, los microorganismos
aislados con mayor frecuencia y señalados en los seres humanos de Estados Unidos de América
fueron Salmonella, serotipos Typhimurium (Serogrupo B), Enteritidis (D). ASPECTOS
EPIDEMIOLÓGICOS: Los reservorios principales de las salmonelas no tifoídicas son animales que
incluyen aves de corral, ganado, reptiles y mascotas. Los vehículos principales de la transmisión son
alimentos de origen animal, como la carne de aves de corral, reces, pescados, huevos y productos
lácteos. Se ha dicho que en algunos brotes intervienen otros alimentos como frutas, verduras y
productos de panadería; los alimentos en cuestión por lo común quedaron contaminados por
contacto con un producto animal, o a veces por un ser humano infectado. Otros mecanismos de
transmisión han incluido la ingestión de agua contaminada; el contacto con reptiles infectados
(Como tortuguitas, iguanas, lagartijas, serpientes) y la exposición a medicamentos contaminados,
colorantes e instrumentos médicos. A diferencia de los serotipos no tifoídicos, S. serotipo Typhi
aparece sólo en seres humanos y su presencia denota contacto directo con una persona infectada o
con un objeto contaminado por un portador. La fiebre tifoidea, a pesar de que es poco común en
Estados Unidos (Unos 400 casos al año) es endémica en muchos países. En consecuencia, las
infecciones de ese tipo en dicho país por lo común fueron producto de contagio durante viajes a
otras naciones. Las cifras de ataque de infección por Salmonella, específica de cada grupo de edad,
alcanzan su máximo en personas menores de cuatro años, con una máxima o pico en los primeros
meses de vida.

E. COLI SÍNDROME URÉMICO HEMOLÍTICO. CAUSAS: Cada tipo patológico de E. Coli posee características
específicas de virulencia, algunas codificadas en plásmidos específicos para cada tipo patológico.
Cada tipo patológico posee un conjunto peculiar de antígenos somáticos (O) y flagelares (H). Las
características patógenas son: • La enfermedad causada por E. Coli O157:H7 sigue una evolución
bifásica. La fase intestinal se caracteriza por la formación de la lesión de “Adherencia” y
borramiento, con lo cual surge diarrea secretoria. Después de la fase mencionada sigue la
elaboración de la toxina Shiga, citotóxina potente que también aparece en Shigella Dysenteriae 1)
La acción de la toxina Shiga en células de intestino origina colitis hemorrágica, y la absorción de la
toxina en la circulación causa complicaciones sistémicas, como HUS y secuelas neurológicas. • Las
cepas de EPEC se adhieren a la mucosa del intestino delgado, y a semejanza de E. Coli O157:H7,
originan lesiones “Adherentes” y de borramiento. Las cepas mencionadas históricamente fueron
definidas como miembros de serotipos específicos de E. Coli a los que se achacaron desde el punto
de vista epidemiológico, ser causas de diarrea de niños; en la actualidad una definición patógena
más exacta incluye la capacidad de formar las lesiones de “Adherencia y borramiento” sin que
exista la producción de toxina Shiga. • Las cepas de ETEC colonizan el intestino delgado sin
invadirlo y producen enterotoxinas termolábil o termoestable, o ambas. Las cepas que originan la
enterotoxina termoestable son las que causan casi todos los casos de enfermedad en seres
humanos. • De manera típica, E. Coli enteroinvasora, a semejanza de las especies de Shigella, no
fermentan la lactosa e invaden la mucosa del colon, donde se propagan en sentido lateral e
inducen una respuesta inflamatoria local. • E. Coli enteroagregada ha sido definida por su perfil de
adherencia característico en “pilas de ladrillos” en métodos basados en cultivos celulares. Estos
microorganismos elaboran una o más enterotoxinas y desencadenan daño en la mucosa intestinal.
• Son las que causan casi todos los casos de enfermedad en seres humanos.

Bibliografía:
PRINCIPLES AND PRACTICE OF PEDIATRIC INFECTIOUS DESIASES. SARAH S. LONG. CHURCHILL,
LIVINGSTONE. EDICIÓN 2A. 2002. PÁG. 1188-1192.

FIN DEL CASO CLÍNICO SERIADO


ANÁLISIS DEL CASO CLÍNICO

IDENTIFICACIÓN DEL REACTIVO


Area: PEDIATRÍA
Especialidad: INFECTOLOGIA PEDIÁTRICA
Tema: INFECCIONES NEONATALES
Subtema: ONFALITIS NEONATAL

CASO CLÍNICO SERIADO

RECIÉN NACIDO MASCULINO PRODUCTO DE SEGUNDA GESTA, NACIDO VÍA PARTO EN SU DOMICILIO A LAS 37
SDG TRAS RUPTURA PREMATURA DE MEMBRANAS. ACTUALMENTE EN SU 4º DÍA DE VIDA. ES LLEVADO A LA
UNIDAD DE SALUD DE LA COMUNIDAD POR PRESENCIA DE IRRITABILIDAD Y RECHAZO A LA VÍA ORAL.
DURANTE LA EXPLORACIÓN FÍSICA SE OBSERVA ERITEMA PERIUMBILICAL DE APROXIMADAMENTE 7MM CON
ESCASA SECRECIÓN PURULENTA, LOS SIGNOS VITALES DEL NIÑO SE ENCUENTRAN ESTABLES.

RN.

irritabilidad y rechazo al alimento.

eritema periumbilical de “7mm” con escasa


secreción purulenta.

239 - ES EL PRINCIPAL FACTOR DE RIESGO QUE PUDO CONTRIBUIR A LA ENFERMEDAD DEL PACIENTE:

PREMATUREZ LO PRIMERO QUE DEBERÁS HACER ES DETERMINAR EL DIAGNÓSTICO CLÍNICO QUE EN ESTE
CASO SE TRATA DE UNA ONFALITIS. La ONFALITIS corresponde a la infección del muñón
umbilical en el recién nacido. Se caracteriza por edema peri-umbilical, eritema e hipersensibilidad
con o sin secreción de olor fétido. Con base en la extensión de la infección se reconocen tres
categorías clínicas: 1. Solo descarga purulenta. 2. Celulitis y linfangitis de la pared abdominal. 3.
Extensión de la inflamación a tejido celular subcutáneo y fascia profunda. Los signos clínicos que
acompañan a esta patología son: fiebre (>38°C) o hipotermia (<36°C), temperatura inestable,
ictericia, taquicardia, llenado capilar retardado, dificultad respiratoria o apnea, distensión
abdominal con ruidos intestinales ausentes, irritabilidad, letargo, mala succión, hipotonía o
hipertonía. LA PREMATUREZ Y EL BAJO PESO AL NACER SON FACTORES DE RIESGO PARA LA
PRESENCIA DE ONFALITIS, PERO NO CORRESPONDEN AL PRINCIPAL FACTOR DE RIESGO.
PARTO NO Los siguientes son factores de riesgo para el desarrollo de ONFALITIS: - Manipulación inadecuada
ESTÉRIL del cordón umbilical. - Aplicación en el cordón umbilical por costumbres culturales de sustancias
no apropiadas (talco, aceite de palma, ceniza de tabaco, agua, leche materna, saliva, semillas,
especias, aceite de mostaza, etc.). - Ruptura prolongada de membranas. - Infección materna. -
Parto no estéril. - Prematurez. - Neonatos con sistemas inmunes débiles o deficiente (síndrome de
adhesión leucocitaria deficiente o LAD). - Neonatos hospitalizados sometidos a procedimiento
invasivos (cateterización umbilical). - Bajo peso al nacimiento. EL PARTO NO ESTÉRIL ES UN
FACTOR DE RIESGO PARA QUE EL RECIÉN NACIDO PRESENTE ONFALITIS, PERO NO EL PRINCIPAL.

MALAS SE CONSIDERA EL PRINCIPAL FACTOR DE RIESGO RESPONSABLE DE LA ONFALITIS SON LAS


PRÁCTICAS PRÁCTICAS NO HIGIÉNICAS DEL CORDÓN UMBILICAL.
HIGIÉNICAS
DEL CORDÓN
UMBILICAL

RUPTURA Debe considerarse una RUPTURA PROLONGADA DE MEMBRANAS cuando se produce en más de
PROLONGADA 18 horas antes de iniciarse el trabajo de parto. SE CONSIDERA UN FACTOR DE RIESGO PARA
DE ONFALITIS, PERO NO ES EL MÁS IMPORTANTE.
MEMBRANAS

Bibliografía:

http://www.cenetec-difusion.com/CMGPC/SS-483-11/ER.pdf

240 - ES EL AGENTE ETIOLÓGICO MÁS FRECUENTEMENTE ASOCIADO A ESTA PATOLOGÍA:

STAPHYLOCOCCUS Después del nacimiento, la piel del RN, incluyendo el muñón umbilical es colonizado
COAGULASA principalmente por bacterias no patógenas tales como el Staphylococcus coagulasa negativo
NEGATIVO y bacilos Diphtheroids. EL STAPHYLOCOCCUS COAGULASA NEGATIVO FORMA PARTE DE LA
FLORA NORMAL DE LA PIEL DEL RECIÉN NACIDO.

STAPHYLOCOCCUS Los organismos aeróbicos son los causantes más frecuentes, dentro de los que se incluyen -
AUREUS Staphylococcus aureus (EL MÁS COMÚN). - Staphylococcus epidermidis. - Streptococcus del
grupo A. - Escherihia coli. - Proteus. EL STAPHYLOCOCCUS AUREUS ES EL AGENTE CAUSAL
MÁS COMÚN DE ONFALITIS.

CLOSTRIDIUM Más de un tercio de los casos de onfalitis están asociados a infecciones anaeróbicas causadas
PERFRINGENS por: - Bacteroides fragilis. - Peptostreptococcus. - Clostridium perfringens. LOS AEROBIOS
SON LOS MICROORGANISMOS QUE MÁS A MENUDO PRODUCEN ONFALITIS, POR TAL
MOTIVO SE DESCARTA ESTA OPCIÓN.

STREPTOCOCCUS Las bacterias patógenas como Coliforme y Streptococcus también pueden estar presentes en
SP la piel del recién nacido pudiendo producir infección del muñón umbilical. LOS
ESTREPTOCOCOS PUEDEN INFECTAR EL MUÑÓN UMBILICAL PERO NO SON LA PRINCIPAL
CAUSA.

Bibliografía:

http://www.cenetec-difusion.com/CMGPC/SS-483-11/ER.pdf

241 - CON BASE EN EL CUADRO CLÍNICO EL MANEJO DEBERÁ CONSISTIR EN:

ANTIBIÓTICO Con base en la GPC el manejo del recién nacido deberá ser tópico, en casos leves sin
ORAL complicaciones; o intravenosos en aquellos que se consideren de riesgo para complicación o con
datos francos de complicación. LA GPC NO CONSIDERA LOS ANTIBIÓTICOS ORALES COMO
PARTE DEL MANEJO DE LA ONFALITIS, esto muy probablemente por la gravedad de las
complicaciones y la vulnerabilidad del recién nacido.
ANTIBIÓTICO En onfalitis con área menor de 5 milímetros periumbilical con “datos sistémicos”, se deberá
INTRAVENOSO comenzar a dar tratamiento intravenoso y derivar a un segundo nivel de atención para la
valoración por un especialista. Ante la presencia de datos de complicación se recomienda
siempre el uso de antimicrobianos intravenosos los cuales incluyen: aminoglucósidos,
clindamicina, cefotaxima, metronidazol e imipenem.

ANTIBIÓTICO En onfalitis con área menor a 5 milímetros periumbilical sin datos sistémicos, se recomienda dar
LOCAL + tratamiento local y medidas higiénico-dietéticas. Cuando existe onfalitis “sin signos de
MEDIDAS complicación” solo se aplicará tratamiento local del tipo ungüento de neomicina o de
HIGIÉNICAS mupirocina.

ANTIBIÓTICO En onfalitis mayor de 5 milímetros periumbilical con o sin datos sistémicos, se debe iniciar
INTRAVENOSO tratamiento intravenoso y derivar URGENTE a segundo o tercer nivel de atención.
+ ATENCIÓN
ESPECIALIZADA
URGENTE

Bibliografía:

http://www.cenetec-difusion.com/CMGPC/SS-483-11/ER.pdf

FIN DEL CASO CLÍNICO SERIADO


ANÁLISIS DEL CASO CLÍNICO

IDENTIFICACIÓN DEL REACTIVO


Area: MEDICINA INTERNA
Especialidad: NEUMOLOGÍA
Tema: ENFERMEDADES PULMONARES OBSTRUCTIVAS Y RESTRICTIVAS
Subtema: INTOXICACIONES RESPIRATORIAS

CASO CLÍNICO CON UNA PREGUNTA

MUJER DE 55 AÑOS QUE ES RESCATADA DE SU CASA QUE SE ENCONTRABA INCENDIÁNDOSE. NO SE


ENCUENTRAN QUEMADURAS PERO SI UNA GRAVE ALTERACIÓN DEL ESTADO DE ALERTA. USTED SOSPECHA
INTOXICACIÓN POR MONÓXIDO DE CARBONO.

MUJER DE 55 AÑOS.

rESCATADA DE SU CASA QUE SE


ENCONTRABA INCENDIáNDOSE.

-.

GRAVE ALTERACIÓN DEL ESTADO DE


ALERTA. SOSPECHA INTOXICACIÓN POR
MONóXIDO DE CARBONO.

-.

242 - EL MECANISMO FISIOPATOLÓGICO PRINCIPAL POR EL CUAL SE PRODUCE LA INTOXICACIÓN POR


ESTE COMPUESTO SE EXPLICA POR:

LESIÓN REPASO INTOXICACIÓN POR MONOXIDO DE CARBONO. El monóxido de carbono (CO) es


CELULAR incoloro, inodoro, insípido, no irritante, lo cual facilita el proceso de intoxicación debido a que
DIRECTA. no despierta fenómenos de alergia que le permitan al paciente crear conciencia de la presencia
del tóxico. Es producido por la combustión incompleta de algún material que contenga carbono.
La exposición humana incluye la inhalación de humo en los incendios, el exosto de los
automóviles, pobre ventilación al contacto con carbono, kerosene o gas de estufas, hornos o
calderas y el hábito del cigarrillo. Se calcula que quien fuma dos paquetes de cigarrillos por día,
tiene un nivel promedio de carboxihemoglobina de 5,9%. Un cigarrillo produce entre 40-100 mL
de monóxido de carbono, por lo que varias cajetillas pueden elevar esta cifra hasta 1-2 litros.
MAYOR FISIOPATOLOGÍA DE LA INTOXICACIÓN POR MONÓXIDO DE CARBONO. El monóxido de
AFINIDAD A LA carbono se une a la hemoglobina con una afinidad 250 veces mayor que el oxígeno, lo cual
HEMOGLOBINA. resulta en una saturación reducida de la oxihemoglobina y en disminución de la capacidad de
transportar oxígeno a los tejidos. Además, la curva de disociación de la oxihemoglobina es
desplazada hacia la "izquierda". Las personas que sufren de anemia corren un peligro mayor de
intoxicarse, pues el monóxido de carbono impide la liberación del oxígeno de la oxihemoglobina
no alterada. El monóxido de carbono puede inhibir directamente la citocromo oxidasa. La
toxicidad es consecuencia de hipoxia celular e isquemia, por lo que no importa el peso corporal
de la persona que esté expuesta, y tampoco el número de personas presentes, sino que cada uno
de ellos está igualmente expuesto al riesgo. DOSIS TÓXICA La concentración máxima permitida
en los sitios de trabajo para el monóxido de carbono debe ser de 25 ppm con un tiempo de
trabajo promedio de ocho horas. Los niveles considerados como inmediatamente dañinos son de
1.200 ppm en adelante. Varios minutos de exposición a 1000 ppm (0.1%) pueden resultar en
50% de saturación de la carboxihemoglobina. EL MONÓXIDO DE CARBONO TIENE UNA MAYOR
AFINIDAD POR LA HEMOGLOBINA QUE EL OXÍGENO; PUEDE SUSTITUIR A ESTE ÚLTIMO Y
DISMINUIR SU APORTE.

INCREMENTO CUADRO CLÍNICO. Los síntomas de intoxicación se originan predominantemente en los órganos
DE RADICALES con alto consumo de oxígeno como el cerebro y el corazón. El proceso de intoxicación puede
LIBRES DE simular cualquiera de las encefalopatías conocidas. La mayoría de los afectados presenta cefalea
OXIGENO. con sensación de pulsación de las arterias temporales, mareo, náuseas y vómito. Pueden ocurrir
fenómenos sensoriales auditivos y visuales. Los pacientes con enfermedad coronaria pueden
presentar angina o infarto del miocardio. DIAGNÓSTICO. El diagnóstico no es difícil si hay una
historia de exposición, por ejemplo en individuos encontrados en un garaje cerrado con un
motor de auto funcionando. Aunque no son signos específicos, se observa coloración rojo-cereza
de la piel, o la sangre venosa de color rojo brillante. La máquina de gases arteriales sanguíneos
mide la presión parcial de oxígeno disuelta en plasma (PO2), pero la saturación de oxígeno es
calculada a partir de la PO2 y por tanto no es evaluable en los pacientes con intoxicación por
monóxido de carbono. La oximetría de pulso también puede dar resultados falsos positivos,
porque no es capaz de distinguir entre la oxihemoglobina y la carboxihemoglobina. NIVELES
ESPECÍFICOS Es necesario obtener una concentración específica de carboxihemoglobina.

DESVIACIÓN DE TRATAMIENTO. MEDIDAS DE EMERGENCIA Y SOPORTE. • Mantener la vía aérea y la ventilación.


LA CURVA DE Si también ha ocurrido inhalación de humo, considerar la intubación rápida para proteger la vía
HEMOGLOBINA aérea. • Líquidos endovenosos y corrección de hipotensión, que se presenta con alta frecuencia
A LA DERECHA. en este tipo de intoxicación. • Tratar el coma y las convulsiones. • Monitorización continua
electrocardiográfica por varias horas después de la exposición. • Debido a que el humo contiene
a veces algunos otros gases tóxicos, considerar la posibilidad de intoxicación por cianuro,
presencia de metahemoglobinemia y daño por gases irritantes. DROGAS ESPECÍFICAS •
Administrar oxígeno en la concentración más alta posible (100%). Oxígeno aspirado al 100%
tiene una velocidad de eliminación de CO a partir de la hemoglobina de aproximadamente una
hora, comparado con seis horas en habitación aireada. Usar la máscara o el flujo de oxígeno alto
con reservorio o administrar el oxígeno por tubo endotraqueal. El tratamiento se lleva hasta
alcanzar niveles de carboxihemoglobina menores de 5%.

Bibliografía:
1. GUÍA DE REFERENCIA RÁPIDA, DIAGNÓSTICO Y TRATAMIENTO DE LA INTOXICACIÓN AGUDA POR
MONÓXIDO DE CARBONO. MÉXICO: SECRETARIA DE SALUD, 2011. 2. GUÍA DE PRÁCTICA CLÍNICA,
DIAGNÓSTICO Y TRATAMIENTO DE LA INTOXICACIÓN AGUDA POR MONÓXIDO DE CARBONO. MÉXICO:
SECRETARIA DE SALUD, 2011. 3. LONGO DL, FAUCI AS, KASPER DL, HAUSERSL, JAMESON JL, LOSCALZOJ.
HARRISON. PRINCIPIOS DE MEDICINA INTERNA, 18A EDICIÓN. MC GRAW HILL. NEW YORK, USA. 2012, PP
860.

http://www.cenetec.salud.gob.mx /descargas/gpc/CatalogoMaestro/487_GPC_IntoxicacixnMonxxidoCarbono/GPC_Intoxicacion_Aguda_por_CO_2011.pdf
Salir

MUJER DE 26 AÑOS DE EDAD SIN ANTECEDENTES DE IMPORTANCIA. ES LLEVADA


AL SERVICIO DE URGENCIAS PORQUE MIENTRAS SE ENCONTRABA EN UNA
FIESTA DIO 5 FUMADAS A UN CIGARRO DE MARIHUANA PRESENTANDO DE
INMEDIATO MALESTAR GENERAL, MAREO Y COMENZÓ A DECIR COSAS "RARAS"
COMO QUE ALGO MALO LE PASARÍA, MOSTRÁNDOSE A SU VEZ TEMEROSA DE
SER APRENDIDA POR LA POLICÍA. A LA EXPLORACIÓN SE ENCUENTRA
DESHIDRATADA, CON TAQUICARDIA Y CON PUPILAS DILATADAS. TRES HORAS
DESPUÉS LA PACIENTE SE ENCUENTRA ESTABLE SIN ALTERACIONES
APARENTES.

243 - EL DIAGNÓSTICO CLÍNICO MÁS PROBABLE ES:


TRASTORNO BIPOLAR.
TRASTORNO PSICÓTICO INDUCIDO POR DROGAS Y ALCOHOL.
INTOXICACIÓN POR CANNABIS.
ATAQUE DE PÁNICO E HISTERIA CONVERSIVA.

Bibliografía:
1. LONGO DL, FAUCI AS, KASPER DL, HAUSERSL, JAMESON JL, LOSCALZOJ. HARRISON. PRINCIPIOS DE
MEDICINA INTERNA, 18A EDICIÓN. MC GRAW HILL. NEW YORK, USA. 2012, PP 3557-3558. 2. PAPADAKIS MAXINE
A, MCPHEE STEPHEN J. DIAGNÓSTICO CLÍNICO Y TRATAMIENTO. 52ª EDICIÓN. NUEVA YORK. 2013, PP 1084-
1085.

SEGURA DUDA NI IDEA

244 - EL MANEJO INDICADO EN ESTE CASO ES CON:

ANTIPSICÓTICO.
ANSIOLÍTICO.
ANTIDEPRESIVO.
PSICOTERAPIA.

Bibliografía:
1. GUÍA DE PRÁCTICA CLÍNICA, PREVENCIÓN, DETECCIÓN Y CONSEJERÍA EN ADICCIONES PARA
ADOLESCENTES Y ADULTOS EN EL PRIMER NIVEL DE ATENCIÓN. MÉXICO: SECRETARIA DE SALUD,
26/MARZO/2015. 2. PAPADAKIS MAXINE A, MCPHEE STEPHEN J. DIAGNÓSTICO CLÍNICO Y TRATAMIENTO. 52ª
EDICIÓN. NUEVA YORK. 2013, PP 1085.

http://www.cenetec.salud.gob.mx/descargas/gpc/CatalogoMaestro/023_GPC_Adicciones1erNivel/SSA_023_08_EyR.pdf

SEGURA DUDA NI IDEA


Salir

LACTANTE FEMENINA DE 20 MESES DE EDAD, SIN ESQUEMA DE VACUNACIÓN.


INICIA SU PADECIMIENTO ACTUAL DESDE HACE UNA SEMANA CON FIEBRE
CUANTIFICADA DE 38.5°C, RINORREA HIALINA Y POSTERIORMENTE
MUCOPURULENTA, EPISTAXIS Y SE AGREGA DISFAGIA. MOTIVO POR LO CUAL
ACUDE A URGENCIAS. A LA EXPLORACIÓN CON ESCORIACIONES FÉTIDAS EN
NARINAS Y LABIO SUPERIOR, EN LA BOCA SE OBSERVAN MEMBRANAS
SANGRANTES DE COLOR GRIS OBSCURO. EL CUELLO SE MUESTRA
INCREMENTADO DE VOLUMEN, A EXPENSAS DE LAS LINFADENOPATÍAS.

245 - EL DIAGNÓSTICO MÁS PROBABLE DE LA PACIENTE ES:


FARINGOAMIGDALITIS POR DIFTERIA
MONONUCLEOSIS INFECCIOSA
HERPANGINA
FARINGOAMIGDALITIS ESTREPTOCÓCICA

Bibliografía:
INFECTOLOGÍA CLÍNICA PEDIÁTRICA. NAPOLEÓN GONZÁLEZ SALDAÑA. MC. GRAW HILL. EDICIÓN 7A. 2003.
PÁG. 829-831.

SEGURA DUDA NI IDEA

246 - EL ESTUDIO QUE DEBE REALIZARSE A LA PACIENTE PARA CONFIRMAR EL


DIAGNÓSTICO ES:

CULTIVO PARA HERPES


SEROLOGÍA PARA EPSTEIN BARR
CULTIVO DE LOEFFLER
CULTIVO EN AGAR SANGRE

Bibliografía:
INFECTOLOGÍA CLÍNICA PEDIÁTRICA. NAPOLEÓN GONZÁLEZ SALDAÑA. MC. GRAW HILL. EDICIÓN 7A. 2003.
PÁG. 830-831.

SEGURA DUDA NI IDEA

247 - EL TRATAMIENTO INICIAL QUE DEBERÁ RECIBIR LA PACIENTE ES CON:

ANTITOXINA ESPECÍFICA
PENICILINA
INTUBACIÓN OROTRAQUEAL
VACUNA ANTIDIFTÉRICA

Bibliografía:
1. INFECTOLOGÍA CLÍNICA PEDIÁTRICA. NAPOLEÓN GONZÁLEZ SALDAÑA. MC. GRAW HILL. EDICIÓN 7A. 2003.
PÁG. 830. 2. KLIEGMAN R, STANTON B, GEME J, SCR N, BEHRMAN R. NELSON TRATADO DE PEDIATRÍA, 19ª
EDICIÓN. ELSEVIER. ESPAÑA 2013.
SEGURA DUDA NI IDEA
ANÁLISIS DEL CASO CLÍNICO

IDENTIFICACIÓN DEL REACTIVO


Area: PEDIATRÍA
Especialidad: CRECIMIENTO Y DESARROLLO
Tema: ALTERACIONES DE LA NUTRICIÓN
Subtema: MARASMO

CASO CLÍNICO SERIADO

LACTANTE MENOR DE 2 MESES DE EDAD CON ANTECEDENTE DE PESO BAJO AL NACER. ES LLEVADO A
CONSULTA DE CONTROL MENSUAL. LA MADRE REFIERE DIFICULTAD PARA ALIMENTARLO DEBIDO A QUE SU
PEZÓN NO ESTÁ BIEN FORMADO. A LA EXPLORACIÓN SE ENCUENTRA TEMPERATURA DE 34.8°C,
DESNUTRICIÓN SEVERA, HIPOTRÓFICO, HIPOACTIVO, PIEL SECA Y ARRUGADA. CON RETRASO EN EL
DESARROLLO PSICOMOTOR EN LA ESFERA DE MOTOR GRUESO.

2 meses de edad (muy importante la edad).

peso bajo al nacer, di cultad para


alimentarlo.

Datos clásicos de desnutrición severa por


pobre consumo calórico.

248 - ES LA PRINCIPAL CAUSA DE CONSUMO DE ENERGÍA EN ESTE NIÑO:

CONSUMO El consumo de energía en general se enfoca a 5 puntos especiales. 1. CONSUMO METABÓLICO EN


METABÓLICO REPOSO (CMR) o Gasto energético en reposo o Metabolismo basal: es el gasto que se realiza
EN REPOSO. durante el reposo y en ayuno. Es mayor en niños que en adultos debido a que la relación
superficie corporal y peso es mayor en niños, una parte de la energía basal del niño se destina al
crecimiento, el tamaño relativo de las vísceras y el cerebro es mayor en los niños. El pico más alto
de este consumo se encuentra en el primer año de vida y después disminuye de forma gradual
hasta llegar a la edad adulta.

NECESIDADES 2. NECESIDADES PARA EL CRECIMIENTO: toda síntesis de tejidos requiere forzosamente de


PARA EL energía. Durante los primeros 4 meses de edad este tipo de consumo tiene su pico más alto ya
CRECIMIENTO. que, en este tiempo se requiere casi un tercio del consumo total calórico para este fin. Al final del
segundo año de vida este nivel se reduce hasta llegar al 1 y 2% del total de las calorías.
PÉRDIDA DE 3. PÉRDIDA DE ENERGÍA A TRAVÉS DE LAS EXCRESIONES: a través de las excreciones se eliminan
ENERGÍA A ciertos compuestos importantes por ejemplo; nitrógeno por la orina, proteínas y grasas por las
TRAVÉS DE heces. Estas pérdidas corresponden aproximadamente al 10% del consumo energético normal.
EXCRECIONES.

ACCIÓN 4. EFECTO TÉRMICO DE LOS ALIMENTOS, también llamado Acción Dinámica Específica: el
DINÁMICA metabolismo en reposo aumenta en el periodo postprandial, especialmente cuando la comida es
ESPECÍFICA. rica en proteínas, y tarda varias horas en regresar a su estado basal. La cantidad de energía que se
utiliza para este fin corresponde al 5 a 10% del consumo calórico total.

Bibliografía:
1. MCINERNY T.TRATADO DE PEDIATRÍA, 1A EDICIÓN. EDITORIAL MÉDICA PANAMERICANA. ARGENTINA,
2011.

249 - UNA VEZ INICIADO SU MANEJO SE DEBERÁ CONSIDERAR ADECUADA LA SIGUIENTE GANANCIA
PONDERAL:

5G/KG DÍA. El manejo inicial del niño desnutrido grave será en medio hospitalario.

10G/KG/DÍA. En todo niño desnutrido que inicia manejo médico nutricional se deberá vigilar la ganancia de peso.
Se considera adecuada 10g/Kg/d

15G/KG/DÍA. En caso de ganancia de peso insuficiente se descartarán de manera general los siguientes problemas
: - Inadecuada frecuencia en la alimentación. - No se alimenta correctamente en las noches. - Aporte
energético inadecuado, no se ajusta el aporte de calorías proporcionalmente al aumento de peso
conseguido. - Intolerancia digestiva a la alimentación implementada. - Mala dinámica de
alimentación entre el niño(a) y cuidador(a). - Errores en la preparación de la fórmula o alimento
preparado in situ, mal estado o almacenamiento de los ingredientes, aspecto, higiene y sabor del
alimento administrado.

20G/KG/DIA. En niños que no recuperan el peso esperado siempre se descartarán infecciones subyacentes que
frecuentemente pasan desapercibidas: - Infecciones de vías urinarias. - Otitis media. - Tuberculosis.
- Giardiasis.

Bibliografía:
1. GUÍA DE PRÁCTICA CLÍNICA, DIAGNÓSTICO Y TRATAMIENTO DE LA DESNUTRICIÓN EN MAYORES DE
CINCO AÑOS EN EL PRIMER NIVEL DE ATENCIÓN. MÉXICO: SECRETARIA DE SALUD; 2008. 2. DIAGNÓSTICO
Y TRATAMIENTO DE LA DESNUTRICIÓN EN MENORES DE CINCO AÑOS DE EDAD EN EL PRIMER NIVEL DE
ATENCIÓN. EVIDENCIAS Y RECOMENDACIONES; GUÍA DE PRÁCTICA CLÍNICA. MÉXICO, CENETEC:2018.

http://www.cenetec-difusion.com/CMGPC/SS-119-08/ER.pdf

FIN DEL CASO CLÍNICO SERIADO


ANÁLISIS DEL CASO CLÍNICO

IDENTIFICACIÓN DEL REACTIVO


Area: MEDICINA INTERNA
Especialidad: INMUNOALERGIA
Tema: RINITIS ALÉRGICA
Subtema: RINITIS ALÉRGICA

CASO CLÍNICO SERIADO

MUJER DE 35 AÑOS CON ANTECEDENTE DE INMUNODEFICIENCIA PRIMARIA. INICIA SU PADECIMIENTO HACE


2 DÍAS CON LA DISFONÍA CONTINUA Y TOS SECA. SE REALIZA LARINGOSCOPIA QUE MUESTRA EDEMA Y
ERITEMA EN AMBAS CUERDAS VOCALES.

MUJER DE 35 AÑOS.

INMUNODEFICIENCIA PRIMARIA.

HACE 2 DÍAS CON LA DISFONÍA CONTiNUA


Y ToS SECA.

-.

SE REALIZA LARINGOSCOPÍA QUE


MUESTRA EDEMA Y ERITEMA EN AMBAS
CUERDAS VOCALES.

250 - EL DIAGNÓSTICO CLÍNICO MÁS PROBABLE ES:

PAPILOMATOSIS PAPILOMATOSIS LARÍNGEA - Es considerada como la neoplasia benigna más común en


LARÍNGEA. población pediátrica y adultos jóvenes. Ésta es una enfermedad causada por
papilomavirus, siendo los más comunes el tipo 6 y 11, mismos que son responsables de los
condilomas genitales. - La localización más común es la glotis y la subglotis, tiende a
producir lesiones exofíticas y puede presentar diseminación traqueobroncoalveolar en
pocos casos. Los papilomas traqueales pueden ser localizados o diseminados. Tienen una
apariencia de “hueva de salmón”. Son estrictamente intraluminales y pueden obstruir la
totalidad de la vía aérea. - El diagnóstico se realiza con visualización indirecta
(laringoscopia y/o fibrobroncoscopía) y toma de biopsia. NO HAY DESCRIPCIÓN DE
LESIONES EXOFÍTICAS CLÁSICAS DURANTE LA LARINGOSCOPÍA.
EPIGLOTITIS. EPIGLOTITIS - Es una enfermedad infecciosa que progresa rápidamente poniendo en riesgo
la vida por obstrucción de la vía aérea por lo que se considera una urgencia médica. -
Involucra la epiglotis y estructuras adyacentes: superficie lingual posterior, tejidos blandos
contiguos y pliegues aritenoidepiglóticos. - Más común en edad escolar y adolescencia. EL
INICIO ES SÚBITO Y DE EVOLUCIÓN RÁPIDA LO QUE AUNADO A LA EDAD NO
CORRESPONDEN AL CUADRO CLÍNICO DE LA PACIENTE.

LARINGITIS AGUDA LARINGITIS AGUDA INESPECÍFICA - Las laringitis agudas inespecíficas difusas, son muy
INESPECÍFICA. comunes, representando la gran mayoría de las consultas por disfonía aguda tanto en los
servicios de urgencia como en atención primaria. El motivo de consulta, además de la
disfonía, suele ser un cuadro catarral inespecífico, con febrícula, odinofagia, sensación de
sequedad en la garganta y tos seca persistente. - A la exploración cervical, no aparecen
adenopatías significativas o bien podemos tener alguna adenopatía yugulodigástrica
inespecífica inflamatoria. - A la exploración laríngea indirecta con espejillo o con
endoscopio, tendremos una mucosa laríngea uniformemente enrojecida y congestiva, con
aspecto reseco, pero con normalidad de la anatomía supraglótica y glótica; la movilidad de
las cuerdas vocales esta conservada, si bien podemos observar cierta dificultad para el
cierre completo debido a la inflamación de la mucosa. - El proceso suele durar unos días,
pero puede alargarse debido a la sobreinfección bacteriana, como se detalla a
continuación al comentar la etiología, entonces la tos que inicialmente era seca, se hace
productiva, con expectoración mucopurulenta. Debemos reseñar igualmente, que las
laringitis agudas inespecíficas aparecen muy frecuentemente en el contexto de un catarro
de vías respiratorias altas, apareciendo primero en forma de coriza, y descendiendo luego
hacia orofaringe y laringe. El cuadro tiene buen pronóstico en general. LA DISFONÍA
ACOMPAÑADA DE TOS SECA FUNDAMENTAN EL CUADRO CLÍNICO.

LARINGOTRAQUEITIS. LARINGOTRAQUEITIS - Puede aparecer como cuadro de debut o bien como complicación
de la laringitis subglótica, y supone el edema de toda la mucosa del árbol respiratorio
inferior a la glotis, pudiendo incluir entonces subglotis, tráquea e incluso bronquios
pulmonares. - Es un cuadro que puede ser muy grave, y es lo que clásicamente se conocía
como “Crup verdadero”. - El cuadro de obstrucción respiratoria es mucho más llamativo, y
además del estridor, presentará grados variables de broncoespasmo, por lo que la
traqueotomía de urgencias será insuficiente para solucionar la dificultad respiratoria,
debiendo instaurarse la oxigenoterapia intensiva. - La etiología de estos cuadros, en el
caso de la laringitis subglótica, puede desencadenarse tras infección vírica de cualquier
tipo, pues la conformación histológica subglótica la hace especialmente sensible a mínimas
inflamaciones; en cuanto a las bacterias, Haemophylus influenza será el más frecuente. ES
UNA PATOLOGÍA RESPIRATORIA COMÚN EN LOS INFANTES CON DATOS DE
OBSTRUCCIÓN RESPIRATORIA POR LO QUE NO CORRESPONDE A NUESTRA PACIENTE.

Bibliografía:
1. LONGO DL, FAUCI AS, KASPER DL, HAUSERSL, JAMESON JL, LOSCALZOJ. HARRISON. PRINCIPIOS DE
MEDICINA INTERNA, 18A EDICIÓN. MC GRAW HILL. NEW YORK, USA. 2012, PP 265. 2. PAPADAKIS MAXINE A,
MCPHEE STEPHEN J. DIAGNÓSTICO CLÍNICO Y TRATAMIENTO. 52ª EDICIÓN. NUEVA YORK. 2013, PP 233.

251 - EL TRATAMIENTO INDICADO EN ESTE CASO ES:

NEBULIZACIONES TRATAMIENTO DE LARINGOTRAQUEITIS - La nebulización precisa flujos de oxigeno o aire


CON BUDESONIDA Y de 5l/min para que las partículas se depositen en la laringe. - Se demostró mejoría con el
ANTINFLAMATORIOS uso de corticoides a las 6 y 12 horas de su uso, además de mejores resultados con el uso de
epinefrina racémica. - Se ha visto mejoría con el uso de dosis única de dexametasona 0.6
mg./Kg/Peso intramuscular (IM), en los tres primeros grados de laringitis, reduciendo la
severidad de los síntomas como también la necesidad de terapia concomitante con
epinefrina racémica, pudiendo utilizar la vía oral, aunque la IM es necesario en caso de
vómitos o dificultad respiratoria severa. - Se puede utilizar budesonida en nebulización, a
dosis de 2 mg. disminuye el riesgo de hospitalización y más rápida disminución de la
sintomatología. NO CORRESPONDE AL MANEJO DE LA PATOLOGÍA DIAGNOSTICADA.
NEBULIZACIONES La Epinefrina racémica da buenos resultados en el manejo de la laringotraqueitis. NO
CON EPINEFRINA CORRESPONDE AL MANEJO DE LA PATOLOGÍA DIAGNOSTICADA.
RACÉMICA

RESECCIÓN La resección quirúrgica está indicada en papilomatosis o tumoraciones de diversos tipos a


QUIRÚRGICA Y éste nivel. NO CORRESPONDE AL MANEJO DE LA PATOLOGÍA DIAGNOSTICADA.
REPOSO DE VOZ

REPOSO DE VOZ Y El tratamiento de la Laringitis Aguda Inespecífica en el adulto se basa en: - Reposo de voz -
ANTIINFLAMATORIOS Líquidos abundantes - AINEs - Mucolíticos si hay secreción abundante - Antibióticos sólo si
es bacteriana - Prohibición del tabaco. EL MANEJO SE BASA EN ANTIINFLAMATORIOS Y
MEDIDAS GENERALES. Recuerda que... en todo tipo de laringitis está indicado el reposo de
la voz.

Bibliografía:
1. LONGO DL, FAUCI AS, KASPER DL, HAUSERSL, JAMESON JL, LOSCALZOJ. HARRISON. PRINCIPIOS DE
MEDICINA INTERNA, 18A EDICIÓN. MC GRAW HILL. NEW YORK, USA. 2012, PP 265. 2. PAPADAKIS MAXINE A,
MCPHEE STEPHEN J. DIAGNÓSTICO CLÍNICO Y TRATAMIENTO. 52ª EDICIÓN. NUEVA YORK. 2013, PP 233.

FIN DEL CASO CLÍNICO SERIADO


ANÁLISIS DEL CASO CLÍNICO

IDENTIFICACIÓN DEL REACTIVO


Area: CIRUGÍA
Especialidad: PROCTOLOGÍA Y UROLOGÍA
Tema: PATOLOGÍA DE COLÓN, ANO Y RECTO
Subtema: FISTULA PERIANAL

CASO CLÍNICO CON UNA PREGUNTA

HOMBRE DE 28 AÑOS DE EDAD, CON ANTECEDENTE DE ESTREÑIMIENTO CRÓNICO, DOLOR A LA


DEFECACIÓN Y DESCARGA A PESAR DE NO HABER EVACUADO. DURANTE LA EXPLORACIÓN ENCUENTRA
UNA FÍSTULA PERIANAL INTERESFINTERIANA.

28 años.

estreñimiento crÓnico.

dolor al defecar, descarga A PESAR DE NO


HABER EVACUADO.

fÍstula perianal interes nteriana.

-.

252 - EL TRATAMIENTO DE ELECCIÓN EN ESTE CASO SERÍA:

FISTULOESCLEROSIS. LA SIMPLE OBLITERACIÓN DEL TRAYECTO FISTULOSO CON ALGÚN MATERIAL NO ES


SUFICIENTE EN EL TRATAMIENTO DE LA FÍSTULA PERIANAL. Desde el punto de vista
fisiopatológico, la fístula es un trayecto epitelizado y es justo esta característica la que no
permitirá el adecuado manejo de la misma con esclerosis, por otro lado no se ha encontrado
el material ideal para la obliteración del conducto fistuloso.

FISTULORRAFIA. El cierre primario con sutura del trayecto fistuloso "no ha demostrado efecto terapéutico
adecuado", pues presenta un alto índice de recidiva sin legrar el trayecto y abrirlo.
FISTULOTOMÍA. La FÍSTULA ANAL es una enfermedad caracterizada por la presencia de un conducto de
paredes fibrosas afectadas que comunica una cripta anal o el lumen del recto con la piel. El
orificio localizado en la cripta anal se denomina primario o interno, y el cutáneo, secundario
o externo. Las fístulas simples son generalmente interesfintéricas o transesfintéricas bajas,
generalmente con un trayecto único entre el oficio primario y el secundario. La fístula
compleja tiene un trayecto que comprende más del 30% a 50% del esfínter externo
(transesfintérica alta, supraesfintérica o extraesfintérica), es de localización anterior en
mujeres, la padece un paciente con incontinencia, existe enfermedad inflamatoria intestinal
o el antecedente de cirugías previas anorrectales. Se presentan en el 50% de los casos que
han desarrollado absceso anorrectal. Los antecedentes de importancia en este caso son:
patología intestinal asociada, eventos que comprometen la función del esfínter, cirugía
anorrectal, trauma obstétrico, procesos infecciosos en el área anorrectal. Por lo general se
presentan con descarga a través del orifico externo, en ocasiones con dolor. A la exploración
física se observa el orifico externo con descarga y se palpa trayecto con fibrosis. EL
TRATAMIENTO IDEAL DE LA FÍSTULA PERIANAL DE LA FÌSTULA SIMPLE, O DE AQUELLA QUE
NO INVOLUCRA UNA CANTIDAD IMPORTANTE DEL ESFÍNTER ANAL ES LA FISTULOTOMÍA,
bajo anestesia regional, se coloca al paciente en posición proctológica, se identifica el
orificio externo e idealmente el orifico interno, para hacerlo se puede infundir a través del
orificio externo peróxido de hidrógeno e identificar el orificio interno , se cánula
gentilmente con estilete y se incide el trayecto con electrocauterio, se legra el piso de la
fístula y se deja abierta la herida para cierre por segunda intención. Una segunda corriente
marsupializa los bordes, lo que favorece una recuperación más rápida y disminuye las
recidivas así como las alteraciones funcionales. Con la fistulotomía el procentaje de recidiva
va del 2-9%, con alteración funcional del 0-17%. LA FISTULOTOMÍA ES EL TRATAMIENTO DE
ELECCIÓN PARA EL MANEJO DE LA FÍSTULA RECTAL GRACIAS A SU BAJO PORCENTAJE DE
RECIDIVA Y SU MENOR AFECCIÓN DE LA FUNCIÓN RECTAL.

FISTULECTOMÍA. La fístula anal simple puede tratarse con fistulectomía; sin embargo, el porcentaje de
afectación funcional y el índice de recidivas es mayor con este abordaje, por lo que es
preferible la fistulotomía. LA FISTULECTOMÍA PROVOCA MAYOR RECIDIVA Y AFECTACIÓN
DE LA FUNCIÓN, POR TANTO NO SE CONSIDERA EL TRATAMIENTO DE ELECCIÓN.

Bibliografía:
1. GUÍA DE PRÁCTICA CLÍNICA, DIAGNÓSTICO Y TRATAMIENTO QUIRÚRGICO DE LA FÍSTULA ANAL EN EL
ADULTO. MÉXICO: SECRETARIA DE SALUD; 12 DICIEMBRE 2013. 2. GUÍA DE REFERENCIA RÁPIDA,
DIAGNÓSTICO Y TRATAMIENTO QUIRÚRGICO DE LA FÍSTULA ANAL EN EL ADULTO. MÉXICO: SECRETARIA
DE SALUD; 12 DICIEMBRE 2013. 3. BRUNICARDI F, ANDERSEN D, BILLIAR T, Y COLS. SCHWARTZ PRINCIPIOS
DE CIRUGÍA, 9A EDICIÓN. MC GRAW HILL. 2011, PP 1064-1065.

http://www.cenetec.salud.gob.mx/descargas/gpc/CatalogoMaestro/200_SSA_09_fistula_anal/GPC_SSA_200-09_FISTULA_ANAL_No_sexalar_actualizada.pdf
ANÁLISIS DEL CASO CLÍNICO

IDENTIFICACIÓN DEL REACTIVO


Area: GINECOLOGÍA Y OBSTETRICIA
Especialidad: GINECOLOGÍA
Tema: DOLOR PELVICO
Subtema: ENDOMETRIOSIS

CASO CLÍNICO CON UNA PREGUNTA

MUJER DE 32 AÑOS DE EDAD, CUYO CUADRO CLÍNICO SE CARACTERIZA POR TENER UN PATRÓN CÍCLICO
REGULAR CON DOLOR PÉLVICO INTENSO, DISPAREUNIA, ESTERILIDAD Y DISMENORREA.

Mujer de 32 años de edad.

Patrón cíclico REGULAR.

Dolor pélvico, dispareunia, esterilidad y


dismenorrea.

-.

-.

253 - EL DIAGNÓSTICO CLÍNICO MÁS PROBABLE ES:

ENFERMEDAD La ENFERMEDAD PÉLVICA INFLAMATORIA aguda consiste en la infección del útero, trompas de
PÉLVICA Falopio y estructuras pélvicas adyacentes, no asociadas con cirugía o embarazo. Los criterios
INFLAMATORIA. clínicos para el diagnóstico. • Dolor abdominal con o sin rebote. • Sensibilidad a la movilización
del cérvix. • Sensibilidad anexial. Los tres criterios anteriores son necesarios para establecer el
diagnóstico, con uno o más de los siguientes: Extendido de Gram positivo de endocérvix, para
diplococos Gram negativos intracelulares temperatura mayor de 38°C. Leucocitosis (mayor de
10.000 por c.c.) Material purulento (Positivo para leucocitos), en la cavidad peritoneal obtenida
por culdocentesis o laparoscopia. NO CUMPLE CON LOS CRITERIOS CLÍNICOS PARA ESTE
DIAGNÓSTICO, NO EXISTEN SIGNOS DE INFECCIÓN.
OVARIO El SÍNDROME DE STEIN-LEVENTHAL u OVARIO POLIQUÍSTICO se caracteriza por ciclos
POLIQUÍSTICO. anovulatorios que dan como resultado ovarios poliquísticos hipertróficos, amenorrea u
oligomenorrea, obesidad, hirsutismo e infertilidad. Histológicamente se caracteriza por la
presencia de pequeños quistes foliculares llenos de líquido debajo de la engrosada y fibrosa
corteza superficial. CX: - Resistencia insulínica: se manifiesta por mayor grado de resistencia
insulínica e hiperinsulinemia, situación que puede condicionar hiperglucemias. - Anovulación:
la secreción inapropiada de gonadotropinas provoca ciclos anovulatorios, con mayor
producción de hormona luteinizante (LH) y de hormona foliculoestimulante (FSH). -
Hiperandrogenismo: la hiperilsulinemia y la elevación de LH estimula la producción de
andrógenos en las células de la teca ovárica. - Hirsutismo: se manifiesta con la presencia de
vello grueso, oscuro y terminal distribuido en un patrón masculino. - Acné: el aumento de
andrógenos hiperestimula los receptores androgénicos en la unidad pilosebásea con lo cual
aumenta la producción de sebo lo cual favorece la formación de comedones. Las pruebas de
laboratorio revelan elevación sérica de andrógenos, aumento en la proporción de hormona
luteinizante contra la hormona del folículo, anormalidades en los lípidos y resistencia a la
insulina. LOS CICLOS MENSTRUALES SON CÍCLICAS REGULARES, NO CUMPLE CON CRITERIOS.

ENDOMETRIOSIS. Los síntomas que presentan las pacientes son: 1. Dolor: (Dispareunia y dismenorrea) Es el
síntoma más frecuente de la endometriosis. Se localiza principalmente en el abdomen, en la
región lumbar, rectal, dolores radiantes a ambas piernas, pero en especial la derecha y en la
pelvis, que puede ser de leve a severo. 2. Hipermenorrea: Consiste en sangrados menstruales
muy abundantes tanto en el tiempo como en la cantidad de sangre. A veces la metrorragia
ocurre fuera de la menstruación. 3. Infertilidad: Dificultad o imposibilidad de quedar
embarazada. 4. Trastornos intestinales: Como diarrea, estreñimiento, dolor al defecar o
proctalgia. 5. Astenia: También se llama fatiga. 6. Amenorrea: Consiste en ausencia menstrual,
debido a que el sangrado es interno en la cavidad abdominal. DEBES RECORDAR LA TRIADA
CLÁSICA DE ENDOMETRIOSIS: DISMENORREA, DISPAREUNIA E INFERTILIDAD.

CERVICITIS Los síntomas son: • Sangrado vaginal anormal; después de las relaciones sexuales, después de
CRÓNICA. la menopausia, entre menstruaciones. • Leucorrera vaginal inusual; que no desaparece, de color
gris, blanco o amarillo, y puede tener olor. • Dispareunia. • Dolor vaginal y pélvico. NO SE
MENCIONA LA PRESENCIA DE LEUCORREA EN LA PACIENTE.

Bibliografía:
1. GUÍA DE PRÁCTICA CLÍNICA, DIAGNÓSTICO Y TRATAMIENTO DE LA ENDOMETRIOSIS. MÉXICO:
SECRETARIA DE SALUD; MARZO 2013. 2. SCHONGUE J, SCHAFER J, HALVORSON L, HOFFMAN B,
BRADSHAW K, CUNNINGHAM G. WILLIAMS GINECOLOGÍA, DE LA 1A EDICIÓN EN INGLÉS. MC GRAW HILL.
USA. 2009.

http://www.cenetec.salud.gob.mx/descargas/gpc/CatalogoMaestro/207_SSA_10_ENDOMETRIOSIS/EyR_SSA_207_09.pdf
ANÁLISIS DEL CASO CLÍNICO

IDENTIFICACIÓN DEL REACTIVO


Area: PEDIATRÍA
Especialidad: URGENCIAS PEDIÁTRICAS
Tema: URGENCIAS CARDIOVASCULARES PEDIÁTRICAS
Subtema: VALORACIÓN GENERAL DE LAS CARDIOPATÍAS

CASO CLÍNICO CON UNA PREGUNTA

RECIÉN NACIDA DE 10 DÍAS DE VIDA QUE ACUDE A SU PRIMERA VISITA MÉDICA. DURANTE LA EXPLORACIÓN
USTED LE AUSCULTA UN SOPLO POR LO QUE LE SOLICITARÁ ESTUDIOS COMPLEMENTARIOS.

recién nacida de 10 días de vida

AUSCULTA UN SOPLO. en función de la


valoración clínica del soplo, las
manifestaciones cardiovasculares y el
crecimiento y desarrollo del paciente se
determinará la conducta diagnóstica que
se deberá seguir en estos pacientes

254 - EN CASO DE CONFIRMARSE UNA CARDIOPATÍA, POR FRECUENCIA ES MÁS PROBABLE QUE SE
TRATE DE:
UNA ETIOLOGÍA La etiología todavía no es clara en muchas de las cardiopatías, y se consideran
COMUNICACIÓN tres principales causas: genética, factores ambientales y multifactorial, en la que se
INTERAURICULAR asociarían factores genéticos y ambientales. Dentro de los de etiología genética, aparte de
las cromosomopatías conocidas se han identificado defectos genéticos y moleculares
específicos que contribuyen a las malformaciones cardiacas. De tal manera, se han
identificado mutaciones de un solo gen en malformaciones cardiacas aisladas, como la
estenosis aórtica supravalvular y la coartación aórtica, o asociadas a síndromes
malformativos, como: Alagille, Marfan, Noonan o Holt Oram. También, síndromes de
microdeleciones cromosómicas han sido implicadas en las malformaciones cardiacas, como
en el síndrome de DiGeorge, o en el síndrome de Williams-Beuren. Es muy interesante en
este aspecto, el estudio publicado por Mary Ella Pierpont y cols. En que hace una revisión
actualizada sobre las bases genéticas de las cardiopatías, donde se pueden encontrar, por un
lado, un algoritmo de las diferentes malformaciones cardíacas asociadas a las
cromosomopatías (deleciones, trisomías, monosomías...) y, por otro lado, un algoritmo
extenso de las causas genéticas, asociaciones y características clínicas de las principales
cardiopatías. Dentro de las causas ambientales responsables de malformaciones cardiacas,
están: 1. las enfermedades maternas, como la diabetes pregestacional, la fenilcetonuria, el
lupus eritematoso y la infección por VIH; 2. exposición materna a drogas (alcohol,
anfetaminas, hidantoínas y otras); y 3. exposición a tóxicos, como: disolventes orgánicos,
lacas, pinturas, herbicidas, pesticidas y productos de cloración. El riesgo de padecer una
cardiopatía en relación con estos factores no hereditarios está detallado en el estudio de
Kathy J. Jenkins, en que se valora el riesgo relacionado con las diferentes drogas y agentes
químicos y físicos.

UNA Las CARDIOPATÍA CONGÉNITA (CC) más frecuentes según los diferentes autores, A NIVEL
COMUNICACIÓN MUNDIAL son, por orden de frecuencia: CIV, CIA, el ductus permeable, estenosis pulmonar
INTERVENTRICULAR (EP), coartación aórtica (CoAo), tetralogía de Fallot y estenosis aórtica (EAo). Y después, la
transposición de grandes arterias, el canal atrioventricular y el síndrome del corazón
izquierdo hipoplásico. LA COMUNICACIÓN INTERVENTRICULAR OCUPA EL PRIMER LUGAR
DE FRECUENCIA EN CARDIOPATÍAS CONGÉNITAS A NIVEL MUNDIAL, PERO NO EN
NUESTRO PAÍS. OJO: si el reactivo especifica a nivel mundial, deberás tener este dato en
cuenta.

LA PERSISTENCIA En México en diversos estudios como el siguiente, reportan otro orden: Un análisis de 2257
DEL CONDUCTO pacientes con cardiopatía congénita realizado en el Hospital de Cardiología del Centro
ARTERIOSO Médico Nacional Siglo XXI, mostró que la persistencia del conducto arterioso representó
20% de los casos, situación muy explicable por la altura a la que, con respecto al nivel del
mar, está la Ciudad de México y zonas conurbadas; le siguió la comunicación interatrial
(16.8%); comunicación interventricular (11%); tetralogía de Fallot y atresia pulmonar con
comunicación interventricular (9.3%); coartación aórtica y estenosis pulmonar (3.6%)
respectivamente y la conexión anómala total de venas pulmonares (3%). CON BASE EN LA
GPC LA PERSISTENCIA DEL CONDUCTO ARTERIOSO (PCA) ES LA CARDIOPATÍA CONGÉNITA
MÁS FRECUENTE EN MÉXICO. REPASO. Se llama PCA a la persistencia de la permeabilidad
del conducto arterioso > 6ta semana de vida. La incidencia PCA aumenta directamente en
prematuros, predomina en el sexo femenino con una relación 2:1. En este momento las
malformaciones congénitas representan la 2da causa de mortalidad en menores de 5 años y
específicamente las malformaciones cardiacas aisladas se encuentran en noveno lugar. La
PCA se encuentra dentro de las primeras dos cardiopatías congénita mas frecuentes en
pacientes con síndrome de Down con una incidencia hasta en 58%. En un alto porcentaje
(40%) la PCA se encuentra asociado a otras cardiopatías entre las cuales se encuentran
principalmente la comunicación interventricular, interauricular, válvula aortica bivalva,
estenosis pulmonar (valvular y supravalvular), coartación de aorta.
UNA DETECCIÓN DE UNA CC EN EL RECIEN NACIDO. - Anamnesis. En primer lugar, es importante
TRANSPOSICIÓN DE la anamnesis de las enfermedades maternas (diabetes, drogas, tóxicos), antecedentes de CC
GRANDES VASOS o de otras anomalías cardiacas familiares y antecedentes obstétricos (si se ha efectuado
ecocardiografía fetal, sufrimiento fetal, infecciones...). También hay que valorar el momento
de aparición de los síntomas o signos. Así, preguntaremos sobre la respiración, ya que la
taquipnea suele ser frecuente en muchas cardiopatías, la forma de alimentarse, si rechaza el
alimento, si suda con las tomas, si gana peso. - Exploración. Habrá que investigar la cianosis,
y diferenciarla de la cianosis periférica secundaria a un enlentecimiento del flujo sanguíneo
por el frío, hipovolemia o shock. Se considera patológica una PO2 inferior a 60 mmHg o una
saturación menor al 92% respirando aire ambiente. La auscultación tiene un valor limitado
en el RN, pues puede haber CC sin soplos. Hay que prestar atención a la frecuencia cardiaca
y el ritmo; así, en el RN, los límites normales están entre 80-170/m, y en los dos primeros
años oscila de 80-130/m. Es importante la palpación de los pulsos periféricos
simultáneamente, para descartar una Co Ao. En el RN y lactante, es más fácil palpar los
axilares que los radiales y en ocasiones, se palpan mejor los pedios que los femorales. La
palpación torácica de un frémito es indicativa de cardiopatía. Una hepatomegalia superior a
3 cm probablemente sea secundaria a insuficiencia cardiaca congestiva (ICC). Una frecuencia
respiratoria por encima de 60/m debe ponernos en alerta para descartar CC. El diagnóstico
se basará principalmente en la ecocardiografía-Doppler, que se hará siempre que haya la
sospecha. El ECG nos puede orientar en algunas cardiopatías y cuando se sospeche arritmia.
La Rx de tórax será útil para valorar el tamaño cardiaco, el flujo pulmonar y el arco aórtico.

Bibliografía:
1. TRATADO DE PEDIATRÍA. NELSON. MC-GRAW-HILL. INTERAMERICANA. EDICIÓN 15. 1997. PAG. 1610-1612. 2.
GUÍA DE PRÁCTICA CLÍNICA. DIAGNÓSTICO Y TRATAMIETNO DE LA PERSISTENCIA DEL CONDUCTO
ARTERIOSO EN NIÑOS, ADOLESCENTES Y ADULTOS . MÉXICO: SECRETARÍA DE SALUD, 2009.

http://www.cenetec.salud.gob.mx /descargas/gpc/CatalogoMaestro

/380_GPC_PERSISTENCIA_DEL_CONDUCTO_ARTERIOSO/GER_PERSISTENCIA_CONDUCTO_ARTERIOSO.pdf
ANÁLISIS DEL CASO CLÍNICO

IDENTIFICACIÓN DEL REACTIVO


Area: MEDICINA INTERNA
Especialidad: GERIATRÍA Y GERONTOLOGÍA
Tema: MALTRATO EN EL ADULTO MAYOR
Subtema: MALTRATO EN EL ADULTO MAYOR

CASO CLÍNICO SERIADO

DURANTE LA SESIÓN MENSUAL DE ADULTOS MAYORES DE SU COMUNIDAD, LOS ANCIANOS SE QUEJAN DE


ESTRÉS Y SENSACIÓN DE MALTRATO PSICOLÓGICO POR PARTE DE SUS CUIDADORES. COMO ESTRATEGIA DE
ACCIÓN SE HA PROPUESTO INICIAR UN PROGRAMA DE ATENCIÓN PARA PREVENIR EL MALTRATO Y LA
INSTAURACIÓN DE UN PLAN DE REHABILITACIÓN INTEGRAL EN LOS ANCIANOS AFECTADOS.

Adultos mayores.

Estrés y maltrato psicológico.

-.

-.

-.

255 - ES LA ESTRATEGIA CONTRA EL MALTRATO QUE INCLUYE DENTRO DE SUS ACCIONES LA


REHABILITACIÓN DEL PACIENTE :

APOYO El APOYO EMOCIONAL es el contacto social y afectivo que recibe la persona de manera
EMOCIONAL. coordinada y en momentos críticos. EL APOYO EMOCIONAL INCORPORA LA AFECTIVIDAD CON
LOS ESTADOS CRÍTICOS DEL INDIVIDUO.

CALIDAD DE La CALIDAD DE VIDA es la percepción que un individuo tiene de su lugar en la existencia, en el


VIDA. contexto de la cultura y del sistema de valores en los que vive y en relación con sus objetivos, sus
expectativas, sus normas y sus inquietudes. Se trata de un concepto muy amplio que se ve
afectado de modo complejo por la salud física del sujeto, su estado psicológico, su nivel de
independencia, sus relaciones sociales, así como su relación con los elementos esenciales de su
entorno. LA CALIDAD DE VIDA RELACIONA AL INDIVIDUO CON SU MEDIO.
ATENCIÓN ATENCIÓN INTEGRAL es el proceso de atención a la salud que comprende las esferas biológica,
INTEGRAL. psicológica, social y ambiental del individuo, la familia y la población, en aspectos de prevención
primaria, secundaria y terciaria, curación, rehabilitación y atención social a la salud. LA ATENCIÓN
INTEGRAL ES LA ÚNICA ESTRATEGIA QUE AL INCLUIR LA PREVENCIÓN TERCIARIA TOMA EN
CUENTA LA REHABILITACIÓN DE LOS PACIENTES.

APOYO El APOYO INFORMATIVO es la información verbal, escrita relativa a las redes de apoyo formal,
INFORMATIVO. programas gerontológicos y servicios accesibles y disponibles para las personas adultas mayores.
EL APOYO INFORMATIVO NO VA MÁS ALLÁ DE SÓLO OFRECER ALTERNATIVAS AL PACIENTE
PARA SOLICITAR ATENCIÓN SIN QUE ESTO INFLUYA DIRECTAMENTE EN SU ESTADO O
DECISIONES.

Bibliografía:
1. GUÍA DE PRÁCTICA CLÍNICA, DETECCIÓN Y MANEJO DEL MALTRADO EN EL ADULTO MAYOR. MÉXICO:
SECRETARIA DE SALUD; 2013.

http://www.cenetec.salud.gob.mx/descargas/gpc/CatalogoMaestro/057_GPC_MaltratoAdultoMayor/MAM_EVR_CENETEC.pdf

256 - EL PROFESIONAL ESPECIALIZADO QUE DEBERÁ OCUPARSE DE LA REHABILITACIÓN DE LOS


PACIENTES Y CUIDADORES ES:

EL GERIATRA. GERIATRÍA es la especialidad médica encargada del estudio y la atención (preventiva y curativa)
de los principales problemas de salud y sus repercusiones sobre la funcionalidad física, mental y
social de las personas adultas mayores.

EL ACTIVADOR El ACTIVADOR FÍSICO en general es aquel que mediante actividades deportivas busca dar
FÍSICO. respuesta a las necesidades de prevención y acceso a la salud en su dimensión física y social. Está
enfocado a todas las edades de la vida, pero no rehabilita, sólo promueve.

EL GERONTOLOGÍA es la ciencia que estudia el envejecimiento en sus aspectos biológicos,


GERONTÓLOGO. psicológicos y sociales.

EL GERICULTISTA es el técnico profesional que proporciona atención integral al adulto mayor,


GERICULTISTA. aplica técnicas específicas de rehabilitación, da información referente a fomento a la salud y
asesoría al familiar y/o al cuidador.

Bibliografía:
1. GUÍA DE PRÁCTICA CLÍNICA, DETECCIÓN Y MANEJO DEL MALTRADO EN EL ADULTO MAYOR. MÉXICO:
SECRETARIA DE SALUD; 2013.

http://www.cenetec.salud.gob.mx/descargas/gpc/CatalogoMaestro/057_GPC_MaltratoAdultoMayor/MAM_EVR_CENETEC.pdf

FIN DEL CASO CLÍNICO SERIADO


ANÁLISIS DEL CASO CLÍNICO

IDENTIFICACIÓN DEL REACTIVO


Area: MEDICINA INTERNA
Especialidad: INFECTOLOGÍA
Tema: INFECCIONES FEBRILES SISTÉMICAS
Subtema: HIV Y SIDA

CASO CLÍNICO SERIADO

HOMBRE DE 63 AÑOS DE EDAD, VIH POSITIVO, ACUDE A CONSULTA AL PRESENTAR UN CUADRO COMPATIBLE
CON HERPES ZÓSTER EN CARA, REFIRIENDO INTENSO DOLOR NOCTURNO.

Masculino de 63 años DE EDAD.

vih, herpes zóster en cara.

Re riendo intenso dolor nocturno.

-.

-.

257 - EL SIGUIENTE MEDICAMENTO HA DEMOSTRADO DISMINUIR LA INTENSIDAD DEL DOLOR, SIN


DISMINUIR EL RIESGO DE PRESENTAR NEURITIS POSTHERPÉTICA.
PREDNISONA REPASO: El HERPES ZÓSTER es una enfermedad infecciosa aguda que afecta nervios periféricos y la
piel de los dermatomas correspondientes. Se presenta como una erupción vesicular que incluye
uno o dos dermatomas adyacentes caracterizado frecuentemente por dolor en piel de tipo
ardoroso días antes de la erupción. El herpes zoster es la expresión clínica de la reactivación del
virus de la varicela zoster que permanece como infección latente en ganglios nerviosos de los pares
craneales y de todo el neuroeje después de haber padecido varicela, que es la infección primaria. La
reactivación del virus es un evento que se inhibe si la respuesta inmune es normal, pero cuando
esta declina por procesos de evejecimiento, tratamiento inmunosupresor u otras casusas de
inmunodepresión (VHI), la reactivación viral se lleva a cabo, el virus se replica, produce ganglionitis
don daño neuronal, manifestaciones cutáneas y, respuesta inflamatoria local y sistémica. Los
elementos clínicos del herpes zóster son primordialmente: - Dolor o parestesias ardorosas en la
zona afectada. - Distribución en varios dermatomas. - Aparición de exantema característico,
pápulas, vesículas, pústulas y finalmente costra y cicatriz pigmentada. - Limitación al trayecto del
nervio afectado. - Alodinia en la zona de distribución, el dolor puede provocarse hasta con roce de
algodón. EL MANEJO DEL DOLOR ES PRIMORDIAL EN LA ENFERMEDAD DEL HERPES ZÓSTER. El
tratamiento combinado de aciclovir con glucocorticoides no es mejor que el de aciclovir solo para
el alivio de las lesiones agudas, ni en la prevención de la neuralgia postherpética, algunos casos
aislados no diabéticos, no hipertensos y sin úlcera péptica, pueden beneficiarse, pero este beneficio
no es reconocido en general. EL USO DE GLUCOCORTICOIDES PARA EL MANEJO DE LOS SÍNTOMAS
DEL HERPES SIMPLE NO ESTÁ BIEN ESTABLECIDO, ADEMÁS DE QUE DEBERÁ ASOCIARSE SIEMPRE
CON ACICLOVIR.

ACICLOVIR El ACICLOVIR tiene efectividad probada para el manejo del HERPES ZÓSTER. - Reduce de 3 a 2 días
la liberación de partículas virales de las lesiones. - Las nuevas lesiones seden en menos de 10 días
en el 70% de los tratados, y en el 50% de los no tratados. - EL 50% de los pacientes sanas en 7.1
días comparado con el 50% de los no tratados. - Acelera la curación del exantema “disminuye la
intensidad y duración del dolor agudo”. El Aciclovir y el Famciclovir, "no han demostrado prevenir
la Neuropatía Herpética (NPH)" al compararse con placebo en el desenlace de NPH presente a 6
meses del inicio del Hepes Zoster (HZ). Aunque se sabe que hay mejoría de la intensidad del dolor
en los pacientes tratados con Aciclovir a 2 y 4 meses. EL ACICLOVIR HA DEMOSTRADO
DISMINUCIÓN DEL DOLOR PERO NO PREVENCIÓN DE LA NEUROPATÍA POR HERPES ZOSTER.

FLUOXETINA Los recursos con los que se cuenta para el manejo del dolor en el HERPES ZÓSTER son: analgésicos
comunes, analgésicos opiodes, moduladores del dolor y tratamientos invasivos. Los moduladores
del dolor están indicado en aquellos casos que no mejoran completamente con a administración de
opioides, para este fin están indicados: gabapentina o pregabalina, antidepresivos tricíclimos y
glucocorticoides. LA FLUOXETINA NO ESTA RECOMENDADA COMO PARTE DEL MANEJO DEL
DOLOR EN HERPES ZÓSTER.

BRIVUDINA Los principales objetivos del tratamiento del HERPES ZÓSTER SON: 1. Supresión de la replicación
viral, especialmente en inmunocomprometidos y lo que tienen afección oftálmica. 2. Tratamiento
del dolor agudo. 3. Prevención y tratamiento de la neuralgia postherpética. 4. Atención de las
comorbilidades. ADEMÁS DEL MANEJO DEL DOLOR, QUE ES PRIMORDIAL EN ESTOS CASOS, SE
TIENEN POR OBJETIVO PREVENIR Y TRATAR LA NEURALGIA POSTHERPÉTICA. El uso de antivirales
sistémicos en todos los pacientes con HERPES ZÓSTER está justificado en: - Mayores de 50 años. -
Dolor moderado a grave. - Exantema moderado o grave. - Afección de cabeza y extremidades. EL
MANEJO CON ANTIVIRALES ESTÁ BIEN JUSTIFICADO EN ESTE CASO DEBIDO A QUE PRESENTA
LESIONES EN LA CARA. La BRIVUDINA ha sido probada con igual eficacia que el Aciclovir, tienen
una administración más sencilla pero con la limitante de mayor costo. No ha sido aprobada por la
FDA pos su efecto adverso al inhibir el catabolismo de pirimidinas fluorinadas y causar supresión
fatal de la médula ósea. INDEPENDIENTEMENTE DE QUE TENGA EFECTOS SIMILARES AL
ACICLOVIR, NO DEBERÁS TOMARLA EN CUENTA COMO OPCIÓN TERAPÉUTICA POR SUS EFECTOS
ADVERSOS Y ALTO COSTO.

Bibliografía:
1. GUÍA DE PRÁCTICA CLÍNICA, PREVENCIÓN, DIAGNÓSTICO Y TRATAMIENTO DEL HERPES ZÓSTER EN EL
ADULTO. MÉXICO: SECRETARIA DE SALUD, 2008.

http://www.cenetec.salud.gob.mx/descargas/gpc/CatalogoMaestro/217_SSA_09_Herpes_Zoster/EyR_SSA_217_09.pdf

258 - A PESAR DEL TRATAMIENTO 3 MESES DESPUÉS, EL PACIENTE PRESENTA NEURITIS HERPÉTICA. EN
ESTE CASO USTED INDICARÍA:
PULSOS DE Los siguientes moduladores del dolor se han usado en casos que no mejoran
METILPREDNISOLONA completamente con opiodes: 1. Gabapentina o pregabalina. 2. Antidepresivos tricíclicos. 3.
Glucocorticoides.

TRAMADOL TRAMADOL - En la neuralgia postherpética es de utilidad el tramadol. - Debe vigilarse


estrechamente su uso en pacientes seniles por los efectos indeseables, el riesgo de caídas y
de desorientación.

GABAPENTINA GABAPENTINA - Gabapentina y Pregabalina son útiles en el tratamiento del dolor. - La


Gabapentina es útil en cualquier grado de neuritis. SI BIEN, LA NEURITIS POSTHERPÉTICA
PUEDE SER BIEN TRATADO CON TRAMADOL, CARBAMAZEPINA Y ANTIDEPRESIVOS
TRICÍCLICOS (MENOS FLUOXETINA) LA GABAPENTINA ES LA ÚNICA RESPUESTA
CORRECTA DENTRO DE LAS OPCIONES PARA ÉSTE MANEJO.

FLUOXETINA Los antidepresivos tricíclicos, mejoran en algún grado la NPH. Se ha utilizado con buena
respuesta la Venlafaxina y Nortriptilina. La Fluoxetina (no es un antidepresivo triciclico),
no está recomendada para el manejo de la NPH.

Bibliografía:
1. GUÍA DE PRÁCTICA CLÍNICA, PREVENCIÓN, DIAGNÓSTICO Y TRATAMIENTO DEL HERPES ZÓSTER EN EL
ADULTO. MÉXICO: SECRETARIA DE SALUD, 2008.

http://www.cenetec.salud.gob.mx/descargas/gpc/CatalogoMaestro/217_SSA_09_Herpes_Zoster/EyR_SSA_217_09.pdf

FIN DEL CASO CLÍNICO SERIADO


ANÁLISIS DEL CASO CLÍNICO

IDENTIFICACIÓN DEL REACTIVO


Area: MEDICINA INTERNA
Especialidad: PSIQUIATRÍA
Tema: TRANSTORNO DE LA ALIMENTACIÓN Y ADICCIONES
Subtema: ADICCIONES Y DROGADICCIÓN

CASO CLÍNICO CON UNA PREGUNTA

HOMBRE DE 19 AÑOS DE EDAD, QUE ES LLEVADO A SU CONSULTA CON APARENTES TRASTORNOS DEL
COMPORTAMIENTO SECUNDARIOS A LA INHALACIÓN DE COCAÍNA.

hombre de 19 años.

consumo de cocaína.

trastornos del comportamiento


secundarios.

-.

-.

259 - LOS EFECTOS DE ESTA SUSTANCIA SOBRE EL COMPORTAMIENTO DE LA PERSONA QUE LA


CONSUME ES DEBIDO A QUE:

DISMINUYE LOS El uso crónico de Alcohol se ha observado en relación a la larga de la disminución de los
NIVELES DE niveles de glutamato.
GLUTAMATO.

INHIBE LA Esta opción esta en relación hacia algunos medicamentos anticonvulsivos que tienen este
PRODUCCIÓN DE efecto, pero no mencionados en el uso de la cocaína.
GABAPENTINA.

BLOQUEA LOS Los endocannabinoides se han descrito con la interacción de los canales de calcio, lo que
CANALES DE CALCIO Y puede ejercer efectos sobre los neurotransmisores, pero de igual forma no se mencionan
LA CAPTURA DE estos efectos con el consumo de cocaína.
NEUROTRANSMISORES.
BLOQUEA LA La COCAÍNA es un estimulante y analgésico local con potentes propiedades
RECAPTURA DE vasocontritoras. Sus efectos dependen de sus efectos en las neuronas dopaminérgicas en
DOPAMINA Y el sistema mesolímbico. Ahí la cocaína intensifica las concentraciones sinápticas de los
SEROTONINA. neurotransmisores monoaminíticos, “DOPAMINA, NORADRENALINA Y SEROTONINA”
por medio de la “unión a la proteína transportadora” en las neuronas presinápticas y
“BLOQUEANDO SU RECAPTACIÓN”. Tiene un efecto activador específico sobre las vías
dopaminérgicas mesolímbicas o mesocorticales. Las vías dopaminérgicas como el área
tegmental ventral, el lóbulo frontal, el septum, la amígdala y el núcleo accumbens, son
importantes en el reforzamiento de la conducta.

Bibliografía:
1. LONGO DL, FAUCI AS, KASPER DL, HAUSERSL, JAMESON JL, LOSCALZOJ. HARRISON. PRINCIPIOS DE
MEDICINA INTERNA, 18A EDICIÓN. MC GRAW HILL. NEW YORK, USA. 2012, PP 3556.
ANÁLISIS DEL CASO CLÍNICO

IDENTIFICACIÓN DEL REACTIVO


Area: PEDIATRÍA
Especialidad: INFECTOLOGIA PEDIÁTRICA
Tema: INFECCIONES VÍAS RESPIRATORIAS ALTAS
Subtema: OTITIS EXTERNA AGUDA

CASO CLÍNICO SERIADO

PREESCOLAR FEMENINA DE 5 AÑOS DE EDAD, NADADORA, PADECIMIENTO DE 24 HRS CON DOLOR EN OÍDO
DERECHO, FIEBRE DE 38 °C, Y SALIDA DE SECRECIÓN BLANQUECINA POR EL CONDUCTO. A LA EXPLORACIÓN
CON DOLOR A LA PRESIÓN DEL TRAGO Y A LA MOVILIZACIÓN DEL PABELLÓN. CONDUCTO AUDITIVO
HIPERÉMICO, CON ESCASA SECRECIÓN BLANQUECINA. MEMBRANA TIMPÁNICA DE COLOR PERLADO.

PREESCOLAR DE 5 AÑOS

NADADORA, ANTECEDENTE MUY


IMPORTANTE PARA OTITIS EXTERNA

MUY IMPORTANTES DOLOR A LA PRESIóN


DEL TRAGO Y A LA MOVILIZACIóN DEL
PABELLóN, CLÁSICOS DE LA OTITIS
EXTERNA.

VISUALIZACIÓN DEL CONDUCTO


EXTERNO HIPERÉMICO Y MEMBRANA
NORMAL

260 - A LA PACIENTE SE LE REALIZA EL DIAGNÓSTICO DE:

MIRINGITIS MIRINGITIS La miringitis es la inflamación de la membrana timpánica. Es frecuente observar


AMPOLLOSA hiperemias timpánicas con otalgia en el curso de cuadros víricos de vías altas, sin presencia de
exudado del oído medio. Una vez más, el timpanómetro nos ofrecerá el diagnóstico diferencial
entre simple miringitis y OMA. Estas formas catarrales no deben recibir tratamiento antibiótico,
aunque debe vigilarse su evolución, dado que pueden evolucionar hacia OMA. La visualización de
bullas timpánicas (que hay que diferenciar de la presencia de niveles líquidos retrotimpánicos
propios de formas crónicas de otitis media con exudado) asociada a un intenso dolor es diagnóstica
de miringitis bullosa o ampollosa. La miringitis ampollosa es una forma de otitis media con
participación de la membrana timpánica, que si bien clásicamente se consideró debida a
Mycoplasma Pneumonae, se ha observado que es causada más frecuentemente por los gérmenes
habituales productores de OMA (neumococo, Haemophilus).
OTITIS OTITIS EXTERNA AGUDA. La clínica se caracteriza fundamentalmente por el dolor, precedido a
EXTERNA veces de prurito, con los clásicos signos del trago y del pabellón muy positivos. Suele ser unilateral.
AGUDA Habitualmente, hay disociación clínico-otoscópica, puesto que el niño tiene un dolor muy superior
a los escasos o nulos signos inflamatorios que pueden visualizarse en fases iniciales.
Posteriormente, aparece hiperemia de intensidad variable de la piel del conducto auditivo externo
y estrechamiento hasta del 50% de la luz del mismo, con dificultades para visualizar la membrana
timpánica. Es frecuente la hipoacusia y no es rara la supuración. La fiebre no es común y cuando
aparece suele traducir infección mixta estafilococo-pseudomonas. Puede acompañarse de
adenopatías preauriculares. LA PACIENTE PRESENTA UN CUADRO TÍPICO, LA MEMBRANA
TIMPÁNICA NORMAL ES FUNDAMENTAL PARA ESTE DIAGNÓSTICO.

OTITIS OTITIS MEDIA AGUDA (OMA). El diagnóstico diferencial con la OMA no siempre es fácil,
MEDIA especialmente cuando hay supuración que impide la valoración de la membrana timpánica. El dolor
AGUDA suele mejorar cuando hay perforación en la OMA y en cambio empeora con la otorrea en la otitis
externa. En caso de duda, el timpanómetro es un aliado imprescindible, al verificarnos si existe o no
perforación timpánica con la valoración del volumen de oído medio. Los signos del trago y del
pabellón son propios de la otitis externa y no de la OMA. En la OMA suele haber un antecedente
catarral y es más frecuente la fiebre. La otitis externa es más propia del verano.

MASTOIDITIS MASTOIDITIS. Si la otitis externa cursa con edema periauricular, debe diferenciarse de la
mastoiditis, en la que suele haber un episodio previo de OMA, hipoacusia, dolor a la presión de la
apófisis mastoidea y ausencia de signo del trago. En la otitis externa, se conserva el surco
postauricular, que puede borrarse en la mastoiditis.

Bibliografía:
ATENCIÓN PRIMARIA EN PEDIATRÍA. ROBERT A. HOEKELMAN. MOSBY. EDICIÓN 3A. 1997. PAG. 1475.

261 - EL TRATAMIENTO DE ESTA PACIENTE ES CON ANALGÉSICOS Y:

ANTIBIÓTICO TÓPICO OTITIS EXTERNA AGUDA. El tratamiento fundamental es el analgésico, puesto que el dolor
Y ESTEROIDES es el signo capital. La utilización de paracetamol o ibuprofeno a dosis adecuadas suele ser
TÓPICOS suficiente. Es útil el calor local. En casos más importantes, se obtiene buena respuesta con
la asociación de paracetamol y codeína. Si hay mucho material orgánico y exudado, debe
procederse a una limpieza del conducto auditivo con cureta o irrigación y, si no es posible
hacerlo en la consulta, el paciente debe ser remitido al especialista ORL. De no hacerse esta
limpieza, el ATB tópico no va a contactar con la pared del conducto y el tratamiento va a
fracasar. Si el conducto está permeable, hay que administrar ATB local. Clásicamente, se
han utilizado preparados que asocian aminoglucósidos (neomicina, polimixina o colistina)
con corticoides, a razón de 2 gotas, 4 veces al día durante 7 días. No hay estudios que
comparen la ventaja adicional de la utilización de corticoides con respecto al uso solo de
ATB. Para el uso de estos preparados clásicos, hay que tener la absoluta certeza de que la
membrana timpánica no está perforada, Por otra parte, se han descrito reacciones de
sensibilización a los aminoglucósidos tópicos. El ciprofloxacino ótico es igualmente eficaz,
y pese a que su coste es muy superior, parece la opción más recomendable en la
actualidad, por su mejor tolerancia, dosificación más cómoda (2 veces al día) y ausencia de
efectos secundarios y sensibilización. El niño con la habitual otitis externa leve puede
seguir bañándose, siempre que no bucee en el curso del tratamiento.

DRENAJE Y MASTOIDITIS AGUDA. Ésta se caracteriza por extensión del proceso infeccioso a las
ANTIBIOTICOTERAPIA celdillas mastoideas con acumulación de secreción purulenta, osteítis y destrucción de las
INTRAVENOSA trabéculas óseas formando una cavidad común. Ésta se maneja como un absceso mediante
drenaje (mastoidectomía) y antibioticoterapia intravenosa.
ANALGÉSICOS Y OTITIS MEDIA AGUDA (OMA) El tratamiento de la OMA debe empezar por una correcta
ANTIBIÓTICO VÍA analgesia. La utilización o no de ATB debe fundamentarse en un correcto diagnóstico de
ORAL. OMA (con la presencia de dolor o su equivalente en lactantes) y en no tratar otoscopias
positivas, sino la combinación de clínica específica y signos otoscópicos sugestivos,
preferentemente confirmados con otoscopia neumática, y si es posible, con
timpanometría. La amoxicilina a altas dosis es el tratamiento de 1ª elección. La cefuroxima
es la mejor opción si se utiliza una cefalosporina. Los macrólidos no deben utilizarse, salvo
en caso de alergia anafiláctica a la penicilina. Ceftriaxona puede ser una opción a
considerar en caso de precisarse la vía parenteral o como último recurso previo a la
derivación hospitalaria.

ANTIHISTAMÍNICOS Y MIRINGITIS AMPOLLOSA. El tratamiento inicial será idéntico al de la OMA. El fracaso


ESTEROIDES TÓPICOS terapéutico deberá hacer considerar la posibilidad de instaurar un macrólido.

Bibliografía:
ATENCIÓN PRIMARIA EN PEDIATRÍA. ROBERT A. HOEKELMAN. MOSBY. EDICIÓN 3A. 1997. PAG. 1475.

FIN DEL CASO CLÍNICO SERIADO


ANÁLISIS DEL CASO CLÍNICO

IDENTIFICACIÓN DEL REACTIVO


Area: MEDICINA INTERNA
Especialidad: GASTROENTEROLOGÍA
Tema: PATOLOGÍA HEPÁTICA Y PANCREATICA
Subtema: PATOLOGIA DE PANCREAS

CASO CLÍNICO CON UNA PREGUNTA

MUJER DE 60 AÑOS CON DIAGNÓSTICO DE CÁNCER DE PÁNCREAS LIMITADO A LA COLA.

mujer de 60 años de edad.

cancer de pancreas limitado a la cola.

-.

-.

-.

262 - EL TRATAMIENTO DE ELECCIÓN SERÍA:

RESECCIÓN El ADENOCARCINOMA DE PÁNCREAS se define por la aparición de características de


QUIRÚRGICA malignidad en las células ductales que pueden, como en el resto de los cánceres, diseminarse
TOTAL. localmente o hacia otros órganos del cuerpo (metástasis); estas células se localizan con
mayor frecuencia en la cabeza de la glándula. Actualmente la cirugía más común para cáncer
de páncreas es la pancreatoduodenectomía de Whipple que está indicada para cáncer de la
cabeza de páncreas. Anteriormente se recomendaba la pancreatectomía total para los
pacientes con cáncer de páncreas, actualmente se recomienda más la parcial ya que la total
no ha mostrado mejorar la sobrevida y da como resultado insuficiencia pancreática tanto
endocrina como exocrina que ameritarán tratamiento. ACTUALMENTE SE PREFIEREN LAS
CIRUGÍAS PARCIALES A LA RESECCIÓN TOTAL.
QUIMIOTERAPIA. Los pacientes con cáncer de páncreas con enfermedad metastásica son candidatos para
quimioterapia, sin embargo este tratamiento nunca es curativo, solo es paliativo. Se debe
valorar la respuesta del tumor a la quimioterapia mediante estudios de imagen seriados,
marcador sérico ca 19-9 y síntomas relacionados con el tumor. Solo dos agentes
quimioterapéuticos se han asociado con aumento de la sobreviva y son el 5 fluoracilo y
gemcitabina. Es importante mencionar que aunque se utilice quimioterapia la sobrevida de
estos pacientes no se modifica de manera importante, solo 5 a 6 meses máximo. LA
QUIMIOTERAPIA ESTÁ CONSIDERADA TRAS LA PANCREATECTOMÍA Y EN OCASIONES
DUODENO- PANCREATECTOMÍA.

BRAQUITERAPIA. Esta técnica esta indicada para el tratamiento paliativo del dolor en los pacientes con cáncer
de páncreas. NO CORRESPONDE AL TRATAMIENTO DE ELECCIÓN.

PANCREATECTOMÍA Los casos de CÁNCER DE COLA DE PÁNCREAS deben ser tratados con resección de la cola del
PARCIAL. páncreas y asegurar que los bordes estén libres malignidad. También se deben tomar
ganglios linfáticos para descartar enfermedad metastásica. La resección quirúrgica es el
único tratamiento potencialmente curativo del cáncer de páncreas. Generalmente esta
enfermedad cuando se presentan síntomas ya está avanzada por lo que son candidatos para
pancreatectomía solo 15 a 20% de ellos. SE PREFIEREN LAS CIRUGÍAS DELIMITADAS AL
TUMOR, RARAMENTE ES NECESARIA UNA PANCREATECTOMÍA TOTAL.

Bibliografía:
1. GUÍA DE PRÁCTICA CLÍNICA, DIAGNÓSTICO Y TRATAMIENTO DEL ADENOCARCINOMA DE PÁNCREAS EN
EL ADULTO. MÉXICO: SECRETARIA DE SALUD; 2009.

http://www.cenetec.salud.gob.mx/descargas/gpc/CatalogoMaestro/324_IMSS_10_Adenocarcinoma_pancreas/EyR_IMSS_324_10.pdf
ANÁLISIS DEL CASO CLÍNICO

IDENTIFICACIÓN DEL REACTIVO


Area: PEDIATRÍA
Especialidad: INFECTOLOGIA PEDIÁTRICA
Tema: NEUMONÍAS
Subtema: NEUMONÍAS

CASO CLÍNICO CON UNA PREGUNTA

PACIENTE PREESCOLAR DE 4 AÑOS DE EDAD, LA CUAL PRESENTA UN CUADRO DE NEUMONIA BACTERIANA.

PREESCOLAR DE 4 AÑOS

DIAGNÓSTICO DE NEUMONÍA
BACTERIANA

263 - LA SIGUIENTE ASOCIACIÓN TIENE EL VALOR PREDICTIVO POSITIVO MÁS ALTO PARA REALIZAR EL
DIAGNÓSTICO DE NEUMONÍA:

FIEBRE-TOS- La clínica de las infecciones respiratorias altas y bajas es a menudo superponible; por lo que,
CREPITANTES basándose sólo en la clínica, no siempre es fácil establecer o descartar con seguridad el
diagnóstico de neumonía. La falta de fiebre tiene un valor predictivo negativo del 97% si la
temperatura no se ha modificado con antitérmicos. La asociación fiebre-tos-crepitantes es la que
tiene mayor valor predictivo positivo.
TOS- REPASO NEUMONÍA. Debe sospecharse una neumonía bacteriana en niños menores de 3 años
DIFICULTAD con fiebre mayor a 38.5° C, junto a retracciones costales y/o taquipnea mayor a 50/min, y estaría
RESPIRATORIA- indicada la práctica de radiografía de tórax. En niños mayores de esta edad, el distrés respiratorio
SIBILANCIAS tiene mayor valor que otros signos clínicos. La ausencia de todos los signos físicos mencionados
(auscultación anormal, taquipnea y distrés respiratorio) hace poco probable la neumonía y
podría evitarse la práctica de la radiografía de tórax. Debiera incluirse la pulsioximetría entre las
determinaciones iniciales, sobre todo en niños con estado general afectado o dificultad
respiratoria, ya que aumenta la sensibilidad para detectar la presencia de neumonía y ayuda a
valorar su gravedad. No es imprescindible la práctica de una radiografía para iniciar el
tratamiento en el niño con firme sospecha clínica de neumonía, pero ayuda a confirmar el
diagnóstico. En nuestro medio, parece razonable seguir recurriendo a la radiografía de tórax si
existe una sospecha clínica fundada de neumonía. La probabilidad de encontrar una radiografía
alterada en caso de fiebre sin sintomatología respiratoria es del 1-6%.

TAQUIPNEA- REPASO NEUMONÍA. De los signos físicos, la taquipnea, con un valor predictivo negativo del
TAQUICARDIA 85%, y el distrés respiratorio, son los mejores indicadores de la posibilidad de padecer una
Y DIFICULTAD neumonía. La severidad de la taquipnea está además relacionada con la gravedad del proceso. La
RESPIRATORIA taquipnea es el signo más útil para identificar neumonía en los niños entre 3 meses y 5 años,
aunque tiene una baja sensibilidad y especificidad en fases precoces de la enfermedad, si es éste
el único signo clínico que se valora. OJO LA TAQUIPNEA POR SI SÓLA ES DE GRAN VALOR.

FIEBRE ALTA- REPASO NEUMONÍA. La tríada: fiebre alta con escalofríos, tos y dolor en punta de costado, típica
TOS-DOLOR EN del adulto, puede encontrarse a veces en niños mayores, pero suele estar ausente o ser
PUNTA DE indemostrable en niños de menor edad. La radiografía está indicada: • En niños pequeños, para
COSTADO descartar o confirmar la sospecha clínica de neumonía, aun en ausencia de signos respiratorios. •
En niños mayores, sólo ante la presencia de clínica compatible (fiebre y signos respiratorios). • En
ambos casos, aun en ausencia de signos respiratorios, en el contexto de una fiebre prolongada e
inexplicada por otra causa.

Bibliografía:
TRATADO DE INFECCIONES EN PEDIATRÍA. NAPOLEÓN GONZÁLEZ SALDAÑA. MC GRAW-HILL
INTERAMERICANA. EDICIÓN 7A. 2004. PAG. 559,560.
ANÁLISIS DEL CASO CLÍNICO

IDENTIFICACIÓN DEL REACTIVO


Area: MEDICINA INTERNA
Especialidad: INFECTOLOGÍA
Tema: INFECCIONES FEBRILES SISTÉMICAS
Subtema: HIV Y SIDA

CASO CLÍNICO CON UNA PREGUNTA

MUJER DE 28 AÑOS DE EDAD, CON ANTECEDENTE DE EMBARAZO DE 10 SEMANAS DE GESTACIÓN, PRUEBA


DE VIH RÁPIDA HACE 2 SEMANAS NEGATIVA. ACUDE AL SERVICIO DE URGENCIAS SOLICITANDO SER
VALORADA, PUES REFIERE QUE HACE 5 DÍAS FUE VIOLADA EN LA CALLE POR AGRESOR DESCONOCIDO. SE
MUESTRA TEMEROSA ANTE LA POSIBILIDAD DE ADQUIRIR INFECCIÓN POR VIH Y SOLICITA SE LE ADMINISTRE
TRATAMIENTO PREVENTIVO.

28 añoS

Embarazo de 10 SDG, prueba rápida de VIH


negativa hace 2 semanas.

-.

-.

Solicita tratamiento preventivo para VIH.

264 - LA SIGUIENTE CONDICIÓN CONTRAINDICA LA PROFILAXIS PPE-VIH EN ESTE MOMENTO:


NO HAY Se denomina EXPOSICIÓN ACCIDENTEAL AL VIH al contacto no deseado ni esperado que se
EVIDENCIA DE produce con un “líquido potencialmente infectante” ya sea por vía parenteral a través de una
QUE LA lesión percutánea, por piel intacta o por membranas mucosas como las de la boca, ojos, vagina o
FUENTE SEA recto. Por su parte, la PROFILAXIS POST-EXPOSICIÓN al VIH (PPE-VIH) es la administración de
SEROPOSITIVA tratamiento antirretroviral (ARV) lo más temprano posible tras una “exposición accidental al VIH”
PARA VIH para evitar la infección por este virus, a través de una prescripción en las primeras 24 horas de la
exposición (idealmente en las primeras dos horas) con una combinación de tres ARV que deberá
administrarse durante 4 semanas; cuyos principales objetivos son suprimir al máximo la carga
viral, aunque sea muy baja y, evitar el deterioro de la respuesta celular del sistema inmunológico
para prevenir o abortar la infección temprana. Lo cual se logra mediante el uso de terapia ARV
triple para PPE-VIH en todas las exposiciones significativas. AUNQUE NO EXISTA EVIDENCIA DE
QUE LA FUENTE SEA SEROPOSITIVA PARA VIH, UNA VIOLACIÓN SIGNIFICA UNA EXPOSICIÓN A
LÍQUIDOS CORPORALES POTENCIALMENTE INFECTADOS. La aplicación de la PPE-VIH está
indicada en personas que estuvieron expuestas con pacientes VIH positivas o con probación con
factores de riesgo en los que se desconoce su serología para VIH o donde la seroprevalencia de
VIH se considera suficiente para justificar la toxicidad y costo del tratamiento. Los principales
factores de riesgo para este caso son: - Hombres que tienen sexo con hombres. - Hombres que
tienen sexo con hombres y mujeres trabajadores sexuales. - Personas que usan drogas
intravenosas. - Personas con historia de haber estado en la cárcel. - Personas provenientes de
países donde la seroprevalencia de VIH es 1% o mayor. - Exposición con personas que son parejas
sexuales de alguno de los grupos anteriores. - Personas con “antecedente de violación”. EL
ANTECEDENTE DE VIOLACIÓN SE CONSIDERA FACTOR DE RIESGO PARA EXPOSICIÓN CON EL
VIH, POR TAL MOTIVO DEBERÁ INICIARSE PROFILAXIS POST-EXPOSICIÓN al VIH (PPE-VIH)
INDEPENDIENTEMENTE DE QUE NO HAYA EVIDENCIA DE QUE LA FUENTE SEA SEROPOSITIVA
PARA VIH.

HAN PASADO NO está indicada la PROFILAXIS POST-EXPOSICIÓN al VIH (PPE-VIH) en las siguientes
MÁS DE 72HRS condiciones. 1. Si la persona expuesta (quien se accidenta) es VIH positivo. 2. Si la exposición no
DEL pone en riesgo la transmisión. - Exposición de piel intacta a fluidos potencialmente
MOMENTO DE contaminados. - Exposición a líquidos corporales no considerados de riesgo de transmisión
LA (heces, saliva, orina y sudor). - Exposición a líquidos corporales de paciente conocido como
EXPOSICIÓN negativo a VIH, a menos que la persona se identifique con exposición de riesgo reciente y así
encontrarse en el periodo de ventana para el diagnóstico. 3. Si el tiempo de exposición es mayor
de 72 horas. 4. Cuando el paciente no autorice el inicio de PPE-VIH. NO HAY EVIDENCIA QUE LA
ADMINISTRACIÓN DE ANTIRRETROVIRALES DESPUÉS DE LAS 72HRS DE LA EXPOSICIÓN A VIH
SEA EFECTIVA PARA PREVENIR LA INFECCIÓN. Las recomendaciones actuales para la PPE-VIH
incluyen Tenofovir/Emtricitabina más Raltegravir o Dolutegravir como esquema inicial preferido
debido a su excelente tolerabilidad, potencia probada y fácil administración, lo que mejora la
adherencia. Otros medicamentos alternativos, según el caso, son Zidovudina combinada con
Lamivudina en sustitución de Tenofovir combinado con Emtricitabina. Las alternativas del
Inhibidor de Integrasa son: Inhibidores de Proteasa (Atazanavir con Ritonavir, Lopinavir/Ritonavir
o Darunavir con Ritonavir), o Etravirina, entre otros. IMPORTANTE: - Cuando el trabajador
expuesto recibe PPE-VIH por riesgo alto al desconocer la serología para VIH del paciente fuente,
esta profilaxis se deberá suspender si se confirma que el “paciente fuente” es VIH negativo. -
Cuando el “paciente fuente” se conoce VIH positivo, para la selección del esquema de PPE-VIH
deberá tomarse en cuenta la historia de tratamientos ARV, la evolución de la carga viral, los
estudios genotípicos o fenotípicos de resistencia si están disponibles, y consultar con un experto
en el tratamiento de pacientes con VIH.

NO SE HA RECUERDA QUE las personas con antecedente de violación son consideradas como de alto riesgo
COMPROBADO para adquirir VIH. Hasta no demostrar lo contrario la sospecha de violación, sobre todo de
QUE HUBO agresor desconocido, es razón suficiente para considerarse en riesgo.
CONTACTO
DIRECTO CON
LÍQUIDOS
CORPORALES

EL ESTADO Si se decide iniciar la profilaxis PRE-VIH, se debe realizar una biometría hemática completa,
GRAVÍDICO DE pruebas de función renal, pruebas de función hepática y en su caso prueba de embarazo. No es
LA PACIENTE necesario esperar los resultados para iniciar la atención profiláctica, únicamente con relación a la
prueba de embarazo si se considera no utilizar Efavirenz en el esquema por su capacidad
potencialmente teratogénica. EL EMBARAZO NO CONTRAINDICA EL USO DE RETROVIRALES
PARA EXPOSICIÓN A VIH, RECUERDA QUE AÚN LAS EMBARAZADAS SEROPOSITIVAS DEBEN
RECIBIR TRATAMIENTO CON ÉSTOS DURANTE EN CASO DE INFECCIÓN.
Bibliografía:
1. PREVENCIÓN, DIAGNÓSTICO Y TRATAMIENTO DE LA EXPOSICIÓN LABORAL AL VIH EN TRABAJADORES
DE LA SALUD. GUÍA DE EVIDENCIAS Y RECOMENDACIONES: GUÍA DE PRÁCTICA CLÍNICA. MÉXICO.
SECRETARÍA DE SALUD. 16/03/2017.

http://www.cenetec-difusion.com/CMGPC/IMSS-241-12/ER.pdf
ANÁLISIS DEL CASO CLÍNICO

IDENTIFICACIÓN DEL REACTIVO


Area: GINECOLOGÍA Y OBSTETRICIA
Especialidad: GINECOLOGÍA
Tema: ENFERMEDADES DE TRANSMISIÓN SEXUAL E INFECCIONES PÉLVICAS
Subtema: ENFERMEDAD INFLAMATORIA PÉLVICA

CASO CLÍNICO SERIADO

MUJER DE 26 AÑOS DE EDAD CON ANTECEDENTE DE USO DE DIU DESDE HACE 4 MESES. PRESENTA
SANGRADO INTERMENSTRUAL Y LEUCORREA DE 2 MESES DE EVOLUCIÓN. HACE 3 DÍAS COMIENZA CON
DOLOR PÉLVICO, DISPAREUNIA, FIEBRE, NÁUSEAS Y MALESTAR GENERAL. A LA EXPLORACIÓN FÍSICA
PRESENTA TEMPERATURA 39 °C, TAQUICARDIA, PALIDEZ EN PIEL, FACIES DOLOROSA, DOLOR ABDOMINAL
INTENSA DE PREDOMINIO EN HIPOGASTRIO Y FOSA ILÍACA IZQUIERDA, RESISTENCIA MUSCULAR Y
DISMINUCIÓN DE RUIDOS INTESTINALES. SE REALIZA ECOGRAFÍA TRANSVAGINAL DONDE SE APRECIA UNA
COLECCIÓN EN EL FONDO DE SACO DE DOUGLAS DE 6 X 5 CM COMPATIBLE CON TUMORACIÓN O ABSCESO
DEPENDIENTE DE ANEXO IZQUIERDO CON BORDES MAL DEFINIDOS Y LÍQUIDO LIBRE. EL DOPPLER REVELA
AUMENTO DE FLUJO EN OVARIO IZQUIERDO.

MUJER 26 años.

sexualmente activa, DIU.

dolor pélvico, dispareunia, ebre, náuseas y


malestar general.

ebre, taquicardia, palidez en piel, facies


dolorosa, datos de irritación peritoneal.

US con datos de absceso pélvico y líquido


libre en cavidad.

265 - ES LA MANIFESTACIÓN CLÍNICA MÁS FRECUENTEMENTE ENCONTRADA EN PACIENTES CON ESTA


PATOLOGÍA:

DOLOR RECUERDA EN CASOS COMO ESTE QUE NO TE MENCIONAN EL DIAGNÓSTICO DEBERÁS DEFINIR
ABDOMINAL. ESTE COMO PRIMER PASO, DE AHÍ SEGUIR LA LÍNEA DEL DIAGNÓSTICO PARA RESOLVER EL RESTO
DE REACTIVOS. En un estudio de cohorte presentado en las GPC se demostró que los síntomas más
comunes en la ENFERMEDAD PÉLVICA INFLAMATORIA (EPI) son: • Dolor abdominal (90%), •
Leucorrea (70%), • Sangrado irregular (40%), • 30% de las pacientes presentaba antecedente de
Dispositivo Intrauterino (DIU).
LEUCORREA. Se sugiere realizar examen bimanual en población sexualmente activa con riesgo de enfermedad de
transmisión sexual y dolor pélvico ante los siguientes datos de EPI: dolor abdominal bajo,
dispareunia, leucorrea, sangrado transvaginal anormal y fiebre.

SANGRADO La ENFERMEDAD INFLAMATORIA PÉLVICA es un síndrome clínico que consiste en dolor abdominal
IRREGULAR. bajo, flujo vaginal y que se presenta por la infección ascendente de gérmenes procedentes del
cérvix con mayor incidencia en las mujeres con prácticas sexuales de riesgo con un espectro de
gravedad, desde muy leve hasta potencialmente letal, incluyendo endometriosis, paremetritis,
salpingitis, ooforitis, absceso tuboovarico y peritonitis. Los principales gérmenes involucrados son
la Chlamydia tracomatis y Neisseria gonorrheae, ocasionalmente se asocian agentes anaerobios y
bacterias facultativas encontradas en la vaginosis bacteriana. NINGÚN SIGNO O SÍNTOMA SE
CONSIDERA ESTÁNDAR DE ORO PARA EL DIAGNÓSTICO DE EPI.

FIEBRE. “Debe considerarse caso sospechoso, toda mujer que presente dolor abdominal bajo con o
síntomas acompañantes”.

Bibliografía:
1. GUÍA DE REFERENCIA RÁPIDA, DIAGNÓSTICO Y TRATAMIENTO DE LA ENFERMEDAD INFLAMATORIA
PÉLVICCA EN MUJERES MAYORES DE 14 AÑOS CON VIDA SEXUAL ACTIVA. MÉXICO: SECRETARIA DE
SALUD; 2009. 2. GUÍA DE PRÁCTICA CLÍNICA, DIAGNÓSTICO Y TRATAMIENTO DE LA ENFERMEDAD
INFLAMATORIA PÉLVICCA EN MUJERES MAYORES DE 14 AÑOS CON VIDA SEXUAL ACTIVA. MÉXICO:
SECRETARIA DE SALUD; 2009.

http://www.cenetec.salud.gob.mx/descargas/gpc/CatalogoMaestro/072_GPC_EnfInfPelvica/ENF_INFLAMATORIA_PELVICA_EVR_CENETEC.pdf

266 - CON BASE EN LOS HALLAZGOS Y MANIFESTACIONES CLÍNICAS ENCONTRADAS EN LA PACIENTE SU


SEVERIDAD DEBERÁ CLASIFICARSE COMO:

GRADO La clasificación de ENFERMEDAD PÉLVICA INFLAMATORIA (EPI) orienta al tipo de intervención a realizar;
I. de acuerdo a la respuesta inflamatoria sistémica y la presencia de abdomen agudo. 1. Grado I (Leve) NO
COMPLICADA: • Sin masa anexial • Ni datos de abdomen agudo, ni irritación peritoneal

GRADO 2. Grado II (Moderada) COMPLICADA: • Con masa anexial o absceso que involucra trompas y/o ovarios. •
II. Con o sin signos de irritación peritoneal.

GRADO 3. Grado III (Grave) DISEMINADA A ESTRUCTURAS EXTRA PÉLVICAS: • Absceso tubo-ovárico roto o pelvi
III. peritonitis • Con datos de respuesta sistémica LA PACIENTE TIENE DATOS CLÍNICOS E IMAGENOLÓGICOS
DE PERITONITIS CON ATAQUE AL ESTADO GENERAL.

GRADO Tomar en cuenta que la clasificación de EPI considera la respuesta inflamatoria sistémica y la presencia de
IV. abdomen agudo para la identificación de la gravedad y orientación sobre el tipo de intervención a
realizar. NO EXISTE GRADO IV EN LA CLASIFICACIÓN DE LA ENFERMEDAD PÉLVICA INFLAMATORIA.

Bibliografía:
1. GUÍA DE REFERENCIA RÁPIDA, DIAGNÓSTICO Y TRATAMIENTO DE LA ENFERMEDAD INFLAMATORIA
PÉLVICCA EN MUJERES MAYORES DE 14 AÑOS CON VIDA SEXUAL ACTIVA. MÉXICO: SECRETARIA DE
SALUD; 2009. 2. GUÍA DE PRÁCTICA CLÍNICA, DIAGNÓSTICO Y TRATAMIENTO DE LA ENFERMEDAD
INFLAMATORIA PÉLVICCA EN MUJERES MAYORES DE 14 AÑOS CON VIDA SEXUAL ACTIVA. MÉXICO:
SECRETARIA DE SALUD; 2009.

http://www.cenetec.salud.gob.mx/descargas/gpc/CatalogoMaestro/072_GPC_EnfInfPelvica/ENF_INFLAMATORIA_PELVICA_EVR_CENETEC.pdf

267 - EL HOSPITAL NO CUENTA CON MEDICAMETOS DE PRIMERA LÍNEA PARA ESTE CASO POR LO QUE
DEBERÁ ADMINSITRARSE EL SIGUIENTE ESQUEMA:
METRONIDAZOL OJO: DEBES PONER ESPECIAL ATENCIÓN AL REACTIVO, "EL HOSPITAL NO CUENTA CON
Y MEDICAMENTOS DE PRIMERA LÍNEA" Y ADEMÁS CONSIDERAR QUE SE TRATA DE UNA "EPI
LEVOFLOXACINO. GRADO III". El tratamiento debe incluir antimicrobianos útiles contra: • Neisseria gonorrhoeae
• Chlamydia trachomatis • Gram negativos • Anaerobios y estreptococos El METRONIDAZOL
puede agregarse a cualquier esquema básico de manejo en EPI ambulatoria a dosis de 500mg
vía oral 2 veces al día por 14 días para cubrir anaerobios. La LEVOFLOXACINA está indicada en
caso de que la terapia con cefalosporinas no fuese posible y existe bajo riesgo de gonorrea a
dosis de 500mg una vez al día por 14 días en casos de manejo AMBULTORIO. DADO QUE EL
CUADRO CLÍNICO CORRESPONDE A UNA EPI GRAVE EL MANEJO AMBULATORIO NO ESTÁ
INDICADO.

OFLOXACINO Y De acuerdo a la NOM y las GPC vigentes, el tratamiento para la EPI en pacientes ambulatorios
CLINDAMICINA. (leve a moderada) es: • Ofloxacina 400 mg VO cada 12 horas o • Levofloxacina 500 md diario
por 14 días más metronidazol 500 mg VO cada 12 horas por 14 días o • Clindamicina 450 mg
VO cada 6 horas por 14 días; EL OFLOXACINO CORRESPONDE AL MEDICAMENTO DE ELECCIÓN
PARA EL MANEJO DE EPI AMBULATORIA POR TANTO NO APLICA EN EPI GRAVE.

CEFTRIAXONA Y De acuerdo a la NOM, el tratamiento para la EPI en pacientes hospitalizados consiste en: •
DOXICICLINA. Ceftriaxona 250 mg im más doxiciclina 100 mg vo cada 12 horas por 14 días, o • Cefotetan 2 g
iv cada 12 horas más doxiciclina 100 mg vo cada 12 horas por 14 días. CORRESPONDE AL
MANEJO DE “PRIMERA ELECCIÓN” EN PACIENTES HOSPITALIZADOS, DEBE ADMINISTRARSE
COMO DOBLE ESQUEMA. “LA PREGUNTA REFIERE QUE NO LO TIENEN EN ESTE MOMENTO”

CLINDAMICINA Y De acuerdo a la NOM el régimen parenteral alternativo para el manejo de la EPI es: •
GENTAMICINA. Clindamicina a dosis de 900mg IV cada 8hrs con • Gentamicina a dosis de impregnación IV o IM
de 2mg/kg y de mantenimiento 1.5mg/gl cada 8hrs. CORRESPONDE AL MANEJO
ALTERNATIVO IDEAL PARA EL MANEJO DE LA EPI GRAVE DE FORMA HOSPITALARIA.

Bibliografía:
1. GUÍA DE REFERENCIA RÁPIDA, DIAGNÓSTICO Y TRATAMIENTO DE LA ENFERMEDAD INFLAMATORIA
PÉLVICCA EN MUJERES MAYORES DE 14 AÑOS CON VIDA SEXUAL ACTIVA. MÉXICO: SECRETARIA DE
SALUD; 2009. 2. GUÍA DE PRÁCTICA CLÍNICA, DIAGNÓSTICO Y TRATAMIENTO DE LA ENFERMEDAD
INFLAMATORIA PÉLVICCA EN MUJERES MAYORES DE 14 AÑOS CON VIDA SEXUAL ACTIVA. MÉXICO:
SECRETARIA DE SALUD; 2009.

http://www.cenetec.salud.gob.mx/descargas/gpc/CatalogoMaestro/072_GPC_EnfInfPelvica/ENF_INFLAMATORIA_PELVICA_EVR_CENETEC.pdf

FIN DEL CASO CLÍNICO SERIADO


ANÁLISIS DEL CASO CLÍNICO

IDENTIFICACIÓN DEL REACTIVO


Area: PEDIATRÍA
Especialidad: URGENCIAS PEDIÁTRICAS
Tema: URGENCIAS QUIRÚRGICAS Y PATOLOGIAS DE RESOLUCIÓN
QUIRÚRGICA
Subtema: INVAGINACIÓN INTESTINAL

CASO CLÍNICO SERIADO

LACTANTE MASCULINO DE 5 MESES DE EDAD QUE ES LLEVADO A URGENCIAS PORQUÉ DESDE HACE 16 HRS
PRESENTA FIEBRE DE 39°C, IRRITABILIDAD, VOMITOS DE CONTENIDO GASTROBILIAR EN NUMERO DE 4 EN 16
HRS, DOLOR ABDOMINAL Y EVACUACIONES DISMINUIDAS DE CONSISTENCIA CON SANGRE. A LA
EXPLORACION PRESENTA FIEBRE DE 38.5°C, IRRITABILIDAD CON LLANTO INTENSO, ABDOMEN DOLOROSO A
LA PALPACION MEDIA Y PROFUNDA,CON MASA EN FORMA DE FORMA DE MORCILLA EN MESOGASTRIO Y
PERISTALSIS INCREMENTADA. SE REALIZA TACTO RECTAL CON SALIDA DE EVACUACIÓN CON SANGRE EN
JALEA DE GROSELLA.

lactante masculino de 5 meses

desde hace 16 hrs. con presencia de ebre


de 39°C, irritabilidad, vómitos de contenido
gastrobiliar en número de 4 en 16 hrs, dolor
abdominal y evacuaciones disminuidas de
consistencia con sangre

ebre de 38.5°c, irritabilidad con llanto


intenso, abdomen doloroso a la palpación
media y profunda , masa en forma de
morcilla en mesogastrio y peristalsis
incrementada.Tacto rectal con evacuación
en "jalea de grosella"

268 - EL DIAGNÓSTICO CLÍNICO MÁS PROBABLE ES:


GASTROENTERITIS La GASTROENTERITIS INFECCIOSA es una inflamación y/o disfunción del intestino
INFECCIOSA producida por un germen o sus toxinas, que da lugar a una alteración de su capacidad
para regular la absorción y secreción de sales y agua, produciendo diarrea por la
inhibición de la absorción o la estimulación de la secreción.Pueden deberse a multitud de
microorganismos patógenos entre los que se incluyen bacterias, virus y parásitos.La
gastroenteritis vírica es la causa más frecuente de diarrea infecciosa aguda en los niños
pequeños.La mayoría de los episodios de diarrea aguda son autolimitados y no necesitan
evaluación.El rotavirus es la causa más común de diarrea severa y gastroenteritis
deshidratante en niños. La diarrea constituye la cuarta causa muerte en menores de 5
años.Se conoce que alrededor del 95% de los niños se habrán infectado por lo menos una
vez al cumplir entre los 3 y 5 años de edad siendo mucho más frecuente, severa y
peligrosa antes de los 2 años edad y afecta sobre todo a niños que acuden a guarderías y
aquellos que viven en situación con hábitos higiénicos deficientes, aunque ningún niño o
niña están excentos de padecerla.Los principales síntomas de esta enfermedad son el
vómito y la diarrea, que pueden durar entre 1 y 7 días y presentar entre 20 o 30
evacuaciones por día, lo que provoca una deshidratación severa.Otro síntoma puede ser
la fiebre que se presenta en alrededor de la mitad de los casos.

ENTEROCOLITIS ENTEROCOLITIS PSEUDOMEMBRANOSA Es una infección del colón con una donde existe
PSEUDOMEMBRANOSA una proliferación excesiva de la bacteria Clostridium difficile.La bacteria Clostridium
difficile normalmente está presente en el intestino; sin embargo, puede proliferar en
exceso al tomar antibióticos. Las bacterias liberan una potente toxina que causa los
síntomas. El revestimiento del colon se inflama y sangra y toma una apariencia
característica llamada seudomembranas.La ampicilina, la clindamicina y las
cefalosporinas son los antibióticos más comunes asociados con esta enfermedad en los
niños. La colitis seudomembranosa es rara en bebés menores de 12 meses, gracias a que
tienen los anticuerpos protectores de la madre y debido a que la toxina no causa la
enfermedad en la mayoría de los bebés.La mayoría de los casos de colitis
seudomembranosa suceden mientras la persona está en el hospital, debido a que las
bacterias se pueden diseminar de un paciente a otro.Los principales síntomas son: dolor
abdominal tipo cólico(de leves a severos),heces con sangre,fiebre,diarrea acuosa (a
menudo de 5 a 10 veces por día)

INVAGINACIÓN La INVAGINACIÓN INTESTINAL es la introducción de una porción del intestino dentro del
INTESTINAL segmento del mismo inmediatamente distal. Es la causa mas frecuente de obstrucción
intestinal en los menores de 1 año de edad, con mayor frecuencia entre los 5° y 9° mes de
vida. Se puede presentar fuera de esta edad secundario a un divertículo de meckel,
tumores, adenitis mesentérica, pólipos etc. Pueden contar con el antecedente de cuadro
de infecciones respiratorias o intestinales. Los síntomas que aparecen en estos pacientes
son: dolor abdominal tipo cólico, diarrea con moco y sangre (apariencia de jalea de
grosella), vómitos, que inicialmente son gastroalimentario pero conforme avance el
tiempo de evolución se torna biliares o incluso fecaloides. A la exploración física se palpa
una masa en forma de morcilla o salchicha en cualquier parte del abdomen, puede
encontrarse prolapso rectal y al tacto rectal la presencia de evacuación con sangre. LA
PRESENCIA DE UNA MASA PALPABLE + EVACUACIÓN EN SANGRE DE GROSELLA SON
ELEMENTALES PARA SOSPECHAR UNA INVAGINACIÓN INTESTINAL.
ALERGIA A LAS La ALERGIA A LAS PROTEÍNAS DE LA LECHE es cuando un individuo, tras la ingesta de
PROTEÍNAS DE LA lácteos manifiesta síntomas que se pueden englobar dentro de las reacciones adversas a
LECHE alimentos y en ese proceso hay un mecanismo inmunológico comprobado.La leche
común esta constituida de proteínas carbohidratos, azúcar (lactosa), grasas, vitaminas y
minerales. Es la proteína la que ocasiona la reacción alérgica en la alergia a la proteína de
la leche de vaca. Esta alergia puede desarrollarse tanto en niños alimentados
exclusivamente con seno materno como en los que toman fórmula. Sin embargo, los que
se alimentan exclusivamente con leche materna, tienes menos probabilidades de
desarrollar alergias de cualquier tipo. Ocasionalmente, los bebes alimentados con seno
materno pueden desarrollar alergia a la proteína de la leche de vaca debido a las
pequeñas cantidades que pasan a la leche materna en una madre que si consume la leche
de vaca y/o sus derivados. En otros casos, algunos niños se sensibilizan a las proteínas de
la leche de vaca que le transfiere la madre por la leche materna pero no manifiestan
reacción sino hasta después que empiezan a tomar leche de vaca por ellos mismos. Los
síntomas más frecuentes pueden ser generales o específicamente gastroenterológicos.
Las manifestaciones generales se refieren a un cuadro de anafilaxia mediado por IgE, de
aparición muy rápida y cuya sintomatología puede ser intensa y violenta, acompañarse
de náuseas, vómitos, diarrea, fatiga, pérdida del conocimiento, llegando al shock y a la
muerte si no se brinda el tratamiento oportuno. En el caso de las gastrointestinales
podemos encontrar: a) lesiones orales por contacto al alimento, manifestándose por
prurito y edema de los labios y resto de la cavidad bucal; b) enteropatía, cuyos síntomas
más frecuentes son diarrea crónica con diferentes grados de malabsorción y falla del
crecimiento pondoestatural; c) gastroenteritis eosinofílica, que consiste en un cuadro
típico de náuseas, vómitos, dolor abdominal, diarrea, y que se caracteriza además por
presentar en la biopsia una infiltración eosinofílica de las paredes gástricas y/o
intestinales en ausencia de vasculitis, y eosinofilia en sangre periférica; d) colitis alérgica,
observada generalmente en lactantes menores, presentándose como rectitis o colitis
eosinofílica, con diarrea y sangre fresca en deposiciones; e) cólicos del lactante menor: el
10 a 15% de los pacientes que presentan cólicos responden al retirar la leche de vaca.
Ocasionalmente se han visto cuadros de reflujo gastroesofágico, constipación y
alteraciones del sueño como síntomas de esta patología.

Bibliografía:
1. DIAGNÓSTICO Y TRATAMIENTO PEDIATRICO. WILLIAM. MANUAL MODERNO. EDICIÓN 17. 2005. PAG. 616-
617.

269 - EL TRATAMIENTO QUE SE DEBE INICIAR EN ESTE PACIENTE ES CON:

ANTIBIOTICOTERAPIA En los pacientes con ENTEROCOLITIS PSEUDOMEMBRANOSA el tratamiento inicial es la


suspensión de los medicamentos antibióticos que provocaron la enfermedad. El
Metronidazol es considerado actualmente el tratamiento de primera línea, ya que es
barato, efectivo y con la administración oral del mismo se obtienen similares resultados en
cuanto a respuesta y tasa de recurrencias que al utilizar vancomicina. La dosis utilizada
varia entre 250-500 mg 4 veces al día o 500-750 mg 3 veces al día por 7-10 días. No se
recomienda su uso en niños pequeños y embarazadas. En los pacientes que no presentan
mejoría al cabo de 2-3 días y se han descartado otras patologías, debe cambiarse el
esquema antibiótico a vancomicina ya que algunas cepas de C.Difficile son resistentes a
metronidazol. La utilización de Vancomicina por vía oral se asocia con una tasa de
respuesta cercana al 90%, y es el tratamiento preferido en pacientes severamente
enfermos o en aquellos que no responden a la terapéutica inicial con metronidazol. La
dosis utilizada es de 125 mg 4 veces al día por 7-14 días, y debido a que es pobremente
absorbida por el tubo digestivo pueden obtenerse altas concentraciones en materia fecal
sin efectos adversos sistémicos.

CAMBIO DE Los pacientes con ALERGIA A LAS PROTEÍNAS DE LA LECHE alimentados al seno materno
FORMULA LÁCTEA pueden continuar su tratamiento con seno materno exclusivo, en el caso de que el niño sea
alimentado con leche maternizada, se deberá cambiar su leche por leche de soya o
hidrolizados de proteínas.
HIDRATACIÓN VÍA En los pacientes con GASTROENTERITIS INFECCIOSA, principalmente la ocasionada por
ENDOVENOSA Y rotavirus sobre todo en pacientes menores de 2 años de edad, se debe de implementar
VIGILANCIA medidas higienico-dietéticas e hidratación oral en su domicilio, pero la infección
ocasionada por este virus , la principal complicación es la deshidratación que incluso
puede llegar a deshidratación severa, choque y muerte. Cuando hay una deshidratacion de
moderada a severa es necesario la hospitalización para tratar la deshidratación con
líquidos intravenosos ,corregir el desequilibrio hidroelectrólitico y una vigilancia estrecha
para detectar cualquier complicación.

INSUFLACIÓN DE LA INSUFLACIÓN DE COLON O ENEMA CON BARIO ES EL ESTUDIO RADIOLÓGICO QUE


COLON O ENEMA NOS AYUDA EN EL DIAGNÓSTICO Y EL TRATAMIENTO EN LOS PACIENTES CON
CON BARIO INVAGINACIÓN INTESTINAL. El estudio se puede realizar con bario , medio hidrosoluble y
aire. Se realiza en los pacientes con invaginación intestinal de menos de 24 hrs de
evolucíon, con adecuado estado hídrico, y un estado neurológico normal. Contraindicado
en pacientes con somnolencia, deshidratación moderada a severa(choque), datos de
abdomen agudo (perforación).

Bibliografía:
1. DIAGNÓSTICO Y TRATAMIENTO PEDIATRICO. WILLIAM. MANUAL MODERNO. EDICIÓN 17. 2005. PAG. 616-
617.

FIN DEL CASO CLÍNICO SERIADO


ANÁLISIS DEL CASO CLÍNICO

IDENTIFICACIÓN DEL REACTIVO


Area: PEDIATRÍA
Especialidad: CRECIMIENTO Y DESARROLLO
Tema: ALTERACIONES DEL CRECIMIENTO
Subtema: TALLA BAJA

CASO CLÍNICO CON UNA PREGUNTA

RECIÉN NACIDO DE 2 DÍAS, ATENDIDA EN HOSPITAL PRIVADO DONDE NO LE REALIZAN EL TAMIZ NEONATAL.

recién nacido de 2 días de nacido

no se le realiza tamiz neonatal

--

--

--

270 - USTED LE EXPLICA A LA MADRE LA IMPORTANCIA DE DICHA PRUEBA DADO A QUE DETECTA
PRINCIPALMENTE LA SIGUIENTE ENFERMEDAD:

HIPOTIROIDISMO El tamizaje rutinario para hipotiroidismo congénito en neonatos ha sido considerado como
una práctica clínica costo-efectiva desde que el tratamiento temprano con hormona tiroidea
de reemplazo demostró que puede prevenir daño cerebral. En México a partir de 1988, la
secretaria de salud emitió la norma técnica que estableció la prevención del retardo mental
causado por hipotiroidismo congénito a través de la realización del examen de tamiz a todos
los recién nacidos y quedo incorporada con carácter de obligatoriedad en la norma oficial
mexicana en 1995. DEBERÁS ELEGIR ESTA RESPUESTA COMO CORRECTA YA QUE EL
HIPOTIROIDISMO ES LA PATOLOGÍA "MÁS FRECUENTEMENTE" DIAGNOSTICADA POR
MEDIO DEL TAMIZ NEONATAL.
HIPERINSULINISMO PRUEBAS DE LAS QUE CONSTA EL TAMIZ BÁSICO: • HIPOTIROIDISMO CONGÉNITO. •
Fenilcetonuria. PRUEBAS DE LAS QUE CONSTA EL TAMIZ AMPLIADO: • Deficiencia de
Biotinidasa. • Hiperplasia suprarrenal congénita. • GALACTOSEMIA. • DEFICIENCIA DE
GLUCOSA 6- FOSFATO- DESHIDROGENASA. • HIPOTIROIDISMO CONGÉNITO. • Fibrosis
quística. • Diferentes trastornos en el metabolismo de los aminoácidos. • Diferentes
trastornos en el metabolismo de ácidos orgánicos y ácidos grasos. "No incluye el diagnóstico
de hiperinsulinismo".

DEFICIENCIA DE El déficit de la Glucosa-6-fosfato deshidrogenasa (G-6-PDH) es la eritroenzimopatía más


GLUCOSA 6 común genéticamente determinada y la mejor conocida, tanto clínica como molecularmente.
FOSFATO Las personas portadoras de deficiencias de G-6-PDH al ser enfrentadas a sustancias
"oxidantes" (cuya lista es bastante grande) desarrollan episodios de hemodiálisis de
intensidad variable. Este mismo fenómeno puede ocurrir cuando sufren infecciones
preferentemente bacterianas. También se ha señalado una mayor frecuencia y severidad de
los episodios de ictericia neonatal en los portadores de esta anomalía. LA DEFICIENCIA DE
GLUCOSA 6 FOSFATO ES UNA PATOLOGÍA DETECTADA POR TAMIZ AMPLIADO PERO NO
CONSTITUYE LA PRIMERA EN ORDEN DE FRECUENCIA.

GALACTOSEMIA Se trata de una rara enfermedad autosómica recesiva caracterizada por la reducida capacidad
de convertir galactosa de la dieta en glucosa. Debido a la deficiencia de alguna de las tres
enzimas necesarias para la canalización de esta conversión (galactoquinasa, galactosa 1-
fosfato uridiltransferasa [GALT], uridindifosfato galactosa epimerasa [UDP]) puede estar
asociada con enfermedad clínica. La mayoría de los casos son resultado de la deficiencia de
GALT. Los síntomas generalmente aparecen en los primeros días o semanas de vida y puede
ser mortal debido a que causa hepatotoxicidad aguda, diátesis hemorrágica y predisposición
a sepsis por Escherichia Coli. LA GALACTOSEMIA CONSTITUYE UNA ENTIDAD "RARA" QUE
PUEDE SER DETECTADA POR LA PRUEBA DE TAMIZ NEONATAL.

Bibliografía:
1. ATENCIÓN PRIMARIA. CONCEPTOS, ORGANIZACIÓN Y PRÁCTICA CLÍNICA. MARTÍN ZURRO A. ELSEVIER.
EDICIÓN 5. 2003. PAG. 555-56.
Salir

ESCOLAR DE 6 AÑOS DE EDAD, ES LLEVADA A URGENCIAS POR LA PROBABLE


INGESTA ABUNDANTE DE SALICILATOS. A LA EXPLORACIÓN FÍSICA SE
ENCUENTRA CON TEMBLOR, SUDORACIÓN PROFUSA, ENROJECIMIENTO,
HIPERVENTILACIÓN, NÁUSEAS, VÓMITO Y SANGRADO GASTROINTESTINAL.

271 - EL MECANISMO FISIOPATOLÓGICO QUE DESENCADENA LA INTOXICACIÓN


POR SALICILATOS SE EXPLICA POR:

DESACOPLAMIENTO DE LA FOSFORILACIÓN OXIDATIVA


FORMACIÓN DE METABOLITOS INTERMEDIOS REACTIVOS
QUE EVITA LA DEGRADACIÓN DE ACETILCOLINA
PRODUCCIÓN DE DAÑO MITOCONDRIAL
Bibliografía:
1. NELSON TRATADO DE PEDIATRÍA. BEHRMAN. ELSEVIER. EDICIÓN 17A. 2004. PAG. 2366-68.

SEGURA DUDA NI IDEA


ANÁLISIS DEL CASO CLÍNICO

IDENTIFICACIÓN DEL REACTIVO


Area: GINECOLOGÍA Y OBSTETRICIA
Especialidad: OBSTETRICIA
Tema: DIABETES GESTACIONAL
Subtema: DIABETES GESTACIONAL

CASO CLÍNICO CON UNA PREGUNTA

MUJER DE 36 AÑOS, GESTA 3, CON ANTECEDENTE DE DOS ÓBITOS DESCONOCIENDO LA CAUSA. USTED
CONSIDERA LA POSIBILIDAD DE DIABETES GESTACIONAL.

-.

GESTA 3, ÓBITOS 2, SE CONSIDERA


POSIBILIDAD DE DIABETES GESTACIONAL.

-.

-.

-.

272 - EN ESTE CASO LO INDICADO ES SOLICITAR:

CURVA DE Se recomienda practicar pruebas de detección de diabetes gestacional en mujeres con:


TOLERANCIA Antecedentes obstétricos adversos de causa desconocida (óbito, aborto, polihidramnios, defectos
A LA congénitos, parto pretérmino, mortinato, macrosómico, hipoglucemia o hipocalcemia neonatal);
GLUCOSA síntomas sugestivos de diabetes mellitus, hiperglucemia aislada, glucosuria de repetición,
CON CARGA antecedentes de infertilidad, antecedente de Diabetes mellitus gestacional, antecedente de diabetes
DE 50 GRS mellitus familiar, edad materna avanzada, multiparidad, moniliasis recurrente y obesidad. LA
DE PACIENTE ES UNA MUJER AÑOSA, CON ANTECEDENTE DE ÓBITOS LO CUAL RESPALDA LA TOMA
GLUCOSA. DE ESTA MUESTRA. La prueba de tamiz para diabetes gestacional se realiza con una carga de 50mg
de glucosa no importando la hora ni el tiempo de ayuno o postprandio. No requiere preparación de
la paciente. Se medirá la glucemia a la hr y será positiva con un resultado mayor o igual a 130mg/dl
requiriendo entonces una curva de tolerancia a la glucosa. Si el resultado es mayor o igual a
180mg/dl será diagnóstico de diabetes gestacional y se deberá evitar la curva de tolerancia a la
glucosa. LA ADMINISTRAR UNA CARGA ORAL DE 50GR DE GLUCOSA NO ES PROPIAMENTE UNA
CURVA DE TOLERANCIA A LA GLUCOSA ES UN TEST DE SCREENING QUE SE LLAMA DE O'SULLIVAN.
CURVA DE Actualmente a nivel mundial considerando la recomendación de la OMS se tiene que utilizar la
TOLERANCIA curva de tolerancia a la glucosa con 75 g de glucosa y toma de muestra a los 0, 60 y 120 minutos
A LA por cuestiones de menor costo y resultados similares en cuanto a sensibilidad y especificidad. DE
GLUCOSA ACUERDO A LA OMS DEBE ESTANDARIZARSE A NIVEL INTERNACIONAL LA PRUEBA DE
CON CARGA TOLERANCIA A LA GLUCOSA CON 75GR PARA EMBARAZADAS.
DE 75 GRS
DE
GLUCOSA.

CURVA DE Criterios diagnósticos de O’ Sullivan y Mahan corresponde a la determinación de concentraciones


TOLERANCIA séricas de glucosa durante la Curva de Tolerancia a la Glucosa oral (CTGO) con 100 g a los 0, 60 y
A LA 120 minutos tras la ingesta de la glucosa. LA ADMINISTRACIÓN DE 100GR ES LA MÁS USUAL EN
GLUCOSA NUESTRO PAÍS, SIN EMBARGO DE ACUERDO A LOS LINEAMIENTOS INTERNACIONALES YA NO
CON CARGA DEBE UTILIZARSE.
DE 100 GRS
DE
GLUCOSA.

CURVA DE Una administración por arriba de los 100 gr de glucosa como carga para valorar la cuerva de
TOLERANCIA tolerancia a la glucosa no ha mostrado datos distintos de los que se encuentran a partir de la
A LA administración de 75 gr y si muestra incremento importante en los costos de la prueba.
GLUCOSA
CON CARGA
DE 125 GRS
DE
GLUCOSA.

Bibliografía:
1. DIAGNÓSTICO Y TRATAMIENTO DE LA DIABETES EN EL EMBARAZO. EVIDENCIAS Y RECOMENDACIONES.
GUÍA DE PRÁCTICA CLÍNICA. MÉXICO: SECRETARÍA DE SALUD; 03/11/2016. 2. CUNNINGHAM G, LEVENO K,
BLOMM S, HAUTH J, RPUSE D, SONG C. WILLIAMS OBSTETRICIA, 23A EDICIÓN. MC GRAW HILL. USA. 2011
EN ESPAÑOL.

http://www.cenetec-difusion.com/CMGPC/IMSS-320-10/ER.pdf
ANÁLISIS DEL CASO CLÍNICO

IDENTIFICACIÓN DEL REACTIVO


Area: PEDIATRÍA
Especialidad: INFECTOLOGIA PEDIÁTRICA
Tema: ENFERMEDADES EXANTEMÁTICAS
Subtema: VARICELA

CASO CLÍNICO SERIADO

RECIÉN NACIDA DE 10 DÍAS DE VIDA. ANTECEDENTE DE RUPTURA PREMATURA DE MEMBRANAS DE 4 HRS.


OBTENIDA POR PARTO EUTÓCICO. PADECIMIENTO DE 2 DÍAS DE EVOLUCIÓN CON FIEBRE DE 39°C, RECHAZO
A LA VÍA ORAL, IRRITABILIDAD E ICTERICIA POR LO QUE ES LLEVADA A URGENCIAS. A LA EXPLORACIÓN CON
ICTERICIA GENERALIZADA, SECRECIÓN BLANQUECINA EN AMBOS OJOS, LESIÓN ULCERADA EN CÓRNEA
DERECHA, MOVIMIENTOS TÓNICOS DE EXTREMIDADES SUPERIORES, CHUPETEO, VESÍCULAS CONFLUENTES
EN TRONCO, HEPATOMEGALIA DE 3-3-4 CM Y ESPLENOMEGALIA DE 3CM.

Femenino de 10 días de vida

OBTENIDA POR PARTO EUTÓCICO.

Rechazo a la vía oral, irritabilidad.


Compromiso neurológico.

Ictericia, lesiones oculares, crisis convulsivas


focalizadas, dermatosis diseminada
compatible con herpes.
Hepatoesplenomegalia. Compromiso
cutáneo, ocular, neurológico y
gastrohepático

273 - EL DIAGNÓSTICO MÁS PROBABLE ES:


INFECCIÓN POR La TOXOPLASMOSIS es la infección producida por el parásito intracelular estricto
TOXOPLASMOSIS Toxoplasma gondii, que se encuentra en la naturaleza y su hábitat es el intestino del gato.
Presenta 3 formas: Quiste tisular o hístico, que se encuentran en las carnes de vacuno y
cordero mal cocidas; ooquistes, forma infectante que se encuentra en las deposiciones del
gato; y trofozoítos. En niños puede ser de tipo congénita o adquirida. La “infección aguda
adquirida después del nacimiento” puede ser asintomática, pero se piensa que es resultado
de la persistencia crónica y persistente de quistes en los tejidos del hospedador, en tanto que
la “toxoplasmosis congénita” es una infección del recién nacido debida al paso del parásito,
de la madre infectada (al comer carne mal cocida con quistes y/o alimentos contaminados
con deposiciones de gatos) al producto, a través de la placenta. Se calcula que, en promedio,
sólo el 33% de las mujeres infectadas transmitirá el parásito al feto, el resto, dará a luz a
niños no infectados. La edad gestacional es determinante en la transmisión transplacentaria
del parásito al feto, pues se sabe que entre más cercana sea ésta al parto, mayor será la
gravedad de la infección en el recién nacido. Así bien, durante el primer trimestre se calcula
una transmisión alrededor del 15%, mientras que en el tercer trimestre la transmisión será
máxima alcanzando el 65%. Las lesiones provocadas por el parásito se asocian con muerte
celular y necrosis focal causada por los trofozoitos en replicación, y se caracterizan por una
intensa respuesta inflamatoria mononuclear en todos los tejidos infectados. En los lactantes
puede permanecer asintomática al momento del nacimiento, para posteriormente
manifestarse con signos y síntomas variados, siendo la linfadenopatía el signo más común.
La triada clásica de esta enfermedad la conforman: coriorretinitis, hidrocefalia y
calcificaciones cerebrales; éstas últimas suelen ser múltiples y aparecer por todo el cerebro
(difusas), incluso en la zona periventricular, pero con mayor afinidad por el núcleo caudado,
los ganglios basales, el plexo coroideo y la región subependimaria. Otros signos y síntomas
asociados son: anasarca fetal, muerte perinatal en feto con bajo peso, prematuridad,
cicatrices retinianas periféricas, ictericia persistente, trombocitopenia leve, pleocitosis de
líquido cefalorraquídeo y convulsiones. LA INFECCIÓN POR TOXOPLASMOSIS NO JUSTIFICA
LAS LESIONES DÉRMICAS DE LA PACIENTE.

INFECCIÓN La mayoría de los recién nacidos (RN) infectados con VIRUS DEL HERPES SIMPLE (HVS)
DISEMINADA POR presenta sintomatología, sin embargo, la media de diagnóstico varía entre los 5 y los 7 días
HERPES VIRUS posparto, ya que las manifestaciones clínicas de la enfermedad son inicialmente
inespecíficas. El 10 % de los RN, pese a ello, desarrollan sintomatología en el primer día de
vida. La infección neonatal por VHS se clasifica en tres categorías clínicas: 1. Enfermedad
cutánea/ocular/oral conocida como SEM disease (skin/eye/mouth): Es la forma menos severa
y típicamente se manifiesta por lesiones cutáneas en forma de vesículas aisladas o
agrupadas, bulas o denudación cutánea. Pueden seguir apareciendo lesiones cutáneas,
incluso tras el inicio del tratamiento. Se puede acompañar de úlceras bucales y de
queratoconjuntivitis o coriorretinitis como hallazgos oculares predominantes. Pueden
producirse manifestaciones cutáneas recidivantes en algunos de estos pacientes y parece,
que ello se relaciona con anomalías neurológicas lo que podría sugerir una afectación del
sistema nervioso central (SNC) indetectada. 2. Enfermedad localizada del SNC: La fiebre suele
estar presente en el 44 % de estos pacientes y puede ser la única manifestación clínica.
Además, pueden presentar hipotermia, letargia, apnea, irritabilidad, convulsiones focales o
generalizadas, opsoclonus, signos piramidales o pérdida del reflejo de succión. En un tercio
de estos pacientes no existen lesiones orales o cutáneas asociadas que sugieran el
diagnóstico. El análisis del LCR muestra una pleocitosis mononuclear, a pesar de que se han
demostrado casos con predominio neutrofílico e incluso con recuentos celulares normales.
Suele existir hiperproteinorraquia con glucosa normal o ligeramente disminuida. Es
frecuente el hallazgo de alteraciones electroencefalográficas focales o generalizadas.
Aproximadamente el 15 % de los casos de encefalitis por VHS tipo 2 fallecen, pero la
mortalidad de la encefalitis por VHS-1 es prácticamente nula. Dos tercios de los
supervivientes tienen secuelas neurológicas a largo plazo. 3. Enfermedad diseminada: Se
considera enfermedad diseminada si existe evidencia de afectación visceral incluyendo
hepatitis, neumonitis o coagulación intravascular diseminada. Los órganos más afectados son
el hígado, el corazón, los pulmones, las glándulas suprarrenales y el tracto digestivo. En dos
tercios de los casos diseminados existe también afectación del sistema nervioso central. La
tasa de mortalidad es del 55 % pese al tratamiento con Aciclovir y del 80 % sin tratamiento.
La quinta parte de los que sobreviven presentan graves secuelas neurológicas. SE DEBE
CONSIDERAR LA POSIBILIDAD DE INFECCIÓN HERPÉTICA EN TODO NEONATO CON
SOSPECHA CLÍNICA DE SEPSIS, RESULTADOS NEGATIVOS DE LOS CULTIVOS BACTERIANOS Y
DISFUNCIÓN HEPÁTICA. LA AFECCIÓN, PRÁCTICAMENTE GENERALIZADA DEL PACIENTE,
ASOCIADA A VESÍCULAS EN LA PIEL DIRIGEN EL DIAGNÓSTICO DE UNA INFECCIÓN POR
HERPES VIRUS DISEMINADA.
INFECCIÓN POR La INFECCIÓN POR CITOMEGALOVIRUS es la causa más común de infección neonatal,
CITOMEGALOVIRUS presentando el 1-2 % del total de recién nacidos (RN). En el caso de una infección congénita,
un 6 a 19% presentan la llamada enfermedad por inclusión citomegálica: Los síntomas son
hepatoesplenomegalia, púrpura, ictericia, coriorretinitis, petequias, microcefalia,
calcificaciones cerebrales, con mayor afinidad periventriculares, y hernia inguinal. En el caso
connatal, lo más frecuente e importante es un cuadro de neumonitis intersticial subfebril. Los
síntomas también pueden ser de presentación tardía, con un neonato aparentemente sano;
en este caso hay retraso psicomotor e hipoacusia en un 10% de los casos. La infección
adquirida al nacer el producto o poco después, por las secreciones cervicouterinas de la
gestante o de la leche materna, generalmente no se acompaña de enfermedad clínica. La
infección que es consecuencia de transfusión de sangre o hemoderivados de donantes
seropositivos (que tienen virus citomegálico) a pequeños pretérmino, ha ocasionado
síntomas sistémicos, incluida la enfermedad de la porción baja de las vías respiratorias. Existe
cierta superposición entre estas tres categorías clínicas ya que se pueden apreciar lesiones
cutáneas en todas ellas y la enfermedad diseminada suele asociarse también con afectación
neurológica. IMPORTANTE: En series amplias de casos de infección neonatal por VHS las
vesículas cutáneas (El 68 % de los RN infectados las presentan) y las convulsiones fueron los
dos hallazgos clínicos más sugestivos de infección neonatal por Herpes. EL COMPROMISO
NEUROLÓGICO + LESIONES DÉRMICAS + HEPATOMEGALIA SUGIEREN LA PRESENCIA DE
HERPES VIRUS, DEJANDO A UN LADO LA POSIBILIDAD DE QUE SE TRATE DE UNA
INFECCIÓN POR CITOMEGALOVIRUS

INFECCIÓN Aunque la VARICELA durante este período suele ser poco frecuente, estimándose en torno a
GENERALIZADA 7 casos cada 1.000 embarazos, y a pesar de que la mayoría de los niños de mujeres que
POR VIRUS contraen la varicela durante la gestación nacen sanos, es importante conocer que pueden
VARICELA-ZOSTER darse las siguientes situaciones: a. Si la varicela aparece en las primeras 20 semanas de
gestación, en el 2% de los casos se produce una embriopatía característica, conocida como
embriopatía varicelosa, cuyo dato más frecuente es la atrofia de una extremidad o la
presencia de lesiones cutáneas cicatrizales en distribución dermatómica. Se ha descrito
también: atrofia cerebral, hidrocefalia e hipoplasia del diafragma, entre otras muchas
malformaciones. b. Si la madre padece la varicela entre las 20 y las 39 semanas de gestación,
puede producirse una varicela intrauterina, lo que explica la posibilidad de que el niño tenga
luego un herpes zóster sin haber padecido previamente la varicela en la vida extrauterina. c.
Si la erupción aparece en la madre entre 5 días antes y 2 días después del parto, o aparece en
el niño entre los 5 y 10 días de vida, el resultado es una varicela congénita grave con una
mortalidad del 30%. Por el contrario, si la madre padece la erupción al menos 5 días antes del
parto, o aparece en los niños en los primeros 5 días de vida, la varicela congénita será leve.
Ello es debido a que ha habido tiempo suficiente para la producción de los anticuerpos
maternos necesarios para proporcionar protección específica al recién nacido. POR
PRINCIPIO, NO EXISTE ANTECEDENTE DE VARICELA EN LA MADRE DURANTE EL EMBARAZO
O EN ESTE MOMENTO.

Bibliografía:
INFECTOLOGÍA NEONATAL. NAPOLEÓN GONZÁLEZ SALDAÑA. MC GRAW-HILL. EDICIÓN 2A. 2006. PÁG. 152.
KLIEGMAN R, STANTON B, GEME J, SCR N, BEHRMAN R. NELSON TRATADO DE PEDIATRÍA, 19ª EDICIÓN.
ELSEVIER. ESPAÑA 2013. LONGO DL, FAUCI AS, KASPER DL, HAUSERSL, JAMESON JL, LOSCALZOJ.
HARRISON. PRINCIPIOS DE MEDICINA INTERNA, 18A EDICIÓN. MC GRAW HILL. NEW YORK, USA. 2012.

274 - EL TRATAMIENTO QUE DEBE DE INICIARSE DE INMEDIATO ES CON

SULFADIAZINA INFECCIÓN POR TOXOPLASMOSIS No hay vacuna para su prevención. La prevención se basa en
mantener a gatos alejados de la casa y especialmente de la comida así comer la carne cocida. En la
embarazada el tratamiento es con Espiramicina; en el neonato es con Pirimetamina más
Sulfadiazina, a lo cual se agrega ácido folínico, además se debe realizar control hematológico por
la posibilidad de anemia aplásica. LA SULFADIAZINA FORMA PARTE DEL TRATAMIENTO PARA EL
NEONATO CON INFECCIÓN POR TOXOPLASMOSIS.
VALACICLOVIR 1.- GESTANTE CON VARICELA ENTRE 21º DÍA PREPARTO Y 5º DÍA PREPARTO. A. Gestante: Sólo
antivíricos (Aciclovir) en enfermedad severa. B. Neonato: • Aislamiento (de los no inmunes) y
vigilancia. • En neonatos con clínica de varicela: Administración de Aciclovir (45mg/Kg/día /4
dosis /IV/más de 7 días. • Se puede plantear el uso de Aciclovir en prematuros. 2.- GESTANTE
CON VARICELA ENTRE 5º DÍA PREPARTO Y 3º DÍA POSTPARTO: A. Gestante: • Intentar retrasar el
parto para aumentar el paso de anticuerpos al feto. • Aislamiento de la madre durante 7 días, sin
lactar durante este tiempo B. Neonato: • Administración de Inmunoglobulina específica anti
varicela-zoster (Varitectâ) a dosis de 1cc/Kg /IV lento (velocidad máxima de 1ml / minuto, puede
diluirse con solución salina isotónica). • Aislamiento (de los no inmunes) y vigilancia estricta. • En
neonatos con clínica de varicela: administración de Aciclovir 45mg/Kg/día /4 dosis /IV/más de 7
días. • Se puede plantear el uso de Aciclovir en prematuros. 3 .- GESTANTE CON VARICELA
DESPUÉS DEL 3º DÍA POSTPARTO: A. Madre: • Aislamiento durante 7 días, sin lactar durante este
tiempo. B. Neonato: • Observación clínica y aislamiento (de los no inmunes) • Administración de
Inmunoglobulina específica anti varicela- zoster. • En neonatos con clínica de varicela:
administración de Aciclovir 45mg/Kg/día /4 dosis /IV/más de 7 días. EL VALACICLOVIR ES UN
ANTIVIRAL SIMILAR AL ACICLOVIR, PERO CON EFECTOS TERAPÉUTICOS MENORES. TANTO EN LA
VARICELA COMO EN EL HERPES SIMPLE, NO SE RECOMIENDA COMO FÁRMACO DE PRIMERA
LÍNEA.

ACICLOVIR NEONATOS CON INFECCIÓN POR HERPES VIRUS SIMPLE (HVS). El ACICLOVIR parenteral es el
tratamiento más indicado contra infecciones por HSV en neonatos. Debe administrarse a todos
los recién nacidos con infección por HSV, sean cuales sean las manifestaciones y los signos
clínicos. Se observan los mejores resultados en términos de morbilidad y mortalidad en lactantes
con enfermedad circunscrita a la piel, ojos y boca. Muchos neonatos tratados de encefalitis por
HSV sobreviven, pero muchos quedan con notables secuelas neurológicas. En promedio, 25% de
los neonatos con enfermedad diseminada terminan por fallecer a pesar del tratamiento antivírico.
En las infecciones perinatales, el tratamiento debe ser precoz y con dosis altas de aciclovir hasta
que haya evidencia de resolución. IMPORTANTE: en tratamientos largos con aciclovir existe
riesgo por neutropenia por lo que se recomienda monitorización estricta. EL TRATAMIENTO
ANTIVIRAL ESTÁ INDICADO EN TODOS LOS NEONATOS INFECTADOS CON HERPES VIRUS
INDEPENDIENTEMENTE DEL GRADO DE LA ENFERMEDAD DADO EL RIESGO ALTO DE
COMPLICACIONES Y MORTALIDAD.

GANCICLOVIR El ganciclovir es beneficioso para tratar la retinitis causada por infección adquirida o recurrente
por virus CITOMEGÁLICO en sujetos infectados por VIH. En Estados Unidos de América, se ha
aprobado el uso de dicho fármaco para tratar la retinitis grave en adultos inmunodeficientes. Los
escasos datos en niños sugieren que su inocuidad y eficacia son semejantes a las observadas en
adultos. Siempre deberás considerar al ganciclovir como el fármaco de elección para el
tratamiento de infección por citomegalovirus. En Estados Unidos de América también, se ha
aprobado la combinación de foscarnet sódico y clorhidrato de valganciclovir para tratar la
retinitis por virus citomegálico en adultos, y constituyen fármacos de uso alternativo. Sólo en
adultos. EL GANCICLOVIR ES UNA BUENA OPCIÓN TERAPÉUTICA PARA EL MANEJO DE LA
RETINITIS SECUNDARIA A INFECCIÓN POR CITOMEGALOVIRUS.

Bibliografía:
1. INFECTOLOGÍA NEONATAL. NAPOLEÓN GONZÁLEZ SALDAÑA. MC GRAW-HILL. EDICIÓN 2A. 2006. PÁG.
152.

275 - EN ESTA PACIENTE LA DURACIÓN DEL TRATAMIENTO DEBE SER POR:


7 MANEJO DEL RN EXPUESTO (HIJO DE MADRE CON LESIONES ACTIVAS DE HERPES GENITAL): 1- EN CASO
DÍAS DE PARTO VAGINAL. Dado que el riesgo de los RN expuestos durante el parto varía ampliamente,
dependiendo de muchas circunstancias, la decisión de tratar a los RN asintomáticos de forma empírica con
Aciclovir EV es controvertida. En los hijos de madres con una recurrencia genital activa, se estima que el
riesgo de infección neonatal es inferior al 5%, sin embargo, en los casos de primoinfección materna genital
el riesgo puede ser superior al 50%. El manejo por tanto de los RN asintomáticos dependerá de la
categorización de la infección materna en: • Primoinfección activa • Infección recurrente activa • Madre con
status serológico (Infección primaria o recurrente) desconocido. La distinción anterior puede ser en
ocasiones difícil de realizar ya que el primer episodio clínico puede no corresponder siempre a una
primoinfección, puesto que con frecuencia ésta es asintomática. En todos los casos anteriores se deben
obtener cultivos del RN (Incluyendo urocultivo, coprocultivo, exudado rectal y oral y aspirado nasofaríngeo,
hemocultivo y PCR viral en LCR entre las primeras 24 y 48 horas. 2- EN CASO DE CESÁREA. Se recomienda
realizar los mismos cultivos que en el RN expuesto con parto vaginal y el mantenimiento de una vigilancia
clínica cuidadosa. Si aparece sintomatología o se obtiene positividad en algún cultivo se debe iniciar el
tratamiento con Aciclovir EV. Por último, señalar que debido a que la infección neonatal herpética puede
manifestarse entre las dos y tres semanas de vida, los padres de los niños asintomáticos expuestos deben
conocer las presentaciones clínicas posibles de la enfermedad y ante la aparición de cualquier signo o
síntoma de alerta debe acudir a un centro sanitario.

14 AISLAMIENTO. • Neonatos con infección por VHS: Se deben mantener con medidas de aislamiento de
DÍAS contacto si presentan lesiones mucocutáneas. • Neonatos expuestos a VHS durante el parto: En los RN hijos
de madres con lesiones herpéticas activas, deben mantenerse medidas de aislamiento de contacto mientras
dure el período de incubación. Algunos expertos consideran que son innecesarias las medidas de
aislamiento en RN expuestos, si el parto fue mediante cesárea y la ruptura de membranas se produjo menos
de cuatro horas antes del parto.

21 TRATAMIENTO ANTIVIRAL EN INFECCIÓN DISEMINADA POR HERPES SIMPLE PERINATAL. La dosis de


DÍAS Aciclovir es de 60 mg/kg al día, en tres fracciones, aplicadas por vía intravenosa durante 14 días si la
enfermedad se limitó a la piel, los ojos y la boca y, de 21 días si el cuadro es diseminado y afecta el SNC. Al
terminar el tratamiento surgen a veces recaídas de enfermedades de la piel, los ojos, la boca y el SNC. El
tratamiento óptimo de tales recurrencias no se ha definido. Está en fase de evaluación la utilidad de la
terapia supresora a largo plazo o intermitente con Aciclovir para neonatos con enfermedad de la piel, los
ojos y la boca. Los lactantes con ataques de ojos atribuibles a HSV deben recibir un colirio tópico (1 a 2% de
trifluridina, 0.1% de yododesoxiuridina ó 3% de vidarabina), así como la terapia antivírica parenteral. A
pesar de la reciente utilización de Aciclovir a dosis altas la morbimortalidad por VHS continúa siendo
inaceptablemente alta. El mejor pronóstico lo presentan los pacientes con enfermedad buco-óculo-cutánea.
Una revisión de 72 RN (50% con enfermedad herpética diseminada y 35% con enfermedad neurológica
localizada) tratados entre 1990 y 1997 con Aciclovir a dosis de 60mg/kg/d durante 21 días mostró una
mortalidad del 31% a los 24 meses en los casos diseminados y del 6% en los de encefalitis. Sólo en el 30% de
los supervivientes de casos de afectación neurológica tuvieron un desarrollo normal, frente a un 83% de los
supervivientes de una enfermedad diseminada. DEBIDO A QUE SE TRATA DE UNA INFECCIÓN DISEMINADA
(SISTÉMICA) ES NECESARIO EXTENDER EL TRATAMIENTO POR 21 DÍAS.

28 RESISTENCIA DEL VHS AL ACICLOVIR. La resistencia del VHS al Aciclovir es poco frecuente, con la excepción
DÍAS de los pacientes afectos de SIDA, aunque se han descrito casos excepcionales neonatales de resistencia al
mismo. Se conocen tres mecanismos de resistencia. El más común consiste en una mutación que provoca un
déficit de la timidina quinasa viral, enzima que lleva a cabo el primer paso en la transformación del Aciclovir
a su forma activa en el organismo. Las infecciones causadas por VHS resistente al Aciclovir en el adulto
pueden ser tratadas con foscarnet, cidofovir o vidarabina. Con excepción de la vidarabina la experiencia de
empleo de los otros agentes en neonatos es mínima.

Bibliografía:
INFECTOLOGÍA NEONATAL. NAPOLEÓN GONZÁLEZ SALDAÑA. MC GRAW-HILL. EDICIÓN 2A. 2006. PÁG. 152.

FIN DEL CASO CLÍNICO SERIADO


ANÁLISIS DEL CASO CLÍNICO

IDENTIFICACIÓN DEL REACTIVO


Area: MEDICINA INTERNA
Especialidad: GERIATRÍA Y GERONTOLOGÍA
Tema: INMOVILIDAD Y ÚLCERAS POR PRESIÓN
Subtema: INMOVILIDAD Y ÚLCERAS POR PRESIÓN

CASO CLÍNICO SERIADO

MUJER DE 65 AÑOS DE EDAD QUE REQUIERE ORIENTACIÓN PARA PREVENIR LA APARICIÓN DE ÚLCERAS DE
DECÚBITO YA QUE SE ENCUENTRA PROGRAMADA PARA COLOCACIÓN DE PRÓTESIS DE CADERA.

Mujer de 65 años de edad.

sera sometida a intervención quirúrgica de


reemplazo de cadera.

-.

-.

-.

276 - ENTRE LAS MEDIDAS PREVENTIVAS QUE DEBERÁ INDICARLE USTED LA SIGUIENTE MEDIDA HA
DEMOSTRADO NO SER ÚTIL EN SU PREVENCIÓN:

USO DE La ÚLCERA POR PRESIÓN (escara por presión, escara de decúbito o úlcera de decúbito), es una
CREMAS lesión de origen isquémico localizada en la piel y en los tejidos subyacentes con pérdida cutánea,
HIDRATANTES. que se produce por presión prolongada o fricción entre los planos duros, uno que pertenece al
paciente, y otro externo a él. En su desarrollo se conjugan dos mecanismos; la oclusión vascular
por la presión externa y el daño endotelial a nivel de la micro-circulación. En su formación
participan tres tipos de fuerzas: presión, fricción y cizallamiento. RECUERDA: Es importante
MANTENER LA PIEL HIDRATADA, ya que la piel reseca es un factor de riesgo para el desarrollo
de úlceras de presión. Se recomienda una crema lubricante sin alcohol aplicarla de forma regular
a la piel.
EVITAR LA PIEL La piel debe limpiarse cuando este sucia por orina o heces en intervalos regulares. Tanto la orina
HÚMEDA. como las heces, son abrasivos para la piel. También el contacto con las secreciones de los
drenajes de heridas, pueden ser abrasivos. Se recomiendan toallas o calzoncillos para absorber la
humedad, también se pueden utilizar recubrimientos impermeables al agua vaselinados para
proteger a la piel de la orina y las heces. SE DEBE EVITAR QUE LA PIEL SE ENCUENTRE HÚMEDA.
OJO: no debe estar húmeda por agentes externos, pero deberá estar hidratada.

ASEO DIARIO ES IMPORTANTE MANTENER SIEMPRE LA PIEL EN GENERAL LIMPIA Y SECA. OJO: seca no quiere
DE LAS decir que deshidratada. REPASO: Factores de riesgo para la formación de úlceras por presión: 1.
SALIENTES INTRÍNSECOS: inmovilidad, alteraciones respiratorias y/o circulatorias, insuficiencia vascular,
ÓSEAS. presión arterial baja, insuficiencia cardiaca, vasoconstricción periférica, alteraciones endoteliales,
anemia, septicemia, medicación, inmunosupresión y desnutrición/deshidratación. 2.
EXTRÍNSECOS: Humedad, perfumes o lociones que contengan alcohol, superficie de apoyo,
masajes en la piel que cubre a las salientes óseas, presencia de sondas (vesical, naso-gástrica,
férulas y yesos).

MASAJE DE LA NO DEBEN DARSE MASAJES SOBRE LAS PROMINENCIAS ÓSEAS, ya que son dañinos para los
PIEL EN tejidos. En los pacientes con alto riesgo de padecer úlceras por presión o en estadio I, aplicar
PROMINENCIAS suavemente ácidos grasos hiperoxigenados en las zonas de riesgo "SIN DAR MASAJE".
ÓSEAS.

Bibliografía:
1. GUÍA DE PRÁCTICA CLÍNICA, PREVENCIÓN Y TRATAMIENTO DE ÚLCERAS POR PRESIÓN A NIVEL
HOSPITALARIO. MÉXICO: SECRETARIA DE SALUD, 2009.

http://www.cenetec.salud.gob.mx/descargas/gpc/CatalogoMaestro/105_GPC_Ulcpresion1NA/ULCERA_DECUBITO_1ER_NIVEL_EVR_CENETEC.pdf

277 - PARA ALIVIAR LA PRESIÓN SOBRE LOS TROCÁNTERES, TOBILLOS Y TALONES ESTA
CONTRAINDICADO EL USO DE:

CUÑAS Las personas que están postradas, se recomiendan usar almohadillas especiales para disminuir la
DE presión en salientes óseas.
ESPUMA.

COJINES Se pueden usar cojines o almohadillas que tengan un grosor de 1 a 4 pulgadas de grosor o también
DE existen los cojines de Jay o ROHO.
ESPUMA.

COLCHÓN Tanto los colchones neumáticos como los de agua, se ha demostrado reducen el riesgo de desarrollar
DE AGUA. úlceras de presión al general una distribución homogénea de la presión.

COJINES No deben usarse donas, pues estas tienen mayor riesgo de daño que otros dispositivos. Las donas o
EN flotadores, concentran la presión sobre la zona corporal que queda en contacto con ellas provocando
FORMA edema y congestión venosa, lo que favorece la formación de úlceras de presión.
DE DONA.

Bibliografía:
1. GUÍA DE PRÁCTICA CLÍNICA, PREVENCIÓN Y TRATAMIENTO DE ÚLCERAS POR PRESIÓN A NIVEL
HOSPITALARIO. MÉXICO: SECRETARIA DE SALUD, 2009.

http://www.cenetec.salud.gob.mx/descargas/gpc/CatalogoMaestro/105_GPC_Ulcpresion1NA/ULCERA_DECUBITO_1ER_NIVEL_EVR_CENETEC.pdf

278 - EN CASO DE PRESENTAR LA PACIENTE ULCERAS POR PRESIÓN A NIVEL DEL TALÓN. SERÁ
IMPORTANTE POR SU FRECUENCIA Y TRASCENDENCIA PREVENIR LA PRESENTACIÓN DE LA SIGUIENTE
COMPLICACIÓN:
OSTEOMIELITIS. LA PRINCIPAL COMPLICACIONES QUE SE PRESENTA EN PERSONAS CON ÚLCERAS DE
PRESIÓN, SON LAS INFECCIONES. Esto es como resultado de colonización bacteriana que
posteriormente puede diseminarse a los tejidos contiguos. Puede haber infección desde la
piel, tejidos blandos hasta osteomielitis. La "osteomielitis" se desarrolla en aproximadamente
30% de las personas que tienen úlceras de presión por un periodo de 4 meses.

OSTEOARTRITIS. La osteoartritis, es la enfermedad articular degenerativa más frecuente de los adultos. No es


una consecuencia de la postración o inmovilidad.

OSTEOPOROSIS. La inmovilidad a largo plazo, puede generar osteoporosis, pero en caso de esta paciente
debido a su patología, se espera movilizarla a mas tardar a un mes de haber sido intervenida.
En caso que esta paciente ya tenga osteoporosis, no sería una complicación de la
inmovilización, sino una comorbilidad de la paciente que si podría alterar la evolución de la
cirugía y la recuperación de la misma. ES UNA POSIBILIDAD QUE SE CONSIDERA SÓLO EN
CASOS DE MUY LARGA INMOVILIZACIÓN, NO SE JUSTIFICA PARA ESTE CASO.

MICROFRACTURAS. No se asocia con la inmovilización. Puede asociarse con traumas y más probable en pacientes
que ya tienen establecida osteoporosis.

Bibliografía:
1. GUÍA DE PRÁCTICA CLÍNICA, PREVENCIÓN Y TRATAMIENTO DE ÚLCERAS POR PRESIÓN A NIVEL
HOSPITALARIO. MÉXICO: SECRETARIA DE SALUD, 2009.

http://www.cenetec.salud.gob.mx/descargas/gpc/CatalogoMaestro/105_GPC_Ulcpresion1NA/ULCERA_DECUBITO_1ER_NIVEL_EVR_CENETEC.pdf

FIN DEL CASO CLÍNICO SERIADO


ANÁLISIS DEL CASO CLÍNICO

IDENTIFICACIÓN DEL REACTIVO


Area: PEDIATRÍA
Especialidad: INFECTOLOGIA PEDIÁTRICA
Tema: TÉTANOS
Subtema: TÉTANOS

CASO CLÍNICO CON UNA PREGUNTA

ESCOLAR MASCULINO DE 10 AÑOS CON PROBABLE DIAGNÓSTICO DE TÉTANOS.

Masculino de 10 años de edad

Diagnóstico de tétanos.

279 - EL DIAGNÓSTICO DE TÉTANOS SE CONFIRMARÍA REALIZANDO:

CITOQUÍMICO DE TÉTANOS. MANIFESTACIONES CLÍNICAS: El tétanos generalizado (Trismo), es un cuadro


LÍQUIDO neurológico que incluye dicha manifestación (Trismo), y espasmos musculares graves. Es
CEFALORRAQUÍDEO causado por la neurotoxina producida por la bacteria anaerobia Clostridium tetani en una
herida contaminada. Su comienzo es gradual y abarca uno a siete días, y los síntomas
evolucionan hasta espasmos musculares generalizados muy intensos que suelen ser
agravados por cualquier estímulo externo. Los espasmos graves persisten durante una
semana o más, y en personas que se recuperan ceden en un periodo de semanas. El tétanos
localizado se manifiesta en la forma de espasmos musculares locales en zonas vecinas a la
herida. La variante cefálica es una disfunción de los pares craneales, que proviene de heridas
infectadas en la cabeza y el cuello. Los dos trastornos preceden al tétanos generalizado.
CAUSAS: Clostridium tetani, que es el bacilo del tétanos, es un bacilo esporógeno, anaerobio
y Gram Positivo. Contamina heridas y no causa destrucción tisular ni una respuesta
inflamatoria. La forma vegetativa produce una endotoxina potente codificada por plásmido
(Tetanospasmina) que se liga a gangliósidos en la unión mioneural del músculo de fibra
estriada y a las membranas neuronales de la médula espinal, lo que bloquea los impulsos
inhibitorios a las motoneuronas. No se ha corroborado con tanta exactitud la acción de la
toxina tetánica en el encéfalo y el sistema nervioso simpático.
HISTORIA CLÍNICA MÉTODOS DIAGNÓSTICOS: El diagnóstico del tétanos se hace sobre bases clínicas al
Y EXPLORACIÓN descartar otras causas de espasmos tetánicos, como sería la tetania hipocalcémica,
reacciones por fenotiazinas, intoxicación por estricnina e histeria. Habrá que hacer intentos
para identificar C. tetani en cultivos; sin embargo, el índice de confirmación diagnóstica es
inadecuado y el hecho de que un cultivo sea negativo, no descarta la presencia de la
enfermedad. No debe utilizarse la concentración de antitoxina sérica protectora para
descartar la posibilidad de tétanos. LOS INTENTOS POR CONFIRMAR EL DIAGNÓSTICO DE
TÉTANOS POR MEDIO DE ESTUDIOS DE LABORATORIO HAN RESULTADO POCO
CONCLUYENTES. Rara vez se aísla el microorganismo en el sitio de infección, y por lo general
no hay una respuesta de anticuerpos detectable.

BIOMETRÍA REPASO. ASPECTOS EPIDEMIOLÓGICOS TÉTANOS: El tétanos es una enfermedad de


HEMÁTICA Y distribución mundial; es más frecuente en climas cálidos y en meses de verano, en parte por
ELECTROLITOS la mayor frecuencia de heridas contaminadas, vinculadas con tales sitios y estaciones. El
microorganismo que habita normalmente en la tierra y los intestinos de animales y seres
humanos tiene una distribución amplísima en el entorno, especialmente en sitios en que es
frecuente la contaminación con las excretas. Las heridas, identificadas o no, son los sitios de
multiplicación del microorganismo y su elaboración de toxina. Las expuestas a un mayor
peligro son las contaminadas, en particular las que tienen tejidos desvitalizados y
traumatismos profundos y puntiformes. El tétanos neonatal es frecuente en muchos países
en desarrollo en que las mujeres no son vacunadas adecuadamente contra la enfermedad, y
se siguen todavía prácticas antihigiénicas para el cuidado del cordón umbilical. La
vacunación activa amplia contra el tétanos ha modificado los aspectos epidemiológicos de la
enfermedad en Estados Unidos, país en el cual se han notificado desde 1995 menos de 50
casos al año. El tétanos no es transmisible de una persona a otra. El periodo de incubación
varía de dos días a meses, y casi todos los casos surgen en término de 14 días. En los
neonatos, el periodo de incubación va de cinco a 14 días; lapsos más breves se han vinculado
con heridas contaminadas más intensamente, con un cuadro más grave y un pronóstico peor.

TINCIÓN DE GRAM REPASO. TRATAMIENTO TÉTANOS: • El concentrado inmunoglobulínico antitetánico


EN LÍQUIDO humano (Tetanus Immune Globulin, TIG) se recomienda para tratar la enfermedad, en una
CEFALORRAQUÍDEO sola dosis total de 3 000 a 6 000 U para niños y adultos. No se ha definido la dosis
terapéutica óptima, y dosis incluso de 500 U han sido eficaces y han ocasionado menos
molestias al paciente. Se pueden aplicar por vía intramuscular los preparados regulares.
Algunas autoridades recomiendan infiltrar parte de la dosis en la zona alrededor de la herida
a pesar de que no se ha demostrado la eficacia de esta estrategia. Son antagónicos los
resultados de estudios de los beneficios de la administración intrarraquídea de TIG. El
preparado que se usa en Estados Unidos no ha sido aprobado ni formulado para uso
intrarraquídeo o intravenoso. • En países en que no se distribuye TIG puede obtenerse
antitoxina tetánica equina, producto que desde hace algún tiempo ha sido retirado del
mercado en Estados Unidos de América. La antitoxina equina se administra después de
practicar pruebas adecuadas de sensibilidad y emprender desensibilización, si es necesario
sensibilidad en reacciones a sueros de animales. • El concentrado inmunoglobulínico
intravenoso contiene anticuerpos contra el tétanos y cabe pensar en su uso, si no se dispone
de TIG. En los Estados Unidos de América, la Food and Drug Administration no ha aprobado
el uso de TIG para tal finalidad ni se ha precisado la dosis. • Es necesario limpiar y desbridar
de manera apropiada todas las heridas, en particular si hay necrosis extensa. En el tétanos
neonatal no conviene la extirpación amplia del muñón umbilical. • Son de gran importancia
las medidas de apoyo y la farmacoterapia para erradicar los espasmos tetánicos. • La
administración oral o intravenosa de metronidazol (30 mg/kg al día a intervalos de 6 hrs;
dosis máxima, 4 g/día) es eficaz para disminuir el número de formas vegetativas de C. tetani
y es el antimicrobiano más indicado. Otra posibilidad sería usar penicilina G parenteral (100
000 U/kg al día a intervalos de 4 a 6 hrs; dosis máxima, 12 millones de unidades/día). Se
recomienda un ciclo de 10 a 14 días.

Bibliografía:
1. INFECTOLOGÍA CLÍNICA PEDIÁTRICA. NAPOLEÓN GONZÁLEZ SALDAÑA. MC GRAW HILL
INTERAMERICANA. EDICIÓN 7A. 2003. PÁG. 835-853. 2. HEYMANN, DAVID L. EL CONTROL DE LAS
ENFERMEDADES TRANSMISIBLES, 19A EDICIÓN. OPS. WASHINGTON, D.C. 2011.
ANÁLISIS DEL CASO CLÍNICO

IDENTIFICACIÓN DEL REACTIVO


Area: CIRUGÍA
Especialidad: OFTALMOLOGÍA
Tema: ENFERMEDADES DE LOS PÁRPADOS, APARATO LAGRIMAL Y LA
CONJUNTIVA
Subtema: BLEFARITIS, ORZUELO, CHALAZIÓN

CASO CLÍNICO SERIADO

MUJER DE 23 AÑOS DE EDAD QUIEN REFIERE COMO ÚNICO ANTECEDENTE HABER CAMBIADO DE
COSMÉTICOS HACE 3 DÍAS, PRESENTA DOLOR INTENSO EN PÁRPADO DE OJO DERECHO. A LA EXPLORACIÓN
SE OBSERVA EDEMA PALPEBRAL DERECHO Y ERITEMA EN EL BORDE DE LAS PESTAÑAS CON LESIÓN
PAPULAR ÚNICA EN MISMA ZONA, A LA EVERSIÓN PALPEBRAL SE OBSERVA CONJUNTIVA ERITEMATOSA.

-.

Uso de cosméticos.

Dolor intenso en párpado de ojo derecho.

Edema palpebral derecho, eritema en el


BORDE DE LAS PESTAÑAS con lesión
papular única en misma zona, eritema de
conjuntiva tarsal.

-.

280 - CORRESPONDE AL DIAGNÓSTICO CLÍNICO DE LA PACIENTE:

ORZUELO. ORZUELOS: Es la inflamación aguda estafilocócica del folículo de una pestaña o glándula
relacionada. Son muy frecuentes en la población general, pero más aún en los afectos de blefaritis
marginal. Se presentan con dolor, enrojecimiento local, y a veces un foco de pus central, se
distingue entre: 1. Orzuelo externo: Se afecta la glándula de Zeiss (Sebácea), o la de Moll
(Sudorípara apocrina especial). 2. Orzuelo interno: Afectación de la glándula de Meibomio. Es más
doloroso y la tumefacción puede interesar a la conjuntiva palpebral. Características clínicas: -
Absceso, hiperemia. - Edema localizado o difuso del borde palpebral. - Dolor localizado. -
Queratinización del conducto glandular. - Pápula o pústula en el borde palpebral con o sin secreción
en conjuntiva, pero con la punta inflamada y eritematosa. RECUERDA LA CARACTERÍSTICA DEL
ORZUELO EXTERNO ES QUE APARECE EL ÁREA DE LOS FOLÍCULOS PILOSOS DE LAS PESTAÑAS.
CHALAZIÓN. CHALAZIÓN: Es un granuloma inflamatorio crónico de una glándula de Meibomio, a causa de la
obstrucción de su conducto de salida con retención de secreciones. Favorecen su aparición los
orzuelos de repetición y las blefaritis. Se forma un nódulo rojizo, indoloro y sin signos de
inflamación de localización más frecuente en la conjuntiva tarsal superior. Pueden resolverse
espontáneamente, aunque no es lo más frecuente, siendo subsidiarios de la inyección de
triamcinolona intralesional, o bien, la excisión quirúrgica. Ante la aparición de chalazión recidivante,
pensar en la posibilidad de un carcinoma de glándulas sebáceas. Características clínicas: - Nódulo
subcutáneo en el tarso bien definido, NO DOLOROSO, elevado, no doloroso de 2-8 mm de diámetro.
- La eversión palpebral puede mostrar granuloma conjuntival externa. - Lesión única o múltiple. -
Puede ser recurrente. - Puede drenar a través de la piel. - Algunas veces produce visión borrosa por
astigmatismo inducido. RECUERDA LA CARACTERÍSTICA DEL CHALAZIÓN ES LA PRESENCIA DE
“NODULACIONES” INDOLORAS O POCO DOLOROSAS.

BLEFARITIS. La BLEFARITIS es una enfermedad crónica inflamatoria, multifactorial y frecuente de los párpados.
Se clasifica como: 1. ANTERIOR: cuando afecta la parte externa del párpado donde se implantan las
pestañas. Se reconocen en ésta dos tipos primordiales. - Seborreica: Es debida a un exceso de
secreción de las glándulas de Zeiss pudiendo asociarse a inflamaciones de estas (Orzuelos). Es
frecuente su asociación con la dermatitis seborreica. En el borde anterior de los párpados se aprecia
un brillo céreo, eritema leve ha moderado y pestañas grasientas. Presenta "escamas blandas,
amarillentas" que no dejan úlcera al arrancarlas. - Estafilocócica: Está causada por una infección
estafilocócica de la base de las pestañas. En el borde palpebral anterior se aprecian vasos sanguíneos
dilatados y escamas duras y quebradizas que rodean las raíces de las pestañas (Collaretes), y que
cuando se arrancan dejan una pequeña úlcera sangrante. 2. POSTERIOR: cuando afecta los orificios
de las glándulas de Meibomio. En ella se aprecia un eritema del borde posterior del párpado
asociado a lágrima oleosa y a la acumulación de una secreción espumosa. La expresión de las
glándulas de Meibomio produce secreción abundante. Se asocia frecuentemente con el acné
rosáceo. EN LA BLEFARITIS NO HAY PRESENCIA DE NÓDULOS Y LAS LESIONES SE LOCALIZAN EN EL
BORDE DE LAS PESTAÑAS.

PTERIGIÓN. PTERIGIÓN es una enfermedad de origen y patogenia desconocida, uni o bilateral, que se caracteriza
por crecimiento de tejido fibrovascular anormal de tejido conjuntival, de forma triangular, que
invade la córnea a partir de la conjuntiva bulbar; con base localizada en la periferia y el ápex hacia la
córnea, puede ser nasal o temporal y consta de 3 áreas: la cabeza, el cuello y el cuerpo. NO GUARDA
NINGUNA RELACIÓN CON LOS PÁRPADOS, PUES SE TRATA DE UNA FORMACIÓN DE TEJIDO
FIBROSO EN LA CONJUNTIVA.

Bibliografía:
1. GUÍA DE PRÁCTICA CLÍNICA, DIAGNÓSTICO Y MANEJO DEL ORZUELO Y CHALAZIÓN. MÉXICO:
SECRETARIA DE SALUD; 2009. RECUPERADO DE
HTTP://WWW.CENETEC.SALUD.GOB.MX/CONTENIDOS/GPC/CATALOGOMAESTROGPC.HTML 2. RIORDAN P,
CUNNINGHAM E. VAUGHAN Y ASBURY OFTALMOLOGÍA GENERAL, 18A EDICIÓN. MC GRAW HILL LANGE.
2012.

http://www.cenetec.salud.gob.mx/descargas/gpc/CatalogoMaestro/316_GPC_ORZUELO_Y_CHALAZION/chalazion_EVR_CENETEC.pdf

281 - LOS HALLAZGOS CLÍNICOS SON EXPLICADOS POR LA AFECTACIÓN DE:

LAS La inflamación AGUDA de las glándulas de MEIBOMIO se relaciona con el ORZUELO INTERNO y la
GLÁNDULAS inflamación CRÓNICA con la BLEFARITIS.
DE
MEIBOMIO.

LA El PTERIGIÓN se caracteriza por crecimiento de tejido fibrovascular anormal de tejido conjuntival,


CONJUNTIVA de forma triangular, que invade la córnea a partir de la CONJUNTIVA BULBAR.
BULBAR.

LAS El ORZUELO EXTERNO ocurre en la superficie de la piel, en el borde de los párpados. EL ORZUELO
GLÁNDULAS EXTERNO AFECTA FOLÍCULOS PILOSOS O ASOCIADA CON GLÁNDULAS DE ZEIZ Y MOLL.
DE ZEIZ Y
MOLL.
LA El ORZUELO INTERNO se presenta sobre la conjuntiva tarsal. Ambos tipos pueden ser secuela de la
CONJUNTIVA blefaritis aguda (meibomitis) y requiere escisión y drenaje del absceso. La escisión está indicada
TARSAL. cuando la lesión no se resuelve espontáneamente o con terapia médica. EL ORZUELO INTERNO
AFECTA LAS GLÁNDULAS DE MEIBOMIO Y SE PRESENTA SOBRE LA CONJUNTIVA TARSAL.

Bibliografía:
1. GUÍA DE PRÁCTICA CLÍNICA, DIAGNÓSTICO Y MANEJO DEL ORZUELO Y CHALAZIÓN. MÉXICO:
SECRETARIA DE SALUD; 2009. RECUPERADO DE
HTTP://WWW.CENETEC.SALUD.GOB.MX/CONTENIDOS/GPC/CATALOGOMAESTROGPC.HTML 2. RIORDAN P,
CUNNINGHAM E. VAUGHAN Y ASBURY OFTALMOLOGÍA GENERAL, 18A EDICIÓN. MC GRAW HILL LANGE.
2012.

http://www.cenetec.salud.gob.mx/descargas/gpc/CatalogoMaestro/316_GPC_ORZUELO_Y_CHALAZION/chalazion_EVR_CENETEC.pdf

282 - EN ESTE MOMENTO USTED DEBERÁ INDICAR:

ANTIBIÓTICO Los ANTIBIÓTICOS SISTÉMICOS “NO” son necesarios a menos que exista enfermedad asociada
SISTÉMICO como celulitis, que es raro, acné rosácea y blefaritis o diminución de las glándulas de meibomio
que pueden ocasionar cuadros graves o recurrentes.

ANTIBIÓTICO El tratamiento de la BLEFARITIS consiste en: - Blefaritis anterior: sulfacetamina tópica,


TÓPICO + hipromelosa al 0.5%. En algunas ocasiones se puede agregar tobramicina y prednisolona o
PREDNISOLONA. dexametasona. - Blefaritis posterior: se dispone de sulfacetamida con prednisolona a dosis de
reducción a partir de 7 días, hipromelosa, alcohol polivinílico y cualquier de los siguientes
antibióticos orales: tetraciclinas, doxiciclina, eritromicina o metronidazol. CORRESPONDE AL
TRATAMIENTO DE ELECCIÓN DE LA BLEFARITIS.

TRATAMIENTO La mayoría de los casos de CHALAZIÓN requieren procedimientos más invasivos como
QUIRÚRGICO. esteroides intralesionales y cirugía. La opción disponible y más viable de realizar la incisión y
CURETAJE en lesiones mayores a 6mm. EL TRATAMIENTO QUIRÚRGICO DE ELECCIÓN PARA EL
CHALAZIÓN ES LA INCISIÓN CON CURETAJE. Otra de las patologías candidata a tratamiento
quirúrgico es el PTERIGIÓN el cual consiste en RESECCIÓN DEL PTERIGIÓN CON COLOCACIÓN
DE AUTOINJERTO CONJUNTIVAL.

HIGIENE DE LOS El ORZUELO en muchos de los casos tiene una recuperación espontánea. Se recomienda como
PÁRPADOS Y tratamiento inicial el uso de compresas húmedas tibias (3 a 4 veces al día por 5 a 10 minutos)
ANTIBIÓTICO acompañadas de masaje con antibiótico en ungüento para acelerar la curación. La higiene del
TÓPICO. borde palpebral es muy importante, se puede realizar con un isopo de algodón pero no más allá
de la unión muco-cutánea, para ello se puede utilizar champú de bebé o solución de
bicarbonato de sodio. Los antibióticos tópicos recomendados para este caso son el cloranfenicol
o cuando no se disponga de este, la eritromicina. Los antibióticos sistémicos no son necesarios a
menos de que se asocie a otras patologías que lo requieran (celulitis, acné, blefaritis…).

Bibliografía:
1. GUÍA DE PRÁCTICA CLÍNICA, DIAGNÓSTICO Y MANEJO DEL ORZUELO Y CHALAZIÓN. MÉXICO:
SECRETARIA DE SALUD; 2009. RECUPERADO DE
HTTP://WWW.CENETEC.SALUD.GOB.MX/CONTENIDOS/GPC/CATALOGOMAESTROGPC.HTML 2. RIORDAN P,
CUNNINGHAM E. VAUGHAN Y ASBURY OFTALMOLOGÍA GENERAL, 18A EDICIÓN. MC GRAW HILL LANGE.
2012.

http://www.cenetec.salud.gob.mx/descargas/gpc/CatalogoMaestro/316_GPC_ORZUELO_Y_CHALAZION/chalazion_EVR_CENETEC.pdf

FIN DEL CASO CLÍNICO SERIADO


ANÁLISIS DEL CASO CLÍNICO

IDENTIFICACIÓN DEL REACTIVO


Area: GINECOLOGÍA Y OBSTETRICIA
Especialidad: GINECOLOGÍA
Tema: PATOLOGÍA BENIGNA Y MALIGNA DE OVARIO
Subtema: PATOLOGIA MALIGNA DE OVARIO

CASO CLÍNICO CON UNA PREGUNTA

FEMENINA 65 AÑOS DE EDAD, OBESA CON IMC DE 31, GESTA 2, ABORTOS 2. ACUDE A LA CONSULTA POR
PRESENCIA DE POLIURIA, DIFICULTAD PARA LA EVACUACIÓN Y SINTOMATOLOGÍA DIGESTIVA NO ESPECÍFICA
DESDE HACE 4 MESES. A LA EXPLORACIÓN ES DIFÍCIL DELIMITAR EL TAMAÑO Y LA FORMA DEL ÚTERO Y LOS
ANEXOS POR PRESENCIA DE UNA MASA ABDOMINAL, DE APROXIMADAMENTE 25 CENTÍMETROS DE
DIÁMETRO DE CONSISTENCIA RENITENTE.

la edad es un factor de riesgo importante


para un tumor de ovario maligno. más de la
mitad de estos tumores se presentan en
mayores de 63 años.

edad, obesidad, son antecedentes


importantes.

sintomatología digestiva y urinaria, mas


factores de riesgo de tanta importancia
para cáncer de ovario deben orientarte al
diagnóstico.

imc de 31, más el hallazgo de la masa


abdominal debe guiarnos a considerar
como principal posibilidad diagnóstica al
cáncer de ovario.

-.

283 - EL DIAGNÓSTICO CLÍNICO MÁS PROBABLE ES:

ABSCESO Los ABSCESOS PARATUBARIOS se presentan principalmente en pacientes en "edad


PARATUBARIO. reproductiva", como complicación a patologías propias de esa edad, como embarazo ectópico
o enfermedad pélvica inflamatoria. LA EDAD DE LA PACIENTE Y NO CONTAR CON FACTORES
DE RIESGO QUE NOS EXPLICARAN ESTA POSIBILIDAD DIAGNÓSTICA.
ENDOMETRIOMA. ENDOMETRIOMA. Se trata de una enfermedad de la "mujer en período fértil" y aunque la
mayor incidencia aparece entre los 30-45 años. La endometriosis puede localizarse en cualquier
parte del aparato genital y también en localizaciones extragenitales como ligamentos
redondos, uterosacros, anchos, fondos de saco vesicouterinos, Douglas, septo rectovaginal y
en lugares alejados de la pelvis como pulmón, intestino, vejiga, pleura y extremidades. La
localización más frecuente es la uterina y ovárica. Entre las dos suman el 75% de las
endometriosis. A pesar de ser múltiples los síntomas asociados con endometriosis, en muchas
mujeres cursa de forma asintomática. Los aspectos clínicos cardinales asociados con la
endometriosis son: dolor (dismenorrea, coitalgia, dolor pélvico crónico) e infertilidad. Las
sintomatología producida no está en relación con el tamaño de la lesión, así grandes
endometriomas pueden cursar de forma asintomática mientras que pequeños implantes
peritoneales pueden causar síntomas intolerables para la mujer. La dismenorrea es el síntoma
más característico de la enfermedad; tiene tres características: adquirida, es decir, de nueva
aparición, de intensidad progresiva e intramenstrual. LA EDAD Y EL CUADRO CLÍNICO
DESCARTAN COMPLETAMENTE ESTA POSIBILIDAD.

MIOMATOSIS MIOMATOSIS UTERINA. Los LEIOMIOMAS UTERINOS (mioma, miofibroma, fibromioma,


UTERINA. leiomiofibroma, fibroliomioma, fibroma y fibroide) son tumores benignos conformados por
músculo liso del miometrio con cantidades variables de tejido conectivo fibroso. FACTORES DE
RIESGO Y EPIDEMIOLOGIA: Son los tumores más comunes del útero y la pelvis femenina. La
incidencia citada es del 50% hallada en estudios post mortem. Sabemos que es más frecuente
en mujeres de raza negra (3.75 a 1), con predominio entre los 25 a 45 años. Los leiomiomas
pueden ser únicos, aunque la mayoría son múltiples. Se desarrollan frecuentemente en el
cuerpo del útero y con menor frecuencia en el cérvix. El cambio más común es la degeneración
hialina. La mayoría de los miomas son asintomáticos; Sensación de plenitud o presión en el
bajo abdomen, compresión de vísceras pelvianas, hemorragia o flujo no relacionados con la
menstruación, dificultad o dolor al orinar, aumento del perímetro, abdomen agudo en torsión,
aborto espontáneo. EL CUADRO CLÍNICO Y LA EDAD DESCARTAN ESTA PATOLOGÍA. LA EDAD
PRÁCTICAMENTE DESCARTA ESTA POSIBILIDAD.

TUMOR EL RIESGO DE PADECER CÁNCER DE OVARIO AUMENTA CON LA EDAD. Los cánceres de ovario
OVÁRICO. son poco comunes en las mujeres menores de 40 años, y la mayoría de los cánceres ováricos se
origina después de la menopausia. La mitad de todos los cánceres de ovario se encuentran en
mujeres de más de 63 años. RELACIÓN OBESIDAD Y CÁNCER DE OVARIO. Varios estudios han
analizado la relación entre la obesidad y el cáncer de ovario. En general, parece que las mujeres
obesas (aquellas con un índice de masa corporal de al menos 30) tienen un mayor riesgo de
cáncer de ovario. Un estudio de la Sociedad Americana Contra El Cáncer encontró una tasa
mayor de muertes de cáncer de ovario en las mujeres obesas. El riesgo aumentó 50% en las
mujeres de mayor peso. El cáncer de ovario en etapas iniciales tiende a causar síntomas que
con más frecuencia son ocasionados por otras razones. Estos síntomas incluyen distención del
abdomen (debido a una masa o acumulación de líquido), presión en la pelvis o dolor
abdominal, dificultad para ingerir alimentos o sensación rápida de plenitud al comer, y/o
síntomas urinarios (urgencia o frecuencia). La mayoría de estos síntomas también pueden ser
causados por otras condiciones menos graves. Estos síntomas pueden ser más graves cuando
son causados por cáncer de ovario, aunque esto no siempre es así. Para cuando el cáncer de
ovario se considera una posible causa de estos síntomas, probablemente ya se haya propagado
más allá de los ovarios. También, algunos tipos de cáncer de ovario pueden propagarse
rápidamente a la superficie de órganos cercanos. De cualquier forma, si los síntomas se
atienden sin demora, las probabilidades de que la enfermedad se diagnostique en sus etapas
iniciales y de que se trate con éxito podrían ser mayores. Examen ginecológico: sólo el 5% de
los blastomas anexiales malignos son detectados por la palpación bimanual. • Examen físico
general y de territorios ganglionares inguinales, axilares, supraclaviculares y cervicales. Ante la
presencia de adenopatías sospechosas se efectúa punción con aguja fina. Si en la paciente ya es
posible detectar la masa en la exploración, sumado a los factores de riesgo y sintomatología, la
probabilidad de que se trate de cáncer de ovario es muy alta.

Bibliografía:
1. GUÍA DE PRÁCTICA CLÍNICA, TRATAMIENTO MÉDICO Y QUIRÚRGICO DEL CÁNCER EPITELIAL DEL
OVARIO. MÉXICO: SECRETARIA DE SALUD; 2010. 2. GUÍA DE REFERENCIA RÁPIDA, TRATAMIENTO MÉDICO Y
QUIRÚRGICO DEL CÁNCER EPITELIAL DEL OVARIO. MÉXICO: SECRETARIA DE SALUD; 2010. 3. SCHONGUE J,
SCHAFER J, HALVORSON L, HOFFMAN B, BRADSHAW K, CUNNINGHAM G. WILLIAMS GINECOLOGÍA, DE LA
1A EDICIÓN EN INGLÉS. MC GRAW HILL. USA. 2009, PP 716-717.

http://www.cenetec.salud.gob.mx/descargas/gpc/CatalogoMaestro/468-11_Ca_ovario/IMSS-468-11_GER_Cxncer_de_Ovario.pdf
ANÁLISIS DEL CASO CLÍNICO

IDENTIFICACIÓN DEL REACTIVO


Area: GINECOLOGÍA Y OBSTETRICIA
Especialidad: OBSTETRICIA
Tema: COMPLICACIONES DEL EMBARAZO Y DEL PARTO
Subtema: ATENCIÓN DEL PARTO Y DISTOCIAS

CASO CLÍNICO SERIADO

DURANTE SU VALORACIÓN OBSTÉTRICA EN LA UNIDAD TOCOQUIRÚRGICA ENCUENTRA A MUJER DE 40


AÑOS, GESTA 4, PARTOS 3, CON EMBARAZO DE 41 SEG. CON TRABAJO DE PARTO ACTIVO. A LA EXPLORACIÓN
PRESENTA TALLA DE 1.55 ÍNDICE DE MASA CORPORAL DE 30, FONDO UTERINO A 33 CM, PRESENTACIÓN
CEFÁLICA, DORSO IZQUIERDO, EN ESTE MOMENTO SE ENCUENTRA EN SEGUNDO PLANO, VARIEDAD DE
POSICIÓN OCCIPITO IZQUIERDA ANTERIOR, CON 6 CENTÍMETROS DE DILATACIÓN.

-.

TALLA 1.55.

-.

-.

-.

284 - CON BASE EN EL CUADRO CLÍNICO EN CASO DE PRESENTAR UNA DISTOCIA ESTA SERÁ:

POR Las DISTOCIAS POR VARIEDAD de posición son la occipito-transversa y occipito- posterior. La
VARIEDAD DE forma de encajamiento más frecuente es la OCCIPITO-ILIACA IZQUIERDA- ANTERIOR, LA CUAL SE
POSICIÓN CONSIDERA NORMAL Y ESTA PRESENTE EN EL FETO DE LA PACIENTE.

DE HOMBROS La DISTOCIA DE HOMBROS se produce cuando, tras la salida de la cabeza fetal, se detiene la
progresión del parto. Los factores de riesgo para la distocia de hombros son aquellos que
provocan un crecimiento fetal aumentado o una pelvis reducida. Los factores relacionados con la
estenosis pélvica, la baja talla, y los defectos de la marcha de la madre. LA DISTOCIA DE
HOMBROS ES MÁS FRECUENTE EN MUJERES DE TALLA BAJA.
DE Las DISTOCIAS DINÁMICAS son aquella producidas por la existencia de una actividad uterina
CONTRACCIÓN defectuosa o ineficaz para conseguir la dilatación cervical y/o descenso de la presentación.
Generalmente se asocian a otras condiciones patológicas como trabajo de parto prolongado,
analgesia o sedación de la madre, parto prematuro, ruptura uterina, sobredistension uterina, etc.
LAS DISTOCIAS DE CONTRACCIÓN GENERALMENTE SE ASOCIAN A UNA PATOLOGÍA DEL
TRABAJO DE PARTO NO HAY DATOS EN LA PACIENTE QUE SUGIERAN ASOCIACIÓN A ÉSTA.

DE PARTE La distocia de partes blandas es la dificultad del trabajo de parto originado por alguna de las
BLANDAS estructuras q componen alcanal de parto, o las causadas por la presencia de órganos o tejidos, q
al encontrarse enla pelvis materna obstruyen y desplazan la presentación. Esta es sumamente baja
en comparación con otros grupos de distocias, teniendo una incidencia del 0.2%. ES LA DISTOCIA
MENOS FRECUENTE DEL TRABAJO DE PARTO RAZÓN POR LA CUAL SE DESCARTA COMO
POSIBILIDAD DIAGNÓSTICA.

Bibliografía:
1. CUNNINGHAM G, LEVENO K, BLOMM S, HAUTH J, RPUSE D, SONG C. WILLIAMS OBSTETRICIA, 23A
EDICIÓN. MC GRAW HILL. USA. 2011 EN ESPAÑOL, PP 481-483.

285 - SE PRESENTA LA DISTOCIA CONSIDERADA POR LO QUE USTED DEBERÁ:

PRACTICAR La distocia de partes blandas se soluciona de manera inmediata empleando técnicas como la
EPISIOTOMÍA episiotomía, vaginotomía y cesáreas como las más empleadas. LA EPISIOTOMÍA ES LA TÉCNICA
BILATERAL. MÁS USADA EN LAS DISTOCIAS DE PARTES BLANDAS, NO CORRESPONDE AL TIPO DE DISTOCIA
QUE PODRÍA PRESENTAR LA PACIENTE.

REALIZAR EL MANEJO PRIMARIO DE LA DISTOCIA DE HOMBROS SE REALIZA MEDIANTE LA APLICACIÓN DE


MANIOBRAS MANIOBRAS ESPECIALES PARA MODIFICAR LAS DIMENSIONES DE LA PELVIS Y/O EL FETO. Las
ESPECIALES. Maniobras de primer nivel son manipulaciones externas que tratan de modificar la estática
(dimensiones) y la dinámica (posiciones) de la pelvis y del feto. Incluyen Maniobra de McRoberts y
Maniobra de Manzzati (el 60 % de las distocias de hombros se solucionan con estas dos maniobras).
Las Maniobras de segundo nivel suponen manipulaciones internas que tratan de modificar la
estática (dimensiones) o la dinámica (posiciones) del feto. Incluyen la Maniobra de Rubin II-Woods,
Maniobra de Jacquemier y Maniobra de Gaskin. Las Maniobras de tercer nivel incluyen fractura
deliberada de la clavícula fetal, Maniobra de Zavanelli, Sinfisiotomía e Histerectomía.

APLICAR El FÓRCEPS OBSTÉTRICO es un instrumento diseñado para ayudar a que nazca la cabeza del feto.
FÓRCEPS. Solo se utiliza para acelerar el parto o para corregir anormalidades en la relación cabeza del
producto y la pelvis materna, y que interfieran en el descenso de la cabeza de producto. EL USO DE
LOS FÓRCEPS SE LIMITA PARA CORREGIR LAS DISTOCIAS DE CABEZA DEL PRODUCTO, NO ES DE
UTILIDAD EN LA DISTOCIA DE HOMBROS.

PRACTICAR La Maniobra de Zavanelli usada en la distocia de hombros es la más espectacular por que supone la
CESÁREA. reintroducción de la cabeza en el canal del parto y la extracción fetal mediante cesárea. Existen
publicados, con éxito en numerosos casos, probablemente otros muchos intentos fracasados no se
han publicado, y también hay casos en los que se describe la enorme dificultad y la morbilidad
subsiguiente. Seria una maniobra extrema, quizás como primera consideración en aquellos casos en
que el desprendimiento de la cabeza fetal es parcial y apenas alcanza a que se vea la boca, cuando
la relajación uterina, la reposición y la extracción mediante cesárea podría ser los menos traumático
para el feto y para la madre. ES UNA MANIOBRA DE RESCATE EXTREMA CUANDO EL RESTO DE LAS
MANIOBRAS PARA CORREGIR LA DISTOCIA DE HOMBROS FRACASARAN.

Bibliografía:
1. CUNNINGHAM G, LEVENO K, BLOMM S, HAUTH J, RPUSE D, SONG C. WILLIAMS OBSTETRICIA, 23A
EDICIÓN. MC GRAW HILL. USA. 2011 EN ESPAÑOL, PP 483-485.

FIN DEL CASO CLÍNICO SERIADO


ANÁLISIS DEL CASO CLÍNICO

IDENTIFICACIÓN DEL REACTIVO


Area: GINECOLOGÍA Y OBSTETRICIA
Especialidad: OBSTETRICIA
Tema: DIABETES GESTACIONAL
Subtema: DIABETES GESTACIONAL

CASO CLÍNICO CON UNA PREGUNTA

MUJER DE 25 AÑOS CON PESO Y TALLA NORMAL, CON DIAGNÓSTICO RECIENTE DE DIABETES GESTACIONAL
EN SU SEMANA 29 DE EMBARAZO.

-.

PESO Y TALLA NORMAL, DIABETES


GESTACIONAL

-.

-.

-.

286 - LA SIGUIENTE INSULINA NO TIENE SUFICIENTE EVIDENCIA CIENTÍFICA PARA SER UTILIZADA EN
ESTE CASO:

INSULINA Los análogos de insulina LISPRO y ASPART, así como la INSULINA HUMANA DE ACCIÓN RÁPIDA, han
LISPRO. sido investigadas en el embarazo demostrando efectividad, seguridad clínica, mínimo transporte a
través de la placenta y no se ha informado teratogenicidad. Este tipo de medicamentos son categoría
B de la FDA. SI BIEN, EN GENERAL, NO EXISTE RECOMENDACIÓN ACTUAL POR LAS GPC PARA EL USO
DE LOS ANÁLOGOS DE INSULINA DURANTE EL EMBARAZO, LOS ANÁLOGOS DE ACCIÓN RÁPIDA
(LISPRO Y ASPART) PUEDEN RESULTAR BENEFICIOSOS EN MUJERES QUE NO LOGRAN METAS,
GRACIAS A QUE TIENEN UN MÍNIMO TRANSPORTE A TRAVÉS DE LA PLACENTA.
INSULINA La DIABETES GESTACIONAL es un padecimiento caracterizado por la intolerancia a los carbohidratos
RÁPIDA. con diversos grados de severidad, que se reconoce por primera vez durante el embarazo, y que puede
o no resolverse después de éste. Factores de riesgo. La insulina es el tratamiento de elección en
cualquier tipo de diabetes durante el embarazo. La mayoría de las preparaciones de insulina utilizadas
hoy en día han demostrado ser seguras y promueven un adecuado control glucémico. IMPORTANTE:
El tratamiento farmacológico de la DIABETES GESTACIONAL se debe considerar cuando la dieta y el
ejercicio no logran las cifras meta (menor de 95mg/dl en ayuno y 120mg/dl en postprandial de 2 hrs)
para el control de la glucosa en sangre en un periodo de 2 semanas. Es aconsejable combinar insulina
intermedia (NPH) con insulina de acción rápida a fin de evitar la hiperglucemia postprandial y simular
la secreción de insulina fisiológica durante todo el día. Se adiciona insulina rápida (insulina humana
regular) cuando no se alcanza la meta terapéutica de la glucemia posprandial. La combinación de
insulina de acción rápida e intermedia debe ser administrada 30 minutos antes del desayuno y de la
cena. LA INSULINA RÁPIDA (REGULAR) ES IDEAL PARA COMPLEMENTAR EL TRATAMIENTO EN
AQUELLAS MUJERES GESTANTES QUE NO CUMPLEN METAS CON INSULINA INTERMEDIA (NPH)
SOLA.

INSULINA En pacientes que no logran cumplir las metas de control glucémico con cambios de estilo de vida y
NPH. terapia medica nutricional, se recomienda el inicio de terapia con insulina NPH. LA INSULINA DE
ACCIÓN INTERMEDIA (NPH) ES LA ÚNICA APROBADA PARA EL USO DE LA TERAPIA BASAL Y ES
CONSIDERADA COMO EL ESTÁNDAR DE CUIDADO PARA LA DIABETES Y EMBARAZO.

INSULINA Algunos estudios sugieren que los análogos de insulina de acción lenta como glargina y detemir
GLARGINA. podrían ayudar al control de la diabetes en mujeres embarazadas; sin embargo, “no existen ensayos
clínicos controlados a gran escala que afirmen su seguridad y eficacia” a largo plazo. La insulina
glargina pertenece a la clasificación C de la FDA. NO EXISTE SUFICIENTE EVIDENCIA PARA
RESPALDAR EL USO DE INSULINA GLARGINA DURANTE EL EMBARAZO, POR LO TANTO, NO ES
RECOMENDABLE SU USO, PUESTO QUE NO EXISTE GARANTÍA EN SU PERFIL DE SEGURIDAD.
RECUERDA: estos son reactivos tipo ENARM donde en ocasiones es necesario elegir la mejor
respuesta correcta, si bien, en general, no existe recomendación para los análogos de insulina durante
el embarazo, el uso de Glargina es el menos estudiado, además de tener al parecer, un perfil menos
favorable de seguridad, razón por la que deberás elegir esta respuesta como correcta.

Bibliografía:
1. DIAGNÓSTICO Y TRATAMIENTO DE LA DIABETES EN EL EMBARAZO. EVIDENCIAS Y RECOMENDACIONES.
GUÍA DE PRÁCTICA CLÍNICA. MÉXICO: SECRETARÍA DE SALUD; 03/11/2016.

http://www.cenetec-difusion.com/CMGPC/IMSS-320-10/ER.pdf
ANÁLISIS DEL CASO CLÍNICO

IDENTIFICACIÓN DEL REACTIVO


Area: PEDIATRÍA
Especialidad: URGENCIAS PEDIÁTRICAS
Tema: URGENCIAS NEUROLÓGICAS PEDIÁTRICAS
Subtema: CRISIS CONVULSIVAS ABORDAJE EN EL PRIMER NIVEL DE
ATENCIÓN

CASO CLÍNICO CON UNA PREGUNTA

NIÑA DE 8 AÑOS DE EDAD CON ANTEDENTE DE EPILEPSIA, EN TRATAMIENTO CON DIFENILHIDANTOÍNA


HASTA HACE UNA SEMANA, DESDE ENTONCES NO TOMA ANTICONVULSIVANTES POR INICIATIVA DE LOS
PADRES. ES LLEVADA A LA UNIDAD DE SALUD PORQUE DESDE HACE 5 MINUTOS PRESENTA CRISIS
CONVULSIVA GENERALIZADA.

Niña de 8 años.

Epilepsia, suspendió tratamiento


anticonvulsivo.

Crisis convulsiva tónico clónica


generalizada de 5 minutos de duración

Aún en período ictal.

287 - EL MEDICAMENTO DE ELECCIÓN QUE DEBE DE ADMINISTRARSE EN ESTE MOMENTO A LA


PACIENTE ES:

DIAZEPAM. El manejo de una crisis convulsiva que se prolonga por más de 5 minutos debe iniciar de
forma temprana, desde el traslado al hospital el paciente puede recibir benzodiacepinas
(lorazepam o diazepam). Tanto el lorazepam como el DIAZEPAM intravenosos son útiles y
seguros en el control de las crisis convulsivas tónico clónicas, pero el lorazepam tiene la
ventaja sobre el diazepam de proporcionar un tiempo de acción más prolongado. El
lorazepam tiene una eficacia del 80% para yugular una crisis convulsiva, la del DIAZEPAM
es del 70%. En el caso de crisis convulsivas generalizadas, si después de 5 minutos no ha
cedido espontáneamente la crisis, se recomienda como tratamiento de elección lorazepam
por vía intravenosa (IV), intramuscular (IM) o rectal (VR) para yugularla. En caso de no
contar con lorazepam se puede administrar como alternativa DIAZEPAM por vía
intravenosa o rectal. SI BIEN EL DIAZEPAN ESTÁ BIEN INDICADO PARA ESTE CASO, NO ES
LA OPCIÓN MÁS CORRECTA, PUESTO QUE EXISTE UNA BENZODIACEPINA MÁS EFICAZ
DENTRO DE LAS OPCIONES DE RESPUESTA (LORAZEPAM).
LORAZEPAM. En primer lugar debes tener en cuenta el momento en que se encuentra la paciente,
“periodo ictal”, lo cual es básico para elegir el medicamento a utilizar. Existen
anticomiciales de primera elección para el manejo ambulatorio de pacientes con crisis
convulsivas parciales o generalizadas, y otros que están recomendados para el manejo
específico de los pacientes con crisis convulsivas generalizadas en periodo ictal, es decir,
para su uso en el periodo agudo de la enfermedad. LAS BENODIAZEPINAS SON EL
TRATAMIENTO DE PRIMERA ELECCIÓN EN EL MANEJO INICIAL DE LAS CRISIS
CONVULSIVAS EN LA FASE ICTAL. El manejo de una crisis convulsiva que se prolonga por
más de 5 minutos debe de iniciar de forma temprana inclusive desde el traslado a una
unidad hospitalaria alguna benzodiacepina (lorazepam o diazepam). Tanto el lorazepam
como el diazepam (por vía intravenosa) son igualmente eficaces para el control de las crisis
colvulsivas en la fase ictal. DEBERÁS ELEGIR AL LORAZEPAM PARA ESTE CASO, YA QUE
TIENE LA VENTAJA SOBRE EL DIAZEPAM DE TENER UN TIEMPO DE ACCIÓN MÁS
PROLONGADO Y REDUCIR EL RIESGO DE FRACASO EN EL CESE DE LAS CONVULSIONES.

DIFENILHIDANTOÍNA. Las BENZODIACEPINAS son el tratamiento de primera elección para el tratamiento de las
crisis convulsivas en fase ictal. LA DIFENILHIDANTOÍNA NO CORRESPONDE AL
TRATAMIENTO DE ELECCIÓN EN ESTE CASO.

VALPROATO DE Si después de administrar dos dosis de benzodiacepinas la crisis convulsiva continua, el


SODIO. siguiente paso es utilizar monoterapia con antiepiléptico de segunda línea (fenitoína,
fofenitoina, valproato de sodio o fenobarbital) a dosis de impregnación. Si el estado
epiléptico continúa después de 60 minutos de haber iniciado la crisis convulsiva al manejo
se le agregará un anticonvulsivo de tercera línea. Si el estado epiléptico se prolonga más
allá de 90 minutos se considera inducir coma barbitúrico. EL VALPROATO DE SODIO PUEDE
APLICARSE SI HAY PERSISTENCIA DE LAS CONVULSIONES TRAS LA ADMINISTRACIÓN DE
2 DOSIS DE BARBITÚRICOS.

Bibliografía:
1. GUÍA DE PRÁCTICA CLÍNICA, TRATAMIENTO DE LA PRIMERA CRISIS CONVULSIVA EN NIÑOS. MÉXICO:
SECRETARIA DE SALUD; 2009

http://www.cenetec.salud.gob.mx/descargas/gpc/CatalogoMaestro/244_IMSS_09_PRIMERA_CRISIS_CONVULSIVA_NINOS/EyR_IMSS_244_09.pdf
ANÁLISIS DEL CASO CLÍNICO

IDENTIFICACIÓN DEL REACTIVO


Area: PEDIATRÍA
Especialidad: INFECTOLOGIA PEDIÁTRICA
Tema: GASTROENTERITIS INFECCIOSA
Subtema: GEPI VIRAL

CASO CLÍNICO CON UNA PREGUNTA

LACTANTE DE 8 MESES, ES LLEVADA A URGENCIAS POR PRESENTAR VÓMITOS ABUNDANTES EN NÚMERO DE


15, EVACUACIONES LIQUIDAS ABUNDANTES, SIN MOCO NI SANGRE, EN NÚMERO DE 14 EN LAS ÚLTIMAS 24
HORAS. A LA EXPLORACIÓN: TA 50/20, FC 100XMIN FR 35 X MIN. LLENADO CAPILAR DE 6 SEG, OJOS
HUNDIDOS, ABDOMEN CON PERISTALSIS INCREMENTADA. NO DATOS DE IRRITACIÓN PERITONEAL.

lactante de 8 meses

--

VÓMITOS ABUNDANTES EN NÚMERO DE


15, EVACUACIONES LíQUIDAS
ABUNDANTES, SIN MOCO NI SANGRE, EN
NÚMERO DE 14 EN LAS ÚLTIMAS 24
HORAS

TA 50/20, FC 100XMIN FR 35 X MIN.


LLENADO CAPILAR DE 6 SEG, OJOS
HUNDIDOS, ABDOMEN CON PERISTALISIS
INCREMENTADA. NO DATOS DE
IRRITACION PERITONEAL

--

288 - EL DIAGNÓSTICO MAS PROBABLE DE ESTE LACTANTE ES:


DESHIDRATACIÓN CARACTERÍSTICAS DESHIDRATACIÓN MODERADA • Pérdida del peso corporal. Menor de 5 a
MODERADA POR 5-9%, lactantes 3-7%, mucosas secas, irritabilidad, ojos hundidos, poca lágrima, inicia pliegue
DIARREA en piel, llenado capilar retardado, fontanela hundida, sed aumentada, orina disminuida,
OSMÓTICA taquicardia leve, TA normal-baja, respiración normal-baja. EL PACIENTE TIENE DATOS CLAROS
DE HIPOTENSIÓN, TAQUICARDIA E HIPOPERFUSIÓN TISULAR QUE "NO" SE CLASIFICAN EN
LA DESHIDRATACIÓN MODERADA. En la infección por Rotavirus se activa el sistema nervioso
entérico con secreción de fluidos y electrolitos (diarrea secretora). Un mecanismo tardío es la
muerte de los enterocitos, aparentemente por apoptosis, que contribuye a la diarrea osmótica.
EN LA DIARREA AGUDA POR ROTAVIRUS LA DIARREA ES SECRETORA POR LO QUE NO
CORRESPONDE A LA FASE ACTUAL DE LA ENFERMEDAD.

SEPSIS POR El síndrome disentérico (o diarrea disenteriforme o simplemente disentería) es una entidad
DISENTERIA clínica más específica que se caracteriza por evacuaciones numerosas, de escasa cantidad,
compuestas fundamentalmente de moco y sangre con poca materia fecal, casi siempre
acompañadas de cólicos, pujo, tenesmo y fiebre, e incluso, en ocasiones, de estado
toxiinfeccioso. LA DIARREA DEL PACIENTE NO TIENE MOCO NI SANGRE POR LO QUE N
CORRESPONDE A ÉSTA OPCIÓN DIAGNÓSTICA.

CHOQUE SHOCK HIPOVOLÉMICO POR PÉRDIDA DE LÍQUIDOS Y ELECTROLITOS. • La pérdida de líquidos


HIPOVOLÉMICO disminuye el volumen intravascular y la precarga. La pérdida de agua en grandes volúmenes
POR DIARREA de líquido genera descompensación, que se manifiesta a través de la disminución del estado
PROBABLEMENTE de conciencia, taquicardia, perfusión inadecuada de la piel con prolongación del tiempo de
VIRAL llenado capilar, oliguria y, por último, hipotensión. • El shock no hemorrágico se desarrolla en
pacientes con diarrea, vómitos, pérdida de orina, pérdidas por evaporación, terceros espacios
de líquido 8peritonitis, edema) y quemaduras. • "La causa más frecuente de diarrea aguda en
menores de 5 años es la gastroenteritis por rotavirus". EL PACIENTE TIENE DATOS DE CHOQUE
HIPOVOLÉMICO ASOCIADO A GEPI.

CHOQUE CAUSAS DE CHOQUE HIPOVOLÉMICO: • Pérdidas gastrointestinales, • Producción de orina


HIPOVOLÉMICO excesiva, administración de diuréticos, • Administración de manitol, • Hipoalbuminemia, •
POR Quemaduras, • Pérdidas de líquido en terceros espacios (peritonitis, edema), • Pérdida de
INTOXICACIÓN sangre traumática. LA INTOXICACIÓN ALIMENTARIA POR SI SOLA NO PRODUCE CHOQUE
ALIMETARIA HIPOVOLÉMICO A MENOS QUE DESENCADENE UNA DIARREA PROFUSA.

Bibliografía:
PRINCIPLES AND PRACTICS OF PEDIATRIC INFECTIOUS DISEASES. SARAH S. LONG. CHURCHILL,
LIVINGSTONE. EDICIÓN 2A. 2002. PAG. 1105-1109.
ANÁLISIS DEL CASO CLÍNICO

IDENTIFICACIÓN DEL REACTIVO


Area: MEDICINA INTERNA
Especialidad: NEUROLOGÍA
Tema: CEFALEA
Subtema: CEFALEA Y MIGRAÑA

CASO CLÍNICO SERIADO

HOMBRE DE 32 AÑOS DE EDAD, CON ANTECEDENTE DE PADECER CEFALEAS RECURRENTES DESDE LOS 14
AÑOS DE EDAD. INGRESA AL SERVICIO DE URGENCIAS POR PRESENTAR CEFALEA DE 6 HORAS DE
EVOLUCIÓN, FRONTOORBITARIA, PUNZANTE, PRECEDIDA DE HIPEROSMIA, FOTOFOBIA Y TINITUS Y
ACOMPAÑADA DE NÁUSEAS Y VÓMITOS. A LA EXPLORACIÓN PRESENTA SIGNOS VITALES NORMALES Y SE
ENCUENTRA NEUROLÓGICAMENTE INTEGRO.

HOMBRE DE 32 AÑOS.

CEFALEAS RECURRENTES DESDE LOS 14


AÑOS DE EDAD.

CEFALEA DE 6 HORAS DE EVOLUCIÓN,


FRONTOORBITARIA, PUNZANTE,
PRECEDIDA DE HIPEROSMIA, FOTOFOBIA
Y TINITUS Y ACOMPAÑADA DE NÁUSEAS Y
VÓMITOS. SINTOMATOLOGÍA PREVIA QUE
DEBE SER CONSIDERADA AURA.

SIGNOS VITALES NORMALES Y SE


ENCUENTRA NEUROLóGICAMENTE
INTEGRO.

-.

289 - EL DIAGNÓSTICO CLÍNICO MÁS PROBABLE ES:


MIGRAÑA La MIGRAÑA es el subtipo de cefalea primaria severa más común. Se caracteriza por ser “unilateral”,
CON AURA. pulsátil, puede presentarse en minutos u horas, de intensidad moderada a severa, se asocia con
“náusea y/o vómito y/o sensibilidad a la luz y al ruido, incapacitante y se agrava con la actividad
física”. Se clasifica por la presencia o ausencia de aura. Criterios diagnósticos para MIGRAÑA CON
AURA. A. Al menos dos crisis que cumplen los criterios B y C. B. Uno o más de los síntomas de aura
siguientes totalmente reversibles: 1. Visuales. 2. Sensitivos. 3. Del habla o del lenguaje. 4. Motores. 5.
Troncoencefálicos. 6. Retinianos. C. Al menos dos de las siguientes cuatro características: 1.
Progresión gradual de al menos uno de los síntomas de aura durante un periodo mayor a 5 minutos
y/o dos o más síntomas se presentan consecutivamente. 2. Cada síntoma de aura tiene una duración
de entre 5 y 60 minutos. 3. Al menos uno de los síntomas de aura es unilateral. 4. El aura se
acompaña o sigue, antes de 60 minutos de la cefalea. D. Sin mejor explicación por otro diagnóstico
de la ICHD-III (Clasificación internacional de cefaleas) y se ha descartado un accidente isquémico
transitorio. LA PRESENCIA DE HIPEROSMIA, FOTOFOBIA Y TINITUS, PREVIO A LA INSTALACIÓN DE
LA MIGRAÑA ES LA CLAVE PARA FUNDAMENTAR ESTE DIAGNÓSTICO.

CEFALEA EN La CEFALEA EN RACIMOS (CR) es una de las cefaleas más terriblemente invalidantes. Desde el punto
RACIMOS. de vista clínico los pacientes que la sufren describen un dolor atroz, lancinante referido a la región
periocular y que se acompaña de una serie de síntomas y signos, fundamentalmente de disfunción
autonómica, como lagrimeo, edema palpebral o rinorrea, por citar sólo algunos, que facilitan su
diagnóstico. Sin embargo, a pesar de su característico perfil clínico continúa siendo pobremente
reconocida y mal controlada. Se caracteriza por la aparición de ataques de dolor periocular,
estrictamente unilaterales, de gran intensidad junto con signos autonómicos locales homolaterales,
como lagrimeo, rinorrea o edema palpebral. La clasificación actual de la Internacional Headache
Society (IHS) sitúa la CR en el apartado 3 junto al resto de cefaleas trigémino-autonómicas. Se
conoce también como Hemicránea angioparalítica, Faciocefalalgia autonómica, Vasodilatación
simpática hemicefálica, Neuralgia de Horton, cefalea histamínica o cefalea en acúmulos (del inglés:
Cluster headache) o en tandas, calificativos estos últimos que hacen referencia al agrupamiento de
los ataques del dolor en periodos de tiempo que denominamos racimos. De esa manera quedan
definidas las características cardinales de la CR: periodicidad de los ataques, dolor periocular de gran
intensidad y signos autonómicos acompañantes. Aunque su frecuencia de aparición es baja, resulta
crucial conocer su manejo terapéutico debido a que el dolor atroz que ocasiona es uno de los más
invalidantes que se conocen. Acepciones del estilo de “cefalea en racimos” expresan de forma
sumamente gráfica lo que su presencia puede conllevar a quienes la padecen. Distinguimos dos
formas de presentación. La forma episódica (CRE), en la cual se alternan los racimos con periodos
libres de dolor que habitualmente se prolongan durante meses o años; esta entidad mantiene unos
criterios bien definidos y, por lo general, no ofrece problemas especiales en el diagnóstico o
tratamiento. Por otro lado, la forma crónica (CRC) presente en el 10% de los casos, se caracteriza
porque los ataques duran más de un año o porque los periodos asintomáticos son inferiores a un
mes. Una pequeña parte de ellos no responden al tratamiento médico, representado las formas de
CRC refractaria que habitualmente conllevan un difícil manejo terapéutico. EN LA CEFALEA EN
RACIMOS EL DOLOR EN ESTOS CASOS ES PERIOCULAR, UNILATERAL ACOMPAÑADO DE SIGNOS DE
DISFUNCIÓN AUTONÓMICA, LO CUAL NO ENCAJA CON EL CUADRO CLÍNICO DEL PACIENTE.

CEFALEA La CEFALEA TENSIONAL es el subtipo de cefalea primaria más común. El dolor es típicamente
TENSIONAL. “bilateral”, opresivo, y de intensidad leve a moderada. “No se presentan náuseas” y no se agrava con
la actividad física. Se pueden asociar la sensibilidad a la luz, al ruido o hipersensibilidad pericraneal.
Se distinguen dos tipos: 1. Cefalea tensional episódica infrecuente: Episodios de cefalea poco
frecuentes, de localización típicamente “bilateral”, con dolor tensivo u opresivo de intensidad leve a
moderada y con una duración de minutos a días. Este dolor no empeora con la actividad física
habitual ni está asociado con náuseas, pero podrían presentarse fotofobia o fonofobia. 2. Cefalea
tensional episódica frecuente: Episodios frecuentes de cefalea, de localización típicamente
“bilateral”, con dolor tensivo u opresivo de intensidad leve a moderada y con una duración de
minutos a días. Este dolor no empeora con la actividad física habitual ni está asociado con náuseas,
pero podrían presentarse fotofobia o fonofobia. La fisiopatología de la CTT es multifactorial,
involucra mecanismos periféricos y centrales (sensibilización). Los hallazgos clínicos más
importantes en la CTT son el aumento de la sensibilidad miofascial pericraneal y el aumento de la
contracción muscular a la palpación. También participan factores emocionales por modificación del
sistema límbico, incremento del tono vascular por hiperactividad del sistema simpático, cambios en
la presión intracraneana del líquido cefalorraquídeo o del sistema venoso intracraneano, y fallas en
la inhibición supraespinal por disfunción del sistema serotoninérgico. EN LA CEFALEA TENSIONAL EL
DOLOR ES GENERALIZADO, NO JUSTIFICA LA PRESENCIA DE SÍNTOMAS PREMONITORIOS COMO EN
EL CASO DEL PACIENTE.
MIGRAÑA Criterios diagnósticos para MIGRAÑA SIN AURA. A. Al menos cinco crisis que cumplen los criterios B
SIN AURA. a D. B. Episodios de cefalea de entre 4 a 72hrs de duración (no tratados o tratados sin éxito). C. La
cefalea presenta al menos dos de las siguientes cuatro características. 1. Localización unilateral. 2.
Carácter pulsátil. 3. Dolor de intensidad moderada o severa. 4. Empeorada por la actividad física o
que condiciona el abandono de ésta. D. Al menos uno de los siguientes durante la cefalea: 1.
Náuseas y/o vómitos. 2. Fotofobia y fonofobia. E. Sin mejor explicación por otro diagnóstico de la
ICHD-III (Clasificación internacional de cefaleas). LA PACIENTE PRESENTA SÍNTOMAS
PREMONITORIOS PREVIOS A LA CEFALEA (HIPEROSMIA, FOTOFOBIA Y TINITUS) QUE CONSTITUYEN
EL AURA, RAZÓN POR LO QUE ESTA RESPUESTA ES INCORRECTA.

Bibliografía:
1. MANEJO DE CEFALEA TENSIONAL Y MIGRAÑA EN EL ADULTO. EVIDENCIAS Y RECOMENDACIONES.
MÉXICO: SECRETARÍA DE SALUD. 2. LONGO DL, FAUCI AS, KASPER DL, HAUSERSL, JAMESON JL,
LOSCALZOJ. HARRISON. PRINCIPIOS DE MEDICINA INTERNA, 18A EDICIÓN. MC GRAW HILL. NEW YORK,
USA. 2012, PP 115-117.

http://www.cenetec-difusion.com/CMGPC/IMSS-047-08/ER.pdf

290 - EL PACIENTE YA HA SIDO TRATADO CON LOS MEDICAMENTOS DE PRIMERA ELECCIÓN, SIN
RESULTADOS POSITIVOS, EN ESTE CASO DEBERÁ INDICAR:

OXÍGENO. Los fármacos empleados para tratar las formas de CEFALEA EN RACIMOS CRÓNICA Y EPISÓDICA
son similares, si bien, ciertos medicamentos y sus posibles combinaciones pueden ser más útiles
para tratar las formas crónicas. En el tratamiento de la CR distinguimos una terapia para yugular
los ataques agudos y una terapia preventiva para evitar en lo posible la aparición de dolor o
disminuir su duración, frecuencia e intensidad durante los periodos activos. Siempre se debe
comenzar recordando al paciente que debe suprimir los factores precipitantes del ataque en el
caso de que los hubiere. Por ejemplo, los enfermos deben dejar el alcohol, sobre todo si padecen
la forma crónica. También puede ser necesario suspender o modificar fármacos vasodilatadores.
Algunos enfermos evitan dormir la siesta y otros procuran no viajar en avión o ascender a alturas
durante los periodos activos. En aquellos pacientes que viajen a grandes alturas, se han
notificado casos de buena respuesta a la Acetazolamida durante 4 días, empezando 2 días antes.
De forma adicional, se debe instar a los pacientes a evitar el tabaquismo debido al uso
concomitante de medicamentos vasoconstrictores como triptanes o ergotamínicos. Al igual que
en la migraña, el tratamiento se puede dividir en tratamiento sintomático agudo de las crisis
individuales y tratamiento profiláctico, el cual puede incluir una primera parte que
denominamos fase de inducción. Tratamiento del ataque. El tratamiento agudo del ataque es
común en las variedades episódica y crónica. En primer lugar, recordar que los A.I.N.E.S, los
analgésicos comunes y los opiáceos no suelen tener ninguna efectividad en las crisis de CR.
TRATAMIENTO DE ELECCIÓN. Oxígeno 7 litros por minuto durante 15 min por mascarilla.
Sumatriptán 6 mg por vía subcutánea. Sumatriptán 20 MG intranasal Lidocaína intranasal
Lidocaína 10%, 20-60 mg en gotas o aerosol nasal
ZOLMITRIPTÁN. TRATAMIENTO FARMACOLÓGICO DE MIGRAÑA. - Acetaminofén es el medicamento de primera
línea para cefaleas leves a moderadas. Para la remisión del dolor, no se debe suspender su uso
hasta que se logre su efecto máximo (650 - 1300 mg/cada 4 hrs en máximo 2 dosis en las crisis
agudas de cefalea). Sólo (100mg) o combinado con metoclopramida (10mg) es el tratamiento de
elección para ataques agudos de migraña. - Ácido acetilsalicílico de 1000mg (650 a 1300
mg)/cada 4 horas en 2 tomas al día en ataques agudos de cefalea. Se recomienda en
combinación con 10mg de metoclopramida para reducir los síntomas de náuseas, vómito,
fotofobia y fonofobia. - Ibuprofeno es uno de los medicamentos de primera línea para cefaleas
en casos moderados (400 a 1200 mg cada 6 hrs sin exceder 10 días) - Zolmitriptán 2.5-5mg para
el tratamiento abortivo de pacientes con migraña. - Eletriptán a 40mg, se recomienda para el
ataque agudo de migraña, con mejora de la respuesta sobre todo a las 2hrs y a las 24hrs de su
ingesta. IMPORTANTE: - Actualmente el Naproxeno NO se recomienda como opción de
tratamiento para la migraña, ya que otros analgésicos de uso común reportan mejores
resultados. - Aunque el uso de Diclofenaco a 50mg, está recomendado durante el ataque agudo
de migraña, se debe tomar en cuenta que sólo una minoría de los pacientes presenta
desaparición del dolor con este medicamento. - El uso de zolmitriptán 10mg se asocia con mayor
número de eventos adversos, siendo estos en su mayoría de intensidad moderada y transitoria.
ACTUALMENTE SE RECOMIENDA EL USO DE TRIPTANES PARA EL TRATAMIENTO ABORTIVO DE
LA MIGRANA DURANTE EL ATAQUE AGUDO. El ZOLMITRIPTAN se administra por vía sublingual
en aquellos pacientes intolerantes a la vía oral como primera línea o en aquellos en los cuales se
han utilizado AINES sin mejoría clínica, cuando los pacientes no muestran mejoría puede
agregarse una segunda dosis o Ketorolaco 30 a 60mg. RESUMEN. Los TRIPTANES son un grupo
de fármacos que se emplean en el tratamiento del ataque agudo de migraña. OJO: No son útiles
en otro tipo de cefaleas (salvo en las cefaleas en racimos histaminérgicas tipo cluster), ni como
preventivos para evitar los episodios migrañosos. Los triptanes se comportan como fármacos
específicos y selectivos en el tratamiento sintomático de las crisis de migraña. Son específicos
porque actúan directamente bloqueando los mecanismos potencialmente implicados en la
génesis de la migraña. Son selectivos porque su espectro de acción se circunscribe a modular la
acción un grupo limitado de receptores serotoninérgicos; en ello se diferencian de los agentes
ergóticos que actúan también sobre otros receptores y sus reacciones adversas son más
numerosas, frecuentes y persistentes.

AMITRIPTILINA. Con anterioridad, este medicamento era considerado de segunda línea para el tratamiento del
ataque agudo de migraña cuando los AINES no resultaban eficaces, sin embargo, EN LA
ACTUALIDAD HA SIDO DESPLAZADA POR LOS TRIPTANES. Aunque es el único antidepresivo
tricíclico probado para el tratamiento agudo de la migraña, su uso se restringe al TRATAMIENTO
PROFILÁCTICO DE LA MIGRAÑA. Se prefiere en pacientes con comorbilidades tales como
depresión, insomnio y dolor neuropático. Es de suma importancia la monitorización semanal de
la TA en los pacientes en que se utiliza, así como el fondo de ojo, peso e IMC. LA AMITRIPTILINA
ACTUALMENTE SE RESTRINGE AL TRATAMIENTO PROFILÁCTICO DE LA MIGRAÑA.

PROPANOLOL. Tratamiento PROFILÁCTICO de la migraña: 1. BETA BLOQUEADORES: SE CONSIDERAN EL


TRATAMIENTO DE ELECCIÓN PARA LA PROFILAXIS DE LA MIGRAÑA. Propanolol es el
recomendado a dosis de 40 mg/día. 2. ANTIDEPRESIVOS TRICÍCLICOS: están recomendados,
pero siempre se deben tomar en cuenta sus efectos adversos. La amitriptilina es el único de los
tricíclicos con eficacia probada en el tratamiento de migraña. 3. ANTICOLVULSIVOS: Topiramato
es el más estudiado, se recomienda a razón de 100mg/día para el manejo profiláctico. Se sugiere
iniciar con 25mg/día e ir aumentando la dosis 25 a 50mg por semana hasta alcanzar un máximo
de 100mg dos veces al día o la dosis máxima tolerada y vigilar los efectos adversos. El Valproato
se ha usado con éxito para reducir la frecuencia de cefaleas y es razonablemente bien tolerado
en pacientes adultos con migraña episódica. No está recomendado durante la gestación por su
efecto teratogénico.

Bibliografía:
1. MANEJO DE CEFALEA TENSIONAL Y MIGRAÑA EN EL ADULTO. EVIDENCIAS Y RECOMENDACIONES.
MÉXICO: SECRETARÍA DE SALUD. 2. LONGO DL, FAUCI AS, KASPER DL, HAUSERSL, JAMESON JL,
LOSCALZOJ. HARRISON. PRINCIPIOS DE MEDICINA INTERNA, 18A EDICIÓN. MC GRAW HILL. NEW YORK,
USA. 2012.

http://www.cenetec-difusion.com/CMGPC/IMSS-047-08/ER.pdf

FIN DEL CASO CLÍNICO SERIADO


ANÁLISIS DEL CASO CLÍNICO

IDENTIFICACIÓN DEL REACTIVO


Area: PEDIATRÍA
Especialidad: CRECIMIENTO Y DESARROLLO
Tema: ALTERACIONES DEL CRECIMIENTO
Subtema: PUBERTAD PRECOZ Y RETARDADA

CASO CLÍNICO CON UNA PREGUNTA

ESCOLAR FEMENINA DE 7 AÑOS DE EDAD, ES LLEVADA A LA CONSULTA POR LA PRESENCIA DE TELARQUIA


BILATERAL. A LA EXPLORACIÓN CON TANNER MAMARIO II, PÚBICO DE I Y EDAD ÓSEA DE 10 AÑOS. SE
REALIZA TAC DE CRANEO QUE SE REPORTA NORMAL. SE SOLICITAN NIVELES DE HORMONA LUTEINIZANTE
QUE SE REPORTAN CON UNA CONCENTRACIÓN MAYOR A LA NORMAL.

escolar de 7 años de edad

femenina

telarquia bilateral

tanner mamario ii, púbico de i

edad ósea de 10 años. tac cráneo normal lh


mayor a la normal.

291 - LA CONDUCTA A SEGUIR EN ESTA PACIENTE DEBE SER:

VIGILANCIA TELARQUIA PRECOZ: Se llama telarquia precoz benigna a la aparición de desarrollo mamario
PERIÓDICA Y aislado en una niña menor de 8 años, sin ningún signo de pubertad evidenciable. Puede ser una
REPOSICIÓN patología benigna sin trascendencia clínica o el inicio de una pubertad precoz verdadera.
ENZIMÁTICA Cuando la edad de presencia es muy temprana es obligatorio descartar siempre una etiología
RECOMBINANTE tumoral; el hamartoma de hipotálamo es una de las causas más frecuentes.

VIGILANCIA SITUACIÓN HORMONAL: Generalmente los niveles de estrógenos y LH (hormona luteinizante)


PERIÓDICA Y son prepuberales, sin embargo, los de la hormona FSH (hormona estimulante de los folículos)
TRATAMIENTO basal y tras su estimulación con el GnRH (hormona liberadora de gonadotropinas) son mayores
QUIRÚRGICO que en controles normales. Se distinguen dos tipos de pubertad precoz: una que depende de la
Secreción de GnRH, que es la pubertad precoz verdadera o central, y otro tipo que es
independiente de la secreción de GnRH, que es la pseudopubertad precoz o pubertad precoz
periférica.
VIGILANCIA DIAGNÓSTICO Y TRATAMIENTO: No debemos olvidar que la telarquia es el primer signo de una
PERIÓDICA Y pubertad precoz y, además, si aparece en niñas menores de 4 años será obligatorio descartar
EDUCACIÓN una patología neurológica. Este cuadro no precisa tratamiento, sólo se debe hacer observación y
NUTRICIONAL diagnóstico diferencial con una pubertad precoz central, una pseudopubertad o una exposición
a estrógenos.

VIGILANCIA Las causas de pubertad independiente de GnRH incluyen un origen gonadal, o adrenal, o
PERIÓDICA, Y ectópico, o una fuente exógena de producción hormonal. Las causas de origen gonadal incluyen
CONSEJO el síndrome de McCune-Albright, la pubertad precoz masculina familiar y los tumores. La
GENÉTICO enfermedad de McCune-Albright es un síndrome de rara aparición caracterizado por una
displasia fibrosa poliostótica, manchas café-au-lait y varias hiperfunciones endocrinas que
conducen comúnmente a una pubertad precoz. En ocasiones puede acompañarse de otros tipos
de alteraciones. En el 95 % de los casos reportados, el paciente es del sexo femenino. DADO QUE
NO SE ASOCIA A TUMORES PRODUCTORES DE GnRH, DEBE MANTENERSE VIGILANCIA
ESTRECHA Y CONSEJO GENÉTICO A LOS PADRES ANTE LA POSIBILIDAD DE ALGÚN SÍNDROME
ASOCIADO A PUBERTAD PRECOZ.

Bibliografía:
NELSON. TRATADO DE PEDIATRÍA. BEHRMAN. MC GRAW HILL. EDICIÓN 16. 2001. PAG. 1841-1850.
ANÁLISIS DEL CASO CLÍNICO

IDENTIFICACIÓN DEL REACTIVO


Area: MEDICINA INTERNA
Especialidad: NEFROLOGÍA
Tema: INSUFICIENCIA RENAL AGUDA
Subtema: INSUFICIENCIA RENAL AGUDA INTRINSECA Y NEFROPATÍAS

CASO CLÍNICO CON UNA PREGUNTA

MUJER DE 41 AÑOS DE EDAD, CON DIAGNÓSTICO DE LUPUS ERITEMATOSO SISTÉMICO DESDE LOS 30 AÑOS
DE EDAD, HACE SEIS MESES PRESENTÓ UNA FUERTE RECAÍDA DE LA ENFERMEDAD CON ASOCIACIÓN DE
NEFRITIS LÚPICA. POR TAL MOTIVO, SE INICIÓ TRATAMIENTO CON CICLOS DE CICLOFOSFAMIDA +
METILPREDNISOLONA. DURANTE LA CONSULTA DE CONTROL, SE OBSERVA MARCADA PANCITOPENIA EN LA
BIOMETRÍA HEMÁTICA.

41 años.

¥ lupus eritematoso sistémico + nefritis, en


tratamientocon metilprednisolona +
ciclofosfamida.

marcada pancitopenia.

292 - CONSIDERAMOS QUE LA CICLOFOSFAMIDA ESTÁ LESIONANDO LAS CÉLULAS PRECURSORAS


SANGUÍNEAS DEBIDO A QUE ES:
UN AGENTE Existen diversos fármacos que modifican directamente la estructura del ADN dentro de los que
ALQUILANTE destacan: los agentes alquilantes, compuestos de platino y las bleomicinas. AGENTES
ALQUILANTES. Los más reconocidos son la CICLOFOSFAMIDA, la clormetina, el melfalán, el
clorambucilo y la tiotepa. Son moléculas electrofílicas que son atacadas por sitios nucleófilos
del ADN. Dependiendo del agente, la alquilación puede tener lugar en los átomos de nitrógeno
o de oxígeno de la base, del esqueleto de fosfatos o de una proteína asociada al ADN. Repaso.
En pacientes con nefritis lúpica, se recomienda la administración combinada de bolos
mensuales de metilprednisolona (dosis 1gr/m2) + bolos de ciclofosfamida (dosis 1gr/m2 de
superficie corporal total) para lograr remisión renal y reducir la probabilidad de recaída. La
administración de ciclofosfamida se asocia con incremento de infecciones graves, herpes
zoster, falla ovárica y toxicidad hematológica (aplasia de médula ósea). ES POSIBLE QUE LA
CICLOFOSFAMIDA LESIONARÁ LAS CÉLULAS PRECURSORAS SANGUÍNEAS, YA QUE SE
CONSIDERA UN AGENTE ALQUILANTE.

UN COMPUESTO COMPUESTOS DE PLATINO. Los agentes más reconocidos de este grupo son el cisplatino,
DE PLATINO carboplatino y oxaliplatino. El cisplatino forma enlaces cruzados intercatenarios entre residuos
adyacentes de guanina, dando lugar a lesiones en el ADN.

UN INHIBIDOR INHIBIDORES DE TOPOISOMERASA. Dentro de este grupo se engloban las camptotecinas, las
DE LA antraciclinas, las epipodofilotoxinas y la amsacrina. Estos compuestos impiden la correcta
TOPOISOMERASA función de las topoisomerasas celulares obligándolas a participar en la destrucción del ADN.

UNA BLEOMICINAS. Las bleomicinas son una familia de glucopéptidos naturales sintetizados por
BLEOMICINA una especie de Streptomyces con actividad citotóxica importante. La bleomicina se une al ADN
y quela el hierro, dando lugar a la formación de radicales libres que producen roturas de una
sola cadena y de doble cadena en el ADN.

Bibliografía:
1. GOLDAN D, TASHJIAN A, ARMSTRONG E, AMSTRONG A. PRINCIPIO DE FARMACOLOGÍA, 3ª EDICIÓN.
WOLTERS KLUWER LWW. ESPAÑA. 2012. GUÍA DE PRÁCTICA CLÍNICA. DIAGNÓSTICO Y TRATAMIENTO DE
LA NEFROPATÍA LÚPICA EN PACIENTES MAYORES DE 18 AÑOS DE EDAD. MÉXICO: SECRETARÍA DE
SALUD.2009.

http://www.cenetec.salud.gob.mx/descargas/gpc/CatalogoMaestro/173_GPC_NEFROPATIA_LUPICA/IMSS_173_09_EyR_Nefropatia_lupica.pdf
ANÁLISIS DEL CASO CLÍNICO

IDENTIFICACIÓN DEL REACTIVO


Area: GINECOLOGÍA Y OBSTETRICIA
Especialidad: GINECOLOGÍA
Tema: LESIONES BENIGNAS Y MALIGNAS DE LA MAMA
Subtema: ENFERMEDAD FIBROQUÍSTICA

CASO CLÍNICO CON UNA PREGUNTA

MUJER DE 23 AÑOS DE EDAD, SOLTERA Y SIN VIDA SEXUAL ACTIVA. ACUDE A CONSULTA POR PRESENTAR
TUMORACIÓN MAMARIA IZQUIERDA EN EL CUADRANTE INFERIOR EXTERNO ÚNICA Y DOLOROSA DURANTE
LA MENSTRUACIÓN.

Ante un nódulo mamario, la edad no


determina la conducta a seguir.

-.

-.

MANEJO DEL NÓDULO MAMARIO. ANTE


LA APARICIÓN DE UN NÓDULO MAMARIO
DEBEMOS DESCARTAR. SIEMPRE LA
MALIGNIDAD, YA SE TRATE DE UNA
PACIENTE QUE PRESENTE O HAYA
PRESENTADO ANTERIORMENTE QUISTES,
FIBROADENOMAS U OTROS NÓDULOS
BENIGNOS. LA EXPLORACIÓN ES MUY
IMPORTANTE. SON SOSPECHOSOS DE
MALIGNIDAD LOS NÓDULOS MAL
DEFINIDOS, POCO MÓVILES, DE FORMA
ESTRELLADA, QUE INFILTREN O
RETRAIGAN LA PIEL O DEFORMEN LA
MAMA.

Se derivará al especialista: 1. nódulo de


reciente aparición o sin diagnosticar. 2.
signo de sospecha de malignidad tras la
exploración clínica. 3. cualquier signo de
sospecha de malignidad en las pruebas de
imagen. 4. Si es necesario realizar alguna
otra maniobra diagnóstica de un nódulo,
aunque sea para con rmar la benignidad
(PAAF o Biopsia).

293 - USTED SOLICITARÁ EL SIGUIENTE ESTUDIO PARA CONFIRMAR SU DIAGNÓSTICO:

ULTRASONIDO La ECOGRAFÍA es de gran utilidad para definir si una MASA QUÍSTICA O SÓLIDA, y forma parte
DE MAMA. de casi todos los algoritmos para el diagnóstico. Ciertas características de los tumores sólidos
como los bordes irregulares, los ecos internos o una relación entre espesor y altura menor de 1.7
cm sugieren cáncer. Es de gran utilidad para determinar datos sugestivos de mastopatía
fibroquística, sin embargo ante la presencia de una masa única palpable se sugiere biopsia. POR
LA EDAD PODRÍA SER QUE PENSARAS EN EL ULTRASONIDO COMO UNA BUENA OPCIÓN Y POR
SU PUESTO QUE ESTÁ INDICADO QUIZÁ INCLUSIVE ANTES DE LA BIOPSIA PARA UBICAR BIEN
LA LESIÓN PERO ESTE NO TE DARÁ LA "CONFIRMACIÓN DIAGNÓSTICA".

MASTOGRAFÍA. Los estudios de imagen de un TUMOR SOSPECHOSO comienzan con una mastografía, que
comprende amplificación, compresión o la toma de proyecciones adicionales a la oblicua lateral,
y craneal caudal que son las habituales. A diferencia de la mastografía de detección, la
mastografía diagnóstica se realiza en mujeres de cualquier edad. Los resultados de las imágenes
diagnósticas se deben resumir según la clasificación del Breast Imaging Reporting and Data
System (BI-RADS). Las lesiones que se clasifican como BI-RADS 5 son altamente sugestivas de
cáncer y mayor al 95% de los casos, al final se diagnóstica cáncer. Una clasificación baja reduce la
posibilidad de cáncer. Recuerda; que la mastografía de tamizaje está indicada en MUJERES
MAYORES DE 40 AÑOS debido a que la densidad del tejido mamario en mujeres jóvenes no
permite visualizar lesiones por éste método. RECUERDA EN NINGÚN CASO LA MASTOGRAFÍA
ESTÁ INDICADA ANTES DE LOS 25 AÑOS DE EDAD AUNQUE TUVIERA ANTECEDENTES DE
RIESGO.

BIOPSIA DE LA La combinación de la EXPLORACIÓN FÍSICA con los ESTUDIOS DE IMAGEN y la BIOPSIA CON
LESIÓN. AGUJA se denomina prueba triple. Cuando estos tres métodos sugieren una lesión benigna o un
cáncer mamario, se dice que la prueba triple es concordante. Una prueba triple benigna
concordante tiene una precisión del 99 % y los tumores mamarios que caen dentro de esta
categoría se pueden mantener en observación realizando exploraciones clínicas a intervalos de
seis meses. Cuando cualquiera de los tres estudios sugiere la posibilidad de cáncer, el tumor se
debe extirpar sin importar los resultados de las otras dos. Siempre es recomendable ofrecer la
ablación de un tumor mamario bien estudiado, incluso, después de obtener un resultado
concordante en la prueba triple, puesto que los tumores mamarios causan gran ansiedad. Toda
lesión mamaria "Única" palpable (mayor de 1cm) debe ser biopsiada para determinar su clase
histopatológica. OJO: LA PREGUNTA ES CLARA, "PARA CONFIRMAR EL DIAGNÓSTICO", LA
PRUEBA CONFIRMATORIA ES LA BIOPSIA.

NIVELES Como ya lo debiste de haber constatado, esta opción no tiene nada que ver en el estudio de la
SÉRICOS DE paciente con nódulo mamario. Es muy importante que adviertas que los tres estudios anteriores
PROLACTINA. son útiles y necesarios en el estudio del nódulo mamario, sin embargo, la pregunta menciona
diagnóstico definitivo que implica benignidad o malignidad, lo que sólo se puede lograr con el
estudio histopatológico de la lesión. EN PACIENTES CON GALACTORREA, CEALEA E
INFERTILIDAD PODRÍA SER DE UTILIDAD PARA DESCARTAR HIPERPROLACTINEMIA.

Bibliografía:
1. SCHONGUE J, SCHAFER J, HALVORSON L, HOFFMAN B, BRADSHAW K, CUNNINGHAM G. WILLIAMS
GINECOLOGÍA, DE LA 1A EDICIÓN EN INGLÉS. MC GRAW HILL. USA. 2009, PP 272-273.
ANÁLISIS DEL CASO CLÍNICO

IDENTIFICACIÓN DEL REACTIVO


Area: MEDICINA INTERNA
Especialidad: NEFROLOGÍA
Tema: INSUFICIENCIA RENAL AGUDA
Subtema: INSUFICIENCIA RENAL AGUDA POSTRENAL

CASO CLÍNICO SERIADO

HOMBRE DE 65 AÑOS DE EDAD, CON ANTECEDENTE DE HIPERTENSIÓN ARTERIAL. HACE 48 HRS INICIA CON
SINTOMATOLOGÍA URINARIA, DISMINUCIÓN DEL VOLUMEN URINARIO, HASTA LA ANURIA
APROXIMADAMENTE DESDE HACE 12 HRS. REFIERE INTENSOS DESEOS DE ORINAR, DOLOR EN HIPOGASTRIO,
QUE SE IRRADIA A AMBAS FOSAS RENALES. A LA EXPLORACIÓN SE ENCUENTRA TA 140/90 MMHG , F.C: 110 X´,
F.R: 34 X´, TEMP. 38.62º C. TACTO RECTAL: DOLOROSO CON MASA PALPABLE. LABORATORIO CON LEUCOCITOS
8,000, NEUTRÓFILOS 65 %, CREATININA 2.5, BUN 40, GLUCOSA 120 MG/DL. EGO: SIN MUESTRA.

HOMBRE DE 65 AÑOS.

HIPERTENSIÓN ARTERIAL.

48 HRS con sINTOMATOLOGÍA URINARIA.


DISMINUCIÓN DEL VOLUMEN URINARIO,
HASTA LA ANURIA APROXIMADAMENTE
DESDE HACE 12 HRS. REFIERE INTENSOS
DESEOS DE ORINAR, DOLOR EN
HIPOGASTRIO, QUE SE IRRADIA A AMBAS
FOSAS RENALES.

TACTO RECTAL: DOLOROSO CON MASA


PALPABLE. PROBABLEMENTE PRóSTATA.

"CREATININA 2.5", "BUN 40". DATOS DE


INSUFICIENCIA RENAL AGUDA.

294 - EL SIGUIENTE ES EL EVENTO FISIOPATOLÓGICO QUE EXPLICA EL DAÑO RENAL DEL PACIENTE:
INFLAMACIÓN La NEFRITIS TÚBULO INTERSTICIAL AGUDA (NTIA) se define como la inflamación aguda de
AGUDA DE LOS los túbulos e intersticio renales. La causa más frecuente de NTIA es farmacológica, con una
TÚBULOS E incidencia de hasta el 71% en algunas series, fundamentalmente la NTIA ligada a
INTERSTICIO antibióticos (hasta en un tercio de los casos), aunque también puede asociarse con
RENALES. trastornos autoinmunes, infecciosos o neoplásicos. Las manifestaciones clínicas suelen ser
inespecíficas, como náuseas y vómitos o malestar general. La tríada clásica consistente en
exantema, fiebre y eosinofilia se presenta sólo en el 10% de los casos. NO SE MENCIONA EL
ANTECEDENTE DE ADMINISTRACIÓN DE ANTIBIÓTICOS O PROCESOS INFECCIOSOS
PREVIOS.

AUMENTO DE LA DEBES TENER EN CUENTA EN PRIMER LUGAR QUE DE ACUERDO AL CASO CLÍNICO EXISTE
PRESIÓN UNA MASA (PROBABLEMENTE PRÓSTATA) QUE PUEDE ESTAR PROVOCANDO
INTRATUBULAR OBSTRUCCIÓN DIRECTA U OBSTRUCCIÓN POR EFECTO DE MASA. Las causas de FALLO
RENAL Y RENAL AGUDO (FRA) obstructivo difieren según la edad. En la infancia predominan las
VASOCONSTRICCIÓN. anomalías anatómicas, en la edad adulta, la litiasis y en pacientes mayores las causas son la
"hiperplasia prostática" y las neoplasias pelvianas: vejiga, próstata, útero y recto. El FRA
obstructivo es más prevalente en pacientes de edad avanzada. El origen de la obstrucción
puede ser intraluminal, intramural o extraluminal. Entre las causas intraluminales se
incluyen la obstrucción intratubular (hematuria, mioglobinuria, cristaluria asociada con
fármacos como sulfamidas y Aciclovir, etc.) y la obstrucción del tracto urinario por litiasis,
coágulos, necrosis papilar o agregados fúngicos. Las causas intramurales son el adenoma
prostático, las estenosis uretrales, las anomalías congénitas, tumores y la microvejiga
secundaria a tuberculosis o radioterapia. Las causas extraluminales se deben
fundamentalmente a neoplasias y fibrosis retroperitoneal. La clínica es variable y va a
depender del lugar de la obstrucción, de si es completa o parcial, del carácter agudo o
crónico de la obstrucción, etc. Es frecuente el dolor debido a la distensión de la vejiga,
sistema colector o cápsula renal, aunque suele ser mínimo o ausente en las obstrucciones
parciales o en aquellas que se establecen lentamente. El sitio de la obstrucción determinará
la localización del dolor. El volumen urinario es variable. Ante toda anuria brusca o bien si
existen períodos alternativos de anuria/poliuria se debe descartar, en primer lugar, un FRA
obstructivo. El flujo urinario normal o incluso elevado no excluye un cuadro obstructivo
como origen del FRA. LA ANURIA MÁS LA MASA PROBABLEMENTE PROSTÁTICA GUÍAN
NUESTRO DIAGNÓSTICO A UNA INSUFICIENCIA RENAL AGUDA POSTRRENAL POR
OBSTRUCCIÓN URETRAL.

ISQUEMIA E INFARTO EMBOLIA RENAL. Se trata de una entidad poco frecuente, pero es importante la realización
RENAL. de un diagnóstico precoz ya que se trata de una entidad con mal pronóstico (con una
mortalidad cercana al 30%), y es potencialmente reversible. Cursan con fallo renal agudo
(FRA) los casos de oclusión completa de las 2 arterias renales y de la arteria en caso de
riñón único. Clínicamente se presenta como FRA anúrico, y las concentraciones urinarias de
sodio, creatinina y urea se acercan a las concentraciones plasmáticas. En las situaciones de
oclusión parcial, puede presentarse deterioro funcional en relación con la zona renal
infartada. Pueden asociarse también dolor abdominal en los flancos, periumbilical o en la
fosa lumbar, fiebre, náuseas o vómitos. NORMALMENTE EXISTE EL ANTECEDENTE DE
TROMBOEMBOLISMO, LO QUE NO PRESENTA NUESTRO PACIENTE. LA PRESENCIA DE
MASA EN EL TACTO RECTAL NOS DESCARTA ESTE DIAGNÓSTICO.
EFECTO TÓXICO NECROSIS TUBULAR AGUDA NEFROTÓXICA. El riñón es un órgano muy vascularizado y
LOCAL. presenta una gran capacidad de concentración de sustancias tóxicas en el túbulo. Por ello
es especialmente vulnerable a la acción tóxica de numerosas sustancias, fármacos y
compuestos orgánicos. También es importante destacar la sinergia que existe entre la
hipoperfusión renal y la exposición a determinados agentes nefrotóxicos. Además de la
NTA nefrotóxica, los fármacos pueden inducir FRA por otros mecanismos. Los
aminoglicósidos son policationes, por lo tanto, presentan una pobre absorción digestiva,
una escasa unión a albúmina y una rápida excreción renal mediante filtración glomerular.
Producen nefrotoxicidad en el 10-20% de los casos por acumulación en la corteza renal. Se
presenta como una NTA no oligúrica entre 7-10 días tras el inicio del tratamiento. En
general tiene buen pronóstico, presentando lenta recuperación de función renal, aunque se
han descrito casos de nefritis túbulo-intersticial irreversible en pacientes con tratamientos
prolongados a altas dosis. Parece que tobramicina es menos nefrotóxica que gentamicina y
que estreptomicina es poco nefrotóxica, pero dado el importante riesgo de ototoxidad se
utiliza muy poco. De forma experimental, amikacina es la menos nefrotóxica. Para limitar
el riesgo de nefrotoxicidad se recomienda la administración de los fármacos en una dosis
única diaria, a excepción de tobramicina, con la que no se han evidenciado diferencias en
cuanto a su acumulación renal en dosis única respecto a infusión continuada. Las
cefalosporinas, la anfotericina B y polimixinas pueden condicionar NTA. La anfotericina B
característicamente produce nefrotoxicidad cuando se administra una dosis acumulada por
encima de los 2 g. Las formas liposomales de fármaco parece que pueden ser menos
nefrotóxicas. Los pigmentos orgánicos como hemoglobina y mioglobina pueden causar
FRA. El primer caso se produce en situaciones que condicionen anemia hemolítica como la
exposición a ciertos metales, transfusiones incompatibles o circuitos de circulación
extracorpórea. En estas situaciones no suele visualizarse orina sonrosada (ya que el pm de
la Hb es de 65.000). En el caso de la rabdomiólisis, tanto el plasma como la orina son
sonrosados y es característica la detección de «hemoglobina», en realidad mioglobina, en
la tira de orina sin visualización de hematíes en el sedimento. En ambas situaciones deben
instaurarse medidas encaminadas a la prevención del FRA como la alcalinización de la orina
y el aumento de volumen urinario (mediante expansión de volumen y diurético, si es
preciso). La lista de fármacos nefrotóxicos es larga y en ella se incluyen fármacos
inmunosupresores, como el grupo de los anticalcineurínicos; los solventes orgánicos que
clásicamente se han empleado en intentos autolíticos; metales, herbicidas o antivirales
como el foscarnet. NO EXISTEN ANTECEDENTES QUE SUSTENTEN ESTA SOSPECHA
DIAGNÓSTICA.

Bibliografía:
1. LONGO DL, FAUCI AS, KASPER DL, HAUSERSL, JAMESON JL, LOSCALZOJ. HARRISON. PRINCIPIOS DE
MEDICINA INTERNA, 18A EDICIÓN. MC GRAW HILL. NEW YORK, USA. 2012, PP 2299. 2. PAPADAKIS MAXINE
A, MCPHEE STEPHEN J. DIAGNÓSTICO CLÍNICO Y TRATAMIENTO. 52ª EDICIÓN. NUEVA YORK. 2013, PP 903.

295 - EL ESTUDIO DE ELECCIÓN PARA CORROBORAR EL DIAGNÓSTICO ES:

GAMAGRAMA Y EMBOLIA RENAL. En la analítica sanguínea suelen evidenciarse datos de citólisis: elevación
TOMOGRAFÍA de LDH, fosfatasa alcalina y GOT. Son útiles para el diagnóstico, técnicas como el eco-
RENAL. Doppler y el renograma isotópico, aunque el diagnóstico de certeza se realiza por TC de alta
resolución o arteriografía.

QUÍMICA En un paciente afectado de fallo renal agudo (FRA) es fundamental el análisis de orina. La
SANGUÍNEA Y existencia de proteinuria moderada-severa es más frecuente en el FRA parenquimatoso
EXAMEN GENERAL secundario a glomerulonefritis, vasculitis, enfermedades sistémicas y nefritis túbulo
DE ORINA. intersticial aguda (NTIA). El sedimento en la NTA puede mostrar cilindros granulosos y/o
hialinos de coloración parduzca debido a detritos celulares del epitelio tubular y la
presencia de proteína de Tamm-Horsfall. Un sedimento más activo con diversos cilindros,
hematuria y leucocitaria orienta hacia otras formas de FRA parenquimatoso. La eosinofiluria
es típica de la NTIA inducida por fármacos.
ESTUDIO DEL NEFRÍTIS TÚBULO INTERSTICIAL AGUDA. Habitualmente se llega al diagnóstico a través de
SEDIMENTO la historia de exposición a un determinado fármaco (lo que resulta muy complicado en
URINARIO. muchas ocasiones, ya que los pacientes suelen estar polimedicados), y a los hallazgos de
laboratorio característicos (datos de fallo renal agudo, eosinofilia, EFNa alrededor del 1% y
presencia en el sedimento urinario de leucocitos, hematíes, cilindros leucocitarios y en
algunas ocasiones eosinofiluria y manifestaciones similares a las del síndrome de Fanconi).
El diagnóstico únicamente puede confirmarse mediante "biopsia renal", que en ocasiones se
retrasa en espera de que la función renal pueda recuperarse tras la suspensión del fármaco,
aunque algunos estudios afirman que la instauración precoz de tratamiento con
corticoesteroides mejora el pronóstico y se evidencia una mayor recuperación funcional.

USG La ECOGRAFÍA ABDOMINOPÉLVICA es la primera prueba diagnóstica que debe realizarse


ABDOMINOPÉLVICO. para valorar la morfología y el tamaño renal y prostático, descartar signos que indiquen
dilatación de las vías urinarias y litiasis, y valorar la existencia de retenciones agudas de
orina o de importantes residuos postmiccionales. Posteriormente, se valora la realización de
una TC para tener una mejor definición de la etiología de la obstrucción o bien en aquellos
casos que por la clínica sean altamente sospechosos de FRA obstructivo, pero en los que
aún no se observa dilatación pielocalicial en la ecografía. DADO QUE SE CONSIDERA UNA
INSUFICIENCIA RENAL POSTRENAL LO QUE HAY QUE CONFIRMAR O DESCARTAR ES LA
CAUSA DE LA OBSTRUCCIÓN Y ÉSTE SE REALIZA MEDIANTE EL ULTRASONIDO.

Bibliografía:
1. PAPADAKIS MAXINE A, MCPHEE STEPHEN J. DIAGNÓSTICO CLÍNICO Y TRATAMIENTO. 52ª EDICIÓN.
NUEVA YORK. 2013, PP 903.

296 - EL TRATAMIENTO QUE DEBERÁ ESTABLECERSE DE INMEDIATO EN EL PACIENTE ES:

COLOCACIÓN Si la obstrucción transcurre a nivel uretral, causando hidronefrosis, la colocación de un catéter


DE CATÉTER endoluminal doble J puede resolver el cuadro. LA OBSTRUCCIÓN ES URETRAL, POR LO QUE NO
DOBLE J. DEBE INDICARSE ESTE PROCEDIMIENTO. Indicaciones del uso del catéter doble J en la litiasis
urinaria. Indicaciones terapéuticas: • Infección urinaria sobreañadida a la obstrucción. • Anuria
obstructiva. • Obstrucción con enfermedad renal crónica. • Cólico nefrítico refractario. •
Tratamiento quimiolítico. • Litiasis obstructiva en las embarazadas. • Iatrogenia en cirugía
endoscópica. Indicaciones profilácticas: • Profilaxis de la obstrucción uretral después de la
litotricia extracorpórea.

COLOCACIÓN El tratamiento inicial es bien conocido en el manejo de OBSTRUCCIÓN POR HIPERPLASIA


DE SONDA PROSTÁTICA. Debe colocarse una sonda uretral, a ser posible tipo Foley. Si se prevé que va a ser
TRANSURETRAL. necesaria durante poco tiempo, puede utilizarse una de látex (máximo 3-4 semanas). Las de
silicona presentan mayor resistencia a la infección y se toleran mejor; puede retrasarse el
recambio hasta 3 meses. Las sondas de menor calibre producen menos molestias. No debe
usarse una sonda Foley mayor de 14-16 Ch salvo que haya algún motivo (coágulos, etc.) que lo
haga recomendable. Si aparecen dificultades para el sondaje uretral, debe avisarse al urólogo.
Una lesión de uretra conlleva un importante riesgo de estenosis posterior, y si a través de la
misma se produce extravasación de orina hay riesgo de infección y sepsis. Es importante
recordar que tras la resolución de la obstrucción aparece una fase poliúrica en muchos casos,
que requiere de un control hemodinámico y analítico estricto. CORRESPONDE A LA MEDIDA
PRIMARIA INMEDIATA PARA MEJORAR EL ESTADO DEL PACIENTE.

CISTOSTOMÍA EN CASO DE QUE NO PUEDA REALIZARSE EL SONDAJE URETRAL, SE REALIZARÁ UN DRENAJE


SUPRAPÚBICA. POR PUNCIÓN SUPRAPÚBICA DE LA VEJIGA. Existe riesgo de interposición de asas intestinales
en quienes tienen antecedentes de cirugía abdominal compleja previa, y está desaconsejada en
los pacientes con neoplasia vesical, por el riesgo de siembra neoplásica en el trayecto del tubo.

NEFROSTOMÍA En caso de obstrucción uretral con hidronefrosis será necesaria la realización de nefrectomía
DE URGENCIA. percutánea. ESTA SERÍA YA LA ÚLTIMA OPCIÓN DE MANEJO QUIRÚRGICO.

Bibliografía:
1. LONGO DL, FAUCI AS, KASPER DL, HAUSERSL, JAMESON JL, LOSCALZOJ. HARRISON. PRINCIPIOS DE
MEDICINA INTERNA, 18A EDICIÓN. MC GRAW HILL. NEW YORK, USA. 2012, PP 2306-2307. 2. PAPADAKIS
MAXINE A, MCPHEE STEPHEN J. DIAGNÓSTICO CLÍNICO Y TRATAMIENTO. 52ª EDICIÓN. NUEVA YORK. 2013,
PP 903.

FIN DEL CASO CLÍNICO SERIADO


ANÁLISIS DEL CASO CLÍNICO

IDENTIFICACIÓN DEL REACTIVO


Area: MEDICINA INTERNA
Especialidad: PSIQUIATRÍA
Tema: TRANSTORNOS DE ANSIEDAD
Subtema: TRANSTORNO OBSESIVO-COMPULSIVO

CASO CLÍNICO SERIADO

FEMENINA DE 34 AÑOS DE EDAD DE PROFESIÓN PEDIATRA, SOLTERA, EXITOSA EN SU TRABAJO, ACUDE AL


SERVICIO DE URGENCIAS CON IDEAS DE MUERTE. DESDE HACE DOS MESES, LLEGA A SU MENTE LA IDEA DE
TENER RELACIONES SEXUALES CON SU HERMANO LO QUE LE HA HECHO CONFESARSE EN FORMA
REPETIDA. AL OBSERVAR QUE DICHA SINTOMATOLOGÍA NO CEDE, DECIDE DEJAR DE IR A SU CASA. YA
RECIBIÓ TERAPIA DE APOYO CON UN PSICÓLOGO Y NO MEJORÓ DESPUÉS DE UN MES DE TRATAMIENTO.

femenina de 34 años de edad.

pediatra, soltera, terapia psicológica sin


mejoría.

ideas de muerte desde hace 2 meses, ideas


intrusivas de tener relaciones sexuales con
hermano, compulsiones de confesarse.

-.

-.

297 - EL DIAGNÓSTICO CLÍNICO MÁS PROBABLE ES:

ESQUIZOFRENIA. La ESQUIZOFRENIA es un síndrome heterogéneo que se caracteriza por perturbaciones del


lenguaje, la percepción, el pensamiento, la relación social, el afecto y la voluntad. Suele
aparecer en los últimos años de la adolescencia de inicio incidioso y de mal pronóstico. Los
pacientes con esquizofrenia pueden presentar signos positivos (desorganización conceptual,
ideas delirantes o alucinaciones) o síntomas negativos (deterioro funcional, anhedonia,
expresión emocional reducida, alteraciones de la concentración y decremento de las relaciones
sociales) y deben tener al menos dos de éstos síntomas durante un periodo mínimo de un mes y
síntomas continuos de la enfermedad al menos durante seis meses para cumplir los criterios
diagnósticos formales. NO CUMPLE CON EL CUADRO CLÍNICO NI CON LOS TIEMPOS DE
EVOLUCIÓN PARA DETERMINAR ESTE DIAGNÓSTICO.
TRASTORNO EL TRASTORNO OBSESIVO COMPULSIVO (TOC) SE CLASIFICA EN EL DSM-IV COMO UN
OBSESIVO TRASTORNO DE ANSIEDAD, MANIFESTADO CON OBSESIONES, COMPULSIONES O AMBAS, QUE
COMPULSIVO. CAUSAN PERTURBACIÓN SIGNIFICATIVA EN EL ÁREA SOCIAL O PERSONAL. El Trastorno
Obsesivo Compulsivo suele iniciarse en los primeros años de la vida adulta., pero también
puede tener inicio en la infancia. La mitad de los pacientes tiene síntomas a los 25 años y las tres
cuartas partes a los 30. Se halló un inicio más precoz en los hombres. Las OBSESIONES se
definen como pensamientos, impulsos o imágenes recurrentes y persistentes que se
experimentan en algún momento del trastorno como intrusos e inapropiados, y causan
ansiedad o malestar significativos. - Los pensamientos, impulsos o imágenes no se reducen a
simples preocupaciones excesivas sobre problemas de la vida real. - La persona intenta ignorar
o suprimir estos pensamientos, impulsos o imágenes, o bien intenta neutralizarlos mediante
otros pensamientos o actos. - La persona reconoce que estos pensamientos, impulsos o
imágenes obsesivos son el producto de su mente las compulsiones se definen por
comportamientos o actos mentales de carácter repetitivo, que el individuo se ve obligado a
realizar en respuesta a una obsesión o con arreglo a ciertas reglas que debe seguir estrictamente
el inicio de los síntomas generalmente es gradual pero a veces es brusco. - Puede comenzar
como pensamientos intrusivos, como imágenes violentas que entran en la mente. - También
puede comenzar con rituales, comportamientos repetitivos. - Las obsesiones y compulsiones
pueden ser más de una y pueden cambiar en el transcurso del tiempo. - Los síntomas múltiples
son la norma. Los pacientes pueden resistirse a describir sus síntomas por vergüenza; los que
divulgan sus obsesiones reconocen que sus ideas o impulsos son irracionales. SI TE DAS CUENTA
SE DESCRIBE PERFECTAMENTE LA OBSESIÓN EN ESTA PACIENTE.

TRASTORNO El TRASTORNO POR ESTRÉS surge como una respuesta tardía y diferida a una situación o
POR ESTRÉS. acontecimiento estresante (de duración breve o prolongada) de naturaleza excepcionalmente
amenazante o catastrófica, que causaría por sí misma un malestar generalizado en casi cualquier
persona. NO HAY ANTECEDENTE DE EXPOSICIÓN A UNA SITUACIÓN TRAUMANTE TAL COMO
EXPOSICIÓN AMENAZA DE MUERTE, LESIÓN O PÉRDIDA FAMILIAR A PARTIR DEL CUAL SE
GENEREN LOS SÍNTOMAS.

DEPRESIÓN Para considerar DEPRESIÓN MAYOR debe tener al menos 5 de los siguientes aspectos: 1. Estado
MAYOR. de ánimo deprimido casi todo el día, la mayor parte de los días. (NO LO PRESENTA) 2. Pérdida
marcada del interés o del placer por todas o casi todas las cosas, casi todo el día o a mayor parte
de los días. (NO LO PRESENTA) 3. Pérdida de peso significativa en ausencia de dieta de
adelgazamiento o ganancia ponderal. (NO LO PRESENTA) 4. Insomnio o hipersomnia casi todos
los días. (NO LO PRESENTA) 5. Agitación o retardo psicomotor casi todos los días. (NO LO
PRESENTA) 6. Cansancio o pérdida de energía casi todos los días. (NO LO PRESENTA) 7.
Sensación de inutilidad o culpa excesiva o inadecuada casi todos los días. (PRESENTA CULPA) 8.
Disminución de la capacidad de pensar o concentrarse, o indecisión casi todos los días. (NO LO
PRESENTA) 9. Ideas de muerte recurrentes, ideas de suicida recurrente sin plan específico, o
intento de suicidio, o plan específico de suicidio. (IDEAS SUICIDAS). ÚNICAMENTE CUMPLE CON
LAS IDEAS SUICIDAD Y EL SENTIMIENTO DE CULPA, NO CUBRE CRITERIOS PARA ESTE
DIAGNÓSTICO.

Bibliografía:
1. LONGO DL, FAUCI AS, KASPER DL, HAUSERSL, JAMESON JL, LOSCALZOJ. HARRISON. PRINCIPIOS DE
MEDICINA INTERNA, 18A EDICIÓN. MC GRAW HILL. NEW YORK, USA. 2012, PP 3535. 2. PAPADAKIS MAXINE
A, MCPHEE STEPHEN J. DIAGNÓSTICO CLÍNICO Y TRATAMIENTO. 52ª EDICIÓN. NUEVA YORK. 2013, PP 1041.

298 - EL TRATAMIENTO INDICADO EN ESTA PACIENTE SERA CON:

BENZODIAZEPINAS. Las BENZODIAZEPINAS se utilizan a veces para el trastorno obsesivo-compulsivo, a pesar


de que generalmente se cree que es ineficaz para esta indicación, sin embargo, la eficacia
fue encontrada en un estudio pequeño. Las benzodiacepinas pueden ser consideradas
como una opción de tratamiento en los casos resistentes al tratamiento. En cuanto al
tratamiento farmacológico de elección en estos pacientes son los inhibidores selectivos de
la recaptación de serotonina (ISRS) como paroxetina, sertralina, fluoxetina, escitalopram y
la fluvoxamina. SE UTILIZAN SÓLO CUANDO EL TRATAMIENTO DE PRIMERA ELECCIÓN
NO HA SIDO EFICAZ.
TERAPIA La indicación más frecuente de la TERAPIA ELECTROCONVULSIVA (TEC) es en el trastorno
ELECTROCONVULSIVA. depresivo mayor, para el cual es el tratamiento más rápido, eficaz disponible. La terapia
electroconvulsiva (TEC) se ha intentado con frecuencia en pacientes con Trastorno
Obsesivo Compulsivo (TOC) grave los beneficios logrados han sido ocasionales.
Probablemente, los pacientes que respondieron a la TEC tenían depresión primaria con
síntomas secundarios de trastorno obsesivo compulsivo (TOC), o exacerbación del
trastorno obsesivo compulsivo (TOC) primario debido a la superposición de depresión
secundaria. LA TERAPIA ELECTROCONVULSIVA NUNCA DEBE CONSIDERARSE COMO
TRATAMIENTO DE PRIMERA LÍNEA, SE UTILIZA SÓLO EN PACIENTES GRAVES QUE NO
HAN RESPONDIDO AL MANEJO FARMACOLÓGICO Y PSICOTERAPÉUTICO.

INHIBIDORES LOS FÁRMACOS DE ELECCIÓN PARA EL TRATAMIENTO DEL TRASTORNO OBSESIVO


SELECTIVOS DE LA COMPULSIVO (TOC) SON LOS INHIBIDORES SELECTIVOS DE LA RECAPTURA DE
RECAPTACIÓN DE SEROTONINA (ISRS), que mostraron eficacia del 40% a 60%. Igualmente se ha usado la
SEROTONINA. clomipramina que es un antidepresivo tricíclico. Hasta el momento, se han aprobado para
esta enfermedad, la clomipramina, la fluvoxamina, la fluoxetina, la sertralina y la
paroxetina; también parecen eficaces el citalopram, la mirtazapina y la venlafaxina. De
acuerdo con la información disponible, la dosis de antidepresivos para tratar el TOC es
mayor a la utilizada para un episodio depresivo u otros trastornos de ansiedad. En
general, la respuesta a estos fármacos comienza a las 8 a 12 semanas, por lo que se debe
esperar mayor tiempo que en la depresión (máxima respuesta a las 4 a 6 semanas); dado
que la respuesta óptima se produce entre las semanas 10 y 12, se recomienda esperar 10
semanas antes de aumentar la dosis. Una vez alcanzada la dosis máxima del fármaco, se
recomienda su mantenimiento, al menos por 3 meses. RECUERDA: SIEMPRE INDICARLOS
A LA PAR DE TERAPIA CONDUCTUAL.

NEUROLÉPTICOS. En los casos resistentes al tratamiento se pueden agregar otros fármacos serotoninérgicos,
como la buspirona, o con un neuroléptico o una benzodiazepina. Incluso en algunos casos
muy graves se ha utilizado la estimulación cerebral profunda. LOS NEUROLÉPTICOS NO
SON EL MEDICAMENTO DE PRIMERA LÍNEA PARA EL TRASTORNO OBSESIVO
COMPULSIVO, PERO PUEDEN SER DE AYUDA EN COMBINACIÓN CON EL TRATAMIENTO
DE ELECCIÓN EN CASOS GRAVES.

Bibliografía:
1. LONGO DL, FAUCI AS, KASPER DL, HAUSERSL, JAMESON JL, LOSCALZOJ. HARRISON. PRINCIPIOS DE
MEDICINA INTERNA, 18A EDICIÓN. MC GRAW HILL. NEW YORK, USA. 2012, PP 3535. 2. PAPADAKIS MAXINE
A, MCPHEE STEPHEN J. DIAGNÓSTICO CLÍNICO Y TRATAMIENTO. 52ª EDICIÓN. NUEVA YORK. 2013, PP 1044.

FIN DEL CASO CLÍNICO SERIADO


ANÁLISIS DEL CASO CLÍNICO

IDENTIFICACIÓN DEL REACTIVO


Area: MEDICINA INTERNA
Especialidad: PSIQUIATRÍA
Tema: TRANSTORNO DE LA ALIMENTACIÓN Y ADICCIONES
Subtema: ADICCIONES Y DROGADICCIÓN

CASO CLÍNICO CON UNA PREGUNTA

HOMBRE DE 25 AÑOS DE EDAD, CON EL DIAGNÓSTICO DE DEPENDENCIA A COCAÍNA. AL LLEGAR A


URGENCIAS PRESENTA DATOS DE SOBREDOSIS AGUDA.

hombre de 25 años.

dependencia a cocaína.

-.

-.

-.

299 - EN ESTE CASO USTED INDICARÍA:

FLUMAZELINO El FLUMAZELINO es un antagonista específico de las BENZODIAZEPINAS. La NALOXONA es el


Y NALOXONA. medicamento de elección para el tratamiento de la sobredosis por OPIOIDES.

HALOPERIDOL En la intoxicación por FENCICLINIDA se ha administrado HALOPERIDOL para suprimir la conducta


Y psicótica. La OLANZAPINA es un antipsicótico se utiliza en algunos casos de intoxicación aguda
OLANZAPINA. por alcohol.

DIAZEPAM Y La COCAÍNA es un estimulante producto de la planta de la coca. La base libre constituye un


PROPANOLOL. derivado más puro y potente llamado crack. La TOXICIDAD POR COCAÍNA produce un estado
hiperadrenérgico caracterizado por hipertensión, taquicardia, convulsiones tónico, clónicas,
disnea y arritmias ventriculares. El DIAZEPAM intravenoso, resulta eficaz para las convulsiones y el
PROPANOLOL para el tratamiento de las arritmias ventriculares.

CLORURO El tratamiento de la sobredosis por METANFETAMINA en buena medida sintomático. El CLORURO


AMONICO Y AMÓNICO puede ser útil para acidificar la orina y potenciar la eliminación de la droga.
CLONAZEPAM.
Bibliografía:
1. LONGO DL, FAUCI AS, KASPER DL, HAUSERSL, JAMESON JL, LOSCALZOJ. HARRISON. PRINCIPIOS DE
MEDICINA INTERNA, 18A EDICIÓN. MC GRAW HILL. NEW YORK, USA. 2012, PP 3557.
ANÁLISIS DEL CASO CLÍNICO

IDENTIFICACIÓN DEL REACTIVO


Area: PEDIATRÍA
Especialidad: URGENCIAS PEDIÁTRICAS
Tema: PATOLOGÍA NEONATAL
Subtema: TAQUIPNEA TRANSITORIA DEL RECIEN NACIDO

CASO CLÍNICO CON UNA PREGUNTA

RECIEN NACIDO PREMATURO DE 30 SEMANAS DE GESTACION OBTENIDO POR CESAREA POR RUPTURA
PREMATURA DE MEMBRANAS. CON UN APGAR DE 7-8, CON UN SILVERMAN ANDERSEN DE 2, QUE SE HA IDO
INCREMENTANDO CONFORME PASA LOS MINUTOS.

recién nacido, prematuro de 30 semanas


de gestación.

embarazo pretérmino de 30 semanas,


obtenido por cesárea por ruptura
prematura de membranas, con un apgar
de 7-8, con un silverman de 2, con un
incremento progresivo.

-----

-----

-----

300 - EN EL SÍNDROME DE DIFICULTAD RESPIRATORIA:


DISMINUYE LA El SÍNDROME DE DIFICULTAD RESPIRATORIA (MEMBRANA HIALINA) es una enfermedad
DISTENSIBILIDAD caracterizada por inmadurez del desarrollo anatómico y fisiológico pulmonar del recién nacido
PULMONAR Y “prematuro”, cuyo principal componente el déficit transitorio de surfactante por disminución
AUMENTAN LAS de la síntesis, alteraciones cualitativas o aumento de su inactivación. La pérdida de la función
PRESIONES tensoactiva produce colapso alveolar, con pérdida de la capacidad residual funcional (CRF), que
COLAPSANTES dificulta la ventilación y altera la relación ventilación perfusión, por aparición de atelectasias.
ALVEOLARES. EL PULMÓN SE HACE MÁS RÍGIDO (CUESTA DISTENDERLO) Y TIENDE FÁCIL Y RÁPIDAMENTE
AL COLAPSO, AUMENTANDO EL TRABAJO Y EL ESFUERZO RESPIRATORIO. Este aumento del
esfuerzo no podrá mantenerse debido a la limitación de la fuerza muscular que afecta a la
función del diafragma y facilita que la pared torácica sea más débil y con tendencia a
deformarse, lo que dificulta la ventilación y el intercambio gaseoso. Se produce cianosis por
hipoxemia secundaria a las alteraciones de la ventilación-perfusión y se retiene CO2 por
hipoventilación alveolar. Todo ello produce acidosis mixta, que aumenta las resistencias
vasculares pulmonares y favorece la aparición de un cortocircuito derecha izquierda a nivel del
ductus y del foramen, aumentando la hipoxemia. En el pulmón aparecen micro-atelectasias
difusas, edema, congestión vascular y lesión del epitelio respiratorio, más evidente en los
bronquiolos terminales, con aspecto hepatizado y poco aireado. El edema alveolar, rico en
proteínas, inactiva el surfactante precisando elevadas presiones para la apertura de los alvéolos
colapsados, que son superiores a 25-30 cm de H2O para los alvéolos de menor radio.

LA TENSIÓN A medida que avanza la edad gestacional se sintetizan más fosfolípidos que se almacenan en
SUPERFICIAL SE las células alveolares tipo II. Estos agentes tensoactivos de superficie, son liberados hacia los
ENCUENTRA alveolos, donde “disminuyen la tensión superficial”, y ayudan a mantener la estabilidad
REDUCIDA Y alveolar al evitar el colapso de los pequeños espacios aéreos al final de la espiración. Debido a
EXISTE la inmadurez las cantidades producidas o liberadas de sustancias tensoactivas son insuficientes
ATELECTASIA para cubrir las demandas postnatales, de ahí que, EN LUGAR DE DISMINUIR LA TENSIÓN
ALVEOLAR SUPERFICIAL, ÉSTA “AUMENTA” EN PACIENTES PREMATUROS. La pérdida de la función
SUBSECUENTE. tensoactiva produce colapso alveolar, con pérdida de la capacidad residual funcional (CRF), que
dificulta la ventilación y altera la relación ventilación perfusión, por aparición de atelectasias.

AUMENTA LA En los pacientes con Enfermedad de Membrana Hialina se encuentra DISMINUIDA LA


RESISTENCIA Y ELASTICIDAD PULMONAR (DISTENSIBILIDAD), debido a la ausencia o disminución del líquido
DISMINUYE LA surfactante, lo que trae consigo todas las alteraciones a nivel pulmonar dando la dificultad
DISTENSIBILIDAD respiratoria en los recién nacidos. En los neonatos pretérmino, la gran distensibilidad de la
PULMONAR. pared torácica ofrece MENOS RESISTENCIA A LA TENDENCIA NATURAL DEL PULMÓN A
COLAPSARSE que la que ofrece la pared torácica de un recién nacido maduro. En los
prematuros, la limitación de la fuerza muscular afecta la función del diafragma y facilita que la
pared torácica sea más débil y con tendencia a deformarse, lo que contribuye a una mayor
dificultad para la ventilación y el intercambio gaseoso. SI LA DISTENSIBILIDAD ESTA
DISMINUIDA ES OBVIO QUE LA RESISTENCIA DEL PULMÓN TAMBIÉN, Y ELLO, SUMADO A LAS
CARACTERÍSTICAS DE LA PARED TORÁCICA DEL PREMATURO, CONTRIBUYE A QUE EL
PULMÓN SEA FACILMENTE COLAPSABLE.

DISMINUCIÓN En los pacientes con TAQUIPNEA TRANSITORIA DEL RECIÉN NACIDO (TTRN) la cantidad de
EN LA líquido que cubre el potencial espacio aéreo pulmonar durante la vida fetal es de 20-30 ml/kg,
DISTENSIBILIDAD es secretado por el pulmón a razón de 4-6 ml/kg/hr gracias a un gradiente electroquímico que
PULMONAR, es producido por una bomba de cloro; sus características iónicas son muy distintas a las del
ATRAPAMIENTO líquido amniótico o el plasma, el líquido pulmonar contiene grandes cantidades de cloro, poco
DE AIRE bicarbonato y un número casi nulo de proteínas; la producción de este líquido disminuye días
ALVEOLAR Y antes del nacimiento y el residuo restante debe ser absorbido vía linfática o sanguínea durante
AUMENTO EN LA los primeros días de vida para permitir que el intercambio gaseoso pulmonar se realice de
RESISTENCIA DE manera apropiada. El bajo número de proteínas en el líquido pulmonar juega un papel
LA VÍA ÁREA. importante en el favorecimiento de su absorción, ya que la mayor presión oncótica del
intersticio pulmonar atrae el líquido presente en el espacio aéreo. El retraso en la absorción de
líquido pulmonar va a producir el cuadro clínico presente en la TTRN, la acumulación del
líquido produce una DISMINUCIÓN EN LA DISTENSIBILIDAD PULMONAR, ATRAPAMIENTO DE
AIRE ALVEOLAR Y UN AUMENTO EN LA RESISTENCIA DE LA VÍA ÁREA, lo que produce la
dificultad respiratoria característica, además de aumentar el riesgo de edema pulmonar. Las
situaciones involucradas en la disminución de la absorción son: hipoproteinemia, hipertensión
pulmonar y disfunción ventricular ya que aumentan la presión venosa central disminuyendo la
absorción del líquido por el sistema linfático. A DIFERENCIA DE LA TAQUIPNEA TRANSITORIA
DEL RECIÉN NACIDO, DONDE HAY HIPERAIREACIÓN ALVEOLAR; EN EL SÍNDROME DE
DIFICULTAD RESPIRATORIA EL ALVEOLO SE ENCUENTRA COLAPSADO.

Bibliografía:
1. GUÍA DE PRÁCTICA CLÍNICA. DIAGNÓSTICO Y TRATAMIENTO DE SÍNDROME DE DIFICULTAD
RESPIRATORIA EN EL RECIÉN NACIDO. MÉXICO: SECRETARÍA DE SALUD. 2009. 2. KLIEGMAN R, STANTON B,
GEME J, SCR N, BEHRMAN R. NELSON TRATADO DE PEDIATRÍA, 19ª EDICIÓN. ELSEVIER. ESPAÑA 2013. 3.
FISIOLOGÍA. LINDA S. CONSTSANZO. MC.GROW HILL. EDICIÓN 2DA. 2000. PAG. 181.

http://www.cenetec.salud.gob.mx/descargas/gpc/CatalogoMaestro/137_GPC_SINDROME_DIF_RESP/IMSS_137_08_EyR_SINDROME_DIF_RESP.pdf
ANÁLISIS DEL CASO CLÍNICO

IDENTIFICACIÓN DEL REACTIVO


Area: CIRUGÍA
Especialidad: TRAUMATOLOGÍA Y ORTOPEDIA
Tema: INFECCIONES OSEAS Y ARTICULAES
Subtema: OSTEOMIELITIS Y ARTRITIS SÉPTICA

CASO CLÍNICO CON UNA PREGUNTA

FEMENINA DE 30 AÑOS, HACE 10 MESES SUFRIÓ ACCIDENTE AUTOMOVILÍSTICO, QUEDANDO PARAPLEJICA.


DEBIDO A LA INMOVILIDAD DESARROLLO ÚLCERA SACRA QUE NO HA CURADO. A LA EXPLORACIÓN CON
ÚLCERA SACRA DE 15 X 15 CM CON MATERIAL PURULENTO FÉTIDO Y TEJIDO NECRÓTICO.

FEMENINA DE 30 AÑOS.

paraplejia.

Exploración con Úlcera sacra de 15 x 15 cm


con material purulento fétido y tejido
necrótico.

301 - EL ESTUDIO DE PRIMERA LÍNEA NECESARIO PARA EVALUAR LA EXTENSIÓN DE LA INFECCIÓN ES:

BIOPSIA DEL La BIOPSIA DEL BORDE solo serviría para determinar los cambios histológicos presentes en la
BORDE. lesión, los cuales macroscópicamente son evidentes, necrosis e infección. En dado caso, más que la
biopsia estaría indicada la toma de CULTIVOS Y ANTIBIOGRAMA para determinar el agente causal
y así comenzar con la antibioticoterapia más adecuada. LAS BIOPSIAS SON DE UTILIDAD PARA
OBTENER LOS CULTIVOS Y EN AQUELLOS CASOS EN LOS QUE SE SOSPECHA MALIGNIDAD,
AUNQUE, PARA AMBOS CASOS NO NECESARIAMENTE SE REQUIERE UNA BIOPSIA DEL BORDE.
GAMAGRAFÍA. En este caso no sería necesaria, debido a que el foco de infección está claramente definido y la
extensión de la misma puede deducirse con ayuda de la radiografía simple. La gammagrafía
muestra aumento o disminución del recambio óseo, requiere de un marcador radiactivo que se
administra vía intravenosa. En el caso de haber una sospecha de infección deberá realizarse una
gamagrafía trifásica, es decir, con tomas seriadas a tiempo después de haber aplicado el marcador
radiactivo (al momento y 3 a 4hrs después); en el caso de sospechas neoplásicas únicamente se
tomará a las 3 o 4hrs. DE NINGUNA MANERA PUEDE SER CONSIDERADO UN ESTUDIO DE
PRIMERA LÍNEA, PUESTO QUE ES COSTOSA, POCO ACCESIBLE, IMPLICA EXPOSICIÓN A MATERIAL
RADIACTIVO Y ADEMÁS SE REQUIEREN DE TOMAS SERIADAS.

RADIOGRAFÍA El estudio más simple y de fácil acceso para determinar la extensión de la lesión y/o la infección es
DE COLUMNA. la RADIOGRAFÍA. Para este caso, aunque no se especifica el tiempo de evolución de la lesión, se
infiere que se trata de un proceso crónico, debido al tamaño de la lesión y la presencia de material
necrótico, de ahí que exista una alta sospecha de infección al hueso (osteomielitis) por extensión.
El sacro, como bien sabemos forma parte de la columna vertebral y por lo tanto el hueso por
contigüidad se vería más afectado, así como las vértebras de la columna lumbar. Como estudios
de primera línea se recomienda el uso de radiografías simples con técnica de partes blandas, en
proyección antero posterior y lateral del sitio afectado. A partir del séptimo día de iniciada la
lesión ósea (osteomielitis), las radiografías simples pueden mostrar elevación del periostio, lisis,
esclerosis y reducción de densidad ósea. DE LAS OPCIONES QUE AQUÍ SE MUESTRAN LAS
RADIOGRAFÍAS SIMPLES, SON DE PRIMERA ELECCIÓN PARA VALORAR LA LESIÓN ÓSEA.
IMPORTANTE: Actualmente se considera a la resonancia magnética con gadolinio el estudio de
elección para el diagnóstico de osteomielitis; este estudio puede mostrar cambios en el hueso que
no se observan con facilidad en las radiografías en etapas tempranas de la lesión (menos de 15
días). EL ESTUDIO DE PRIMERA ELECCIÓN PARA ESTE CASO SON LAS RADIOGRAFÍAS SIMPLES,
PUESTO QUE EL TAMAÑO Y CARACTERÍSTICAS DE LA LESIÓN SUGIEREN UN TIEMPO DE
EVOLUCIÓN LARGO.

TOMOGRAFÍA. Si existieran datos de afección a órganos contiguos (intrapélvicos) o incluso abdomen,


definitivamente estaría indicada la TAC puesto que ante la presencia de infección se pueden
desarrollar abscesos los cuales en algún momento sería necesario drenar. LA TOMOGRAFÍA PUEDE
AYUDARNOS A VALORAR LA EXTENSIÓN, PERO NO SE CONSIDERA DE PRIMERA LÍNEA, DE
HECHO, EN TODO CASO SE PREFERIRÍA LA RESONANCIA MAGNÉTICA.

Bibliografía:
1. PREVENCIÓN, DIAGNÓSTICO Y MANEJO DE LAS ÚLCERAS POR PRESIÓN EN EL ADULTO. EVIDENCIAS Y
RECOMENDACIONES: GUÍA DE PRÁCTICA CLÍNICA. MÉXICO: INSTITUTO MEXICANO DEL SEGURO SOCIAL; 2
DE DICIEMBRE DE 2015. 2. DOHERTY G. DIAGNÓSTICO Y TRATAMIENTO QUIRÚRGICO, 13A EDICIÓN. MC
GRAW HILL LANGE. 2011.

http://www.cenetec-difusion.com/CMGPC/IMSS-104-08/ER.pdf
ANÁLISIS DEL CASO CLÍNICO

IDENTIFICACIÓN DEL REACTIVO


Area: PEDIATRÍA
Especialidad: CRECIMIENTO Y DESARROLLO
Tema: ALTERACIONES DE LA NUTRICIÓN
Subtema: OBESIDAD

CASO CLÍNICO CON UNA PREGUNTA

ESCOLAR DE 9 AÑOS DE EDAD, CON PRESENCIA DE OBESIDAD DESDE HACE UN AÑO Y ACTUALMENTE BAJO
TRATAMIENTO DIETÉTICO.

escolar de 9 años de edad

obesidad desde hace 1 año, actualmente en


tratamiento dietético

--

--

--

302 - USTED EXPLICA LA IMPORTANCIA DE LA LEPTINA EN EL TRATAMIENTO DE LA OBESIDAD. LA


LEPTINA SE PRODUCE PRINCIPALMENTE EN EL:

TEJIDO LEPTINA: • Debido al hecho de que la leptina es producida en grandes proporciones en el tejido
ADIPOSO adiposo, sus niveles circulantes son proporcionales a las reservas adiposas del organismo. • Su
producción es dependiente del buen estado nutricional y de maduración de los adipocitos, y de la
intensidad y regulación del metabolismo del carbohidrato en estas células. • La leptina ha sido
considerada importante en el desarrollo de la obesidad por influir tanto en la ingesta como en el
gasto energético. LA LEPTINA ES PRODUCIDA PRINCIPALMENTE EN EL TEJIDO ADIPOSO.

INTESTINO FISIOLOGÍA DE LA LEPTINA: • La leptina promueve la reducción de la ingesta energética por medio de
DELGADO la señal de saciedad en el cerebro. • La hormona estimula el “lipostato hipotalámico” enviando una
señal de que existe tejido adiposo suficiente, provocando, por lo tanto, reducción en la ingesta de
alimentos y aumento en el gasto energético.

HÍGADO • La leptina promueve la reducción de la ingesta energética por medio de la señal de saciedad en el
cerebro. • Inyecciones de leptina inhiben la síntesis y la liberación del NPY (neuropéptido Y), quizá el
principal mediador de su efecto sobre el apetito.
PÁNCREAS • La leptina, por medio de señalización en nivel hipotalámico, también favorece la lipólisis en el tejido
adiposo conduciendo los nutrientes para el músculo, resultando en balance energético positivo y
reducción de la adiposidad.

Bibliografía:
1. LEHNINGER PRINCIPLES OF BIOCHEMISTRY. NELSON DL. WORTH PUBLISHERS. EDICIÓN 3A. 2000. PAG.
887-896.
ANÁLISIS DEL CASO CLÍNICO

IDENTIFICACIÓN DEL REACTIVO


Area: MEDICINA INTERNA
Especialidad: INFECTOLOGÍA
Tema: INFECCIONES FEBRILES SISTÉMICAS
Subtema: HIV Y SIDA

CASO CLÍNICO CON UNA PREGUNTA

MUJER DE 43 AÑOS DE EDAD, LA CUAL ACUDE A CONSULTA SOLICITANDO SE LE REALICEN LAS PRUEBAS
PERTINENTES PARA DESCARTAR VIH, YA QUE HACE 1 SEMANA SU ESPOSO CON QUIEN AÚN TIENE VIDA
SEXUAL, FUE DIAGNOSTICADO CON ESTE PADECIMIENTO.

43 años.

esposo diagnosticado con VIH hace 1


semana.

303 - ES LA PRUEBA QUE DE RESULTAR POSITIVA SERÍA CONFIRMATORIA DE INFECCIÓN POR VIH TRAS
HABER REALIZADO PRUEBAS DE ESCRUTINIO PRIMERO:

PRUEBA DE Las pruebas serológicas de detección de anticuerpos contra VIH son útiles para investigar el
ELISA diagnóstico de infección. Se cuenta con prueba de ELISA (enzimoinmunoanálisis de adsorción)
que es un ESTUDIO DE ESCRUTINIO. Esta prueba se utiliza de manera convencional para la
detección sistemática de VIH en todo aquel paciente con factores de riesgo para infección por
este virus. Tienen una sensibilidad del 99.5%. La prueba suele determinarse con resultados
positivos (reacción intensa), negativos (reacción nula) o indeterminados (reacción parcial). PARA
EVITAR FALSOS POSITIVOS EN UNA PRUEBA DE ELISA PARA VIH DEBERÁN REALIZARSE PRUEBAS
CONFIRMATORIAS CON INMUNOTRANSFERENCIA.
PRUEBA DE La prueba de WESTERN BLOT (WB) es un estudio confirmatorio para infección por VIH que se
WESTERN basa en el método de inmunofluoresencia. La prueba confirmatoria más utilizada es la de
BLOT inmunofluorescencia, es ideal para pacientes con ELISA positiva o indeterminada pero no se
recomienda como prueba sistemática. ES LA EL GOLD ESTANDAR PARA LA CONFIRMACIÓN DE
INFECCIÓN POR VIH. Su especificidad cuando se combina con un ELISA es del 99.9%. OJO: sus
resultados son indeterminados al inicio de la infección, infección por VIH-2, enfermedad
autoinmunitaria, embarazo y administración reciente de toxoide tetánico. NO DEBE REALIZARSE
COMO PRUEBA INICIAL EN NINGÚN CASO.

DETECCIÓN DE La prueba rápida de detección de anticuerpos para VIH produce resultados en 10 a 20 minutos,
ANTICUERPOS tiene la ventaja de ser fácil de realizar y barata. Toda prueba rápida positiva debe confirmarse
CONTRA VIH mediante pruebas serológicas de laboratorio. DE NINGUNA MANERA UNA PRUEBA RÁPIDA
PUEDE SER CONFIRMATORIA.

RECUENTO Los estudios que deben incluirse en la evaluación inicial de un paciente con infección por
LEUCOCITARIO VIH(SIDA para estadificar la enfermedad y apoyar la selección del esquema antiretroviral (ARV)
DE T CD4 son: - Anticuerpos para VIH - Cuenta de linfocitos TCD4 - Carga viral - Biometría hemática
completa, química sanguínea, niveles de transaminasas, nitrógeno uréico creatinina y examen
general de orina - Serología para hepatitis A, B y C - Glucosa y lípidos en ayuno EL RECUENTO
LEUCOCITARIO DE TCD4 ES DE UTILIDAD PARA LA EVALUACIÓN EN PACIENTES QUE YA SE
CONOCEN VIH POSITIVOS Y NO COMO PRUEBA DIAGNÓSTICA.

Bibliografía:
1. TRATAMIENTO ANTIRRETROVIRAL DEL PACIENTE ADULTO CON INFECCIÓN POR EL VIH. GUÍA DE
EVIDENCIAS Y RECOMENDACIONES: GUÍA DE PRÁCTICA CLÍNICA. MÉXICO. SECRETARÍA DE SALUD.
16/03/2017. 2. LONGO DL, FAUCI AS, KASPER DL, HAUSERSL, JAMESON JL, LOSCALZOJ. HARRISON.
PRINCIPIOS DE MEDICINA INTERNA, 18A EDICIÓN. MC GRAW HILL. NEW YORK, USA. 2012. 3. PAPADAKIS
MAXINE A, MCPHEE STEPHEN J. DIAGNÓSTICO CLÍNICO Y TRATAMIENTO. 52ª EDICIÓN. MC GRAW HILL
EDUCATION, LANGE. USA. 2013.

http://www.cenetec-difusion.com/CMGPC/IMSS-245-09/ER.pdf
ANÁLISIS DEL CASO CLÍNICO

IDENTIFICACIÓN DEL REACTIVO


Area: GINECOLOGÍA Y OBSTETRICIA
Especialidad: GINECOLOGÍA
Tema: LEUCORREA
Subtema: VAGINOSIS BACTERIANA

CASO CLÍNICO CON UNA PREGUNTA

MUJER DE 23 AÑOS DE EDAD, CON VIDA SEXUAL ACTIVA. ACUDE A SU CONSULTORIO REFERIDA DEL CENTRO
DE SALUD, CON DIAGNÓSTICO DE INFECCIÓN VAGINAL POR CHLAMYDIA PARA RECIBIR TRATAMIENTO.
ACTUALMENTE SE ENCUENTRA SIN NINGUNA SINTOMATOLOGÍA.

mujer de 23 años de edad.

vida sexual activa.

Asintomática, elemento muy característico


de esta infección.

-.

Diagnóstico de infección vaginal por


chlamydia.

304 - EN EL PLAN DE MANEJO DE LA PACIENTE, SERÁ IMPORTANTE CONSIDERAR LA SIGUIENTE


CARACTERÍSTICA EPIDEMIOLÓGICA DE LA CHLAMYDIA:
ES EL CAUSANTE DE MÁS DEL Sin un diagnóstico y/o un tratamiento adecuado la infección por CHLAMYDIA
70% DE SALPINGITIS AGUDAS. puede progresar hacia los órganos genitales internos por simple diseminación
vertical, afectando el endometrio, las trompas de Falopio y puede llegar a la
cavidad abdominal originando un cuadro clínico conocido como Enfermedad
Inflamatoria Pélvica. Si no es tratada, la infección puede avanzar y causar graves
problemas reproductivos y de salud con consecuencias a corto y largo plazo. Al
igual que la enfermedad, el daño que causa la Chlamydia es a menudo
"Silencioso". En las mujeres, si la infección no es tratada, puede propagarse al
útero o a las trompas de Falopio, y causar Enfermedad Inflamatoria Pélvica
(EIP). "Esto ocurre en alrededor del 10 al 15 % de las mujeres que tienen
Chlamydia, y no han recibido tratamiento." La Chlamydia también puede causar
infección en las trompas de Falopio, sin presentar ningún síntoma. La EIP y la
infección “Silenciosa” en la parte superior del aparato genital pueden causar
daño permanente en las trompas de Falopio, el útero y el tejido circundante. El
daño puede ocasionar dolor pélvico crónico, infertilidad o embarazo ectópico
que puede resultar mortal. La Chlamydia también puede aumentar la
probabilidad de adquirir la infección por el VIH, si hay exposición al virus. LA
INFECCIÓN POR CHLAMYDIA PUEDE PERSISTIR, DANDO ENTRE OTRAS
COMPLICACIONES SALPINGITIS (10-15%), O RESOLVERSE DE MANERA
ESPONTÁNEA. LA CONDICIÓN PARA EL PLAN DE MANEJO ES QUE: UNA VEZ
QUE SE DIAGNOSTICA INFECCIÓN POR CHLAMYDIA DEBE TRATARSE AÚN
ESTANDO LA PACIENTE ASINTOMÁTICA YA QUE LA ENFERMEDAD POR LO
GENERAL ES SILENTE.

FRECUENTEMENTE SE ASOCIA A Se ha sugerido que las mujeres que toman anticonceptivos orales son más
MÉTODOS ANTICONCEPTIVOS propensas al desarrollo de Cervicitis, debido en parte a la presencia de ectropión
DE BARRERA. cervical. EL USO DE ANTICONCEPTIVOS DE BARRERA OFRECE UNA ACCIÓN
PROTECTORA CONTRA LA ENFERMEDAD.

EN MUJERES CON VIDA SEXUAL Prueba de ELISA (Ensayo de inmuno-absorción ligado a enzimas) para
ACTIVA, LA PRUEBA DE Chlamydia Trachomatis, desarrollado en muestras de exudado, tiene como
MICROINMUNOFLUORESCENCIA ventaja una gran especificidad (90-97 %), y regular sensibilidad (60-80 %), se
ES PRECISA EN UN 80%. adecua al tamizaje de grandes poblaciones. Ensayos de hibridación de ADN, su
utilidad es casi similar a la prueba de ELISA y mantiene similares niveles de
especificidad y sensibilidad, su utilidad es mayor en pruebas de diagnóstico en
muestras uretrales y rectales. Ensayos de amplificación de material genético,
que utilizan las pruebas PCR (Reacción en Cadena de Polimerasa) o LCR
(Reacción en Cadena de Ligasa), para ampliar el material genético de la bacteria
obtenido a partir de muestras de orina y/o de secreciones. Es altamente
específico y sensible, detectando hasta el 90-95% de las infecciones, es
considerado el método de elección para el diagnóstico de la infección. Para
ayudar a prevenir las graves consecuencias de la infección por Chlamydia, se
recomienda que las mujeres sexualmente activas de 25 años de edad o menos,
se realicen una prueba de detección de la Chlamydia al menos una vez al año.
También se recomienda que las mujeres mayores de 25 años con factores de
riesgo de contraer Chlamydia (Por ejemplo, si tienen una nueva pareja sexual o
múltiples parejas sexuales) se realicen la prueba de detección anualmente.
Todas las mujeres embarazadas deben hacerse una prueba de detección de la
Chlamydia. Las complicaciones entre los hombres son poco comunes. En
ocasiones, la infección se propaga al epidídimo (El conducto que transporta el
semen desde los testículos) y causa dolor, fiebre y, rara vez, esterilidad. LA
PRUEBA DE INMINOFLUORESCENCIA NO ES ESPECÍFICA EN ESTE CASO.
LA MAYORÍA DE LAS PACIENTES La infección por Chlamydia Trachomatis se clasifica como de transmisión sexual.
CON ESTÁ INFECCIÓN SON La transmisión vertical puede ocurrir en el momento del parto y puede provocar
ASINTOMÁTICAS. oftalmía neonatal o neumonitis en el recién nacido. A la infección por
Chlamydia se le conoce como la enfermedad "Silenciosa" porque la mayoría de
las personas infectadas no presentan síntomas. Cuando se manifiestan, los
síntomas aparecen generalmente entre 1 y 3 semanas después del contagio. En
las mujeres, la bacteria infecta inicialmente el cuello uterino y la uretra (El
conducto urinario). Las mujeres con síntomas podrían presentar flujo vaginal
anormal o una sensación de ardor al orinar. Algunas mujeres siguen sin tener
signos ni síntomas aun cuando la infección se propague del cuello uterino a las
trompas de Falopio (Los conductos que transportan los óvulos desde los ovarios
hasta el útero); otras mujeres presentan dolor en la parte inferior del vientre,
dolor de espalda, náusea, fiebre, dolor durante las relaciones sexuales o
sangrado entre los períodos menstruales. La infección Chlamydial del cuello
uterino puede propagarse al recto. La amoxicilina es una alternativa efectiva,
para las mujeres con infección genital por Chlamydia Trachomatis durante el
embarazo, es efectiva la eritromicina. Sin embargo, la falta de datos apropiados
sobre la efectividad a mayor plazo de la amoxicilina en términos del riesgo de la
infección neonatal causa preocupación sobre el uso de rutina en la práctica
clínica. Desde el punto de vista del manejo de la paciente, es muy importante
que reconozcas que esta infección es frecuentemente asintomática, lo que no
contraindicará en ningún momento el comienzo del tratamiento. LA MAYOR
PARTE DE LOS PACIENTES INFECTADOS POR CHLAMYDIA NO MANIFIESTAN
SIGNOS NI SÍNTOMAS.

Bibliografía:
1. GUÍA DE PRÁCTICA CLÍNICA, PREVENCIÓN Y DIAGNÓSTICO OPORTUNO DE LA INFECCIÓN DEL TRACTO
GENITOURINARIO INFERIOR POR CHLAMYDIA TRACHOMATIS. MÉXICO: SECRETARIA DE SALUD; MARZO
2013.

http://www.cenetec.salud.gob.mx/descargas/gpc/CatalogoMaestro/006_GPC_Inf.porChlamydia/SSA_006_08_EyR.pdf
ANÁLISIS DEL CASO CLÍNICO

IDENTIFICACIÓN DEL REACTIVO


Area: GINECOLOGÍA Y OBSTETRICIA
Especialidad: GINECOLOGÍA
Tema: AFECCIONES BENIGNAS Y MALIGNAS DEL UTERO
Subtema: HIPERPLASIA ENDOMETRIAL Y CÁNCER DE ENDOMETRIO

CASO CLÍNICO CON UNA PREGUNTA

MUJER DE 29 AÑOS DE EDAD. RECIÉN CASADA, QUE ACUDE A SU CONSULTA SOLICITANDO UN MÉTODO DE
PLANIFICACIÓN FAMILIAR.

Mujer de 29 años. Al considerarse protector,


en este momento no es importante la
edad.

El antecedente familiar de cáncer


endometrial es importante siempre y
cuando se encuentre mutación en los
genes mlh1 y msh2.

El sangrado posmenopáusico orienta


mayormente al diagnóstico de cáncer
endometrial.

-.

Recuerda, el CA-125 es considerado el único


marcador tumoral de utilidad clínica en el
cáncer de endometrio. Es útil sobre todo
para vigilancia de la respuesta terapéutica.

305 - USTED LE RECOMIENDA, ANTICONCEPTIVOS HORMONALES COMBINADOS POR SU MECANISMO


DE ACCIÓN Y PORQUE DISMINUYEN EL RIESGO DE:
CÁNCER DE El tratamiento estrogénico sin oposición, es un factor estimulante potencial de cáncer de
ENDOMETRIO. endometrio. Por fortuna, el potencial maligno del estrógeno administrado en forma continua
o secuencia es reconocido desde hace más de 30 años. En la actualidad es raro encontrar una
mujer cuyo útero se haya sometido a estrógeno sin oposición por años. En lugar de eso se
prescribe tratamiento hormonal combinado con estrógenos más progestina para mujeres
posmenopáusicas que conservan el útero, a fin de disminuir el riesgo de cáncer endometrial. El
uso de anticonceptivos orales por un periodo de al menos un año, confiere un descenso en el
riesgo de cáncer endometrial de hasta 30 a 50%, y la reducción del riesgo se prolonga por 10 a
20 años. En esencia, la progestina de estos preparados tiene un efecto biológico protector en
el endometrio. LOS ANTICONCEPTIVOS ORALES EVITAN LA HIPERTROFIA ENDOMETRIAL CON
LO QUE DISMINUYEN EL RIESGO DE CÁNCER EN ESTE TEJIDO. La inclusión de progestágenos a
las terapias hormonarles disminuye hasta en 1.5 la posibilidad de desarrollar hiperplasia
endometrial y cáncer endometrial.

CÁNCER DE CÁNCER DE OVARIO Factores de riesgo: A mayor número de ciclos menstruales: menarca
OVARIO. precoz, menopausia tardía, nuliparidad, primer hijo tras los 30 años, exposición a tratamientos
estimulantes de la ovulación. Edad, antecedentes personales o familiares con cáncer de ovario
o mama. IMPORTANTE: En efecto, los esteroides sexuales femeninos se han relacionado con
cierto efecto protector para el cáncer de ovario y endometrio. Sin embargo de acuerdo a
diversos estudios no se aconseja el uso de anticonceptivos orales para la prevención del cáncer
de ovario en mujeres con factores de riesgo; además de que se deberá tener en cuenta que la
mayor parte de los casos de cáncer de ovario se diagnostican durante la gestación.

CÁNCER CÁNCER DE CÉRVIX Factores de riesgo: La infección por el virus del papiloma humano, es
CERVICOUTERINO. causa necesaria pero no suficiente. El contacto sexual también es causa necesaria.
Multiparidad: Actividad sexual precoz, Múltiples parejas sexuales, Pareja sexual de riesgo.
Nivel socioeconómico bajo. Infecciones asociadas (herpes simple, VIH, Chlamydia),
anticoncepción hormonal. TABAQUISMO. EL CÁNCER CERVICOUTERINO NO SE RELACIONA
CON FACTORES HORMONALES.

CÁNCER DE CÁNCER DE MAMA. Factores de riesgo: Mayor edad, clase social alta, nuliparidad, primer
MAMA. embarazo a mayor edad, menarca precoz, menopausia tardía, antecedentes personales o
familiares sobre todo de cáncer de mama, ovario o endometrio. Obesidad e
hipercolesterolemia. 10% BRCA1, BRCA2. El tratamiento hormonal estrógenos más
progestágenos (Sólo Estrógenos, Aumenta el de Endometrio, Pero No el de Mama).
Mastopatía fibroquística, consumo de alcohol, grasas saturadas, radiaciones, prolactina alta,
estatura alta. LA EXPOSICIÓN A TERAPIAS DE REEMPLAZO HORMONAL DE TIPO
ESTROGÉNICO AUMENTAN EL RIESGO DE CAMA.

Bibliografía:
1. GUÍA DE REFERENCIA RÁPIDA, DIAGNÓSTICO Y TRATAMIENTO DE LA HIPERPLASIA ENDOMETRIAL EN
MUJERES POSMENOPÁUSICAS EN EL SEGUNDO NIVEL DE ATENCIÓN. MÉXICO: SECRETARIA DE SALUD;
2012. 2. GUÍA DE PRÁCTICA CLÍNICA, DIAGNÓSTICO Y TRATAMIENTO DE LA HIPERPLASIA ENDOMETRIAL
EN MUJERES POSMENOPÁUSICAS EN EL SEGUNDO NIVEL DE ATENCIÓN. MÉXICO: SECRETARIA DE
SALUD; 2012. 3. SCHONGUE J, SCHAFER J, HALVORSON L, HOFFMAN B, BRADSHAW K, CUNNINGHAM G.
WILLIAMS GINECOLOGÍA, DE LA 1A EDICIÓN EN INGLÉS. MC GRAW HILL. USA. 2009, PP 701.

http://www.cenetec.salud.gob.mx/descargas/gpc/CatalogoMaestro/223_SSA_09_hiperplasia_endometrial/GPC_223-09_Hiperplasia_endometrialEVR.pdf
ANÁLISIS DEL CASO CLÍNICO

IDENTIFICACIÓN DEL REACTIVO


Area: MEDICINA INTERNA
Especialidad: CARDIOVASCULAR
Tema: ALTERACIONES DEL RITMO
Subtema: TAQUIARRITMIAS Y SINCOPE

CASO CLÍNICO SERIADO

MUJER DE 71 AÑOS, HIPERTENSA DESDE HACE 10 AÑOS. ACUDE A CONSULTA POR PRESENTAR MAREO Y
PALPITACIONES DE 12 HORAS DE EVOLUCIÓN. A LA EXPLORACIÓN SE ENCUENTRA TA 160/100 MMHG, FC
145/MIN, FR 18/MIN, PALIDEZ DE TEGUMENTOS Y RUIDOS CARDIACOS ARRÍTMICOS. SE SOLICITA ECG QUE
REPORTA: ONDA P AUSENTE Y RESPUESTA VENTRICULAR IRREGULAR.

MUJER DE 71 AÑOS.

HIPERTENSA DE 10 AÑOS DE EVOLUCIÓN.

PRESENTA MAREO Y PALPITACIONES DE


12 HORAS DE EVOLUCIÓN.

TA 160/100 MMHG, FC 180/MIN, FR 18/MIN,


PALIDEZ DE TEGUMENTOS Y RUIDOS
CARDÍACOS ARRÍTMICOS.

ECG CON ONDA P AUSENTE Y RESPUESTA


VENTRICULAR IRREGULAR.

306 - CORRESPONDE AL DIAGNÓSTICO CLÍNICO Y ELECTROCARDIOGRÁFICO DE ESTA PACIENTE:


FLUTTER AURICULAR. El FLUTER AURICULAR se caracteriza por una frecuencia auricular entre 250 y 350 lpm. Por
lo general, la frecuencia ventricular es la mitad de la auricular, es decir, alrededor de 150
lpm debido al bloqueo en el nudo AV. Las ondas clásicas del aleteo se manifiestan como
actividad auricular regular en dientes de sierra, más llamativa en las derivaciones inferiores.
LA CARACTERÍSTICA MÁS IMPORTANTE DEL FLUTTER AURICULAR ES LA IMAGEN EN
DIENTES DE SIERRA.

FIBRILACIÓN La FIBRILACIÓN AURICULAR se caracteriza en el electrocardiograma por la existencia de


AURICULAR. oscilaciones irregulares de la línea de base, las que traducen depolarizaciones auriculares
múltiples y desorganizadas (entre 400 y 700 por min) Otra característica fundamental es la
presencia de un respuesta ventricular totalmente irregular. La frecuencia auricular aumenta
enormemente (400-700 lpm), afortunadamente el periodo refractario no permite la
conducción de todos los impulsos, pero de cualquier forma el latido ventricular es elevado
(170-200 lpm), lo que acorta la diástole y por eso agrava los efectos hemodinámicos de la
obstrucción mitral favoreciendo la hipertensión venocapilar y el edema pulmonar. LA
FIBRILACIÓN AURICULAR SE CARACTERIZA POR UNA ACTIVIDAD AURICULAR
DESORDENADA SIN ONDAS P BIEN DEFINIDAS EN EL ECG.

TAQUICARDIA El término TAQUICARDIA SUPRAVENTRICULAR (TSV) incluye una variedad de ritmos que
SUPRAVENTRICULAR. tienen en común un origen por encima de la bifurcación del haz de His. Cobija, por lo
tanto, a las taquicardias originadas en la aurícula, en la unión AV (nodo AV, haz de His) y a
las que incorporan una vía accesoria aurículoventricular. Usualmente, las TSV presentan
complejos QRS angostos con una duración del complejo QRS < 120 ms. LA TAQUICARDIA
SUPRAVENTRICULAR INCLUYE TODAS LAS PATOLOGÍAS QUE SE ORIGINAN EN LA
AURICULA Y LA UNIÓN AV, POR LO QUE EL TÉRMINO ES MUY GENERAL Y SE DESCARTA
DE SER LA RESPUESTA CORRECTA.

TAQUICARDIA La TAQUICARDIA VENTRICULAR se define como tres o más extrasístoles ventriculares a una
VENTRICULAR. frecuencia de más de 120 lpm. La anchura del intervalo QRS en estos casos es > 120 mseg.
NO SE REFIEREN EXTRASÍSTOLES EN EL TRAZO ECG DE LA PACIENTE POR LO QUE NO
CORRESPONDE A ÉSTE DIAGNÓSTICO.

Bibliografía:
1. GUÍA DE PRÁCTICA CLÍNICA, DIAGNÓSTICO Y TRATAMIENTO DE LA FIBRILACIÓN AURICULAR. MÉXICO:
SECRETARIA DE SALUD, FEBRERO 2011.

http://www.cenetec.salud.gob.mx/descargas/gpc/CatalogoMaestro/014_GPC_FibrilacionAuricular/SS_014_08_EyR.pdf

307 - DURANTE LA REVALORACION DE LA PACIENTE, PRESENTA PÉRDIDA DEL ESTADO DE ALERTA. ES


EL TRATAMIENTO DE ELECCIÓN EN ESTE CASO:

AMIODARONA. AMIODARONA: produce incremento en la duración del potencial de acción (incrementa el


espacio QT) y del periodo refractario, tanto del miocardio auricular como ventricular, y lo es
también en los haces anómalos; por lo que suprime los mecanismos de reentrada. Indicaciones:
- Extrasístoles auriculares y ventriculares. - Prevención de arritmias en los síndromes de
preexitación. - Previene la muerte súbita en pacientes que presentan extrasístoles ventriculares
frecuentes, precoces, en salvas o carreras de taquicardia ventricular no sostenida y quienes han
presentado fibrilación ventricular. SU PRINCIPAL INDICACIÓN SON EXTRASÍSTOLES Y
FIBRILACIÓN VENTRICULAR.

VERAPAMIL. Produce disminución de la frecuencia ventricular durante la fibrilación auricular y el aleteo y


otras SVT en ausencia; VT idiopática. EL VERAPAMILO SE UTILIZA COMO MANEJO INICIAL
SIEMPRE Y CUANDO NO HAYA DATOS DE COMPROMISO HEMODINÁMICO, POR ÉSTA RAZÓN
NO SE ELIGE COMO RESPUESTA CORRECTA.
DESFIBRILACIÓN. Desfibrilación, esto es la terminación de la fibrilación ventricular mediante un estimulo
eléctrico de alta energía y asincrono. Cuando la desfibrilación es exitosa el corazón toma su
marcapaso normal (nodo SA) para terminar con los marcapasos ectópicos que están causando
FV. Indicación: - Taquicardia ventricular - Taquicardia ventricular sin pulso - Taquicardia
ventricular sostenida con pulso, si el paciente está inconsciente - La FV fina la cual asemeja a la
asistolia en el monitor. SU INDICACIÓN PRINCIPAL ES FIBRILACIÓN VENTRICULAR, NO
CORRESPONDE AL DIAGNÓSTICO CLÍNICO DE LA PACIENTE.

CARDIOVERSIÓN. Cardioversión la terminación de una taquiaríitmia por medio de un estimulo sincronizado con
los complejos QRS. La Cardioversión Eléctrica consiste en la administración de un choque
eléctrico sincronizado con la onda R del ECG con un desfibrilador. La Cardioversión Eléctrica
está indicada en la reversión a ritmo sinusal de una arritmia cardíaca no autolimitada cuando: -
falla el tratamiento farmacológico - de forma urgente cuando la arritmia se acompaña de
compromiso hemodinámico. Las arrítmias cardíacas que se pueden tratar mediante
Cardioversión Eléctrica son: - arritmias supraventriculares: taquicardia auricular paroxística,
fluutter auricular, fibrilación auricular, taquicardia ventricular con pulso. DADO QUE LA
PACIENTE MUESTRA DATOS DE COMPROMISO HEMODINÁMICO ESTÁ INDICADA LA
CARDIOVERSIÓN.

Bibliografía:
1. GUÍA DE PRÁCTICA CLÍNICA, DIAGNÓSTICO Y TRATAMIENTO DE LA FIBRILACIÓN AURICULAR. MÉXICO:
SECRETARIA DE SALUD, FEBRERO 2011.

http://www.cenetec.salud.gob.mx/descargas/gpc/CatalogoMaestro/014_GPC_FibrilacionAuricular/SS_014_08_EyR.pdf

FIN DEL CASO CLÍNICO SERIADO


ANÁLISIS DEL CASO CLÍNICO

IDENTIFICACIÓN DEL REACTIVO


Area: CIRUGÍA
Especialidad: CIRUGÍA ABDOMINAL
Tema: SANGRADO DE TUBO DISGESTIVO ALTO Y BAJO
Subtema: HEMORRAGIA DIGESTIVA INFERIOR

CASO CLÍNICO CON UNA PREGUNTA

HOMBRE DE 29 AÑOS SIN ANTECEDENTES DE IMPORTANCIA, SE ENCUENTRA EN PROTOCOLO POR


DIAGNÓSTICO DE ANEMIA FERROPENIA SIN ENCONTRAR LA CAUSA APARENTE, SE SOSPECHA SANGRADO DE
TUBO DIGESTIVO BAJO.

masculino de 29 años de edad.

ANEMIA FERROPÉNICA.

-.

SE SOSPECHA SANGRADO DE TUBO


DIGESTIVO BAJO.

-.

308 - ANTE LA SOSPECHA DIAGNÓSTICA Y POR ORDEN DE FRECUENCIA USTED ESTA OBLIGADO A
BUSCAR:
ÚLCERAS O El SANGRADO DE TUBO DIGESTIVO BAJO se define como aquel sangrado originado por
VÁRICES lesiones en el tubo digestivo en forma distal al ligamento de Treitz, es decir, entre el intestino
RECTALES. delgado y el ano. Resultado del sangrado de tubo digestivo bajo se puede encontrar anemia o
inestabilidad hemodinámica de acuerdo a la pérdida de sangre. Generalmente está ocasionado
por malformaciones vasculares, diverticulos y neoplasias. Las ENFERMEDADES PERIANALES que
suelen ocasionar sangrado corresponden a hemorroides y fístulas perianales principalmente. 1.
HEMORROIDES: son plexos vasculares anteriovenosos y se encuentran a través de todo el canal
anal. Sólo ocasionan síntomas cuando se produce dilatación o ingurgitación. Su prevalencia va
del 4 al 86%. Se clasifican en internas y externas. Las externas, se sitúan por debajo de la línea
dentada y están cubiertas por epitelio escamoso. Generalmente se presentan como uno o varios
nódulos que suelen ser asintomáticos, aunque pueden causar prurito, que puede ser intenso si
se trombosan. En ciertos casos de hemorroides externas trombosadas el coágulo de sangre
puede eliminarse de forma espontánea a través de una ULCERACIÓN CUTÁNEA, pudiendo
provocar sangrado. Las alteraciones hemorroidales internas, por su parte, son la rectorragia y el
prolapso. En ocasiones, a consecuencia de irritación de la mucosa prolapsada, se acompañan de
prurito y/o mucosidad asociada. Raramente se manifiestan como trombosis. LAS ÚLCERAS Y
VÁRICES RECTALES SON PATOLOGÍAS QUE PUEDEN ACOMPAÑARSE DE HEMORRAGIA PERO
NO FORMAN PARTE DE LAS PRIMERAS CAUSAS DE SANGRADO DE TUBO DIGESTIVO BAJO.

ENFERMEDAD La COLITIS NO INFECCIOSA está ocasionada por: enfermedad de Crohn, colitis ulcerativa,
DE CROHN. isquémia y por radiación. Esta es una causa de sangrado de tubo digestivo, pero no la más
frecuente. Las más comunes son la colitis ulterativa (2-4%) y la enfermedad de Crohn (6%) del
total de casos de hematoquesia. La colitis ulcerativa afecta el recto sigmoides en
aproximadamente 80% de los casos, ocasionando pancolitis en 10% de los que la padecen.

DIVERTICULOSIS. La DIVERTICULOSIS es la presencia de múltiples divertículos falsos en el intestino. En cerca del


80% de los casos se presenta asintomática y cuando llega a dar datos clínicos la generalidad es
que sea como consecuencia de complicaciones propias tales como sangrado o diverticulitis. Es
la causa más frecuente de sangrado de tubo digestivo bajo (STDB) que se asocia del 20 al 55%
de los caso, aunque sólo el 3 a 15% de los casos de diverticulosis presenta sangrado. La forma
más común de este sangrado es la hematoquezia. Es ocasionada por la erosión de los vasos en
el cuello del divertículo. El 75% de los divertículos están localizados en el colon izquierdo, pero
la mayor parte de las veces (50-90%) la hemorragia procede de divertículos en el colon derecho.
Existen dos factores principales en la génesis de los divertículos: - Contracciones peristálticas
excesivas con elevación anormal de la presión intraluminal. - Defectos focales en la pared de la
mucosa normal del colon. Los pacientes con sospecha de sangrado de tubo digestivo bajo
deben ser sometidos a estudios de colonoscopía para descartar el diagnóstico. LA
DIVERTICULOSIS ES LA PRIMERA CAUSA DE SANGRADO DE TUBO DIGESTIVO BAJO.

COLITIS La COLITIS INFECCIOSA se ocasiona principalmente por: amibiasis, disentería, tuberculosis y


INFECCIOSA. otras menos frecuentes (campylobacter, E. coli, Clostridium difficile). Clínicamente se asocia a
dolor abdominal, tipo cólico y diarrea de características sanguinolentas. SI BIEN CONDICIONA
EN ALGUNAS OCASIONES HEMORRAGIA EN EL TUBO DIGESTIVO BAJO, NO ES LA CAUSA MÁS
FRECUENTE DE SANGRADO A ESTE NIVEL.

Bibliografía:
1. GUÍA DE PRÁCTICA CLÍNICA, ATENCIÓN INTEGRAL EN PACIENTES CON SANGRADO DE TUBO DIGESTIVO
BAJO EN LOS TRES NIVELES DE ATENCIÓN. MÉXICO: SECRETARIA DE SALUD; 2010. RECUPERADO DE
HTTP://WWW.CENETEC.SALUD.GOB.MX/CONTENIDOS/GPC/CATALOGOMAESTROGPC.HTML

http://www.cenetec.salud.gob.mx/descargas/gpc/CatalogoMaestro/306_GPC_STDBajo/ISSSTE-306-10-EYR_SANGRADO_DE_TUBO_DIGESTIVO_BAJO.pdf
ANÁLISIS DEL CASO CLÍNICO

IDENTIFICACIÓN DEL REACTIVO


Area: GINECOLOGÍA Y OBSTETRICIA
Especialidad: GINECOLOGÍA
Tema: DOLOR PELVICO
Subtema: ENDOMETRIOSIS

CASO CLÍNICO CON UNA PREGUNTA

MUJER DE 27 AÑOS DE EDAD, CON TRATAMIENTO ACTUAL DE ENDOMETRIOSIS. LA PACIENTE ACTUALMENTE


RECIBE TRATAMIENTO CON MEDICAMENTO QUE INHIBE LA OVULACIÓN, A TRAVÉS DE LA INHIBICIÓN DE LA
FSH Y LH.

Mujer de 27 años de edad.

En tratamiento actual de endometriosis


con inhibidores de ovulación.

-.

-.

-.

309 - EN ÉSTE CASO LA PACIENTE DEBE ESTAR RECIBIENDO EL SIGUIENTE TRATAMIENTO CON:

BROMOCRIPTINA. La BROMOCRIPTINA es un derivado de la Ergolina clasificado dentro de los agonistas D2,


Dopaminérgicos que se usa para el tratamiento de trastornos hipofisarios y la Enfermedad de
Parkinson. Uno de los efectos Dopaminérgicos sobre la hipófisis, es el antagonismo de la
producción de prolactina por los lactotrofos. SE INDICA PARA HIPERPROLACTINEMIA, NO
TIENE RELACIÓN CON EL CASO NI CON EL MECANISMO DE ACCIÓN.

TIBOLONA. La TIBOLONA es un esteroide sintético que se comporta como una pro droga con
características farmacológicas singulares pues, dependiendo de las enzimas específicas
presentes en los distintos tejidos, se transforma en tres metabólicos activos, cada uno con un
efecto hormonal diferente; por lo mismo, sus efectos hormonales varían de un tejido a otro,
ejerciendo en la mayoría de ellos actividad estrogénica, en otros progestágena, y androgénica
en algunos. Tratamiento de los síntomas de la menopausia natural o artificial. Prevención de
osteoporosis en estados de deficiencia estrogénica SE INDICA PARA LOS SÍNTOMAS DEL
CLIMATERIO, NO TIENE RELACIÓN CON EL CASO NI CON EL MECANISMO DE ACCIÓN.
AMPICILINA. La AMPICILINA es una penicilina del grupo de los beta-lactámicos con efecto bactericida.
Actúa inhibiendo la última etapa de la síntesis de la pared celular bacteriana uniéndose a unas
proteínas específicas llamadas PBPs (Penicillin-Binding Proteins) localizadas en la pared
celular. Al impedir que la pared celular se construya correctamente, la ampicilina ocasiona, en
último término, la lisis de la bacteria y su muerte. ES UN ANTIBIÓTICO NO TIENE RELACIÓN
CON EL CASO CLÍNICO NI CON EL MECANISMO DE ACCIÓN.

DANAZOL. El DANAZOL es un andrógeno débil, pero también se han observado acciones


antiandrogénicas, progestogénicas, antiprogestogénicas, estrogénicas y antiestrogénicas.
Interfiere con las síntesis de esteroides gonadales, posiblemente por inhibición de la enzima
separadora de la cadena lateral de colesterol y de otras enzimas de la esteroidogénesis. Puede
también inhibir la acumulación del AMP cíclico en las células de las capas granulosa y lútea, en
respuesta a las hormonas gonadotrópicas. Se ha observado inhibición en la mitad del ciclo
donde se genera la producción de hormona foliculostimulante (FSH), hormona luteinizante
(LH) y una reducción en la secreción de LH. En el periodo postmenopausia, el Danazol
disminuye los niveles medios plasmáticos de estas gonadotropinas. Dentro del amplio rango
de acción del Danazol sobre las proteínas plasmáticas, se encuentran: Incremento de
protrombina, plasminógeno, antitrombina III, alfa 2, macroglobulina, inhibidor de esterasa C1,
eritropoyetina, así como disminución de fibrinógeno, globulinas unidas a tiroides y unidas a
hormonas sexuales. El DANAZOL incrementa la proporción y concentración de testosterona
libre (No unida a proteínas) en plasma. Los efectos supresores del DANAZOL sobre el eje
hipotálamo-hipófisis gonadal son reversibles. La actividad cíclica se restablece normalmente
dentro de los 60 a 90 días siguientes a la suspensión del tratamiento. En el tratamiento de la
endometriosis. DANAZOL transforma el endometrio en inactivo y atrófico causando también,
regresión del tejido endometrial ectópico. Como se ha comprobado por la endoscopia, se
produce desaparición completa de las lesiones endometriales en la mayoría de los casos. EL
DANAZOL ESTÁ INDICADO PARA EL MANEJO DEL DOLOR EN LA ENDOMETRIOSIS; SU
PRINCIPAL ACCIÓN ES LA SUPRESIÓN DEL PICO DE HORMONA LUTEINIZANTE EN LA MITAD
DEL CICLO, CREANDO UN ESTADO ANOVULATORIO CRÓNICO.

Bibliografía:
1. SCHONGUE J, SCHAFER J, HALVORSON L, HOFFMAN B, BRADSHAW K, CUNNINGHAM G. WILLIAMS
GINECOLOGÍA, DE LA 1A EDICIÓN EN INGLÉS. MC GRAW HILL. USA. 2009, PP 237.
ANÁLISIS DEL CASO CLÍNICO

IDENTIFICACIÓN DEL REACTIVO


Area: PEDIATRÍA
Especialidad: URGENCIAS PEDIÁTRICAS
Tema: URGENCIAS ORTOPÉDICAS
Subtema: FRACTURAS PATOLÓGICAS Y TUMORES OSEOS

CASO CLÍNICO CON UNA PREGUNTA

PACIENTE MASCULINO DE 12 AÑOS DE EDAD, AL CUAL LE DIAGNOSTICARON OSTEOSARCOMA EN FEMUR


DERECHO HACE 8 DIAS. YA CUENTA CON EL PROTOCOLO DE ESTUDIO COMPLETO.

MUY IMPORTANTE, ADOLESCENTE.

DIAGNÓSTICO DE OSTEOSARCOMA DE
FEMUR (MIEMBRO INFERIOR).

310 - EL TRATAMIENTO QUE DEBE DE INICIAR EL PACIENTE ES CON:

QUIMIOTERAPIA SARCOMA OSTEOGÉNICO U OSTEOSARCOMA. El sarcoma osteogénico es el tumor óseo más


frecuente. Puede presentarse a cualquier edad, pero más del 75% de los casos se presentan
entre los 12 y los 25 años. Existe un claro predominio masculino en su incidencia. La máxima
frecuencia de presentación en la segunda década de la vida y coincidiendo con el estirón
puberal, sugiere una relación entre el crecimiento óseo acelerado y el desarrollo. El
osteosarcoma aparece preferentemente en la zona metafisaria de los huesos de más rápido
crecimiento en la adolescencia (fémur distal, tibia proximal, húmero proximal).

RADIOTERAPIA Etiología y bases genéticas Las radiaciones ionizantes son un agente causal claramente
Y establecido y son responsables de un 3% de estas neoplasias. El intervalo entre la irradiación y la
QUIMIOTERAPIA aparición del tumor oscila entre 4 y 40 años. También, la administración de quimioterapia con
agentes alquilantes se ha asociado a la aparición de sarcomas osteogénicos. En pacientes de más
de 40 años, la enfermedad de Paget se asocia al sarcoma osteogénico en el 2% de los casos. La
predisposición genética más clara a la presentación de este tumor se da entre los pacientes con
retinoblastoma. Los datos disponibles apoyan que el locus específico del retinoblastoma,
situado en el cromosoma 13, está también implicado en el osteosarcoma.
CIRUGIA Y Clínica y datos de laboratorio A menudo, un traumatismo sobre la región afecta origina el
RADIOTERAPIA descubrimiento de la lesión. Con frecuencia transcurren períodos largos, a veces de varios
meses, hasta llegarse a un diagnóstico correcto. Más del 10% de los pacientes presentan
metástasis en el momento del diagnóstico, casi siempre pulmonares, y con menor frecuencia:
óseas, ganglionares, hepáticas y cerebrales. Los estudios históricos han demostrado, como en
los TFEw, que prácticamente todos los pacientes sin enfermedad diseminada detectable al
diagnóstico, tienen ya micrometástasis subclínicas. La elevación de la fosfatasa alcalina y de la
lactodehidrogenasa aparece en más del cuarenta por ciento de los pacientes. Este último
parámetro es de gran significación pronóstica, dado que los niveles más elevados aparecen en
pacientes con enfermedad diseminada y se correlacionan inversamente con la supervivencia. La
velocidad de eritrosedimentación tiene menor trascendencia pronóstica. Estudios radiológicos e
isotópicos La radiología convencional muestra la presencia de lesiones permeativas con
destrucción del patrón trabecular normal y con márgenes difusos. Es frecuente la neoformación
ósea periostal, así como la afectación de partes blandas. La TAC y la RMN proporcionan una
información más precisa de la extensión tumoral (especialmente de la extraósea), necesaria para
la planificación del tratamiento quirúrgico. Es imprescindible practicar a todos los pacientes una
TAC torácica para detectar la presencia de metástasis pulmonares y/o pleurales. Los estudios
isotópicos con tecnecio y gadolinio, además de en la valoración inicial de la extensión de la
enfermedad, son útiles en el seguimiento posterior de los paciente para descubrir recidivas
locales o metástasis óseas.

CIRUGÍA Y FACTORES PRONÓSTICOS En cuanto a la localización del tumor primario, las lesiones axiales,
QUIMIOTERAPIA como la pélvica, tienen peor pronóstico que las de las extremidades, fundamentalmente por la
mayor dificultad de lograr en ellas el control quirúrgico de la lesión. También, se considera
actualmente que el volumen tumoral tiene importancia pronóstica. El más importante de los
factores conocidos, el grado de respuesta a la quimioterapia en la pieza quirúrgica del tumor
primario III), no es accesible en el momento del diagnóstico. TRATAMIENTO El tratamiento
actual del sarcoma osteogénico se aborda desde una perspectiva multidisciplinaria, combinando
la cirugía oncológica ortopédica y la poliquimioterapia. La extirpación quirúrgica del tumor ha
sido el único tratamiento disponible hasta hace tres décadas. Sin embargo, cerca del 90% de los
pacientes con enfermedad aparentemente localizada al diagnóstico desarrollaban metástasis,
fundamentalmente pulmonares, y fallecían antes de dos años tras el diagnóstico. Los ensayos
iniciales con quimioterapia sistémica en los años setenta del pasado siglo, condujeron a la
aplicación de quimioterapia preoperatoria. Además de intentar evitar la progresión de las
micrometástasis, se conseguía una disminución del tumor primario que facilitaba la cirugía
conservadora del miembro. LA CIRUGÍA ES EL TRATAMIENTO DE PRIMERA LÍNEA, SIEMPRE
DEBERÁ IR ACOMPAÑADO DE QUMIOTERAPIA, YA QUE ELLO SE HA RELACIONADO CON
MEJOR PRONÓSTICO GRACIAS A QUE REDUCE EL RIESGO DE METÁSTASIS.

Bibliografía:
NIÑO CON CÁNCER. DR. ROBERTO RIVERA LUNA. EDITORES DE TEXTOS MEXICANOS. EDICIÓN 1A. 2007.
PAG. 149,150.
ANÁLISIS DEL CASO CLÍNICO

IDENTIFICACIÓN DEL REACTIVO


Area: GINECOLOGÍA Y OBSTETRICIA
Especialidad: OBSTETRICIA
Tema: COMPLICACIONES DEL EMBARAZO Y DEL PARTO
Subtema: CESAREA Y DETERIORO FETAL

CASO CLÍNICO SERIADO

FEMENINA DE 23 AÑOS, SECUNDIGESTA, CESÁREA PREVIA, CON EMBARAZO GEMELAR DE 35 SDG, ACUDE AL
SERVICIO DE URGENCIAS POR PRESENCIA DE 3 CONTRACCIONES EN 1 HORA CON UNA DURACIÓN DE 20
SEGUNDOS CADA UNA, A SU LLEGADA PRESENTA SALIDA DE LÍQUIDO TRASVAGINAL CLARO ABUNDANTE, EL
ULTRASONIDO REPORTA PLACENTA A 5CM DEL ORIFICIO CERVICAL INTERNO, ÍNDICE DE LÍQUIDO
AMNIÓTICO MENOR A 5, LLAMA LA ATENCIÓN LA PRESENCIA DE BRADICARDIA EN UNO DE LOS PRODUCTOS.

25 AÑOS.

SIN CONTROL PRENATAL.

SALIDA DE LIQUIDO TRASVAGINAL


ABUNDANTE HACE 6HRS.

TRABAJO DE PARTO FASE ACTIVA.

US OLIGOHIDRAMNIOS,
TOCCARDIOGRAFIA CON DESCENSOS
BRUSCOS DE LA FCF.

311 - SE INDICA CESAREA URGENTE CUYA INDICACIÓN EN ESTE CASO ES:

PLACENTA PREVIA. La PLACENTA PREVIA es la complicación obstétrica consistente en la implantación anormal


placentaria, la cual ocurre a nivel del segmento uterino y que, en ocasiones, cubre parcial o
totalmente el orificio cervical interno, de tal modo que obstruye el paso del feto durante el
parto. - Placenta previa total: cubre el orificio cervical interno en su totalidad. - Placenta
previa parcial: cubre parcialmente el orifico cervical interno. - Placenta previa marginal: el
borde de la placenta queda al margen del orificio cervical interno. - Implantación baja de la
placenta: placenta implantada en el segmento uterino inferior cerca del orificio cervical
interno. NO CUMPLE CON LOS CRITERIOS PARA ESTA PATOLOGÍA.
SUFRIMIENTO Las principales indicaciones operación CESÁREA son: desproporción cefalopélvica, cesárea
FETAL. previa, sufrimiento fetal, ruptura prematura de membranas y presentación distinta de la
cefálica. Cuando el SUFRIMIENTO FETAL está presente existe siempre indicación de
interrumpir el trabajo de parto y extraer tan rápidamente al feto como lo permitan las
condiciones médico-obstétricas del caso. La BRADICARDIA, la arritmia y las alteraciones de
intensidad de los tonos cardíacos sí indican siempre la extracción rápida del feto. ES CLARO
QUE HAY PRESENCIA DE RUPTURA PREMATURA DE MEMBRANAS CON
OLIGOHIDRAMNIOS SECUNDARIO. DEBES ELEGIR ESTA RESPUESTA COMO CORRECTA YA
QUE DESCRIBE LA SITUACIÓN EMERGENTE EN ESTE MOMENTO Y DE DONDE DERIVA EL
MANEJO MÉDICO INMEDIATO.

OLIGOHIDRAMNIOS. La principal causa que lleva al oligohidramnios es la rotura prematura de membranas


fetales, que coincide con el caso de la paciente. El nacimiento del producto vía cesárea es
frecuente en esta condición con lo cual se mejora el pronóstico para el recién nacido.
CUANDO EL ÍNDICE DE LÍQUIDO AMNIÓTICO ES MENOR A 5 SE DIAGNOSTICA
OLIGOHIDRAMIOS. ES EVIDENTE LA PRESENCIA DE ÉSTE ANTE UNA RUPTURA DE
MEMBRANAS PERO NO ES EL DIAGNÓSTICO DE PRINCIPAL IMPORTANCIA SINO UN
FACTOR DESENCADENANTE DEL SUFRIMIENTO FETAL.

RUPTURA La RUPTURA PREMATURA DE MEMBRANAS es la ruptura del corion y del amnios antes de
PREMATURA DE que se inicie el trabajo de parto. En la madre existe el riesgo de infección sobre todo cuando
MEMBRANAS. la RPM es prolongada. Si el parto no se inicia en forma espontánea dentro de las 24 horas se
procederá a la interrupción del embarazo. ES ADECUADO HABLAR DE UNA RUPTURA
PREMATURA DE MEMBRANAS EN ESTE MOMENTO, SIN EMBARGO NO ES EL DIAGNÓSTICO
DE MAYOR IMPORTANCIA, ES UN FACTOR DESENCADENANTE DEL SUFRIMIENTO FETAL.

Bibliografía:
1. CUNNINGHAM G, LEVENO K, BLOMM S, HAUTH J, RPUSE D, SONG C. WILLIAMS OBSTETRICIA, 23A
EDICIÓN. MC GRAW HILL. USA. 2011 EN ESPAÑOL, PP 429-431.

312 - SE INDICA CESÁREA DE URGENCIA. CORRESPONDE AL TIPO DE HISTEROTOMÍA DE ELECCIÓN EN


ESTE CASO:

CORPORAL OJO. Para poder contestar esta pregunta deberás de partir de algunos puntos importantes: -
CLÁSICA. Cesárea previa. - Embarazo gemelar - “Placenta a 5cm del orificio cervical interno”. Además, habrá
que recordar que las anomalías en la inserción de la placenta se clasifican de acuerdo a la distancia
entre el borde placentario y el orificio cervical interno en: 1. De inserción baja: el borde
placentario se encuentra en el segmento inferior a menos de 7cm del orificio cervical interno. 2.
Marginal: el borde placentario alcanza los márgenes del orificio cervical interno. 3. Central parcial:
la placenta cubre el orificio cervical interno cuando el cuello se encuentra cerrado, pero cuando
hay una dilatación igual o mayor a 3cm solo cubre parcialmente al orificio cervical interno. 4.
Central total: la placenta cubre la totalidad del orificio cervical interno aun con dilatación
avanzada. EN ESTE CASO SE TRATA DE UNA PLACENTA DE INSERCIÓN BAJA QUE SE ENCUENTRA
EN EL “SEGMENTO INFERIOR”, POR LO TANTO, DEBEMOS EVITAR LAS INCISIONES A ESTE NIVEL
POR EL RIESGO DE PODER LESIONAR LA PLACENTA, PROVOCANDO SANGRADO. NOTA: si bien, la
lesión sólo podría suceder en aquellas placentas de inserción baja en la cara anterior, el
ultrasonido no nos ofrece detalles a este respecto y debemos elegir la mejor respuesta sólo con
base en los datos que se nos otorgan. CESÁREA CORPORAL CLÁSICA: La incisión es vertical se
realiza en el “cuerpo uterino”. Sus indicaciones más frecuentes son: cáncer cérvico-uterino invasor,
embarazo pretérmino, situación fetal transversa con dorso inferior, histerorrafia corporal previa,
procesos adherenciales o varicosos importantes en el segmento inferior, placenta previa de cara
anterior, cesárea posmortem, miomatosis uterina de grandes elementos y cuando después de la
cesárea se realizará una histerectomía. AL TRATARSE DE UNA PLACENTA DE INSERCIÓN BAJA,
QUE SE ENCUENTRA INSTALADA EN EL SEGMENTO INFERIOR, SE PREFIERE REALIZAR UNA
CESÁREA CORPORAL, LO QUE GARANTIZA NO LESIONAR LA PLACENTA.
FÚNDICA CESÁREA FÚNDICA CORPORAL O CORPORAL ALTA. La técnica quirúrgica aplicada consiste
CORPORAL fundamentalmente en abordar el útero a través del cuerpo lo más superior posible. Su indicación
ALTA. principal es placenta acreta y en general el resto de las indicadas para la cesárea corporal clásica.
Incluye la placenta previa de cara anterior. RECUERDA: - En caso de paciente programada para
operación cesárea por placenta previa y/o acretismo placentario, realizar histerotomía corporal
anterior o fúndica y “nunca a nivel del segmento”. - En embarazos múltiples la incisión uterina
debe ser lo suficientemente amplia para permitir el paso atraumático de los fetos, teniendo
siempre la opción a ampliarla en caso de ser necesario. LA INCISIÓN FÚNDICA CORPORAL ALTA
PUEDE SER UNA BUENA OPCIÓN PARA EVITAR EL SEGMENTO, SIN EMBARGO, OFRECE POCAS
VENTAJAS PARA LA EXTENSIÓN Y ABORDAJE EN PACIENTES CON EMBARAZOS MÚLTIPLES, POR
ESO SE PREFIERE INICIAR EN UNA CESÁREA CORPORAL QUE PUEDE EXPONER PERFECTAMENTE A
LOS PRODUCTOS Y TENGA LA VENTAJA DE PODER EXTENDERSE DE MANERA VERTICAL EN
DIRECCIÓN AL FONDO UTERINO.

TRASNVERSAL CESÁREA SEGMENTO ARCIFORME O TRANSVERSAL (KERR): Es la técnica quirúrgica más usada por
DE KERR. sus múltiples ventajas. Al realizarse una incisión transversal del segmento inferior. LA PLACENTA
EN ESTA PACIENTE SE UBICA EN EL SEGMENTO INFERIOR (INSERCIÓN BAJA) POR LO QUE CON
ESTE TIPO DE CESÁREA AUMENTA EL RIESGO DE INCIDIR SOBRE ELLA.

SEGMENTO CESÁREA SEGMENTO CORPORAL (BECK): La incisión es vertical y se realiza sobre el segmento y
CORPORAL. parte del cuerpo uterino. Sus principales indicaciones son: embarazo pretérmino, embarazo
gemelar, situación fetal transversa con dorso inferior, presentación pélvica, placenta previa en la
cara anterior del útero, anillo de retracción e histerorrafias corporales previas. IMPORTANTE: la
cesárea tipo Beck ofrece la ventaja de permitir ampliar la incisión uterina vertical hacia arriba lo
que ofrece un abordaje más amplio que puede ir desde el segmento hasta el fondo, lo que la hace
muy recomendable para embarazos múltiples. EN ESTE CASO NO SE ELIGE COMO LA MEJOR
OPCIÓN PUESTO QUE LA PLACENTA ES DE INSERCIÓN BAJA, LO QUE SIGNIFICA QUE SE
ENCUENTRA EN EL SEGMENTO INFERIOR PUDIENDO LESIONARSE CON ESTE TIPO DE INCISIÓN.

Bibliografía:
1. REDUCCIÓN DE LA FRECUENCIA PARA OPERACIÓN CESÁREA. EVIDENCIAS Y RECOMENDACIONES;
GUÍA DE PRÁCTICA CLÍNICA. MÉXICO, INSTITUTO MEXICANO DEL SEGURO SOCIAL: 2014. 2. DIAGNÓSTICO Y
TRATAMIENTO OPORTUNO DE LA PLACENTA PREVIA EN EL 2º Y 3ER TRIMESTRE DEL EMBARAZO EN EL
SEGUNDO Y TERCER NIVEL DE ATENCIÓN. EVIDENCIAS Y RECOMENDACIONES; GUÍA DE PRÁCTICA
CLÍNICA. MÉXICO, SECRETARÍA DE SALUD:2008. HTTP://WWW.CENETEC-DIFUSION.COM/CMGPC/ISSSTE-
124-08/ER.PDF 3. CUNNINGHAM G, LEVENO K, BLOMM S, HAUTH J, RPUSE D, SONG C. WILLIAMS
OBSTETRICIA, 23A EDICIÓN. MC GRAW HILL. USA. 2011 EN ESPAÑOL. 4. DECHERNEY A, NATHAN L, LAUFER
N, ROMAN A. DIAGNÓSTICO Y TRATAMIENTO GINECOOBSTÉTRICOS, 11ª EDICIÓN. MC GRAW HILL
EDUCATION, LANGE. MÉXICO. 2014.

http://www.cenetec-difusion.com/CMGPC/IMSS-048-08/ER.pdf

FIN DEL CASO CLÍNICO SERIADO


ANÁLISIS DEL CASO CLÍNICO

IDENTIFICACIÓN DEL REACTIVO


Area: GINECOLOGÍA Y OBSTETRICIA
Especialidad: OBSTETRICIA
Tema: ENFERMEDAD HIPERTENSIVA DEL EMBARAZO
Subtema: PREECLAMPSIA

CASO CLÍNICO CON UNA PREGUNTA

MUJER DE 40 AÑOS DE EDAD CON ANTECEDENTE DE PREECLAMPSIA Y QUE CURSA ACTUALMENTE CON
EMBARAZO DE 34 SEG Y PREECLAMPSIA SEVERA.

MUJER ADULTA DE 40 AÑOS.

Recuerda los factores de riesgo para


preeclampsia: hipertensión crónica o
enfermedad renal preexistente, diabetes
mellitus pre-existente, enfermedades del
tejido conectivo como lupus, trombo lia
congénita o adquirida, obesidad, edad
mayor a 40 años, embarazo múltiple y
antecedente de preeclampsia. La paciente
cursaba con dos muy importantes.

-.

-.

-.

313 - EN ESTE TIPO DE PACIENTE LA SITUACIÓN FISIOPATOLÓGICA QUE DETERMINA LA APARICIÓN DE


PREECLAMPSIA-ECLAMPSIA ES LA:
DISFUNCIÓN La INSUFICIENCIA RENAL POSTPARTO IDIOPÁTICA O NEFROESCLEROSIS MALIGNA
ENDOTELIAL POSTPARTO, que puede presentarse desde el primer día hasta varias semanas después
SECUNDARIA AL DÉFICIT del parto. Se desconoce su etiología, se cree que puede ser secundaria a una infección
DE FACTOR RELAJANTE viral previa, a retención de restos placentarios, a la administración de fármacos como
ENDOTELIAL los anticonceptivos orales, o derivados del cornezuelo de centeno. Se presenta con
DEPENDIENTE DEL oliguria, anuria, azoemia, datos de anemia hemolítica microangiopática o coagulopatía
ÓXIDO NÍTRICO. por consumo, inducida probablemente por una DISFUNCIÓN ENDOTELIAL
SECUNDARIA AL DÉFICIT DE FACTOR RELAJANTE ENDOTELIAL DEPENDIENTE DEL
ÓXIDO NÍTRICO. A nivel renal hay alteración en los capilares glomerulares y alteración
en las arteriolas. Su pronóstico es reservado. Esta es una causa de insuficiencia renal
aguda y debe considerarse en la paciente posparto con insuficiencia renal aguda (IRA)
sin causa aparente. ES UNA CARACTERÍSTICA DE LA INSUFICIENCIA RENAL POSTPARTO
IDIOPÁTICA, EL HECHO DE SER POSTPARTO, LA DESCARTA COMPLETAMENTE.

PRESENCIA DE SÍNDROME NEFRÓTICO Se caracteriza por la presencia de proteinuria mayor de 3 g/día,


GLOMERULOESCLEROSIS, albúmina sérica menor de 3 g/dL, edema e hipercolesterolemia. Su incidencia es en 1 de
HIPERTROFIA CELULAR Y cada 1,500 embarazos, su etiología es variada, aunque es más frecuente por
CAMBIOS preeclampsia, nefritis lúpica, nefropatía diabética, glomerulopatía proliferativa o
DEGENERATIVOS membranoproliferativa, nefrosis lipídica, nefritis hereditaria, trombosis de la vena renal
TUBULARES. y amiloidosis. Los datos más importantes para diferenciarlas son la hipertensión (que
sugiere preeclampsia), el sedimento urinario (los cilindros eritrocitarios sugieren
glomerulonefritis aguda), los anticuerpos antinucleares y la disminución del
complemento indicarían lupus. Los procesos infecciosos en pacientes embarazadas
raramente producen síndrome nefrótico. En la década pasada han aumentado los
reportes de síndrome nefrótico en pacientes con infección por el virus de la
inmunodeficiencia humana (VIH); este es caracterizado por proteinuria severa y rápida
progresión a falla renal terminal, las características histológicas en la biopsia renal son
GLOMERULOESCLEROSIS, HIPERTROFIA CELULAR Y CAMBIOS DEGENERATIVOS
TUBULARES. Esto se ha visto sobre todo en poblaciones afroamericanas, por lo que
debemos sospechar esta entidad en pacientes embarazadas e infectadas por el VIH. El
tratamiento es etiológico y, como medidas conservadoras, la dieta debe ser alta en
proteínas de alto peso molecular, baja en colesterol y en grasas saturadas, restringir el
aporte de sodio, administrar heparina (ya que la tendencia a desarrollar trombosis
incrementa con el embarazo y la hipoalbuminemia). La administración de albúmina es
controvertida, sólo si el edema progresara a anasarca y la albúmina fuera menor de 2 g
se puede indicar. Los antibióticos se administran de forma profiláctica por la alta
incidencia de infecciones urinarias. UNA CAUSA IMPORTANTE DE SÍNDROME
NEFRÓTICO EN EL EMBARAZO ES LA PREECLAMSIA, SI TE FIJAS BIEN, LA OPCIÓN DE
RESPUESTA SE REFIERE A LOS HALLAZGOS PATOLÓGICOS RENALES DE UN SÍNDROME
NEFRÓTICO NO AL MECANISMO FISIOPATOLÓGICO, LO QUE LA DESCARTA COMO
RESPUESTA CORRECTA.

LIBERACIÓN DE Fisiopatología del SÍNDROME HELLP. Como en la preeclampsia y en otros procesos


PROTEÍNAS COMO asociados a trombopenia y microangiopatía, el fenómeno inicial probablemente sea
FIIBRONECTINA QUE una LESIÓN SISTÉMICA DE CÉLULAS ENDOTELIALES por diferentes mecanismos
PRODUCEN ACTIVACIÓN (hipertensión, alteraciones en la placentación, complejos inmunes). Sobre estos vasos
Y ADHESIÓN DE LAS lesionados se produciría un desequilibrio entre las sustancias vasodilatadoras
PLAQUETAS AL (prostaciclina) y vasoconstrictoras (tromboxano A2, serotonina, endotelina-1) a favor
SUBENDOTELIO. de estas últimas que tendrían un papel fundamental en la patogenia del proceso. Por
otra parte, la alteración endotelial se acompaña de la LIBERACIÓN DE PROTEÍNAS DE
LA MATRIZ CELULAR COMO FIBRONECTINA Y FACTOR DE VON WILLEBRAND que
producirían una activación y adhesión de las plaquetas al subendotelio así como una
agregación plaquetaria y trombopenia. La trombopenia también podría deberse a la
activación de la coagulación y del sistema fibrinolítico que puede acompañarse de
cuadros de coagulación intravascular diseminada (CID), con el consiguiente aumento de
la destrucción plaquetaria. La activación del sistema de coagulación se asociaría a una
disminución de los anticoagulantes fisiológicos como la antitrombina III y la proteína C,
todo lo cual contribuiría a la formación de microtrombos en la circulación y la
subsiguiente disfunción orgánica generalizada. DEBERÁS RECORDAR QUE TODA
PACIENTE CON PREECLAMPSIA PUEDE LLEGAR A COMPLICARSE CON ÉSTE SÍNDROME,
PERO NO ES CORRESPONDE PROPIAMENTE ESTA RESPUESTA A LA PREECLAMPSIA.
LESION DEL ENDOTELIO La PREECLAMPSIA-ECLAMPSIA constituyen un síndrome que se caracteriza por:
DE LOS VASOS hipoperfusión tisular generalizada relacionada con una respuesta vascular anormal
SANGUÍNEOS. placentaria que se asocia con un aumento en las resistencias vasculares sistémicas,
DAÑO ENDOTELIAL, cambios metabólicos, consumo plaquetario, aumento en la
respuesta inflamatoria y activación del sistema de coagulación. Hay distintas teorías en
cuanto a la etiología de éste síndrome sin embargo la más aceptada es la lesión
endotelial de la cual se desprenden el resto de ellas. Si bien en mujeres portadoras de
preeclampsia la actividad de renina plasmática es menor a la de embarazadas normales,
la hiperreninemia podría tener un efecto potenciador sobre las arterias hiperreactivas.
RECUERDA QUE EXISTEN VARIAS TEORÍAS CON RESPECTO A LA FISIOPATOLOGÍA DE
LA PREECLAMSIA PERO TODAS ELLAS CONVERGEN EN EL DAÑO ENDOTELIAL.

Bibliografía:
1. CUNNINGHAM G, LEVENO K, BLOMM S, HAUTH J, RPUSE D, SONG C. WILLIAMS OBSTETRICIA, 23A
EDICIÓN. MC GRAW HILL. USA. 2011 EN ESPAÑOL, PP 713-714.
ANÁLISIS DEL CASO CLÍNICO

IDENTIFICACIÓN DEL REACTIVO


Area: CIRUGÍA
Especialidad: URGENCIAS
Tema: TRAUMA DE TÓRAX
Subtema: TÓRAX INESTABLE Y CONTUSIÓN PULMONAR

CASO CLÍNICO SERIADO

FEMENINA DE 40 AÑOS DE EDAD, SUFRE CAÍDA DESDE UNA ESCALERA DE 2 METROS RECIBIENDO GOLPE
DIRECTO EN LA REGIÓN COSTAL, POR LO QUE ES LLEVADA A URGENCIAS. A LA EXPLORACIÓN CON TENSIÓN
ARTERIAL DE 110/70, FRECUENCIA CARDIACA DE 90 POR MINUTO, FRECUENCIA RESPIRATORIA DE 40 POR
MIN. SE ENCUENTRA AGITADO, RESPIRACIONES RÁPIDAS Y SUPERFICIALES, A LA PERCUSIÓN CLARO
PULMONAR, RUIDOS RESPIRATORIOS PRESENTES EN AMBOS HEMITÓRAX, TRAQUEA CENTRAL, PRESENTA
CREPITACIÓN EN REGIÓN COSTAL DERECHA. ASIMETRÍA DE LOS MOVIMIENTOS RESPIRATORIOS. SE LE
ADMINISTRA OXÍGENO SUPLEMENTARIO A 10 LTS/MIN. Y SOLUCIONES CRISTALOIDES INTRAVENOSAS.
RADIOGRAFÍA DE TÓRAX NO CONCLUYENTE.

Mujer de 40 años de edad.

Caída desde una escalera de dos metros,


con trauma torácico.

-.

Hemodinámicamente estable,
compromiso respiratorio con 40
respiraciones por minuto. Respiración
rápida y super cial, asimétrica, crepitación
a la palpación de región costal. La
percusión es normal, ruidos respiratorios
presentes y la traquea es central. No se
integra ningún síndrome pleuropulmonar,
con la exploración exclusivamente se
puede descartar neumotórax y hemotórax.

Radiografía no concluyente.

314 - ESTA PACIENTE CURSA CON EL DIAGNOSTICO DE:


TÓRAX El TRAUMA DE TÓRAX contribuye con cerca del 25% de las muertes en los pacientes
INESTABLE. traumatizados; se debe tener en cuenta que los traumatismos cerrados alcanzan una mortalidad
mucho mayor cuando son severos. El tórax inestable o tórax batiente o volet costal, se define
como la fractura de 3 o 4 costillas en más de dos segmentos de su longitud ocasionando un
fenómeno paradójico en la caja torácica colapsándolo en inspiración y expandiéndolo en
espiración. Es importante reconocer que se asocia a múltiples lesiones torácicas como contusión
pulmonar, hemotórax, neumotórax y a nivel extratorácico con lesiones abdominales, encefálicas y
lesiones en extremidades inferiores. La frecuencia del tórax inestable es baja alcanzando en las
diferentes series con un total de 851 pacientes, un rango entre el 1.5% y 10.1% en su mayoría
derivados de accidentes de tránsito; otras asociadas son caídas, accidentes de trabajo, agresiones
y lesiones deportivas. Después de evaluar el ABC en el paciente, se define si presenta insuficiencia
respiratoria representada en taquipnea y aumento del trabajo respiratorio, alteración del estado
de conciencia, agitación psicomotriz, etcétera. En el tórax se evalúa la existencia de movimientos
paradójicos, se ausculta en busca de alteraciones del murmullo vesicular que representen
hemotórax o neumotórax; una clave para observar el tórax inestable es mirar de perfil el
movimiento torácico; esto se puede realizar en un plano horizontal sobre los pies. El estudio de
imagen inicial en estos pacientes, es la radiografía portátil de tórax, la cual se realiza en
proyección antero posterior, con el inconveniente de ser deficiente en la evaluación de los arcos
posteriores y laterales de las costillas, si el paciente presenta compromiso multiorgánico con gran
severidad del trauma se sugiere realizar tomografía torácica y abdominal simple y con contraste,
con la ventaja adicional de poder evaluar estructuras vasculares y sólidas en las diferentes
cavidades. Son las lesiones más frecuentes en el trauma de tórax, se pueden presentar en el
trauma cerrado o en heridas por proyectil de arma de fuego. Su DIAGNÓSTICO se realiza por
medio del examen físico. Con la palpación se localiza la zona dolorosa y en algunas circunstancias
se encuentra crepitación o signo de la tecla en la costilla afectada. En la radiografía de tórax no
siempre se ven las fracturas, especialmente si no están desplazadas. La radiografía de reja costal
tiene poca aplicación. La gammagrafía ósea es muy útil después de las primeras 24 horas. Cuando
se lesionan las dos primeras costillas se debe sospechar lesión de grandes vasos, y cuando se
fracturan las últimas se debe descartar lesión diafragmática o intraabdominal. En su manejo, el
control del dolor es lo más importante. EL TÓRAX INESTABLE OCURRE CUANDO UN SEGMENTO
DE LA PARED TORÁCICA PIERDE LA CONTINUIDAD ÓSEA COMO ÉSTA, ÉSTA ES LA OPCIÓN
CORRECTA ,DEBIDO A QUE LOS DATOS CLÍNICOS SON COMPATIBLES CON ESTE DIAGNÓSTICO.

CONTUSIÓN La CONTUSIÓN PULMONAR se observa tanto en casos de trauma cerrado, como en los de trauma
PULMONAR. penetrante, especialmente en heridas con proyectiles de alta velocidad. Se han planteado varias
hipótesis sobre el mecanismo, tales como el aumento brusco de la presión intra-alveolar causado
por una sobre expansión intra-pulmonar que produce ruptura de los alvéolos, o la continuación
de la onda de contusión por el efecto de inercia que se inicia con el trauma que destruye los
alvéolos. Todo lo anterior lleva a una lesión de la membrana alveolo-capilar que produce
hemorragia intra-alveolar, edema del intersticio, atelectasias en áreas adyacentes, aumento de la
producción de moco, disminución en la producción de surfactante y aumento de la permeabilidad
capilar. También la onda que se produce en el trauma puede llevar a laceración pulmonar y a los
cambios que caracterizan a la contusión pulmonar. Estos cambios pueden progresar con el
transcurso del tiempo si hay continuidad en el sangrado parenquimatoso o resangrado. Todo lo
anterior conduce a un aumento de peso de los pulmones con incremento de la resistencia arterial,
del contenido del surfactante, disminución de la distensibilidad (Compliance) y del intercambio
gaseoso, que lleva a la hipoxemia. El paciente presenta DISNEA, CIANOSIS y HEMOPTISIS, en el
examen físico se pueden escuchar ESTERTORES O AUSENCIA DE RUIDOS RESPIRATORIOS. En la
radiografía, se observan infiltrados alveolares por la hemorragia dentro de los alvéolos que se va
instaurando progresivamente entre las 6 y las 12 horas post trauma, y duran aproximadamente
una semana. El mejor método para la evaluación de la contusión pulmonar es la tomografía axial
computadorizada (TAC). Los pacientes con contusión pulmonar deben ser hospitalizados para
observación cuidadosa y monitoría respiratoria y cardiovascular. Si la contusión afecta a más del
30% del parénquima, se maneja en la Unidad de Cuidado Intensivo. Se inicia suplemento de
oxígeno para mantener la PaO2 >60 mmHg, se controla el dolor y se realiza terapia respiratoria
vigorosa. Cuando la respiración del paciente no es adecuada, se procede con intubación
orotraqueal y ventilación mecánica. La utilización de esteroides y de antibióticos sigue causando
controversia, el antibiótico está indicado cuando se presentan signos de infección. La contusión
pulmonar es una patología grave. Dependiendo de la magnitud, tiene una mortalidad que oscila
entre el 11% y el 78%. NO CORRESPONDE AL CUADRO CLÍNICO DE LA PACIENTE.
HEMOTÓRAX. El HEMOTÓRAX, es la "acumulación de sangre en el espacio pleural", lo cual resulta de una "lesión
del parénquima pulmonar", de la "pared torácica", de los "grandes vasos", del "corazón" o del
"diafragma". Se puede producir tanto en casos de trauma cerrado como de trauma penetrante, y
la causa principal es la lesión del parénquima pulmonar. Se detecta en la radiografía de tórax,
cuando se han acumulado más de 300 ml de sangre con borramiento del ángulo costofrénico o
apariencia de líquido en el espacio pleural. HEMOTÓRAX MASIVO Se produce por la rápida
acumulación de sangre en el espacio pleural, su principal causa es la lesión del hilio pulmonar o de
los vasos sistémicos de la reja costal. El paciente se puede presentar con inestabilidad
hemodinámica y ausencia de ventilación en un hemitórax; el cuadro clínico es secundario a la
hipovolemia y a la alteración V/Q que lleva a shock y dificultad respiratoria severa. El ATLS del
American College of Súrgenos, define el hemotórax masivo como el drenaje de 1.500 ml de
sangre en el espacio pleural en el momento de la colocación del tubo o un drenaje mayor de 200
ml por hora en 2 a 4 horas. DEBIDO A LA ACUMULACIÓN DE SANGRE EN EL ESPACIO PLEURAL
LOS RUIDOS RESPIRATORIOS ESTARÍAN AUSENTES.

NEUMOTÓRAX El NEUMOTÓRAX A TENSIÓN resulta de lesión de la caja torácica o del pulmón que establece un
A TENSIÓN. mecanismo valvular de una sola vía, el cual a su vez produce acumulación y atrapamiento del aire
en el espacio pleural, hasta crear una gran presión positiva que causa colapso total del pulmón y
desplazamiento del mediastino con angulación de las cavas y disminución del retorno venoso;
todo esto se manifiesta en inestabilidad hemodinámica que pone en grave peligro la vida del
paciente. Se debe sospechar en todo paciente con inestabilidad hemodinámica; establecido el
neumotórax a tensión. El paciente presenta AUSENCIA DE RUIDOS RESPIRATORIOS e
hiperresonancia en el hemitórax afectado y distensión de las venas del cuello con desplazamiento
de la tráquea. NO CORRESPONDE AL CASO CLÍNICO.

Bibliografía:
1. DIAGNÓSTICO Y TRATAMIENTO DEL TRAUMATISMO DE TÓRAX EN ADULTOS. EVIDENCIAS Y
RECOMENDACIONES. GUÍA DE PRÁCTICA CLÍNICA. MÉXICO; CENETEC. 2017. 2. COMITÉ DE TRAUMA DEL
COLEGIO AMERICAN DE CIRUJANOS, PROGRAMA AVANZADO DE APOYO VITAL EN TRAUMA PARA
MÉDICOS. ATLS, 7A EDICIÓN. USA. PP 110.

http://www.cenetec-difusion.com/CMGPC/SS-447-11/ER.pdf

315 - LA PACIENTE SE DETERIORA A PESAR DEL TRATAMIENTO MÉDICO IMPLEMENTADO; SE TOMA


GASOMETRÍA ARTERIAL PRESENTANDO HIPOXEMIA E HIPERCAPNEA, POR LO QUE LA MEDIDA
TERAPÉUTICA DE URGENCIA ES REALIZAR:

TORACOSTOMÍA El manejo del HEMOTÓRAX consiste en el drenaje del espacio pleural, generalmente mediante
CON SONDA. un tubo de toracostomía; el sangrado del parénquima se detiene con la expansión pulmonar, la
baja presión intravascular y la alta concentración de tromboplastina tisular. El hemotórax
simple es aquel que no cumple los criterios de hemotórax masivo.

AUMENTAR EL El MANEJO AGRESIVO DEL DOLOR es un punto clave en el manejo de estos pacientes. Las
APORTE DE O2 Y opciones son manejo endovenoso, bloqueo de nervios intercostales, bloqueo torácico
ANALGESIA. paravertebral o analgesia por catéter epidural. El control del dolor debe ser uno de los
objetivos iníciales en el paciente con fracturas costales, los medicamentos de elección son los
opiodes preferiblemente los más potentes como el fentanyl en goteo o sulfato de morfina para
aplicación en bolos, las dosis se titulan de acuerdo al grado de dolor y la evaluación con la
escala análoga del dolor. El bloqueo de nervios intercostales es efectivo en el manejo inicial y
cuando el número de fracturas no es muy grande (Mayor utilidad en fracturas únicas); se
sugiere utilizar bupivacaina para prolongar el efecto, si no se cuenta en el servicio con éste
medicamento, se puede utilizar lidocaína. Es importante recordar que es fácil llegar a la dosis
toxicas de estos medicamentos. El bloqueo torácico paravertebral es otra opción útil en
asociación con analgesia peridural. Ha mostrado mayor efectividad en el manejo del dolor y los
medicamentos más utilizados son: Indometacina y ketorolaco, la desventaja es la perpetuación
de la injuria renal. El tratamiento de elección es la analgesia peridural. Si bien el manejo
analgésico es fundamental, además del aporte de oxígeno, la paciente presenta deterioro de la
función respiratoria que indica de inmediato la ventilación mecánica.
DESCOMPRESIÓN El tratamiento del NEUMOTÓRAX A TENSIÓN; consiste en la descompresión urgente y rápida
CON AGUJA DE del espacio pleural, lo cual se logra con la colocación de una aguja o catéter en el segundo
GRUESO espacio intercostal, sobre la línea medio axilar, a fin de convertir un neumotórax a tensión en
CALIBRE. un neumotórax simple. Posteriormente se coloca un tubo de toracostomía a succión o drenaje
cerrado.

INTUBACIÓN El objetivo del MANEJO MÉDICO DEL TÓRAX INESTABLE es mantener adecuada ventilación, los
ENDOTRAQUEAL elementos claves son el manejo del dolor y el mejoramiento de la oxigenación. La ventilación
CON es otro de los pilares de manejo en la inmovilización interna del tórax, la presión positiva fue la
VENTILACIÓN primera estrategia utilizada, inicialmente descrita por Avery en 1956. Cullen planteó el uso de
ASISTIDA. ventilación mecánica intermitente con buenos resultados. En la década de los sesentas y
setentas, se pregonaba la utilización de traqueotomías tempranas con ventilación mecánica
con resultados favorables para la disfunción respiratoria, pero con aumento de la incidencia de
neumonías y otras infecciones asociadas a ventilación mecánica prolongada. Las indicaciones
para la vía aérea definitiva son: 1. Necesidades de proteger la vía aérea: pérdida del
conocimiento, fracturas maxilofaciales graves, riesgo de aspiración (hemorragia, vómito),
riesgo de obstrucción (hematoma de cuello, lesión traqueal o laríngea, estridor). 2.
Necesidades de ventilación: apnea (parálisis neuromuscular, pérdida de la conciencia), esfuerzo
respiratorio inadecuado (taquipnea, hipoxemia, hipercarbia, cianosis), traumatismo de cráneo
cerrado severo con necesidad de hiperventilación. En la ventilación mecánica se pueden utilizar
sistemas invasivos y no invasivos. Sin embargo, se sugiere la intubación profiláctica en los
pacientes que cumplen con criterios. LA PACIENTE PRESENTA 3 DE 4 DATOS DE ESFUERZO
RESPIRATORIO (TAQUIPNEA, HIPOXIA, HIPERCAPNIA Y CIANOSIS) POR LO QUE ESTÁ
INDICADA LA INTUBACIÓN.

Bibliografía:
1. DIAGNÓSTICO Y TRATAMIENTO DEL TRAUMATISMO DE TÓRAX EN ADULTOS. EVIDENCIAS Y
RECOMENDACIONES. GUÍA DE PRÁCTICA CLÍNICA. MÉXICO; CENETEC. 2017. 2. COMITÉ DE TRAUMA DEL
COLEGIO AMERICAN DE CIRUJANOS, PROGRAMA AVANZADO DE APOYO VITAL EN TRAUMA PARA
MÉDICOS. ATLS, 7A EDICIÓN. USA. PP 110.

http://www.cenetec-difusion.com/CMGPC/SS-447-11/ER.pdf

FIN DEL CASO CLÍNICO SERIADO


ANÁLISIS DEL CASO CLÍNICO

IDENTIFICACIÓN DEL REACTIVO


Area: GINECOLOGÍA Y OBSTETRICIA
Especialidad: OBSTETRICIA
Tema: COMPLICACIONES DEL EMBARAZO Y DEL PARTO
Subtema: OLIGO Y POLIHIDRAMNIOS

CASO CLÍNICO CON UNA PREGUNTA

MULTIGESTA DE 25 AÑOS DE EDAD CON ANTECEDENTE DE PÉRDIDAS GESTACIONALES DE REPETICIÓN.


ACTUALMENTE CON UN EMBARAZO DE 25 SEMANAS POR FUM. A LA EXPLORACIÓN FÍSICA CON FONDO
UTERINO MAYOR AL ESPERADO PARA LA EDAD GESTACIONAL, FRECUENCIA CARDIOFETAL DENTRO DE
LÍMITES NORMALES. SE SOLICITA ULTRASONIDO POR SOSPECHA DE POLIHIDRAMNIOS.

25 años.

pérdidas gestacionales de repetición


(sugiere malformaciones fetales que
pueden asociarse a polihidramnios).

--

fondo uterino mayor al esperado para la


edad gestacional.

USG

316 - ES EL ÍNDICE DE PHELAN A PARTIR DEL CUAL SE CONFIRMARÍA EL DIAGNÓSTICO:

23 El líquido amniótico tiene múltiples funciones para la gestación dentro de las que destacan: promover el
CM desarrollo pulmonar fetal fisiológico, evitar la compresión del cordón umbilical y crear un espacio físico
propicio para que el esqueleto fetal adquiera su forma normal.

25 El POLIHIDRAMNIOS es el aumento en el volumen del líquido amniótico. En la mayoría de los casos responde
CM a un aumento gradual de la cantidad de líquido más que a un proceso agudo; aunque es posible que aparezca
de esta última forma dando lugar a una distensión aguda que se lleva a cabo en unos cuantos días. Según el
índice de Phelan se define al polihidramnios como la presencia de un índice mayor a 24 cm. EL
POLIHIDRAMNIOS CORRESPONDE A UN ÍNDICE DE LÍQUIDO AMNIÓTICO O ÍNDICE DE PHELAN MAYOR A
24-25 CM.
27 El método más adecuado para medir la cantidad de líquido amniótico es el ecográfico mediante la valoración
CM del índice de líquido amniótico o índice de Phelan. Esta valoración resulta de la suma de las mediciones
verticales de las acumulaciones más grandes de líquido en cada uno de los cuatro cuadrantes uterinos
equivalentes.

29 Se considera que de manera normal, el líquido amniótico alcanza un volumen máximo alrededor de la semana
CM 36 llegando a ser de hasta 1L, situación que va en decremento hasta llegar a menos de 200 a la semana 42.

Bibliografía:
1. CUNNINGHAM G, LEVENO K, BLOMM S, HAUTH J, RPUSE D, SONG C. WILLIAMS OBSTETRICIA, 23A
EDICIÓN. MC GRAW HILL. USA. 2011. 2. DECHERNEY A, NATHAN L, LAUFER N, ROMAN A. DIAGNÓSTICO Y
TRATAMIENTO GINECOOBSTÉTRICOS, 11ª EDICIÓN. MC GRAW HILL EDUCATION, LANGE. MÉXICO. 2014.
ANÁLISIS DEL CASO CLÍNICO

IDENTIFICACIÓN DEL REACTIVO


Area: GINECOLOGÍA Y OBSTETRICIA
Especialidad: OBSTETRICIA
Tema: ENFERMEDAD HIPERTENSIVA DEL EMBARAZO
Subtema: ECLAMPSIA

CASO CLÍNICO CON UNA PREGUNTA

MUJER DE 16 AÑOS, CON EMBARAZO DE 34 SDG. PRESENTA DESDE HACE 6 HORAS CEFALEA, ACÚFENOS Y
FOSFENOS. SE DETECTA TA DE 130/105. DURANTE LA EXPLORACIÓN LA PACIENTE PRESENTA CRISIS
CONVULSIVA TÓNICO-CLÓNICA GENERALIZADA, QUE ES TRATADA CON SULFATO DE MAGNESIO.

adolescente.

-.

-.

crisis convulsiva tónico clónica


generalizada.

-.

317 - EL SIGUIENTE DATO SERÍA INDICATIVO DE MUY MAL PRONÓSTICO EN LA PACIENTE:

COMA El ESTADO DE COMA es un estado severo de pérdida de consciencia, que puede resultar de una
POSTERIOR gran variedad de condiciones incluyendo las intoxicaciones (drogas, alcohol o tóxicos),
PERSISTENTE. anomalías metabólicas (hipoglucemia, hiperglucemias, cetosis), enfermedades del sistema
nervioso central, accidente vascular cerebral, traumatismo cráneo-encefálico, convulsiones e
hipoxia. EL DETERIORO EN EL ESTADO DE CONCIENCIA CONSTITUYE EL "PRINCIPAL
INDICATIVO" DE MAL PRONÓSTICO EN LA PACIENTE CON ECLAMPSIA.

CRISIS El manejo de las crisis convulsivas durante el embarazo debe realizarse con sulfato de
CONVULSIVA magnesio independientemente de la duración de las mismas. Las crisis convulsivas son el signo
CON DURACIÓN clave del diagnóstico de eclampsia y el pronóstico de la paciente depende del manejo
MAYOR DE 10 posterior y la recuperación de la paciente. IMPORTANTE: TODOS LOS DATOS MENCIONADOS
MINUTOS. SON ALARMANTES DURANTE LA EVALUACIÓN CLÍNICA DE LA PACIENTE, ES NECESARIO QUE
ELIJAS EL MÁS IMPORTANTE PARA ACERTAR A LA RESPUESTA CORRECTA.
ESTADO El ESTATUS EPILÉPTICO es una situación de convulsión persistente en el tiempo, o bien cuando
EPILÉPTICO. las crisis comiciales se repiten frecuentemente sin permitir recuperar el nivel de conciencia o la
normalidad intercrisis. Menos del 1% de las pacientes epilépticas presentan un status
epiléptico durante la gestación. La definición estándar es de más de 30 minutos, pero
probablemente es necesario el tratamiento en toda crisis convulsiva que persista más de 5-10
minutos. EL ESTADO EPILÉPTICO ES UNA COMPLICACIÓN POCO FRECUENTE EN EL EMBARAZO
Y DEBE TRATARSE DEL MISMO MODO QUE LAS CRISIS CONVULSIVAS AISLADAS, NO
DEPENDE DE ELLO EL PRONÓSTICO SIEMPRE Y CUANDO SE RECUPERE LA PACIENTE.

ARREFLEXIA Y La ARREFLEXIA es la ausencia de reflejos osteotendinosos en un paciente. Las PARESIAS son la


CUADRIPARESIAS. disminución de la fuerza muscular, que al referir cuadriparesia se refiere a las cuatro
extremidades. Ambos hallazgos podrían ser más bien explicados por una probable
intoxicación por sulfato de magnesio, que debe ser considerado más frecuente en pacientes
tratadas con sulfato de magnesio e insuficiencia renal. El 2-10% de las pacientes con
preeclampsia presentan convulsiones, configurando un cuadro de eclampsia. El 25% de los
casos se presentan en el puerperio. La prevención de las convulsiones se efectuará en todas las
pacientes que cursan con preeclampsia grave. Para estos fines, se utilizará el sulfato de
magnesio -ampollas de 1.25 gr-, en dosis iniciales, por vía intravenosa lentamente, de 2-4 gr,
seguidos de una infusión a razón de 1-2 gr/horas, continuando hasta 24 horas después de
parto o cesárea, o luego de la última convulsión. Mientras se administra sulfato de magnesio
deberá controlarse: a) La diuresis, en caso de disminuir el ritmo diurético, aumentar el aporte
parenteral de líquidos, si la paciente desarrolla oligoanuria deberá suspenderse. b) El reflejo
patelar, evaluado cada hora. La arreflexia coincide con niveles séricos de magnesio
suficientemente elevados como para suspender su administración (8-10 mg/dl
aproximadamente). El reflejo patelar carece de valor luego de una anestesia peridural, en estos
casos se utilizará el reflejo bicipital La concentración sérica. Los valores terapéuticos son 4-8
mg/dl, suspender la infusión con valores por encima de 8 mg/dl. La apnea sobreviene con
niveles superiores a 15 mg/dl, y el paro cardíaco con valores mayores de 25 mg/dl. En caso de
intoxicación por sulfato de magnesio, el antídoto es el gluconato de calcio al 10%, en dosis de
10 ml a pasar en 3 minutos por vía intravenosa. El tratamiento de las convulsiones se efectúa
sulfato de magnesio.

Bibliografía:
1. CUNNINGHAM G, LEVENO K, BLOMM S, HAUTH J, RPUSE D, SONG C. WILLIAMS OBSTETRICIA, 23A
EDICIÓN. MC GRAW HILL. USA. 2011 EN ESPAÑOL, PP 721-724, 749.
ANÁLISIS DEL CASO CLÍNICO

IDENTIFICACIÓN DEL REACTIVO


Area: MEDICINA INTERNA
Especialidad: CARDIOVASCULAR
Tema: PATOLOGÍA MIOCÁRDICA
Subtema: INSUFICIENCIA CARDIACA

CASO CLÍNICO SERIADO

MUJER DE 75 AÑOS HIPERTENSA DESDE HACE 30 AÑOS. SEDENTARIA Y OBESA. ACUDE A CONSULTA POR
PRESENTAR SÚBITAMENTE DISNEA. A LA EXPLORACIÓN SE ENCUENTRA FC 135/MIN, TA 150/100,
INGURGITACIÓN YUGULAR GRADO IV Y EDEMA HASTA RODILLAS.

MUJER DE 75 AÑOS.

HIPERTENSA DESDE HACE 30 AÑOS,


SEDENTARIA Y OBESA.

PRESENTA SÚBITAMENTE DISNEA.

FC 135/MIN, TA 150/100, INGURGITACIÓN


YUGULAR GRADO IV Y EDEMA HASTA
RODILLAS.

-.

318 - EL DIAGNÓSTICO CLÍNICO MÁS PROBABLE ES:


INSUFICIENCIA CRITERIOS DE FRAMINGHAM PARA EL DIAGNÓSTICO DE INSUFICIENCIA CARDÍACA
CARDÍACA CONGESTIVA (ICC). 1. Criterios Mayores: • Disnea paroxística nocturna • Distensión venosa
CONGESTIVA. yugular • Estertores • Cardiomegalia • Edema agudo de pulmón • Ritmo de galope por tercer
ruido • Aumento de la presión venosa (>16 cmH2O) • Reflujo hepatoyugular positivo 2.
Criterios Menores • Edema en miembros • Tos nocturna • Disnea de esfuerzo • Hepatomegalia
• Derrame pleural • Capacidad vital disminuida en un tercio • Taquicardia (120 lpm) 3. Criterio
Mayor o Menor • Pérdida de peso 4.5 kg después de 5 días de tratamiento 4. Para establecer el
diagnóstico clínico de insuficiencia cardíaca congestiva según estos criterios, se necesitan
como mínimo un criterio mayor y dos menores. LA PACIENTE PRESENTA 2 CRITERIOS
MAYORES (DISTENSIÓN VENOSA YUGULAR Y DISNEA) Y CRITERIOS MENORES (EDEMA EN
MIEMBROS, DISNEA Y TAQUICARDIA) LO QUE CONFIRMA LA SOSPECHA CLÍNICA. NOTA: La
GPC más actual para la insuficiencia cardíaca muestra datos que estiman la sensibilidad y
especificidad de los principales marcadores clínicos de congestión de la siguiente manera:
disnea de esfuerzo (66 y 52%), Ortopnea (66 y 47%), Edema (46 y 73%), Distensión venosa
yugular en reposo (70 y 79%), S3 (73 y 42%) y, para los marcadores radiográficos,
cardiomegalia (97 y 10%, redistribución (60-68%) y edema intersticial (60 y 73%). LA
PACIENTE PRESENTA EL MARCADOR CON MAYOR SENSIBILIDAD Y ESPECIFICIDAD
(DISTENSIÓN VENOSA YUGULAR EN REPOSO) SUMADO A OTROS MARCADORES COMO SON
DISNEA Y EDEMA

COR PULMONALE. El cor pulmonale se define como un aumento de tamaño del ventrículo derecho (right
ventricle, RV) secundario a alteraciones del pulmón, del tórax y la ventilación o la circulación
pulmonares. A veces provoca insuficiencia ventricular derecha y una elevación de la presión
telediastólica transmural del ventrículo derecho. CX: La insuficiencia ventricular derecha aguda
causa palidez, diaforesis, hipotensión y pulso rápido y débil. Las venas del cuello están
distendidas y con frecuencia muestran ondas v prominentes de insuficiencia tricuspídea. El
hígado puede ser pulsátil, estar aumentado de tamaño y doloroso a la palpación. El soplo
sistólico de la insuficiencia tricuspídea en el borde esternal puede ir acompañado de un galope
presistólico. Los gases arteriales con frecuencia muestran una PaO2 reducida por desequilibrio
ventilación/perfusión y una PaCO2 baja por hiperventilación. POR PRINCIPIO DE CUENTAS LA
PACIENTE PRESENTA HIPERTENSIÓN LO CUAL DESCARTA ÉSTA POSIBILIDAD DIAGNÓSTICA.

TROMBOEMBOLIA EMBOLIA PULMONAR: La disnea es el síntoma más frecuente de la PE y la taquipnea el signo


PULMONAR. más frecuente. Mientras la disnea, el síncope, la hipotensión o la cianosis indican una PE
masiva, el dolor pleurítico, la tos o la hemoptisis suelen indicar una embolia pequeña situada
distalmente cerca de la pleura. • En la exploración física, las personas jóvenes y que antes
estaban sanas sólo parecen ansiosas pero por lo demás están muy bien, incluso con una PE
anatómicamente grande. Quizá experimenten nada más disnea con el esfuerzo moderado.
Suelen carecer de los signos "clásicos" como taquicardia, fiebre de grado bajo, distensión de
las venas del cuello o un componente pulmonar intensificado como segundo ruido cardíaco.
En ocasiones sobreviene bradicardia paradójica. • En los pacientes más viejos que se quejan de
molestias torácicas vagas, el diagnóstico de PE puede no ser aparente excepto cuando existan
signos de insuficiencia cardíaca derecha. Desgraciadamente, dado que los síndromes
isquémicos coronarios agudos son tan comunes, puede pasar inadvertida la posibilidad de una
PE potencialmente fatal y los pacientes son dados de alta del hospital después de excluir el
infarto de miocardio con determinaciones seriadas de enzimas cardíacas y electrocardiograma.
LA AUSENCIA DE DOLOR DESCARTA ÉSTA POSIBILIDAD DIAGNÓSTICA AUNQUE AL SER UNA
PACIENTE ANCIANA SIEMPRE DEBES TENERLA EN MENTE COMO ALTERNATIVA
DIAGNÓSTICA.

CRISIS CRISIS HIPERTENSIVA (CH): es toda elevación tensional aguda que motive una consulta
HIPERTENSIVA. médica urgente. De forma arbitraria sería una elevación grave de la presión arterial (PA)
diastólica superior a los 120 ó 130 mmHg o de sistólica por encima de 210 mmHg. Y en un
sentido estricto de la palabra, este término, tan solo indicaría la detección de una elevación de
las cifras tensionales de un sujeto por encima de aquellas que le son habituales. LA TA
DIASTÓLICA NO CUMPLE CON LOS CRITERIOS ESTABLECIDOS, QUE AUNQUE VARÍAN CON
CADA AUTOR EN NINGÚN CASO SE CONSIDERA POR DEBAJO DE 120MMHG.

Bibliografía:
1. GUÍA DE PRÁCTICA CLÍNICA, DIAGNÓSTICO Y TRATAMIENTO DE LA INSUFICIENCIA CARDÍACA AGUDA.
MÉXICO: SECRETARIA DE SALUD. 2. LONGO DL, FAUCI AS, KASPER DL, HAUSERSL, JAMESON JL,
LOSCALZOJ. HARRISON. PRINCIPIOS DE MEDICINA INTERNA, 18A EDICIÓN. MC GRAW HILL. NEW YORK,
USA. 2012, PP 1904-1905. 3. PAPADAKIS MAXINE A, MCPHEE STEPHEN J. DIAGNÓSTICO CLÍNICO Y
TRATAMIENTO. 52ª EDICIÓN. NUEVA YORK. 2013, PP 398-399. 4. DIAGNÓSTICO Y TRATAMIENTO DE LA
INSUFICIENCIA CARDÍACA AGUDA EN PACIENTES ADULTOS. EVIDENCIAS Y RECOMENDACIONES; GUÍA DE
PRÁCTICA CLÍNICA. MÉXICO, CENETEC:2018.

http://www.cenetec-difusion.com/CMGPC/SS-219-09/ER.pdf

319 - A LA PACIENTE SE LE DEBE INDICAR:

ANTIARRÍTMICOS. Un metaanálisis ha encontrado que los pacientes con ICC y arritmias ventriculares tienen
riesgo elevado de sufrir muerte súbita, siendo la ICC sintomática el mayor predictor
individual. La Amiodarona es el antiarrítmico más estudiado y ha demostrado ser muy
efectivo en suprimir las arritmias ventriculares sin deprimir la contractilidad miocárdica,
además ha reducido la mortalidad general en un 19%, siendo mayor la reducción por causa
cardiovascular, ICC y arritmias ventriculares. No obstante SÓLO DEBE PRESCRIBIRSE EN
PACIENTES QUE TIENEN ASOCIADA A LA ICC UNA TAQUIARRITMIA VENTRICULAR
EVIDENTE. Sin embargo como no existe consenso sobre el papel de los antiarrítmicos
convencionales en el tratamiento de la ICC severa, SU USO NO DEBE SER RUTINARIO. LA
PACIENTE NO CUMPLE CON LOS CRITERIOS PARA APLICACIÓN DE ANTIARRÍTMICOS.

INOTRÓPICOS. DIGOXINA: Cada vez hay menos lugar para los digitálicos y otros medicamentos inotrópicos
en el tratamiento de la ICC. Se deben prescribir para pacientes con ritmo sinusal y
sintomáticos a pesar del tratamiento estándar, así como para los que tienen asociado una
fibrilación auricular rápida, pues permiten mantener la frecuencia cardíaca dentro de límites
normales. SI BIEN LOS IECAs Y DIURÉTICOS CONSTITUYEN EL ESTÁNDAR DE TRATAMIENTO
DE LA ICC, NO SE ENCUENTRAN EN LAS OPCIONES DE RESPUESTAS, SE ELIGE A LOS
INOTRÓPICOS COMO CORRECTA AL SER LA TERAPIA CONSECUTIVA DEL TRATAMIENTO
ESTÁNDAR EN PACIENTES SINTOMÁTICOS. NOTA: Basado en los últimos lineamientos
vigentes en nuestro país, el paciente con insuficiencia cardíaca con datos de congestión
deberá ser clasificado en: paciente húmero y caliente (congestión + adecuada perfusión) y
paciente húmero y frío (congestión + mala perfusión); para los primeros se recomienda
iniciar tratamiento con diuréticos o en su defecto, hiperfiltración cuando existe resistencia a
diuréticos, mientras que, para los pacientes húmedos fríos, se recomienda tratamiento con
inotrópicos, considerando el uso de vasopresores en casos refractarios.

ANTICOAGULANTES. ANTICOAGULANTES. Los pacientes con ICC tienen un incremento del riesgo de
tromboembolismo arteriales y accidentes cerebrovasculares, ya que se ha asociado la baja
fracción de eyección ventricular con complicaciones embólicas. Los datos disponibles NO
PERMITEN ACONSEJAR EL USO DE ANTIGOAGULANTES Y ANTIAGREGANTES
PLAQUETARIOS DE FORMA RUTINARIA porque los mismos son contradictorios y no existen
ensayos clínicos diseñados que valoren la eficacia de estos medicamentos. Un enfoque
práctico sería la administración de los mismos SOLO SI COEXISTEN ENFERMEDADES
ASOCIADAS como fibrilación auricular, la acinesia marcada de la pared anterior o un trombo
ventricular detectado por ecocardiografía, en dónde está demostrada su eficacia. NO
CUMPLE CON CRITERIOS QUE JUSTIFIQUEN EL USO DE ANTICOAGULANTES.

INHIBIDORES DE DATOS IMPORTANTES DEL MANEJO ESTÁNDAR DE ICC. • IECA: Se han convertido en la
FOSFODIESTERASA. piedra angular del tratamiento de la ICC, por ello todo paciente con esta enfermedad (grado
I, II, III, y IV) debe estar tratado con estos medicamentos (recomendación A), al menos que
existan contraindicaciones para su uso. Los IECAs alcanzan todos los objetivos terapéuticos
propuestos para la ICC. • DIURÉTICOS: tienen un importante lugar para el tratamiento
sintomático de la ICC, en particular en los síntomas por retención de líquido, al reducir y
mantener la volemia. Las tiazidas (Clortalidona, Hidroclorotiazida), los diuréticos del asa
(Furosemida) y los inhibidores de la aldosterona (Espironolactona) son los más utilizados.
Generalmente estos medicamentos forman parte del tratamiento estándar de la ICC
(recomendación A) asociados a los IECAs y a otros medicamentos en pacientes sintomáticos.

Bibliografía:
1. GUÍA DE PRÁCTICA CLÍNICA, DIAGNÓSTICO Y TRATAMIENTO DE LA INSUFICIENCIA CARDÍACA AGUDA.
MÉXICO: SECRETARIA DE SALUD. 2. LONGO DL, FAUCI AS, KASPER DL, HAUSERSL, JAMESON JL,
LOSCALZOJ. HARRISON. PRINCIPIOS DE MEDICINA INTERNA, 18A EDICIÓN. MC GRAW HILL. NEW YORK,
USA. 2012, PP 1906-1913. 3. PAPADAKIS MAXINE A, MCPHEE STEPHEN J. DIAGNÓSTICO CLÍNICO Y
TRATAMIENTO. 52ª EDICIÓN. NUEVA YORK. 2013, PP 401-405. 4. DIAGNÓSTICO Y TRATAMIENTO DE LA
INSUFICIENCIA CARDÍACA AGUDA EN PACIENTES ADULTOS. EVIDENCIAS Y RECOMENDACIONES; GUÍA DE
PRÁCTICA CLÍNICA. MÉXICO, CENETEC: 2018.

http://www.cenetec-difusion.com/CMGPC/SS-219-09/ER.pdf

FIN DEL CASO CLÍNICO SERIADO


ANÁLISIS DEL CASO CLÍNICO

IDENTIFICACIÓN DEL REACTIVO


Area: PEDIATRÍA
Especialidad: URGENCIAS PEDIÁTRICAS
Tema: URGENCIAS CARDIOVASCULARES PEDIÁTRICAS
Subtema: FIEBRE REUMÁTICA

CASO CLÍNICO SERIADO

ADOLESCENTE FEMENINA DE 13 AÑOS DE EDAD, CON HISTORIA CLÍNICA DE FARINGOAMIGDALITIS DE


REPETICIÓN, PADRE CON ANTECEDENTE DE CARDIOPATÍA REUMÁTICA. DESDE HACE 2 SEMANAS PRESENTA
POLIARTRITIS QUE INICIA EN MUÑECA DERECHA QUE TRAS SU RESOLUCIÓN ESPONTÁNEA SE PRESENTA EN
RODILLA CONTRALATERAL Y TOBILLOS; DESDE ESTA MAÑANA LA MADRE HA NOTADO MOVIMIENTOS
INVOLUNTARIOS EN BOCA Y MANOS QUE DESAPARECEN DURANTE EL SUEÑO, LOS CUALES SE HAN IDO
INCREMENTANDO EN HEMICUERPO DERECHO, DIFICULTANDO LA MARCHA Y LA COMUNICACIÓN. SE
SOSPECHA QUE SE TRATA DE FIEBRE REUMÁTICA. SE TOMA EXUDADO FARÍNGEO COMO PARTE DEL
PROTOCOLO.

Adolescente femenina (es más común que


presenten corea)

Faringoamidalitis de repetición, padre con


cardiopatía reumática.

POLIARTRITIS ASIMÉTRICA MIGRATORIA


(CARACTERÍSTICA), movimientos
involuntarios en boca y manos (corea).

Se toma exudado faríngeo.

320 - EL TIPO DE MEDIO EN EL QUE DEBE CULTIVARSE LA MUESTRA DEL EXUDADO ES EN:

AGAR EL ESTÁNDAR DE ORO PARA EL DIAGNÓSTICO DE ESTREPTOCOCO BETA HEMOLÍTICO DEL GRUPO A ES
SANGRE. EL CULTIVO FARÍNGEO EN PLACA DE AGAR SANGRE.

THAYER Toda secreción vaginal se debe cultivar en medios habituales para cualquier bacteria inespecífica sin
MARTIN. embargo, hay algunas bacterias que requieren cultivos especiales como la Neisseria Gonorrhoae que se
cultiva en medio Thayer-Martin, la Gardnerella en medio Cassman, la cándida en médio de Saboraud y
a Clamydia en cultivo de células de McCoy. EL CULTIVO DE THAYER MARTIN ES DE UTILIDAD PARA LA
SIEMBRA DE NEISSERIA GONORRHOAE.
AGAR En la actualidad el medio Moredum, de agar carbón que contiene ciclohexamina y espectinomicina, es
CARBÓN. altamente específico de Bordetella parapertussis; se debe considerar que este medio inhibe el
crecimiento de Bordetella Pertussis. EL AGAR CARBÓN ES IDEAL PARA LA SIEMBRE DE BORDETELLA
PARAPERTUSSIS.

CARY El CLARY BLAIR es un MEDIO DE TRANSPORTE uno propiamente un medio de siembra, está
BLAIR. recomendado enviar la muestra de exudado faríngeo en medio este medio de transporte para siembra
de agar de cordero al 5% preferentemente en las primeras dos horas de la toma de muestra y antes de
las 24hrs.

Bibliografía:
1. GUÍA DE PRÁCTICA CLÍNICA, PREVENCIÓN Y DIAGNÓSTICO OPORTUNO DE FIEBRE REUMÁTICA.
MÉXICO: SECRETARIA DE SALUD; 2008. 2. GONZÁLEZ N, TORALES A. GÓMEZ D. INFECTOLOGÍA CLÍNICA
PEDIÁTRICA, 8A EDICIÓN. MC GRAW HILL. MÉXICO, 2011.

http://www.cenetec.salud.gob.mx/descargas/gpc/CatalogoMaestro/149_GPC_FIEBRE_REUMATICA/SSA_149_08_EyR_Fiebre_reumatica.pdf

321 - CORRESPONDE AL MEDICAMENTO DE ELECCIÓN PARA EL MANEJO NEUROLÓGICO DE ESTA


PACIENTE :

LAMOTRIGINA. La FIEBRE REUMÁTICA es una enfermedad inflamatoria, causada por una reacción
inmunológica previa a una infección faríngea por Estreptococo beta hemolítico del grupo
A que afecta principalmente el corazón, articulaciones, piel, tejido celular subcutáneo y
sistema nervioso central. Su complicación más seria, la cardiopatía reumática, puede
resultar como consecuencia, una vez resuelto el episodio agudo.

DIFENILHIDANTOÍNA. La identificación de la Corea corresponde a la presencia de movimientos involuntarios


incoordinados especialmente en manos, pies, lengua y cara, que desaparecen con el sueño
y pueden afectar un solo lado del cuerpo (hemicorea). La corea asociada a la fiebre
reumática afecta con mayor frecuencia a las mujeres adolescentes.

ÁCIDO VALPRÓICO. Un dato importante es que la Corea se asocia a carditis en un 71% de los casos por lo que
es necesario solicitar electrocardiograma y ecocardiograma en todos aquellos pacientes
que la presentan. EL ÁCIDO VALPRÓICO ESTÁ INDICADO EN CASOS REFRACTARIOS TRAS
LA ADMINISTRACIÓN DE CARBAMAZEPINA.

CARBAMAZEPINA. EL MEDICAMENTO INICIAL PARA LA COREA SEVERA ES LA CARBAMAZEPINA. En casos


refractarios se recomienda el ácido valpróico. La inmunoglobulina intravenosa es
recomendada para la corea severa refractaria por su recuperación más rápida; sin
embargo, no reduce la incidencia de enfermedad valvular a largo plazo.

Bibliografía:
1. GUÍA DE PRÁCTICA CLÍNICA, PREVENCIÓN Y DIAGNÓSTICO OPORTUNO DE FIEBRE REUMÁTICA.
MÉXICO: SECRETARIA DE SALUD; 2008.

http://www.cenetec.salud.gob.mx/descargas/gpc/CatalogoMaestro/149_GPC_FIEBRE_REUMATICA/SSA_149_08_EyR_Fiebre_reumatica.pdf

FIN DEL CASO CLÍNICO SERIADO


ANÁLISIS DEL CASO CLÍNICO

IDENTIFICACIÓN DEL REACTIVO


Area: CIRUGÍA
Especialidad: CIRUGÍA ABDOMINAL
Tema: PATOLOGÍA ABDOMINAL RESOLUCIÓN QUIRÚRGICA DE URGENCIA
Subtema: OCLUSIÓN INTESTINAL

CASO CLÍNICO SERIADO

HOMBRE DE 53 AÑOS CON ANTECEDENTE DE CIRUGÍA POR APENDICITIS AGUDA PERFORADA HACE 3 AÑOS.
ACUDE A URGENCIAS POR DOLOR ABDOMINAL DE 36 HORAS DE EVOLUCIÓN, INTENSO, GENERALIZADO Y
ACOMPAÑADO DE VÓMITOS DE COLOR VERDE. SE ENCUENTRA DESHIDRATADO, EL ABDOMEN DISTENDIDO,
DOLOROSO A LA PALPACIÓN. SE AUSCULTAN RUIDOS INTESTINALES DE LUCHA. ACUDE CON FACULTATIVO
QUIEN INDICA ANALGÉSICO Y ANTIBIÓTICO.

masculino de 53 años de edad.

apendicectomía por apendicitis perforada


hace 3 años.

dolor abdominal de 36 hrs de evoluciÓn,


intenso, generalizado, acompañado de
vómitos gastrobiliares.

deshidratación, abdomen distendido,


doloroso a la palpación. ruidos intestinales
de lucha.

-.

322 - EL DIAGNÓSTICO CLÍNICO MAS PROBABLE ES:


PERFORACIÓN La PERORACIÓN DIVERTICULAR se produce por una micro o macro perforación de un divertículo,
DIVERTICULAR. la cual puede ser simple o complicada, produciendo inflamación (Diverticulitis), perforación libre,
abscesos, peritonitis generalizada, obstrucción, fístulas o sangrado. Cuando se ocluye el cuello
del divertículo se produce un aumento de la presión de la luz y sobreinfección, que causa la
perforación intestinal, que lleva a la diverticulitis aguda. La Diverticulosis sintomática se
caracteriza por episodios de dolor abdominal inespecífico, localizado en hemiabdomen izquierdo
(fosa iliaca izquierda) que es típicamente cólico, pero puede ser continuo y a menudo es aliviado
con la defecación o la expulsión de gases. Los episodios de dolor abdominal pueden ser
desencadenados por la ingesta. Puede ir acompañado de distensión abdominal, fiebre,
leucocitosis, náuseas, vómitos y alteración del hábito intestinal, generalmente estreñimiento,
aunque puede presentar episodios de diarrea. El dolor agudo, estará en relación con la magnitud
del proceso inflamatorio, plastrón o absceso peridiverticular. El examen físico puede revelar
empastamiento o masa palpable, dolor a la palpación sin defensa, ni signos de reacción
peritoneal. La maniobra de San Martino o tacto rectal es importante pudiendo revelar dolor en
fondo de saco de Douglas o masa palpable. La perforación comprende a la presencia de
abscesos, peritonitis o retro peritonitis, como consecuencia de una perforación diverticular libre
o insuficientemente bloqueada. La perforación libre en la cavidad peritoneal origina una
peritonitis aguda difusa grave. En la Rx de pie se evidencia el neumoperitoneo. La
retroperitonitis difusa por perforación retroperitoneal es una complicación grave y de una
mortalidad muy elevada. En toda perforación diverticular se forma al comienzo un pequeño
absceso peridiverticular que luego se reabsorbe o se drena espontáneamente en el intestino. En
ocasiones, como consecuencia de un bloqueo insuficiente de la infección local, pueden formarse
colecciones purulentas voluminosas (abscesos) en diferentes localizaciones. Las localizaciones
más frecuentes del absceso intraperitoneal son fosa iliaca izquierda, Douglas, mesocolon y
espacio subfrénico, aunque también puede ser retroperitoneal y exteriorizarse por la fosa
lumbar. La cirugía urgente está indicada en la peritonitis y retroperitonitis difusas, abscesos intra
y retroperitoneales, la hemorragia incoercible y la obstrucción colónica completa. La perforación
acompañada de peritonitis purulenta o estercorácea es una emergencia quirúrgica, que requiere
un tratamiento precoz una vez lograda la resucitación del enfermo, con aseo peritoneal y
resección del segmento comprometido para luego practicar una colostomía. La operación más
utilizada en esta instancia es la operación de Hartmann, que consiste en la resección del
sigmoides, cierre del recto y abocamiento del colon proximal como una colostomía. La simple
colostomía sin resección del segmento comprometido debe ser evitada a toda costa ya que
conlleva una mortalidad muy superior a la de la resección debido a la persistencia de la
contaminación peritoneal por el segmento afectado. EL PACIENTE CON PERFORACIÓN
DIVERTICULAR PRESENTA "DATOS DE IRRITACIÓN PERITONEAL" Y EN OCASIONES SE PALPA
TUMORACIÓN RELACIONADA A PLASTRÓN; ESTOS SIGNOS NO ESTÁN PRESENTES EN NUESTRO
PACIENTE.

ENFERMEDAD El término ENFERMEDAD INFLAMATORIA INTESTINAL (EII) incluye dos trastornos de etiología
INFLAMATORIA desconocida, la colitis ulcerosa (CU) y la enfermedad de Crohn (EC), que se definen según
INTESTINAL. criterios clínicos, radiológicos, endoscópicos e histológicos. Ambas cursan de forma crónica con
brotes de actividad inflamatoria. La CU afecta exclusivamente a la mucosa del colon en extensión
variable, generalmente de predominio distal, de forma continua. La EC es un trastorno
inflamatorio crónico transmural que puede afectar cualquier segmento del tracto
gastrointestinal desde la boca hasta el ano con carácter discontinuo. En los casos en que es
imposible diferenciar entre CU y EC se habla de colitis indeterminada (CI). Los síntomas más
frecuentes de la colitis ulcerosa. Las manifestaciones clínicas son: diarrea líquida y
frecuentemente sanguinolenta o con moco asociada a urgencia, tenesmo, dolor abdominal
cólico, fiebre y pérdida de peso. Se consideran signos graves: fiebre elevada, afectación del
estado general, retraso del crecimiento, distensión abdominal, timpanismo y dolor a la palpación
abdominal. Puede manifestarse como una colitis fulminante con más de 6 deposiciones al día,
con sangre abundante, anemia, hipoalbuminemia, alteraciones hidroelectrolíticas, fiebre y
taquicardia. El abdomen puede estar distendido y doloroso a la palpación. Estas formas pueden
evolucionar a megacolon tóxico con dilatación colónica superior a 6 cm asociada a cuadro
séptico, gran distensión y dolor abdominal. Puede producirse como complicación una colitis
fulminante, tras una colonoscopia o enema de bario, o por uso de anticolinérgicos o por retirada
rápida de corticoides. Tanto la colitis grave como el megacolon tóxico presentan un alto riesgo
de perforación, sepsis y hemorragia masiva. La complicación más grave y frecuente en la CU de
larga evolución es el cáncer. ESTE TIPO DE ENFERMEDADES TIENEN UNA EVOLUCIÓN CRÓNICA,
LO CUAL NO CORRESPONDE AL CASO CLÍNICO.
OBSTRUCCIÓN La OCLUSIÓN INTESTINAL se define como la interrupción del tránsito intestinal, lo que impide
INTESTINAL expulsar gases y heces por el recto. EL bloqueo de la luz intestinal se da en su mayoría por
POR factores extrínsecos (adherencias, hernia inguinal, hernia de pared estrangulada) y en pocas
ADHERENCIAS. ocasiones por obstrucción de la luz. La OBSTRUCCIÓN INTESTINAL POR ADHERENCIAS
intraperitoneales puede ser parcial o total, dependiendo del grado de extensión y rigidez de las
mismas, siendo éstas la causa más frecuente de obstrucción Las bridas pueden obstruir un asa
intestinal por angulación, por torsión o, la más frecuente, por constricción. La obstrucción
intestinal por bridas se presenta en un lapso variable después de la cirugía previa. Presencia de
dolor abdominal tipo cólico, vómitos, distensión abdominal y no expulsión de heces ni gases.
Esto puede variar en orden de aparición e intensidad en dependencia del nivel de la oclusión. Se
deberá precisar la presencia de cicatrices quirúrgicas abdominales y examinar los orificios
herniarios. La auscultación abdominal es de gran importancia. Puede encontrarse aumento de los
ruidos hidroaéreos en las oclusiones mecánicas, que son susceptibles de estar disminuidos o
ausentes en el íleo-paralítico adinámico y en las fases terminales o de adinamia del íleo
mecánico. El tacto rectal es obligatorio. Las ayudas diagnósticas están dirigidas a detectar la
presencia de obstrucción intestinal, su nivel y severidad. La radiografía simple de abdomen y el
tránsito intestinal detectan la presencia de obstrucción aunque sin definir en forma exacta su
causa. La TAC abdominal además de brindar la información sobre la presencia de obstrucción
mecánica puede descartar las otras causas menos frecuentes de obstrucción, tales como tumores,
hernias, etc. La laparoscopia es un procedimiento quirúrgico tanto diagnóstico como terapéutico
para las adherencias intraperitoneales Su eficacia diagnóstica no tiene discusión, pero como
tratamiento tiene una alta tasa de recidiva. LAS ADHERENCIAS INTRAABDOMINALES POR
INTERVENCIONES QUIRÚRGICAS PREVIAS, SOBRE TODO AQUELLAS QUE REQUIRIERON LAVADO
PERITONEAL (COMO APENDICITIS PERFORADA) REPRESENTA EL 75% DE LOS CASOS DE
OBSTRUCCIÓN INTESTINAL. OJO: EN REACTIVOS COMO ESTE DEBES TENER MUCHO CUIDADO
EN LOS ANTECEDENTES Y LIGARLOS AL PADECIMIENTO ACTUAL.

VÓLVULUS DE EL VÓLVULUS DE CIEGO SUELE OBSERVARSE EN PACIENTES MÁS JÓVENES y causa menos de
CIEGO. 20% de los vólvulos de colón. Se debe a una fijación anormal del colón derecho al retroperitoneo
que origina que el ciego se mueva libremente. El síntoma más común de vólvulo cecal es el dolor
abdominal. También se observan náuseas y vómitos, obstipación y diarrea. Clínicamente el
paciente parece tener una obstrucción del intestino delgado. Un enema con contraste
hidrosoluble puede mostrar obstrucción de la columna a nivel del vólvulo. El tratamiento es la
destorsión quirúrgica sola o con cecopexia o cecostomía.

Bibliografía:
1. GUÍA DE PRÁCTICA CLÍNICA, TRATAMIENTO QUIRÚRGICO DE OCLUSIÓN INTESTINAL POR ADHERENCIAS
POSTQUIRÚRGICAS EN EL ADULO EN SEGUNDO NIVEL DE ATENCIÓN. MÉXICO: SECRETARIA DE SALUD;
2012. RECUPERADO DE
HTTP://WWW.CENETEC.SALUD.GOB.MX/CONTENIDOS/GPC/CATALOGOMAESTROGPC.HTML 2.
BRUNICARDI F, ANDERSEN D, BILLIAR T, Y COLS. SCHWARTZ PRINCIPIOS DE CIRUGÍA, 9A EDICIÓN. MC
GRAW HILL. 2011, PP 988-992.

http://www.cenetec.salud.gob.mx /descargas/gpc/CatalogoMaestro/359_GPC_OclusixnIntestinal/GPC_EYR_OCLUSIxN_INTESTINAL.pdf

323 - PARA CORREGIR LA DESHIDRATACIÓN SE INDICA CARGA CON HARTMAN PARA POSTERIORMENTE
INDICAR:

ANTIBIÓTICOS Y Los ANTIBIÓTICOS son útiles en algunas formas clínicas de la enfermedad inflamatoria
ANTIINFLAMATORIOS. intestinal (EII), porque pueden disminuir la inflamación crónica de la mucosa intestinal al
disminuir la concentración luminal de las bacterias y modular la respuesta inmune
mediada por células. La antibioterapia parenteral de amplio espectro esta indicada en
caso de colitis severa, megacolon tóxico y complicaciones intestinales de la EC, como los
abscesos. Los corticoesteroides fueron los primeros fármacos en utilizarse ampliamente
como tratamiento primario de la EII. Los mayormente utilizados son la prednisona y la
metilprednisolona, que están disponibles para ser administrados por vía oral o
intravenosa (acción sistémica) y la budesonida que en nuestro medio esta disponible en
cápsulas y enemas (acción local). Los corticoesteroides sistémicos son muy efectivos para
controlar la enfermedad activa, induciendo la remisión en un 60-90% de los casos. NO
ESTÁN INDICADAS EN ESTE MOMENTO, PUEDEN SER DE UTILIDAD EN ALGUNAS
FORMAS AGUDAS DE ENFERMEDAD INFLAMATORIA INTESTINAL. ADEMÁS SI PONES
ATENCIÓN EN EL CASO CLÍNICO, YA FUERON INDICADOS.
DESCOMPRESIÓN La primera medida después del ABC de la reanimación se debe de colocar una sonda
NASOGÁSTRICA. nasogástrica para descompresión. En las oclusiones de asas delgadas: sin resección
intestinal, la descompresión se efectuará por sonda nasogástrica, haciéndola progresar, en
caso de ser posible, hasta el intestino delgado para facilitar su aspiración. Debe evitarse
en lo posible la enterotomía descompresiva. En las oclusiones de colon (resueltas por
sección delas bridas, quelotomías o destorsión de vólvulos): se realizará la descompresión
en el acto quirúrgico, mediante una sonda colocada en el recto. En las oclusiones resueltas
por resección y anastomosis primaria, se llevará a cabo la descompresión antes de la
anastomosis. En caso de que el tratamiento conservador no sea exitoso, el paciente
deberá llevarse a una laparotomía para adherenciolisis quirúrgica y medidas de
prevención para evitar las adherencias recidivantes. ES NECESARIO EXTRAER DE FORMA
CONSTANTE EL AIRE DEL INTESTINO A TRAVÉS DE UNA SONDA NASOGÁSTRICA, DE ESTE
MODO SE LOGRA DISMINUIR LAS NÁUSEAS, EL DOLOR Y EL RIESGO DE ASPIRACIÓN POR
VÓMITO.

CIRUGÍA URGENTE. En los pacientes con perforación requieren de una cirugía urgente, esta indicada en la
peritonitis y retroperitonitis difusas, abscesos intra y retroperitoneales, la hemorragia
incoercible y la obstrucción colónica completa. Son datos que sugieren que el manejo
conservador no es suficiente para el manejo de los pacientes con oclusión intestinal:
persiste el íleo mecánico intestinal por más de 3 días, si el drenaje por sonda al tercer día
es mayor a 500 ml, en mayores de 40 años, adherencia compleja, si hay obstrucción
completa del intestino delgado (sin evidencia de aire en el intestino grueso), y CPK (mayor
o igual a 130UI/L). Son indicadores directos de cirugía: - Todo paciente con presencia de
líquido intraperitoneal, edema del mesenterio y signo de las heces en intestino delgado
y/o intestino desvascularizado en una tomografía de abdomen indica que el paciente es
candidato a laparotomía exploradora. - Dolor abdominal en escala analógica visual mayor
a 4, resistencia vascular, leucocitos de >10,000, proteína C reactiva 75mg/L predicen la
necesidad de laparatomía de emergencia al momento de la admisión. - Si el cuadro no se
ha resuelto con medidas conservadoras por 48 a 72 hrs, a pesar de utilizar medio de
contraste hidrosoluble. LAS ADHERENCIAS PUEDEN LIBERARSE MEDIANTE CIRUGÍA
LAPAROSCÓPICA O ABIERTA, ÉSTO EN CASO DE NO MEJORAR CON EL MANEJO MÉDICO
CONSERVADOR. LA PACIENTE NO TIENE DATOS QUE INDIQUEN CIRUGÍA DIRECTA EN
ESTE MOMENTO.

DESVOLVULACIÓN. En las formas subagudas y agudas se propicia la destorsión colonoscópica que ha sido
publicado por Anderson y col. Cuando el paciente debe ser intervenido quirúrgicamente
se seguirán las normas preoperatorias establecidas para los enfermos ocluidos. Los pasos
a seguir durante la intervención son: desvolvulación de las asas comprometidas,
valoración de los segmentos isquémicos y análisis de la viabilidad de los mismos. Si el
intestino comprometido está viable se prefiere la desvolvulación y la fijación del asa a la
pared abdominal. Este tipo tratamiento tiene una mortalidad muy baja y una recidiva que
oscila entre el 5 y el 20%, pero si al mismo tiempo se efectúa una cecostomía, la recidiva
es mínima. CORRESPONDE AL TRATAMIENTO DE ELECCIÓN EN CASO DE VÓLVULUS DE
CIEGO.

Bibliografía:
1. GUÍA DE PRÁCTICA CLÍNICA, TRATAMIENTO QUIRÚRGICO DE OCLUSIÓN INTESTINAL POR ADHERENCIAS
POSTQUIRÚRGICAS EN EL ADULO EN SEGUNDO NIVEL DE ATENCIÓN. MÉXICO: SECRETARIA DE SALUD;
2012. RECUPERADO DE
HTTP://WWW.CENETEC.SALUD.GOB.MX/CONTENIDOS/GPC/CATALOGOMAESTROGPC.HTML 2.
BRUNICARDI F, ANDERSEN D, BILLIAR T, Y COLS. SCHWARTZ PRINCIPIOS DE CIRUGÍA, 9A EDICIÓN. MC
GRAW HILL. 2011, PP 988-992.

http://www.cenetec.salud.gob.mx /descargas/gpc/CatalogoMaestro/359_GPC_OclusixnIntestinal/GPC_EYR_OCLUSIxN_INTESTINAL.pdf

FIN DEL CASO CLÍNICO SERIADO


ANÁLISIS DEL CASO CLÍNICO

IDENTIFICACIÓN DEL REACTIVO


Area: MEDICINA INTERNA
Especialidad: DERMATOLOGÍA
Tema: SÍNDROME PRURIGINOSO
Subtema: TIÑAS

CASO CLÍNICO CON UNA PREGUNTA

MUJER DE 38 AÑOS CON DIAGNÓSTICO ACTUAL DE TIÑA CORPORIS Y PEDIS. DESEA SABER LA MEDIDA DE
PREVENCIÓN MÁS EFICAZ PARA EVITAR CONTAGIAR A SUS FAMILIARES.

femenina de 38 años de edad.

diagnóstico de tiña corporis y pediS. quiere


conocer la medidas de prevención para
evitar el contagio.

-.

-.

-.

324 - USTED LE INDICARÁ EN SUS FAMILIARES EL USO DE:

QUERATOLÍTICOS. REPASO. La TIÑA es una infección de la piel, piel cabelluda, uñas o pelo causada por hongos
dermatofíticos que invaden el estrato corneo y usan la queratina como nutriente. Existen tres
géneros de dermatofitos: Trichophyton, Epidermophyton y Microspurum. Los sitios de
formación de artroconidia, los cuerpos que forman esporas de los dermatofitos, clasifican las
especies que causan tiña capitis. Especies de ectotrix forman conidia alrededor del eje del pelo
y debajo su cutícula. Especies de endotrix presentan artroesporas en el eje del pelo y las
especies fávicas tienen hifas en paralelo dentro y al rededor del eje del pelo.
JABONES REPASO. TIÑA CORPORIS: inicialmente se presenta como pápulas brillantes, rojas que se
LIGEROS. diseminan formando lesiones anulares, pruriginosas; el centro suele ser claro para
hiperpigmentarse cuando la lesión avanza. TIÑA CRURIS: se manifiesta como una lesión con
márgenes eritematosos y delgados marcadamente delimitados y pruriginosos. TIÑA PEDIS:
frecuentemente se presenta como un tejido macerado que afecta plantas y parte lateral de los
pies con un componente hiperqueratólico. TIÑA UNGUEAL: afecta las placas ungueales y son
muy resistentes al tratamiento. La uña se torna distrófica, engrosada, decolorada y quebradiza.
El término onicomicosis se refiere a cualquier infección micótica de las uñas.

HUMECTANTES. • Las micosis cutáneas se suelen producir en presencia de factores predisponentes que facilitan
el desarrollo y el crecimiento de las colonias de hongos. • Es importante evitar la aparición de
estos factores. Estas medidas serán preventivas si nos encontramos en ausencia de clínica o
coadyuvantes al tratamiento específico cuando hay enfermedad. • Las medidas más
importantes que se deben tomar para prevenir las dermatomicosis son: buena higiene
personal, uso de prendas de vestir transpirables, sobre todo el calzado, y protección de las
manos y pies en actividades laborales y deportivas. AL CONTRARIO, LOS MEDIOS HÚMEDOS
FAVORECEN LA TRANSMISIÓN DE LOS HONGOS.

SECANTES. El tratamiento tópico inespecífico de la tiña son productos astringentes, antisépticos y


secantes. • Se utilizan como coadyuvantes en las micosis cutáneas con inflamación intensa y/o
exudación. Su acción antifúngica es la suma de las acciones desinfectantes, antiexudativa y
queratolítica. Su principal ventaja es su bajo costo. • Su uso se restringe a casos aislados, y los
más utilizados son el permanganato potásico en concentraciones entre 1/8.000 y 1/10.000 (es
el que tiene mayor actividad antifúngica), el sulfato de cinc y el sulfato de cobre. LOS
PRODUCTOS ASTRINGENTES CIERRAN LOS POROS, SECAN LA PIEL Y LA DESINFLAMAN CON
LO QUE FAVORECEN AMBIENTES POCO PROPICIOS PARA LAS MICOSIS.

Bibliografía:
1. HEYMANN, DAVID L. EL CONTROL DE LAS ENFERMEDADES TRANSMISIBLES, 19A EDICIÓN. OPS.
WASHINGTON, D.C. 2011, PP 130-131.
ANÁLISIS DEL CASO CLÍNICO

IDENTIFICACIÓN DEL REACTIVO


Area: CIRUGÍA
Especialidad: OTORRINOLARINGOLOGÍA
Tema: PATOLOGÍA DEL OÍDO
Subtema: OTITIS MEDIA AGUDA

CASO CLÍNICO CON UNA PREGUNTA

HOMBRE DE 26 AÑOS DE EDAD, QUE ACUDE A SU CONSULTA CON OTALGIA DERECHA DE 2 DÍAS DE
EVOLUCIÓN.

-.

-.

otalgia DE 2 DÍAS DE EVOLUCIÓN.

-.

-.

325 - DURANTE LA EXPLORACIÓN, EL DIAGNÓSTICO DE OTITIS MEDIA CRÓNICA SE PODRÁ CONFIRMAR


SI SE ENCUENTRA:

OTORREA La OTITIS EXTERNA AGUDA es la inflamación difusa del canal auditivo externo que puede
CON involucrar el pabellón auricular o la membrana timpánica, ocasionada por una infección
MEMBRANA secundaria a la ruptura de la barrera mecánica que proporciona el cerumen en presencia del
TIMPÁNICA aumento de la temperatura y humedad. Se caracteriza por el inicio rápido de la sintomatología
INTEGRA. (generalmente menos de 48 horas). Los síntomas de inflamación del conducto auditivo son:
otalgia severa con irradiación temporomandibular y craneofacial (puede ocurrir también dolor
auricular intenso a la más leve manipulación de la zona o durante la masticación), prurito ótico,
plenitud ótica, dolor mandibular y disminución auditiva. Los signos que se encuentran a la
exploración son: hipersensibilidad del trago, pabellón auricular o ambos (signo del trago positivo);
eritema y edema difuso del conducto auditivo, otorrea (puede o no estar presente), eritema de
membrana timpánica, celulitis del pabellón auricular y piel adyacente así como linfadenitis
regional. LA PRESENCIA DE OTORREA EN EL CONDUCTO AUDITIVO EXTERNO CON MEMBRANA
ÍNTEGRA DIRIGE EL DIAGNÓSTICO DE OTITIS EXTERNA AGUDA.
MEMBRANA La OTITIS MEDIA AGUDA se define como la presencia sintomática de exudado (generalmente,
TIMPÁNICA pero no obligadamente, purulento) en el oído medio de presencia aguda (24-48 horas). Los
ABULTADA E síntomas más característicos son: otalgia, fiebre e irritabilidad, otorrea; y/o alguna de las
HIPERÉMICA. siguientes alteraciones de la membrana timpánica: inflamación, engrosamiento y/o
abombamiento, opacidad, presencia de bulas, depósito de fibrina, coloración blanco amarillenta y
ausencia de movimiento a la neumo-otoscopia. EL ABULTAMIENTO DE MEMBRANA TIMPÁNICA
ES UN FRANCO SIGNO DE PRESENCIA DE INFLAMACIÓN DEL OÍDO MEDIO Y MIENTRAS NO SE
PERFORE LA MEMBRANA O TENGA MENOS DE 3 MESES DE EVOLUCIÓN SE CONSIDERA AGUDA.

PERFORACIÓN La OTITIS MEDIA CRÓNICA supurada se define como un exudado persistente o intermitente a
TIMPÁNICA través de una membrana timpánica que no está intacta (es decir, perforación o tubo de
CON timpanostomía). La perforación crónica de la membrana timpánica puede ocurrir sin supuración y
OTORREA. con frecuencia se denomina otitis media crónica inactiva. Se caracteriza por inflamación crónica
(más de 3 meses) de la mucosa del oído medio y mastoides, perforación de la membrana
timpánica y otorrea; en muchos de los casos el paciente refiere hipoacusia. LA PRESENCIA DE
PERFORACIÓN TIMPÁNICA DIRIGE EL DIAGNÓSTICO DE OTITIS MEDIA CRÓNICA. RECUERDA QUE
EL RESTO DE SÍNTOMAS EN LA OTITIS MEDIA PUEDEN ESTAR PRESENTES INDEPENDIENTEMENTE
DEL TIEMPO DE EVOLUCIÓN. IMPORTANTE: aún no se ha revisado al paciente y los antecedentes
son escasos, o más bien nulos. Si bien a primera vista no pareciera un cuadro crónico, la pregunta
se dirige específicamente a la característica de una OTITIS CRÓNICA. Es común encontrar reactivos
así en ENARM por lo que te recomiendo leas atentamente las preguntas para no errar en tu
respuesta.

ERITEMA DEL El ERITEMA SIMPLE DE CONDUCTO AUDITIVO EXTERNO dirige el diagnóstico hacia una otitis
CONDUCTO externa aguda.
AUDITIVO.

Bibliografía:
1. GUÍA DE PRÁCTICA CLÍNICA, DIAGNÓSTICO Y TRATAMIENTO DE LA OTITIS EXTERNA AGUDA EN
ADULTOS. MÉXICO: SECRETARIA DE SALUD; 2010. RECUPERADO DE
HTTP://WWW.CENETEC.SALUD.GOB.MX/CONTENIDOS/GPC/CATALOGOMAESTROGPC.HTML 2. LALWANI K,
DIAGNÓSTICO Y TRATAMIENTO EN OTORRINOLARINGOLOGÍA. CIRUGÍA DE CABEZA Y CUELLO, 2A
EDICIÓN. MC GRAWHILL LANGE. 2008, PP 660-661.

http://www.cenetec.salud.gob.mx/descargas/gpc/CatalogoMaestro/443_GPC_Otitis_externa_aguda/GER_Otitis_externa_aguda.pdf
ANÁLISIS DEL CASO CLÍNICO

IDENTIFICACIÓN DEL REACTIVO


Area: PEDIATRÍA
Especialidad: INFECTOLOGIA PEDIÁTRICA
Tema: GASTROENTERITIS INFECCIOSA
Subtema: GEPI VIRAL

CASO CLÍNICO SERIADO

PREESCOLAR DE 3 AÑOS DE EDAD, INICIA SU PADECIMIENTO HACE 12 HRS CON EVACUACIONES DIARRÉICAS
EN NÚMERO DE 3 Y DOLOR ABDOMINAL. LA MADRE LE AUTOMEDICA TRIMETROPRIM HACE 2 HORAS. A LA
EXPLORACIÓN SE ENCUENTRA AFEBRIL, ACTIVO, REACTIVO, MUCOSA ORAL SECA, ABDOMEN BLANDO,
PERISTALSIS AUMENTADA, LLENADO CAPILAR DISTAL INMEDIATO. SE INDICA PLAN A DE HIDRATACIÓN ORAL
Y MEDIDAS HIGIÉNICO DIETÉTICAS. SE LE EXPLICA A LA MADRE QUE NO ES NECESARIO EL USO DE
ANTIBIÓTICOS YA QUE SEGURAMENTE LA ETIOLOGÍA EN ESTE CASO ES VIRAL.

preescolar 3 años.

automediación de trimetropim

3 evacuaciones diarreicas en 12 horas.

deshidratación leve.

plan A de hidratación oral, se sospecha


etiología viral.

326 - LA FRECUENCIA DE GASTROENTERITIS DE ETIOLOGÍA VIRAL A ESTA EDAD SE ESTIMA EN:

20 A La DIARREA es una alteración en el movimiento característico del intestino con un incremento en el


30%. contenido de agua, volumen o frecuencia de las evacuaciones. Una disminución de la consistencia líquida o
blanda y un incremento de la frecuencia de los movimientos intestinales con mayor o igual a tres
evacuaciones en un día. Clínicamente se define a la DIARREA AGUDA como la disminución en la consistencia
de las evacuaciones (líquidas o semilíquidas) o el incremento de la frecuencia de las mismas (más de 3 en
24hrs), que puede acompañarse de fiebre o vómitos, su duración es por lo general menos de 7 días y no
mayor de 14 días. OJO: No se considera diarrea a las heces de consistencia sólida, ni a las de menor
consistencia o pastosas en bebés que son amamantados.

40 A La diarrea es un síntoma de infección, ocasionada por muy diversos organismos víricos, bacterianos y
50%. parásitos, la mayoría de los cuales se transmiten por agua contaminada y, que generalmente se asocia
frecuentemente con náuseas, vómito y cólico abdominal.
70 A Los agentes infecciosos son la causa más común de gastroenteritis aguda. LOS VIRUS, PRINCIPALMENTE
80%. ESPECIES DE ROTAVIRUS, SON RESPONSABLES DEL 70 A 80% DE LOS CASOS DE DIARREA INFECCIOSA EN
EL MUNDO DESARROLLADO. ALGUNAS BIBLIOGRAFÍAS CONTEMPLAN PORCENTAJES MENORES, PERO EN
TODOS LOS CASOS SE CONSIDERA LA ETIOLOGÍA VIRAL POR ARRIBA DEL 50% DE LOS CASOS. Los
ROTAVIRUS se encuentran dentro de los agentes causales más frecuentes de diarrea grave en niños
pequeños en todo el mundo según la OMS. RESUMEN DE INFECCIONES POR ROTAVIRUS. ETIOLOGÍA:
rotavirus de ARN segmentado, de cadena doble, de la familia Rotaviridae, con menos al menos 7 grupos
antigénicos diferenciados (A a G). Los virus del grupo A son los de mayor presencia a nivel mundial.
EPIDEMIOLOGÍA: se ha modificado poco a poco con la aplicación de la vacuna. Anteriormente, en nivel
mundial, se consideraba la segunda causa de muerte en menores de 5 años y, la principal causa de en
promedio 3 episodios de diarrea en este grupo de edad en países de ingresos bajos, hasta el 2017, según la
OMS. En México, se observó una reducción de hasta el 50% del número de muertes por diarrea de niños
menores de 5 años de edad atribuido directamente al uso de la vacuna. El virus se encuentra presente en
alta titulación en las heces de pacientes infectados varios días antes y varios días después de la aparición de
la enfermedad clínica. La transmisión es fecal-oral, además, se ha reconocido la transmisión por fómites
como un posible mecanismo de transmisión. La transmisión a través de agua contaminada o alimentos se ha
reportado en brotes con poca frecuencia. En climas templados el virus es más prevalente durante los meses
más fríos. El periodo de incubación oscila entre 1 a 3 días. MANIFESTACIONES CLÍNICAS: inicio agudo
manifestado por fiebre y vómitos, seguidos de diarrea acuosa 24 a 48hrs después de los síntomas iniciales y,
que persisten entre 3 a 8 días. En los casos moderados puede haber deshidratación, alteraciones
electrolíticas y/o acidosis. En presencia de inmunocompromiso, la infección suele manifestarse como una
diarrea persistente. DIAGNÓSTICO: No es posible realizar el diagnóstico por pruebas clínicas o no
específicas. Para este fin se dispone de inmunoensayos enzimáticos (EIA), inmunocromatográfia y ensayos
de aglutinación de látex para la determinación del antígeno de rotavirus del grupo A en heces. También
pueden identificarse estos virus en heces con microscopio electrónico, por electroforesis y tinción de plata,
por ensayo de reacción en cadena de polimerasa con transcriptasa inversa (RT-PCR) estándar o en tiempo
real para la detección de ARN genómico viral y por cultivo viral. TRATAMIENTO: no existe terapia antivírica,
el tratamiento se aplica en función del estado hidroelectrolítico del paciente. La administración de
inmunoglobulina humana por vía oral (inmunización pasiva) ha tenido buenos resultados para disminuir la
propagación viral y acortar el tiempo de duración del cuadro; este tipo de estrategia se ha usado como
protección en recién nacidos con bajo peso y niños inmunodeficientes.

80 A Varios patógenos bacterianos explican cerca del 10 al 20% de los casos de diarreas en niños, el 10% puede
90%. atribuirse a Escherichia coli diarreogénica. PUNTOS IMPORTANTES QUE RECORDAR: - La gastroenteritis viral
es de corta duración y está asociada a mayor riesgo de vómito y deshidratación. - La gastroenteritis
bacteriana se asocia más frecuentemente a dolor abdominal grave y a veces con diarrea sanguinolenta.

Bibliografía:
1. PREVENCIÓN, DIAGNÓSTICO Y TRATAMIENTO DE DIARREA AGUDA EN PACIENTE DE 2 MESES A 5 AÑOS
DE EDAD, EN EL PRIMER Y SEGUNDO NIVEL DE ATENCIÓN. EVIDENCIAS Y RECOMENDACIONES: GUÍA DE
PRÁCTICA CLÍNICA. MÉXICO: CENETEC. MARZO 2018. 2. GONZÁLEZ N, TORALES A, GÓMEZ D.
INFECTOLOGÍA CLÍNICA PEDIÁTRICA, 8A EDICIÓN. MC GRAW HILL. MÉXICO, 2011. 3. HEYMANN, DAVID L. EL
CONTROL DE LAS ENFERMEDADES TRANSMISIBLES, 19A EDICIÓN. OPS. WASHINGTON, D.C. 2011. 4. RED
BOOK: 2015 REPORT OF THE COMMITTEE ON INFECTIOUS DISEASES, 30ST EDICION.

http://www.cenetec-difusion.com/CMGPC/GPC-SS-156-18/ER.pdf

327 - ES LA MEDIDA DE PREVENCIÓN PRIMARIA ESPECÍFICA PARA ESTA ENFERMEDAD EN NIÑOS DE


ESTE GRUPO DE EDAD:

LAVADO DE Dentro de la historia natural de la enfermedad se contempla la prevención en diferentes casos;


MANOS. en el periodo prepatogénico se realiza la prevención primaria, en el periodo patogénico se debe
promover prevención secundaria y terciaria. En este sentido la PREVENCIÓN PRIMARIA
comprende dos grandes rubros: 1. Promoción a la salud. 2. Promoción específica. OJO: según la
OPS, no se ha comprobado la eficacia de otras medidas preventivas, sólo del uso de las vacunas,
esto derivado de que actualmente se sabe que los fómites juegan un papel importante en la
transmisión y ante ello las medidas para evitar la transmisión fecal-oral pueden ser insuficientes.
CAMPAÑAS DE La PROMOCIÓN A LA SALUD incluye para este caso: - Educación sanitaria. - Orientación
EDUCACIÓN higiénica para prevenir contactos. - Provisión de condiciones adecuadas de vivienda (drenaje,
PARA eliminación de excretas, red de agua). - Campañas educativas y a favor de la vacunación
ENFERMEDADES específica. - Exámenes periódicos para detectar niveles de cloro en el agua domiciliaria. -
DIARRÉICAS. Mejoramiento de los niveles de atención médica en la comunidad. - Acercamiento de los
servicios. - Promoción y capacitación del uso de vida suero oral.

VACUNA La PREVENCIÓN PRIMARIA O PROTECCIÓN “ESPECÍFICA” incluye: - Aplicación de la vacuna


CONTRA antirotavirus en menores de 8 meses, se recomiendan 2 dosis (a los 2 y 4 meses). - Evitar la
ROTAVIRUS. automedicación. - Búsqueda intencionada de casos en la población sana. - Vigilancia de
contactos sanos. Con respecto a las medidas preventivas de protección específica, existen dos
vacunas orales vivas atenuadas contra el rotavirus: la derivada de una sola cepa común de
rotavirus humano y la de un rotavirus reagrupado bovino-humano, que están disponibles a nivel
internacional y precalificadas por la OMS; ambas consideradas sumamente eficaces en la
prevención de la enfermedad gastrointestinal grave. En México, se observó una reducción de
hasta el 50% del número de muertes por diarrea de niños menores de 5 años de edad atribuido
directamente al uso de la vacuna, mientras que, para Estados Unidos, se estima que a partir de
2008 la reducción ha sido dramática llegando hasta un 75% de los casos de hospitalización. En
evaluaciones de control de casos en ese país, se ha descubierto que las vacunas (series
completas) son aproximadamente 85 a 96% eficaces contra la enfermedad de rotavirus grave
que requiere hospitalización; también se registro una reducción importante en el número de
visitas a la consulta externa atribuible a gastroenteritis. VACUNA ANTIROTAVIRUS. Es una
preparación de virus atenuados, de origen humano y/o animal, producida en cultivo celular o
bien a través de la construcción de virus con re-arreglos genéticos, protege contra
gastroenteritis graves causadas por rotavirus. - Se debe vacunar a todos los menores de 8 meses
de edad. - Esquema: dos o tres dosis dependiendo del laboratorio fabricante. La primera dosis
debe aplicarse a los dos meses de edad (puede iniciarse a las seis semanas de vida) y la última
dosis antes de cumplir los 8 meses de edad. El intervalo sugerido es de 2 meses entre cada dosis,
con un mínimo de cuatro semanas. - Dosis: 1.5 o 2 ml, dependiendo de la presentación de la
vacuna y el laboratorio fabricante. - Contraindicaciones: no debe administrarse a personas con
hipersensibilidad conocida después de la administración previa de la vacuna o a cualquier
componente de la vacuna. Personas con malformaciones congénitas no corregidas del tracto
gastrointestinal (divertículo de Meckel) que pudieran predisponer a invaginación intestinal.
Pacientes con intolerancia hereditaria a la fructosa, mala absorción de glucosa o galactosa o
insuficiencia de sacarosa-isomaltosa e inmunodeficiencia combinada severa. Pacientes con
antecedentes de intususcepción y alergia al látex. IMPORTANTE: al elegir esta respuesta deberás
considerar en primer lugar que esta es la única que cumple con los criterios de ser una medida
de PREVENCIÓN PRIMARIA, además de ser ESPECÍFICA. Por otro lado, aunque sea una vacuna
que se aplique antes de los 8 meses de edad, su efecto repercute durante varios años, se espera
que al menos durante los primeros 5 años de vida, momento de más alta incidencia de GEPI viral
complicada. Por lo tanto, LA MEDIDA DE PROTECCIÓN ESPECÍFICA PARA LA GASTROENTERITIS
EN MENORES DE 5 AÑOS ES, POR EXCELENCIA, LA VACUNA CONTRA EL ROTAVIRUS.

TOMA DE La PREVENCÍÓN SECUNDARIA se divide en: diagnóstico temprano (examen clínico, uso de
ISÓPO RECTAL auxiliares diagnósticos), tratamiento específico de la enfermedad y limitación de la incapacidad.
A TODO NIÑO La PREVENCIÓN TERCIARIA incluye la rehabilitación específica según cada enfermedad.
CON DIARREA.

Bibliografía:
1. PREVENCIÓN, DIAGNÓSTICO Y TRATAMIENTO DE DIARREA AGUDA EN PACIENTE DE 2 MESES A 5 AÑOS
DE EDAD, EN EL PRIMER Y SEGUNDO NIVEL DE ATENCIÓN. EVIDENCIAS Y RECOMENDACIONES: GUÍA DE
PRÁCTICA CLÍNICA. MÉXICO: CENETEC. MARZO 2018. 2. NORMA OFICIAL MEXICANA NOM-036-SSA2-2012,
PREVENCIÓN Y CONTROL DE ENFERMEDADES. APLICACIÓN DE VACUNAS, TOXOIDES, FABOTERÁPICOS
(SUEROS) E INMUNOGLOBULINAS EN EL HUMANO. 3. GONZÁLEZ N, TORALES A, GÓMEZ D. INFECTOLOGÍA
CLÍNICA PEDIÁTRICA, 8A EDICIÓN. MC GRAW HILL. MÉXICO, 2011. 4. HEYMANN, DAVID L. EL CONTROL DE
LAS ENFERMEDADES TRANSMISIBLES, 19A EDICIÓN. OPS. WASHINGTON, D.C. 2011. 5. RED BOOK: 2018
REPORT OF THE COMMITTEE ON INFECTIOUS DISEASES, 31ST EDICION.

http://www.cenetec-difusion.com/CMGPC/GPC-SS-156-18/ER.pdf

FIN DEL CASO CLÍNICO SERIADO


ANÁLISIS DEL CASO CLÍNICO

IDENTIFICACIÓN DEL REACTIVO


Area: MEDICINA INTERNA
Especialidad: NEUMOLOGÍA
Tema: ENFERMEDADES PULMONARES OBSTRUCTIVAS Y RESTRICTIVAS
Subtema: ENFERMEDAD PULMONAR OBSTRUCTIVA CRÓNICA Y
ENFERMEDAD INTERSTICIALES

CASO CLÍNICO CON UNA PREGUNTA

HOMBRE DE 65 AÑOS DE EDAD, CON ANTECEDENTE DE HABER TRABAJADO EN UNA MINA POR 40 AÑOS.
ACTUALMENTE SE ENCUENTRA EN TRATAMIENTO POR SILICOSIS PULMONAR.

HOMBRE DE 65 AÑOS.

HABER TRABAJADO EN UNA MINA POR 40


AÑOS.

en TRATAMIENTO POR SILICOSIS


PULMONAR.

-.

-.

328 - LA TERAPÉUTICA EMPLEADA EN EL PACIENTE CON ESTA PATOLOGÍA ESTÁ ENCAMINADA


PRINCIPALMENTE A:
MEJORAR LA REPASO SILICOSIS PATOGENIA. Este tipo de enfermedad ocupacional resulta por la inhalación
COMPLIANCE repetida de polvo de sílice (SiO2) o cristales de cuarzo. El riesgo silicógeno se ha relacionado
PULMONAR. con determinadas actividades profesionales como los trabajadores de minas, cortadores de
piedra, industrias siderometalúrgica y de la cerámica, trabajadores del cristal, fundiciones,
trabajadores de canteras sobre todo de granito, trabajadores en túneles o espacios cerrados
incluso los del gremio de la albañilería. Dentro de estas diferentes ocupaciones, los
trabajadores que emplean el chorro de arena para desincrustar las piezas metálicas o limpiar
las piezas de la fundición son los que más se han relacionado directamente con el desarrollo
de silicosis. La mayoría de las partículas de polvo filtradas por la vía aérea superior son
aclaradas por el epitelio ciliado de las grandes vías. Si estas defensas se encuentran
disminuidas las partículas de menos de 10 micrómetros de diámetro pueden depositarse en
las vías de pequeño tamaño y pueden producir una respuesta inflamatoria que desencadena
la enfermedad. Las partículas de sílice son altamente fibrogénicas y citotóxicas comparadas
con otras partículas como las de carbón. Las partículas de sílice serían fagocitadas por los
macrófagos alveolares, que desencadenarían la cascada inflamatoria con múltiples
mediadores inflamatorios, citoquinas, y factores de crecimiento. De esta forma se pondría en
marcha una maquinaria que a pesar de dejar la exposición, la enfermedad evolucionaría a un
estado de fibrosis irreversible. Estos cambios en la arquitectura pulmonar no sólo producirían
una enfermedad intersticial. Tras la realización de estudios epidemiológicos, hoy es sabido
que el sílice puede desarrollar una limitación crónica al flujo aéreo que cumpliría criterios de
EPOC tipo enfisema y bronquitis crónica. Estas alteraciones se han observado en trabajadores
jóvenes y sin signos radiológicos de silicosis.

PREVENIR TRATAMIENTO DE LA SILICOSIS. Como en otros tipos de neumoconiosis, no se dispone de un


COMPLICACIONES. tratamiento eficaz en la actualidad, la única alternativa es la "prevención", "cese de la
exposición al agente causal" y "prevención de las complicaciones" asociadas. En cambio, esta
medida incluso a veces es ineficaz, ya que la enfermedad puede seguir evolucionando tras
años de suspensión de la exposición. En el caso de que el paciente tenga síntomas de
obstrucción del flujo aéreo, se pueden administrar glucocorticoides. Si en el contexto de una
silicosis surgen complicaciones, el tratamiento irá dirigido a paliar éstas. NO EXISTE UN
TRATAMIENTO ESPECÍFICO PARA LA SILICOSIS, LAS ALTERNATIVAS DE TRATAMIENTO
ESTÁN DIRIGIDAS A DAR SOPORTE AL PACIENTE.

MEJORAR LA CURSO CLÍNICO. Si la exposición es muy intensa y en un corto período de tiempo, como
HEMATOSIS. ocurre en la forma de silicoproteinosis o silicosis aguda, el paciente presenta un cuadro de
disnea progresiva en unas semanas o meses con importante pérdida de peso, y progresión
hacia la insuficiencia respiratoria y la muerte en menos de 2 años. Si por el contrario la
exposición ha sido más larvada como en el caso de la silicosis crónica o simple, pueden
aparecer síntomas en un periodo de 10 a 20 años de la exposición, como tos, expectoración,
disnea como síntoma tardío, y puede presentarse presentarse episodios de hemoptisis
repetidas de escasa cuantía. La forma complicada o fibrosis masiva progresiva ocurre incluso
cuando la exposición a sílice ha cesado, hay compromiso clínico con disnea de esfuerzo y tos.
El polvo puede producir bronquitis o grandes nódulos linfáticos que pueden comprimir la
tráquea y los bronquios. En ocasiones la asociación con el tabaquismo puede potenciar o
acelerar la sintomatología inicial. La exploración física suele ser anodina, salvo en ocasiones
cuando existe cierta obstrucción al flujo aéreo; en ese caso pueden auscultarse roncus y
sibilancias, así como cuando existe cierto grado de fibrosis se escucharán estertores
crepitantes posterobasales.
DISMINUIR LA CRITERIOS DIAGNÓSTICOS. Historia de exposición a polvo de sílice (el periodo de inducción
RESISTENCIA desde la exposición hasta el hallazgo de nódulos silicóticos en la radiografía es generalmente
ALVEOLAR. de 10 años. Periodos más cortos están asociados a exposiciones más intensas y agudas y
puede desarrollar alteraciones en un periodo de meses después de la exposición) y uno o
varios de los siguientes criterios: • Radiografía de tórax u otras técnicas de imagen que
muestren alteraciones compatibles con silicosis (alteraciones radiográficas compatible con la
clasificación ILO). • Hallazgos histopatológicos característicos de la silicosis (los nódulos
silicóticos con particular birrefringencia demostrada por luz polarizada. En el caso de la
silicosis aguda material PAS positivo en el interior de los alvéolos). DIAGNÓSTICO
DIFERENCIAL Cuando nos encontramos ante un paciente con un patrón radiológico
anteriormente citado y existe el antecedente profesional de exposición, es fácil establecer un
diagnóstico. Pero en el caso de que los antecedentes de exposición no sean lo suficientemente
evidentes, los estudios radiológicos nos pueden abrir un amplio abanico de diagnósticos
diferenciales. 1. Infección por diseminación hematógena: cuando en la radiología se
evidencien discretos nódulos dispersos en ambos campos pulmonares de 1 a 5 mm de
diámetro, es decir un patrón miliar, el diagnóstico diferencial se puede realizar con la
tuberculosis miliar o con infecciones fúngicas como la histoplasmosis, sin embargo, el hecho
de que el paciente no presente signos de una enfermedad sistémica lo hacen poco probable.
2. Enfermedades autoinmunes con afectación pulmonar. Las manifestaciones clínicas de una
colagenopatía sistémica son lo suficientemente evidentes antes de un patrón intersticial
pulmonar radiológico, además la positivización de autoanticuerpos, así como la existencia de
glomerulonefritis puede orientar al diagnóstico. 3. Histiocitosis X o granuloma eosinofílico
pulmonar: aunque también produce un patrón nodular en la radiografía, la mayoría de los
paciente son jóvenes o de mediana edad, entre 20 y 40 años, el 90% tiene antecedentes de
tabaquismo. Se produce una evolución de los granulomas hacia fibrosis con la consiguiente
formación de quistes y además las adenopatías hiliares son raras. 4. Sarcoidosis: esta
patología granulomatosa multisistémica, se presenta en pacientes entre 20 y 40 años, y
produce también adenopatías hiliares en cáscara de huevo (eggshell) con opacidades
pulmonares, por lo que los antecedentes de exposición en este caso son de gran importancia
en el diagnóstico diferencial, además del tiempo de exposición y la temprana edad. 5.
Neumonitis por hipersensibilidad: los antecedentes de exposición a alérgenos propios de la
neumonitis, así como la clínica sistémica nos reorientaran al diagnóstico de esta entidad. 6.
Otras neumopatías ocupacionales: se deberá descartar otros tipos de neumoconiosis como la
del minero de carbón, la asbestosis y la beriliosis. 7. Diseminación hematógena neoplásica
como una linfangitis carcinomatosa.

Bibliografía:
1. GUÍA DE PRÁCTICA CLÍNICA, DIAGNÓSTICO Y TRATAMIENTO DE NEUMOCONIOSIS POR SÍLICE. MÉXICO:
SECRETARIA DE SALUD, 2009 2. GUÍA DE REFERENCIA RÁPIDA, DIAGNÓSTICO Y TRATAMIENTO DE
NEUMOCONIOSIS POR SÍLICE. MÉXICO: SECRETARIA DE SALUD, 2009

http://www.cenetec.salud.gob.mx/descargas/gpc/CatalogoMaestro/382_IMSS_10_Neumoconiosis_por_Silice/GPC_Neumoconiosis_Silice.pdf
ANÁLISIS DEL CASO CLÍNICO

IDENTIFICACIÓN DEL REACTIVO


Area: GINECOLOGÍA Y OBSTETRICIA
Especialidad: OBSTETRICIA
Tema: COMPLICACIONES DEL EMBARAZO Y DEL PARTO
Subtema: ATENCIÓN DEL PARTO Y DISTOCIAS

CASO CLÍNICO CON UNA PREGUNTA

FEMENINA DE 25 AÑOS DE EDAD, GESTA 3, PARTOS 1, CÉSAREA 1 (HACE 5 AÑOS). CURSA CON EMBARAZO DE
38 SEMANAS DE GESTACIÓN Y TRABAJO DE PARTO. DURANTE LA VALORACIÓN EN PÉLVICA EN ADMISIÓN SE
LE CONSIDERA PÉLVIS LÍMITE.

Mujer de 25 años.

g3, p1, c1.

-.

PÉLVIS LIMÍTROFE.

-.

329 - EL PROCEDIMIENTO QUE DEBERÁ INDICARSE PARA INTEGRAR EL DIAGNÓSTICO ES:

PELVIMETRÍA La PELVIMETRÍA ha sido utilizada para pronosticar la necesidad de cesárea en mujeres cuyos fetos
ÓSEA. tienen presentación cefálica, por lo que puede influir en la atención clínica. No se puede
establecer el pronóstico de parto vaginal exitoso en algún embarazo, sólo con base en la
pelvimetría radiográfica, ya que su sensibilidad y especificidad es muy baja. SE CONSIDERA DE
UTILIDAD LIMITADA PARA LA ATENCIÓN DEL TRABAJO DE PARTO EN LAS PRESENTACIONES
CEFÁLICAS POR LO QUE HA CAÍDO EN DESUSO.

TOMOGRAFÍA Las ventajas de la PELVIMETRIA POR TAC en comparación con la pelvimetría ósea incluyen, menor
AXIAL exposición a radiación, mayor presición y mayor desempeño. AUNQUE ES UN MÉTODO MUY
COMPUTADA CONFIABLE PARA DIAGNOSTICAR LA DESPROPORCIÓN CEFALOPÉLVICA EL COSTO LA DESCARTA
DE PELVIS. COMO MEDIDA DIAGNÓSTICA PRIMARIA.
PRUEBA DE La PRUEBA DE TRABAJO DE PARTO es el procedimiento obstétrico a que se somete una
TRABAJO DE parturienta con relación cefalopélvica límite y mediante su vigilancia y conducción, sin riesgo
PARTO. materno; tiene por objeto conseguir la evolución del trabajo de parto, venciendo obstáculos
previstos y presumiblemente franqueables. LA PRUEBA DE TRABAJO DE PARTO ESTA INDICADA
EN LA PRESENCIA DE PELVIS LIMÍTROFE, IDEAL EN ÉSTA PACIENTE DEBIDO A QUE TIENE UN
PARTO PREVIO.

ULTRASONIDO Dados los resultados poco claros de la pelvimetria ósea se creo un método para descubrir la
OBSTÉTRICO. desproporción cefalopélvica al comparar la circunferencia de la cabeza y abdominal fetales, con
los planos de entrada y medio de la pelvis materna, a lo que se le llamo índice fetopélvico. Para
los cálculos de las dimensiones fetales se utiliza la ecosonografía. EL ULTRASONIDO OBSTÉTRICO
ES DE UTILIDAD ÚNICAMENTE PARA DETERMINAR EL DIÁMETRO CRANEAL DEL FETO REQUIERE
DE UNA RADIOGRAFÍA MATERNA PARA REALIZAR LA COMPARACIÓN MATERNO-FETAL, ESTE
MÉTODO NO HA RESULTADO SER PREDICTIVO DE DESPROPORCIÓN CEFALOPÉLVICA.

Bibliografía:
1. GUÍA DE PRÁCTICA CLÍNICA, REDUCCIÓN DE LA FRECUENCIA DE OPERACIÓN CESÁREA. MÉXICO:
INSTITUTO MEXICANO DEL SEGURO SOCIAL; 2014. 2. CUNNINGHAM G, LEVENO K, BLOMM S, HAUTH J,
RPUSE D, SONG C. WILLIAMS OBSTETRICIA, 23A EDICIÓN. MC GRAW HILL. USA. 2011 EN ESPAÑOL, PP 570-
571.

http://www.cenetec.salud.gob.mx/descargas/gpc/CatalogoMaestro/048_GPC_Cesarea/IMSS_048_08_EyR.pdf
ANÁLISIS DEL CASO CLÍNICO

IDENTIFICACIÓN DEL REACTIVO


Area: PEDIATRÍA
Especialidad: URGENCIAS PEDIÁTRICAS
Tema: URGENCIAS ORTOPÉDICAS
Subtema: LUXACIONES Y FRACTURAS DE MIEMBRO SUPERIOR

CASO CLÍNICO SERIADO

LACTANTE FEMENINA DE 12 MESES DE EDAD, AL IR CAMINANDO DE LA MANO DE SU PADRE SUFRE TIRÓN


DEL BRAZO, CON EL CODO EN EXTENSIÓN Y PRONACIÓN. POSTERIORMENTE PRESENTA IRRITABILIDAD,
LLANTO, DOLOR A LA MOVILIZACIÓN Y LIMITACIÓN A LA FUNCIÓN.

lactante menor de 5 años

el mecanismo de lesion ...CODO EN


EXTENSIÓN Y PRONACIÓN...

DOLOR A LA MOVILIZACIÓN DEL CODO


NO MUEVE LA EXTREMIDAD

-rx no revelan datos evidentes en el codo


de niñera

330 - EL DIAGNÓSTICO MÁS PROBABLE ES:

ELONGACIÓN Aunque el mecanismo descrito en el caso clínico podría ser causante de una elongación del plexo
DEL PLEXO braquial, no se comentan más datos clínicos que nos orienten hacia dicha patología, tales como
BRAQUIAL disminución de la fuerza muscular en el miembro torácico que puede ser distal o proximal
(parálisis de herb o de klumpque) o incluso una parálisis total del miembro superior la cual daría
más aun la sospecha de lesión completa del plexo braquial. Debido a que no contamos con dichos
síntomas podemos descartar tal patología.

FRACTURA Para que un lactante presente fractura de algún hueso debe haber un mecanismo de lesión mas
METAFISARIA severo que una simple tracción de su brazo, a menos que tenga patología de fondo como lo es la
DE RADIO osteogenesis imperfecta, de lo cual no se mencionan datos en el presenta caso. por lo tanto
podemos descartar esta opción.
SUBLUXACIÓN En este caso debemos tomar en cuenta las palabras comentadas en el caso: "...SUFRE TIRON DEL
DE CABEZA DE BRAZO, CON EL CODO EN EXTENSIÓN Y PRONACIÓN..." Este es el mecanismo de lesión
RADIO patognomónico de la subluxación de la articulación radio cubital proximal también conocida
como...CODO DE NIÑERA...

LUXACIÓN DE Al igual que en las opciones anteriores no contamos con datos comentado en el planteamiento
CODO del caso que nos hagan sospechar de tal lesión, además, igual que en las fracturas, se necesita de
un mecanismo de lesión más severa para causar dicha lesión principalmente a la presencia de una
hiperlaxitud ligamentaria en el lactante y a que la forma de la cabeza del radio antes de los 5 años
aun no es la manera madura de la misma, es decir, normalmente la cabeza del radio debe tener
una forma cónica con la cabeza más ancha que la base lo cual hace que el ligamento anular
permanezca en su sitio al momento de la pronosupinación. En el caso de los lactantes la forma de
la cabeza radial aun presenta el mismo diámetro en la base que en la cabeza por lo tanto no hay
nada que impida la salida del ligamento anular al momento de la pronación anexando una
tracción del codo presentándose así la subluxación de dicha articulación.

Bibliografía:
DIAGNÓSTICO Y TRATAMIENTO EN URGENCIAS. SAUNDERS. MANUAL MODERNO. EDICIÓN 3A. 1999. PAG.
366.

331 - LOS ESTUDIOS RADIOLÓGICOS EN ESTA PACIENTE SON NECESARIOS PARA:

DEFINIR EL Los estudios radiológicos son útiles en el caso de sospecha de lesiones óseas para confirmar el tipo
TRAZO DE de fractura, el lugar exacto de la lesión y determinar así el tratamiento más adecuado a seguir, en
FRACTURA este caso la subluxación de la cabeza radial siendo una patología propia de la infancia y lactancia
no presentara cambios evidentes en la radiografía de la articulación.

DESCARTAR El objetivo de solicitar estudios radiológicos será el de descartar lesiones del tipo fracturas no así
LESIÓN ÓSEA para corroborar el diagnóstico de subluxación de la cabeza del radio, pues como se menciona
anteriormente no se observaran cambios radiológicos en este tipo de lesiones.

ESTUDIAR En este caso no se sospecha de luxación del codo aun que si este fuera el caso...
LUXACIÓN independientemente de la EF, la clínica que nos indicaría que pudiera existir la misma por la
incongruencia de la articulación del codo la radiografía en lateral evidenciaría sin duda la
deformidad.

CORROBORAR Generalmente no se asocia a lesión cervical.


LESIÓN EN
CERVICALES

Bibliografía:
DIAGNÓSTICO Y TRATAMIENTO EN URGENCIAS. SAUNDERS. MANUAL MODERNO. EDICIÓN 3A. 1999. PAG.
366.

332 - EL MANEJO A SEGUIR EN ESTA PACIENTE ES CON:

REDUCCIÓN DE LUXACIÓN El tratamiento en si es la reducción de la subluxación mediante la manipulación del


codo, en general las reducciones cerradas de la fracturas y/o luxaciones pretenden
reproducir inversamente el mecanismo de la lesión por lo tanto si el mecanismo por el
cual se lesiono la articulación fue la pronación... para la reducción se deberá realizar
una supinación del codo.

ELECTROESTIMULACIONES No tendrán ningún efecto en la patología que estamos tratando.


TRACCIÓN Y SUPINACIÓN Para corregir la SUBLUXACIÓN DE LA CABEZA DEL RADIO se tira con suavidad del
antebrazo cogido por la muñeca y colocando el pulgar de la otra mano sobre la cara
anterior de la cabeza del radio. Se va girando lentamente el antebrazo de modo que el
pulgar del niño se vaya alejando de su cuerpo, a la vez que flexionamos el codo. El
desbloqueo tiene lugar cuando notamos un resalte o clic en el lugar donde tenemos
colocado el dedo pulgar. El niño recupera de inmediato la movilidad y si la lesión era
reciente, deja de doler y el niño de llorar.

MANIPULACIÓN Y YESO Si se trata del primer episodio y se ha resuelto en las primeras 12 horas, no es
necesario inmovilizar la extremidad. Se inmovilizaría si a pesar de hacer la maniobra
correcta y notar el "clic o resalte", persiste con dolor. También podría ser necesario
inmovilizar 2 ó 3 semanas si hay episodios recurrentes. Lo normal es que no haga
falta, no será necesario inmovilizar con aparato de yeso completo, puede ser
suficiente con una férula o incluso con un cabestrillo.

Bibliografía:
DIAGNÓSTICO Y TRATAMIENTO EN URGENCIAS. SAUNDERS. MANUAL MODERNO. EDICIÓN 3A. 1999. PAG.
366.

FIN DEL CASO CLÍNICO SERIADO


ANÁLISIS DEL CASO CLÍNICO

IDENTIFICACIÓN DEL REACTIVO


Area: PEDIATRÍA
Especialidad: CRECIMIENTO Y DESARROLLO
Tema: CRECIMIENTO Y DESARROLLO NORMAL
Subtema: ESCOLAR

CASO CLÍNICO CON UNA PREGUNTA

CON EL FIN DE DETECTAR OPORTUNAMENTE RETRASO EN EL DESARROLLO DE PACIENTES EN EDAD


ESCOLAR, SE LE HA SOLICITADO EVALUAR A LOS NIÑOS DE LA PRIMARIA DE LA COMUNIDAD EN LA QUE
REALIZA SU SERVICIO SOCIAL.

escolares de primaria.

EVALUAR DESARROLLO

333 - CORRESPONDE A LA ESCALA DE EVALUACIÓN DEL DESARROLLO PARA NIÑOS DE ESTA EDAD:

DE La ESCALA DE DESARROLLO DE GESELL es específica para la valoración del desarrollo en niños de 5 a 9


GESELL. años y para adolescente de 10 a 19 años.

DE El criterio más comúnmente utilizado para valorar la madurez sexual en el adolescente es la de los
TANNER. ESTADIOS DE TANNER la cual evalúa desarrollo genital, del vello púbico y de las mamas. LOS
ESTADIOS DE TANNER SON ESPECÍFICOS PARA EVALUAR LA MADUREZ SEXUAL EN ADOLESCENTES.

DE El desarrollo neurológico del niño en las áreas motriz gruesa, motriz fina, lenguaje y socialización se
DENVER. evalúa por medio de la PRUEBA DE DENVER REVISADA la cual se evalúa hasta los 24 meses de edad.
LA PRUEBA DE DENVER EVALÚA EL DESARROLLO NEUROLÓGICO HASTA LOS 2 AÑOS DE EDAD.
DE La determinación de la edad ósea es un método bien utilizado para valorar el grado de madurez
BLIZZARD. fisiológica o biológica de un niño. El método más utilizado para este fin es el de Greulich-Pyle
esquematizado por Blizzard (esquemas de Blizzard); en él se analiza el desarrollo del hombro, codo,
mano, cadera y rodilla y pie desde el nacimiento hasta los 12 a 13 años. LOS ESQUEMAS DE BLIZZARD
SON UTILIZADOS PARA DETERMINAR LA EDAD ÓSEA EN NIÑOS DESDE EL NACIMIENTO HASTA LOS
13 AÑOS DE EDAD.

Bibliografía:
1. GUÍA DE PRÁCTICA CLÍNICA, CONTROL Y SEGUIMIENTO DE LA NUTRICIÓN Y EL CRECIMIENTO Y
DESARROLLO DEL NIÑO MENOR DE 5 AÑO. MÉXICO: SECRETARIA DE SALUD; 2009. 2. NORMA OFICIAL
MEXICANA NOM -008-SSA2-1993, CONTROL DE LA NUTRICIÓN, CRECIMIENTO Y DESARROLLO DEL NIÑO Y
DEL ADOLESCENTE. CRITERIOS Y PROCEDIMIENTOS PARA LA PRESTACIÓN DEL SERVICIO.
ANÁLISIS DEL CASO CLÍNICO

IDENTIFICACIÓN DEL REACTIVO


Area: GINECOLOGÍA Y OBSTETRICIA
Especialidad: OBSTETRICIA
Tema: COMPLICACIONES DEL EMBARAZO Y DEL PARTO
Subtema: PARTO PREMATURO

CASO CLÍNICO SERIADO

MUJER DE 18 AÑOS CON EMBARAZO DE 32 SEG. ACUDE AL SERVICIO DE URGENCIAS POR PRESENTAR DESDE
HACE 2 DÍAS MAREOS, DOLOR EN EPIGASTRIO CON IRRADIACIÓN A AMBOS FLANCOS, POLIAQUIURIA Y
DISURIA. A LA EXPLORACIÓN SE ENCUENTRA FONDO UTERINO A 27 CM DEL BORDE SUPERIOR DEL PUBIS,
PRODUCTO ÚNICO LONGITUDINAL CEFÁLICO, CON FRECUENCIA CARDIACA FETAL DE 145 LATIDOS POR
MINUTO, DOLOR A LA PALPACIÓN EN EPIGASTRIO, HIPOGASTRIO E HIPOCONDRIO DERECHO. DURANTE LA
EXPLORACIÓN PRESENTA 4 CONTRACCIONES EN 20 MINUTOS DE 30 SEGUNDOS CADA UNA. EN EL TACTO
VAGINAL CÉRVIX INTERMEDIO BLANDO CON 80% DE BORRAMIENTO.

-.

embarazo 32sdg.

-.

ACTIVIDAD UTERINA 4 CONTRACCIONES


EN 20 MINUTOS, CÉRVIX INTERMEDIO,
BLANDO CON 80% DE BORRAMIENTO.

-.

334 - EL DIAGNOSTICO CLÍNICO MÁS PROBABLE ES:


INMINENCIA La ECLAMPSIA es la fase convulsiva de la enfermedad hipertensiva (preeclampsia) y se encuentra
DE ECLAMPSIA. entre las más graves manifestaciones de la enfermedad. A menudo es precedido por
acontecimientos premonitorios, tales como dolores de cabeza e hiperreflexia pero puede ocurrir
en ausencia de signos. OJO: aunque generalmente se presenta en el contexto de una
preeclampsia, la eclampsia se presenta tras una serie de signos y síntomas que puede o no tener
hipertensión o proteinuria o edema o una combinación de todos. Es recomendable el
interrogatorio sobre síntomas clínicos premonitorios que pueden ayudar a predecir la aparición
de eclampsia, entre ellos se encuentran: cefalea occipital o frontal persistente, fosfenos,
fotofobia, epigastralgia, dolor en cuadrante superior derecho o ambos y alteraciones del estado
mental. Los signos y síntomas de alarma para eclampsia o INMINENCIA DE ECLAMPSIA son, en
conjunto, el evento clínico que precede a la eclampsia y que, por sus características, sugieren la
evolución rápida hacia la fase convulsiva. Se considera en presencia de TA mayor a 185/112mm
Hg, proteinuria mayor a 10g, estupor, irritabilidad, pérdida parcial o total de la visión, dolor
epigástrico en barra, hiperreflexia generalizada o la triada náuseas, vómito y cefalea. Por su
gravedad, estas pacientes deberán ser manejadas como eclámpticas. NOTA: si bien, este término
de inminencia de eclampsia no es contemplado actualmente en la GPC, los lineamientos de la
Secretaría de Salud y algunos otros autores lo definen así y será importante que lo conozcas. LA
PACIENTE REFIERE PRESENCIA DE DOLOR EPIGÁSTRICO EN BARRA QUE AL SER EXPLORADA
SOLO SE CORROBORA CON IRRADIACIÓN A HIPOCONDRIO DERECHO SIN MÁS DATOS QUE
FUNDAMENTEN ÉSTA OPCIÓN DIAGNÓSTICA.

PREECLAMPSIA. Los DESÓRDENES HIPERTENSIVOS EN EL EMBARAZO pueden ser clasificados como (Magee L,
2014): a) Hipertensión Preexistente. - Con condiciones comórbidas. - Con evidencia de
preeclampsia b) Hipertensión gestacional. - Con condición comórbida. - Con evidencia de
preeclampsia. c) Preeclampsia. d) Otras Formas Hipertensivas: - Hipertensión transitoria -
Hipertensión de bata blanca - Hipertensión enmascarada. Se entiende por PREECLAMPSIA a la
evidencia de preeclampsia puede aparecer muchas semanas después del inicio de la hipertensión
gestacional. Está definida como hipertensión que se presenta en el embarazo (TA sistólica igual o
mayor a 140 mm Hg o TA diastólica igual o mayor a 90mm Hg en 2 ocasiones con un margen de
al menos 4 horas después de la semana 20 de gestación en una mujer con presión arterial normal
antes del embarazo) y uno o más de los siguientes puntos: Proteinuria de nueva aparición (igual
o mayor a 300 mg en recolección de orina de 24 hrs) o Labstix con lectura de 1+ solo si no se
dispone de otro método cuantitativo o Relación proteína/creatinina mayor a 0.28 mg/dL o en
ausencia de proteinuria, 1 o más condiciones adversas. En la actualidad el Colegio Americano de
Ginecólogos y Obstetras (ACOG) integra a la preeclampsia con y sin características de severidad,
lo que anteriormente se conocía como preeclampsia leve y severa, y es recomendable, según la
GPC actual que, en la mujer con embarazo y trastorno hipertensivo, sea clasificada la presencia o
ausencia de características de severidad de acuerdo con los criterios establecidos por este
Colegio. EN LA PREECLAMPSIA PUEDE PRESENTARSE DOLOR EN HIPOCONDRIO DERECHO CON
ELEVACIÓN DE LAS CIFRAS TENSIONALES Y PROTEINURIA, ESTOS ÚLTIMOS DATOS NO ESTÁN
PRESENTES EN NUESTRA PACIENTE.

ENFERMEDAD La embarazada experimenta con frecuencia a lo largo de la gestación, síntomas originados en


ÁCIDO trastornos del aparato digestivo. Algunos de ellos como el reflujo gastroesofágico o la
PÉPTICA. constipación, se explican por modificaciones fisiológicas bien conocidas; otros, sin embargo,
como la hiperémesis gravídica, tienen una patogenia casi desconocida. Puede presentarse con
frecuencia nauseas, dolor en hipogastrio y pirosis. SI BIEN EL DOLOR QUE PRESENTA LA
PACIENTE SE UBICA PRINCIPALMENTE EN HIPOGASTRIO Y PUEDE SER A CAUSA DE UNA
ENFERMEDAD ÁCIDO PÉPTICA, NO CONSTITUYE EL DIAGNOSTICO PRINCIPAL YA QUE AL HABER
CAMBIOS CERVICALES ESO DEBE SER EL PUNTO PRINCIPAL DE ATENCIÓN Y DIAGNÓSTICO.

AMENAZA DE El parto pretérmino se caracteriza por la presencia de contracciones uterinas (> 4 en 20 minutos
PARTO o >8 en una hora) y cambios cervicales (<20mm o fibronectina fetal positiva asociado a longitud
PRETÉRMINO. cervical entre 29 a 20 mm) documentados con membranas amnióticas integras entre las 20.1 a
las 36 semanas con 6 días. Se establece diagnóstico de AMENAZA DE PARTO PRETÉRMINO con
presencia de contracción uterinas con una frecuencia de 2 en 10 minutos o 4 en 20 minutos y 8
en 60 minutos acompañadas de cualquiera de los siguientes: Dilatación cervical mayor o igual a 3
centímetros, borramiento mayor o igual a 50% ó cambios cervicales por medición de longitud
cervical detectados por estudios de seguimiento sonográfico. DADO QUE LA PACIENTE
PRESENTA CONTRACCIONES CON CAMBIOS CERVICALES EN UNA EDAD GESTACIONAL MENOR
A 37 SDG SE DOCUMENTA BIEN EL DIAGNÓSTICO DE AMENAZA DE PARTO PRETÉRMINO.

Bibliografía:
1. PREVENCIÓN, DIAGNÓSTICO Y TRATAMIENTO DE PARTO PRETÉRMINO. EVIDENCIAS Y
RECOMENDACIONES. GUÍA DE PRÁCTICA CLÍNICA. MÉXICO, CENETEC; 2017. 2. PREVENCIÓN,
DIAGNÓSTICO Y TRATAMIENTO DE LA PREECLAMPSIA EN EL SEGUNDO Y TERCER NIVELES DE ATENCIÓN.
EVIDENCIAS Y RECOMENDACIONES. GUÍA DE PRÁCTICA CLÍNICA. MÉXICO, SECRETARÍA DE SALUD;
09/03/17. 3. CUNNINGHAM G, LEVENO K, BLOMM S, HAUTH J, RPUSE D, SONG C. WILLIAMS OBSTETRICIA,
23A EDICIÓN. MC GRAW HILL. USA. 2011 EN ESPAÑOL, PP 804, 814.

http://www.cenetec-difusion.com/CMGPC/IMSS-063-08/ER.pdf

335 - EL MANEJO INICIAL DE ESTA PACIENTE DEBE CONSISTIR EN:

HIDRATACIÓN Y 1. HIDRATACIÓN. No se recomienda la hidratación y reposo estricto en cama como


ADMINISTRACIÓN tratamientos específicos e intervenciones que disminuyan el parto pretérmino ya que un
DE SULFATO DE análisis demostró que NO EXISTE SUFICIENTE EVIDENCIA PARA EL USO DE LA
MAGNESIO. HIDRATACIÓN COMO PARTE DEL MANEJO EN PACIENTES CON TRABAJO DE PARTO
PRETÉRMINO. 2. SULFATO DE MAGNESIO. Es el medicamento de elección para la prevención
y tratamiento de las crisis convulsivas durante el embarazo. DADO QUE LA PACIENTE NO
PRESENTA CONVULSIONES NO ESTÁ INDICADO ESTE MANEJO. IMPORTANTE: Se ha
sugerido que el uso de sulfato de magnesio en concentraciones suficientemente altas logra
alterar la contractilidad del endometrio, actuando como un antagonista de calcio. Por lo
cual se le han atribuido propiedades para inhibir el trabajo de parto. Existen diversos
estudios que sugieren una dosis de impregnación de 4gr seguida de una infusión de 2g/h
para detener el trabajo de parto. SI BIEN, ALGUNOS CONSIDERAN LAS PROPIEDADES DE
ESTE FÁRMACO COMO TOCOLÍTICO, LAS GPC ESTABLECEN SU RECOMENDACIÓN POR
24HRS EN MUJERES CON PARTO PRETÉRMINO PARA REDUCIR EL RIESGO DE PARÁLISIS
CEREBRAL EN EL PRODUCTO, MÁS QUE COMO UN TOCOLÍTICO.

INHIBIDORES DE El uso de inhibidores de receptores H2 y los antiácidos son seguros durante la gestación y
RECEPTORES H2 Y sustituyen al uso de inhibidores de la bomba de protones por su seguridad. Los síntomas
PROTECTORES DE característicos del reflujo gastroesofágico son muy frecuentes durante el embarazo, en
MUCOSA GÁSTRICA. especial la pirosis y es ahí donde están indicados. SI BIEN NO PODEMOS OLVIDARNOS DE
LA POSIBILIDAD DE ENFERMEDAD ÁCIDO PÉPTICA ASOCIADA AL EMBARAZO, EL MANEJO
INICIAL DEBE ENCAMINARSE EN EVITAR EL PARTO PRETÉRMINO ASEGURANDO UNA
MADUREZ FETAL EN LO POSIBLE.
INDUCTORES DE 1. TERAPIA TOCOLÍTICA Puede retrasar el parto pretérmino, aunque no ha demostrado que
MADURACIÓN mejoren los resultados neonatales. Los objetivos de la terapia tocolítica son: - Otorgar un
PULMONAR Y tiempo suficiente para la administración de los corticosteroides prenatales. - Permitir la
TERAPIA referencia de la paciente con PP a una unidad hospitalaria de 3er nivel. OJO: La tocolisis
TOCOLÍTICA puede ser considerada para las mujeres con sospecha de parto prematuro que han tenido un
embarazo sin complicaciones; sin embargo, la tocolisis no debe usarse cuando existe una
contraindicación para prolongar el embarazo. La decisión de uso del agente tocolítico que el
clínico elija debe basarse en las características individuales de cada paciente. La evidencia
apoya el uso de fármacos tocolíticos a corto plazo (48 horas) para prolongar el embarazo
durante al menos 48 horas que permita la administración de esteroides prenatales y que
pueda permitir el transporte de la madre a un centro de atención especializada y la
administración de sulfato de magnesio para reducir el riesgo de parálisis cerebral. En este
sentido, se recomienda desde la semana 23.5 y hasta la 33.6 retrasar el nacimiento 48 para
permitir el manejo con esteroides para maduración pulmonar fetal o la referencia a un
segundo nivel de atención. Se consideran como agentes tocolíticos de primera elección los
antagonistas de oxitocina, bloqueadores de canales de calcio, beta miméticos y AINES; otros
tocolíticos no se recomiendan por sus efectos adversos. OJO: en gestaciones menores o
iguales a 28 semanas, se recomienda continuar la terapia por más de 48hrs y hasta por dos
semanas; mientras que, para embarazos mayores de 29 semanas no se debe extender la
terapia tocolítica por más de 48hrs. 2. INDUCTORES DE MADURACIÓN PULMONAR.
Actualmente, con base en la GPC para la prevención, diagnóstico y tratamiento del parto
pretérmino, establece que la TERAPIA ANTENATAL CON CORTICOESTEROIDES para la
maduración pulmonar debe administrarse a mujeres con embarazos menores a 38 semanas”.
OJO: ANTERIORMENTE SE RECOMENDABA EL USO DE CORTICOESTEROIDES HASTA LA
SEMANA 33.6 DE GESTACIÓN; SIN EMBARGO, SEGÚN LA ÚLTIMA VERSIÓN DE LA GPC
PARA PARTO PRETÉRMINO (2017), ESTOS LINEAMIENTOS HAN SIDO CAMBIADOS Y SERÁ
IMPORTANTE QUE LOS TENGAS EN CUENTA. Indicaciones: - Mujeres con embarazo entre las
24 y 34 SDG que presenten: parto pretérmino, embarazo múltiple, ruptura de membranas. -
Mujeres con embarazo de 34 a 36.6 SDG que presenten: riesgo de parto pretérmino en los
próximos 7 días y que no hayan recibido previamente terapia antenatal con
corticoesteroides. Indicaciones dosis de rescate: - Mujeres con embarazo de 34 a 36.6SDG
que presenten: riesgo de parto pretérmino en los próximos 7 días y que hayan recibido
terapia antenatal con corticoesteroides en más de 14 días. Contraindicaciones: La
administración tardía de la terapia antenatal con corticoesteroides no está indicada en
mujeres con diagnóstico de infección intrauterina (corioamnionitis). Condiciones especiales:
- La administración de corticoesteroides prenatales en la paciente con parto pretérmino
inminente no debe ser utilizado como indicación para el uso de tocolisis en un intento de
retrasar el parto. - No se debe posponer el parto pretérmino medicamente indicado en la
paciente grave para la administración de la terapia antenatal con corticoesteroides. - Con
base en el contexto clínico, puede administrarse la terapia antenatal con corticoesteroides
en casos de parto prematuro indicado que presenten embarazo entre 34 a 36 semanas con
gestación múltiple. - Son recomendables en todas las pacientes con preeclampsia antes de
las 36SDG. Dosis recomendada de corticoesteroides: - Betametasona: 2 dosis, cada una de
12mg cada 24hrs IM. - Dexametasona: 4 dosis, cada una de 6mg cada 12hrs IM.
CORRESPONDE AL MANEJO IDEAL EN ESTA PACIENTE.

HIDRATACIÓN Y 1. La HIDRATACIÓN NO HA DEMOSTRADO BENEFICIOS en el manejo de las pacientes con


ANTIHIPERTENSIVO. amenaza de parto pretérmino. 2. Los ANTIHIPERTENSIVOS están reservados para el USO
EXCLUSIVO DE LA ENFERMEDAD HIPERTENSIVA DEL EMBARAZO en todos sus niveles, la
paciente no presenta datos que respalden este diagnóstico.

Bibliografía:
1. PREVENCIÓN, DIAGNÓSTICO Y TRATAMIENTO DE PARTO PRETÉRMINO. EVIDENCIAS Y
RECOMENDACIONES. GUÍA DE PRÁCTICA CLÍNICA. MÉXICO, CENETEC; 2017. 2. CUNNINGHAM G, LEVENO K,
BLOMM S, HAUTH J, RPUSE D, SONG C. WILLIAMS OBSTETRICIA, 23A EDICIÓN. MC GRAW HILL. USA. 2011
EN ESPAÑOL, PP 822-826.

http://www.cenetec-difusion.com/CMGPC/IMSS-063-08/ER.pdf

FIN DEL CASO CLÍNICO SERIADO


ANÁLISIS DEL CASO CLÍNICO

IDENTIFICACIÓN DEL REACTIVO


Area: MEDICINA INTERNA
Especialidad: HEMATOLOGÍA
Tema: ANEMIAS
Subtema: HEMOGLOBINOPATÍAS

CASO CLÍNICO CON UNA PREGUNTA

MASCULINO DE 24 AÑOS CON SOSPECHA DIAGNÓSTICA DE ANEMIA DE CÉLULAS FALCIFORMES.

MASCULINO DE 24 AÑOS.

-.

-.

-.

SOSPECHA DIAGNÓSTICA DE ANEMIA DE


CÉLULAS FALCIFORMES.

336 - LA HEMOGLOBINA MAS ABUNDANTE EN LOS PACIENTES CON ÉSTE DIAGNÓSTICO ES LA:

C. Hemoglobinopatía C La Hb C se caracteriza por la sustitución del ácido glutámico de la posición 6 de la cadena


beta por lisina. Es una hemoglobinopatía propia del África occidental, pero puede encontrarse con cierta
frecuencia en España. El estado homocigoto (CC) se caracteriza por una ligera anemia hemolítica crónica con
esplenomegalia. El estado heterocigoto (AC) no produce trastorno alguno. Aunque la Hb C tiende a cristalizar
en condiciones de hipoxia, no produce crisis vasooclusivas como las de la Hb S. La morfología eritrocitaria se
caracteriza por la aparición de dianocitos. La presencia de Hb C interfiere en la determinación por
cromatografía en columna de la Hb A (cuyo aumento es característico de la beta talasemia heterocigota). ESTE
TIPO DE HEMOGLOBINOPATÍA ES FRECUENTEMENTE ENCONTRADA EN CONJUNTO DE LA S TIPO S.
S. DEFINICIONES: • Enfermedad falciforme: es un trastorno hereditario de la sangre caracterizado por una
anomalía de la hemoglobina que consiste en una alteración estructural de la cadena b de la globina debida a la
sustitución de un único aminoácido (ácido glutámico por valina) originando la hemoglobina S (HbS). • Rasgo
falciforme: El niño es portador del gen defectuoso, HbS, pero también tiene algo de hemoglobina normal,
HbA. Esto se denomina HbAS. Los niños que tienen el rasgo drepanocítico normalmente no presentan ningún
síntoma de la enfermedad. • Anemia células falciformes: La mayoría o toda la hemoglobina normal (HbA) del
niño está cambiada por hemoglobina falciforme (HbS). Esto se denomina HbSS. Es la forma más común y más
severa de las variedades de células falciformes. LOS SÍNDROMES DREPANOCÍTICOS (FALCIFORMES) SE
PRODUCEN POR UNA MUTACIÓN DEL GEN DE LA GLOBINA B QUE SUSTITUYE POR VALINA EL SEXTO
AMINOÁCIDO, EL ÁCIDO GLUTÁMICO ORIGINANDO LA HBS.

J. Hemoglobinopatía J Se caracteriza por la sustitución de la glicina en posición 16 de la cadena beta por ácido
aspártico. Es una Hb de migración rápida. No produce ningún trastorno en estado heterocigoto. Endémica en
Europa, la Hb J es relativamente frecuente en Cerdeña y puede encontrarse en España.

D. La Hb D no produce trastorno alguno en estado heterocigoto. El estado homocigoto, muy infrecuente, produce
una discreta anemia hemolítica. La movilidad electroforética de la Hb D es la misma que la de la Hb S. La Hb E
es muy frecuente en el sudeste asiático. El estado homocigoto no pro-duce alteraciones clínicas, pero el
hemograma es semejante al de las talasemias. El estado heterocigoto provoca sólo microcitosis discreta.

Bibliografía:
1. LONGO DL, FAUCI AS, KASPER DL, HAUSERSL, JAMESON JL, LOSCALZOJ. HARRISON. PRINCIPIOS DE
MEDICINA INTERNA, 18A EDICIÓN. MC GRAW HILL. NEW YORK, USA. 2012, PP 854-855.
ANÁLISIS DEL CASO CLÍNICO

IDENTIFICACIÓN DEL REACTIVO


Area: GINECOLOGÍA Y OBSTETRICIA
Especialidad: OBSTETRICIA
Tema: ENFERMEDAD HIPERTENSIVA DEL EMBARAZO
Subtema: PREECLAMPSIA

CASO CLÍNICO SERIADO

PRIMIGESTA DE 16 AÑOS DE EDAD CON EMBARAZO DE 30SDG POR FUM CONFIABLE. ACUDE A CONSULTA DE
CONTROL PRENATAL, REFIERE DOLOR ABDOMINAL DE PREDOMINIO EN EPIGASTRIO, CEFALEA LEVE Y
ADECUADA MOVILIDAD FETAL. MANIFIESTA NO HABER ORINADO DESDE HACE 36 HORAS. A LA
EXPLORACIÓN TA 135/80, A LA PALPACIÓN DOLOR EN EPIGASTRIO, ALTURA UTERINA ACORDE A EDAD
GESTACIONAL, PRODUCTO ÚNICO, FRECUENCIA CARDIOFETAL DE 150 LATIDOS POR MINUTO, EXTREMIDADES
INFERIORES CON EDEMA +, REFLEJOS OSTEOTENDINOSOS AUMENTADOS. EL ULTRASONIDO REVELA
PLACENTA GRADO 2 DE GRANNUM. EXAMEN DE ORINA HACE 48HRS CON PROTEINURIA +++.

16 años.

PRIMIGESTA, ADOLESCENTE. Embarazo de


30SDG.

Dolor abdominal, cefalea. anuria(36hrs).

Re ejos osteotendinosos aumentados.

PROTEINURIA.

337 - LO MÁS PROBABLE ES QUE EN ESTE MOMENTO LA PACIENTE ESTE CURSANDO CON:

HIPERPERFUSIÓN La PREECLAMPSIA es un síndrome idiopático heterogéneo multisistémico que se presenta


TISULAR después de la semana 20 de gestación, se caracteriza por hipertensión arterial y proteinuria.
GENERALIZADA. Constituye una causa importante de morbimortalidad materna y perinatal. EN LA
PREECLAMPSIA SE ESPERA UNA “HIPOPERFUSIÓN TISULAR”.
DISMINUCIÓN Los hallazgos clínicos de la PREECLAMPSIA se pueden manifestar como síndrome materno
DE LAS (HIPERTENSIÓN Y PROTEINURIA con o sin manifestaciones multisistémicas) y/ o síndrome fetal
RESISTENCIAS consistente en oligohidramnios, restricción en el crecimiento intrauterino (RCIU) y alteraciones
PERIFÉRICAS. de la oxigenación. Además, estas manifestaciones se pueden presentar antes de las 37 semanas
de gestación, al término del embarazo, en el trabajo de parto o en el puerperio. LAS
RESISTENCIAS PERIFÉRICAS EN LA PREECLAMPSIA ESTÁN AUMENTADAS. RECUERDA QUE
ESTO ES UNA CARACTERÍSTICA DE CUALQUIER TIPO DE HIPERTENSIÓN.

AUMENTO DEL RECUERDA: LO PRIMERO A CONSIDERAR AL CONTESTAR UN REACTIVO ES EL DIAGNÓSTICO,


CONSUMO SI BIEN LAS CIFRAS TENSIONALES PARECEN NO DIAGNÓSTICAS DE PREECLAMPSIA EN ESTE
PLAQUETARIO. CASO TIENES UNA MUJER CON ALTO RIESGO + SIGNOS DE ALARMA + PROTEINURIA. La
PREECLAMPSIA se asocia a DAÑO ENDOTELIAL y por lo tanto puede presentar manifestaciones
prácticamente en todos los aparatos y sistemas, y el feto no es la excepción. Se caracteriza por:
hipoperfusión tisular generalizada relacionada con una respuesta vascular anormal placentaria
que se asocia con un aumento en las resistencias vasculares sistémicas, daño endotelial,
cambios metabólicos, CONSUMO PLAQUETARIO, aumento en la respuesta inflamatoria y
activación del sistema de coagulación.

DISMINUCIÓN Se cree que durante la PREECLAMPSIA hay una mala tolerancia inmunitaria de la madre a los
DE LA antígenos placentarios y fetales derivados del padre. Este trastorno se considera está mediado
RESPUESTA por mecanismos inmunitarios; se cree que esta es una de las razones por la cual la preeclampsia
INFLAMATORIA. se presenta mayormente en primigestas que en aquellas que han tenido un evento obstétrico
previo. LA RESPUESTA INFLAMATORIA EN LA PREECLAMPSIA SUELE ESTAR
CARACTERÍSTICAMENTE AUMENTADA.

Bibliografía:
1. GUÍA DE PRÁCTICA CLÍNICA, ATENCIÓN INTEGRAL DE PREECLAMPSIA EN EL SEGUNDO Y TERCER NIVEL
DE ATENCIÓN. MÉXICO: SECRETARIA DE SALUD; 2011. 2. CUNNINGHAM G, LEVENO K, BLOMM S, HAUTH J,
RPUSE D, SONG C. WILLIAMS OBSTETRICIA, 23A EDICIÓN. MC GRAW HILL. USA. 2011.

http://www.cenetec.salud.gob.mx/descargas/gpc/CatalogoMaestro/020_GPC_Preeclampsia/SS_020_08_EyR.pdf

338 - EL DIAGNÓSTICO CLÍNICO ES:

HIPERTENSIÓN Con base en la última modificación a la GPC para preeclampsia (2017), la HIPERTENSIÓN
GESTACIONAL. GESTACIONAL está definida como la hipertensión que se desarrolla por primera vez después de la
semana 20 de gestación con presión arterial sistólica mayor o igual que 140mmHg y/o presión
arterial diastólica mayor o igual que 90mmHg. Deberá además subclasificarse, en caso de cumplir
con los criterios, como: - Con condición comórbida: por ejemplo, Diabetes mellitus I o II pre
gestacional o enfermedad renal, garantizan un control de la presión sanguínea fuera del
embarazo por su asociación con riesgo cardiovascular elevado. - Con evidencia de preeclampsia:
mismos criterios que para la evidencia en hipertensión preexistente. PUESTO QUE EXISTE
PROTEINURIA NO PUEDE CLASIFICARSE COMO HIPERTENSIÓN GESTACIONAL.
PREECLAMPSIA OJO: LA CLASIFICACIÓN DE LAS ENFERMEDADES HIPERTENSIVAS DURANTE EL EMBARAZO FUE
LEVE. RECIENTEMENTE MODIFICADA. Los DESÓRDENES HIPERTENSIVOS EN EL EMBARAZO pueden ser
clasificados como (Magee L, 2014): a) Hipertensión Preexistente. - Con condiciones comórbidas. -
Con evidencia de preeclampsia b) Hipertensión gestacional. - Con condición comórbida. - Con
evidencia de preeclampsia. c) Preeclampsia. d) Otras Formas Hipertensivas: - Hipertensión
transitoria Hipertensión de bata blanca e) Hipertensión enmascarada. La evidencia de
preeclampsia puede aparecer muchas semanas después del inicio de la hipertensión gestacional.
Está definida como hipertensión que se presenta en el embarazo (TA sistólica mayor o igual que
140 mmHg o TA diastólica mayor o igual que a 90mmHg en 2 ocasiones con un margen de al
menos 4 horas después de la semana 20 de gestación en una mujer con presión arterial normal
antes del embarazo) y uno o más de los siguientes puntos: Proteinuria de nueva aparición (igual
o mayor a 300 mg en recolección de orina de 24 hrs) ó Labstix con lectura de 1+ solo si no se
dispone de otro método cuantitativo ó Relación proteína/creatinina ? a 0.28 mg/dL ó en ausencia
de proteinuria, 1 ó más condiciones adversas. En la actualidad el Colegio Americano de
Ginecólogos y Obstetras (ACOG) integra a la preeclampsia con y sin características de severidad,
lo que anteriormente se conocía como preeclampsia leve y severa, y es recomendable, según la
GPC actual que, en la mujer con embarazo y trastorno hipertensivo, sea clasificada la presencia o
ausencia de características de severidad de acuerdo con los criterios establecidos por este
Colegio. ESTA CLARO QUE SE TRATA DE UNA PREECLAMPSIA. LA PACIENTE PRESENTA VARIAS
CARACTERÍSTICAS DE SEVERIDAD, POR LO QUE NO PUEDE CLASIFICARSE COMO UNA
PREECLAMPSIA SIN CARACTERÍSTICAS DE SEVERIDAD (ANTES DENOMINADA PREECLAMPSIA
LEVE).
PREECLAMPSIA La PREECLAMPSIA CON CRITERIOS DE SEVERIDAD (anteriormente denominada preeclampsia
SEVERA. severa) es aquella que se acompaña de uno o más de los siguientes criterios: - Síntomas
maternos: “cefalea” persistente o “de novo”; alteraciones visuales o cerebrales; “epigastralgia” o
dolor en hipocondrio derecho; dolor torácico o disnea, signos de disfunción orgánica, y en caso
de “hipertensión severa” (sistólica mayor o igual que 160 y/o diastólica mayor o igual que 110
mm Hg); edema agudo pulmonar o sospecha de desprendimiento placentario. - Alteraciones de
laboratorio: Elevación de creatinina sérica (> 1.1 mg/dL), incremento de AST o ALT (> 70 IU/L) o
deshidrogenasa láctica; disminución de plaquetas < 100,000/ mm3. Las complicaciones severas
asociadas a órgano o sistema afectado en la preeclampsia con datos de severidad abarcan
principalmente: - Sistema nervioso central: TA mayor o igual que 160 de presión sistólica y/o
mayor o igual que 110 mm Hg de presión diastólica “en dos ocasiones con al menos 15 minutos
de diferencia en el mismo brazo”, con la paciente en reposo y sin uso de antihipertensivos
previos; eclampsia, síndrome de leucoencefalopatía posterior reversible, ceguera cortical,
desprendimiento de retina, Glasgow menor a 13, Accidente vascular cerebral, ataque isquémico
transitorio, déficit neurológico reversible. - Cardiorrespiratorio: Hipertensión severa no
controlada (utilizando 3 agentes antihipertensivos), saturación de oxígeno menor a 90, necesidad
de oxígeno suplementario para mantener la saturación por arriba de 50% por 1 hora, intubación,
edema agudo pulmonar, soporte inotrópico positivo, isquemia miocárdica o infarto. -
Hematológico: elevación del conteo de leucocitos, elevación del tiempo de protrombina o
púrpura trombocitopénica, conteo plaquetario disminuido menor a 100000/microlitro. - Renal:
lesión renal agudo (creatinina >1.1mg/dl o al doble de la concentración sérica de creatinina),
indicación de diálisis. - Hepático: disfunción hepática (INR >2 en ausencia de coagulación
vascular diseminada), elevación de deshidrogenasa láctica >600UI/ml, “hematoma hepático” o
ruptura. - Unidad feto placentaria: desprendimiento prematuro de placenta normoinserta, onda A
reversa del ductus venoso, muerte fetal. NOTA: la presencia de hipertensión severa, dolor en
epigastrio, dolor en hipocondrio derecho, náusea y vómito severo aún, cuando no se demuestre
de manera inmediata proteinuria, incrementan la certeza diagnóstica de preeclampsia y ameritan
hospitalización urgente, preferentemente en unidades que cuenten con terapia intensiva.
IMPORTANTE: recuerda que estos reactivos son tipo ENARM y que, aunque la tensión arterial es
un signo definitivo de preeclampsia, es probable encontrar en el examen algún reactivo como
este con cifras de TA por debajo de 140/90mmHg, por lo cual te recomendamos tener en cuenta
los siguientes puntos al hacer el análisis de este caso. 1. Si bien, las cifras de TA son elementales
para hacer el diagnóstico de preeclampsia, por definición y sobre todo en pacientes sintomáticas,
como es el caso, deberán revisarse al menos “en dos ocasiones con al menos 15 minutos de
diferencia en el mismo brazo”. 2. Los datos clínicos de epigastralgia, anuria y proteinuria +++
son, por si solos, un foco de alarma importante para considerar la presencia de preeclamsia, por
tal motivo no podemos descartar esta patología con una sola toma de TA. 3. En pacientes con
signos tan graves, como se muestra en el caso y, con cifras tensionales “aparentemente
normales” siempre habrá de descartarse hemorragia, especialmente hematoma hepático, lo que
pudiera estar llevando a la paciente a un estado de choque, de ahí que las cifras tensionales
parezcan dentro de los parámetros normales, pero en realidad se están reduciendo por la
evolución propia de esta complicación. 4. Todas las opciones diagnósticas están dirigidas a
enfermedades hipertensivas del embarazo, por lo que, únicamente con los datos que el caso
clínico te ofrece deberás elegir la respuesta más correcta, la cual claramente es una
PREECLAMPSIA SEVERA. EL CONJUNTO DE SIGNOS Y SÍNTOMAS QUE PRESENTA LA PACIENTE,
ESPECIALMENTE LA EPIGRASTRALGIA, ANURIA Y PROTEINURIA, FUNDAMENTAN ESTE
DIAGNÓSTICO.

ECLAMPSIA. La ECLAMPSIA es la fase convulsiva de la enfermedad hipertensiva (preeclampsia) y se encuentra


entre las más graves manifestaciones de la enfermedad. A menudo es precedido por
acontecimientos premonitorios, tales como dolores de cabeza e hiperreflexia, pero puede ocurrir
en ausencia de signos. OJO: aunque generalmente se presenta en el contexto de una
preeclampsia, la eclampsia se presenta tras una serie de signos y síntomas que puede o no tener
hipertensión o proteinuria o edema o una combinación de todos. Es recomendable el
interrogatorio sobre síntomas clínicos premonitorios que pueden ayudar a predecir la aparición
de eclampsia, entre ellos se encuentran: cefalea occipital o frontal persistente, fosfenos,
fotofobia, epigastralgia, dolor en cuadrante superior derecho o ambos y alteraciones del estado
mental. NO SE REFIERE LA PRESENCIA DE CONVULSIONES EN ESTE MOMENTO.

Bibliografía:
1. PREVENCIÓN, DIAGNÓSTICO Y TRATAMIENTO DE LA PREECLAMPSIA EN EL SEGUNDO Y TERCER
NIVELES DE ATENCIÓN. EVIDENCIAS Y RECOMENDACIONES. GUÍA DE PRÁCTICA CLÍNICA. MÉXICO,
SECRETARÍA DE SALUD; 09/03/17.

http://www.cenetec-difusion.com/CMGPC/S-020-08/ER.pdf
FIN DEL CASO CLÍNICO SERIADO
ANÁLISIS DEL CASO CLÍNICO

IDENTIFICACIÓN DEL REACTIVO


Area: CIRUGÍA
Especialidad: TRAUMATOLOGÍA Y ORTOPEDIA
Tema: INFECCIONES OSEAS Y ARTICULAES
Subtema: OSTEOMIELITIS Y ARTRITIS SÉPTICA

CASO CLÍNICO SERIADO

ESCOLAR DE 11 AÑOS, CON ANTECEDENTE DE TRAUMATISMO DIRECTO EN MUSLO DERECHO, HACE 12 HORAS
PRESENTA DOLOR AGUDO EN LA ZONA DISTAL DEL MUSLO MOTIVO POR LO QUE ACUDE A LA CONSULTA. A
LA EXPLORACIÓN CON TEMPERATURA DE 38°C, INFLAMACIÓN EN EL TERCIO INFERIOR DEL MUSLO
DERECHO Y DOLOR A LA MOVILIZACIÓN, LIMITACIÓN FUNCIONAL DE CADERA Y RODILLA. BIOMETRÍA
HEMÁTICA CON LEUCOCITOSIS Y LA VELOCIDAD DE SEDIMENTACIÓN GLOBULAR AUMENTADA.
RADIOGRAFÍAS DE FÉMUR NORMALES.

La edad es útil para determinar el agente


etiológico mas probable.

DOLOR AGUDO EN LA ZONA DISTAL DEL


MUSLO.

Hipertermia de 38°c In amación en el


tercio inferior del muslo derecho y dolor a
la movilización, limitación funcional de
cadera y rodilla.

leucocitosis vsg aumentada (indicativa de


proceso in amatorio, infeccioso o tumoral).

339 - EL GERMEN PATÓGENO MÁS PROBABLE DE ESTA PATOLOGÍA ES:

STAPHILOCOCUS En este caso debemos tomar en cuenta los datos que nos están proporcionando, es importante
AUREUS. hacer notar que el cuadro clínico concuerda y nos hará sospechar de un proceso infeccioso por
lo que debemos pensar en las causas más probables de infección en este zona, OSTEOMIELITIS
O ARTRITIS SÉPTICA. Tanto a la edad escolar como en el adolescente, el agente etiológico más
común que provoca esta infección es el estafilococo. TANTO PARA ARTRITIS COMO PARA
OSTEOMIELITIS EL AGENTE CAUSAL MÁS FRECUENTE ES EL S. AUREUS. Recuerda, la edad será
fundamental para sospechar sobre otros agentes etiológicos.
ENTEROBACTER. En el caso de la OSTEOMIELITIS la etiología por edad se divide de la siguiente manera: 1.
Neonatos (menores de 4 meses): S. Aureus, Enterobacter Sp, y Streptococcus grupo A y grupo
B. 2. Lactantes y preescolares (4 m a 4 años): S. Aureus, Streptococcus Pyogenes, Haemophilus
Influenzae, y especies de Enterobacter. 3. Escolares y adolescentes (de 4-19 años): S. Aureus
(80%), S. Pyogenes, H. influenzae, y especies de Enterobacter. 4. Adultos: S. Aureus y
ocasionalmente Enterobacter o especies del Streptococcus. En el caso de la ARTRITIS SÉPTICA:
El germen causal más frecuente es el estafilococo Auereus en el 70% a 80% de los casos, en
segunda frecuencia está el estreptococo y gonococo. Los gérmenes Gram negativos son menos
frecuentes (Escherichia Coli, Pseudomonas, Proteus, Haemophilus influenza, Serratia) pero hay
una mayor incidencia de infecciones articulares por ellos, en los recién nacidos y lactantes,
especialmente cuando cursan con sepsis a Gram negativos; lo mismo ocurre en pacientes de
edad avanzada, donde encontramos la mayor parte de las infecciones urinarias y sistémicas.
Entonces partimos de que ambas patologías infecciosas son causadas en mayor frecuencia por
el estafilococo tomando en cuenta la edad del paciente, por lo tanto con ello podemos
descartar el resto de las opciones que aunque también son agentes causales de infección no son
los más frecuentes en la zona anatómica comentada ni en la edad del paciente y debido a que la
opción de respuesta comentada es S. Aureus la patología de que debemos deducir se trata es de
osteomielitis aguda.

ESCHERICHIA CUADRO CLÍNICO: La osteomielitis se produce con frecuencia durante la infancia, con una
COLI. media de edad de 6 años. Es excepcional en el período neonatal. Se encuentra preferentemente
en las metáfisis más fértiles: cerca de la rodilla, por debajo del codo. En 30 a 40% de los casos,
se observa la aparición de una lesión menor en los días anteriores a la aparición de la infección.
En el caso típico, es un varón (preferencia 3/1) que se queja de dolor en el extremo de un hueso
largo. El dolor tiene un comienzo súbito, ocasionalmente acompañado de fractura y una
deficiencia funcional de la extremidad afectada. La suave movilización de la articulación
adyacente es posible. El síndrome infeccioso está marcado con una fiebre superior a 38°,
deterioro del estado general, escalofríos y sudoración. En la fase inicial, los signos locales son
inespecíficos. Más adelante en el curso de la infección, hay un aumento del volumen e
inflamación localizada. Por lo general, no se palpan linfadenopatías. Otros eventos que pueden
ser hallados en una osteomielitis en particular en un contexto febril incluyen: un estado de
agitación o postración; pseudoparálisis de un miembro de un bebé o recién nacido;
convulsiones; claudicación.

PSEUDOMONAS PATOGENIA: generalmente los microorganismos se han diseminado hasta el hueso por vía
AERUGINOSA. hematógena, por ejemplo, a través del torrente sanguíneo. También se esparcen
contagiosamente al hueso desde áreas locales de infección, así como la celulitis o bien por un
trauma penetrante, incluyendo causas iatrogénicas como reemplazos articulares o la fijación
interna de fracturas o de canalizaciones (Endodoncias, Tratamientos de Conducto) de la raíz de
los dientes. Una vez que el hueso está infectado, los leucocitos entran en la zona infectada, y en
su intento de eliminar los organismos infecciosos, liberan enzimas que corren ail hueso. La pus
se extiende por los vasos sanguíneos del hueso, ocluyéndolos y formando abscesos lo cual priva
al hueso del flujo sanguíneo, y se forman en la zona de la infección áreas de hueso infectado
desvitalizado, conocido como secuestro óseo, que forma la base de una infección crónica. A
menudo, el organismo tratará de crear nuevo hueso alrededor de la zona de necrosis. El nuevo
hueso así resultante recibe el nombre de involucrum. En el examen histológico, estas áreas de
necrosis de los huesos son la base para distinguir entre la osteomielitis aguda y la osteomielitis
crónica. La osteomielitis es un proceso infeccioso que abarca la totalidad de los huesos y sus
componentes, incluida la médula ósea y el periostio. Cuando es crónica puede dar lugar a
esclerosis ósea y deformidad. En los niños, por lo general, se afectan usualmente los huesos
largos, mientras que en los adultos se afectan más comúnmente las vértebras y la pelvis. OJO:
EN EL CASO ESPECIAL DE PSEUDOMONAS AERUGINOSA TENDRÍAS QUE PENSAR
OBLIGADAMENTE EN UN PACIENTE CON INMUNOCOMPROMISO PREVIO.

Bibliografía:
1. GUÍA DE PRÁCTICA CLÍNICA, PREVENCIÓN, DIAGNÓSTICO Y TRATAMIENTO DE LA ARTRITIS SÉPTICA
AGUDA EN NIÑOS Y ADULTOS. MÉXICO: SECRETARIA DE SALUD; 2011. 2. GUÍA DE PRÁCTICA CLÍNICA,
PREVENCIÓN, DIAGNÓSTICO Y TRATAMIENTO DE LA ARTRITIS SÉPTICA AGUDA EN NIÑOS Y ADULTOS.
MÉXICO: SECRETARIA DE SALUD; 2011.

http://www.cenetec.salud.gob.mx/descargas/gpc/CatalogoMaestro/368-10_Artritis_Sxptica_Aguda/IMSS-368-10_GER_Artritis_sxptica.pdf

340 - EN CASO DE DEJARSE A LIBRE EVOLUCIÓN SE ESPERA ENCONTRAR CAMBIOS RADIOLÓGICOS EN


UN MÍNIMO DE TIEMPO DE.

3 Se deben tomar radiografías simples con técnica de partes blandas, en proyección antero posterior y lateral
DÍAS. sel sitio afectado. RECUERDA: en las etapas tempranas de la enfermedad, la RESONANCIA MAGNÉTICA
suele mostrar cambios en el hueso más pronto que las radiografías simples.

7 Se utiliza la TÉCNICA PARA PARTES BLANDAS debido a que el proceso osteomielítico NO MUESTRA
DÍAS. ALTERACIONES ÓSEAS EN LOS PRIMEROS DÍAS de iniciado el proceso infeccioso, por lo que lo único que
puede observarse es el aumento de las partes blandas profundas y adyacentes al hueso afectado.

15 Las radiografías simples son útiles para el diagnóstico tardío (más de 15 días) de osteomielitis hematógena
DÍAS. aguda. IMPORTANTE: se sugiere realizar radiografías simples tempranas (menos de 15 días) en pacientes
con sospecha de osteomielitis hematógena aguda para establecer diagnóstico diferencial.

21 Recuerda en como auxiliar secundario se puede utilizar el ULTRASONIDO en etapas tempranas de la


DÍAS. enfermedad, en él se puede observar fluidos subperiósticos y yuxtacorticales y cambios en tejidos blandos.

Bibliografía:
GUÍA DE PRÁCTICA CLÍNICA. PREVENCIÓN, DIAGNÓSTICO Y TRATAMIENTO DE LA OSTEOMIELITIS
HEMATÓGENA AGUDA EN POBLACIÓN GENERAL PARA EL PRIMER Y SEGUNDO NIVELES DE ATENCIÓN.
MÉXICO: SECRETARÍA DE SALUD, 25/SEPTIEMBRE/2014.

http://www.cenetec.salud.gob.mx/descargas/gpc/CatalogoMaestro/111_GPC_Osteomielitisaguda/SSA_111_08_EyR.pdf

341 - DESPUÉS DE 36 HRS DE INICIADOS LOS ANTIBIÓTICOS EL PACIENTE PERSISTE CON


MANIFESTACIONES LOCALES Y SISTÉMICAS, POR LO QUE SE LE DEBE REALIZAR:

COLOCACIÓN DE Gracias a que ya se ha conformado el diagnóstico de OSTEOMIELITIS podemos decir que la


SISTEMAS DE colocación de sistemas de irrigación succión no sería el tratamiento más adecuado y en el
IRRIGACIÓN. determinado caso en que se hubiera llegado al diagnóstico de ARTRITIS SÉPTICA hoy en día
están contraindicados tales sistemas ya que se consideran un foco de entrada de gérmenes
puesto que dejan en contacto la articulación con el medio externo a través de las cánulas de
irrigación succión.

TRACCIÓN DE LA La TRACCIÓN DE EXTREMIDADES puede ser cutánea o esquelética y tiene otras indicaciones tal
PIERNA. es el caso de REDUCCIÓN DE FRACTURAS de cadera, estabilización momentánea de fracturas
de fémur o como tratamiento posterior a una luxación de cadera pero no así para procesos
infecciosos.

INMOVILIZACIÓN La INMOVILIZACIÓN como ya se menciono anteriormente solo estaría indicada por un periodo
DE LA corto en el caso de sospechar de procesos inflamatorios o postraumáticos, NO EN LOS CASOS
EXTREMIDAD. DE PATOLOGÍA INFECCIOSA.

DESBRIDACIÓN La OSTEOMIELITIS requiere de una terapia de antibióticos prolongada, llevando de semanas a


DEL FOCO ÓSEO. meses. Para este fin, a menudo se coloca una línea intravenosa central. La osteomielitis
también puede requerir el desbridamiento quirúrgico. Los casos graves pueden conducir a la
pérdida de una extremidad. Los antibióticos de primera línea de elección son generalmente
glucopéptidos como la vancomicina, determinados por la historia del paciente y las diferencias
regionales al respecto de los organismos infecciosos. Se pueden utilizar también rifampicina y
tetraciclinas. EN CASO DE FALLA ANTIBIÓTICA ESTÁ INDICADA LA DESBRIDACIÓN
QUIRÚRGICA.

Bibliografía:
1. BRUNICARDI F, ANDERSEN D, BILLIAR T, Y COLS. SCHWARTZ PRINCIPIOS DE CIRUGÍA, 9A EDICIÓN. MC
GRAW HILL. 2011, PP 1550-1551.

FIN DEL CASO CLÍNICO SERIADO


ANÁLISIS DEL CASO CLÍNICO

IDENTIFICACIÓN DEL REACTIVO


Area: PEDIATRÍA
Especialidad: INFECTOLOGIA PEDIÁTRICA
Tema: HEPATITIS INFECCIOSA
Subtema: HEPATITIS A

CASO CLÍNICO SERIADO

ESCOLAR FEMENINA DE 10 AÑOS DE EDAD, HACE UNA SEMANA PRESENTA FIEBRE DE 39 °C, ATAQUE AL
ESTADO GENERAL, ASTENIA, ADINAMIA, DOLOR ABDOMINAL Y SE AGREGA HOY ICTERICIA CONJUNTIVAL Y DE
TEGUMENTOS, VÓMITOS. A LA EXPLORACIÓN CON ATAQUE AL ESTADO GENERAL, ICTERICIA GENERALIZADA,
DOLOR ABDOMINAL EN HIPOCONDRIO DERECHO, HÍGADO DE 4-4-5 CM Y BAZO DE 4 CM.

Escolar de 10 años de edad

Astenia, adinamia y dolor abdominal

Síndrome infeccioso, ictericia,


hepatomegalia más hepatalgia en paciente
pediátrico; es hepatitis hasta no demostrar
lo contrario

342 - LO MÁS PROBABLE ES QUE LA PACIENTE ESTE CURSANDO CON UNA INFECCIÓN POR:
VIRUS MONONUCLEOSIS INFECCIOSA. La mononucleosis infecciosa, es una infección causada por el virus de
EPSTEIN- Epstein-Barr. Se manifiesta en forma típica por fiebre, faringitis exudativa, linfadenopatía,
BARR hepatoesplenomegalia y linfocitosis atípica. Las manifestaciones patológicas son muy amplias y varían
desde un cuadro sin síntomas hasta una infección letal. Las infecciones por lo común pasan
inadvertidas en lactantes y niños de corta edad. Puede surgir alguna erupción y es más frecuente en
niños tratados con ampicilina y con otras penicilinas. Entre las complicaciones del sistema nervioso
central (SNC) están meningitis aséptica, encefalitis y síndrome de Guillain-Barré. Complicaciones raras
son rotura del bazo, trombocitopenia, agranulocitosis, anemia hemolítica, síndrome hemofagocítico,
orquitis y miocarditis. La réplica del virus de Epstein-Barr (Epstein-Barr virus, EBV) por linfocitos B y la
linfoproliferación resultante suelen ser inhibidas por las respuestas de células T y células citolíticas
naturales. En personas que tienen inmunodeficiencias de tipo celular congénitas o adquiridas, puede
haber una infección diseminada letal o linfomas de células B. NO SE DESCRIBE LA PRESENCIA DE
LINFADENOPATÍAS NI FARINGITIS, LO QUE HACE POCO PROBABLE ESTE DIAGNÓSTICO.

VIRUS HEPATITIS C. Los signos y los síntomas de la infección por el virus de hepatitis C (Hepatitis C virus,
HEPATITIS HCV) son prácticamente idénticos a los de las hepatitis A o B. El cuadro agudo tiende a ser menos
C intenso y su comienzo es más insidioso, y casi todas las infecciones son asintomáticas. Se observa
ictericia en menos de 20% de los pacientes, y las anormalidades en las pruebas de función hepática por
lo común son menos intensas que las que surgen en personas con infección por virus de hepatitis B. La
infección persistente con HCV se observa en 50 a 60% de los niños infectados, incluso sin que haya
pruebas bioquímicas de ataque del hígado. Casi todos los niños con infección crónica son
asintomáticos. A pesar de que la hepatitis crónica aparece en 60 a 70 % de los adultos infectados,
algunos datos indican que menos del 10% de los niños infectados terminan por mostrar hepatitis
crónica, y que menos de 5% muestran al final cirrosis. La infección por HCV es la principal razón para
la práctica de trasplante de hígado en adultos en Estados Unidos de América. CAUSAS: El virus de
hepatitis C es un virus pequeño de RNA de un solo filamento y es miembro de la familia de Flavivirus.
Existen múltiples genotipos y subtipos de dicho virus. Epidemiología. Se ha calculado que la
prevalencia de infección por HCV en la población general estadounidense es de 1.8%. La
seroprevalencia es de 0.2 % en menores de 12 años y 0.4% en adolescentes de 12 a 19 años. La
seroprevalencia varía de una población a otra según los factores de riesgo concomitantes. La infección
se propaga más bien por exposición parenteral a sangre de sujetos infectados por HCV. Las mayores
seroprevalencias de la infección por HCV (60 a 90%) se observan en personas con exposición
percutánea directa, grande o repetitiva a la sangre o los hemoderivados, como serían los usuarios de
drogas inyectables y hemofílicos tratados con concentrados del factor de coagulación producidos
antes de 1987. Las prevalencias son moderadamente altas en sujetos con exposiciones percutáneas
directas frecuentes pero pequeñas, como las personas sometidas a hemodiálisis (10 a 20 %). Las
prevalencias más bajas se detectan en individuos con exposiciones percutáneas o de mucosas, no
manifiestas, como individuos con comportamiento sexual de alto riesgo (1 a 10 %) y en sujetos con
exposiciones percutáneas esporádicas, como los profesionales asistenciales (1%). Se ha calculado que
la seroprevalencia en embarazadas estadounidenses va de 1 a 2%. El peligro de transmisión de madre
a hijo (perinatal) es de 5 a 6%, en promedio, y la transmisión se observa sólo en mujeres que tienen
RNA de HCV en el momento del parto. El periodo de incubación de la hepatitis C (enfermedad) es de
seis a siete semanas, en promedio, con límites de dos semanas a seis meses. El lapso que media, desde
la exposición hasta la aparición de la viremia suele ser de una a dos semanas. LA CLAVE PARA
DESCARTAR HEPATITIS C SON LOS MECANISMOS DE TRANSMISIÓN; LA MATERNA, SE DESCARTA
POR EL TIEMPO DE INCUBACIÓN Y EDAD DE LA PACIENTE Y, OTRAS VÍAS DE TRANSMISIÓN, PORQUE
EN NINGÚN MOMENTO SE SEÑALA ALGÚN FACTOR DE RIESGO ASOCIADO (TRANSFUSIONES,
ACTIVIDAD SEXUAL DE RIESGO).
VIRUS HEPATITIS B. Las personas infectadas del virus de hepatitis B (Hepatitis B virus, HBV) pueden mostrar
HEPATITIS signos y síntomas variados, que incluyen un cuadro subagudo con manifestaciones inespecíficas
B (anorexia, náuseas o malestar generalizado), hepatitis clínica con ictericia o la forma letal fulminante.
Es frecuente que haya seroconversión asintomática, y la posibilidad de que surjan los síntomas de la
enfermedad depende de la edad de la persona. En niños de corta edad, es más frecuente la infección
anictérica o asintomática. En los comienzos de la evolución pueden surgir manifestaciones
extrahepáticas, como artralgias, artritis, erupciones maculosas, trombocitopenia o acrodermatitis
papulosa (Síndrome de Gianotti-Crosti), antes de que surja la ictericia. Es difícil distinguir entre la
hepatitis B aguda y otras formas de hepatitis aguda por virus; con base sólo en los signos y síntomas
clínicos o datos inespecíficos de estudios de laboratorio. El elemento determinante del riesgo de que
el trastorno evolucione hasta volverse crónico es la edad en que ocurrió la infección aguda. Más de 90
% de los lactantes infectados en fase perinatal terminarán por mostrar infección crónica (HBV). De 25
a 50 % de los niños infectados cuando tenían entre uno y cinco años de edad terminarán siendo
enfermos crónicos, en tanto que sólo entre el 6 al 10% de niños mayores y adultos con la infección
aguda terminarán por mostrar la infección crónica. Etiología: La partícula infectante es un
hepadnavirus de 42 mm. de diámetro que contiene DNA. Entre los componentes importantes de tal
partícula están los antígenos de superficie, el antígeno central y el antígeno e. El anticuerpo contra el
antígeno de superficie (Anti-HBs) protege de la infección por dicho virus. Epidemiología: La hepatitis B
se transmite por sangre o líquidos corporales, que incluyen exudado de heridas, semen, secreciones
cervicouterinas y saliva. La sangre y el suero son los líquidos que contienen el número máximo de
virus, en tanto que la saliva contiene el número más bajo. Las personas con infección crónica son los
reservorios primarios de la enfermedad. Entre los mecanismos frecuentes de transmisión están la
exposición percutánea y de mucosas a líquidos corporales infectantes, uso compartido de jeringas o
agujas no esterilizadas; contacto sexual con una persona infectada y exposición perinatal a la madre
infectada. La propagación directa, es decir, de persona a persona, puede surgir en situaciones que
incluyen el contacto interpersonal por largo tiempo, como cuando una persona con infección crónica
reside en un núcleo familiar. En el núcleo en cuestión, la transmisión no sexual surge más bien de un
niño a otro, y los niños de corta edad estarán expuestos al máximo riesgo de ser infectados. Se
desconocen los mecanismos exactos de transmisión directa de un niño a otro; sin embargo, la forma
más posible de transmisión quizá sea el contacto interpersonal frecuente de la piel o las mucosas no
intactas con secreciones sanguinolentas, o tal vez saliva. También se observa transmisión al compartir
objetos inanimados como toallas y sábanas, maquinillas de rasurar o cepillos dentales. El virus de la
hepatitis B sobrevive en el entorno una semana o más, pero es inactivado por los desinfectantes de
uso común, como serían los desinfectantes y blanqueadores caseros diluidos al 1:10 con agua. El virus
en cuestión no es transmitido por la vía fecal-oral. La transmisión perinatal del virus es muy eficiente y
suele ocurrir por exposiciones de sangre durante la primera y segunda fases del parto. Es rara la
transmisión en útero, y causa menos de 2 % de las infecciones perinatales, en casi todos los estudios.
El peligro de que un producto se contagie de HBV de su madre infectada es de 70 a 90 % en el caso de
hijos de madres que tienen los antígenos de superficie y e; el riesgo es de 5 a 20 % si el pequeño es
hijo de una madre sin el antígeno e. Otros niños de corta edad en peligro de infectarse incluyen: 1) Los
contactos del núcleo familiar de personas con infección crónica por HBV; 2) Niños internados en
instituciones para discapacitados del desarrollo; 3) Niños a quienes se practica hemodiálisis, y 4)
Menores con coagulopatías y otros que reciben repetidamente hemoderivados. La infección aguda por
HBV surge más a menudo en adolescentes y adultos en Estados Unidos de América. Los grupos de
máximo riesgo incluyen quienes usan drogas inyectables, personas con múltiples compañeros sexuales
y varones jóvenes homosexuales. ESTE DIAGNÓSTICO ES POCO PROBABLE YA QUE NO SE REFIEREN
FACTORES DE RIESGO PARA HEPATITIS B, ACTUALMENTE LA VACUNA AL NACIMIENTO DISMINUYE
DE MANERA IMPORTANTE LA TRANSMISIÓN PERINATAL, ADEMÁS DE QUE ESTA PACIENTE NO SE
ENCUENTRA DENTRO DE LOS GRUPOS DE MÁXIMO RIESGO.
VIRUS HEPATITIS A. De manera característica, la infección por el virus de hepatitis A (Hepatitis A virus, HAV)
HEPATITIS incluye un cuadro agudo que cede por sí solo, compuesto de fiebre, malestar generalizado, ictericia,
A anorexia y náuseas. La infección sintomática por dicho virus se observa en cerca de 30% de niños
infectados que tienen menos de seis años de vida; pocos de ellos mostrarán ictericia. En niños de
mayor edad y adultos, la infección casi siempre es sintomática y se prolonga algunas semanas; en 70%
de los casos, en promedio, aparece ictericia. Se observan a veces casos de enfermedad prolongada o
recidivante que puede durar incluso seis meses. La hepatitis fulminante es rara, pero es más frecuente
en sujetos con alguna hepatopatía primaria; no se observa infección crónica. Etiología: La partícula
infectante es un virus de RNA clasificado como miembro del grupo Picornavirus. Epidemiología: El
mecanismo común de transmisión es el de persona a persona, que es consecuencia de la
contaminación fecal y la ingestión del patógeno (La vía fecal-oral). La edad en que se produce la
infección varía con el estado socioeconómico y las condiciones de vida. En países en desarrollo donde
la infección es endémica, casi todos los individuos se infectan en el primer decenio de su vida. Entre
los casos de infección de hepatitis A notificados a los CDC, las fuentes de infección identificadas
incluyen contacto personal muy cercano con una persona infectada con el virus de hepatitis A,
contactos domésticos o personales en un centro de atención pediátrica, viajes internacionales, un
brote en que se sabe que el germen fue transmitido por alimentos o agua, actividad homosexual en
varones y el uso de drogas inyectables. Rara vez la partícula se transmite por transfusión de sangre, o
de la madre a su recién nacido, es decir, transmisión vertical. En contadas ocasiones la infección ha
provenido de primates no humanos y que no nacieron en cautiverio. En cerca de la mitad de los casos
notificados, es imposible detectar su origen. Es probable que muchos de estos casos de origen
desconocido hayan provenido del contagio fecal-oral con personas que tenían infecciones
asintomáticas y en particular niños de corta edad. El periodo de incubación es de 15 a 50 días, con un
promedio de 25 a 30 días. ESTA ES LA HEPATITIS INFECCIOSA MÁS COMÚN Y CONCUERDA CON EL
CUADRO CLÍNICO DESCRITO EN EL CASO CLÍNICO.

Bibliografía:
RUDOLPH'S PEDIATRICS. ABRAHAM M. RUDOLPH. APPLETON AND LANGE. EDICIÓN 20A. 1996. PÁG.
647,648,649.

343 - EL ESTUDIO NECESARIO PARA CONFIRMAR EL DIAGNÓSTICO ES:

PCR PARA MÉTODOS DIAGNÓSTICOS HEPATITIS C: Los dos tipos principales de métodos para el diagnóstico
VIRUS de laboratorio de infecciones por virus de la hepatitis C (HCV), son las mediciones de anticuerpos
HEPATITIS C en busca de anticuerpo contra HCV y estudios de ácido nucleico (Nucleic acid tests, NAT) para
detectar RNA de HCV. El diagnóstico por medio de los títulos de anticuerpos comprende la
práctica de inmunoanálisis enzimático inicial; la repetición de los resultados positivos se confirma
por medio de inmunotransferencia recombinante (Recombinant Immunoblot Assay, RIBA). Los dos
métodos detectan el anticuerpo de inmunoglobulina (Ig) G; no se cuenta con métodos para
valorar IgM. Los NAT diagnósticos aprobados en Estados Unidos por la Food and Drug
Administration (FDA) para la detección cualitativa de HCV utilizan la reacción en cadena de
polimerasa-transcriptasa inversa (Reverse transcriptase-polymerase chain reaction, RT-PCR). Es
posible detectar el RNA del virus de hepatitis C en suero o plasma, en término de una a dos
semanas después de exposición al virus y semanas antes de que comiencen las anormalidades en
las enzimas hepáticas o la aparición de anticuerpo contra HCV. En la práctica clínica se utilizan
comúnmente los métodos para identificar RNA de HCV, como la cadena de polimerasa-
transcriptasa inversa, en el diagnóstico inicial de la infección, para identificar la infección en
lactantes desde los comienzos de la vida (Transmisión perinatal), momento en que los anticuerpos
séricos de la madre interfieren en la capacidad de detectar los anticuerpos producidos por el
lactante, y para vigilar a pacientes que reciben antivíricos.

IGM CONTRA MÉTODOS DIAGNÓSTICOS HEPATITIS A: En el comercio se cuenta con pruebas serológicas para
VIRUS DE LA detectar anticuerpos de inmunoglobulina IgM totales específicos contra el virus de la hepatitis A
HEPATITIS A (HAV). La IgM sérica aparece desde el comienzo de la enfermedad y desaparece en término de
cuatro meses, pero puede persistir seis meses o más. La presencia de IgM en suero denota
infección actual o reciente, aunque pueden surgir resultados positivos falsos. Se puede detectar
IgG contra HAV, poco después de que aparezca IgM. La presencia del anticuerpo contra HAV sin
que surja IgM como anticuerpo contra HAV denota infección pasada e inmunidad. IMPORTANTE:
LA SENSIBILIDAD DE IGM PARA HEPATITIS A ES DEL 100%.
ANTICUERPOS MÉTODOS DIAGNÓSTICOS MONONUCLEOSIS INFECCIOSA: Es posible aislar el virus de Epstein-
HETERÓFILOS. Barr de las secreciones bucofaríngeas, pero en el laboratorio de diagnóstico corriente por lo
común no se practican las técnicas para tal finalidad, y el aislamiento del virus no indica
obligadamente la presencia de una infección aguda. Por tal razón, el diagnóstico depende de
métodos serológicos. Los practicados con mayor frecuencia son estudios inespecíficos en busca de
anticuerpo heterófilo, que incluye la prueba de Paul-Bunnell y la reacción de aglutinación en
laminilla. El anticuerpo heterófilo es predominantemente la inmunoglobulina (Ig) M, que surge en
las primeras dos semanas de enfermedad para desaparecer poco a poco en un lapso de seis meses.
Los resultados de las pruebas para valorar el anticuerpo mencionado suelen ser negativos en niños
menores de cuatro años con infección por EBV, pero identifican alrededor de 90% de los casos
(Probados por métodos serológicos específicos de EBV) en niños de mayor edad y en adultos. Un
dato característico, aunque inespecífico, es el incremento absoluto del número de linfocitos
atípicos en la segunda semana de la enfermedad, con mononucleosis infecciosa. Sin embargo,
detectar más de 10% de linfocitos atípicos junto con la positividad de una prueba de anticuerpo
heterófilo se considera como datos que permiten diagnosticar una infección aguda. Los estudios
serológicos del virus de Epstein-Barr particularmente son útiles para valorar a pacientes que tienen
mononucleosis infecciosa sin anticuerpo heterófilo. En tales enfermos quizá también convenga
buscar otros antígenos víricos, en particular el virus citomegálico. En estudios de investigación,
métodos como el cultivo de muestras de saliva o la búsqueda de EBV mononucleares de sangre
periférica, la hibridación de DNA in situ o la reacción en cadena de polimerasa, permitirán
identificar la presencia de EBV o su DNA, y quizá se pudiera asignar una participación causal del
virus en un síndrome como el de linfoproliferación.

ANTIGENO DE MÉTODOS DIAGNÓSTICOS HEPATITIS B: Se cuenta en el comercio con métodos serológicos para
SUPERFICIE detectar los antígenos de superficie y e (HBsAg y HBeAg). También se cuenta con técnicas para
DEL VIRUS DE detectar anticuerpos contra el antígeno de superficie (anti-HBs), anticuerpos totales contra el
HEPATITIS B antígeno central (anti-HBc), anticuerpo IgM contra HBc y anticuerpo contra el antígeno e.
Además, se dispone de métodos de hibridación y de amplificación génica (Reacción en cadena de
polimerasa, métodos para DNA ramificado) para detectar y cuantificar el DNA de virus de hepatitis
B (HBV). Durante la infección aguda se puede detectar el antígeno de superficie de hepatitis B. Si
la infección cede por sí sola, en casi todos los pacientes desaparece el antígeno s antes de que se
detecte la presencia de anticuerpos contra él (Ha recibido el nombre de fase de ventana de la
infección). El anticuerpo IgM contra el antígeno central es altamente específico para corroborar el
diagnóstico de infección aguda, porque está presente ya desde los comienzos de la infección y en
la fase de ventana de niños mayores y adultos. Sin embargo, el anticuerpo contra el antígeno
central por lo común no se identifica en lactantes infectados en etapa perinatal. Las personas con
infección crónica tienen en su circulación el antígeno de superficie y el anticuerpo contra el
antígeno central; en raras ocasiones también hay anticuerpo contra el antígeno de superficie. Los
anticuerpos contra los antígenos de superficie (s) y central (c) se detectan en personas cuya
infección no ha mostrado resolución, en tanto que el anticuerpo contra el antígeno s, sólo aparece
en individuos que han recibido la vacuna contra hepatitis B. La presencia del antígeno e, en el
suero guarda relación con un número mucho mayor de títulos de HBV y una mayor infectividad.
Los métodos para detectar DNA de HBV y antígeno e, son útiles para seleccionar a candidatos que
recibirán antivíricos y vigilar su respuesta a tal farmacoterapia.

Bibliografía:
RUDOLPH'S PEDIATRICS. ABRAHAM M. RUDOLPH. APPLETON AND LANGE. EDICÓN 20A. 1996. PÁG.
647,648,649.

FIN DEL CASO CLÍNICO SERIADO


ANÁLISIS DEL CASO CLÍNICO

IDENTIFICACIÓN DEL REACTIVO


Area: MEDICINA INTERNA
Especialidad: INFECTOLOGÍA
Tema: INFECCIONES FEBRILES SISTÉMICAS
Subtema: SEPSIS Y CHOQUE SÉPTICO

CASO CLÍNICO SERIADO

MUJER DE 80 AÑOS DE EDAD, CON DIAGNÓSTICO DE CÁNCER DE PÁNCREAS; DESDE HACE 24 HORAS
PRESENTA FIEBRE DE 39ºC, SOMNOLENCIA Y DISMINUCIÓN DEL ESTADO DE ALERTA. A LA EXPLORACIÓN SE
ENCUENTRA FC 150/MIN, FR 30/MIN, TEMPERATURA 40°C, T.A. 70/40MMHG; ICTÉRICA, CON POCA RESPUESTA A
ESTÍMULOS Y DOLOR A LA PALPACIÓN PROFUNDA EN HIPOCONDRIO DERECHO. LABORATORIOS:
LEUCOCITOS 34,000, HEMOGLOBINA 16 GR/DL. BILIRRUBINAS TOTALES DE 25 MG/DL A EXPENSAS DE
BILIRRUBINA DIRECTA. ULTRASONIDO HEPÁTICO CON DILATACIÓN INTRA Y EXTRAHEPÁTICA DE LA VÍA BILIAR.
LA PACIENTE NO RESPONDE A LA INFUSIÓN DE LÍQUIDOS POR LO QUE SE INICIA MANEJO CON AMINAS
VASOACTIVAS.

Mujer de 80 años de edad.

cáncer de páncreas.-.

cuadro clínico con una evolución de 24


horas con somnolencia, disminución del
estado de alerta y ebre de 39.5ºC.

taquicardica, Taquipnea, ebre,


hipotensión. Ictérica, poca respuesta a
estímulos, dolor a la palpación profunda en
hipocondrio derecho.

leucocitosis de 34,000, bilirrubina total de


25 mg/dl a expensas de la directa.
Ultrasonido hepático con dilatación de la
vía biliar intra y extrahepática.

344 - LA PACIENTE PRESENTA ES ESTE MOMENTO DATOS COMPATIBLES CON:

BACTEREMIA El termino BACTEREMIA se refiere a la existencia de existencia de bacterias cultivables en el


torrente sanguíneo.
CHOQUE El CHOQUE SÉPTICO; es cuando un paciente tiene criterios de sepsis acompañado de hipotensión,
SÉPTICO que no revierte a pesar de una terapia de reanimación hídrica adecuada y que amerita
tratamiento con fármacos vasopresores. EL DIAGNÓSTICO CORRECTO ES CHOQUE SÉPTICO POR
LA FALTA DE RESPUESTA A LA REPOSICIÓN HÍDRICA Y ADMINISTRACIÓN DE AMINAS
VASOACTIVAS.

SÍNDROME DE La RESPUESTA INFLAMATORIA SISTÉMICA es la presentación de al menos dos de los siguientes


RESPUESTA criterios: 1. Temperatura > 38º C o < 36º C. 2. Frecuencia cardiaca > 90 latidos por minuto. 3.
INFLAMATORIA Frecuencia respiratoria > 20 respiraciones por minuto o una PaCO2 < 32 mmHg. 4. Leucocitosis >
SISTÉMICA 12 000 o < 4,000 células/mm3 o > 10% de bandas (células inmaduras) vasopresores. LOS
PACIENTES DESARROLLAN UNA RESPUESTA INFLAMATORIA SISTÉMICA QUE DEBE SER
CLASIFICADA EN ESTE CASO COMO CHOQUE SÉPTICO POR LA FALTA DE RESPUESTA A LA
REPOSICIÓN HÍDRICA Y ADMINISTRACIÓN DE AMINAS VASOACTIVAS.

SEPSIS GRAVE La presencia de alteraciones generales inflamatorias asociadas a infección documentada o su


sospecha establece el diagnóstico clínico de SEPSIS, que para su confirmación requiere estudios
de laboratorio. La presencia de manifestaciones de disfunción orgánicas secundarias a
hipoperfusión en un paciente séptico, establecen el diagnóstico clínico de SEPSIS GRAVE, que
para su confirmación diagnóstica requiere de estudios de laboratorio. La sepsis grave; se define
como sepsis asociada con disfunción de órganos a distancia del sitio de infección, con
hipoperfusión o hipotensión. LA HIPOTENSIÓN EN LA SEPSIS GRAVE SE REVIERTE CON LA
ADMINISTRACIÓN DE SOLUCIONES INTRAVENOSAS.

Bibliografía:
1. GUÍA DE PRÁCTICA CLÍNICA, DIAGNÓSTICO Y TRATAMIENTO DE LA SEPSIS GRAVE Y CHOQUE SÉPTICO
EN EL ADULTO. MÉXICO: SECRETARIA DE SALUD, 2010. 2. LONGO DL, FAUCI AS, KASPER DL, HAUSERSL,
JAMESON JL, LOSCALZOJ. HARRISON. PRINCIPIOS DE MEDICINA INTERNA, 18A EDICIÓN. MC GRAW HILL.
NEW YORK, USA. 2012, PP 2223.

http://www.cenetec.salud.gob.mx/descargas/gpc/CatalogoMaestro/084_GPC_Sepsisgveychoqueseptico/SepsisGrave_y_Choque_ER_CENETEC.pdf

345 - LOS CAMBIOS HEMODINÁMICOS SON EXPLICADOS POR LAS SIGUIENTES ALTERACIONES:

VASODILATACIÓN Actualmente se sabe que la causa de la disfunción orgánica en paciente con sepsis es
MEDIADA POR secundaria "disfunción de la microcirculación en los tejidos". Esta distribución inadecuada
CITOCINAS Y del flujo sanguíneo a los tejidos a nivel de la microcirculación, puede contribuir a una
DISFUNCIÓN inadecuada utilización de oxígeno en los tejidos afectados. También puede contribuir un
MICROCIRCULATORIA defecto en la función mitocondrial con disminución de la producción de ATP. Aunque la
evidencia actual sugieren que la disfunción mitocondrial y de la microcirculación
contribuyen de forma significativa a la falla orgánica que se produce en la sepsis aún
existen muchas preguntas sin respuesta. Se ha propuesto que el óxido nítrico induce
"vasodilatación" en la microcirculación. También el descubrimiento de las citocinas
proinflamatorias fue seguida rápidamente de evidencia que estas proteínas de forma
individual o en conjunto pueden inducir sepsis severa y choque séptico. De hecho se ha
demostrado que los pacientes con sepsis severa no solo tienen estos mediadores
inflamatorios sino que al mismo tiempo tienen varias citocinas antiinflamatorias. LA
ALTERACIÓN EN LA MICROCIRCULACIÓN ESTÁ CARACTERIZADA POR VASODILATACIÓN.

DESEQUILIBRIO En caso de los pacientes con choque séptico existe una reducción del volumen circulante
ENTRE PÉRDIDAS E efectivo, no por perdidas, sino más bien por extravasación de líquidos a los tejidos. Estos
INGRESOS DE pacientes no se encuentran deshidratados. NO HAY DATOS CLÍNICOS QUE SUGIERAN
LÍQUIDO CON PÉRDIDAS.
DESHIDRATACIÓN
CONSECUENTE

GASTO CARDIACO Uno de los mecanismos de producción de la hipotensión en los pacientes con sepsis y
BAJO CON choque séptico es una disminución importante de las resistencias periféricas, debido a la
RESISTENCIAS vasodilatación generalizada de la microcirculación. Al mismo tiempo estos pacientes
PERIFÉRICAS tienen en un inicio un aumento del gasto cardiaco con la finalidad de asegurar la perfusión
AUMENTADAS tisular. LA DILATACIÓN VASCULAR PRODUCIDA DURANTE EL CHOQUE SÉPTICO
DISMINUYE LAS RESISTENCIAS PERIFÉRICAS.
LIBERACIÓN DE La existencia de bacterias en la circulación es un término que se conoce como bacteremia.
BACTERIAS A LA Y nuevamente en la sepsis y choque séptico uno de los mecanismos fisiopatológicos en su
SANGRE CON producción es la vasodilatación de la microcirculación. No hay vasoconstricción. LA
REPERCUSIÓN CARACTERÍSTICA EN ESTE CASO ES VASODILATACIÓN.
SISTÉMICA Y
VASOCONSTRICCIÓN

Bibliografía:
1. LONGO DL, FAUCI AS, KASPER DL, HAUSERSL, JAMESON JL, LOSCALZOJ. HARRISON. PRINCIPIOS DE
MEDICINA INTERNA, 18A EDICIÓN. MC GRAW HILL. NEW YORK, USA. 2012, PP 2224-2226.

FIN DEL CASO CLÍNICO SERIADO


ANÁLISIS DEL CASO CLÍNICO

IDENTIFICACIÓN DEL REACTIVO


Area: PEDIATRÍA
Especialidad: CRECIMIENTO Y DESARROLLO
Tema: CRECIMIENTO Y DESARROLLO NORMAL
Subtema: CRECIMIENTO Y DESARROLLO FETAL

CASO CLÍNICO CON UNA PREGUNTA

PRIMIGESTA ADOLESCENTE CON ANTECEDENTE DE ABUSO DE COCAÍNA HASTA HACE 1 MES. ACUDE AL
SERVICIO DE GINECOLOGÍA Y OBSTETRICIA ENVIADA DE SU UNIDAD DE SALUD POR SOSPECHA DE
RESTRICCIÓN EN EL CRECIMIENTO INTRAUTERINO BASADO EN UN FONDO UTERINO MENOR A LA EDAD
GESTACIONAL CALCULADA (32SDG). SE LE HA SOLICITADO UN ESTUDIO ECOGRÁFICO CON LOS SIGUIENTES
HALLAZGOS: PESO ESTIMADO MENOR A LA PERCENTILA 10, OLIGOHIDRAMNIOS, VARIABILIDAD DE LA
FRECUENCIA CARDIO-FETAL MENOR A 5 LATIDOS POR MINUTO EN AUSENCIA DE MEDICACIÓN SEDANTE.

adolescente.

primigesta con abuso de cocaína hasta


hace un mes.

sospecha de restricción en el crecimiento


intrauterino.

fondo de saco menor al esperado para la


edad gestacional.

peso estimado por debajo del percentil 10


(DX RCIU), oligohidramnios y variabilidad
cardio-fetal menor a 5 latidos por minuto
(CRITERIOS DE GRAVEDAD, QUE
CONDUCEN AUTOMÁTICAMENTE A LA
CATEGORÍA MÁS GRAVE “TIPO V”).

346 - CON BASE EN LOS DATOS APORTADOS, LA RESTRICCIÓN DEL CRECIMIENTO INTRAUTERINO EN
ESTE CASO DEBE CLASIFICARSE COMO:

TIPO A. FETO PEQUEÑO PARA LA EDAD GESTACIONAL: Peso fetal estimado (PFE) mayor al percentil 3, pero
II menor al percentil 10; evaluación Doppler de las arterias uterinas normal. B. RESTRICCIÓN EN EL
CRECIMIENTO INTRAUTERINO (RCIU). TIPO I: Peso fetal estimado (PFE) menor al percentil 3, razón cerebro-
placentaria por arriba del percentil 5 e índice de pulsatilidad medio de las arterias menor al percentil 95.
TIPO II: Peso fetal estimado (PFE) menor al percentil 10, razón cerebro-placentaria menor al percentil 5 o
índice de pulsatilidad de las arterias uterinas mayor al percentil 95.
TIPO TIPO III: Peso fetal estimado (PFE) menor al percentil 10, flujo diastólico ausente en arteria umbilical (AEDV)
III en más del 50% de los ciclos en asa libre del cordón en ambas arterias.

TIPO TIPO IV: Peso fetal estimado (PFE) menor al percentil 10, razón cerebro placentaria menor al percentil 5 y un
IV índice de pulsatilidad de la arteria cerebral media menor al percentil 5.

TIPO TIPO V: Peso fetal estimado (PFE) menor al percentil 10 con uno o más de los siguientes criterios de
V gravedad: - Perfil biofísico (PBF) menor o igual a 4/10 o menor o igual a 6/10 con oligohidramnios. - Prueba
sin estrés (NST) tipo II (variabilidad menor a 5 latidos por minuto en ausencia de medicación sedante y/o
patrón desacelerativo). - Flujo diastólico reverso en la arteria umbilical (en más del 50% de ciclos en asa libre
de cordón en ambas arterial). - Índice de pulsatilidad del ductus venoso (DV) mayor al percentil 95 y/o
pulsaciones venosas dicrotas y persistentes en la vena umbilical (dos determinaciones con diferencia de
12hrs).

Bibliografía:

http://www.cenetec-difusion.com/CMGPC/IMSS-500-11/ER.pdf
ANÁLISIS DEL CASO CLÍNICO

IDENTIFICACIÓN DEL REACTIVO


Area: MEDICINA INTERNA
Especialidad: HEMATOLOGÍA
Tema: ANEMIAS
Subtema: ANEMIA POR DEFICIENCIA DE HIERRO

CASO CLÍNICO CON UNA PREGUNTA

PACIENTE MASCULINO DE 65 AÑOS CON DIAGNÓSTICO DE SÍNDROME ANÉMICO Y QUE COMIENZA ESTUDIO
POR PROBABLE ANEMIA SECUNDARIA A DEFICIENCIA DE HIERRO. LA BIOMETRÍA HEMÁTICA SUGIERE ESTE
DIAGNÓSTICO.

MASCULINO DE 65 AÑOS.

SÍNDROME ANÉMICO.

-.

SÍNDROME ANÉMICO.

BIOMETRIA HEMÁTICA QUE SUGIERE


ANEMIA POR DEFICIENCIA DE HIERRO.

347 - SE ESPERA SE ELEVE CON RAPIDEZ TRAS EL TRATAMIENTO AÚN CUANDO LOS DEPÓSITOS DE
HIERRO NO ESTÉN REPUESTOS POR COMPLETO:

LA La SATURACIÓN DE TRANSFERINA (Tfsat) indica el porcentaje de puntos de unión al hierro


SATURACIÓN ocupados y REFLEJA EL TRASPORTE DE HIERRO MÁS QUE SU ACUMULACIÓN. Su disminución
DE indica un hierro sérico bajo con respecto al número de receptores y, por tanto, sugiere reservas de
TRANSFERINA hierro bajas. La Tsfat varía por los mismos factores que influyen en la TIBC (capacidad total de
SÉRICA. fijación al hierro).

LOS NIVELES La FERRITINA es un compuesto de almacenamiento del hierro y su valor sérico se correlaciona con
DE FERRITINA las reservas totales en el organismo, disminuyendo por debajo de 10-12 ng/dl cuando éstas se
SÉRICA. depleccionan. Es un reactante de fase aguda, por lo que puede estar aumentada en caso de
infección o inflamación. LA CONCENTRACIÓN DE FERRITINA AUMENTA RÁPIDAMENTE TRAS
INICIAR UN TRATAMIENTO CON HIERRO Y ALCANZA VALORES NORMALES MUCHO ANTES DE
QUE SE REPONGAN POR COMPLETO LOS DEPÓSITOS DE HIERRO, POR LO QUE NO ES VÁLIDA
PARA EVALUAR LA EFICACIA AL TRATAMIENTO.
EL VOLUMEN La concentración de hemoglobina y el hematocrito son generalmente utilizados para el cribado de
CORPUSCULAR la deficiencia de hierro pero son marcadores tardíos y su valor predictivo disminuye cuando la
MEDIO. prevalencia de anemia ferropénica es baja. El VCM es normal en aproximadamente un tercio de
los pacientes con anemia ferropénica, ya que en los casos leves la anemia es normocítica y
normocrómica. NO SE UTILIZA COMO VALOR PREDICTIVO NI DIAGNÓSTICO YA QUE EN LA
MAYORÍA DE LOS CASO ESTÁ NORMAL.

LA La CAPACIDAD TOTAL DE FIJACIÓN AL HIERRO (TIBC) mide la disponibilidad de puntos de unión


CAPACIDAD al hierro en la transferrina. Es, por tanto, un medidor indirecto de los niveles de transferrina y
TOTAL DE AUMENTA CUANDO LA CONCENTRACIÓN DE HIERRO DISMINUYE. Está disminuida en la
FIJACIÓN AL malnutrición, la inflamación, la infección crónica y el cáncer.
HIERRO.

Bibliografía:
1. LONGO DL, FAUCI AS, KASPER DL, HAUSERSL, JAMESON JL, LOSCALZOJ. HARRISON. PRINCIPIOS DE
MEDICINA INTERNA, 18A EDICIÓN. MC GRAW HILL. NEW YORK, USA. 2012, PP 453. 2. PAPADAKIS MAXINE A,
MCPHEE STEPHEN J. DIAGNÓSTICO CLÍNICO Y TRATAMIENTO. 52ª EDICIÓN. NUEVA YORK. 2013, PP 492.
ANÁLISIS DEL CASO CLÍNICO

IDENTIFICACIÓN DEL REACTIVO


Area: PEDIATRÍA
Especialidad: CRECIMIENTO Y DESARROLLO
Tema: CRECIMIENTO Y DESARROLLO NORMAL
Subtema: PREESCOLAR

CASO CLÍNICO SERIADO

DURANTE SU AÑO DE SERVICIO SOCIAL SE LE SOLICITA REALIZAR UN ESTUDIO DE DIAGNÓSTICO DEL


ESTADO NUTRICIONAL DE LOS NIÑOS EN ETAPA PREESCOLAR DADO EL ALTO ÍNDICE DE OBESIDAD EN SU
COMUNIDAD.

preescolares.

ALTO ÍNDICE DE OBESIDAD EN LA


COMUNIDAD.

Se solicita un estudio de diagnóstico del


estado nutricional.

348 - PARA REALIZAR ESTA ACTIVIDAD SERÁ NECESARIO SABER QUE EL MEJOR PARÁMETRO QUE
DEFINE LA OBESIDAD EN ESTOS NIÑOS SERÁ:

EL PESO, LA LA SOMATOMETRÍA DEL NIÑO INCLUYE: TALLA, PESO Y PERÍMETRO CEFÁLICO. Para medir la
TALLA Y EL longitud de los niños deben usarse infantómetro para menores de 2 años y estadímetro a partir de
PERÍMETRO los 24 meses; estas medidas deberán realizarse sin zapatos y expresamente en centímetros.
CEFÁLICO. CORRESPONDE A LAS MEDIDAS GENERALES QUE SE DEBEN OBTENER EN TODO NIÑO.

EL PESO Para la cuantificación del peso se utiliza una báscula pesa-bebé en niños menores de 2 años y una
PARA LA báscula de plataforma en mayores de 2 años. La medición debe realizarse sin ropa y expresarse en
TALLA. kilogramos.

EL ÍNDICE EL ÍNDICE DE MASA CORPORAL (IMC) ES EL PARÁMETRO QUE MEJOR DEFINE LA OBSESIDAD EN EL
DE MASA NIÑO Y ADOLESCENTE. Se acepta internacionalmente desde los 2 años de edad con una elevada
CORPORAL. sensibilidad y especificidad para la detección de sobrepeso y obesidad. En niños mayores de 2 años
es recomendable calcular el IMC y comparar el resultado con las gráficas de percentiles
correspondientes establecidas por la OMS.
EL PESO Los niños con sobrepeso y obesidad deben considerarse de ALTO RIESGO para desarrollar
PARA LA Hiperinsulinemia y Dislipidemia y a largo plazo enfermedades cardiovasculares de ahí la importancia
EDAD. de una detección temprana.

Bibliografía:
1. CONTROL Y SEGUIMIENTO DE LA SALUD EN LA NIÑA Y EL NIÑO MENOR DE 5 AÑOS EN EL PRIMER
NIVEL DE ATENCIÓN. EVIDENCIAS Y RECOMENDACIONES: GUÍA DE PRÁCTICA CLÍNICA. MÉXICO:
SECRETARÍA DE SALUD, 2 DE DICIEMBRE DE 2015.

http://www.cenetec-difusion.com/CMGPC/IMSS-029-08/ER.pdf

349 - TRAS LA EVALUACIÓN, TODOS AQUELLOS NIÑOS QUE RESULTEN CON SOBREPESO Y OBESIDAD SE
LES DEBERÁ PROGRAMAR CITAS CON LA SIGUIENTE FRECUENCIA:

CADA De acuerdo a la NOM el niño sano menor de 5 años se recomienda recibir el siguiente número de
6 consultas de acuerdo a su edad: - En el menor de 28 días (2 consultas): una a los 7 y otra a los 28 días. - En
MESES. el menor de 1 año (5 consultas como mínimo): a los 2, 4, 6, 9 y 12 meses de edad. - En el niño de 1 a 4
años: debe recibir una consulta cada 6 meses. LOS NIÑOS SANOS PUEDEN SER CITADOS CADA 6 MESES
PARA SEGUIMIENTO GENERAL.

CADA Cuando se habla de un control más estricto que incluya vigilancia de la nutrición, crecimiento y desarrollo
3 psicomotor del niño se recomienda ampliar el número de consultas a: - En el recién nacido (2 consultas): a
MESES. los 7 y 28 días. - Del mes al año (12 consultas): 1 cada mes. - De 1 a 3 años (4 consultas por año): cada 3
meses. - De 3 a 5 años (2 consultas por año): cada 6 meses. LOS LACTANTES MAYORES QUE AMERITAN
UN SEGUIMIENTO ESTRICTO PODRÁN EVALUARSE CADA 3 MESES.

CADA La frecuencia de seguimientos en el niño tiene una amplia variabilidad de acuerdo a las condiciones
2 especiales de cada uno. El objetivo de citar a los niños sanos para su supervisión es detectar temprana y
MESES. oportunamente desviaciones en su crecimiento y desarrollo. RECUERDA QUE LA FRECUENCIA DE LAS
CONSULTAS DE SEGUIMIENTO SE IRÁ AJUSTANDO A LAS NECESIDADES DE CADA NIÑO.

CADA Cuando el crecimiento y estado nutricional del niño se encuentra con desnutrición leve o moderada,
MES. sobrepeso o talla ligeramente baja, se deben citar a los niños con intervalos cortos de cada 30 días para
los menores de 5 años. Todo niño con desviaciones somatométricas por IMC se deberán evaluar mediante
seguimiento mensual y considerar la evaluación de un especialista para ofrecer una investigación más
profunda de las posibles causas y dar medidas correctivas en la alimentación. TODO NIÑO CON
DESVIACIÓN DE LA NORMALIDAD DEL IMC DEBERÁ SER EVALUADO MENSUALMENTE.

Bibliografía:
1. CONTROL Y SEGUIMIENTO DE LA SALUD EN LA NIÑA Y EL NIÑO MENOR DE 5 AÑOS EN EL PRIMER
NIVEL DE ATENCIÓN. EVIDENCIAS Y RECOMENDACIONES: GUÍA DE PRÁCTICA CLÍNICA. MÉXICO:
SECRETARÍA DE SALUD, 2 DE DICIEMBRE DE 2015. 2. NORMA OFICIAL MEXICANA NOM-031-SSA2-1999,
PARA LA ATENCIÓN A LA SALUD DEL NIÑO.

http://www.cenetec-difusion.com/CMGPC/IMSS-029-08/ER.pdf

FIN DEL CASO CLÍNICO SERIADO


ANÁLISIS DEL CASO CLÍNICO

IDENTIFICACIÓN DEL REACTIVO


Area: MEDICINA INTERNA
Especialidad: ONCOLOGÍA
Tema: OTROS TUMORES
Subtema: TUMORES DE TIROIDES

CASO CLÍNICO CON UNA PREGUNTA

MUJER DE 31 AÑOS CON EMBARAZO DE 10 SEMANAS DE GESTACIÓN. HACE UNA SEMANA SE LE REALIZÓ
BIOPSIA A NÓDULO TIROIDEO, REPORTÁNDOSE POSITIVA PARA MALIGNIDAD.

MUJER DE 31 AÑOS.

EMBARAZO DE 10 SEMANAS DE
GESTACIÓN.

-.

-.

BIOPSIA de NóDULO TIROIDEO,


REPORTáNDOSE POSITIVA PARA
MALIGNIDAD.

350 - EN CASO DE REQUERIR TRATAMIENTO QUIRÚRGICO DEL TUMOR, ESTE DEBERA REALIZARSE:

DE El CÁNCER DIFERENCIADO DE TIROIDES es una neoplasia maligna que se desarrolla en las células
INMEDIATO. foliculares de la glándula tiroides, representa el 1 a 3% de los tumores, del 80 al 90% son carcinomas
papilares y del 10 al 15% carcinomas foliculares. Es el cáncer endocrinológico más frecuente, sin
embargo, es raro en comparación con otros tipos de cánceres. El diagnóstico de cáncer de tiroides se
hace en base a una biopsia de un nódulo tiroideo o después de que el nódulo es removido durante
la cirugía.

ANTES DE La biopsia del nódulo tiroideo durante el embarazo está determinada por niveles de TSH normales o
LA SEMANA elevados. Es recomendable vigilar clínica y por ultrasonido la progresión del nódulo tiroideo
20 DE positivo o sospechoso y la decisión quirúrgica debe ser tomada por el equipo multidisciplinario.
GESTACIÓN.
DESPUÉS DE En caso de resultado por biopsia positiva para malignidad en el embarazo realizar ultrasonido
LA SEMANA periódico y en caso de crecimiento realizar cirugía después de la semana 24 de gestación o en caso
24. de no mostrar progresión puede realizarse al término del embarazo. EL TRATAMIENTO EN ESTE
CASO DEBE SER QUIRÚRGICO DESPUÉS DE LA SEMANA 24 DE GESTACIÓN.

UNA VEZ En la mujer embarazada con neoplasia de tiroides, es recomendable vigilar clínica y por ultrasonido
TERMINADO la progresión del nódulo tiroideo positivo o sospechoso, y la decisión quirúrgica debe ser tomada
EL por el equipo multidisciplinario. SE ESPERA AL FINAL DEL EMBARAZO, PARA EL MANEJO
EMBARAZO. QUIRÚRGICO DE LA NEOPLASIA DE TIROIDES, SÓLO SI NO HAY DATOS DE PROGRESIÓN EN
ESTADIOS TEMPRANOS.

Bibliografía:
1. GUÍA DE PRÁCTICA CLÍNICA, DIAGNÓSTICO Y TRATAMIENTO DEL TUMOR MALIGNO DE TIROIDES
(CÁNCER DE TIRIDES BIEN DIFERENCIADO). MÉXICO: SECRETARIA DE SALUD, 2009.

http://www.cenetec.salud.gob.mx/descargas/gpc/CatalogoMaestro/166_GPC_TUMOR_MALIGNO_TIROIDEO/Gpc_tumor_tiroideo.pdf
ANÁLISIS DEL CASO CLÍNICO

IDENTIFICACIÓN DEL REACTIVO


Area: MEDICINA INTERNA
Especialidad: REUMATOLOGÍA
Tema: TRANSTORNOS MEDIADOS POR MECANISMOS INMUNITARIOS
Subtema: ARTRITIS IDIOPÁTICA JUVENIL

CASO CLÍNICO SERIADO

HOMBRE DE 19 AÑOS DE EDAD, CON ANTECEDENTE DE INTERVENCIÓN ODONTOLÓGICA HACE 2 SEMANAS.


ACUDE A CONSULTA POR PRESENCIA DE DOLOR INTENSO EN LAS ARTICULACIONES DISTALES DE LA MANO
DERECHA DE 2 MESES DE EVOLUCIÓN; REFIERE ADEMÁS QUE, POR LAS MAÑANAS, LE ES COMPLICADO
MOVILIZAR LA RODILLA DEL MISMO LADO Y QUE ESTO DURA AL MENOS UNA HORA. DURANTE LA
EXPLORACIÓN FÍSICA SE OBSERVA LIGERA CLAUDICACIÓN, DACTILITIS E INFLAMACIÓN EN 3 DE 5
ARTICULACIONES INTERFALÁNGICAS DISTALES DE LA MANO DERECHA, CON CAMBIOS UNGUEALES EN LA
MISMA MANO; LA RODILLA AFECTADA ESTÁ EDEMATOSA Y SE DESPIERTA DOLOR A LA PALPACIÓN. SE
SOLICITAN LABORATORIOS.

19 años.

intervención odontológica (distractor).

dolor en articulaciones DISTALES de la


mano, rigidez en rodilla derecha= afección
unilateral, asimétrica.

claudicación, DACTILITIS, in amación de


las articulaciones interfalángicas DISTALES,
CAMBIOS UNGUEALES, rodilla edematosa
y dolorosa.

351 - EL DIAGNÓSTICO MÁS PROBABLE ES:


ARTRITIS La ARTRITIS REUMATOIDE (AR) es una enfermedad inflamatoria, crónica autoinmune y sistémica
REUMATOIDE de etiología desconocida; su principal órgano blanco es la membrana sinovial; se caracteriza por
inflamación poliarticular y simétrica de pequeñas y grandes articulaciones, con posible compromiso
sistémico en cualquier momento de su evolución. Cx. Se debe sospechar en presencia de 4 de 7
criterios de clasificación. 1. Rigidez matutina: rigidez matutina articular que dura al menos 1hr. 2.
Artritis de 3 o más grupos articulares: al menos 3 grupos articulares deben estar inflamados
simultáneamente y ser objetivados por un médico. 3. Artritis de articulaciones de las manos: al
menos una articulación de las manos debe estar inflamada (carpo, metacarpofalángicas,
interfalángicas proximales). 4. Artritis simétrica: afectación simultánea del mismo grupo articular
en ambos lados del cuerpo. 5. Nódulos reumatoides: nódulos subcutáneos en prominencias óseas,
superficies de extensión o en zonas yuxta-articulares observados por un médico. 6. Factor
reumatoide en suero: presencia de valores elevados de factor reumatoide por cualquier método
dan un resultado en controles inferior al 5%. 7. Alteraciones radiológicas: típicas de la artritis
reumatoide en radiografías posteroanteriores de las manos. Debe existir erosión u osteoporosis
yuxta-articular clara y definida en articulaciones afectada. LA ARTRITIS REUMATOIDE SUELE SER
“SIMÉTRICA” Y LAS ARTICULACIONES DE LA MANO QUE MÁS SE AFECTAN SON LAS
“PROXIMALES”, NO LAS DISTALES.

ARTRITIS La ARTRITIS IDIOPÁTICA JUVENIL es un término amplio que describe un grupo clínicamente
IDIOPÁTICA heterogéneo de artritis de etiología desconocida, que persiste por más de seis semanas y que
JUVENIL comienza “antes de los 16 años de edad”. Involucra diferentes categorías de enfermedades, cada
una de ellas con distintas formas de presentación, signos, síntomas clínicos y pronóstico. Los siete
subgrupos principales de artritis idiopática juvenil incluyen: 1. Oligoarticular (27-56%). 2.
Poliarticular (más de 5 articulaciones afectadas) con factor reumatoide negativo (11-28%). 3.
Poliarticular con factor reumatoide positivo /2-7%). 4. Artritis séptica (4-17%). 5. Artritis
relacionada con entesitis (3-11%). 6. Artritis psoriásica (2-11%). 7. Artritis indiferenciada (11-21%).
LA ARTRITIS JUVENIL TIENE SU INICIO ANTES DE LOS 16 AÑOS DE EDAD, RAZÓN POR LA CUAL
ESTE PACIENTE NO PUEDE SER INCLUIDO EN ESTE DIAGNÓSTICO.

ARTRITIS La ARTRITIS PSORIÁSICA es una enfermedad inflamatoria sistémica crónica heterogénea que afecta
PSORIÁSICA la piel, uñas y las estructuras musculoesqueléticas como articulaciones, inserciones sinoviales de
tendones y el esqueleto axial. Afecta tanto al esqueleto axial como al periférico, se describen 5
patrones de compromiso clínico: - Poliarticular, con artritis simétrica, semejante a la artritis
reumatoide (15% de los pacientes). - Oligoarticular (4 o menos articulaciones) con artritis
asimétrica (70% de los pacientes). - Compromiso predominante de AID (5% de los pacientes). -
Predominio de espondiloartritis (5% de los pacientes). - Artritis mutilante (5% de los pacientes). A
diferencia de la artritis reumatoide que es de características simétricas y poliarticular, la artritis
psoriásica compromete las articulaciones de forma asimétrica, usualmente como una mono u
oligoartritis, aunque también lo puede hacer como una poliartritis. A menudo afecta las
articulaciones interfalángicas distales, que comúnmente se acompañan de dactilitis (tumefacción
difusa de los dedos) y entesitis (inflamación de la zona de inserción de los músculos, tendones y
ligamentos en el hueso), siendo estos dos rasgos, característicos de la enfermedad. NOTA: En la
mayoría de los casos (60-70%) la psoriasis precede al trastorno articular, hasta en el 20% la
psoriasis y la artritis aparecen al mismo tiempo y, entre el 15 y 20% (menos común) la artritis
precede a la psoriasis. Es más probable que en este paciente se trate de una artritis psoriásica
teniendo en cuenta que: - ES MAYORMENTE DE CARÁCTER ASIMÉTRICO. - MÁS DE 30% DE LOS
PACIENTES PRESENTAN DACTILITIS, Y ES UN SIGNO CLAVE QUE HACE LA DIFERENCIA CON EL
RESTO DE LOS DIAGNÓSTICOS. - CASI SIEMPRE SE ACOMPAÑA DE CAMBIOS UNGUEALES
(HIPERQUERATOSIS, ONICOLISIS).
ARTRITIS La ARTRITIS SÉPTICA (AS) es una emergencia que tiene serias consecuencias si no se diagnostica
SÉPTICA tempranamente y se trata de forma efectiva. Involucra infección bacteriana de una articulación. La
diseminación hematógena bacteriana es la causa más común en niños. Se sospecha cuando tiene un
inicio agudo con dolor y aumento de temperatura en una articulación y limitación del movimiento.
Ocurre con más frecuencia en articulaciones de cadera y rodilla, otras articulaciones son el hombro
y el codo, aunque puede aparecer en cualquier articulación. Factores de riesgo: edad (mayor de 80
o menor de 5 años), nivel socioeconómico bajo, género masculino en niños, prematurez o bajo
peso al nacer, cateterización umbilical, síndrome de dificultad respiratoria neonatal, diabetes
mellitus, hemodiálisis, VIH, sepsis, varicela, ausencia de vacunación contra H. influenza tipo b,
heridas articulares penetrantes, úlceras cutáneas, artritis reumatoide, antecedente de inyección
intra-articular de esteroides, antecedente de cirugía articular menos a 3 meses, prótesis de rodilla o
cadera, infección de tejidos blandos, hemoglobinopatías, inmunodeficiencias primarias,
alcoholismo, uso de drogas intravenosas. Cx: la infección articular puede formar parte de un cuadro
grave de bacteremia con signos típicos de sepsis. Las manifestaciones locales son: claudicación,
pseudoparálisis, dolor articular o referido a la extremidad, aumento de volumen, calor local y/o
inflamación de regiones contiguas a la articulación afectada. Es muy poco probable que se trate de
una artritis séptica en este paciente dado que: - EL ÚNICO FACTOR DE RIESGO, LA INTERVENCIÓN
ODONTOLÓGICA, SE DIO CUANDO LOS SIGNOS DE ARTRITIS YA ESTABAN PRESENTES. - LA
ARTRITIS SÉPTICA AFECTA COMÚNMENTE ARTICULACIONES MÁS GRANDES, Y CON POCA
FRECUENCIA LAS MANOS. - LA EDAD DEL PACIENTE NO CORRESPONDE AL GRUPO MAYORMENTE
AFECTADO.

Bibliografía:
LONGO DL, FAUCI AS, KASPER DL, HAUSERSL, JAMESON JL, LOSCALZOJ. HARRISON. PRINCIPIOS DE
MEDICINA INTERNA, 18A EDICIÓN. MC GRAW HILL. NEW YORK, USA. 2012.

http://www.cenetec-difusion.com/CMGPC/IMSS-369-10/ER.pdf http://www.cenetec-difusion.com/CMGPC/IMSS-368-10/ER.pdf

352 - SU PRESENCIA SE CONSIDERA UN MARCADOR DE SEVERIDAD PARA ESTE CASO:

EDAD MENOR DE Aunque puede manifestarse a cualquier edad, incluyendo la infancia, la artritis psoriásica
20 AÑOS aparece más comúnmente entre los 30 y 50 años de edad. LA EDAD NO ES UN MARCADOR DE
SEVERIDAD, ESTA ENFERMEDAD PUEDE PRESENTARSE A CUALQUIER EDAD.

DACTILITIS LA DACTILITIS, REPORTADA EN UN 14 A 24% DE LOS CASOS, SE ASOCIA COMO UN


MARCADOR DE SEVERIDAD DE LA ENFERMEDAD PARA ARTRITIS PSORIÁSICA. Por su parte, la
enfermedad activa y severa desde el inicio es un predictor de mortalidad.

ASIMETRÍA DE La afección de las articulaciones en la artritis psoriásica suele ser mayormente asimétrica, lo
LAS LESIONES cual ES UNA CARACTERÍSTICA Y NO UN MARCADOR DE SEVERIDAD.

NÚMERO DE Se considera una artritis psoriásica severa cuando existe: - Una artritis periférica con afección
ARTICULACIONES de más de 5 articulaciones, daño radiológico severo, sin respuesta al tratamiento, respuesta
AFECTADAS leve o moderada, severa pérdida de la función física y severo impacto en la calidad de vida. -
Una afección cutánea de la psoriasis mayor al 10% de la superficie corporal, índice de actividad
y severidad de la psoriasis mayor a 10. - Entesitis con pérdida de la función en 2 o más sitios y
falla a la respuesta terapéutica. - Dactilitis con falla a la respuesta. SI BIEN, EL NÚMERO DE
ARTICULACIONES ES UN MARCADOR DE SEVERIDAD, EL PACIENTE TIENE SOLO 4
ARTICULACIONES EN TOTAL AFECTADAS (3 INTERFALÁNGICAS DISTALES + 1 RODILLA), POR
LO QUE NO SE PUEDE CONSIDERAR UNA ARTRITIS SEVERA.

Bibliografía:

http://www.cenetec-difusion.com/CMGPC/IMSS-337-10/ER.pdf

353 - CON BASE EN EL DIAGNÓSTICO, EN LOS ESTUDIOS DE LABORATORIO SE ESPERARÍA ENCONTRAR:


VELOCIDAD DE La velocidad de sedimentación globular (VSG) y la proteína C reactiva (PCR) se encuentran
SEDIMENTACIÓN elevadas en la mayor parte de los casos con ARTRITIS SÉPTICA, generalmente desde los
GLOBULAR Y primeros días de la enfermedad, e incluso en las primeras horas. La asociación de estas dos, con
PROTEÍNA C un cuadro clínico sugestivo y la presencia de leucocitos séricos por arriba de 12,000/mm3
REACTIVA apoya fuertemente el diagnóstico, aunque estos tres parámetros por si solos resulten altamente
ELEVADAS MÁS inespecíficos. VALORES LEUCOCITARIOS POR ARRIBA DE 12,000/MM3, VSG SUPERIOR A 20
LEUCOCITOSIS MM/H Y PCR MAYOR A 20 MG/DL APOYAN EL DIAGNÓSTICO DE ARTRITIS SÉPTICA. Por el
contrario, la normalidad en la VSG y PCR hacen improbable el diagnóstico de artritis séptica en
pacientes con claudicación y derrame articular.

FACTOR LA PRESENCIA DE ANTI-CCP Y FACTOR REUMATOIDE POSITIVOS EN UN MISMO PACIENTE


REUMATOIDE Y ALCANZA UN VALOR PREDICTIVO PARA ARTRITIS REUMATOIDE DEL 100%. Se debe
ANTICUERPOS determinar el factor reumatoide a todo paciente con sospecha clínica de artritis reumatoide;
ANTI-CCP aunque un resultado negativo, no excluye la enfermedad. Los anticuerpos anti-CCP (anti
POSITIVOS péptido cíclico citrulinado) tienen un cociente de probabilidad para el diagnóstico de artritis
reumatoide superior al factor reumatoide. Su aparición puede preceder en años a la
enfermedad y se asocia con un mal pronóstico.

VELOCIDAD DE La artritis psoriásica está clasificada dentro de las espondiloartrosis seronegativas con una usual
SEDIMENTACIÓN ausencia de factor reumatoide (FR) y con una estrecha asociación con el antígeno humano
GLOBULAR Y leucocitario. En general, es posible observar una velocidad de sedimentación globular (VSG) y
PROTEÍNA C proteína C reactiva (PRC) elevadas. UNA CARACTERÍSTICA DE LA ARTRITIS PSORIÁSICA ES SU
REACTIVA NEGATIVIDAD PARA FACTOR REUMATOIDE. RECUERDA: - La elevación de la VSG y la PCR
ELEVADAS CON tienen una estrecha relación con la actividad inflamatoria, lo que las hace altamente
FACTOR inespecíficas, y pueden estar elevadas en cualquiera de los cuatro posibles diagnósticos. - No se
REUMATOIDE cuenta con un estudio diagnóstico específico para el diagnóstico de la artritis psoriásica.
NEGATIVO

ANTICUERPOS Existen títulos elevados de antígenos antinucleares (ANA) en un 40 a 85% de los niños con
ANTINUCLEARES ARTRITIS IDIOPÁTICA JUVENIL oligoarticular o poliarticular, pero no es común en aquellos con
Y FACTOR artritis de inicio sistémico. La presencia de estos antígenos, se han relacionado con mayor
REUMATOIDE riesgo de uveítis crónica. Por su parte, el factor reumatoide (FR) se ha presentado en tan solo el
POSITIVO 5 a 10% de los casos con enfermedad poliarticular. Otro marcador importante es anticuerpo
anti péptico citrulinado (anti-CCP), que en asociación con FR sugieren una enfermedad más
grave. LOS ANTICUERPOS ANTINUCLEARES SE PRESENTAN EN BUENA PARTE DE LOS
PACIENTES CON ARTRITIS IDIOPÁTICA JUVENIL Y SU PRESENCIA GUARDA ESPECIAL RELACIÓN
CON UVEÍTIS CRÓNICA.

Bibliografía:
LONGO DL, FAUCI AS, KASPER DL, HAUSERSL, JAMESON JL, LOSCALZOJ. HARRISON. PRINCIPIOS DE
MEDICINA INTERNA, 18A EDICIÓN. MC GRAW HILL. NEW YORK, USA. 2012.

http://www.cenetec-difusion.com/CMGPC/IMSS-195-10/ER.pdf http://www.cenetec-difusion.com/CMGPC/IMSS-368-10/ER.pdf

FIN DEL CASO CLÍNICO SERIADO


ANÁLISIS DEL CASO CLÍNICO

IDENTIFICACIÓN DEL REACTIVO


Area: GINECOLOGÍA Y OBSTETRICIA
Especialidad: GINECOLOGÍA
Tema: PATOLOGÍA DE VULVA Y VAGINA
Subtema: LESIONES BENIGNAS DE VULVA

CASO CLÍNICO SERIADO

MUJER DE 70 AÑOS DE EDAD, QUE ACUDE A LA CONSULTA AL PRESENTAR PRURITO VULVAR, DISPAREUNIA Y SENSACIÓN DE ARDOR VAGINAL. A LA
EXPLORACIÓN FÍSICA SE ENCUENTRAN LESIONES BLANCAS A NIVEL DE LA VULVA Y SE EXTIENDEN AL PERINÉ. LA ÚLTIMA CITOLOGÍA, DE HACE 2 AÑOS, FUE
NEGATIVA CON ATROFIA.

70 AÑOS.

-.

"prurito vulvar", dispareunia y sensación de


ardor vaginal.

Presencia de lesiones "blancas" en región


vulvar que "se extienden al periné".

ÚLTIMA CITOLOGÍA HACE 2 AÑOS:


NEGATIVA CON ATROFIA.

354 - EL DIAGNÓSTICO CLÍNICO MÁS PROBABLE ES:

VULVOVAGINITIS LO PRIMERO QUE DEBES HACER EN ESTE CASO, ES DISTINGUIR EL TIPO DE LESIÓN VULVAR QUE SE PRESENTA. Las LESIONES VULVARES clínicamente se
ATRÓFICA. pueden clasificar por color, patrón y localización. 1. Color: - Blanquesinas: infección por VPH, liquen esclerosos, neoplasia intraepitelial vulvar. -
Eritematosas: infecciones, psoriasis, liquen plano, dermatitis de contacto, endometriosis, hemangiomas, enfermedad de Paget, candidiasis. -
Oscuras/marrones. 2. Patrón: quiste, pápula, placa, verruga, vesícula o ámpula, úlcera y tumor. 3. Localización: áreas provistas o desprovistas de vello.
VULVOVAGINITIS ATRÓFICA. La VULVOVAGINITIS ATRÓFICA resultante de la carencia de estrógenos sobre la vagina y vulva se manifiesta con sequedad,
perdida de la elasticidad y hasta retracción vaginal. Esto lleva a la dispareunia, en especial en pacientes que no mantienen relaciones sexuales frecuentes.
Clínicamente se observa una mucosa vulvovaginal adelgazada, brillante, dejando ver en algunos casos la vascularización subyacente. En algunos casos la
piel vulvar puede presentarse como tejido adelgazado con equimosis, como un área “blanca” con “agrietamiento” de la piel. Esta deficiencia estrogénica
también provoca en la uretra atrofia de la red venosa, adelgazamiento del epitelio, que lleva a una posición defectuosa de la uretra y a una reducción en
la presión de cierre uretral. Esto origina un aumento en la incidencia de infecciones urinarias e incontinencia urinaria. La citología del cérvix y vagina
reporta células epiteliales inmaduras con o sin inflamación. IMPORTANTE: ante la paciente en la peri o postmenopausia que refiere sequedad vaginal y/o
dispareunia se debe sospechar atrofia genitourinaria por hipoestrogenismo. EL CASO CLÍNICO NO REPORTA CARACTERÍSTICAS CLÍNICAS VAGINALES
QUE SUGIERAN ESTE DIAGNÓSTICO, SÓLO LESIONES VULVARES. IMPORTANTE: la paciente presenta además datos de atrofia descrita por citología, pero
la distribución de la lesión es más concordante con liquen escleroso.
LIQUEN Los TRASTORNOS EPITELIALES NO NEOPLÁSICOS DE LA PIEL Y MUCOSA VULVAR incluyen: - Liquen escleroso. - Liquen simple crónico. - Otras
ESCLEROSO. dermatosis. El LIQUEN ESCLEROSO corresponde al más común de estos trastornos. Es secundario a un proceso inflamatorio crónico benigno de la vulva,
relacionado con la deficiencia de vitamina A y procesos autoinmunitarios. Algunas teorías sugieren un exceso de la enzima elastasa con disminución de la
actividad de la enzima 5-alfa-reductasa, lo cual impide la transformación de testosterona en dihidrotestosterona (hormona trófica de la piel), teniendo
como resultado un adelgazamiento de la piel. Situación que justifica que se utilizara con anterioridad la testosterona tópica como tratamiento. CX. Se
presenta en mujeres que rebasan los 60 años de edad, cuando se presenta en mujeres jóvenes o durante la infancia, suele tener una resolución
espontánea. Clínicamente las pacientes suelen referir prurito, dolor vulvar, dispareunia o simplemente lesiones blancas asintomáticas en la vulva. EF:
Durante el proceso evolutivo de la enfermedad, las lesiones aparecen con la siguiente evolución durante la FASE AGUDA: 1. Eritema y edema de vulva. 2.
Placas blancas de liquenificación e hiperqueratosis. 3. Unión de las placas blancas. 4. Prurito con rascado. 5. Telangiectasias y hemorragias por rascado. 6.
Erosiones, fisuras y ulceraciones secundarias. La FASE CRÓNICA por su parte también tiene una evolución progresiva característica: 1. Piel blanca,
adelgazada y arrugada (apariencia de papel para cigarros). 2. Aglutinación de las partes anteriores de los labios menores, bilateral, que llegan a cubrir e
clítoris provocando fimosis. 3. Contracción de las estructuras vulvares con estenosis del introito (antes llamada craurosis). 4. “Afección perianal en forma
de ocho” (rodeando vulva y ano). 5. Formación de islas de tejido hiperplásico dentro del tejido atrófico de la lesión. DX: El diagnóstico es clínico y se
confirma por estudio histoatológico donde descartan cinco características esenciales: 1. Delgada capa hiperqueratósica. 2. Adelgazamiento de la capa
epitelial. 3. Aplanamiento de las papilas (crestas interpapilares). 4. Homogeneización de los estromas. 5. Profunda infiltración linfocítica. Estas lesiones
obligan a hacer diagnóstico diferencial con vitíligo, liquen simple crónico y psoriasis. PX: se relaciona con una evolución maligna en un 3 a 5% de los
casos, lo que obliga a tomar biopsia en todos los casos, con especial atención en aquellos que no mejoran con el tratamiento. IMPORTANTE: ante placas
“blancas” nacaradas, “intensamente pruriginosas” con piel fina, frágil, con zonas purpúricas, con un patrón típico en forma de 8, rodeando la zona vulvar
y “perianal”, se debe descartar liquen escleroso. LA EDAD DE LA PACIENTE, CON PRESENCIA DE LESIONES BLANCAS, CON PRURITO INTENSO QUE SE
EXTIENDE EN PERINÉ, SON CLÁSICAS DE LIQUEN ESCLEROSO.

LEUCOPLASIA El color del epitelio y de las lesiones en vulva está definido por el espesor de la capa de queratina de la zona, la vascularización de la dermis, el grosor de
VULVAR. la epidemis subyacente y la cantidad de pigmento (melanina o pigmento sanguíneo. Las lesiones blancas liquenoides que aparecen en la vulva se han
relacionado en gran medida con maceración de la capa de queratina engrosada debida a un aumento en la humedad de la zona. Estas lesiones son de
cuidado pues pueden enmascarar neoplasias ocultas con lesiones hiperplásicas visibles. Otras causas de lesiones albinas a nivel vulvar se relacionan con
ausencia de pigmentación como lo es en el vitíligo y en algunos trastornos hereditarios; o bien por cicatrices hipopigmentadas después de que sana una
ulceración. IMPORTANTE: El término LEUCOPLASIA es muy antiguo y ha quedado en desuso, inicialmente a las lesiones liquenoides blancas que se
encontraban en vulva se les denominaba leucoplasia, craurosis vulvar y vulvitis senil. Hacia la década de los 70’s este término fue sustituido por
trastornos epiteliales no neoplásicos de la piel y mucosa vulvar; esto con el fin de diferenciarlos de aquellos relacionados con neoplasias, situación que
deberá corroborarse por medio de biopsia. Cuando estas lesiones presentan datos de “atipia” a nivel microscópico se les definió como distrofia con
atipia, pero este término también ha quedado en desuso para llamarles ahora neoplasias intraepiteliales vulvares (VIN). Así bien los TRASTORNOS
EPITELIALES NO NEOPLÁSICOS DE LA PIEL Y MUCOSA VULVAR incluyen entonces al liquen escamoso, el liquen simple y otras dermatosis de la vulva. LAS
LEUCOPLASIAS, GENERALMENTE HACEN REFERENCIA A LESIONES BLANCAS, SE USA DE FORMA GENÉRICA PARA TODA LESIÓN ALBINA
INDEPENDIENTEMENTE DE SU CAUSA. ACTUALMENTE ESTA EN DESUSO, ES PREFERIBLE REFERIRSE A LAS PATOLOGÍAS DE FORMA ESPECÍFICA. El
tratamiento de la leucoplasia, dependerá del diagnóstico final que se haga de las lesiones.

DISQUERATOSIS DISQUERATOSIS hace referencia a la alteración en la pigmentación normal de las células de la epidermis, condicionada por presencia de células que
POR VIRUS DEL contienen queratina antes de alcanzar su madurez y sin llegar a la superficie. En el caso de los condilomas está condicionado por infección por el VIRUS
PAPILOMA DEL PAPILOMA HUMANO (VPH). El VPH es actualmente un agente frecuente de infección en vía genitales, se asociado además con cáncer a nivel cervical
HUMANO. y con menor frecuencia en vulva. El periodo de incubación puede ser desde semanas hasta años. Las lesiones no neoplásicas, condilomas, se asocian con
serotipos del VPH 6 y 11 principalmente, mientras que aquellas neoplásicas se asocian con los subtipos 16, 1, 45, 31 y 52. - Los condilomas típicos tienen
un crecimiento exofítio blanco o papilomatoso, que suele agruparse dando una imagen en coliflor. - Los condilomas papilomatosos floridos, se observan
como lesiones blancas elevadas con proyecciones digitiformes que a menudo contienen capilares. - Los condilomas planos, en pico o invertidos, se
observan como lesiones blancas sin relieve o con poco relieve que sólo se observan mediante colposcopía. Mediante colposcopía también se puede
observar un patrón en mosaico con punteado lo que sugiere neoplasia intraepitelial vaginal (VIN) y que debe descartarse mediante biopsia. Las lesiones
generalmente son asintomáticas, pocas veces la paciente refiere prurito. Deberá hacerse en estos casos diagnóstico diferencial con molusco contagioso,
quistes epidérmicos y de queratina; en el caso del condiloma plano, deberá diferenciarse con sífilis. En todos los casos se recomienda estudio
colposcópico con toma de citología del cuello uterino. Se deben solicitar al mismo tiempo pruebas para otras infecciones de transmisión sexual (sífilis,
hepatitis B y C, chlamydia y VIH). CARACTERÍSTICAMENTE SON ASINTOMÁTICAS, ES MÁS COMÚN ENCONTRAR LESIONES EXOFÍTICAS, CUANDO SON
PLANAS MUCHAS VECES SÓLO SON VISIBLES MEDIANTE COLPOSCOPÍA.

Bibliografía:
1. GUÍA DE PRÁCTICA CLÍNICA, ABORDAJE DIAGNÓSTICO DE LOS DESÓRDENES BENIGNOS DE LA VULVA. MÉXICO: SECRETARIA DE SALUD; 2013. 2. DECHERNEY
A, NATHAN L, LAUFER N, ROMAN A. DIAGNÓSTICO Y TRATAMIENTO GINECOOBSTÉTRICOS, 11ª EDICIÓN. MC GRAW HILL EDUCATION, LANGE. MÉXICO. 2014.

http://www.cenetec.salud.gob.mx/descargas/gpc/CatalogoMaestro/IMSS_630_13_DESORDENESBENIGNOSVULVA/630GER.pdf

355 - EL TRATAMIENTO INDICADO EN ESTE CASO ES:

ÁCIDO Entre los agentes tópicos para el tratamiento de las VERRUGAS GENITALES está la Podofilina, el ACIDO TRICLOROACÉTICO y Bicloroacético. 1.
TRICLOROACÉTICO. Autoaplicación: El tratamiento más comúnmente utilizado es la auto aplicación de podofilina, el cual es un agente antimitótico, interfiriendo en la
división celular y dañando los tejidos en los cuales las células se están reproduciendo. Se utiliza la Podofilina al 10 a 25 % en tintura de benzoína. 2.
Aplicación médica: se debe de aplicar por un médico el ácido bicloroacético o ácido Tricloroacético (Proteolíticos) solución al 50% a 80%, con menor
concentración en cérvix y vagina, y con mayor concentración en vulva. Se realiza la aplicación semanal hasta que las lesiones desaparezcan. Ya se
demostró que la aplicación de interferón dentro de la lesión es efectiva en casos que no responden al tratamiento. Se pueden utilizar agentes
quimioterapéuticos como el ungüento de 5-fluorouracilo o bleomicina en forma de inyecciones dentro de la lesión como tratamiento de segunda
línea. TRATAMIENTO INMUNOLÓGICO. El IMIQUIMOD es un producto tópico inmunomodulador con actividad antiviral, ya que induce la formación
de citocinas en el sitio de la lesión, entre ellos el Interferón. Además, se ha visto que el ARN-m L1 y el ADN del VPH se encuentran significativamente
disminuidos después del tratamiento. La crema de Imiquimod al 5% se aplica tres veces por semana y el tiempo promedio para la desaparición de las
lesiones es de 12 semanas. La recidiva varía del 6 al 19% a 3 meses de seguimiento. Se recomienda en “lesiones pequeñas” o después de la escisión
quirúrgica del condiloma, aplicándolo sobre el área cicatrizada. TRATAMIENTO QUIRÚRGICO. Se recomienda en lesiones extensas, particularmente
aquellas que son pediculadas. Las opciones incluyen criocirugía, destrucción electroquirúrgica, escisión y vaporización con láser. - La electrocirugía
con asa diatérmica se recomienda para pacientes con condiloma acuminado genital, anal u oral, como aplicación única. - La vaporización está indicada
en lesiones muy extensas tanto anales como genitales, su costo es elevado y tienen una baja accesibilidad. EL ÁCIDO TRICLOROACÉTICO ES UN
AGENTE TÓPICO INDICADO EN EL MANEJO DE LOS CONDILOMAS POR INFECCIÓN DEL VIRUS DEL PAPILOMA HUMANO.

UNGÜENTOS CON Medidas que han quedado EN DESUSO y que debes conocer en el manejo del LIQUEN ESCLEROSO: pomada de testosterona al 2%, inyección
ANDRÓGENOS Y intralesional con alcohol, vulvectomía. Se describen porque algunas fuentes aún las incluyen aunque ya no forman parte de la estrategia actual. ES
ESTEROIDES. USO DE TESTOSTERONA TÓPICA YA NO SE USA PARA EL MANEJO DEL LIQUEN ESCLEROSO.

ESTEROIDES El tratamiento inicial del LIQUEN ESCLEROSO está encaminado a reducir el ciclo comezón-rascado, lo cual se logra mediante el aseo con jabón neutro,
TÓPICOS Y evitar ropa ajustada y secado de la zona. 1. Manejo médico. - Antihistamínicos. - Esteroides tópicos: dipropionato de clobetasol al 0.05%, en
ANTIHISTAMÍNICOS. cantidades pequeñas, dos veces al día dos semanas. 2. Manejo quirúrgico: se utiliza para liberar la fimosis del clítoris, el estrechamiento introital, o
cuando se asocian lesiones malignas. La extirpación de fibras nerviosas y de la piel afectada, son medidas extremas que se utilizan cuando ninguna
medida logra aminorar el prurito. EL USO DE ANTIHISTAMÍNICOS Y ESTEROIDES TÓPICOS ESTÁN INDICADOS PARA EL MANEJO DEL LIQUEN
ESCLEROSO.
TERAPIA El tratamiento de la VULVOVAGINITIS ATRÓFICA, una vez que se hayan excluido otro tipo de afecciones, consiste en la administración local y
HORMONAL CON sistémica de estrógenos para las lesiones vaginales. IMPORTANTE: En el caso de las vulvares, se recomienda siempre realizar un estudio muy
ESTRÓGENOS Y meticuloso para descartar otras patologías. INDICACIONES DEL USO DE TERAPIA HORMONAL (TH): - Síntomas vasomotores (bochornos,
PROGESTÁGENOS. sudoraciones o taquicardias). - Atrofia vulvovaginal (dolor al coito, dispareunia, resequedad). - Prevención de osteoporosis en protmenopáusica en
pacientes con factores de riesgo.LA TERAPIA HORMONAL HA MOSTRADO SER EL TRATAMIENTO DE PRIMERA OPCIÓN PARA EL CONTROL DE LOS
SÍNTOMAS VASOMOTORES Y “ATROFIA UROGENITAL” DEL CLIMATERIO. Revisiones clínicas muestran que la terapia hormonal reduce la frecuencia
de síntomas vasomotores de un 80 % a 90 %, requiriéndose generalmente poco tiempo de uso, por lo que los riesgos son bajos. La TH combinada
consiste en la prescripción de estrógenos y progestágeno para el control del síndrome climatérico. La TH combinada está indicada en mujeres con
útero íntegro para reducir el riesgo de hiperplasia o cáncer de endometrio. La elección del esquema de administración de la TH combinada dependerá
de la elección de la paciente en cuanto a si desea o no continuar con sangrados cíclicos.

Bibliografía:
1. DECHERNEY A, NATHAN L, LAUFER N, ROMAN A. DIAGNÓSTICO Y TRATAMIENTO GINECOOBSTÉTRICOS, 11ª EDICIÓN. MC GRAW HILL EDUCATION, LANGE.
MÉXICO. 2014. 2. . GUÍA DE PRÁCTICA CLÍNICA, TRATAMIENTO DEL CONDILOMA ACUMINADO EN MUJERES EN EDAD REPRODUCTIVA EN LOS TRES NIVELES DE
ATENCIÓN. MÉXICO: SECRETARIA DE SALUD; JUNIO 2013.
HTTP://WWW.CENETEC.SALUD.GOB.MX/DESCARGAS/GPC/CATALOGOMAESTRO/ISSSTE_658_13_CONDILOMAACUMINADOMUJERES/GPC_ER_CONDILOMA_ACUMINADO.PDF
3. GUÍA DE PRÁCTICA CLÍNICA, DIAGNÓSTICO Y TRATAMIENTO DE LA PERIMENOPAUSIA Y POSTMENOPAUSIA. MÉXICO: SECRETARIA DE SALUD; 2013.
HTTP://WWW.CENETEC.SALUD.GOB.MX/DESCARGAS/GPC/CATALOGOMAESTRO/019_GPC_CLIMATYMENOP/SS_019_08_EYR.PDF

FIN DEL CASO CLÍNICO SERIADO


ANÁLISIS DEL CASO CLÍNICO

IDENTIFICACIÓN DEL REACTIVO


Area: CIRUGÍA
Especialidad: OTORRINOLARINGOLOGÍA
Tema: PATOLOGÍA DEL FARINGE Y LARINGE
Subtema: FARINGOAMIGDALITIS AGUDA

CASO CLÍNICO CON UNA PREGUNTA

MUJER DE 28 AÑOS DE EDAD, QUE ACUDE A SU CONSULTA DESESPERADA PORQUE EN LOS ÚLTIMOS 6
MESES HA PRESENTADO TRES CUADROS DE INFECCIÓN DE VÍAS RESPIRATORIAS SUPERIORES, A PESAR DE
HABER CUMPLIDO CON LOS TRATAMIENTOS INDICADOS POR LOS MÉDICOS QUE LA ATENDIERON. DESPUÉS
DE SU VALORACIÓN USTED DIAGNOSTICA UNA FARINGITIS AGUDA.

adulto de 28 años.

cuadros faríngeos de repetición,


cumpliendo tratamientos previos
adecuadamente.

-.

faringitis aguda.

-.

356 - CON RESPECTO A LAS ETIOLOGÍAS ANTERIORES Y ACTUAL USTED PODRÁ DECIRLE CON TODA
SEGURIDAD QUE:

EN MÁS DEL 90% DE La FARINGOAMIGDALITIS es una infección de la faringe y amígdalas que se caracteriza por
LOS PACIENTES LA garganta roja de más de cinco días de duración, afecta a ambos sexos y a todas las edades
ETIOLOGÍA ES VIRAL. pero es mucho más frecuente en la infancia. Las causas que condicionan la faringitis aguda
son los cambios bruscos de temperatura a finales del otoño y principios del invierno.
Existen factores de riesgo para desarrollar la enfermedad y factores que predisponen al
progreso hacia complicaciones como enfermedades inmunológicas, pacientes con
tratamientos antibióticos previos, infección persistente y falla del tratamiento. Se ha
documentado que la etiología más común de este padecimiento ES DE ORIGEN VIRAL
CON UN 90% en adultos, con infección bacteriana agregada, por lo que su tratamiento es
inicialmente sintomático durante los primeros tres días. La rinorrea, tos húmeda, disfonía
y conjuntivitis sugieren origen viral. Igual que la presencia de vesículas. LA GRAN
MAYORÍA DE LOS CUADROS DE FARINGITIS SON DE ORIGEN VIRAL.
A SU EDAD LA La faringitis es comúnmente causada por virus, pero aproximadamente 15% de los
ETIOLOGÍA MÁS episodios pueden deberse Estreptococo Beta Hemolítico del grupo A (EBHGA). En estos
FRECUENTE ES casos habitualmente se encuentra exudado blanquecino en amígdalas, adenopatía
BACTERIANA. cervical, ausencia de rinorrea y tos y fiebre mayor de 38°C; el diagnóstico clínico con 3 de
estos datos ha demostrado una sensibilidad de más de 75%. Los signos y síntomas típicos
de EBHGA se caracterizan por fiebre, adenopatía cervical anterior, hiperemia e hipertrofia
amigdalina con exudado purulento, cefalea, dolor abdominal, y otros ocasionales como
vómito, anorexia y rash o urticaria. La prueba de antígeno rápido de inmunoensayo se
utiliza para identificar EBHGA Tiene una sensibilidad de 61% al 95%, y especificidad de 88-
100%. Es barata y rápida. El cultivo faríngeo sigue siendo el estándar de oro para
confirmar el diagnóstico clínico de faringitis estreptocócica aguda, tiene una sensibilidad
de 90% -95%. El principal objetivo del tratamiento de la faringoamigdalitis por
estreptococo es prevenir el desarrollo de fiebre reumática y las complicaciones
supurativas. El tratamiento temprano contra EBHGA reduce el tiempo de sintomatología a
1-2 de los típicos 3 a 7. La penicilina es el antibiótico de elección. Diversos estudios
apoyan que la penicilina en diferentes dosis o vías de administración sigue siendo el
antibiótico de elección. En faringoamigdalitis aguda estreptocócica se recomienda el uso
de: - Penicilina compuesta de 1, 200,000 UI una aplicación cada 12 horas por dos dosis,
seguidas de 3 dosis de penicilina procaínica de 800,000 UI cada 12 horas intramuscular.
NO IMPORTANDO LA EDAD, LA ETIOLOGÍA MÁS FRECUENTE DE LA FARINGITIS AGUDA
ES DE TIPO VIRAL.

A SU EDAD LA Si después de tres días el paciente no presenta mejoría e incluso puede ser que empeore,
ETIOLOGÍA MÁS se puede considerar que pudiera estar cursando con una infección mixta, viral y
FRECUENTE ES MIXTA. bacteriana. EN NINGÚN CASO ESTA PODRÍA CONSIDERARSE LA ETIOLOGÍA MÁS
FRECUENTE. RECUERDA EN MENORES DE 4 AÑOS EN EL 99% DE LOS CUADROS DE
FARINGITIS LA ETIOLOGÍA ES VIRAL.

A SU EDAD NO EXISTE La fiebre reumática y la glomerulonefritis son complicaciones no supurativas de la


PELIGRO DE faringitis por EBHGA y requieren ser tratada con terapia antibiótica apropiada. Es la
COMPLICARSE CON principal razón para iniciar el tratamiento temprano cuando es detectado el
UNA Streptococcus. La Glomerulonefritis Aguda Postestreptocócica (GNAPE) es la inflamación
GLOMERULONEFRITIS. aguda del glomérulo renal posterior a la infección por estreptococos del grupo A. Se
manifiesta en la clínica como un síndrome nefrítico de inicio repentino caracterizado por
hematuria, la cual puede ser micro o macroscópica, hipertensión arterial de leve a severa,
oliguria, edema, elevación de azoados y en algunas oportunidades proteinuria. No toda
esta sintomatología va a estar presente en todos los casos. La GNAPE podría evitarse
poniendo énfasis en el diagnóstico etiológico temprano de las infecciones cutáneas y rino-
sinuso-faríngeas, sobre todo en la población pediátrica que es la más susceptible, sin
minimizar la importancia de esos cuadros como ocurre en la práctica diaria
implementando a posteriori la terapéutica antibiótica adecuada. ESTA ES UNA
ENFERMEDAD DESCRITA PRINCIPALMENTE EN LOS PACIENTES EN EDAD PEDIÁTRICA; SIN
EMBARGO, NO PUEDE DESCARTARSE COMPLETAMENTE A OTRAS EDADES.

Bibliografía:
1. GUÍA DE PRÁCTICA CLÍNICA, DIAGNÓSTICO Y TRATAMIENTO DE LA FARINGOAMIGDALITIS AGUDA.
MÉXICO: SECRETARIA DE SALUD; 2009. RECUPERADO DE
HTTP://WWW.CENETEC.SALUD.GOB.MX/CONTENIDOS/GPC/CATALOGOMAESTROGPC.HTML

http://www.cenetec.salud.gob.mx/descargas/gpc/CatalogoMaestro/073_GPC_Faringoamgaguda/Faringo_ER_CENETEC.pdf
ANÁLISIS DEL CASO CLÍNICO

IDENTIFICACIÓN DEL REACTIVO


Area: GINECOLOGÍA Y OBSTETRICIA
Especialidad: OBSTETRICIA
Tema: ENFERMEDADES QUE COMPLICAN EL EMBARAZO
Subtema: ENFERMEDADES VIRALES DURANTE EL EMBARAZO

CASO CLÍNICO SERIADO

SECUNDIGESTA EN LA TERCÉRA DÉCADA DE LA VIDA CON EMBARAZO DE 26 SDG, INICIA SU PADECIMIENTO


ACTUAL HACE 12 HRS CON ARTRALGIAS, MIALGIAS, FIEBRE DE 38 A 39°C, TOS PRODUCTIVA, CORIZA Y
RINORREA HIALINA. ANTE LA SOSPECHA INFECCCIÓN POR VIRUS DE LA INFLUENZA SE TOMA PRUEBA
RÁPIDA RESULTANDO POSITIVA.

-.

embarazo de 26SDG

12hrs de evolución, artralgias, mialgias,


ebre, tos productiva, coriza y rinorrea
hialina.

-.

prueba rápida para in uenza positiva.

357 - PARA CONFIRMAR EL CASO COMO INFLUENZA ES NECESARIO REALIZAR LA SIGUIENTE PRUEBA:

REACCIÓN EN CADENA DE La REACCIÓN EN CADENA DE LA POLIMERASA CON TRANSCRIPCIÓN REVERSA (RT-


LA POLIMERASA CON PCR) es la PRUEBA MÁS SENSIBLE Y ESPECÍFICA PARA EL DIAGNÓSTICO DE LA
TRANSCRIPCIÓN INVERSA. INFLUENZA, sus resultados están disponibles en alrededor de 4 a 6 horas. Siempre
que se tenga una prueba de tamiz o preliminar positiva para influenza deberá
confirmase por medio de una prueba confirmatoria como lo es la PCR (reacción en
cadena de la polimerasa) o el cultivo viral.

INMUNOFLUORESCENCIA. La tinción por INMUNOFLUORESCENCIA directa o indirecta de anticuerpos para la


detección de antígenos de influenza se emplea como estudio de tamizaje. Tiene una
sensibilidad y especificidad levemente menores que el aislamiento viral cultivado.
PRUEBAS DE DETECCIÓN Las pruebas rápidas para la detección de antígenos proveen resultados en 10 a 30
DE ANTÍGENOS. minutos, su desventaja es que tienen una sensibilidad baja en comparación con la RT-
PCR y el cultivo viral.

ENZIMOINMUNOANÁLISIS. El AISLAMIENTO VIRAL no es un estudio de tamizaje sin embargo está bien indicado
en temporadas bajas para la aparición de la influenza. Las PRUEBAS SEROLÓGICAS no
se recomiendan para detectar la infección por virus de la influenza humana en el
tratamiento de los cuadros agudos. La prueba de ENZIMOINMUNOANÁLISIS es una
prueba rápida por lo que no se considera confirmatoria.

Bibliografía:
1. GUÍA DE PRÁCTICA CLÍNICA, PREVENCIÓN DIAGNÓSTICO Y TRATAMIENTO DE LA INFLUENZA
ESTACIONAL. MÉXICO: SECRETARIA DE SALUD; 2010. 2. CUNNINGHAM G, LEVENO K, BLOMM S, HAUTH J,
RPUSE D, SONG C. WILLIAMS OBSTETRICIA, 23A EDICIÓN. MC GRAW HILL. USA. 2011

http://www.cenetec.salud.gob.mx/descargas/gpc/CatalogoMaestro/384_SS_09_influenza_estacional/EyR_SS_384_09.pdf

358 - DE CONFIRMARSE INFECCIÓN POR EL VIRUS DE LA INFLUENZA TIPO B DEBERÁ INICIARSE EL


TRATAMIENTO CON EL SIGUIENTE MEDICAMENTO:

RIMANTADINA. De acuerdo a las pautas de sensibilidad de los antivirales: - La infección por influenza A
estacional (H1N1) debe tratarse con zanamivir o un adamantano (de preferencia rimantadina) -
La infección por virus de la influenza A (H3N2) deberá tratarse con oseltamivir o zanamivir, NO
SE DEBE ADMINISTRAR AMANTADINA EN ESTE CASO DE INFLUENZA. - Si no se conoce el tipo
de influenza deberá tratarse con zanamivir o una combinación de oseltamivir y rimantadina. - La
infección por virus de la influenza B deberá tratarse sólo con oseltamivir o zanamivir.

OSELTAMIVIR. Actualmente se reconocen 2 tipos de antivirales: 1. Los ADAMANTANOS comprenden la


amantadina y la rimantadina. 2. Los INHIBIDORES de la neuraminidaza. Actualmente el
tratamiento de elección en México es con Oseltamivir a dosis de 75mg cada 12 hrs por 5 días vía
oral. También se recomienda para la prevención a dosis de 75mg diarios por 10 días vía oral.
IMPORTANTE: los fármacos antivirales que están indicados para el manejo de la influenza se
clasifican como categoría C, sin embargo se administran durante el embarazo debido a que los
beneficios potenciales superan los riesgo.

AMANTADINA. No se tiene mucho conocimiento sobre el uso de antivirales para la influenza durante el
embarazo. Todos ellos son fármacos categoría C tienen su indicación cuando los beneficios
potenciales superan los riesgos. DEBES TENER EN CUENTA QUE ACTUALMENTE SE REPORTA
UNA RESISTENCIA DE MÁS DEL 90% PARA AMANTADINA EN EL TRATAMIENTO DE LA
INFLUENZA.

ZANAMIVIR. La embarazada que estuvo en contacto con un paciente con diagnóstico de influenza o sospecha
de la misma deberá recibir tratamiento profiláctico con zanamivir u oseltamivir por 10 días. EL
OSELTAMIVIR CORRESPONDE, AL MENOS EN NUESTRO PAÍS LA PRIMERA OPCIÓN
TERAPÉUTICA Y EN SEGUNDO LUGAR EL ZANAMIVIR PARA ESTE CASO.

Bibliografía:
1. GUÍA DE PRÁCTICA CLÍNICA, PREVENCIÓN DIAGNÓSTICO Y TRATAMIENTO DE LA INFLUENZA
ESTACIONAL. MÉXICO: SECRETARIA DE SALUD; 2010. 2. CUNNINGHAM G, LEVENO K, BLOMM S, HAUTH J,
RPUSE D, SONG C. WILLIAMS OBSTETRICIA, 23A EDICIÓN. MC GRAW HILL. USA. 2011.

http://www.cenetec.salud.gob.mx/descargas/gpc/CatalogoMaestro/384_SS_09_influenza_estacional/EyR_SS_384_09.pdf

FIN DEL CASO CLÍNICO SERIADO


ANÁLISIS DEL CASO CLÍNICO

IDENTIFICACIÓN DEL REACTIVO


Area: CIRUGÍA
Especialidad: OFTALMOLOGÍA
Tema: TRAUMATISMOS OCULARES
Subtema: CONTUSIONES, LACERACIONES Y QUEMADURAS

CASO CLÍNICO CON UNA PREGUNTA

MUJER DE 17 AÑOS DE EDAD, QUE ACUDE AL SERVICIO DE URGENCIAS. POR QUE DESDE HACE TRES HORAS
POSTERIOR A GOLPE CONTUSO EN OJO IZQUIERDO, PRESENTA DOLOR DEL OJO A LA MOVILIZACIÓN Y
DISMINUCIÓN DE LA VISIÓN. EN LA EXPLORACIÓN. SE OBSERVA, AGUDEZA VISUAL FRANCAMENTE
DISMINUIDA, DERRAME SUBCONJUNTIVAL Y A LA OFTALMOSCOPÍA SE APRECIA CÁMARA ANTERIOR
TRANSPARENTE, REFLEJO ROJO AUSENTE Y NO ES POSIBLE OBSERVAR EL FONDO.

Adolescente de 17 años de edad.

trauma ocular.

Re ere disminución de la visión, elemento


fundamental.

Pérdida del re ejo rojo; lo que sugiere


pérdida de la transparencia vítrea
principalmente, siendo muy difícil
visualizar el fondo de ojo.

-.

359 - LA COMPLICACIÓN SECUNDARIA QUE PUEDE EXPLICAR LA SINTOMATOLOGÍA DEL PACIENTE, Y


DEBERÁ DESCARTARSE DE INMEDIATO ES:
HIFEMA Se denomina HIFEMA a la presencia de sangre en la "cámara anterior del ojo", la cual puede ser
PROGRESIVO. difícil de detectar si solamente existe una pequeña cantidad de ella, y en casos extremos se llena
completamente. En la mayoría de los casos, el hifema puede ser visto con una lámpara de bolsillo
ya que refleja daño de los vasos sanguíneos del iris. Usualmente indica trauma intraocular grave.
Entre las consecuencias del hifema, se encuentra glaucoma en el 7% de los pacientes, además de
opacidades corneales. El hifema puede ser resultado de una lesión ocular subyacente grave. Aun
en el caso de un pequeño sangrado, frecuentemente ocurre un nuevo sangrado espontáneo dentro
de los primeros cinco días, lo cual puede condicionar un hifema total, por lo que el paciente debe
ser referido al especialista. Ambos ojos deben ser ocluidos, el paciente usualmente hospitalizado,
colocado en reposo, y reevaluado frecuentemente. En presencia de hifema, la subsiguiente
aparición de dolor generalmente indica un nuevo sangrado y/o glaucoma agudo. LA PRESENCIA
DE HEMORRAGIA A NIVEL DEL IRIS (HIFEMA) SE PUEDE OBSERVAR A SIMPLE VISTA, EN ESTE
CASO NO SE DESCRIBE ASÍ.

CELULITIS. La CELULITIS ORBITARIA es la inflamación de los tejidos orbitarios que rodean al ojo. El tejido
periorbital se puede inflamar a través de dos formas: - Traumatismos: Que a pesar de ser más
frecuente en la primera infancia, se pueden producir a cualquier edad. En este grupo se incluye las
lesiones producidas por picadura de insectos. - Como foco secundario a partir de una bacteriemia,
sobre todo en niños pequeños (entre 3 y 36 meses) que están expuestos a mayor riesgo de
contraer bacteriemia a neumococo. Para su valoración es importante tener en cuenta que la órbita
está rodeada por: el piso del seno frontal, la pared interna del seno etmoidal y el techo del seno
maxilar. El seno etmoidal es la fuente más común de infección orbitaria debido a que éste se
separa de la órbita, solamente por una lámina papirácea. En la adolescencia, el seno frontal
también puede ser una fuente de celulitis orbitaria. Debido a que la sinusitis es un pre-requisito
para la celulitis orbitaria no traumática, este proceso se da más en niños mayores. La celulitis
periorbitaria se manifiesta con eritema, induración, edema y/o calor del tejido periorbital. En
aquellas lesiones secundarias a trauma o picadura de insecto es posible observar la lesión primaria
en la piel. Estos pacientes en general no presentan signos de enfermedad sistémica, tales como
fiebre y leucocitosis; y a las manifestaciones básicas de celulitis se agregan proptosis, quemosis
(edema de la conjuntiva bulbar), oftalmoplejía y disminución de la agudeza visual. A veces, debido
a la imposibilidad del operador de examinar el ojo, se debe solicitar una TAC de órbita con
contraste. En ella los hallazgos compatibles con celulitis orbitaria son: proptosis, inflamación de los
músculos oculares, absceso subperióstico y sinusitis ipsilateral o bilateral. El compromiso sinusal es
necesario para hacer el diagnóstico de celulitis orbitaria (infecciosa), pero no, así de celulitis
periorbitaria (traumática). AGENTE ETIOLÓGICO El germen causante depende de la patogénesis de
la infección. En celulitis periorbitaria secundaria a traumatismos, los gérmenes más frecuentes son
Staphylococcus Auereus y Estreptococo pyogenes. La celulitis secundaria a bacteriemia es
frecuentemente producida por el Estreptococo Pneumonae, sobre todo en niños entre 3 y 36
meses de edad. El Haemophilus I. dejó de ser el germen causal más frecuente desde la aplicación
de la vacuna conjugada. En la celulitis orbitaria que, como dijimos, es secundaria a un cuadro de
sinusitis aguda o crónica, el germen causal es el mismo que el de dicha entidad e inclusive puede
ser poli microbiana. TRATAMIENTO La CELULITIS PERIORBITARIA POSTRAUMÁTICA no
complicada generalmente se trata con antibióticos vía oral que cubren gérmenes Gram Positivos.
Como por ejemplo cefalexina, dicloxacilina o clindamicina. EL CASO CLÍNICO NO REFIERE DATOS
DE CELULITIS DURANTE LA EXPLORACIÓN FÍSICA.
HEMORRAGIA Un TRAUMA CONTUSO, también puede producir hemorragia. Esto se presenta ocasionalmente
VITREA. después del daño a un vaso sanguíneo y hemorragia dentro del humor vítreo, lo cual resulta en
una repentina y profunda pérdida de la vista. El examen fundoscópico puede ser imposible y se
pierde el reflejo rojo que se ve con la luz del oftalmoscopio. Los pacientes con esta lesión, deben
ser puestos en reposo con el ojo cubierto con un parche y enviados a un oftalmólogo. LA CLAVE ES
LA AUSENCIA DEL REFLEJO ROJO Y LA INCAPACIDAD PARA OBSERVAR EL FONDO QUE
TRADUCEN CAMBIOS EN LA TRANSPARENCIA DEL HUMOR VÍTREO. La HEMORRAGIA VÍTREA se
define como la presencia de sangre en la cavidad vítrea normal o por ruptura de nuevos casos de
retina, estas se producen cuando la sangre atraviesa la membrana interna o la hialoides posterior y
penetra en la cavidad vítrea. La hemorragia, de acuerdo a su ubicación, puede ser: - Intravítrea: en
las sustancias del vítreo. - Retrovítrea (subhialidea): cuando se produce en el espacio entre el
vítreo y la superficie de la retina. Se le llama hemorragia vítrea simple cuando se produce en
ausencia de otra patología intravítrea. Se trata de una indicación relativa para cirugía de vítreo. El
diagnóstico clínico de la hemorragia vítrea se establece con la disminución de la agudeza visual
(AV) y la exploración del fondo de ojo que en ocasiones permite observar la sangre y el nervio
óptico como faro en la niebla y en otras simplemente no permite observar el fondo de ojo. El dato
clínico que con mayor frecuencia refieren los pacientes con hemorragia vítrea es la baja visual
monocular no dolorosa acompañada de miodesopsias (puntos, rayas, telón, etc.). Se pueden
agregar fotopsias en algunos casos, y desgarro y/o desprendimiento de retina. La evaluación del
vítreo debe buscar la inserción o desprendimiento posterior y evaluar la periferia con depresión
escleral en busca de desgarros retinianos. Es necesaria la evaluación del segmento anterior pero
sobre todo con medios opacos, medición de presión intraocular. El tratamiento para las
hemorragias vítreas es conservador e incluye reposo con posición en semifowler y evitar
maniobras de Valsalva. Los pacientes con hemorragia vítrea sin otras enfermedades vítreas son
candidatos a manejo quirúrgico. Para ello se recomienda un manejo conservador previo de: 1) 10
semanas cuando el ultrasonido no revela desgarro retiniano, 2) 2 semanas en presencia de
desgarro, 3) 1 semana cuando el ultrasonido reporta desprendimiento de retina. IMPORTANTE:
todo paciente con factores de riesgo para hemorragia vítrea (en este caso traumatismo) y datos
clínicos que sugieran la enfermedad deberán ser valorados de inmediato por oftalmología ante el
riesgo de presentar desprendimiento de retina concomitante.

HEMORRAGIA Un trauma contuso también puede producir hemorragia en la retina. El paciente puede o no tener
DE RETINA. disminución de la agudeza visual, dependiendo de si la mácula está involucrada. La hemorragia
superficial de la retina aparece de color rojo cereza, las lesiones más profundas se observan de
color gris. En este caso no es posible observar la retina, lo que no descarta la hemorragia, pero
tampoco poderla confirmar. IMPORTANTE: todo paciente con factores de riesgo para hemorragia
vítrea (en este caso traumatismo) y datos clínicos que sugieran la enfermedad deberán ser
valorados de inmediato por oftalmología ante el riesgo de presentar desprendimiento de retina
concomitante.

Bibliografía:
1. GUÍA DE PRÁCTICA CLÍNICA, DIAGNÓSTICO Y TRATAMIENTO DE LA HEMORRAGIA VÍTREA. MÉXICO:
SECRETARIA DE SALUD; 2009. RECUPERADO DE
HTTP://WWW.CENETEC.SALUD.GOB.MX/CONTENIDOS/GPC/CATALOGOMAESTROGPC.HTML 2. RIORDAN P,
CUNNINGHAM E. VAUGHAN Y ASBURY OFTALMOLOGÍA GENERAL, 18A EDICIÓN. MC GRAW HILL LANGE.
2012.

http://www.cenetec.salud.gob.mx/descargas/gpc/CatalogoMaestro/177_GPC_HEMORRAGIA_VITREA/Gpc_hemorragia_vitrea.pdf
ANÁLISIS DEL CASO CLÍNICO

IDENTIFICACIÓN DEL REACTIVO


Area: PEDIATRÍA
Especialidad: INFECTOLOGIA PEDIÁTRICA
Tema: INFECCIONES VÍAS RESPIRATORIAS ALTAS
Subtema: ADENOIDITIS AGUDA

CASO CLÍNICO CON UNA PREGUNTA

PREESCOLAR DE 4 AÑOS DE EDAD CON ANTECEDENTE DE INFECCIONES RESPIRATORIAS ALTAS


RECURRENTES. LA MADRE REFIERE QUE DESDE HACE UN PAR DE MESES TIENE DIFICULTAD PARA RESPIRAR
POR LA NARIZ, POR LO QUE LO HACE POR LA BOCA Y PRESENTA RONQUIDOS NOCTURNOS. DESDE HACE
DOS DÍAS CON FIEBRE, SECRECIÓN PURULENTA NASAL Y HALITOSIS. A LA EXPLORACIÓN FÍSICA CON
AMÍGDALAS HIPERTRÓFICAS CONFLUENTES.

4 años.

infecciones respiratorias altas recurrentes.

respiración oral, ronquidos, ebre,


secreción purulenta nasal, halitosis.

amígdalas hipertró cas con uentes.

360 - CON BASE EN SU SOSPECHA DIAGNÓSTICA, LA SIGUIENTE ES LA COMPLICACIÓN MÁS GRAVE QUE
PUEDE PRESENTAR EL PACIENTE:

ABSCESO Las infecciones periamigdalinas se manifiestan como celulitis o abscesos francos de la región
PERIAMIGDALINO superior y lateral de la cápsula amigdalina. Suelen asociarse a infecciones amigdalinas de
repetición y son polimicrobianas, incluyendo aerobios y anaerobios. Los síntomas iniciales son
dolor faríngeo unilateral, otalgia ipsilateral, babeo y trismo. A la exploración se observa
desplazamiento caudal y medial de la amígdala afectada secundaria a tumefacción del pilar
amigdalino anterior y del paladar. El diagnóstico se hace mediante tomografía computada o
aspiración con aguja, esta última con envío de muestra para cultivo. EL ABSCESO
PERIAMIGDALINO SE RELACIONA MÁS CON INFECCIONES PROPIAS DE LA AMÍGDALA, MÁS
QUE DE LA ADENOIDES.
TROMBOSIS DEL La TROMBOSIS SÉPTICA DEL SENO CAVERNOSO responde a un proceso tromboflebítico de las
SENO venas intracraneales de origen infeccioso, vinculado principalmente a sinusitis y menos
CAVERNOSO frecuentemente a meningitis, otitis media, infecciones odontológicas, faríngeas, mastoiditis y
focos sépticos distales. El manejo consiste habitualmente en el mantenimiento de medidas
neuroprotectoras, la anticoagulación y el manejo intensivo de la infección. La recanalización
venosa, independientemente de su causa, suele lograrse a los 3 meses, los menos de los casos
requieren ampliación del tratamiento anticoagulante por 6 meses. LA TROMBOSIS DEL SENO
CAVERNOSO ES UNA COMPLICACIÓN EN LA QUE EL FOCO PRIMARIO MAYORMENTE
RELACIONADO ES LA SINUSITIS Y ÉSTE NO NECESARIAMENTE DEBE ESTAR VINCULADO A
PATOLOGÍA ADENOIDEA.

COR PULMONALE LO PRIMERO QUE DEBES HACER ES DEFINIR EL DIAGNÓSTICO QUE, PARA ESTE CASO,
HIPERTROFIA ADENOIDEA CON ADENOIDITIS AGUDA. La ADENOIDITIS EN NIÑOS comprende
tres grandes grupos: I. ADENOIDEOS DE TIPO RESPIRATORIO, II. ADENOIDEOS DE TIPO
AURICULAR (Otitis media supurada) y III. ADENOIDEOS DE TIPO INFECTANTE (infecciones
descendentes de las vías aéreas e infecciones de tubo digestivo). I. ADENOIDEOS DE TIPO
RESPIRATORIO: La obstrucción nasal es el primer síntoma que llama la atención de los padres,
el niño respira mal, mantiene la boca abierta y ronca por la noche y no se suena; parece tener
la nariz llena sin poder evacuarla. Todos estos trastornos se acentúan durante los corizas. A
veces, la facies es tan típica que se la denomina facies adenoidea: boca abierta, labio abierto
retraído, dejando los dientes al descubierto, bóveda palatina ojival y dientes mal colocados,
por atrofia del macizo óseo de la cara. Consecuencias de la obstrucción nasal: - VOZ NASAL
(rinolalia): El niño pronuncia mal las nasales, dice babá en lugar de mamá. - CEFALEA: Falta de
atención, inapetencia intelectual: un niño que después de desenvolverse bien hasta los 8 o 10
años se vuelve hepático, distraído, de comprensión lenta. Esto le ocurre desde que le han
atacado las vegetaciones. - TOS FARÍNGEA: Las vegetaciones determinan la producción de
pequeñas granulaciones en la pared posterior de la faringe, que provocan accesos de tos
incoercibles. - TRASTORNOS DEL DESARROLLO ESQUELÉTICO: Tórax retraído, hundimiento del
esternón, raquitismo. Se detiene el desarrollo del niño, el cual deja de crecer. - TRASTORNOS
REFLEJOS: Terrores nocturnos, pesadillas, el niño se orina en la cama, etc. Estos signos clásicos
generalmente se presentan. Con sólo interrogar a los padres de los adenoideos, se
evidenciarán algunos de estos síntomas. II. ADENOIDEOS DE TIPO AURICULAR: Hay ciertos
pacientes que no presentan ninguno de los signos procedentes, ni la menor obstrucción nasal,
pero aquejan trastornos auditivos, cuya causa ha escapado a la observación. Sordera
intermitente, zumbido, otalgias, a veces muy intensas. Estos síntomas aparecen siempre o
aumentan durante las corizas. En el niño operado, al igual que en el adulto, pueden persistir
“vegetaciones tubáricas” cuya importancia se ha revelado en aeronáutica; los portadores de
vegetaciones tubáricas presentan con frecuencia sordera ligera, latente, pero sobre todo al
viajar en avión, especialmente durante los descensos rápidos acusan un dolor auricular
violento, sordera y vértigo, circunstancias que muestran la aero-otitis. II. ADENOIDEOS DE
TIPO INFECTANTE. Hay niños que en el curso de corizas sufren de adenoiditis aguda con
fiebre, obstrucción y secreción purulenta nasal y faríngea, adenopatías submaxilares y
producción a distancia de: a) Otitis media supuradas de repetición. b) Infecciones
descendentes de las vías aéreas. Los accesos de falso crup, consecutivos a corizas, a menudo
son consecuencia de las vegetaciones, al igual que las laringitis o bronconeumonías, que tienen
el mismo origen. c) Numerosas infecciones del tubo digestivo, gastrointestinales, de la primera
infancia, cuando no se produce por causa alimenticia, tienen origen faríngeo. El enfermo no
sabe expectorar, deglute mucosidades sépticas que provocan los trastornos gastrointestinales.
CLARAMENTE LA PACIENTE PRESENTA SINTOMATOLOGÍA COMPATIBLE CON UNA
ADENOIDITIS AGUDA DE TIPO INFECTANTE. Los síntomas más característicos de la adenoiditis
son secundarios a la obstrucción del paso del aire por la nariz, al estar su salida posterior
obstruida. Es decir: • Respiración con la boca abierta de forma continua. • Ronquidos mientras
se duerme. • Timbre de voz característico, al tener la nariz tapada. • Halitosis por las mañanas.
• Congestión nasal y moqueo persistentes. • Infecciones de oídos (otitis) repetidas. Otra
complicación más de esta patología es la presencia de apneas durante el sueño (apnea
obstructiva del sueño). Excepcionalmente la obstrucción respiratoria y las apneas pueden
conducir a insuficiencia cardíaca (cor pulmonale), aunque infrecuente, es una complicación
muy grave. AFORTUNADAMENTE LA INSUFICIENCIA CARDÍACA SECUNDARIA A PROBLEMAS
RESPIRATORIOS (COR PULMONALE) ES UNA COMPLICACIÓN POCO COMÚN, PERO DEBES
CONSIDERARLA COMO LA MÁS GRAVE DE ÑAS OPCIONES PUESTO QUE PUEDE CONDUCIR A
FALLO CARDÍACO Y MUERTE.
SINUSITIS Las ADENOIDES son un agregado único de tejido linfático localizado entre el tabique nasal y la
CRÓNICA pared posterior de la faringe. Está separada por las estructuras adyacentes por una delgada
cápsula de tejido fibrótico; contiene criptas, pero son mucho más sencillas que las de las
amígdalas. La hipertrofia de adenoides junto con otras enfermedades como la rinitis alérgica,
enfermedad por reflujo gastroesofágico, inmunodeficiencias adquiridas o congénitas, pólipos
nasales, fibrosis quística, otitis media, bronquitis y tumores o cuerpos extraños, se han
relacionado con la presencia de RINOSINUSITIS bacteriana aguda y predisponentes para
eventos recurrentes o crónicos. En este sentido, la obstrucción adenoidea puede repercutir
sobre el movimiento normal de secreciones, condicionando problemas de inflamación crónica
o aguda recurrente que conllevan a sinusitis. LA SINUSITIS CRÓNICA PUEDE ASOCIARSE A
HIPERTROFIA ADENOIDEA, SIN EMBARGO, NO CONSTITUYE LA COMPLICACIÓN MÁS GRAVE.
A CONSIDERAR: las infecciones de vías respiratorias recurrentes predisponen a la hipertrofia
adenoidea por la presencia de acúmulo de tejido fibroso y, a la inversa, la hipertrofia
adenoidea puede predisponer a cronicidad de algunas infecciones respiratorias altas (otitis
media y sinusitis).

Bibliografía:
1. KLIEGMAN R, STANTON B, GEME J, SCR N, BEHRMAN R. NELSON TRATADO DE PEDIATRÍA, 19ª EDICIÓN.
ELSEVIER. ESPAÑA 2013.
ANÁLISIS DEL CASO CLÍNICO

IDENTIFICACIÓN DEL REACTIVO


Area: GINECOLOGÍA Y OBSTETRICIA
Especialidad: OBSTETRICIA
Tema: COMPLICACIONES DEL EMBARAZO Y DEL PARTO
Subtema: PARTO PREMATURO

CASO CLÍNICO SERIADO

MUJER DE 28 AÑOS GESTA 2 ABORTOS 1, CON EMBARAZO DE 30 SEG PRESENTA SANGRADO FRESCO E
INDOLORO TRANSVAGINAL. A LA EXPLORACIÓN LA ENCUENTRA CON SIGNOS VITALES ESTABLES, SIN
ACTIVIDAD UTERINA Y YA SIN PÉRDIDAS TRASVAGINALES.

-.

-.

SANGRADO FRESCO E INDOLORO


TRAnSVAGINAL.

SIGNOS VITALES ESTABLES SIN ACTIVIDAD


UTERINA Y SIN PÉRDIDAS VAGINALES.

-.

361 - EN ESTA PACIENTE ESTARÍA INDICADO PRINCIPALMENTE COMENZAR MANEJO CON:

PROGESTERONA. La placenta previa es una causa importante de hemorragia en el tercer trimestre del embarazo,
se presenta con sangrado vaginal rojo rutilante, indoloro e intermitente que coincide con el
cuadro clínico de la paciente. La PROGESTERONA desempeña un papel fundamental en la
preparación del útero para la implantación del óvulo recientemente fertilizado. De acuerdo a la
OMS el progestágeno debe eliminarse de la lista de tratamientos para prevenir el aborto
espontáneo, puede ser de utilidad en el aborto recurrente asociado a deficiencia de ésta
hormona. La progesterona se ha utilizado por mucho tiempo con coadyuvante del manejo del
aborto y su indicación se reserva durante el primer trimestre del embarazo. LA PACIENTE SE
ENCUENTRA EN EL TERCER TRIMESTRE DE GESTACIÓN DONDE A PESAR DE LA HEMORRAGIA
NO TIENE NINGUNA INDICACIÓN TERAPÉUTICA LA PROGESTERONA.
ESTRÓGENOS. Los ESTRÓGENOS no deben ser administrados durante el embarazo. El uso de estrógenos
durante el embarazo precoz puede causar serias lesiones en los embriones. Estudios en animales
sugieren que cercano al parto hay un aumento de estrógenos y disminución de progesterona
con la finalidad de promover la maduración fetal. LOS ESTROGENOS ESTAN
CONTRAINDICADOS EN EL EMBARAZO, Y AUNQUE SU USO, AUN NO CLARO EN LA
MADURACIÓN CERVICAL, PUDIERA HACERSE EN EL FINAL DEL EMBARAZO SE
CONTRAINDICAN EN LA PACIENTE YA QUE PRESENTA INMADUREZ FETAL.

INDOMETACINA La TERAPIA TOCOLÍTICA Puede retrasar el parto pretérmino, aunque no ha demostrado que
RECTAL. mejoren los resultados neonatales. Los objetivos de la terapia tocolítica son: - Otorgar un
tiempo suficiente para la administración de los corticosteroides prenatales. - Permitir la
referencia de la paciente con PP a una unidad hospitalaria de 3er nivel. OJO: La tocolisis puede
ser considerada para las mujeres con sospecha de parto prematuro que han tenido un embarazo
sin complicaciones; sin embargo, la tocolisis no debe usarse cuando existe una contraindicación
para prolongar el embarazo. La decisión de uso del agente tocolítico que el clínico elija debe
basarse en las características individuales de cada paciente. La evidencia apoya el uso de
fármacos tocolíticos a corto plazo (48 horas) para prolongar el embarazo durante al menos 48
horas que permita la administración de esteroides prenatales y que pueda permitir el transporte
de la madre a un centro de atención especializada y la administración de sulfato de magnesio
para reducir el riesgo de parálisis cerebral. En este sentido, se recomienda desde la semana 23.5
y hasta la 33.6 retrasar el nacimiento 48 para permitir el manejo con esteroides para
maduración pulmonar fetal o la referencia a un segundo nivel de atención. Se consideran como
agentes tocolíticos de primera elección los antagonistas de oxitocina, bloqueadores de canales
de calcio, beta miméticos y AINES; otros tocolíticos no se recomiendan por sus efectos adversos.
OJO: en gestaciones menores o iguales a 28 semanas, se recomienda continuar la terapia por
más de 48hrs y hasta por dos semanas; mientras que, para embarazos mayores de 29 semanas
no se debe extender la terapia tocolítica por más de 48hrs. La indometacina es usualmente
administrada por vía oral a dosis inicial de 50mg seguida de 25 a 50mg cada 6 horas por 48
horas. Su uso está limitado a gestaciones menores de 32 semanas. La evidencia muestra que los
inhibidores de la prostaglandina fueron más eficaces para disminuir la posibilidad de parto
pretérmino por 48 horas en comparación con placebo. Considerándose dentro de los 3 mejores
tocolíticos. LA INDOMETACINA RECTAL SE UTILIZA COMO AGENTE TOCOLÍTICO PARA
PREVENIR Y DETENER LAS CONTRACCIONES UTERINAS.

TERBUTALINA. La TERBUTALINA es un agonista B adrenérgico. Los agonistas adrenérgicos administrados por


vía oral o parenteral podrían dar lugar a relajación de la musculatura uterina e inhibición en el
parto por acción sobre receptores beta-2, de forma que la terbutalina se ha empleado en
ocasiones por vía parenteral para evitar partos prematuros durante el tercer trimestre, siempre
que no existan complicaciones. La TERBUTALINA al igual que otros adrenérgicos-beta-
miméticos, se usan con cierta reserva debido a la toxicidad cardíaca que producen en la madre.
Se utiliza casi sólo como una medida de rescate para ganar tiempo, siempre que no existan
otras alternativas terapéuticas y no deberá utilizarse por más de 72hrs. La GPC establece que su
uso deberá estar acompañado de un estricto control de la TA y de los líquidos. Los efectos
adversos asociados son: taquicardia, edema agudo de pulmón, hierglucemia, isquemia
miocárdica y hipokalemia. La vía de administración es subcutánea. SE UTILIZA COMO ÚLTIMO
RECURSO PARA EL MANEJO DE LA AMENAZA DE PARTO PRETÉRMINO PRINCIPALMENTE
COMO RELAJANTE UTERINO.

Bibliografía:
1. PREVENCIÓN, DIAGNÓSTICO Y TRATAMIENTO DE PARTO PRETÉRMINO. EVIDENCIAS Y
RECOMENDACIONES. GUÍA DE PRÁCTICA CLÍNICA. MÉXICO, CENETEC; 2017. 2. CUNNINGHAM G, LEVENO K,
BLOMM S, HAUTH J, RPUSE D, SONG C. WILLIAMS OBSTETRICIA, 23A EDICIÓN. MC GRAW HILL. USA. 2011
EN ESPAÑOL. 3. DECHERNEY A, NATHAN L, LAUFER N, ROMAN A. DIAGNÓSTICO Y TRATAMIENTO
GINECOOBSTÉTRICOS, 11ª EDICIÓN. MC GRAW HILL EDUCATION, LANGE. MÉXICO. 2014. 4. PREVENCIÓN
PRIMARIA Y TAMIZAJE DEL PARTO PRETÉRMINO EN EL PRIMER NIVEL DE ATENCIÓN. EVIDENCIAS Y
RECOMENDACIONES: GUÍA DE PRÁCTICA CLÍNICA. MÉXICO: CENETEC; 2018.

http://www.cenetec-difusion.com/CMGPC/IMSS-063-08/ER.pdf

362 - LA INTERRUPCIÓN INMEDIATA DEL EMBARAZO ESTARÍA INDICADO SI SE ENCONTRARA:


SUFRIMIENTO Cuando se presenta sangrado transvaginal sugerente de placenta previa, lo primero es
FETAL. confirmar el diagnóstico, si el estado materno-fetal es estable, no existe indicación para
interrumpir el embarazo. Si hay presencia de sufrimiento fetal está indicado realizar cesárea
con previa inducción de maduración fetal. RECUERDA QUE LA TOCOLISIS SE REALIZA CON
LA FINALIDAD DE APLAZAR UN POCO EL TRABAJO DE PARTO PARA GANAR TIEMPO PARA
LA APLICACIÓN DE INDUCTORES DE MADURACIÓN PULMONAR, SI EN ESE TRANSCURSO
HUBIERA SUFRIMIENTO FETAL, NO TENDRÍA CASO CONTINUAR CON LA GESTACIÓN.

OLIGOHIDRAMNIOS. Se define al OLIGOHIDRAMNIOS como la presencia de un índice de líquido amniótico


inferior a 5 o de una máxima columna vertical inferior a 2 cm. La principal causa de
oligohidramnios es malformación fetal; si se diagnostica ésta tempranamente, se informará
del pronóstico y riesgo a los padres y estos podrían acogerse a la interrupción legal del
embarazo antes de la semana 20. Si deciden seguir adelante con la gestación debemos
realizar amniocentesis/cordocentesis para estudio de cariotipo y valorar el estudio de la
función renal en orina fetal. Ante la presencia de oligohidramnios debe se mantendrá una
conducta expectante hasta las 40 semanas de gestación si el control de bienestar fetal es
normal. Por encima de las 37 semanas ante condiciones cervicales favorables valorar la
finalización de la gestación. EN EL TERCER TRIMESTRE DE EMBARAZO CON
OLIGOHIDRAMNIOS NO ESTÁ INDICADA LA INTERRUPCIÓN DEL MISMO, LA CONDUCTA ES
ESPECTANTE A MENOS QUE HAYA DATOS DE SUFRIMIENTO FETAL ANTES DE LLEGAR A
TÉRMINO.

CERVICOVAGINITIS. Las INFECCIONES DEL TRACTO GENITAL femenino son la causa de muchos casos de parto
pretérmino, no se encuentra una indicación clara en la interrupción del embarazo. Ante la
presencia de cervicovaginitis en el tercer trimestre la indicación es otorgar tratamiento
específico para la infección y mantener una terapia expectante hasta lograr madurez fetal, a
menos que haya sufrimiento fetal antes. LA CERVICOVAGINITS NO ES UNA INDICACIÓN DE
INTERRUPCIÓN DEL EMBARAZO.

RESTRICCIÓN DEL Se denomina RESTRICCIÓN DEL CRECIMIENTO INTRAUTERINO cuando el peso de un recién
CRECIMIENTO. nacido está por debajo del percentil lo para su edad gestacional. Si se detecta que la unidad
uteroplacentaria no esté funcionando adecuadamente y que el feto puede estar mejor
afuera que adentro del vientre materno, se decidirá la interrupción del embarazo. Cuando
se cuenta con recursos de neonatología adecuados se puede interrumpir el embarazo desde
las 32 a 34 semanas, no antes, de preferencia por cesárea, con resultado neonatal
satisfactorio. LA RESTRICCIÓN DEL CRECIMIENTO INTRAUTERINO ESTÁ INDICADA SÓLO SI
SE FUNDAMENTA BIEN UN DETERIORO PLACENTARIO QUE PONGA EN RIESGO AL FETAL.
POR LO QUE NO CONSTITUYE UNA INDICACIÓN DE INTERRUPCIÓN INMEDIATA.

Bibliografía:
1. CUNNINGHAM G, LEVENO K, BLOMM S, HAUTH J, RPUSE D, SONG C. WILLIAMS OBSTETRICIA, 23A
EDICIÓN. MC GRAW HILL. USA. 2011 EN ESPAÑOL, PP 826.

FIN DEL CASO CLÍNICO SERIADO


ANÁLISIS DEL CASO CLÍNICO

IDENTIFICACIÓN DEL REACTIVO


Area: MEDICINA INTERNA
Especialidad: INFECTOLOGÍA
Tema: INFECCIONES GASTROINTESTINALES
Subtema: DIARREA AGUDA

CASO CLÍNICO CON UNA PREGUNTA

FEMENINO DE 35 AÑOS DE EDAD, CON ANTECEDENTE DE DIARREA ACUOSA DE 2 MESES DE EVOLUCIÓN.


DURANTE SU ESTUDIO SE DIAGNOSTICA INTOLERANCIA A LA LACTOSA.

Femenino de 35 años de edad.

Intolerancia a la lactosa.

Diarrea acuosa de 2 meses de evolución.

-.

-.

363 - EL MECANISMO FISIOPATOLÓGICO QUE PRODUCE LA DIARREA ES DEBIDO A QUE EL DISACÁRIDO:

ESTIMULA LAS Las DIARREAS SECRETORAS se deben a "alteraciones del transporte de los líquidos y electrolitos",
VELLOSIDADES a través de la mucosa intestinal. Clínicamente se caracterizan por ser muy voluminosas, acuosas,
típicamente indoloras, y persistentes a pesar del ayuno. Como no hay mala absorción de solutos,
la osmolaridad fecal depende de la normalidad de los electrolitos endógenos sin que exista
diferencia osmótica fecal. Causas de diarrea secretora son algunos laxantes, ingesta de etanol por
largo tiempo, algunos fármacos y toxinas, ablación, enfermedades o fístulas intestinales, tumores
hormonógenos (Carcinoide, VI Poma, cáncer de la médula tiroidea, mastocitosis, gastrinoma,
adenoma velloso colorrectal), entre otras. LA ALTERACIÓN DEL TRANSPORTE DE LÍQUIDOS Y
ELECTROLITOS A NIVEL INTESTINAL SE DA A MENUDO POR EFECTO DE ALGUNAS TOXINAS U
OTRAS ENFERMEDADES PROPIAS DEL INTESTINO, NO RELACIONADAS CON LA LACTOSA.

SE En realidad no ocurre un intercambio de azúcares, más bien, independientemente del efecto


INTERCAMBIA osmótico de la lactosa, ésta es "fermentada por las bacterias intestinales" y esta fermentación
POR OTROS contribuye al desarrollo de algunos de los síntomas.
AZÚCARES
NO SE La MALA ABSORCIÓN DE CARBOHIDRATOS a causa de defectos congénitos o adquiridos de las
HIDROLIZA Y disacaridasas y otras enzimas del borde en cepillo de los enterocitos, producen "diarrea osmótica"
ES OSMÓTICO con un pH bajo. Una de las causas más frecuentes de diarrea crónica en los adultos es la
deficiencia de lactasa, que afecta a 75 % de la población no blanca de todo el mundo; la mayoría
de estos pacientes dejan de tomar productos lácteos por propia iniciativa, basándose en su
experiencia. Algunos azúcares, como el sorbitol, se absorben siempre mal, y hay muchos
medicamentos, chicles o caramelos endulzados con estos azúcares no absorbibles, cuya ingestión
va seguida de diarrea. En ausencia o deficiencia de actividad de las disacaridasas, los disacáridos
de la dieta "no son hidrolizados" y por lo tanto tampoco absorbidos. Lo mismo ocurre con la
malabsorción de monosacáridos. Rápidamente la flora bacteriana colónica se adapta a la llegada
de azúcares e inicia la hidrólisis utilizando su propia maquinaria enzimática. Esto resulta en la
producción de ácidos grasos volátiles (acético, butírico y propiónico) y gases (metano, dióxido de
carbono e hidrógeno). "La presencia de disacáridos o monosacáridos en el intestino grueso tiene
además efecto osmótico", a veces suficiente como para producir diarrea. La producción de gases
es responsable de la aparición de flatulencia, meteorismo y dolor abdominal; la presencia de
ácidos grasos por otra parte, explica la acidificación de las deposiciones, que resulta en valores de
pH menores a 5,5. LA DIARREA PRODUCIDA POR LA INTOLERANCIA A LA LACTOSA ES DE TIPO
OSMÓTICA.

ES UTILIZADO Los síntomas producidos fuera de la diarrea en los pacientes con intolerancia a la lactosa son
POR LA FLORA producto de la fermentación de la lactosa que no se absorbió por las bacterias intestinales. Éstos
NORMAL son distensión abdominal, flatulencias y dolor, pero más bien, ésto es una consecuencia de la
mala absorción, no son directamente la causa de la diarrea. NO ES LA CAUSA DE DIARREA SINO
DE OTROS SÍNTOMAS COMO DISTENSIÓN Y FLATULENCIAS.

Bibliografía:
1. LONGO DL, FAUCI AS, KASPER DL, HAUSERSL, JAMESON JL, LOSCALZOJ. HARRISON. PRINCIPIOS DE
MEDICINA INTERNA, 18A EDICIÓN. MC GRAW HILL. NEW YORK, USA. 2012, PP 2465-2466. 2.. PAPADAKIS
MAXINE A, MCPHEE STEPHEN J. DIAGNÓSTICO CLÍNICO Y TRATAMIENTO. 52ª EDICIÓN. NUEVA YORK. 2013,
PP 577-578.
ANÁLISIS DEL CASO CLÍNICO

IDENTIFICACIÓN DEL REACTIVO


Area: MEDICINA INTERNA
Especialidad: INMUNOALERGIA
Tema: RINITIS ALÉRGICA
Subtema: RINITIS ALÉRGICA

CASO CLÍNICO CON UNA PREGUNTA

DURANTE SU ROTACIÓN COMUNITARIA EN UN CENTRO DE SALUD SE LE SOLICITA REALIZAR UNA GUÍA DE


PRÁCTICA CLÍNICA PARA LA ATENCIÓN DE LAS ENFERMEDADES ALÉRGICAS MÁS COMUNES.

-.

-.

-.

-.

-.

364 - CON BASE EN SU INVESTIGACIÓN USTED CONSIDERARÁ A LA ENFERMEDAD ALÉRGICA QUE MÁS
DEMANDA DE CONSULTA PRODUCE EN EL PRIMER NIVEL DE ATENCIÓN A LA SIGUIENTE:

URTICARIA. La URTICARIA tiene gran variedad de presentaciones clínicas y causas. Es un síndrome reaccional
de piel y mucosas que se caracteriza por la presencia de ronchas o placas eritemato- edematosas
y transitorias de tamaño variable. Las lesiones son ocasionadas por edema vasomotor transitorio
y circunscrito de la dermis que dura algunas horas; puede ser recidivante. El angioedema se
manifiesta típicamente como un edema asimétrico causado por la presencia de plasma dentro de
tejido celular subcutáneo y mucosas. Es una de las enfermedades dermatológicas más frecuentes.
Se calcula que la urticaria aguda afecta a 20% de la población a escala mundial, su origen puede
ser inmunológico, no inmunológico o desconocido. Predomina en mujeres de 40 a 50 años de
edad. Afecta 6-7% de los niños preescolares y a 17% de los niños con dermatitis atópica. La
mayoría de los pacientes (aproximadamente 50%) padece tanto urticaria como angioedema, 40%
sólo urticaria y 10% sólo angioedema. Tiene diversas clasificaciones la más frecuentemente
utilizada es con base al tiempo de evolución, se divide en aguda (menos de 6 semanas) o crónica
(más de 6 semanas). En la urticaria crónica la causa específica se determina en no más de 30% de
los casos.
ASMA. El ASMA es la enfermedad crónica más frecuente en la infancia y adolescencia. La importancia de
esta patología radica en el carácter de enfermedad crónica que afecta a la calidad de vida, al
ausentismo escolar y en los elevados costos sanitarios que genera. Se estima que un elevado
porcentaje del costo que origina el asma está ocasionado por su inadecuado control. La
trascendencia de esta patología se destaca en su elevada prevalencia y su gran variabilidad,
siendo su prevalencia entre 8 y 20%. En México, no se cuenta con estudios epidemiológicos
acerca de la prevalencia del padecimiento en niños menores de cinco años de edad. Sin embargo,
los datos proporcionados por el Sistema Único de Información para la Vigilancia Epidemiológica,
dependiente de la Dirección General de Epidemiología de la Secretaría de Salud, señalaron que
entre 2001 y 2006 la tendencia del asma en los niños menores de cinco años de edad tuvo un
incremento de 31%, ya que pasó de 479.44 a 627.95 por 100,000 habitantes en ese lapso.

RINITIS La RINITIS ALÉRGICA a es una enfermedad inflamatoria crónica de la mucosa nasal mediada por
ALÉRGICA. anticuerpos IgE alérgeno-específicos, con participación de diversas células, citosinas y
mediadores. LA RINITIS ALÉRGICA ES LA PRIMERA CAUSA DE CONSULTA EN ALERGOLOGÍA EN
MUCHOS PAÍSES INCLUYENDO MÉXICO, Y UNA DE LAS DIEZ PRINCIPALES EN LA ATENCIÓN
PRIMARIA. LA PREVALENCIA MUNDIAL DE RINITIS ALÉRGICA HA AUMENTADO EN LAS ÚLTIMAS
DÉCADAS Y LA RINITIS ALÉRGICA ES LA PRIMERA CAUSA DE CONSULTA EN ALERGOLOGÍA EN
MUCHOS PAÍSES INCLUYENDO MÉXICO Y UNA DE LAS DIEZ PRINCIPALES EN LA ATENCIÓN
PRIMARIA. LA PREVALENCIA MUNDIAL DE RINITIS ALÉRGICA HA AUMENTADO EN LAS ÚLTIMAS
DÉCADAS Y SE ESTIMA ENTRE 10 Y 25%. Los síntomas principales, desencadenados por la
exposición a alérgenos, son la rinorrea, obstrucción nasal, prurito, estornudos en salva, los cuales
remiten espontáneamente o con tratamiento. Frecuentemente los pacientes con rinitis alérgica
presentan síntomas conjuntivales y de asma.

CONJUNTIVITIS La primavera es la época del año en que los síntomas de la CONJUNTIVITIS ALÉRGIA estacional
PRIMAVERAL. empeoran. Puede afectar a personas de cualquier edad, aunque la máxima incidencia se da entre
los 18 y los 35 años, sin diferencias entre los dos sexos. La incidencia de padecimientos alérgicos
oculares varía marcadamente dependiendo de la región geográfica, aunque es más común en
lugares de clima cálido. En México se desconoce la incidencia de la conjuntivitis alérgica; sin
embargo, un 80% de los pacientes también tiene rinitis alérgica, asma o dermatitis atópica; en un
porcentaje menor es concomitante con alergia a fármacos y alimentos o urticaria. En la infancia
se manifiesta ocasionalmente como enfermedad única o relacionada con otros padecimientos
alérgicos. Existe normalmente un factor genético, por lo que personas con antecedentes
familiares de alergia tienen más probabilidades de desarrollarla. Los alérgenos más comunes en
esta afección son los pólenes de los árboles (olivo, plátano, ciprés, etc.) o de gramíneas. El picor
es el principal síntoma de esta enfermedad. Además, puede presentarse lagrimeo, ojos rojos,
hinchazón de los párpados, secreciones, dolor ocular y, en casos severos, fotofobia y visión
borrosa.

Bibliografía:
1. GUÍA DE PRÁTICA CLÍNICA, DIAGNÓSTICO Y TRATAMIENTO DE RINITIS ALÉRGICA. MÉXICO: SECRETARIA
DE SALUD; 2009. 2. GUÍA DE REFERENCIA RÁPIDA, DIAGNÓSTICO Y TRATAMIENTO DE RINITIS ALÉRGICA.
MÉXICO: SECRETARIA DE SALUD; 2009.

http://www.cenetec.salud.gob.mx/descargas/gpc/CatalogoMaestro/041_GPC_RinitisAlergica/IMSS_041_08_EyR.pdf
ANÁLISIS DEL CASO CLÍNICO

IDENTIFICACIÓN DEL REACTIVO


Area: PEDIATRÍA
Especialidad: URGENCIAS PEDIÁTRICAS
Tema: URGENCIAS NEUMOLÓGICAS PEDIÁTRICAS
Subtema: ASMA

CASO CLÍNICO CON UNA PREGUNTA

PREESCOLAR DE 3 AÑOS DE EDAD, ESTABA SOLO EN UN CUARTO JUGANDO, PRESENTA SÚBITAMENTE TOS,
DISFONÍA, CIANOSIS, ALETEO NASAL Y SIBILANCIAS MOTIVO POR EL CUAL ES LLEVADO A URGENCIAS.

PREESCOLAR DE 3 AÑOS DE EDAD

ESTABA SOLO EN UN CUARTO JUGANDO

PRESENTA SúBITAMENTE TOS, DISFONíA,


CIANOSIS, ALETEO NASAL Y SIBILANCIAS.

--

--

365 - EL CUADRO CLÍNICO CORRESPONDE A:

EPIGLOTITIS AGUDA La EPIGLOSITIS AGUDA ha sido reconocida como una enfermedad rápidamente progresiva
que puede amenazar la vida y que afecta a niños en la mayoría de los casos. Es de origen
infeccioso, el organismo causal más común es el Hemophylus influenzae tipo B. CX. Los tres
signos más comunes son: fiebre de inicio súbito, dificultad para respirar e irritabilidad. Los
niños usualmente llegan con apariencia tóxica, con signos clínicos de obstrucción de la vía
aérea superior, estridor inspiratorio, taquipnea y sialorrea; hay disfagia y el habla se ve
limitada por el dolor. NO SE DESCRIBE EN EL PACIENTE ANTECEDENTE DE INFECCIONES,
FIEBRE NI DOLOR.
LARINGOTRAQUEITIS LARINGOTRAQUEITIS es el proceso inflamatorio de origen infeccioso que abarca laringe,
AGUDA tráquea y que invade rinofaringe llegando también a involucrar la parte inferior del árbol
respiratorio, produciendo diversos grados de obstrucción. CX: El inicio de la enfermedad
sugiere un resfriado común. Los síntomas iniciales son obstrucción nasal, rinorrea, coriza y
fiebre; después de 12 a 48 horas, los signos de obstrucción respiratoria superior hacen su
aparición, la tos es perruna, hay disfonía y el estridor se hace progresivo; la fiebre fluctúa
entre 37.8 y 40 grados centígrados, la progresión de la enfermedad y el grado de
obstrucción es variable. SE CARACTERIZA POR TOS "PERRUNA" Y ANTECEDENTE DE
INFECCIÓN DE VÍAS RESPIRATORIAS LO QUE NO CORRESPONDE AL CASO CLÍNICO.

CRÍSIS ASMÁTICA CRISIS ASMÁTICA es el empeoramiento rápido de los síntomas y signos del asma que no
mejora con el aumento de la dosis de la medicación usada habitualmente. CX: tos,
dificultad para respirar, dificultad para hablar, sibilancias. NO HAY ANTECEDENTE DE
ASMA NI EXPOSICIÓN A SITUACIONES QUE PUDIERAN PROVOCAR UNA CRISIS ASMÁTICA.

CUERPO EXTRAÑO CX CUERPO EXTRAÑO EN VÍAS AÉREAS Se describen tres fases clínicas distintas tras la
EN LAS VÍAS AÉREAS aspiración de un cuerpo extraño. 1. Período inmediato postaspiración, se manifiesta por
una tos súbita, violenta, con asfixia, cianosis, ahogo, estridor y sibilancias. 2. Período
asintomático que puede ser de minutos a meses de duración, dependiendo de la
localización del cuerpo extraño, del grado de obstrucción que produce y de la reacción
inflamatoria que genere. 3. Se manifiesta la patología derivada de la reacción de cuerpo
extraño generada: inflamación- infección, dando síntomas como tos crónica,
expectoración, fiebre, sibilancias y, a veces, hemoptisis. EL ANTECEDENTE DE ESTAR
JUGANDO SIN VIGILANCIA AUMENTA LA SOSPECHA DIAGNÓSTICA. EL PACIENTE
PRESENTA EL CUADRO CARACTERÍSTICO DE LA FASE AGUDA.

Bibliografía:
TRATADO DE PEDIATRÍA. NELSON. MC GRAW HILL. EDICIÓN 16. 2000. PAG. 1400-1401.
ANÁLISIS DEL CASO CLÍNICO

IDENTIFICACIÓN DEL REACTIVO


Area: MEDICINA INTERNA
Especialidad: NEUROLOGÍA
Tema: DEMENCIAS
Subtema: DEMENCIA Y ENFERMEDAD DE ALZHEIMER

CASO CLÍNICO CON UNA PREGUNTA

HOMBRE DE 70 AÑOS DE EDAD, ES LLEVADO A CONSULTA POR SUS FAMILIARES AL NOTAR PÉRDIDA
IMPORTANTE DE LA MEMORIA A LARGO Y CORTO PLAZO. DURANTE SU VALORACIÓN SE DETERMINA SUFRE
DE DEMENCIA.

70 años. (principal factor de riesgo


envejecimiento).

-.

Pérdida importante de memoria a corto y


largo plazo.

DEMENCIA.

-.

366 - ES EL TIPO MÁS COMÚN DE DEMENCIA:

DEMENCIA La DEMENCIA FRONTOTEMPORAL suele comenzar a partir de los 50 años y en este grupo de
FRONTOTEMPORAL. edad, su prevalencia es similar a la de Alzheimer.

ENFERMEDAD DE La ENFERMEDAD DE ALZHEIMER, es una enfermedad neurológica progresiva e irreversible


ALZHEIMER. que afecta al cerebro, ES LA CAUSA MÁS COMÚN DE DEMENCIA y se caracteriza por pérdida
progresiva de la memoria y un progresivo deterioro de las actividades básicas de la vida
diaria así como cambios de conducta. Es la causa más común de demencia en países del
mundo occidental. Alrededor de 10% de todas las personas de 70 años o más muestran
amnesia notable y en más de la mitad de los casos, tal deficiencia es causada por AD. Sin
embargo, la enfermedad puede aparecer en cualquier decenio de la vida adulta. Se ha
calculado que en Estados Unidos el coste anual por cuidar a un solo paciente con AD en
etapa avanzada de su trastorno es de unos 50 000 dólares. La enfermedad también impone
una gran carga emocional a los parientes y cuidadores. La AD muy a menudo comienza con
la pérdida sutil de la memoria, seguida de demencia de evolución lenta, que dura varios
años.
DEMENCIA La DEMENCIA VASCULAR es el síndrome que cursa con demencia secundaria a lesiones
VASCULAR. cerebrales causadas por Enfermedad Cerebrovascular (ECV). Para definir DEMENCIA
VASCULAR se requiere de tres requisitos: el paciente debe cumplir con criterios de
demencia, debe existir la evidencia de Enfermedad Cerebrovascular por historia, exploración
y técnicas de neuroimagen y ambos requisitos deben estar razonablemente relacionados.
Son hechos sugerentes de DEMENCIA VASCULAR: el deterioro intelectual agudo en los tres
meses siguientes a un accidente vascular cerebral ,con evolución fluctuante o escalonada,
historia de alteración de la marcha y caídas frecuentes, signos positivos en la exploración
neurológica, predominio de déficit cognitivos subcoriales sobre los corticales así como la
presencia de factor de riesgo cardiovascular.

DEMENCIA POR La demencia puede acompañar al alcoholismo crónico y ser consecuencia de la desnutrición
ALCOHOLISMO. coexistente, sobre todo de vitamina B, de la cual predomina la deficiencia de tiamina.

Bibliografía:
1. GUÍA DE PRÁCTICA CLÍNICA, DIAGNÓSTICO Y TRATAMIENTO DE LA DEMENCIA TIPO ALZHEIMER.
MÉXICO: SECRETARIA DE SALUD; 2010. 2. LONGO DL, FAUCI AS, KASPER DL, HAUSERSL, JAMESON JL,
LOSCALZOJ. HARRISON. PRINCIPIOS DE MEDICINA INTERNA, 18A EDICIÓN. MC GRAW HILL. NEW YORK,
USA. 2012.

http://www.cenetec.salud.gob.mx/descargas/gpc/CatalogoMaestro/393_IMSS_10_Demencia_Alzheimer/EyR_IMSS_393_10.pdf
ANÁLISIS DEL CASO CLÍNICO

IDENTIFICACIÓN DEL REACTIVO


Area: CIRUGÍA
Especialidad: CIRUGÍA ABDOMINAL
Tema: SANGRADO DE TUBO DISGESTIVO ALTO Y BAJO
Subtema: HEMORRAGIA DIGESTIVA SUPERIOR

CASO CLÍNICO SERIADO

HOMBRE DE 70 AÑOS CON PÉRDIDA DEL ESTADO DE ALERTA CON ALIENTO ALCOHÓLICO IMPORTANTE, ES
LLEVADO AL SERVICIO DE URGENCIAS POR PARAMÉDICOS DE AMBULANCIA AL SER ENCONTRADO TIRADO
EN LA CALLE. A SU INGRESO SE ENCUENTRA CON PALIDEZ IMPORTANTE +++, ESTUPOROSO Y CON LOS
SIGUIENTES SIGNOS VITALES, TA 50/20 MM HG, FC 130 POR MINUTO, FR 30 POR MINUTO, TEMPERATURA 35.6°C.
POSTERIOR A LA INFUSIÓN DE 2 LITROS DE CRISTALOIDES SE LOGRA LA REANIMACIÓN.

masculino de 70 años de edad.

pérdida del estado de alerta, aliento


alcohólico.

-.

palidez importante +++, estuporoso,


hipotensión 50/20 mm hg,taquicardia
130xmin, fr de 30xmin, temperatura de
35.6°c. se logra reanimar con la
administración de 2 lts. de cristaloides.

-.

367 - EL DIAGNÓSTICO CLÍNICO MAS PROBABLE ES:


CETOACIDOSIS PARA PODER CONTESTAR PREGUNTAS COMO ESTA DEBERÁS PRIMERO ANALIZAR Y
DIABÉTICA. DETERMINAR EL DIAGNÓSTICO CLÍNICO QUE EN ESTE CASO ES UN ESTADO DE CHOQUE. La
CETOACIDOSIS DIABÉTICA (CAD) constituye todavía una causa importante de morbilidad en
pacientes diabéticos mal tratados o inadecuadamente instruidos. La incidencia anual varía entre
4-8 episodios por cada 1000 pacientes al año y es causa del 20 al 30% de las formas de
presentación de una diabetes tipo 1 e inicialmente se caracteriza por una producción aumentada
de cuerpos cetónicos con elevadas concentraciones plasmáticas de los ácidos acetoacéticos e
hidroxibutírico. Los Factores precipitantes de la Cetoacidosis Diabética son: Abandono o errores
en la administración de la Insulina, inicio de una Diabetes Juvenil, infecciones como las
neumonías o de vías respiratorias altas, amigdalitis, infecciones Urinarias, Colecistitis, y Otras. Las
causas vasculares como Infarto de Miocardio, Enfermedad Cerebro Vascular, Traumatismos
graves, Estrés Psíquico y algunos fármacos: Glucocorticoides, diazóxido, difenilhidantoina, etc. Se
describe un típico dolor abdominal difuso que también tiende a desorientar al médico hacia un
cuadro abdominal agudo pero que muchas veces desaparece al estabilizar clínicamente al
paciente. No obstante es importante descartar patologías abdominales como la pancreatitis,
apendicitis y colecistitis aguda entre otros. Los vómitos y la poliuria muy frecuentes llevan a la
deshidratación del individuo a extremos tales que pueden desencadenar hipovolemia franca que
a su vez desencadena hipotensión, oliguria e insuficiencia renal aguda asociados a mal pronóstico
en estos sujetos. Hoy en día se reconocen como complicaciones graves de la CAD a la acidosis
respiratoria, el choque, la presencia de arritmias, la insuficiencia cardiaca, la hipocalcemia; el pH
de ingreso menor de 7,10 e insuficiencia renal aguda y todas las complicaciones anteriores
implican alto riesgo de mortalidad, por lo que estos pacientes idealmente deben ser manejados
en una Unidad de Cuidados Intensivos. ESTÁ CLARO QUE EL CUADRO CLÍNICO ES COMPATIBLE
CON UN CHOQUE HIPOVOLÉMICO, CIRCUNSTANCIA QUE NO PUEDE SER SECUNDARIA A UNA
CETOACIDOSIS DIABÉTICA.

EVENTO La ENFERMEDAD VASCULAR CEREBRAL(EVC) es una alteración neurológica, se caracteriza por su


VASCULAR aparición brusca, generalmente sin aviso, con síntomas 24 horas o más, causando secuelas y
CEREBRAL. muerte. Destaca como la causa más común de incapacidad en adultos y es la quinta de causa de
muerte en nuestro país. En Evento cerebral (EVC) puede ocurrir cuando una arteria se obstruye
produciendo interrupción o pérdida repentina del flujo sanguíneo cerebral, o bien, ser el
resultado de la ruptura de un vaso, dando lugar a un derrame. Los tres tipos principales de EVC
son Trombóticos, embólicos y hemorrágicos. Los signos de alarma de un EVC son: parestesias en
extremidades superiores e inferiores, parálisis de la cara, en uno o ambos lados del cuerpo y de
forma repentina, visión borrosa, incapacidad repentina para comunicarse, cefalea súbita. Existen
factores de riesgo para el EVC como lo son: hipertensión arterial, problemas cardíacos, diabetes,
hipercolesterolemia, tabaquismo. NO JUSTIFICA LA HIPOTENSIÓN Y TAQUICARDIA QUE
PRESENTA NUESTRO PACIENTE.

INTOXICACIÓN La INTOXICACIÓN ETÍLICA AGUDA (IEA) es el trastorno orgánico más común inducido por alcohol
ETÍLICA. y la intoxicación aguda más frecuente en nuestro medio. La Intoxicación Etílica Aguda (IEA) es un
síndrome clínico producido por el consumo de bebidas alcohólicas de forma brusca y en cantidad
superior a la tolerancia individual de la persona. Dependiendo de la cantidad ingerida de alcohol
y de la tolerancia, el curso puede oscilar desde leve desinhibición, hasta coma, depresión
respiratoria y muerte. Afecta al 1.1% de la población, sobre todo varones entre 19-28 años. La
dosis letal 50 es 5 gr. /l con ingesta aproximada de alcohol de 3 gr/ Kg. peso. La mortalidad por
coma etílico es del 5%. En la IEA el riesgo de sufrir TCE y Hematoma Subdural es más del doble y
está aumentado el de convulsiones, intentos autolíticos e intoxicaciones combinadas. El alcohol es
prioritariamente un depresor de la transmisión nerviosa en el S.N.C. y presenta tolerancia cruzada
con otros psicótropos depresores del S.N.C. Las alteraciones del S.N.C. son los primeros síntomas
que aparecen en el paciente. Desde el punto de vista psicopatológico lo más relevante son los
cambios conductuales que aparecen tras la ingestión de alcohol. En la IEA típica si relacionamos la
sintomatología con las concentraciones de etanol en sangre encontramos diferentes estadios en
una persona que no haya desarrollado tolerancia al alcohol. En conjunto los efectos de
“pseudoexcitación”, sensación de euforia, optimismo, aumento de sociabilidad, conducta
espontánea y menos autocontrolada y sobrevaloración personal se asocian a alteraciones
importantes del rendimiento psicomotor, disminución de habilidad psicomotora fina, aumento
del tiempo de reacción a estímulos y pérdida de la capacidad de concentración. Deterioro de la
acomodación y de la capacidad para seguir objetos, reducción del campo visual y alteración de la
visión periférica y en casos severos conlleva mayor depresión central con sintomatología más
relevante a nivel psicológico y psicomotor, torpeza expresiva y motora (disartria y ataxia),
pérdida de reflejos, sopor hasta llegar coma, depresión bulbar, e incluso, muerte. ES CLARO UN
ESTADO ETÍLICO EN ESTE MOMENTO, CIERTAMENTE SE PRODUCE DESHIDRATACIÓN PERO NO
AL GRADO DE UN ESTADO DE CHOQUE.
HEMORRAGIA La HEMORRAGIA DIGESTIVA ALTA (HDA) se define como el sangrado originado en el tracto
DIGESTIVA digestivo superior, proximalmente al ángulo de Treitz, y constituye la urgencia
ALTA. gastroenterológica más importante. Las dos causas más frecuentes son la HDA por úlcera péptica
y la secundaria a hipertensión portal. La úlcera péptica continúa siendo la causa más frecuente de
HDA, constituyendo entre el 37-50% de los casos, siendo dos veces más frecuente el sangrado por
úlcera duodenal que por úlcera gástrica. La magnitud del sangrado se ha correlacionado con el
tamaño (> 2 cm) y la localización de la úlcera, debido a la erosión de grandes vasos,
principalmente las localizadas en cara posterior del bulbo duodenal (arteria gastroduodenal) y en
la porción proximal de la curvatura menor gástrica (arteria gástrica izquierda).Generalmente se
manifiesta por sangrado externo, el cual puede ser masivo, en la forma de: hematemesis = vómito
de sangre. Puede ser de sangre fresca, o de sangre alterada con apariencia de posos de café; la
hematemesis indica pérdida de por lo menos un 25% del volumen sanguíneo. Melena =
evacuación rectal de sangre de color negro. El color negro se debe a su alteración y digestión en
el tracto gastrointestinal; hematoquezia = evacuación rectal de sangre roja viva. El tránsito
intestinal rápido puede resultar en hematoquezia y no melena, a pesar del origen alto del
sangrado. Es característica la inestabilidad hemodinámica: sensación de debilidad al asumir la
posición de pies, o aun pérdida del sentido ("desmayo") y caída de su propia altura, e hipotensión
ortostática, lo cual se acompaña de notoria palidez. Usualmente hay historia de melena. En un
sangrado por úlcera duodenal el sangrado puede dejar al paciente hemodinámicamente inestable
(hipotensión, taquicardia, hipotermia) como en el caso del paciente, y que a pesar de la
restitución con líquidos (cristaloides) nuevamente manifiesta hipotensión arterial, lo que nos
orienta a que el paciente seguramente continuaba con sangrado activo. ES CLARO QUE EL
PACIENTE SE ENCUENTRA EN ESTADO DE CHOQUE EL CUAL MUY PROBABLEMENTE ES DE TIPO
HIPOVOLÉMICO PUES RESPONDE A LA ADMINISTRACIÓN DE CRISTALOIDES. DADO QUE EL
PACIENTE SE ENCUENTRA EN ESTADO ETÍLICO ES PROBABLE QUE CURSE CON UN PROBLEMA
CRÓNICO HEPÁTICO QUE DERIVE EN VÁRICES ESOFÁGICAS Y SANGRADO DE TUBO DIGESTIVO
ALTO. OJO: DE LOS CUATRO DIAGNÓSTICOS PRESENTADOS ES LA ÚNICA CAUSA QUE JUSTIFICA
EL CUADRO CLÍNICO.

Bibliografía:
1. BRUNICARDI F, ANDERSEN D, BILLIAR T, Y COLS. SCHWARTZ PRINCIPIOS DE CIRUGÍA, 9A EDICIÓN. MC
GRAW HILL. 2011, PP 98-102.

368 - TREINTA MINUTOS DESPUÉS PRESENTA NUEVAMENTE CAÍDA DE LA TENSIÓN ARTERIAL. EN ESTE
MOMENTO SE DEBERÁ INDICAR:
LAVADO LAS PRIORIDADES EN TODO PACIENTE CON CHOQUE HIPOVOLÉMICO HEMORRÁGICO SON: 1.
GÁSTRICO , ASEGURAR LA VÍA AÉREA, 2. CONTROLAR EL ORIGEN DE LA HEMORRAGIA, 3. REANIMACIÓN
PANENDOSCOPIA DEL VOLUMEN INTRAVENOSO. En los pacientes con sangrado de tubo digestivo alto, el
Y TRANSFUSIÓN manejo inicial es con la colocación de sonda nasogástrica y el lavado gástrico. La aspiración de
SANGUÍNEA. sangre y coágulos de la cavidad gástrica permitirá realizar la exploración endoscópica de una
forma más segura y eficaz. Además, la colocación de la sonda nasogástrica ayuda a localizar el
origen del sangrado en aquellos pacientes con melenas o hematoquezia sin hematemésis. La
presencia de sangre roja o posos de café confirma el origen alto del sangrado y, en
determinados estudios, han mostrado un significado pronóstico, con una tasa de mortalidad
del 9% si se aspira posos de café frente al 18% si el aspirado es sangre roja, o bien la presencia
de una lesión de alto riesgo de sangrado. Idealmente, en los casos de sangrado activo o
inestabilidad hemodinámica la endoscopia se debe realizar inmediatamente tras la
estabilización hemodinámica del paciente. La transfusión en estos pacientes es vital dado a que
la pérdida sanguínea es importante y necesita la restitución con hemoderivados. Si el paciente
presenta una hemorragia masiva y choque hipovolémico, debemos valorar la indicación de
cirugía urgente con una eventual endoscopia preoperatoria debe de realizarse de forma
precoz, idealmente en las primeras 12-24 horas, ya que se asocia a una mayor eficacia
diagnóstica, nos permite detectar aquellos pacientes con lesiones de bajo riesgo y que pueden
ser dados de alta precozmente, y en aquellos pacientes con lesiones de alto riesgo nos permite
aplicar una terapia hemostática, lo que determina una reducción de la estancia hospitalaria,
menores requerimientos transfusionales, disminución de la incidencia de resangrado y cirugía
y un aumento de la supervivencia. EL PROPÓSITO DE LA REANIMACIÓN ES LOGRAR UNA
TENSIÓN ARTERIAL MEDIA DE POR LO MENOS 90MMHG, LO CUAL SE LOGRA COMO MEDIDA
PRIMARIA APLICANDO SOLUCIONES CRISTALOIDES Y HEMODERIVADOS. LA
PANENDOSCOPÍA POR SU PARTE PERMITIRÁ UBICAR LA ZONA DE SANGRADO Y OFRECER
UNA MEDIDA TERAPÉUTICA SI ES POSIBLE.

TERAPIA INICIAL El manejo de la CETOACIDOSIS DIABÉTICA requiere principalmente de la corrección de la


CON LÍQUIDOS E deshidratación, de la hiperglicemia y del desbalance electrolítico. Para iniciar el manejo es
INFUSIÓN CON indispensable el manejo de: a. Líquidos: La terapia inicial con líquidos va dirigida hacia la
INSULINA. expansión del volumen intravascular y restaurar la perfusión renal en los pacientes
severamente deshidratados. En ausencia de compromiso cardíaco se debe iniciar con solución
salina isotónica al 0,9% infundida a una velocidad de 15-20 cc/ K lo que en el adulto promedio
equivale a 1 a 1,5 litros durante la primera hora. Autores sugieren la administración de 500
ml/hora por las siguientes 2-4 Horas seguidas de 250 ml/hora por 8 horas más o hasta obtener
estabilidad hemodinámica para entonces pasar al uso de solución 0,45% (sin dextrosa). A
menos que exista hipocalcemia, el tratamiento de elección para la Cetoacidosis diabética
moderada y severa lo constituye la instalación de una infusión de insulina cristalina. Aun
cuando en adultos se recomienda la administración previa de un bolo de insulina cristalina
endovenoso de 0,15 unidades/Kg, en niños dicho bolo no es recomendable. Al momento de
corregir una eventual hipocalcemia se procede preparar una infusión continua de insulina
cristalina y se inicia en ambos grupos de pacientes a una velocidad de 0.1 unidades/Kg/h (5-7
unidades/h en adultos).

CARGA CON En los pacientes con INTOXICACIÓN ETÍLICA el manejo es básicamente conservador. La actitud
SOLUCIÓN terapéutica es implementar medidas de reanimación para estabilizar las constantes vitales y
GLUCOSADA AL mantener equilibrio hidroelectrolítico y ácido base por vía parenteral. Se puede colocar al
50%. paciente en posición en decúbito lateral del paciente. Se administran soluciones glucosadas,
20-40 ml al 50% dado a previenen que aparezca hipoglucemia. El Control de Temperatura
corporal también es importante por las hipotermias, dado a que inferiores a 31ºC pueden
producir coma. Y obviamente soluciones intravenosas a requerimientos altos para garantizar
buena hidratación y porque diluye el etanol hasta en un 20%. RECUERDA: LA REANIMACIÓN
EN CASO DE INTOXICACIÓN ETÍLICA SE REALIZA CON SOLUCIONES GLUCOSADAS Y NO CON
CRISTALOIDES.

INTUBACIÓN Y El apoyo de la vía aérea y la ventilación asistida se recomiendan en pacientes con


TOMOGRAFÍA. ENFERMEDAD VASCULAR CERBRAL isquémica aguda que presenten un estado de alerta
disminuido o disfunción bulbar que ocasione compromiso de la vía aérea . TODOS LOS
PACIENTES CON SOSPECHA DE ENFERMEDAD VASCULAR CEREBRAL ISQUÉMICA AGUDA (MÁS
DE UNA HORA DE EVOLUCIÓN) DEBEN SER SOMETIDOS A ESTUDIO DE IMAGEN CEREBRAL DE
INMEDIATO para saber el origen o causa del EVC y se recomienda la realización de tomografía
computada (TAC) para la mayoría de los pacientes en la fase aguda de la enfermedad vascular
cerebral isquémica.

Bibliografía:
1. BRUNICARDI F, ANDERSEN D, BILLIAR T, Y COLS. SCHWARTZ PRINCIPIOS DE CIRUGÍA, 9A EDICIÓN. MC
GRAW HILL. 2011, PP 98-102.

FIN DEL CASO CLÍNICO SERIADO


ANÁLISIS DEL CASO CLÍNICO

IDENTIFICACIÓN DEL REACTIVO


Area: MEDICINA INTERNA
Especialidad: NEUMOLOGÍA
Tema: ENFERMEDADES PULMONARES OBSTRUCTIVAS Y RESTRICTIVAS
Subtema: ENFERMEDAD PULMONAR OBSTRUCTIVA CRÓNICA Y
ENFERMEDAD INTERSTICIALES

CASO CLÍNICO CON UNA PREGUNTA

EN LA COMUNIDAD DONDE SE ENCUENTRA REALIZANDO SU SERVICIO SOCIAL LE ENCOMIENDAN REALIZAR


UNA CAMPAÑA DE PREVENCIÓN CONTRA EL NEUMOCOCO EN PACIENTES CON EPOC.

-.

CAMPAÑA DE PREVENCIÓN CONTRA EL


NEUMOCOCO EN PACIENTES CON EPOC.

-.

-.

-.

369 - LA INTERVENCIÓN MAS EFICIENTE QUE DEBERÁ IMPLEMENTAR PARA LA PREVENCIÓN PRIMARIA
DE LA NEUMONÍA NEUMOCÓCICA EN ESTE GRUPO DE PACIENTES, CONSISTIRÁ EN:

HACER EL REPASO. Vacuna de la influenza. Efectividad • Previene o atenúa la enfermedad en jóvenes y


AISLAMIENTO ancianos. • Su eficacia es del 70 a 90% en personas sanas menores de 65 años. • Para los más
HOSPITALARIO DE viejos con enfermedades crónicas su eficacia es menor. • Disminuye la presencia de neumonía
LOS CASOS en el 53%, hospitalización en el 50% y muerte en el 68%. Recomendaciones para su uso. •
DIAGNOSTICADOS. Personas mayores de 65 años. • Residentes de asilos, a los que tienen enfermedades crónicas:
pulmonares, cardiovasculares, diabetes, etc. • Pacientes que requieren atención médica
regular o han estado hospitalizados en años previos. • Enfermeras, médicos y personal que
atiende a pacientes con alto riesgo de adquirir la infección. • Personas con VIH. • Niños con
asma. Contraindicaciones y precauciones • Reacción anafiláctica al huevo y derivados. •
Enfermedad febril aguda. Dosis • La revacunación es necesaria cada año. • Debe aplicarse a
principios de septiembre y a mediados de octubre. RECUERDA QUE LA VACUNA CONSTITUYE
LA MEJOR INTERVENCIÓN EN LA PREVENCIÓN DE COMPLICACIONES EN PACIENTES CON
ENFERMEDADES CRÓNICAS PREVIAS TALES COMO EPOC.
ASPIRACIÓN Programa de vacunación en pacientes con EPOC. El objetivo de las vacunas en el manejo de
CONSTANTE DE pacientes con EPOC es el mejor control y prevención de las exacerbaciones infecciosas. Las
SECRECIONES EN vacunas que se recomiendan son las que contienen virus inactivados vivos o muertos porque
PACIENTES CON son más efectivas en pacientes con EPOC de edad avanzada. Las vacunas recomendadas son
SÍNTOMAS las que están diseñadas para prevenir la infección por influenza y la infección por
RESPIRATORIOS Streptococcus Pneumonae. La infección por el virus de la influenza incrementa la morbilidad
AGUDOS. de los pacientes con EPOC. Por ello se recomienda la vacunación anual ya que reduce las tasas
de hospitalización y de infecciones respiratorias bajas en individuos de edad avanzada. Debe
considerarse la profilaxis con amantadina o rimantadina en pacientes previamente no
vacunados durante los períodos considerados como de alto riesgo de contraer la infección
por influenza tipo A (por ejemplo durante el invierno y cuando hay familiares enfermos). Los
inhibidores de neuraminidasa, oseltamivir, zanamivir, pueden ser efectivos en individuos con
menos de 2 días de la duración de los síntomas aunque este último deber ser evaluado
debido a la posibilidad de desencadenar broncoespasmo en pacientes con enfermedad
pulmonar.

APLICACIÓN DE Inmunoterapia oral. Los agentes inmunoestimulantes representan una clase de medicamentos
INMUNOTERAPIA que contienen antígenos derivados de extractos de varias bacterias y que producen
ORAL EN estimulación del componente no específico del sistema inmune. Dentro de una gran gama de
PACIENTES DE inmunoestimulantes, el OM-85 BV, el agente con mayor número de estudios y al que se
RIESGO. tomará como prototipo, se encuentra constituido por 8 diferentes especies de bacterias
usualmente presentes en las vías aéreas inferiores: Haemophilus influenzae, Streptococcus
Pneumonae, Klebsiella Pneumonae y ozaenae, Staphylococcus Auereus, Streptococo
Pyogenes y viridans, y Neisseria catarrhalis. Su posible mecanismo de acción es activando
macrófagos pulmonares y aumentando la presentación del antígeno a linfocitos T. La
activación de macrófagos produce un incremento de respuesta inmune inespecífica. Al menos
en un ensayo clínico controlado, los inmunoestimulantes han demostrado una disminución en
el número de exacerbaciones, hospitalizaciones y días de estancia. Evidentemente se requiere
de mayor investigación que compruebe los hallazgos anteriores. LOS
INMUNOESTIMULADORES HAN DEMOSTRADO UNA DISMINUCIÓN EN EL NÚMERO DE
EXACERVACIONES, HOSPITALIZACIONES Y DÍAS DE ESTANCIA EN PACIENTES CON
INFECCIONES CRÓNICAS O PULMONARES RECURRENTES, PERO NO HAY EVIDENCIA
SUFICIENTE.

VACUNAR A LOS Vacuna neumocócica. Recomendaciones para su uso. • Aplicarla a todos los pacientes de 65
MAYORES DE 65 años o más. • Personas de 64 o menos que tengan una enfermedad crónica (cardiovascular,
AÑOS DE EDAD Y pulmonar, diabetes, alcoholismo, cirrosis, asplenia o si viven en ambientes que contribuyan al
DE 64 O MENOS desarrollo de una neumonía. • Residentes de asilos con enfermedades crónicas, de cualquier
CON ENFERMEDAD edad. • En los que tienen VIH, leucemia, linfoma, mieloma, cáncer. En los que reciben
CRÓNICA. tratamiento inmunosupresor prolongado, incluyendo esteroides. • Pacientes hospitalizados
de alto riesgo (antes del alta hospitalaria). Efectividad. • Previene enfermedades invasivas
(bacteremia, meningitis, etc.). • Eficacia en enfermedades crónicas del 56 al 81%. • En
inmunocompetentes mayores de 65 años su efectividad es del 75%. • La eficacia en
inmunosuprimidos es menos clara, pero se recomienda su uso. • La respuesta a la vacuna
disminuye entre los 5 a 10 años. Contraindicaciones y precauciones. o Se desconoce su
seguridad en el embarazo Dosis • En mayores de 65 años o más, no repetir la dosis. •
Revacunación simple en los mayores de 65 años, quienes fueron vacunados por primera vez
cuando tenían menos de 65 años. • En pacientes con asplenia, revacunar a los 5 años si tienen
más de 10 años de edad. LA VACUNA REDUCE LA PRESENTACIÓN DE NEUMONÍA EN LA
POBLACIÓN APLICADA, Y EN LOS PREVIAMENTE ENFERMOS PULMONARES REDUCE LAS
COMPLICACIONES Y EXACERVACIONES DE LA ENFERMEDAD. La vacuna antineumocóccica
reduce en un 43% el número de hospitalizaciones y un 29% el número de muertes por todas
las causas en pacientes con EPOC. La combinación de vacuna antiinfluenza con
antineumocóccica disminuye un 63% el riesgo de hospitalización por neumonía y un 81% el
riesgo de muerte.

Bibliografía:
1. GUÍA DE REFERENCIA RÁPIDA, DIAGNÓSTICO Y TRATAMIENTO DE LA ENFERMEDAD PULMONAR
OBSTRUCTIVA CRÓNICA. MÉXICO: SECRETARIA DE SALUD, 2009 2. GUÍA DE PRÁCTICA CLÍNICA,
DIAGNÓSTICO Y TRATAMIENTO DE LA ENFERMEDAD PULMONAR OBSTRUCTIVA CRÓNICA. MÉXICO:
SECRETARIA DE SALUD, 2009

http://www.cenetec.salud.gob.mx/descargas/gpc/CatalogoMaestro/037_GPC_EPOC/IMSS_037_08_EyR.pdf
ANÁLISIS DEL CASO CLÍNICO

IDENTIFICACIÓN DEL REACTIVO


Area: PEDIATRÍA
Especialidad: URGENCIAS PEDIÁTRICAS
Tema: URGENCIAS ORTOPÉDICAS
Subtema: DISPLASIA DEL DESARROLLO (CONGÉNITA) DE LA CADERA

CASO CLÍNICO SERIADO

RECIÉN NACIDO MASCULINO QUE TIENE UNA HERMANA DE 3 AÑOS DE EDAD CON ANTECEDENTE DE
DISPLASIA DEL DESARROLLO DE LA CADERA. SE OBTUVO POR VÍA ABDOMINAL CON PRESENTACIÓN
CEFÁLICA. PESO DE 3,800GR Y TALLA DE 50CM. LA MADRE SOLICITA INFORMACIÓN CON RESPECTO A LA
POSIBILIDAD DE QUE SU HIJO PRESENTE EL MISMO DEFECTO CONGÉNITO QUE SU HERMANA.

RN masculino.

Hermana con antecedente de displasia del


desarrollo de la cadera.

370 - EL NIVEL DE RIESGO DEL PACIENTE PARA PRESENTAR DISPLASIA DEL DESARROLLO DE LA
CADERA ES:

SIN RIESGO. La DISPLASIA EN EL DESARROLLO DE LA CADERA TÍPICA (DDC), anteriormente conocida como
luxación congénita de cadera, integra anormalidades anatómicas que afectan la articulación
coxofemoral del niño (a) incluyendo el borde normal del acetábulo (displasia) y mala posición de la
cabeza del fémur, causando desde la subluxación hasta la luxación, afectando el desarrollo de la
cadera durante los períodos embriológicos, fetal o infantil. Todos los recién nacidos deben
estudiarse sistemáticamente mediante la exploración física de la cadera en la primera semana de
vida. La evaluación de la cadera del recién nacido se evalúa mejor cuando el niño se encuentra
tranquilo e incluirá: - Evaluación de la proporción y pliegues de la piel de las extremidades
inferiores. - Identificación de deformidades. - Maniobras de Barlow y Ortolani.

RIESGO Se recomienda investigar de forma intencionada aquellos factores que se relacionan con la
BAJO. presencia de displasia del desarrollo de la cadera de acuerdo al nivel de riesgo: - RIESGO BAJO:
niños varones sin factor de riesgo o varones con antecedentes familiares positivos.
RIESGO - RIESGO INTERMEDIO: niñas sin factores de riesgo, niños varones con presentación pélvica.
INTERMEDIO.

RIESGO - RIESGO ALTO: niñas con antecedentes familiares positivos, niñas con presentación pélvica.
ALTO.

Bibliografía:
1. GUÍA DE PRÁCTICA CLÍNICA, DIAGNÓSTICO Y TRATAMIENTO OPORTUNO DE LA DISPLASIA DEL
DESARROLLO DE CADERA. MÉXICO: SECRETARIA DE SALUD; 2012.

http://www.cenetec.salud.gob.mx/descargas/gpc/CatalogoMaestro/141_GPC_HEMOFILIA_PEDIATRICA/Imss_RR.pdf

371 - ES LLEVADO NUEVAMENTE A LOS 6 MESES DE EDAD A REVISIÓN, DEBERÁ SOSPECHARSE


DISPLASIA DE CADERA SI PRESENTA:

SIGNOS DE Los datos clínicos de utilidad para el diagnóstico de DISPLASIA EN EL DESARROLLO DE LA


BARLOW Y CADERA (DDC) en niños de “2 a 3” meses son la prueba de Ortolani, Barlow, asimetría de los
ORTOLANI pliegues y limitación para la abducción de la cadera afectada. LAS MANIOBRAS DE BARLOW Y
POSITIVOS ORTOLANI NO SON CONFIABLES A PARTIR DE LOS 3 MESES DE EDAD pudiendo dar falsos
negativos.

ALARGAMIENTO Los datos clínicos de utilidad para el diagnóstico de DDC en niños MAYORES A TRES MESES de
APARENTE DE LA edad son: - Limitación de la abducción de la cadera. - Asimetría de pliegues de la cara interna
EXTREMIDAD de los muslos y glúteos. - Signo de Galleazzi. - ACORTAMIENTO DE LA EXTREMIDAD
AFECTADA. AFECTADA. - Signos de pistón.

SIGNO DE EN NIÑOS MAYORES DE SEIS MESES DE EDAD SE RECOMIENDA EL SIGNO DE PISTÓN Y


GALLEAZZI GALLEAZZI. El signo de Galleazzi se explora al doblar las rodillas del niño y observarlas desde
POSITIVO los pies, la que corresponde a la cadera luxada es más baja y/o más corto el muslo ipsilateral.

SIGNO DE Durante la DEAMBULACIÓN, se pueden identificar los signos de Trendelemburg, la marcha


TRENDELENBURG. tipo Duchenne y el signo de Lloyd Roberts, característicos de DDC. ES UN SIGNO TARDÍO QUE
SE OBSERVA CUANDO YA HAY MARCHA.

Bibliografía:
1. GUÍA DE PRÁCTICA CLÍNICA, DIAGNÓSTICO Y TRATAMIENTO OPORTUNO DE LA DISPLASIA DEL
DESARROLLO DE CADERA. MÉXICO: SECRETARIA DE SALUD; 2012.

http://www.cenetec.salud.gob.mx/descargas/gpc/CatalogoMaestro/091_GPC_Displasiacaderaped/SSA_091_08_EyR.pdf

FIN DEL CASO CLÍNICO SERIADO


ANÁLISIS DEL CASO CLÍNICO

IDENTIFICACIÓN DEL REACTIVO


Area: GINECOLOGÍA Y OBSTETRICIA
Especialidad: OBSTETRICIA
Tema: HEMORRAGIA POSTPARTO Y PUERPERIO ANORMAL
Subtema: INFECCIONES POSTPARTO Y PUERPERALES

CASO CLÍNICO SERIADO

REGRESA PACIENTE DE 32 AÑOS DE EDAD A VALORACIÓN AL SERVICIO DE URGENCIAS. ACTUALMENTE


CURSANDO EL QUINTO DÍA DE PUERPERIO POSTCÉSAREA, PRESENTAR FIEBRE DE 38.5 GRADOS. EN LA
EXPLORACIÓN SE ENCUENTRA DISTENSIÓN ABDOMINAL Y ÚTERO DOLOROSO A LA MOVILIZACIÓN.

-.

quinto día puerperio postcesárea.

ebre 38.5·C.

distensión abdominal, y útero doloroso a la


movilización.

-.

372 - EL DIAGNÓSTICO CLÍNICO MAS PROBABLE ES:

SALPINGOOFORITIS. La SEPSIS PUERPERAL es la infección del tracto genitalque ocurre entre el período
transcurrido entre el nacimiento y los 42 días postparto y la presencia de un o más de los
siguientes daros clínicos: dolor pélvico, flujo vaginal anormal, flujo fétido trasnsvaginal,
retardo en la involución uterina y/o fiebre. LO PRIMERO QUE DEBES DEFINIR ES QUE LA
PACIENTE TIENE DATOS SUGESTIVOS DE SEPSIS PUERPERAL, PARA DESPUÉS DETERMINAR
LA CAUSA MÁS PROBABLE. La SALPINGITIS y SALPINGOOFORITIS puerperal se produce
cuando la infección puerperal se propaga a las trompas de Falopio y/o ovarios. En algunas
condiciones poco frecuentes se desarrollan abscesos tuboováricos como complicación de la
infección puerperal. Los síntomas y signos son de aparición tardía, entre el octavo y el
décimo día del puerperio y entre los más frecuentes tenemos: fiebre elevada, dolor en
hemiabdomen inferior, irritación peritoneal, subinvolución uterina, presencia o no de masas
anexiales. Ésta condición se da casi siempre de forma unilateral con síntomas de 1 a 2
semanas después del parto. EL CUADRO CLÍNICO ES MÁS TARDÍO CON MANIFESTACIONES
UNILATERALES QUE NO CORRESPONDEN NI EN TIEMPO NI EN SIGNOS CON EL CUADRO
CLÍNICO.
ABSCESO PÉLVICO. En algunas mujeres que presentan matritis (inflamación del útero) después de la cesárea, la
celulitis parametrial es pronunciada y forma un área de induración que recibe el nombre de
flemon, dentro de las hojas del ligamento ancho. En raras ocasines un flemon puede ser
supurativo y formar un tumor del ligamento ancho que sobresale por arriba del ligamento
inguinal, consituyendo un ABCESO PÉLVICO. Se puede sospechar de ésta infección cuando la
fiebre persiste durante más de 72hrs pese a medicamentos antimicrobianos. ES UNA
COMPLICACIÓN GENERALMENTE PRECEDIDA DE ENDOMETRITIS, ES POCO FRECUENTE.

ENDOMETRITIS. La ENDOMETRITIS es la inflamación del endometrio. Se manifiesta entre el segundo y quinto


días del puerperio y constituye la causa más frecuente de infección puerperal, acompañada
de fiebre en este periodo. Los síntomas y signos más frecuentes son: fiebre elevada de 38-
40·C, calofríos, útero subinvolucionado, blando y doloroso, loquios abundantes, oscuros
(achocolatados o seropurulentos).La infección por anaerobios es causa de intensa fetidez.La
endometritis postparto es 10 veces más frecuente después de una cesárea. LA
ENDOMETRITIS PUERPERAL CONSTITUYE LA CAUSA MÁS FRECUENTE DE INFECCIÓN
PUERPERAL.

PELVIPERITONITIS. La PERITONITIS y PELVIPERITONITIS es un cuadro grave que se instala cuando la infección


se propaga a la pared pélvica y/o peritoneo, se presenta a finales de la primera semana del
puerperio. Los síntomas y signos más frecuentes son: fiebre elevada, generalmente mayor
de 40·C, compromiso del estado general, náuseas y vómitos en cantidad variable, dolor en
hipogastrio con localización preferente hacia una u otra región anexial. A la exploración se
pueden detectar la presencia de colecciones líquidas, generalmente purulentas y de masas
anexiales fijas o móviles en hemiabdomen inferior y saco de Douglas. Es infrecuente que
ocurra peritonitis después de la cesárea, pero casi de manera invariable la anteceden
metritis (inflamación del útero), necrosis de la incision uterina y dehiscencia. Otras causas
pueden ser por lesión de órganos abdominales durante la cesárea pero es menos
infrecuente. ES UNA COMPLICACIÓN GRAVE CON ATAQUE AL ESTADO GENERAL, LA
PACIENTE PRESENTA DATOS BIEN LOCALIZADOS QUE DESCARTAN ÉSTA POSIBILIDAD.

Bibliografía:
1. GUÍA DE PRÁCTICA CLÍNICA, DIAGNÓSTICO Y TRATAMIENTO DE LA SEPSIS PUERPERAL. MÉXICO:
SECRETARIA DE SALUD; 2009. 2. CUNNINGHAM G, LEVENO K, BLOMM S, HAUTH J, RPUSE D, SONG C.
WILLIAMS OBSTETRICIA, 23A EDICIÓN. MC GRAW HILL. USA. 2011 EN ESPAÑOL, PP 661-663.

http://www.cenetec.salud.gob.mx /descargas/gpc/CatalogoMaestro/272_GPC_Diagnostico_tratamiento_de_SEPSIS_PUERPERAL/RER_SEPSIS_PUERPERAL.pdf

373 - EL TRATAMIENTO INDICADO EN ESTA PACIENTE ES:

CLINDAMICINA El tratamiento de las INFECCIONES PUERPERALES empírico al inicio y debe revalorarse a las
Y 72hrs. El tratamiento inicial de una infección puerperal postcesñarea debe enfocarse a la flora
GENTAMICINA. mixta teniendo en cuenta una cobertura contra anaeróbios. La CLINDAMICINA es "la base" del
tratamiento de las infecciones puerperales. La asociación Clindamicina-gentamicina tienen un
índice de resolución de 95%. LA CLINDAMICINA CONSTITUYE EL MEDICAMENTO IDEAL PARA
LAS INFECCIONES PUERPERALES Y DEBE ASOCIARSE CON GENTAMICINA PARA AMPLIAR EL
ESPECTRO EN INFECCIÓN POSTCESÁREA.

CEFOTAXIMA E El IMIPENEM es un carbapenem con cobertura de amplio espectro cntra la mayor parte de los
IMIPENEM. patógenos que causan metritis. Se administra combinado con cilastatina, que inhibe el
metabolismo renal del imipenem. si bien esta combinación es efectiva en la mayor arde de los
casos de metritis, conviene reservarla para las infecciones más graves desde el punto de vista
médico y económico. EL IMIPENEM ESTÁ BIEN INDICADO EN INFECCIONES PUERPERALES, DEBE
COMBINARSE CON CILASTATINA PERO SIEMPRE TÓMALO EN CUENTA SÓLO EN CASOS
GRAVES.
AMIKACINA Y Las CEFALOSPORINAS están indicadas como monoterapia cuando ésta se inicia empiricamente
CEFTAZIDIMA. hasta tener los resultados de antibiograma. Al igual que otros antibióticos de amplio espectro se
utilizarán acorde al resultado del antibiograma o cuando el proceso no haya remitido con los
esquemas básicos. LA ASOCIACIÓN DE UNA CEFALOSPORINA DE TERCERA GENERACIÓN A UN
AMINOGLUCÓSIDO ES CORRECTA PARA EL MANEJO DE INFECCIÓN PUERPERAL SIN EMBARGO,
DEBES TOMAR EN CUENTA LA PRESENCIA DE GÉRMENES ANAEROBIOS ASOCIADOS AL
PUERPERIO POSTCESÁREA POR LO QUE NO ESTÁ INDICADO ÉSTE ESQUEMA.

METRONIDAZOL El metronidazol tienen una buena actividad contra anaerobios, combinado con ampicilina y un
Y aminoglucósido ofrece cobertura contra la mayor parte de los microorganismos encontrados en
ERITROMICINA. las infecciones pélvicas graves. El uso de eritromicina está reservado a los paciente con alergia a
penicilina. EL METRONIDAZOL ES ÚTIL CONTRA LA MAYOR PARTE DE ANAEROBIOS, RECUERDA
EN CASO DE INDICARLO DEBE SER UN TRIPLE ESQUEMA QUE INCLUYA AMPICILINA Y UN
AMINOGLUCÓSIDO PARA LOGRAR BUENA RESPUESTA TERAPÉUTICA.

Bibliografía:
1. CUNNINGHAM G, LEVENO K, BLOMM S, HAUTH J, RPUSE D, SONG C. WILLIAMS OBSTETRICIA, 23A
EDICIÓN. MC GRAW HILL. USA. 2011 EN ESPAÑOL, PP 663-664.

FIN DEL CASO CLÍNICO SERIADO


ANÁLISIS DEL CASO CLÍNICO

IDENTIFICACIÓN DEL REACTIVO


Area: MEDICINA INTERNA
Especialidad: PSIQUIATRÍA
Tema: TRANSTORNO DE LA ALIMENTACIÓN Y ADICCIONES
Subtema: ANOREXIA Y BULIMIA

CASO CLÍNICO CON UNA PREGUNTA

MUJER DE 23 AÑOS DE EDAD QUE ACUDE A CONTROL Y SEGUIMIENTO PORQUE DESDE HACE 2 MESES
CUENTA CON EL DIAGNÓSTICO DE BULIMIA.

mujer de 23 años.

diagnóstico de 2 meses de evolución de


bulimia.

-.

-.

-.

374 - ADEMÁS DE LA TERAPIA NUTRICIONAL QUE REQUERIRÁ LA PACIENTE LOS SIGUIENTES FÁRMACOS
HAN DEMOSTRADO SER ÚTILES EN EL TRATAMIENTO DE ESTE TIPO DE PACIENTES:

ANTIDEPRESIVOS Los ANTIDEPRESIVOS TRICÍCLICOS, NO están sugeridos para trastornos de conducta


TRICÍCLICOS. alimentaria, su acción incrementar los neurotransmisores de serotonina y noradrenalina. Su uso
recomendado en: enuresis, ansiedad, insomnio, dolor neuropático, depresión resistente.

ESTIMULANTES Los estimulantes del apetito, están INDICADOS EN ANOREXIA NERVIOSA, al ser un estimulante
DE APETITO. se establece en casos restrictivos. Presenta las propiedades de estimular el apetito y producir
un aumento de peso corporal, gracias a que el fármaco disminuye la utilización de la glucosa
por el organismo, lo que tiene como consecuencia una disminución de la actividad del centro
de la saciedad del hipotálamo; en estas condiciones, desaparece o disminuye el efecto inhibidor
que el centro medial de la saciedad ejerce sobre el centro lateral del apetito, por lo cual su
actividad aumenta, presentando por ende, un incremento del apetito.

ANSIOLÍTICOS. Las indicaciones de estos medicamentos son dirigidos hacia síntomas de ansiedad, trastornos
de ansiedad, por lo que pudiesen usarse en conjunto a los inhibidores de la recaptura de
serotonina en comorbilidades del espectro ansioso.
INHIBIDORES La FLUOXETINA es el medicamento de elección, se ha observado en relación a disminución de
SELECTIVOS DE los atracones, la dosis recomendable es de 20mg/dìa la cual puede incrementarse en casos de
LA RECAPTURA bulimia hasta 60mg/dìa. AUNQUE PRÁCTICAMENTE TODOS LOS ANTIDEPRESIVOS SON ÚTILES
DE SEROTONINA. EN ESTE CASO, SÓLO LA FLUOXETINA, UN INHIBIDOR DE LA RECAPTURA DE SEROTONINA,
ESTÁ AUTORIZADA POR LA FDA PARA EL MANEJO DE ESTE TRASTORNO.

Bibliografía:
1. LONGO DL, FAUCI AS, KASPER DL, HAUSERSL, JAMESON JL, LOSCALZOJ. HARRISON. PRINCIPIOS DE
MEDICINA INTERNA, 18A EDICIÓN. MC GRAW HILL. NEW YORK, USA. 2012, PP 641.
ANÁLISIS DEL CASO CLÍNICO

IDENTIFICACIÓN DEL REACTIVO


Area: PEDIATRÍA
Especialidad: INFECTOLOGIA PEDIÁTRICA
Tema: ENFERMEDADES EXANTEMÁTICAS
Subtema: MONONUCLEOSIS INFECCIOSA

CASO CLÍNICO CON UNA PREGUNTA

ADOLESCENTE DE 11 AÑOS DE EDAD, ES LLEVADO A CONSULTA MÉDICA CON CUADRO APARENTE DE


FARINGITIS AGUDA DE 3 DÍAS DE EVOLUCIÓN, LA CUAL YA FUE TRATADA DE MANERA SINTOMÁTICA SIN
MEJORÍA CLÍNICA. A LA EXPLORACIÓN FÍSICA SE OBSERVA PACIENTE ASTÉNICO, FEBRIL, TINTE ICTÉRICO EN
PIEL Y CONJUNTIVAS ++, OROFARINGE ERITEMATOSA, ADENOMEGALIAS CERVICALES Y AXILARES
BILATERALES. LA BIOMETRÍA HEMÁTICA REVELA LINFOCITOSIS CON 20% DE LINFOCITOS ATÍPICOS.

Adolescente de 11 años de edad.

Faringitis aguda de 3 días de evolución sin


mejoría clínica.

Asténico, febril, orofaringe eritematosa,


adenomegalias cervicales y axilares
bilateral, ictericia ++.

Linfocitosis con 20% de atípicos. CLÁSICO


DE MONONUCLEOSIS INFECCIOSA.

375 - EL SIGUIENTE MEDICAMENTO ESTRÍA CONTRAINDICADO EN ESTE CASO:


IBUPROFENO. LO PRIMERO QUE DEBERÁS HACER EN CASOS COMO ESTE, ES TENER EN CUENTA EL
DIAGNÓSTICO CLÍNICO. La MONONUCLEOSIS INFECCIOSA es un síndrome clínico generalmente
ocasionado por el Virus de Epstein Barr (EBV); hasta el 7% de los casos se ha asociado a
Citomegalovirus. El EBV es un gama-herpes virus que infecta al 90% de la población mundial, su
medio de transmisión es a través de la saliva. La Mononucleosis infecciosa se caracteriza por la
triada clásica en un 98% de los casos: - Fiebre. - Linfadenopatía. - Datos faríngeos. Además de la
triada clásica hay otros muchos datos que se asocian a esta enfermedad entre los que destacan:
Petequias en el paladar (50%), adenopatía inguinal, adenopatía axilar y adenopatía
retroauricular. Las manifestaciones clínicas aunque no tan frecuentes suelen llamar la atención
en los casos de mononucleosis múltiple, lo más frecuente es observar un exantema eritematosso
maculopapular que puede ser también de tipo urticariforme, buloso, morbiliforme, vesicular,
petequial y purpúrico. Cuando un exantema se asocia a ictericia, artralgias, conjuntivitis y
síntomas faríngeos deberás descartar siempre la presencia de Mononucleosis Múltiple.

PARACETAMOL. El tratamiento de la Mononucleosis Infecciosa es sintomático e incluye: - Mantener hidratación


adecuada. - Antiinflamatorios no esteroideos: ibuprofeno o paracetamol; ambos eficaces, bien
tolerados y seguros en niños y adultos. Precaución con el uso de ácido acetilsalicílico en niños
por el riesgo de Síndrome de Reye. EL PARACETAMOL E IBUPROFENO SON LOS
ANTIINFLAMATORIOS DE ELECCIÓN PARA EL MANEJO SINTOMÁTICO DE LA MONONUCLEOSIS
MÚLTIPLE.

AMOXICILINA. Cuando un exantema se asocia a ictericia, artralgias, conjuntivitis y síntomas faríngeos deberás
descartar siempre la presencia de Mononucleosis Múltiple. Las recomendaciones generales para
el manejo de los pacientes con MONONUCLEOSIS MÚLTIPLE son: - Informar al paciente y/o
cuidador sobre los signos de alarma: dolor abdominal y dificultad respiratoria. - Evitar deportes
de contacto durante al menos 1 mes después del diagnóstico así como actividades de riesgo. -
Reposo relativo. - NO PRESCRIBIR ANTIBIÓTICOS, ESPECIALMENTE AMPICILINA Y
AMOXICILINA, y en general todos los BETALACTÁMICOS dado que quien recibe este tipo de
antibióticos se complica con exantema.

ACICLOVIR. El ACICLOVIR es una droga que inhibe la síntesis de DNA viral en forma competitiva, algunos de
los virus de grupo herpes son sensibles. En el caso de Virus de Epstein Barr la concentración
mínima inhibitoria será de 10 a 80 veces mayor a la requerida para inhibir la replicación del
Herpes virus simple. El Aciclovir NO HA DEMOSTRADO EFICACIA PARA EL TRATAMIENTO
CONTRA EL VIRUS DE EPSTEIN BARR. NO ESTÁ INDICADO SU USO PERO TAMPOCO
CONTRAINDICADO COMO TAL POR SU ASOCIACIÓN CON ALGUNA COMPLICACIÓN.

Bibliografía:
1. GUÍA DE PRÁCTICA CLÍNICA, DIAGNÓSTICO DIFERENCIAL DE LOS EXANTEMAS INFECCIOSOS. MÉXICO:
SECRETARIA DE SALUD; 2011. 2. GUÍA DE PRÁCTICA CLÍNICA, DIAGNÓSTICO Y TRATAMIENTO DE LA
MONONUCLEOSIS INFECCIOSA. MÉXICO: SECRETARIA DE SALUD; 2011.

http://www.cenetec.salud.gob.mx/descargas/gpc/CatalogoMaestro/588_GPC_Exantemasinfecciososenlainfancia/588GER.pdf
ANÁLISIS DEL CASO CLÍNICO

IDENTIFICACIÓN DEL REACTIVO


Area: GINECOLOGÍA Y OBSTETRICIA
Especialidad: GINECOLOGÍA
Tema: CLIMATERIO Y MENOPAUSIA
Subtema: CLIMATERIO Y MENOPAUSIA

CASO CLÍNICO CON UNA PREGUNTA

MUJER DE 50 AÑOS DE EDAD, CON ANTECEDENTE DE TROMBOSIS VENOSA. ACUDE POR PRESENTAR
SÍNTOMAS SEVEROS VASOMOTORES Y ATROFIA UROGENITAL.

mujer de 50 años probablemente en la


perimenopáusia.

el dato clave es el antecedente de


trombosis venosa, que contraindica el uso
te terapia hormonal, principalmente con
estrógenos.

sintomatología de perimenopausia.

-.

-.

376 - CONSIDERANDO EL RIESGO DE TROMBOSIS ESTA INDICADA LA TERAPIA HORMONAL:

CON Entre las COMPLICACIONES DE LA TERAPIA HORMONAR se encuentran: • Cáncer hormono-


PROGESTÁGENOS. dependiente (endometrial y de mama). • Sangrado uterino anormal no diagnosticado. •
Insuficiencia venosa complicada. • Insuficiencia hepática. • Litiasis vesicular. • Trombofilias. •
Antecedentes de eventos tromboembólicos. • Dislipidemias. LA PACIENTE PRESENTA
ANTECEDENTE DE EVENTOS TROMBOEMBOLÍTICOS, LO QUE CONTRAINDICA LA TERAPIA
HORMONAL “DE CUALQUIER TIPO”. Tanto los estrógenos como las progesteronas vía oral
están contraindicadas cuando existen antecedentes trombóticos en las pacientes.

COMBINADA En las MUJERES CON ÚTERO, los PROGESTÁGENOS se combinan con estrógenos Para reducir
CONTÍNUA. el riesgo de cáncer endometrial, De hecho, se puede prescribir diariamente progestágenos con
los estrógenos, lo que se denomina terapia continua. Sin embargo este método genera
amenorrea. Se indica en pacientes con útero. INCLUIR ESTRÓGENOS EN ESTE CASO ESTÁ
CONTRAINDICADO.
CON “Los estrógenos por vía oral están contraindicados en pacientes con antecedentes de
ESTRÓGENOS VÍA tromboembolismo porque al ser metabolizados en hígado estimulan la producción hepática
TRANSDÉRMICA. de factores de la coagulación”. Sin embargo los parches transdérmicos, omiten el efecto de
primer paso en el hígado y ofrecen la conveniencia de una administración menos frecuentes.
EXISTE EVIDENCIA DE QUE LA TERAPIA HORMONAL ORAL MUESTRA UN MAYOR RIESGO DE
TROMBOMEBOLISMO CASI 4 VECES MÁS E INCREMENTO DE LOS LÍPIDOS SANGUÍNEOS EN
COMPARACIÓN CON LA VÍA TRANSDÉRMICA. Si bien, lo mejor para la paciente sería utilizar
terapias no hormonales, en caso de requerir una terapia hormonar la vía transdérmica sería la
indicada porque “es menos trombogénica” que aquellas por vía oral. DATOS IMPORTANTES: -
El estradiol administrado por vía oral se absorbe a nivel del tracto gastrointestinal y es
rápidamente metabolizado por el hígado para formar estrona y sus conjugados. - El estradiol
transdérmico es absorbido a través de la piel evitando el paso a nivel hepático y por ello la
dosis por esta vía es menor que la oral. - El estradiol por vía transdérmica tiene menor efecto
sobre colesterol plasmático y no aumenta los niveles de triglicéridos Por todas estas razones,
debes elegir los estrógenos vía trasndérmica como correctos en este caso. IMPORTANTE:
Debes considerar que aunque existen también presentación transdérmica para progesterona,
ésta no está indicada como tratamiento para reducir la atrofia urogenital en mujeres
postmenopáusicas.

CÍCLICO TERAPIA CÍCLICA COMBINADA. Se administran estrógenos durante 25 días de cada mes y
COMBINADO. algún progestágeno durante los últimos 10 días. Los medicamentos se retiran durante cinco
días, después de los cuáles se produce descamación y hemorragia endometrial. Este
tratamiento cíclico es el que más se utiliza durante la transición menopáusica, mientras que el
tratamiento continuo se prefiere en las mujeres después de la menopausia. Al incluir
estrógenos estaría contraindicado.

Bibliografía:
1. GUÍA DE REFERENCIA RÁPIDA, DIAGNÓSTICO Y TRATAMIENTO DE LA PERIMENOPAUSIA Y
POSTMENOPAUSIA. MÉXICO: SECRETARIA DE SALUD; 2013. 2. GUÍA DE PRÁCTICA CLÍNICA, DIAGNÓSTICO Y
TRATAMIENTO DE LA PERIMENOPAUSIA Y POSTMENOPAUSIA. MÉXICO: SECRETARIA DE SALUD; 2013. 3.
SCHONGUE J, SCHAFER J, HALVORSON L, HOFFMAN B, BRADSHAW K, CUNNINGHAM G. WILLIAMS
GINECOLOGÍA, DE LA 1A EDICIÓN EN INGLÉS. MC GRAW HILL. USA. 2009, PP 1334-1335.

http://www.cenetec.salud.gob.mx/descargas/gpc/CatalogoMaestro/019_GPC_ClimatyMenop/SS_019_08_EyR.pdf
ANÁLISIS DEL CASO CLÍNICO

IDENTIFICACIÓN DEL REACTIVO


Area: PEDIATRÍA
Especialidad: INFECTOLOGIA PEDIÁTRICA
Tema: ENFERMEDADES EXANTEMÁTICAS
Subtema: MONONUCLEOSIS INFECCIOSA

CASO CLÍNICO SERIADO

MASCULINO DE 12 AÑOS DE EDAD, CON PADECIMIENTO DE 3 SEMANAS DE EVOLUCIÓN CON FIEBRE DE


38.5°C, ASTENIA, ADINAMIA, CEFALEA, DIAGNOSTICÁNDOLE FARINGOAMIGDALITIS BACTERIANA RECIBIENDO
TRATAMIENTO CON AMOXICILINA; SIN PRESENTAR MEJORÍA, AGREGÁNDOSE EXANTEMA, ADENOMEGALIAS
CERVICALES Y DISTENSIÓN ABDOMINAL POR HEPATO-ESPLENOMEGALIA.

Masculino de 12 años de edad

Padecimiento de 3 semanas de evolución

Triada: Faringoamigdalitis, ebre y


adenopatías, exantema posterior a la
administración de amoxicilina.
Hepatoesplenomegalia.

377 - EL DIAGNÓSTICO MÁS PROBABLE EN NUESTRA PACIENTE ES:


FIEBRE Antecedentes a investigar en sospecha de fiebre reumática. 1. Faringoamigdalitis por EBHGA.
REUMÁTICA 2. Condiciones de la vivienda adversas. 3. Familiares con fiebre reumática. Síntomas y signos
más comunes de la fiebre reumática. Criterios de Jones: El diagnóstico de fiebre reumática, se
establece con la presencia, en un primer episodio, de dos criterios mayores o de un criterio
mayor y dos menores, y en un episodio recurrente, además, con tres criterios menores, más la
evidencia de un cuadro precedente de infección por estreptococo beta hemolítico del grupo A.
Criterios mayores. Poliartritis. Usualmente es asimétrica y migratoria, pero puede ser aditiva,
que afecta grandes articulaciones. Corea. Consiste en movimientos involuntarios,
incoordinados, especialmente en manos, pies, lengua y cara, que desaparecen con el sueño y
pueden afectar un solo lado del cuerpo (Hemicorea). Afecta con mayor frecuencia a las
mujeres en la adolescencia. Se observa en signos clínicos como la presión de la lechera, el
cuchareo, el signo del pronador y la incapacidad para mantener la protrusión de la lengua.
Carditis. Afecta principalmente las válvulas mitral y aórtica. Al principio produce regurgitación
valvular. Se presenta como un soplo holosistólico apical con o sin soplo de flujo
mesodiastólico, o un soplo diastólico temprano en la base del corazón (Regurgitación aórtica).
Presenta un aspecto característico en la ecocardiografía, que confirma el diagnóstico. Nódulos
subcutáneos. Son raros (Menos del 2 %) y altamente específicos de fiebre reumática. Miden de
0.5 a 2.0 cm. de diámetro, son nódulos redondos, firmes, libremente móviles e indoloros que
se agrupan sobre los codos, muñecas, rodillas, tobillos, el tendón de Aquiles, el occipucio y las
apófisis vertebrales posteriores. Duran una a dos semanas después del inicio del cuadro de
fiebre reumática. Eritema marginado. También tiene una frecuencia menor del 2 % y resulta
altamente específico de fiebre reumática. Consiste en máculas o pápulas rosa brillante que
blanquean a la presión, son indoloras y se extienden hacia los lados con un patrón circular o
serpiginoso. Criterios menores. Fiebre: Se identifica con la medición de la temperatura oral,
timpánica o axilar, mayor de 38ºC. Intervalo P-R prolongado, en el electrocardiograma, de
acuerdo con la edad: • 3 a 12 años, más de 0.16 segundos. • 12-16 años, más de 0.18 segundos.
• Mayor de 17 años, más de 0.20 segundos. • Elevación de reactantes de fase aguda el nivel de
PCR debe ser de 30 mcg/l o el de VSG 30 mm/hora. El paciente no cumple con ningún criterio.

LEPTOSPIROSIS MANIFESTACIONES CLÍNICAS: Es una enfermedad febril aguda con manifestaciones diversas
que son consecuencia de vasculitis generalizada. La intensidad del trastorno varía de un
cuadro sistémico que cede por sí sólo (En alrededor de 90 % de los pacientes) hasta un cuadro
letal que incluye ictericia, insuficiencia renal y neumonitis hemorrágica. Sea cual sea su
gravedad, el comienzo se caracteriza por síntomas inespecíficos que incluyen fiebre,
escalofríos, cefalea, náuseas, vómitos y una erupción transitoria. Las manifestaciones clínicas
más características son la congestión de conjuntivas sin secreción purulenta (30 a 40 % de los
casos) y mialgias de la pantorrilla y regiones lumbares (80 % de los casos). Por todo lo
comentado, la fase inicial “Septicémica” por lo común dura tres a siete días, y después de ella
surge una segunda fase “Mediada por mecanismos inmunitarios”. En algunos pacientes, las
dos fases están separadas por un periodo breve a febril (Uno a tres días). Los signos que casi
siempre surgen en la fase mediada por inmunidad incluyen fiebre, meningitis aséptica,
congestión de conjuntivas, uveítis, dolor muscular al tacto, adenopatía y erupción purpúrica.
En promedio, 10 % de los pacientes, tienen cuadro grave que incluye ictericia y disfunción
renal (Síndrome de Weil), neumonitis hemorrágica, arritmias cardiacas y colapso circulatorio
que origina un índice de letalidad de 5 a 40 %. La duración global del cuadro sintomático en
ambas fases de la enfermedad varía de menos de una semana a varios meses. CAUSAS: La
leptospirosis es causada por espiroquetas del género Leptospira. En épocas pasadas las
serovariedades que originaban la enfermedad eran clasificadas dentro de la especie Leptospira
interrogans. Sin embargo, por medio de la subtipificación genética, se ha sabido que existen
varias especies particulares dentro de éste grupo patógeno de Leptospira. ASPECTOS
EPIDEMIOLÓGICOS: Los reservorios de la especie Leptospira incluyen animales salvajes y
domésticos de muy diversa índole que pueden dispersar durante varios años el germen, y aun
así, permanecer asintomáticos. Los microorganismos excretados por orina, líquido amniótico o
placenta de los animales son viables en la tierra o el agua, semanas o meses. Los seres
humanos se infectan por el contacto de sus superficies mucosas o piel excoriada, con tierra,
agua o tejidos animales contaminados. Los individuos predispuestos por sus labores incluyen
quienes trabajan en rastros y con alcantarillados, veterinarios, granjeros y personal militar.
Normalmente en estos casos se considera siempre la exposición laboral a animales.
FIEBRE TIFOIDEA En la Salmonella serotipo Typhi y, otros serotipos de éste microorganismo pueden causar un
cuadro bacteriémico tardío, y de duración larga que se conoce como fiebre intestinal o
tifoidea. El cuadro comienza de manera gradual, con manifestaciones como fiebre, síntomas
generalizados (Cefalea, malestar general, anorexia y letargia), dolor espontáneo y a la
palpación en el vientre, hepatomegalia y esplenomegalia, “Manchas color de rosa” y cambios
en el estado psíquico. La fiebre intestinal puede ser un cuadro febril leve impreciso en niños de
corta edad, en que a veces hay bacteriemia sostenida o intermitente. El estreñimiento puede
ser una de las primeras manifestaciones. En los niños a menudo surge diarrea. El cuadro
faríngeo no tendría relación con este caso. El dolor abdominal también sería característico.

MONONUCLEOSIS La MONONUCLEOSIS INFECCIOSA se manifiesta en forma típica por fiebre, faringitis


INFECCIOSA exudativa, linfadenopatía, hepatoesplenomegalia y linfocitosis atípica. Las manifestaciones
patológicas son muy amplias, y varían desde un cuadro sin síntomas hasta infección letal. Las
infecciones por lo común pasan inadvertidas en lactantes y niños de corta edad. Puede surgir
alguna erupción, y es más frecuente en niños tratados con ampicilina y con otras penicilinas.
Entre las complicaciones del sistema nervioso central (SNC) están meningitis aséptica,
encefalitis y síndrome de Guillain-Barré. Complicaciones raras son rotura del bazo,
trombocitopenia, agranulocitosis, anemia hemolítica, síndrome hemofagocítico, orquitis y
miocarditis. CON LOS DATOS CLÍNICOS EXPUESTOS EN EL CASO CLÍNICO, ES POSIBLE
CONSIDERAR ESTA COMO LA OPCIÓN CORRECTA. ES MUY IMPORTANTE EL DATO DEL
EXANTEMA UNA VEZ ADMINISTRADA LA AMOXICILINA.

Bibliografía:
PRINCIPLES AND PRACTICE OF PEDIATRIC INFECTIOUS DISEASES. SARAH S. LONG. CHURCHILL,
LIVINGSTONE. EDICIÓN 2A. 2002. PÁG. 1059-1065.

378 - PARA APOYAR SU DIAGNÓSTICO DEBERÁ SOLICITAR:

CULTIVO EXÁMENES DE LABORATORIO PARA FIEBRE REUMÁTICA. El estándar de oro para el diagnóstico de
FARÍNGEO EBHGA, es el cultivo faríngeo en placa de agar sangre de cordero al 5 %. La muestra debe enviarse
al laboratorio, en medio de transporte Cary Blair o medio de transporte de Amies. Determinación
de antiestreptolisina O como evidencia de infección previa por EBHGA: • 4-5 años, 120UI/ ml. • 6-9
años, 480 UI/ml. • 10-14 años, 320 UI/ml

SEROLOGÍA MÉTODOS DIAGNÓSTICOS PARA LEPTOSPIROSIS. Es posible aislar Leptospira de muestras de


PARA sangre o líquido cefalorraquídeo en la fase septicémica inicial del trastorno, y en muestras de orina
LEPTOSPIRA después de 7 a 10 días de la enfermedad. Sin embargo, el aislamiento puede ser muy difícil,
porque se necesitan medios y técnicas especiales e incubación, incluso durante 16 semanas.
Además, es escasa la sensibilidad del cultivo de diagnóstico. Por tales razones, siempre se
obtendrán muestras de suero para facilitar el diagnóstico serológico. En la segunda semana de la
enfermedad por lo común surgen los anticuerpos y se les mide por los inmunoensayos
comerciales; sin embargo, a veces el incremento del título de las inmunoglobulinas puede
retrasarse o no aparecer en algunos pacientes. La microaglutinación, que es el método serológico
confirmatorio, se practica sólo en laboratorios especializados y para ella se necesitan muestras de
suero de fase aguda y de convalecencia. Las técnicas inmunohistoquímicas detectan anticuerpos
leptospíricos en tejidos infectados. Se ha creado una reacción en cadena de polimerasa para
detectar Leptospira, pero se le practica sólo en laboratorios especializados.
ANTICUERPOS MÉTODOS DIAGNÓSTICOS PARA MONONUCLEOSIS INFECCIOSA. Es posible aislar el virus de
HETERÓFILOS Epstein-Barr de las secreciones bucofaríngeas, pero en el laboratorio de diagnóstico corriente por
lo común no se practican las técnicas para tal finalidad, y el aislamiento del virus no indica
obligadamente la presencia de una infección aguda. Por tal razón, el diagnóstico depende de
métodos serológicos. Los practicados con mayor frecuencia son estudios inespecíficos en busca de
"anticuerpo heterófilo", que incluye la prueba de Paul-Bunnell, y la reacción de aglutinación en
laminilla. El anticuerpo heterófilo es predominantemente la inmunoglobulina (Ig) M, que surge en
las primeras dos semanas de enfermedad para desaparecer poco a poco en un lapso de seis meses.
Los resultados de las pruebas para valorar el anticuerpo mencionado suelen ser negativos en niños
menores de cuatro años con infección por EBV, pero identifican alrededor de 90 % de los casos
(Probados por métodos serológicos específicos de EBV) en niños de mayor edad, y en adultos. Un
dato característico, aunque inespecífico, es el incremento absoluto del número de linfocitos
atípicos en la segunda semana de la enfermedad, con mononucleosis infecciosa. Sin embargo,
detectar más de 10 % de linfocitos atípicos junto con la positividad de una prueba de anticuerpo
heterófilo se considera como datos que permiten diagnosticar una infección aguda. LOS ESTUDIOS
SEROLÓGICOS DEL VIRUS DE EPSTEIN-BARR PARTICULARMENTE SON ÚTILES PARA VALORAR A
PACIENTES QUE TIENEN MONONUCLEOSIS INFECCIOSA SIN ANTICUERPO HETERÓFILO.
IMPORTANTE: En tales enfermos quizá también convenga buscar otros antígenos víricos, en
particular el virus citomegálico.

BIOMETRÍA MÉTODOS DIAGNÓSTICOS PARA SALMONELOSIS. La identificación de salmonellas en cultivos de


HEMÁTICA, excrementos, sangre, orina y material obtenido de focos de infección refuerza el diagnóstico. La
REACCIONES gastroenteritis se diagnostica por medio del cultivo de excrementos. Se cuenta con métodos
FEBRILES rápidos que utilizan inmunoanálisis enzimáticos, aglutinación de látex, sondas de DNA y
anticuerpos monoclonales, y se utilizan en algunos laboratorios. No se recomienda aplicar
métodos serológicos en busca de aglutininas de Salmonella (Pruebas febriles o prueba de Widal).
Como puedes ver actualmente esta respuesta es errónea en todos los sentidos.

Bibliografía:
PRINCIPLES AND PRACTICE OF PEDIATRIC INFECTIOUS DISEASES. SARAH S. LONG. CHURCHILL,
LIVINGSTONE. EDICIÓN 2A. 2002. PÁG. 1059-1065.

379 - DURANTE LA REVALORACIÓN, ES INMINENTE LA PRESENCIA DE UNA COMPLICACIÓN, EN ESTE


CASO SE DEBERÁ ADMINISTRAR:

ERITROMICINA TRATAMIENTO FIEBRE REUMÁTICA. Hay dos métodos terapéuticos necesarios en los pacientes
con fiebre reumática aguda: Antibioticoterapia antiestreptocócica y Tratamiento de las
manifestaciones clínicas de la enfermedad. Al momento del diagnóstico, a todos los pacientes
con fiebre reumática aguda, se les tratará como si tuviesen una infección por estreptococo del
grupo A, se haya o no se haya recuperado el microorganismo mediante cultivo. Se comenzará
de inmediato la antibioticoterapia estándar: Un régimen completo de 10 días en adultos con
penicilina V oral (500 mg dos veces al día) o eritromicina (250 mg cuatro veces al día) en los
pacientes con alergia a la penicilina. Muchos optan por penicilina G benzatínica intramuscular
(Una sola inyección intramuscular de 1.2 millones de unidades) para tratar la probable
infección estreptocócica. Manejo inicial mientras se realiza la referencia y seguimiento de
casos. Hasta que el diagnóstico esté confirmado, es recomendable tratar el dolor articular con
paracetamol. Los salicilatos son recomendados como primera línea de tratamiento debido a la
amplia experiencia en fiebre reumática aguda. Deberá iniciarse en pacientes con artritis o
artralgias severas tan pronto como el diagnóstico se haya confirmado. El naproxeno ha sido
usado exitosamente en pacientes con FRA y es una alternativa segura en caso de intolerancia a
salicilatos. El medicamento inicial para el manejo de la corea severa, es la carbamazepina. En
casos refractarios, se puede utilizar ácido Valproico. La inmunoglobulina intravenosa es
recomendada para tratar la corea severa refractaria, por la recuperación más rápida; sin
embargo, no reduce la incidencia de enfermedad valvular a largo plazo. Para el control de la
carditis, el esteroide de elección es la prednisona o prednisolona. La metilprednisolona se
indica en casos severos. El cuidado dental rutinario es muy importante en pacientes con
historia de fiebre reumática y/o cardiopatía reumática y resulta primordial previo a cirugía
valvular, en donde debe valorarse y tratarse toda la patología oral-dental.
CIPROFLOXACINA TRATAMIENTO FARMACOLÓGICO DE LA FIEBRE TIFOIDEA. Para el tratamiento ambulatorio y
hospitalario de niños y adultos con fiebre tifoidea (casos aislados, casos fuera de brote
epidémico o cuando el germen sea sensible con base al patrón de susceptibilidad de S. typhi)
se recomiendan los siguientes antimicrobianos como fármacos de primera línea: 1.
Ciprofloxacina: • Niños: 15 a 20 mg/Kg/día VO c/12 h por 7 días • Adultos: 500 mg VO c/12 h
por 7 días 2. Cefixima: • Niños: 15 a 20 mg/Kg/día VO c/12 h por 14 días • Adultos: 200 mg VO
c/12 h por 14 días 3. Cloranfenicol: • Niños: 50 a 75 mg/kg/día VO c/6 h por 14 días (no
exceder 3 g) • Adultos: 500 mg VO c/6 h por 14 días (no exceder 3 g) Cuando no es posible
utilizar los fármacos de primera línea las alternativas son: 1. Ampicilina: • Niños: 50 a 100
mg/Kg/día VO c/6 h por 14 días • Adultos: 1gr VO cada 6hs 2. Amoxicilina: • Niños: 50 a 100
mg/Kg/día VO c/6 h por 14 días • Adultos: 1gr VO cada 8hs 3. Trimetropima –sulfametoxazol: •
Niños: 4 a 10 mg/Kg/día (en base a trimetoprima) VO c/12 h por 14 días • Adultos: 160 mg (en
base a trimetroprim) VO c/12 h por 14 días Se recomienda utilizar azitromicina en los casos de
brote epidémico de fiebre tifoidea o cuando se trate de resistencia probada de S typhi a
fármacos de primera línea; siempre y cuando exista la posibilidad de ofrecer tratamiento
ambulatorio: • Niños: 10 mg/kg/día VO cada 24 hrs por 7 días. • Adultos: 500 mg VO cada 24
hrs por 7 días Las cefalosporinas de tercera generación están indicadas en los siguientes casos:
1. Falla al tratamiento ambulatorio inicial 2. Resistencia a fármacos de primera línea 3.
Imposibilidad para administrar los antimicrobianos por vía oral 4. Fiebre tifoidea complicada
(Ver anexo 6.3; cuadro 2) 5. Recaída de la enfermedad Cefotaxima: • Niños: 40 a 80 mg/Kg/día
IV c/8 h por 14 a 21 días • Adultos: 1 a 2 g IV c/6 - 8 h por 14 a 21 días Ceftriaxona: • Niños: 50
a 75 mg/Kg/día IV o IM c/12 a 24 h por 14 a 21 días • Adultos: 2 a 4 g IV o IM c/12 a 24 h por
14 a 21 días Se sugiere el cambio a tratamiento por vía oral sólo cuando se conozca que la cepa
aislada es susceptible a cualquiera de las alternativas por vía oral. Para el tratamiento de
erradicación del estado de portador los antimicrobianos de primera elección son: 1.
Ciprofloxacina: • Niños: 15 a 20 mg/Kg/día VO c/12 h por 28 días • Adultos: 750 mg VO c/12 h
por 28 días 2. Amoxicilina: • Niños: 100 mg/Kg/día VO c/6 h por 6 semanas • Adultos: 1gr VO
cada 6hs por 6 semanas 3. Trimetoprima – Sulfametoxazol (TMP/SMZ): • Niños: 4 a 10
mg/Kg/día (TMP) VO c/12 h por 6 semanas • Adultos: 160 mg (TMP) VO c/12 h por 6 semanas
4. Ampicilina: • Niños: 100 mg/Kg/día VO c/6 h por 6 semanas • Adultos: 1gr VO cada 6hs por 6
semanas En las mujeres gestantes con fiebre tifoidea se recomienda el tratamiento con
ampicilina, amoxicilina o cefalosporinas de tercera generación en las dosis previamente
descritas. Para el control de la fiebre es recomendable el uso de ibuprofeno o paracetamol.

PENICILINA TRATAMIENTO PARA LEPTOSPIROSIS. El fármaco más indicado para sujetos que necesitan
hospitalización es la penicilina intravenosa. La penicilina G acorta el lapso de los síntomas
sistémicos y la persistencia de las anormalidades en estudios de laboratorio, y puede evitar la
aparición de leptospiruria. Como ocurre con otras espiroquetosis, a veces surge después de
emprender la terapia con penicilina, una reacción de Jarisch-Herxheimer (Reacción febril
aguda acompañada de cefalea, mialgias y agravamiento del cuadro clínico, que dura menos de
24 h). Si la enfermedad no es intensa, con doxiciclina ingerible se ha acortado la evolución del
padecimiento, y también la frecuencia con que surge la leptospiruria. No debe utilizarse
doxiciclina en embarazadas ni en niños menores de ocho años por el peligro de que queden
con manchas en sus dientes. Otro fármaco por usar en niños menores de ocho años es la
amoxicilina oral.

PREDNISONA TRATAMIENTO PARA MONONUCLEOSIS INFECCIOSA. Es importante que la persona no


participe en deportes por contacto hasta que se recupere totalmente de la mononucleosis
infecciosa y, no se le palpe el bazo. No se administrará ampicilina ni amoxicilina a individuos
en quienes se sospeche la enfermedad mencionada porque los fármacos originarán erupciones
morbiliformes, no alérgicas en una elevada proporción de sujetos con mononucleosis. La
administración de un ciclo breve de "corticosteroides" pudiera tener efecto beneficioso en los
síntomas agudos, pero ante sus posibles efectos adversos habrá que destinar su uso
exclusivamente a pacientes con complicaciones como la inflamación amigdalina extraordinaria
con obstrucción inminente de vías respiratorias, esplenomegalia masiva, miocarditis, anemia
hemolítica o síndrome hemofagocítico. La dosis de prednisona suele ser de 1 mg/kg de peso al
día, ingerida (Máximo, 20 mg si el niño pesa más de 10 kg), durante siete días con disminución
gradual ulterior. El Aciclovir posee actividad antivírica in vitro contra EBV, pero no ha tenido
utilidad probada en los síndromes linfoproliferativos contra dicho virus. Disminuir las dosis de
inmunosupresores podría ser una estrategia favorable en sujetos con linfoproliferación
inducida por EBV, como el caso de los trastornos linfoproliferativos después de trasplantes.
LOS CORTICOESTEROIDES ESTÁN BIEN INDICADOS EN PRESENCIA DE SÍNTOMAS AGUDOS
SECUNDARIOS A MONONUCLEOSIS INFECCIOSA.

Bibliografía:
PRINCIPLES AND PRACTICE OF PEDIATRIC INFECTIOUS DISEASES. SARAH S. LONG. CHURCHILL,
LIVINGSTONE. EDICIÓN 2A. 2002. PÁG. 1059-1065.

FIN DEL CASO CLÍNICO SERIADO


ANÁLISIS DEL CASO CLÍNICO

IDENTIFICACIÓN DEL REACTIVO


Area: PEDIATRÍA
Especialidad: URGENCIAS PEDIÁTRICAS
Tema: URGENCIAS CARDIOVASCULARES PEDIÁTRICAS
Subtema: VALORACIÓN GENERAL DE LAS CARDIOPATÍAS

CASO CLÍNICO CON UNA PREGUNTA

LACTANTE DE 9 MESES CON UNA COMUNICACIÓN INTRAVENTICULAR DETECTADA DESDE EL NACIMIENTO. ES


INGRESADA AL HOSPITAL CON DATOS CLÍNICOS DE INSUFICIENCIA CARDÍACA. SE INICIA MANEJO CON
DIGOXINA.

lactante de 9 meses de edad

Diagnóstico de Comunicación
Interventricular desde el nacimiento

Presenta datos clínicos de Insu ciencia


Cardíaca

Presenta datos clínicos de Insu ciencia


Cardíaca

----

380 - UNO DE LOS MECANISMOS FARMACOLÓGICOS PRINCIPALES DE LA DIGOXINA QUE JUSTIFICAN SU


USO EN LA PACIENTE ES SU EFECTO DE:

AUMENTO DEL En pacientes con insuficiencia cardiaca (IC), digoxina disminuye la frecuencia ventricular tanto
TONO en el ritmo sinusal (RS), debido al retiro de la estimulación simpática, como en la fibrilación
PARASIMPÁTICO auricular (FA) por "incremento del tono parasimpático". Estas drogas producen también
aumento del flujo sanguíneo, descenso de la resistencia vascular, venodilatación y reducción de
la presión venosa central y del ritmo cardíaco. La vasodilatación resulta del incremento en el
gasto cardíaco (GC) y de la directa disminución de la vasoconstricción simpática mediada por
los barorreflejos. En sujetos con severa obstrucción de arterias coronarias, la administración
intravenosa (IV) de digoxina puede producir vasoconstricción e isquemia miocárdica transitoria
en la circulación coronaria. Los digitálicos normalizan los mecanismos directos de los
barorreceptores que se hallan en la IC. Además, puede existir una reducción en los niveles de
norepinefrina plasmática, de aldosterona sérica y de la actividad de la renina plasmática. En
pacientes con IC y retención de líquidos, digoxina induce diuresis al incrementar el GC y la
hemodinamia renal, inhibiendo la reabsorción tubular de sodio y aumentando la secreción de
péptidos natriuréticos auriculares.
AUMENTO DE LA Farmacología DIGOXINA. Se excreta exponencialmente con una eliminación vida-media de 36-
ACTIVIDAD 48 horas en pacientes con función renal normal, produciendo una pérdida de aproximadamente
SIMPÁTICA un tercio de las reservas corporales por día. La excreción renal de digoxina es proporcional al
ritmo de filtración glomerular y así, al clearance de creatinina. Cuando las pérdidas diarias están
acompañadas de una toma diaria, se alcanza un estado de equilibrio con la terapia de
mantenimiento diario. En pacientes con función renal normal que están comenzando con esa
droga, esto puede llevar 5-7 vidas-medias ó 7-10 días. Digoxina tiene un índice terapéutico
extremadamente bajo, y su uso debe ser cuidadosamente monitoreado a través de los niveles
séricos en sangre. El nivel sérico óptimo es de 0,5 a 1,0 ng/mL. Esto se alcanza con una dosis
prescripta de 0,125 mg a 0,25 mg/día (toma oral). En la población geriátrica y en aquellos con
disfunción renal, la dosis puede ser incluso más baja, como 0,125 mg en días alternos. Las
muestras de sangre para la medición de digoxina sérica deben obtenerse al menos de 6 a 8
horas luego de la última dosis. Generalmente, la toxicidad clínica manifiesta tiende a aflorar en
concentraciones séricas mayores a 2,0 ng/mL. Debe sospecharse toxicidad digitálica siempre
que los pacientes que consuman dicha droga se presenten con molestias gastrointestinales
(náuseas, vómitos, diarrea), con síntomas neurológicos (confusión mental, temblores en las
extremidades) o cardiovasculares (bloqueos aurículo-ventriculares -AV-, latidos ventriculares
prematuros, taquicardia auricular con bloqueos AV variables). En estos individuos debe retirarse
la droga, y los niveles séricos de digoxina. El incremento del automatismo del tejido cardíaco en
respuesta a los digitálicos se ve aumentada en la hipokalemia, y algunos abogan por la
administración de potasio vía oral para ritmos ectópicos auriculares, AV o ventriculares. La
disponibilidad generalizada de los fragmentos Fab policlonales de alta afinidad, anticuerpos
específicos para digoxina, puede salvar a los pacientes con arritmias por toxicidad digitálica
potencialmente mortales.

BLOQUEO DE LA Beneficios clínicos de la DIGOXINA en la insuficiencia cardíaca. Varios estudios clínicos con
ACTIVIDAD digitálicos han documentado una mejoría en la sintomatología de los pacientes con IC. También
ELÉCTRICA mejoran la clase funcional (CF) de la IC, aumentan la capacidad de ejercicio y el consumo de
oxígeno (VO2), mejoran los parámetros hemodinámicos en reposo y en ejercicio, aumentan la
fracción de eyección ventricular izquierda (FEVI) en reposo y en ejercicio, y reducen la
frecuencia cardíaca (FC). Estos beneficios ocurren independientemente del ritmo cardíaco o de
la etiología de la IC.

AUMENTO DE LA Recomendaciones de las guías internacionales para la insuficiencia cardíaca crónica. La


ACTIVIDAD European Society of Cardiology (ESC) recomienda el uso de digoxina (clase I, nivel de evidencia
ELÉCTRICA C) en pacientes en ritmo sinusal con IC sintomática y FEVI?40%. También recomienda el uso
previo de beta bloqueantes para el control inicial del ritmo ventricular en un paciente con FA
rápida en IC descompensada, clase IIa y nivel de evidencia B. Las guías afirman que es
obligatorio el monitoreo de electrolitos séricos y de la función renal. La ESC contraindica
digoxina en pacientes con bloqueo cardíaco de segundo y tercer grado, en síndromes de
preexcitación y si hay evidencia previa de intolerancia a la digoxina. En pacientes con función
renal normal la concentración de digoxina debe ser controlada tempranamente durante la
terapia crónica. El American College of Cardiology (ACC) y la American Heart Association (AHA)
recomiendan considerar la posibilidad de sumar digoxina al tratamiento de los pacientes con
síntomas persistentes de IC, durante la terapia con diuréticos, e IECA o ARA y beta bloqueantes.
Si el paciente está tomando digoxina y no un IECA o beta bloqueante, no debe suspenderse el
tratamiento con digitálicos, pero debe iniciarse una terapia adecuada con antagonistas
neurohormonales. Digoxina es prescripta de manera rutinaria en pacientes con IC y FA crónica,
pero los beta bloqueantes usualmente son más efectivos cuando se adicionan a digoxina en el
control de la respuesta ventricular, particularmente durante el ejercicio. De acuerdo a estas
guías, digoxina no está indicada como terapia principal para la estabilización de pacientes con
una exacerbación aguda de síntomas de IC, incluyendo retención de líquidos o hipotensión. La
recomendación del uso de digitálicos, en estas guías, es actualmente IIa.

Bibliografía:
1. THE PHARMACOLOGICAL BASIS OF THERAPEUTICS. BRUNTON IL, LAZO JS, PARKER KL. MC GRAW HILL.
EDICIÓN ELEVEN. 2005. PAG. 886-889.
ANÁLISIS DEL CASO CLÍNICO

IDENTIFICACIÓN DEL REACTIVO


Area: PEDIATRÍA
Especialidad: URGENCIAS PEDIÁTRICAS
Tema: URGENCIAS QUIRÚRGICAS Y PATOLOGIAS DE RESOLUCIÓN
QUIRÚRGICA
Subtema: APENDICITIS AGUDA

CASO CLÍNICO CON UNA PREGUNTA

PREESCOLAR DE 3 AÑOS, POSTOPERADO POR UNA APENDICITIS COMPLICADA. DURANTE SU


RECUPERACIÓN PRESENTA UNA FISTULA ENTEROCUTANEA, QUE REQUERIRÁ COMO PARTE DEL
TRATAMIENTO AYUNO PROLONGADO.

preescolar de 3 años de edad

operado de una apendicectomía por una


apendicitis complicada

presencia de stula enterocutánea

fístula enterocutánea

381 - DURANTE EL AYUNO PROLONGADO EL MUSCULO DEL NIÑO UTILIZA COMO PRINCIPAL FUENTE DE
ENERGÍA A:

LA HORMONA La Hormona de crecimiento por la somatomedina nos ayuda a sintetizar proteínas,impide se


DE consuma glucosa por las células. Es hiperglucemiante tiene una acción lipolítica (es la que ahora
CRECIMIENTO nos interesa). Es lipolítica, diabetógena y cetogénica por su acción sobre las proteínas durante el
ayuno y se postula que tiene un preponderante papel protector de su metabolismo. Su secreción
es irregular, variando a medida que el ayuno progresa; ésto depende entre otras cosas del stress
que presenta el ayuno a nivel hipotalámico y cortical o hipotalámico solamente. Los efectos de la
STH difieren en músculo y tejido adiposo. En el músculo, la STH antagoniza la acción de la
insulina, inhibiendo la glucolisis, y en tejido adiposo produce aumento de la oxidación de la
glucosa. Esta Hormona interviene en la SEGUNDA FASE.
LOS El cerebro no funciona sin glucosa. Esta se formaría en el proceso de la neoglucogénesis en el
AMINOÁCIDOS hígado y en el riñón, ya que: La insulina que disminuye la glucogénesis y aumenta la utilización de
los aminoácidos para la formación de proteínas, al estar disminuída produciría el efecto contrario.
Los aminoácidos se utilizan para formar a partir de ellos glucosa. EN LAS PRIMERAS ETAPAS DEL
AYUNO SE UTILIZAN LOS ÁCIDOS GRASOS Y CUERPOS CETÓNICOS.

LOS CUERPOS En condiciones de ayuno el HÍGADO forma aceto-acetato y beta-hidroxibutirato a partir del
CETÓNICOS acetil-CoA formado tras la oxidación de ácidos grasos. Permite la liberación de SH-CoA para que
continúe la beta-oxidación. Estos CUERPOS CETÓNICOS se transportan por la sangre otros tejidos
que los oxidarán por el ciclo del ácido cítrico para producir energía.

LOS ÁCIDOS LOS ÁCIDOS GRASOS QUE SE MOVILIZAN DEL TEJIDO ADIPOSO CONSTITUYEN UNA BUENA
GRASOS Y FUENTE ENERGÉTICA QUE SE UTILIZARÁ CON PREFERENCIA A LA GLUCOSA EN LA MAYORÍA DE
CUERPOS LOS TEJIDOS. En el hígado, la oxidación de los ácidos grasos aporta la mayor parte del ATP
CETÓNICOS necesario para la gluconeogénesis. Sin embargo, en el estado de ayuno, sólo una pequeña parte
del acetilCoA que se libera en la b-oxidación entra en el ciclo del ácido cítrico para su completa
oxidación. El destino principal de esta molécula es la formación hepática de cuerpos cetónicos
que se liberan a la sangre y que se captan en los tejidos que pueden utilizarlos como fuente
energética. En el cerebro, aunque constituyen el combustible alternativo a la glucosa, los cuerpos
cetónicos no satisfacen por completo las necesidades energéticas de sus células, para las cuales es
siempre necesario el suministro del monosacárido. En el MUSCULO esquelético, los CUERPOS
CETÓNICOS evitan que se produzca la hidrólisis de las proteínas, ya que, a medida que los ácidos
grasos se oxidan en el hígado, aumenta la concentración de los cuerpos cetónicos en el plasma y,
en consecuencia, las células demandan menos glucosa y menos aminoácidos gluconeogénicos. En
estas condiciones, no se activa la proteólisis ni tiene lugar, por tanto, la destrucción del
fundamental tejido muscular. LOS ÁCIDOS GRASOS SE MOVILIZAN DE MANERA GENERAL COMO
ALTERNATIVA PARA PRODUCIR ENERGÍA EN LA MAYORÍA DE LOS TEJIDOS, INCLUIDO EL
MÚSCULO; MIENTRAS QUE LOS CUERPOS CETÓNICOS SON UNA ALTERNATIVA ESPECÍFICA EN EL
MÚSCULO.

Bibliografía:
BIOCHEMISTRY. DAVIDSON. HARWAL. EDICIÓN 3. 1994. PAG. 516.
ANÁLISIS DEL CASO CLÍNICO

IDENTIFICACIÓN DEL REACTIVO


Area: MEDICINA INTERNA
Especialidad: ENDOCRINOLOGÍA
Tema: URGENCIAS ENDOCRINAS
Subtema: SÍNDROME HIPEROSMOLAR HIPERGLUCÉMICO NO CETÓSICO

CASO CLÍNICO CON UNA PREGUNTA

MUJER DE 65 AÑOS CON DIABETES MELLITUS DESDE HACE 20 AÑOS. LA ENCUENTRAN SEMIINCONSCIENTE
EN SU DOMICILIO POR LO QUE ES LLEVADA INMEDIATAMENTE A URGENCIAS. A LA EXPLORACIÓN SE
ENCUENTRA TA NORMAL, FC 110/MIN, FR 25/MIN, SIN RESPUESTA A ESTÍMULOS, RESPUESTA PLANTAR
EXTENSORA, MUCOSA ORAL MUY SECA, PULSOS LIGERAMENTE DISMINUÍDOS DE INTENSIDAD, GODETE
POSITIVO. NA 154 MEQ/L, K 3.8 MEQ/L, CL 106 MEQ/L, CO2 22 MEQ/L, GLUCOSA 620 MG/DL, BUN 49 MG/DL.

mujer de 65 años de edad.

diabética desde hace 20 años.

se encuentra con alteración en el estado de


conSciencia.

normotensa, taquicardica, fr 25/min, sin


respuesta a estímulos, respuesta plantar
extensora, mucosa oral deshidratada,
pulsos disminuidos de intensidad.

sodio 154 meq/l, potasio 3.8 meq/l, co2 33


meq/l, glucosa 620 mg/dl bun 49 mg/dl.

382 - EL DIAGNÓSTICO CLÍNICO MAS PROBABLE ES:

DESHIDRATACIÓN Los principales síntomas de la hipernatremia son neurológicos y consisten en alteraciones del
E estado de conciencia, debilidad, irritabilidad neuromuscular, déficit neurológico focal y, en
HIPERNATREMIA ocasiones, coma o convulsiones. También puede haber poliuria o sed. En esta paciente
evidentemente tanto la deshidratación como la hipernatremia pueden contribuir de manera
significativa a las manifestaciones clínicas del paciente, sin embargo las alteraciones en el
estado de hidratación y en las concentraciones séricas de sodio son secundarias a la
hiperglucemia.
ENFERMEDAD Las manifestaciones clínicas de una enfermedad vascular cerebral son muy amplias y
VASCULAR dependen de la zona afectada y su extensión. Sin embargo por los antecedentes de la
CEREBRAL. paciente, los síntomas y los estudios de laboratorio lo primero a descartar es se trate de un
estado hiperosmolar hiperglucémico. Aunque en un porcentaje de pacientes la causa
desencadenante del estado hiperosmolar pueden ser EVC agudos.

CETOACIDOSIS. La CETOACIDOSIS DIABÉTICA puede ser el complejo sintomático inicial que culmina en el
diagnóstico de DM de tipo 1, pero ocurre con más frecuencia en personas que experimentan
diabetes establecida. A menudo son prominentes náuseas y vómitos. El dolor abdominal
puede ser intenso La hiperglucemia produce glucosuria, deficiencia de volumen y taquicardia.
Puede ocurrir hipotensión a causa de la deficiencia de volumen combinada con vasodilatación
periférica. Dos signos clásicos de este trastorno son respiración de Kussmaul y aliento afrutado
(por acidosis metabólica y aumento de los cuerpos cetónicos). Letargo y depresión del sistema
nervioso central pueden evolucionar hasta el coma en caso de cetoacidosis diabética grave. ES
MÁS COMÚN EN DIABETES TIPO 1 O EN DIABETES TIPO 2 CON INSULINOTERAPIA EN
PACIENTES MÁS JÓVENES CON UN FACTOR DESENCADENANTE (ALCOHOLISMO,
INFECCIONES).

ESTADO El paciente prototípico en ESTADO HIPEROSMOLAR HIPERGLUCÉMICO (HHS) es un anciano


HIPEROSMOLAR. con DM de tipo 2 que tiene antecedentes de varias semanas de duración con poliuria, pérdida
de peso y decremento de la ingestión oral que culminan en confusión mental, letargo o coma.
Los datos de la exploración física reflejan deshidratación profunda e hiperosmolaridad y
revelan hipotensión, taquicardia y trastorno del estado mental. Es notable la ausencia de
síntomas como náuseas, vómitos y dolor abdominal, así como de la respiración de Kussmaul
característica de la cetoacidosis diabética. RECUERDA QUE ESTA COMPLICACIÓN ES MÁS
FRECUENTE EN PACIENTES CON DM2. OJO: En el ENARM hemos notado el uso indistinto de
HCO3 y CO2 total en los casos clínicos, por lo que debes poner especial atención a este valor.
En virtud de las características de disociación del ácido carbónico (H2CO3) a pH corporal, el
dióxido de carbono disuelto se encuentra casi de manera exclusiva en bicarbonato; por lo
tanto, con propósitos clínicos, el contenido total de dióxido de carbono es equivalente
(+-3meq/L) a la concentración del bicarbonato, y de ahí que en muchas ocasiones se reporte
de forma indiferente como HCO3 o CO2 en sangre. Lo anterior se explica con el siguiente
esquema: H + HCO3 <--> H2CO3 <--> CO2+ H2O Las formas clásicas con las cuales se reporta
este estudio son: examen de bicarbonato, HCO3-, examen de bióxido de carbono, TCO2, CO2
total o prueba de CO2 en suero. Según el autor, los valores normales oscilan entre 22-23 a 28-
29mEq/L o mmol/L.

Bibliografía:
1. LONGO DL, FAUCI AS, KASPER DL, HAUSERSL, JAMESON JL, LOSCALZOJ. HARRISON. PRINCIPIOS DE
MEDICINA INTERNA, 18A EDICIÓN. MC GRAW HILL. NEW YORK, USA. 2012, PP 2979. 2. GUÍA DE PRÁCTICA
CLÍNICA. DIAGNÓSTICO Y TRATAMIENTO DEL SÍNDROME HIPERGLUCÉMICO HIPEROSMOLAR EN
ADULTOS DIABÉTICOS TIPO 2 EN EL SEGUNDO Y TERCER NIVELES DE ATENCIÓN. MÉXICO: SECRETARIA
DE SALUD: OCTUBRE 2013. 3. PAPADAKIS MAXINE A, MCPHEE STEPHEN J. DIAGNÓSTICO CLÍNICO Y
TRATAMIENTO. 52ª EDICIÓN. NUEVA YORK. 2013.

http://www.cenetec.salud.gob.mx/descargas/gpc/CatalogoMaestro/160_GPC_SINDROME_HIPEROSMOLAR/SSA-160-09_HIPERGLUCxMICO_HIPEROSMOLAR_DM-EVR.pdf
ANÁLISIS DEL CASO CLÍNICO

IDENTIFICACIÓN DEL REACTIVO


Area: MEDICINA INTERNA
Especialidad: ENDOCRINOLOGÍA
Tema: TRANSTORNOS DEL METABOLISMO Y DE LAS VITAMINAS
Subtema: TRANSTORNOS DEL METABOLISMO DE LAS LIPOPROTEÍNAS

CASO CLÍNICO CON UNA PREGUNTA

HOMBRE DE 45 AÑOS A QUIÉN SE LE DESCARTO RECIENTEMENTE SÍNDROME METABÓLICO, SIN EMBARGO


DURANTE SUS ESTUDIOS SE DETECTA HIPERTRIGLICERIDEMIA.

hombre de 45 años.

se descarto síndrome metabólico.

-.

-.

HIPERTRIGLICERIDEMIA.

383 - EL MEDICAMENTO DE MAYOR UTILIDAD PARA EL TRATAMIENTO DE ESTE PACIENTE ES:

EZETIMIBA. La EZETIMIBA es un inhibidor de la absorción de colesterol. Este medicamento evita la


absorción intestinal del colesterol de la dieta y biliar sin afectar la absorción de triglicéridos o
vitaminas liposolubles. INDICADO EN HIERCOLESTEROLEMIA, NO TIENE EFECTO SOBRE
TRIGLICÉRIDOS.

ROSUVASTATINA. Es importante saber que los pacientes que tienen diabetes tipo 2 o síndrome metabólico, la
pérdida de peso, ejercicio y los agentes que disminuyen la glucosa son importantes para la
reducción de los triglicéridos. Si no se mejora el control glucémico generalmente la
hipertrigliceridemia persiste a pesar de iniciar tratamiento. El medicamento más utilizado es la
metformina aunque también se puede utilizar pioglitazona o rosiglitazona. Estos aumentan la
sensibilidad de la insulina reducen los niveles de triglicéridos a través de aumentar su
eliminación plasmática. LA ROSUVASTATINA PUEDE DISMINUIR LOS NIVELES DE
TRIGLICÉRIDOS DE FORMA SECUNDARIA, PERO NO ES SU INDICACIÓN PRINCIPAL.
COLESTIPOL. El COLESTIPOL es un medicamento que quela los ácidos biliares. Estos son polímeros que se
unen a los ácidos biliares con carga negativa junto con las sales biliares en el intestino delgado.
Esto interrumpe la circulación entero-hepática de los ácidos biliares produciendo un aumento
en la conversión de colesterol en la bilis en el hígado. Esto resulta en una disminución de la
cantidad de colesterol en los hepatocitos que promueve un aumento en los receptores de LDL
y aumenta la eliminación de LDL de la circulación. INDICADO EN HIERCOLESTEROLEMIA, NO
TIENE EFECTO SOBRE TRIGLICÉRIDOS.

GEMFIBROZIL. Existen varios medicamentos para reducir los triglicéridos. Tanto la niacina como los fibratos
producen una reducción de los mismos de 40 a 50%. Sin embargo los fibratos son mejor
tolerados que la niacina ya que aproximadamente la mitad de los pacientes que toman esta
última presentan efectos secundarios por lo que los fibratos se prefieren para el tratamiento.
LOS FIBRATOS CORRESPONDEN AL TRATAMIENTO DE ELECCIÓN EN HIPERTRIGLICERIDEMIA.

Bibliografía:
1. DIAGNÓSTICO Y TRATAMIENTO DE LAS DISLIPIDEMIAS (HIPERCOLESTEROLEMIA) EN EL ADULTO.
EVIDENCIAS Y RECOMENDACIONES: GUÍA DE PRÁCTICA CLÍNICA. MÉXICO: SECRETARÍA DE SALUD.
03/11/2016. 2. LONGO DL, FAUCI AS, KASPER DL, HAUSERSL, JAMESON JL, LOSCALZOJ. HARRISON.
PRINCIPIOS DE MEDICINA INTERNA, 18A EDICIÓN. MC GRAW HILL. NEW YORK, USA. 2012, PP 3156-3161.

http://www.cenetec-difusion.com/CMGPC/IMSS-233-09/ER.pdf
ANÁLISIS DEL CASO CLÍNICO

IDENTIFICACIÓN DEL REACTIVO


Area: MEDICINA INTERNA
Especialidad: NEUROLOGÍA
Tema: EPILEPSIA
Subtema: CRISIS CONVULSIVAS PARCIALES Y GENERALIZADAS

CASO CLÍNICO CON UNA PREGUNTA

HOMBRE DE 45 AÑOS CON DIAGNÓSTICO DE EPILEPSIA DESDE HACE 3 AÑOS. ACTUALMENTE SE CONSIDERÁ
AL NEUROTRANSMISOR EXCITATORIO MÁS ABUNDANTE DEL SISTEMA NERVIOSO CENTRAL COMO
FUNDAMENTAL EN LA GENESIS DE ALGUNOS TIPOS DE EPILEPSIA.

HOMBRE DE 45 AÑOS.

DIAGNÓSTICO DE EPILEPSIA DESDE HACE


3 AÑOS.

GéNESIS DE ALGUNOS TIPOS DE


EPILEPSIA.

-.

-.

384 - ESTE NEUROTRANSMISOR ES:

LA EPILEPSIA. Estudios epidemiológicos indican que entre 0.5 y 1% de la población mundial padece
ACETILCOLINA. epilepsia y se considera que entre 1 y 3 % de la población tendrá epilepsia durante su vida. En
México, la prevalencia de pacientes con epilepsia es de 10 a 20 por cada 1000, lo cual sugiere que
en nuestro país existen por lo menos un millón de personas con alguna forma de epilepsia. La
epilepsia es una alteración del sistema nervioso central (SNC) caracterizada por un incremento y
sincronización anormales de la actividad eléctrica neuronal, que se manifiesta con crisis
recurrentes y espontáneas así como por cambios electroencefalográficos. En general, su
diagnóstico implica detectar una anormalidad epileptogénica persistente del cerebro que es
capaz de generar actividad paroxística espontáneamente. Lo anterior difiere de un cerebro que ha
tenido una crisis aguda como una respuesta natural a una alteración o pérdida de la homeostasis.
En condiciones normales, la actividad nerviosa se mantiene en un estado de equilibrio dinámico
regulado por procesos neuronales inhibitorios y excitatorios. Un desequilibrio entre estos
mecanismos puede producir epilepsia. LA ACETILCOLINA ES EL PRINCIPAL NEUROTRANSMISOR
RELACIONADO CON MIASTENIA GRAVE.
LA LAS CRISIS EPILÉPTICAS PARCIALES O DE INICIO FOCAL. Las crisis parciales, focales o relacionadas
SEROTONINA. con localización se caracterizan porque la actividad eléctrica anormal inicia y se queda restringida
en cierta área cerebral y forma el foco epiléptico, pudiendo restringirse a un hemisferio cerebral.
A su vez, las crisis parciales se clasifican teniendo en cuenta si se altera o no la conciencia. Las
crisis parciales simples son aquellas en las que la conciencia no se ve alterada; mientras que en las
crisis parciales complejas existe pérdida de la conciencia y generalmente se involucra al sistema
límbico, que incluye estructuras como el hipocampo y la amígdala. No obstante, una crisis parcial
simple puede convertirse en una crisis parcial compleja o en una generalizada. LOS MECANISMOS
DE GENERACIÓN DE LA EPILEPSIA FOCAL. La capacidad de algunas poblaciones de neuronas de
generar descargas sincrónicas, excesivas y de alta frecuencia determina el desarrollo de la
epileptogénesis focal y depende de la interacción de varios factores que incluyen los siguientes
mecanismos: 1) Generación de descargas en ráfaga. Ésta (trenes de potenciales de acción) es un
mecanismo de amplificación de las señales, ya que las neuronas que generan múltiples impulsos
en respuesta a un estímulo tenderían a liberar más neurotransmisores desde su terminal sináptica.
2) Disminución de mecanismos inhibitorios. La inhibición postsináptica está ampliamente
distribuida en los circuitos corticales y es el mecanismo de control que previene el desarrollo de
descargas epilépticas sincrónicas. 3) Potenciales postsinápticos excitatorios. Los circuitos
sinápticos excitatorios en estructuras corticales representan un tercer elemento que determina el
desarrollo de la epileptogénesis focal. Los potenciales postsinápticos excitatorios (PPE) participan
en este proceso y son evidentes cuando la inhibición está disminuida. LA SEROTONINA SE
RELACIONA CON ALTERACIONES EN EL ESTADO DEL ÁNIMO Y VÍAS DEL DOLOR.

LA LAS CRISIS EPILÉPTICAS GENERALIZADAS O DE INICIO GENERALIZADO. En las crisis


DOPAMINA. generalizadas, la descarga incontrolable de las neuronas involucra a ambos hemisferios
cerebrales. La crisis epiléptica inicia en un área del cerebro y se propaga a toda la masa encefálica.
La conciencia se altera como posible manifestación inicial y se presentan conductas motoras
bilaterales. El patrón electroencefalográfico ictal es bilateral al inicio y presumiblemente refleja
descarga neuronal, la cual involucra a ambos hemisferios. Las crisis generalizadas pueden además
subdividirse en crisis convulsivas (crisis tónico-clónicas, tónicas y clónicas) y en crisis no
convulsivas (crisis de ausencia, mioclónicas y atónicas). LOS PERÍODOS DE LA ACTIVIDAD
EPILÉPTICA. La actividad epiléptica se caracteriza por tres períodos: 1) El período ictal o ictus, que
corresponde a la crisis epiléptica per se, es un evento intermitente y breve que puede durar desde
segundos hasta minutos y que presenta un patrón electroencefalográfico hipersincrónico
asociado con cambios conductuales. Las crisis recurrentes inducen mecanismos homeostáticos
que actúan para disminuir la hiperexcitabilidad. Estos mecanismos terminan con el evento ictal,
previenen la propagación de la actividad epiléptica y mantienen el estado interictal. Los factores
fisiológicos que influyen en la transición del estado interictal al ictal son la sincronización y la
excitabilidad neuronal. 2) El período postictal es aquel que se presenta inmediatamente después
de la crisis epiléptica y cuya duración varía desde unos minutos hasta días. El período postictal
sigue a la mayoría de las crisis convulsivas parciales y generalizadas. Durante este período se
presenta la depresión postictal, que se caracteriza por la inmovilidad corporal y representa el
umbral de una crisis epiléptica (refractariedad postictal). Además, se observan algunas
alteraciones conductuales tales como paresias (parálisis ligera o incompleta), automatismos, así
como amnesia anterógrada y analgesia, las cuales se asocian a la activación del sistema de los
opioides endógenos. Otro sistema involucrado en el período postictal es el Gabaérgicos. 3) El
período interictal es el transcurrido entre crisis y crisis. Se ha propuesto que los cambios
asociados con éste son consecuencia de mecanismos inhibitorios que se desarrollan para
disminuir la hiperexcitabilidad epiléptica e impedir la aparición de nuevas crisis. LA DOPAMINA
SE RELACIONA CON LA ENFERMEDAD DE PARKINSON.
EL El GLUTAMATO es un aminoácido que está implicado en la mayoría de las funciones normales del
GLUTAMATO. Sistema Nervioso Central (SNC), es el mayor mediador de señales excitatorias y de la plasticidad
del Sistema Nervioso, pero también puede ser altamente neurotóxico. Debido a las múltiples
acciones fisiológicas en las que interviene su concentración en el espacio extracelular no debe
sobrepasar ciertos límites, para ello la homeostasis de los sistemas glutamatérgicos
(metabolismo, mecanismos de liberación, receptores y transportadores) están finamente
regulados. El glutamato debe de estar presente en concentraciones correctas, en el momento y en
el lugar correcto. La excitotoxicidad es un término acuñado para describir las modalidades
degenerativas neuronales producidas por un exceso de glutamato en el espacio extracelular, este
aumento induce una sobreactivación de los receptores de glutamato, cuyo resultado es la muerte
celular, tanto neuronal como glial (atrocitos, oligodendrocitos y microglia). El glutamato en este
caso actúa como una neurotoxina, pudiendo representar una vía final común en afecciones
neurológicas. En afecciones agudas se produciría un fenómeno de excitotoxicidad aguda y en
afecciones neurológicas crónicas de excitotoxicidad lenta o crónica. La concentración de
glutamato en la hendidura sináptica depende de la cantidad de glutamato liberado, de la
velocidad a la que es liberado, y de la velocidad con que es eliminado de la hendidura sináptica.
La acumulación sináptica de cantidades elevadas de glutamato y su acción prolongada sobre los
receptores de glutamato postsinápticos podría deberse a: un aumento de la liberación por
episodios de sobreexcitación (epilepsia), destrucción tisular (traumatismos), o a la alteración de
los mecanismos de recaptación por fallo de las proteínas trasportadores de glutamato. EL
GLUTAMATO ES EL PRINCIPAL NEUROTRANSMISOR EXITADOR, SE ENCUENTRA LOCALIZADO
POR TODO EL SISTEMA NERVIOSO INCLUSO EN LAS CÉLULAS PIRAMIDALES CORTICALES.

Bibliografía:
1. LONGO DL, FAUCI AS, KASPER DL, HAUSERSL, JAMESON JL, LOSCALZOJ. HARRISON. PRINCIPIOS DE
MEDICINA INTERNA, 18A EDICIÓN. MC GRAW HILL. NEW YORK, USA. 2012, PP 94, 3227.
ANÁLISIS DEL CASO CLÍNICO

IDENTIFICACIÓN DEL REACTIVO


Area: MEDICINA INTERNA
Especialidad: ENDOCRINOLOGÍA
Tema: DIABETES MELLITUS
Subtema: DIABETES MELLITUS TIPO II

CASO CLÍNICO CON UNA PREGUNTA

DURANTE SU ROTACIÓN POR EL SERVICIO DE OFTALMOLOGÍA HA NOTADO UN INCREMENTO CONSIDERABLE


EN LOS PACIENTES CON DETERIORO EN LA VISIÓN.

-.

DETERIORO DE LA VISIÓN.

-.

-.

-.

385 - LO MÁS PROBABLE ES QUE LA MAYORÍA DE LOS CASOS SEA A CAUSA DE:

DEFECTOS La DIABETES MELLITUS (DM) es definida como un grupo de enfermedades metabólicas


CONGÉNITOS. caracterizada por defectos en la secreción de insulina, de su acción o ambas; y que tienen como
fenotipo la hiperglucemia.

TRAUMATISMOS. La Diabetes Mellitus (DM) ocupa en México el primer lugar como causa de defunción y por
desgracia las tendencias de morbi- mortalidad para esta enfermedad van en aumento.

GLAUCOMA. La historia natural de la DM incluye situaciones que comprometen el control de los pacientes y
condicionan la presentación de complicaciones agudas y crónicas. La DM se asocia con el
desarrollo de lesiones microvasculares (nefropatía, retinopatía y neuropatía) y macrovasculares
(cardiopatía isquémica, enfermedad vascular cerebral y enfermedad vascular periférica.
DIABETES. La DM es la primera causa de ceguera, de insuficiencia renal crónica y de amputación no
traumática de miembros inferiores; condiciona muertes prematuras e implica costos de
atención y hospitalización. La RETINOPATÍA DIABÉTICA es la presencia de lesione
microvasculares en la retina de una persona con diabetes. Se caracteriza por la presencia de
microaneurismas, hemorragias, exudados duros, manchas algodonosas, alteraciones
microvasculares, arrosaramiento venosos, neovasos y tejido fibroso; que aparecen como
complicación crónica de la diabetes. IMPORTANTE: la retinopatía diabética es la primera causa
de ceguera prevenible en la población económicamente activa.

Bibliografía:
1. GUÍA DE PRÁCTICA CLÍNICA, DIAGNÓSTICO, METAS DE CONTROL AMBULATORIO Y REFERENCIA
OPORTUNA DE PREDIABETES Y DIABETES MELLITUS TIPO 2 EN ADULTOS EN EL PRIMER NIVEL DE
ATENCIÓN. MÉXICO: SECRETARIA DE SALUD; 2012. 2. LONGO DL, FAUCI AS, KASPER DL, HAUSERSL,
JAMESON JL, LOSCALZOJ. HARRISON. PRINCIPIOS DE MEDICINA INTERNA, 18A EDICIÓN. MC GRAW HILL.
NEW YORK, USA. 2012. 3. PAPADAKIS MAXINE A, MCPHEE STEPHEN J. DIAGNÓSTICO CLÍNICO Y
TRATAMIENTO. 52ª EDICIÓN. MC GRAW HILL EDUCATION, LANGE. USA. 2013.

http://www.cenetec.salud.gob.mx/descargas/gpc/CatalogoMaestro/093_GPC_Diabmellitus2/SSA_093_08_EyR.pdf
ANÁLISIS DEL CASO CLÍNICO

IDENTIFICACIÓN DEL REACTIVO


Area: GINECOLOGÍA Y OBSTETRICIA
Especialidad: GINECOLOGÍA
Tema: CLIMATERIO Y MENOPAUSIA
Subtema: PROLAPSO VAGINAL Y CISTOCELE

CASO CLÍNICO SERIADO

MUJER DE 50 AÑOS DE EDAD QUE ACUDE A CONSULTA POR INCONTINENCIA FECAL OCASIONAL, SENSACIÓN
DE CUERPO EXTRAÑO VULVAR Y CONSTIPACIÓN HABITUAL. A LA EXPLORACIÓN GENITAL SE APRECIA PARED
VAGINAL ANTERIOR NORMAL Y REDUNDANCIA DE PARED VAGINAL POSTERIOR.

Mujer postmenopáusica.

incontinencia fecal ocasional, que nos


habla de compromiso rectal.

clave. sensación de cuerpo extraño en


vagina.

clave. redundancia de pared vaginal


posterior igual a rectocele.

-.

386 - EL DIAGNÓSTICO CLÍNICO MÁS PROBABLE ES:

CISTOCELE. El CISTOCELE es el prolapso de le vejiga urinaria a través de la PARED VAGINAL ANTERIOR. Los
factores de riesgo para el prolapso de la pared vaginal anterior (cistocele) e incontinencia urinaria de
esfuerzo incluyen: predisposición genética, multiparidad, “menopausia”, edad avanzada, cirugía
pélvica previa, ejercicio de alto impacto, alteraciones de los tejidos conectivos, asociados con
incremento en la presión abdominal (obesidad, constipación) y tosedoras crónicas. El único síntoma
específico para el cistocele es la sensación de cuerpo extraño a nivel vaginal. A la exploración física
bimanual se evidencia la protrusión de la pared vaginal anterior con la maniobra de Valsalva, está
indicado realizarla de piel en caso de no visualizar prolapso. EN EL CISTOCELE LA PARED VAGINAL EN
COMPROMISO ES LA ANTERIOR, Y NO LA POSTERIOR COMO SE MENCIONA EN EL CASO CLÍNICO.
PROLAPSO Es la CAÍDA O DESPLAZAMIENTO DEL ÚTERO desde su posición normal en la cavidad pélvica dentro
UTERINO. de la vagina. Datos clínicos: • Sensación de pesadez o tracción en la pelvis • Sensación de "estar
sentada en una bola pequeña" • Dolor de espalda bajo • Protrusión desde la abertura vaginal (en
casos que van de moderado a severo) • Relación sexual difícil o dolorosa A la exploración
esperaríamos encontrar: • Un examen pélvico (con la mujer haciendo esfuerzo) muestra protrusión
del cuello uterino dentro de la parte inferior de la vagina (prolapso leve), protrusión más allá del
introito o abertura vaginal (prolapso moderado) o protrusión de todo el útero más allá del introito o
abertura vaginal (prolapso severo). NO SE MENCIONA PROTRUSIÓN DE LA APERTURA CERVICAL
QUE JUSTIFIQUE QUE LA MASA SEA DE ESTE ORIGEN.

RECTOCELE. OJO. De acuerdo con el reporte del “Sub-comité de Estandarización de la Sociedad Internacional de
Continencia” (ICS), es el descenso de uno o más de los siguientes: pared vaginal anterior, pared
vaginal posterior, el ápex de la vagina (cérvix/útero) o la cúpula vaginal posterior a histerectomía. El
prolapso puede estatificarse del I al IV de acuerdo al sistema de cuantificación de prolapso de
órganos pélvicos (CPOP). El prolapso de órganos pélvico puede ocurrir con asociación de
incontinencia urinaria y otras disfunciones del tracto urinario inferior y en ocasiones puede
enmascarar la incontinencia. Podemos observar prolapsos de las paredes vaginales anteriores,
apicales y posteriores. Lo más habitual es encontrar una combinación de descenso en los tres
compartimientos. El nombre específico que se le da a cada representación clínica de los prolapsos
depende del órgano prolapsado por encima de la pared vaginal correspondiente. Así, podemos
observar colpouretrocele, colpocistocele, colpouretrocistocele, colporectocele, prolapso uterino y
colpoenterocele. EN ESTE CASO AL TRATARSE DE LA PARED VAGINAL POSTERIOR, SERÍA UN
RECTOCELE, LA CLAVE PARA REALIZAR EL DIAGNÓSTICO DIFERENCIAL LA DIO LA EXPLORACIÓN
FÍSICA. En todos los casos de prolapso de los órganos pélvicos, el síntoma clínico principal es la
sensación del cuerpo extraño en la vagina.

DISTOPIA RECUERDA: el descenso de la vejiga se llama cistocele; el de uretra uretrocele, y de la fascia pélvica se
GENITAL nombra como defecto paravaginal, en mayor o menor grado, a través de la pared vaginal anterior. La
ANTERIOR. distopia genital es sinónimo de prolapso de los organos pélvicos. Al referirse como anterior
estaríamos hablando de un cistocele. Como puedes ver dos respuestas son iguales pero utilizando un
distinto nombre.

Bibliografía:
1. GUÍA DE PRÁCTICA CLÍNICA, DIAGNÓSTICO Y TRATAMIENTO DEL PROLAPSO DE LA PARED VAGINAL
ANTERIOR CISTOCELE E INCONTINENCIA URINARIA DE ESFUERZO. MÉXICO: SECRETARIA DE SALUD; 2009.
2. SCHONGUE J, SCHAFER J, HALVORSON L, HOFFMAN B, BRADSHAW K, CUNNINGHAM G. WILLIAMS
GINECOLOGÍA, DE LA 1A EDICIÓN EN INGLÉS. MC GRAW HILL. USA. 2009, PP 534.

http://www.cenetec.salud.gob.mx /descargas/gpc/CatalogoMaestro/263_GPC_CISTOCELE/Cistocele_EVR_CENETEC_IUE_VERIF_MZO.pdf

387 - EL TRATAMIENTO MÁS RECOMENDABLE EN ESTE CASO SERÍA CON:

LAXANTES. Es controversial el uso de laxantes, si el rectocele se asocia con estreñimiento, podría ser parte del
manejo médico de la paciente. Sin embargo los cirujanos recomiendan otorgar medicamentos que
disminuyan el tránsito intestinal con el fin de reducir la estimulación rectal que propicie mayor
prolapso. En cualquiera de los casos, este manejo no sustituye el manejo quirúrgico.

PESARIO Los DEFECTOS DEL COMPONENTE ANTERIOR (cistocele, defectos vaginales; con o sin incontinencia
VAGINAL. urinaria) la reparación quirúrgica está dirigida al sitio afectado. El uso de un PESARIO es una
alternativa no quirúrgica que puede considerarse en pacientes con prolapso de la pared vaginal
anterior (CISTOCELE), sin importar el estadio.
EJERCICIOS Estos estarían indicados en los casos de CISTOCELE que se acompaña de incontinencia urinaria de
PERINEALES. esfuerzo. • Los ejercicios de Kegel se recomiendan como tratamiento no invasivo en mujeres con
incontinencia urinaria de esfuerzo. El principio de los ejercicios de Kegel es fortalecer los músculos
del piso pélvico y, en consecuencia, mejorar el funcionamiento del esfínter uretral para disminuir las
pérdidas de orina. Aparte de los beneficios relacionados con las pérdidas de orina, estos ejercicios
también se han recomendado para recuperar el tono de los músculos y de la vagina después del
parto, encontrándose además que su práctica habitual tiene efectos beneficiosos en la sexualidad. El
objetivo es contraer y relajar de manera repetida el músculo pubococcígeo. Ejercitar los músculos
del suelo de la pelvis durante cinco minutos tres veces al día puede significar una gran diferencia en
el control de la vejiga. El ejercicio fortalece los músculos que sostienen la vejiga y mantienen los
otros órganos de la pelvis en su lugar. LOS EJERCICIOS DE KEJEL ESTÁN INDICADOS EN CASOS DE
INCONTINENCIA URINARIA.

CIRUGÍA. Los DEFECTOS DEL COMPONENTE POSTERIOR del soporte pélvico (rectocele y anomalías del esfínter
anal), se puede tratar mediante manejo médico y quirúrgico. -Tratamiento médico: medicamentos
que disminuyan el tránsito intestinal, agentes de engrosamiento (volumétricos),
biorretroalimentación y terapéutica con estimulación eléctrica. - Tratamiento quirúrgico: contempla
la colporrafia para corregir los defectos de la fascia posterior, esta puede hacerse de forma primaria
o con la ayuda da injertos; también se puede realizar esfinteroplastía o reparación del prolapso
rectal. En pacientes sin actividad sexual puede tenerse como opción la colpocleisis (cierre vaginal), o
la colpectomía (resección vaginal). En casos específicamente de RECTOCELE se realizará
COLPOPERINEORRAFIA, con defectos moderados debe además realizarse plastía de la cuña perineal
con miorrafia de los músculos transversos superficiales del periné. LOS RECTOCELES REQUIEREN DE
TRATAMIENTO QUIRÚRGICO PARA SU RESOLUCIÓN COMPLETA.

Bibliografía:
1. SCHONGUE J, SCHAFER J, HALVORSON L, HOFFMAN B, BRADSHAW K, CUNNINGHAM G. WILLIAMS
GINECOLOGÍA, DE LA 1A EDICIÓN EN INGLÉS. MC GRAW HILL. USA. 2009, PP 551. 2. DOHERTY G.
DIAGNÓSTICO Y TRATAMIENTO QUIRÚRGICO, 13A EDICIÓN. MC GRAW HILL LANGE. 2011.

FIN DEL CASO CLÍNICO SERIADO


ANÁLISIS DEL CASO CLÍNICO

IDENTIFICACIÓN DEL REACTIVO


Area: MEDICINA INTERNA
Especialidad: CARDIOVASCULAR
Tema: CARDIOPATÍA ISQUÉMICA
Subtema: INFARTO AGUDO DEL MIOCARDIO

CASO CLÍNICO CON UNA PREGUNTA

MASCULINO DE 51 AÑOS QUE INGRESA A SALA DE CHOQUE CON DIAGNÓSTICO DE PROBABLE INFARTO
AGUDO AL MIOCARDIO.

MASCULINO DE 51 AÑOS.

DIAGNÓSTICO DE PROBABLE INFARTO


AGUDO AL MIOCARDIO.

-.

-.

-.

388 - EL PRIMER MARCADOR DE NECROSIS MIOCÁRDICA QUE USTED ESPERARÍA ENCONTRAR ELEVADO
ES:
FRACCIÓN MB DE LA En un plazo de 4 a 8 h, aumentan los niveles de fosfocinasa de creatinina (creatine
CREATINFOSFOCINASA. phosphokinase, CK), que se normalizarán entre las 48 y 72 h). Un inconveniente
importante de medir la CK total es que no señala de manera específica la presencia de
Síndrome Coronario Agudo con Elevación del Segmento ST (STEMI), dado que el nivel de
la enzima puede aumentar con el traumatismo de músculos de fibra estriada. • Después
de una inyección intramuscular, por ejemplo, aumenta dos o tres veces la cifra total de
CK; tal ambigüedad puede originar el diagnóstico erróneo de STEMI en las personas a
quienes se ha aplicado por vía intramuscular un narcótico contra el dolor retroesternal
de origen no cardíaco. • Otras causas posibles del incremento de CK total son: 1)
trastornos de músculos de fibra estriada, como distrofias musculares, miopatías y
polimiositis; 2) cardioversión eléctrica; 3) hipotiroidismo; 4) accidente apoplético; 5)
operaciones quirúrgicas, y 6) daño de músculo estriado a causa de traumatismo,
convulsiones e inmovilización duradera. • La isoenzima MB de la fosfocinasa de
creatinina tiene la ventaja, respecto a la CK total, de que no aparece en concentraciones
relevantes en tejido extracardíaco y es muchísimo más específica. LA CK Y SU FRACCIÓN
MB SE LIBERA A LAS 4 A 8HRS POSTERIORES AL IAM, NO ES LA PRIMERA ENZIMA EN
ELEVARSE.

MIOGLOBINA. • La mioglobina es liberada en la sangre unas horas después de haber comenzado el IAM.
• Dicho pigmento es uno de los primeros marcadores cardíacos que aumentan por
encima de los límites normales después del IAM, pero no tiene especificidad cardíaca y se
excreta rápidamente en la orina, de modo que los niveles en sangre retornan a sus
intervalos normales en las primeras 24 h de haber comenzado el infarto. • Es el
biomarcador más rápido (1 a 2 horas) para demostrar lesión celular aguda, su elevación
máxima se observa entre las 6 y 12 horas después del inicio de los síntomas y regresa a la
normalidad en las siguientes 24 horas. AUNQUE POCO ESPECÍFICA, LA MIOGLOBINA ES
DE LOS PRIMEROS MARCADORES EN CARDÍACOS EN APARECER EN SANGRE.

TROPONINA T. Es posible encontrar troponina sérica en muchas condiciones clínicas como insuficiencia
cardíaca, hipotensión asociada a arritmias, enfermos en estado crítico, miocarditis,
pericarditis, tromboembolia pulmonar, hipertensión arterial pulmonar crónica grave,
traumatismo cardíaco, estados avanzados de insuficiencia renal, etc. • Su principal valor
es la alta sensibilidad para identificar infartos pequeños, se eleva entre 4 a 10 horas
después del inicio de los síntomas, con elevación máxima a las 12 ó 48 horas y puede
persistir hasta por 21días. • Se recomiendan tres determinaciones, una basal en
urgencias, a las 6 y 12 horas. Cualquier valor elevado de troponina se relaciona con
mayor riesgo para eventos adversos cardiovasculares. LAS TROPONINAS SE ELEVAN
ENTRE LAS 4 Y 10 HRS POSTINFARTO, NO SON LOS PRIMEROS MARCADORES EN
APARECER.

TROPONINA I . RECUERDA: por la baja especificidad de la mioglobina se requieren determinaciones de


troponina simultáneas para confirmar daño miocárdico y eliminar falsos positivos. Y la
titulación de todas éstas preferentemente a las 4, 8 y 12hrs.

Bibliografía:
1. LONGO DL, FAUCI AS, KASPER DL, HAUSERSL, JAMESON JL, LOSCALZOJ. HARRISON. PRINCIPIOS DE
MEDICINA INTERNA, 18A EDICIÓN. MC GRAW HILL. NEW YORK, USA. 2012, PP 2030-2031.
ANÁLISIS DEL CASO CLÍNICO

IDENTIFICACIÓN DEL REACTIVO


Area: GINECOLOGÍA Y OBSTETRICIA
Especialidad: OBSTETRICIA
Tema: DIABETES GESTACIONAL
Subtema: DIABETES GESTACIONAL

CASO CLÍNICO SERIADO

PRIMIGESTA MEXICANA DE 33 AÑOS DE EDAD, CON ANTECEDENTE DE PADRE DIABÉTICO. SOLICITA


CONSULTA DE CONTROL PRENATAL POR AMENORREA DE 13 SDG POR FUM. ACTUALMENTE CON PESO
ADECUADO PARA LA TALLA Y EDAD GESTACIONAL. SE ABRE EXPEDIENTE CLÍNICO Y SE CITA LO ANTES
POSIBLE A CONSULTA CON USG TRANSVAGINAL, LABORATORIOS PRENATALES GENERALES Y PRUEBA DE
TAMIZ PARA DIABETES.

33 años (el riesgo aumenta en mayores a


30 años).

MEXICANA, familiar de primer grado con


diabetes.

--

--

PRUEBA DE TAMIZ PARA DIABETES.

389 - ES LA PRUEBA DE TAMIZ INDICADA PARA DESCARTAR DIABETES EN ESTA PACIENTE:

GLUCOSA Existen 4 formas de realizar el diagnóstico de DIABETES PREGESTACIONAL (DMG). 1. Glucemia


SÉRICA EN en ayuno mayor o igual a 126 mg/dL en dos ocasiones. 2. Glucemia plasmática al azar mayor o
AYUNO igual de 200 mg/dL. 3. Glucosa plasmática 2hrs poscarga oral de 75gr. 4. HbA1c mayor o igual a
6.5%. LA GLUCOSA SÉRICA EN AYUNO ES IDEAL PARA IDENTIFICAR DIABETES
PREGESTACIONAL, SE RECOMIENDA EN LA PRIMER CONSULTA PRENATAL O ANTES DE LAS
13SDG.
CARGA CON 50 El diagnóstico de Diabetes Gestacional (DMG) se puede realizar mediante dos modelos, en un
GRS DE paso o en dos pasos: 1. EN UN PASO: - Realizar CTGO (curva de tolerancia a la glucosa oral) con
GLUCOSA carga de 75gr. en mujeres previamente sin diagnóstico de DMG. La CTGO debe realizarse en la
mañana con un ayuno de 8 horas. 2. EN DOS PASOS: - Realizar tamizaje con 50 gr de glucosa (no
se requiere ayuno) con medición de la glucosa 1 hora poscarga, en embarazadas entre las 24 a 28
semanas de gestación. Si los niveles plasmáticos de glucosa a la hora son ? 140*mg/dl se procede
a CTGO. La CTGO debe realizarse en ayuno, con carga oral de 100 gr de glucosa y mediciones en
ayuno, 1, 2 y 3 horas poscarga. Se realizará el diagnóstico con 2 valores por arriba de los valores
plasmáticos de referencia (ayuno 95-105mg/dl, 1hr 180-190mg/dl, 2hrs 155-165mg/dl, 3hrs 140-
145mg/dl). - Realizar el tamiz o CTGO entre la semana 24 y 28 de gestación en mujeres de
moderado y alto riesgo que tuvieron resultado de glucosa de ayuno normal en la primera visita
prenatal. LA PRUEBA DE TAMIZAJE PARA DIABETES GESTACIONAL SE REALIZA CON UNA CARGA
DE 50MG/DL.

CURVA DE Aunque con anterioridad se utilizaban determinaciones con cargas de glucosa de 100 mg.,
TOLERANCIA A actualmente la OMS recomienda utilizar la curva de tolerancia a la glucosa con 75 grs. de glucosa
LA GLUCOSA y muestras a los 0, 60 y 120 minutos será diagnosticada cuando se utiliza como prueba en un
solo paso cuando sea: igual o mayor a 92mg/dl en ayuno, igual o mayor a 180mg/dl en 1hr
poscarga, igual o mayor a 153mg/dl en 2hrs poscarga. LA CURVA DE TOLERANCIA A LA
GLUCOSA ES UN MÉTODO DIAGNÓSTICO DE LA DIABETES GESTACIONAL, NO UNA PRUEBA DE
TAMIZAJE.

GLUCOSA La glucosa capilar postprandial no se considera una prueba de tamiz para ningún tipo de
CAPILAR diabetes, ni confirmatoria para el diagnóstico de la misma. EL USO DE LA GLUCOSA CAPILAR SE
POSTPRANDIAL LIMITA AL AUTOMONITOREO DE PACIENTES YA DIAGNOSTICADOS.

Bibliografía:
1. DIAGNÓSTICO Y TRATAMIENTO DE LA DIABETES EN EL EMBARAZO. EVIDENCIAS Y RECOMENDACIONES.
GUÍA DE PRÁCTICA CLÍNICA. MÉXICO: SECRETARÍA DE SALUD; 03/11/2016.

http://www.cenetec-difusion.com/CMGPC/IMSS-320-10/ER.pdf

390 - ES LA META TERAPÉUTICA DE GLUCOSA SÉRICA EN AYUNO QUE DEBE TENER LA PACIENTE EN
CASO DE RESULTAR CON DIAGNÓSTICO DE DIABETES:

60 A Las metas terapéuticas de glucosa sanguínea para pacientes con DIABETES GESTACIONAL son muy
95 estrictas y consisten en: - Glucosa en ayuno menor a 95mg/dl. - Glucosa 1hrs posprandia igual o menor a
MG/DL 140mg/dl. - Glucosa 2hrs posprandial igual o menor a 120mg/dl. - HbA1c igual o menor a 6.0% - Glucosa
antes de dormir y en la madrugada 60mg/dl. IMPORTANTE: dado que el manejo es muy riguroso, se
deberá tener especial atención ante el riesgo de hipoglucemia y restricción en el crecimiento intrauterino.

95 A Es un parámetro muy alto para pacientes con diabetes gestacional, debes tener en cuenta que las metas
110 deben ser más estrictas que en una paciente no gestante. Las metas terapéuticas de glucosa sanguínea
MG/DL para pacientes con DIABETES PREGESTACIONAL son: - Glucosa en ayuno menor a 90mg/dl. - Glucosa 1hrs
posprandia igual o menor a 100mg/dl. - Glucosa 2hrs posprandial igual o menor a 120mg/dl. - HbA1c
igual o menor a 6.0% - Glucosa antes de dormir y en la madrugada 60mg/dl.

110 A El término PREDIABETES se refiere a un estado metabólico con alteraciones en la sensibilidad a la insulina
126 y de la función de la célula beta pancreática, en el que las concentraciones séricas de glucosa oscilan entre
MG/DL los valores de referencia normales y elevados (100-125 mg/dl). Estos pacientes no llegan a tener cifras
diagnósticas de diabetes, pero sus parámetros son muy altos para considerarse normales.

126 A 126 a 140 mg/dl Toda mujer con niveles séricos por arriba 126 mg/dl en ayuno en dos ocasiones deberán
140 ser diagnosticadas como diabéticas. ESTAS CIFRAS SE ENCUENTRAN MUY POR ARRIBA DE LAS METAS
MG/DL DESEADAS PARA UNA PACIENTE EN CONTROL.

Bibliografía:
1. DIAGNÓSTICO Y TRATAMIENTO DE LA DIABETES EN EL EMBARAZO. EVIDENCIAS Y RECOMENDACIONES.
GUÍA DE PRÁCTICA CLÍNICA. MÉXICO: SECRETARÍA DE SALUD; 03/11/2016.

http://www.cenetec-difusion.com/CMGPC/IMSS-320-10/ER.pdf
FIN DEL CASO CLÍNICO SERIADO
ANÁLISIS DEL CASO CLÍNICO

IDENTIFICACIÓN DEL REACTIVO


Area: PEDIATRÍA
Especialidad: INFECTOLOGIA PEDIÁTRICA
Tema: BRONQUIOLITIS
Subtema: BRONQUIOLITIS

CASO CLÍNICO SERIADO

LACTANTE DE 8 MESES DE EDAD, CON DIAGNÓSTICO DE RESFRIADO COMÚN HACE 24HRS. ES LLEVADO AL
SERVICIO DE URGENCIAS POR DIFICULTAD RESPIRATORIA Y RECHAZO A LA VÍA ORAL. A LA EXPLORACIÓN
CON POLIPNEA, TIRAJE INTERCOSTAL Y SIBILANCIAS BILATERALES GENERALIZADAS. LA PLACA DE TÓRAX
MUESTRA ESPACIOS INTERCOSTALES ENSANCHADOS CON HORIZONTALIZACIÓN DE LOS ARCOS COSTALES,
DIAFRAGMA DESCENDIDO, APLANADO Y HORIZONTAL, HIPERCLARIDAD PULMONAR MARCADA.

8 meses.

Dx de resfriado común hace 24hrs

di cultad respiratoria y rechazo a la vía oral.

datos de di cultad respiratoria.

síndrome de rarefacción pulmonar.

391 - EL MANEJO INICIAL QUE DEBERÁ RECIBIR EL PACIENTE SERÁ CON:

INOTRÓPICOS Y INSUFICIENCIA CARDÍACA. El manejo habitual de la insuficiencia cardíaca a esta edad,


DIURÉTICOS requerirá del ingreso del paciente a una unidad de cuidados intensivos y dependerá
principalmente de la causa específica. El manejo básico incluirá un inotrópico, diurético y
vasodilatados cuando así lo amerite.
ANTIBIÓTICOS Y NEUMONÍAS. Dada la dificultad para diferenciar las neumonías víricas y las bacterianas,
BRONCODILATADORES incluso si se dispone de pruebas complementarias (analítica básica y radiología simple de
tórax), no siempre al alcance del médico de Atención Primaria, en la práctica, las
neumonías suelen tratarse con antibióticos. Muchas guías de práctica clínica recomiendan
no administrar antibióticos cuando se sospecha o se constata una neumonía en niños
pequeños (menores de 2 años) con síntomas respiratorios leves. Sin embargo: • El
neumococo, el micoplasma y las clamidias son frecuentes agentes de neumonía incluso
en las edades en que mayoritariamente es originada por virus. o Son frecuentes las
infecciones mixtas. o Cuanto más pequeño es el niño, más probable es que la neumonía
sea causada por virus, pero también es mayor el riesgo de no tratar con antibióticos una
neumonía de origen bacteriano.
NEBULIZACIONES Y LO PRIMERO QUE DEBES HACER ES IDENTIFICAR EL DIAGNÓSTICO QUE, PARA ESTE
BRONCODILATADORES CASO, ES UNA BRONQUIOLITIS. El diagnóstico de la BRONQUIOLITIS se basa
principalmente en los síntomas que se presentan en un lactante menor de 2 años y que
coincide con una epidemia de VRS en la comunidad. El cuadro comienza como un catarro
de vías altas con estornudo, tos, rinorrea y en ocasiones febrícula o fiebre; y en el
transcurso de 2 ó 3 días se intensifica la tos, aparecen los síntomas de obstrucción de la
vía respiratoria inferior con aumento del trabajo respiratorio, taquipnea e irritabilidad. A
la exploración física, podemos encontrar: taquipnea, aumento del trabajo respiratorio,
con utilización de músculos accesorios con retracciones costales (tiraje subcostal e
intercostal), y en la auscultación pulmonar podemos encontrar sibilantes espiratorios o
crepitantes finos inspiratorios bilaterales. En muchas ocasiones, los sibilantes pueden
escucharse con el oído desnudo sin ayuda del estetoscopio. No existe un patrón
radiológico típico en la bronquiolitis. Los hallazgos radiológicos más comunes son: el
atrapamiento aéreo. A pesar de que la bronquiolitis constituye la infección respiratoria
más frecuente durante la lactancia y de su elevada morbilidad, se ha avanzado muy poco
en su tratamiento, siendo la mayoría de éstos controvertidos y con escasa o nula
evidencia científica. Medidas de soporte - Mantener una adecuada hidratación por vía
oral, con ingesta de líquidos en tomas pequeñas y frecuentes; en aquellos pacientes con
bronquiolitis leve y que no tienen problemas para alimentarse. Los lactantes que
presentan dificultad respiratoria con taquipnea, tiraje, aleteo nasal, etc., tienen más
riesgo de aspiración bronquial y pueden necesitar fluidos intravenosos. • Corregir la
hipoxia administrando oxígeno suplementario a los pacientes con una saturación inferior
al 95%. • Antitérmicos si hay fiebre. • Posición semiincorporada en decúbito supino. •
Humidificación/nebulización templada: existen pocos estudios que evalúen el papel de
esta medida en el tratamiento de la bronquiolitis y ninguno de ellos ha podido demostrar
su beneficio. Además, el hecho de que son muy pocas las gotas de vapor que alcanzan la
vía respiratoria inferior y la posibilidad de efectos adversos desaconsejan su uso. •
Broncodilatadores inhalados: el uso de agentes broncodilatadores continúa siendo
controvertido. Estudios bien diseñados (ECA) no han podido demostrar, de manera
consistente, que dichos agentes produzcan un beneficio en el tratamiento de la
bronquiolitis. Aunque en algunos casos la bronquiolitis puede ser el preludio de asma, en
la mayoría de los casos el uso de terapias inhaladas y otros tratamientos eficaces para
tratar el broncoespasmo característico del asma no tendrá eficacia para tratar el edema
de la vía aérea típica de la bronquiolitis. • Beta 2 agonistas: en una revisión sistemática de
doce ECAs comparándolos con placebo, ocho no pudieron demostrar beneficios clínicos
después de su administración. Tres estudios demostraron beneficios clínicos, a corto
plazo (30 y 60 minutos), como mejores puntuaciones de los escores clínicos frecuencia
respiratoria, frecuencia cardíaca y saturación de oxígeno. En un estudio se observó un
empeoramiento en el grupo con tratamiento. Los beta 2 agonistas no reducen la tasa de
hospitalizaciones ni la estancia hospitalaria. No existe, por consiguiente, suficiente
evidencia para recomendar el uso sistemático de salbutamol inhalado A pesar de la falta
de datos que justifiquen el uso de broncodilatadores inhalados en el tratamiento de la
bronquiolitis, la experiencia clínica nos muestra que en algunos pacientes se observa una
mejoría clínica tras la administración de un broncodilatador; lo difícil es identificar, a
priori, qué pacientes se podrán beneficiar y cuáles no. Quizá los antecedentes personales
o familiares de atopia serían el único dato útil, aunque en ningún caso fiable. Por lo
tanto, la administración de una dosis de prueba con un broncodilatador y valorar la
respuesta clínica sería una opción aceptable; únicamente, en aquellos casos en los que se
obtenga respuesta clínica estaría indicado continuar con el tratamiento. Con respecto a
qué broncodilatador usar, se recomienda el salbutamol ya que la falta de estudios
consistentes y los posibles efectos secundarios hacen poco recomendable la adrenalina
nebulizada. EN LA PRÁCTICA HOSPITALARIA EN MÉXICO, ADEMÁS DE LA
SUPLEMENTACIÓN CON OXÍGENO, HUMIDIFICACIÓN Y NEBULIZACIONES SIMPLES,
ANTE LA PRESENCIA DE SIBILANCIAS, SIEMPRE SE AGREGA SALBUTAMOL AL MANEJO
DE LOS PACIENTES. POR ESTA RAZÓN SE CONSIDERA ESTA LA RESPUESTA ADECUADA.
NOTA: el resto de las opciones de respuesta te permitirán distinguir los principales
diagnósticos diferenciales para este caso y cómo se deben abordar desde el punto de
vista terapéutico.
AMBIENTE HÚMEDO LARINGITIS. En los procesos de laringitis leves, suelen ser procesos benignos que no
FRÍO Y ESTEROIDES requieren medidas terapéuticas. En las laringitis más graves sí existe consenso en el uso
INTRAMUSCULARES de corticoides y adrenalina nebulizada, aunque continúa el debate sobre la dosis y
posología de ambos fármacos. Recordar que la nebulización de sustancias en el
tratamiento de las laringitis agudas precisa de un flujo de aire de 4-6 litros por minuto,
para favorecer el depósito de las mismas en la laringe. Es importante destacar la
importancia de evitar en las laringitis situaciones que incrementan y facilitan la agitación
del niño, pues esto conduce a un empeoramiento de la sintomatología (estridor, tos
perruna). Se recomiendan medidas que tranquilicen (estar en brazos de los padres)
durante la exploración y tratamiento de esta patología.

Bibliografía:
1. NELSON TRATADO DE PEDIATRÍA. RICHARD BEHRMAN INTERAMERICANA. EDICIÓN 15A. 1997. PAG. 1521-
1523.

392 - CON BASE EN SU DIAGNÓSTICO, DEBE INDICARSELE A LA PACIENTE EL SIGUIENTE MEDICAMENTO


ESPECÍFICO:

RIBAVIRINA BRONQUIOLITIS. La ribavirina es un agente antiviral y su uso está limitado a pacientes con
bronquiolitis grave o que presentan algún factor de riesgo, inmunodeficiencias, prematuridad,
enfermedad cardíaca o pulmonar previa, etc. ACTUALMENTE SE MANTIENE MUY LIMITADO SU
USO, EN MÉXICO PRÁCTICAMENTE NO SE UTILIZA, SIN EMBARGO, EN LA LITERATURA
INTERNACIONAL LO SIGUEN CONSIDERANDO UNA OPCIÓN TERAPÉUTICA. OJO: la pregunta
indica el tratamiento “más específico”, lo cual coincide con esta respuesta dado que la etiología
es viral.

DIGOXINA INSUFICIENCIA CARDÍACA. La digoxina pertenece al grupo de los glucósidos cardíacos,


sustancias con amplia distribución en el reino animal donde habitualmente actúan como
toxinas, de entre los cuales es el más utilizado en la práctica clínica. Hoy en día la digoxina es
uno de los fármacos más ampliamente usados en la esfera cardiovascular con especial
protagonismo en la insuficiencia cardíaca y en las taquiarritmias de origen auricular. La digital
es un inhibidor potente y selectivo de la subunidad alfa de la ATPasa que posibilita el
transporte de Sodio-Potasio (ATPasa Na+/K+) a través de las membranas celulares. Esta unión
es de características reversibles. Los efectos más importantes de dicha inhibición de la ATPasa
Na+/K+ sobre el sistema cardiovascular son: • Efecto inotrópico positivo: la inhibición de la
ATPasa Na+/K+ a nivel de la célula cardíaca provoca un aumento del sodio intracelular, que a
su vez se intercambia por calcio extracelular con un efecto global de aumento de la presencia
de calcio intracelular. Este aumento del calcio disponible intracelular provoca un aumento de la
velocidad y grado de acortamiento de los sarcómeros a nivel cardíaco con el consiguiente
aumento de la contractilidad global del músculo cardíaco. Hay que tener en cuenta que este
aumento de la contractilidad, y por lo tanto del gasto cardíaco, no se produce a expensas de un
aumento importante del consumo de oxígeno por parte del músculo cardíaco. • Efecto
cronotrópico negativo: la digital actúa también sobre el sistema nervioso autónomo
aumentando la estimulación parasimpática e inhibiendo la simpática a nivel del corazón.
Además, sobre el propio sistema de conducción disminuye el automatismo y aumenta los
periodos refractarios, disminuyendo por lo tanto la frecuencia sinusal y retrasando la
conducción a través del nodo AV.

DEXAMETASONA LARINGITIS. Corticoides. La eficacia de los corticoides en el tratamiento de las laringitis en sus
diferentes modalidades de administración (oral, intramuscular o nebulizada) ha sido
demostrada en diferentes estudios comparados con el uso del placebo: • Disminuye la
intensidad de la clínica del paciente. • Disminuye el número de visitas a urgencias/ingresos
hospitalarios. • Disminuye el uso de adrenalina nebulizada. No se ha demostrado una eficacia
terapéutica superior en la asociación de corticoides orales/intramusculares, junto a inhalados.
Corticoide sistémico. La dexametasona en dosis única es la pauta de elección, por eficacia y
seguridad conocida.
CEFTRIAXONA NEUMONÍA. El siguiente esquema está indicado en neumonía comunitaria sin complicación:
Menores de 3 años. Manejo ambulatorio: Amoxicilina-clavulanato. Manejo hospitalario:
Penicilina, Ceftriaxona o cefotaxima o Amoxicilina-clavulanato Mayores de 3 años. Manejo
ambulatorio: Ceftriaxona o macrólidos. Alternativa Amoxicilina clavulanato o cefuroxima.
Manejo hospitalario: Penicilina, Ceftriaxona o cefotaxima o Amoxicilina-clavulanato

Bibliografía:
NELSON TRATADO DE PEDIATRÍA. RICHARD BEHRMAN. INTERAMERICANA. EDICIÓN 15A. 1997. PAG. 1521-
1523.

FIN DEL CASO CLÍNICO SERIADO


ANÁLISIS DEL CASO CLÍNICO

IDENTIFICACIÓN DEL REACTIVO


Area: MEDICINA INTERNA
Especialidad: GASTROENTEROLOGÍA
Tema: ENFERMEDADES DEL INTESTINO DELGADO Y GRUESO
Subtema: INTESTINO IRRITABLE

CASO CLÍNICO CON UNA PREGUNTA

FEMENINA DE 35 AÑOS, REFERIDA DE SU COMUNIDAD CON DIAGNÓSTICO DE PROBABLE COLON IRRITABLE.

femenino de 35 años de edad.

probablemente sÍndrome de colon irritable.

-.

-.

-.

393 - HALLAZGO CLÍNICO INDISPENSABLE PARA CONFIRMAR EL DIAGNÓSTICO:

AUSENCIA DE Solo entre 25 y 50% de los pacientes con IBS se quejan de dispepsia, pirosis, náuseas y vómitos,
PIROSIS, por lo que aunque estos síntomas pueden estar presentes en esta enfermedad no son el hallazgo
DISPEPSIA, clínico principal.
NÁUSEAS Y
VÓMITO
DOLOR El SÍNDROME DE INTESTINO IRRITABLE (SII) se define como una afección crónica de dolor o
ABDOMINAL molestia abdominal asociado a alteraciones en el hábito intestinal, durante al menos 3 días por
CÓLICO mes en los últimos 3 meses, sin causa orgánica que lo justifique. De acuerdo al patrón en los
EPISÓDICO hábitos defecatorios, el síndrome de intestino irritable (SII) se clasifica en tres subtipos: 1. SII con
constipación. Heces duras más del 25% de las veces y deposiciones disminuidas de consistencia
menos de 25% de las veces). 2. SII con diarrea. Deposiciones disminuidas de consistencia más del
25% de las veces y heces duras menos del 25% de las veces. 3. SII mixto. Heces duras más del 25%
de las veces y deposiciones disminuidas de consistencia más del 25% de las veces). El DOLOR
ABDOMINAL suele describirse como sensación de calambres con intensidad y localización
variables, asociado a periodos de exacerbación. La intensidad del dolor puede variar desde ligero
a severo. Asimismo, factores como el estrés emocional y la alimentación pueden exacerbarlo,
mientras que la defecación a menudo lo alivia. POR DEFINICIÓN EN SÍNDROME DE INTESTINO
IRRITABLE ES UN CUADRO CRÓNICO DE DOLOR O MOLESTIA ABDOMINAL QUE SE ACOMPAÑA
DE OTROS SÍNTOMAS INTESTINALES.

AUSENCIA DE A veces, la heces se acompaña de grandes cantidades de moco. No obstante no hay inflamación.
MOCO Y El sangrado no es característico del síndrome de colon irritable a menos que haya hemorroides, y
SANGRE EN no ocurren malabsorción ni pérdida de peso.
LAS HECES

SÍNTOMAS El dolor suele ser exacerbado por las comidas o el estrés emocional y aliviado por la expulsión de
QUE SE gases o excremento.
INCREMENTAN
CON LA
DEFECACIÓN

Bibliografía:
1. GUÍA DE PRÁCTICA CLÍNICA, DIAGNÓSTICO Y TRATAMIENTO DEL SÍNDROME DE COLON IRRITABLE.
MÉXICO: SECRETARIA DE SALUD; MARZO 2015.

http://www.cenetec.salud.gob.mx/descargas/gpc/CatalogoMaestro/042_GPC_ColonIrritable/IMSS_042_08_EyR.pdf
ANÁLISIS DEL CASO CLÍNICO

IDENTIFICACIÓN DEL REACTIVO


Area: CIRUGÍA
Especialidad: PROCTOLOGÍA Y UROLOGÍA
Tema: PATOLOGÍA DE COLÓN, ANO Y RECTO
Subtema: FISTULA PERIANAL

CASO CLÍNICO CON UNA PREGUNTA

HOMBRE DE 38 AÑOS DE EDAD, QUE ACUDE A SU CONSULTA REFERIDO DEL CENTRO DE SALUD CON
DIAGNÓSTICO DE QUISTE PILONIDAL. A LA EXPLORACIÓN LO ENCUENTRA EN FASE AGUDA, INFECTADO Y
ABSCEDADO.

38 AÑOS PACIENTE JOVEN.

MASCULINO, MAS COMÚNMENTE SE


PRESENTA EN HOMBRES JÓVENES.

-.

-.

-.

394 - EN ESTE PACIENTE EL TRATAMIENTO INDICADO ES:

PUNCIÓN Y EL MANEJO PRIMARIO DE ABSCESO PILONIDAL ESTABLECIDO CONSISTE EN EL DRENAJE


ASPIRACIÓN DE INMEDIATO. El QUISTE PILONIDAL se presenta por lo general como un absceso en la región
CONTENIDO. sacrococcígea, el aspirado del contenido del quiste pilonidal generará recidiva inmediata,
aún en caso de drenaje, excisión y cierre primario el índice de recidiva va del 22-39% de
acuerdo a la serie consultada. La aspiración y el uso de antibióticoterapia empírica puede
resolver de manera satisfactoria un absceso pilonidal agudo en el 95% de los casos en los
que está indicado y permitir un procedimiento electivo subsecuente. Los criterios de
inclusión para realizar el tratamiento por aspiración y posteriormente el tratamiento
electivo son pacientes con absceso pilonidal agudo, sin datos de sepsis, no diabéticos o
inmunocomprometidos y sin cambios necróticos de la piel. LA SIMPLE ASPIRACIÓN DEL
MATERIAL PURULENTO NO ES UN TRATAMIENTO ADECUADO, SE REQUIERE UNA
INCISIÓN DE DRENAJE SUFICIENTEMENTE AMPLIA Y LA ADMINISTRACIÓN DE
ANTIBIÓTICOS DE FORMA INDIVIDUALIZADA.
ADMINISTRACIÓN "El uso de antibióticos NO es suficiente" para el control de un quiste pilonidal infectado, las
DE ANTIBIÓTICOS concentraciones tisulares de los mismos en el medio de un absceso son pequeñas y la
SISTÉMICOS. mayor parte de las veces no generan la actividad antimicrobiana necesaria, el tratamiento
del absceso es el drenaje.

LAVADO CON Como se ha mencionado al abordar el tema de enfermedad pilonidal el tratamiento


ANTISÉPTICOS Y definitivo de la misma es la "excisión del seno" y tejido afectado, en la fase aguda el
EMPAQUETAMIENTO "drenaje del absceso" es lo indicado permitiendo resolver el proceso infeccioso agudo, el
CON GASAS CON empaquetamiento de la región con gasas con isodine no es una alternativa para el manejo
ISODINE. de la fase aguda.

DRENAJE ABIERTO Y El paciente frecuentemente se presenta con aumento de volumen y temperatura en la


MARSUPIALIZACIÓN. región sacrococcígea media, con dolor, fluctuación en ocasiones descarga purulenta
espontánea. El tratamiento se divide en dos fases: 1. En la fase aguda el tratamiento esta
orientado al "drenaje del absceso", se puede realizar con un procedimiento ambulatorio
con anestesia local, y en caso necesario uso de antibióticos. El drenaje del mismo se realiza
con una incisión sobre la porción de mayor fluctuación realizando aseo de la cavidad del
absceso y marsupializando los bordes con el objetivo de que el absceso no vuelva a
techarse y recidivar. 2. Una vez resuelta la fase aguda existen dos corrientes: - Vigilancia y
en caso de recidiva tratamiento quirúrgico. - Tratamiento quirúrgico desde el primer
evento. Existen múltiples modalidades de abordaje, por lo general incluyen excisión de
seno pilonidal completo, hasta la fascia presacra y legrado del tejido de granulación, la
herida puede dejarse abierta y esperar cierre por segunda intención con una tasa de
recidiva del 2-3% con el inconveniente de un tiempo de recuperación de 4-8 semanas, en el
caso de cierre primario la recuperación es más rápida en 3-5 semanas; sin embargo, el
porcentaje de recidiva reportado es de 11-29%. Existen tratamientos que por su
complejidad no se utilizan frecuentemente, en los que se incluyen aquellos en los que se
requiere un cirujano plástico para rotar colgajos para cubrir la zona de excisión. EL
TRATAMIENTO DE ELECCIÓN PARA EL ABSCESO PILONIDAL DEBE INICIAR CON DRENAJE Y
ANTIBIÓTICOS PARA DESPUÉS REALIZAR LA MARSUPIALIZACIÓN.

Bibliografía:
1. GUÍA DE PRÁCTICA CLÍNICA, DIAGNÓSTICO Y TRATAMIENTO DE LA ENFERMEDAD PILONIDAL EN
PACIENTES ADULTOS EN LOS TRES NIVELES DE ATENCIÓN. MÉXICO: SECRETARIA DE SALUD; 27 JUNIO
2013. 2. GUÍA DE REFERENCIA RÁPIDA, DIAGNÓSTICO Y TRATAMIENTO DE LA ENFERMEDAD PILONIDAL
EN PACIENTES ADULTOS EN LOS TRES NIVELES DE ATENCIÓN. MÉXICO: SECRETARIA DE SALUD; 27 JUNIO
2013. 3. BRUNICARDI F, ANDERSEN D, BILLIAR T, Y COLS. SCHWARTZ PRINCIPIOS DE CIRUGÍA, 9A EDICIÓN.
MC GRAW HILL. 2011, PP 1067-1068.

http://www.cenetec.salud.gob.mx/descargas/gpc/CatalogoMaestro/issste_662_13_enfpilonidal/issste_662_13_gpc_er_enfermedad_pilonidal.pdf
ANÁLISIS DEL CASO CLÍNICO

IDENTIFICACIÓN DEL REACTIVO


Area: MEDICINA INTERNA
Especialidad: NEUROLOGÍA
Tema: ENFERMEDADES CEREBROVASCULARES
Subtema: SECUELAS Y REHABILITACIÓN EVC

CASO CLÍNICO CON UNA PREGUNTA

HOMBRE DE 48 AÑOS DE EDAD, DESDE HACE 6 MESES EN REHABILITACIÓN POR SECUELAS SECUNDARIAS A
EVC ISQUÉMICO. ACUDE A CITA DE TERAPIA PARA EVALUACIÓN DE RESULTADOS Y REPLANTEAMIENTO DE
OBJETIVOS. A LA EXPLORACIÓN FÍSICA CON CAPACIDAD PARA LA MARCHA DE MANERA INDEPENDIENTE, SIN
AYUDA DE OTRA PERSONA, PERO CON USO DE BASTÓN. PUEDE LEVANTARSE SOLO DE LA SILLA Y LA CAMA.
ES CAPAZ DE IR AL BAÑO, VESTIRSE Y ASEARSE SOLO, PERO SOLICITA AYUDA PARA RASURARSE Y COCINAR.
AL SUBIR Y BAJAR ESCALERAS, REQUIERE DEL SOPORTE DE OTRA PERSONA, SOBRE TODO CUANDO NO
TIENEN PASAMANOS.

48 años.

secuelas secundarias a EVC isquémico.

marcha solo con bastón, puede moverse, ir


al baño y vestirse; necesita ayuda para
rasurarse, cocinar y subir escaleras

395 - EL GRADO DE DISCAPACIDAD DEL PACIENTE DEBERÁ SER CLASIFICADO COMO:

MUY LEVE La ESCALA DE RANKIN MODIFICADA permite de una forma fácil clasificar a los pacientes con
secuelas de EVC con base en su grado de discapacidad, incluye 7 niveles que van del cero al seis.
NIVEL 0. ASINTOMÁTICO. NIVEL 1. MUY LEVE: puede realizar tareas y actividades habituales “sin
limitaciones”. EL PACIENTE SI PRESENTA LIMITACIONES, POR TAL MOTIVO NO PUEDE CLASIFICARSE
COMO DISCAPACIDAD MUY LEVE.
LEVE NIVEL 2. LEVE: incapacidad para realizar algunas actividades previas, pero pueden valerse por sí
mismos, sin necesidad de ayuda. EL PACIENTE AÚN REQUIERE AYUDA PARA ALGUNAS DE SUS
ACTIVIDADES (SUBIR ESCALERAS), RAZÓN POR LA QUE NO PUEDE CONSIDERARSE DISCAPACIDAD
LEVE.

MODERADA NIVEL 3. MODERADA: requiere de algo de ayuda, pero pueden caminar solos. EL PACIENTE TIENE
NECESIDAD DE AYUDA PARA ALGUNAS ACTIVIDADES EN ESPECIAL, PERO EN GENERAL PUEDE
CAMINAR SOLO Y REALIZAR LA MAYORÍA DE LAS TAREAS.

GRAVE NIVEL 4. MODERADAMENTE GRAVE: dependientes de actividades básicas de la vida diaria, pero sin
necesidad de supervisión continua (necesidades personales sin ayuda). EL PACIENTE REALIZA LA
MAYOR PARTE DE LAS ACTIVIDADES DE LA VIDA DIARIA SOLO Y SIN SUPERVISIÓN (VESTIRSE,
COMER, IR AL BAÑO), POR ESO NO SE CLASIFICA AQUÍ. NIVEL 5. GRAVE: totalmente dependientes,
requieren asistencia continua. NIVEL 6. MUERTE.

Bibliografía:

http://www.cenetec-difusion.com/CMGPC/IMSS-114-08/ER.pdf
ANÁLISIS DEL CASO CLÍNICO

IDENTIFICACIÓN DEL REACTIVO


Area: CIRUGÍA
Especialidad: CIRUGÍA ABDOMINAL
Tema: PATOLOGÍA ABDOMINAL RESOLUCIÓN QUIRÚRGICA DE URGENCIA
Subtema: APENDICITIS AGUDA

CASO CLÍNICO CON UNA PREGUNTA

HOMBRE DE 32 AÑOS DE EDAD QUE INGRESA AL SERVICIO DE URGENCIAS CON DIAGNÓSTICO DE ABDOMEN
AGUDO. DURANTE SU VALORACIÓN SE CONCLUYE QUE SE TRATA DE UN CUADRO DE APENDICITIS AGUDA
POR LO QUE SE DECIDE EL TRATAMIENTO QUIRÚRGICO.

32 años de edad.

-.

-.

ABDOMEN AGUDO.

APENDICITIS AGUDA.

396 - DURANTE EL TRANSOPERATORIO USTED ENCUENTRA UN APÉNDICE NORMAL LA CONDUCTA A


SEGUIR SERÁ:

CONCLUIR LA Se ha reconocido por largo tiempo que es adecuado que todo cirujano realice laparotomías
CIRUGÍA. exploradoras con sospecha de apendicitis en las que el apéndice finalmente se identifique
macroscópicamente sin patología, las series anteriores reportaban índices de laparotomía
blanca hasta del 20%; sin embargo, con el advenimiento de los estudios de laboratorio y
gabinete actuales se dice que el porcentaje de laparotomías blancas puede variar entre el 4-
8%. Lo anterior se explica porque no hay método diagnostico 100% sensible y específico, por
lo tanto si toda la cirugía que ingresa a quirófano con diagnóstico de probable apendicitis se
confirma apendicitis en el estudio histopatológico, es muy probable que algunos casos en los
que nuestra conclusión diagnóstica se descartó apendicitis, finalmente terminó siendo
patología apendicular y ameritó manejo quirúrgico.
REVISIÓN DE En caso de identificar un apéndice macroscópicamente normal, el cirujano deberá realizar una
CAVIDAD Y inspección general de la cavidad abdominal, si la cirugía se realiza desde el punto de vista
APENDICECTOMÍA laparoscópico la revisión es más sencilla, sin fue a través una cirugía abierta, deberá por lo
INCIDENTAL. menos explorar el hueco pélvico principalmente en mujeres para descartar patología
ginecoobstétrica, útero y anexos, y deberá revisarse la porción del íleon terminal para
descartar la posibilidad de divertículo de Meckel que es una de los diagnósticos diferenciales.
A pesar de resultados controvertidos, las guías práctica clínica de nuestro país y la mayoría de
los cirujanos recomienda realizar apendicectomía a pesar de aspecto macroscópicamente
normal las razones son: Un bajo índice de complicaciones en los pacientes con apendicectomía
incidental. En caso de volver a presentar dolor en fosa iliaca derecha, la posibilidad de
apendicitis se ha descartado. En algunos estudios hasta en el 26% de los casos en donde el
cirujano describió un apéndice macroscópicamente normal había datos de apendicitis
incipiente. SIEMPRE QUE SE REALIZA UNA CIRUGÍA POR SOSPECHA DE PATOLOGÍA
APENDICULAR SIN DATOS PATOLÓGICOS EN EL TRANSOPERATORIO SE ESTÁ OBLIGADO A
REALIZAR REVISIÓN DE CAVIDAD. LA APENDICECTOMÍA INCIDENTAL ESTÁ JUSTIFICADA POR
LA ALTA INCIDENCIA DE APENDICITIS AGUDA EN LA POBLACIÓN GENERAL.

LAVADO DE EL LAVADO DE CAVIDAD ESTÁ INDICADO EN CASOS DE PERITONITIS GENERALIZADAS CON


CAVIDAD. OBJETIVO DE DISMINUIR LA CARGA BACTERIANA el American College of Surgeons
recomienda el lavado de cavidad de forma sistemática por cuadrantes siguiendo las manecillas
del reloj, iniciando por el cuadrante superior derecho, con solución fisiológica a 36 grados
para evitar hipotermia con entre 8 y12 L con lo que se logra disminuir la posibilidad de
absceso residual. NO SE JUSTIFICA EL LAVADO PERITONEAL EN ESTE CASO.

APENDICECTOMÍA Ya hemos comentado que la apendicectomía incidental se recomienda por las razones antes
INCIDENTAL SIN explicadas, en caso de no identificar patología apendicular, realizar apendicectomía sin
REVISIÓN DE realizar revisión de cavidad podría provocar una evolución tórpida al no identificar otra
CAVIDAD. patología que podría tener resolución quirúrgica. EN CASOS COMO ESTE SIEMPRE SE DEBE
REALIZAR REVISIÓN DE CAVIDAD.

Bibliografía:
1. BRUNICARDI F, ANDERSEN D, BILLIAR T, Y COLS. SCHWARTZ PRINCIPIOS DE CIRUGÍA, 9A EDICIÓN. MC
GRAW HILL. 2011, PP 1087-1088.
ANÁLISIS DEL CASO CLÍNICO

IDENTIFICACIÓN DEL REACTIVO


Area: MEDICINA INTERNA
Especialidad: INFECTOLOGÍA
Tema: INFECCIONES GASTROINTESTINALES
Subtema: AMEBIASIS INTESTINAL

CASO CLÍNICO CON UNA PREGUNTA

CAMPESINO DE 43 AÑOS DE EDAD, CON ANTECEDENTE DE ALCOHOLISMO CRÓNICO. REFIERE DESDE HACE
UNA SEMANA HIPERTERMIA CUANTIFICADA POR ARRIBA DE 38.5°C DE PREDOMINIO VESPERTINO, DOLOR
ABDOMINAL EN FLANCO DERECHO Y DIARREA SÓLO EN 1 O 2 OCASIONES. A LA EXPLORACIÓN FÍSICA
TEMPERATURA DE 39°C, DISNEICO, ICTERICIA GENERALIZADA ++, ABDOMEN GLOBOSO CON HEPATOMEGALIA
DOLOROSA, CON BORDE A 5CM POR DEBAJO DEL REBORDE COSTAL. LOS ESTUDIOS DE LABORATORIO
REVELAN ANEMIA NORMOCÍTICA NORMOCRÓMICA, LEUCOCITOSIS, ENZIMAS HEPÁTICAS ELEVADAS Y
ELEVACIÓN DE LA FOSFATASA ALCALINA. LA RADIOGRAFÍA DE TÓRAX MUESTRA ELEVACIÓN DEL
HEMIDIAFRAGMA DERECHO.

43 años.

alcoholismo crónico.

UNA semana con ebre mayor a 38.5°C


vespertina, dolor abdominal en aNco
derecho, diarrea escasa.

ebre, ictericia ++, hepatomegalia dolorosa.

anemia normocítica normocrómica,


leucocitosis, enzimas hepáticas elevadas,
fosfatasa alcalina elevada, Rx tórax con
elevación de hemidiafragma derecho.

397 - COMO PARTE DEL PROTOCOLO DE ESTUDIO DEL PACIENTE DEBERÁ SOLICITARSE EN ESTE
MOMENTO:
TOMOGRAFÍA El ABSCESO HEPÁTICO AMEBIANO es una necrosis enzimática de los hepatocitos y múltiples
AXIAL microabscesos que confluyen, formando habitualmente una cavidad única, conteniendo líquido
COMPUTADA homogéneo que puede variar en color, desde la crema blanca hasta el de ”pasta de anchoas”.
CUADRO CLÍNICO: - Presentación aguda, con una o dos semanas de fiebre (38.5 a 39.5º C) de
predominio vespertino y nocturno, acompañada de escalofríos y diaforesis. - La diarrea, en forma
recurrente, está presente en menos de un tercio de los pacientes. - Disentería en los meses
anteriores. - Ocasionalmente, los pacientes tienen una presentación más crónica con meses de
fiebre, pérdida de peso y dolor abdominal. - La fiebre es de inicio abrupto de 38 a 40º C
vespertina. - La hepatomegalia dolorosa debajo de las costillas o en los espacios intercostales es
un hallazgo típico. LA TOMOGRAFÍA NO SERÍA EL ESTUDIO DE PRIMERA ELECCIÓN EN ESTE
CASO, DEBERÁ REALIZARSE PREVIO ULTRASONIDO HEPÁTICO.

RADIOGRAFÍA DIAGNÓSTICO: La presencia de factores de riesgo y una lesión sospechosa, son suficientes para
DE ABDOMEN pensar que existe una infección amebiana, al menos, a la espera de resultados de otras pruebas.
LA RADIOGRAFÍA DE ABDOMEN NO ES ESPECÍFICA PARA ESTE CASO.

ULTRASONIDO LO PRIMERO EN ESTE CASO ES DEFINIR EL DIAGNÓSTICO PROBABLE. - EL PACIENTE TIENE


HEPÁTICO FACTORES DE RIESGO PARA ABSCESO HEPÁTICO AMEBIANO (EDAD, ALCOHOLISMO). - EL
TIEMPO DE EVOLUCIÓN QUE TIENE CON FIEBRE DESCARTA UNA HEPATITIS VIRAL AGUDA. -
DEBES PONER ATENCIÓN AL SIGNO DE DISNEA Y LA ELEVACIÓN DEL HEMIDIAFRAGMA
DERECHO PARA FUNDAMENTAR TU DIAGNÓSTICO. - RECUERDA QUE LA CAUSA MÁS
FRECUENTE DE ABSCESOS HEPÁTICOS EN NUESTRO MEDIO ES EL DADO POR LA AMEBIASIS
EXTRAINTESTINAL. ESTUDIOS DE IMAGEN: Se deben solicitar en pacientes con cuadro clínico
sugestivo, radiografía de tórax, ultrasonido hepático y de acuerdo con lo encontrado en el
ultrasonido solicitar tomografía de abdomen. - El ULTRASONIDO es un estudio de bajo costo para
detectar abscesos medianos o grandes, es útil para su punción guiada. - La tomografía es útil para
detectar abscesos pequeños. - Otros hallazgos inespecíficos comunes, incluyen una radiografía de
tórax anormal con elevación del hemidiafragma y/o derrame pleural derecho, así como
proteinuria. UNA VEZ QUE SE SOSPECHA DE UN ABSCESO HEPÁTICO AMEBIANO, DEBERÁ
INDICARSE EL ULTRASONIDO COMO AUXILIAR DIAGNÓSTICO DE IMAGEN. OJO: De ninguna
manera el ultrasonido puede fundamentar el diagnóstico final, ya que en algunos casos puede
tratarse de enfermedades neoplásicas del hígado, pero sí es el estudio de imagen de elección de
primera línea para este padecimiento.

PANEL VIRAL ESTUDIOS DE LABORATORIO: - Los pacientes con absceso hepático amebiano generalmente
DE HEPATITIS tienen leucocitosis (más de 10.000/mm3), sin eosinofilia. Se presenta anemia hasta en un 30% de
los casos. - Las pruebas de función hepática revelan una fosfatasa alcalina elevada en 80% de los
casos y las transaminasas hepáticas también pueden elevarse en una tercera parte de los casos. EL
PANEL VIRAL QUEDA DESCARTADO, PUES NO ES NECESARIO ANTE LA SOSPECHA DE UN
ABSCESO HEPÁTICO AMEBIANO.

Bibliografía:
1. GUÍA DE PRÁCTICA CLÍNICA, DIAGNÓSTICO Y DEL ABSCESO HEPÁTICO AMEBIANO NO COMPLICADO.
MÉXICO: SECRETARIA DE SALUD; 2014. 2. LONGO DL, FAUCI AS, KASPER DL, HAUSERSL, JAMESON JL,
LOSCALZOJ. HARRISON. PRINCIPIOS DE MEDICINA INTERNA, 18A EDICIÓN. MC GRAW HILL. NEW YORK,
USA. 2012.

http://www.cenetec.salud.gob.mx/descargas/gpc/CatalogoMaestro/432_GPC__Absceso_hepatico/GER_Absceso_hepxtico_amebiano.pdf
ANÁLISIS DEL CASO CLÍNICO

IDENTIFICACIÓN DEL REACTIVO


Area: MEDICINA INTERNA
Especialidad: HEMATOLOGÍA
Tema: TRANSTORNOS DE LA HEMOSTASIA
Subtema: HEMOFILIA A Y B

CASO CLÍNICO SERIADO

MASCULINO DE 18 AÑOS DE EDAD, CON DIAGNÓSTICO DE HEMOFILIA. HACE UNA SEMANA, MIENTRAS
ACUDÍA A UN CONCIERTO, SE VIO INVOLUCRADO EN UNA ESTAMPIDA, CON LO QUE SUFRIÓ MÚLTIPLES
CONTUSIONES, QUE RESULTARON EN GRAVES HEMATOMAS, HEMORRAGIA CEREBRAL Y AL MENOS DOS
HEMARTROSIS. DURANTE EL PASE DE VISITA NOCTURNO EL FAMILIAR REFIERE QUE, A ÚLTIMAS FECHAS, EL
SERVICIO DE HEMATOLOGÍA LO HA CATALOGADO CON UN TIPO DE HEMOFILIA MUY AGRESIVA.

18 años.

HEMOFILIA DE TIPO AGRESIVA.

398 - SE CONSIDERA EL TIPO DE HEMOFILIA CLÍNICAMENTE MÁS AGRESIVA:

A La HEMOFILIA es una enfermedad hemorrágica hereditaria caracterizada por la deficiencia funcional o


cuantitativa del factor VIII (hemofilia A) o del factor IX (hemofilia B) de la coagulación, esto debido a un
defecto en los genes que se encuentran localizados en el brazo largo del cromosoma X. Desde el punto de vista
clínico, la hemofilia A y la hemofilia B son indistinguibles. El fenotipo de la enfermedad se correlaciona con la
actividad residual del factor VIII y IX de coagulación, clasificándola en: - Leves: mayor de 5-40% - Moderada: de
1 a 5% - Grave: menor a 1% LA HEMOFILIA A CLÍNICAMENTE TIENDE A SER MÁS AGRESIVA QUE LA HEMOFILIA
B EN IGUALES CONDICIONES DE CONCENTRACIÓN PLASMÁTICA DEL FACTOR CORRESPONDIENTE.
B Se estima una frecuencia de 1:5,000 por cada varón nacido vivo. La hemofilia A es más frecuente que la B (80-
85% y de 15-20% respectivamente). Una razón por la que la HEMOFILIA TIPO A RESULTA MÁS AGRESIVA, es
porque a nivel mundial se informa que la prevalencia de los inhibidores es de 20-30% para los pacientes con
hemofilia A grave y de 3 a 5% para los de hemofilia B grave. Siendo el desarrollo de anticuerpos contra factores
de la coagulación VIII y IX exógenos, la complicación más grave, problemática y costosa de la exposición al
tratamiento de reemplazo en pacientes con hemofilia hereditaria. Esta complicación dificulta la resolución de
las hemorragias, ya que es la principal causa de falla al tratamiento con el factor deficiente y no se logra el
control de las mismas, como ocurre en los pacientes sin esta complicación. Los pacientes con inhibidores tienen
mayor grado de artropatía, disminución de la calidad de vida con un elevado costo del tratamiento, así como el
incremento de la morbi-mortalidad. La presencia de inhibidores limita la realización de procedimientos
quirúrgicos y pone en peligro la vida del paciente afectado

C La denominada HEMOFILIA C por deficiencia del factor de coagulación XI se hereda como un rasgo recesivo, lo
que resulta en defectos heterocigóticos u homocigóticos. Aun cuando las concentraciones del factor de
coagulación son bajas, no guarda una relación directa con el grado de las manifestaciones clínicas.
GENERALMENTE SE MANIFIESTA COMO UNA HEMORRAGIA LEVE, y ésta se asocia mayormente a traumatismos
o cirugías que exponen al paciente a una mayor tendencia a sangrar.

D NO EXISTE UNA HEMOFILIA TIPO D. RESUMEN ADICIONAL. Será importante que recuerdes algunos datos
importantes de la enfermedad de Von Willebrand (EvW), que corresponde a un diagnóstico que aparece a
menudo como diferencial en algunos reactivos de este tema. La enfermedad de Von Willebrand (EvW) es una
enfermedad hemorrágica hereditaria que se caracteriza por hemorragias mucocutáneas de intensidad variable y
que afecta primordialmente la hemostasia primaria en su interacción plaqueta, factor de von Willebrand y
endotelio. El factor de vW tiene un sitio de unión del factor VIII, con lo que prolonga su semivida en la
circulación. Por tal motivo, se recomienda realizar diagnóstico diferencial para Enfermedad de Von Willebrand
en caso de deficiencia de F VIII. La EvW tipo 2N tiene un cuadro clínico semejante a la hemofilia tipo A, salvo
por antecedentes familiares que muestran afección en mujeres, algunos autores la denominan HEMOFILIA
AUTOSÓMICA.

Bibliografía:
1. LONGO DL, FAUCI AS, KASPER DL, HAUSERSL, JAMESON JL, LOSCALZOJ. HARRISON. PRINCIPIOS DE
MEDICINA INTERNA, 18A EDICIÓN. MC GRAW HILL. NEW YORK, USA. 2012. 2. PAPADAKIS MAXINE A,
MCPHEE STEPHEN J. DIAGNÓSTICO CLÍNICO Y TRATAMIENTO. 52ª EDICIÓN. NUEVA YORK. 2013.

http://www.cenetec-difusion.com/CMGPC/IMSS-178-09/ER.pdf http://www.cenetec-difusion.com/CMGPC/IMSS-737-15/ER.pdf

399 - CON BASE EN ESTE TIPO DE HEMOFILIA, ENCONTRAREMOS DEFICIENCIA EN EL SIGUIENTE FACTOR
DE LA COAGULACIÓN:

XI La deficiencia del factor de coagulación XI, también denominado HEMOFILIA C, es un trastorno hemorrágico
raro, que se presenta en la población general con una frecuencia de 1 en un millón. Esta frecuencia se ve
incrementada en poblaciones de judíos asquenazíes e iraquíes. EN LA HEMOFILIA C EL FACTOR DE
COAGULACIÓN QUE SE ENCUENTRA ALTERADO ES EL XI.

X LOS TRASTORNOS HEREDITARIOS DEBIDOS A DEFICIENCIAS “DIFERENTES” DE LOS FACTORES DE


COAGULACIÓN VIII, IX Y XI SON EXTREMADAMENTE RAROS; su presentación clínica es muy variable y va
desde asintomáticos (alteraciones del factor VII), hasta potencialmente fatales (deficiencia del factor X y XIII).

IX El diagnóstico de certeza de hemofilia se hace a través de la determinación de FVIII:C para la HEMOFILIA A y


FIX:C para la HEMOFILIA B que se encuentran disminuidos o disfuncionales. SE DENOMINA “HEMOFILIA B”
CUANDO EL FACTOR DE COAGULACIÓN IX SE ENCUENTRA DISMINUIDO O DEFICIENTE.

VIII Los pacientes con sospecha clínica de hemofilia deben ser sometidos a dos fases principales de estudio para
su diagnóstico. - La PRIMERA FASE DE ESTUDIO del paciente con hemofilia debe incluir la determinación de
las pruebas de escrutinio con: Biometría hemática completa, Tiempo de hemorragia (TH), Tiempo de
tromboplastina parcial activada (TTPa) y tiempo de protrombina (TP) y Tiempo de Trombina (TT). Los
resultados mostraran un alargamiento del TTPa mayor a 10 segundos en relación al valor del testigo, con
plaquetas y TP normales. - La SEGUNDA FASE DE ESTUDIO incluye la determinación de los factores de
coagulación. Para detectar HEMOFILIA A con la deficiencia de factor VIII coagulante (F.VIII:C) o HEMOFILIA B
con factor IX coagulante (F.IX:C). SE DENOMINA “HEMOFILIA A” CUANDO EL FACTOR DE COAGULACIÓN VIII
SE ENCUENTRA DISMINUIDO O DEFICIENTE.
Bibliografía:
LONGO DL, FAUCI AS, KASPER DL, HAUSERSL, JAMESON JL, LOSCALZOJ. HARRISON. PRINCIPIOS DE
MEDICINA INTERNA, 18A EDICIÓN. MC GRAW HILL. NEW YORK, USA. 2012.

http://www.cenetec-difusion.com/CMGPC/IMSS-774-15/ER.pdf http://www.cenetec-difusion.com/CMGPC/IMSS-141-08/ER.pdf

FIN DEL CASO CLÍNICO SERIADO


ANÁLISIS DEL CASO CLÍNICO

IDENTIFICACIÓN DEL REACTIVO


Area: CIRUGÍA
Especialidad: URGENCIAS
Tema: QUEMADURAS
Subtema: QUEMADURAS ELECTRICAS Y QUÍMICAS

CASO CLÍNICO CON UNA PREGUNTA

EN EL CENTRO DE SALUD SE DAN PLATICAS ORIENTANDO A LOS USUARIOS SOBRE LA PREVENCION DE LAS
QUEMADURAS EN EL HOGAR.

-.

-.

-.

-.

-.

400 - EN LA PLÁTICA SE COMENTA QUE HAY ALGUNAS QUEMADURAS QUE SON MAS GRAVES DE LO
QUE APARENTAN. ESTE TIPO DE QUEMADURAS SON LAS:
ELÉCTRICAS. QUEMADURAS ELÉCTRICAS El traumatismo eléctrico se produce por el paso de corriente a través del
organismo. La mayoría de los accidentes eléctricos son por corrientes alternas (domésticos). El daño
va a depender de varios factores como: tipo de corriente (alterna o continua siendo la primera más
lesiva), intensidad, recorrido a su paso por el organismo, tiempo de contacto y resistencia de los
tejidos. TIPOS DE TRAUMATISMOS ELÉCTRICO: DIRECTO: paso de la corriente por el organismo. -
Bajo voltaje (<1000 V): es el 80% de los traumatismos eléctricos siendo especialmente frecuente en
el ámbito doméstico y en los niños. Las lesiones son pequeñas y profundas quemaduras dístales
(manos, boca) y arritmias graves. - Alto voltaje (>1000 V): produce lesiones graves como
quemaduras de diversos grados, afectación multiorgánica, destrucción tisular, etc. Similar al
síndrome por aplastamiento. INDIRECTO O ARCO VOLTAICO: se debe al campo magnético que se
produce alrededor de las líneas de alta tensión (> 10000 V). FLASH ELÉCTRICO: se trata de una
quemadura por llama. Es una lesión térmica. RAYO: produce parada respiratoria y muerte inmediata.
Lesiones cutáneas en arborización típica. ESTE TIPO DE QUEMADURAS CON FRECUENCIA SON MÁS
GRAVES DE LO QUE APARENTAN EN LA SUPERFICIE. EL CUERPO SIRVE COMO CONDUCTOR DE
ENERGÍA ELÉCTRICA Y EL CALOR QUE SE GENERA PRODUCE LA LESIÓN TÉRMICA TISULAR. LA
DIFERENCIA EN PÉRDIDA DE CALOR DESDE LA SUPERFICIE HASTA LOS TÉJIDOS PROFUNDOS ES LA
CAUSA DE QUE SE OBSERVE UNA PIEL RELATIVAMENTE NORMAL COEXISTIENDO CON NECROSIS
MUSCULAR PROFUNDA. La rabdomiólisis causa liberación de mioglobina, la cual puede producir
insuficiencia renal. El manejo inmediato de un paciente con quemadura eléctrica significativa incluye
atención de la vía aérea y ventilación, establecimiento de una línea intravenosa, monitoreo
electrocardiográfico y la colocación de una sonda vesical. Si la orina está oscura, se debe suponer
que hay presencia de hemocromógenos. No se debe esperar la confirmación por el laboratorio para
iniciar el tratamiento de mioglobinuria. Se debe aumentar la administración de líquidos para
asegurar un gasto urinario de por lo menos 100ml/hora en el adulto. Si el pigmento no se aclara
luego de aumentar la administración de líquidos, deben administrarse 25g de manitol en forma
inmediata y agregar 12.5g a cada litro subsecuente, con el fin de mantener la diuresis.

QUÍMICAS. QUEMADURAS QUÍMICAS Se define como el daño cutáneo agudo provocado por irritación directa,
corrosión y/o calor generado por agentes químicos. Las lesiones estarán en función de: –
Concentración del producto. – Tipo de reacción que se produzca. – Volumen que se aplique. –
Duración del contacto con el producto. Las lesiones causadas por ácido provocan necrosis
coagulativa, mientras que los álcalis provocan necrosis licuefactiva. Junto a esto es importante
recordar los efectos sistémicos de los ácidos y álcalis: – Metabólicos: Hipocalcemia (ácido
fluorhídrico y oxálico). – Hepatorrenal: (Ácido tánico, pícrico, fórmico y fósforo). – Respiratorio:
(Inhalación de vapores de amoniaco). 1. Quemaduras por ácidos • Suelen ser dolorosas, pasando
desde una leve lesión eritematosa hasta lesión severa con escara y aspecto de piel curtida. • El
tratamiento de urgencia es retirar a la víctima de la zona de exposición lavando con abundante agua
la zona afectada (de 1 a 10 minutos). También podemos utilizar soluciones diluidas con bicarbonato
sódico. Caso de lesiones retirar los esfacelos y desbridar la herida. • Respecto a los principios
generales del tratamiento no difieren del resto de quemaduras. Quemaduras por álcalis. • Se
producen con mayor frecuencia en el ambiente doméstico y su mecanismo es deshidratación celular
y saponificación de grasa subcutánea. De comienzo presentan menos lesiones que los ácidos pero a
largo plazo son más lesivos. • Clínicamente podemos observar desde un eritema a escaras con
aspecto más blando que los ácidos y tendencia a la autolisis. El dolor es más leve si bien puede
aumentar cuando penetra el álcali. • El tratamiento sería irrigar con abundante agua y durante un
período más prolongado que los ácidos (30- 60 minutos). Desbridar las heridas y el resto igual que
las demás quemaduras.

POR QUEMADURAS POR RADIACIÓN O RADIODERMITIS Son lesiones accidentales que obedecen a la
RADIACIÓN. aplicación de mayor dosis de lo adecuado en una exposición o la repetición con demasiada
frecuencia de exposiciones medias o pequeñas. Suelen provocar quemaduras que van desde un
primer hasta un cuarto grado con diversos tipos de lesiones en piel y órganos internos. Los cuadros
clínicos más graves pueden cursar con muerte súbita, astenia, fiebre, anemia, agranulocitosis, etc. Si
bien de forma crónica pueden provocar cáncer, cataratas, leucosis, necrosis ósea, etc. El tratamiento
de las lesiones será el típico de una quemadura siendo muy importante la prevención para evitar
patologías crónicas irreversibles.

POR LÁSER. Produce un efecto térmico y fotoquímico y a partir de 100oC destruye tejido. Los órganos más
sensibles son córnea, cristalino y retina, aunque sobre la piel puede producir cáncer. También puede
provocar electrocución, explosión o generación de gases tóxicos.

Bibliografía:
1. GUÍA DE PRÁCTICA CLÍNICA, ATENCIÓN PREHOSPITALARIA AL PACIENTE “GRAN QUEMADO ADULTO”.
MÉXICO: SECRETARIA DE SALUD; 2013. 2. GUÍA DE REFERENCIA RÁPIDA, ATENCIÓN PREHOSPITALARIA AL
PACIENTE “GRAN QUEMADO ADULTO”. MÉXICO: SECRETARIA DE SALUD; 2013. 3. COMITÉ DE TRAUMA DEL
COLEGIO AMERICANO DE CIRUJANOS, PROGRAMA AVANZADO DE APOYO VITAL EN TRAUMA PARA
MÉDICOS. ATLS, 7A EDICIÓN. USA. PP 244.

http://www.cenetec.salud.gob.mx/descargas/gpc/CatalogoMaestro/ISSSTE_678_13_Manejo_prehospitalario_quemaduras/GER678.pdf

También podría gustarte